You are on page 1of 1968

MCQ

NOVEMBER
2020
-
NOVEMBER
2008
NOVEMBER 2020
1. Patient with long QT syndrome and doctor needs an antibiotic for UTI. Where to search for
information about QT prolongation drugs:
a) Pubmed
b) CPS
c) Medline
d) Canadian Cardiovascular Society.
e) CredibleMeds

2. Pt. comes with small flakes of silver


- grey scales in head, she feels her
scalp is very dry & itchy. Dr excluded
psoriasis or seborrhoea and diagnosed
her with dandruff. All these
recommendations are true except:
a) Use cold air humidifier
b) Use Ketoconazol shampoo
c) Increase sun exposure
d) Use Hyrocortisone solution
e) Use coal-tar shampoo

3. How to improve the quality of the services provided to Pts with depression?
a) Provide the medication in a blister pack
b) Make sure the pt. receives his medication on time, no late refills
c) Follow-up call with patients to check the improvement of their symptoms

4. What is the least antidepressant causes nausea?


a) Mirtazapine
b) Citalopram
c) Sertraline
d) Venlafaxine
e) Fluoxetine
The most commonly observed adverse events related to the use of mirtazapine tablets (5% or greater drug‐
related incidence for mirtazapine tablets and at least twice that of placebo) were somnolence (54% vs. 18%),
increased appetite (17% vs. 2%), weight gain (12% vs. 2%) and dizziness (7% vs. 3%).
Digestive System: frequent: vomiting, anorexia; infrequent: eructation, glossitis, cholecystitis, nausea and
vomiting, gum hemorrhage, stomatitis, colitis, liver function tests abnormal.
Rare: tongue discolouration, ulcerative stomatitis, salivary gland enlargement, increased salivation,
intestinal obstruction, pancreatitis, aphthous stomatitis, cirrhosis of liver, gastritis, gastroenteritis, oral
moniliasis, tongue edema.
Adverse Effect Management
GI upset (nausea, constipation, diarrhea) Usually diminishes after 1–2 wk of therapy
Nausea: fluvoxamine (37%), venlafaxine (37%), Nausea: once-daily dosing; take with food; have
paroxetine (26%), sertraline (26%), vilazodone (24%), small, frequent meals; avoid strong smells
vortioxetine (23%), desvenlafaxine (22%), citalopram Constipation: increase fibre intake, fluids, laxatives,
(21%), fluoxetine (21%), duloxetine (20%) exercise
Constipation: fluvoxamine (18%), venlafaxine (15%), Diarrhea: have small, frequent meals; take
paroxetine (14%), duloxetine (11%) probiotics; limit caffeine, alcohol, spicy food; fluid
Diarrhea: vilazodone (29%), sertraline (18%), replacement
paroxetine (11%)

5. Pt. had GERD, doctor put him on long term PPI (3 months), came to the pharmacy after 2
months for refill and informed you that he has improved. what to recommend:
a) Tell him to stop the medication
b) Tell him to see his doctor for assessment and see if he can stop the medication
c) Tell the pt. he needs to take if for 6 months

6. 55 year with Heart problem, he had GERD and worsened over 3 months, what to
recommend?
a) Refer to physician
b) Give PPIs for 2 weeks
c) Give H2
d) Give antacids

7. Patient 65 years old has shingles in his eyes and come after 5 days what to counsel (Post
Herpatic neuritis). What do you recommend for this pxt?
a) Trifluridine eye drop
b) Famciclovir tab
c) Topical Acyclovir
d) No treatment as 5 days is too late
CTC: To prevent ocular complications, treat patients with ocular zoster even if the rash has been present for
up to 7 days. Promptly refer patients with ocular complications to an ophthalmologist.

8. Whats the most important risk factor for shingles


a) Age >65
b) Exposure to varicella zoster
c) Exposure to HSV
9. What to counsel?
a) Don’t let it get wet till cure
b) Take vaccine
c) Avoid contact with any person had not experienced chicken pox or avoid contact
with one not immunized
d) Don't put water on lesions till resolved

10. He is asking you about recommendation to PREVENT postherpetic neuralgia: What to


recommend
a) Pregabalin
b) Give capsaicin cream
c) Heat compresses
d) Give low dose acyclovir before exposure to the trigger
e) Avoid trigger
Consider acyclovir prophylaxis in healthy patients who are at increased risk of moderate to severe varicella
such as those ≥12 years of age, pregnant women, patients with chronic cutaneous or pulmonary disorders,
patients receiving long‐term salicylate treatment and in those receiving short or intermittent courses of oral
or inhaled corticosteroids.

11. Patient later developed PHN, what is the Drug of choice for pain?
a) Pregabalin
b) Propranolol
c) Nortiptyline
d) Tramadol

12. Diabetic pt. receiving insulin lispro 7 units before meals at bedtime & at breakfast. He
wakes up in the morning with high BG reading (dawn phenomenon), how to manage him?
a) Increase bed time insulin NPH dose
b) Decrease bed time insulin NPH dose
c) Increase Lispro before dinner
d) Decrease Lispro before dinner

13. Female suffer from migraine, her medical profile includes Asthma, CHF, diabetic and
depression and take mirtazapine. Which migraine prophylaxis could help her?
a) Amitryptiline
b) Propranolol
c) Verapamil
d) Sumatriptan
e) Ketorolac
14. While doing narcotic reconcilliation, the pharmacist noticed that there is 50 pills loss in the
actual narcotic inventory compairing to the computer system while doing inventory check.
What is the first action to take?
a) Call the police
b) Notify the college of pharmacy
c) Inform the local authorities
d) Report to the office of narcotics and cotrolled substances
e) Check the purchase invoices and sale reports and compare them to the system

15. Where to look for information about strontium use in the treatment of osteoporosis:
a) Medline (Pubmed in another version)
b) CPS
c) Martindale
d) Rxtx

16. What is the weekly dose of Alendronate (another version version residronate) for
Osteoporosis prophylaxis?
a) 35 mg
b) 70 mg
c) 150 mg
Alendronate Increases bone mass & prevents hip and spine (vertebral compression fractures) fractures
Prevention: 5 mg daily PO.
Treatment: 10 mg daily PO or 70 mg once weekly PO
S.E: Usually minimal: GI symptoms, altered taste, nighttime leg cramps.
Rare: atypical fractures, Osteonecrosis of the jaw (ONJ), acutephase reactions involving fever
and lymphopenia, joint or muscle pain, skin reactions, ocular effects.
Although available evidence does not prove a causal link between oral bisphosphonates and
ONJ, advise patients to complete elective dental work if possible before starting therapy.
Safety in impaired renal function (ClCr <35 mL/min) is unknown. C.I: Hypocalcemia
Take on empty stomach, at least 30 min before the first food or drink (other than plain
water) of the day or any other medication. Take with a full glass of water and do not eat or lie
down for 30 min after taking (to decrease risk of esophageal ulceration).
A 1- to 3-year drug holiday may be appropriate for moderate-risk patients (10-year fracture
risk ≤20%) who were prescribed alendronate for 3–5 years based solely on low BMD.

17. Clincial practice guidelines, which one is true?


a) Updated annually/yearly
b) A comprehensive information reviewed from expert
c) Rely on evidence-based data
d) Only RCT
18. COPD patient, truck driver, he is heavy smoker for the last 10 years and drinks 4-5cups if
coffee per day, BMI 24, has CAD, HTN & depression. His medications include Escitalopram,
SABA and tiotropium puffers to control his COPD, he came to the pharmacy complainig that
his rescue puffer is not controling his symptoms any more and he has to use it about 15 times a
day, he mentioned that his tiotrpium is working well, what should be your first response?
a) Review inhalation technique
b) Refer to his family doct to add ICS (maybe this is the answer)
c) Give him salbutamol refill
d) Give him glycopyronium

19. Few days later he admitted to the emergency due to increased purulent discharge, dyspnea
and thickening of his sputum also he confirmed that he wasn’t in the hospital before and he has
penicillin allergy, what is your recommendation?
a) Amox/clav
b) SMX/TMP
c) Azithromycin
d) Doxycycline 200mg loading dose, then 100-BID.
e) Amoxicillin
Another version
COPD pt with cystic fibrosis got exacerbation what’s the best treatment? I choose FQs

20. What is the best thing to do to help him after leaving the hospital?
a) Lung Rehabilation
b) O2 supply at home
c) Give him Roflumilast
d) Give him LABA
After an acute exacerbation, recommend initiation of pulmonary rehabilitation within 4 weeks to reduce the
chances of further exacerbations.
Consider early referral of symptomatic patients to pulmonary rehabilitation programs.

21. After 2 weeks he came to the pharmacy and told you that he decided to quit smoking and
already stareted to use nocotin lozenges (7pieces 2mg daily) but he still feels craving during
stressfull times. What should you give him?
a) Long-acting nicotine patches
b) Bupropion
c) Varenicline
d) Nicotine lozenges + inhaler
22. After few days, the patient started to feel fine tremors in his hands what could be the
reason?
a) Nicotin withdrwal
b) Caffeine toxicity due to reduced nicotine level
c) Poor control of nicotin level

23. Pt. is traveling to South America, he has received Dukoral vaccine 3 years ago, and today he
brings a Rx of Ciprofloxacin 500mg BID X 5 days, why you refuse to fill the Rx?
a) Drug resistance in south America
b) Dukoral will be enough to cover bacterial and viral infection
c) Wrong medication
d) Long duration of treatment
Fluoroquinolones Start prophylactic on the 1st day in the area of risk and continued for 1-2 days after return
Ciprofloxacin home, to maximum 3 wk total.
Levofloxacin Adults: Prophylaxis: 500 mg once daily PO
Treatment: 500 mg BID PO × 3 days or 500–1000 mg PO × 1 dose
Children: Not recommended
S.E: Infrequent GI disturbance, CNS effects, skin rash. May cause tendinopathy, peripheral
neuropathy and CNS disorders; in rare cases, these side effects may be persistent and/or
disabling. If symptoms resolve, there is no need to complete treatment.
Q. Not recommended in southeast Asia (Thailand, India, Nepal and Indonesia) and Indian
subcontinent due to antibiotic resistance.

24. What would be suitable counseling for dukoral?


a) It’s effective against food borne bacteria and viruses
b) If he’s travelling again within 5 years of taking it, he only needs booster dose
c) Take with food
d) Should be taken within 2 days before traveling
e) Don’t eat or drink any thing 1 hour before or after
f) Acidic drinks, coffee and nicotine increase the vaccine absorbtion
Vaccines Contains the nontoxic B subunit of cholera toxin, which has significant homology with the
Vibrio cholerae toxin of ETEC and is approved in Canada for prevention of TD caused by ETEC.
whole cell/ Prevention of enterotoxigenic E. coli diarrhea:
recombinant Adults and children ≥2 y: Primary immunization: 2 doses PO; 2nd dose administered within
cholera toxin B 7–42 days after the 1st dose and at least 1 wk before reaching destination
subunit vaccine Booster: 1 dose every 3 months if the risk is continuous
Q. Dukoral Q. Taken orally on an empty stomach (1 h before or 1 h after eating or drinking)
Inactivated S.E: Abdominal pain, diarrhea, nausea and vomiting.
May consider for prevention of TD in persons with chronic illnesses (e.g., HF, insulin-
dependent DM, IBD, chronic kidney disease), or in those with immune suppression
25. Patient height is 165 cm and her weight is 75 kg. She is using phenytoin 300 mg once daily;
she needs to start on oral contraceptive. She asks for recommendation for contraception
a) MPA injection
b) Barrier
c) Combined oral contraceptive with > 35 Estrogen
d) Patch

27. 26 yrs old lady on phenytoin (70kg) came after unprotected sex 6 days ago, she is asking for
emergency contraceptive, what is the risk factor of contraceptive failure:
a) Phenytoin
b) Weight
c) Several time usage of it
d) Interaction with other drugs
For patients taking a hepatic enzyme–inducing medication, it is preferable to take a nonhormonal EC, e.g.,
copper IUD. For women unable or unwilling to use the copper IUD, it is an option to take a total of 3 mg (2 ×
1.5 mg tablets) levonorgestrel as a single dose ASAP after unprotected intercourse.

28. What to give her:


a) Ullipristol
b) Plan B
c) Copper IUD
Copper-T IUD The only method to provide ongoing contraception to prevent future need for emergency
Flexi-T, Liberte, contraception. Interferes with implantation after fertilization.
MonaLisa Use within 7 days of unprotected intercourse as an emergency contraceptive.
It is the most effective method of emergency contraception available.
S.E: Major: salpingitis, uterine perforation, cervical perforation, endometrial embedding,
menorrhagia, pain, infection, ectopic pregnancy.
Contraindications: Absolute: pregnancy, undiagnosed vaginal bleeding, stenosed cervix,
copper allergy, current PID or STI, cervical or endometrial cancer, inability to place or retain
device. Relative: 2–28 days postpartum (to decrease risk of expulsion).
Pregnancy test is recommended if normal menstrual bleeding does not occur by day 21
following treatment.

29. A mother came with her son 25 months for AOM, he had it 11 months ago and another one
2 months back, he was prescribed Amoxicillin each time. Which of the following is not an
option to treat Acute Otitis Media?
a) Amoxicillin
b) Azithromycin
c) Cloxacillin
d) Cefuroxime
30. What is your advice to her concerning childs fever and ear pain?
a) Send her to the doctor to change the antibiotic
b) Tell her fever and symptoms will subside by 24-72 hours of using treatment.
c) Put cold pads so it will vaporize and relief the pain and fever
d) She can compress the ear with cloth soaked with cool water to ease the pain

31. Women work as airplane pilot and she take lorazepam. She came for early refill, what the
pharmacist should do:
a) Tell her that she should not drive plan 6 hours of taking lorazepam
b) Tell her that she maybe suffering from lorazepam abuse & don’t give early refills

32. Which one cause hyperglycemia?


a) Levothyroxine.
b) Hydrochlorothiazide.
c) Naproxen.
d) Donepzil

33. Pt. tells you that she heard about multivitamins can help to prevent age related macular
degeneration (she has a family history as her sister had AMD), she doesn’t have any eye
problem, what to recommend?
a) You don’t need to take any supplement
b) Recommend her to take supplement containing beta carotene
c) Recommend her to take supplement without beta carotene because it increases the risk of
lung cancer in smokers and non – smokers

34. 52 years old Female Patient with “osteoarthritis” and likes to swim 3times/week. She works
as a hairdresser, which is 2 km away from home. She smokes half a pack of cigarettes per day.
She felt once when she was young (7 years old) and broke her arm, her mother had a hip
fracture. She takes 1 cup of alcohol on weekends. What is her risk factor for developing
“osteoporosis”?
a) Smoking
b) Alcohol
c) Her mother’s fracture
d) Osteoarthritis

35. Whats is the recommendation


a) Walk to work
b) Remove risk of falls
c) Increase swimming to 3 times per weak
36. What is the DXA score that gives indication for osteoporosis?
a) -1.5
b) -2
c) -2.5

37. A 56 years old lady gets menopause 6 years ago; she has family history of breast cancer
(mother and Sister) and family history of hip fracture. Her mammography recently done is
correct, and her bone mass test score is - 3, she has asthma and use long time inhaled
fluticasone, she had GERD also. She recently had a hip fracture and she is afraid she will also
have osteoporosis later in her life. She is having a healthy life and drinks 3 cups of milk day.
What do you advise her?
a) Take vitamin D daily
b) Make bone mass density yearly

38. She went to the MD. Which of the following can he prescribed to prevent hip fractures?
a) Raloxifene
b) Teraparatide
c) Alendronate 10 mg daily
d) Hormonal therapy

39. Raloxifene side effect?


a) Dyslipidemia
b) VTE
c) Hypercalcemia
d) Uterine bleeding

40. Fentanyl patches. What is true?


a) Do not apply the new one before removing the old one
b) Discard the used one in the recycle bin or in the garbage after folding
c) Cut it into 4 quarter to get less doses
d) Used for acute pain

41. In a community pharmacy wt is correct about compounding


a) Electronic balance is more sensitive than the manual balance or torsion balance
b) Cylinder has to be kept sterilized on the shelf
c) Cream to be prepared on the paper
d) Use glass slab for ointments
e) Replace cylinders every year
Another version
What is wrong about combounding at pharmacy:
a) Glass is better than paper in preparation.
b) Bowls should be always sterile (or clean) even when not working.
c) Electronic balance is more accurate than manual one.
d) Preparation should be done under supervision of pharmacist.

42. 66 years patient living in a rural area developed ischemic stroke. His wife took him to the
nearest clinic, symptoms since 3 hrs including facial drooling and can’t move his arm. CT scan
was done and Hemorrhage bleeding was excluded. He supposed to be transferred to another one
(more advanced hospital) which is 3 hours away from the first one.
His blood glucose 11.6 mmole
His blood pressure is 180/110
He smokes, drink alcohol, BMI=28. Has heart failure, BP and taking Ramapril
Physician asking pharmacist to help to give this pt something until he reaches the emergency
Which one is suitable?
a) ASA 325 mg STAT
b) LMWH
c) Unfractionated heparin
d) Alteplase

43. Same above case, when you recommend this medication, you explain to the pt. the
risk/benefit of using the medication. As a pharmacist you uphold which ethic?
a) Beneficence
b) Non- maleficence
c) Autonomy
d) Justice

44. Doctor decided to transport him to a cardiac facility, what to monitor for while being
transported?
a) Heart rate
b) ST segment elevation
c) Breathing rate
d) Hypertension
e) DM
45. Patient has been discharged and the physician has decided to start him and a medication to
lower his risk of a stroke, which is the best option?
a) Rivaroxaban
b) ASA 81 mg
c) Dabigatran
d) Warfarin

46. pt with A.fib, end-stage renal failure and has hypertension, he was taking Enoxaparin, he
was discharged from hospital. On which drug he’ll continue?
a) Warfarin
b) Dabigatran
c) ASA
d) Rivaroxaban

47. In Hypothyroidism - lab test criteria, what could be decreased?


a) TSH
b) Free T4
c) T3
d) TIBC
e) Hematocrit

48. Lady diagnosed with depression and she is on paroxetine she is complain from chronic
constipation, when the pharmacist chat with her she mentioned that she is taking bisacodyl for
long time and now she is taking 5 pill every night without counseling the physician. What do
you recommend?
a) Switch the antidepressant
b) Send her to the doctor for more investigation
c) Increase the dose
d) Give senna
e) Increase fluid & fiber intake

49. Which antibiotic not to give in Pyelonephritis?


a) Aminoglycoside
b) Nitrofurantoin
c) Amxicillin clavulanic
d) SMX/TMP
e) Levofloxacin
50. Pt. comes to the pharmacy to return a box of insulin he received it yesterday from your
pharmacy because it was expired. What is the best action to do to prevent such incidence to
happen in the future?
a) Ask technician to check expiry date everytime before dispensing medication to the pt.
b) Put the near expiry date in the front and the far expiry date in the back of the shelf
c) Routinely check the expiry date of medications every 6 months
d) Check the expiry before buying from wholesaler
e) Put alarm on system screen

51. Pharmacy manager wants to implement something regarding policy of operation (something
like that) what will he do? Or What should be included in the pharmacy policy and procedures?
a) Ask staff to wear non- Latex gloves at time of patient vaccination
b) Ask staff to regularly do a minimum amount of med check or number of Rx that should
be done to by the pharmacist during his shift
c) All staff should report any interpersonal problems to the manager
d) During fire the staff should make sure they lock the pharmacy take their belonging before
meeting at the fire exit point

52. What is the first thing you can do to ensure staff safety while compounding suspension for
chlorambusil in vertical laminar hood or in a biological safety cabinet or aseptic cabinet?
a) Wear a 95 mask and face shield
b) Put gloves and wash your hands after prepartion
c) Its okay for a pregnant woman as long as she is wearing N95 mask
d) Pregnant women shouldn’t take part
e) Spill kit should be near the place of handling it

53. Old lady was working in her garden outdoors in sun for long time. She came to the
pharmacy with erythema and headache. Which of the following is a sign of heat stroke?
a) Sweating
b) Dizziness
c) Hot/dry skin
d) Edema
e) Numbness
54. A young adult went to fishing trip with his friends but did not apply sunscreen to his face or
his feet. Now, he came to the pharmacy complain from sun burns on his face and the surface of
his feet & it was 2nd degree burns, all of these medications you can give to him except?
a) Cortisone topical
b) Ibuprofen analgesic
c) Topical diphenhydramine
d) Topical antibiotic
e) Oatmeal bath

55. A lady has HIV, toxoplasmosis and sulfa allergy, in the hospital the nurse gave her
SMX/TMP, but the pharmacist discovered that. What is your immediate action?
a) Notify the on-call physician who was treating the patient for assessment
b) Call her husband
c) Start incidence report in the hospital private system
d) Conduct a meeting with nurse
e) Make a procedure in the pharmacy (system alarm)

56. Patient on Clozapine for 3 month his blood lab work is normal. He comes to the pharmacy
late on Friday, asks for refills for his Clozapine, however when asked about his blood work he
stated that he didn’t have time to do it and his physician is not available for 2 days. What can
you do in this scenario?
a) Give refill for 2 days and inform him to come back on Monday with blood work for
follow up
b) Refill all as his blood work is return to normal
c) Refer him to the emergency to do blood work
d) Tell him that you can’t dispense without lab work.

57. What to monitor for the above patient?


a) WBC
b) Neutrophils
c) Bilirubin
d) Lymphocyetes
58. Patient is a smoker & used to smoke half of packet every day and currently on clozapine.
Come for routine chek up at clinic following lab test value.
 Whiteblood cell 12000 (5000-11000)
 Neutrophil count 1.7 (normal 2.8-7.7)
 RBC normal
 Eosinophil less
 Lymphocytes 0
Q what abnormality patient face?
a) WBC
b) Neutrophills
c) Lymphocyetes
d) Red blood cells

59. Now, he wants to quite smoking what is your recommendation to change clozapine dose
a) Increase clozapine dose as increase in hepatic enzymes induction
b) Decrease clozapine dose as decrease hepatic enzymes induction
c) Increase clozapine dose as increase hepatic enzymes metabolism
d) Decrease clozapine dose as increase hepatic enzymes metabolism

60. What do you recommend for him to quit smoking, he have tried everything
a) Varenicline
b) Bupropion
c) Nicotine patch
d) Nicotine gum

61. Which one is the least SE of clozapine in comparing to other antipsychotics:


a) Anticholinergic effect
b) Extrapyramidal side effect
c) Postural hypotension
d) Pericarditis

62. A lady comes to the pharmacy, she takes COC and complaining of white patches on her
back. She said that she visits salon for tanning but these patches remain white. What is your
concern OR What’s your diagnosis?
a) It is tinea corporis
b) It is a photosensitivity due to COC
c) Psoriasis
d) She is applying sunscreen not evenly
63. Which of the following medication doesn’t cause SIADAH?
a) Amitriptyline
b) Sertraline
c) Paroxetine
d) Venlafaxine
e) Mertazapine

64. SE of quetiapine:
a) HTN
b) Hyperglycemia
c) Hyperprolactinemia
d) Hypertriglyceridemia

65. Which drug has the least weight gain among antipsychotics?
a) Lurasidone
b) Risperidone
c) Ziprasidone

66. Off label use of medication, where to look?


a) Canadian drug data base
b) CPS
c) Manufacturer information
d) Micromedex

67. Diabetic pt. has pancreatitis, he smokes, drink coffee, alcohol. He is receiving metformin,
but his HbA1c is 9. What is the cause of pancreatitis?
a) Smoking.
b) Alcohol.
c) Obesity
d) Ethnicity
e) Diabeties
Q. Acute pancreatitis is acute inflammation (Non-infectious) of the pancreas (and,
Acute pancreatitis sometimes, adjacent tissues).
The most common triggers are gallstones and alcohol intake.
The severity of acute pancreatitis is classified as mild, moderately severe, or severe
based on the presence of local complications and transient or persistent organ failure
Causes
Disorders: hypertriglyceridemia, alcoholism & bulimia nervosa.
Drug induced: EtoH, CHC (Estrogen/ EE  ↑ TG), Anti-HIV medictions (didanosine,
PIs), isotretinoin ↑ TG, incretin-based drugs DPP - 4 Inhibitors (Alogliptin, Linagliptin,
Saxagliptin & Sitagliptin), (GLP-1) Agonists (Dulaglutide, Semaglutide, Liraglutide &
Lixisenatide
In acute pancreatitis serum lipase and serum amylase are elevated however serum lipase
are slightly more sensitive in both major causes of pancreatitis gallstone and alcoholic
associated acute pancreatitis.
Acute Physiologic Assessment and Chronic Health Evaluation (APACHE) II Scores
should be calculated on admission and daily for the first 72 hours after admission.
An APACHE II Score of 8 or higher at baseline or in the first 72 hours is suggestive of
severe acute pancreatitis and is predictive of a worse clinical course.
Supportive care with fluid resuscitation, pain control and nutritional support
Prophylactic abx are NOT recommended regardless of type and severity

68. What to give to control his diabetes?


a) Acarbose
b) Insulin
c) Linagliptin
d) Liraglutide

69. Which diabetes drug shouldn’t be taken in a patient having acute pancreatitis?
a) Insulin
b) Metformin
c) Saxagliptin
d) Liraglutide

70. What to monitor for pancreatitis?


a) Lipase
b) Amylase
c) Bilirubin
d) ALT & AST

71. The dr wants to start empiric therapy for pancreatitis and asks you what’s the best regimen ?
a) Piperacillin/Tazobactam
b) Ampicillin/Metronidazole
c) No AB are currently needed.
d) Ceftriaxone
e) Ciprofloxacine.
72. Pt with hypoglycemia and the glucose reading around 4, Wife is asking what to do?
a) Give 1 liter of soft drink
b) Take 3 - 4 tablespoons of honey
c) Take 3 - 4 tablespoons dextrose
d) S.C glucagon

73. Same pt. had retinopathy and neuropathy, the pharmacist is teaching him about sick days,
which of the following he shouldn’t do?
a) Stop his insulin
b) Double the insulin dose
c) Measure his ketone bodies regularly
d) Go to ER

74. You are a pharmacy manager, and one technician came to you complaining that another
technician misuses his sick leave. How to handle?
a) Tell the technician it is his right to use the allowed sick leave and you can’t do any thing
b) Call the other technician for a meeting
c) Confront both the complainer with the accuser
d) Check the sick leave history schedule and see if there’s any suspicious action

75. Female pt. has cancer and will undergo a hysterectomy, Dr. called you asking what AB to
give as a prophylaxis. She has penicillin allergy. What do you recommend?
a) Gentamycin + Clindamycin
b) Vancomycin
c) Ceftriaxone
d) Metronidazole + Clindamycin

76. Pregnant lady diagnosed with HTN and using salbutamol for asthma, what do u recommend
to start for her BP
a) Labetalol
b) Nifedipine XL
c) Verapamil
d) Hydrochlorthiazide

77. Same patient was prescribed ASA 81; she came to the pharamacy to ask if its safe to use it?
a) Yes, use it as ASA protects from preeclampsia
b) Yes, use it as ASA prevents stroke
c) No, there is no benefit from using ASA
d) It can’t be taken by pregnant woman
78. What is the one that mean not upholding justice?
a) Calling only subset of pts to talk about medication shortage.

79. Long scenario about patient has severe bleeding after takin dabigatran (anticoagulant). Now
this patient transfered to hospital but the antidote which is idaracizumab is expensive and not
covered by insurance plan, what will the pharmacist do?
a) Give protamine sulfate instead (wrong as it is heparin antidote).
b) Wait till tomorrow to negotiate with insurance company (something like that).
c) Ask for idracizumab from university hospital nearby
Another version
A patient with Dabigatran overdose was transported to local clinic which doesn’t have
idarucizumab (antidote). What shows non-maleficience?
a) The pharmacist makes sure that he’s not taking other anticoagulants for DVT.
b) Calling and asking to stop any other anticoagulant he was taking

80. Dabigatran councelling:


a) Take it as a whole. Don’t crush or chew
b) Take with food to increase its absorption
Direct Inhibit both clot-bound and free thrombin lla.
Thrombin Patients >75 y, use with caution & consider reducing dose to 150 mg BID PO daily.
Inhibitors For patients >75 y with 1 or more risk factors for bleeding consider 110 mg BID PO.
Dabigatran, S.E: Bleeding, dyspepsia. Antidote: Idurucizumab
Pradaxa. Q. The capsule should NOT be chewed, broken, opened or crushed.
Use with caution with other drugs acting on Pgp e.g., quinidine.
Dabigatran is recommended to be discontinued 24 hours prior to endoscopic procedures,
irrespective of renal functional status in low bleeding risk procedures.
In high bleeding risk procedures or surgeries, dabigatran is recommended to be discontinued
48–72 hours prior in normal renal function and mild renal function (CrCl > 50 mL/minute).
In moderate renal impairment (CrCl of 30–49 mL/minute), discontinue 72–96 hours before
high-risk endoscopic procedure.
If severe renal impairment (CrCl < 29 mL/min), discontinue 96–144 hours before endoscopy.
Increased risk of bleeding with ClCr < 30 mL/min

81. Terazocin, what is NOT true?


a) Take it at bed time
b) Might cause nasopharingitis
c) Causes renal problem
d) It should be given at low dose then titrated up to therapeutic dose
82.You have subs. A conc. is 4% to be added to subs B to have a final volume of 50 ml. After
mixing both subs. The conc. of subs. A in the final solution was 0.1 %. Calculate the amount of
subs. B in mg in each dose if you divided the total solution to 5 small doses. Conc. of subs B is
10% w/v.
Answer:
C1*V1 = C2*V2 4%*V1 = 0.1%*50
V1 = 0.1%*50/4% = 1.25 ml which is the final volume of subs. A
Volume of subs. B = 50 ‐1.25 = 48.75 ml
10 gm ‐‐‐‐‐‐‐ 100 ml X gm ‐‐‐‐‐‐ 48.75 ml X = 48.75 * 10/100 = 4.875 gm
4.875gm /5 doses = 0.975 gm in each dose.

83.Patient in hospital is taking IV Cotrimoxazole (each ml contains 80 mg SMX / 16mg TMP)


0.5 ml Q6hr. Dr. want to shift him to liquid (200 mg SMX / 40 mg TMP) in each 5 ml, how
many of liquid is needed to provide the equivalent dose as was taken IV.
Answer:
IV Cotrimoxazole 0.5 ml Q6hr = 0.5 * 4 = 2 ml /Day
80 mg ‐‐‐‐‐‐‐ 1 ml X mg ‐‐‐‐‐‐‐ 2 ml X= 2*80/1 = 160 mg
Liquid:
200 mg ‐‐‐‐‐‐‐ 5 ml 160 mg ‐‐‐‐‐‐‐ Y ml Y = 160*5/200 = 4 ml/Day

84. Tobramycin IV 400 mg/24 hr was given to a patient, peak conc. is > 20mg & rough conc.
required is < 0.5 mg. post dose peak was 28 mg/L.& after 10 hr. was 7 mg/L. find the T1/2 &
the correct dose.
Answer:
Log C = Log Cₒ - (k*t /2.303)
Log 7 = Log 28 - (10K /2.303) k=0.14
T1/2= 0.693 / K = 0.693 / 0.14 = 4.99 hr (5 hr.)
400 mg / 24 hr -------- 28 mg/ L X mg / 24 hr ------- 20 mg / L
X = 20*400 / 28 = 285.7 mg
285.7 mg ------- 24 hr 400 mg -------- Y Y = 400*24/285.7 = 33.6 hr.

85. Patient receives 1 gm Vancomycin IV, T 1∕2 of Vancomycin is 3 days. After 3 days later,
He received 1 gm Vancomycin IV, the blood concentration was 15mmol/L, what is the steady
state trough concentration of Vancomycin
a) 49
b) 30
c) 59
d) 45
Answer:
Steady state is the trough steady state con = 5 t1/2
After 3 days (1 T1/2), conc. was 15 m. mole/L.
So, at time of adminstration, it was 30 m. mole/L.
He is receiving 1gm every 3 days. so, we add the conc of 30m.mole to the conc after the half life

86. Patient was given Drug X 08:00. At 10:00, levels were 60. At 22:00, levels were 45, what is
T1/2?
a) 10 h
b) 12 h
c) 18 h
d) 24 h
Answer:
Log C=log Cₒ - KT /2.303 Log 45=log 60 - K*12/ 2.303 K = 0.02397
T½ = 0.693/0.02397 = 28.9 hr.

87. Child needs TPN with 0.5 micromole/L of Selenium, your stock is 0.25 micromole/L & 40
microgram /ml, how much of 40 μg/ml of selenium would you add if you want to make 482mL
(M. Wt of selenium is 79).
Answer:
40 microgram ------- 1 ml X microgram ------- 1000 ml
X = 1000*40/1 = 40000 microgram / L. No. of m. mole = 40000 / 79 = 506.32 m. mole /L.

X + Y = 482 ml Y = 0.25/ (505.82 + 0.25) * 482 = 0.238 L.

88. Which floroqinolones doesn’t need renal dose adjustment in patients with renal failure?
a) Moxifloxacin
b) Levofloxacin
c) Ciprofloxacin
d) Ofloxacin
89. What pt need treatment for hyperglycemia:
a) Pt taking insulin forgot his insulin dose. Reading for glucose is 5
b) Pt the pharmacist dispensed wrong order of 50 units insulin instead of 5 units before
meals and 10 units bid.
c) Pt taking oral medication for diabeties and took 2 pills of glyburide instead of 1
d) Pt missed his long-acting insulin at night.

90. End stage metastatic cancer patient is using LMWH for VTE prophylaxis, he told his
onchologist that he doesn't want to take injections and he is ready to take oral treatment what
should be the onchologist response?
a) Swich the patient to warfarin and respect his autonomy
b) Stop LMWH as it’s his responsibility and wait for the VTE to happen
c) Continue prescriping LMWH and leave the decision to the patient either to take it or not
d) Prescribe long acting LMWH to be injected every other day
e) Recommend LMWH and force him to take
f) Recommend one day LMWH once daily instead of BID

91. Patient BSA 1.2m2 and given doxorubicin 95mg/day. If the max dose per day for (adult) is
400mg/m2 BSA, what is the max dose you should give?
Answer
400mg ---- 1 m2 X ----- 1.2 m2
X = 400 * 1.2 = 480mg

92. Mother came with her baby has diaper rash she tried vasline got worse she tried zinc oxide
25% for 2 days and show small improvement, asking you what’s your recommendation?
a) Give clotrimazole cream
b) Give H. Cortisone 0.5% cream
c) Ask he to keep using zinc that will make it better it’s just need time

93. Lady came for compression stocking. She had heart failure, diabeties (taking metformine –
sitagliptine) using ACEI, gabapentine. Complain of swollen bilatral legs not comfortable but no
pain. What is the cause of swelling?
a) DVT.
b) Gabapentin.
c) Heart failure.
d) Sitagliptin.

94. What community pharmacy should apply as per health canada act portability?
a) Having medical coverage in any place across the country
95. What not to include in nasogastric tube?
a) Suspension
b) Film coated tablets
c) Enteric coated tablet
d) Powder

96. Patient on different drugs, got insomnia, which drug is responsible?


a) Duloxetine
b) Morphine
c) Sertraline
d) Gabapentin
e) Temazepam
SNRI Inhibits serotonin and norepinephrine reuptake at all doses.
Duloxetine 30–60 mg once daily PO Maximum dose: 120 mg/day (divided BID)
Cymbalta S.E: Nausea, headache, drowsiness, insomnia, dizziness, dry mouth.
Q. Do not use in patients with severe renal impairment (ClCr <30 mL/min).
Drug Interactions: Alcohol, CNS depressants. MAO inhibitors may cause serotonin
syndrome (severe reaction: tremor, agitation, hypomania, hypertension).
Tramadol may also increase risk of serotonin syndrome.
Do not use with potent CYP1A2 inhibitors (ciprofloxacin, fluvoxamine, ketoconazole).
CYP2D6 inhibitors (e.g., SSRIs) may increase duloxetine levels.

97. Ginkgo bilopa is used for


a) Memory impairment
b) Depression
c) Headache

98. A Technician made an Error while entering the drug. He entered Levothyroxin instead of
Leviteracitam. Where you’ll report this error?
a) NASIR
b) ISMP
c) CADTH

99. Patient wants 1500 tab of pseudoepherdine (approx. 30 packs each contain may be 100 mg or
more) to take with him outside canada for his friends for cold treatment, what you should respond
a) Contact his doctor to get a refill
b) Refuse to dispense because it is a prescribed drug
c) Refuse because it is a precursor of amphetamine
d) It’s illegal to buy pseudophedrine for family and friends
e) The quantity is too large so it’s suspicious
100. Which of the following assessment tools is used for depression?
a) PHQ-9
b) MoCA
c) MMSE

101. In which pt we don’t need to adjust the dose of aminoglycoside?


a) Pt 42 yo with low kidney function.
b) 80 yo pt with COPD.
c) 50 yo pt with pancreatitis.

102. A technician complaining that pharmacist is treating him with disrespect. As a pharmacy
manager, what to do?
a) Make a meeting and let the technicians face the pharmacist
b) Inform the pharmacist to suggest solution for the situation
c) Put policy that staff should respect each other
d) Speak with him directly about what happen
e) Speak with him in general about the best practice inside the pharmacy

103. Pharmacist working in community pharmacy had rosacea and one of his friends gave him
cream and it works good with hem and his rosacea has disappeared. The pharmacist decided to
buy a large amount of this cream to sell it as a private business. Which of the following
represent a conflict of interest?
a) To put assign that there is a new cream for rosacea and his phone no.
b) To discuss with the patients while dispensing other prescription and tell them that he has
a new cream for rosacea
c) To telephone all patients that have been using metronidazole for rosacea and try to
convince them with the new product which is not covered by their insurance plan.

104. 14 weeks Pregnant diabetic lady got itching and vaginal discharge. She came to the
pharmacy asking why she got this itching despite of using douching continuously for the last 3
months, what is the risk factor?
a) Pregnancy
b) Female gender
c) Diabetes
d) Douching
105. What should be the 1st question to ask her?
a) Do you have dysuria or painful urination?
b) Do you have vaginal discharge?
c) For how long you have these symptoms?
d) Do you have fever?
e) What is your blood glucose reading?

106. The doctor found out that she is having GARDINELLA VAGINALIS infection, he
diagnosed her with bacterial vaginitis, what is your recomendation for ttt?
a) Metronidazole 500 mg po tab bid for 7 days
b) Fluconazole 150 mg po tab
c) Miconazole X 7 days
d) Clindamycin vaginal cream

107. 2 days later she came with prescriptions wrote metronidazole PO, what do you think about
the treatment?
a) It’s right treatment
b) She needs just topical
c) No need for treatment

108. What do you suggest regarding monitoring?


a) No need for monitoring
b) Monitor 2 weeks after finishing ttt, every month during pregnancy until delivery

109. A patient has been complaining from vomiting for 5 days with a history of Nausea for 3
months. he says his vomiting increases after eating pizza. He’s also drinks a lot of coffee and
smokes but alcohol is not too much. He is on Carbamazepine 400 mg CR for epilepsy but he’s
been seizure free for 10 years. What to do?
a) Give him omeprazole OTC
b) Give him H2 blocker
c) Refer to doctor
d) Non pharm measures; avoid eating pizza, coffee and alcohol
110. Same patient is tested positive for H. Pylori. He came back with a prescription for H.
Pylori: Amoxicillin 500 Bid, dexlansoprazole 40 bid, Clarithromycin 500 bid. Upon entering
the prescription, the computer showed an alarm of interaction between Carbamazepine &
Clarithromycin. What to do?
a) Stop CBZ temporarily
b) Reduce the dose of clarithromycin to 500 once daily
c) Call doctor to switch him to another antibiotic
d) Decrease the dose of clarithromycin

111. Same patient, the dexlansoperazole is expensive for the patient because its not covered
with his insurance and he can not pay, you have the ability to change it to pantoprazole which is
less expensive, what to do?
a) Change it to pantoprazole 40 mg and document in the patient profile and fax the
change to the physician

112. After 2 months he came to you and says he is good and asks if he can stop pantoprazole
and he says that he have been trying to stop smoking and started with nicotine gum lately and
feels tremor in his hands and insomnia:
a) Tell him he can stop pantoprazole after talking with his physician and the tremor is
a nicotine overdosage

113. Pharmacist will do presentation about Guillane barre syndrome, what to include in the
presentation:
a) Influenza vaccine may rarely cause it

114. What is the needle size of influenza vaccine?


a) 1 cm, 5/8 inch
b) 1.5 cm, 7/8 inch
c) 2.4 cm, 2 inches
d) 2.5 cm, 1 inch

115. Pt travelling to the Dominican & wants a Twinrix vaccine, what to counsel?
a) Transmission of hepatitis B via sexual contact type contraception.
b) Transmission of hepatitis B via sexual contact is unusual.
c) Twinrix can prevent Guillain-Barre Syndrome
d) Hep A Respiratory droplets transmission
116. He would like to take the accelerated Twinrix dose, which of these regimens is correct?
a) 0, 1, 6 days
b) 0, 7, 21, 365 days
c) 0, 2, 4, 6 days

117. What is correct regarding Clopidogrel


a) Rare side effect “leukemia “
b) In transient ischemia, it is not a first line
c) Leukopenia is a rare side effect

118. What is the difference between intention to treat and per protocol?
a) Per protocol is the method usually used
b) Per protocol involve all participants from the beginning
c) Per protocol it doesn’t allow for confirmative difference
d) Intention to treat includes all patients even the ones who dropped out of the trial

119. FMEA analysis definition “to predict future error”

120. Patient start using senimet (levodopa – carbidopa), he is using propranolol. What to monitor?
a) CBC
b) Blood pressure
c) Serum creatinine
d) Bradycardia
Symptomatic postural hypotension can occur when SINEMET is added to the treatment of a patient
receiving antihypertensive drugs. Therefore, when therapy with SINEMET is started, dosage adjustment of
the antihypertensive drug may be required.

121. What is a nested case control study


a) A cohort study in a case control study
b) A case control in a cohort study
c) A randomized controlled study
Nested Case Control Studies: Similar to case control study EXCEPT:
 The case and control subjects are obtained from the COHORT (not the population) of an ongoing
PROSPECTIVE cohort study (the case‐control study is NESTED in the cohort study)
 The study is prospective in nature
 For every case that appears in the cohort, the researcher selects 1‐ 4 control subjects
122. Lurasidone:
a) Low risk of weight gain
b) Initiate at 20 mg/day
c) Titrates to lowest effective dose to decrease side effect
d) Take with food
Should be administered with food (at least 350 calories independent of fat content). Tablets should not be
crushed or cut; they should be swallowed whole.
Doses below 40 mg have not been shown to be effective in patients with schizophrenia.
In bipolar depression, the recommended starting dose is 20 mg given once daily as monotherapy or as
adjunctive therapy with lithium or valproate

123. Lurasidone monitoring main concern is?


a) Akathisia
b) Insomnia
c) Potassium level
d) Glucose level
The most common adverse reactions (incidence ≥5% and at least twice the rate of placebo) in adolescent
patients (13 to 17 years) treated with LATUDA were somnolence, nausea, akathisia, extrapyramidal
symptoms (non‐akathisia, 40 mg only), rhinorrhea/rhinitis (80 mg only) and vomiting.

124. Patient taking medroxy progesterone acetate (MPA), which one is the most common long
term side effect of MPA?
a) Osteoporosis
b) Thrombolytic event
c) Osteomalcia
d) Endometric cancer
e) Breast tenderness
Medroxy Inject 150 mg within first 5 days of onset of
progesterone menses (to be effective immediately).
acetate Interval between injections must not exceed 13
Depo-Provera wk. Condoms needed for STI protection
Excellent for women who should avoid high
estrogen doses, e.g., migraine sufferers.
S.E: Breast tenderness, insomnia or
somnolence, fatigue, mood changes, e.g.,
depression or irritability, weight gain,
menstrual irregularities, decreased libido, skin
sensitivity reactions, hyperpyrexia, acne.
Long-term use: decrease in BMD (monitor Ca),
delayed return of fertility.
Consider other options in women with risk for osteoporosis.
Interaction with griseofulvin (advise backup barrier method during therapy).
Carbamazepine, phenytoin, protease inhibitors, phenobarbital, rifampicin, St. John's wort,
topiramate may decrease progestin serum concentrations.
C.I: Absolute: pregnancy, unexplained vaginal or urinary tract bleeding, current diagnosis of
breast cancer, known sensitivity to MPA or to the vehicle. Relative: severe cirrhosis, active viral
hepatitis, benign hepatic adenoma.

125. A lady complaining from spotting on day 14 -17 of her cycle while on Alesse (EE 20 mcg /
levonorgestril 0.1 mg) and her cycle every 23 - 28 days. What to do?
a) Increases her estrogen
b) Increases her progesterone
c) Decrease her estrogen
d) Decrease her progesterone

126. A 25-years lady asking about her risk factor for future DVT as she has family history of
thrombosis, she is on OCPs. If we want do test to know his risk of thrombosis, what test do you
recommend for checking her risk
a) D-dimer test
b) Factor V Leiden
c) INR

127. Rivaroxiban counseling


a) Adjustment in renal failure
b) Take with meal to increase bioavailability
c) Don’t take with antacid
d) Keep in original container

128. Type of analysis that converts benefits and risk to monetary values and compares them:
a) Cost effectiveness analysis
b) Cost Benefit analysis
c) Cost minimization analysis
129. Horizontal laminar flow hoods:
a) Air should flow in the back of the hood.
b) Clean horizontal from top to down.
c) Allow pharmacists only to work with hazardous drugs
d) The sample should be 6 inch away from walls
e) Nothing should hinder the flow of air behind the sample

130. According to ISMP, what is the safest tall man letter?


a) PredniSone - prednisolone
b) morPHINE - hydromorPHONE
c) CEFuroxime - ceFIXIME
d) Morphine - HYDROmorphine

131. Which of these is a problem when using Medline as a reference while keeping in mind that
you are concerned about ‘’publication bias’’?
a) Too many results
b) Indexing of abstracts
c) Small database
a) Different language
b) Lacks to publish negative results

132. Patient complaining from edema on his both legs which of his medication is NOT related
to his edema?
a) Pregabaline
b) Pioglitazone
c) Amlodipine
d) Spironolactone

133. Diabetic patient has infection and severe pain reaching to his bone, what infection he might
have?
a) Osteomyelitis
b) Cellulitis
c) Infected decubitus ulcer

134. What is the causative organism for his infection?


a) S. Aureus
b) E. coli
c) S. pnemonia
135. Same patient, this infection has foul smelling, so we should give treatment that covers:
a) Gm -ve
b) Gm -ve and Gm +ve
c) Gm +ve
d) Anaerobic
Empiric therapy for early infections should target Staph. aureus and the beta‐hemolytic streptococci.
If the patient presents with severe necrotic tissue and foul‐smelling wounds, use empiric therapy targeting
Gram Negative and Anaerobic pathogens, especially if the patient has been soaking the feet. These cases
will require a surgical opinion as antibiotics will be unlikely sufficient.

136. What is the treatment for previous pt:


a) Amoxi/clav.
b) Muprocin
c) Hydrocortisone.
d) Vancomycin + gentamycin.
Q. Osteomyelitis Palpation of bone at base of ulcer Treatment may require 4–6 weeks of parenteral
S. aureus (MSSA or MRSA) is the most common pathogen, or several months of oral antimicrobial therapy.
but other less virulent organisms may be pathogenic, e.g., Oral Options
coagulase-negative staphylococci.  Amoxi/clav, Cephalexin, Clindamycin,
Treat with IV therapy or long-term oral antimicrobial Cloxacillin, Doxycycline, Linezolid,
therapy using agents that are well absorbed from the GI tract Moxifloxacin, SMX/TMP.
and have good distribution to bone and tissue  Ciprofloxacin, levofloxacin, SMX/TMP
Surgical débridement indicated to remove necrotic debris, plus clindamycin or metronidazole.
abscess or sequestrum Parenteral Options
Therapy should base on culture results whenever possible.  3rd generation cephalosporin plus PO/IV
If MRSA present / suspected, add vancomycin or linezolid. Clindamycin or PO/IV metronidazole
If clinical improvement isn’t observed (e.g. resolution of  Daptomycin, Linezolid, Carbapenem,
erythema, edema, heat, draining sinus, coverage of bone Piperacillin/tazobactam.
with soft tissue), consult with a specialist.

137. A doctor called you as a pharmacist to advice one of his pt confused with her multi
medication, she has filled her drugs for 3months, what to advice to reduce confusion?
a) Follow up calls for monthly refill
b) Weekly calendar
c) Easily open bottles
d) Drug splitter
e) Monthly blister pack
138. Pt has bipolar and taking Li. He has developed toxicity because he suddenly changed his
regular habits and drinks a lot of coffee and eat junk food (certain problems suddenly happened
to him) also less sleep, asking about most factor leads to such a problem
a) Change in caffeine intake
b) Diet (high salt intake)
c) Ibuprofen

139. What will be the first signs of Li toxicity?


a) Hand tremors
b) Hyperglycemia

140. What to monitor for lithium? Serum creatinine

141. 27 yo male, came to the clinic asking for PRE- Exposure prophylaxis against HIV. He
tested negative for HIV. His history includes multiple sexual relationships (300 partners males
and females) without condom. He treated before from gonorrhea and chlamydia. He’s currently
living with a male roommate who is HIV positive & a previous heroin addict. What is the
highest risk factor to catch HIV?
a) Multiple sex partners and condomless
b) His roommate
c) His past diagnosis of gonorrhea

142. What’s the pre-exposure prophylaxis?


a) Emtricitabine / Tenofovir
b) Emtricitabine / Tenofovir AF / Raltegravir
c) Emtricitabine/ Tenofovir AF / Elvitegravir / Cobicistat.

143. How should he use the treatment?


a) Take it within 24 hours of high-risk behavior.
b) Take it with one month of high-risk behavior.
c) Take it daily regardless of high-risk behavior.

144. For how long he should take the regimen?


a) Every day for 4 weeks
b) 3 days / week
c) For 6 months
d) Daily
145. How long after his first follow-up visit should he be tested for HIV using serology?
a) 1 month
b) 3 months
c) 6 months
d) 12 months.

146. Long case pt was taking amiodarone and digoxin, Bisoprolol & citalopram. Which
medication of the following the patient need to stop because it may cause QT prolongation?
a) Citalopram
b) Amiodarone
c) Digoxin
d) Bisoprolol

147. After a while pt got digoxin toxicity because of what?


a) Amiodarone interaction with digoxin
Amiodarone increases digoxin concentrations by inhibiting the P‐glycoprotein (P‐gp) mediated efflux
transporter on the apical surface of the intestinal enterocyte, biliary canalicular membrane and the renal
tubular cells that facilitates the elimination of digoxin from the body.

148. Second day morning the digoxin conc is 2.9 n, what to do if K level after digifab is 4.4?
a) Give another digifab
b) Give both k and digifab
c) Do nothing only monitor
d) Give potassium (his potassium was already 4.4 after the digifab)

149. What can cause pulse rate to increase to the pt


a) Amiodarone
b) Bisoprolol
c) Diltiazem

150. Mom come for her 7 years old boy that has allergic rhinitis for more than 3 weeks now.
What to advice?
a) Give him dephenhydramine.
b) Give him pseudoephidrine.
c) Refer to dr because more than 2 weeks.
d) Give acetaminophen if there is fever.
151. Which of the following has the increased risk to develop DVT?
a) Patient in admitted to the hospital for toe amputation
b) Elective surgery of hysterectomy in 55 years old
c) Elective surgery of orthopedic hip replacement in 70 years old man

152. Patient has done gastroctomy. Now, he has low hemoglobin, MCV 104 (Normal 102),
what test should you require?
a) Check for iron
b) Check for vit B12
c) TSH test
d) CBC and Differential
e) Folic acid

153. pt diagnosed with seizures the doctor started him on phenytoin and it’s work good with
him at first time, now pt start drinking alcohol more than before, they wrote a lot about the pt
life style I cannot recall and he start to feel symptoms like (toxicity). They asking what do you
think the cause?
a) Avoid alcohol to minimize hepatic toxicity
b) Contraindicated with caffeine
c) The medication shouldn’t be crushed or chewed
d) Do a weekly phenytoin level until therapeutic dose achieved
e) Take with food or without food

154. KW is a diabetes patient with CrCl 28 ml/min comes to the pharmacy with symptoms of
shallow breathing, discomfort and decreased appetite. Upon investigation you knew that he
takes NPH 25 Units BID and metformin for diabetes and citalopram for depression, he also uses
garlic to reduce his cholesterol. What possibly caused these symptoms?
a) High dose NPH
b) Citalopram
c) Metformin
d) Garlic

155. Baby 18-month-old, recent immigrant from china. She developed rash over her face and
trunk and high fever, she attends day care and leave in a basement room with her mom and dad
and brother, she was diagnosed with chickenpox, what is her risk factor
a) Day care attendence
b) Age
c) Being new immigrant
156. Double doctoring case, a pt came Wz RX oxycodone from Doc X, then after two days
came Wz another narcotic RX from Doc Y, what to do
a) Call the second prescriber

157. A nurse called from long term care facility and said that she collected turbid malodorous
urine from 81 years old female and the urine culture results showed bacterial gtowth of E.
COLI, what should be your first response?
a) Ask the nurse if the patient has any symptoms
b) Isolate the patient
c) Start antibiotic therapy

158. What is the appropriate treatment for this patient


a) Ciprofloxacin
b) Fosfomycin
c) Nitrofurantoin
d) SMX/TMP
e) No antibiotic is required

159. Obese female patient, she is a heavy smoker & wants to quite smoking but worrid about
her weight as smoking distract her from eating junk food, in which stage is the patient?
a) Precontemplation
b) Contemplation
c) Preparation
d) Action

160. Which of the followin represent the difinition of NNT?


a) Number of reductions in BP in mmHg when using a drug vs placeebo
b) No of full-term pregnant women who gave birth to a healthy baby when using infertility
treatment
c) No of migrains in migraine prophylaxis treatment

161. Pregnant women in 14 weeks develop UTI and has penicillin allergy. She says she doesn’t
have pain or dysuria and no fever. What is the preferred treatment?
a) Cephalexin
b) Nitrofurantoin
c) SMX/TMP
d) She is asymptomatic no need to treat
e) Amoxicillin
f) Levofloxacin
162. Patient will take biphosphonate. When counseling about ORAL biphosphonates, which of
the following is a side effect that the patient should be alerted to?
a) Osteonecrosis of the Jaw
b) Atypical fracture
c) Weight gain
d) Sedation

163. Which calcium supplement is suitable with PPI?


a) Ca carbonate
b) Ca Citrate
c) Ca gluconate
d) Ca sulfate

164. To consider 2 drugs interchangeable what should we consider to be equal?


a) C max
b) T max
c) Vd
d) Bioavailability
The generic is bioequivalent when it meets the following criteria:
 AUC: must be between 80% ‐ 125% of the AUC of the brand name
 Cmax: must be between 80% ‐ 125% of the Cmax of the brand name
 The drug contents within +/‐ 5% of the brand name

165. Which of these can be delegated to the technician to be without independently the
pharmacist supervision?
a) Best possible med history (BPMH)
b) Medication reconcilliation
c) Monitoring patient taking methadone
d) Educate about OTC med

166. Impetigo case: 7-year-old kid presents to the pharmacy with her father, father complains
that a mosquito bit her this morning and the area is red, inflamed and crusted. What is an
optimal OTC medication that a pharmacist can recommend?
a) Mupirocin
b) Fucidin
c) Topical antihistamine
d) Topical Hydocortisone 0.5%
167. What non-pharmacologic measure is appropriate?
a) Petroleum and cold compresses
b) Cover the wound
c) Use a tight bandage around it
d) Remove the crust using hot compress

168. 4 days after, the father presents to the pharmacy again with his daughter and complains that
it got worse because his daughter keeps on scratching the area, what is the best action to do now?
a) Recommend a stronger steroid
b) Recommend tacrolimus
c) Tell him to wait
d) Diphenhydramine tab
e) Refer to family physician
Systemic antibiotic therapy Should be considered if the condition is widespread, the patient is
immunocompromised or has valvular heart disease, there are signs of fever or bacteremia or there is a lack
of improvement after 24–48 hours of topical therapy

169. When the pharmacist asked what happen, the father said that his daughter scratch it while
the daughter denied it. What to document following the assessment?
a) Patient’s condition got worse due to scratching
b) Case got worse after recommending a treatment
c) That the father and the daughter said opposite facts
a) That the treatment failed to heal the disease

170. Health Canada withdraw a medication (injection) distributed in one city only
a) Cold chain breached during transportation
b) Impurities discovers in the medication
c) Expensive drug.
d) Shortage in raw materials

171. Lady with severe back pain can’t tolerate nausea and vomiting of Tylenol #1 (codeine +
acetaminophen), what is your recommendation
a) Iboprufen
b) Fentanyl patch
c) Acetaminophen
d) Ketorolac inj
e) Morphine SC
172. A female patient with chronic pain. shows list of medications including multiple pain
killers: Morphine SR 100 mg BID, Morphine SR 60 mg once, Morphine IR 5 mg PRN,
Gabapentin 300 mg, Temazepam QHS. She has Hyperalgesia. The dr wants to step down her
opioids & then discontinue them to treat hyperalgesia & he asks the pharmacist what to do.
Which medication regimen she should be shifted to:
a) Morphine SR 30 mg 3 times daily then decrease the dose by 30 % weekly
b) Morphine SR 90 mg BID then decreases the dose by 10 % weekly
c) Keep IR morphine and make SR 30 BID and target tapering 10% every 2 weeks
d) Keep IR morphine and give hydromorphone 6 bid and target tapering 10% every 2 weeks
e) Stop morphine IR and gradually reduce the dose of Morphine SR over weeks
f) Discontinue everything

173. Same above case, the pt develop hyperalgesia & Dr. wants to stop morphine to manage his
pain. How you will gona explain this to the patient with a friendly language & easy language:
a) Tell the pt that you are suffering from hyperalgesia du to high dose / prolonged morphine
use
b) Tell the pt. that long term use of some pain mediations causes increase sensitivity to
pain and that’s what happened to you, and Dr. wants to stop the medication because
it causes you the pain
c) the pain killler suppose to reduce the pain but in your case, it acts opposite way
d) You Have Hyperalgesia. Your body produces abnormal response to non-painful stimuli.
We have to cut off your pain medications.
e) Your medications caused you a condition in which your body over responds to pain so
we have to slowly reduce your medications.
f) In some pts the body get tolerance from using the opioid for a long time
g) In some pts body get sensitive to pain from a normal stimulus that in a certain time the
opioid dose starts not to be effective and pt starts feeling the pain
h) Your body developed something called “Hyperalgia “and we need to step down your
opioid dose

174. Case a Pt with CAP coming with purulant discharge, has penicillin allergy. Dr decided to
treat him as an out patient. Which antibiotic is suitable for him?
a) Amoxiclav
b) Cephalixin
c) Doxycycline 100 BID then once for 5 days
d) Clarithromycin
e) Levofloxacillin
175. What to monitor for CAP improvement?
a) WBC
b) Self symptoms monitoring
c) Chest X ray
d) Chest pain
e) No nausea vomiting

176. Which medication cases Esophagitis the most?


a) Doxycycline
b) Metronidazole
c) Fluconazole
d) Dabigatran
e) SMX/TMp
Instances of esophageal lesions (esophagitis and ulcerations), sometimes severe, have been reported in
patients receiving doxycycline. The patients must be instructed to take Doxycycline Capsules
/ Doxycycline Tablets with a full glass of water, to keep in orthostatic position after the administration and
not to go to bed within 1‐2 hours after the intake. If symptoms such as dysphagia and retrosternal pain
occur, Doxycycline Capsules / Doxycycline Tablets should be discontinued and an esophagic lesion must be
investigated
Doxycycline Capsules / Doxycycline Tablets should not be prescribed to patients with obstructive esophagic
pathology, such as stenosis and achalasia.

177. Vaccine against Herpes and shingles prevention was compared to placebo
Incidence of disease with vaccine. Placebo. RRR
Herpes. 0.3 0.6 51%
Shingles. 3% 1%. 66%
Calculate NNT of vaccine for shingles prevention?
Answer
ARR = CER – EER = 3% - 1% = 2% NNT = 1/ARR = 1 / 2% = 50

178. From the numbers above, what does RRR 64 % mean?


a) Patients who take herpes vaccine will have 51% less risk of developing the disease
compared to placebo.
b) Pts taking vaccine are 64 % less likely to get shingles compared to placebo group
c) Patients on vaccine are 34 % less likely to get infection (something like that)

179. Contraindications to give vaccine by pharmacist


a) Severe febrile fever
b) Pregnant lady
180. Patient lost her med that was refilled as 90 tabs, from 48 days her insurance doesn’t cover
the lost drugs and can not pay for his drug. What to do?
a) Charge the province benefit insurance
b) Tell her that she has to pay for the med
c) Give her the med and take it from the insurance as an early refill
d) Ask the company to give exception
e) Give him some free and ask the insurance to pay when the time is due

181. Cancer patient in advanced stage prescribed sunitinib that is very expensive and his
insurance doesn’t cover his medications. He is poor and can’t afford paying for his drugs, so he
decided to stop the treatment. How can you help him to show beneficiencec?
a) Contact the manufacturer to give some drug through compassionate program
b) Give him drug for free if you can
c) Request medication through SAP
d) Ask the dr to change the medication

182. Special Access program (SAP) and pt doesn’t have money:


a) Government pays for drug.
b) Pharmacist can charge credit monthly plan for it.
c) Pharmacist can ask manufacture if can help this pt.

183. Which most cause tachycardia


a) Amiodarone
b) Digoxin
c) Verapamil
d) Propafenone
The adverse drug reactions reported for CCBs reflect differences in their pharmacology, so that the type and
frequency of events may differ between the dihydropyridines, nondihydropyridines and flunarizine. In
therapeutic dosages, most adverse effects are transient and mild in severity, occur within the first few
weeks of therapy, or during periods of dose titration. Often adverse effects do not require drug
discontinuation and occasionally may be minimized with dosage adjustment. Adverse effects may be
exaggerated in older patients (over 65).
The most common adverse effects and those that most frequently result in discontinuation are
cardiovascular and nervous system effects related to the vasodilatory action of CCBs, e.g., pedal edema,
flushing, palpitations, headache. These are reported more often with dihydropyridines, but can occur when
nondihydropyridines are used. Serious side effects include angina, heart failure (due to negative inotropic
and chronotropic effects associated more often with nondihydropyridines), pulmonary edema, tachycardia
(caused by reflex activation of the sympathetic system and also peripheral vasodilation), bradycardia,
excessive hypotension, skin rashes, arthritis and transient blindness (for nifedipine). Extrapyramidal effects,
galactorrhea and mental depression are specifically associated with flunarizine.
184. Which of these medications require sales report or Which one is under narcotic regulation?
a) Lorazepam
b) Butorphanol
c) Phenytoin
d) Buprenorphine patch

185. For how long you should keep the records?


a) 1 year
b) 2 years
c) 10 days

186. Patient did a dental extraction and has pain, his dentist prescribed narcotic drug, he has a
prescription for 250 tablets Tylenol 1 for 6 months use, the pharmacist refused to dispense it
due to?
a) Not a proper/clear direction to the pharmacist
b) Quantity is too much than normal

187. What mistake that can lead to hyperglycemia:


a) Miss enterpretation of glargine 4 IU instead of 40 IU.
b) Dispensing of 2.5 IU instead of 5 IU lispro
c) Increase bed time NPH to 3 doses
d) Stop enteral feeding while still giving insulin
e) Take basal insulin with no rapid acting before meals

188. A new study for new anti-hyperlipidemic drug, what will be the most important thing that
mean that drug is promising:
a) Reduction in LDL
b) Reduction of stroke
c) Reduction of Myocardiac infarction
d) Decrease ischemic cardiovascular disease

189. Drug X started to see rise in selling and not included in the provincial formulary, why
could be the reason of the increased sales or an increase in doctor prescribing x ACEi that is not
in benefit formulary in the area and the pharmacist wanted to adjust his inventory accordingly,
why do u think this happened?
a) Academic detailing with the physician
b) Some pts heard about drug X in an American advertisement
c) Anticipated shortage of the drug.
d) Shortage in ACEi that is leading and normally prescribed
190. A cancer patient taking morphine CR and IR for palliative care and is going home. What is
true about fear of abuse?
a) Minimal in palliative care
b) Give her little supply to ensure there is no abuse

191. Drug shortage nation – wide. What could be the reason?


a) Some changes in quality assurance
b) Decrease in the drug price
c) Breach in transportation from wholesaler to pharmacies

192. You are a pharmacy manager in a hospital, shortage of ferrous sulphate as national recall,
what to do: or Back order of Fe-Sucrose in hospital, what you do:
a) Automated policy to replace it with ferrous-Dextran
b) Take all Fe-sucrose from all wards back to pharmacy
c) Talk to patient who take sucrose and convert it to dextran
d) Send memo to nurses and unit with this shortage
e) Go to all hospitals units to return to pharmacy

193. Patient has bacterial infection


 Sensitive to: cloxacillin, cephalosporin
 Resistant to penicillin
What type of resistance:
a) B-lactamase
b) Change in B-lactam binding site
c) Efflux

194. Nurse works unstable shifts sometimes in the morning and sometimes in the evening is to
be started on an insulin, however she is concerned about hypoglycemia because she works
unstable shifts and would like an insulin which will be suitable for any shift change, what is a
good option?
a) NPH BID morning and evening
b) Detemir OD morning
c) Degludec OD evening
d) Glargine OD evening
195. Which patient the pharmacist can treat but needs to refer or What minor ailment that will
still needs referral?
a) Hemorrhoids stage 3 that is retractable and no bleeding
b) 2 yellow nailed with fingers involvement
c) Pregnant women in her last trimester with mild heart burn
d) 10 years girl with mild allergy for three weeks

196. 45 yrs patient with C. difficle and has Diarrhea more than 8 times /day, no vomiting or
other complication. Patient buys food from vendor - WBC 14 – Scr 85 – BP 128/80. 5 months
ago, he was in hospital & was prescribed Moxifloxacin 400 IV one dose followed by step down
to oral for 7 days and patient is on pantoprazole for one year, what kind of infection?
a) Mild
b) Severe
c) Severe with distention
d) Fulminant

197. What is a good initial treatment?


a) Vancomycin 125 mg QID
b) Fidoxamicin 200mg QID
c) Rifaximin 100 mg BID
d) Vancomycine PO + Metronidazole IV

198. What’s the Risk factor of C. Diff in this pt.?


a) Age
b) Moxifloxacin
c) Pantoprazole

199. What’s the goal of therapy of C. diff?


a) Decrease dehydration within the first 24 hrs
b) Decrease diarrhea by day 4
c) Balance the electrolyte
d) Stop vomiting

200. What is the target to confirm treatment success or How to monitor for improvement of the
case?
a) Negative C.difficile in stool after 10 days
b) PCR test done 1 week after
c) No monitoring required
d) Reduction in number of daily stools over the next 5 days
201. Pt 71 kg taking digoxin and concentration was 1.5 ng/l. Dr decided pt should given
antidote fab. How much fab should be given if we want to give 4 divided doses of fab:
Dose of fab = concentration of digoxin ng/l × weight
Answer
Dose of fab = concentration of digoxin ng/l × weight
Dose of fab = 1.5 / 71 = 0.021mg * 4 doses = 0.085mg

202. What is the error can be happened due to relying on technology?


a) Drop-down menu to select the drug by writing the first 2 letters only
b) Dispensing error by bringing wrong medicine from the shelf
c) Write first 2 letters of medication, a pop screen came down, so choose another one by
mistake
d) 2 patients name similar so choose one instead of the other.

203. What type of insulin is glargine?


a) Long-acting insulin
b) Short acting insulin
c) Intermediate insulin

204. For how long can we keep insulin out of fridge (at room temperature):
a) 6 weeks
b) 28 days
c) 14 days
d) 21 days

205. Olanzapine side effect?


a) Hypoglycemia
b) Weight gain
c) Hypokalemia

206. A patient who has GAD and recently suffered from depression. The doctor started him on
bupropion, which side effect is it most important to monitor for in that patient?
a) Nausea and vomiting
b) Agitation
c) Lethargy
d) Weight gain.
e) Sun downing.
Adverse events commonly encountered during the clinical development of bupropion hydrochloride were
headache, constipation, dry mouth, nausea, dizziness, insomnia, tremor and tinnitus.
207. Dementia goal of treatment?
a) Reduce repetitive questioning
b) Improve the pt quality of life
c) Stop disease progression

208. A pharmacist was chosen for a role of getting consents from people to do clinical trial on
them and when he read the details, he discovered that there is a rare but serious side effect to the
drug not mentioned in the consent so if he wants to act with non maleficence what should he do
a) Talk to investigators to review the policy and consent and add that side effect to it
b) Ask the patient to do research on the drug before they do the trial
c) Refuse to do that role and give it to another person
d) Ignore since it is not serious side effect

209. A child with ADHD on methylphenidate was staying with his grand mother. The
grandmother believes her grandson is find and doesn’t need to take any medications. She
refused to give her grandson his medicine and called the pharmacist to ask him why her
grandson is taking this medication. What could be the pharmacist response?
a) Tell her he has ADHD
b) Tell her he needs this medication and that you are obliged to call his parents if she
continues to refuse giving him the medication.
c) The pharmacist advice the grandmother to talk to his parents and discuss the
matter with them
d) The pharmacist should educate the grandmother about the disease

210. Patient in the hospital and she has allergy from SMX/TMP written in her profile, the
doctor prescribes her SMX/TMP, while the pharmacist reading her file, the nurse took
SMX/TMP and give it to the patient which cause her allergy, what to do:
a) Call the doctor and tell him about what happen
b) Call her husband and explain to him whats happen
c) Do internal incident report in the hospital report about what happen

211. Pt. receive Azithromycin + Ampicillin. what to counsel?


a) Avoid dairy products
b) Photosensitivity, avoid sun exposure
c) If you develop fever, go and see your doctor
212. A patient got paralysis in his left arm. His family brought him to the hospital after 3.5 hrs
and he will be transported to another hospital that’s 3 hrs away. The pharmacist explains the
risks & benefits of treatment to the patient. What ethical priniciple is upheld?
a) Beneficience
b) Autonomy
c) Fidelity

213. What is the councelling of using Symbicort Turbohaler?


a) If you hear whistle sound that means that you took the correct dose
b) If you wash the aero chamber, dry it with a piece of clothes
c) Take deep forceful inhalation OR Take a deep breath and inhale strongly
d) You can keep two doses at a time in the chamber for later use
e) Rinse your mouth once per month

214. A clinic and a pharmacy decided to monitor pts’ INR in the pharmacy and give the
feedback of the patients monitoring to that Clinic. One day One of the pt had an INR of 6. What
could be the pharmacist’s first question?
a) Do you have any bleeding?
b) Did you change your diet?
c) Did you double your warfarin dose?

215. Same case: How you fulfill successful INR monitoring in your pharmacy?
a) By monitoring pts INR and keep the INR within the required range (2-3)
b) By helping pts not to experience any bleeding symptoms

216. Physician decided to start bisphosphonates; he is saying that the bisphosphonates will
reduce the risk by 40% if her risk is 15%, how much it will be reduced smthng like this can’t
remember exactly
a) 9
b) 10
c) 7

217. A diabetic lady with preserved heart failure she was on prenidopril 2 mg together with
other drugs her kidney function is normal and her blood pressure is 132/89. Why the doctor
wants to increase the prendopril to 10 mg
a) Prevent nephropathy progression
b) For the heart failure preservation
c) For her HTN
218. Manger wants to divide some work between pharmacy technician & Assisstant:
1. Data entry (DE)
2. Filling
3. Check appropriateness (CA)
4. Dispensing medication (DM)
5. Counsel patient (CP)
6. Technical med check
Which choice will represent the best time management for these work assessments?
a) Ph. Tech. 1, 3 & 5
b) Assistant 2, 5 & 6
c) ph. Tech. 1, 4 & 5
d) Assistant 1, 2 & 4
Ignore choices
Pharmacist makes 3 & 5 & 4, Tech makes 6, Assistant 1 & 2
Acc to choices given
Pharmacist 3 and 5
Assistant 1 and 2 (tech can perform 2 also)

219. Pharmacist was treating tech in a bad way, what to do as a pharmacy manager?
a) Arrange meeting for both of them together so they can discuss it
b) Arrange a meeting between you and the pharmacist and discuss the issue without
pinpointing on the pharmacist action in specific
c) Let tech talk directly to the pharmacist
d) Ignore the whole situation

220. Patient was vaccinated, site of injection got swollen, hot and hardened, what is the type of
this reaction?
a) Guillain-Barre syndrome
b) Arthus reaction
c) Cellulitis
An Arthus reaction is a local vasculitis associated with deposition of immune complexes and activation of
complement. Immune complexes form in the setting of high local concentration of vaccine antigens and high
circulating antibody concentration.

221. Which need pharmacist intervention or Which one has prescribing mistake:
a) Hydromorphone 1mg, 1-2 mg prn 4 times a day
b) Salbutamol 100microgram 1 puff bid x 30 days
c) Fluticasone 1 buff BID (No concentration)
222. What to consider to include in hospital formulary?
a) Being included in the province benefit formulary
b) Therapeutic index
c) Evidence from clinical trials

223. Dr. ordered a loading dose of Phenytoin 20mg/Kg infused with a rate of 0.5 mg/Kg/min
for a 6 years old child whose weight is 33-pound. If you've it in 100 ml bag. What is the rate of
pumb that you must be adjusted in ml/hr.?
Answer
Rate of infusion = 0.5 mg / kg per min
Loading dose = 20 mg / kg So, it will need = 40 minutes
Now all dose is dissolved in 100 ml
So, if 100 ml needs 40 minutes, then how many mls in 1 hr
So, rate will be adjusted to 150 ml per hour

224. To make 100 gm of ointement we need 60 ml mineral oil, how much mineral oil needed to
make 480 mg of ointement " specific gravity 0.9
Answer
100 ——— 60 0.48 ——— X X = 0.288 ml
0.9 = wt/0.288 Wt = 0.2592 gram

225. You have in your pharmacy 400 mg potassium permanganate tablets. You are requested to
prepare 2 L of a potassium permanganate solution such that the patient will dilute this 1 in 10 to
obtain a 0.005% solution suitable for wound washing. How many of these tablets would you
dissolve in a small amount of water before making the solution up to a final volume of 2 L with
water
Answer
Final strength after dilution equals to 0.005 %, But this one diluted 10 times
So original = 0.005* 10 = 0.05 %. This means 0.05 gm in 100 ml = 0.5 gm in liter (0.5 gm in
1000ml)
But our solution is 2 liters. So, we have to use 0.5 *2= 1 gm
As tablet is 400 mg, so 1 gm potassium will be 2.5 tablets

226. Pt got ascites & he is chronic alcoholism, Dr gave spironolactone didn’t work, what’s your
recommendations?
a) Add Furosemide
b) Paracentesis
c) Add amiloride
227. How to monitor ttt effectiveness?
a) Regain consciousness
b) Monitor body weight
c) Increase urine output
A in case of encephalopathy when patient lost
his consciousness

228. 5 days later he went to the ER, Pt.


Case got worse and peritoneal fluid got
infection. What infection do you think pt
has now?
a) Cholengitis
b) Spontaneous bacterial peritonitis
c) Pancreatitis

229. Now, he treated, what’s your advice


to the pt?
a) Change healthy life style
b) Give ABX as prophylactic
c) Correct liver function

230. Pt taking S.C Heparin. He complains about bruising on his skin. What could be the reason
of this bruising?
a) He’s injecting Heparin in his abdomen
b) Injecting 45-90
c) Rubbing skin after injection
d) After injecting, you leave the needle for 10 sec before removing it
Heparin can be administered by the sc or iv route. The im route can also be used, but is usually avoided due
to the increased risk of bruising and bleeding. Preparations of the appropriate strength and volume should
be selected based on the route of administration.
The recommended site for injection is into the fat of the lower abdomen. This should be at least 5
centimeters away from your belly button and out towards your sides.
Prior to injection, wash your hands and cleanse (do not rub) the selected site for injection with an alcohol
swab. Select a different site of the lower abdomen for each injection.
Remove the needle cover by pulling it straight off the syringe. If adjusting the dose is required, the dose
adjustment must be done prior to injecting.
Sit or lie down in a comfortable position and gather a fold of skin with your thumb and forefinger.
Then holding the syringe at a right angle to the skin folded between your thumb and forefinger, insert the
needle as far as it will go. Hold the skin fold throughout the injection process. Once the needle has been
inserted, the syringe should not be moved. Push the plunger to inject. Be sure the syringe is empty and the
plunger is pushed all the way down before removing the syringe.
Remove the needle at a right angle, by pulling it straight out. A protective sleeve will automatically cover
the needle.
NOTE: The safety system allowing release of the protective sleeve can only be activated when the syringe
has been emptied by pressing the plunger all the way down.
You can now let go of the skin fold and apply light pressure to the skin at the injection site for several
seconds with an alcohol swab. This action will help lessen any oozing or bleeding. Do not rub the injection
site. You should then safely dispose of the syringe and needle with its protective sleeve, so they remain out
of reach of children.

231. Pt came to the pharmacy with a metolazone prescription the assistant prepared the
prescription and the Technician check it and by mistake they gave MTX instead. After 2 weeks
the pt was in the ER for MTX toxicity asking about the symptoms of MTX toxicity?
a) Nausea
b) Palpitation
Oral overdose of MTX is often due to incorrect dosage and administration (e.g., daily rather than weekly
administration by patients). Symptoms include leukopenia, thrombocytopenia, anemia, pancytopenia, bone
marrow suppression, mucositis, stomatitis, oral ulceration, nausea, vomiting, GI ulceration and GI bleeding.
Symptoms of intrathecal overdose include headache, nausea and vomiting, seizure or convulsion and acute
toxic encephalopathy. Leucovorin calcium (also known as folinic acid) is used to counteract toxicity of
inadvertent overdosage of methotrexate. Administer as soon as possible.

232. What is the test that you should order for this pt?
a) CBC
b) ALP
c) TSH
233. How to prevent like this mistake in the future?
a) Let the pharmacist review all the new prescriptions and counsel the pt and make
sure that he understands his new medication

234. It was a long case taking duloxetine asking about the serious SE?
a) Bleeding
b) Hypotension
c) Hypernatremia
d) Renal failure

235. Question about hazard ratio or relative risk reduction asking about the confidence interval.
Gave us confidence interval for 4 study asking which one in significant?
a) HR = 0.75 and C.I = (0.25 TO 1.28)
b) RR= 1.314 and C.I = (0.90 TO 1.35)
c) HR = 0.56 and C.I = (0.45 to 0.78)
• RR/HR/OR
 If Cl contains 1 (95% CI 0.7 – 1.2)  there is NO difference = Accept NULL hypothesis = P > 0.05
 If Cl does NOT contain 1 (95% CI 1.1 – 1.8 or 0.3 – 0.8)  There IS difference = Reject the null
hypothesis = P </= 0.05

236. Patient is taking Atorvastatin 10 mg for dyslipidemia, later the doctor started him on
fenofibrate. A couple of days later the patient complained of muscle pain to the doctor stopped
his treatments. What is the initial next step?
a) Start Niacin
b) Start Fenofibrate
c) Start Atorvastatin and fenofibrate
d) Start atorvastatin 10 mg
237. Published bias effects what?
a) Meta analysis

238. DM pt her diabetes was well controlled she has IDA start on iron and she took ASA now
as prophylaxis. Now, her A1c start to be higher than before, what do you think the cause?
a) Iron supplies
b) ASA
c) Anemia
Factors that can affect A1C
Factor Increased A1C Decreased A1C
Erythropoiesis Iron defeciency. Use of erythropoietin, iron or B12
B12 defeciency Reticulocytosis.
Decreased erythropoiesis Chronic liver disease
Altered hemoglobin Fetal hemoglobin. Hemoglobinopathies. Methemoglobin
Variable change in A1C Genetic determinants
Altered glycation Alcoholism Ingestion of aspirin, vitamin C or vitamin E
Chronic renal failure Hemoglobinopathies
Decreased erythrocyte pH Increased erythrocyte pH
Erythrocyte destruction Increased erythrocyte lifespan: Decreased erythrocyte lifespan:
Assays Splenectomy Chronic renal failure
Hemoglobinopathies
Splenomegaly
Rheumatoid arthritis
Antiretrovirals Ribavirin Dapsone

239. Complications of HTN affect all of these except?


a) Erectile dysfunction
b) Tricuspide Valve
c) Ischemic Stroke
d) Retinopathy
e) Myomegaly (Hypertrophy)

240. Pt has diagnosed with depression and he has history of insomnia, what’s your
recommendation for treatment?
a) Mirtazapine
b) Bupropion
c) Venlafaxine
241. Pt diagnosed with depression he is using bupropion, what’s the natural product that may
cause DI that you should advise your pt to avoid? Or Which herbal know to cause serotonin
syndrome with anti depressant?
a) St. John worth
b) Vit E
c) Co-Q10

242. A patient on a high dose corticosteroid for long periods almost 6 months. What side effect
that should be monitored?
a) Skin photosensetivity
b) Mood changes
c) Hypoglycemia
d) Weight Loss
e) Osteoporosis

243. Asthma patient is suffering from osteoporosis due to long term corticosteroids therapy,
what is the modifiable agent to decrease his risk to develop osteoporosis
Many choices but the right is to change corticosteroids to montelukast

244. A female patient was recently diagnosed with diabetes (HBA1c 12%). She come with a
new prescription for insulin. She also has SLE & Osteoporosis. Her medications include
Prednisone, Alendronate 70 once weekly & Ca supplement. What’s the pharmacist’s concern?
a) She’s taking a drug for which there’s no indication.
b) There’s drug-drug interaction
c) There’s disease-drug interaction
Between corticosteroids and diabetes
245. After a couple of weeks her husband calls stating that she’s suffering from bilateral leg
edema & elevated BP. Her physician wants to start her on anti-hypertensive. What’s the least
favorable for her condition?
a) HCTZ
b) Nadolol
HCTZ cause hyperglycemia and hypercalcemia & she is taking Ca supplement

246. Blood pressure medication cause hyperlipidemia


a) Nadolol
b) Hydrochlorithaizide
247. Which of the following diseases if present in one of the pharmacy staff you have to remove
him from tasks requiring direct patient contact?
a) Conjunctivitis
b) Hepatitis C
c) Cellulitis
d) Pyelonyphritis

248. What could cause a patient to use undesired ingredients when selecting a product?
a) Use of an extension line for a product

249. Pharmacist saw a technician counseling on a medicine correctly on phone, what is the
appropriate response?
a) Wait and then give him punishment
b) Advice the tech not to do it again in the future
c) Continue hearing and see if it is ok
d) Take the phone from his hand & continue counseling yourself

250. Which of the following prescriptions is legally correct:


a) Testosterone cap BID mitte 60 refill * 2 every month

251. In which patient population is measuring vancomycin peak levels important?


a) Stage 2 CKD
b) Measuring peak conc for vancomycin is not clinically important

252.
Ingredient A 307 mg
Ingredient B 5 mg
Clarithromycin 500 mg
Ingredient D 188 mg
What’s the amount of clarithromycin in ml that would be required to prepare 450 gm of this
mixture (Specific gravity of clarithromycin is 0.98)?

253. Gives clearance & Rate for 4 drugs and asks which drug has highest saturated state conc.
Cl K time-1 R
Drug A
Drug B
Drug C
Drug D
254. A dr wants a 10% solution of drug x but you have stock solution 50% & 5 % … you also
want to use all the amount you have of the 5 % solution (150 ml) … what quantity of 50 %
solution should be added to the whole quantity of 5 % solution to make the mixture?
Answer
Using allegation method

X = 5 * 150 / 40 = 18.75ml

255. You’re in a province that allows pharmacist to prescribe … how would you contact the
patient’s dr to tell him you prescribed something to that patient?
a) Call clinic and tell assistant to put you through to the dr
b) Call clinic and leave msg with assistant
c) Give patient a note

256. Side effects of co-trimoxazole


a) Hypoglycemia
Sulfamethoxazole 800/160 mg (2 regular-strength tablets or 1 double-strength tablet) Q12H PO
/ Trimethoprim S.E: Rash, pruritus. May cause hyperkalemia; risk increased in elderly & renally impaired
(SMX/TMP) patients. Hypersensitivity reactions, N & V, diarrhea, rash, false increase in serum
creatinine, renal impairment, neutropenia, thrombocytopenia, anemia, agranulocytosis.
Hyperkalemia with ACEIs & ARBs; monitor serum potassium & renal function if used
concomitantly. Increases phenytoin levels; increased myelosuppression with methotrexate.
Increase INR with warfarin, hypoglycemia with sulfonylureas, increased nephrotoxicity
with cyclosporine.

257. Bevacizumab side effect?


a) VTE
Venous thrombo Traditional Routine prophylaxis with ASA, Q. Vitamin K antagonists e.g warfarin
embolic events cytotoxic warfarin or low molecular are used with caution as cancer patients
(deep vein drugs, weight heparin may be required are susceptible to wide fluctuations in
thrombosis, Bevacizumab, for patients receiving regimens INR.
pulmonary Tamoxifen, containing dexamethasone and Q. Low molecular weight heparins
embolism) Sunitinib, thalidomide or its analogues. preferred for long-term anticoagulation.
(within weeks to Thalidomide &
months) Analogues
258. Which is wrong regarding ISMP
a) HCTZ 12.5mg PO QD
b) Digoxin 0.625mg PO daily
c) Levothyroxin 25mcg PO Daily

259. Patient is diabetic taking a metformin 500 bid and has heart failure, what is wrong
medication?
a) Linagliptin
b) Acarbose
c) Rosiglitazone
Contraindicated in patients with NYHA Class I, II, III and IV heart failure. Patients with severe heart failure
(including NYHA Class III and IV cardiac status) were not studied during the clinical trials. Thiazolidinediones,
like AVANDIA, alone or in combination with other antidiabetic agents, can cause fluid retention, which can
exacerbate or lead to congestive heart failure. The fluid retention may very rarely present as rapid and
excessive weight gain. All patients should be monitored for signs and symptoms of adverse reactions
relating to fluid retention and heart failure, including excessive, rapid weight gain; dyspnea; and/or edema.
In particular, patients who are at risk for heart failure including those receiving concurrent therapy which
increases insulin levels (i.e., sulfonylureas) should be closely monitored
AUGUST 2020 – 2 VERSIONS

1. Type 2 Diabetic patient using many mediaction, has a hypoglycemia at the morning (after
wake up), what medication could be the reason?
a) Metformin
b) Exenatide
c) Pioglitazone
d) Repaglinide
Repaglinide is capable of inducing hypoglycemia. Proper patient selection, dosage, and instructions to the
patient are important to avoid hypoglycemic episodes. Hepatic insufficiency may cause elevated repaglinide
levels in the blood and may also diminish gluconeogenic capacity, both of which increase the risk of serious
hypoglycemic reactions.
Elderly, debilitated or malnourished patients, and those with adrenal, pituitary or hepatic insufficiency are
particularly susceptible to the hypoglycemic action of glucose‐lowering drugs. Hypoglycemia may be difficult
to recognize in the elderly, and in people who are taking beta‐adrenergic blocking drugs.
Hypoglycemia is more likely to occur when caloric intake is deficient or when meals are skipped. Given the
preprandial dosing regimen, patients taking Repaglinide can adjust dosing according to their changing meal
patterns, thereby reducing the risk of hypoglycemia when meals are missed.
Hypoglycemia is also more likely to occur after strenuous or prolonged exercise, when alcohol is ingested, or
when more than one glucose‐lowering drug is used.
Hypoglycemia in pioglitazone monotherapy is rare. The risk of hypoglycemia is increased
when pioglitazone is used in combination with insulin or a sulfonylurea.

2. Diabetes DM1 case very long. Diabetes patient 37 yrs & pregnant. With retinopathy,
nephropathy & protienurea, she took basal reading 3 times before & basal at bed time & PBG.
Morning before mean 7 - 7.5 After 2 hrs 7 - 7.2
Lunch before 7 - 7.5 After 2 hrs 3.5 - 4 Bedtime 10 - 8
She takes Glargine & glulisine with each meal. How to regulate her regimen?
a) Reduce her glargine dose
b) Reduce her glulisine dose
c) Increase her glargine dose
d) Increase her glulisine dose
We should correlate hypoglycemia first, so decrease before lunch glulisine

3. What should be her A1C target?


a) Less than 7.5 because of her nephropathy
b) Less than 7.5 because of her pregnancy
c) More than 7 because of hypoglycemia
d) Less than 7 because of retinopathy
Diabetic retinopathy is the most common cause of new‐onset blindness in the working‐age population.
Optimal glycemic control has been shown to reduce the development and progression of retinopathy in
patients with type 1 and type 2 diabetes.
The target blood pressure for people with diabetes is less than 130/80 mm Hg.

4. After a period of time he developed retinopathy, what to add for protection?


a) Statins
b) ACEIs
c) Gabapentin
Retinopathy: Refers to retinal vascular disease, or damage to the retina caused by abnormal blood flow.
Risk factors:
 longer duration of diabetes, elevated HbA1c, increased blood pressure, dyslipidemia, anemia, pregnancy
(in type 1 diabetes), proteinuria & severe retinopathy itself.
Investigations:
 Screen/evaluate for retinopathy annually beginning 5 years after the onset of diabetes in all patients with
type 1 diabetes (at or after age 15).
 Screen for retinopathy in all patients, including children and adolescents, at the time of diagnosis of type 2
diabetes. Follow-up every 1–2 years unless severity dictates more frequent evaluation tailored to the
severity of the retinopathy.
 Evaluate women with diabetes during pregnancy planning, in the 1st trimester, throughout the pregnancy
as needed and within the first year postpartum.
Nonpharmacologic Choices:
 Patients with moderate visual loss should have spectacle correction, vision aids and other measures to
promote independence and quality of life.
 May be treated with retinal photocoagulation (laser therapy) and vitreoretinal surgery
Pharmacologic Choices:
 Fenofibrate, in addition to statin therapy, may be used in patients with type 2 diabetes to slow the
progression of retinopathy.
 Anti-VEGF (ranibizumab, aflibercept and off-label use of bevacizumab) are first line in management of
centre involving diabetic macular edema. Local (intraocular) therapy with anti-VEGF shown to both
improve vision and reduce macular edema.
 Focal macular laser only preserves existing vision. Used when central vision is not involved.
 Steroids are alternatives in the management of diabetic macular edema, including intraocular injections of
triamcinolone, dexamethasone and fluocinolone. The disadvantage of intraocular steroids is ↑ rates of
glaucoma & cataract formation.

5. 67-year-old male smoker, has Cancer and taking cisplatin & dexamethasone. He has bad
history of montion sickness, he came complaining for severe N&V. What is the risk for N&V?
a) Cisplatin therapy
b) Sleep depriviation
c) Age
d) Smoking
e) History of motion sickness
f) Gender
Potential Intravenous Chemotherapy Agents Oral Chemotherapy Agents
High Carmustine, Cisplatin, Mechlorethamine, Hexamethylmelamine, Procarbazine
(>90%) Streptozocin, Cyclophosphamide (>1500 mg/m2),
CTC: Although medication is the most likely cause of nausea and vomiting in a patient receiving cancer
treatment rule out other potential causes (e.g., fluid/electrolyte abnormalities, bowel obstruction, CNS or
hepatic metastases, infections, radiation therapy). Other drugs (e.g., opioids, digoxin, antibiotics) may cause
or exacerbate nausea and vomiting; therefore, a thorough medication history is essential.
Some chemotherapeutic agents are more likely to cause nausea
and vomiting than others. It is important to consider the
emetogenic potential, the dose of the chemotherapy agents and
the expected pattern of emesis of the chemotherapy regimen
when choosing antiemetics.
Patient‐specific factors such as <50 years of age, female gender,
and a history of motion sickness, nausea in pregnancy, or
chemotherapy‐induced nausea and vomiting (CINV) may raise the
risk of experiencing nausea and vomiting; therefore, antiemetic
regimens must be tailored to the individual patient.

6. His wife knew that, if she learned how to give her husband the drug, they will no need to go
to the clinic and he can take the drugs at home. What the advice you provide her about how to
handle these medications?
a) Wash hands before and after touching both drugs
b) Wear gloves & mask before cisplatin, wash hands before & after dexamethasone
c) Wear gloves for both medications
d) No special handling is required

7. Same Pt, the pt given ondansetron and aprepitant 125 mg in Day 1 and 80 mg on Day 2 and
day 3, he also took dexamethasone, he was also on chlorpromazine but he suffered from
vomiting in Day 2 and Day 3. What to do for the next cycle? No nabilone in the options
a) Increase dose of aprepitant on day 2 and three
b) Increase dose of chlorpromazine
c) Add olanzapine from the 1st day
d) Change ondansetron to granisetron
Olanzapine is a 2nd generation antipsychotic agent that antagonizes several neurotransmitters receptors
involved in the emetic pathway, including dopamine, 5‐HT, histamine and acetylcholine muscarine.
Regimens containing olanzapine have demonstrated efficacy in preventing acute and delayed nausea and
vomiting resulting from highly emetogenic (HEC) and moderately emetogenic chemotherapy (MEC), and
olanzapine monotherapy has proven effective in the treatment of breakthrough nausea and vomiting.
Expert groups support the use of olanzapine as part of 2‐drug to 4‐drug regimens for the prevention of CINV
and as monotherapy for the treatment of breakthrough nausea and vomiting.
8. 4 hours before the next dose, patient has nausea. What do you give before the next dose to
prevent NV?
a) Chlorpromazine
b) Metoclopramide
c) Increase dexamethasone dose
d) Lorazepam
Lorazepam and alprazolam are the most commonly used, and have been studied in cases of anticipatory
nausea. They are usually used in combination with other antiemetics.

9. What is the goal of therapy of the above patient?


a) Decrease nausea and decrease vomiting
b) Decrease nausea and stop vomiting
c) Stop nausea and decrease vomiting
d) Stop nausea and vomiting
Goals of Therapy
 Prevent or minimize acute (starting within 24 hours of chemotherapy), delayed (starting >24 hours
after chemotherapy) and anticipatory (starting before chemotherapy as a conditioned response)
nausea and vomiting to maintain quality of life, help patient adherence with active treatment and
avoid treatment delays
 Decrease incidence & severity of nausea & vomiting (once it has occurred) & maintain patient comfort
 Prevent complications, such as esophageal tears, dehydration, anorexia, malnutrition, weight loss,
pathological bone fractures, metabolic alkalosis, chloride and potassium depletion
10. After taking steroid dexamethasone for 1 week, what to expect as SE?
a) Hyperglycemia
b) Osteoporosis
c) Glucoma
d) Acne

Aprepitant and fosaprepitant may increase serum concentration of dexamethasone, so, decrease dose of
dexamethasone (usually reduced from 20 mg to 8–12 mg).

11. LP is experiencing purulent discharge from the nose, congestion, his temperature is normal.
The symptoms have been affecting his sleep daily for the last 3 weeks. Patient profile shows
that LP was taking amoxicillin and clarithromycin 3 weeks ago for H. Pylori. The physician
diagnosed LP with rhinosinusitis. What is the type of rhinosinusitis do the patient have?
a) Moderate Intermittent
b) Moderate Persistent
c) Severe Intermittent
d) Severe Persistent

12. The doctor would like to prescribe antibiotic for his Rhinosinusitis. What is the best
recommendation for this patient?
a) Amoxicillin
b) Cephalexin
c) Azithromycin
d) Levofloxacin
e) Clindamycin
CTC: If a patient has received antibiotics (especially macrolides or fluoroquinolones) within the past 3
months, choose a different antibiotic/antibiotic class to treat acute bacterial rhinosinusitis as there is a
higher risk of multidrug‐resistant S. pneumoniae
Antibiotics Not Routinely Recommended as Empiric Therapy for Acute Bacterial Rhinosinusitis
Cephalexin No activity against penicillin intermediate/resistant Streptococcus pneumoniae.
No activity against Haemophilus influenzae or Moraxella catarrhalis
Cefixime No activity against penicillin intermediate/resistant S. pneumoniae.
Excellent activity against H. influenzae and M. catarrhalis.
May be an option but only in combination with clindamycin
Ceftriaxone Routine use of this agent is not recommended due to potential for increased resistance to third-
generation cephalosporins. May be an option in patients with severe acute bacterial
rhinosinusitis who have failed therapy. Three days of IM/IV therapy is recommended, as a
single dose is not as effective in eradicating penicillin-resistant S. pneumoniae.
Clindamycin No activity against H. influenzae or M. catarrhalis. Reasonable activity against S. pneumoniae.
Can be used in acute rhinosinusitis but only in combination with cefixime.
Can be used as an alternative to Amoxicillin + Clavulanate in chronic cases.
Ciprofloxacin Suboptimal coverage of S. pneumoniae
Macrolides Poor activity against H. influenzae.
Significant macrolide resistance in S. pneumoniae and Streptococcus pyogenes.
SMX/TMP Poor activity against S. pneumoniae and H. influenzae. No activity against S. pyogenes.

Another version: Sinusitis case with 2 weeks symptoms + penicillin allergy (she has anaphylactic
allergy with cefazolin), took azithromycin 1-month ago, what to give:
a) Clarithromycin (not used)
b) Clindamycin (not used)
c) Doxycycline 200 mg then 100 mg bid
d) Moxifloxacin (very last resort)
Recommended Empiric Therapy for Bacterial Rhinosinusitis in children and adults
Children Adult
Clinical Acute Rhinosinusitis (symptoms < 4 wk and ≤ 3 episodes/y)
Presentation Refer to ENT specialist if ≥4 episodes/y
Usual Pathogens Strep. Pneumoniae, M. catarrhalis, H. influenzae, Occasionally Staph. aureus, S.
pyogenes, anaerobes
Recommended First-line: First-line:
Empiric Therapy Standard-dose amoxicillin × 10 days or Amoxicillin × 5–7 days
high-dose amoxicillin × 10 days
Penicillin allergy: B-lactam allergy:
≤8 y: clindamycin + cefixime × 10 days Doxycycline 5–7 days
>8 y: doxycycline × 10 days
Severe infection or immunocompromised: Severe infection or immunocompromised:
amoxicillin/clavulanate (7:1) + amoxicillin/clavulanate (7:1) ± amoxicillin
amoxicillin × 10 days × 5–7 days
Nonsevere beta-lactam allergy: Nonsevere beta-lactam allergy:
ceftriaxone × 10 days Ceftriaxone × 5–7 days
Severe beta-lactam allergy / anaphylaxis: Severe b-lactam allergy/anaphylaxis:
levofloxacin × 10 days levofloxacin × 5 days
Failure of First Line Amoxicillin/clavulanate (7:1) ± Amoxicillin/clavulanate ± amoxicillin × 5–
Agents: amoxicillin × 10 days 10 days
(No improvement after Nonsevere beta-lactam allergy: Beta-lactam allergy:
7 days of antibiotic clindamycin + cefixime × 10 days or Levofloxacin or moxifloxacin × 5-10 days
therapy or clinical ceftriaxone × 10 days
deterioration or Severe beta-lactam allergy/anaphylaxis:
recurrence within 3 m)
levofloxacin × 10 days
Comments Consider resistant organisms, especially penicillin-resistant S. pneumoniae and
ampicillin-resistant H. influenzae

13. What question you will ask about to differentiate between sinusitis and allergic rhinitis
a) Facial pain & Fever
b) Cough
c) Headache
d) Congestion
e) Interfere with sleeping

14. What is the appropriate action?


a) Refer her because 2 weeks symptom
The preferred initial strategy is watchful waiting without initiation of antibiotic therapy and with the use of
analgesics/antipyretics and decongestants when needed.
Approximately 70% of cases of acute rhinosinusitis will resolve within 2 weeks without antibiotic treatment;
however, if symptoms continue for longer than 7 days after diagnosis or worsen at any time, antibiotic
therapy should be considered.
Therapy should be anywhere between 7‐14 days. For chronic sinusitis, up to 3 weeks is required.
Short courses in several antibiotics shown efficacy (azithro 1‐3 days, trimox 3days, amoxiclav 5 days).
A study comparing 3‐7 vs 6 ‐ 10 days courses found no differences, but with fewer ADRs in shorter case
In adults with uncomplicated acute rhinosinusitis, a treatment duration of 5–7 days is recommended.
For children, data on shorter durations of therapy are lacking; so, it is recommended to treat for 10 days.
Improvement may take up to 7 days, complete resolution of symptoms may take up to 14 days.

15. Same patient complains about


nocturnal cough, she is using ACEI
and PPI for GERD, cause of her
cough?
a) Post nasal drip
b) GERD
c) ACE side effect
d) Reactive airways

16. AW 55-year-old female came complaining of dyspnea and excessive sputum production and
cough. She suffered from the symptoms daily for 3 months, her BMI 24.5 (they give you the
weight and height to calculate the BMI), waist circumference 105 cm, sedentary lifestyle and
history of smoking, patient profile shows she is on ventolin QID prn and tiotropium once daily.
Patient confirmed she has not been to the hospital before, how to know its not controlled or How
you assess the case that this asthma is uncontrolled?
a) Wake up at night or nocturnal sympt >2 / week
b) Ask pt how many days absent form work/school last yr because of asthma
c) Can’t perform his usual excersise
d) How many times did you have exacerbation last year?
e) Ask of symptoms of dyspnea sputum production and temperature
f) How do you take ventolin and tiotropium?
17. They asked about the staging, which stage of asthma is she are?
a) Mild persistent
b) Mild intermitant
c) Moderate persistent
d) Severe persistent
18. The same patient on SABA PRN and ICS, dr said he wheeze and Rxed him cough syrup,
what is next?
a) Increase dose of ICS
b) Pt should not take it
c) Add LABA to ICS
d) Should start oral cortisone

19. He went to emergency room ER


with asthma, what to give
a) Glycopyrenium
b) IV Corticosteroids
c) SABA
d) IV Formetrol.
Systemic corticosteroids should be
administered to the patient within 1 hour
of presentation.
20. International recall 3 for a specific medication (ranitidine) because the lot number and
excipients weren’t written on the Packs but medication is good. What should be done?
a) Publish this information in the pharmacy front, so all the staff know about that.
b) Tell the staff if any of the Pt ask about that recall, refer them to the pharmacy manager to
address all their questions
c) Send memo to all drs & Look for alternative
d) Call pt to return it back
e) Call manufacturer to know the reason, but you still can use it in pharmacy
f) Check DIN if you have the recalled medication & return what you have on hand,
but don’t call the Pt who got the medication because the recall is not that dangerous

21. UTI case, Pregnant woman patient came with


several symptoms to the pharmacist. What should the
pharmacist ask to rule out pyelonephritis infection?
a) Flank Pain
b) Hematuria
c) Suprapubic pain
d) Frequency
e) Costovertebral pain
f) Dysuria
Suprapubic pain happens in your lower abdomen near where your hips and many important organs, such as
your intestines, bladder, and genitals, are located.
Flank pain refers to discomfort in your upper abdomen or back and sides. It develops in the area below the
ribs and above the pelvis. Usually, the pain is worse on one side of your body.
Costovertebral pain often mimics cardiopulmonary‐related pain. Common causes are acceleration or
deceleration injuries, blunt trauma to the chest or spine, and overuse or misuse. Subluxation or dislocation
of the joint can occur in severe trauma. The joint is affected most notably in ankylosing spondylitis

22. Lab culture shows Gram negative (lactose fermenting: positive)


a) Klebsiella Pneumonia
b) Pseudomonas Aeruginosa
c) Proteus mirabilis
Klebsiella pneumoniae is a Gram‐negative, non‐motile, encapsulated, lactose‐fermenting, facultative
anaerobic, rod‐shaped bacterium. It appears as a mucoid lactose fermenter on MacConkey agar.
E. coli is a Gram‐negative, facultative anaerobic, rod‐shaped, lactose‐fermenting, coliform bacterium of
the genus Escherichia that is commonly found in the lower intestine of warm‐blooded organisms
(endotherms)
P. aeruginosa has few nutritional requirements and can adapt to conditions not tolerated by other
organisms. It does not ferment lactose or other carbohydrates but oxidizes glucose and xylose.
23. The doctor prescribed Moxifloxacin. Why this prescription is inappropriate?
a) Moxifloxacin has a risk of teratogenicity
b) Moxifloxacin is not right for the indication
Systemic fluoroquinolones are generally not recommended for use in pregnancy due to reports of cartilage
damage in juvenile animals. Fluoroquinolones have not been proven to increase the risk of malformations in
the developing fetus; however, it is generally recommended that these agents not be used during
pregnancy, especially during the 1st trimester, as safer alternatives are usually available. Ophthalmic
products are considered acceptable due to low systemic bioavailability.
Mild to E. coli (90%), P. mirabilis & K. pneumoniae (5%) Fluoroquinolone Amoxi/clav PO or
moderate Occurs in women who experience recurrent (ciprofloxacin, SMX/TMP PO or
pyelonephritis uncomplicated UTIs but at lower frequency than levofloxacin, Trimethoprim PO
OR Acute cystitis. norfloxacin), all for 10-14 days
Nonobstructive Classic presentation includes fever, N&V, flank PO × 7–14 days
Pyelonephritis pain with or without associated irritative urinary
Severe symptoms. Aminoglycoside Fluoroquinolone IV
pyelonephritis Patients who present with UTI with only lower IV ± ampicillin IV × 10–14 days or
urinary tract symptoms or asymptomatic for initial therapy; 3rd generation
bacteriuria occasionally have associated occult if appropriate, step cephalosporin IV ±
renal infection. down to oral aminoglycoside IV
Bacteremic infection occurs most frequently in therapy as in mild × 10–14 days or
diabetic women or women >65 y. to moderate Carbapenem IV ×
Urine Culture: always recommended. infections in order 7–14 days for
Consider blood cultures. to complete 10–14 ESBL-producing
days organisms

24. What is the appropriate antibiotic to take


a) Ciprofloxacin
b) Ceftriaxone
c) Amoxycillin
d) Gentamycin
Management of Treat asymptomatic bacteriuria & symptomatic cystitis with 3 - 7 days course of amoxicillin
UTI during (if the organism is known to be susceptible), amoxicillin/ clavulanate, cephalexin, fosfomycin
Pregnancy tromethamine or nitrofurantoin with appropriate follow-up.
Nitrofurantoin is usually avoided near term because of the risk of inducing hemolytic anemia in
the fetus or newborn, especially in those with G6PD deficiency; however, this toxicity is rare.
Fosfomycin tromethamine may also be used in pregnancy when the organism is susceptible.
Ceftriaxone is the preferred empiric therapy for treating pyelonephritis in pregnancy. Step
down to oral therapy once the patient is stabilized and urine culture results are available.
Avoid trimethoprim and SMX/TMP in the 1st trimester of pregnancy.
Avoid SMX in the last 6 weeks of pregnancy & Avoid fluoroquinolones in pregnancy.
25. Which of the following diseases you can step down from IV therapy to oral after 2 days:
a) Meningitis
b) Osteomyelitis
c) Pyelonephritis
d) Pneumonia
e) Candidemia
f) Febrile neutropenia pt
g) Cancer pt
https://www.ncbi.nlm.nih.gov/pmc/articles/PMC4008927/
There are mainly three types of IV to PO conversions.
 Sequential therapy: It refers to the act of replacing a parenteral version of a medication with its oral
counterpart of the same compound. For instance, conversion of inj. pantoprazole 40 mg OD (once
daily) to tab. pantoprazole 40 mg OD
 Switch therapy: It describes the conversion of an IV medication to a PO equivalent; within the same
class and has the same level of potency, but of a different compound. For example, switch over from
inj. ceftriaxone 1 g BD (bis in die) to tab. cefixime 200 mg BD, switch over from inj. pantoprazole 40
mg BD to tab. rabeprazole 20 mg BD
 Step down therapy: It refers to the conversion from an injectable medication to an oral agent in
another class or to a different medication within the same class where the frequency, dose, and the
spectrum of activity (in the case of antibiotics) may not be exactly the same. For example, conversion
of inj. cefotaxim 1 g to tab. ciprofloxacin 500 mg, switch over from inj. heparin to tab. warfarin.

26. KS is a 6-year-old girl who comes to the pharmacy with her mother. Her main complaint is
having honey crusts on her lips spreading around her nose & chin. She also developed a rash
that is covered by honey-color crusted drainage on her left dorsal hand flexures and shoulder for
the last 4 days. Based on these, the physician diagnosed her condition as non-bollus impetigo.
He then recommended skin hygiene by cleansing normal saline or soap/water or antiseptic agent
twice daily to decrease number of pathogenic bacteria on the skin. On top of this, doctor plans
to prescribe antibiotics. What is the first therapy for infection?
a) Cephalexin po 7-10 days
b) Mupirocin for 7-10 days apply 3 times daily
c) Metronidazole for 3 days
d) Clindamycin 300mg TID for 7-10 days
Nonbullous impetigo typically manifests as clusters of vesicles or pustules that rupture and develop a honey‐
colored crust (exudate from the lesion base) over the lesions.
Bullous impetigo is similar except that vesicles typically enlarge rapidly to form bullae. The bullae burst and
expose larger bases, which become covered with honey‐colored varnish or crust
27. One-week later, KS came back with no improvement, what is the most appropriate action?
a) Swap to exudate and test for bacterial infection
b) Swap to exudate and test for HSV viral infection
c) Swap to exudate and test for fungal infection
d) Continue mupirocin
e) Drainage
Diagnosis of impetigo and ecthyma is by characteristic appearance. Cultures of lesions are indicated only
when the patient does not respond to empiric therapy. Patients with recurrent impetigo should have nasal
culture. Persistent infections should be cultured to identify MRSA
Alternative diagnostic possibilities are key to consider in recurrent cases or those that do not respond
quickly to treatment. Tinea is a very common and often missed mimic. Careful and full skin examination is
required to rule out tinea as a cause of impetigo‐like infection of the skin. Unlike impetigo, tinea almost
never occurs on the upper lip.
Herpetic impetigo also is a very common and often missed mimic. Viral culture is recommended in atopic
patients, who are the high‐risk group for this presentation. In particular, those patients with extensive
eczema are most at risk and should be considered for this testing.

28. Then the dr Rx cephalexin 500mg QID for 14 days what is the problem?
a) Wrong medicine for the disease
b) Might with long duration
29. A family of 35-year-old man diabetic, wife and 2 years old child are traveling to a country
with known malaria in Africa. They came to the pahrmacy asking for prophylaxis, what is the
organism we are protecting from?
a) Plasmodium species
b) Trepenoma pallidum
c) Borrelia burgdorferi
d) Klebsiella Pneumonia

30. What is correct about malaria prophylaxis?


a) Malaria prophylaxis is only recommended to immunocompromised patient
b) Prophylaxis should begin 1-2 weeks before travel, during trip & 4-6 weeks after trip
c) Repellent is effective, repellent like > 10% DEET and Acarbidin
d) Apply DEET once only from dusk to dawn
e) Apply a combination of sunscreen and DEET
Another version
Patient is traveling to east africa for hiking in woods. What would be the most important non
pharmacological advice?
a) Insect propellants will protect against stinging and biting insects
b) Add drops of alcohol to your drinks in order to sterilize them
c) Do not eat fruits and do not drink sealed carbonated beverages
d) Do not travel to high risk areas unless for an emergency.
propellants will protect against biting insects not stinging insects
Ingredient Comments
Q. Use before outdoor activity during the main hours of malarial transmission.
DEET Vapour thought to have offensive smell or taste to insects and repels mosquitoes.
(N, N-diethyl- Effective against mosquitoes, black flies, ticks, chiggers and fleas.
m- Duration of effect depends on concentration: 30% DEET = 5 – 8 h, 20% DEET = 4 – 6.5 h,
toluamide) 15% DEET = 3.5– 5.5 h, 10% DEET = 2.5– 4.5 h, 5% DEET = 1.5–2.5 h.
Available as Concentration 5– 30% Apply sparingly to clothing or exposed skin.
lotion, pump According to Health Canada guidelines: < 6 months: Not recommended
and aerosol If travelling to area with high risk of arthropod associated disease, up to 10% may be applied
sprays, to children < 6 months. The American Academy of Pediatrics now supports the use of DEET
towelette 30% in children as young as 2 months of age.
6 months – 2 y: ≤10%; maximum once daily.
>2–12 y: ≤10%; maximum TID. >12 y: ≤ 30%; maximum TID.
Children <12 y, do not use on daily basis for >1-month duration.
DEET has been rarely associated with neurologic side effects in children exposed to high
concentrations (>35%) and prolonged use.
S.E: Irritating to mucous membranes and open wounds. Contact dermatitis (rash, redness,
itching) has been reported. Absorbed through intact skin, and systemic effects are related to the
amount absorbed. If ingested, may lead to seizures, hypotension, angioedema or death.
Concurrent application of sunscreen may lower SPF but efficacy of DEET appears to be
maintained. Sunscreen should be applied first & allowed to penetrate before DEET application.
If application of DEET is delayed after the sunscreen has been applied, the sunscreen product
maintains more of its original SPF.
No evidence that the use of DEET by pregnant or breastfeeding women poses a health hazard to
unborn babies or breastfed infants/children. Avoid excessive or prolonged use.
Icaridin Affect the insect's ability to detect the host by concealing attractants emitted by hosts or by
(also known as changing the insect's ability to smell them. Effective against mosquitoes, ticks and black flies.
picaridin) Picaridin (icaridin) 20% is comparable to DEET across the majority of mosquito vectors, but
Available as lasts for 8 hours and has greater acceptability due to less odour and greasiness.
spray, aerosol Duration of effect: 10% = 5 h for mosquitoes and 7 h for ticks. 20% = 7 h for mosquitoes and 8
or towelette h for ticks and black flies.
All ages ≥ 6 months: Up to 20%. 10 – 20% Apply directly to skin, avoid eye contact.
Reapply 10% after 5 h, up to QID. Reapply 20% after 7 h, up to BID
< 6 months: Not recommended. If travelling to area with high risk of arthropodassociated
disease, up to 10% may be applied to children < 6 months.
Low toxicity. Nonirritating to skin, but should be kept out of eyes and mouth.
No evidence that the use of icaridin by pregnant or breastfeeding women poses a health hazard
to unborn babies or breastfed infants/children. No allergic reactions reported.
P-menthane Effective against mosquitoes, biting flies and gnats (not effective against ticks).
3,8-diol (PMD) Effective for <2 hours at concentrations of 10%. Duration of effect is about 2 h for mosquitoes
synthetic and 5 h for black flies.
derivative of PMD is not recommended for children < 3 years of age and should be considered a second-
oil of lemon line option when DEET or picaridin cannot be used. 10 % ≥ 3 y: Maximum BID.
eucalyptus No evidence that the use of PMD by pregnant or breastfeeding women poses a health hazard to
lotion unborn babies or breastfed infants/children
Other options  Use bed nets, preferably impregnated with permethrin.
 Use mosquito coils, aerosolized insecticides or electrically operated insecticide
generators containing pyrethroids.
 Wear clothes covering exposed skin, weather permitting.
 Permethrin-impregnated clothing adds an additional measure of protection.
 Sleep in an air-conditioned or screened room if possible.

31. Same case. They want a chemoprophylaxis for malaria, but the father has depression &
Seizure. What agent to avoid?
a) Atovaquine
b) Chloroquine
c) Mefloquine
d) Primaquine
Mefloquine Used to prevent malaria in travellers going to areas reporting chloroquine-resistant P. falciparum
Administer with a meal and with at least 240 mL of water
Start at least 1 wk prior to exposure and continue weekly for 4 wk after leaving the endemic area
Adults: 250 mg base (1 tablet) once/wk PO
If it is not possible to initiate mefloquine 1 wk prior to exposure, a loading dose can be given to
rapidly achieve the effective levels:
Loading dose, >45 kg: 250 mg base daily PO for 3 days, then 250 mg base weekly thereafter
S.E: Common: dizziness, nausea, vomiting, diarrhea, headaches, sinus bradycardia, nightmares,
insomnia, mood alteration, anxiety, irritability, hair loss, skin rash.
Rare: permanent dizziness, vertigo, tinnitus, and loss of balance; seizures; psychosis; thrombotic
thrombocytopenia purpura.
Contraindicated in patients with a history of seizures, depression, generalized anxiety disorder,
psychosis, schizophrenia or other psychiatric disorders, self‐endangering behaviour, suicide
attempts or suicidal ideations.
Contraindicated in patients who are hypersensitive to mefloquine or structurally related
compounds, e.g., chloroquine, quinidine, quinine.
Loading dose is associated with an increased risk of depression.
When possible, mefloquine may be started 4 wk prior to departure to allow time to assess for
adverse effects prior to travel since 70% of severe adverse reactions occur within first 3 doses.
Drug of choice for pregnant women who require prophylaxis in chloroquine resistant area.

32. They want dukoral vaccine to prevent traveller diarrhea, what is right councelling?
a) Do not eat 1 hour before and after taking medication
b) Not effective to prevent cholera, don’t take it
c) Only for the husband
d) To be given to all family member including infants
e) 3 doses protocol before 2 weeks of travelling and between the doses at least 2 weeks
f) Take 2 doses. Second dose one day before travel
Vaccines Contains the nontoxic B subunit of cholera toxin, which has significant homology with the
Vibrio cholerae toxin of ETEC and is approved in Canada for prevention of TD caused by ETEC.
whole cell/ Prevention of enterotoxigenic E. coli diarrhea:
recombinant Adults and children ≥2 y: Primary immunization: 2 doses PO; 2nd dose administered within
cholera toxin B 7–42 days after the 1st dose and at least 1 wk before reaching destination
subunit vaccine Booster: 1 dose every 3 months if the risk is continuous
Q. Dukoral Q. Taken orally on an empty stomach (1 h before or 1 h after eating or drinking)
S.E: Abdominal pain, diarrhea, nausea and vomiting.
May consider for prevention of TD in persons with chronic illnesses (e.g., HF, insulin-
dependent DM, IBD, chronic kidney disease), or in those with immune suppression

33. Dukoral Vaccine exposed to room temperature for 3 days. what would be your initial action?
a) Call the manufacture for advice on what to do?
b) Return back to fridge until expiry and call manufacture for credit loss
c) Discard of the vaccine as it is no longer effective
d) Just return it to the fridge
e) Ask who make this and take disciplinary action with him
Store in the refrigerator at 2° to 8°C (35° to 46°F). Do not freeze. The vaccine can be stored at room
temperature (up to 25°C) for up to two weeks on one occasion only. After mixing with the buffer solution
the vaccine should be consumed within 2 hours.

34. Hospital pharmacy have more morphine dose errors. How to avoid?
a) Alarm on system
b) Labeling
c) Meeting & creating Policy
d) Separation
e) Double check
https://www.ismp.org/resources/high-alert-medication-feature-reducing-patient-harm-opiates

35. A pharmacy manager did a physcial count of morphine, he found 30 tabs more in inventory.
What can be the cause for this?
a) You entered Rx on system but didnt count them
b) You dispensed Oxycodone instead
c) Error due to proceeding pescription on system
d) You prepared 30 tabs but patient didn’t take it & you forget to cancel on the system.
Narcotic Reconciliation
Most shortages or overages can be accounted for; however, when large amounts of a drug are missing, the
owner or DM may want to notify the police and if the pharmacy belongs to a chain, the head office should
be contacted as per pharmacy policy.
Large quantities of missing narcotics are usually associated with an internal theft or diversion problem.
Similarly, overages may be an indication of poor record keeping processes or false billings. Where large
quantities are involved, the DM or owner should begin by reviewing all systems and procedures including
ordering, receiving, storage, and final dispensing of the drug.

36. Patient came to the pharmacy with a Rx of Morphine IR. What is the main concern of the
pharmacist? What opioid RX you suspect more?
a) Rx includes other non opioid drugs
b) Patient address in the same region of the pharmacy
c) pt comes early in the day at weekday (not weekend)
d) Patient coming once pharmacy open
e) pt has profile at the pharmacy
f) Rx date not consistent with urgency of the medication
IR means we need it for it’s rapid and immediate pain‐relieving effect, so if patient came after several days
from the date it was written, I will suspect
37. Best reference for multiple IV drugs compounding or Which of the following references can
be used for IV incompatibility??
a) Micromedex
b) AHFS
c) Martindale
d) CPS
Micromedex Alternative to CPS: Drug monographs, Drug identification, interactions, IV compatibility,
American Calculations, Patient education, off label uses & Toxicology
Micromedex, pre-2019 CPS and LexiComp have drug identification tools.

38. In a hospital patient has received 1 gm Vancomycin, after 2 hr the blood conc. was 35
m.mol/L, after 72 hr the conc. was 17 m.mol/L & he has received a second dose of 1 gm
Vancomycin. Peak conc. is 20 m.mol/L, trough 15 m.mol/L. When should he take the third dose
after the 2nd dose?
a) 2 days
b) 3 days
c) 5 days
Answer:
Log C = log Co ‐ k*t /2.303 k = (2.303 / t) * log (Co / C) k = (2.303/70) * log (35/17) = 0.010 hr‐1
Log C = log Co ‐ k*t /2.303 Log 35 = log Co ‐ (0.010*2) / 2.303 log Co = 1.544 + 0.008 = 1.552
Co = 35.7 m.mol/L So, upon the second dose: Co = 35.7 + 17 (remains from the 1st dose) = 52.7 m.mol /L
C = 15 m.mol / L. (trough conc. at which the 3rd dose has to be taken)
k = 0.010 hr ‐1 Log C = log Co ‐ k*t /2.303
t = (2.303 / k) * log (Co / C) t = (2.303/0.010) * log (52.7/15) =125.679 hr = 5.23 days
So, the 3 rd dose has to be taken 5 days after the 2nd dose

39. A patient is receiving vancomycin. His drug levels and known pharmacokinetic parameters
are provided. Vancomycin ke = 0.17 hr-1. Vancomycin trough = 32 mcg/mL
How many hours will it take for his vancomycin trough to reach 20 mcg/mL?
Answer:
Log C = log C0 – kt/2.303 Log 20 = log 32 ‐ 0.17t/2.303 T = 2.765 hours.

40. Calculation about Vancomycin is given in a dose 500 mg q12h, after 2hrs of infusion (@11
o'clock) the conc. was 30 mg, after 7 hrs (@18 o'clock) the conc was 7 mg, what is the T ½?
a) 3.5 hour
b) 1.7 hour
Answer:
C1 = 30 mg C2 = 7 mg T1 = 11 T2 = 18 Difference in infusion time = 18 – 11 = 7 hours
Log C= log C0 ‐ kt/2.303 Log 7 = log 30 – k (18‐11) /2.303
K= 0.2079 T1/2 = 0.693/0.2079 = 3.45 hr
41. Vancomycin 1.5 g is infused over 2 hours. Steady state plasma concentration After 2 hours
was 30mg/L. Serum level after 8 hours was 17.1mg/L. Calculate Vd?
Answer:
K = In Cp1 – In Cp2 / T1 – T2
K = In 30 – In 17 / 6hr = 3.40 ‐ 2.83 / 6 = 0.57 / 6 = 0.095hr
K = In Cp1 – In Cp2 / T1 – T2
0.095hr = In Cp1 – In 30 / 2
In Cp1 = 2 * 0.095hr + 3.40 = In 3.59 = 36.2
Volume of distribution = total dose / initial conc of drug
Vd = 1500mg / 36.2 = 41.4 L

42. Patients receive vancomycin 1 gm q12 hours gives steady state conc of 1 mg/dl. Physicians
want to increase the concentration to 1.5 mg/dl. What should be the dose of the drug?
a) 500 mg q 12 hour
b) 750 mg q 8 hour
c) 1 gm q 8 hours
d) 1.5 gm q 12 hours
e) 2 gm once daily
1 gm /12 hrs ‐‐‐‐‐‐ 1mg/dl X ‐‐‐‐ 1.5 mg /dl

43. Vancomycin calculation, conc was 17 mg/ml when t1 (given). And conc was 28 when t2
(given also) and want the trough between 15-20 mg/ml, when should we give next dose??
I calculated the t 1/2, it was 14hrs
Answer:
log 17 = log 28 – k (t1 ‐ t2) / 2.303 Then calculate k Then, T1/2 = 0.693/k
We will give the second dose when we reach the trough value
Put the conc desired 15 mg /ml and calculate the time according to given data in the problem.

44. 469. Female patient is 29 years old, 100 lb,


has serum creatinine 135%, calculate ClCr?
wt = 100/2.2 = 45.45kg
CrCl = (140 – 29) * 45.45 / 72*135% = 5044.95 / 97.2 =
51.903 * .085 = 44.118 = 0.44ml/min
45. A physician would like to switch this patient to fentanyl. Currently, the patient is on oral
Hydromorph Contin 9 mg 8AM, 9 mg 2PM, 10 mg 8PM with hydromorphone 3 mg q4h prn
(use average 6 mg a day). What should be the strength of fentanyl?
http://nationalpaincentre.mcmaster.ca/opioid/cgop_b_app_b08.html
I think the table in the above link was given (not supposed to memorize these values)
Answer:
patient daily consumption of hydromorphone Cr formula = 9+9+10=28 mg IR Formula = 6 mg
Total = 28+6= 34 mg equivalent to (34* 5) = 170 mg morphine
Then decrease the dose (30 to 50 %) to allow for incomplete cross tolerance
It will be 85 mg (if 50 % dose reduction)
From table 85 mg morphine = 25 mcg / hr (fentanyl patches). As 60‐134 mg morphine = 25 mcg /hr fentanyl

46. Pt ask about an evidence based natural medicine for his BPH. Which ref.
a) RxTx
b) Pubmed
c) Natural medicine Database
Licensed herbal product database and wrrite prostate in somewhere he explains how to make search)

47. Pt ask about if she can use grapefruit seeds for her daughter disease (can’t remember the
disease), which ref.
a) CTMA
b) Licensed Natural Medicine Database

48. Test A has less (false positive/total


positive) than test B. or Drug X false
positive is less in test A than in tets B
(Ref.) that means?
a) Test A has more specificity
b) Test A has less specificity
c) Prevalency is less in A than B
d) Prevalency is more in A than B
Specificity = true ‐ve / (true +ve + true ‐ve)

49. Delegation to technician. Which can technician do freely or Technician can’t do all except?
a) Counselling on non medicated drugs
b) PBMH
c) Taking Verbal narcotic prescription
d) Advise OTC, dextromethorphan
e) Counselling on schedule 2 medication
f) Counselling on inhaler device
g) Doing independent final technical check after the filling
h) Medication reconciliation
a) Methadone ingestion monitoring

50. Cholinesterase inhibitors used for?


a) Parkinsonism
b) Left bundle block
c) Urinary retention
Doctors prescribe cholinesterase inhibitors to individuals with dementia caused by Alzheimer's disease and
dementia. Doctors also prescribe them to individuals to treat Lewy Body dementia, Parkinson's disease
dementia, glaucoma, myasthenia gravis, and schizophrenia.
Dementia is also common in patients with PD. Cholinesterase inhibitors (e.g., donepezil, rivastigmine) have a
modest impact on improving dementia, but careful observation for deterioration in motor function is required

51. 45 years old teacher with parkinsonism, she was embarrassed as she has poor hand writing
& falls many times and they gave the symptoms (it seems to be severe). Now, she is working
part time instead of full time. What is your advice for this patient?
a) Start treatment to avoid delaying of
disease progression
b) Do not start treatment because of the
side effects of the medication
c) Start the treatment as quality of life
is affected
d) Change her work

52. What is most appropriate


treatment or what do you to alleviate
his symptoms?
a) Pramipexole
b) Selegiline
c) Amantadine
d) Entacapone
53. Parkinson patient taking
levodopa and carbidopa IR in
the morning and SR at night,
he developed high muscular
rigidity & dyskinesia, how to
avoid wearing off?
a) Shift from IR to CR
b) Add entacapone
c) Add Amantadine
d) Decrease 25% of dose
e) Shift both to SR
If both wearing off & dyskinesia,
go with amantadine
If wearing off only B & E are right

DVT cases 7 qs
54. An Obese, elderly Patient had femur fracture surgery, after
stay in hospital for femur surgery, he developed DVT (lots of
distractors). What risk factor that caused DVT?
a) Immobilization
b) Obesity
c) Age
d) HTN
e) Dyslipidemia

55. In hospital, he is on LMWH enoxaparin, the Pt will be


discharged, what should be the next step?
a) Continue LMWH for at least 5 days
with warfarin, then stop LMWH
b) Continue LMWH
c) Heparin
d) Stop enoxaparin & give dabigatran

56. How long will you treat?


a) 1 month
b) 3 months
c) 6 months
d) 1 year
57. RW 64-year-old male is taking warfarin 2 mg daily for
stroke prevention. He has a mechanical valve. He is planning
to get his wisdom tooth extracted in 15 days. What is your
advice?
a) No need to stop warfarin and give him mouth wash
tranexamic acid
b) Stop warfarin 1 day before the appointment
c) Stop warfarin for 3 days before the appointment
d) Stop warfarin for 5 days before the appointment
e) On the same day of surgery

58. He has a problem in frequent monitoring of INR and asks for probability to change it to
another anticoagulant. If doctor wants to switch between warfarin and rivaroxaban, stop
warfarin and start rivaroxaban, what is the appropriate action to take?
a) Stop warfarin and start Rivaroxaban when INR 1
b) Stop warfarin and start Rivaroxaban when INR < 2.5
c) Stop warfarin and start Rivaroxaban after 12 hours
d) Stop warfarin and after 5 days start Rivaroxaban
e) Stop INR monitoring while keeping the patient on warfarin
Transitioning from warfarin to rivaroxaban:
 Discontinue warfarin and initiate rivaroxaban as soon as INR falls to <3 (US) or ≤ 2.5 (Canadian).
Transitioning from warfarin to apixaban:
 Discontinue warfarin and initiate apixaban as soon as INR falls to <2 (US labeling).
Transitioning from warfarin to dabigatran:
 Discontinue warfarin and initiate dabigatran as soon as INR falls to <2 (US labeling).
Transitioning from warfarin to edoxaban:
 Discontinue warfarin and initiate edoxaban as soon as INR falls to ≤2.5 (US labeling).
Transitioning from warfarin to parenteral anticoagulation: Stop warfarin and start the parenteral
anticoagulant when INR is as close as possible to the lower end of the targeted INR range.

58. Rivaroxaban 10 mg presciption came to pharmacy, could be for which case or in which of
the following you can use rivaroxaban 10 mg daily?
a) Valvular atrial fibrillation
b) DVT after Hip replacement surgery
c) DVT after femur surgery
d) Loading dose… (cannot remember)
59. Which of the following statements is correct regarding DVT. Patient is taking rivaroxaban
20 mg once daily:
a) Take with food
b) Cause constipation
c) Tablets can be Chewed or swallowed (crushed and mixed with applesauce)
d) Avoid antacids (no relation mentioned)
e) Rivaroxaban can be taken if CrCl is less than 30
f) Rivaroxaban should be started a high dose in the first 3 weeks
g) Rivaroxaban should not be taken in liver dysfunction
h) Rivaroxaban is not indicated for initial treatment for DVT
Administer doses ≥15 mg with food; doses of 2.5 mg and 10 mg may be administered without regard to meals.
For nonvalvular atrial fibrillation, administer with the evening meal.
For patients who cannot swallow whole tablets, the tablets may be crushed and mixed with applesauce
immediately prior to use; immediately follow administration of the 15 mg and 20 mg tablets with food (2.5
mg and 10 mg tablets may be administered without regard to food).
Missed doses:
Patients receiving 15 mg twice daily dosing who miss a dose should take a dose immediately to ensure 30
mg of rivaroxaban is administered per day (two 15 mg tablets may be taken together); resume therapy the
following day as previously taken. Patients receiving 2.5 mg twice daily who miss a dose should take a
single 2.5 mg dose at the next scheduled time; then resume therapy as usual. Patients receiving once‐daily
dosing who miss a dose should take a dose as soon as possible on the same day; resume therapy the
following day as previously taken.
SIDE EFFECTS
As XARELTO acts on the blood clotting system, most side effects are related to signs of bruising or bleeding.
In some cases, bleeding may not be obvious, such as unexplained swelling.
Patients treated with XARELTO may also experience the following side effects: nausea, vomiting, stomach
ache, constipation, diarrhea, indigestion, and decreased general strength and energy.
Monitoring and Laboratory Tests
Measuring PT using Neoplastin reagent, or Factor‐Xa assay using rivaroxaban ‐specific calibrators and
controls, may be useful to inform clinical decisions in these circumstances.
Although XARELTO therapy will lead to an elevated INR, depending on the timing of the measurement, the
INR is not a valid measure to assess the anticoagulant activity of XARELTO.
At recommended doses, XARELTO affects the measurement of the aPTT and Heptest. These tests are not
recommended for the assessment of the pharmacodynamic effects of XARELTO.
Contraindications
 Clinically significant active bleeding, including gastrointestinal bleeding
 conditions at increased risk of clinically significant bleeding, eg, recent cerebral infarction
(hemorrhagic or ischemic), active peptic ulcer disease with recent bleeding.
 Concomitant systemic treatment with strong inhibitors of both CYP 3A4 and P‐glycoprotein (P‐gp),
such as ketoconazole, itraconazole, posaconazole, or ritonavir.
 Concomitant treatment with any other anticoagulant, including
o unfractionated heparin (UFH), except at doses used to maintain a patent central venous or
arterial catheter, low molecular weight heparins (LMWH), such as enoxaparin and dalteparin.
o heparin derivatives, such as fondaparinux, and
o oral anticoagulants, such as warfarin, dabigatran, apixaban, edoxaban, except under
circumstances of switching therapy to or from XARELTO.
 Hepatic disease (including Child‐Pugh Class B and C) associated with coagulopathy, and having
clinically relevant bleeding risk.
 Pregnancy, Nursing women & Hypersensitivity to XARELTO or to any ingredient in the formulation.

60. A mother went to a hiking trip with her two sons, the youngest is 9 months. She is asking
about insect repellent icaridin, what is true about it?
a) Protect against ticks, but not mosquitoes
b) Icaridin 20% for age < 6 months to 12 years
c) Icaridin 10% for age < 6 months to 12 years
Icaridin Affect the insect's ability to detect the host by concealing attractants emitted by hosts or by
(also known changing the insect's ability to smell them. Effective against mosquitoes, ticks and black flies.
as picaridin) Duration of effect: 10% = 5 h for mosquitoes and 7 h for ticks.
20% = 7 h for mosquitoes and 8 h for ticks and black flies.
Available as spray, aerosol or towelette. No allergic reactions reported.
10 – 20% Apply directly to skin, avoid eye contact.
< 6 months: Not recommended. All ages ≥ 6 months: Up to 20%.
Reapply 10% after 5 h, up to QID. Reapply 20% after 7 h, up to BID
Low toxicity. Nonirritating to skin, but should be kept out of eyes and mouth.
If travelling to area with high risk of arthropodassociated disease, up to 10% may be
applied to children < 6 months.
No evidence that the use of icaridin by pregnant or breastfeeding women poses a health hazard
to unborn babies or breastfed infants/children

61. The mom also asked about sunscreen for her family.
a) Advice children to play outside if it is cloudy
b) Use light clothes
c) Give all sunscreen except 9
months child
d) Give all of them sunscreen
SPF 45 to apply it 10
minutes before exposure
e) Give them sunscreen SPF
30 to be applied 30
minutes before exposure.
62. one of her chilren got lost in the woods and when she found him, there was a tick attached
to his skin. After 24 hours, he developed rash and erythema on legs, she came to pharmacy
asking which problem her son has?
a) Lyme disease
b) Poison ivy (contact dermatitis)
c) Atopic dermatitis
d) Eczema
a) Allergy to icaridin
Allergic In ACD, the primary symptom is intense pruritus; pain is usually the result of excoriation or
contact infection. Skin changes range from transient erythema through vesiculation to severe swelling
with bullae, ulceration, or both. Changes often occur in a pattern, distribution, or combination
dermatitis
that suggests a specific exposure, such as linear streaking on an arm or leg (eg, due to
brushing against poison ivy) or circumferential erythema (under a wristwatch or waistband).
Linear streaks are almost always indicative of an external allergen or irritant.
Any surface may be involved, but hands are the most common surface due to handling and
touching potential allergens. With airborne exposure (eg, perfume aerosols), areas not
covered by clothing are predominantly affected. The dermatitis is typically limited to the site
of contact but may later spread due to scratching and autoeczematization (id reaction). In
systemically induced ACD, skin changes may be distributed over the entire body.
The eruption usually begins within 24 to 48 hours after exposure to the allergen.
Atopic Age < 6 m - 2 y: chest, face, neck, diaper area: Cycles of itching and scratching. Red, raised
dermatitis blisters with oozing, dry skin
Age >4 - 10 y: scattered - neck, wrist, elbow, knee: Less acute & oozing; dry papules,
thickened, periorbital edema and erythema
Adolescents and adults: flexor areas, hands: Dry, thickened, hyperpigmented plaques
Triggers
Extreme heat or cold, rapid temperature changes, sweating, irritant or occlusive clothing
(wool or nylon), soaps and detergents, greases, infections, environmental allergens,
Physiologic and psychosocial stress. Anxiety,
Genetically associated, and higher risk of inheritance from mother than father.
Increased in black and mixed-race compared with Caucasians. More common in those of
higher socio- economic status, children from small families & who live-in privately-owned
properties
Lyme Early localized Lyme disease:
disease This stage is characterized by an acute illness that manifests on
average 7–10 days following the tick bite. The most common
manifestation is the localized erythema migrans (EM) skin
lesion.
It typically begins as a macule or papule at the site of the tick
bite then subsequently enlarges to reach a threshold diameter of at least 5 cm.
Q. At least two-thirds of solitary EM lesions are initially erythematous plaques, with or
without an enhanced central erythema, but many possible variations have been described
including a “bull’s eye” appearance, blistering or necrotic areas, and blue or purple hues.
63. What is your advice about
treatment?
a) Topical corticosteroids
b) Mupirocin cream
c) Zno 15% cream
d) Refer to assess

64. A mother went to a hiking trip with her two sons, the youngest is 9 months, they took
sunscreen. Her children have atopic dermatitis. After returning from trip with few hour’s, they
developed skin rash, the mother found insect crawling on her 9 months bayby’s clothes & she
took it before it inserts her mouth piece into her son’s leg. she is worried that the rash is because
of lyme disease, what is the best response you will give as she is concerned about lyme disease?
a) Don’t go to forest next time to avoid ticks
b) Don’t worry as the ticks should be conducted on the skin with 72 hours
c) Don’t worry lyme disease is preventable as one dose of antibiotic can do the job
d) I understand your concern, the tick needs to be in the skin for 36 hrs, it is not lyme
disease as you took the tic out before the bite
e) It is exacerbation of eczema as you used chemical sunscreen
f) The symptoms are not for lyme disease but for poison ivy
g) You can make a Lyme titer test to make sure

65. After the mother removed the black insect; she suspected lyme and went to the pharmacy
because of the resulting eczema. What is your advice about treatment?
e) Topical corticosteroids
f) Mupirocin cream
g) Zno 15% cream
h) Refer to assess

66. Same case: patient wants to read more about ticks, where to tell her about where she can
find information?
a) Public health authority
67. What is the drug of choice for Zollinger Ellison syndrome?
a) Proton pump inhibitors
b) H2 receptor antagonists
A gastrinoma is a gastrin‐producing tumor usually located in the pancreas or the duodenal wall. Gastric acid
hypersecretion and aggressive, refractory peptic ulceration result (Zollinger‐Ellison syndrome).
Diagnosis is by measuring serum gastrin levels. Treatment is proton pump inhibitors and surgical removal.

68. He took Omeprazole 40mg BID but no improvement, what is the appropriate action?
a) Change to Pantoprazole 80 mg BID
b) Add Sucralfate PO
c) Add Octreotide S.C
Proton pump inhibitors are the drugs of choice (eg, omeprazole or esomeprazole 40 mg orally 2 times a
day). The dose may be decreased gradually once symptoms resolve and acid output declines.
A maintenance dose is needed; patients need to take these drugs indefinitely unless they undergo surgery.
Octreotide injections, 100 to 500 mcg subcutaneously twice a day to 3 times a day, may also decrease
gastric acid production and may be palliative in patients not responding well to proton pump inhibitors. A
long‐acting form of octreotide (20 to 30 mg IM once a month) can be used.
Another version
Zollinger Ellison syndrome. Patient was on PPI 40 mg daily; he is not controlled. What is your
recommendation:
a) Change to esomeprazole
b) Use IV PPI
c) Increase dose of Omeprazole to 80 mg daily
d) Add octreotide
Another version
Zollinger Elison syndrome pt, he took PPI and he is better now and SXs controlled, what is
your advice?
a) D/C PPI
b) Stay on it indefinitely

69. Baby with diaper rash with red tomatoe shaped, and painful (when touching, the baby
becomes irritated), what should you counsel his mom:
a) Frequent washing with soap and water
b) Shift milk formula to another one
c) Expose as possible as to dry air between diaper change
d) Use Talc powder
70. What should we give?
a) Clotrimazole 1%
b) Hydrocortisone 0.5%
c) Zn oxide 15%
d) Topical nystatin
e) Zn oxide 40%
They supposed to describe the rash more
to decide the specific treatment
See the algorithm & know ho to treat each
type of rash.

71. Ph manager want to use automatic


software in the pharmacy. What steps
in ph that this software will not help?
a) In Dispensing
b) When ph want to see DDI
(appropraiet of the RX)
c) When preparing the rx
Software has nothing to do with preparing
Rxs.

HIV case (5 questions):


72. Male patient 34 years old, have 300 sexual
contacts (males and females) last year. He treated
before from gonorrhea and chlamydia. Living
with a roommate who has HIV, slept with many
girls without condom. What is the highest risk
factor to catch HIV?
a) Multiple sex partners
b) His roommate
c) Not wearing condoms
d) His past diagnosis of gonorrhea

73. He wants to use pre-exposure prophylaxis.


a) Emtricitabine / Tenofovir
b) Emtricitabine/ Tenofovir AF / Raltegravir
c) Emtricitabine/ Tenofovir AF / Elvitegravir / Cobicistat.
74. When he should start taking anti HIV medication dose?
a) As soon as possible
b) Within 72 hours
c) Before HIV exposure and continuing daily after

75. How frequently you monitor him?


a) Once monthly
b) 3 times monthly
c) Every 3 months
d) Every 6 months
e) Every One year
Patients who initiate PrEP should be assessed for efficacy and safety at 1 month, 3 months and then every 3 months
thereafter. HIV testing (in addition to testing for other STIs) must be performed every 3 months.

76. For how long it is the regimen?


a) Every day for 4 weeks
b) 3 days / week
c) For 6 months
d) Till he is no longer at high risk
Discontinuing PrEP
 It is not clear how long a person needs to keep taking medication before it can be safely stopped. HIV
protection decreases over 7–10 days following discontinuation of PrEP.
 Guidelines suggest MSM with a long duration of PrEP adherence may require only 2 additional days of
therapy following their last exposure. However, in the absence of better evidence, individuals with other risks
(e.g., PWID, women) may take PrEP for up to 28 days, based on extrapolation of data from PEP.
 The optimal timing for HIV testing following discontinuation of PrEP is unknown. Since PrEP may delay the
detection of HIV antibodies, it is suggested that HIV testing at 8 weeks is reasonable.

77. Patient diagnosed with Hep. B acute virus infection, what to councel?
a) Stop alcohol
b) Avoid Marijuana
c) Take multivitamins
d) Give IG to her son 6yrs
plus vaccination
e) Interferon B
78. Randomised Clinical trials RCTs should be reported before it starts, researchers do this
reporting to?
a) Reduce publication bias
b) Reduce attribution bias
c) Make sure it does not complete a previous one
Publication bias Occurs in published academic research. It occurs when the outcome of an experiment or research
study influences the decision whether to publish or otherwise distribute it.

Another version: When performing systematic review. In order to minimize the bias, which of
the following is correct:
a) Intention to treat.
b) Randomized Controlled trial.

79. A study comparing two drugs differences with 0.17 CI from -2.5. to 1.02
a) Study is non inferior
b) Study shows equivalence
c) Study is not non inferior, show superior results
Another version:
Question about non inferiority study question between a new drug and brand name. P value was
given: results are significant but the brand is still superior to the new drug
When you are checking difference between drugs check for 0 if it exists in the confidence interval then no
difference. If they mentioned HR, RR or OR check for 1 if its there no difference also

80. Another question about type of bias Test only used in ICU to detect pneumonia, dr want to
use it for all patients in all wards in hospital what type of Bias
a) Performance bias

81. Asymmetric funnel plot indicates relation between ttt efficacy and study precision?
a) Publication bias
82. GK is a 13-week pregnant female 35-year-old. CrCl 28 ml/min, suffering from dysuria,
hematuria, frequency, suprapubic discomfort and urgency with sulfa allergy & hyperkalemia.
She is diagnosed with cystitisWhat is the appropriate treatment for her?
a) Amoxicillin
b) Nitrofurantoin
c) SMX/TMP
d) Fosfomycin
Nitrofurantoin: Not recommended for treatment of pyelonephritis and contraindicated if ClCr < 30mL/min.
SMX/TMP: contraindicated in sulpha allergy.
Before the administration of Fosfomycin to patients with severe renal impairment (creatinine clearance <30
mL/min) or patients undergoing hemodialysis, the physician should evaluate if the potential benefits of the
drug outweigh the potential risks to the patient, as Fosfomycin is principally excreted by the kidney

83. Related to the previous case, what is true?


a) No culture retest is required after eradication
b) Follow up culture test should be performed twice during pregnancy after infection
eradication
c) Follow up culture test should be performed one to two weeks after antibiotic
completion and monthly until delivery
d) Culture test should be performed before treatment and a follow up culture is required
after treatment and before birth.
CTC: Screen pregnant women for asymptomatic bacteriuria early in pregnancy (wk 12–16) and treat if
asymptomatic bacteriuria is confirmed on 2 consecutive cultures. If left untreated, bacteriuria in pregnancy
can progress to pyelonephritis and cause adverse effects to the fetus.
Antibiotic treatment has been shown to reduce the incidence of pyelonephritis, rate of preterm delivery and
risk of low birth‐weight babies. Upon completion of therapy, follow‐up with a urine culture 1–2 weeks later,
and then monthly until the baby is born.

84. What could be the causative agent for complicated UTI. Culture indicates lactose
fermenting gram negative bacilli
a) E. Coli
b) P. mirabilis
c) P. aeruginosa
d) S. Pneumonia
85. Priority to dispense, Nurse bring 4 prescriptions stat, which one you prepare first?
a) Alteplase IV for stroke (another version is tenecteplase)
b) Zolindronic acid for the malignant hypercalcemia
c) Clopidogrel loading dose for STEMI
d) Methotrexate oral post PI
e) Clarithromycin for cap
f) IV levothyroxin
g) Bisphosphonate (pamidronate) for hyperparathyroid
h) Ciprofloxacin for neutropenic cancer pt
Loading dose of ASA in STEMI also is correct as STAT. Others are ASAP
‫ﻃ ﺐ ﻫﻮ ﻏﺎﻟ ﺎ ﺣ ﺠ ﺐ ﻟﻚ ار ــﻊ اﺧﺘ ﺎرات وﺣ ﻄﻠﺐ ﻣﻨﻚ اﻻوﻟ ﺎت‬
‫ﻻن ﻣﺶ ﻣﻤﻜﻦ ﺠ ﺐ ﻞ اﻻﺧﺘ ﺎرات دي ﻣﻊ ﻌﺾ‬
‫ اﻫﻢ ﺣﺎﺟﻪ اﻟﺤﺎﺟﺎت اﻟ ﻟﻴﻬﺎ‬time frame ‫ﺗﻤﺸﻴﻬﺎ اﻻول‬
Alteplase should be given within 4,5 hours from the start of stroke
Then clopidogrel loading dose for STEMI as this patient is waiting for PCI (primary should be done within 90
min)
Then give all priorities for all Intravenous medications as these patients generally in ICU (icu patients are
most important than other patients who can wait)
Then give a priority for unit dose patients (normal ward patients) like their medications of antibiotics or
chronic drugs
Finally, chronic stable patient either single doses or weekly like zaldronic acid annual injections or
biphosphonate monthly doses least important
Take care of ward STAT require always highest priority
86. MF is 50 years old pt with chronic kidney disease, DVT history and also type 2 diabetes,
patient measurements: A1C 7.8, LDL 1.9, Ca 2.7 mmol/L (normal range 2.2 - 2.7 mmol/L),
Phosphate: 6 mg/dL (normal range: 2.5 - 4.5 mg/dL), GFR= 60, Crcl: 54 ml/min.
Drugs: Insulin NPH BID 24 Unit, Lispro 12.15.13 Unit before the main meals, Metformin 500
TID. What is the most appropriate recommendation for MF to decrease cardiovascular events?
a) Rosuvastatin 10 mg
b) Empagliflozin
c) Dapagliflozin
d) Aspirin
e) Linagliptin
f) ACEIs

CV Protection To promote optimal cardiovascular health, the treatment of all patients with diabetes should
in People with include: Optimal blood glucose control based on individualized target, Optimal blood
Diabetes pressure of less than 130/80 mm Hg, Optimal lipid control, Healthy body weight, Regular
physical activity Smoking cessation & Healthy diet.
Statin therapy is recommended if: ≥ 40 years of age, clinical cardiovascular disease,
microvascular complications, diabetes present for >15 years and age >30 years, presence of
other risk factors (male, smoker, family history of premature cardiovascular disease [CVD],
ECG abnormalities, ACR >2 mg/mmol).
For those not at LDL-C goal despite statin therapy, a combination of a statin with a second-
line therapy (ezetimib or the PCSK9 inhibitor evolocumab) may be used.
ACE inhibitor or ARB is recommended if: ≥55 years of age with additional cardiovascular
risk factor or end-organ damage (albuminuria, retinopathy, left ventricular hypertrophy),
clinical cardiovascular disease & microvascular complications.
Low-dose ASA (81–162 mg) once daily should be used for secondary prevention of CVD in
those who already have established CVD to prevent cardiovascular events.
ASA should not be routinely used for primary prevention of CVD in people with diabetes, but
may be used in those with additional cardiovascular risk factors.
Clopidogrel 75 mg once daily may be used in those unable to tolerate ASA.
Antihyperglycemic agent with demonstrated cardiovascular outcome benefit (empagliflozin,
canagliflozin, liraglutide) should be added to reduce the risk of major cardiovascular events.
The use of semaglutide has also resulted in a reduction of major adverse CV events.

87. According to the previous profile, what should be added to MF regimen:


a) Ca Carbonate
b) Sevalamer
c) Na bicarbonate
d) Na polystyrene sulfonate
Sevelamer is used to lower high blood phosphorus (phosphate) levels in patients who are on dialysis due to
severe kidney disease.

88. Now, she complains about biletral edema in her legs, what could be the reason?
a) Dapagliflozin
b) Empagliflozin
c) Rosiglitazone
d) Linagliptin
Individual TZDs may differ in their effects on serum lipids. Both agents (Pioglitazone, rosiglitazone) are
associated with weight gain due to increased subcutaneous fat deposition, fluid retention and edema, which
is likely the cause of the increased incidence of heart failure in patients receiving TZDs.
Other reported adverse effects include worsening
macular edema and an increased risk of fractures,
particularly of the hip and wrist.

89. What is his “CKD” BP target?


a) <130/80
b) <140/90
c) <135/85
90. Inventory management (numbers provided is for explanation only)
You a pharmacist in community pharmacy, you are doing regular check for your narcotic stock
and you found the following?
Oxycodone Morphine Hydoromorphone

Previous stock 1420 X2 X3

Purchase 320 Y2 Y3

Sales 1393 Z2 Z3

Damages/Lost 1 tablet crushed none None

Physical Count 344 A2 A3

Which of the following match the results you found?


a) 2 tablet shortage of oxycodone, and nothing in the others
b) 2 tablet shortage of oxycodone, 3 overage of HM and 2 shortage of morphine
c) 2 tablet overage of oxycodone, 3 shortage of HM and 2 overages of morphine
d) 2 tablet shortage of oxycodone, 3 shortage of HM and 2 shortage of morphine
Answer
Observe given numbers in the above table and detect where is the shortage
Starting stock + purchase =
Ending stock + sales + damage/lost =
Compare between both of them

91. Opoid conversion case: pt was on oxycodone IR 30mg bid and 5mg prn Q4-6h. patient use
4 doses for breakthrough pain per day. Pain is not controlled and dr want to change to CR
therapy. They gave a table of morphine equivalent for conversion. Pt had no history of drug
abuse, alcohol consumption no tolerance. What is the best CR therapy the dr will give? (you
will be provided with the equivalence, no need to memorize)
a) Morphine 5mg/ml, 5 ml Q4h
b) Morphine 5 mg/ml, 5 ml Q6h
c) Hydromorphone 12mg bid and 1 mg PRN Q6h
d) Hydromorphone 6mg bid and 5mg PRN
I am not sure about the new therapy name if it was hydromorphone or something else. I went as
pt has not problem tolerance so I decrease the dose by 50% and gave 10% of total daily dose for
breakthrough pain
92. 22 yr female with multiple partners and takes depot prog for 89 days; she came after 6 days
of unprotected sex, asking for emergency contraceptive. She uses phenytoin as anti-seizure.
What do you give as emergency contraceptive?
a) Plan B
b) Ulipristal
c) IUD-copper.
d) Take DMP again
Ulipristal acetate 30 mg was approved in Canada for prevention of pregnancy when taken within 120 hours
(5 days) of unprotected intercourse or known or suspected contraceptive failure.
The copper IUD is the most effective method of EC available. Postcoital insertion of a copper IUD can be
considered up to 7 days after unprotected intercourse. Prior to insertion, it is important to exclude pre‐
existing pregnancy.
A single dose of levonorgestrel 1.5 mg used within 24 hours of unprotected intercourse prevents 95% of
expected pregnancies. Efficacy is highest if treatment is provided within 24 hours; it can be taken up to 5
days after unprotected intercourse, though the effectiveness declines with increasing delay between
unprotected intercourse and treatment initiation.

93. Pt has focal seizure (partial), she has HLA-


B positive. What should be given?
a) Carbamezapine
b) Phenobarbital
c) Valproic acid
d) Levetiracetam
e) Phenytoin
Seizure Type First Choice Alternative Mono-
Monotherapy therapy or Add-on
Focal (partial) or unclassified tonic-clonic seizures CarLa LevOx Brivaracetam
The electrical disturbance is limited to a Carbamazepine Clobazam
specific area of one cerebral hemisphere Lamotrigine Eslicarbazepine
(side of the brain). Levetiracetam Gabapentin
Without impairment of awareness (simple Oxcarbazepine Lacosamide
partial seizures): N.B; Perampanel
motor, sensory, autonomic or cognitive lamotrigine is Phenobarbital
features, usually brief (<60 second) better tolerated Phenytoin
With impairment of awareness (complex partial seizures): than Primidone
characterized by behavioural arrest, often manifested as a blank carbamazepine Topiramate
stare, usual duration of 1–2 minutes, frequently accompanied by and similarly Valproic acid /
motor automatisms, e.g., lip smacking and chewing movements, effective in Divalproex
brief postictal confusion is common, can begin without impairment obtaining Vigabatrin
of awareness before progressing or can have impairment of long-term
awareness at onset, commonly misdiagnosed as absence seizures seizure freedom
94. What should we monitor for Levetiracetam?
a) Blurred vision
b) Depression
c) Weight loss
Monitoring Parameters: CNS depression (impaired coordination, ataxia, abnormal gait, weakness, fatigue,
dizziness, and somnolence); psychiatric and behavioral symptoms (aggression, agitation, anger, anxiety,
apathy, confusion, depersonalization, depression, emotional lability, hostility, hyperkinesias, irritability,
nervousness, neurosis, suicidal thoughts and personality disorder); diastolic blood pressure in children 1
month to <4 years; CBC (in patients who experience significant weakness, pyrexia, recurrent infections or
coagulation disorders

95. Calculation: 3 products give clearance and Kel. Which with high Vd.
Answer
For any drug use this formula CLt = Vd * k el

97. Gives 3 clinical trials with different P values, asking about which is significant results. Hint
the smaller P values, the more significant the results
The table last column p value
1 mortality - 2 morbidity - 3 hypo K - 4 parathyroid - 5 hypo P - 6 Osteopo
P value for each
0.11, 0.005, 0.054, 0.02, 0.001
The q w is sig? Compare the P value in the specific events choose the right that match ..
U need to cal each drug risk reduction to decide

98. Statistics P value. It was less than 0.05 what does it mean?
Another one: Statistics case about which test is used to measure distance walked by a heart
failure patient when comparing two treatments.

99. 72 yrs old patient has controlled hypertension, CKD with crcl 28.5, dyslipidemia, MI,
intermittent claudication, what is his CHADS2 Score?
a) 1
b) 4
c) 5
d) 2
100. Doctor wants to start treatment for
stroke protection, what is the best
choice?
a) Warfarin
b) Aspirin
c) Rivaroxaban
d) Clopidogrel
Warfarin  better than riva because
of low CrCl

101. Pt diabetic with renal dysfunction.


Which medication should stop?
a) Metformin
b) Insulin
c) Canagliflozin
d) Liraglutide
102. What could be side effect of Invokana (Canagliflozin)?
a) Urine Infection
b) Hypertension
c) Hypokalemia
d) Pancreatitis
Sodium- Canagliflozin 100–300 mg once daily PO Less effective in moderate and
Glucose Co- Dapagliflozin 5–10 mg once daily PO ineffective in severe renal impairment.
transporter Empagliflozin 10–25 mg once daily PO S.E: Increased risk of genitourinary
2 (SGLT2) Ertugliflozin 5–15 mg once daily PO infections; reduced intravascular volume
Inhibitors Take on Empty stomach in the morning resulting in hypotension &
They↓ glucose reabsorption, ↑ urinary glucose excretion, hyperkalemia.
↓ blood glucose. SGLT2 also decreases reabsorption of D.I: Loop diuretics increase risk of
sodium and causes osmotic diuresis. hypotension

103. A female patient with history of DM, hypothyroidism on Levothyroxin 125 mcg and
hypertension on Enalapril [BP 135/87], wants to get pregnant. She also on atorvastatin. Her doctor
told her to stop ACE once she is pregnant, what other medication she should stop too?
a) Enalapril
b) Atorvastatin
c) Metformin
d) Levothyroxin
Statins are contraindicated in pregnancy because cholesterol synthesis is thought to be essential for normal
fetal development. Women of childbearing age using a statin should be informed of the potential risks.
Management of dyslipidemia is a chronic process, such that discontinuation of the statin during pregnancy
poses no immediate risk to the mother. Dietary measures are considered appropriate to manage
dyslipidemia during pregnancy.

104. What can you do about the BP medications?


a) Switch to Nifedipine XL
b) Switch to irbesartan
c) No need for medication and monitor BP
First line ‐ Strong safety record: Methyldopa, Labetalol, Oxprenolol, acebutolol, Pindolol, Propranolol,
Metoprolol, Nifedipine XL, Clonidine, Hydralazine

105. What is your recommendation regarding Levothyroxine dose?


a) Decrease dose once she becomes pregnant
b) Lots of variation so should get TSH every 6-8 weeks
c) Should increase her dose twice a week as soon as she finds out she is pregnant (or
increase by 50%)
d) Test T4 levels
Women with known hypothyroidism who are taking thyroid hormone replacement therapy should be
advised to increase their thyroid hormone dose by 2 extra tablets per week immediately following positive
pregnancy test.
Thyroid‐binding globulins increase during pregnancy so requirements for L‐T4 replacement may increase by
up to 50% during pregnancy to maintain TSH between 2.5 mU/L and the lower limit of the normal range.
Throughout pregnancy, women on thyroid hormone replacement should check TSH Q4 wk in 1st trimester,
Q6 wk until ~32 wk gestation and 4–6 wk after any dosage adjustment.

106. Now, she came with Rx 0.5 mg folic acid & ask you if she needs higher dose of folic?
What concentration of Folic acid should she take?
a) Yes, as pregnant should take 1 mg
b) Yes, as type 2 DM should take 1 mg
c) Yes, as hypothyroidism pt should take 1 mg
d) Yes, as HTN should take 1 mg
Diabetes and Pregnancy
Both type 1 and type 2 diabetes mellitus can occur in women of childbearing age, while gestational diabetes
mellitus (GDM) is a condition that develops during pregnancy.
Pre-existing diabetes (type 1 or 2) increases the risk of miscarriage, perinatal mortality, fetal macrosomia, and
congenital malformations (due to hyperglycemia in the first weeks of gestation), while GDM increases the risk
of fetal hyperinsulinemia, heavier birth weight, higher rates of cesarean deliveries and neonatal hypoglycemia.
In the months preceding conception, women should follow these risk-reduction activities:
 Begin folic acid supplementation at least 3 months prior to conception. Initial dose 5 mg daily. After 3
months' gestation, the dose is reduced to 0.4–1 mg daily and continued throughout the pregnancy and for
a minimum of 6 months postpartum.
 Undergo eye exam because pregnancy can accelerate retinopathy resulting from poor glycemic control.
 Switch to insulin regimen from non-insulin antihyperglycemics and aim to achieve HbA1c level of <7%.
 Undergo screening for cardiovascular disease and chronic kidney disease (CKD).
 Discontinue any teratogenic medications such as ACE inhibitors, ARBs or statins.

107. DM case; type 2 for 10 yrs, his reading good, what he should do?
a) Self monitoring decrease complication
b) Self monitoring increase management of diabetes
c) Should increase his monitoirng
d) No need as his HbA1c is good
If he is not using insulin and his A1C is ok. So, no need for self monitor. Just regular A1c check.
For people with type 2 diabetes treated with healthy behaviour interventions, with or without noninsulin
antihyperglycemic agents, the effectiveness and frequency of monitoring BG in improving glycemic control is
less clear. A series of recent meta‐analyses, all using different methodologies and inclusion criteria, have
generally shown a small benefit to reducing A 1 C in those individuals performing SMBG compared to those
who did not. SMBG has been demonstrated to be most effective in persons with type 2 diabetes within the
first 6 months after diagnosis. Also, of significance, there is no evidence that SMBG affects one's
satisfaction, general well‐being or general health‐related quality of life.
108. KM aged 65 years old patient with diabetic type II & HF. He used to feel tired, pain and
dyspnea after walking 9 blocks, now he feels tired after 2 blocks on ground level, Waist
circumference 105cm with a history of smoking a pack 20-years, but stopped 7 years ago, His
father died of CVS diseases aged 51. Drinks 2 coffees per day. Patient profile shows: Ramipril
5 mg BID Atenolol 50 mg. What the modifiable risk factors for this patient:
a) Smoking
b) Waist circumference
c) Family history
d) Caffeine
Waist Circumference is a constant measure of abdominal obesity. It has recently been shown that mesenteric
adipose tissue inflammation is more related to metabolic consequences of obesity. It has been proven through
following up the evolution of MS components is that the intraabdominal fat mass predisposes to development
of HPT. Interestingly, this was independent of BMI and occurred even in individuals with BMI < 25 kg/m2.
The WHO stated that WC > 94 cm in men and > 80 cm in women is associated with increased risk of metabolic
complications and the risk is significantly increased with a WC > 102 cm in men and > 88 cm in women.
CTC: Additional risk factors include low HDL‐C, impaired fasting glucose, increased waist circumference,
cigarette smoking and hypertension.

109. According to NYHA classification


where can u categorize this patient
a) I
b) II
c) III
d) IV

110. Same patient was many medications, Nitroglycerin, ACEI, B blocker atenolol, furosemide
40 mg am and 20 mg at night, what is the initial step that should be done with this patient?
a) Switch to Bisoprolol
b) Switch to Metoprolol
c) Switch to Propranolol
d) Discontinue Atenolol
e) Change atenolol to Ivabradine.
a) Switch to Sotalol
b) Switch to Carvedilol
c) Increase the dose
d) Keep same dose of Atenolol
Switch to bisoprolol, Switch to carvedilol, D/C atenolol  All are right answers
Beta‐blockers improve symptoms and reduce the risk of hospitalization and death in patients with HFrEF.
They are recommended in all patients with an LVEF ≤40%. Prescribe only the beta‐blockers that have been
shown to reduce mortality: bisoprolol, carvedilol and metoprolol succinate (not available in Canada).
Metoprolol tartrate is available in Canada, but has not been shown to reduce mortality in patients with HF.
In fact, mortality and hospital admissions were significantly more frequent in patients treated with
metoprolol tartrate (target dose 50 mg BID) than with carvedilol (25 mg BID) in a large randomized trial.
The target dose of metoprolol tartrate used in this trial was lower than the target dose of controlled‐release
metoprolol succinate that significantly decreased mortality in a large placebo‐controlled trial.
Nebivolol has been studied in HF, but it has not been convincingly shown to reduce mortality.
In HF patients, initiate beta‐blockers at a very low dose and slowly titrate the dose at 2‐ to 4‐week intervals.
Beta‐blockers should not be initiated, or their doses increased, while patients are acutely decompensated.
Nevertheless, once the patient is clinically stable, beta‐blockers should be carefully initiated with close
monitoring before the end of the hospitalization.
Monitor blood pressure and heart rate before initiating a beta‐blocker and before any increase in dose.
Watch for the signs and symptoms of HF decompensation when initiating or increasing the dose of a beta‐
blocker, paying particular attention to the daily morning weight.
Beta‐blockers should not be stopped abruptly.
Refer patients with severe HF (NYHA class III–IV) to an HF specialist for initiation of a beta‐blocker

110. Same patient has high K level 5.8, HP 140/90, still have moderate edema. What is the best
to add to his medication to reduce mortality?
a) Spironolactone
b) Increase furosemide dose
c) Digoxin
d) Hydralazine / nitrate
Mineralocorticoid receptor antagonists (eplerenone or spironolactone) should not be used in patients with a
baseline potassium >5 mmol/L, serum creatinine >221 mcmol/L or creatinine clearance <30 mL/minute.
Monitor vital signs, serum creatinine and potassium at 3 days and 7 days after initiating or titrating the dose
of MRAs, and repeat as necessary until the potassium level and renal function are stable. Monitoring should
then be performed monthly for 3 months and then every 3 months.
Eplerenone, unlike spironolactone, does not produce gynecomastia and is therefore the agent of choice in
individuals who have experienced this adverse effect.
Eplerenone appears to have a similar risk of hyperkalemia and renal dysfunction as spironolactone, so
should not be used as a substitute in these situations. Prospective comparative data between these agents
is limited. The choice is left to the discretion of the clinician and will likely be influenced by reimbursement
considerations.
Nitrates/Hydralazine
The combination of isosorbide dinitrate plus hydralazine reduces mortality and morbidity in black patients
with NYHA class III–IV HF and is recommended in addition to standard therapy (ACE inhibitor, beta‐blocker
with or without MRA) in this setting. Use of this combination may also be considered in black patients with
NYHA class II HF and in other HF patients who do not tolerate ACE inhibitors, ARBs or an ARNI.
Nitrate monotherapy is valuable in treating symptoms of angina, paroxysmal nocturnal dyspnea and
orthopnea. Nitrates have not been shown to reduce mortality in the absence of hydralazine.
Digoxin
Digoxin improves symptoms and reduces the risk of hospitalization for exacerbations of HFrEF, but does not
reduce mortality in patients with persistent moderate to severe symptoms (NYHA class II–IV) while on ACE
inhibitor therapy.
Diuretics
Diuretics are recommended to control signs and symptoms of volume overload. Thiazide diuretics can be
used in patients with minimal fluid retention, but loop diuretics, usually furosemide, are required in most
patients.

111. This patient was in waiting area and he hear your counsel other patient about sildenafil, he
approaches you and ask you for same drug, as he is having sexual issue too, what the best way
to handle it?
a) You can’t take this medicine with NG
b) Its a private conselling you are not supossed to hear it
c) Your medication causes this ED
d) You take care next time when counselling so pt not overhead you
e) Tell him you will cooperate with his family doctor to deal with this issue.
f) Explain & discuss the available options for the pt
Another version
What works for beneficence of the patient?
a) Discuss with his physician the addition of Sildenafil. 1st patient beneficence
b) Make sure to council the 2nd patient privately. 2nd Patient confidentiality

110. GH is a 63-year-old male with history of ACS but no CHADS2 risk factors. He has aspirin
allergy. He got NSTEMI, what do you recommend for secondary prevention:
a) Aspirin
b) Dipyridamole
c) Clopidogrel
d) Warfarin
e) Rivaroxaban
ACS: Antiplatelet Drugs Aspirin, clopidogrel, prasugrel, ticagrelor, ticlopidine, and glycoprotein (GP) IIb/IIIa
inhibitors are examples of antiplatelet drugs. All patients are given aspirin 160 to 325 mg (not enteric‐
coated), if not contraindicated (eg, life‐threatening active bleeding), at presentation and 81 mg once a day
indefinitely thereafter. Chewing the first dose before swallowing quickens absorption. Aspirin reduces short‐
and long‐term mortality risk. If aspirin cannot be taken, clopidogrel 75 mg orally once a day or ticlopidine 250
mg orally twice a day may be used. Clopidogrel has largely replaced ticlopidine for routine use because
neutropenia is a risk with ticlopidine and white blood cell count must be monitored regularly.

111. GH came back after 2 weeks complaining of severe chest pain radiating to his left
shoulder, he used tadalafil 12hours before. He visits your pharmacy after diagnosed or
discharged form hospital because of angina, the prescription was NG SL to be used prn, patient
falls in the pharmacy. What will be your initial response?
a) Call 911
b) Call 911 & administer 2 crushed tablet of 80 mg ASA
c) Call 911 and administer nitroglycerin SL
d) Call 911, administer ASA 2 tablet 80 mg & NG
Systolic and diastolic blood pressure may be significantly reduced following coadministration of nitrates and
phosphodiesterase 5 inhibitors.
The manufacturers of sildenafil, tadalafil and vardenafil
recommend that these drugs not be used in combination
with nitrates. In situations where nitrate use is required in
a patient also receiving a phosphodiesterase 5 inhibitor,
effect on blood pressure can be reduced if there is
sufficient time between doses. Separate doses of nitrates
and sildenafil and vardenafil by at least 24 hours. Allow 48
hours between tadalafil administration and nitrates.

112. The pt was ok & discharged from


hospital, what the best to add to him to
manage his angaina
a) Beta blocker
b) Amlodipine
c) Bisoprolol

113. Then he came after 2 months with


leg edema. What medication is the cause?
a) Pregabalin
b) Clopidogrel
c) Amitriptyline
d) Lidocaine
e) Tramadol
>10%: Cardiovascular: Peripheral edema. Endocrine & metabolic: Weight gain. Gastrointestinal: Xerostomia
Nervous system: Dizziness, drowsiness, fatigue, headache. Ophthalmic: Blurred vision, visual field loss
Monitoring Parameters
Measures of efficacy (pain intensity/seizure frequency); degree of sedation; symptoms of myopathy;
creatine kinase (as clinically indicated); symptoms of ocular disturbance; weight gain/edema; skin integrity
(in patients with diabetes); signs and symptoms of suicidality (eg, suicidal thoughts, anxiety, depression,
behavioral changes); platelet count (as clinically indicated)

114. HF LVEF less 35 %, to improve the


cardiac rhythm add?
a) Sotolal
b) Metoprolol
c) Amiodarone

115. Post MI + hypertension + LVEF 35


%. What to give post MI
a) Clopidogrel + ASA + Bisoprolol +
Eplernone + ACEI
b) Ticogrelol + ASA + Atenolol +
Spiranolactone + ARB

116. A 53-year-old man presents at your pharmacy with complaints of painful and swollen
joints, fever, chest pain, hair loss, mouth ulcers, swollen lymph nodes, feeling tired, and a
red rash, dr diagnosed him with SLE, what NON pharmacological options to advise him?
a) Smoking cessation
b) Decrease salt and caffeinated
beverages
c) Use moisturizer while going out
in hot weather

117. What initial treatment to give


orally?
a) Hydroxychloroquine
b) High dose corticosteroid
c) MTX
Considered baseline therapy for the majority of patients with SLE.
May take up to 2 months to see an effect.
200–400 mg daily PO. Max: 5 mg/kg/day based on actual body weight
May increase digoxin levels, may increase effect of betablockers.
Ophthalmologic assessment required Q1–5 y, depending on risk factors.
118. She came back with a joint attack &
Flare ups; now what to give?
a) Methotrexate
b) Sulfasalazine
c) Mycophenolate

119. What test to monitor Methotrexate?


a) Chest X-ray
b) TB
c) CK
d) TSH
e) Eye examination
f) Skin rash.
Baseline assessment should include complete
CBC with differential and platelet counts, hepatic
enzymes, renal function tests and chest xray.
Monitor hematology at least monthly, and hepatic enzymes and renal function every 1–2 months.

120. after 2 weeks, the patient calls and said he is experiencing stomatitis, headache, nausea and
vomiting after taking high dose of MTX, what is the antidote you recommend?
a) Leucoverin
b) Naloxone
c) Flumazenil
d) Penicillamine
Oral overdose is often due to incorrect dosage & administration (e.g., daily rather than weekly administration).
Symptoms include leukopenia, thrombocytopenia, anemia, pancytopenia, bone marrow suppression, mucositis,
stomatitis, oral ulceration, nausea, vomiting, GI ulceration and GI bleeding. Symptoms of intrathecal overdose
include headache, nausea and vomiting, seizure or convulsion and acute toxic encephalopathy.
Recommended Management
Leucovorin calcium (also known as folinic acid) is used to counteract toxicity of inadvertent overdosage of
methotrexate. Administer as soon as possible.
Glucarpidase is a recombinant bacterial enzyme that inactivates extracellular methotrexate and can rapidly
lower serum methotrexate levels by 95% within 15 minutes of administration. It is available through the
Health Canada Special Access Programme. Administration time of leucovorin and glucarpidase should be
separated. Leucovorin should be continued after administration of glucarpidase.
If overdosage is massive, hydration and urinary alkalinization may be necessary to prevent precipitation of
methotrexate and its metabolites in the renal tubules. Peritoneal dialysis does not improve methotrexate
elimination. Acute intermittent hemodialyis using a high‐flux dialyzer has achieved significant clearance of
methotrexate. Accidental intrathecal overdosage may require intensive systemic support, high‐dose leucovorin
(systemic, not intrathecal), alkaline diuresis and rapid CSF drainage and ventriculolumbar perfusion.
121. She now finished her course for treatment with a complete resolution of her case, she told
you that she wants to be a pregnant but you told her she needs to wait 3 months after the last
dose of MTX. She is concerned about pre-eclampsia. What to give to her if she is getting
pregnant to prevent preeclampsia?
a) Use 81 mg ASA
b) Use Azathioprine
c) Give her Prednisone
a) Washout period with cholestyramine
Recommendations for the management of patients with SLE during pregnancy depend on the presence of
antiphospholipid antibodies with or without antiphospholipid antibody syndrome. Low‐dose ASA is given
empirically to women with SLE for preeclampsia prevention and is safe in pregnancy starting around 12
weeks’ gestation with continuation to term.
Women with antiphospholipid antibody without overt antiphospholipid antibody syndrome may be treated
with ASA with a low‐dose heparin (either unfractionated or low‐molecular weight heparin). Women with
antiphospholipid syndrome are treated with prophylactic full dose heparin in addition to low‐dose ASA, as
the combination improves live birth rate and prevents preeclampsia respectively.
Patients on methotrexate and mycophenolate are counselled to avoid pregnancy. These medications should
be stopped 3 months (methotrexate) or 6 weeks (mycophenolate) prior to attempting conception.
Recent small cohort studies showed no adverse pregnancy outcomes after preconception low‐dose MTX
exposure in males or females. Women should still be switched to a low‐risk medication e.g., azathioprine or
hydroxychloroquine, and observed for disease activity over 6 months prior to conception

122. 3 straight calculations for NNT (one of them NNH) and ARR. Drug A and Drug B and
given fatal rate and CV side effect, ask about NNH (number need to harm)

123. 25 yrs old guy complains from fever, severe headache, stiff neck or back pain and/or
photophobia, feeling unwell with associated vomiting, loss of balance, disorientation, confusion
or altered level of consciousness. He did the test & Lab report show that CSF has High protein,
Low glucose; High WBC & G+ve diplococci organism. Dr diagnosed him with bacterial
meningitis & no other medical issues. What is the most causative organism?
a) Strep pneumonia
b) N meningitidis
c) Listeria monocytogens
d) GBS
Streptococcus Gram-positive, lancet-shaped cocci. Usually they are seen as pairs of cocci (diplococci), but
pneumoniae they may also occur singly and in short chains.
appearance Nonmotile, non-spore forming
Neisseria Gram-negative cocci, typically appear in pairs with the opposing sides flattened (a "kidney
meningitidis bean" appearance, with the long axes of the cell parallel)
appearance nonmotile, non-spore forming
Listeria small, Gram-positive, short rods or coccobacilli
monocytogenes motile at 30°C (flagella, a characteristic tumbling motility in fluid media), nonmotile at 37°C
appearance non-spore-forming
S. aureus Gram-positive cocci in grape-like clusters. Nonmotile, non-spore-forming

124. Empirical ttt. What to give?


a) Ceftraixone + Ampicillin
b) Ceftriaxone + vancomycin
c) Ceftriaxone + Gentamicin

125. A patient punctured while he was in


gardening & developed osteomyelitis.
Culture show Gm-ve rode-shaped,
which bacteria
a) S aureus
b) Streptococcus pyogens
c) P aeruginosa
Streptococcus Gram-positive cocci in chains (when cultivated in liquid media)
pyogenes Nonmotile, non-spore-forming, often encapsulated (capsule composed of hyaluronic acid)
S. aureus Gram-positive cocci in grape-like clusters. Nonmotile, non-spore-forming
Pseudomonas Pseudomonas aeruginosa is a gram-negative, rod-shaped, asporogenous, and
aeruginosa monoflagellated bacterium that has an incredible nutritional versatility.
It is a rod about 1-5 µm long and 0.5-1.0 µm wide.

126. Which is the proper treatment without


wound Debridement?
a) Cefazolin 10 – 14 days
b) Cefazolin 4 – 6 weeks
c) Gentamycin 10 – 14 days
d) Cefuroxime 4 – 6 weeks
Penetrating Trauma: as in puncture wound of
foot, caused by P. aeruginosa, S. aureus
 For Children: cloxacillin or cefazolin plus
ceftazidime; Alternative: extended
spectrum penicillin and beta‐lactamase inhibitor combination
 For Adults: fluoroquinolone, extended spectrum penicillin, beta‐lactamase inhibitor combination.
 Duration of antibacterial therapy should be minimum of 4 weeks; many authorities recommend 6
weeks (especially for vertebral osteomyelitis). More severe initial presentation, extensive bone
involvement, slow resolution of systemic and local signs indicates a 6‐week course.
 In osteomyelitis following penetrating injury, 10–14 days of treatment is sufficient if adequate
débridement has been performed.
127. Very similar calculation, just different numbers
Ampicillin 5,000,000 units when added to 23 ml of water yield 200,000 units / ml. how much
water needed to make it 250,000 units /0.5 ml.
Answer:
200,000 Us ‐‐‐‐‐‐‐ 1 ml 5,000,000 Us ‐‐‐‐‐‐‐ X ml
X = 1*5,000,000/200,000 = 25 ml amount of Ampicillin = 25 ‐ 23 = 2 ml.
250,000 Us ‐‐‐‐‐‐‐ 0.5 ml 5,000,000 Us ‐‐‐‐‐‐‐ X ml
X = 5,000,000*0.5/250,000 = 10 ml amount of water = 10 ‐ 2 = 8 ml

128. A mother came with her 6-year-old child


with symptoms fever, irritability, otalgia, and
ear discharge that indicates AOM, which one
is common pathogens for AOM
a) Strep pneumoniae, H. influenzae and
M. catarrhalis
b) Strep pneumoniae, H. influenzae and S.
aureus
c) N. meningitidis, H. influenzae and S.
aureus

129. The child has allergy to penicillin with


red patch on his limbs, didn’t take any
antibiotic in the last year, what you will use?
a) Amoxiclav
b) Azithromycin
c) Clindamycin
d) Cefuroxime auxetille

130. One of the staff forget vaccine outside the fridge, what should be done or Fridge went off
for 36 hours, what to do?
a) Punish the staff member who did that
b) Call manufacture to check if the vaccines are still ok
c) Put them back in the fridge till the expiry date then back to the manufacturer
d) Just return them back to the fridge
Managing cold‐chain breaches
1. ISOLATE the vaccines in the refrigerator and label “DO NOT USE”
2. DOCUMENT: Date of incident, the issue, estimated range of temperatures vaccine was exposed to,
Duration of exposure, Manufacturer expiry date and lot number
3. NOTIFY pharmacy administration and all other appropriate authorities
4. ASSESS usability of vaccine, by pharmacist in consultation with the manufacturer
5. DISCARD exposed vaccine if necessary
6. EVALUATE pharmacy procedures to determine how to avoid future cold‐chain disruptions
7. MAKE POLICY CHANGES as necessary and monitor changes for effectiveness

131. You received a package of insulin, you forgot to put it immediately in the fridge so it
remained for a while outside; when you checked its temp., it was 8C. What to do?
a) Punish the staff member who did that
b) Quarantine and Call / Contact manufacture to check if the insulin is still ok
c) Put them back in the fridge till the expiry date then back to the manufacturer
d) Return them back to the fridge and use till expired.

132. Pt came to pharmacy and afraid of taking influenza vaccine, because of its reaction or side
effects. What should the pharmacist do?
a) Respect her choice and leave her
b) Enforce social obligation aspect, and how vaccination will protect the whole society
c) Tell her you are ready to address any of her concerns
There should be no obligation, everything should go smoothly with patient approval.

133. What should the pharmacist ask the Pt before giving him influenza vaccination?
a) If Pt has egg allergy
b) If Pt is pregnant
c) If Pt has previous vaccine reaction

134. A very angry customer comes to your pharmacy and complains that you have dispensed
wrong medication. What is appropriate first step in resolving this problem?
a) Calm down patient & Offer private counselling area
b) Be assertive
c) Acknowledge his anger, ask and verify what is error, if there is error in dispensing,
apologize and correct it.
d) Ignore and let him go
Another version: Pt is angry as his RX isn't ready yet by its schedule time because technician
has workload, what to do?
a) Discipline the technician
b) You should tell the technician to be accountable
Tips for Remember that feelings of hostility are rarely personal; the patient may be under a great deal of
interacting stress
with the  Acknowledge anger, let patients vent their anger & Stay calm
angry patient  Lower your voice, speak slowly and maintain eye contact
 If you are at fault, agree; agreeing often diffuses anger
 Avoid defensiveness (which can aggravate the situation)
135. A mother is afraid from vaccinating her child because of needle pain, what should you tell
her before injecting him?
a) Give diclofenac 12.5 supp.
b) Breastfeed him before and after vaccination (before, during, after vaccination)
c) Lay child supine before giving vaccine (hold you baby)
d) Cold compresses
e) Use topical anesthetic Lidocaine 10 -20 min before (20 to 60 minutes)
f) Give Acetaminophen 10-15 mg kg 30 min before vaccination
g) Tell him it is not painful
h) Tell nurse to hold him while he is in upright position

136. Breastfeeding Pt with vaginal itching, odorless, white-cottage discharge, dr diagnosed her
with vulvovaginal candidiasis, she has it before and this isn’t the1st time, she is also on
warfarin. She is asking for your recommendation, what to give?
a) Fluconazole oral 150 cap single dose
b) Clotrimazole 500 mg vaginal tablet X 1 dose
c) Clotrimazole 1% cream external use 7 day
d) Refer to doctor
e) Metronidazole 500 mg vaginal tablet 7 days
C is wrong not external
Clotrimazole, Vaginal tablet and cream:
Recurrent infection (≥4 episodes/y): extend treatment period
to 10– 14 days then maintain with clotrimazole 500 mg
vaginal tablets once monthly for at least 6 months.
Menstruation is not an indication to stop treatment.
Q. Safe for use in pregnancy; 7–14 days treatment period
may be necessary.
Follow up if symptoms persist despite treatment or recur
within 2 months of onset.

137. Patient took clozapine injection IV, you need to


give it immediately as first dose, which first sign to
monitor or what you need to monitor for the 1st 2 days?
a) SJS
b) Neutropenia/ agranulocytosis
c) NMS
d) Hypotension
On the first day, CLOZARIL should be given at a 12.5 mg dose (one‐half of a 25 mg tablet) once or twice,
followed by one or two 25 mg tablets on the second day. If well tolerated, the dosage may be increased in
daily increments of 25 mg to 50 mg, achieving a target dose of 300‐450 mg/day by the end of two weeks.
Subsequent dosage increases should be made no more than once or twice weekly, in increments not to
exceed 100 mg. Cautious titration and a divided dosage schedule are necessary to minimize the risks of
hypotension, seizure and sedation

138. Asthmatic Female patient


complains about foreign body
sensation in the eye, sandy or
scratchy, burning, itchy or tired. The
patient present by himself after 48
Hours to the pharmacist and from
her symptoms, you noticed that it is
dry eye condition, what to give her?
a) Eye lubricant or artificial
tears
Carboxymethylcelleulose
b) Polysporin eye drops
c) Refer for ciprofloxacin E.D
d) Give oxymetazoline E.D
139. After a period of time, the patient came with other eye complain. She has halos around
lights, decreased visual acuity & headache. She is also taking COC for about 6 years now. You
refered her to doctor & after investigation, dr told her that she has glucoma. What is the Goal of
therapy for her?
a) Preserve optic nerve function
b) Prevent from progressing to closed angle glucoma
c) Prevent spontaneous detachment of optic nerve
d) Prevent ARMD

140. Her doctor is looking for medication for one of his patients who has glaucoma. The patient
is currently on salbutamol and flovent. He also suffers from a sulpha allergy. He decided on
Prostaglandin analogue travaprest what would be the MOA?
a) Decrease IOP by increasing outflow
of aqueous humor
b) Decrease IOP by Suppressing
formation of aqueous humor
c) Decrease IOP by by inhibiting
formation of aqueous humor

141. Travoprost, what is the counselling?


a) Increase iris pigmentation and
eyelash length
b) Store in room temperature, don’t
refrigerate
c) Can be used while wearing contact
lenses
d) Use one drop twice daily, and if missed
dose, use it ASAP.
e) Can cause headache and decrease in night vision
a) Exacerbate patient asthma, thus requiring more salbutamol

142. Spironolactone will cause Hypokalemia with


a) Hypoaldosteronism
b) Bisoprolol
c) Corticosteroids
d) TMP-SMX
ALDACTONE is indicated for treatment of hypokalemia, when other measures are considered inappropriate
or inadequate. It is also indicated for the prophylaxis of hypokalemia in digitalis therapy when other
measures are inadequate or inappropriate.
143. Case about Trichomoniasis, 3 q. Female patient came with symptoms Characterized by
inflammation, Pruritus, odour, off-white or yellow, frothy wet discharge. The main cause?
a) Inflammation in vagina
b) Change in vaginal flora
c) Normal flora defect

144. Oral treatment should be


a) Cefixime
b) Metronidazole
c) Fluconazole
d) Clindamycin

145. If her boy friend refuse to wear a condom. What you should tell him
a) It is a sexual transmitted and you have to wear condom
b) No problem, no need to wear condom transmission rate of the infection is low
c) Prophylaxis dose of metronidazole will prevent transmission
Trichomoniasis Characterized by inflammation, Pruritus, odour, off-white or yellow, frothy wet discharge.
Caused by Vaginal pH >4.5, -ve "Whiff" test, +ve PMN = polymorphonucleocyte
Trichomonas Treatment with oral metronidazole (2 g PO × single dose or 500
vaginalis. mg BID PO × 7 days) is recommended for all patients with possible
exception of asymptomatic pregnant women.
Intravaginal metronidazole is ineffective in treatment.
There are no effective alternative topical or systemic treatments
available for those patients who cannot use metronidazole.
S.E: GI upset, urethral burning, dark or reddish-brown discoloration of urine, metallic taste
Treat current partners irrespective of symptoms; efficacy increases if partner is also treated,
no sexual contact until patient and partner finished treatment and are asymptomatic
Can be used in pregnancy or breastfeeding; some clinicians advise withholding breastfeeding
for 12–24 h after a 2 g dose.
146. Which one causes urine discoloration?
a) 5-fluorouracil
b) Doxurubicin
Doxorubicin may give a reddish color to your urine, tears, and sweat. This effect may start in the first hours
after treatment and may last up to several days. This is a normal effect of the drug and should not be
mistaken for blood in your urine. Temporary hair loss may occur.

147. Pt discharged from hospital but couldn’t do routine daily activities. Which one could help?
a) Social worker
b) Occupational Therapist
c) Physiotherapist

148. EC 50 years female diabetic has penicillin allergy diagnosed with CAP went to hospital.
The Physician decided to treat her as an outpatient. which microorganism is cocci positive?
a) S pneumonia
b) N gonorrhea
c) H Influenza

149. The dr asks the pharmacist for treatment recommendation what should be the response:
a) Doxycycline
b) Moxifloxacin PO
c) Amoxicilline clavulanate plus clarithromycin
d) Ciprofloxacin IV
Empiric Therapy
 For CAP of moderate severity, there is no difference between beta-lactam alone, macrolide and beta-
lactam, or fluoroquinolone therapy. A systematic review showed no difference between macrolides &
fluoroquinolones but fewer adverse events with clarithromycin than erythromycin. Another review
showed no difference between macrolides and fluoroquinolones for death.
 For outpatients, amoxicillin as first choice or either amoxicillin/clavulanate or doxycycline as 2nd choice.
 For severe pneumonia, 3rd generation cephalosporin (ceftriaxone or cefotaxime) in combination with
clarithromycin is a rational empiric regimen. However, it should be noted that macrolide antibiotics
(e.g., azithromycin, clarithromycin) and FQs may cause QT interval prolongation and caution is advised.
 For inpatients: Systematic review evidence for inpatients showed no difference between macrolides
and fluoroquinolones for death and no difference for death whether or not atypical coverage was
used. In order to reduce increasing fluoroquinolone resistance and prevent adverse events (e.g., QT
interval prolongation), use of a respiratory fluoroquinolone should be reserved for when cephalosporins
or penicillins cannot be used.
 Piperacillin/tazobactam should be used only for severe pneumonia or in patients at high risk for
resistant pathogens, e.g., P. aeruginosa.
150. What is the recommended treatment duration:
a) 10 days
b) 7 days
c) 14 days
d) 21 days
Duration of Antibiotic Therapy
 For patients who are well enough to be treated on an ambulatory basis, a minimum of 5 days of
antibiotic therapy is required.
 Patients who are hospitalized, who respond to treatment within 48 hours and who have no complications
may be treated for 5–10 days.
Specific etiologies may require longer treatment, such as:
 21 days for severe legionnaires’ disease & pneumonia caused by P. aeruginosa.
 14 days for bacteremic aerobic gram-negative bacilli pneumonia & Empyema that requires drainage.
 Prolonged therapy is necessary when a lung abscess complicates pneumonia.

151. EC returned back with complications that require ICU admission what would be the best B
lactam for him:
a) Piperacillin/Tazobactam
b) Meropenem
c) Ceftazidime
d) Ceftriaxone

153. Physician recommended the pneumococcal vaccine to EC but she is informing you that she
is afraid of taking the vaccine what would be your response:
a) Tell her that she has to take the vaccine as recommended by the physician for her safety.
b) Tell her that she has to receive the vaccine to protect the public
c) Inform her about benefits and risks of the vaccine
d) Do not give her the vaccine

154. As indication for EC improvement. all will be monitored except:


a) Respiratory rate
b) Chest X ray
c) Sputum production
d) O2 saturation
e) Fever & Temperature
Chest X ray after 1 month of discharge to role out cancer risk especially in people over 50
Switch patients from IV to oral antibiotics when the following criteria are met: GI tract is functioning
normally, hemodynamically stable; 2 temperature readings are normal, normalized white blood cell count;
subjective improvement in cough and shortness of breath; able to consume oral medications.
Discharge the patient when the following criteria are met in addition to those above: absence of
complications from the pneumonia (e.g., empyema); absence of complications from comorbid illnesses (e.g.,
MI); absence of complications from treatment (e.g., severe adverse drug reactions); physiological stability as
indicated by an oxygen saturation of ≥92% while breathing room air for those who do not have COPD (for
patients with COPD, a return to baseline status is desirable), pulse rate of <100 beats/minute and
respiratory rate ≤24 breaths/minute

155. Calculation case; pt on dexamethasone


 Dexamethasone 15mg for 21 days
 Dexamethasone 12.5mg for 21 days
 Dexamethasone 10 mg for 21 days
Then decrease the daily dose by 1 mg per day every 4 weeks.
Pt has private insurance and his plan cover 100 days only, you decided to give 98 days therapy.
You have in stock dexamethasone 5mg and dexamethasone 1mg, how many tablets of each
doses you will give to make his regimen the simplest the possible? Options give were number
tablets of each dose. (Forward case and time consuming)
Answer
Now, you will dispense 98 days only to patient according to his insurance coverage
So, 98 – (21+21+21) = 35 days
These 35 days will be supplied as the following:
9 mg dexamethasone (4 weeks) & 8 mg dexamethasone (1 week).
Now we will calculate the tablets required from 5 mg and 1 mg strength
(3*21) + (2.5 *21) +(2*21) = 63+ 52.5+42=157.5 tablet
From 5 mg to get 9 mg dexa you will need (1*28) =28
From 1 mg to get 4 mg (9‐5) you will need (4*28) = 112
To calculate 8 mg (from 5 mg you need (1*7) and from 1 mg (3*7)
From 5 mg only 7 and from 1 mg 21 tablets
So total from 5 mg = 157.5 +28+7=193 tablet From 1 mg you need = 112+21=133 tablets.

156. Non-sterile compound beyond use date BUD


a) 40 days on refrigerated
b) 30 days on refrigerated
c) 1 day at room temperature
expired day 40 days ‫ ﻛﺎﻥ ﺍﻗﻞ ﻭﺍﺣﺪ ﻓﻴﻬﻢ‬rx ‫ﻛﺎﻥ ﺟﺎﻳﺐ ﺷﻮﻳﺔ ﺍﺩﻭﻳﺔ ﻭﺟﻨﺐ ﻛﻞ ﻭﺍﺣﺪ‬
157. 9 years girl with meningitis what empiric therapy
a) Cefatrixone + vancomycin
Age Group Bacteria Empiric Antibacterial Regimen
Infants <1 month Streptococcus agalactiae (Group B Cefotaxime + Ampicillin
streptococcus), E. coli, Listeria monocytogenes Add gentamicin if early neonatal
(rare), Neisseria meningitidis (rare), S. meningitis suspected due to synergy
pneumoniae (rare), Other Enterobacteriaceae. for Group B streptococcal infections
Children ≥1 month S. pneumoniae, N. meningitidis, S. agalactiae Ceftriaxone or cefotaxime +
(Group B streptococcus), Haemophilus Vancomycin
influenzae type b, E. coli (rare), L.
monocytogenes (rare), Other Enterobacteriaceae
Adults < 60 y E. coli S. pneumoniae N. meningitidis
L. monocytogenes (rare)

158. Her mother not belive in vaccine, but she uses it for her girl, now the girl has problem in
hearing & not talking well, her mother belive because of her vaccine, to whom doctor should
you refer?
a) Audioligst
b) Early child education specialist
c) Psychatric
d) Family physician
e) Speech language patholigist
Parents may suspect a hearing deficit if their child ceases responding appropriately to noises or voices or
does not understand or develop speech.
Because hearing deficits impair language development, hearing problems must be remedied as early as
possible. The clinician therefore should seek parental input about hearing at every visit during early
childhood and be prepared to do formal testing or refer to an audiologist whenever there is any question of
the child’s ability to hear.
Audiometry can be done in the primary care setting; most other audiologic procedures (eg, otoacoustic
emission testing, brain stem auditory evoked response) should be done by an audiologist. Conventional
audiometry can be used for children beginning at about age 3 years; young children can also be tested by
observing their responses to sounds made through headphones, watching their attempts to localize the
sound, or observing them complete a simple task.
An audiologist is a licensed hearing health care professional who specializes in the diagnosis and treatment
of hearing loss and balance disorders in adults and children. You can think of an audiologist primarily as a
“hearing doctor.” Most audiologists have completed a doctor of audiology (Au.
159. An old patient in Long care unit with many diseases, Dyslipidemia, Hypothyriodism &
STEMI, also he did PCI. His medications profile includes Rosuvastatin, Levothyroxine, Ca
carbonate, Clonazepam & ramipril. The physician called you and asked you to helping her to
take her medication & you offered him blister-packs for his medication. What should concern
the pharmacist in the dosset that that technician prepared?
a) Alendronate at lunch time
b) Iron in the morning
c) Rosuvastatin 20 mg at bed time
d) Levothyroxine at morning
e) Ca carbonate at lunch
f) Amiloride in the morning
The recommended dosage is one 70 mg tablet once weekly or one 10 mg tablet once daily.
FOSAMAX must be taken at least one‐half hour before the first food, beverage, or medication of the day
with plain water only. Other beverages (including mineral water), food, and some medications are known to
reduce the absorption of FOSAMAX. Waiting less than 30 minutes will lessen the effect of FOSAMAX by
decreasing its absorption into the body.
FOSAMAX should only be taken upon arising for the day. To facilitate delivery to the stomach and thus
reduce the potential for esophageal irritation, a FOSAMAX tablet should be swallowed with a full glass of
water (200‐250 mL). To facilitate gastric emptying, FOSAMAX oral solution should be followed by at least 60
mL (a quarter of a cup) of water. Patients should not lie down for at least 30 minutes and until after their
first food of the day. FOSAMAX should not be taken at bedtime or before arising for the day. Failure to
follow these instructions may increase the risk of esophageal adverse experiences.
All patients must receive supplemental calcium and Vitamin D, if dietary intake is inadequate.
Amiloride is administered orally with food or milk to decrease adverse GI effects.

160. Pharmacist wants to end relationship with a patient, whom to inform


a) The pharmacist in charge
b) Call the doctor to take permission
c) Move file to next pharmacy
d) Just end and do nothing
e) The college
f) The physician
https://www.ocpinfo.com/regulations‐standards/practice‐policies‐guidelines/ending‐relationship/
The pharmacist will consider the patient’s condition and availability of alternative services when making the
decision to terminate the pharmacist/patient relationship.
The patient relationship cannot be terminated without good reason, proper notice, and an opportunity
given to the patient to obtain another pharmacist’s/pharmacy’s services before discontinuation.
The pharmacist must ensure that the decision to terminate care does not infringe a prohibited ground within
the meaning of the Ontario Human Rights Code.
Where several pharmacists work together, it may be appropriate to plan in advance how terminations will
be executed, and whether another member is available to provide patient care.
1. Communicate the decision
Depending on the reason for the termination, the member will communicate the decision to terminate
service in writing, unless the patient has no fixed address or the pharmacy does not have a current address
on file. When communicating the decision in person, it is important to maintain acoustical privacy while
ensuring that both the patient and staff members are safe.
The patient should be clear about the availability of refills or other professional services until he or she is
able to obtain services from another pharmacist. Based on an evaluation of the patient’s condition,
determine whether to notify his/her prescriber(s) of the change in pharmacy or rely on the patient to do so.
2. Provide a reasonable amount of time for the patient to find a new pharmacist
The amount of time provided for the patient to find a new pharmacist will be reflective of the condition of
the patient, his or her special needs and availability of services in the local community. Advise the patient of
measures that will assist the transition including record transfers and providing information directly to the
next provider, as required. If no refills remain on file, provide the patient with a patient profile
report/medication history for his/her information.
3. Document
Document the decision and rationale for the termination according to the Documentation Guidelines and
retain a copy of the patient’s letter. A summary of the type of information that could be included in the
patient’s letter is attached (Appendix 1).
4. Advise staff members
Let the appropriate staff members know of the decision to terminate the patient relationship and the period
in which services will continue to be provided, if any.
Summary Information – Letter of Termination
A written communication to the patient regarding a termination of the pharmacist/patient relationship
contains the patient’s name, the pharmacist’s name and the name of the pharmacy; and additional
information including, for example:
 Affirmation and rationale for the decision to terminate the relationship and date chosen as the last
day of care;
 Direction to the patient to obtain services at another pharmacy and offer to transfer prescriptions;
 Confirmation that prescriber(s) will be informed of the decision in the event that verbal prescriptions
are received, if relevant, and/or a recommendation that the patient inform his/her prescriber(s)
directly;
 Acknowledge attachment of patient profile/medication history (if applicable); and
 Any other information considered relevant.

161. According to agreement between pharmacy and wholesaler, only non narcotic and non
refrigerated items can be returned to the supplier. Which of the following medication could be
returned to wholesaler?
a) Epoetin alpha (needs fridge)
b) Etanercept inj
c) Testosterone patch
d) Evra® norelgestromin—ethinyl estradiol patch
e) Latanoprost eye drop
f) Nalbuphine
g) Fludrocortisone tablet
h) Vaccines
i) DMPA injection
DMPA Monograph: Do not store above 25°C. Do not refrigerate or freeze.
Evra: Store between 15‐25°C. Do not refrigerate or freeze. Store patches in their protective pouches inside
the original box. Apply patch immediately upon removal from its packaging.

162. Which needs preparation under sterile condition:


a) Eye drops
b) Dialysis solution
c) Rectal sup
d) Topical antibiotic cream
e) Otic drop
f) Nasal drop
g) Oral antibiotics
As the quality of water in the dialysis fluid varies considerably, and in view of the fact that endotoxin or
active derivatives can cause acute side effects in patients, the dialysis fluid must be sterile. ... With
ultrafiltration of dialysis fluid, we can obtain sterile dialysate, which is endotoxin free.

163. Antibiotic medication for child the dose was wrong because the weight was entered to
computer system as pounds not KG how to prevent future errors:
a) Recheck patient weight with parents during counselling
b) Let technician double check calculation after pharmacist
c) Update computer system so that it only can take pounds not KG entry for weight
،، ‫ﺍﻟﻤﺸﻜﻠﺔ ﺍﻧﻪ ﺩﺧﻠﻬﺎ ﻋﻠﻰ ﺍﻟﻜﻤﺒﻴﻮﺗﺮ ﻏﻠﻂ ﻻﻥ ﺍﻟﻜﻤﺒﻴﻮﺗﺮ ﻣﻤﻜﻦ ﺗﺪﺧﻞ ﻋﻠﻴﻪ ﻛﺠﻢ ﺍﻭ ﺑﺎﻭﻧﺪ‬،، ‫ﺍﻟﻤﺸﻜﻠﺔ ﻣﺶ ﻓﻰ ﺍﻟﻜﺎﻟﻜﻮ‬
‫ﻓﺎﺣﻨﺎ ﺣﻨﻮﺣﺪ ﺍﻻﺩﺧﺎﻻﺕ ﻋﻠﺸﺎﻥ ﻣﻨﺘﻠﻐﺒﻄﺶ ﺑﻌﺪ ﻛﺪﻩ‬

164. You are doing presentation for elderly diabetic patient, what NOT to do
a) Private patient counselling

165. Most ovicidal lice treatment or What give 100% ovacidal activity?
a) Isopropyl myristate
b) Permethrin
c) Pyrethrin
d) Crotamiton
e) Dimethicone
166. Minimum time to keep dimethicone on hair
a) 4 hrs
b) 8 hrs
c) 14 hrs
Dimeticone Noninsecticidal, physically acting agent; penetrates spiracles causing suffocation or inhibition of
50% NYDA water excretion resulting in gut rupture from osmotic stress. Cure rate: 97%
Ovicidal activity: 100% but 2nd application still recommended due to imperfect application.
Spray carefully all-over dry hair. Massage in until hair is completely wetted with solution. Leave
solution on hair. After 30 min, comb hair with a lice comb.
Allow solution to dry on hair for at least 8 h and then wash. Repeat after 8–10 days.
May cause local irritation. Caution around open flames/sources of ignition.
Not recommended for infants or children younger than 2 y.
Resistance to product is unlikely as it has a physical mode of action.
No data on safety during pregnancy and breastfeeding.

167. Scabies patient has ragweed allergy what to use


a) Crotamtion
b) Isopropyl myristate
c) Pyrethrin
Pyrethrins are natural components of chrysanthemum flowers, with strong insecticidal activity; pyrethroids
are synthetic and natural relatives of pyrethrin; and permethrin is a commonly used synthetic pyrethroid.
Pyrethrins are combined with a piperic acid derivative (piperonyl butoxide) to enhance efficacy.

168. Non-Pharm for scabies treatment


a) Wash clothes in 50c before wearing
b) No need to treat Other family memebers

169. Long story about obese lady with diabetic, she asked
if she can use liraglutide, she can not use it because or
Patient given liraglutide what medicine to stop
a) Linagliptin
b) Rosiglitazone
DPP ‐ 4 Inhibitors & Glucagon‐Like Peptide‐1 (GLP‐1) Agonists work on the same system, so, don’t combine them

170. Trial was conducted between 2 drugs: parathyroid hormone and ca


Odd ratio is not used for RCT but used for case control study
171. Patient with hyperkalemia 5.9. ECG normal. What will you give?
a) Insulin plus dextrose
b) Calcium gluconate
c) Sodium polystyrene
d) Lactulose
Cation-exchange Promote exchange of Na+ and Ca++ for K+, respectively, in bowel; and also bind Ca & Mg.
resins Because sodium is exchanged for potassium when sodium polystyrene sulfonate is used,
Ca polystyrene sodium overload (Hypernatremia) may occur.
sulfonate, Used in Patients with serum potassium < 6 mEq/L (< 6 mmol/L) and no ECG abnormalities
Na polystyrene Administer at least 3 hours before or 3 hours after administration of other oral medications
sulfonate Calcium polystyrene sulfonate  Use if sodium overload is a concern.
S.E: Constipating; risk of bowel ulceration or necrosis. Magnesium and calcium can be lost
during treatment, monitor for all applicable electrolyte disturbances.
Avoid administration of sorbitol with calcium polystyrene sulfonate as it causes colonic
necrosis & intestinal obstruction.
The duration of drug contact with the mucosa may be a risk factor. A cleansing enema
(sodium-free) is recommended to reduce this risk.
Avoid with Digoxin, Laxatives such as magnesium hydroxide or aluminium carbonate and
Immunosuppressant drugs.
Lithium & Thyroxine: possible decrease of their absorption.

172. Dr prescribed Na Polystyrene, Due to entry problem, the patient received high dose 12 g of
polystyrene, the technician entered it as 3 times/day instead of 3 times / week. He took it
already, what should be expected?
a) Hyperkalemia
b) Hyercalcemia
c) Hyperphosphatemia
d) Hypernatremia
Sodium polystyrene sulfonate is not absorbed from the gastrointestinal tract. As the resin passes through
the gastrointestinal tract, the resin removes the potassium ions by exchanging it for sodium ions.
The average daily adult dose of the resin is 15 to 60 grams. This is provided by administering 15 grams
(approximately 4 level teaspoons) of KAYEXALATE one to four times daily. Since the in vivo efficiency of
sodium‐potassium exchange resins is approximately 33 per cent, about one third of the resin's actual sodium
content is being delivered to the body
OVERDOSAGE: Biochemical disturbances resulting from overdosage may give rise to clinical signs and
symptoms of hypokalemia, including irritability, confusion, delayed thought processes, muscle weakness,
hyporeflexia, and eventually frank paralysis. Apnea may be a serious consequence of the progression.
Electrocardiographic changes may be consistent with hypokalemia; cardiac arrhythmia may occur.
Hypocalcemic tetany may occur.
Appropriate measures should be taken to correct serum electrolytes (potassium, calcium). The resin should
be removed from the alimentary tract by appropriate use of laxatives or enemas.
173. How to decrease risk for future order daily frequency or What is best way to prevent
further error to happen?
a) Double check the dose every time you dispense this medication
b) A poster you put so all pharmacy staff read about this error
c) Meet all staff and educate them about risk of this type of error on pt
d) Speak with technician who made the error privately
e) POP Up screen on the computer system whenever an Rx of polystyrene is entered on the
system to avoid multiple daily dosing
f) Prevent auto refill for this medication
g) Put fluorescent label on shelf

174. Sodium polystyrene sulphonate it interacts with what (it was a case for a patient with mild
hyperkalemia)
a) Furosemide
b) Levothyroxine
c) Amlodipine
d) Warfarin
e) Metformin
f) Amiodarone
Aluminum hydroxide: intestinal obstruction due to concretions of aluminum hydroxide has been reported
when aluminum hydroxide was combined with the resin.
Digitalis drugs: the toxic effects of digitalis on the heart, especially various ventricular arrhythmias and A‐V
nodal dissociation, are likely to be exaggerated if hypokalemia is allowed to develop.
Non‐absorbable cation‐donating antacids and laxatives: systemic alkalosis has been reported after cation‐
exchange resins were administered orally in combination with non‐absorbable cation‐donating antacids and
laxatives such as magnesium hydroxide and aluminum carbonate.
Lithium: possible decrease of lithium absorption.
Thyroxine: possible decrease of thyroxine absorption.

175. Fever 38.7 in kid, what to NOT counsel patient


a) Avoid excesive cloth
b) Don’t wake up the child to give him medication
c) Wave the child rectally given acetaminophen would cause eradication of disease
d) Erratic rectal absorption
e) Wake child at night and give medicine to maintain fever control
f) Give acetaminophen every 4 hrs
There were 2 things that we shouldn’t do  Not give suppository and don’t give alternative 2 antipyretics
paracetamol and ibuprofen
176. Mother came with 15-month baby and has fever, no history of febrile seizure, which
method is the best to measure his tempreture
a) Rectal
b) Axillary
c) Mouth
d) Frontal

177. Where to find IV compatibility between ceftriaxone and pantoprazole


a) Handbook on injectable drugs
b) Pubmed
IV compatibility American, Hospital practice, Publisher: ASHP; Covers injectable product information,
Handbook on routes of administration, other drug information, and detailed compatibility and stability
Injectable Drugs information on commercial and investigational drugs.
- Trissel Trissel's, King's guide, Online Lexi-Comp and Micromedex provide IV compatibility

178. Patient is asking about traveler’s diarrhea vaccine, her husband has Crohn’s disease and is
controlled for 2 years on azathioprine, what to advice?
a) Vaccine is not routine but recommended for husband
b) Vaccine is recommended
c) Simply wash your hands with soap
Travel to the Indian subcontinent is associated with the highest risk of contracting enteric fever. There are
two available vaccines against S. typhi: the live attenuated oral vaccine containing the S. typhi strain Ty21a
(Ty21a vaccine) and the parenteral capsular polysaccharide vaccine based on the S. typhi Vi antigen (Vi
vaccine). Thus, the Vi vaccine is recommended for IBD patients. It is available for children ≥ 2 years old,
conferring protection 7 d after injection with a maximum neutralizing antibody concentration demonstrated
28 d after vaccination. The gastroenterologist should discuss with the patient the efficacy of the vaccine and
reinforce the necessity of strict food and water precautions. The same control measures are required to
prevent cholera and all diarrheal illnesses. However, when access to clean water and sanitation are not
guaranteed, cholera vaccine should be administered, conferring 85% short‐term protection, and 60%
protection up to 3 years following vaccination. IBD patients taking immunosuppressants should receive the
oral‐killed vaccine licensed in more than 20 countries, including the European Union (Dukoral®). Another
available oral‐killed vaccine (Vabiotech, ORC‐Vax®) was initially licensed only in Vietnam
Vaccines Contains the nontoxic B subunit of cholera toxin, which has significant homology with the
Vibrio cholerae toxin of ETEC and is approved in Canada for prevention of TD caused by ETEC.
whole cell/ Prevention of enterotoxigenic E. coli diarrhea:
recombinant Adults and children ≥2 y: Primary immunization: 2 doses PO; 2nd dose administered within
cholera toxin B 7–42 days after the 1st dose and at least 1 wk before reaching destination
subunit vaccine Booster: 1 dose every 3 months if the risk is continuous
Q. Dukoral Q. Taken orally on an empty stomach (1 h before or 1 h after eating or drinking)
Inactivated S.E: Abdominal pain, diarrhea, nausea and vomiting.
May consider for prevention of TD in persons with chronic illnesses (e.g., HF, insulin-
dependent DM, IBD, chronic kidney disease), or in those with immune suppression
Immunization with DUKORAL should be deferred in the presence of acute gastrointestinal
illness or acute febrile illness to avoid superimposing adverse effects from the vaccine on
the underlying illness or mistakenly identifying a manifestation of the underlying illness as
a complication of vaccine use. A minor illness such as mild upper respiratory infection is
not reason to defer immunization.

179. Fosphenytoin will be out of stock for indifinite period, what to do?
a) Call manufacturer to see alternative and stock up the alternative
b) Automatically shift the orders from fosphenytoin to phenytoin
c) Keep the fosphenytoin for patient that need it for short period
d) Ask when will be available

180. Question about violating Veracity


a) Dr didn’t tell the patient about medication side effect

181. Wife came with Gonorrhea Rx, asks you how she got this disease, you know that her
husband came to the pharmacy last week with gonorrhea Rx, which reply will uphold veracity?
a) It is a sexually transmitted disease
b) Go and ask your husband or discuss with your partner (uphold confidentiality)
c) Ask the doctor
d) Check public health

182. Question about violating Justice


a) To tell father about 16 years old daughter medication
b) To keep medicines that you know it will be out of stock for relatives that have a
prescription

183. Obese Female pt with BMI 32 (weight given and height and you need to calculate BMI)
smoking 2.5 packs/day for one year, HTN and list of unrelevant risk factor for heart. She is 5
out of 10 confident in quitting smoking. What is the best approach for her (precontemplation
and contemplation principles)?
a) Give info about consequences of obesity on heart disease
b) Discuss the treatment option
c) Discuss non-pharma measures like physical acitivity
d) Give motivational material
Contemplation: patient is considering quitting, typically in next 6 months to a year. Motivate and Assist.
 Encourage patients to think about their own pros and cons for smoking versus quitting
 Provide encouragement and positive reinforcement of their desire to quit and reassurance about any
perceived deterrents, e.g., “It is great that you are thinking about quitting. That is the first step
towards success.
184. She comes again asking for NRT specifically nicotine patches. What is the appropriate
patch concentration for her case?
a) 7 mg patch
b) 28 mg patch
c) 14 mg patch
d) 21 mg patch
Q. General dosing instructions involve 6 wk of use of highest strength (21 mg for Nicoderm or Habitrol, 15
mg for Nicorette) followed by 2 wk at the intermediate strength then 2 wk at the lowest strength.
Q. Off‐label uses include total daily doses of nicotine up to 35 mg per day for smokers previously using 21–40
cigarettes a day, and up to 40 mg per day for smokers previously using more than 40 cigarettes a day, with
reported safety and improved efficacy.
Patients must desire to stop smoking and should be instructed to stop smoking immediately as they begin
using NICORETTE INVISIPATCH (Nicotine Transdermal System) therapy. If the patient is unable to avoid
cigarette smoking within 2 weeks of starting treatment, NICORETTE therapy should be stopped, since few
additional patients in clinical trials were able to quit after this time. The duration of treatment should not
exceed 12 weeks. If a dose is missed, the patch should be applied immediately and removed at bedtime. A
new patch should be applied the next morning.

185. Now, she wants bupropion for increasing efficacy of quitting smoking, but her insurance
doesn’t cover it & she can’t pay for it, she then brings Rx for other medication and ask to
replace it with bupropion. He wanted to quit smoking and bring Rx for depression, how the ph
will uphold fidelity?
a) Dispense Rx as new indication
b) Refuse to despense it
c) Call/Fax the Dr to ask him about
d) Don’t dispense and call insurance to tell them about the fraud
e) Change to covered drug like Varenicline “Champix”
it’s a fraud, there’s rules to wht covered and wht not, if you dispensed it for smoking under
depression diagnosis it’s fraud
‫ ﺍﻥ ﺍﻟﺪﻛﺘﻮﺭ ﻛﺘﺒﻪ ﻛﺪﻩ ﻓﻌﻼ ﻋﻠﺸﺎﻥ ﻳﺪﺧﻞ ﺍﻟﺘﺎﻣﻴﻦ ﻭﻫﻮ‬،، ‫ ﺍﻟﺴﺆﺍﻝ ﺩﻩ ﺟﺎﻟﻰ ﺑﺎﻟﻈﺒﻂ ﺑﺼﻴﻐﺔ ﺗﺎﻧﻴﺔ‬for smoking ‫ ﺍﻟﻤﻔﺮﻭﺽ‬،،
‫ﻛﺼﻴﺪﻟﻰ ﺗﺘﺼﺮﻑ ﺍﺫﺍﻯ‬
‫ ﻗﺎﻟﻰ ﺍﻟﻤﻔﺮﻭﺽ ﺍﻟﺘﺎﻧﻴﺔ ﻛﻤﺎﻥ‬،، ‫ ﻗﻠﺘﻠﻪ ﻭﺍﻟﺘﺄﻣﻴﻦ ﻣﺎﻫﻮ ﻣﻤﻜﻦ ﻳﺮﻭﺡ ﻟﺼﻴﺪﻟﻴﺔ ﺗﺎﻧﻴﺔ‬، ‫ﺍﻧﺎ ﺳﺄﻟﺖ ﺍﺳﺎﻣﺔ ﻗﺎﻟﻰ ﻣﺎﺗﺼﺮﻓﻴﺶ ﻭﺧﻼﺹ‬
‫ ﺍﻧﺎ ﺍﻗﺘﻨﻌﺖ ﺑﺮﺃﻳﻪ ﺻﺮﺍﺣﺔ‬،، ‫ ﺍﻟﺼﺢ ﺍﻧﻚ ﻣﺎﺗﺼﺮﻓﻴﺶ ﻭﺧﻼﺹ‬،، ‫ﺗﺮﻓﺾ‬
The ethical principle of Accountability (Fidelity) refers to the healthcare professional’s fiduciary duty to be a
responsible and faithful custodian of the public trust.
Application
Pharmacists and Pharmacy Technicians maintain the public trust by ensuring that they act in the best
interest of their patients and society. In order to fulfill their fiduciary duty to maintain the public trust:
A. Members practice within their scope of practice, in accordance with their Code of Ethics, Standards of
Practice and all relevant legislation, policies and guidelines and only when competent to do so.
B. Members refrain from participating in unethical business practices.
C. Members avoid conflict of interest.
Another one
Bupropion for ttt of smoking cessation, which one is true?
a) Use 1-2 weeks before selecting quit
date
What will be the most effective regimen for
Smoking cessation?
a) Bupropion
b) NRT patch
c) NRT Inhaler plus gum
d) Varenicline

186. Phamacy revenue decreased so manger asks pharmacists to do MED check for patients.
This patient is taking 3 medications but has no DRP and did med check 2 months ago but
because she is taking 3 medications you have to do MED check to be paid by GOVT, what is
the problem?
a) Paternalism
b) Beneficience
c) Professional integrity
d) Conflict of interest

187. Sterile aseptic technique: Laminar flow Hood best way to use it?
a) Use horizontal and vertical for anti cancer medication
b) Operate the hood for 3 hours before working
c) Disinfect every 48 hours energy if not using it
d) Work at least 6 inch inside hood or put items to be prepared 6 inch inside cabinet
e) Disinfect many times per day
f) You can use it sterilize eye drops
g) Turn it on 24 hours before work
Another version
Which of the following is true about biological safety cabinet?
a) Preparation should be done 3 inches away from HEPA filters
b) Preparations should be done 6 inches away from the edge
c) Should be turned on 1 hour prior to using it
d) Should be cleaned once daily
188. 72-year-old patient MI & mechanical heart valve & low Cr Cl, on warfarin, he heard about
new oral treatment, he does not like to INR test, what to do?
a) He should stay on warfarin because of valve
b) Can’t take DOAC because of low Cr Cl
Anticoagulants Superior to clopidogrel plus ASA for prevention of
Vitamin K vascular events in patients with atrial fibrillation at
Antagonists high risk for stroke.
Acenocoumarol Prevent cerebral and systemic emboli in patients
Warfarin with acute MI, valvular and nonvalvular atrial
fibrillation, and prosthetic cardiac valves.
Patients with nonvalvular AF & prior TIA/stroke
require higher target INR of 3 instead of 2.5.
S.E: Bleeding. Skin necrosis. Coumarin
anticoagulants are contraindicated in pregnancy
Many potential interactions. Substrate for CYP2C9
and other isoenzymes.
Warfarin is the preferred coumarin anticoagulant
(extensive published experience).

189. Patient he has history of hypothyroidism & Smoking 30 yrs ago. He takes warfarin in the
morning and ginkgo biloba. Last night, he forgot to take the warfarin in the morning but he took
it at night. Last week his physician prescribed aspirin for him. Yesterday, he was out with his
friends and he had 3 alcoholic drinks. Today in the morning his INR measurement is 4. Which
of the following affects the INR reading?
a) Acute alcohol Intake.
b) Ginkgo Biloba.
c) Late night dose of warfarin or She takes warfarin night then she measures it in the morning
d) Hyperthydoidism
e) Duloxetine
f) Aspirin
190. What to do?
a) Omit 2 doses
b) Give oral vit. K
c) Omit dose and repeat INR before
2nd dose
d) Dec dose by 50%
e) Dec dose and follow up

191. She not want to measure INR, then she


selected Point of care INR to measure it at
home. Where to find info about or which
organization will give best evidence?
a) CADTH
b) ISMP
c) Public health organization
d) NSIR
e) COMPUS
f) CIHI
Tdwar feen 3ala source yedyy recommednations le ghaaz be2es el INR we mokrna bl laboratory
192. Patient prescription for DVT treatment after discharge from hospital, which one attracts
pharmacist attention?
a) Enoxaparin for 5 days no refill
b) Warfarin 2 mg for 5 days no refill
c) Tylenol 30 days no refill
It should be at least for 3 months

193. A long case, pharmacist took verbal prescription, write down .5 mg, what is the reason of
erroring?
a) Leading zero missed
Dose Designations
and Other Information Intended Meaning Misinterpretation Correction
Trailing zero after
decimal point 1 mg Mistaken as 10 mg if the decimal point is not seen Do not use trailing zeros for doses
(e.g., 1.0 mg) ** expressed in whole numbers
“Naked” decimal point Use zero before a decimal point
(e.g., .5 mg) ** 0.5 mg Mistaken as 5 mg if the decimal point is not seen
when the dose is less than a
whole unit
Abbreviations such as mg. mg The period is unnecessary and could be mistaken as the number 1 if Use mg, mL, etc. without a
or mL. with a period written poorly terminal period
following the abbreviation mL
Drug name and dose run Inderal 40 mg Mistaken as Inderal 140 mg Mistaken as Tegretol 1300 mg Place adequate space between
together (especially Tegretol 300 mg the drug name, dose, and unit of
problematic for drug measure
names that end in “l” such
as Inderal40 mg;
Tegretol300 mg)
Numerical dose and unit 10 mg The “m” is sometimes mistaken as zero or two zeros, risking a 10- Place adequate space between
of measure run together 100 mL to 100-fold overdose the dose and unit of measure
(e.g., 10mg, 100mL)
Large doses without 100,000 units 100000 has been mistaken as 10,000 or 1,000,000; 1000000 has Use commas for dosing units
properly placed commas 1,000,000 units been mistaken as 100,000 at or above 1,000, or use
(e.g., 100000 units; words such as 100
1000000 units) "thousand" or 1 "million" to
improve readability
Drug Name Abbreviations Intended Meaning Misinterpretation Correction
194. All of the following is considered a dangerous abbreviation according to ISMP except:
a) Salbutamol 100mcg
b) Alendronate q weekly
c) Heparin 5000 IU once daily
d) Paroxetine 20 mg PO HS
e) Citalopram 20 mg PO QD
f) Insulin lispro 10 units Subcut BID before meals

195. According to ISMP, which one is a safe abbreviation?


a) QD
b) IU
c) HS
d) TID

196. Pt with red itchy patch on his leg, scaly & raised from borders but not from the middle and
clear center, what is the diagnosis?
a) Psoriasis
b) Cellulitis
c) Erythema migrans
d) Atopic dermatitis
e) Tenia

197. The oral agent for Psoriasis is


a) Methotrexate
b) Sulfasalazine
c) 6-MP
d) Azathioprine
198. All of the following requires special consideration to avoid errors in the pharmacy, Except:
a) Insulin (high alert)
b) Methotrexate (high alert)
c) Glyburide (look alike with glipizide: glipiZIDE – glyBURIDE)
d) Estrogen

199. List of Drugs: which one is N controlled targeted Rx?


a) Zolmitriptan tab
b) Testosterone amp (Controlled part III)
c) Hydrocodone tab (Straight N)
d) Clonazepam (targeted substance)
e) Fentanyl (Straight N)
f) Methamphetamine (Controlled part I)

200. When two drugs are biosimilar, this means that both drugs are?
a) Same dose, strength & route
b) They are similar to generics & they should be considered equivelent to innovative brand
c) They need bioequivelent studies to show their equivelency to their innovative brand
d) The manufacture should have their own clinical studies that shows drug efficacy to
be interchangeable
e) Can be done on automatic substitution for a brand
f) Can be compared to the brand in clinical trials.
Another version: Biosimilars, what is correct?
a) Biosimilars has the same active ingredient, AUC & Cmax
b) Biosimilars are interchangeable with biologics once approved
c) Biosimilars has the same DNA sequence of originating biologics
d) Biosimilars requires new clinical trials in order to approved by HC
Biosimilars or subsequent entry biologics (SEBs)
 Biosimilars (previously called subsequent entry biologics) are biologic drugs that are made by a
different manufacturer than that of the reference (innovator) biologic.
 Unlike generic drugs, biosimilars are not identical to the reference biologic drug and are therefore
not necessarily interchangeable.
 A biological drug is a large protein molecule that is produced through recombinant DNA technology.
 The manufacturing process is propriety and unique; it is impossible to exactly replicate the original
molecule and therefore biosimilars should not be considered generic medicines.
 Biosimilars have a separate approval process through Health Canada that requires clinical trials to
demonstrate their benefit and safety prior to being approved for a specific indication
201. Which medication need narcotic label (all were narcotic)?
a) Codeine 8mg/ Acetaminophen/ Butalbital/ Caffeine
b) Phenobarbital/ Codeine/ Caffeine/ ASA
c) Buprenorphine
d) Tramadol
e) Dexamphtamine
f) Fiorenal C

202. Expired drug that does not need witness for destruction?
a) Ketamine
b) Nabilone
c) Butorphanol
d) Clonazepam
a) Clozapine

203. Maximum allowed time for zolpidem according to health Canada or Maximum days of
zopiclone prescription
a) 10 days
b) 1 month
Q. Although not a benzodiazepine, the cyclopyrrolone zopiclone acts at the benzodiazepine receptor
Zopiclone and has similar therapeutic and adverse effects
Adults: Initial dose: 3.75 mg HS PO. If needed, titrate to 5 –7.5 mg HS PO. Maximum 7.5 mg/day.
Geriatrics: 3.75 mg HS PO. Increase to 5 mg once daily if needed and tolerated.
Maximum 5 mg/day in elderly, patients with hepatic, renal impairment, or taking a strong CYP3A4
inhibitor.
The lowest dose for shortest duration is recommended. Treatment should rarely exceed 7–10 days.
Re-evaluate the patient if treatment beyond 2–3 weeks is required.
Due to rapid onset, administer just before retiring at night when there is sufficient time for a full
night's sleep. Use lowest effective dose. Use only when full night’s sleep is possible.
Advise patients of risk of next-day impairment, even if feeling fully awake.
Allow at least 12 hours between bedtime dose and any activity requiring mental alertness, e.g.,
driving. Advise patients to wait ≥12 h before driving or operating machinery.
S.E: Impaired cognitive function and, rarely, anterograde amnesia and transient global amnesia.
Bitter/metallic taste. May cause dosedependent, next-day impairment of activities requiring
alertness, including driving a car, despite the patient feeling fully awake.
Minimal additive effects with low doses of alcohol.
CYP3A4 inhibitors (e.g., clarithromycin, itraconazole, ritonavir) may increase its plasma levels.
CYP3A4 inducers (carbamazepine, phenytoin, St. John’s wort) decrease zopiclone plasma levels.
Monitor patient for increased or reduced response and adjust dose accordingly.
Does not accumulate; tolerance may be delayed and rebound insomnia may be reduced.
Eszopiclone, the active (S+) isomer of zopiclone, shares similar pharmacologic properties to the
racemic compound but is more potent. Not available in Canada.
204. Calculation: patient is taking infusion 500 Q 8 hrs concentration was 57 but the peak
should be 20, for how long do we have to stop the infusion?
Most probably it will be solved by Log C = log C0 –kt /2.303

205. CDR: acknowledge


CADTH Common Drug Review (CDR)
 Reviews drugs and makes reimbursement recommendations to Canada's federal, provincial, and
territorial public drug plans, with the exception of Quebec, to guide their drug funding decisions.
 Conducting reviews of the clinical, cost-effectiveness (pharmacoeconomic analysis), and patient
evidence for drugs and providing formulary listing recommendation.
 Gives options for drug prices to compare between.
 After drug approval in Canada, Federal and P/T public plans must decide whether the drug will be
eligible for public reimbursement  CDR provides drug reimbursement recommendations for listing (or
for not listing) and advice to federal, and P/T public drug plans.
 Food and Drug Act requirements must be met. Otherwise, a marketing authorization is NOT granted and
re-submission is needed. After approval, regulatory controls and post-market surveillance continues.
 National review in which the drug sponsor requests a CDR to have the patented drugs covered by the
publicly funded drug plans; administered by CADTH
 CDR submission eligibility: Any new drug, new combination products, drugs with new indications that
have received NOC or a NOC with conditions (NOC/c) and new drugs with a pending NOC or NOC/c

206. Patient take infusion over 30 minutes every 12 hours, after one hour of infusion start C was
x, and 30 minutes before next dose the C was Y, doctor want the concentration to be Z, how
long to you have to wait before giving the next dose?
Not enough data

207. Diabetic patient, has depression, penicillin allergy, Smoker for 30 years, suffer from
cough, wheezing and emphysema. He is diagnosed with asthma & COPD exacerbation, there is
no previous exacerbation or hospital visit. He also has Qt prolongation, atrial Fibrillation, what
to give?
a) Amoxycillin
b) Levofloxacin
c) Doxycyclin
Group Symptoms & risk factors Probable pathogen First choice antibiotic
Simple < 4 exacerbations in the past year M. catarrhalis  Amoxicillin
exacerbations Increased sputum purulence + at S. pneumonia  Doxycycline
(COPD least 1 of: Haemophilus spp.  SMX/TMP
without risk 1) Increased sputum volume All for 5–7 days
factors) 2) Worsening dyspnea
Complicated As in simple exacerbation + at M. catarrhalis Preferred:
exacerbations least 1 of: S. pneumonia  Amoxicillin/clavulanate
(COPD with 1) FEV1 < 50% predicted Haemophilus spp.  Cefuroxime axetil
risk factors) 2) ≥ 4 exacerbations per year Klebsiella spp. Both for 5 – 10 days
3) Ischemic heart disease Other gram-negative  Levofloxacin × 5 days
4) Use of home O2 pathogens Alternative:
5) Chronic oral corticosteroid Pseudomonas spp.  Azithromycin × 3 days
6) Antibiotic use in previous Higher probability of  Clarithromycin × 5–10
3 months betalactam resistance days

208. Now, he has > 4 exacerbation in the year & he was already taking doxycycline before,
what to give?
a) Ciprofloxacin
b) Levofloxacin
c) TMP-SMX
d) Amoxicillin/clavulanate

209. Duration of Dexamethasone dose for the exacerbation of COPD?


a) 3 days
b) 5 days
c) 7 days
d) 10 days
Severe COPD patients gain additional benefit from a short course of systemic corticosteroids. Oral
corticosteroids improve lung function and shorten length of hospital stay in all patients and reduce risk of
early relapse or hospitalization for subsequent acute exacerbations
Oral corticosteroids should be given within 30 days of an acute exacerbation to reduce the risk of further
exacerbations. Administration beyond 30 days is not recommended.
A 5‐day course of oral prednisone 30–40 mg/day or equivalent is sufficient and has shown outcomes
comparable to a 10‐ to 14‐day course. Tapering is unnecessary for oral CS courses that last <2 weeks.
There is no role for oral corticosteroid maintenance therapy for patients with COPD. There is no advantage
to using IV corticosteroids.
210. He is using on salbultamol 2 puff Q4-6H and ICS, this patient in need for
a) Symbicort
b) Fluticasone
c) Ipratropium
d) Tiotropium
Long‐Acting Muscarinic Antagonists
The LAMA tiotropium is a first‐line agent for managing persistent symptoms and moderate to severe airflow
obstruction, as it decreases exacerbations and hospitalizations.
A single daily 18 mcg dose via a breath‐activated DPI or two 2.5 mcg inhalations (total dose of 5 mcg) via
SMI gives maximal anticholinergic activity for a full 24 hours.
When compared with ipratropium, more sustained effects on pulmonary function, activity‐related dyspnea,
and quality of life, as well as fewer acute exacerbations, were noted. When given in combination with
pulmonary rehabilitation, tiotropium improved treadmill walking endurance time.
Retrospective studies have shown an increased risk of cardiovascular death, MI or stroke associated with
both ipratropium and tiotropium. This was not confirmed in a 4‐year randomized controlled trial that
evaluated efficacy of tiotropium and also included safety parameters as secondary endpoints.
The FDA and others have extensively reviewed available data and have concluded there is no increased risk
of stroke, MI or death associated with tiotropium DPI. However, ipratropium appears to be associated with
an increase in cardiovascular events.
Glycopyrronium is a rapid‐onset LAMA delivered via DPI. It has a faster onset of action than tiotropium. The
currently licensed dose is 50 mcg once daily. Phase III studies have established improvement in dyspnea and
quality of life and reduced risk of exacerbation compared with placebo; when compared with tiotropium,
similar efficacy and safety data were observed. The bronchodilation effect is seen on day 1 of
administration. Improvements in exercise endurance and quality of life are seen over time.
The LAMA aclidinium is dosed at 400 mcg twice daily and has effects similar to those of tiotropium and
glycopyrronium on lung function.
Umeclidinium is a once daily LAMA delivered via DPI. A Cochrane review found that it reduced the likelihood
of developing moderate exacerbations requiring oral steroids and/or antibiotics (NNT = 18); however, a
reduction in exacerbations requiring hospitalization was not observed. This therapy also reduced the
frequency of rescue inhaler use while improving quality of life, COPD‐related symptoms and lung function.
All LAMAs have been found to reduce the rate of acute exacerbations leading to hospitalization.
Check the patient inhaler technique of Spiriva and decide which steps to be corrected
Tiotropium It has been shown to have value as an add-on therapy to improve lung function and
Long-acting decrease exacerbations in patients with symptomatic asthma despite maintenance
(LAMAs) treatment with high-dose ICS + LABA, medium-dose ICS, low-dose ICS and
R
Spiriva medium-dose ICS ± LABA.
Respimat Soft mist inhaler SMI: 2.5 mcg/actuation: 2 actuations inhaled once daily.
Q. Prime befor use. Breathe out slowly and fully. Close your lips around the
mouthpiece without covering the air vents. Point the inhaler towards the back of
throat. While taking in a slow, deep breath, PRESS the dose release button and
continue to breathe in slowly.
S.E: Dry mouth, metallic taste; mydriasis and glaucoma if released into eye
211. Uncontrolled COPD measured by?
a) Nocturnal symptoms
b) Days absent from work
c) mMRC Modified Medical Research
Council
d) GINA

Other versions:
COPD taking genuair aclidinium bromide? know the use
How to use Genuair:
 Remove the cap from the inhaler.
 Press the coloured button all the way down and release.
 Check that the colour control window is green. This means that the inhaler is ready.
 Breathe out fully, away from the inhaler.
 Seal your lips around the mouthpiece and breathe in strong and deep.
 Keep breathing in, even after you have heard the inhaler ‘click’.
 Remove the inhaler from your mouth and hold your breath for 5‐10 seconds.
 Breathe out.
 Check that the colour control window has turned from green to red. This tells you if you have inhaled
correctly. If the window has not turned red, breathe in another strong, deep breath
 Replace the cap.

COPD use Titropium not affect change to what? LAMA + LABA comb
Patient with depression and copd and penicillin allergy he is using lama/ laba/ ics. He got
COPD exceserbation, which antibiotic to give
a) Amox
b) Cephalexin
c) Doxycylin

Dr need to add another drug to his copd medication regimen:


a) Rufimilast
b) Prednisone
c) Thiophylline

212. AF patient age above 65, on apixaban 5 mg bid, diabetic, and came with RX after PCI for
STEMI, resident calling to take your advice for prevention of future MI, your answer is?
a) Stop apixaban 5mg and start ASA and clopidogrel
b) Add ASA and Clopidogrel to apixaban
c) Add ASA to Apixaban
d) Add clopidogrel to apixaban

213. Case of chronic bacterial prostatis CBP, what is the duration of the ttt or Case about
treatment for prostatitis outpatient, Treatment should be
a) Ciprofloxacin 4 week
b) Tamsulsin 4 weeks
c) NSAIDs 6 weeks
d) SMX/TMP 4 weeks
Chronic E. coli (80%), Klebsiella spp, P. aeruginosa, Fluoroquinolone SMX/TMP
bacterial Proteus spp, other PO × 4–6 wk PO × 4–6 wk
prostatitis Common cause of recurrent UTIs in older men;
CBP increases with age. Intermittent urinary infection
presenting as cystitis; history of recurrent UTIs.
Acute bacterial E. coli, Enterobacteriaceae, P. aeruginosa, Aminoglycoside Fluoroquinolone IV
prostatitis Staphylococcus aureus, others IV ± cloxacillin IV or PO × 4 wk
ABP Acute onset chills, fever, perineal and low back ± ampicillin (if or
pain, irritative and obstructive voiding. Enterococcus is a SMX/TMP PO × 4
The prostate is tender, swollen, indurated and concern) wk
warm. IV Cloxacillin is
Prostatic massage is not recommended because it useful only if S.
may cause bacteremia. aureus is known to
Obtain urine specimen before initiating antibiotic be present.

Another version: the question came without diagnosis, only symptoms like Acute onset chills,
fever, perineal and low back pain, irritative and obstructive voiding. The prostate is tender,
swollen, indurated and warm. And asked about the suitable treatment?
a) Ciprofloxacin 4 week
b) Tamsulsin 4 weeks
c) NSAIDs 6 weeks
d) SMX/TMP 4 weeks

‫ﺑﺼﺮﻑ ﺍﻟﻨﻈﺮ ﻋﻦ ﺍﻟﺴﺆﺍﻝ ﺍﻟﺤﺘﻪ ﺩﻱ ﻣﻬﻤﻪ ﻳﻌﻨﻲ ﻟﻮ ﺟﺎﻟﻚ ﺍﺧﺘﻴﺎﺭﻳﻦ ﻳﻨﻔﻌﻮﺍ ﺍﺑﻌﺪ ﻋﻦ ﺍﻟﻔﻠﻮﺭﻭﻛﻴﻨﻮﻟﻮﻥ ﻭﺍﺧﺘﺎﺭ ﺍﻟﺘﺎﻧﻰ ﻓﻰ ﺣﺎﻟﺔ ﺍﺫﺍ‬
.‫ﻛﺎﻧﻮ ﻣﺘﺴﺎﻭﻳﻴﻦ ﻳﻌﻨﻰ ﺍﻻﺗﻨﻴﻦ ﻓﺮﺳﺖ ﻻﻳﻦ ﺍﻭ ﺳﻜﻨﺪ ﻻﻳﻦ‬

214. Frail old man has low body mass & comes asking about zostavax vaccine, where to
administer vaccine?
a) Anterolateral thigh at 90
b) Anterolateral thigh at 45  zostavax injected SC
c) Buttock at 90
d) Buttock at 45
Vaccines given SQ (subcutaneous) route: MMR, MMRV, VAR, and ZVL (Zostavax).
Vaccines given IM (intramuscular) route: DTaP, DT, Hib, hepA, hepB, HPV, IIV, MCV, PCV, rabies, Td, Tdap
and RZV (Shingrix).

215. Rheumatoid arthritis patient on Methotrexate 0.15 mg/ ml, wants to take shingrix shingles
vaccine, what to advise her:
a) No, it is Contraindicated
b) You can take it safely
c) Wait for at least 1 month and take it
d) Wait for at least 3 month and take it
Live vaccines available in Canada that are contraindicated in immunosuppressed IBD patients include
intranasal influenza, measles‐mumps‐rubella (MMR), smallpox, oral typhoid, yellow fever and varicella.
Live vaccines should not be given to patients using immunosuppressive therapy until 3 months after these
therapies are stopped. While herpes zoster is also a live vaccine, it is not used to elicit a primary immune
response; so, it may be considered safe to administer to patients receiving low‐dose immunosuppressive
therapy: methotrexate ≤0.4 mg/kg/week, azathioprine ≤3.0 mg/kg/day, 6‐mercaptopurine ≤1.5 mg/kg/day,
prednisone <20 mg/day (or equivalent).

216. what to monitor after 2-4 weeks of starting MTX


a) ALT
b) ALP
Baseline assessment should include complete blood count (CBC) with differential and platelet counts,
hepatic enzymes, renal function tests and chest x‐ray. Monitor hematology at least monthly, and hepatic
enzymes and renal function every 1–2 months. Transient liver function test abnormalities are observed
frequently after methotrexate administration and are usually not cause for modification of therapy.
Persistent liver function test abnormalities and/or decreased serum albumin may be indicators of serious
liver toxicity. If symptoms of possible methotrexate‐induced lung disease present (persistent dry,
nonproductive cough and/or dyspnea, with or without fever), pulmonary function tests and chest x‐ray may
be considered. Radiograph abnormalities occurring with methotrexate therapy include interstitial and
alveolar infiltrates, hilar adenopathy and pleural effusion, occasionally progressing to fibrosis, scarring and
honeycomb changes.

217. She is still experiencing pain,


what to do?
a) Give azathioprine with MTX
b) Give hydroxychloroquine with
MTX
c) Change to etanercept
d) Infliximab
e) Leflunomide
After MTX, we make triple therapy or add
biologics
TNF‐alpha inhibitors (TNFi) were the first
biologics introduced for the treatment of
RA and include adalimumab, certolizumab,
etanercept, golimumab and infliximab.

218. Same patient after adding the 2nd


medication, he is still using naproxen a lot, what to do?
a) Use ustekinumab
b) Use etanercept
Patients with inadequate or loss of response to TNFi therapy may be switched to a biologic with a different
mechanism of action or to another TNFi. There is low‐quality evidence that switching to a biologic with a
different mechanism of action is preferred over a second TNFi in patients with inadequate response to the
first TNFi. Patients with inadequate response to the second TNFi should be switched to a biologic with a
different mechanism of action.
TNF‐alpha inhibitors (TNFi) were the first biologics introduced for the treatment of RA and include
adalimumab, certolizumab, etanercept, golimumab and infliximab.
Tocilizumab and sarilumab are interleukin‐6 (IL‐6) inhibitors indicated for patients with moderate to severe
disease who have an inadequate response to csDMARDs. Anakinra blocks the interleukin‐1 (IL‐1) receptor.
Other Biologics: abatacept & rituximab

219. What is isde effect of ustekinumab? Vulvovaginal Infection


Used for Plaque Psoriasis, Psoriatic Arthritis & crohn’s disease
>10%:
Immunologic: Antibody development (3% to 12%; associated with reduced efficacy in psoriasis patients)
Infection: Infection (psoriasis: 27% to 72%; infection, severe: ≤3%) Respiratory: Nasopharyngitis
1% to 10%:
 Dermatologic: Acne vulgaris (Crohn disease: 1%), pruritus (2% to 4%)
 Gastrointestinal: Abdominal pain (ulcerative colitis: 7%), dental disease (infection: 1%), diarrhea
(ulcerative colitis: 4%), nausea (3%), vomiting (Crohn disease: 4%)
 Genitourinary: Urinary tract infection (Crohn disease: 4%), vaginal mycosis (Crohn disease: ≤5%),
vulvovaginal candidiasis (Crohn disease: ≤5%)
 Hematologic & oncologic: Malignant neoplasm (excluding nonmelanoma: ≤2%), skin carcinoma
(nonmelanoma including squamous cell carcinoma; psoriasis: 2%)
 Infection: Influenza (ulcerative colitis: 6%). Local: Erythema at injection site (1% to 5%)
 Nervous system: Depression (psoriasis: 1%), dizziness (psoriasis: 2%), fatigue (3% to 4%), headache
 Neuromuscular & skeletal: Arthralgia (psoriatic arthritis: 3%), asthenia (Crohn disease: 1%), back
pain (psoriasis: 2%)
 Respiratory: Bronchitis (Crohn disease: 5%), pharyngolaryngeal pain (psoriasis: 2%), sinusitis (3% to 4%)
 Miscellaneous: Fever (ulcerative colitis: 5%)
Monitoring Parameters:
Tuberculosis screening (prior to initiating and periodically during therapy); CBC; ustekinumab‐antibody
formation; monitor for signs/symptoms of infection, reversible posterior leukoencephalopathy syndrome
(RPLS), and squamous cell skin carcinoma.

220. She complains that she cannot do normal chores at home, you will refer her to:
a) Social worker
b) Occupational therapist
c) Physician
d) Nurse
221. Case about pt has symptoms in his/her joint/s, which symptom will help to differentiate
rheumatoid arthritis over osteoarthiritis? Not sure about the options (make sure you know the
symptoms of all of them)
a) One joint
b) Swollen and inflamed joint
c) Multiple joints involvement
d) Forgot this
Osteoarthritis Rheumatoid Arthritis (RA)
Degenerative joint disease caused by a breakdown  A chronic systemic autoimmune disease manifesting
of the cartilage between bones, and degradation of as a symmetric and erosive polyarthritis.
articular cartilage in synovial joints.  Occurs when body’s immune system attacks tissue
Osteoarthritis can impact any joint but typically lining and results in joints causing cartilage to erode.
occurs in the spine (cervical or lumbar spine facet  It can cause pain, stiffness and fatigue. Joint
joints), hands, hips or knees. destruction resulting in disability and premature
Mainly effects on weight bearing joints mortality.
Patients with OA suffer mostly from pain,  Onset can occur at any age, including childhood, but
stiffness, discomfort and joint function most frequently starts between ages of 40 and 50.
impairment.  Affects women 3 times more frequently than men.
Investigations  Patients with RA have an increased risk of CV
 History: joint stiffness usually lasting <30 disease and CV mortality and depression.
minutes, with joint pain and/or dysfunction  Patients with RA typically have multiple swollen
on certain movements, absence of signs or joints, in a symmetric distribution, with involvement
symptoms of other types of inflammatory of the hands and feet.
arthritis.  The diagnosis of RA should be considered in
 Physical examination: pain, stiffness and patients with joint swelling or inflammatory joint
limitation of both passive and active pain, although the differential diagnosis of joint
movement of joint, crepitus, deformity, swelling is broad. To make a diagnosis of RA,
muscle atrophy, ligament tenderness. objective joint swelling (by clinical exam or imaging)
 Imaging: x-rays and diagnostic ultrasound, must be present. Patients with RA typically have
Chondrocalcinosis (calcium pyrophosphate multiple swollen joints in a symmetric distribution
dihydrate [CPPD] deposition) may be seen with involvement of the hands and feet. Patients
in the joint cartilage areas of patients with also report morning stiffness greater than 30 minutes.
some metabolic disorders which predispose Investigations:
to secondary OA, MRI and CT scan used if  + ve Anti-CCP = antibody to cyclic citrullinated
MRI contraindicated. protein, CBC, ↑ CRP, ↑ ESR, +ve RF = rheumatoid
 Laboratory tests: blood work for factor, liver and renal blood panel, Radiographs of
inflammatory markers (e.g., CBC, - ve hands and feet.
rheumatoid factor, normal ESR & CRP) if Physical examination:
other inflammatory arthritides are  Number of actively inflamed/swollen joints.
suspected, joint fluid analyses.  Mechanical joint problems: loss of motion, crepitus,
instability, malalignment and/or deformity.
 Extra-articular manifestations: dry eyes, nodules,
pulmonary findings, carpal tunnel syndrome.
222. OA case: 78 years old patient, no smoking and her weight is normal. She has OA in both
Knees, what is first choice?
a) Topical diclofenac
b) Diclofenac capsule
c) Capsaicin patch
Standard pharmacological OA management involves a stepwise approach consisting of initial use of topical
analgesics, followed by oral analgesics (e.g., acetaminophen, NSAIDs, opioids) and injectable agents.
Topical analgesics are an alternative treatment option, with the potential for a reduced risk of side effects
compared to oral therapy. Topical NSAIDs may be especially useful in at‐risk groups including the elderly,
who are at an increased risk of GI, renal, cardiovascular and other NSAID adverse effects. Topical NSAIDs
have been shown to be superior to placebo and equal to oral NSAIDs in the treatment of OA in a subset of
patients. The most common side effects are cutaneous reactions.

223. what non pharmacological advice to tell


her to do?
a) Acupuncture
b) Stretching exercise
Exercise and Physiotherapy: are the key initial
management strategies in OA.
Exercise programs can also reduce pain, improve physical function and reduce the need for total hip
replacement by 44% in patients with hip OA. Aquatic exercise can provide short‐term benefits for hip and
knee OA pain. The benefits of swimming can be similar to cycling exercises
Acupuncture around an affected joint may offer temporary relief but the effect may not be sustained.

224. She returns back complains of too much pain although she takes almost 4 gm
acetaminophen daily, what is your recommendation:
a) Celecoxib
b) Naproxen
c) Intra-articular Hyaluronic acid
d) Intra-articular Corticosteroids
Naproxen has a higher risk of GI
adverse effects than ibuprofen, but
have the lowest risk among NSAIDs
for cardiovascular events.
In monograph, it is used with
gastroprotection.
225. Counselling for cyclosporine eye drops
RESTASIS (cyclosporine) ophthalmic emulsion, 0.05% w/v is indicated for the treatment of moderate to
moderately severe, aqueous deficient dry eye disease, characterized by moderate to moderately severe:
ocular staining, reduction in tear production and fluctuating visual symptoms, such as blurred vision.
Contraindications:
Patients who are hypersensitive to this drug. Patients with active ocular infections. Patients should be
advised to avoid touching the tip of the vial to the eye or any surface, as this may contaminate the emulsion.
RESTASIS should not be administered while the patient is wearing contact lenses. If contact lenses are worn,
they should be removed prior to the administration of the emulsion. Lenses may be reinserted 15 minutes
after the administration of RESTASIS.
The most common adverse event following the use of RESTASIS (cyclosporine) ophthalmic emulsion, 0.05%
w/v is ocular burning. RESTASIS may cause transient blurred vision due to its emulsion formulation.
Recommended dose is one drop of RESTASIS, instilled twice a day in each eye approximately 12 hours apart.
If a dose of this medication is missed, it should be taken as soon as possible. However, if it is almost time for
the next dose, the missed dose should be skipped and the regular dosing schedule resumed. Doses should
not be doubled. The dose should not exceed two drops in the affected eye(s) daily.
Before using, gently shake the vial by tipping it up and down a few times until the emulsion is white and
appears the same throughout the vial. RESTASIS may be used together with artificial tears. Wait 15 minutes
between using RESTASIS and the artificial tear product.
Each individual, single‐use vial should be used immediately after opening for administration to one or both
eyes, and the remaining contents discarded immediately after administration.
Should be stored at 15‐25°C. Patients should be instructed to keep unused vials within the resealable tray.

226. Question about zostavax and shingrex, which from the below is right
a) Shingrix is live attenuated vaccine (recombinant vaccine)
b) Zostavax is 2 dose vaccine (single shot)
c) If you get zostavax then Shingrex will maximize the Benefit and protection
d) Shingrix is two doses only
e) They are Schedule 3 (schedule 2, BTC)
Vaccines, viral
varicella virus Prevention of varicella Avoid in pregnancy.
vaccine, live (chickenpox) Adverse effects: local pain, swelling, redness. A rash, resembling
attenuated Dosage: varicella zoster virus infection, may occur, although it is generally
Varilrix, ≥12 months: 2 doses minor and self-limited.
Varivax III SC. The second shot The need for booster doses is unclear, and still under study.
should be given at Do not administer the vaccine to immunocompromised patients
ages 4-6 years. (especially those with T-cell immunodeficiency).
Q. Prevention of herpes Avoid in pregnancy. Not recommended in pediatric patients.
zoster virus zoster (shingles) Adverse effects: local pain, swelling, redness. A rash, resembling
vaccine, live Dosage: varicella zoster virus infection, may occur, although it is generally
attenuated ≥ 60 y: 1 dose SC minor and self-limited.
Zostavax II Can be considered for The need for booster doses is unclear, and still under study.
those ≥ 50 y Do not administer the vaccine to immunocompromised patients
(especially those with T-cell immunodeficiency).
Can coadministered (in different site) with pneumococcal vaccine
Can be given at least 1 y following last episode of herpes zoster
Zostavax II should be stored refrigerated at a temperature of 2 to
8°C or colder until it is reconstituted for injection.
Discard if reconstituted vaccine is not used within 30 minutes.
zoster virus Prevention of herpes No data on use in pregnant women.
vaccine, nonlive zoster (shingles) Adverse effects include pain, redness and swelling at the injection
recombinant, Dosage: site; myalgia, fatigue, headache, shivering, fever, and nausea,
adjuvanted ≥50 y: 2 doses IM vomiting, diarrhea and/or abdominal pain.
Shingrix (0.5 mL administered The need for booster doses following the primary vaccination
at 0 and 2–6 months schedule has not been established.
IM) Limited data on use in immunocompromised adults.
Wait a minimum of 8 weeks after a persreceived Zostavax to give
Shingrix.
 zostavax — 1 year — shingrix,
 shingrix — 8 weeks — zostavax,
 shingles — 1 year — zostavax
Another version
52-year-old patient came to the pharmacy to take a Shingrix vaccine for herpes zoster. While he
was a child, he got chicken pox. What will you tell him?
a) Shingrix is recommended for patients 60-year-old of age or older
b) Shingrix is taken in 3 doses 0 & 2-6month regimen
c) Shingrix could be taken one year after zostavax
d) No need for Shingrix vaccine as the patient had chickenpox before

226. Infective endocarditis caused by MRSA, what is the dose of vancomycin?


a) 1500 IV loading doses, followe by 1g twice daily and get sample after 2 days to check
trough between 10-15
b) 1500 IV loading doses, followe by 1g every 8hrs and get sample after 4 days to check
trough between 10-15
c) 1500 IV loading doses, followe by 1g twice daily and get sample after 2 days to check
trough between 15-20
d) 1500 IV loading doses, followe by 1g TID and get sample after 4 days to check trough
between 15-20
Vancomycin Sampling Peak: 25–40 mg/L. Dosage should be stable for 20–
IV Peak: 30 min–2 h after 1-h infusion. Trough: 15–20 30 h.
Trough: ≤ 5 min prior to next dose. mg/L. Time to steady state: 32 hr.
Usually obtain trough levels. Half-life about 6 h in patients
Resampling: samples taken on day 2 or with normal renal function,
3 of treatment, repeated weekly. prolonged in RF
Pathogen Regimen Duration Comments
Antibiotic treatment regimens for staphylococcal endocarditis 40%
Staphylococci - No Prosthetic Material
Methicillin - Cloxacillin or 6 wk May use 2 wk  if the infective endocarditis is
sensitive Cefazolin uncomplicated and right-sided only.
Methicillin - Vancomycin or 6 wk Vancomycin trough concentrations of 10–20 mg/L
resistant Daptomycin as suggested. Drug of choice for treatment of MRSA IE and in
an alternative in patients with a type-1 hypersensitivity reaction to beta-
selected patients lactams
Staphylococci - Prosthetic Valve
Methicillin - Cloxacillin or ≥ 6 wk Gentamicin peak and trough concentrations are suggested to
sensitive Cefazolin + First 2 wk be maintained at 3–4 mg/L and <1 mg/L respectively
Rifampin +
Gentamycin
Methicillin - Vancomycin + ≥ 6 wk Vancomycin 15 mg/kg Q12H IV
resistant Rifampin + First 2 wk trough concentrations of 10–15 mg/L suggested
Gentamycin

227. How to store vaccine in fridge or How to manage a vaccine in the fridge, to keep it at its
proper temperature
a) Put vaccine in cardboard in fridge
b) Put filled water bottles in the door and lower shelf with label on them not for use
c) Keep it in a bar size fridge, just for
pharmaceuticals
d) Adjust the temp from 0-8C
e) Regular size fridge but only keep it
in the door
f) Remove any frost from the fridge
g) Measure temprature twice a week

228. Which medicine


associated with
photosensitivity
a) TMP-SMX
b) Minocycline
229. Nail fungal infection case (Onychomycosis) and patient is using many drugs and lots of
details. Dr prescribed him oral terbinafine, which one is baseline monitoring?
a) ALT
b) Nausea, vomiting
c) Headache
Liver function tests at baseline and periodically during treatment; CBC (if used >6 weeks; immunosuppressed
patients only); taste and/or smell disturbances

230. Terbinafine councel. Choose the correct statement regarding terbinafine tablets
a) With or without food at the same time per day
b) Avoid alcohol
c) Their main side effects are GIT side effects 5%.
d) It is okay to still keep breastfeeding while on terbinafine tablets
e) Terbinafine tablets can be used in pregnant women
f) Store this medication in the fridge
g) Should withhold statin for the duration of therapy with terbinafine. (no interaction)
Allylamine More effective for dermatophyte OM (tinea unguium) with fewer serious side effects and drug
Antifungals interactions than itraconazole or fluconazole.
Duration: Fingernails: 6–12 wk. Toenails: 12– 24 wk. Children: 10–<20 kg: 62.5 mg daily PO
Terbinafine 20–40 kg: 125 mg daily PO. Adults & Children > 40kg: 250 mg daily PO.
Lamisil, S.E: Gastrointestinal upset, headaches, minor rashes, sensory loss of smell or taste, hearing
disturbances and rarely, serious or fatal hepatotoxicity.
Mycological Cimetidine decreases and rifampicin increases systemic clearance of terbinafine.
cure rate = Terbinafine inhibits CYP2D6 enzymes and may decrease serum concentration of cyclosporine
70%. and formation of tamoxifen's active metabolites.
Clinical cure May increase serum concentration of atomoxetine, bupropion, fluoxetine, paroxetine, tramadol
rate = 38%. and TCAs (monitor for toxicity). May decrease therapeutic effect of codeine.
Q. Obtain baseline and mid treatment serum aminotransferase level.
Continuous terbinafine regimen is more effective than pulse regimen.
Terbinafine hydrochloride is excreted in breast milk; therefore, mothers receiving oral
treatment with TERBINAFINE should not breast feed.
Unless the potential benefits outweigh any potential risks, oral terbinafine hydrochloride
should not be used during pregnancy.
Another version
Terbinafine better taken to improve absorption?
a) With lipids
b) Low lipids diet
c) Not affected.
Administer tablets without regard to meals. Administer granules with food; sprinkle granules on a spoonful
of pudding or other soft, nonacidic food (eg, mashed potatoes); swallow entire spoonful without chewing;
do not mix granules with applesauce or other fruit‐based foods.
231. Patient with hurt burn; prescribed PPI. When to assess?
a) After 1 week
b) After 1 month
c) After 3 months
Initial pharmacologic management of dyspepsia in the absence of alarm symptoms focuses on testing for H.
pylori and eradicating it if present, or empirically treating symptoms with a standard‐dose PPI for 4–8
weeks.

232. KL 35-year-old female has symptoms of heartburn with endoscopy negative H. Pylori. He
was on Omeprazole 40 mg. Symptoms relieved after 8 weeks of treatment. One week later he
came back to the pharmacy with the same symptoms. What will be your initial response?
a) Send a request to the Dr. to repeat the same 8-week course
b) Send patient to repeat endoscopy
c) Send a request to the doctor for a step-down PPI treatment
d) Double the dose
233. Prescription for antibiotic suspension. Total rx was something which was rounded up to
5.25 gm. Of drug. In the dry powder when u add 127 ml water it gave 50 mg/ml conc. Of total
150 ml. Father wants 350mg/5 ml as he was concerned about large volume to be govien to the
child. How you ll prepare?
150 ‐127 = 23ml displacement.
That 23ml was 50mg/ml = 23 * 50 / 150ml = 7.67g round to 7.5g of powder.
Cross multiply: 7.5g / 23ml = 5.25 g / X. X = 16.1ml displaced with 5.25g of powder.
For 350mg/5ml = 70 mg/ml. Solve for mL
70mg ‐‐‐‐‐ 1ml 5250mg (5.25gm) ‐‐‐‐‐ Xml
X = 5250mg * 1 / 70 = 75ml. Then 75ml – 16ml = 59mL to add.

234. Doctor prescribed 100 mg elemental iron each day for lady, which of the following
formulations should be dispensed to her?
a) 300mg/5ml of ferrous fumerate
b) 300mg/5ml of ferrous gluconate
c) 300mg/5ml of ferrous sulphate
ANSWER
ferrous fumerate elemental iron =33%
ferrous gluconate =12%
ferrous sulphate =20 %
so, answer will be A

235. Marian Q22. A physician prescribed a fortified eye drops for a patient Tobramycin 13.5mg
/ml gtt ou twice daily. How much of the 40 mg/ml stock solution should be added to 5ml of
0.3% to get the desired concentration?
ANSWER
0.3 % means 0.3 gm ‐‐‐‐‐‐‐ 100 ml X gm ‐‐‐‐‐‐‐ 1 ml
X = 1*0.3/100 = 0.003 gm = 3mg So 0.3% = 3 mg / ml
by allegation method:

X = 5 * 10.5 / 26.5 = 1.98 ml

236. Marian Q73: Dose to be given as an infusion 900mg, 50mg/hr initially then increase by 50
mg/hr every 30 min until maximum 400mg/hr. how many hours needed?
ANSWER
50 ml /hr = 25 ml/ 30 min
50 initially + 50 (at 1.0 hr) – 150 (2 hr) – 200 (3 hr) ‐ 250 (4hr) – 300 (5) ‐ 350 (5 hr) ‐ 400mg (6 hr)
237. Perpetual inventory management?
a) Physical count of all present products that is done periodically
b) Count On computer system
c) Check the shelves
Methods of Inventory management
Method Description Comments
Visual Check the shelves for items with low quantity Time consuming and not accurate
Want Writing down what needs to be ordered & keeping Lag time between ordering & receiving
Book track of minimum no. of units to keep in inventory drugs
Keep index of various manufacturers (ie. in hospital) Issue with understocking due to long lag
to know where to order from time
Periodic Inventory management, or control that is done Limitation: # of products on paper does not
periodically coincide with # of products on shelf due to
Physical count carried out of all present products theft, loss, damage
Perpetual Perpetual inventory is a method of accounting Computer can suggest ordering amount
for inventory that records the sale or purchase based on previous records of orders &
of inventory immediately through the use of usage
computerized point-of-sale systems and enterprise Advantage:
asset management software Accurate inventory (point of sale data &
shelf inventory)
Point-Of- Provides efficient inventory control of merchandise in the front store.
Sale (POS) This system has the ability to track inventory from the time it is received to the time it is sold.

238. 2 allegation calculations

239. Prednisolone tab counting: 20 mg Q a.m for 2 weeks. Then decrease 2.5 mg every week
until discontinued. How many tablets if you have 5 mg tablet?
ANSWER
20 mg (4 *5 for 14 days) = 56 tablets
Then decrease by 2.5 mg (half tablet each week)  So (3.5 +3+2.5+2+1.5+1+.5) * 7 days = 14 *7= 98
Total required = 56+98=154 tablets

240. 3-4 drugs with KCl, which drug will stay in the serum for longer time? Find t1/2
ANSWER
calculate T1/2 for each drug T ½ = .693 / k
The longer half life will stay longer time in the circulation.

241. Patient dead, a family member came to return his unused medications (different drugs &
Narcotics), which one is the apprrpriate action to take as a pharmacist?
a) Destroy all
b) Discard narcotics, return others to the shelf
c) Put only tablets on the shelf and discard others
242. LV is a 53-year-old female who had DVT 10 years ago,
breast cancer, depression, lipid problems & insomnia. She
comes to the pharmacy with symptoms of sweating, hot
flashes and lethargy. Her life style indicates that she drinks 2
coffee per day and she is non smoker with occasional
alcohol drinking, where in SOAP we put these pt symptoms?
a) Subjective
b) Objective
c) Assessment
d) Plan

243. Which of the following symptoms requires referral?


a) Occasional vaginal bleeding
b) Low BMD
c) Hot flashes

244. Non- Pharmacological: What is your best advice for her


hot flashes:
a) Weight bearing exercise
b) Drinking Chamomile before going to bed
c) Decrease coffee intake
d) Decrease humidity and sleep in cold temperature
e) Soya isoflavone
Nonprescription and herbal products including black cohosh, dong quai, evening primrose oil, flaxseed,
ginseng, hops, maca, omega‐3 fatty acids, pine bark, pollen extract and puerpuria all have either negative,
insufficient or inconclusive data and should not be recommended until further data is available. Note that
case reports link black cohosh to liver damage.
Vitamin E (400–800 units daily) has a marginal effect on vasomotor symptoms.
A meta‐analysis of plant‐based therapies used for menopausal symptoms suggests that dietary and
supplemental soy isoflavones may improve hot flash scores and vaginal dryness but not night sweats.
Whole soybean contains equal amounts of genistein and daidzein and a smaller amount of glycitein; all
components may have different therapeutic outcomes. Gut bacteria metabolize daidzein to S‐equol, which
has been developed for women who are unable to metabolize daidzein and may hold therapeutic promise,
but further studies are warranted.
Exercise, yoga, acupuncture and paced respiration are measures that have been taken to reduce the severity
and/or frequency of menopausal symptoms. However, randomized‐controlled trials have demonstrated
such measures are no better than placebo.
While exercise and yoga do not impact vasomotor symptoms directly, yoga may have a positive impact on
quality of life during menopause. As well, all women should be advised to exercise (at least 150 minutes
moderate‐intensity exercise/week) to improve mood, reduce cardiovascular risk and improve bone health.
Traditionally, common‐sense measures have been promoted to address vasomotor symptoms, which include
cooling techniques (e.g., dressing in layers, using fans, lowering the ambient temperature) and avoidance of
triggers (e.g., spicy foods, hot drinks, caffeine, alcohol). While such measures are not harmful, there is a lack
of clinical evidence demonstrating benefit

245. Which of the following


medication is most appropriate for
LV for her hot flushes?
a) HRT Estrogen/Progesterone
b) Venlafaxine
c) Clonidine
d) Pregabalin
e) Duloxetine

245. Diagnostics, Lab test and chest


X-ray represent which in SOAP?
a) Subjective
b) Objective
c) Assessment
d) Plan
SOAP Meaning Example
SUBJECTIVE • What the patient reports Signs & symptoms, when they
• Info from patient’s perspective started, etc.
OBJECTIVE • What the provider reports/measures Diagnostics, Laboratory results,
(Findings) x-ray, vital signs BP 150/90, SrCr
(80), FBG 6.5 mmol.
ASSESSMENT Interpretation of subjective & objective information Working diagnosis
If writer is physician, assessment will be a disease state Drug interaction due to CYP450
or condition diagnosis & explain reason for diagnosis.
The pharmacist SOAP notes, assessment will identify
DTP and explains why DTP needs to be corrected.
PLAN • Action plan Discontinue Drug X and initiate
• Monitoring parameters Drug Y

246. When can you do alternative therapeutics without asking the doctor?
a) Generic.
Another version
Insurance company wants to reduce cost, what will interfere with physician autonomy or Wht
compromise physician autonomy?
a) Brand - Generic substitution // interchange
b) Therapeutic interchangeability
Once a generic drug is approved through the Abbreviated New Drug Submission Process, it becomes
immediately interchangeable with the reference product it was compared to this eliminates any delay
determining interchangeability.

247. Long case regarding Zika virus. patient want to travel to cuba with her husband & their 3
children that are 12 years, 8 years and 2 years, what to recommend as insect repellent
a) Citronella twice daily (every 2 hours)
b) DEET 30% (not for less than 12 year- Canadian guidelines)
c) DEET 10% 3 times daily
d) Eucalyptus oil

248. What is true about DEET?


a) DEET protect from sting insect & bite insect
b) Repellent should be reapplied after 2 hrs
c) The one containing citronella is more effective
d) DEET provide protection in highly wooded areas
e) Apply DEET before sunscreen
Propellants will protect against biting insects not stinging insects
249. Her husband got infected with zika virus. They have sexual contact without any protection.
Now, she wants to get pregnant what to advise:
a) Men semen is affected for several moths use condome for protection & wait 6 months
b) No problem with zika virus and pregnancy, you can go be pregnant
c) Just use insect repellant
d) Zika virus stays in semen
The Zika virus can be transmitted by infected men to their sexual partner and unborn child. The use of male
latex condoms or abstinence from sexual intercourse is recommended for men infected with the Zika virus
for 6 months prior to attempting conception or for the duration of pregnancy
Prevention of sexual transmission
RNA of the Zika virus has been detected in semen up to 281 days after the onset of symptoms. However,
detection of viral RNA does not necessarily indicate the presence of infectious virus. In a recent study,
infectious virus was detected in a few individuals 30 days after illness onset but, in general, shedding of
infectious Zika virus appeared to become much less common with time and was limited mainly to the first
few weeks after illness onset; however, there is still a possibility of later transmission. Because Zika virus can
be transmitted via semen, the CDC recommends people use condoms or practice abstinence if one or both
partners live in or have traveled to an area with current or past Zika virus transmission.
This recommendation applies whether or not people have symptoms because most Zika virus infections are
asymptomatic, and when symptoms do develop, they are usually mild.
 Man, with pregnant partner: Abstain from sexual activity, or use condoms and avoid sharing sex toys
for the duration of the pregnancy
 Man traveled to area at risk of Zika with or without female partner: Abstain from sexual activity or
use condoms for 3 months after return (or the start of symptoms)
 Woman traveled to area at risk of Zika without male partner: Abstain from sexual activity or use
condoms for 2 months after return (or the start of symptoms)
If using condoms, they should be used from start to finish every time during vaginal, anal, and oral sex.
Although no cases of woman‐to‐woman sexual transmission have been reported, the CDC recommends that
all pregnant women who have a female sex partner who has traveled to or resides in an area with Zika use
barrier methods every time during vaginal, anal, and oral sex, or abstain from sex during the pregnancy,
and avoid sharing sex toys.

Case about case seizure: Complicated case of phenytoin with link reference from rxtx. She had
symptoms of high dose of phyntoin and conc of phynitoin was high & they asking about
reason? and many Question i don't remeber them.
250. Female has tonic-clonic seizure, she received phenytoin, fluconazole & other medications.
Few weeks ago, she got Cachexia (loss of muscle or muscle wasting and weakneess); visual
disturbances and fell from the stairs. Phenytoin level was ordered and was very high. Which
pharmacokinetic property of phenytoin lead to that toxicity?
a) Protein binding of phenytoin.
b) Fluconazole interaction with phenytoin
251. Now she has an urgent flare seizure (status epilepticus) and admitted to emergency room.
What is the medication you recommend?
a) IV Carbamezapine
b) IV Phenobarbital
c) IV Valproic acid
d) IV Levetiracetam
e) IV Phenytoin
Time since Management
symptom onset
0–5 min History, physical examination. Oral airway, oxygen. Consider intubation.
Venous blood (glucose, blood counts, electrolytes, calcium, renal and liver function, AED
blood levels, consider drug screen).
Arterial blood gases. Monitor ECG, pulse oximetry, blood pressure
6–10 min Start 2 large-bore IV saline infusions. Vit. B1 Thiamine 100 mg IM or IV (prior to dextrose).
50 mL dextrose 50% IV (if low or unknown blood glucose).
Lorazepam 4 mg IV; repeat once if seizure persists, or
If no IV access: midazolam 10 mg IM/intranasal/buccal; repeat once if seizures persist, or
Diazepam 20 mg PR; can use rectal gel formulation or IV solution.
11–30 min Phenytoin 20 mg/kg IV (maximum rate = 50 mg/min, slower in elderly patients and monitor
ECG) or phenobarbital 20 mg/kg IV (50–75 mg/min).
If seizures persist, consider admission to intensive care unit and treatment with: midazolam
10–20 mg IV bolus, then 0.05–0.4 mg/kg/h or propofol 1–2 mg/kg IV bolus followed by
infusion at 2–5 mg/kg/h.
31–60 min Obtain neuroimaging (CT, MRI) if etiology of SE not known.
Consider lumbar puncture if a possibility of meningitis-encephalitis.
Admit to intensive care unit, obtain expert advice, arrange EEG.
252. After returning home, the pt has problem doing everyday activities, what specialist you
would advice to him?
a) Occupational therapiest
b) Social worker

253. Same case she needs your recommendation for contraceptive. What the reason of
possibility of failure any combined COC?
a) Her BMI above above 25
b) Phenytoin 300 mg daily

254. Pt got live Flu vaccine in right arm, developed anaphylactic reaction, where to inject epipen?
a) 0.5ml SC Left arm 45 degree
b) 0.3ml IM Right arm 45 degree
c) 0.3ml IM Left arm 90 degree
d) 0.5ml SC IM Right arm 90 degree

255. What to document allergy to vaccine? In medical profile

256. LX is a 75-year-old male with dementia, BPH and hypertension, experiencing symptoms
of urinary incontinence. The patient had BPH in the last 2 years and his prostate was removed 6
years ago. His profile shows that he is on:
Donepezil 5 mg in the morning - Mirabegron - Metoprolol OR bisoprolol 50 mg - HCTZ -
Tamsulosin-dutasteride. What might cause a drug interaction with donepezil
a) Mirabegron
b) Metoprolol
c) Tamsulosin
d) HCTZ
e) Dutasteride
Q. Selective and have greater affinity for AchEi in brain than periphery.
Donepezil Reduces the hydrolysis of acetylcholine, increasing the amount available in the synaptic cleft.
Piperidine- Donepezil was effective in 3- to 6-month trials in patients with mild to moderate Alzheimer
based disease (MMSE score of 10 – 26). It was also effective in moderate to severe Alzheimer disease
Centrally (MMSE 0 – 17), and is the only cholinesterase inhibitor approved for all disease severities
active Initial daily dose 5mg, taken at night. Can be taken in the morning if sleep disturbances occur
reversible, non Monitor treatment effects 2 weeks after initiating therapy or increasing dosage, then every
competitive. 3 months. After 4 weeks, try increasing to the target dose of 10 mg/day. Adjust dose after 4 wk
S.E: Cholinesterase inhibitors: theoretically, these agents may lower seizure threshold, increase
the risk of GI ulceration or bleeding, or exacerbate COPD or asthma.
Donepezil: >10%: headache, nausea, diarrhea. <10%: vomiting, anorexia, fatigue, sleep
disturbance, syncope, muscle cramps, urinary frequency. Bradycardia (uncommon), heart block
(uncommon), rhabdomyolysis (uncommon), neuroleptic malignant syndrome (uncommon).
Cholinesterase inhibitors: theoretical concern regarding antagonistic effect of combined therapy
with cholinesterase inhibitors and drugs with anticholinergic activity.
Additive bradycardia when combined with BBs or CCBs; few reports of actual interactions.
Toxicity may be increased by inhibitors of CYP2D6 or CYP3A4 such as paroxetine,
erythromycin, prednisone, grapefruit juice.
Effectiveness may be reduced by inducers of CYP2D6 or CYP3A4 such as carbamazepine,
phenytoin, rifampin.
Patients should be carefully monitored for muscle pain, tenderness or weakness and darkened
urine, particularly if accompanied by malaise or fever. Blood creatine phosphokinase (CPK)
levels should be assessed in patients experiencing these symptoms.
Close monitoring for adverse effects in patients with hepatic disease, reanal diseases & geriatrics
being treated with donepezil hydrochloride is therefore recommended.

257. What should be removed from LX regimen:


a) Mirbegron
b) Dutasteride
c) Metoprolol
d) Tamsolusin

258. Donepezil counselling, all are true except?


a) Must Swallow a whole tablet in the morning on empty stomach
b) It will show effect after 3- 6 months at least
c) It will prevent the disease from progressing
d) Contraindicated in case of Left branch Bundle block
e) Most common side effects are nausea, diarrhea, insomnia, vomiting, muscle cramp,
fatigue and anorexia.
f) Blood creatine phosphokinase (CPK) levels should be assessed in patients
experiencing muscle pain because of possible rhabdomyolysis
259. 7 years old patient has atopic dermatitis. Patient tried hydrocortisone 1% but only had
partial response. Family doctor wants to start him on Pimecrolimus but is not aware of risk of
cancer associated with the drug. What should you tell the kid’s mother when she comes in to fill
the prescription to uphold Beneficence?
a) Fill prescription and don’t mention risk of cancer If it is used for short term it is ok
b) Tell mother to take her son to visit a dermatologist If less than 2 years or after calcineurin
treatment failure
c) Call family doctor and recommend switching patient to mometasone before pimecrolimus

260. His mother told you that he can’t sleep from rash. What to do for this?
a) Diphenhydramine syrup for 7 days
b) Drink green tea
c) Tell her to use Vaseline or petroleum after cortisone
d) Give her gravol and send her to see doctor
Diphenhydramine OTC labeling: When used for self‐medication, do not use in children <6 years, to make a
child sleep, or with any other diphenhydramine‐containing products (including topical products)
Sleep Disturbance
Patients with poorly controlled AD often report poor or disturbed sleep due to itch and irritation.
Disease severity correlates with sleep disturbance, making improved treatment of underlying AD the most
important intervention to improve sleep. Although there is no evidence for them effectiveness in the
treatment of AD, some sources recommend short‐term intermittent use of first‐generation antihistamines
(e.g., diphenhydramine, hydroxyzine) for sleep disturbance in AD due to their sedating properties, but not as
a substitute for better disease control.

261. Here are the average measurements of PL 54-year-old type I diabetes mellitus.
His insulin regimen includes 10 units of lispro before breakfast and 12 units before lunch and
14 units before dinner and 10 units of NPH twice daily in the morning and at the bedtime .PL
also receives Ramipril 5 mg BID for hypertension. His blood pressure is 140/85.
Fasting  8.5 After Breakfast  11 After Lunch  4.2
After Dinner  9 At bedtime  7.2
What the best recommendation for this patient:
a) Increase bedtime NPH by 2 units
b) Decrease bedtime NPH by 2 units
c) Decrease pre launch lispro by 2 units and keep monitoring after lunch by 2 hours
d) Decrease pre breakfast lispro by 2 units
262. He is already on ACEI Enalapril. What could be done to PL to control his high blood
pressure:
a) Add Irbesartan
b) Change it to perindopril
c) Add Bisoprolol with given dose
a) Add amlodipine 2.5 mg
d) Add Chlorothalidone 12.5 mg
No BB (mask glucose reading) or thiazides (↑ glucose) because of diabetes.
263. Few weeks later, PL went to the clinic to measure his blood pressure and it was still 140/85
despite him being sure that his readings for the last week at home were within normal range.
What could be the reason for this reading at office:
a) Drinking coffee one hour before taking the reading
b) Measuring the blood pressure from both arms.
c) Discussing his insulin regimen during measurement
d) Cuff was too loose (big cuff size)
e) Taking Naproxen in the morning
f) Used Tadalafil 12 hours before

264. He got angioedema from enalapril; now his dr want to switch to ARB, how to switch?
a) Stop, wait & switch.
b) Switch directly
c) Continue with small dose.
Practical tips & tricks on when and how to change from ACE inhibitors to ARBs
 When RAAS blockade is indicated, ACE inhibitors should be used as first‐line treatment
 There are currently no compelling indications for the use of ARBs routinely as first‐line treatment
 The combination of ACE inhibitors/ARBs is contraindicated in the vast majority of patients
 When RAAS blockade is needed but ACE inhibitors are not well tolerated due to a persistent dry cough,
ARBs can be considered as an alternative (ARBs should be avoided as an alternative to ACE inhibitors in
patients who develop severe renal insufficiency or hyperkalaemia as adverse effects of this treatment)
In case of a switch from ACE inhibitors to ARBs, it seems reasonable to stop ACE inhibitors and start ARBs
the following day at an equivalent dose.
ACEIS ‐‐‐ ARB: ‫ﺡ ﻧﺪﻯ ﺗﺎﻧﻰ ﻳﻮﻡ ﻻﻧﻪ ﺩﻭﺍ ﺿﻐﻂ ﺍﻭ ﻗﻠﺐ ﻋﺎﺩﻯ ﺍﻟﺮﺍﺟﻞ ﺧﺪ ﺍﻟﻨﻬﺎﺭﺩﻩ ﺍﻟﺪﻭﺍ ﻑ ﻳﺎﺧﺪ ﺑﻚ ﺡ ﺍﻟﺠﺪﻳﺪ ﻭ ﺧﻼﺹ‬
ACEIS to ARNI = 36 hrs ARB to ARNI ‫ﻋﺎﺩﻯ ﻣﺎ ﻓﻴﺶ ﻣﺪﻩ ﻣﻌﻴﻨﻪ‬
Isosorbide dinitrate plus hydralazine: considered in black patients with NYHA class III – IV and in other HF
patients who do not tolerate ACE inhibitors, ARBs or an ARNI.

265. Hypertension may affect the below organs damage EXCEPT


a) Retinopathy
b) Ventricular hypertrophy.
c) Cerebrovascular accident.
d) Nephropathy
e) Sexual dysfunction
f) Neuropathy  only happens only if accompanied with diabetes
Elevated BP leads to cardiovascular and renal complications. Important sequelae are stroke, myocardial
infarction, atrial fibrillation, heart failure, chronic kidney disease and dementia. No pathologic changes
occur early in hypertension. Severe or prolonged hypertension damages target organs (primarily the
cardiovascular system, brain, and kidneys), increasing risk of Coronary artery disease (CAD) and myocardial
infarction (MI), Heart failure, Stroke (particularly hemorrhagic), Renal failure
266. Female patient complain that she become anxious and get panicked. She worked in a
company where she has to do a lot of presentations, face many clients and she feels like
fleeing from such situations. Doctor dignosed her as having GAD and wants to start non
pharmacological therapy, what do you recommend?
a) Cognitive behavioural therapy (CBT)
b) Exercise
c) Group therapy
Psychotherapies: Cognitive behavioural therapy (CBT), behavioural activation (BA) and interpersonal
therapy (IPT ‐ preferable when major interpersonal issues present) are considered first‐line
recommendations. Newer versions of CBT and IPT have evolved and show particular promise;
 Mindfulness‐based CBT (mCBT): effective in preventing relapse into depression and acceptance.
 Commitment therapy (ACT): has some efficacy in acute depression.
Motivational interviewing: effective for treatment of substance abuse, not on the depressive symptoms.

267. Lately her situation is worsening even she went to emergency for chest pain. Doctors
couldn’t find any abnormality in her heart. Last week she got 8 panic attack and now doctor
wants to initiate a 6 weeks trial of anti anxiety medication. He prscribed her citalopram 20
mg, she complains now about agitation, what to do?
a) Sertraline
b) Sertraline + lorazepam
c) Venlafaxine
d) Reduce citalopram to 10 mg
e) Increase citalopram it to 40 mg
f) Venlafaxine + clonazepam
Another scenario: Pt suffers from depression. Dr Rx Citalopram 10 mg. After 10 days, she
suffered more anxiety. What to do?
a) Increase Dose
b) Decrease Dose
c) Switch to Venlafaxine
d) Switch to Paroxetine
This question was written with doses
‫ﻻﺯﻡ ﻧﺴﺘﻨﻰ ﻋﻠﺸﺎﻥ ﻧﻘﺪﺭ ﻧﻘﻴﻢ ﺍﻟﺪﻭﺍء ﺷﻐﺎﻝ ﻭﻻ ﻻ ﺑﺲ ﻓﻰ ﺍﻟﺴﺆﺍﻝ ﺍﻟﻐﺒﻰ ﺩﻩ ﻫﻮ ﻏﺎﻟﺒﺎ ﻋﺎﻳﺰ ﻳﻌﺮﻑ ﺍﻧﺖ ﻋﺎﺭﻑ ﺍﻟﺠﺮﻋﺎﺕ ﻭﻻ‬
‫ﻻ‬
For GAD, escitalopram initial should be 10 mg and when case got agitated it’s SE and
should be gone after 2 wks or we can add BZ for first 6-8 wk
For depression initial 10-20 mg, so if increased anxiety may be decrease dose but switching
in same class if SE can’t b tolerated
Some benzodiazepines have demonstrated efficacy in the treatment of anxiety disorders, although they
generally, do not help with comorbidities like depression. They have also been shown to be useful in
relieving acute anxiety, agitation and panic attacks at the beginning of treatment while waiting for
antidepressant treatment to take effect, as well as to mitigate anxiety and agitation flare‐ups that may
occur when beginning to take an antidepressant.
It is generally recommended to use them as a short‐term measure during the first few weeks of
treatment only, if possible. However, benzodiazepines are sometimes continued for a few months if they
are effective and well tolerated in the absence of antidepressant response, until an effective treatment is
found. In the elderly, benzodiazepines should be used with caution because, due to age‐related
pharmacokinetic changes, older patients are more likely to experience adverse effects, such as
psychomotor and cognitive impairment, and present a higher risk of falls and fractures. Benzodiazepines
are not recommended in the presence of a comorbid substance use disorder or a history of one.

268. She becomes pregnant and is stressed about her job and scared she’ll lose her job and
having hard time maintaining her expenses. She is asking pharmacist to help her with her
medication expenses and other issues. Which is the best option to look for help?
a) Social worker
b) Occupational therapiest
c) Manufacturers program
d) Give discount from pharmacy

269. Female Patient came with BV bacterial vaginosis symptoms fishy odour, grey or milky,
thin, creamy or copious discharge, she is sexually active, shy to tell her partner about her
symptoms ask you advice?
a) Give nonprescription fluconazole
b) Refer for metronidazole therapy
c) Clotrimazole, Vaginal tablet
Bacterial Characterized by fishy odour, grey or milky, thin, creamy or copious
vaginosis discharge. No inflammation or Pruritus
Caused by Fishy odour often intensified after addition of 10% (KOH).
replacement of Vaginal pH >4.5 (5-6), +ve "Whiff" test, -ve PMN
normal vaginal Treatment is unnecessary in asymptomatic women except if undergoing
flora by intrauterine device (IUD) insertion, gynecologic surgery, therapeutic abortion, upper genital
overgrowth of tract instrumentation or if the woman is at high risk of preterm delivery.
anaerobic Q. Risk factors: Intrauterine devices (IUD), Sexual intercourse with new or multiple
Mycoplasma partners, Lack of lactobacilli & Douching.
and Gardnerella Treatment of male sexual partners is not indicated and does not prevent recurrence.
vaginalis Antibiotic treatment can be offered to all symptomatic women with bacterial vaginosis.
Clindamycin (Preferred treatment), 2% vaginal cream 1 applicatorful 5g daily PV ×7 days
S.E: Vulvovaginitis. Not recommended in high-risk pregnancy.
Topical clindamycin has been associated with adverse outcomes in the newborn when used
in pregnancy. Delay treatment until completion of menstrual period.
Contains mineral oil and may decrease effectiveness of condoms and diaphragms.
Clindamycin, oral (Alternative treatment): 300 mg BID PO × 7 days
Can be used in pregnant and breastfeeding women; retest 1 month after completion to
confirm efficacy of therapy. S.E: GI upset, Clostridium difficile colitis, diarrhea.
Q. Metronidazole, oral (Preferred treatment): Preferred dose: 500 mg BID PO × 7 days,
Alternative dose: 2 g PO × single dose (associated with higher failure rate)
Recurrent bacterial vaginosis (≥3 episodes/y): 500 mg BID PO × 10–14 days.
Can be used in pregnancy or breastfeeding. Some clinicians advise against breastfeeding for
12–24 h after a 2 g dose. Retest 1 month after treatment to ensure therapeutic success.
Metronidazole vag. gel (Preferred treatment): 1 applicatorful (5 g) daily PV × 5 days
Recurrent bacterial vaginosis (≥3 episodes/y): 1 applicatorful (5 g) daily PV × 10 days
followed by 5 g PV twice weekly × 4–6 months
S.E: Vag. discharge, yeast infection, vulva/vaginal irritative symptoms, pain & discomfort.
Not recommended in high-risk pregnancy as it does not prevent preterm birth.
Topical treatment has similar cure rate as oral metronidazole.

270. Same patient, you will report to public health, what is the best action you do as a
pharmacist regarding her?
a) Report her case as BV is STI, and advise her to abstain from sex
b) Report her case as BV is STI and tell her you can still have sex with your partner
c) Don’t report her case as BV is not STI, advice to abstain from sex
d) Don’t report her case as BV is not STI & tell her you can still do sex with partner

271. Gout case: patient on Colchicine and allopurinol for one year now. There are no attacks
and Uric acid level is in range. He developed STEMI and so many details. What your
recommendation now?
a) Stop colchicine & keep allopurinol
b) Keep colchicine and stop allopurinol
c) Keep both Allopurinol and Cholchicine
d) Stop both agents
Allopurinol may have protective effects over ischemic reperfusion injury and reduce infarct size. ... In
patients admitted with STEMI who are candidates of thrombolytic therapy, allopurinol is associated with
better 90‐minute ST resolution, lower enzymatically determined infarct size, and in‐hospital MACE.
HMG‐CoA Reductase Inhibitors (Statins): Colchicine may enhance the myopathic (rhabdomyolysis) effect
of HMG‐CoA Reductase Inhibitors (Statins). Colchicine may increase the serum concentration of HMG‐
CoA Reductase Inhibitors (Statins). Risk C: Monitor therapy
If case about colchicine and statin: keep allopurinol and decrease dose of colchicine
Another version
Gout case: patient on Colchicine and allopurinol for one year now. There are no attacks and
Uric acid level is in range. What your recommendation now?
a) Stop colchicine & keep allopurinol
b) Keep colchicine and stop allopurinol
c) Keep both Allopurinol and Cholchicine
d) Stop both agents

272. DS 55-year-old male with low BMD it was borderline, the patient was smoking
occasionally. What is the appropriate advice for him?
a) Give Ca 1200 mg and VitD 1000 Unit
b) Start alendronate 65 mg per week
c) Start cyclical Pamidronate / CA
d) Start Denosumab
BMD measurements and fracture risk stratification: V.V.I
 BMD of the spine and hip by dual x-ray absorptiometry (DXA) is the preferred method of assessing
bone mass. Other methods (heel ultrasound) are acceptable if DXA is not available.
 The WHO created DXA-BMD definition of osteoporosis as a BMD T-score ≤ −2.5 (2.5 standard
deviations below a normal young adult reference mean).
 Osteoporosis Canada recommends calculating estimated 10-year risk of osteoporotic fracture in
women, which is derived by combining age and epidemiologic data with DXABMD measurements.
 Two fracture risk calculators have been developed, validated and are now recommended as an aid to
osteoporosis management for the Canadian population: FRAX (Fracture Risk Assessment Tool) and
CAROC (Canadian Association of Radiologists and Osteoporosis Canada). The estimated risk is
reported as low (<10%), moderate (10–20%) or high (>20%). Risk categories are intended only for
assessment of the as-yet-untreated patient

273. What is the modifiable risk factor for the above patient?
a) Smoking
b) Alcohol
c) Ex smoker
d) Family history.
274. Dose of denosumab in osteoporosis?
a) 60 mg once every 6 months S.C.
b) 120 mg every 4 wk SC
c) 60 mg once every 6 months IM.
d) 120 mg every 4 wk IM
Recommended Dose and Dosage Adjustment
Multiple Myeloma and Bone Metastasis from Solid Tumours
 Recommended dose is 120 mg administered as a single subcutaneous injection once every 4 weeks.
Giant Cell Tumour of Bone
 The recommended dose of is 120 mg administered as a subcutaneous injection once every 4
weeks with a loading dose of 120 mg on days 8 and 15 of the first month of therapy.
Hypercalcemia of Malignancy Refractory to Intravenous Bisphosphonate
 The recommended dose of is 120 mg administered as a subcutaneous injection once every 4
weeks with a loading dose of 120 mg on days 8 and 15 of the first month of therapy.
Treatment to Increase Bone Mass in Men with Osteoporosis at High Risk for Fracture
 The recommended dose is a single SC injection of 60 mg, once every 6 months
Missed Dose: If a dose is missed, administer the injection as soon as the patient is available. Thereafter,
injections should be scheduled every 4 weeks from the date of the last injection. Keep refrigerated.

275. Patient on Methadone, he comes regularly to the pharmacy to administer his daily dose,
today after you witness his dose. He asks to take the kit (it has syringe, condom, etc) upon
asking why you want the kit and you are on methadone therapy. He said it is for a friend.
What will be your response in order to uphold beneficience?
a) Give kit and ask he to come tomorrow as usual to take his methadone dose
b) Give kit and tell him no more methadone for you
c) Don’t give the kit and ask him to come back for methadone dose tomorrow
d) No kit and no more methadone

276. Dr Rx note theifted, he is only eligible to Rx narcotics. which drug is not inculded?
a) Ketamine
b) Pentazocin
c) Tylenol No.4 (Codeine Phosphate. 60 mg + Acetaminophen. 300 mg)
d) Dextroamphetamine

278. A 4-year-old child is having asthma and his parents inquires about the appropriate
technique for the aerochamber:
a) You can take 2 doses at same time
b) Breath normally
c) When you hear whistle, it means you received the right dose
d) Wear the aerochamber and breath 5 to 6 breaths after releasing the dose
e) You can spray the dose in advance and leave it for subsequent doses
f) Wash the aerochamber and dry it using piece of cloth
When you hear whistle, you are using it too fast
279. Community pharmacy with high workload, you should do medical councelling for large
number of patients, what to do?
a) To delegate med reco to Ph tech to gather info
b) To delegate tech to Councel non-Rx drugs
c) To apologize for pt due to workload

280. Rx for pharmacist by error select Methotrexate 25 mg instead of Methotrexate 10 mg on


central processing unit CPU, then the tech completes the Rx upon the selection, what to do to
decrease the error?
a) Educate technician
b) Remove 25 mg from CPU
c) Ask for 2 pt identifies
d) Make independent double check
Human factors A patient went to an outpatient clinic on the weekend for treatment of shoulder pain and
Confirmation bias was prescribed Diclofenac. It was interpreted as Diflucan by the pharmacy staff member
Lack of who did the order entry. (The staff member thought Diclofenac & Diflucan were the same.)
independent To avoid incidents related to confirmation bias, indications for each medication should be
double checks included on the prescription.
Knowledge deficit Independent double checks should be performed throughout the entire pharmacy
workflow.
281. Two case about patients have LVH, hyperalgesia due to morphine and need taper, how
to deal? (many scenarios and select the lay language)
Reductions in opioids can be carried out in many ways
1. Fast –
 Simply stopping your opioids immediately, or reducing rapidly over a few days or weeks will result
in more severe withdrawal symptoms, but the worst will be over in a relatively short period of time.
 This method is best carried out in a medically supervised withdrawal center.
 Ask your doctor if such a center exists in your community.
2. Slow –
 Gradual dose reductions of 5 to 10% of the dose every 2-4 weeks with frequent follow-up with your
doctor is the preferred method for most people.
 If you are taking any short- acting opioids it may be preferable to switch your total dose to long
acting opioids taken on a regular schedule. This may make it easier for you to stick to the withdrawal
plan. A pharmacist can help lay out a schedule of dose reductions.
3. Methadone or buprenorphine-naloxone –
 Another strategy that may result in less severe withdrawal is a switch to methadone or
buprenorphine-naloxone and then gradually tapering off.
 This requires a doctor trained to use these medications but can be an alternative to the “Slow”
method noted above.

282. Q about Vaccine interchangability, which one is


right?
a) MMR has no interchangeable
b) Diluent of vaccine has no interchangeable
c) Live zostavax inactivated
d) Vaccine from different companies are normally
interchangeability
e) Vaccines from the same manufacturer share the
same additives.
https://www.canada.ca/en/public-health/services/publications/healthy-living/canadian-immunization-guide-
part-1-key-immunization-information/page-7-principles-vaccine-interchangeability.html

283. 5 y flu trivalent out of stock? give quadrivalet


Flu Vaccine for 5-year-old child (differences about trivalent and quadrivalent flu vaccines)
Influenza vaccines
If a child (aged less than 9 years) requires 2 doses of influenza vaccine in the same influenza season, it is
preferable to use the same type of vaccine (trivalent inactivated [TIV], quadrivalent inactivated [QIV], or
live attenuated influenza [LAIV]) for both doses. However, if the child is eligible for TIV, QIV or LAIV, and
the type of vaccine used for the first dose is not available; TIV, QIV or LAIV may be used for the second
dose. If the vaccine provider administers TIV or QIV for both doses, vaccines from different
manufacturers can be used for the first and second dose.
284. Patient with Splenectomy, more susceptible to which M.O:
a) Streptococcus Pneumonia
b) Staph. Aureus
c) Influenza Virus
d) Neisseria Meningitis
What is this patient’s risk of infection after splenectomy?
Elective splenectomy is indicated in the management of certain medical conditions. Patients who have
undergone splenectomy are at risk of overwhelming postsplenectomy infection, which is characterized by
sepsis, meningitis or both, and carries a fatality rate of 50%–70%. These episodes occur in patients who
have had splenectomy at a rate of 0.2%–0.5% per year, with a lifetime risk of about 5%. Streptococcus
pneumoniae is the most common pathogen (> 50%), followed by other encapsulated bacteria such
as Haemophilus influenzae and Neisseria meningitidis, and less commonly by gram‐negative organisms
such as Escherichia coli, and species of Salmonella and Pseudomonas.

285. After splenectomy, he needs all of these vaccines except?


a) PCV 13
b) PCV 23
c) Flu Vaccine
d) Yellow fever
e) Tdap vaccine
286. Mirabegron for urinary incontenance. The patient wants to know about the side effects
of mirabegron. Which of the following is TRUE?
a) Blurred vision
b) Constipation
c) Hypotension
d) QT prolongation

287. Rosacea case. What the true option about rosacea?


a) No improvement could be seen with topical metronidazole before 4 months
b) Oral doxycycline is used incase of no response with topical metronidazole
c) Brimonidine is used once daily for one month
d) Topical Metronidazole should be used 3 months to get improved
e) Isotretinoin is a drug of first choice to treat rosacea
Oral antibiotics have been shown to be effective and may be added to topicals when the response is
inadequate or the condition is moderate to severe. The rationale for the use of antibiotics resides with
their anti‐inflammatory benefits, rather than their antimicrobial properties. They are generally used for
up to 3 months and then reassessed. Other oral antibiotics (including minocycline, trimethoprim,
azithromycin, erythromycin and metronidazole) do not have high‐quality evidence supporting their use in
rosacea and should be considered only when doxycycline or tetracycline cannot be used. Minocycline has
been associated with rare but serious side effects, resulting in many experts using doxycycline or
tetracycline as first‐line treatments and reserving minocycline for second‐line use when needed. Low‐
dose (subantimicrobial) doxycycline is available and appears to have similar efficacy but less
gastrointestinal effects; it theoretically has less risk of antimicrobial resistance than standard‐dose
doxycycline. It may be an option for patients in whom gastrointestinal effects of standard‐dose
doxycycline are of concern.

288. Then he asked the effect will show after?


a) 3 M
b) 6 M
c) 2 W
d) 4 W
289. Brimonidine in ttt erythromatous rosacea counselling?
a) If you wear contact lenses, remove them first, wait 15 minutes after using eye drops to
put them back into eyes.
b) If gel for rosacea, then apply for whole face not red spots only.
c) Stain on clothes
d) Get onset of action 30 min after application
e) Apply only on papules
Alpha2-adrenergic Apply small pea-sized amount to each of the 5 areas of face (forehead, chin,
Agonists nose, each cheek) avoiding the eyes and eyelids, lips, mouth & inside of nose.
Brimonidine tartrate S.E: Erythema, flushing, skin burning sensation, contact dermatitis, headache.
0.33% gel applied once CV effects may occur due to systemic absorption if applied to damaged skin or
daily accidentally ingested orally.
Keep out of reach of children.
May start to reduce redness within 30 min with peak effect at 3 h.
Wash hands immediately after applying.
Rebound redness/flushing can occur on withdrawal.

290. 18 months kid has runny nose & allergic rhinitis, likes to play outdoors. Father wants
something not to cause drowsiness, what to give?
a) Cetirizine + nasal decongestant
b) Refer
c) Give oral desloratadine
d) DPH
Allergic Rhinitis in Children
Poorly controlled rhinitis impacts childrens’ quality of life significantly; it can lead to reduced learning
performance, anxiety and social difficulties with friends and family.
Treatment of rhinitis symptoms in children is hampered by the difficulty of administering nasal sprays to
children as most do not like the idea of spraying something into their nose. Lower volume nasal sprays
(e.g., fluticasone furoate) tend to be better tolerated and are more effective than oral antihistamines in
children. If antihistamines are used, less‐sedating antihistamines are preferred, as sedating
antihistamines are associated with drowsiness and learning impairment.
Among the nasal preparations, cromoglycate is safe; however, it is not currently available in Canada.
Intranasal corticosteroids are also effective. Oral cough and cold preparations containing decongestants
with or without sedating antihistamines are not to be used in children younger than 6 years of age for
the treatment of viral or allergic rhinitis; Health Canada came to this decision based on limited evidence
of efficacy in viral rhinitis and reported adverse effects.
Desloratadine Safe and well tolerated in children.
Adults and children ≥12 y: 5 mg daily, Children: 6–11 y: 2.5 mg daily, 1–5 y: 1.25
mg daily, 6–11 months: 1 mg daily
S.E: Headache, pharyngitis, dyspepsia. Incidence of diarrhea in children reported as
15-20%. Avoid in patients with hypersensitivity to loratadine.
Pgp inhibitors (e.g., erythromycin, ketoconazole) may increase loratadine levels.
Pgp inducers (e.g., carbamazepine, dexamethasone) may decrease loratadine levels;

291. What is the content for cough syrup for him?


a) Guaifenesin
b) Codeine
c) Fenistil (Dimetindene)
d) Fexofenadine
Since 2008, Health Canada has required manufacturers to relabel nonprescription cough and cold medicines
with certain active ingredients to indicate that they should not be used in children <6 years of age.
Dextromethorphan, guaifenesin and first‐generation antihistamines (including diphenhydramine) contained
in cough and cold products are included in the list of active ingredients in the Health Canada advisory.
Although cough and cold medicines have been used by children for many years, little evidence supports
their effectiveness in this population. Furthermore, Health Canada has advised against the use of these
products in children <6 years of age due to reports of very rare serious side effects as well as misuse and
overdose. Rare but serious potential side effects include seizures, increased heart rate, decreased level of
consciousness, abnormal heart rhythms and hallucinations.
In children ≥6 years of age, dextromethorphan can be used to treat nonproductive cough, though evidence
of efficacy in children is absent. Health Canada recommends that any cough and cold product containing
codeine or other opioids (e.g., hydrocodone, normethadone) be avoided in children <18 years of age.
Health Canada reminds parents and caregivers:
 Do not use over‐the‐counter cough and cold medicines in children under 6.
 Always check the label first to make sure the medication is suitable for your child.
 Do not give children medications labelled only for adults.
 Do not give children aged 6 and up more than one kind of cough and cold medicine (unless under
the advice of a healthcare practitioner). Combining medicines with the same ingredient(s) may
cause side effects.
 Talk to your health care practitioner (e.g. doctor, pharmacist, nurse, etc.) if you have any
questions about using cough and cold medicines in children. These professionals can also help
make sure there are no interactions with other health products your child may be taking.
 A cold is not the same as the flu. Cold medications are not effective against the flu.
 If you have any concern with your child’s condition or if symptoms do not improve within 6 to 10
days, or worsen, consult a health care practitioner.
 Consider these non‐medicinal measures that may provide temporary relief from the symptoms of
coughs and colds:
o Allow the child adequate rest
o Clear nasal passages;
o Ensure plenty of clear fluids (e.g. water, diluted non‐sweetened fruit juice, or clear soups)
to prevent dehydration while keeping the throat moist; and
o Provide a comfortable environment with adequate humidity.

292. PS 45-year-old female having migraine. She stopped coffee 2 weeks ago due to
palpitation which has improved after discontinuing coffee. She is on sumatriptan 100 mg 1
tab in a week PRN and and 2-3 tab of naproxen pain killer in a month. She uses these
medications to control her migraine attacks twice per month. Recently the medications have
become ineffective to control her migraine symptoms. What the reason of uncontrolled
symptoms of migraine:
a) Low dose Sumatriptan
b) Low dose Naproxen
c) Stopping caffeine
d) No treatment for migraine prophylaxis
A sudden decrease in caffeine consumption may lead to a
withdrawal headache; caffeine may help alleviate headache
in some migraine sufferers.
Consider prophylaxis if migraine attacks have a significant
impact on the patient’s quality of life despite appropriate
use of abortive therapies, or if the frequency of attacks puts
the patient at risk of medication‐overuse headache.
There is little evidence on which to recommend an optimal
duration of prophylaxis. Guidelines suggest a trial of at
least 2 months, following dose titration, before assessing
benefit. Advise patients to maintain a headache diary to
monitor headache triggers, frequency and intensity,
menstrual cycle, use of preventive and abortive medications, and side effects.
Successful prophylaxis is usually defined as a ≥50% reduction in headache frequency or days with
headache, though some patients may report improved response to abortive therapy or decreased
headache severity or duration. If there is no benefit after a 2‐month trial at the target/optimal dose, try
a different medication. If prophylaxis is deemed beneficial, continue for 6–12 months then consider
tapering the dose to assess ongoing need. If headache intensity and frequency increase, the dose can be
increased to previously effective levels

293. The most common cause of diarrhea in children is:


a) Vibrio cholerae
b) E. coli
c) Rota virus
d) Pneumococcus
Infection from viruses like rotavirus, bacteria
like salmonella and, rarely, parasites like giardia.
Viruses are the most common cause of a child's
diarrhea. Along with loose or watery stools,
symptoms of a viral gastroenteritis infection often
include vomiting, stomachache, headache, and fever.

294. Rota virus vaccine for the baby, what to counsel?


a) Its life not to be taken
b) Anaphylactic rx
c) Take care when diaper as source of infection being live vaccine.
Vaccines for Rotavirus Prevention
Rotavirus Contains 5 rotavirus strains responsible for about 95% of rotavirus disease in Canada.
pentavalent, 85–98% effective against severe rotavirus disease. Decrease hospitalizations by up to 96%.
live oral First dose usually given between 6 and 12 wk of age; subsequent doses should have 4–10 wk
vaccine between them. Can be given with other routine immunizations
RotaTeq If an incomplete dose is administered, e.g., infant spits or regurgitates vaccine, a replacement
dose is not recommended.
3 total doses. The infant should continue to receive any remaining doses in the recommended series.
S.E: Mild fever, diarrhea, vomiting, otitis media, bronchospasm.
Immunosuppressive therapies (e.g., anti-TNF-alpha agents) passed from the mother to the fetus
may remain in the newborn’s blood for up to 6 months postdelivery; avoid administering any
live vaccines to the newborn until at least 6 months of age.
Rotavirus First dose usually given after 6 wk of age; subsequent dose given at least 4 wk later but no later
monovalent, than 24 wk of age. Can be given with other routine immunizations
live oral 85–98% effective against severe rotavirus disease.
vaccine If an infant spit out or regurgitates most of the vaccine dose, a single replacement dose may be
Rotarix given at the same vaccination visit.
2 total doses S.E: Irritability and diarrhea; intussusception (type of small bowel obstruction; rare).

295. Hand Hygiene: When should a pharmacist wear glove?


a) When vaccinate
b) When counseling
c) Checking patient abdomen for lipoatrophy
d) Standard measuring blood pressure for patient
e) To give SC insulin to patient and determine if there is lipodystrophy
f) To do point of care testing of INR to patient going for dental extraction
g) To teach patient self injection of dalteparin

296. Parents workshop for vaccination about Pneumococcal vaccine. It can ge be given to all
except or Pneumococcal vaccine protects against all except:
a) AOM
b) Pneumonia
c) Endocarditis
d) Meningitis
e) Peritonitis
f) Bronchitis
g) Tonsillitis
h) Septicaemia
i) Diabetes

298. Case of pertussis outbreak. BM is a pregnant female in her 28th week came to the
pharmacy wants to take pertussis vaccine for whooping cough to protect herself and her
infant after delivery. She took pertussis
vaccine when she was 16 yr and want your
advice, what will be your response
a) It is Contraindicated
b) Give her the vaccine
c) Do not give her the vaccine and advice her to take it after delivery
d) Give her the vaccine and tell her to vaccinate her child after delivery
e) Tell her to get vaccinated with her child after delivery.
Vaccination during pregnancy offers the best protection
https://www.cdc.gov/pertussis/pregnant/mom/get‐vaccinated.html
When women get a Tdap vaccine while pregnant, their babies have better protection against whooping
cough than babies whose mothers did not get vaccinated during pregnancy. Getting a Tdap vaccine
between 27 through 36 weeks of pregnancy lowers the risk of whooping cough in babies younger than 2
months old by 78%1.
If you did not get a Tdap vaccine during pregnancy and have never received it before, you can get it after
your baby is born. It will take about 2 weeks before your body develops protection (antibodies) in
response to the vaccine. Once you have protection from the vaccine, you are less likely to give whooping
cough to your newborn while caring for him. But remember, your baby will still be at risk for catching
whooping cough from others.

298. Doctors approach you and asked about QT drug,


where to look?
a) Crediblemed
b) Medline
c) Canadian CV Society.

299. Case RA: patient removed part of colon w drug? IV Infliximab…still he needs action?

300. SE infliximab
Infliximab Approved only in combination with MTX to ↓ immune reaction to murine
Chimeric component.
monoclonal antibody In case of inadequate response to infliximab, MTX doses should be maximized.
composed of human C.I: in patients with known Type I hypersensitivity or anaphylactic reactions to
and murine regions. murine proteins; HF if using > 5 mg/kg/infusion, demyelinating disease,
susceptibility to or presence of serious and/or recurrent infection; SLE is a relative
contraindication.
Storage: Refrigerator (2 to 8°C), do not freeze. Use within 3 hours of reconstitution.
Administer by infusion 3-5 mg/kg at 0, 2, & 6 wk, then every 4-8 week thereafter by
IV.
Most serious side effects respiratory tract infections (Pneumonitis), and
tuberculosis. Neurological problems include dizziness, visual disturbance and
infusion site reaction.

300. SA is diabetic patient experiencing a bilateral painful tingling sensation in his toes.
What will be the first question about?
a) Glycemic control and
neuropathic pain duration
b) HbA1C and fasting Blood
glucose
c) Family history of diabetic
foot
d) Pain worsening with activity
and relieved by rest.
301. 1st line
Empiric therapy for early infections should target Staph. aureus and the beta‐hemolytic streptococci.
Duration of therapy is based on clinical response; however, typical treatment courses for skin and soft
tissue infections range from 7 (mild) to 21 (severe) days, and the treatment of osteomyelitis may require
4–6 weeks of parenteral or several months of oral antimicrobial therapy.
Infection Severity Antimicrobial Agent
Localized infections: Frequently treated with outpatient oral
Neither limb- nor life-threatening antimicrobial therapy (7 days)
Usually associated with cellulitis surrounding an ulcer  Amoxicillin/clavulanate
Purulent debris may be present at the base of the ulcer  Cephalexin, Clindamycin
Usual organisms: aerobic Gram-positive cocci  Cloxacillin, Doxycycline
(Staphylococcus aureus and beta-hemolytic streptococci)  Linezolid - SMX/TMP
More extensive infections: The choice of oral vs. parenteral therapy should
Includes more extensive cellulitis, plantar abscess and deep be guided by the extent of the infection and the
space infections. patient’s overall clinical status
Initial antimicrobial therapy against staphylococci, Oral Options
streptococci, anaerobes and common Enterobacteriaceae.  Amoxicillin/clavulanate, Linezolid,
Empiric treatment targeting Pseudomonas aeruginosa is Moxifloxacin
generally unnecessary unless risk factors present, e.g.,  Ciprofloxacin, levofloxacin, SMX/TMP +
history of foot soaking, severe or chronic infection clindamycin or metronidazole.
Patients who are not toxic may be treated with débridement Parenteral Options
and oral antimicrobial therapy  1st, 2nd or 3rd generation cephalosporin +
Patients who are ill or toxic despite moderate local signs are metronidazole.
treated as having a severe infection:  3rd generation cephalosporin +
Limb- or life-threatening, Frequently polymicrobial Clindamycin
Immediate hospitalization, early surgical débridement and  Daptomycin, Linezolid, Carbapenem,
parenteral antimicrobial therapy Piperacillin/tazobactam.
If MRSA present or suspected, add vancomycin or linezolid
Q. Osteomyelitis Palpation of bone at base of ulcer Treatment may require 4–6 weeks of parenteral
S. aureus (MSSA or MRSA) is the most common pathogen, or several months of oral antimicrobial therapy.
but other less virulent organisms may be pathogenic, e.g., Oral Options
coagulase-negative staphylococci.  Amoxi/clav, Cephalexin, Clindamycin,
Treat with IV therapy or long-term oral antimicrobial Cloxacillin, Doxycycline, Linezolid,
therapy using agents that are well absorbed from the GI tract Moxifloxacin, SMX/TMP.
and have good distribution to bone and tissue  Ciprofloxacin, levofloxacin, SMX/TMP
Surgical débridement indicated to remove necrotic debris, plus clindamycin or metronidazole.
abscess or sequestrum Parenteral Options
Therapy should base on culture results whenever possible.  3rd generation cephalosporin plus PO/IV
If MRSA present / suspected, add vancomycin or linezolid. Clindamycin or PO/IV metronidazole
If clinical improvement isn’t observed (e.g. resolution of  Daptomycin, Linezolid, Carbapenem,
erythema, edema, heat, draining sinus, coverage of bone Piperacillin/tazobactam.
with soft tissue), consult with a specialist.
302. All of the following medication causes pancreatitis except:
a) Metformin
b) Exenatide
c) Empagliflozin
d) Sitagliptin

303. DM case, one 2 antihyperglycemic pt, dr decided to give insulin and call to ask your
advice, which is the best insulin for the patient?
a) 10 IU basal insulin
b) 1 dose NPH around supper time
c) 2 doses NPH to cover his day
d) Increase dose of SU (already max, make sure you have an idea about the dose to tell if
it is high or low)

304. How to measure Blood glucose control after one month?


a) Post prandial readings
b) HbA1C

305. Clostridium difficile case: diarrhea, abdominal crump, < 6 bowel movements/day, fever
38, no confusion, WBC 13.5. Sr Cr 210 (baseline 130). what severity?
a) Mild to moderate
b) Severe complicated
c) Recurrent

306. What is the treatment?


a) Vancomycin 125 mg QID PO ˣ 10 days.
b) Fidaxomicin 200mg PO BID x 5 days
c) Metronidazole 500 mg TID PO ˣ 7 days
d) Vancomycin 500 mg QID PO + Metronidazole 500 mg Q8H IV
307. After two months he got pneumonia and the doctor want to prescribe amoxicillin and he
is afraid from C difficile again and ask for your reference;
a) Reduce the dose
b) Stop amoxicillin
c) Give low-dose amoxicillin
d) Give prophylactic Vancomycin
Patients requiring treatment with antimicrobials during or shortly after the completion of their C. difficile
therapy should receive concomitant oral vancomycin until the antimicrobials are stopped.
Severity Criteria Treatment
Mild Diarrhea plus any additional signs or symptoms Vancomycin 125 mg QID PO ˣ 10 days.
moderate not meeting severe or complicated criteria Fidaxomicin 200mg PO BID x 10 days
disease WBC ≤15 x 10^9/L and a serum creatinine Metronidazole 500 mg TID PO ˣ 10 days
(SCr) level of <133 mcmol/L
Severe Serum albumin < 3 g/dL + 1 of the following: Vancomycin 125 mg QID PO ˣ 10 days or;
disease WBC ≥15 ˣ 10^9/L & Abdominal tenderness Fidaxomicin 200mg PO BID x 10 days
Severe and Any of the following attributable to CDI: Vancomycin 500 mg QID PO +
complicated Admission to ICU for CDI Metronidazole 500 mg Q8H IV +
disease Hypotension ± required use of vasopressors Vancomycin 500 mg in 500 mL saline QID
“Fulminant” Fever ≥38.5 °C. Mental state changes PR as enema
Ileus or significant abdominal distension Surgical consultation suggested
WBC ≥35 ˣ 10^9/L or < 2 ˣ 10^9/L
Serum lactate levels >2.2 mmol/L
End organ failure, e.g., renal failure
Recurrent Recurrent CDI within 8 wk of completion of Repeat metronidazole or vancomycin pulse
CDI therapy regimen. Consider FMT Fecal microbiota
transplantation after 3 recurrences.

308. FR is a cancer patient receiving chemotherapy infusion five days per week. He got a
prescription for Xeloda 1.65 gm / meter square in two divided doses to be administered in
the same days with the chemotherapy infusion. Xeloda is only available only in 2 strengths:
150 and 500 mg. BSA is 1.73-meter square. How many tablets are required to be dispensed
for FR for 6-week supply? round dose to the nearest 50
a) 180 tablet of 500 mg strength & 120 tablets of 150 mg strength
b) 252 tablet of 500 mg strength & 150 tablets of 168 mg strength
c) 168 tablet of 500 mg strength & 180 tablets of 252 mg strength
d) 120 tablet of 500 mg strength & 180 tablets of 150 mg strength
Answer:
Required daily dose of Xeloda for the above patient = 1.65*1.73=2.8545 grams
Patient will need 5 doses per week for 6 weeks supply  So, 5*6=30 doses
Total drug required = 30* 2.8545=85.6535 grams almost 86 grams for the whole peroids
With a quick look to above choices only you will find choices either A OR D (available strengths only
150 & 500 mg)
Choice A = (180*0.5) + (120*0.15) = 108 grams (wrong choice).
Choice D= (120*0.5) + (180*0.15) = 87 grams (right answer).
309. A patient used to put contact lens, came with dry eye. Which sign to confirm dry eye?
a) Gritty sensation

310. What advice? No contact lenses

311. Amiodarone dose related side effects, which side effect occur early?
a) Eye
b) Skin
c) CNS
d) GIT
Because of the extensive distribution of amiodarone in body tissues, and the prolonged time required for
its elimination from the body following discontinuation of long‐term therapy, the relationship between
adverse reactions and dosage and duration of therapy, has not been fully established.
For some adverse reactions—for example, corneal microdeposits—a relationship to dosage and duration
of therapy has been established, so that corneal deposits are reversible with dose‐reduction or with
discontinuation of therapy. However, for other adverse reactions—for example, fibrosing alveolitis or
peripheral neuropathy—the dose relationship and the reversibility of the adverse reaction have not been
established. Certain gastrointestinal reactions (e.g., nausea, vomiting, constipation, and bad taste) and
central nervous system reactions (e.g., fatigue, headaches, vertigo, nightmares, and sleeplessness) occur
frequently at the initiation of therapy when high doses are used. These may disappear on reduction of
the dose. The time and dose relationship of adverse events are under continued study.
The most serious and potentially life‐threatening adverse effects associated with the use of amiodarone
hydrochloride are pulmonary fibrosis, the aggravation of arrhythmias, and cirrhotic hepatitis.
The adverse reactions most frequently requiring discontinuation of amiodarone hydrochloride have
included pulmonary infiltrates or fibrosis, paroxysmal ventricular tachycardia, congestive heart failure,
and elevation of liver enzymes. Other symptoms causing discontinuations less often have included visual
disturbances, solar dermatitis, blue skin discoloration, hyperthyroidism, and hypothyroidism.

312. Patient on amiodarone with other medication, which statin can be use?
a) Rosuvastatin
b) Atorvastatin
c) Lovastatin
d) Simvastatin
Can be used with pravastatin or rosuvastatin
HMG‐CoA reductase inhibitors that are CYP3A4 substrates (including simvastatin and atorvastatin) in
combination with amiodarone have been associated with reports of myopathy/rhabdomyolysis.

313. Patient is taking cyclosporine, what DDI problem?


a) Atorvastatin
b) Propranolol
c) Clindamycin
d) Erythromycin
Cyclosporine may reduce the clearance of digoxin, colchicine, prednisolone and HMG‐CoA reductase
inhibitors (statins) and etoposide.
314. Labeling requirement of baclofen
a) May cause drowsiness or dizziness
b) Avoid Grape fruit
c) Can be stopped abruptly
d) Avoid caffeine containing beverage
e) Can cause Abnormal hepatic function & Palpitation
Baclofen may be associated with adverse effects such as dizziness, sedation, somnolence and visual
impairment which may impair the patient’s reaction. Patients experiencing these adverse reactions
should be advised to refrain from driving or using machines. Patients should also be cautioned that the
CNS effects of baclofen may be additive to those of alcohol and other CNS depressants.
No studies have been performed in patients with hepatic impairment receiving baclofen therapy. As
baclofen does not undergo predominant hepatic metabolism, its pharmacokinetics is unlikely to be
altered to a clinically significant level in patients with hepatic impairment.

315. SSRI + tamadol + anti epiliptic ⇒ Inc seizers threshold, sedation, syndrome.

316. Pregnant + Anxiety. What to give?


a) Citalopram
b) Venlafaxine
c) Paroxetine

317. She used Citalopram 20 mg for 2 months, no response what is the appropriate ttt?
a) Change to Venlafaxine
b) Change to Paroxetine
c) Increase dose citalopram
d) Change to sertraline
If pharmacologic therapy is indicated, citalopram, escitalopram and sertraline are first‐line options.
Fluoxetine is a second‐line option since its association with major malformations is more controversial.
Paroxetine has been associated with risk for major malformations, and although the clinical magnitude
is questionable, it is still not recommended for perinatal depression, unless the mother was stable on it
prior to conception.
Second‐line options in the treatment of perinatal depression include bupropion, desvenlafaxine, duloxetine,
fluoxetine, fluvoxamine, mirtazapine, TCAs (except clomipramine and doxepin) and venlafaxine.
MAOIs and doxepin should be avoided during pregnancy.
Clomipramine should be considered only if the mother was stabilized on it prior to conception.
Other antidepressants have not been studied sufficiently in pregnancy to make recommendations on
their use (e.g., trazodone, vilazodone, vortioxetine, levomilnacipran).
318. Switching from citalopram to venlafaxine? cross tapering

319. 12-month baby with protien milk allergy & has enteropathy, what to give?
a) Soy milk
b) Lactose free milk
c) Pasturised whole cow milk
d) Extensively hydrolyzed cow milk
Hydrolyzed Can be either partially or extensively hydrolyzed. The 2 types cannot be used interchangeably.
Protein Partially hydrolyzed whey formulas
Formulas Contain lactose and are less expensive and more palatable than extensively hydrolyzed protein–
containing formulas. Used commonly as a substitute for or supplement to breast milk.
The perceived benefits of partially hydrolyzed whey formulas, such as fewer spitting-up episodes
and softer stools, are likely related to the beta-lactoglobulin (which remains soluble in the
stomach) moving faster to the upper jejunum.
These benefits may be more pronounced in children with underlying GERD.
Extensively hydrolyzed protein–containing formulas
Contain casein proteins that have been heat treated and enzymatically hydrolyzed into peptide
chains and free amino acids. More expensive and less palatable than milk-based formulas.
They are recommended for infants with intolerance to intact cow's milk protein and soy protein.
Extensively hydrolyzed formulations benefit infants with malabsorptive diseases such as short
bowel syndrome, liver disease, cystic fibrosis and intractable diarrhea.
Infants with cholestasis and lymphangiectasia also benefit from extensively hydrolyzed
formulations, especially those containing a higher percentage of medium-chain triglycerides.
Extensively hydrolyzed protein– containing formulas are lactose-free, and the carbohydrate is
usually corn syrup solids, cornstarch and occasionally sucrose.
Infants who cannot exclusively breastfeed and who are at high risk of allergy and atopic disease
should be given extensively hydrolyzed protein formula, which may delay or prevent occurrence.
320. Vegetarian Patient with Diabetes, anemia taking a lot of drugs erythropoetin, 2 ferrous
sulfate BID, iron level 130, Lab low Hb, normal MCV & ferritin. What the right action?
a) Stop epeotin
b) Continue epoetin and stop ferrous
c) Stop both
d) Continue both
Iron must be administered with ESA therapy to avoid depletion.

321. According to his lab. Tests, which type of anemia he suffers or he has deficiency in?
a) Thiamine
b) Cyanocobalamin (Pernicious anemia vitamin B12 deficiency)
c) Ferrous
d) Pyridoxine

322. What is the rare but serious side effect of Fluoroquinolone group?
a) HF
b) Tendonitis
c) Aortic aneurysm
d) Pseudotumor cerebri
Tendinitis and tendon rupture are rare but potentially disabling reactions associated with all systemic
fluoroquinolones (ciprofloxacin, levofloxacin, moxifloxacin, norfloxacin). The Achilles tendon is most
commonly affected, but tendinopathy may involve any tendon including the quadriceps, femoris, gluteal,
triceps, rotator cuff, finger flexors, or the thumb tendon as well as plantar fascia. Sudden severe pain is
usually the initial symptom. The reaction is often bilateral, which can aid in diagnosis. Onset can be
early, even the 1st day of starting treatment, while tendon ruptures may occur up to 6 months after
discontinuation. Rupture of the shoulder, hand and Achilles tendons have required surgery. Recovery
may be slow with prolonged disability over days to months. Patients who are over 60 years of age,
taking corticosteroids, undergoing hemodialysis, transplant recipients or with a history of
fluoroquinolone‐induced tendinopathy are at increased risk. One case occurred with ophthalmic
administration of moxifloxacin. If symptoms occur, discontinue therapy immediately and advise rest of
the afflicted area to prevent worsening of the condition.
QTc interval prolongation, torsades de pointes, palpitations, cardiac arrest, ventricular arrhythmia and
ventricular tachycardia; aortic aneurysm and dissection.
CNS effects: Fluoroquinolones have been associated with an increased risk of CNS effects, including
seizures, increased intracranial pressure (including pseudotumor cerebri), dizziness, and tremors. Status
epilepticus cases have been reported. Use with caution in patients with a history of seizures, with known
or suspected CNS disorder (severe cerebral arteriosclerosis, previous history of convulsion, reduced
cerebral blood flow, altered brain structure, or stroke), or with other risk factors that may predispose to
seizures or lower the seizure threshold (eg, certain drug therapy, renal dysfunction). Discontinue if
seizures occur and institute appropriate therapy.
Quinolone most SE questiosn, dysglycemia, QT prolongation, tendonitis and newly one is retinal
detachment
NOT cause Nephrotoxicity
323. Case: female with genital herpes simplex. ttt? Acyclovir dose….
One of the most common sexually transmitted infections, caused by HSV‐2, which is transmitted through
contact with genital skin, sores or fluids.
May be asymptomatic or present as papules and vesicles on the groin, pubic area, genital area, anus,
rectum or buttocks that can rupture and lead to painful ulcers.
Primary Treatment is effective in reducing the severity and duration of symptoms if initiated up to 7 days
Episodes after onset. Topical antiviral therapy has no effect in primary episodes of genital herpes
of Genital Oral acyclovir is effective, but severe genital herpes requires IV acyclovir 5mg/kg Q8h x 5-10days.
Herpes Famciclovir and valacyclovir have comparable efficacy and tolerability to that of oral acyclovir.
The simplicity of famciclovir and valacyclovir dosing regimens is an advantage over acyclovir.
Recurrent In immunocompetent patients, treat with oral acyclovir, famciclovir or valacyclovir.
Genital Shorter courses (1–3 days) initiated within hours of symptom onset appear to be effective as 5-day
Herpes treatments for episodes of recurrent genital herpes in healthy individuals.
For individuals with recurring symptomatic genital herpes (e.g., ≥ 4 episodes per year), chronic
suppressive therapy with oral acyclovir, famciclovir or valacyclovir for 2–12 months decreases the
number of recurrences.
Valacyclovir, 500 mg or 1000 mg, is the only treatment approved for once-daily dosing.
Tenofovir, a nucleotide reverse transcriptase inhibitor used to treat HIV, effective against HSV-2.
Interrupt suppression periodically to evaluate the need for continued treatment as the frequency of
outbreaks tends to decrease over time.
One strategy is to stop every 3–6 months and to await 2 recurrences. A second 3- to 6-month course
would be appropriate only if these 2 recurrences are close together (≤2 months apart). This strategy
can be continued indefinitely
Genital Primary: In HIVpatients, first episodes may be more severe & prolonged than in non-HIV infected
Herpes IV and oral regimens recommended for immunocompetent patients.
and HIV Some experts recommend higher doses and longer duration of therapy than that used in HIV-
negative patients, as oral acyclovir 400 mg 3–5 times daily, until healing complete.
Recurrent: treat with acyclovir 200–400 mg 5 times per day, famciclovir 500 mg BID or
valacyclovir 1000 mg BID for 5–7 days. Shorter 1- and 3-day courses have not tested.
Oral acyclovir does not seem to reduce transmission of HSV-2 from HIV-1/HSV-2 co-infected
individuals to their partners. It is important that optimal antiretroviral therapy be coadministered to
prevent reductions in immune function as indicated by CD4 count and HIV viral load.
Herpes Refers to the inflammation of the rectal mucosa caused by HSV infection.
proctitis It can be acquired by anal intercourse or through oral-anal contact.
Symptoms include anorectal pain and mucopurulent or bloody rectal discharge.
Oral acyclovir shown to reduce healing time for first-episode herpes proctitis from 14 to 5 days.

324. Then, she is having multiple partners and got sore, may be syphilis. ttt? Ampicillin

325. Patient comes with Single large non-painful chancer on penis (clear syphilis diagnosis).
What is the best treatment?
a) Cefixime 800 mg PO one dose +
Azithromycin 1 gm one dose
b) Azithromycin 1 gm single dose
c) Penicillin G benzathine one dose
d) Ceftriaxone 2 gm single dose IM
326. Pt central BP 137/80, PR 80, pot 5, on bisoprolol + ACEI, what to add?
a) Change to valsartan
b) Change furosemide to spironolactone

327. 1st pychosis event for how long ttt?

328. Timolol + latanoprost eye drops. Which of the following would not preserve its
stability? Or; All of the following affects stability, EXCEPT?
a) Use for more than 4 weeks after opening
b) Touching the tip of the eye
c) Store in the refrigerator after opening
d) Keep away from light

329. What is not correct about timolol? After open keep in fridge

330. All of the following are roles of Public health agency of canada PHAC, except?
a) Chronic Disease prevention & control
b) Emergency infection management
c) Promotion of travel infection prevention
d) Managing public health insurance plan
331. You are working at a pharmacy that make very low profit. Which ethic is upholding
when you make a drug review for a patient and didn’t make any change on his medication?
a) Autonomy
b) Veracity
c) Fedility
d) Conflict of interest

332. What is the best strategy to avoid administering oral medication via parenteral route?
a) Use amber syringe instead of transparent ones
b) Use a syringe that is incompatible with the injectable part
c) Put a label on metric and non-metric syringe with oral use only

333. How to dispense oral liquid in hospital pharmacy


a) Syringe with for oral use only
b) Syringe which can not be attached to any needle used in hospital
c) Amber bottle
d) Disposable bottle
334. Employer offer to his employee, insurance plan with monthly premium & give him one
month to reply. He should pay 100 dollars per month. You checked his profile and found out
he only used 2 antibiotics in last year. He wants your opinion, what to tell him?
a) Don’t take the offer
b) When he takes the offer, it will decrease the premium for other employees
c) He should know first if he would be excluded, if he did not reply in one month

335. Which of the following should be prepared under horizental flow hood?
a) Vincristine
b) Amoxicillin suspension
c) Eye drops
d) Doxorubicin

336. Technician received call for verbal order & made error by writing the drug name wrong
but he discovered it from the dose & how many to take it per day & then you called the
physician & corrected it, what to prevent this error?
a) Only pharmacist receive the call
b) Technician read it back to the prescriber over the phone policy or Repeat order
after doctor
c) Tech do it under pharmacist supervision
d) Take the order only if the pharmacy is quiet
e) Pharmacist has to double check

337. HIV patient on triple therapy in the hospital got 2 antiretrovirals instead of 3 at his
room. What to do to avoid this error from happening in the future:
a) Educational seminars for nurses to teach them about the importance of the
combination of 3 antiretrovirals in the management of HIV
b) Put a pop-up screen in the computer systems to prevent dispensing less than 3
antiretroviral medication
Another version
Veracity case: AIDS patient given only 2 out of 3 of his drugs in hospital.
a) Call and inform his Dr about the error.
338. Pt takes Cyanocobalamin injection for pernicious anemia. Manufacturer stopped production
for a long time with no plan to be in the market in the near future. What should you do:
a) Give the patient PO high dose cyanocobalamin
a) Order cyanocobalamin by SAP
b) Call manufacturer to ask him to reproduce cyanocobalamin
c) Wait till it come
Vitamin B12 has traditionally been given parenterally because deficiency is most often due to
malabsorption, and most cases of malabsorption are attributable to pernicious anemia with its lack of
intrinsic factor. High‐dose oral vitamin B12 therapy is effective, feasible and cost‐effective, but
limitations include patient adherence and the need for more attentive monitoring.

339. Which is the best way used by the pharmacist to communicate circle of care about patient
penicillin allergy:
a) Include penicillin allergy in Medication Administration Record
b) Call all physician and nurses to inform them about the allergy
c) Update patient profile in the pharmacy

340. Diabetes + Glucoma with High BP, was on cortisone for a while.
Pt on fluticasone inhaler & eye condition? What is interact? fluticasone inc IOP

341. Case PEG1 dose is high as twice day


A patient with drugs one of PEG was high twice was the choice
When to visit the Dr? Temp or diarrhea
‫ ﺍﻟﻠﻲ ﻓﺎﻛﺮﺍﻩ ﺍﻧﻪ ﻛﺎﻥ ﻛﺎﺗﺐ ﺍﻟﺒﺮﻭﻓﺎﻳﻞ ﺑﺘﺎﻉ ﺍﻟﺮﺍﺟﻞ ﻭﺍﻻﺩﻭﻳﻪ ﺍﻟﻠﻲ ﺑﻴﺎﺧﻮﺩﻫﺎ ﻭﻛﺎﺗﺒﻠﻚ ﺍﻧﻪ ﺟﺎﻱ ﺏ‬symptoms ‫ﻣﺶ ﻓﺎﻛﺮﺍﻫﺎ‬
‫ ﺑﺮﺿﻪ ﻭﺑﻴﻘﻮﻟﻚ ﺩﻱ ﻣﻦ ﺍﻳﻪ ﻭﻛﺎﻥ ﻣﻦ ﺿﻤﻦ ﺍﻻﺧﺘﻴﺎﺭﺍﺕ‬high dose peg ‫ﺩﻱ ﺍﻟﻠﻲ ﺍﺧﺘﺮﺗﻪ‬
‫ﻫﻮ ﺍﻟﻠﻲ ﻛﺎﻥ ﻣﻠﻔﺖ ﺍﻧﻪ ﻛﺎﻥ ﺑﻴﺎﺧﺪﻩ ﺑﻘﺎﻟﻪ ﻣﺪﻩ ﺣﻠﻮﻩ ﻭﻣﺮﺗﻴﻦ ﻓﺎﻟﻴﻮﻡ‬
‫ ﻓﻜﺎﻥ ﺍﻟﻔﻜﺮﻩ ﻫﻮ ﻣﺎﺷﻲ ﻋﻠﻴﻪ ﻟﻴﻪ ﻛﻞ ﺩﻩ ﻭﻣﺎﻋﺘﻘﺪﺵ ﻛﺎﻥ ﻓﻴﻪ‬opioid ‫ ﻓﺄﺩﻭﻳﺘﻪ ﺍﻭ ﺍﻱ ﺣﺎﺟﻪ ﺑﻨﺪﻱ ﻣﻌﺎﻫﺎ‬laxative ‫ﻓﺘﺮﻩ ﻁﻮﻳﻠﻪ‬
‫ﻭﻛﺎﻥ ﻣﻦ ﺿﻤﻦ ﺍﻻﺳﺌﻠﻪ‬
what to change in his medication?
‫ﺳﺎﻋﺘﻬﺎ ﺍﻧﺎ ﻓﺎﻛﺮﻩ ﻭﻗﻔﺘﻬﻮﻟﻪ ﺑﻼﺵ ﻭﺟﻊ ﺩﻣﺎﻍ‬
In the event of overdosage, diarrhea would be the expected major event. If an overdose of
drug occurred without concomitant ingestion of fluid, dehydration due to diarrhea may
result. In the event of overdose, medication should be terminated and free water
administered.
342. Council of drospirenone
Drospirenone is related to the aldosterone antagonist spironolactone and has both progestational and
antiandrogenic activity. A Cochrane review of 5 trials shows drospirenone‐containing COCs may benefit
women with premenstrual dysphoric disorder (PMDD), but there is limited evidence of benefit for milder
disease or for greater than 3 cycles of use. Several observational studies concluded that drospirenone
containing COCs may confer a higher risk of venous thromboembolism than COCs containing other
Progestins.
COCs Comments
Combined estrogen (35 mcg ethinyl estradiol) S.E: Major: thromboembolism (rare), stroke, hypertension,
& progestin, monophasic retinal artery thrombosis, MI, benign liver tumor, cholelithiasis.
Ethinyl estradiol 35 mcg/ norethindrone 0.5 mg Watch for danger signals ACHES: abdominal pain, chest pain,
or 1 mg headaches, eye problems, severe leg pain  Consult physician.
Ethinyl estradiol 35 mcg/ norgestimate 0.25 mg – Common: breakthrough bleeding/spotting, amenorrhea, nausea
Cyclen / vomiting, bloating, chloasma, breast tenderness, mood
Combined estrogen (30 mcg ethinyl estradiol) changes such as depression, headaches.
& progestin, monophasic May increase cyclosporine levels or hepatotoxicity.
Ethinyl estradiol 30 mcg / desogestrel 0.15 mg – May decrease lamotrigine levels.
Marvelon Drospirenone: Risk of hyperkalemia in patients prone to
Ethinyl estradiol 30 mcg / drospirenone 3 mg – increased K+, e.g., renal disease, concomitant ACEI, ARB, K-
Yasmin (for acne also) sparing diuretics, NSAID. Q. Check K+ level after 1st cycle (or
Ethinyl estradiol 30 mcg / ethynodiol diacetate 2 1 month).
mg - Demulen 30 May increase risk of VTE compared to LNG containing OCs.
Ethinyl estradiol 30 mcg / levonorgestrel 0.15 mg Significant pharmacokinetic interaction with rifampin,
- Min-Ovral griseofulvin (advise backup barrier method during therapy).
Ethinyl estradiol 30 mcg / norethindrone acetate Monitor INR with concurrent oral anticoagulant use.
1.5 mg Carbamazepine, phenytoin, protease inhibitors, phenobarbital,
Combined estrogen (20 mcg ethinyl estradiol) St. John's wort, topiramate may decrease ethinyl estradiol /
& progestin, monophasic progestin serum concentrations.
Ethinyl estradiol 20 mcg / drospirenone 3 mg – Contraindications:
Yaz (for acne) Absolute: history of MI or ischemic heart disease, complicated
Ethinyl estradiol 20 mcg/ drospirenone 3 mg/ valvular heart disease, cerebrovascular disease, current or past
levomefolate calcium 0.451 mg - Yaz Plus history of VTE, known thrombogenic mutation, severe
Ethinyl estradiol 20 mcg/ levonorgestrel 0.1 mg – cirrhosis, liver tumor, breast cancer, DM with microvascular
Alesse complications, migraines with aura, < 6 wk postpartum if
Ethinyl estradiol 20 mcg/ norethindrone 1 mg – breastfeeding, smoker >35 y (≥15 cigarettes/day), HPT
Minestrin (systolic BP ≥160 mm Hg or diastolic BP ≥100 mm Hg),
Combined estrogen (10 mcg ethinyl estradiol) known coagulation-factor deficiency.
& progestin, monophasic - LOLO Relative: estrogen hypersensitivity, migraine, gallbladder
Ethinyl estradiol 10 mcg / norethindrone 1 mg disease, high BMI/weight. Low-dose COCs are relatively
Combined estrogen and progestin, triphasic contraindicated if BP ≥140/90.
Ethinyl estradiol 25 mcg / desogestrel 0.1 Lower-dose COCs are method of choice for most young
mg/0.125 mg/0.15 mg – Linessa couples, especially for teens, if combined with condoms;
Ethinyl estradiol 35 mcg / norethindrone 0.5 mg/1 products with lower dose of ethinyl estradiol have increased
mg/0.5 mg – Synphasic safety, ↓ side effects; condoms needed for STI protection.
Ethinyl estradiol 30 mcg/40 mcg/30 mcg/ Patients with diarrhea or breakthrough bleeding may be at
levonorgestrel 0.05 mg/0.075 mg/0.125 mg higher risk of contraceptive failure.
Starting OC’s
1st tablet either on the day of menses (eliminates need for alternate means of contraception) or take 1st tablet on
first Sunday after beginning of menses and use extra contraception method for 1st 7 days of OC’s
OCs are best taken in evening before bedtime, to decrease side effects as nausea, breast tenderness, Q. chloasma
(exacerbated by sunlight when estrogen concentration is high and can be prevented by use of sunscreen & wide
hats).

343. Ph want to open non sterile compounding in pharmacy, which of the following is correct:
a) Everybody should be trained
b) You only need to get one training
c) Person who does compounding should be assessed periodically
d) Person who knows how to do it, no need for certificate
e) One technician who is familiar with compounding should be responsible for
compounding
f) Compounding technician should receive annual compounding certificate
g) Compounding operation should be performed only under pharmacist supervision

344. You are a pharmacist in hospital which prepare sterilized stuff, what is the best way to
manage the preparations?
a) You as a pharmacist should supervise all magistrates
b) You let experienced tech supervise fresh tech graduate
c) You hire experienced tech in sterile preparation and let him do everything by himself
d) Pharmacist or tech should do the calculation before the tech prepare the magistrate
preparation

345. Patient come with new RX has many Drug interaction and uncommon doses. Which
from below represent collaboration
a) Send fax to doctors with your concern
b) Talk to the patient about your concerns
c) Print your concern and document everything in the file of the patient
346. What is the purpose of medication reconciliation:
a) To avoid contraindications and possible side effects
b) To give the patient a list of his medication
c) To consolidate the medication and to minimize duplications and errors
d) Decrease medication error
e) Assess pt adherence
What are the benefits of MedRec?
 MedRec conducted in a primary care clinic significantly reduced (from 26% to 6%) the proportion of
visits with missing medication lists and reduced prescription medication errors by more than 50%
 After MedRec was implemented in 4 academic primary care clinics, completeness of medication lists
improved from 20.4% to 50.4% (p < 0.001)
 Among patients who received MedRec 3 to 7 days post discharge, there was a statistically
significant decrease in readmission rates at days 7 and 14
 Reduces preventable drug‐related adverse events

347. You are a pharmacist in pain control clinic, what is the most effective way to detect any
opioid abuse?
a) Check for early refills
b) Make urine test for all patient every 3 months
c) Make PHQ9 for all patient every visit to detect any mental disease.
If aberrant behaviours continue, prescribers should respond with more intensive monitoring to reduce
risk, e.g., dispense weekly or even twice weekly if running out early, increase frequency of urine drug
testing if unexpected results occur.

348. 25 old pt have nausea, vomiting & abdominal pain, that not relieve with medication
only with hot shower and bath. What to ask him?
a) Cannabis use (hyperemesis syndrome)
b) Worsen with hot shower
c) Do have GERD or not
Cannabis hyperemesis syndrome (CHS) has been reported in individuals who have used cannabis
chronically for many years.
Symptoms include episodes of severe abdominal pain, nausea and vomiting that are relieved by
bathing/showering in hot water.
Treatment includes rehydration and supportive care as well as slow tapering of cannabis and
psychological counselling; antinauseants or benzodiazepines have conflicting evidence for symptom
relief.

349. How do u prevent pain when you inject?


a) You squeeze hand and inject
b) Put it on the upper arm to lower tissue penetration
350. ITT alpha 1 error
Intent to treat (ITT) analysis
 Means all patients who were enrolled and randomly allocated to treatment
are included in analysis and are analyzed in the groups to which they were
randomized. “Once randomized always analyzed”
 It is a statistical approach used for RCTs that taken into consideration all
subjects in treatment and control group including any non-adherence to
the study protocol (e.g. taking/administering treatment by the patient,
dropout or protocol deviation). It ignores noncompliance, withdrawal,
protocol deviation, or anything happened after randomization.
 Advantages: Avoid overestimation of drug effect, maintain sample size,
reduce type I error & more consistent with clinical practice.
 Disadvantages: susceptible to type II error & dilution of the drug effect.
Non-intent to treat analysis (Non-ITT): Aim to estimate the effect of treatment as delivered or as received
(as opposed to assigned) to account for non-adherence. Can be used as interim.
A per-protocol analysis: Opposite end of the spectrum from ITT analysis. It is an interpretation of
randomized clinical trial results that removes data from patients who didn't comply with the protocol.
Imagine a trial designed to test experimental drug A against standard treatment B. If some patients drop out
of the trial before investigators can measure the primary outcome, a per-protocol analysis wouldn't include
their results with patients who completed treatment. Its results represent the best-case treatment results that
could be achieved if the study sample were retained and remained compliant with treatment.

351. Pt can’t remember wording, what resource would you look in? Micromedex
Micromedex American. Alternative to CPS: Drug monographs, Drug identification, interactions, IV
compatibility, Calculations, Patient education, off label uses & Toxicology
Micromedex, pre‐2019 CPS and LexiComp have drug identification tools.
352. Vaccine info? Public health unit
Public Health Units
 Public health units must conduct cold chain incident and annual routine inspections.
 Contact your public health unit for assistance with vaccine storage and handling practices.
 Public health units are required by the Vaccine Storage and Handling Protocol under the Ontario
Public Health Standards, issued under the authority of the Health Protection and Promotion Act
to respond to reports of all cold chain incidents and to inspect premises, at least once annually,
where provincially funded vaccines are stored.
 The purpose of cold chain incident and routine (annual) inspections is to: Ensure the proper
management of vaccine inventories; Provide education strategies to minimize vaccine wastage;
Reduce provincially funded vaccine wastage; and Promote vaccine safety and effectiveness.

353. Deprescribing of omeprazole 20, PPI used for long time patient 24 yrs use. He wants to
stop, how to do this?
a) Every other day
b) Omperazole 10 then 5
c) Stop at once without taper
Omeprazole
Delayed-release Do not crush or split. Swallow whole. Do not open capsule.
capsules: 20 mg Tapering Options:
Losec Capsules  Reduce from twice daily to daily
 Reduce from daily to every other day
 On-demand dosing
Delayed-release Do not crush or split. Swallow whole.
tablets: 10 mg, 20 Tapering Options:
mg Losec Tablets,  Reduce from twice daily to daily
Omeprazole, other  Reduce from 20 mg daily to 10 mg daily
generics  Reduce from daily to every other day
 On-demand dosing

354. Patient palliative care wants to stop all the medication except that helps him to improve
quality of life. Which of the following medication will be stopped?
a) Aspirin
b) Gabapentin
c) Duloxetine
d) Tylenol 2
e) Clonazepam
f) Ondansetron
END of LIFE care: Therapeutic Choices
Certain principles are essential for optimal symptom management in the palliative and end‐of‐life
setting:
 Any symptom is as distressing as a patient claims it to be.
 Treatment risks, benefits and alternatives need to be discussed in the context of the dying
patient’s values, culture, goals and fears.
Medication needs to be reviewed; the 3 general principles of prescribing at the end of life include:
- stop non‐essential drugs
- convert essential drugs to parenteral route
- use anticipatory prescribing
Cause of symptoms is irrelevant and investigation is pointless when disease is advanced and death
very near, unless detection would direct a useful change in symptomatic treatment.

355. Composite end point


a) Rare end point
b) You can use it after collect data
c) 3 endpoints together
Nov 17: What is true about composite endpoint of a study EXCEPT?
a) Lots of question is answered
b) Rare parameters can be included in the composite endpoint
c) Lots of factor are considered in the composite endpoint
d) Outcome is very simple
Composite endpoint Surrogate (marker) endpoint
It is endpoint that is a combination of multiple clinical endpoints. In treatment of some diseases, it may take
Ex: studies of clopidogrel for the prevention of vascular ischemic very long time to observe the definitive
events use combinations of MI, stroke, death, and re-hospitalization endpoint (e.g., death).
as components of composite endpoints. A surrogate endpoint is a measure that is
It can be primary or secondary composite endpoints: predictive of the clinical event but takes a
 Primary: the main measurements for a trial; they answer most shorter time to observe.
important questions in the trial. Ex, if endpoint of a study is The definitive endpoint often measures
“cure”, then composite endpoint might be cure or remission. clinical benefit whereas surrogate endpoint
 Secondary: the secondary objectives in the trial. Ex, a drug tracks the progress or extent of disease.
designed to cure/put into remission a disease might also have It measures effect of a specific treatment
measures of whether chronic pain and quality of life improved. that may correlate with a real clinical
Advantages of composite endpoints endpoint but does not necessarily have a
 Result in more completed characterization of intervention guaranteed relationship.
effects as there may be interest in a variety of outcomes. Surrogate endpoints could also be used
 Result in higher power and resulting smaller sample sizes in when clinical end-point is too expensive or
event-driven trials since more events will be observed difficult to measure, or not ethical to
(assuming that the effect size is unchanged). measure.
 Reduce bias due to competing risks & informative censoring The criteria for a surrogate marker are:
 Help avoid multiplicity issue of evaluating many endpoints 1. The marker is predictive of the
individually. clinical event
Composite endpoints have several limitations. 2. The intervention effect on the clinical
 Significance of the composite does not necessarily imply outcome manifests itself entirely
significance of the components nor does significance of the through its effect on the marker.
components necessarily imply significance of the composite. For Example:
For example, one intervention could be better on one 1) Hypertension: arterial blood pressure
component but worse on another and thus result in a non-  surrogate for CVA, MI & HF
significant composite. 2) Hypercholestrolemia: cholesterol
 The interpretation can be challenging particularly when the levels  surrogate for atherosclerotic
relative importance of the components differs and the disease
intervention effects on the components also differ. For 3) HIV: CD4 count or viral load 
example, how do we interpret a study in which the overall surrogate for complications of HIV
event rate in one arm is lower but the types of events 4) Glucoma: intraocular pressure 
occurring in that arm are more serious? Higher event rates and surrogate for loss of vision
larger effects for less important components could lead to 5) Diabetes mellitus: blood glucose /
misinterpretation of intervention impact. hemoglobin A1c  surrogate for
 It is also possible that intervention effects for different complications
components can go in different directions. Power can be 6) Albuminuria is a biologically
reduced if there is little effect on some of the components (i.e., plausible surrogate endpoint for the
the intervention effect is diluted with the inclusion of these progression of chronic kidney disease
components).

356. Diabetic + neuropathic pain. what is the non ph. MG of diabetes duration of pain?
Neuropathic pain is often severe and debilitating. Although evidence may be lacking, many
nonpharmacologic strategies (e.g., physiotherapy, mindfulness, yoga, exercise, psychotherapy) can be used
to provide benefit in some patients and should be combined with pharmacotherapy.
Attempt to reduce sleep deprivation and improve physical conditioning to rehabilitate patients and improve
their quality of life.
Patients may minimize chronic neuropathic pain by getting adequate rest and avoiding further aggravation.
Continued medical visits provide important psychological support and hope for desperate patients who
have failed standard therapy and facilitate a trial‐and‐error approach.

357. Pt admitted to hospital experiencing alcohol withdrawal symptoms, what to give initial?
a) Thiamine for treatment/prevention of Wernicke-Korsakoff syndrome
b) Lorazepam for autonomic hyperactivity, agitation/tremor, hallucinations, seizures
c) Phenytoin for seizure
d) Haloperidol for hallucination
e) Clonidine for chills and flushing
Approximately two‐thirds of patients with mild to moderate withdrawal symptoms can be managed
with supportive measures and monitoring. Pharmacologic treatment is always required for moderate to
severe alcohol withdrawal and may be required for about one‐third of patients with mild to moderate
symptoms. Most cases respond well to benzodiazepines, as they reduce the hyperactivity of GABA
receptors that occurs during withdrawal. A benzodiazepine protocol, with dosing based on the patient’s
CIWA‐Ar score, can be used in the primary care setting to alleviate withdrawal symptoms.

358. Naloxone and acambrosate for alcohol abstinence, what is right?


a) Acambrosate is better than naltrxone in alcohol abstinence
b) Naloxone decrease craving more than acambrosate
c) Acambrosate not for complete abstinence
Naltrexone Naltrexone is an opioid antagonist, thought to decrease the euphoria related to endogenous
opioid release upon drinking alcohol, decreasing the amount of the neurotransmitter dopamine,
thus making drinking less pleasurable. Also reduces cravings for alcohol.
Treatment of choice in patients who wish to reduce use but not completely abstining.
Q. 50 mg PO once daily 1-hour befor drinking alcohol. Given potential risk of hepatotoxicity
Contraindicated with concurrent opioid therapy due to precipitation of opioid withdrawal.
Patient must be opioid-free for ≥ 7 days prior to initiation of treatment.
Use with caution if LFTs more than 5× the upper limit of normal
Acamprosate Acamprosate (glutamate and GABA modulator), believed to underlie its ability to relieve
Treatment of symptoms of alcohol withdrawal and reduce the euphoric effects of alcohol.
choice for May be preferred in patients with goal of abstinence.
patients on 666 mg PO TID. Start treatment after ≥ 4 days of alcohol abstinence.
opioid It is the treatment of choice for patients with hepatic insufficiency, since it is renally excreted
therapy. S.E: Diarrhea, vomiting, abdominal pain, pruritus, rash; suicidality (suicidal thoughts /attempts
/completed suicide) has been reported.

359. Which of the following medications is used for chills and flushing during alcohol and
opioid withdrawal
a) Diazepam
b) Nabilone
c) Clonidine
d) Trazodone

360. TTT of opioid withdrawal induced diarrhea?


a) Levodopa
b) Clonidine
c) Octreotide
Alpha2- Use only for acute detoxification. 0.1–0.2 mg Q6– 8H PO PRN for 5–7 days
adrenergic For Autonomic symptoms (sweating, tachycardia, myoclonus).
Agonists Effective for opioid-withdrawal diarrhea.
Clonidine Blunts some withdrawal symptoms, e.g., chills, flushing.
S.E: Sedation, dry mouth, orthostatic hypotension, dizziness.
Additive effects with other CNS depressants such as ethanol and opioids.
Use with caution in those with heart disease and hypertension.
Prolonged use of clonidine may result in rebound hypertension, thus taper slowly.
Maintain fluid intake and monitor for hypotensive effects; hold next dose if blood pressure
<85/55 mm Hg.
Somatostatin analogues can be administered subcutaneously daily (octreotide) or intramuscularly
monthly (octreotide acetate, lanreotide acetate). These agents have been used to control diarrhea
caused by neuroendocrine tumors (VIPoma, carcinoid, medullary carcinoma of the thyroid). Octreotide
also limits idiopathic and infant secretory diarrhea, as well as diarrhea associated with ileostomy, short
bowel syndrome, diabetic neuropathy, chemotherapy, bone marrow transplant, cryptosporidia, graft
versus host disease, and HIV infection. Somatostatin has a short half‐life and requires continuous IV
infusion, which limits its role in the management of diarrhea.

361. College prevent doctor from writing narcotic, what Rx you can receive from this doc?
a) Methylphenidate
b) All other was narcotic options
362. Which of the following medication require a written prescription to be dispensed:
a) Methylphenidate
b) Fiorinal C1/4
c) Granisterone
d) Oxycodone

363. What to not use in?


a) Fligrastim in sickle cell anemia
Chemotherapy‐induced myelosuppression in nonmyeloid malignancies:
Neupogen and filgrastim biosimilars: To decrease the incidence of infection (neutropenic fever) in
patients with nonmyeloid malignancies receiving myelosuppressive chemotherapy associated with a
significant incidence of severe neutropenia with fever

364. TS has a prescription for amitriptyline. By error she received amlodipine from the
pharmacy instead. Few days later she discovered the error what will be the first action by the
pharmacist:
a) Call TS, inform about the error & ask about side effects of the medication
b) Invite her in a private meeting in the pharmacy to discuss the incident
c) Ask her to get the amlodipine back to the pharmacy and change to amitriptyline
d) Call the physician to ask him how to deal with hypotension caused by the error
e) Discipline the technician who is responsible for
Ask about patient safety first

365. Patient has Rx of triazolam and hydromorphone what is the pharmacist concern
a) Short acting benzodiazepine abuse
b) Long acting benzodiazepine abuse
c) Combination of benzo with opioid
CNS depressants (e.g., sedatives, alcohol) may increase CNS depression.
366. You are working in a pharmacy in a province that has obligatory regulation to perform
10 medication check per week for patients who has 3 or more chronic diseases. this will
cause It work-overload. Pharmacy manager is discussing with you about how to execute this
regulation, what should be your response:
a) Perform medication checks for patients who has not more than 3 chronic diseases
b) Perform medication checks and tell other patients that dispensing will take longer
waiting times.
c) Let the technician medication checks in busy hours
d) Ask the manager to hire new staff to perform medication checks
e) Refuse to perform medication checks
Maybe B or D
FOR B
You can offer them delivery to their med or give them buzz so they can Rome around till its
ready or text or call whenever ready. Usually, the wait time at the pharmacy 20-30 min even
if ut will be prepared in 10 min they leave a wide gap
FOR D
‫ﻟﻠﻲ ﺍﻋﺮﻓﻪ ﺍﻥ ﺍﻟﻤﻮﺿﻮﻉ ﺩﻩ ﺍﻟﺤﻜﻮﻣﻪ ﺑﺘﺪﻓﻊ ﻣﺒﻠﻎ ﻛﻮﻳﺲ ﻋﻠﻴﻬﺎ ﻛﻞ ﺳﻨﻪ ﻋﻠﻲ ﻛﻞ ﻣﺮﻳﺾ‬
‫ﻓﻌﺎﺩﻱ ﺍﻥ ﺍﻟﺼﻴﺪﻟﻴﻪ ﺗﺠﻴﺐ ﺣﺪ ﻳﻌﻤﻠﻪ ﻭﻣﺶ ﻫﻴﺒﻘﻲ ﺗﻜﻠﻔﻪ ﺯﻳﺎﺩﻩ ﻋﻠﻲ ﺍﻟﺼﻴﺪﻟﻴﻪ ﺑﺎﻟﻌﻜﺲ ﻫﺘﻜﺴﺐ‬
‫ﻫﻮ ﻣﺶ ﺑﻴﻴﺠﻲ ﻛﻞ ﻳﻮﻡ ﻳﻌﻨﻲ ﻓﻤﺴﻴﺴﺎﺟﺎ ﻛﺎﻥ ﻓﻴﻪ ﺻﻴﺪﻟﻴﻪ ﺑﺘﻌﻤﻠﻪ ﻳﻮﻡ ﺍﻟﺴﺒﺖ ﻭﺑﻴﺪﻭﺍ ﻣﻌﺎﺩ ﻟﻠﻤﺮﺿﻲ ﻳﺠﻮﺍ ﺍﻟﻴﻮﻡ ﺩﻩ‬
‫ﻭﻓﺎﻟﺒﺮﺗﺎ ﻛﺎﻧﻮﺍ ﺑﻴﺨﻠﻮ ﺍﻻﻧﺘﺮﻥ ﻫﻮ ﺍﻟﻠﻲ ﻳﻌﻤﻠﻪ ﻫﻮ ﺍﺻﻼ ﺷﻐﺎﻝ ﺑﺒﻼﺵ ﺑﺲ ﺩﻟﻮﻗﺘﻲ ﻣﻔﻴﺶ ﺍﻧﺘﺮﻥ ﺑﻘﻲ ﺑﺮﺿﻪ ﺑﻴﺠﻲ‬
‫ ﺻﻴﺪﻟﻲ ﻳﻮﻡ ﺍﻭ ﺍﺗﻨﻴﻦ ﻳﻌﻤﻠﻪ ﻭﻛﻞ ﺩﻩ ﺑﻴﺒﻘﻲ‬overlap
‫ﻟﻜﻦ ﺍﻗﻮﻝ ﻟﻠﻤﺮﺿﻲ ﻫﺘﺄﺧﺮ ﻋﻠﻴﻜﻮﺍ ﻁﺐ ﻭﻫﻤﺎ ﻣﺎﻟﻬﻢ ﺗﻌﻄﻠﻬﻢ ﻟﻴﻪ ﻭﺗﺎﺧﺪ ﻭﻗﺖ ﻓﻮﻕ ﺍﻟﻮﻗﺖ ﺍﻟﻄﺒﻴﻌﻲ ﺍﻟﻠﻲ ﺍﻟﻤﻔﺮﻭﺽ ﺗﺎﺧﺪﻩ‬.

367. Patient diagnosed with cancer and doctor prescribed him tamoxifen, he is already on
many other drugs. What’s drug therapy problem?
a) Bupropion
b) Paroxetin
c) Mirtazapine
d) Venlafaxine
It is a pro‐drug requiring metabolic activation by CYP2D6. Low CYP2D6 activity that occurs in patients
harbouring certain CYP2D6 alleles (i.e. *4) or from the chronic use of CYP2D6 inhibitors can lead to
persistent reductions in plasma concentrations of an active metabolite of tamoxifen citrate (endoxifen).
Reduced efficacy on tamoxifen citrate has been reported with concomitant usage of some selective
serotonin reuptake inhibitor (SSRI) antidepressants (e.g. paroxetine, a known CYP2D6 inhibitor).
Concurrent chronic use of CYP2D6 inhibitors that may affect tamoxifen citrate efficacy should be avoided
if possible

368. A patient on fentanyl patch had breakthrough pain and went to the doctor to complain of
pain. Physicians asks you about the best recommendations:
a) Use a higher dose of fentanyl patch
b) Use immediate release morphine for breakthrough pain
If he uses breakthrough dose more than 5‐6 per day, increase the dose. If not, use IR
369. RR has a prescription for clarithromycin but she received lamivudine/abacavir from the
pharmacy by error. When she went home, she realized that she got medication for another
patient. She did not take any of the medication and returned back to the pharmacy.
What will be the initial step by the pharmacy manager to uphold veracity?
a) Contact the other patient to report breach of confidentiality
b) Contact the pharmacist who had checked the prescription
c) Contact the technician who entered the information through the system
d) Report to provincial incident reporting system

370. Pt has narcotic at home after switching and don't use it, what to advise?
a) Bring to pharmacy for disposal
b) Keep it in safe place at home
c) Bring to pharmacy to use by another pts
Another version. Patient wants to refund narcotic:
a) Reject the refund ask him to bring the medication back to the pharmacy

371. Saxenda and wt loss


SAXENDA (liraglutide) is indicated as an adjunct to a reduced calorie diet and increased physical activity
for chronic weight management in adult patients with an initial body mass index (BMI) of:
 30 kg/m2 or greater (obese), or
 27 kg/m2 or greater (overweight) in the presence of at least one weight‐related comorbidity
(e.g., hypertension, type 2 diabetes, or dyslipidemia) and who have failed a previous weight
management intervention.
Limitation of Use:
1. Clinical efficacy and safety data from patients with BMI 27 to 29.9 kg/m2 in the presence of at
least one weight‐related comorbid condition (e.g. hypertension, type 2 diabetes mellitus, or
dyslipidemia) are limited (N=149).
Serious Warnings and Precautions
 Risk of Thyroid C‐Cell Tumours: Liraglutide causes dose‐dependent and treatment‐duration‐
dependent thyroid C‐cell tumours at clinically relevant exposures in both genders of rats and
mice. It is unknown whether liraglutide causes thyroid C‐cell tumours, including medullary thyroid
carcinoma (MTC), in humans, as human relevance could not be ruled out by clinical or nonclinical
studies.
 SAXENDA is contraindicated in patients with a personal or family history of MTC and in patients
with Multiple Endocrine Neoplasia syndrome type 2 (MEN 2). Based on the findings in rodents,
monitoring with serum calcitonin or thyroid ultrasound was performed during clinical trials, but
this may have increased the number of unnecessary thyroid surgeries. It is unknown whether
monitoring with serum calcitonin or thyroid ultrasound will mitigate human risk of thyroid C‐cell
tumours. Patients should be counselled regarding the risk and symptoms of thyroid tumours
372. Counselling on naloxone spray administration?
a) In both nostrils
b) Place the patient on their back, provide support to the back of the neck to allow
the head to tilt back
c) Give the 2nd dose 24 hr apart
d) Maximum 2 dose /day
e) Put the patient on his back with his head tilted backwards
f) Prime dose before administration
g) Insert nozzle in one nostril and close the other one with your hand
h) Give the first dose, if no response, give the other one after 10 mins.
https://www.pharmacists.ca/news-events/news/how-to-use-naloxone-nasal-spray-in-a-few-
simple-steps-new-video-and-infographic/

373. Calculation: how much water to add to prepare antibiotic with a different strength than
in the shelf

374. Pt take alesse for acne, now it is in shortage, what to switch her to?
a) Seasonale
b) Yaz
c) Lolo
375. Dysmenorrhea case: Patient 40 years old had severe period pain (abdomen pain
radiating to thighs + back pain)- starts 2 hours before period- and continues 2-3 days, she
says that when she was in the twenties, she was taking oral contraceptive, what does not go
with Dysmenorrhea diagnosis?
a) Onset of pain
b) Pain location
c) Duration
d) Pain increases with age
Peak onset of primary dysmenorrhea occurs between late teens
and early 20s with prevalence decreasing progressively with age.

376. Cholestatic jaundice testing other than Bilirubin:


a) ALP
b) Aminotransferase

377. How to prevent Insulin syringe errors in hospitals?


a) Put label on metric & non metric syringe (for oral use only)
b) Use syringe that can’t be attached to any tube system
c) Give the drug in cap and multiuse
d) Prepare it in multidose container
e) Single dose and use it to many patients
f) Prefilled syringe prepared by pharmacist only
Another version
Which of the following is correct to avoid insulin errors in a hospital?
a) Nurses should receive good training for dose adjustment
b) Pharmacists should supply prefilled containers for nurses to use
c) Dose calculation should be done by the pharmacist only and prepared in the
pharmacy for each patient
d) Nurse keeps stocks of insulin in wards to prepare and use them

378. Dispensing error. pt took HIV drugs for another pt instead of his Abs, who do discuss
the issue after the pt returned the wrong?
a) Tech who prepared
b) Ph who did the final check
c) Pt who is waiting for his right meds.
We broke his confidentiality

379. New service in pharmacy, point of care testing, when we should do it in pharmacy?
a) When its easy to handle & no need for lab maintenance
b) To tell the pharmacist about how to do it & how to document result & lab work
c) Implement for all diseases and used with all patients
When providing POC testing for patient, pharmacist should remember these points:
 Pharmacist must explain the purpose of the testing to patient or agent & obtain written or verbal
consent, but it cannot be implied consent, before providing point-of-care testing.
 A pharmacist or a pharmacy technician can use a lancing device on a patient for the purposes of the
patient's self-care AND education, or to help the patient self-monitor their chronic illness. The
technician must be under the supervision of a Part A pharmacist.
 A pharmacist is required to document their own name and address, the patient name and address,
the date the test was performed, and confirmation that informed consent was obtained. They may
also choose to document the results, if any, but it is not required by the regulations. However, the
result should be documented if it is being used to adapt or renew a prescription
 In a pharmacy, point-of-care testing typically involves three things that may or may not be in the
pharmacist's scope: performing the test; interpreting the test; and prescribing or modifying a
medication.
 Pharmacists are NOT required to notify a physician of a point-of-care test result. However, it is good
clinical practice to share the result with the physician.
 Pharmacist must follow proper infection control procedures when performing POC testing

380. Old pt afraid of vaccination. what to tell to address her reluctant?


a) You should inforce her for
b) Don’t worry, you will experience no pain & no SE
c) You can ask you any qs about vaccine
d) You will protect community by taking your vaccine

381. Pt has Ca level low what will be high?


a) Albumin
b) Na
c) Po4
d) K

382. Error in pharmacy, pt took drug of another pt. you called him and ask to come to ph bz
of the error and when he came, you returned the wrong drug & give him the right one. what
to do next?
a) Note for the other pt
b) Note to ph who did it
c) Report to college

383. LY has cancer in his eyes complaining of nausea. He has not received chemotherapy yet.
a) Diphenhydramine
b) Domperidone
c) Metoclopramide
d) Scopolamine
The antihistamine dimenhydrinate and the antimuscarinic scopolamine are useful for treating vomiting
due to motion sickness but they are considered no more effective than placebo against CINV.
Prochlorperazine and metoclopramide are the most commonly used dopamine antagonists in the
management of nausea and vomiting in chemotherapy regimens with low emetogenicity or as rescue
agents. Both agents block the dopaminergic receptors in the chemoreceptor trigger zone.
Prochlorperazine may also exhibit serotonin antagonistic activity at higher doses. The availability of a
wide variety of dosage forms (tablet, suppository, injectable) facilitates prochlorperazine use, especially
for outpatients.
Low doses of metoclopramide (10–20 mg) are generally as effective as prochlorperazine; however, in
high doses (1–3 mg/kg), metoclopramide provides significantly higher antiemetic activity. A limitation to
metoclopramide use is the development of dose‐limiting diarrhea and extrapyramidal side effects.

384. CI someone starting a bupropion + naltrexone for (deminish eff of opioid) dulexetine
Bupropion, Naltrexone
GENERALLY, AVOID: Coadministration of naltrexone with other agents known to induce hepatotoxicity
may potentiate the risk of liver injury. Naltrexone, especially in larger than recommended doses (more
than 50 mg/day), has been associated with hepatocellular injury, hepatitis, and elevations in liver
transaminases and bilirubin. Other potential causative or contributory etiologies identified include
preexisting alcoholic liver disease, hepatitis B and/or C infection, and concomitant usage of other
hepatotoxic drugs.
MANAGEMENT: The use of naltrexone with other potentially hepatotoxic agents should be avoided
whenever possible (e.g., acetaminophen; alcohol; androgens and anabolic steroids; antituberculous
agents; azole antifungal agents; ACE inhibitors; cyclosporine (high dosages); disulfiram; endothelin
receptor antagonists; interferons; ketolide and macrolide antibiotics; kinase inhibitors; minocycline;
nonsteroidal anti‐inflammatory agents; nucleoside reverse transcriptase inhibitors; protease inhibitors;
retinoids; sulfonamides; tamoxifen; thiazolidinediones; tolvaptan; vincristine; zileuton; anticonvulsants
such as carbamazepine, hydantoins, felbamate, and valproic acid; lipid‐lowering medications such as
fenofibrate, lomitapide, mipomersen, niacin, and statins; herbals and nutritional supplements such as
black cohosh, chaparral, comfrey, DHEA, kava, pennyroyal oil, and red yeast rice). Patients should be
advised to seek medical attention if they experience potential signs and symptoms of hepatotoxicity such
as fever, rash, itching, anorexia, nausea, vomiting, fatigue, malaise, right upper quadrant pain, dark
urine, pale stools, and jaundice. Periodic monitoring of hepatic function is advisable
Naltrexone, Duloxetine
GENERALLY, AVOID: Coadministration of naltrexone with other agents known to induce hepatotoxicity
may potentiate the risk of liver injury. Naltrexone, especially in larger than recommended doses (more
than 50 mg/day), has been associated with hepatocellular injury, hepatitis, and elevations in liver
transaminases and bilirubin. Other potential causative or contributory etiologies identified include
preexisting alcoholic liver disease, hepatitis B and/or C infection, and concomitant usage of other
hepatotoxic drugs.

385. Depression pt dont want to wt inc or sexual dysf. Which of the following antidepressant
has the least weight gain and sexual dysfunction:
a) Bupropion
b) Olanzapine
c) Mirtazapine
d) Venlafaxine
e) Sertraline
386. Patient complaining of dyspnea & excessive sputum production. It will be written under:
a) Subjective
b) Objective
c) Assessment
d) Plan

387. Case about entry data error the pt took another pt midcation after they discovered this,
they called the pt to ensure that the pt was ok. what to do?
a) Report this to the college of ph
b) Ask the ph who dispensed the meds
c) Ask the technician who prepared the med

388. Long case about someone went to the emergency his eye pupil was normally dilated, he
was found negative to opioids & others and found positive to benzodiazepine & cannabnoids.
He has anion gap, acidosis, his blood glucose was normal, but has dyspnea, what to give him?
a) Flumazenil
b) Fomepozil
Symptoms of mild overdose include drowsiness, impaired coordination, diminished reflexes, confusion
and lethargy. In more serious overdose, symptoms may include ataxia, hypotonia, hypotension,
respiratory depression and coma. Although cardiac arrest has been reported, death from overdose
of benzodiazepines in the absence of concurrent ingestion of alcohol or other CNS depressants is rare.
Recommended Management
Assess for possible co‐ingestion of other substances. Activated charcoal (about 1 g/kg) may be
administered for a recent (<60 minutes), large oral benzodiazepine overdose, as long as the patient is
sufficiently alert to adequately protect their airway. Vital signs and fluid balance should be monitored.
Ensure that an adequate airway is maintained and respiration is assisted as required. Hypotension is not
generally problematic and is usually managed with boluses of isotonic iv fluids.
Flumazenil is a benzodiazepine antagonist that should be used very cautiously, ideally after consultation
with a Poison Control Centre. Potential candidates are patients with severe benzodiazepine toxicity or
overdose who are not benzodiazepine‐dependent and who have not consumed proconvulsant drugs.
Flumazenil is contraindicated in any patient who might have co‐ingested a tricyclic antidepressant or
used benzodiazepines chronically. Sudden benzodiazepine reversal by flumazenil in patients taking
chronic doses can induce withdrawal and precipitate seizures. Flumazenil is generally reserved for the
management of severely symptomatic pure benzodiazepine overdose.
Flumazenil should only be administered when continuous monitoring for recurrence of sedation can be
assured. Flumazenil rapidly reverses the hypnotic‐sedative effects of benzodiazepines. Flumazenil's
effects on respiratory depression are inconsistent; in some studies, residual respiratory depressant
effects were still present despite reversal of sedation. Improved consciousness is expected within the first
several minutes of flumazenil administration, but ventilatory support may be required for respiratory
depression. Flumazenil does not consistently reverse benzodiazepine‐associated amnesia. Dialysis is of
limited value in benzodiazepine overdose.
Antizol (fomepizole) is indicated as an antidote for ethylene glycol (such as antifreeze) or methanol
poisoning, or for use in suspected ethylene glycol or methanol ingestion, either alone or in combination
with hemodialysis
389. Which capsule could be opened and added to feeding tube or which one can put in j
tube or Drugs could be crushed in NJ tube?
a) Dabigatran
b) Finasteride
c) Psyllium
d) Rivaroxaban
For patients who are unable to swallow whole tablets, XARELTO tablets may be crushed and mixed with
applesauce immediately prior to use and administered orally. After the administration of a crushed
XARELTO 15 mg or 20 mg tablet, the dose should be immediately followed by food.
A crushed XARELTO tablet may be also administered via nasogastric (NG) tube. After confirming gastric
placement of the NG tube, the crushed tablet should be suspended in 50 mL of water and administered
via the NG tube after which it should be flushed with water. Because rivaroxaban absorption is
dependent on the site of drug release in the GI tract, avoid administration of XARELTO distal to the
stomach as this can result in reduced absorption and therefore reduced drug exposure. After the
administration of a crushed XARELTO 15 mg or 20 mg tablet, the dose should then be immediately
followed by enteral feeding
‫ﺧﻠﻰ ﺍﻟﻘﺎﻋﺪﻩ ﺍﻧﻚ ﻣﺎ ﺗﺴﺘﺨﺪﻣﺶ‬
SR
Irritating drugs
Enteric coated
Emulsion
‫ ﻭ ﺍﻯ ﺣﺎﺟﻪ ﻣﻤﻜﻦ ﺗﻜﻠﻜﻊ ﻭ ﺗﺴﺪ ﺍﻻﻧﺒﻮﺑﻪ ﺯﻯ ﺍﻝ‬fibers
‫ﻣﺎ ﺗﺤﻔﻈﺶ ﺍﻭﻯ ﻛﻞ ﺣﺎﺟﻪ‬
Another version: Which one can be opened and sprinkled on food?
a) Clindamycin
b) Clopidogrel
c) Dutasetride
d) Hydroxyurea
https://www.ismp‐canada.org/download/safetyBulletins/2013/ISMPCSB2013‐
05_LiquidMedicationsEnteralTube.pdf
There is evidence that certain liquid medications, such as clarithromycin suspension, readily occlude
feeding tubes, especially pediatric enteral tubes. Alternative modes of administration for these drugs may
include crushing tablets to a fine powder (if this can be done safely*) and then admixing the powder with
a sufficient quantity of water or using an alternative ready‐to‐use formulation that does not promote
occlusion
390. Label of teriparatide gp?
a) Take with food
b) Avoid alcohol
c) Keep at room temperature
d) Pt should take in supine or sitting position to avoid risk of orthostatic hypotension
Anabolic A parathyroid hormone (PTH) analogue, is an anabolic agent that demonstrates a steady gain
Agents in bone density and a 50% reduction in osteoporotic fractures.
Teriparatide It may also reduce pain associated with vertebral fractures.
Teriparatide is also a first-line for corticosteroid-treated individual with fractures & low BMD.
Dose: 20 mcg S.C daily for 2 yrs (lifetime exposure). Keep refrigerated, discard after 28 day.
S.E: Nausea, dizziness, leg cramps, hypercalcemia.
Do not use in patients with higher baseline risk of osteosarcoma.
Patients should be in a supine or sitting position during administration due to risk of
orthostatic hypotension.
Limited data available concerning use in renal or hepatic impairment.
The increase in BMD may be lost during the 1st year after stopping the drug unless an
anticatabolic is started.

391. You need to prepare 50 ml 2% solultion or paste, you have 1% and 2.5%, how much
each one? The correct answr was 17 and 33
Answer
Anyway, you will apply V1*C1 = C2* V2
50* 2 %= C2 * V2 (Select any strength given from stocks)
Suppose you will need 17 ml of 2 % So, from the other strength you will need (50 ‐ 17) = 33 mls

392. KN has a prescription for Prednisone 50 mg per day for 2 weeks then decreases dose by
5 mg daily every week until the dose reaches 20 mg per day then decreases by 2.5 mg daily
every week. How many of the 5 mg prednisone tablets are required for this prescription
Answer
to calculate amount required for the whole prescription
50 mg for 2 weeks = (10 *14) =140 tablets of 5 mg each.
Then the dose will be tapered by 5 mg weekly (one tablet tapered every week) = (9+8+7+6+5+4) * 7 days
= 273 tablets (as you have to stop when you reach 20 mg which is equivalent to 4 tablets of 5 mg.
Then next tapering by 2.5 mg (half tablet only till the patient stops the corticosteroids) = (3.5+ 3+ 2.5+ 2+
1.5+ 1+ 0.5) * 7 days = 98 tablets
So total amount needed = 140+273+98=511 tablets.

393. Drug was givin as 2 mcg/min, doctor want to increase the interval by 1mcg/min each (x
minutes). What is the final infusion rate ML/H?
Answer
This problem is missing some data (I think he is asking rate of infusion mg / hour not ML / Hour unless he
gives the drug volum)
2 mcg ‐‐‐‐‐‐‐ min x ‐‐‐‐‐‐ 60 min x = 120 mcg /hours = 120/1000= 0.12 mg / hour
And it increases by 0.06 mg /hour every given interval.
394. They give table for 2 tests for pregnancy, they ask you the right mean of specificity?
I chose negative result test for none pregnant women, D/(D+B)
Exposure present Exposure absent
Outcome Present A B
Outcome Absent C D

How can u calculate specificity?


a) D/C+D
b) B/B+D
c) D/D+B
d) B/A+B

395. Dementia patient. The cognitive function was suddenly worsened and the test result
became to moderate level from mild level. He said he took a medication recently. What can
be the possible medication?
a) Imipramine
396. You receive prescription for
Epoetin alfa, what the problem?
a) Dose daily
b) Dose weekly
c) Dose monthly
d) Dose 3 times weekly

397. A lady was asking or prescribed folic acid 5 mg, what is the right statement?
I choose she should take 5 mg daily because she is obese 35.5 BMI
Treat with Folic acid: 5 mg daily PO × 10 – 12 wk is recommended for:
 Pregnant with history of neural tube defect (NTD) or increased risk due to first‐degree relative
with NTD.
 Women belonging to a high‐risk ethnic group (e.g., Celtic, Northern Chinese, Sikh heritage)
 Those with certain medical conditions (e.g., type I diabetes, therapy with valproic acid or
carbamazepine, BMI > 35 kg/m2, malabsorption disorders).

398. The patient went and get prescription for the doctor what to councel here?
I choose do not use none reversible MOAi

399. PM receives citalopram for depression. He is a smoker and wants to quit. what is your
recommendation:
a) Bupropion
b) Varenicline
c) Nortriptyline
d) Clonidine
Citalopram with buprobion  Increase citalopram conc
Varenicline Act on nicotinic receptors. It is partial agonist that binds selectively to alpha4, beta2,
Champix nicotinic acetylcholine receptors with a greater affinity than nicotine.
0.5 mg daily PO for 3 days then BID for 4 days then 0.5–1 mg BID PO for 12 wk. If 1
mg BID is not tolerated, can reduce to 0.5 mg BID.
Patient should quit smoking 1–2 wk after starting varenicline. If patient is still smoking 4
wk after starting, reassess therapy. Can be continued for additional 12 wk if patient has
benefited.
No tapering necessary when discontinuing. Efficacy is dose-related.
S.E: Q. Nausea (30%); may be mitigated by taking on a full stomach, increasing water
intake or reducing dose. May cause insomnia; take second daily dose at suppertime.
Neuropsychiatric side effects such as suicidal/homicidal ideation have been reported;
monitor closely for changes in mood/behaviour.
Close monitoring by health-care provider for those with pre-existing psychiatric
disorders.
Varenicline, when used in conjunction with NRT, has shown to be more advantageous in
facilitating smoking cessation and equally as safe compared to using only varenicline. It
may increase risk of adverse effects. Further studies are needed to assess long-term
efficacy and safety of the combination.
The combination of varenicline and bupropion was studied in a group of people who
were unable to reduce their smoking by at least 50% after 1 week of NRT.
Compared with varenicline alone, subjects taking varenicline plus bupropion were more
likely to be smokefree at 8–11 weeks. Combination was statistically superior in men, but
not women.
Does not induce CYP enzymes; excreted renally unchanged.
Contraindications: Pregnancy, breastfeeding, and children.
Drug Interactions: Insulin, NRT, warfarin, and theophylline.
Contact doctor if constipation, abdominal pain, appetite changes.

400. What is the duration of treatment for pyelonephritis:


a) 3 days
b) 5 days
c) 10 days
d) 21 days

401. PT is a female pregnant patient who has a prescription folic acid 1 mg daily. She is
taking valproic acid 500 mg BID. She has a question if folic acid dose is high. Which of the
following should be your response?
a) Yes. She should take folic acid 0.5 mg for the whole gestation period
b) Yes. She should have started folic acid 0.5 mg 3 months before pregnancy and
continue during pregnancy
c) NO, she should continue this dose during gestation period
d) NO, she should increase it to 4 mg until week 12 then decrease to 1 mg until 6
weeks postpartum
Women of childbearing potential who take AEDs should receive a daily oral supplementation with a
multivitamin containing 1 mg folic acid, beginning at least 3 months before conception and continuing
until 12 weeks' gestational age, to potentially reduce the risk of AEDs teratogenic effects.
Higher doses (4 mg per day) are recommended in patients who have a history of neural tube defects and
may be considered in those taking valproic acid. From 12 weeks' gestational age, continuing through the
pregnancy, and for 4–6 weeks postpartum or as long as breastfeeding continues, continued daily
supplementation should consist of a multivitamin with 0.4–1 mg folic acid
402. Saxagliptin side effect (no heart side effect mentioned)
The most commonly reported adverse events are nasopharyngitis, bronchitis, hypoglycemia, urinary
tract infection, headache, hypertension and diarrhea.
Dipeptidyl Alogliptin Inhibit DPP-4, Nasopharyngitis, Taken once daily with or without
Peptidase-4 Linagliptin responsible for the hypersensitivity food.
DPP - 4 Saxagliptin inactivation of reactions, rarely Alogliptin & sitagliptin: not inhibit
Inhibitors Sitagliptin incretin hormones pancreatitis. cyt. P450.
Used in type as GLP-1. Prolong Can cause Linagliptin: can be used renal
Not used in 2DM for whom activity of incretin hypoglycemia disease patients), Clearance is
type I metformin increases insulin with sulfonylureas. enhanced by CYP3A4 inducers, e.g.,
diabetes. inappropriate. release in response Q. Caution with rifampin.
Lowers to meals & reduce saxagliptin in Saxagliptin: clearance reduced by
HbA1c by glucagon secretion. patients with strong CYP3A4/5 inhibitors, e.g.,
≤1%. HF. imidazole antifungals, macrolides.
Clearance enhanced by strong
CYP3A4/5 inducers, e.g., rifampin.

403. Reporting Adverse effects to?


a) Medeffect
b) Canada vigilance
c) CADTH
d) NSIR

404. Copayment, markup

405. You have expiry clonazepam


and oxycodone, what to do?
a) Send them back once you get authorization from factory

406. Patient has nocturnal leg cramps, takes quinine sulphate and no effect. The physician
asks you about your recommendation. What should be your response?
Another version: Quinine for night leg problem for 2 years, now she is neutropenia, what to do?
a) Diltiazem
b) Rosuvastatin
c) Clopidogrel
d) Bisoprolol
Drug Comments
Cinchona Use cautiously, only if cramps are frequent, severe and nonpharmacologic measures have failed
Alkaloids Treatment should be interrupted every 3 months to assess further need.
Quinine For patients with severe symptoms in whom other measures have failed: trial of 200–300 mg
sulfate QHS PO × 4–6 wk. May continue if beneficial and well tolerated
Used off-label S.E: Cardiac arrhythmias, dizziness, headache, gait disturbances, GI upset, tinnitus, visual
to manage impairment, potentially fatal thrombocytopenia.
May potentiate effect of warfarin; monitor INR. May decrease digoxin clearance.
nocturnal leg CYP3A4 inhibitors (e.g., ketoconazole) may reduce quinine clearance and increase toxicity.
cramps. CYP3A4 inducers (e.g., rifampin) may increase quinine clearance.
Avoid concurrent use with other drugs that prolong the QTc interval.
Advise patients taking quinine to report any signs of unusual bleeding (e.g., nosebleeds, gum
bleeding, blood in urine/stool, easy bruising or petechiae) to healthcare professional immediately
Not a safe treatment option in pregnancy. Not a Health Canada– approved indication.
Quinine is not recommended for the routine management of nocturnal leg cramps
Diltiazem 30 mg daily, vitamin B complex, gabapentin and verapamil have potential benefit

407. Your pharmacy received stocks of controlled and narcotic drugs. You reviewed the list
and found out they are all expired. What can you do?
a) Send back controlled drugs to refund the money.
b) Send back narcotic drugs to refund the money.
c) Destroy them because you can’t send these back.
d) Separate and label and contact distributor, document and dispose.

408. Amino acid 5% and Dextrose 16.6% 75ml/hr.


lipid infusion 20% over 12 hours in rate 50 ml/h.
Add dextrose ⅔ - ⅓ (dextrose 3.3%, NaCl 0.9%)
at a rate of 100 mL/ hr daily for 24 hrs. Calculate
total Kcal daily? (Amino acid 4 kal/g, Lipid 9 kal/g,
Dextrose 3.4 kal/g)
Answer: this problem may consume > 5 minutes in the
exam
lipid infusion rate
50 ml ‐‐‐ 1‐hour x ‐‐‐ 12‐hour x= 600 ml
so 20 gm ‐‐‐ 100 ml y ‐‐‐‐ 600 ml
y= 600 *20 /100=120 120* 9 K cal = 1080 k cal
for amino acids
5 gm ‐‐‐‐ 100 ml y2 ‐‐‐ (75 ml * 24 hours)
y2 = 90 gm 90* 4 = 360 kcal
in the exam calculate only 2 variables (like lipids and
amino acids) and select according to results you get.
(if he needs them separately) ….
But unfortunally this question is tough and needs to
calculate also dextrose which has 2 infusion rates.
For dextrose (first infusion rate)
16.6 gm ‐‐‐ 100 ml X ‐‐‐ (75 * 24 hours) = 298.8 gm
But you will take only 2 / 3 from that rate. So, amount of delieverd dextrose = 298.8 * 66.6% = 199 gm
199 gm * 3.4 = 676.6 k cal
For dextrose (second infusion)
3.3 gm ‐‐‐ 100 ml Y 2 ‐‐‐‐ (100 ml * 24) second infusion rate Y2 = 79.2 gm
But you will consume only 1/3 of this quantity
So = 79.2 * 33.3 %= 26.4 gm  26.4* 3.4 kcal = 89.76 k cal
So, k cal daily = 1080 + 360 + 676.6 + 89.76 = 2206.36 k cal.
409. In which of the following case you will open the pharmacy and not wait 8 hours to be
open as schedule do you open pharmacy for

410. Community pharmacy software to refill automatic and send SMS and email, what to
consider first? Patient consent

411. Which is higher incidence of narcotic over dose

412. What type of study detect rare disease


a) Case control
b) Cohort study
c) Meta analysis
Rare disease  case control
Rare side effects  case study or case series

413. Which of the following is correct:


a) Confidence interval gets narrower with
increasing the sample size

414. Pharmacy intern went through an argument with a patient because of dispensing error.
After a long time, he said to the patient that the reason for this error is low number of staff at
the time of dispensing. Which is violated?
a) Accountability
b) Patient advocacy
c) Code of excellence
Another version
A patient waits for 30 minutes to have his prescription dispensed, he shows anger to the
pharmacist who replies that the pharmacy is understaff, this reply violates the ethical principle:
a) Veracity
b) Integrity
c) Accountability
d) Service Excellence

416. What the aim of DUR


a) Make recommendation
for public formulary
417. Patient agitated was on .5mg diazepam, nurse administer him 5mg, he is not response to
her, why this error happens?

418. Medication used for old, what reference? Beer criteria

419. Non-maleficence for lady with many drug intraction and need to modify therapy
a) Call doctor and inform him

420. MP 50-year-old male has BPH come to the pharmacy with a Rx of new herbal product
recently approved by health canada. He asks for information about this medication:
a) Take the medication as prescribed by the dr
b) Tell patient about the pros and cons about the medication according to the leaflet
c) Do not take the medication because you have no idea about the medication

421. Elder Japanese couples came to Canada recently, what is the best way when counselling
them (from CTMA)?
a) Speak slowly
b) Use simple language
c) Give written explanation
d) Speak loudly
Tips for interacting with  Slow down your rate of speech
the elderly patient:  Decrease the amount of information given at one time

422. Famous cataract case for both eyes, patient had 3 or 4 eye drops, what is your advice?
a) Pt need to refill only anti-inflammatory ahead of 2nd surgery
b) Pt need to refill only the antibiotic ahead of 2nd surgery
c) Pt need to replace all drops ahead of 2nd surgery
d) Pt can use the same drops for the 2nd surgery

423. Pt on 40 mg atorvastatin, what is the risk factor of myositis?


a) High dose statin
b) 1 dose fluconazole for V candidiasis

424. PE + heparin induced thrombocytopenia, what to give?


a) Rivaroxaban
b) Fondaparinux
c) Dabigatran
d) Enoxaparin
CTC: Argatroban and danaparoid are approved for treatment of heparin‐induced thrombocytopenia
(HIT) in Canada. Limited data indicate that bivalirudin and fondaparinux may also be effective.
Heparin‐induced thrombocytopenia (HIT) occurs in up to 1% of patients receiving unfractionated heparin.
Heparin‐induced thrombocytopenia may occur even when very‐low‐dose heparin (eg, used in flushes to
keep IV or arterial lines open) is used. The mechanism is usually immunologic. Bleeding rarely occurs, but
more commonly platelets clump excessively, causing vessel obstruction, leading to paradoxical arterial
and venous thromboses, which may be life threatening (eg, thromboembolic occlusion of limb
arteries, stroke, acute myocardial infarction).
Heparin should be stopped immediately in any patient who becomes thrombocytopenic and develops a
new thrombosis or whose platelet count decreases by more than 50% while tests are done to detect
antibodies to heparin bound to platelet factor.
Anticoagulation with a nonheparin anticoagulant (eg, argatroban, bivalirudin, fondaparinux) should be
substituted at least until platelet recovery.
Fondaparinux is an acceptable alternative in many patients but given its long 17‐hour half‐life, is not
appropriate in those patients who may soon need a procedure or have a high bleeding risk.
Low‐molecular‐weight heparin (LMWH) is less immunogenic than unfractionated heparin but cannot be
used to anticoagulate patients with heparin‐induced thrombocytopenia because most HIT antibodies
cross‐react with LMWH.
Warfarin should not be substituted for heparin in patients with heparin‐induced thrombocytopenia and,
if long‐term anticoagulation is required, should be started only after the platelet count has recovered.

425. Famous question, how many N, C, T/C, PDL & OTC within the following list? Fiorinal
½ or ¼ (they gave the active composition), alprazolam, methylphenidate, lactulose,
hydromorphone, testosterone, ondansetron, modafinil.
a) 1,1,1,4
b) 0,1,1,5
c) 2,1,0,3
d) 3,1,1,1

426. Son or daughter of a pt (83 yo) came for early refill of narcotic for pain for one of their
parents, cancer patient. Pt doubled the dose so he is out of pills. What is the best action to do?
a) Refuse to fill as it is early
b) Give them the refill they ask
c) Contact dr to change Rx and let dr know about case progression

427. Evolokumab? Keep in fridge


Proprotein Alirocumab, Evolocumab (monoclonal antibodies to PCSK9)
Convertase Result in substantial LDL-C reduction beyond achieved with statins alone.
Subtilisin / Q. New class of lipid-lowering drugs which administered S.C. Refrigerate at 2°C to 8°C.
Kexin Type 9 Considered in very high-risk individuals whose lipid targets have not been reached with
(PCSK9) maximally tolerated lipid lowering medication
Inhibitors S.E: Injection site reactions, nasopharyngitis, upper respiratory tract infection.
Potential neurocognitive impact long term
428. Polymyalgia rhumatica how to treat?
a) Prednisolone
Prompt diagnosis and initiation of corticosteroid therapy is critical for
the prevention of vision loss in patients with giant‐cell arteritis.
Treatment should not be delayed while awaiting a temporal artery
biopsy, which can still be positive up to 4 weeks after the initiation of
corticosteroid therapy.
Aspirin, methylprednisolone inj, methotrexate.
Biologic agent tocilizumab IL6I and abatacept may be considered in
resistant cases of GCA.
Other drugs studied include leflunomide, rituximab and etanercept
and may have steroid‐sparing properties or reduce the risk of relapse
but further quality studies are required

429. Acute bronchitis case, patient has wheezing, no cough, dr prescribed Salbutamol inhaler
and codeine syrup, what DTP?
a) Patient needs antibiotic
b) Patient does not need
cough syrup
c) Patient does not need
salbutamol inhaler

430. Dr wants to add another


drug to his medication:
a) Roflumilast
b) Prednisone
c) Theophylline

431. A care giver came to the pharmacy with Rx for her husband who is taking
dexamethasone and cyclophosphamide tablets. What would you tell her?
a) Wear gloves when handling both dexamethasone and cyclophosphamide
b) Wash hands before and after handling both drugs
c) Wear gloves for dexamethasone and wash hands before and after handling
cyclophosphamide
d) Wear gloves when handling cyclophosphamide and wash hands before and after
handing dexamethasone

432. Case of patient takes too much opioids: hyperalgesia


How to explain hyperalgesia to the patient (use clear non-medical language example)
a) Excessive pain response to a normal pain stimulus
433. Female taking Alesse 20 mg Estrogen + 0.1 mg progesterone, spotting in the second
half: Pick the highest conc of progesterone in the choices (brand names + generics given)
Another version:
spotting after menopause: pick the highest conc. Of Estrogen
Pt take alesse, breakthrough bleeding or spotting, change to? Triphasic

434. Management case: Pregnant woman on Sertraline, worried, how to reassure her?
a) This is the safest medication in pregnancy
b) Sertraline is the most excellent medication in pregnancy

435. Doctor made E-prescriping order for pt to be taken after surgery at 14:00 but pt went to
ward at 14:10 and didn't take what physician ordered, physician discovered it and told you as
pharmacist to help him to process the order and give the pt the medication quickly, wht to do?
a) Delay processing and give the pt the drug without delay
b) Process first then give the pt the drug

436. Hospital pharmacy, you have a pan wide shortage of a drug, what will you do first?
a) Check cases on this medication

437. KW is a diabetes patient with CrCl 28 ml/min comes to the pharmacy with symptoms of
shallow breathing, discomfort and decreased appetite. Upon investigation you knew that he
takes NPH 25 Units BID and metformin for diabetes and citalopram for depression, he also
uses garlic to reduce his cholesterol. What possibly caused these symptoms?
a) High dose NPH
b) Citalopram
c) Metformin
d) Garlic
Lactic acidosis may rarely occur with metformin accumulation. It is a medical emergency and requires
prompt discontinuation of metformin and treatment in a hospital setting. Metformin should not be used
in patients with a history of lactic acidosis.
Lactic acidosis is characterized by elevated blood lactate levels (>5 mmol/L), decreased blood pH,
increased anion gap and increased lactate/pyruvate ratio. It presents initially with nonspecific symptoms
including malaise, somnolence, and abdominal and respiratory distress. With greater severity, it may be
associated with hypothermia, hypotension and resistant bradyarrhythmia. The frequency of metformin‐
associated lactic acidosis is not known, but in data taken from trials, it is estimated to be <4 cases
per 100 000 patient‐years.
Lactic acidosis occurs primarily in patients with renal insufficiency, hepatic dysfunction, or other
conditions involving hypoxemia, dehydration or sepsis. Lactic acidosis may occur in patients who
undergo radiologic studies with radiocontrast dye.
Metformin use in patients with diabetes and heart failure was previously thought to increase the risk of
lactic acidosis. However, newer data suggests metformin does not increase the risk in this population,
and is associated with better cardiovascular outcomes than other antihyperglycemic therapies
438. RR AND MM are two pts at hospital, doctor said order to RR but the order was given
to MM by mistake, what to do to prevent this error?
a) Pt identity must be determined before order process

439. Literature: most recent; Pubmed

440. Digoxin overdose: Micromedex

441. 24-year-old female came to the pharmacy asking for your recommendation for
emergency contraception as she had unsafe sex 6 days ago. What should be your response?
a) Plan B
b) Ulipristal
c) IUD
d) Mifepristone
Copper-T IUD The only method to provide ongoing contraception to prevent future need for
Flexi-T, Liberte, emergency contraception. Interferes with implantation after fertilization.
MonaLisa Use within 7 days of unprotected intercourse as an emergency contraceptive.
It is the most effective method of emergency contraception available.
S.E: Major: salpingitis, uterine perforation, cervical perforation, endometrial
embedding, menorrhagia, pain, infection, ectopic pregnancy.
Contraindications: Absolute: pregnancy, undiagnosed vaginal bleeding, stenosed
cervix, copper allergy, current PID or STI, cervical or endometrial cancer, inability to
place or retain device. Relative: 2–28 days postpartum (to decrease risk of expulsion).
Pregnancy test is recommended if normal menstrual bleeding does not occur by day 21
following treatment.

442. Which of the following symptoms needs referral after a few weeks of coming back from
a vacation in east ASIA?
a) Arthralgia
b) Fever
c) Headache
d) Vomiting

443. Who should take PPI?


a) Patient after endoscopy
b) Patient has GERD for more than 5 days
c) Patient took Ranitidine for 4 days and did not get better

444. Which PPI can be taken with clopidogrel? Pantoprazole


Q. Clopidogrl:
 Similar tolerability to ASA. The benefit of clopidogrel was consistent among patients, regardless of
their TIMI risk score. The major benefits were noted at 30 days, with small additional benefits
observed over the subsequent treatment period (average 8 months).
 Require loading doses for quicker antiplatelet effect.
 Loading dose: pre-PCI or STEMI: 300–600 mg PO with or without food.
 UA/NSTEMI: 300 mg PO (omit if risk of bleeding high). Maintenance: 75 mg daily PO
 No dosage adjustment is necessary for elderly patients or patients with renal impairment
 When treatment is suspended, the platelet system requires time to recover.
 S.E: Bleeding, rash, purpura, abdominal pain, diarrhea, duodenal, gastric or peptic ulcer,
agranulocytosis/ granulocytopenia, aplastic anemia, neutropenia and thrombocytopenia.
 Bleeding risks increase with higher ASA doses. Because of this risk, many hospitals with cardiac
catheterization facilities do not initiate clopidogrel until it is clear that bypass surgery is not needed
for appropriate revascularization
 Clopidogrel less effective in patients who are unable to convert it to active form (dependent on CYP
2C19).
 Caution with NSAIDs & PPIs (except pantoprazole) may reduce clopidogrel efficacy.
 Discontinue clopidogrel or ticagrelor 5 days & prasugrel 7 days prior to bypass surgery to decrease
bleeding.

445. What is contraindicated with hormone replacement therapy?


a) DVT
b) Migraine with aura

446. Shifting from ACEI to valsartan/Sacubitril? 2 days


Angiotensin Preferred to an ACE inhibitor in selected patients with mild to moderate HfrEF.
Receptor- The neprilysin inhibition leads to the accumulation of bradykinin, adrenomedullin and
Neprilysin natriuretic peptides (NPs).
Inhibitor The bioavailability of valsartan is enhanced by sacubitril and superior to enalapril in reducing
Combination the primary end point of death from CV causes or HF hospitalization.
Valsartan / Current Canadian guidelines recommend use of valsartan/sacubitril in place of ACE inhibitor or
Sacubitril ARB in patients with HFrEF who are symptomatic despite optimal medical treatment.
Entresto Q. It is critical to wait 36 hours between the administration of an ACE inhibitor and valsartan /
sacubitril (or switching back to an ACE inhibitor), to minimize the risk of angioedema.
As with ACE inhibitors, the risk of angioedema appears highest in black patients.
Lower risk of hyperkalemia & renal dysfunction than enalapril, but higher rate of hypotension.
Reduce the initial dose by half if risk of hypotension is high.

447. Manufacturer has sent you a notification about drug recall for a specific drug due to
change the label print while there is no change will be done to the ingredients. What should u
do?
a) Call patients asking them to return the medication that they have to the pharmacy and
return all medication on shelf to manufacturer
b) Return only sealed vials on shelf while continue using vials for dispensing.
c) Return all stocks that you have in the pharmacy either they are open or sealed
d) Call the whole saler to purchase more stock of the drug to cover expected drug
shortage

448. DDI: Micromedex


Drug Lexi-Comp Drug interactions, Drug monographs, Drug identification (better than
interactions American CPS), IV compatibility, Calculations, Patient education & Toxicology.
Brand names & Dosing schedules (include Canadian brand names&dosing)
Alternative to Pediatric and Neonatal Dosage Handbook: Off label uses &
Pediatric dosing
Micromedex Alternative to CPS: Drug monographs, Drug identification, interactions, IV
American compatibility, Calculations, Patient education, off label uses & Toxicology
Micromedex, pre-2019 CPS and LexiComp have drug identification tools.
Drug Australian. This publication provides information about Drug-drug and
Interaction drug-food interactions in a quick reference format.
Facts. Tatro Also includes drug significance ratings.

449. Clinical study for effect of two drugs for 24 months. What makes this study better??
a) Longer duration
b) Make the endpoint is to reduce Ml

450. Manufacturer has sent you a notification about drug recall for a specific drug due to
carcinogenic impurities in the vial. What should u do?
a) Call patients asking them to return the medication that they have to the
pharmacy and return all medication on shelf to manufacturer
b) Return only sealed vials on shelf while continue using vials for dispensing.
c) Return all stocks that you have in the pharmacy either they are open or sealed
d) Call the whole saler to purchase more stock of the drug to cover expected drug
shortage

451. Pt has high fever; bloody and mucoid stools, diagnosed with severe traveller diarrhea,
what will you give:
a) Ciprofloxacin
b) Loperamide
c) Bismuth
452. Which one is false regarding Tranexamic acid?
a) Concomitant use of hormonal contraception exacerbates thrombotic risk.
b) Can be used in pregnancy & lactation
c) Need dose adjustment in pt with kidney problems
d) Can cause N & V, diarrhea, visual disturbances, ↓ bp.
e) Used to prevent or reduce bleeding in heavy periods (menorrhagia) in patient
under 18 years of age
Indications Hereditary angioneurotic oedema. Increased local fibrinolysis when the diagnosis is
indicative of hyperfibrinolysis, as with conization of the cervix, dental extraction in patients
with coagulopathies (in conjunction with antihaemophilic factor) epistaxis, hyphaema, and
menorrhagia (hypermenorrhea).
Tranexamic acid mouth wash can be used pre and post dental surgery to decrease bleeding
(mixture – swish 5ml 5 – 10 min before surgery 3‐4 times/day for 1 – 2 days after surgery)
Contra‐ Patients with a history or risk of thrombosis should not be given CYKLOKAPRON
indications (tranexamic acid), unless at the same time it is possible to give treatment with
anticoagulants. The preparation should not be given to patients with acquired disturbances
of colour vision. If disturbances of colour vision arise during the course of treatment the
administration of the preparation should be discontinued.
Patients with active thromboembolic disease, such as DVT, PE and cerebral thrombosis.
Patients with subarachnoid haemorrhage: the limited clinical experience shows that a
reduced risk for re‐bleeding is offset by an increase in the rate of cerebral ischaemia.
Haematuria. Hypersensitivity to tranexamic acid or any of the ingredients.
Warnings Visual disturbances including visual impairment, vision blurred, impaired color vision have been
reported with tranexamic acid. For patients who are to be treated for several weeks with
tranexamic acid, an ophthalmic check‐up is advisable (sharpness of vision, colour vision, fundus,
field of vision, etc.) if possible, before treatment is initiated and regularly during treatments.
Combination hormonal contraceptives are known to increase the risk of venous thromboembolism,
as well as arterial thromboses such as stroke and myocardial infarction. Because CYKLOKAPRON is an
antifibrinolytic, concomitant use of hormonal contraception and CYKLOKAPRON may further
exacerbate this increased thrombotic risk.
Safety during pregnancy has not yet been established. No harmful effects have been reported.
Tranexamic acid is secreted in the mother's milk at a concentration only a hundredth of the
corresponding serum levels (Eriksson et al, 1971). The investigators are of the opinion
that tranexamic acid can be given during lactation without risk to the child.
Clinical experience in menorrhagic children under 18 years of age is not available.
Tranexamic acid may cause dizziness and may influence the ability to drive or use machines.
In patients with serum creatine concentrations of 120 to 250 µmol/L, 15 mg orally or 10 mg
intravenously tranexamic acid per kg body weight twice daily. At serum creatine levels of 250 to 500
µmol/L the dosage should be 15 mg orally or 10 mg intravenously per kg body weight at 24‐hourly
intervals, and at serum creatine levels of 500 µmol/L or more, the same dose should be given at
intervals of 48 hours between doses.
Clinical experience in menorrhagic children under 18 years of age is not available.
Adverse Gastrointestinal symptoms (nausea, vomiting, diarrhea) occur but disappear when dose is reduced.
Effects Isolated cases of dizziness or reduced blood pressure have been reported.
Allergic dermatitis has been reported less commonly.
453. Rx containing hydroquinone 6% and the stock we have is 4%, how much powder
should we use?
Drug X 5%
hydroquinone 6%
1:1 mL of hydroquinone 4% and hydroquinone 0.5%
Mitte 60 gm. The available hydroquinone is 4%. How much hydroquinone powder to add?
Answer according to above data
Strength of hydroquinone needed in the compounding = 6%
That means 6 gm ‐‐‐‐ 100 gm X ‐‐‐‐‐‐‐‐‐ 60 gm X= 3.6 gm
Stock in hand = 4% 3.6 gm ‐‐‐‐‐ 6 % X ‐‐‐‐‐ 4 % Amount needed = 2.4 gm.

454. Pt with acne started Benzoyl peroxide. After how long, treatment should be assessed?
a) 4 weeks
b) 8 weeks
c) 12 weeks
Use the medication only once a day until your skin gets used to it. Use nonprescription acne medication
for 6–8 weeks. Remember, it may take some time before your skin looks better. Try not to get
discouraged!
2‐3 months may be needed to see significant improvement after both topical & systemic agents.

455. XI is female 36 years old on benzoyl peroxide wants to start isotretinoin for her sever
nodular acne, after trying other drugs, how many negative pregnancy tests should be before
start isotritenoin?
a) Zero
b) One
c) Two
d) Three
It is recommended that a woman wait one month after stopping isotretinoin before trying to
become pregnant. Usually, isotretinoin is no longer found in a woman's blood 4‐5 days after the last dose and
most of its by‐products should be gone within 10 days after the last dose. And acitretin 3 years, oral
cotraceptive wait one menstrual cycle, Tetracyclin one week, Spironolactone 5 days

456. What test should be done while on isotrintion? All was strange
a) Serum Creatinine
b) Bilirubin
c) TSH
d) CPK
There was no lipid or liver
Blood potassium increased, blood alkaline phosphatase increased, blood bilirubin increased, blood urea
increased, elevated platelet counts, eosinophil count increased, false positive tuberculosis test, gamma‐
glutamyltransferase abnormal, blood cholesterol increased, glucose urine present, haematocrit
decreased, protein urine, thrombocytopenia, WBC count decreased. Elevations in levels of serum creatine
kinase CPK (monitor)
457. After using Isotretinoin XI is expecting to experience:
a) Hypoglycemia
b) Hyperthyroidism
c) Hyponatremia
d) Hypertriglyceridemia

458. The side effect caused by Isotretinoin is a risk factor of which of the following
conditions:
a) Coma
b) Myxedema
c) SIADH
d) Pancreatitis

459. Which drug needs to keep


sales record?
a) Methadone.

460. Urine incontinence in female and tried different ttt


461. Drug (X) is available as a 20% solution in a 500 mL IV bag. The nurse is supposed to
adjust the infusion pump at a rate of 10 mL/hour and to be doubled every 15 min to a max.
160 mL/hour for 15 min then the infusion stops.
a) What was the dose of the drug in mg? 15500mg
b) How long it will take to reach the maximum dose? 1 hr
Answer A:
10ml ‐‐‐‐‐‐‐‐‐ 1 hr Xml ‐‐‐‐‐‐‐‐‐‐ 15 min X = 10 x 15 /60 = 2.5ml
20 ml/hr ‐‐‐‐‐‐ 15 min ‐‐‐‐‐‐ 5 ml 40 ml/hr ‐‐‐‐‐‐ 15 min ‐‐‐‐‐‐ 10ml
80 ml/hr ‐‐‐‐‐‐ 15 min ‐‐‐‐‐‐ 20 ml 160ml/hr ‐‐‐‐‐ 15 min ‐‐‐‐‐‐ 40ml
2.5 + 5 + 10 + 20 + 40 = 77.5 ml
20 % meaning 20g in 100ml. So, how Xgm in 77.5 ml 77.5 x 20 /100= 15.5 gm = 15500 mg
Answer B:
500ml infusion. Start 0 minute. Rate of 10ml/hr means 2.5ml / 15m, and we double this rate every 15m
So,
10ml/hr  2.5ml/15m 20ml/hr  5ml/15m 40ml/hr  10ml/15m
0m ‐‐‐‐‐‐‐‐‐‐‐‐‐‐‐‐‐‐‐‐‐‐‐‐‐‐‐‐‐‐‐‐‐‐‐‐‐ 15m ‐‐‐‐‐‐‐‐‐‐‐‐‐‐‐‐‐‐‐‐‐‐‐‐‐‐‐‐‐‐‐‐‐‐‐‐‐‐‐‐ 30m ‐‐‐‐‐‐‐‐‐‐‐‐‐‐‐‐‐‐‐‐‐‐‐‐‐‐‐‐‐‐‐‐‐‐‐‐
80ml/hr  20ml/15m 160ml/hr  40ml/15m
45m ‐‐‐‐‐‐‐‐‐‐‐‐‐‐‐‐‐‐‐‐‐‐‐‐‐‐‐‐‐‐‐‐‐‐‐‐ 60m ‐‐‐‐‐‐‐‐‐‐‐‐‐‐‐‐‐‐‐‐‐‐‐‐‐‐‐‐‐‐‐‐‐‐‐‐‐‐‐‐‐ stop
It will take 1 hour to reach the maximum dose. There are 5 concentration and 5 periods of 15 minutes but
the trick is that the question asks for the time to reach the maximum dose so we will not consider the last
15 minutes. This means to reach the maximum dose we need 4 periods of 15 minutes each = 1 hour

462. A 70-year-old patientwas just diagnosed with Parkinson's disease and was placed on
carbidopa/L-dopa 25/100 three times a day by his primary care physician. The patient has
been complaining of Nausea and lightheadness since starting the medication and wants to
stop the medication. The best recommendation to control nausea is to give?
a) Metclopramide
b) Increase cabidopa/ levodopa 25/ 100 to QID
c) Granisetron
d) Diminhydranate
https://www.parkinson.ca/wp-content/uploads/Medications-to-treat-Parkinson%E2%80%99s-disease.pdf
Medications that may worsen symptoms of PD or cause drug‐induced Parkinsonism
463. 5 mg methylnapth tab TID is needed to be switched to solution of 5 mg/ml, knowing
that pure methylphenidate is 5%, so what is the amount of pure methylnapth
Answer
total daily dose = 5 * 3 = 15 mg of regular methylphenidate. (double check the problem is missing some
words to clarify)
To be converted to regular solution
5mg ‐‐‐‐‐ 1 ml 15 mg ‐‐‐‐‐ x So you will consume 3 ml.
But the available pure methylphenidate (stock in hand) = 5 %
5 gm ‐‐‐‐‐ 100 ml 0.015gm (15 mg required daily dose) ‐‐‐‐‐ x mls = .015* 100 /5= 0.3 mls
So, you will need 0.3 mls of pure methyl phenidate
Add diluent till 3 mls (3 ‐ 0.3) = 2.7 mls

464. Css 43, Vd 50, Rate 125. Conc gives efficacy after 20 mg. calculate time needed to give
the next dose.
Answer
According to given data to calculate time needed for next dose
Log c =log C0‐ kt /2.303 (where c = 20 mg and assume C0= Css =43)
The above equation is missing k value
To calculate k Css = R/K*Vd
Where R= rate of infusion 43= 125 /50*k K=R/ CSS* Vd = 125/43*50 = 0.05813
Log 20= log 43 – 0.05813 * t /2.303 T = 13.1709 hours = 13 hours
Another version
A drug infusion rate 120 mg/hr, and Css should be 30 mg, but it was 54 mg. How long
should we stop the infusion to make Css back to 30 mg? Vd was given
Answer
As long as Vd is given Apply law CSS=R/k*vd Where css = 54 Calculate k value
Then apply equation
Log c= log C0 ‐kt/ 2.303 Log 30=log 54 ‐ kt/2.303
And already you know k value  Then calculate the time required

465. Toxic dose of elemental iron is 40mg/kg, pt weight is 44lb. ferrous fumarate tablet
contains 300mg iron. What is the safest dose?
Answer
Weight of pt = 44/2.2 = 20kg
Toxic dose = 40 * 20 = 800mg
Ferrous fumarate contains 33% elemental iron. So, 300mg  100mg elemental iron
300 ‐‐‐‐‐‐‐‐ 100 X ‐‐‐‐‐‐‐‐‐ 800mg
X = 800*300/100 = 2400mg / 300mg = 8 tablets.
As the the toxic dose equals 8 tablets of ferrous fumarate. So, the safest dose is < 8 tablets daily
466. Methadone citalopram interaction? QT prolongation
for accuracy, it was 4 pairs of drugs and which one highest risk of QT prolongation (all pairs
were having QT) methadone and citalopram was highest risk
This is rx file, the BOLD ONE is severe qt prolongation
Using citalopram together with methadone can increase the risk of an irregular heart rhythm that may
be serious and potentially life‐threatening, although it is a relatively rare side effect. You may be more
susceptible if you have a heart condition called congenital long QT syndrome, other cardiac diseases,
conduction abnormalities, or electrolyte disturbances (for example, magnesium or potassium loss due to
severe or prolonged diarrhea or vomiting).

467. question about commission and omission errors


Omission Failure to administer an ordered dose to a patient in hospital, nursing home, or other facility
errors before the next scheduled dose is considered an omission error.
Occur when information essential to filling Rx is missing (ex. drug, dose, or dosage form not
specified on the Rx.)
Errors of omission are a failure to take the correct action. For example, forgetting to order a
cholesterol panel on a patient with diabetes is an example of an omission error.
Commission Include incorrectly specifying the dosage regimen or strength on a prescription.
Errors Errors of commission are also a situation when the wrong action was taken.
For example, prescribing an antibiotic that the patient is known to be allergic to.
Omission errors occur when someone forgets to perform a task, whereas commission errors
occur when someone has performed something that should not have been performed.
Near-misses Near-misses are a situation where an error is just avoided. For example, a nurse is about to
Errors administer an antibiotic into a patient, but just before she opens the tubing she notices that it is
the antibiotic for the patient in the next room.
468. A diabetic patient has different work shift, sometimes morning other time evening, what
best insulin regimen:
a) Glargine in the morning
b) Detemir BID morning and evening
c) Detemir in the evening
d) Degludec in the morning???
e) Glargine morning evening
Day to evening shifts:
 For the client working day to evening shifts, no adjustment is usually needed to AHAs.
 An exception would be if the work involves moderate to high intensity activity. If this is the case,
the recommendations in the increased physical activity section of this guide would be followed.
Day to night shifts:
 For the client working frequent rotating shifts including nights, the night shift is the biggest
challenge.
 Regimens that allow for adjustments to relatively short time intervals help reduce BG variability.
 In cases where an insulin secretagogue or insulin must be used, repaglinide and MDI insulin
regimens offer the most flexibility and are the preferred AHAs for these clients.
 Shift work involving night shifts is a situation where individualization of the plan based on the AHA
regimen, hours of work, type of activity at work and BG patterns and trends cannot be
overemphasized.
The following suggestions are meant as a guide only.
For clients on Oral AHAs:
 AHAs that are not insulin secretagogues or insulin usually do not need to be adjusted for night shifts.
 The timing of insulin secretagogues will likely need to be adjusted. The use of short acting
secretagogues such as repaglinide is preferred.
 If longer acting secretagogues are used, the following are important considerations when making
the switch from days to nights:
o the change in the timing of meals will determine the change in timing of the secretagogue
o the change in the timing of activity level particularly if the work is physical in nature
o frequent BG monitoring is recommended to determine the effect on the individual client’s
BG values of switching from days to nights.
For clients on insulin:
 For clients working night shift, the greatest stability with BG control is obtained from a once‐a‐day
long‐acting insulin analogue given at the same time each day with rapid‐acting insulin given
before meals.
 Initially frequent BG monitoring will be needed to determine if dose adjustments are necessary in
addition to the adjustment in timing of the AHAs.
For clients on one injection of intermediate or long‐acting insulin regimens:
 No adjustment of insulin dose is typically needed especially with long‐acting insulin.
 These clients may also be taking an insulin secretagogue and the timing will need to be adjusted to
changes in meal times.
For clients on two injections per day regimens:
 Clients on 2 injections of basal insulin who work 3 different shifts, may benefit from the following
shift work plan to adjust for days at work and days off.
 This plan can be used as a “starting point and will need individualization using clinical judgment.
469. Which improves QOL in CHF:
a) Digoxin
b) Valsartan / Sacubitril
c) Carvedilol
d) Nitrates/Hydralazine
PARADIGM‐HF, a large multicentre clinical trial that included patients with mild to moderate HF, LVEF
<40%, elevated concentrations of NPs or an HF hospitalization in the past 12 months, serum potassium
<5.2 mmol/L and an eGFR ≥30mL/minute, showed that valsartan/sacubitril was superior to enalapril in
reducing the primary end point of death from cardiovascular causes or HF hospitalization, both these
endpoints individually, and death from any cause, as well as improving quality of life
The combination of isosorbide dinitrate plus hydralazine reduces mortality and morbidity in black
patients with NYHA class III–IV HF and is recommended in addition to standard therapy (ACE inhibitor,
beta‐blocker with or without MRA) in this setting. Use of this combination may also be considered in
black patients with NYHA class II HF and in other HF patients who do not tolerate ACE inhibitors, ARBs or
an ARNI.
Digoxin improves symptoms and reduces the risk of hospitalization for exacerbations of HFrEF, but does
not reduce mortality in patients with persistent moderate to severe symptoms (NYHA class II–IV) while
on ACE inhibitor therapy. Digoxin may be considered in patients with persisting symptoms despite the
use of a beta‐blocker, an ACE inhibitor (or an ARB) and an MRA (or an ARB). It may also be considered for
ventricular rate control in patients with atrial fibrillation that cannot be controlled by beta‐blockers, or in
patients who cannot tolerate beta‐blockers
Beta‐blockers improve symptoms and reduce the risk of hospitalization and death in patients with HFrEF
NOVEMBER 2019
1. Mother comes with 18-month baby constipated for last 2 days, with intermittent constipation
for the last 8 weeks. What to do?
a) Refer to doctor
b) Lactulose
c) Glycerin supp
d) Senna
See Your Health‐Care Provider If you are constipated and:
 Children < 2 years old constipation >2 wks (or no bowel movement for 7 days) despite use of laxative.
 You have not had a bowel movement for 7 days
 You are extremely uncomfortable because you are constipated
 You have pain in your rectum (back passage) or rectal bleeding
 You have a fever, your belly hurts or you feel like throwing up
 Your bowel movements are thin as a pencil
 You have a problem with your bowels that lasts longer than 2 weeks or keeps coming back

2. Female patient is 29 years old, 100 lb, has serum creatinine 135%, calculate ClCr?
wt = 100/2.2 = 45.45kg
CrCl = (140 – 29) * 45.45 / 72*135% = 5044.95 / 97.2 = 51.903 * 0.85 = 44.118 = 0.44ml/min

3. Patient takes Lorazepam 1 mg/ day, physician wants to switch her to diazepam (each 1 mg
Lorazepam equal 5 mg diazepam) each diazepam tab is 2 mg, Rx equivalent dose for 1 week
then taper by decrease dose 10% and use for 2 weeks. How many tabs required?
a) 48 tabs
b) 96 tabs
c) 24 tabs
d) 72 tabs
Lorazepam 1 mg/ day in 1st week = 7mg in 2nd & 3rd week decrease by 10 % = 14mg ‐10 /100 = 12.6 mg
Total lorazepam = 7 + 12.6 = 19.6 Diazepam dose = 19.6 * 5 = 98 / 2 = 48 tab

4. Patient is taking 90 units of LANTUS® (insulin glargine 100 U/mL), physician switched her
to another brand TOUJEO® (insulin glargine 300 U/mL), how many units should he take?
a) 72 units
b) 90 units
c) 45 units
Answer is 72 if he said that dose of lantus is twice daily, so we decrease 20% of the dose.
5. MB female was diagnosed with osteoporosis. She has O.A. in her knee. She broke her arm
when she was young. She works as hair dresser and her work locates 2 km from her home. She
goes for swimming in weekend. She takes one glass of wine at weekend, and smokes 1 pack of
cigarettes daily. What is the risk factor of osteoporosis?
a) Previous fracture
b) Smoking
c) Alcohol
d) Osteoarthritis
Older Adults (≥50 y) Younger Adults (<50 y)
Age ≥65 y Fragility fracture, Prolonged use of
Clinical risk factors for fracture (men age 50–64 y, corticosteroids, use of other high-risk
menopausal women): vertebral compression, fracture fragility, medications, e.g., aromatase inhibitors,
fracture after age 40, prolonged use of corticosteroids, use of androgen deprivation therapy, Hypogonadism
other high-risk medications, e.g., aromatase inhibitors, or premature menopause (<45 y),
androgen deprivation therapy, parent with hip fracture, Malabsorption syndrome, Primary
osteopenia identified on x-ray, current smoking, high alcohol hyperparathyroidism, Other disorders strongly
intake, low body weight (<60 kg) or major weight loss (>10% associated with rapid bone loss or fracture.
of weight since age 25), rheumatoid arthritis. Race (asian & caucasian).

Medications associated with increased risk of fractures:


 Aromatase inhibitors, anticoagulants (unfractionated and LMW heparins), antiretroviral therapy,
cyclosporine, corticosteroid therapy (at least 3 months' cumulative therapy in the previous year at a
prednisone‐equivalent dose ≥7.5 mg daily), loop diuretics, proton pump inhibitors (PPIs), SSRIs,
thiazolidinediones, chemotherapy and high doses of vitamin A.
 Androgen deprivation therapy include leuprorelin, goserelin, triptorelin, histrelin, buserelin, degarelix.
 Depot medroxyprogesterone acetate is associated with bone loss which may be recovered after
treatment stops although data on long‐term use is lacking.
6. She wants to change his lifestyle & her work is located 2 km to his home. What change will
help her to improve her symptoms?
a) Decrease alcohol intake
b) Walk to work everyday
c) Control risk of falls at home
d) Increase swimming activity 3 times weekly
Nonpharmacologic Choices, Recommended for everyone:
 Regular exercise (especially impact type)
 Fall prevention: minimize hazards for falling in the home (e.g., remove throw rugs, install grab bars in
bathrooms, ensure adequate lighting), assess drugs implicated in falls such as benzodiazepines and
other psychotropics, improve strength and balance
 Smoking cessation
 Dietary measures: encourage adequate protein, calcium and vitamin D intake, avoid excessive alcohol
(>2 drinks/day) and caffeine (>4 cups of coffee per day or equivalent)

7. After that she told you that she heard about strontium which is beneficial for her symptoms.
When you searched you found that its indication in OP is not approved. What reference you
used to find information about strontium?
a) Compendium of pharmaceuticals and specialties
b) Micromedex
c) Pub-med
d) Medline
e) Minor ailments
Not available or approved in Canada. Strontium is widely promoted for treatment of osteoporosis.
In Europe, strontium ranelate is available as a prescription drug. In the United States, strontium is a dietary
supplement available as the carbonate, chloride, citrate, gluconate, and sulfate salts.

8. Patient has shingles in his eyes (Post Herpetic neuritis) & come after 5 days, what to counsel?
a) Take vaccine
b) Avoid contact with any person had not experienced chicken pox
c) Don't put water on lesions till resolved
Nonpharmacologic Choices
Keep rash clean and dry to reduce risk of bacterial superinfection.
Prevent transmission of the virus to another person:
 keep the fluid‐filled blisters and rash covered
 wash hands often
 do not touch or scratch the rash
Avoid use of topical antibiotics and dressing with adhesives, as these may cause irritation and delay rash
healing. Use sterile, wet dressings to relieve discomfort in some patients.
9. What to give?
a) Trifluridine eye drop
b) Famciclovir
c) Don’t give any
d) Ophthalmic corticosteroid
e) Ibuprofen
Will, I chose Famcyclovir, as the patient is
suffering from neuritis plus keratitis
triflurodine will be a good choice if it is Herpes
simplex virus infection not zoster
Systemic antiviral nucleoside analogues (acyclovir, famciclovir, valacyclovir) initiated within 72 hours of rash
onset reduce the duration of viral shedding, acute pain and the appearance of new lesions. Increasing age
(patients <50 years of age rarely develop
postherpetic neuralgia), severe pain and
extensive disease correlate with an increased
risk of postherpetic neuralgia.
Oral acyclovir does not significantly reduce the
risk of postherpetic neuralgia, and the effect
of the other nucleoside analogues on this
sequela is not known.
Topical antivirals are NOT effective in the
treatment of herpes zoster. To prevent ocular
complications, treat patients with ocular
zoster even if the rash has been present for up
to 7 days. Promptly refer patients with ocular complications to an ophthalmologist. Famciclovir and
valacyclovir may improve patient adherence because they can be given less frequently, e.g., TID dosing.

10. He also has pain, what to give?


a) Pregabalin
b) Opioid
c) Diclofenac
d) Nortriptyline
e) Tramadol
Prevention of postherpetic neuralgia using antiviral therapy is not absolute. Analgesics, opioids, gabapentin,
pregabalin and corticosteroids may be used in the treatment of acute pain related to herpes zoster.
Corticosteroid therapy improves quality of life (resolution of acute neuritis, uninterrupted sleep and return to
normal activity) but does not accelerate healing or reduce the incidence of postherpetic neuralgia compared
with acyclovir alone.
The risk of immunosuppression with corticosteroids may hinder their use in high‐risk patients, e.g., elderly,
patients with diabetes, hypertension, GI ulcers. Therefore, limit use of corticosteroids to healthy patients with
moderate‐to‐severe pain. For information on postherpetic neuralgia, see Neuropathic Pain.
11. A young adult went to fishing trip with his friends but did not apply sunscreen to his face or
his feet, now he suffers from second degree burns. What NOT to give?
a) Oatmeal
b) Hydrocortisone 1%
c) Diphenhydramine cream
d) Ibuprofen
Nonpharmacologic Choices of Sunburn
Cool baths or wet compresses with tap water or saline for 20 minutes 4–6 times a day provide some relief.
Fluids should be replenished with nonalcoholic beverages. Blisters may require dressings and wound care.
In the case of large blisters, they can be incised and drained with a needle under sterile conditions. The overlying
skin should be left intact to decrease the risk of secondary infection.
Pharmacologic Choices of Sunburn
Calamine lotion and colloidal oatmeal may be soothing. Moisturizers help with dryness and peeling.
Simple analgesics (e.g., acetaminophen, ibuprofen) may provide pain relief.
Pramoxine hydrochloride 1% is a topical anesthetic with low sensitizing potential that may provide short‐term
relief. Avoid topical benzocaine and other “caine” topical anesthetics as they can sensitize the skin.
Topical diclofenac gel applied 6 and 10 hours after irradiation decreased pain, erythema and edema in studies
conducted by the manufacturer.
Evidence of the effectiveness of aloe vera in treating sunburn is limited and conflicting. A systematic review
looking at its use in acute and chronic wounds including burns was inconclusive.
Studies of the treatment of sunburn with topical corticosteroids have shown mixed results. As it is unclear if
there is any benefit, and as sunburn is a self‐limiting condition, they are not generally recommended.
Small studies have examined whether experimentally induced UV erythema and epidermal injury may be
suppressed by oral NSAIDs but the evidence is not convincing and they are not routinely recommended except
as simple analgesics for sunburn pain.
Opioids such as fentanyl and morphine may be required to relieve the pain associated with severe sunburn.
Diphenhydramine cram Monograph:
Therapeutic indications: Symptomatic treatment of allergic conditions e.g.: ‐ Urticaria, pruritus, allergic rashes,
stings and insect bites.
Apply sparingly and infrequently (maximum twice daily) to the affected area. Short term treatment only ‐ not
more than three days. Not recommended for children under six years old.
Contraindications: Known sensitivity to anti‐histamines and benzoates. Do not apply to mucous membranes,
eczematous conditions or where the skin is extensively broken or denuded, or in acute vesicular or exudative
dermatoses. Do not use with any other product containing the same active ingredient.

12. There is a new drug and the pharmacist will work within a team to demonstrate some
information about it, if the team have a booth and the drug is rapped by silver, what will have
an effect on the drug (something like this)
a) Humidity 55%
b) Temperature 29 C
c) UVA high index
13. You are going to talk in a presentation about Naloxone, what to say?
a) Ensure that, the patient has opioid overdose before administration
b) Call the ambulance after applying the first dose
c) If the first dose did not work apply another one in 3-5 (2-3) mints
https://www.ocpinfo.com/library/practice-related/download/Naloxone.pdf
https://harmreduction.org/wp-content/uploads/2016/04/FAQ-pharmacy.pdf
14. Family of 4 person parents 9- and 6-years children is travelling, the 6 years child has
salicylates allergy, father asks about nonpharmacological tips?
a) Use only sealed bottled carbonated or alcoholic beverages
b) Frequent use of alcohol based waterless hand sanitizers
c) Use only hot food from vendors
d) Restrict drinks and water
Prevention
Travelers should dine at restaurants with a reputation for safety and avoid foods and beverages from street
vendors. They should consume only cooked foods that are still steaming hot, fruit that can be peeled, and
carbonated beverages without ice served in sealed bottles (bottles of noncarbonated beverages can contain
tap water added by unscrupulous vendors); uncooked vegetables (particularly including salsa left out on the
table) should be avoided. Buffets and fast food restaurants pose an increased risk. Prophylactic antibiotics are
effective in preventing diarrhea, but because of concerns about adverse effects and development of
resistance, they should probably be reserved for immunocompromised patients.

15. Which drug to use if they get


mild diarrhea? (the pharmacist in a
province where he can prescribe
medication)
a) Bismuth subsalicylate
b) Azithromycin
c) Loperamide
d) Ciprofloxacin
e) Rifaximin
16. What is the organism causing TD?
a) E. coli
b) V.cholera
c) Campylobacter Jejuni
d) Salmonella typhi
Most cases of TD in adults are caused by bacteria, predominantly enterotoxigenic E. coli (ETEC) and
enteroaggregative E. coli (EAEC) as well as Campylobacter, Salmonella and Shigella. Other pathogens include
viruses (norovirus, astrovirus, rotavirus) and protozoa, e.g., Giardia and Cryptosporidium.

17. Babysitter gave 4 months baby 10 mL of 125mg/ml Augmentin instead of 2 ml. What is
your advice?
a) Wait 48 hrs with monitoring vomiting and diarrhea
b) Refer to emergency
c) Give antidote
d) Continue with regular dose
e) Call poison center control
f) Call physician
g) Wake him multi times at night to check the breath
Dosage of amoxicillin trihydrate—clavulanate potassium for Children Aged 12 Weeks (3 months) and Older

Overdosage
For management of a suspected drug overdose, contact your regional Poison Control Centre
Many patients have been asymptomatic following overdosage or have experienced primarily gastrointestinal
symptoms including stomach and abdominal pain, vomiting, and diarrhea. Rash, hyperactivity, or drowsiness
have also been observed in a small number of patients. Amoxicillin crystalluria, in some cases leading to renal
failure, has been observed
Activated charcoal may be administered to aid in the removal of unabsorbed drug. General supported
measures are also recommended.
In the case of overdosage, discontinue CLAVULIN, treat symptomatically, and institute supportive measures as
required. If gastrointestinal symptoms and disturbance of the fluid and electrolyte balances are evident, they
may be treated symptomatically.
Renal impairment appears to be reversible with cessation of drug administration. High blood levels may occur
more readily in patients with impaired renal function because of decreased renal clearance of both amoxicillin
and clavulanate. Both amoxicillin and clavulanate are removed from the circulation by hemodialysis.
18. What gene causes SJS of Carbamazepine?
a) HLA-B*1502
b) HLA-B*5701
Carbamazepine‐induced Stevens Johnson syndrome (SJS) and toxic epidermal necrolysis (TEN) in Han‐Chinese,
Thai and Malaysian populations.

19. Colchicine DDI with statin


a) Muscle weakness
b) Lactic acidosis
c) Myopathy
Colchicine may enhance the myopathic (rhabdomyolysis) effect of HMG‐CoA Reductase Inhibitors (Statins).
Colchicine may increase the serum concentration of HMG‐CoA Reductase Inhibitors (Statins).

20. Dementia case: Patient newly diagnosed with Alzheimer’s and start Donepezil. What not to
give with Donepezil? Donepezil increase Ach so cause bradycardia
a) Ramipril
b) Bisoprolol
Because of their pharmacological action, cholinesterase inhibitors may have vagotonic effects on heart rate
(eg, bradycardia). The potential for this action may be particularly important to patients with "sick sinus
syndrome" or other supraventricular cardiac conduction conditions.
Q. Selective and have greater affinity for AchEi in brain than periphery.
Donepezil Reduces the hydrolysis of acetylcholine, increasing the amount available in the synaptic cleft.
Piperidine- Donepezil was effective in 3- to 6-month trials in patients with mild to moderate Alzheimer disease
based (MMSE score of 10 – 26). It was also effective in moderate to severe Alzheimer disease (MMSE 0
Centrally – 17), and is the only cholinesterase inhibitor approved for all disease severities
active Initial daily dose 5mg, taken at night. Can be taken in the morning if sleep disturbances occur
reversible, Monitor treatment effects 2 weeks after initiating therapy or increasing dosage, then every 3 months.
non After 4 weeks, try increasing to the target dose of 10 mg/day. Adjust dose after 4 wk
competitive. S.E: Cholinesterase inhibitors: theoretically, these agents may lower seizure threshold, increase the
risk of GI ulceration or bleeding, or exacerbate COPD or asthma.
Donepezil: >10%: headache, nausea, diarrhea. <10%: vomiting, anorexia, fatigue, sleep disturbance,
syncope, muscle cramps, urinary frequency. Bradycardia (uncommon), heart block (uncommon),
rhabdomyolysis (uncommon), neuroleptic malignant syndrome (uncommon).
Cholinesterase inhibitors: theoretical concern regarding antagonistic effect of combined therapy with
cholinesterase inhibitors and drugs with anticholinergic activity.
Additive bradycardia when combined with BBs or CCBs; few reports of actual interactions.
Toxicity may be increased by inhibitors of CYP2D6 or CYP3A4 such as paroxetine, erythromycin,
prednisone, grapefruit juice. Effectiveness may be reduced by inducers of CYP2D6 or CYP3A4
such as carbamazepine, phenytoin, rifampin.
Patients should be carefully monitored for muscle pain, tenderness or weakness and darkened urine,
particularly if accompanied by malaise or fever. Blood creatine phosphokinase (CPK) levels should
be assessed in patients experiencing these symptoms.
Close monitoring for adverse effects in patients with hepatic disease, reanal diseases & geriatrics
being treated with donepezil hydrochloride is therefore recommended.

21. Before starting Donepezil, what to do?


a) Decrease dose of Ramipril to avoid hypertension
b) Decrease dose of Bisoprolol to avoid bradycardia
Start donepezil at low dose as usual, baseline ECG and monitor closely HR and ECG

22. After one week, the patient developed insomnia. What is your appropriate suggestion?
a) Reduce the dose of donepezil
b) Stop the medication temporarily and start a later
c) Change to another cholinesterase inhibitor.
d) Take Diphenhydramine
e) Take it in the morning

23. Cancer Patient + DVT what to give?


a) Dalteparin
b) Warfarin
c) ASA
d) No need for treatment
Venous Traditional Routine prophylaxis with Q. Vitamin K antagonists e.g
thromboembolic cytotoxic ASA, warfarin or low warfarin are used with caution as
events drugs, molecular weight heparin cancer patients are susceptible to
(deep vein bevacizumab, may be required for patients wide fluctuations in INR.
thrombosis, tamoxifen, receiving regimens containing Q. Low molecular weight heparins
pulmonary sunitinib, dexamethasone and preferred for long-term
embolism) (within thalidomide and thalidomide or its analogues. anticoagulation.
weeks to months) analogues

24. Case with C. difficle & has Diarrhea more than 6 times /day, no vomiting or other
complication. WBC 14 – Scr 85 – BP 128/80 - He took Moxifloxacin one week ago, what kind?
a) Mild
b) Sever
c) Fulminant
Severity Criteria Treatment
Mild Diarrhea plus any additional signs or Metronidazole 500 mg TID PO ˣ 10 days
moderate symptoms not meeting severe or complicated If unable to take metronidazole, vancomycin
disease criteria 125 mg QID PO ˣ 10 days
9
WBC ≤15 x 10 /L and a serum creatinine If no improvement with metronidazole in 5–7
(SCr) level of <133 mcmol/L days, consider changing to vancomycin
Severe disease Serum albumin < 3 g/dL + 1 of the following: Vancomycin 125 mg QID PO ˣ 10 days or;
WBC ≥15 x 109/L & Abdominal tenderness Fidaxomicin 200mg PO BID x 10 days
Severe and Any of the following attributable to CDI: Vancomycin 500 mg QID PO +
complicated Admission to ICU for CDI Metronidazole 500 mg Q8H IV +
disease Hypotension ± required use of vasopressors Vancomycin 500 mg in 500 mL saline QID
“Fulminant” Fever ≥38.5 °C. Mental state changes PR as enema
Ileus or significant abdominal distension Surgical consultation suggested
9 9
WBC ≥35 x 10 /L or < 2 ˣ 10 /L
Serum lactate levels >2.2 mmol/L
End organ failure, e.g., renal failure
Recurrent Treat the 1st recurrence (2nd occurrence) with the same antibiotic that was used for the initial
CDI episode; however, if recurrence episode is severe, vancomycin should be used.
Recurrent CDI The 2nd recurrence (3rd occurrrence) should be treated with pulsed vancomycin regimen.
within 8 wk of Repeat metronidazole or vancomycin pulse regimen.
completion of Pulsed regimens vary & there is no data to support a specific pulsed regimen.
therapy American College of Gastro enterology proposes vancomycin 125 mg QID PO ˣ 10 days then
125 mg OD every 3 days for 10 doses.
Consider FMT Fecal microbiota transplantation after 3 recurrences.

25. What is the reason of C. difficle? / What is the drug that should be remove?
a) Moxifloxacin
b) Pantoprazole
Risk factors for CDI
 Age > 65 (Elderly). Obesity. Enteral feeding.
Immunosuppression. GI surgery.
Chemotherapy
 Antibiotics use: Ampicillin, amoxicillin, broad
spectrum penicillins, Cephalosporins,
fluoroquinolones, Clindamycin. Therapy with
PPIs, H2RAs (lowers stomach acidity)
 Duration of hospitalization & Contact with infected person

26. What is the treatment?


a) Metronidazole 500 mg TID PO for 5 days
b) Fidaxomicin 200mg PO BID for 7 days days
c) Vancomycin 125 mg QID PO for 10 days
27. What is the specific factor for the
pt. that mean he got better
a) Test 10 days
b) Diarrhea decrease
c) Vomiting decrease

28. A cancer patient has cephalexin &


sulfa allergy. He is going for colon
surgery what is the prophylaxis?
a) Cephalexin
b) Clindamycin + metronidazole
c) Vancomycin
d) Amoxicillin 2 g
e) Gentamicin + Clindamycin
For surgical prophylaxis: 1. Cefazolin single dose + metronidazole 2. Cephamycins: Cefoxitin, Cefotetan
29. Patient male 75 years, has valvular atrial fibrillation, he is Diabetic and his profile includes
medication for hypertension, BPH, GERD and Dementia. Calculate the CHADS2 score?
a) 1
b) 2
c) 3
d) 4
Valvular AFib and nonvalvular
AFib are terms used to
describe AFib caused by two
different factors.
AFib is considered valvular when it's
seen in people who have a heart
valve disorder or a prosthetic heart
valve.
Nonvalvular AFib generally refers
to AFib caused by other things, such
as high blood pressure or stress.

30. Which anticoagulant is suitable for the above patient?


a) Warfarin + ASA
b) Rivaroxaban + ASA
c) Clopidogrel
Oral anticoagulation by DOAC is recommended in patients with persistent or paroxysmal atrial fibrillation
without valvular disease but at high risk of embolic stroke, i.e., having any of the following features: prior
ischemic stroke, transient ischemic attack, or systemic embolism; age >75 years; moderately or severely
impaired left ventricular systolic function or congestive heart failure, history of hypertension, or diabetes
mellitus. For patients at lower risk, individualized treatment is 29 required.
Warafarin for patients with atrial fibrillation and valvular heart disease, especially mitral valve stenosis,
anticoagulation is recommended. For patients with atrial fibrillation and prosthetic heart valves,
anticoagulation is required, with the target INR generally increased, with or without aspirin added, depending
of risk factors related to the replaced valve or inherent to the patient.

30. Patient experiencing Lithium intoxication, what could be the reason?


a) Hyperkalemia
b) Hypophosphatemia
c) Hyponatremia
Lithium is related to Sodium the most: any factor that alter salt and/or water balance Drugs: ACE, NSAIDs,
ARBs, Thiazide diuretics added to established lithium toxicity (loop diuretics, little effect)
Lithium therapy is not recommended in patients with the following conditions, due to increased risk of lithium
toxicity: Renal impairment, Cardiovascular disease, Severe debilitation, Dehydration, Sodium depletion, Brain
damage & Purging disorders
Q. Lithium First-line as monotherapy or in combination with an SGA and should be offered early in the
carbonate course of illness. Effective in preventing manic and depressive relapses.
May be useful for those with high risk of suicide or self harm
Because of overall value, consider lithium first, particularly in younger adults
During acute manic episodes, patients may exhibit increased tolerance to lithium. Therefore,
higher doses may be needed during mania, followed by dose reduction after mania resolves.
Initial: 300 mg TID PO (150 mg BID PO in the elderly). Dose can be titrated up by increments
of 300–600 mg/day every 5–7 days, depending on tolerability and response
Usual: 900–2100 mg/day PO, guided by serum concentrations (target 0.8–1 mmol/L; 0.5–0.8
mmol/L in elderly patients).
The extended-release tablet should be swallowed whole (not to be cut, crushed or chewed).
Narrow therapeutic window. Measure serum concentrations 9–13 h post-dose.
S.E: Tremor (most common), Polydipsia, Hypothermia or hyperthermia, Weight gain and
weight loss, Nausea or vomiting, Diarrhea, Hypothyroidism and hyperthyroidism.
While high blood lithium level produces more response, it involves more risk.
Despite earlier literature recommendations of higher lithium levels, low to medium levels (0.5–
0.8 mmol/L) are increasingly suggested in maintenance, with the switch or addition of another
maintenance medication strategy preferred over pushing the lithium blood level higher.
Once effective dose is reached; single daily dose is preferred.
Steady state reached in 5 days. Onset of lithium 3 weeks. Highly toxic in overdose.
Renal excretion. If GFR 10–50 mL/min  Reduce dose by 25–50%, If GFR <10 mL/min 
Reduce dose by 50–75%
Measure electrolytes and do 24-h urine for ClCr when starting longterm treatment.
Monitor thyroid and renal function at least every 6 months.
Toxic levels may result when adding NSAIDs, ACEIs, ARBs and especially thiazide diuretics.
Reduce lithium dose and check serum level if long-term treatment with these agents is required.
Signs & symptoms of toxicity ataxia, tremor, sedation or agitation, diarrhea, vomiting.
In cases of acute overdose, consultation with a Poison Control Centre is recommended.
Asymptomatic patients should be observed, due to delayed onset of CNS symptoms.
Early symptoms of chronic lithium toxicity can usually be treated by reduction or cessation of
dosage of the drug and resumption of treatment at a lower dose after 24–48 hours.
Whole bowel irrigation with PEG solution may help minimize lithium absorption, particularly
of sustained-release formulations. Activated charcoal does not adsorb lithium but may be of
value if multiple drug ingestion is suspected.
Stop lithium temporarily during acute intermittent illnesses causing fluid and electrolyte losses.
For lithium-associated cognitive impairment, check lithium level and thyroid function.
Lowering the dose or using a slow-release formulation may improve cognitive function.
Patients who experience tremor while taking lithium may benefit from elimination of dietary
caffeine, lithium dose reduction or addition of a beta-blocker such as propranolol or atenolol.
Patients who experience diarrhea while taking slow-release lithium preparations may fare better
with immediate-release formulations.
Drink 8 to 10 glasses of water or other liquids every day.
Keep your salt intake about the same.
Keep your caffeine intake about the same.
Take lithium with food or milk. Avoid alcoholic beverages.
31. Lithium toxicity, what to do?
a) Gastric lavage
b) Carbon charcoal
c) Flunarizine
d) Hemodialysis

32. The drug is excreted by kidney, Digoxin dose 0.125, its level when the withdrawal 1 hr after
injection is 2.5 ng/L. (Drug is first kinetic by infusion). What is the reason?
a) Prolonged t1/2 due to kidney failure
b) Measured the concentration early before the drug being distributed
Measure trough serum concentrations at least 6 h after administration and adjust the dose to maintain the
serum concentration between 0.6 and 1 nmol/L. The idea of digoxin does not measure its serum level
directly after dose, so it is calculated after waiting for about 6 hours for after the dose.

33. If you know that normal level is <2, what to do?


a) Nothing to do because it is early to measure the serum level
b) D/C for 3 days and restart at 0.625
c) Decrease the dose to 0.625
Individualize the dosage based on the patient’s age, weight, renal function and concomitant drugs; the
usual range is 0.0625–0.25 mg daily. Given the narrow therapeutic index of digoxin, pay particular attention
to identifying and preventing potential drug interactions.
Digoxin
 Narrow therapeutic index drug.
 Predominantly renally eliminated (dose 0.12 mg to 0.25 mg), hepatically metabolized into active
metabolite.
 Prolonged distribution phase (6-12 hours)
 Gastrointestinal metabolism
Significant drug interactions:
 Quinidine, verapamil & amiodarone and hypokalemia-causing drugs like furosemide, thiazides,
corticosteroids, macrolides and tetracycline.
 Patients using amiodarone medication should reduce 30 % - 50% of their digoxin dose.
Digoxin dosing is guided by patient’s cardiac and renal function and disease state being treated.
Since digoxin is available in several different dosage forms, the bioavailability (F) must also be considered.
IV = 1; capsule = 0.9; tablets = 0.7; elixir = 0.8
Digoxin Oral Trough sample Heart Failure: Dosage should be stable for 5–7
and Post-load: at least 6 h after 0.6–1.3 nmol/L. days in patients with normal
IV last dose of loading regimen renal function.
(IV or PO). Atrial Fibrillation: Half-life 35–40 h.
For periodic monitoring, 1–2 nmol/L. Time to steady-state prolonged
sample just before next dose. in patients with ↓ renal function.
For suspected toxicity, Levels taken within 6 h of dose
sample anytime. may be artificially elevated.
34. What is the error can be happened due to relying on technology?
a) Drop down menu to select the drug
b) Similar patient names
c) Dispensing error by bringing wrong medicine from the shelf
d) Using first 3 letters of the drug name so take the wrong drug
https://www.ismp.org/news/ismp-publishes-top-10-list-
medication-errors-and-hazards-covered-newsletter
Selecting the wrong medication after entering the first
few letters of the drug name. Entering just the first few
letter characters of a drug name or combination of the
first few letters and product strength can allow the
presentation of similar‐looking drug names on
technology screens, leading to selection errors. This is a
problem that has increased in frequency with the
upswing in technology use. In fact, wrong selection
errors may now rival or exceed those made with
handwritten orders.

35. Manager find recurrent loss in narcotics; he reviewed the videos and discovers a technician
put Oxycodone tablets (1-2 tablets every time - looks like addicted) in her pocket. What to do?
a) Talk to her & Keeps her away from Narcotics
b) Continue reviewing the videos to have more evidence
c) Recommend an addiction treatment center
d) Inform the college
e) Inform the college, the police and fire him

36. Prescription containing codeine, the computer give alert that the pt. had filled it 5 days ago
from another pharmacy, the pharmacist refuses to dispense and the pt. become very angry and
leave, what to document?
a) Patient abuse medication, I refuse to dispense it
b) Pt came with Narcotic prescription he got it 5 days, I refused to dispense and he was rude.
c) Pt came with Narcotic prescription; he got it 5 days ago, so I refused to dispense it

37. Cancer pt. before initiation of Infliximab the doctor needs to insure updated immunization.
Patient takes Rituximab and will start with biological dimers for rheumatoid arthritis, he has
nephritic Lupus syndrome, which vaccine is harm for him or Which vaccine not to give?
a) HBV
b) Pneumococcal
c) MMR
d) Shingrix
Live vaccines contraindicated in immunosuppressed IBD patients include intranasal influenza, measles‐
mumps‐rubella (MMR), smallpox, oral typhoid, yellow fever and varicella. Live vaccines should not be given
to patients using immunosuppressive therapy until 3 months after these therapies are stopped.

38. Framingham cardiac risk score contains all except;


a) Family history
b) Smoker history
c) HDL
d) LDL (obsolete, not used now)
e) BP
If there is a family history of premature CVD, the 10‐
year risk score should be doubled to produce a
‘modified Framingham Risk Score’
The FRS estimates the 10‐year risk of manifesting
clinical CVD (CAD, Stroke, PVD, CHF, cardiac death).
Although not examined in the 2008 model, it is common practice to double the FRS if there is a FHx of
premature CAD in a 1st degree relative (men <55y, women <65y).
*The risk stratification tool for the ESC is the SCORE system which estimates 10y risk of CVD death. Patients
with a 10y risk of CVD death ≥5% are considered high risk. The lipid guidelines recognize risk equivalents as a
distinct category that warrant immediate treatment. For patients with an ESC SCORE ≥ 5% a 3‐month trial of
lifestyle measures is a reasonable starting point. If after 3 months the lipids remain above moderate risk
targets and the SCORE remains ≥ 5% then intensive therapy to reach high risk targets is recommended.

39. A case was given, in recording this in SOAP format what would be recorded in the A section?
a) Drug addition to his old profile
b) Patient symptoms
c) Drug interaction due to CYP 450
SOAP Meaning Example
SUBJECTIVE • What the patient reports Signs & symptoms, when they start
• Info from patient’s perspective Smoking history, angina symptoms,
patient medications
OBJECTIVE • What the provider reports/measures Monitoring parameters like Exercise
(Findings) • Diagnostics, Laboratory results tolerance, lipid profile, BP, HR,
blood glucose, Cholesterol level,
x-ray, SrCr (80)
ASSESSMENT Interpretation of subjective & objective information Working diagnosis
If writer is physician, assessment will be a disease state Drug interaction due to CYP450
or condition diagnosis & explain reason for diagnosis. NYHA, Framingham score
The pharmacist SOAP notes, assessment will identify
DTP and explains why DTP needs to be corrected.
PLAN • Action plan Discontinue Drug X and initiate
• Monitoring parameters Drug Y
40. Drug interaction between Methadone and Citalopram?
a) QT prolongation (Serotonin syndrome was not there)
CPS: Some selective serotonin reuptake inhibitors
(SSRIs) (e.g., sertraline, fluvoxamine) may increase
methadone plasma levels upon co‐administration
with Methadone and result in increased opioid
effects and/or toxicity.
Co‐administration of methadone with a
serotonergic agent, such as a Selective Serotonin
Re‐uptake Inhibitor or a Serotonin Norepinephrine
Re‐uptake Inhibitor, may increase the risk of
serotonin syndrome, a potentially life‐threatening
condition (see Warnings and
Precautions, Serotonin Syndrome).
Monitor for signs and symptoms of serotonin
syndrome/serotonin toxicity (eg, hyperreflexia,
clonus, hyperthermia, diaphoresis, tremor,
autonomic instability, mental status changes)
when these drugs are combined. Patients with other risk factors (eg, higher drug concentrations/doses,
greater numbers of serotonergic agents) are likely at greater risk for these potentially life‐threatening
toxicities.

41. A new drug for osteoporosis was studied in two groups. This medication shows 4.5% of
reduced risk of fracture in group 1 that received the drug and shows 9.5% of reduced risk of
fracture in group 2 that receive the placebo. on ther hand this drug shows 1.4% of development
of esophageal ulcer in group 1 where as 0.28% of esophageal ulcer in group 2. Calculate NNT?
a) 10
b) 20
c) 30
d) 40
NNT = 1/ARR ARR = 9.5% ‐ 4.5% = 5% NNT = 1/5% = 20

42. In hospital patient has received 1 gm Vancomycin, the blood conc was 35 m.mol/L after
2hrs of injection. After 3 days the blood conc. was 17 m.mol/L. He has received a second dose
of 1 gm Vancomycin. Peak conc. is 20 m.mol/L, trough 15 m.mol/L when should he take the
third dose?
a) 1 day after the second dose
b) 2 days after the second dose
c) 3 days after the second dose
d) 5 days after the second dose
Log C = log Co ‐ k*t /2.303 k = (2.303 / t) * log (Co / C)
k = (2.303/70) * log (35/17) = 0.010 hr‐1
Log C = log Co ‐ k*t /2.303 Log 35 = log Co ‐ (0.010*2) / 2.303
log Co = 1.544 + 0.008 = 1.552 Co = 35.7 m.mol/L
So, upon the second dose: Co = 35.7 + 17 (remains from the 1st dose) = 52.7 m.mol /L
C = 15 m.mol / L. (trough conc. at which the 3rd dose has to be taken)
k = 0.010 hr ‐1 Log C = log Co ‐ k*t /2.303
t = (2.303 / k) * log (Co / C) t = (2.303/0.010) * log (52.7/15) =125.679 hr = 5.23 days
So, the 3 rd dose has to be taken 5 days after the 2nd dose

43. The least drug causing weight gain, where can you look??
a) Rx-files
b) CPS
c) Micromedix

44. TPN (Marian question). Child needs TPN with 0.5 micromole/L of Selenium, your stock is
0.25 micromole/L & 40 microgram /ml, how much of 40 µg/ml of selenium would you add if
you want to make 482mL (M. Wt of selenium is 79).
40 microgram ------- 1 ml X microgram -------- 1000 ml
X = 1000*40/1 = 40000 microgram / L. No. of m. mole = 40000 / 79 = 506.32 m. mole /L.
0.25 505.82 ------- X
0.5
506.32 0.25 --------- Y
X+Y = 482 ml Y = 0.25/ (505.82+0.25) * 482 = 0.238 L.

45. Which contravene (violate) with the pharmacist legal scope of practice?
a) Initiate antihypertensive drug
b) Change the dose of levothyroxine
c) Extend prescription of BZD
d) Renew prescription of....
Expanded Scope of Practice for Pharmacists: Roles include:
 Provide immunizations (vaccines) & Improve patient outcomes
 Renew/Extend prescriptions for continuing care (i.e; provide emergency refills)
 Advancing drugs: dispensing a small amount of drug in case of emergencies such as patient travelling
or no clinics opened to get prescription.
 Adapting: change in dose, formulation, regimen, duration and route. Renew or adapt prescriptions with
exception of narcotics and controlled and targeted substance.
 Therapeutic substitution & Prescribing in emergency situations
 Optimize drug therapy for chronic conditions (diabetes, HTN)
 Reduce hospital emergency admissions & Decrease cost to health care system.
 Order blood work /lab tests to assess patient and make recommendations
 Prescribing under delegation, prescribing within a collaborative agreement/relationship
46. Tallman lettering, which one is right?
a) vinCRIStine --- VINBlastine
b) cefTRIAXone --- cefaZOLIN
c) DOBUTamine --- DOPamine
For Vincristine – Vinblastine = vinCRIStine ‐ vinBLAstine
For Ceftriaxone and Cef, = cefTRIAXone ceFAZolin ‐ cefoTEtan – cefOXitin – cefTAZidime
Dobutamine and Dopamine= DOBUTamine – DOPamine

47. Delegation of compounding to assistant, why?


a) He did it before
b) To double check calculations
c) Pharmacist is responsible about safety and accuracy

48. Pharmacy manager decided to change the pharmacy software to be able to include one
original prescription image in the system or manger will change the system to barcode,
what is the benefit of this approach?
a) Decrease all types of errors by time
b) Differentiate between sizes
c) Differentiate the strengths
All are correct but the most important is A. Go always to patient safety and his interest

49. Nurse in ICU prepare vials for injection in an emergency situation. What to say?
a) The person prepared it should inject the patient
b) Don’t prepare more than 3, 4 vials for the same drug
c) Labeling to increase safety of IV preparation
d) For immediate use in hospital

50. Patient is diagnosed with sever GERD, which one is a suitable choice?
a) Endoscopy
b) Screen for H pylori
c) Treatment with PPIs

51. GERD case, Patient has food induced


heart burn and takes H2 antagonist with
no improve.
a) Refer to doctor
b) Refer for endoscopy
c) Take PPI for 8 weeks
d) Take antacid
e) Increase dose of H2 antagonist
Moderate to Severe Gastroesophageal Reflux Disease
If antacids or H2RAs alone are not effective, or if there is endoscopic evidence of erosive esophagitis, the most
effective and common approach is an 8‐week course of any of thePPIs. The goal is to eliminate symptoms,
heal erosive esophagitis and prevent complications

52. Childbearing age patient (on OCP) has generalized tonic colonic seizures and was on
carbamazepine, developed rash what to recommend?
a) Phenytoin
b) Valproic acid
c) Lamotrigine
d) Phenobarbital
Valproic acid has low incidence of rash and less
interaction with OCP.
If there is rash on any of lamo/cbz/phenytoin, do not
use any of them (cross allergy is likely)
Patients receiving anticonvulsant agents are often
sensitive to more than one agent, and cutaneous
reactions are particularly common. It has long been known that the “aromatic” anticonvulsants
(phenobarbital, phenytoin, and carbamazepine) have a high rate of cross‐reactivity for severe adverse
cutaneous reactions.

53. Patient taking Oxycodone 30mg BID and for breakthrough pain Percocet 5mg/4hr PRN (he
usually takes 4 doses per day), patient can't afford ext. release oxycodone so shifted both IR and
ER to hydromorphone. Dr. wants to switch to Hydromorphone BID and morphine PRN. How
much does he need if “30 mg Morphine = 6 mg Hydromorphone = 20 mg Oxycodone”? But
choices included patch and IR tabs.
a) 6 mg HM BID + 4 mg q6h PRN
b) 9 mg HM BID + 1 mg q6h PRN
c) 12 mg HM BID + 2 mg q6h PRN
d) 12 mg HM BID + 4 mg q6h PRN

54. Driving after inhaled cannabis by how long?


a) 4 – 6 hrs
b) 10 – 12 hrs
c) 6 - 8 hrs (consider in post euophoric situation)
d) 24 hrs
No driving for 4 hours (inhalation),
6 hours (ingestion), 8 hours (post‐
euphoric effect), and impairment
may continue for 24 hours
55. Your patient is administering chemotherapy (Cyclophosphamide) and Dexamethasone at
hospital, his wife knew that, if she learned how to give her husband the drug, they will no need
to go to the clinic and he can take the drugs at home. What the advice you provide her about
how to handle these medications?
a) Wash hands before and after touching both drugs
b) Wear gloves before cyclophosphamide, wash hands before & after dexamethasone
c) Wear gloves for both
d) No special handling is required
https://www.caymanchem.com/msdss/11015m.pdf
https://www.baxter.ca/sites/g/files/ebysai1431/files/2018-11/PROCYTOX_PM_SEP072012_EN.pdf

56. Which of the following preparations need sterile condition/biological Cabinet?


a) Eye drop
b) Oral antibiotic

57. Pt takes Capecitabine & pharmacist wants to advise him on product to prevent oral ulcer.
a) Dental floss after each meal
b) Mouth wash Chlorhexidine (contains alcohol)
c) Frequent use saline solution as gargle
d) Check up his dental situation

58. The patient got oral lesion. What to give?


a) Oral Fluconazole
b) Saline
c) Topical Nystatin
Oral lesions, such as ulcers, infections, and inflammation, are common.
Oral candidiasis can be treated with nystatin oral suspension 5 to 10 mL qid, clotrimazole troches 10 mg qid,
or fluconazole 100 mg po once/day.
Mucositis due to radiation therapy can cause pain and preclude sufficient oral intake, leading to undernutrition
and weight loss. Rinses with analgesics and topical anesthetics (2% viscous lidocaine 5 to 10 mL q 2 h or other
commercially available mixtures) before meals, a bland diet without citrus food or juices, and avoidance of
temperature extremes may allow patients to eat and maintain weight. If not, a feeding tube may be helpful if
the small intestine is functional. For severe mucositis and diarrhea or an abnormally functioning intestine,
parenteral alimentation may be needed.

59. The patient takes Ciprofloxacin due to UTI, then he got lesions in his arms and legs (bad
ulcers and sores). What is the reason?
a) Ciprofloxacin
b) Capecitabin
Hand-foot syndrome (HFS) Apply moisturizers at least TID. Dose interruptions and reduction
(erythema, edema, pain, peeling of Avoid prolonged heat exposure. result in greatest benefit.
skin on fingers and toes) (weeks to Urea-based emollients preferred. Manage promptly with
months) Capecitabine, liposomal Avoid irritation and friction from symptomatic treatments and
doxorubicin, fluorouracil ill-fitting shoes & clothing. analgesics.
Hand-foot-skin reaction (HFSR) Vitamin B6 150–200 mg/day may Discontinue therapy if severe or
(same as above + hyperkeratosis at be prescribed (evidence poor). recurrent.
pressure points) (weeks to months) Limit prolonged pressure to Wound care for desquamation and
Axitinib, dabrafenib, pazopanib, weight-bearing areas and treat ulcerations.
regorafenib, sorafenib, sunitinib existing calluses & hyperkeratosis

60. What to monitor for the above patient for capecitabine?


a) Blood pressure
b) Blood glucose
c) Nephritis???
d) Liver cirrhosis
4.4 Special warnings and precautions for use
Dose limiting toxicities include diarrhoea, abdominal pain, nausea, stomatitis and hand‐foot syndrome (hand‐
foot skin reaction, palmar‐plantar erythrodysesthesia). Most adverse reactions are reversible and do not
require permanent discontinuation of therapy, although doses may need to be withheld or reduced.
Diarrhoea. Patients with severe diarrhoea should be carefully monitored and given fluid and electrolyte
replacement if they become dehydrated. Standard antidiarrhoeal treatments (e.g. loperamide) may be used.
Dehydration. Dehydration should be prevented or corrected at the onset. Patients with anorexia, asthenia,
nausea, vomiting or diarrhoea may rapidly become dehydrated. Dehydration may cause acute renal failure,
especially in patients with pre‐existing compromised renal function or when capecitabine is given
concomitantly with known nephrotoxicmedicinal products.
Cardiotoxicity. Cardiotoxicity has been associated with fluoropyrimidine therapy, including myocardial
infarction, angina, dysrhythmias, cardiogenic shock, sudden death and electrocardiographic changes (including
very rare cases of QT prolongation). These adverse reactions may be more common in patients with a prior
history of CAD. Cardiac arrhythmias (including ventricular fibrillation, torsade de pointes, and bradycardia),
angina pectoris, MI, HF and cardiomyopathy have been reported in patients receiving capecitabine.
Hypo‐ or hypercalcaemia.
Diabetes mellitus or electrolyte disturbances.
Hepatic impairment. Capecitabine use should be carefully monitored in patients with mild to moderate liver
dysfunction, regardless of the presence or absence of liver metastasis. Administration of capecitabine should
be interrupted if treatment‐related elevations in bilirubin of >3.0 x ULN or treatment‐related elevations in
hepatic aminotransferases (ALT, AST) of >2.5 x ULN occur.
Renal impairment. The incidence of grade 3 or 4 adverse reactions in patients with moderate renal
impairment (creatinine clearance 30‐50 ml/min) is increased compared to the overall population
Dihydropyrimidine dehydrogenase (DPD) deficiency: DPD activity is rate limiting in the catabolism of 5‐
fluorouracil. Patients with DPD deficiency are therefore at increased risk of fluoropyrimidines‐related toxicity,
including for example stomatitis, diarrhoea, mucosal inflammation, neutropenia and neurotoxicity.
61. A pt. got endocarditis and culture revealed that enterococci faecalis is the causative
organism. Which one is the suitable treatment:
a) Ampicillin + Gentamycin
b) Ceftriaxone + Gentamycin
c) Cefazolin + Rifampin
The Gram‐positive bacteria staphylococci 40% (Staph. aureus, MSSA, MRSA, Coagulase‐negative
staphylococci (e.g. S. epidermidis), streptococci 20% (Viridans streptococci ‐ VGS, S. bovis) and enterococci
10% (E. faecalis, E. faecium) account for the majority of pathogens causing IE.

62. Treatment Duration?


a) 4-6 weeks
b) 2-4 weeks
c) 10 weeks
d) 6-8 weeks
Antibiotic treatment regimens for enterococcal endocarditis 10%
Sensitive to Ampicillin or penicillin G + 4 – 6 wk Native valve:
Penicillin, gentamycin 4-wk therapy recommended for patients with
Gentamicin symptoms of illness <3 months;
and 6-wk therapy recommended for patients with
Vancomycin symptoms >3 months.
Prosthetic valve: 6-wk therapy recommended.
Ampicillin + ceftriaxone 4 – 6 wk Recommended for patients with creatinine
clearance <50 mL/minute pre-existing or
developing during treatment with a gentamicin-
containing regimen.
Vancomycin + gentamycin 6 wk
Sensitive to Ampicillin + ceftriaxone 6 wk
Penicillin, Ampicillin or penicillin G + 4 – 6 wk Native valve:
Streptomycin streptomycin 4-wk therapy recommended for patients with
and symptoms of illness <3 months;
Vancomycin 6-wk therapy recommended for patients with
and Resistant symptoms >3 months.
to Prosthetic valve: 6-wk therapy recommended
Gentamicin Vancomycin + streptomycin 6 wk
Sensitive to Vancomycin Vancomycin + 6 wk
and Aminoglycosides and gentamycin
Resistant to Penicillin
Resistant to Vancomycin Linezolid or ≥ 6 wk The optimal dosing of daptomycin has not been
Aminoglycosides & daptomycin established
Penicillin
63. Pregnant patient and DM, she is using douche for last 3 months, now come with vaginal
itching & odorless white, curdy discharge, what is the risk factor?
a) DM
b) Pregnancy
c) Douching
d) Female gender

64. What is the first question you should ask?


a) Do you have painful urination?
b) Is there any discharge?
c) How long do you have this symptom?
d) When was your last menstrual period?
e) Is it accompanied by unusual odour?

65. What is the treatment?


a) Miconazole ovules 7 days
b) Metronidazole tab (consider this answer in case of bacterial vaginitis)
c) Clotrimazole cream 7 days
d) Clindamycin cream
Vulvovaginal Q. Characterized by inflammation, Stinging/burning, pruritus of vulva and vaginal areas,
candidiasis odourless white, clumpy, curdy “Cottage cheese” discharges.
VVC Vaginal pH <4.5 (normal), -ve "Whiff" test, +ve PMN.
Caused by Risk factors include Chemical irritants (amtiseptics, soaps,
Candida deodorants,), DM, excessive refined carbohydrates diet,
albicans HIV/AIDS, antibiotics, chemotherapy, CS, hormone therapy,
oral contraceptives, tamoxifen, Menses, Pregnancy, Stress,
synthetic undergarments, Tight-fitting clothes. contraceptives (e.g. spermicide diaphragms u,
sponges, intrauterine devices), douching. age between 30 & 40.
Treatment is unnecessary in asymptomatic cases. Use prophylactic topical or oral azoles at
start of antibiotic treatment in women prone to recurrent VVC secondary to antibiotic use.
Recurrent vulvovaginal candidiasis (≥ 4 episodes/year) requires investigation and referral
No treatment for sex partner; but consider treatment in women with recurrent infections
Fluconazole, oral: Uncomplicated infection: 150 mg PO × single dose
Recurrent infection (≥4 episodes/y): 150 mg every 72 h × 3 doses then weekly for 6 m.
S.E: Headache, nausea, abdominal pain, diarrhea, dyspepsia, dizziness. C.I in pregnancy.
If patient is prone to VVC and requires a course of antibiotics, consider prescribing
prophylactic topical or oral antifungals until antibiotic course is completed.
Itraconazole, oral: Recurrent infection (≥4 episodes/y), maintenance treatment (following
induction treatment with topical azole x 10–14 days): 200–400 mg monthly PO × 6 m.
Clotrimazole, Vaginal tablet and cream:
Recurrent infection (≥4 episodes/y): extend treatment period to 10– 14 days then maintain
with clotrimazole 500 mg vaginal tablets once monthly for at least 6 months.
Menstruation is not an indication to stop treatment.
Q. Safe for use in pregnancy; 7–14 days treatment period may be necessary.
Follow up if symptoms persist despite treatment or recur within 2 months of onset.
Miconazole, vaginal ovules or cream, also available in combination with external cream.
Ovules contain hydrogenated vegetable oil and mineral oil, which may decrease
effectiveness of condoms or diaphragms.
Terconazole 0.4% vaginal cream
Uncomplicated infection: 1 applicatorful (5 g) QHS PV × 7 days
S.E: local hypersensitivity, very rare cases of anaphylaxis & toxic epidermal necrolysis TEN
Advise patients to discontinue treatment if symptoms of allergic reaction appear
Boric acid vaginal capsules: reserved for treatment of recurrent or non-albicans VVC.
Recurrent infection (≥4 episodes/y): 300–600 mg gelatin capsules daily PV × 14 days then
maintain with 300 mg capsule PV × 5 days/month beginning the first day of menstrual cycle;
continue for at least 6 months. Useful in treatment of VVC in immunocompromised host.
S.E: Local irritation, vaginal burning; more pronounced with higher dose. 300 mg capsules
are less irritating than 600 mg capsules. Contraindicated in pregnancy.

66. When is the recommended time to start ART for a patient recently diagnosed with HIV?
a) After confirm diagnosis with AIDS
b) Viral load more than 5000/ 10,000
c) ART should be initiated at diagnosis regardless of CD4 count
d) When the CD4 cell count goes below 500
e) When the patient begins to manifest symptoms of advanced HIV

67. What is the most sensitive test for pancreatitis?


a) Amylase
b) Lipase
c) ALT
d) AST
Q. Acute pancreatitis is acute inflammation (Non-infectious) of the pancreas (and, sometimes,
Acute adjacent tissues). The most common triggers are gallstones and alcohol intake.
pancreatitis The severity of acute pancreatitis is classified as mild, moderately severe, or severe based on the
presence of local complications and transient or persistent organ failure
Causes
Disorders: hypertriglyceridemia, alcoholism & bulimia nervosa.
Drug induced: EtoH, CHC (Estrogen/ EE  ↑ TG), Anti-HIV medictions (didanosine, PIs),
isotretinoin ↑ TG, incretin-based drugs DPP - 4 Inhibitors (Alogliptin, Linagliptin, Saxagliptin
& Sitagliptin), (GLP-1) Agonists (Dulaglutide, Semaglutide, Liraglutide & Lixisenatide
In acute pancreatitis serum lipase and serum amylase are elevated however serum lipase are
slightly more sensitive in both major causes of pancreatitis gallstone and alcoholic associated
acute pancreatitis.
Acute Physiologic Assessment and Chronic Health Evaluation (APACHE) II Scores should be
calculated on admission and daily for the first 72 hours after admission.
An APACHE II Score of 8 or higher at baseline or in the first 72 hours is suggestive of severe
acute pancreatitis and is predictive of a worse clinical course.
Supportive care with fluid resuscitation, pain control and nutritional support
Prophylactic abx are NOT recommended regardless of type and severity

68. A 54 years postmenopausal female with risk of osteoporosis, she has hot flashes and renal
failure. What can decrease the risk of vertebral and non-vertebral fracture?
a) Etidronate
b) Raloxifene
c) Conjugated-estrogen/Bazedoxifene
d) Teriparatide
e) Denosumab
Based on the fracture prevention benefit seen in the WHI studies, Osteoporosis Canada guidelines still include
estrogen as a first‐line therapy for prevention of hip, nonvertebral and vertebral fractures but limit the use to
women experiencing vasomotor symptoms. DUAVIVE (conjugated estrogens/bazedoxifene) is indicated in
women with a uterus for treatment of moderate to severe vasomotor symptoms associated with menopause.
In postmenopausal women with osteoporosis, the SC administration of 60 mg of denosumab every 6 m for 36
m significantly reduced the risk of vertebral and nonvertebral fractures and the risk of hip fracture.
Residronate also can be used for vertebral & non‐vertebral fractures.
Raloxifen first‐line option but in the absence of nonvertebral fracture prevention data, consider it only after
the other first‐line therapies have been rejected for individuals at high‐risk of hip fracture
Denosumab is contraindicated in patients with hypocalcemia because it can cause calcium shifts that result
in profound hypocalcemia and adverse effects such as tetany. Osteonecrosis of the jaw and atypical femoral
fractures have been rarely reported in patients taking denosumab.
Patients taking denosumab should not undergo a drug holiday because stopping this drug may cause a rapid
loss in bone mineral density and, importantly, increase the risk of fractures, particularly vertebral fractures,
sometimes multiple. If and when denosumab is discontinued, transition to a bisphosphonate such as
IV zoledronic acid should be considered.

69. List of medication (Since a long time + Ibuprofen, Ibuprofen dose has been increased in the
last 2 days) for a 55 years patient with arthritis and hypertension, what is the risk factor of high
blood pressure?
a) Age
b) OA treatment

70. Pt has insomnia and try OTC drugs, he heard about non pharm, what is your recommendation?
a) CBT
b) Acupuncture
c) Hydrotherapy
d) Meditation
e) Hypnotherapy
Nonpharmacologic Choices
 Good sleep hygiene is the cornerstone of any sleep intervention and is incorporated in the first line
intervention for insomnia, cognitive behavioural therapy for insomnia (CBT‐I).
 Aerobic exercise, a useful modifier of stress and dysphoric moods, also promotes deeper and more
restful sleep.
 Encourage patients with insomnia to eliminate daytime rest periods and increase exercise, such as
brisk walking.
 The quality of evidence for a variety of other nonpharmacologic strategies (e.g., music, foot baths,
sound masking, acupuncture) is low or very low.

71. Patient on Apixaban, scheduled for a colonoscopy after 2 weeks what to do regarding his
anticoagulant therapy?
a) Stop Apixaban now
b) Stop 2 doses before procedure
c) Stop the day of procedure and restart after 6 hours
d) Change to IM Heparin
e) Stop Apixaban 5 days before procedure
72. How will you inform the doctor that, you have stopped Apixaban?
a) Call the doctor and ask his receptionist to give you the doctor
b) Ask the patient to inform the doctor
c) Send a fax
d) Call the clinic and ask them to inform the doctor

73. Patient heard about vitamins for AMD from his friend and ask pharmacist for prevention.
What pharmacist advice?
a) It is not used for prevention
b) It is useful for only dry AMD
c) With no B carotene is good for smokers
d) Only for moderate to severe cases (early to mid stage)
Another version: There was a question on the difference between dry and wet age-related
macular degeneration (AMD) not exactly these words but this gives you an idea
a) Which one is more prevalent? (Dry AMD more prevalent)
b) Which on is more severe? (wet AMD is a more severe condition)
Prevention of Progression
 Advise patients to quit smoking to reduce their risk of AMD. Encourage a balanced diet.
 Numerous vitamin and mineral supplements are marketed to help reduce the risk of AMD. Choosing a
product with safe ingredients in effective strengths requires careful assessment of the evidence.
Vitamins for AMD
People who have a certain form of age‐related macular degeneration (AMD) may benefit from a specific mix of
vitamins and minerals. Taking these nutritional supplements might help slow this eye disease.
About 8 out of 10 people with AMD have the dry form. This condition is due to a breakdown or thinning of
the macula. Dry AMD usually begins when tiny, yellow deposits called drusen form under the retina.
Eventually, the macula may become thinner and stop working properly.
Many people with AMD have drusen. These alone do not cause vision loss. But when drusen grow in size or
number, you are at risk for getting early or intermediate AMD. There are not always symptoms with these
stages of AMD, though people with intermediate AMD might start to notice a blurred spot in their central
vision. Advanced AMD develops when cells in your macula begin to break down. This is when the blurred spot
in your central vision starts getting bigger and darker. That is what robs you of your central vision.
Dry AMD and AREDS Vitamins
AREDS 2 (Age‐Related Eye Disease Study 2) was a very large research study. It looked at taking vitamins and
minerals daily for AMD. This study found that certain nutritional supplements could help some people who
have a lot of drusen. These supplements may also help people who have lost a lot of vision in at least one eye
from AMD. Taking the following nutritional supplements every day may help these people lower their risk of
getting late‐stage or wet AMD: Vitamin C (500 mg) & Vitamin E (400 IU), Lutein (10 mg), Zeaxanthin (2 mg),
Zinc (80 mg), Copper (2 mg)
It is important to remember that nutritional supplements are not a cure for AMD, but they may help to slow
the disease in some people with early‐ to mid‐stage AMD.
74. Dose switch from twice daily dosing of intermediate acting NPH to LA Insulin Detemir,
what is the appropriate recommendation
Do a direct exact dose to dose conversion from NPH to Detemir
Calculate the equivalent dose of the detemir and reduce the dose by 20% (not the exact words
but this the correct option)
https://www.diabetes.org/sites/default/files/2019-08/switching-between-insulin.pdf

75. Menthol 0.42 Camphor 0.14


Triturate with (I can’t remember). Camphor and menthol combination called?
a) Liquid mixture
b) Eutectic mixture
c) Ionic Mixture
d) Liquid Crystalline Mixture

76. This mixture used for


a) Impetigo
b) Herpes Zoster
c) Rosacea
d) Pruritus

77. Which of the following is the first sign or symptom of mild dehydration in children?
a) Sunken eyes
b) Dry Skin
c) Irritability
Early signs of mild Dehydration: Dry Mouth, or lips – dry skin – fewer diapers with darker urine – dizziness
Early signs of severe dehydration: Sunken eyes‐ no tears when crying‐ lethargic‐ irritable and confused
78. Mild fever 38, sore throat, runny nose, congestion in the last 3 days
a) Influenza
b) Pharyngitis
c) Allergic rhinitis
d) Common cold
e) Viral Rhinitis

79. Female weight 80 Kg and smoker. This is the 3rd time to take plan B. What would decrease
plan B effectiveness?
a) Use of amoxicillin
b) Weight more than 80 kg
c) Repeated use of plan B
BACKUP PLAN ONESTEP (levonorgestrel tablets) is not recommended for routine use as a contraceptive. The
pregnancy rate of levonorgestrel tablets is calculated for a single use. If levonorgestrel tablets are used on
more than one occasion, the cumulative pregnancy rate will be higher.
Levonorgestrel Q. Act principally by preventing ovulation or fertilization (by altering tubal transport of
Plan B, sperm and/or ova). In addition, it may inhibit implantation (by altering the endometrium). It
Next Choice, is not effective once the process of implantation has begun.
Available in a 1.5 mg PO (2 × 0.75 mg tablets taken together) as soon as possible after unprotected
pharmacy without intercourse (most effective if taken within 72 h). With or without food.
a prescription 0.75 mg PO Q12H × 2 doses is equally effective (2nd dose can be taken up to 24 h after 1st
dose without significant change in pharmacokinetics).
If you vomit within 2 hours of taking the pill(s), contact health-care provider as a repeat dose
may be required.
S.E: Q. Nausea: 14% - 23%, Abdominal pain: 18%, Fatigue & Headache: 17%, Dizziness &
Breast tenderness: 11%, Vomiting: 6%, Diarrhea: 5% & Irregular menstrual bleeding.
If the symptoms persist for more than 48 hours or are severe, see healthcare professional.
C.I: Pregnancy, allergic to it, or to any of its components, abnormal vaginal bleeding.
Griseofulvin, carbamazepine, phenytoin, protease inhibitors, phenobarbital, St. John's wort,
topiramate may decrease levonorgestrel serum concentrations.
For patients taking a hepatic enzyme–inducing medication, it is preferable to take a
nonhormonal EC, e.g., copper IUD.
For women unable or unwilling to use the copper IUD, it is an option to take a total of 3 mg
(2 × 1.5 mg tablets) levonorgestrel as a single dose ASAP after unprotected intercourse.
Women who do not have access to or do not wish to use alternative EC (such as copper
IUDs) should not be discouraged from using plan B, as it may still provide some benefit.
Next menstrual period at the expected time or within 5 days
Q. Health Canada advisory regarding reduced effectiveness in women weighing 75– 80 kg
and lack of effectiveness in women weighing ≥80 kg, but further evidence is required.
Recommend regardless of BMI.
Hormonal contraception can be started within 24 hours of levonorgestrel EC use.
Backup contraception will be needed for the first 7 days of hormonal contraception.

80. Daughter comes with her mother who has Dementia and urinary incontinence, she is taking
Solifenacin, Donepezil, Indapamide and she say her mother cognitive symptom is deteriorated
for past 3 months, what is the cause of worsening symptoms?
a) Solifenacin
b) Normal progression
c) Indapamide
Anticholinergic side effects of medications can lead to cognitive impairment. A few examples of drug classes
commonly associated with anticholinergic effects are:
 Antiemetics/antivertigo agents, e.g., dimenhydrinate, promethazine, scopolamine
 Antihistamines, e.g., diphenhydramine, hydroxyzine
 Antimuscarinics, e.g., darifenacin, fesoterodine, oxybutynin, solifenacin, tolterodine
 Antipsychotics, e.g., chlorpromazine, clozapine, olanzapine
 TCA, e.g., amitriptyline, clomipramine, desipramine, doxepin, imipramine, nortriptyline, paroxetine

81. She came again to your pharmacy and she is excited for dispensing rivastigmine and said, “I
hope that my mother to get back to her normal life”, what to tell her to show empathy
a) I understand your feeling, but this illness is progressive.
b) Any medication of Alzheimer don not stops the deterioration of this disease.
c) Rivastigmine can treat her condition
d) It will delay the progression of the disease
Cholinesterase Inhibitors: benefits are typically small to moderate, and in many patients, can consist mainly
of disease stabilization. Initiated at 1.5 mg BID and increased to the minimum effective dose of 3 mg BID
after 30 days. The maximum dose is 6 mg BID PO.
Sympathy has been defined in the healthcare literature as an emotional reaction of pity toward
the misfortune of another, especially those who are perceived as suffering unfairly.
In contrast, empathy has been defined as an ability to understand and accurately acknowledge the feelings
of another, leading to an attuned response from the observer
82. Case about Parkinson and urinary incontinence, what to do regarding his medication?
a) Decrease dose of Levodopa
b) Decrease dose of Carbidopa
c) Stop Benztropine
It causes urinary retention and worsens urinary incontinence

“Reduce benztropine (although it is an anticholinergic drug, it can cause overflow incontinence”


“Levodopa might also be associated with urinary incontinence but it is the 1st line therapy in PD

83. Patient with Parkinsonism was taking Levodopa/Carbidopa 100/10 and has nausea, what is
recommended?
a) Give Dimenhydrinate
b) Give Levodopa with strange drugs (can’t remember)
c) Give Levo/Carbi 100/25
d) Give with food
Levodopa may be taken with food early in therapy to ease nausea; it may be taken on an empty stomach in
more advanced disease to help manage motor fluctuations. Since levodopa competes with certain amino
acids, the absorption of levodopa may be impaired in some patients on a high protein diet.

84. There was a question on Levodopa switch from IR and to CR and then Levodopa and
protein containing food interaction
 When patients already receiving levodopa switched to SINEMET®, levodopa must be discontinued
for at least 12 hours or more before SINEMET® is started. SINEMET® should be substituted at a
dosage that will provide approximately 20% of previous levodopa dosage.
 When patients are receiving levodopa monotherapy or SINEMET® (levodopa and carbidopa), this
medication must be discontinued at least 8 hours before therapy with SINEMET® CR is started.
Dosage with SINEMET® CR 200/50 should be substituted at an amount that eventually provides
approximately 10 to 30percent more levodopa per day. The interval between doses should be
prolonged by 30 to 50 percent. This is because when Compared to Sinemet immediate‐release
formulation, bioavailability of Sinemet CR is 25– 30% lower and duration of action 25–30% longer.
 Because entacapone enhances the bioavailability and therefore the central effects of levodopa, it
may be necessary to adjust the dosage of levodopa during the initial days to weeks of entacapone
therapy in order to reduce levodopa‐related dopaminergic adverse reactions, e.g., dyskinesias,
nausea, vomiting and hallucinations. In some cases, it may be necessary to reduce the daily dosages
of levodopa by about 10‐30%. This can be achieved through either reducing the dose of levodopa
preparation itself, or by extending the interval between doses, according to the clinical condition of
the patient.
 A change in diet to foods that are high in protein (such as meat, fish, dairy products, seeds and nuts)
may delay the absorption of levodopa and may not work as well as it should.
85. Mirtazapine side effects all Except?
a) Somnolence at Daytime & Abnormal Dreams
b) Anxiety & Alertness (treat anxiety, cause dizziness and sedation- Depression+ insomnia)
c) Increased Appetite & Weight Gain
d) Dry Mouth & Constipation
e) Tremors & Confusion
f) Edema & Dyspnea
Mirtazapine Mirtazapine acts on both the noradrenergic and serotonergic systems (Alpha2 antagonist and
Remeron potent 5HT2 receptor antagonist - Increases release of norepinephrine and serotonins)
S.E: It has a lower rate of GI and sexual side effects but is associated with sedation and weight
gain (increase appetite). Most weight gains.
Not to be used in patients < 18 years old.
Sedative effects may be potentiated by alcohol or benzodiazepines.
QTc prolongation, torsades de pointes have occurred in patients at risk of QTc prolongation,
patients taking concomitant medications that prolong QTc, or in cases of drug overdose.
Taken daily in the evening. Orally disintegrating tablets can be taken without water.

86. Nursing woman her baby is sucking very well, frequent diaper changes every day (5 times),
the nurse told her that the baby regained his birth weight. She came to pharmacy asking for
vitamins for herself. What to do?
a) Give her Fenugreek tablets
b) She does not need any vitamins
c) Give her multivitamins formula

87. A young lady called the pharmacy and told you that she took Rizatriptan 5 mg half an hour
ago and did not feel any improvement yet, what should you tell her?
a) You should take another dose right now
b) You should take another dose
1 hour if the headache is still
bothering you
c) You should take another dose
2 hour if the headache is still
bothering you
d) The medication will not work
for you, you should see a
doctor
e) After 2 hours if no relief
avoid taking 2nd dose in the
same day (2nd dose if
headache returns, another episode)
Initial dose: 5 mg or 10 mg orally, once. Provided there has been a response to first dose, a second dose may
be administered at least 2 hours later if migraine returns
WARNINGS AND PRECAUTIONS
RIZATRIPTAN ODT should only be used where a clear diagnosis of migraine has been established.
For a given attack, if a patient has no response to the first dose of rizatriptan, the diagnosis of migraine
should be reconsidered before administration of a second dose.
The recommended single adult dose is 5 mg. The maximum recommended single dose is 10 mg.
There is evidence that the 10 mg dose may provide a greater effect than the 5 mg dose
For RIZATRIPTAN ODT administration with liquid is not necessary. Orally disintegrating tablet is packaged in a
blister package. Patients should be instructed not to remove the tablets from the blister package until just prior
to dosing. The tablet should be pushed through the blister with dry hands and the orally disintegrating tablet
placed on the tongue, where it will dissolve and be swallowed with the saliva. Doses should be separated by at
least 2 hours; no more than a total of 20 mg should be taken in any 24‐hour period.
Patients Receiving Propranolol: A single 5 mg dose of RIZATRIPTAN ODT should be used. In no instances
should the total daily dose exceed 10 mg per day, given in two doses, separated by at least two hours

88. Refer Parkinson patient with worsening


speech to?
a) Psychiatric
b) Neurologist
c) Speech specialist

89. Patient consumes the pharmacist time, he needs a long time to remember what he wants to
say due to cognition problem, how to handle the situation?
a) Recommend social worker
b) Recommend speech problem solving
c) Recommend caregiver
d) Recommend occupational therapist

90. A patient will have a Cataract surgery in one eye, after 2 weeks and the other one will be
after 8 weeks of the first eye. Physician prescribed him eye drops / antibiotic, cortisone and
another one (analgesic), why he needs to use this combination?
a) Decrease detachment of lens
b) Decrease risk of infection
c) Prevent the inflammation
d) Reduce the IOP
The goals of the perioperative prophylactic pharmacologic treatment are: Control inflammation, prevent
infection, maintain eye comfort and Promote early visual rehabilitation. The goals of the postoperative
assessment are: Detect intraocular infection in its early stages, detect postoperative uveitis or intraocular
pressure (IOP) elevation and detect other abnormalities in the postoperative course.
91. What should the pharmacist advice the above patient on use of these drops?
a) Shake well
b) Put finger on nasal duct to prevent systemic absorption
c) Put them in the same sequence daily  Wait 3‐5 minutes between drops.

92. After 8 weeks she comes back and says the doctor wrote for her the same 3 drops because
she need to do a surgery for her other eye ,but she thinks she still has the previous eye drops in
home so no need to get new eye drops, what you have to say?
a) Agree for her and let her use the rest that she has
b) Just discard two and use the third one
c) Discard old one and fill the new Rx. (more than 4 weeks)

93. Which of the following is used prospectively to prevent a medication error?


a) Failure Mode and Effect Analysis (FMEA)
b) Root cause Analysis (RCA)
c) Medication Incident report
Root cause analysis RCA
 A systematic approach aimed at discovering the causes of close calls and adverse events for the
purpose of identifying preventative measures.
 RCA teams look beyond human error to identify system issues that contributed to or resulted in the
close call or adverse event. I.e: Used for retrospective study. The goal is to answer what happened,
why did it happen, and what can be done to prevent it from happening again?
 It includes document reviews and interviews with the parties involved in the event.
 Flow diagramming, cause and effect diagramming, and identifying root causes and contributing factors
help to organize the events and determine why an error occurred.
 Based on root causes & contributing factors, actions can be developed to prevent errors from recurring
 Measuring the outcome of an intervention is also planned in order to determine success of the RCA.
 Tools to assist the team include triggering questions, the five rules of causation, and action hierarchy
Failure mode and effects analysis (FMEA)
 A risk management tool to proactively identify and assess the causes and effects of potential failures in
a system, thereby preventing them from happening.
Used for prospective study. The objective of this study was to evaluate effectiveness of FMEA applied
to an academic clinical trial center in a tertiary care setting.

94. The doctor wants to prescribe cortisone drops for a new indication sarcoidosis (off label),
and he wants to know if the effectiveness of corticosteroids in the treatment od sarcoidosis. So,
what to write in the online database search?
a) Corticosteroid – sarcoidosis
b) Ophthalmic drop – sarcoidosis
c) Sarcoidosis – effective drops
Evaluating Internet information: To select information from internet few key criteria to consider.
 Authorship; Who and their credential
 Referencing; Credible references like primary
 Disclosure; Is there any potential conflict of interest on the part of author? (Conflict of interest, publication
bias, research funding sources, research ethics (pregnancy, children, placebo not included & chemo drugs).
 Currency. What is the date of last revision? Start with tertiary.
Search Techniques: Boolean ("advanced") versus non‐Boolean search
 Boolean = supports the use of "AND". OR", "NOT", etc.
 Non‐Boolean = ONLY one keyword could be used; e.g., e‐CPS

95. Source for comparison between PPI?


a) Rx files
b) CPS
c) Medline

96. A patient on opioid withdrawal therapy who uses fentanyl patch, why should we ask the
patient to remove the patch before going in for an MRI scan
a) Some patches contain metal and can cause skin burn
b) There would be increased absorption of fentanyl from the patch causing an overdose
c) Will affect the magnetic fields & the picture
d) The magnetic field will decrease the drug effectiveness
Some transdermal patches containing aluminum or other metal in their nonadhesive backing shouldn’t be
worn during MRI because of skin burn risk. Health care professionals should advise patients wearing
medication patches about procedures for proper removal and disposal before MRI & replacement afterward
Patch Comment
Clonidine Remove before MRI
Catapres-TTS Reapply same patch to another site afterwards
Diclofenac Remove before MRI. Reapply same patch afterward if <2 hours have elapsed since
Flector patch was removed. Apply new patch if longer
Estradiol Remove before MRI
Testosterone For reapplication after the procedure, follow same instructions as for patch falling off
Lidocaine and Remove before MRI
Tetracaine
Fentanyl Remove before MRI
Duragesic After procedure, apply a new patch to an alternate site
Methyl Salicylate OTC product. Remove before MRI
Menthol Salonpas Replace with fresh patch after procedure
Nicotine To avoid confusion, recommend removing patch, regardless clear or opaque, before
MRI. Reapply same patch after procedure
Oxybutynin Removal before MRI is recommended because manufacturer can’t guarantee product
Selegiline doesn’t contain metal
Rivastigmine Consider removal before MRI
Patch hasn’t been studied in patients undergoing MRI
Rotigotine Remove before MRI or cardioversion
Scopolamine Remove before MRI. Place new patch on an alternate site following procedure

97. Where exactly to inject vaccine in


deltoid muscle?
a) Clavicle Apex 4 cm
b) Clavicle Apex 2 cm
c) Axilla
d) 4 cm below Acromion process

98. 6 years boy + daily bed witting, what is your advice


a) Restrict daytime fluids & recommend Regular exercise
b) Parent support and monitoring (non-pharm)
c) Refer to urologist to give medical not for assistance
d) Enuresis alarm (non-pharacological)
e) It is normal for his age
Enuresis alarms: may be tried in motivated and committed patients as young as 5 years of age with
consistent parental involvement and support. Best results may be achieved in children ≥7 years of age when
used properly for 3–4 months.

99. What test to rule out infection?


Urine analysis

100. Diagnosed with enuresis, what to


give?
a) Desmopressin
b) Oxybutynin
c) Amitriptyline

101. A vaccine vial with a residual amount of AL(OH)3, AL(SO4)3, what is their function?
a) To increase the body’s immune response to vaccines (immunogenicity enhancer)
b) To inactivate viruses and toxins  formaldehyde
c) To protect them against freeze-drying or heat  gelatin
d) Antioxidant
e) Preservative
102. A lady came to your pharmacy with her 2-month-old child. Her child was having Diaper
dermatitis. What is your recommendation?
a) Use cotton cloth diapers instead of commercial diaper
b) Allow to air dry properly between changing the diapers
c) Use antifungal (complicated)
d) Use talc powder frequently
e) Increase frequency of washing
Avoid, cornstarch, also no alcoh. Wipes, acid ph cleanser

103. An 18 months old boy, who recently started attending daycare, mother brought to your
pharmacy with an unfilled prescription of recurrent otitis media in the last two months. What
factors that increased his risk of getting otitis media?
a) Age
b) Daycare attending
c) Frequent use of antibiotics

104. What is appropriate therapy


for this child?
a) Low dose amoxicillin for 10
days
b) High dose amoxicillin for
10 days
c) Amox/Clav for 5 days
d) Azithromycin for 3 days

105. Hypertension may affect the


below organs damage EXCEPT
a) Retinopathy
b) Ventricular hypertrophy.
c) Cerebrovascular accident.
d) Nephropathy
e) Sexual dysfunction
f) Neuropathy  only happens only if accompanied with diabetes
Elevated BP leads to cardiovascular and renal complications. Important sequelae are stroke, myocardial
infarction, atrial fibrillation, heart failure, chronic kidney disease and dementia. No pathologic changes
occur early in hypertension. Severe or prolonged hypertension damages target organs (primarily the
cardiovascular system, brain, and kidneys), increasing risk of Coronary artery disease (CAD) and myocardial
infarction (MI), Heart failure, Stroke (particularly hemorrhagic), Renal failure
106. A patient was on methadone maintenance program for addiction treatment did not take her
dose, how many days until we see withdrawal symptoms?
a) 1 day
b) 2 days
c) 3 days
d) 14 hrs
Methadone substitution is the preferred method of managing opioid withdrawal for more seriously addicted
patients because at appropriate doses, it has a long half‐life and less profound sedation and euphoria. Any
physician can initiate methadone substitution during hospitalization or for 3 days in an outpatient setting,
but further treatment is continued in a licensed methadone treatment program. Methadone is given orally
in the smallest amount that prevents severe but not necessarily all symptoms of withdrawal. Typical dose
range is 15 to 30 mg once/day; doses ≥ 25 mg can result in dangerous levels of sedation in patients who
have not developed tolerance.
Symptom scales are available for estimating the appropriate dose. Higher doses should be given when
evidence of withdrawal is observed. After the appropriate dose has been established, it should be reduced
progressively by 10 to 20%/day unless the decision is made to continue the drug at a stable dose
(methadone maintenance). During tapering of the drug, patients commonly become anxious and request
more of the drug.
Methadone withdrawal for addicts who have been in a methadone maintenance program may be
particularly difficult because their dose of methadone may be as high as 100 mg once/day; in these patients,
the dose should be gradually reduced to 60 mg once/day over several weeks before attempting complete
detoxification.
Methadone has been reported to be associated with QTc prolongation and serious arrhythmias including
torsades de pointes (see also Long QT Syndrome and Torsades de Pointes Ventricular Tachycardia). Thus, it
should be used very carefully with appropriate patient evaluation and monitoring during initiation and dose
titration
Methadone Withdrawal Timeline
Days 1‐2 Symptoms of withdrawal usually don’t begin until at least 30 hours after user’s last dose of methadone, and
it may take even longer depending on the amount used. Physical symptoms, such as chills, fever, rapid
heartbeat, and muscle aches, will begin during this time.
Days 3‐8 Over the next week or so, methadone cravings will be strong. Users may face anxiety, body aches and pains,
nausea, and insomnia, as well as irritability and anxiety. Flu‐like physical symptoms will persist. Due to how
long methadone stays in the body, it often takes between 3 and 8 days for symptoms to peak. At this point,
additional symptoms such as depression, vomiting, and cramps begin to appear.
Days After withdrawal peaks, symptoms will begin to subside, although some will remain such as irritability,
9‐15 diarrhea, and physical discomfort. Users may still feel strong drug cravings and depression may set in.
Depression can become severe and some patients may have difficulty feeling pleasure or getting motivated.
Days 15+ Withdrawal symptoms from methadone such as low energy levels, anxiety, trouble sleeping, and cravings
typically persist for 2‐3 weeks. After the 3‐6‐week detox process is over, many former methadone users will
experience post‐acute withdrawal symptoms, or PAWS. PAWS may continue for many months, and in some
cases for up to 2 years. PAWS may include irritability, anxiety, depression, the inability to feel pleasure,
difficulty sleeping, and poor concentration.
107. What side effects should we notice or opioid withdrawal symptoms?
a) Miosis
b) Diarrhea
withdrawal of an opioid analgesic includes body aches, diarrhea, gooseflesh, loss of appetite, nausea,
nervousness or restlessness, anxiety, runny nose, sneezing, tremors or shivering, stomach cramps,
tachycardia, trouble with sleeping, unusual increase in sweating, palpitations, unexplained fever, weakness
and yawning.

108. Elderly Japanese patient and his wife they are newcomers and know very little English,
what can you do to explain for them some info for a new prescription?
a) Speak loudly and tell them counseling points in clear English terms
b) Print medical information sheet in English/French and highlight the important info
c) Speak 1 to 1 and focus only on the patient till agreement
d) Speak slowly in clear simple English terms and show them some figures
e) Speak slowly and let them to repeat what he said
Tips for interacting with  Speak slowly and simply
the patient who does not  Use an interpreter, e.g., caregiver
speak your language:

109. Patient has Asthma and stable angina is


stable on Symbicort- maximum dose
(Budesonide and formoterol) & Bisoprolol 5
mgs and Atorvastatin 20 mg, he is bothered
by chest pain and tightness after exertion,
dizzy & cough. What is the reason?
a) He does not take Salbutamol
b) His Angina is not well controlled

110. What do you recommend?


a) Recommend him Ventolin PRN
b) Recommend him to stop doing exercise
c) Recommend sublingual nitroglycerin

111. Empagliflozin adverse effects?


Constipation, dry mouth, nausea, dry mouth,
Gastroenteritis, Weight decreased, sinusitis &
influenza. Increased risk of genitourinary infections;
Vulvovaginal candidiasis & pruritus
Reduced intravascular volume resulting in
hypotension & hyperkalemia. Hyperlipidemia & dyslipidemia
112. Diabetic patient what to
counsel regarding his foot?
a) Inspect feet daily with a
mirror
b) Wash feet daily with hot
water to clean it
c) Wear large shoes to increase
blood flow to the legs
d) Wear compression stockings

113. Which of these medications should be stored in the fridge?


a) Travoprost eye drops  Store at 2 ‐ 25°C. No refrigeration required.
b) Fludrocortisone tablets
c) Fragmin (Dalteparin) SC injections  LMWH stored at room temperature (15–30°C).
d) Gravol (Diminhydrinate) IM  store at room temperature (15°C – 30°C). Protect from freezing.
e) Medroxyprogestrone acetate  Store at controlled room temperature (15‐30°C).
https://www.medsafe.govt.nz/Consumers/CMI/f/Florinef.pdf
Florinef should be stored in a refrigerator between 2‐8°C. Alternatively, and for a period of 3 months only,
Florinef can also be stored in a cool dry place where the temperature stays below 25⁰C. After this time,
discard all Florinef tablets that have not been refrigerated

114. Which of these medications require sales report?


a) Lorazepam
b) Phenobarbital  no need if it was in preparation
c) Phenytoin
d) Methylphenidate
115. There was a question about a 10 years old boy diagnosed with ADHD and started on
Concerta. The patients ADHD symptoms is well controlled with the titrated dose of Concerta;
however, the father came into the pharmacy to complain that the son is losing weight and has
been refusing his food due to lack of appetite for 2- 3 weeks now. What do you recommend?
a) Switch to atomoxetine
b) Titrate to lower dose of Concerta
c) Switch to a different stimulant
d) Add appetite stimulating supplements in the morning to help improve the child’s
appetite (not exact words but that’s what it implies) or Give high caloric breakfast
Another version:
ADHD case: Mother is asking you for a recommendation, her son started ADHD medication
Concerta 28 mg one month ago and the teacher in school noticed improvement in his learning
and attention, the physician increases the dose to 64 mg and the weight of the child decreased
because he lost the appetite. What do you recommend??
a) Decrease the dose
b) Switch to IR
c) Increase intervals
d) Discontinue medication
e) Give high caloric breakfast
Common, usually transient—continue therapeutic trial: anorexia, insomnia, weight loss, irritability,
dizziness, weepiness, headache, abdominal pain.
Transient—stop and re‐evaluate: “zombie‐like” effects, psychotic reactions (such as hallucinations),
agitation, tachycardia, hypertension, growth failure, rebound hyperactivity, leukopenia, blood dyscrasias.
Monitor patient for suicidal thoughts/ideation; consider a change in treatment if concerns arise.
Overdose symptoms—stop and retitrate: “glassy eyes,” insomnia, hyperactivity.
Significant: sudden cardiac death reported; neurologic symptoms; exacerbation of tics; avoid in patients
with a history of CV conduction disturbances, hypertension, acute psychotic episodes and hyperthyroidism.
If seizures occur, or if frequency increases in patient with controlled epilepsy, stop and re‐evaluate.
Drug holidays (at times of low environmental stresses) may be useful when adverse effects (such as growth
suppression or weight loss >10% of initial body weight) have occurred or when attempting to assess the
continued benefit.
Management of Common Adverse Effects of ADHD Treatment
Adverse Effect Monitoring Management
Appetite suppression Monitor for consistent appetite Maximize nutrition content when
Stimulants, suppression and changes in weight Q patient is not having symptoms of
atomoxetine, 2 wk for the first 2 months, then Q 6 appetite suppression (e.g., evenings,
bupropion months. before the morning dose of stimulant).
In children and adolescents, monitor Reduce portions & ↑ snacking times.
height as well. Consider nutritional meal supplements.
Consider drug holidays on weekends or
during vacations
Cardiovascular (HR, BP) Monitor BP and HR in the first 2 wk They should always be tapered; if
Stimulants  ↑ HR/BP. of starting a stimulant medication, stopped abruptly, they may cause a
Alpha-2 agonists  ↓ then Q 3 months. hypertensive crisis.
HR/BP. ECG not needed routinely if no If significant changes occur in BP, HR
TCAs  tachycardia. previous history of CV disorder. or ECG, discontinue and consider
consulting a cardiologist.
Psychiatric (anxiety, Monitor for difficulties falling asleep, Often worse upon initiation and
irritability, insomnia, staying asleep and/or early morning resolves after 1–2 wk of therapy.
tics) awakenings at 1 wk, then monthly for May need to lower the stimulant dose,
Stimulants, atomoxetine, the first 3 months, then Q 6 months. change time to earlier administration,
bupropion, venlafaxine Caregiver may use Sleep Disturbance add a more sedating medication at
Scale for Children or the Children’s bedtime or discontinue the offending
Sleep Habits Questionnaire to monitor stimulant. Minimize use of caffeine and
at home. other psychostimulants.
Limit stimulating activities (e.g., use of
electronic devices) in the evenings.
Overdosage Acute overdosage include: Dilated pupils, shallow rapid respiration, fever,
hyperpyrexia, chills, rhabdomyolysis, sweating, hyperactive tendon reflexes.
Benzodiazepines are first-line agents in amphetamine overdose for agitation,
movement disorders, seizures, tachycardia, and hypertension.
2nd line includes antipsychotics as chlorpromazine, ziprasidone or haloperidol.

116. CAROC score estimates:


a) Risk of fracture in the next 5 years
b) Risk of fracture in the next 10 years
Osteoporosis Canada recommends calculating an estimated 10‐year risk of osteoporotic fracture in women,
which is derived by combining age and epidemiologic data with DXABMD measurements. Two fracture risk
calculators have been developed, validated and are now recommended as an aid to osteoporosis
management for the Canadian population: FRAX (Fracture Risk Assessment Tool) and CAROC (Canadian
Association of Radiologists and Osteoporosis Canada). Estimated risk is reported as low (<10%), moderate
(10–20%) or high (>20%). Risk categories are intended only for assessment of the as‐yet‐untreated patient

117. Calculation about eye drops Q22 Marian’s file but numbers were different
A physician prescribed a fortified eye drops for a patient Tobramycin 13.5mg /ml gtt ou twice
daily. How much of the 40 mg/ml stock solution should be added to 5ml of 0.3% to get the
desired concentration?
Answer: 0.3 % means 0.3 gm -------- 100 ml X gm ------- 1 ml
X=1*0.3/100=0.003 gm = 3mg So 0.3%=3mg/ml
By allegation method:

X = 5*10.5/26.5 = 1.98 ml
118. If a doctor wants to make a lab test for a rare disease for the patients in his clinic, this lab
test is done in the hospitals. As a pharmacist you will advise him that, the number of the
patients in your clinic is low and this will affect on?
a) PPV (positive predictive values)
b) Sensitivity
c) Specificity
Sensitivity = Tp/real number of patients.
What are sensitivity and specificity?
Sensitivity and specificity are measures of a test's ability to correctly classify a person as having a disease or
not having a disease.
Sensitivity refers to a test's ability to designate an individual with disease as positive. A highly sensitive test
means that there are few false negative results, and thus fewer cases of disease are missed.
The specificity of a test is its ability to designate an individual who does not have a disease as negative. A
highly specific test means that there are few false positive results. It may not be feasible to use a test with low
specificity for screening, since many people without the disease will screen positive, and potentially receive
unnecessary diagnostic procedures.
It is desirable to have a test that is both highly sensitive and highly specific. This is frequently not possible.
Typically, there is a trade-off. For many clinical tests, there are some people who are clearly normal, some
clearly abnormal, and some that fall into the gray area between the two. Choices must be made in establishing
the test criteria for positive and negative results.
What is predictive value?
The probability of having the disease, given the results of a test, is called the predictive value of the test.
Positive predictive value is the probability that a patient with a positive (abnormal) test result actually has the
disease.
Negative predictive value is the probability that a person with a negative (normal) test result is truly free of
disease. Predictive value is an answer to the question: If my patient's test result is positive, what are the chances
that my patient does have the disease?
Predictive value is determined by the sensitivity and specificity of the test and the prevalence of disease in the
population being tested.
Prevalence is defined as the proportion of persons in a defined population at a given point in time with the
condition in question. The more sensitive a test, the less likely an individual with a negative test will have the
disease and thus the greater the negative predictive value. The more specific the test, the less likely an
individual with a positive test will be free from disease and the greater the positive predictive value.
When the prevalence of preclinical disease is low, the positive predictive value will also be low, even using a
test with high sensitivity and specificity. For such rare diseases, a large proportion of those with positive
screening tests will inevitably be found not to have the disease upon further diagnostic testing. To increase the
positive predictive value of a screening test, a program could target the screening test to those at high risk of
developing the disease, based on considerations such as demographic factors, medical history or occupation.
For example, mammograms are recommended for women over the age of forty, because that is a population
with a higher prevalence of breast cancer.
https://www.health.ny.gov/diseases/chronic/discreen.htm
119. Pt 19 years old, starts to feel pain after
the onset of her menstrual period – pain is
7/10, What do you recommend for pain?
a) Hot pad or bottle over the stomach
b) Vitamin B6
c) Naproxen
d) Acetaminophen

120. Patient with DM, MI, HF and has a stent, how will update
and recommend his vaccines, which vaccine he does not need?
a) Pneumococcal
b) Influenza
c) Diphtheria toxoid
d) Tetanus
(if age below 50, exclude shingles)
Routine vaccines in nosocomial infections risk in patients with chronic
diseases: Flu‐ Pneumonia‐ Tdap (Tetanus‐Diphtheria‐ Pertussis)‐ Hep. B
IF THE ADULT IS older THAN 50 YEARS ADD: Shingles vaccine and HAV
may also be considered
Diphtheria is a bacterial infection that can cause myocarditis, damage to the heart muscle. It is important
for patients with heart disease to receive the Tdap vaccine, as diphtheria is one of the components covered
in the immunization.
Encourage patients 50 and older to receive the 2‐dose series of zoster vaccine recombinant, adjuvanted
(Shingrix). The second dose should be administered 2 to 6 months after the first. With the Shingrix shortage,
pharmacists can help ensure that patients receive it through waiting lists and using the CDC’s vaccine finder
to locate a pharmacy that has it in stock.
STRATEGIES TO INCREASE IMMUNIZATION RATES
When individuals are picking up their medications, pharmacists should remind them to get these vaccines
and dispel common myths. Empower patients to take an active role in their health by staying up‐to‐date on
their vaccines. Set up reminder patient phone calls, especially for vaccines such as Shingrix, to ensure that
the second dose is administered. The 4 Pillars Program has been associated with significant increases in
vaccination of high‐risk adults.
121. Tobramycin IV 400 mg/24 hr was given to a patient, Peak concentration is required >
20mg & trough conc. required is < 0.5 mg. post dose peak was 28 mg/L. & after 10 hr. was 7
mg/L. find the T1/2 & the correct dose.
Answer:
I give him 400 mg in order to give me 20 but it gives me 28 but so, it means the dose is not correct i need the
correct dose to give me conc 20 mg
Log C = Log Co ‐ (k*t /2.303) Log 7 = Log 28 ‐ (10K /2.303) k = 0.14
T1/2= 0.693 / K = 0.693 / 0.14 = 4.99 hr (5 hr.)
400 mg / 24 hr ‐‐‐‐‐‐‐ 28 mg/ L X mg / 24 hr ‐‐‐‐‐‐‐ 20 mg / L X = 20*400 / 28 = 285.7 mg
285.7 mg ‐‐‐‐‐‐‐ 24 hr 400 mg ‐‐‐‐‐‐‐ Y Y = 400*24/285.7 = 33.6 hr.
The correct dose is 400 mg/34 hours

122. A lifeguard who worked 5 years outside in the sun, got lesions in his back, irregular edges,
not defined and mottled. which one can be developed?
a) Erythema
b) Edema
c) Hyperpigmentation
d) Melanoma skin cancer (malignant melanoma)
Melanoma Skin Cancer appears to be related to intense and intermittent sun exposure in childhood and
adolescence. It is the rarest type of cancer, but is responsible for the majority of skin cancer deaths. Risk is
increased in blond or red‐headed individuals who have skin that tans poorly and burns easily, those with a
large number of moles, chronic exposure to the sun or those with a past history of sunburns as a child. It
appears as a flat brown or black spot (commonly in a mole or other dark spot) with irregular edges that can
grow larger if left untreated

123. Patient taking opioid (don’t remember name) and


the doctor needs to change it to Fentanyl 50 mg, what
is the pharmacist concern?
a) Starts Fentanyl with high dose so, call the
doctor to change it to Fentanyl 25 mg (not high
dose)
FENTORA is indicated only for the management of breakthrough pain in cancer patients 18 years of age and
older who are already receiving and who are tolerant to continuous opioid therapy for their persistent
baseline cancer pain.
Patients considered opioid tolerant are those who are taking at least 60 mg of oral morphine daily or an
equianalgesic dose of another opioid daily for a week or longer.
All patients starting treatment with FENTORA must begin with titration from the 100‐mcg dose.
This product must not be used in opioid non‐tolerant patients because life‐threatening respiratory
depression and death could occur at any dose in patients not on a chronic regimen of opioids.
For this reason, FENTORA is contraindicated in the management of acute or post‐operative pain, including
headache/migraine, dental pain or use in the emergency room.
Note: FENTORA is contraindicated in all post‐operative pain, including post‐operative cancer pain if
the patient is not already opioid tolerant. The addition of the qualifier “non cancer” may be confusing
as it could be interpreted to mean that FENTORA can be used for post‐operative pain after surgery for
cancer or post‐operatively for cancer pain, both of which can occur in opioid non‐tolerant patients.
The term “post‐operative” already implies that the pain is due to surgery and not to cancer.
FENTORA is intended to be used only in the care of opioid tolerant cancer patients and only by healthcare
professionals who are knowledgeable of and skilled in the use of opioids to treat cancer pain.

124. An old patient in Long care unit with many diseases, Dyslipidemia, Hypothyriodism &
STEMI, also he did PCI. His medications profile includes Rosuvastatin 20 mg at bed time,
Levothyroxine at morning, Ca carbonate at lunch, Clonazepam & ramipril. The physician called
you and asked you to helping her to take her medication, what to do?
a) Dispense his medications monthly
b) Offer him blister-packs for his medication.

125. If his LDL is 3.3, What is the pharmacist recommendation?


a) Increase statin dose to 40 mg
LDL target for him is < 2. So, increase statin dose to achieve this target
HMG‐CoA Reductase Inhibitors (Statins)
Starting a statin early after MI decreases both early and late adverse events. Guidelines suggest
documenting the cholesterol profile within 24 hours of infarction, as LDL cholesterol may be falsely low for
up to 4 weeks postinfarction. Evidence suggests that intensive treatment with higher‐dose statins (e.g.,
atorvastatin 80 mg/day or rosuvastatin 40 mg/day) early after MI confers additional benefits compared
with moderate dosages. The benefit of statins extends beyond lowering of LDL cholesterol levels alone as
they contribute to plaque stabilization by preventing future acute coronary syndromes.
Indefinite continuation of statin therapy is integral to the prevention of ischemic events. Following
MI, all patients are considered to be in the high‐risk group for subsequent events.
Current Canadian Cardiovascular Society guidelines advocate a minimum target LDL‐C of <2 mmol/L.
Muscle‐related intolerance may be confirmed in up to 6% of patients.
In patients intolerant of statins, alternative cholesterol‐lowering agents such as niacin or ezetimibe may be
considered. For patients not achieving target levels despite maximal statin therapy or for those intolerant of
higher statin doses, attempt combination therapy with a statin and extendedrelease niacin.

126. Then he diagnosed with anemia. When should he take Iron tablet?
a) At Breakfast (4 hours apart from
levothyroxine)
b) Before Lunch
c) After Dinner
d) Bedtime on empty stomach (separate
administration by 2 hours)
Ferrous sulfate likely forms a ferric‐thyroxine complex. Administer levothyroxine at least four (4) hours apart
from these agents. (levothyroxine)
Absorption of nonheme iron is optimal in the fasting state, i.e., 1 hour before or 2 hours after food. To
lessen gastrointestinal intolerance, lower doses may be administered initially, and the medication may
be given with or after meals.

127. What should be added to his drugs? Add Aspirin


ASA significantly reduces mortality in patients with STEMI.
If patients are not already receiving ASA, it should be initiated promptly and continued long term.
CTC: ASA is the antiplatelet agent of choice for long‐term secondary prevention due to its effectiveness and
low cost. Indefinite therapy is indicated in all postinfarct patients; low‐dose therapy (75–100 mg) is as
effective as higher doses (300–325 mg). Low‐dose ASA is generally, better tolerated, with lower bleeding
risk. ASA is used in combination with a P2Y12 inhibitor when indicated.
Clopidogrel: The thienopyridine clopidogrel continues to be commonly used after PCI. In MI patients
receiving clopidogrel after PCI, a dose of 150 mg daily for the first 6 days has been shown to be superior and
should be considered. A daily dose of 75 mg should be used after the initial 6 days of high‐dose therapy or
for those patients deemed not to require the initial highdose therapy. For patients with ASA allergy, chronic
clopidogrel therapy may be a substitute.

128. If patient with hypothyroidism, what will be his complain?


a) Cold intolerance
b) Weight loss
c) Diarrhea
d) Oily skin

129. Patient with Dementia, which drug in his regimen will help?
a) Rosuvastatin
b) Levothyroxine
c) Ca carbonate
d) Clonazepam
Dementia also occurs in patients with Parkinson disease, Huntington disease, progressive supranuclear
palsy, Creutzfeldt‐Jakob disease, Gerstmann‐Sträussler‐Scheinker syndrome, other prion disorders,
neurosyphilis, a traumatic brain injury (eg, chronic traumatic encephalopathy), or certain brain tumors located
in cortical or subcortical brain areas involved in cognition. Patients can have > 1 type (mixed dementia).
Some structural brain disorders (eg, normal‐pressure hydrocephalus, subdural hematoma), metabolic
disorders (eg, hypothyroidism, vitamin B12 deficiency), and toxins (eg, lead) cause a slow deterioration of
cognition that may resolve with treatment. This impairment is sometimes called reversible dementia, but
some experts restrict the term dementia to irreversible cognitive deterioration.
Depression may mimic dementia (and was formerly called pseudodementia); the 2 disorders often coexist.
However, depression may be the first manifestation of dementia.
CTC: Using HMG‐CoA reductase inhibitors (statins) solely for the prevention of dementia remains
controversial. Some systematic reviews have shown that statins may have a preventive role, especially in
those at higher risk of dementia, such as carriers of the ApoE4 genotype (which predisposes individuals to a
rise in amyloid‐beta peptide); however, a Cochrane review remains more skeptical.
Other medications including statins, anti‐inflammatory drugs and Ginkgo biloba cannot be recommended
either for the treatment or prevention of Alzheimer disease due to lack of evidence.

130. Which drug in the patient’s medication can put him at risk?
a) Rosuvastatin for diarrhea (constipation)
b) Levothyroxine for sedation (insomnia)
c) Ca carbonate for osteoporosis (Ca supplement)
d) Clonazepam for sleep apnea

131. Patient is taking Clozapine, his daughter came and complained that, her father does not
remember anything, then the pharmacist has contacted his association to discontinue the drug,
who violate the autonomy of the patient?
a) Discussing with a doctor to stop the drug
b) Discussing with a pt to stop the drug
c) Stopping the drug without informing the patient and his daughter
d) Pharmacist did not ask his daughter for permission
The pharmacist when he makes any treatment decisions away from the patient.

132. Which one violates Justice?


a) Give a medicine to your relative while you know it is in shortage
b) Give a medicine to your relative with a discount while other customers not.

133. Which of the following means there is a violation for fidelity?


a) Choose the patient with easy profile to make med review
b) Put condoms behind the counter which annoying for patient
The ministry pays pharmacies a fee to provide MedsCheck medication review services to patients who are
taking three or more prescription medications for a chronic condition(s).
The medication review must be conducted by the community pharmacist as a one‐on‐one in‐person
interview between the patient and the pharmacist in order to qualify for payment through the Health
Network System (HNS). The interview setting must ensure patient privacy.
Members only provide pharmacy care and services that are of good quality and intended to optimize the
patient’s health outcomes and do not compromise patient care for corporate or business interests or
financial gain.

134. Waiting for the patient to give consent for clinical trials to be done on him, he accepts but
asked about consequences, if you told him so you uphold:
a) Non-Maleficence
b) Altruism
c) Fidelity
d) Veracity

135. Symptoms of Dextroamphetamine withdrawal


a) Hallucination (intoxication)
b) Depression
c) Tremor
d) Xerostomia
Withdrawal symptoms: Fatigue, lack of energy, generalized aches, craving, anhedonia, sleepiness
(somnolence), vivid dreams, anxiety or depression, increased appetite, poor memory
Monograph: Dextroamphetamine sulfate causes a lessening of fatigue, an increase in mental activity, an
elevation of mood, and a general feeling of well‐being. However, its indiscriminate use in attempts to
increase capacity for work or to overcome fatigue is undesirable. At high doses, it produces a euphoria,
which upon abrupt withdrawal of the drug reverts to severe depression and lethargy.
No specific treatment is needed when patients stop taking of amphetamines. Blood pressure and mood
should be monitored initially. Patients whose depression persists for more than a brief period after
amphetamines are stopped may respond to antidepressants. Cognitive‐behavioral therapy (a form of
psychotherapy) is effective in some patients. There are no other proven treatments for rehabilitation and
maintenance after detoxification.

136. Symptoms of Tramadol withdrawal? (opioids withdrawal symptoms)


a) Miosis
b) Seizure
Tramadol Monograph: Physical dependence with or without psychological dependence tends to occur with
chronic administration of opioids, including ULTRAM. Withdrawal (abstinence) symptoms may occur
following abrupt discontinuation of therapy. These symptoms may include body aches, diarrhea, gooseflesh,
loss of appetite, nausea, nervousness or restlessness, runny nose, sneezing, tremors or shivering, stomach
cramps, tachycardia, trouble with sleeping, unusual increase in sweating, palpitations, unexplained fever,
weakness and yawning. Do not stop use of ULTRAM abruptly
137. What is drug of choice for peripheral neuropathic pain?
a) Amitriptyline (No Pregabalin)
b) NSAIDS
c) Opioids
d) Acetaminophen
Chronic Q. First-Line Treatment:
Peripheral  TCAs: amitriptyline (more sedating than other TCAs; preferable option if insomnia is
Neuropathic an issue), desipramine, nortriptyline.
Pain  Gabapentinoids: gabapentin, pregabalin (BID dosing is advantage over gabapentin)
Includes:  SNRIs (Venlafaxine, Doluxetine preferred SNRI due to more evidence of efficacy)
post-herpetic Tramadol and topical lidocaine (Not to be used with occlusive covering. Most effective in
neuralgia, postherpetic neuralgia) considered as second line. Combination therapy can be helpful.
diabetic Topical treatments may be helpful as adjunctive therapy combined with an oral analgesic, or as
neuropathy, monotherapy for elderly patients if oral agents are not tolerated.
causalgia, Other sustained release opioids such as morphine, hydromorphone, oxycodone and transdermal
incisional fentanyl can be used third line.
neuralgias Do not use opioids in children or opioid-nave individuals because of the risk of respiratory
following depression.
mastectomy, Other drugs including cannabinoids (e.g., dronabinol (Marijuana), tetrahydrocannabinol /
thoracotomy, cannabidiol (CBD oil)), methadone, tapentadol, lamotrigine, topiramate and botulinum toxin
or bypass. injections by a specialist. Cannabidiol act on CB1 and CB2 receptors.

138. To consider 2 drugs being interchangeable, they should be equal in:


a) Cmax
b) Tmax
c) VD
d) AUC
e) Absolute bioavailability
AUC >> Cmax >>> Tmax (only when onset is important such as pain killers or sedative hypnotics)
Drug Interchangeability, must meet the following requirements:
 Same amount of same active ingredient
 Same clinical significance formulation characteristics
 Comparable pharmacokinetics
 To be administered in the same way as the drug prescribed

139. Patient is taking Spironolactone for HTN and shortage happened, what alternative to give?
a) Amiloride (for hypertension)
b) Eplerenone (for heart failure)
c) Metolazone
d) Furosemide
Spironolactone is a potassium sparing diuretic which is also an aldosterone antagonist with anti‐androgen
activities. In any case, the common uses of spironolactone include: Treatment of Primary Aldosteronism,
Resistant Hypertension, CHF, Ascites in Adults with Cirrhosis & Hirsutism or Hair Loss in Female
In patients who are taking spironolactone for heart failure or ascites with cirrhosis, one may need to replace
with a potassium sparing diuretic that also antagonizes aldosterone. In these situations, eplerenone may be
more suitable alternative, as it is marketed as a selective aldosterone receptor antagonist and with some
evidence for use in heart failure.

140. What is the Side effect of Cytarabine?


a) Conjunctivitis
Conjunctivitis Anthracyclines Artificial tears may have a Usually reversible upon discontinuation
(within days) Cytarabine dilutional effect. of offending agent.
(Dose > 2g/m2) For cytarabine, prednisolone 1% Artificial tears to provide local relief.
Fluoro- or dexamethasone 0.1% eye drops Corticosteroid ophthalmic preparations
pyrimidines Q4–6H started before & continue to manage inflammation.
> 48 h post cytarabine. Ocular pain may require analgesics.
Ophthalmic diclofenac may also
have a role.

141. Patient with hypertension and take drugs, he went to the pharmacy and checked for BP, he
found it low, what was the reason, he takes Sildenafil at the morning:
a) Large cuff size when measure BP
b) Hand was lower than heart position
c) He took Sildenafil yesterday morning so today his pressure will be low
d) He takes Levodopa/Carbidopa
e) He took his medication after food instead of taking them on empty stomach
Of course, sildenafil, on its own, can be another culprit. However, here they address the logic of the human
being (away from pharmacy); therefore, given that sildenafil has been used for 3 months without issues, it's
unlikely to be the culprit.

142. A male patient with acute bronchitis comes to your pharmacy requesting for an antibiotic,
what is the most important counselling tip to give
a) No need for the antibiotic
b) Inhaler corticosteroid
c) You should not take an antibiotic
d) Inhaler Salbutamol
e) Expect cough to last for 10 days
The most common symptom is a cough. Other
symptoms include coughing up mucus, wheezing,
shortness of breath, fever, and chest discomfort.
The infection may last from a few to ten days.
143. A shortage happened, and you call the patient, what is first to ask?
a) Urgency of the drug (assess their need and the urgency will be a logical 1st step)
b) Find alternative
c) Call hospital to bring supply for him
ANOTHER VERSION
The pharmacist knows that, there is one drug will be in shortage, what to do?
a) Inform the doctor
b) Inform the patient
c) Automatic substitution

144. Which one is considered a prescribing cascade?


a) Donepezil  Constipation  Senna
b) Tolterodine  Confusion  Risperidone
c) Amlodipine  edema  furosemide  Hypokalemia then take K supply
d) Hydrochlorothiazide  Hyponatremia  Seizure
e) Mirtazapine  insomnia  Diazepine
Prescribing cascade is defined as the situation in which a first drug administered to a patient causes adverse
event signs and symptoms, that are misinterpreted as a new condition, resulting in a new medication being
prescribed. The cascade may have multiple steps and differ in complexity and severity.
Kcl‐ cause stomach irritation‐ may need PPI
145. For a patient with uncontrolled NYHA III CHF, who took ACEI this morning, if you wish
to switch the patient to ENTRESTO (Sacubitril/Valsartan), when should he take the first dose?
a) Take after 24hours of ACEI dose
b) Take same day in the evening after taking ACEI
c) Take the next day in the evening
d) Take after 2 days
CTC: It is critical to wait 36 hours between the administration of an ACE inhibitor and valsartan/sacubitril (or
switching back to an ACE inhibitor)

146. Cold chain question and proper save of a vaccine in pharmacy


a) Put so near in fridge to each other
b) Put in bar fridge
c) Put bottle of water in fridge door and label them “NOT FOR USE”
d) Put them in leafy boxes

147. You are in the pharmacy & have many medications unused. You want to return them to the
manufacturer. Upon contacting manufacturer, he said that he will take any medication except
that needs to be refrigerated or narcotics. which of the following he can return them?
a) Medroxyprogesterone acetate
b) Acetazolamide
c) Buprenorphine (narcotic)
d) Trifluridine eye drops
e) Prednisolone eye drops
Acetazolamide: Store at room temperature 15‐30°C. Use within 12 hours of reconstitution. Contains no
preservative. Discard unused portion. Reconstituted solution should be stored in refrigerator at 2‐8°C.
PRED FORTE should be stored at 15° to 25°C. Protect from freezing. Store in an upright position.
Medroxyprogesterone acetate: Protect from freezing. Store upright at controlled room temperature 15 to
30°C. Shake well before using. Keep out of reach of children.
Trifluridine eye drops Refrigerate at 2° to 8° C. Protect from light.

148. When she comes to make refill, pharmacist realized she is refilling for some drugs and
others not, when pharmacist asked her about the reason, she said that she can’t pay as she has
no insurance plan, what the pharmacist can recommend?
a) Check with the manufacture if they can help
b) Check with the doctor
c) Try non-pharmacological options instead
d) Social worker program
149. Female patient suffers from chronic kidney disease, DM and dyslipidemia. What should be
her blood pressure target:
a) < 140/90
b) < 130/80
c) < 130 /95

150. Question on sick days in diabetic patient what medication to hold?


a) Gliclazide
b) Insulin
c) Atorvastatin
Sick‐Day Medication List
 When diabetic patients are feeling ill and are unable to intake adequate fluids or have a sudden
decrease in renal functioning (e.g. due to dehydration from vomiting or diarrhea), they should hold the
following medications in order to reduce the risk of worsening renal function and adverse effects of
medications (e.g. hypoglycemia): anti‐hypertensives, anti‐hyperglycemic and anti‐inflammatory
analgesics (which may also be found in cough and cold products).
 "SADMANS" is an acronym used to recall such medications that should be held until patients are able
to eat and drink normally:

151. Diabetic patient taking Metformin and A1C is 8 and obese, what to do?
a) Linagliptin
b) Glyburide
c) Repaglinide
d) Acarbose
152. Now, his blood sugar is under control but he developed hypertension. What is the First line
for the management of hypertension?
a) Thiazide
b) Amlodipine
c) Propranolol
d) ACEI

153. When 2 drugs are biosimilar, this means that


a) They are interchangeable in the formulary
b) Having the same dose & dosage form
CTC (RA chapter) Biosimilars: Biosimilars (previously called subsequent entry biologics) are biologic drugs
that are made by a different manufacturer than that of the reference (innovator) biologic. Unlike generic
drugs, biosimilars are not identical to the reference biologic drug and are therefore not necessarily
interchangeable. A biological drug is a large protein molecule that is produced through recombinant DNA
technology. The manufacturing process is propriety and unique; it is impossible to exactly replicate the
original molecule and therefore biosimilars should not be considered generic medicines. Biosimilars have a
separate approval process.
To be authorized for sale in Canada as a biosimilar, a drug must meet a detailed set of criteria from Health
Canada (for example, similar biochemical structure; pharmacokinetic & pharmacodynamic characteristics)
and must demonstrate safety and efficacy for each indication; in certain situations, it is possible to
extrapolate therapeutic similarity from one indication to another indication.

154. A patient punctured while he was in gardening & developed osteomyelitis. Culture shows
Gram-positive cocci in grape-like clusters. What maybe the responsible microorganism?
a) S. pneumonia
b) S. aureus
c) C. perfringens
d) P. aureginosa
Streptococcus Gram-positive cocci in chains (when cultivated in liquid media)
pyogenes Nonmotile, non-spore-forming, often encapsulated (capsule composed of hyaluronic acid)
S. aureus Gram-positive cocci in grape-like clusters. Nonmotile, non-spore-forming
Pseudomonas Pseudomonas aeruginosa is a gram-negative, rod-shaped, asporogenous, and
aeruginosa monoflagellated bacterium that has an incredible nutritional versatility.
It is a rod about 1-5 µm long and 0.5-1.0 µm wide.

155. Which is the proper treatment:


a) Cefazolin
b) Gentamycin
c) Cefuroxime

156. What is the duration of treatment without wound Debridement?


a) 4 – 6 weeks
b) 10 – 14 days
Duration of antibacterial therapy should be minimum of 4 ‐ 6 weeks (especially for vertebral osteomyelitis).
If there is adequate debridement, 10‐14 days treatment is often adequate.

157. Propafenone can’t be taken in?


a) HTN
b) Heart failure
c) Bradycardia
CTC: Flecainide and propafenone should not be used in patients with coronary artery disease or structural
heart disease. Propafenone exerts both beta blockade and a dose related direct negative inotropic effect on
myocardium. Therefore, PROPAFENONE should not be prescribed in patients with uncontrolled congestive
heart failure where left ventricular output is less than 35%.
Caution should be exercised when using PROPAFENONE in patients with minimal cardiac reserve or in those
who are receiving other drugs with negative inotropic potential.
158. Non pharma for dysmenorrhea severe period pain (may be bleeding also)
CTC: Nonpharmacologic Choices
 Topical heat therapy, e.g., heating pads & Regular exercise may provide some relief
 Yoga may improve menstrual pain and quality of life
 High‐frequency transcutaneous electrical nerve stimulation (TENS) may be helpful.
 Facilitate smoking cessation as smoking is an independent risk factor for dysmenorrhea
 Surgical management of dysmenorrhea may be indicated to treat dysmenorrhea refractory to
medical management, e.g., laparoscopy, pelvic denervation procedures & hysterectomy.
 There is insufficient evidence to recommend behavioural interventions, e.g., progressive relaxation,
or application acupoint stimulation (acupressure or acupuncture) for management of dysmenorrhea.

159. IBD CASE: risk factors of crohn’s


a) Carbohydrates food
Risk factors which have been linked to
developing IBD include age (15‐40 years old
for UC and < 30 for CD), infection, family
history, Caucasian ethnicity, western diet,
psychological stress, and environmental
factors (e.g. bacterial, viral, dietary antigens).
The Western pattern diet (WPD) or standard American diet (SAD) is a modern dietary pattern that is generally
characterized by high intakes of red meat, processed meat, pre‐packaged foods, butter, candy and sweets,
fried foods, conventionally‐raised animal products, high‐fat dairy products, eggs, refined grains, potatoes,
corn (and high‐fructose corn syrup) and high‐sugar drinks, and low intakes of fruits, vegetables, whole grains,
grass‐fed animal products, fish, nuts, and seeds.

160. What is the drug of choice for the induction of remission for a patient with moderate to
severe Crohn’s disease?
a) 5-ASA
b) Azathiopurine
c) Nebulized budesonide
d) Oral prednisolone 10 – 16 wk

161. What to give in maintenance therapy of


Crohn’s disease?
a) Sulfasalazine
b) Prednisone
c) Azathioprine
d) Budesonide
Biologic therapy; Infliximab and adalimumab are
effective in inducing and maintaining remission in patients with moderate to severe CD and in inducing closure
of perianal fistulae. Combining infliximab with azathioprine is more effective than infliximab monotherapy.
162. The same patient is diagnosed with hyperuricemia and doctor prescribed him allopurinol.
He came to you for dispensing it and there is interaction between them, how will you inform the
doctor about it?
a) There is DDI and we need to address this therapy
b) There is DDI and this is contraindicated
c) This contraindication can cause serious complication
d) If the patient is TPMT deficient will has a problem (Thiopurine methyltransferase
deficiency)
Allopurinol Increase Azathioprine effect so decrease dose to 1/4
Several case reports describe increased toxicity of mercaptopurine or azathioprine (which is metabolized to
mercaptopurine) in patients receiving concomitant allopurinol therapy. Significant bone marrow suppression,
including leukopenia, pancytopenia, and thrombocytopenia have occurred. Allopurinol inhibits xanthine oxidas
e, the enzyme responsible for first‐ pass metabolism of mercaptopurine to inactive products, and thus causes
accumulation of mercaptopurine. It has been suggested that intravenous mercaptopurine may not be affected
by allopurinol in the same fashion, however, precautions are still reasonably employed

163. Patient with epilepsy, who do you refer to?


a) Neurologist
b) Occupational therapist
c) Physiotherapist

164. What true about sunscreen


a) Sunscreen SPF 30 provide double protection of sunscreen SPF 15
b) Physical sunscreen SPF 30 protect against UVB only
c) Water resistance 40 min. or 80 min. need second reapplication
d) Apply DEET, and after 20 min apply sunscreen before outdoor activities.
e) Apply before going out by 60 min.
Sun protection factor SPF: a number that tells you how long a sunscreen will protect your skin from sunburn. SPF also
tells you the amount of protection. SPF 15 blocks 93% of UVBSPF 30 blocks 97% of UVBSPF 50 blocks 98% of UVBSPF is
not a measure of UVA protection. Most sunscreens do block UVB and UVA.
Concurrent application of sunscreen with the insect repellent N, N‐diethyl‐meta‐toluamide (DEET) may lower
SPF but the efficacy of DEET appears to be maintained. If application of DEET is delayed after the sunscreen
has been applied, the sunscreen product maintains more of its original SPF.

165. Diabetic pt. On Empagliflozin, got severe pain, nausea, vomiting, polydipsia, polyuria fruit
odour breath. Pharmacist referred him to emergency because he got?
a) Hyper ketoacidosis (DKA)
b) Hyperglycemia
c) Severe Hypoglycemia
Sodium‐glucose co‐transporter 2 (SGLT2) inhibitors (canagliflozin, dapagliflozin, empagliflozin) inhibit SGLT2 in
the proximal tubule of the kidney, which blocks glucose reabsorption, thus causing glycosuria and lowering
plasma glucose. SGLT2 inhibitors may also cause modest weight loss and lowering of blood pressure.
Empagliflozin was shown to decrease cardiovascular events in diabetic patients at high risk for CV disease.
The most common side effects are genitourinary infections, especially mycotic infections. Orthostatic
symptoms can also occur. Reports of diabetic ketoacidosis in patients with both type 1 DM and type 2 DM.

166. what cause the QT prolongation with Citalopram:

167. Patient in hospital is taking IV Cotrimoxazole (each ml contains 80 mg SMX / 16mg


TMP) 0.5 ml Q6hr. Dr. want to shift him to liquid (200 mg SMX / 40 mg TMP) in each 5 ml,
how many of liquid is needed to provide the equivalent dose as was taken IV.
Answer
0.5 ml Q6hr = 0.5 * 4 = 2 ml /Day
SMT: 80 mg -------- 1 ml X mg -------- 2 ml X= 2*80/1 = 160 mg
Liquid: 200 mg --------5 ml 160 mg -------- Y ml Y = 160*5/200 = 4 ml/Day

168. Inventory check for Morphine, Codeine, Oxycodone – the pharmacist bought 1500 tab of
codeine , 1600 tab morphine, 600 tab Hydomorphone, before purchasing the pharmacist were
having 400, 500, 600 tablets from each – then dispensed 200 tab, 400 tab, from them, - some
tablets has been lost and others broken, calculate how much you have ( idea in general). Which
drug is missed and which drug is extra?
Purchase Sales Broken tab. Count

Options was to choose the correct sentence


169. Person is travelling, what is the risk of infection from food?
a) Hepatitis B
b) Hepatitis C
c) Hepatitis A
d) Yellow fever

170. What is the pharmacist role in Euthanasia?


a) Should make sure 2 physicians agreed on eligibility
b) Ensure that the patient gives his consent
Pharmacist Roles in Euthenasia
 Provide information about the process, refer to practitioner BUT do NOT educate the patient on
chemicals that he/she can take to cause death (it is a crime)
 Must be made aware if the Rx is for assisted death
 Must ensure that (but does not need details): Consent was obtained, Eligibility is confirmed &
Second assessment is done
 Counsel practitioner on protocol, time schedule for administration, time required to prepare
prescription, returning unused drugs to the pharmacy
 Dispense in person, no delegation  No adaption
 Must be processed under patient’s name, not for office use
 Pharmacist must report in person to Health Canada (via Canadian MAID Data Collection Portal)
within 30 days of dispensing
 Pharmacist can refuse the service (CPhA requires no mandatory referral)
The Role of the Pharmacist within the Assisted Dying Care Team Pharmacists who choose to participate in
assisted dying have a responsibility to ensure that patients receive the best possible care and guidance
throughout the end‐of‐life process. Pharmacists should be knowledgeable about the protocols for assisted
dying, including requirements for patient consent. Depending upon how assisted dying is provided for in
legislation, protocols may require knowledge of medication management and pharmaceutical care,
including pharmacology, dosage, adverse effects, onset of effect, preparation and stability, storage, and
appropriate administration. Furthermore, the pharmacist has a responsibility to be familiar with all
regulations and requirements of their licensing bodies as they relate to the management and dispensing of
medications for the purpose of assisted dying. (check nurse/ technician/.)
https://www.pharmacists.ca/cpha‐ca/assets/File/CPhA%20AD%20Policy%20‐%20FINAL%20‐%2023%20February%202016%20(2).pdf

171. A prescription about Buprenorphine/Naloxone combination, what is true?


a) Can’t be repeated
b) Transfer Between pharmacies A, B, C and D
c) Transfer for once
d) Transfer for one pharmacy only
e) No transfer
172. Patient takes Calcitriol (Vit D), what he has?
a) Hyperphosphatemia
b) Hyper parathyroid
c) Hypocalcemia

173. HbA1c %, what is the most appropriate condition for glycemic control measured by
glycosylated heamoglobin OR Which one of these HBA1C result is consistent?
a) Pregnant woman in her First trimester
b) African-american female with sickle cell anemia
c) Heart conditions e.g; Heart failure
d) Chemotherapeutics for prostate cancer
e) Man 66 yrs has ESRD (End Stage Renal Disease)
Q. Factors that can affect A1C
Factor Increased A1C Decreased A1C
Erythropoiesis Iron defeciency. Use of erythropoietin, iron or B12
B12 defeciency Reticulocytosis.
Decreased erythropoiesis Chronic liver disease
Altered hemoglobin Fetal hemoglobin. Hemoglobinopathies. Methemoglobin
Variable change in A1C Genetic determinants
Altered glycation Alcoholism Ingestion of aspirin, vitamin C or vitamin E
Chronic renal failure Hemoglobinopathies
Decreased erythrocyte pH Increased erythrocyte pH
Erythrocyte destruction Increased erythrocyte lifespan: Decreased erythrocyte lifespan:
Assays Splenectomy Chronic renal failure
Hemoglobinopathies
Splenomegaly Rheumatoid arthritis
Antiretrovirals Ribavirin Dapsone

174. Comparison between 2 drugs, one reduces the risk by %, other one increases benefit by %,
what is the number needed to reach these results?
One of them increases diabetes by 66% and decreases strokes by 51%, does this drug reduce
stroke by 51% or increases diabetes by 33%? (Relative risk reduction calculation)
Study comparison between 4 drugs - Which one is better?

175. There is a drug, its absorption has been increased by 40% when taken with other drugs,
t1/2 stays as is, the doctor decided to double the dose, what is expected?
a) Tmax will decrease
b) AUC will increase
c) Reach faster to steady state
d) Trough of SS value will decrease
When you increase conc, AUC and Cmax will increase BUT the Tmax and t1/2 remain constant
176. Patient has Torse de points due to?
a) Hypercalcemia
b) Hyperkalemia
c) Hypokalemia
Common causes for torsades de pointes include drug‐induced QT prolongation and less often diarrhea, low
serum magnesium, and low serum potassium or congenital long QT syndrome. It can be seen in
malnourished individuals and chronic alcoholics, due to a deficiency in potassium and/or magnesium.
Torsades de pointes are a specific form of polymorphic ventricular tachycardia in patients with a long QT
interval. It is characterized by rapid, irregular QRS complexes, which appear to be twisting around the
electrocardiogram (ECG) baseline. This arrhythmia may cease spontaneously or degenerate into ventricular
fibrillation. It causes significant hemodynamic compromise and often death. Diagnosis is by ECG. Treatment
is with IV magnesium, measures to shorten the QT interval, and direct‐current defibrillation when
ventricular fibrillation is precipitated.

177. There is an error could happen during


processing of Cefuroxime and Cefotaxime,
how to minimize it?
a) Separate on the shelf
b) Tallman letters

178. Question about a study with dollar value?


Cost Benefit study

179. Patient did a dental extraction and has pain, his dentist prescribed narcotic drug, he has a
prescription for 250 tablets for 6 months use, pharmacist refused to dispense it due to?
a) No written intervals
b) Quantity is too much
It’s a risk of abuse; a suspicious case of Rx forgery Intervals is not a must but it's highly recommended

180. What are a risk factors of myopathy?


a) High dose of Statin
b) Ciprofloxacin
c) Longterm therapy of statin
Patient- and Statin-Related Factors That May Potentiate Myotoxic Effects
Patient Factor Concomitant Medications Statin Properties
 Advanced age (>80 y)  Amiodarone  High dose statin
 Alcohol abuse  Azole antifungals  High bioavailability
 Chronic renal insufficiency (itraconazole, ketoconazole)  Limited protein
 Excessive grapefruit juice intake  Calcium channel blockers binding
 Frailty, small body frame (nondihydropyridines)  High lipophilicity
 Gender (women > men)  Cyclosporine  Potential for drug-
 Hepatic dysfunction  Fibrates (particularly drug interactions
 Hypothyroidism (untreated) gemfibrozil) metabolized by
 Inherited myopathies  HIV protease inhibitors cytochrome P450
 Intercurrent infections  Macrolides (clarithromycin, pathways, especially
 Perioperative periods erythromycin) 3A4 and 2C9
 Vigorous exercise  Nicotinic acid (rarely)
 Vitamin D deficiency

181. What is the following representing Non-maleficence?


a) Call doctor and inform him about drug intraction and the need to modify therapy

182. Pharmacist abuse his break hours, his colleagues complained to the pharmacy manager,
what pharmacy manager should do?
a) Arrange a meeting and discuss the break time
b) Make a warning for the pharmacist
c) Check schedule and confirm if there is an abuse for break hours then take an action

183. All below references can be used for Drug-Drug interaction EXCEPT
a) CPS
b) Lexicomp
c) Remington
d) TC

184. Female patient is taking Risedronate 5mg daily in morning on empty stomach, she came
and complained that, she does not want to take it on empty stomach 30 mints before anything
because she used to drink coffee once wake up, what is the solution?
a) Shift her to Biweekly dose
b) Shift her to Monthly dose
c) Shift her to yearly IV dose
Missed Dose
If a once‐weekly dose (or any etidronate dose) is
missed, advise patients to take it the next
morning, then return to original once weekly
schedule.
If a once‐monthly dose is missed, advise patients
to take it the next morning (if the next month's
dose is >7 days away). If the next month's dose
is within 7 days, wait until the next month's
scheduled dose and return to original once‐
monthly schedule.
185. Corticosteroid Side effect

186. Female has got DVT, she has IUS & flu
& history of smoking from 6 years. Why has
she got DVT??
a) The Progesterone of IUS.
b) Smoking history.
c) She got flu.
Risk Factors for Venous Thrombosis
 Age > 60 years, Obesity
 Cigarette smoking (including passive
smoking)
 Cancer, Myeloproliferative neoplasm (hyperviscosity)
 Estrogen receptor modulators (eg, tamoxifen, raloxifene)
 Heart failure, Hyperhomocysteinemia
 Hypercoagulability disorders: Antiphospholipid antibody syndrome, Antithrombin deficiency, Factor
V Leiden mutation (activated protein C resistance)
 Heparin‐induced thrombocytopenia
 Hereditary fibrinolytic defects, Increase in factor VIII & factor XI
 Paroxysmal nocturnal hemoglobinuria
 Protein C deficiency & Protein S deficiency. Prothrombin G‐A gene variant
 Immobilization
 Indwelling venous catheters, Surgery within the past 3 months
 Trauma & Limb trauma
 Nephrotic syndrome
 Oral contraceptives or estrogen therapy, Pregnancy and postpartum
 Prior venous thromboembolism
 Sickle cell disease
187. Dr. prescribed her LMWH
injection, but she is afraid of needles,
what do you recommend
a) Warfarin
b) Fondaparinux
c) Dabigatran
d) Rivaroxaban

188. How long would you treat her if this


is the first episode of unprovoked DVT?
a) Indefinitely
b) At least 6 months
c) At least 3 months

189. Young female is pregnant in her first trimester, her mother has cancer and take a treatment
but has dementia and needs help with drug administration, what would pharmacist advise her?
a) Don’t help your mom with her drugs because it is teratogenic
b) It is ok you can help her with the drugs administration
c) Help her only with the drugs are not teratogenic

190. Herceptin question


There is a risk of medication errors between HERCEPTIN (trastuzumab) and KADCYLA® (trastuzumab
emtansine). In order to minimize this risk, check the vial labels to ensure that the drug being prepared and
administered is HERCEPTIN (trastuzumab) and not KADCYLA (trastuzumab emtansine).
HERCEPTIN should be prescribed using both the trade name and nonproprietary name.
Trastuzumab alone stops growth of cancer cells by binding to the HER2 receptor, whereas trastuzumab
emtansine undergoes receptor‐mediated internalization into cells, is catabolized in lysosomes where DM1‐
containing catabolites are released and subsequently bind tubulin to cause mitotic arrest and cell death.
Herceptin Monograph: Dosing Considerations
It is important to check the product labels to ensure that the correct formulation (HERCEPTIN or
HERCEPTIN SC) is being administered to the patient as prescribed.
HERCEPTIN SC is not to be used for IV administration and must be administered as a SC injection only.
HERCEPTIN SC should be administered in a hospital setting by health care professionals only.
HERCEPTIN SC is not intended for self‐administration.
There is a risk of medication errors between HERCEPTIN (trastuzumab) and KADCYLA (trastuzumab
emtansine). In order to prevent medication errors, it is important to check the vial labels to ensure that the
drug being prepared and administered is HERCEPTIN (trastuzumab) and not KADCYLA (trastuzumab
emtansine). Ensure that the recommended HERCEPTIN (trastuzumab) dose is administered.
HERCEPTIN should be prescribed using both the trade name and non‐proprietary name. Do not
substitute HERCEPTIN for or with KADCYLA (trastuzumab emtansine).
Recommended Dose and Dosage Adjustment
The recommended fixed dose of HERCEPTIN SC is 600 mg every three weeks irrespective of the
patient’s body weight. No loading dose is required. The dose should be administered over 2‐5 minutes.
The injection site should be alternated between the left and right thigh. New injections should be given at
least 1 inch/2.5 cm from the previous site on healthy skin and never into areas where the skin is red,
bruised, tender, or hard. During the treatment course with HERCEPTIN SC, other medications for SC
administration should preferably be injected at different sites.

191. Acamprosate, what is right about it??


a) Used for Abstinence from alcohol
b) Used to Decrease drinking alcohol

192. When should the first dose of Acamprosate for alcohol withdrawal be taken?
Acamprosate Acamprosate (glutamate and GABA modulator), believed to underlie its ability to relieve
Treatment of symptoms of alcohol withdrawal and reduce the euphoric effects of alcohol.
choice for May be preferred in patients with goal of abstinence.
patients on 666 mg PO TID. Start treatment after ≥ 4 days of alcohol abstinence.
opioid It is the treatment of choice for patients with hepatic insufficiency, since it is renally excreted
therapy. S.E: Diarrhea, vomiting, abdominal pain, pruritus, rash; suicidality (suicidal thoughts /attempts
/completed suicide) has been reported.

193. Which of the following 2nd generation antipsychotics has the least metabolic (dyslipidemia
/ dysglycemic) side effects?
a) Olanzapine
b) Quetiapine
c) Ziprasidone
d) Risperidone
194. KW is a diabetes patient with CrCl 28 ml/min comes to the pharmacy with symptoms of
shallow breathing, discomfort and decreased appetite. Upon investigation you knew that he takes
NPH 25 Units BID, metformin, Canagliflozin and acarbose for diabetes and citalopram for
depression, he also uses garlic to reduce his cholesterol. What possibly caused these symptoms?
a) High dose NPH
b) Citalopram
c) Metformin
d) Garlic
e) Canagliflozin
a) Acarbose
Lactic acidosis is the most common cause of metabolic acidosis in hospitalized patients. Lactate
accumulation results from a combination of excess formation and decreased metabolism of lactate. Excess
lactate production occurs during states of anaerobic metabolism. The most serious form occurs during the
various types of shock. Decreased metabolism generally occurs with hepatocellular dysfunction from
decreased liver perfusion or as a part of generalized shock. Diseases and drugs that impair mitochondrial
function can cause lactic acidosis. Lactic acidosis may rarely occur with metformin accumulation. It is a
medical emergency and requires prompt discontinuation of metformin and treatment in a hospital setting.
Metformin should not be used in patients with a history of lactic acidosis.
Lactic acidosis is characterized by elevated blood lactate levels (>5 mmol/L), decreased blood pH, increased
anion gap and increased lactate/pyruvate ratio. It presents initially with nonspecific symptoms including
malaise, somnolence, and abdominal and respiratory distress. With greater severity, it may be associated
with hypothermia, hypotension and resistant bradyarrhythmia.
Lactic acidosis occurs primarily in patients with renal insufficiency, hepatic dysfunction, or other conditions
involving hypoxemia, dehydration or sepsis. Lactic acidosis may occur in patients who undergo radiologic
studies with radiocontrast dye.
Metformin use in patients with diabetes and heart failure was previously thought to increase the risk of
lactic acidosis. However, newer data suggests metformin does not increase the risk in this population, and is
associated with better cardiovascular outcomes than other antihyperglycemic therapies

195. A 2 months pregnant woman with asymptomatic bacteriuria. Her urine analysis gives
positive UTI but she has no symptoms, what to do what is the best recommendation
a) Do not treat as this does not pose risk to fetus
b) Treat with antibiotics
c) Recommend the patient to drink plenty of cranberry juice

196. If you decide to treat her, what would be the drug of choice
a) SMX/TMP
b) Ciprofloxacin
c) SMX/TMP
d) Nitrofurantoin
Nitrofurantoin is usually avoided near term (38 ‐ 4weeks) because of the risk of inducing hemolytic anemia
in the fetus or newborn, especially in those with G6PD deficiency; however, this toxicity is rare.

197. What is true about insulin in hospital setting?


a) It should be prepared in IV infusion bag and be ready in the ward
b) It should be in multi-doses vials (unit dose)
c) Insulin pens can be used for several patients (false)
d) As insulin pen
https://www.ismp.org/sites/default/files/attachments/2017-11/ISMP138-Insulin%20Guideline-051517-2-WEB.pdf
Very important (must read): Strategies such as tall man lettering with bolded text for the unique letter
characters are used on computer and device screens, pharmacy labels, order forms, storage bins, medication
administration records, and drug information references to help distinguish look‐alike insulin names (e.g.,
HumaLOG and HumuLIN; NovoLOG and NovoLIN). (exam)
2. PHARMACY MANAGEMENT AND DISTRIBUTION OF SUBCUTANEOUS INSULIN
2.2 Processes are in place to differentiate insulin products of different types and concentrations wherever
they are stored.
2.3 Pharmacists confirm the indication before verifying initial insulin orders.
2.4 Insulin vials are removed from the manufacturer’s carton prior to dispensing to patient care areas
2.5 When insulin vials are dispensed (unit‐stock or patient‐specific) and stored in patient care areas
(excluding the patient room), ready‐to‐apply barcoded labels are available to apply to clinician‐prepared
syringes.
2.8 Ideally, insulin pens are dispensed to the clinical units with a patient‐specific, barcode label (that has
been applied in the pharmacy, using a barcode verification process that confirms the correct pen type has
been selected based on the patient’s order) AND steps have been taken to ensure that only the correct
patient‐specific label can be scanned at the bedside.
2.9 A patient‐specific label is affixed on the body of the insulin pen (not on the removable cap), without
obscuring important information on manufacturer labeling or the dose counter/dose window.
3. ADMINISTRATION AND MONITORING OF SUBCUTANEOUS INSULIN
3.1 Patient‐specific insulin pens are stored on clinical units in a manner that prevents their inadvertent use
on more than one patient
3.3 Verbal communication of point‐of‐care blood glucose value results are avoided as much as possible and
are NEVER routinely used as the only source of information when determining insulin doses
3.4 The insulin concentration (e.g., U‐100, U‐200, U‐300) does not follow the name of the insulin on the
medication administration record (MAR) or other medication lists, with the exception of regular insulin U‐
500 (HumuLIN U‐500).
3.5 Appropriately label all clinician‐prepared syringes of subcutaneous insulin, unless the medication is
prepared at the patient’s bedside and is immediately administered to the patient without any break in the
process.
3.6 Prior to subcutaneous insulin administration, the practitioner: a. Confirms that there is an appropriate
indication b. Assesses the patient’s most current blood glucose value c. Assesses the patient for symptoms of
hypoglycemia d. Informs the patient of their most current blood glucose level e. Informs the patient of their
dose, the full name of the product, and the insulin’s intended action
3.7 An individual insulin pen is never used for more than one patient.
3.8 An insulin pen cartridge is never used as a vial
3.9 Barcode scanning is used to verify that a patient‐specific pen is used to administer the correct insulin to
the correct patient.
4. SAFE TRANSITIONS OF CARE FOR PATIENTS RECEIVING SUBCUTANEOUS INSULIN
4.1 Prior to TRANSITIONS OF CARE, a process is in place to ensure that patients will have the necessary
prescriptions, supplies, a follow‐up care plan, and printed instructions for all prescribed insulin and blood
glucose monitoring.
4.3 Healthcare providers primarily prescribe insulin pen devices for outpatients to increase dosing accuracy
and promote self‐care and adherence with their insulin therapy regimen
4.4 Patients discharged on insulin are assessed for understanding of their self‐management, including: a.
Demonstration of proper dose measurement and self‐administration using the same administration device
that will be used at home (e.g., vial and syringe, pen, pump) b. How to monitor blood glucose values c. The
signs and symptoms of hyper‐ and hypoglycemia and how to respond if these symptoms occur d. Common
types of errors possible with their insulin therapy and how to prevent or detect these errors e. The
importance of regular follow‐up with their primary care provider/specialist, including the date of their next
appointment

198. What is considered the most when adding a drug to a hospital formulary
a) Provincial drug list
b) Published evidence of effectiveness
c) Cost of the drug with respect to existing therapy
In Canada, hospital‐based Pharmacy and Therapeutics Committees may function at an individual hospital
level, district or regional health authority level, or provincial level. In this report, the term "Pharmacy and
Therapeutics Committee" (P&TC) refers to a committee responsible for managing drug‐related issues for the
organization represented by the committee. Synonymous terms, such as "Drugs and Therapeutics
Committee" (D&TC) may be used in some Canadian jurisdictions. In this report, the terms "D&TC" and
"P&TC" are used interchangeably. Generally, a P&TC comprises physicians, pharmacists, nurses, and other
health care practitioners, as well as administration and quality assurance representatives and members of
the public. Formulary decisions are made on the basis of empirical evidence; however, factors such as safety
of similar available agents, direct costs, cost offsets, and the total cost of care with a new drug compared
with current care may also significantly impact formulary decisions.

199. There was a question on the side effects of Bupropion (Xyban) for smoking cessation.
Three of the options were known anticholinergic side effects while the other one was out of it
Neuropsychiatric effect (use in smoking cessation): Serious neuropsychiatric events have occurred in patients
taking bupropion for smoking cessation, including changes in mood (eg, depression, mania), psychosis,
hallucinations, paranoia, delusions, homicidal ideation, hostility, agitation, aggression, anxiety, panic,
suicidal ideation, suicide attempt, and completed suicide. The majority occurred during bupropion
treatment; some occurred during treatment discontinuation.
Monitoring Parameters
Body weight; mental status for depression, suicidal ideation (especially at the beginning of therapy or when
doses are increased or decreased), anxiety, social functioning, mania, panic attacks; blood pressure
(baseline and periodically especially when used in conjunction with nicotine transdermal replacement); renal
and hepatic function.
When used for the treatment of ADHD, thoroughly evaluate for cardiovascular risk. Monitor heart rate,
blood pressure, and consider obtaining ECG prior to initiation
Adverse Reactions >10%:
Cardiovascular: Tachycardia (≤11%). Endocrine & metabolic: Weight loss (14% to 23%)
CNS: Insomnia (11% to 40%), headache (25% to 34%), agitation (2% to 32%), dizziness (6% to 22%)
Dermatologic: Diaphoresis (5% to 22%). Neuromuscular & skeletal: Tremor (1% to 21%)
Gastrointestinal: Xerostomia (10% to 28%), constipation (8% to 26%), nausea & vomiting (23%)
Ophthalmic: Blurred vision. Respiratory: Nasopharyngitis (13%), pharyngitis (3% to 13%), rhinitis (12%)

200. There was a question on the drug of choice for rate control for a CHF patient who suffers
an episode of Atrial Fibrillation
a) Amiodarone (consider digoxin first)
b) Propafenone
c) Flecainide
BB‐ non dihydropyridine CCB, digoxin, amiodarone
CTC: Heart Rate Control in Patients with Persistent or Permanent Atrial Fibrillation or Atrial Flutter
With the exception of some patients with intrinsic AV nodal disease (usually elderly individuals), most
patients will need drugs to achieve rate control. Either a beta‐blocker or a CCB (diltiazem or verapamil) is
recommended for initial control of heart rate.
Digoxin should be considered only when response to first‐line agents is inadequate, since it is not
very effective and there are concerns that it may increase mortality. Because of its potential toxicity,
amiodarone should be avoided as a rate control agent.
The medication dose should be titrated to achieve a resting heart rate of <80 bpm and a mean heart rate of
<100 bpm on 24‐hour Holter monitoring.

201. Amiodarone consultation


a) Do not crush
Food increases the rate and extent of
absorption of amiodarone.
Because of the food effect on the
absorption of amiodarone hydrochloride,
administration of AMIODARONE should
be consistent with regard to meals
202. There was a question on Bipolar disorder II. Please try to read this classification properly.
There is BPD I and BPD II. One of them has more episode of mania than depression.
The question was about what to do to manage breakthrough Mania I think
CTC: Specific medication strategies for mania depend on whether the patient is already on maintenance
therapy and is experiencing a breakthrough episode or whether the individual is unmedicated. If the patient
is already taking a first‐line agent such as lithium, divalproex or a second‐generation (“atypical”)
antipsychotic, dosage adjustment may be sufficient after checking blood levels where appropriate. In
moderate‐to‐severe manic episodes, addition of another medication is usually necessary.
In previously unmedicated patients, initiate treatment with a first‐line agent.
When the episode is particularly severe, initiate treatment with a 2‐drug combination such as lithium or
divalproex plus a second‐generation antipsychotic. Continue treatment on any particular regimen for 2
weeks at therapeutic doses before assessing whether a change is necessary.
For manic episodes, the optimal duration of the second‐generation antipsychotic in a 2‐drug combination
has not been determined, although a randomized clinical trial suggests that treatment for 6 months yields
the most favourable risk‐benefit ratio; decrease in relapse rate was similar to that achieved with treatment
for 12 months, and significant weight gain due to metabolic side effects did not occur within 6 months.
Bipolar disorders are classified as
 Bipolar I disorder: Defined by the presence of at least one full‐fledged (ie, disrupting normal social and
occupational function) manic episode and usually depressive episodes
 Bipolar II disorder: Defined by the presence of major depressive episodes with at least one hypomanic
episode but no full‐fledged manic episodes
In cyclothymic disorder, patients have prolonged (> 2‐year) periods that include both hypomanic and
depressive episodes; however, these episodes do not meet the specific criteria for a bipolar disorder.

203. There was a question on Dukoral vaccine. It was about a patient who took Dukoral vaccine
2/3 years ago and comes to your pharmacy for a travel advice because he is planning to travel to
a typhoid and cholera endemic region. What do you advise him?
a) Advise him not to take another dose because the dose he took 3 years ago is still valid
b) Recommend a new and complete dose of dukoral
c) Give a booster dose of dukoral
Vaccines Contains the nontoxic B subunit of cholera toxin, which has significant homology with the
Vibrio cholerae toxin of ETEC and is approved in Canada for prevention of TD caused by ETEC.
whole cell/ Prevention of enterotoxigenic E. coli diarrhea:
recombinant Adults and children ≥2 y: Primary immunization: 2 doses PO; 2nd dose administered within
cholera toxin B 7–42 days after the 1st dose and at least 1 wk before reaching destination
subunit vaccine Booster: 1 dose every 3 months if the risk is continuous
Q. Dukoral Q. Taken orally on an empty stomach (1 h before or 1 h after eating or drinking)
S.E: Abdominal pain, diarrhea, nausea and vomiting.
May consider for prevention of TD in persons with chronic illnesses (e.g., HF, insulin-
dependent DM, IBD, chronic kidney disease), or in those with immune suppression
204. Cancer pt. takes prochlorperazine, olanzapine, dexamethasone, he still has nausea, what to
take?
a) Metoclopramide
b) Nabilone
c) Aprepitant
d) Ondansetron
Nabilone is of limited use because it is available only as an oral formulation and is associated with several
side effects including dry mouth, dizziness, drowsiness, mood alterations, hallucinations, delusions,
tachycardia and hypotension. Higher doses are associated with increased risk of side effects. It is generally
used in refractory nausea and vomiting or in combination with other antiemetics.

205. What will he have got?


a) EPS from prochlorperazine + olanzapine
b) Dexamethasone will cause hyperglycemia
Monitoring Parameters (dexamethasone)
Hemoglobin, occult blood loss, blood pressure, serum potassium, glucose, bone mineral density; IOP with
systemic use >6 weeks; weight and height in children; HPA axis suppression

206. He got constipation & no bowel


movements from 3 days, what can be the
treatment?
a) Psyllium (avoid bulk laxative)
b) PEG
c) Docusate (ineffective in all cases)
d) Refer
https://www.youtube.com/watch?v=mqRwgmDI0gY
(important, clozapine- fatal constipation)
Constipation in chemotherapy:
Constipation may result from opioid use. A stimulant laxative such as senna 2 to 6 tablets po at bedtime
or bisacodyl 10 mg po at bedtime should be initiated when repeated opioid use is anticipated. Established
constipation can be treated with various drugs (eg, bisacodyl 5 to 10 mg po q 12 to 24 h, milk of magnesia
15 to 30 mL po at bedtime, lactulose 15 to 30 mL q 12 to 24 h, Mg citrate 250 to 500 mL po once). Enemas
and suppositories should be avoided in patients with neutropenia or thrombocytopenia.
Olanzapine monograph: Patients should be advised of the risk of severe constipation during olanzapine
treatment and that they should tell their doctor if constipation occurs or worsens, as they may need
laxatives.

CTC: Evidence is lacking to support the effectiveness of the stool softeners docusate sodium and docusate
calcium in the treatment of any type of constipation, including opioid‐induced and functional
constipation; therefore, they are not recommended.
A stimulant (e.g., bisacodyl, senna) or osmotic (e.g., lactulose, polyethylene glycol) laxative should be
started when initiating an opioid in patients who are experiencing constipation or have a history of OIC.

207. Vaccination recommendation a customer of yours travelling to the Caribbean for a


holiday. All except
a) Twinrix
b) Vivotif (Typhoid vaccine)
c) Dukoral
d) Gonococcal Vaccine
The CDC and WHO recommend the following vaccinations for travelers to the Caribbean and Northern
America: hepatitis A, hepatitis B, typhoid, cholera, yellow fever and rabies
Another version: Some one travilling to Tahiti 59 years which vaccine not to be taken?
a) TDaP
b) Pneumonia
c) Meningitis
d) Inflenza
Public health recommendation to Tahiti hepatitis A, hepatitis B, yellow fever measles and influenza

208. Doctor is starting a patient who is 60 kg on Isotretinoin, he wants to start at 0.5 mg/kg/day
for 4 weeks. after the 4 weeks he wants to increase the dose to 1 mg/kg/day. Total cumulative
dose is 120-150 mg/kg. If he starts with 30 mg daily for 4 weeks, what is the next step?
a) 30mg BID for 3 months
b) 30mg BID for 4 months
c) 30mg BID for 5 months
d) 60 mg BID for 3 months
Answer:
minimum cumulative dose = 120mg*60Kg=7200 mg
taken dose = 30mg/daily *28 days = 840 mg
So, the dose should be taken = 7200 - 840 = 6360 mg
If patient takes 30 mg BID = 30mg *2 = 60 mg Daily
No. of days = 6360 / 60 = 106 days = 3.5 months as this is the minimum cumulative dose So the
dose should be used for more than 3.5 month the answer is 30 mg BID for 4 months

209. Inhaler gives 0.21 mcg. each buffer has 200 doses. The pt. Take it … How many days to
be finshed??

210. Anxiety + Insomnia  Clonazepam

211. Pregnant woman didn’t take any supplements. She got anemia because she didn’t take:
a) Iron
b) Folic acid
c) Vit B12

212. Nurse ask about a drug if it needs especial handling, where to look?
a) NIOSH
NIOSH Alert contain sample list of health hazard drugs such as doxorubicin, daunorubicin, vincristine,
vinblastine, cyclosporin or melphalan. Hazardous Drugs ‐ include those that exhibit one or more of the
following characteristics in humans or animals: carcinogenic (causing cancer), teratogenic (causing
developmental damage to a fetus), reproductive toxic (impairs fertility), organ toxic at low doses, genotoxic
& have characteristics similar to an existing hazardous drug
213. T. score & Z. score 10%, what does this means?
The T-score The Z-score
The T-score on your bone density report shows how much your bone A Z-score compares your bone
mass differs from the bone mass of an average healthy 30-year-old density to the average bone density of
adult. A bone density test is like any other medical test or people your own age and gender. For
measurement. The results for the entire population will be distributed example, if you are a 60-year-old
around an average score (the mean). female, a Z-score compares your
A T-score is a standard deviation — a mathematical term that bone density to the average bone
calculates how much a result varies from the average or mean. The density of 60-year-old females.
score that you receive from your bone density (BMD or DXA) test is NOTE: Any post menopausal woman
measured as a standard deviation from the mean. The manufacturers of should always request her T-score
the DXA machines have programmed them to use a formula to rather than just her Z-score
compute these values.
How Often Should I Get Tested? A Z-score is helpful in diagnosing
One standard deviation is equal to a 10–12% difference in bone mass. secondary osteoporosis and is always
If you are exactly equal to the peak bone mass of an average 30-year- used for children, young adults,
old, you do not deviate at all from the average so your T-score would women who are pre-menopausal, and
be 0 standard deviations (SD). If your bones are stronger than the men under age 50. If you have a very
average adult, your bone mass may be +1 or +2 SD indicating that your low Z-score (more than 2 standard
bones have a mass 10–20% above that of the average 30-year-old. If deviations below other individuals
your bones are less dense than the average adult, your standard your age), your doctor should
deviation may be -2 or -3 indicating that your bone mass is 20–30% consider whether other medical
below that of the average 30-year-old. conditions or medications may be
causing lower than expected bone
density.

Although you may have low bone


density when you have your first test,
your doctor cannot tell if you have
lost bone density or if you have
always had lower bone density due to
family or medical history. Your peak
adult bone mass may have been
Definition of Osteoporosis and Osteopenia (low bone density)
below that of the average individual.
Osteoporosis was defined by the World Health Organization in 1994
For example, if you have a T-score of
as a T-score that is 25% lower than the average 30-year-old or 2½
-2.5, it is not appropriate to say that
standard deviations below the mean or a T-score lower than -2.5.
you have lost 25% of your bone
Some people have low bone density. You may hear this called
density unless you had a bone density
osteopenia. Osteopenia is defined by the World Health Organization
test when you reached peak bone
(WHO) as 10% to 25% below an average healthy 30-year-old adult, or
density. There are lab tests that can
a T-score between –1.0 and –2.5 standard deviations below normal.
help your doctor determine if you are
Having a T-score between -1.0 and -2.5 is not “pre-osteoporosis” or a
currently losing bone density.
medical condition, but it is very important for people with low bone
density to develop a good plan to prevent bone loss and osteoporosis.
214. female patient, obese, used medications to loss her weight but she isn’t satisfied. She came
to your pharmacy asking for advice & she don’t want any drugs. You have prepared a natural
compounding for obesity and tried it with some patients and it was effective, what you will do?
a) Tell her about this compounding if she asks about an alternative
b) Don’t tell her even if she asks about an alternative
c) Put a brochure in the pharmacy advertising about this compounding
d) Put an advertisement on facebook page
If you see any drug that can help patient, you should tell him.

215. if she decided to take this compounding, what pharmacist should tell her
a) Not to be used beyond use date (immediate use)

216. Pharmacist wants to delegate some his/her tasks


to tech. Which is the biggest barrier?
a) Pharmacist attitude
b) Technician workspace
c) Some public legislations
d) Pharmacist availability to supervise

217. Pharmacist can do delegation for technician to


which of the following?
a) Counsel for nicotine Spray
b) Counsel for Smoking cessation
c) Counsel for Diabetic readings
d) Counsel for glucometer

218. Specificity and sensitivity meaning (there was a table of numbers)


…………….65% less risk ………………
…………….35% more risk ………………
a volunteer of RCT research, what she is upholding when she does it for the sake of the patient?
219. Assistant and technicians’ responsibilities

220. Drug interaction between 2 drugs (cause hypokalemia)


Hyperkalemia Hypokalemia
Drug-induced: ACEIs, ARBs, BBs, aliskiren, Drug-induced: aminoglycosides, amphotericin B,
aminocaproic acid, cyclosporine, digoxin overdose, antipseudomonal penicillins, beta2-agonists, caffeine,
drospirenone, heparin, K+ supplements, K+ sparing foscarnet, insulin, laxatives, licorice, long-term
diuretics, ketoconazole, NSAIDs, penicillin G corticosteroid therapy, loop and thiazide diuretics,
potassium, trimethoprim or TMP/SMX, theophylline, tocolytic agents.
pentamidine, succinylcholine, tacrolimus, alfalfa and . Diarrhea, vomiting
nettle. . Inadequate dietary intake
. Renal failure, hyporeninemic hypoaldosteronism. . Familial history (Bartter or Gitelman syndrome)
. Diabetes, adrenal insufficiency. . Mineralocorticoid excess, e.g., primary aldosteronism
. Familial history of hyperkalemia . Renovascular disease, Metabolic alkalosis
. Acidosis, Massive transfusions . Osmotic diuresis (diabetes), Hypomagnesemia
. Crush injury, trauma, hemolysis, tumor lysis. . Increased sweat loss, Dialysis/plasmapheresis

221. Which drug that, can be taken in J. tube (enteric coated medication)
a) Dabigatran
b) Finasteride
c) Psyllium
d) Rivaroxaban
Don’t use SR, Irritating drugs, Enteric coated & Emulsion
For patients who are unable to swallow whole tablets, XARELTO tablets may be crushed and mixed with
applesauce immediately prior to use and administered orally. After the administration of a crushed
XARELTO 15 mg or 20 mg tablet, the dose should be immediately followed by food. A crushed XARELTO
tablet may be also administered via nasogastric (NG) tube. After confirming gastric placement of the NG
tube, the crushed tablet should be suspended in 50 mL of water and administered via the NG tube after
which it should be flushed with water. Because rivaroxaban absorption is dependent on the site of drug
release in the GI tract, avoid administration of XARELTO distal to the stomach as this can result in reduced
absorption and therefore reduced drug exposure. After the administration of a crushed XARELTO 15 mg or
20 mg tablet, the dose should then be immediately followed by enteral feeding
222. A 35 years old female is currently using Hydrochlorothiazide 25 mg QD since last 6
months to control her hypertension. Her hypertension is not controlled and Today she comes
with a new prescription of Ramipril 5 mg QD. How will you counsel her to start the new
medication?
a) Start Ramipril from today and stop Hydrochlorothiazide for 7 days to avoid risk of
hypotension
b) Start Ramipril from today and decrease the dose of Hydrochlorothiazide

223. You need 375 mg from 2 compounds, one 250 mg from compound 1 & 125 mg from
compound 2 – compound 1 is 1000 gm powder and compound 2 is 500 mg powder, dissolve
them in 10 ml – its displacement value is 1.4 ml, how much will use from each one?
S1:
If the total is 10 mL
As 1 g in 10 mL so 250 mg X mL
As 0.5 g in 10 mL so 125 mg will be X mL

S2:
If we add 10 mL
1.5 g = 1.4 + 0.7 = 2.1 mL will be displaced.
If we add 10 mL + 2.1 mL = 12.1 mL total
volume stock of 1g A & 0.5 B
As 1 g in 12.1 so 250 mg X mL
As 0.5 g in 12.1 so 125 mg will be X mL

S3:
If we add 10 mL to each compound dissolved
in separate 10 mL, so calculate the stock for each
1g = 1.4 mL + 10 mL =11.4 mL. so, 250 mg in X mL
0.5 = 0.7 mL + 10 mL = 10.7 mL. so, 125 mg in X mL
224. A physician calls the pharmacist to ask whether it is safe to prescribe losartan for a patient
who experienced mild angioedema with perindopril. Which of the following is an appropriate
response for the pharmacist to provide to the physician?
a) Losartan may be safe to use, provided that the patient did not experience airway
obstruction with perindopril.
b) At least one month should elapse after stopping perindopril before starting losartan
c) Therapy with an ARB is contraindicated in a patient who experienced angioedema with
an ACEI
d) The patient should be monitored for 30 minutes after taking the intial dose of
losartan.

225. Which of the following statements about hand hygiene is correct?


a) Antimicrobial gels containing at lest 60% alcohol may be used as an alternative to
soap water.
b) Bar soap is more effective than liquid soap when washing hands that are visibly soiled.
c) It is important to wash jewelry like rings and bracelets as part of the hand hygiene
routine.
d) When using soap and water, it is the temperature of water that determines the
effectiveness of the hand.
MAY 2019
1. What patient shouldn't be prescribed antipsychotics?
a) Lewy body dementia  also dementia of parkinson disease
b) Vascular dementia
c) Alzheimer's disease dementia.
Antipsychotics may worsen psychosis in patients with Lewy body dementia, and can lead to severe
antipsychotic sensitivity reactions that can precipitate irreversible parkinsonism.
If treatment with antipsychotic is necessary, limited evidence suggests using 2nd generation, specifically
quetiapine and olanzapine, may be better tolerated than 1st generation agents.

2. the same patient above cannot sleep; she wakes up many times “explanation of sundown
syndrome”, What to give to this dementia patient that needs to sleep?
a) Mirtazapine
b) Olanzapine
c) Trazodone
d) Melatonin
SARI Trazodone. Manages agitation and disrupted sleep. Trazodone SE: Headache, sedation, dry mouth
BDZs; may cause over-sedation and worsened cognition; last resort.
In severe cases, Lorazepam can be mixed with Haloperidol

3. Another question about dementia, and a patient has depression as well, what should be given
to manage depression?
a) Memantine
b) Lithium
c) Pindolol
d) Sertraline  Take with food to increase availability
Depression SSRIs: sertraline and citalopram.
Early stages  May cause hyponatremia/SIADH (Syndrome Inappropate ADH) in elderly.
of dementia.  SSRIs are preferred to TCAs as less anti-cholinergic and orthostatic hypotension risks.
TCA: desipramine or nortriptyline. Use if there is lack of response or sensitivity to an SSRI.
 Start at low doses, increase dosage until the recommended range is reached.
 Maintain therapy for 4–6 weeks after the first indication of symptomatic improvement

4. A daughter came to your pharmacy and she is excited for dispensing rivastigmine and said, “I
hope that my mother to get back to her normal life”, what to tell her to show empathy
a) I understand your feeling, but this illness is progressive.
b) Any medication of Alzheimer does not stop the deterioration of this disease.
c) Rivastigmine can treat her condition
d) It will delay the progression of the disease
Sympathy has been defined in the healthcare literature as an emotional reaction of pity toward the
misfortune of another, especially those who are perceived as suffering unfairly.
In contrast, empathy has been defined as an ability to understand and accurately acknowledge the feelings
of another, leading to an attuned response from the observer
Cholinesterase Inhibitors: benefits are typically small to moderate, and in many patients, can consist mainly
of disease stabilization. Rivastigmine patch if Nausea as SE.
Monitor treatment effects 2 weeks after initiating therapy or increasing dosage, then every 3 months.
Rivastigmine Inhibits non specific butyrylcholinesterase and reversible acetylcholinesterase or centrally
oral, selective arylcarbamate AchEi. Has short half life 2 hours, but able to inhibit AchEi to 10 hours.
transdermal Because of slow dissociation of carbamate enzyme, it is referred as pseudo-irreversible AchEi.
patch Initiated at 1.5 mg BID and increased to the minimum effective dose of 3 mg BID after 30 days.
The maximum dose is 6 mg BID PO. Not metabolized by cytochrome P450 system.
Oral: Adjust dose monthly; take with breakfast and dinner.
Patch:1 Exelon-5 patch daily; if well tolerated, increase to Exelon-10 patch after at least 4 wk.
If switching from oral rivastigmine, use Exelon-5 for patients taking <3 mg BID, and Exelon-10
for patients taking 3–6 mg BID
S.E: >10%: headache, dizziness, N/V, diarrhea, abdominal pain, anorexia. <10%: insomnia,
fatigue, syncope, dyspepsia, weight loss, UTI, rhinitis. Heart block, delirium & seizures are rare.
Patients should be monitored for symptoms of active or occult GI bleeding, especially those at
increased risk for developing ulcers, e.g., history of ulcer or receiving concurrent NSAIDS.
Patients may lose weight while taking cholinesterase inhibitors, including rivastigmine.
Therefore, the patient's weight should be monitored during therapy with EXELON PATCH.
Considering the possibility of an additive extra-pyramidal effect, the concomitant use of
metoclopramide and rivastigmine is not recommended.
Additive effects leading to bradycardia (which may result in syncope) have been reported with
the combined use of various beta-blockers (including atenolol) and rivastigmine.

5. The same woman comes later complaining that her mother has developed fecal incontinence
and constipation and wonders what the cause is:
a) It is a normal progression of the disease
b) These are side effects of Rivastigmine
Rivastigmine SE diarrhea OR fecal incontinence (later stage)
Memantine SE is constipation (early stage)

6. A patient has bilateral burning leg pain, worsens in the evening and at night. What the cause
of Restless leg syndrome? (Restless Leg Syndrome RLS or Willis-Ekbom disease)
a) Calcium deficiency
b) Iron deficiency
c) Vit B12 deficiency
7. All are Non-Pharm Tips to improve pt with Restless leg syndrome, except?
a) Try baths and massages.
b) Moderate, regular exercise.
c) Try relaxation techniques, such as meditation or yoga.
d) Establish good sleep hygiene
e) Complete Bed rest
Nonpharmacologic Choices
 Engage in mental alertness activities (playing cards or video games or doing crossword puzzles) to
reduce symptoms during times of boredom.
 Abstain from alcohol, caffeine and nicotine. Take hot baths, stretch and exercise moderately.
 Discontinue medications that may be contributing to symptoms, e.g., antidepressants,
antipsychotics, dopamine-blocking antiemetics and sedating antihistamines.
 Minimize aggravating factors such as sleep
deprivation.
 In patients with RLS and varicose veins, consider
sclerotherapy to improve RLS symptoms
 In patients who do not respond to
pharmacotherapy or who experience intolerable
side effects, consider a trial of pneumatic
compression devices (PCDs), garments that are
intermittently inflated and deflated with
compressed air using electrical pneumatic pump.

8. What should be the treatment?


a) Amitryptilline
b) Venlafaxine
c) Prochlorperazine
d) Pramipexole

9. Side effect of Pramipexole


a) Constipation
b) Sweating
c) Edema
d) Insomnia
“there was no gambling or hypersexuality”
Orthostatic hypotension, somnolence, confusion, hallucinations, nausea, vomiting, insomnia, sudden sleep
attack; caution patients about driving or operating dangerous machine. Caution patients about potential
compulsive behaviours such as pathologic gambling or hypersexual behaviour. Risk of parkinsonism
hyperpyrexia syndrome with abrupt discontinuation; taper gradually. Drug holidays not recommended.
10. Patient 80 years old has stress urinary incontinence and hypertension, her medication:
Solifenacin, Atorvastatin, she got Alzheimer prescription for Rivastigmine & docusate sodium.
Then Patient had feacal and worsened urinary incontinence, doctor stops Solifenacin, what the
drug therapy problem (DTP)
a) Wrong ttt from the beginning as solifenacin is ineffective in stress incontinence
b) DDI with Atorvastatin
c) Its anticholinergic side effects inc cognitive dysfunction
d) Its normal progression of disease
e) Effect of medications added

11. A case of 52 yrs old lady with GERD tried many drugs; now she crushes and chew
rabeprazole at night as she suffers more at night, what to advice her?
a) To take rabeprazole morning and Calcium carbonate as addivitve to more contol
b) Call her Dr to change to something like Fast acting lanzoprazole
c) Use rantidine twice daily instead of rabeprazole
If the enteric coating of the tablet is crushed, the medication will be quickly degraded in the stomach before
reaching the duedenum for absorption, rendering the drug ineffective. There are few options if the tablet or
capsule cannot be swallowed whole:
1) Switch to a capsule formulation of omeprazole or lansoprazole where the content can be mixed with
soft food for administration
2) Switch to Pravacid Fastab which is a fast dissolving tablet of lansoprazole.
3) Ask pharmacy to formulate an oral suspension for administration.
Should be Ranitidine if nocturnal

12. She has family history of osteoporosis (parental hip fracture), got a vertebral fracture. What
is the major Risk factor of her osteoporosis?
a) Family history
b) Rabeprazole
He is asking here about the major risk factor, family history here is the main risk factor.
If they asked about what increase the risk factor, should be B
Risk Factors Prompting Assessment for Osteoporosis (Measurement of BMD):
Older Adults (≥50 y) Younger Adults (<50 y)
Age ≥65 y Fragility fracture, Prolonged use of
Clinical risk factors for fracture (men age 50–64 y, corticosteroids, use of other high-risk
menopausal women): vertebral compression, fracture fragility, medications, e.g., aromatase inhibitors, PPIs,
fracture after age 40, prolonged use of corticosteroids, use of androgen deprivation therapy, Hypogonadism
other high-risk medications, e.g., aromatase inhibitors, or premature menopause (<45 y),
androgen deprivation therapy, parent with hip fracture, Malabsorption syndrome, Primary
osteopenia identified on x-ray, current smoking, high alcohol hyperparathyroidism, Other disorders strongly
intake, low body weight (<60 kg) or major weight loss (>10% associated with rapid bone loss or fracture.
of weight since age 25), rheumatoid arthritis. Race (asian & caucasian).
13. To diagnose her with Osteoporosis, her BMD T-Score should be below:
a) 1.5
b) - 2
c) - 2.5
BMD of the spine and hip by dual x-ray absorptiometry (DXA) is
the preferred method of assessing bone mass. Other methods
(heel ultrasound) are acceptable if DXA is not available.
The WHO created DXA-BMD definition of osteoporosis as a
BMD T-score ≤ −2.5 (2.5 standard deviations below a normal young adult reference mean).
Osteoporosis Canada recommends calculating estimated 10-year risk of osteoporotic fracture in women,
which is derived by combining age and epidemiologic data with DXABMD measurements.
Two fracture risk calculators have been developed, validated and are now recommended as an aid to
osteoporosis management for the Canadian population: FRAX (Fracture Risk Assessment Tool) and CAROC
(Canadian Association of Radiologists and Osteoporosis Canada). The estimated risk is reported as low
(<10%), moderate (10–20%) or high (>20%). Risk categories are intended only for assessment of the as-yet-
untreated patient

14. Now, she is about to start Alendronate, what you should tell her about:
a) The risk of atypical fractures
b) Possible non-infectious jaw necrosis
c) To take medicine after meals
d) Not to lie down for 30 min after taking medication
The recommended dosage is: one 70 mg tablet once weekly or one 10 mg tablet once daily.
Administration
APO-ALENDRONATE must be taken at least one-half hour before the first food, beverage, or medication of
the day with plain water only. Other beverages (including mineral water), food, and some medications are
known to reduce the absorption of APO-ALENDRONATE. Waiting less than 30 minutes will lessen the effect
of APO-ALENDRONATE by decreasing its absorption into the body.
APO-ALENDRONATE should only be taken upon arising for the day. To facilitate delivery to the stomach and
thus reduce the potential for esophageal irritation, an APO-ALENDRONATE tablet should be swallowed with
a full glass of water (200 to 250 mL). Patients should not lie down for at least 30 minutes and until after
their first food of the day. APO-ALENDRONATE should not be taken at bedtime or before arising for the day.
Failure to follow these instructions may increase the risk of esophageal adverse experiences.
All patients must receive supplemental calcium and Vitamin D, if dietary intake is inadequate.
Serum calcium and symptoms of hypocalcemia should be monitored during therapy with ALENDRONATE.
Patients should be instructed that if they miss a dose of APO-ALENDRONATE 70 mg once weekly, they should
take one dose on the morning after they remember. They should not take two doses on the same day but
should return to taking one dose once a week, as originally scheduled on their chosen day.
15. A 5-year-old boy, with fever, headache, stiff neck, photophobia, loss of balance, confusion,
no rash, negative brudzinski signs, has bilateral Cochlear implant, got otitis media 3 years ago,
what is the most likely organism for his age?
a) Nisseria meningitidis (-ve)
b) Staph. aureus
c) Strep. Pneumonia (+ve)
Children ≥1 month: S. pneumoniae, N. meningitidis, S. agalactiae (Group B streptococcus), Haemophilus
influenzae type b, E. coli (rare), L. monocytogenes (rare), Other Enterobacteriaceae

16. What is the risk factor of meningitis?


a) Cochlear Implant
b) Age
c) Recurrent respiratoty infections
d) Recurrent Otitis media
Some people say; the reason it's not bilateral cochlear implant because it's date back to 3 years ago.
Recently has recurrent otitis media is the answer. And also, they ask OSAMA: he said I surf a lot I think
cochlear implants. Misbah answer: recurrent otitis media
Dr. habash said it is A, as it is a strange thing inside the body and mostly pose a high risk.
CTC - Conditions increasing risk of invasive pneumococcal disease: chronic cerebral spinal fluid leak,
chronic neurologic condition that may impair clearance of secretions, cochlear implants, chronic cardiac or
pulmonary disease, diabetes mellitus, chronic kidney disease, nephrotic syndrome, chronic liver disease
(including hepatic cirrhosis due to any cause), asthma that required medical care in the preceding 12
months, sickle cell disease or other hemoglobinopathies, congenital immunodeficiencies, anatomic or
functional asplenia, immunocompromising therapy, HIV infection, hematopoietic stem cell transplant
recipient, malignant neoplasms, solid organ or islet transplant candidate or recipient.
The most common pathogens that causes bacterial meningitis are the same pathogens that colonize the
mucosal surface of the respiratory tract; they include Streptococcus pneumoniae, Neisseria meningitidis
and Haemophilus influenzae. During or after a respiratory viral illness, colonization rates in the
nasopharynx will be up to 40%, especially in young children.

Importantly as well, one should be very careful with the Q wording when it comes to risk factors versus
causation. So, for example, if the Q version is about RISK FACTOR (association), cochlear implants remain
the most significant risk factor allowing access of pathogens to the brain, especially that this Q version
is straightforward since AOM is in that scenario a remote history.
However, for a Q version in which you've repeated recent AOM episodes, and the Q is asking clearly,
"what is the most likely CAUSE of the current meningitis? Then the answer will change to AOM, especially
That cochlear implant here appears in the background (remote history). I hope this is clear to everyone.
Flipping scenarios is a very common phenomenon on PEBC exams, and if the candidate is dragged by
previous experience related to similar questions or situations, they're very likely to make mistakes on
those questions. On the other hand, candidates who are going to tackle each and every question based on
their knowledge, understanding, and acquired skills will stand a much better chance.
17. What would be the empiric therapy?
a) Vancomycin + Ceftriaxone
b) Cefotaxime + Ampicillin
c) Amoxycillin
d) Gentamycin + Ceftriaxone
Probable Pathogens and Empiric Therapy Based on Age and Host Characteristics
Age Group Bacteria Empiric Antibacterial Regimen
Infants <1 month Streptococcus agalactiae (Group B Cefotaxime + Ampicillin
streptococcus), E. coli, Listeria monocytogenes Add gentamicin if early neonatal
(rare), Neisseria meningitidis (rare), S. meningitis suspected due to synergy
pneumoniae (rare), Other Enterobacteriaceae. for Group B streptococcal infections
Children ≥1 S. pneumoniae, N. meningitidis, S. agalactiae Ceftriaxone or cefotaxime +
month (Group B streptococcus), Haemophilus Vancomycin
influenzae type b, E. coli (rare), L.
monocytogenes (rare), Other Enterobacteriaceae
Adults < 60 y E. coli S. pneumoniae N. meningitidis
L. monocytogenes (rare)
Adults with S. pneumoniae N. meningitidis H. influenzae Ceftriaxone + Vancomycin +
cellular immune type b L. monocytogenes Ampicillin
deficiency
Humoral immune S. pneumoniae N. meningitidis Ceftriaxone or cefotaxime +
deficiency states H. influenzae type b Vancomycin
Any age: CSF Staph. epidermidis S. aureus S. pneumoniae
leaks or skull N. meningitidis H. influenzae b, S. pyogenes
fractures, Enterobacteriaceae Pseudomonas sp.
head trauma

18. After culture result was H. Influenza, but sensitivity pattern not known yet. What is the
suitable antibiotic?
a) Cefotaxime
b) Ampicillin
c) Gentamicin
d) Vancomycin
Antibacterial Regimens for Specific Pathogens in Uncomplicated Bacterial Meningitis
Pathogen First-line Alternative Duration
S. pneumoniae Penicillin G Ceftriaxone OR 10-14 days
Penicillin sensitive Cefotaxime
S. pneumoniae Cefotaxime or Ceftriaxone Meropenem. 10-14 days
Intermediate / High Consult infectious diseases Consult infectious
Penicillin resistance specialist diseases specialist
Sensitive to Cephalosporins
S. pneumoniae Cefotaxime or Ceftriaxone Meropenem. 10-14 days
Intermediate / High + Vancomycin Consult infectious
Penicillin resistance +/- Rifampin diseases specialist
and 3rd Gen Cephalosporins
N. meningitidis Penicillin G Cefotaxime OR 5-7 days
Penicillin sensitive Ceftriaxone
N. meningitidis Cefotaxime OR Meropenem. 5-7 days
Penicillin resistant Ceftriaxone Consult specialist
H. influenzae Type B Ampicillin Cefotaxime OR 7-10 days
Beta Lactamase Ceftriaxone
NEGATIVE
H. influenzae Type B Ceftriaxone OR Consult infectious 7-10 days
Beta Lactamase POSITIVE Cefotaxime diseases specialist
Group B Streptococcus Penicillin + Gentamicin / Ampicillin or cefotaxime 14 - 21 days
tobramycin for synergy
Listeria monocytogenes Ampicillin + Gentamicin / Sulfamethoxazole At least
tobramycin for synergy Trimethoprim 21 days
Enterobacteria Ceftriaxone or cefotaxime ± Meropenem. 21 days
Gentamicin / tobramycin

19. Child with fever and signs of otitis media for 24 hours. He had one attack previously this
year, Patient also has a penicillin allergy, what is the most appropriate action?
a) Watch and wait
b) Give antibiotics
The strategy of watchful waiting has been recommended for situations in which:
 The child is over 6 months of age. Nonsevere illness (fever < 39ºC, mild otalgia).
 Uncomplicated AOM or history of complicated AOM. No craniofacial anomalies, immunodeficiencies.
 Cardiac or pulmonary disease, down syndrome.
 Parents capable of recognizing worsening illness with ready access to medical care.

20. Later (after a week maybe) he needs an antibiotic, what is the most convenient choice?
a) Ampicillin
b) Amoxicillin / clavulanate
c) Clarithromycin
d) Ciprofloxacin
Alternatives if Alternatives if
Characteristics Therapeutic Tips First choice Treatment Failure Treatment Failure
Noted on Day 3 Noted on Day 10-28
0-6 weeks Investigate for Refer to N/A N/A
bacteremia. emergency for
AOM often due to further
gram-negative assessment; fever
bacteria may be related to
sepsis.
Age 6 weeks to 6 Treat most cases of Standard-dose High Dose High Dose
months AOM with antibiotic amoxicillin or amoxicillin / amoxicillin/clavulanate
for 10 days HD amoxicillin clavulanate or × 10 days or cefprozil
Age ≥ 6 months Consider deferring cefprozil or × 10 days or
with no risk treatment to see if cefuroxime axetil or cefuroxime axetil × 10
factors (no AOM resolves in 24- ceftriaxone IM/IV × days or ceftriaxone
frequent bouts of 48 h only if follow- 3 days IM/IV × 3 days
AOM and no up can be ensured Consider Consider
antibiotics in and if antibacterial tympanocentesis tympanocentesis
previous 3 months) therapy can be
Age ≥ 6 months initiated if High Dose
with risk factors symptoms worsen Amoxicillin
(received <2 y: treat for 10
antibiotics in days
previous 3 m) ≥2 y: treat for 5 days
Any Age Verify AOM High Dose Ceftriaxone IM/IV; 3 High Dose
Frequent AOM Treat AOM episode amoxicillin / days. amoxicillin/clavulanate
for ≥10 days clavulanate Consider × 10 days or
Consider conjugated tympanocentesis. Cefprozil × 10 days or
pneumococcal cefuroxime axetil × 10
vaccine if <5 y days or ceftriaxone
Give influenza IM/IV × 3 days
vaccine yearly, all Consider
ages tympanocentesis
Penicillin Allergy Clarithromycin Clindamycin 10 days
Verify true anaphylactic type allergy: 10 days May consider levofloxacin after consultation
hives; swollen lips, mouth, or throat; OR with an infectious disease specialist
wheezing; hypotension; vomiting and Azithromycin Consider tympanocentesis
diarrhea. Consider penicillin skin testing 3 days

21. Case: Heart failure patient EF 30%. Scenario and medication history included nitroglycerin
and indapamide. What to add to current regimen?
a) Metolazone
b) Ramipril
c) Digoxin
d) Candisartan

22. Which beta blocker can be added:


a) Bisoprolol
b) Atenolol
c) Propranolol
d) Metoprolol succinate
HF BBs are Carvedilol, Bisoprolol and Metoprolol tartarate. Choices did not include carvedilol
23. Doctor decided to add aldosterone antagonist, which one of the following?
a) Eplerenone
b) Furosemide
c) Chlorothalidone
Choices did not include spironolactone
Another version: CASE about Patient on Spironolactone and got gynecomastia, what to give?

24. A patient scenario with medication including ramipril 10mg, telmisartan 40mg, amlodipine
10 mg and vitals including BP 110/70. In one of the questions the patient complains from
dizziness, weakness and syncope. what is the recommended action?
a) Decrease ramipril dose to 5mg
b) Decrease telmisartan dose to 20mg
c) Remove telmisartan (why not?)
d) Change telmisartan to hydrochlorothiazide

25. After a period of time the same patient complained from swollen feet, what is the
responsible drug for this side effect?
a) Amlodipine
b) Telmisartan
c) Ramipril
Hypotension, flushing, peripheral edema.

Case: A female patient with history of hypothyroidism on Synthroid Levothyroxin 125 mcg and
hypertension on Enalapril [BP 135/87], wants to get pregnant. She also takes Atorvastatin.
25. Which medication she needs to stop?
a) Enalapril
b) Atorvastatin (this too)
c) Levothyroxin

26. What can you do about the BP medications?


a) Switch to Nifedipine XL
b) Switch to Irbesartan
c) No need for medication and monitor BP

27. What is your recommendation regarding Levothyroxine dose?


a) Decrease dose once she becomes pregnant
b) Lots of variation so should get TSH every 6-8 weeks
c) Should increase her dose twice a week as soon as she finds out she is pregnant
(or increase by 50%)
d) Test T4 levels
28. A patient had increased HR, TSH decreased, what is the most likely cause?
a) Decreasing dose of levothyroxine
b) Maintaining the same dose
c) Increasing levothyroxine dose
d) Switching to another medication

29. Patient got atrial fibrillation what is the risk factor?


a) High dose of levothyroxine
possible triggers of AF, e.g., alcohol, caffeine, exercise, hyperthyroidism; in most patients, there are no
consistent triggers.

30. Patient with gout, he had history of gout in the family but no acute attacks, he is on colchicine
and allopurinol for 1 year, his serum uric acid level was very slightly above normal. He developed
STEMI and so many details. What do you think?
a) Keep same dose
b) Stop Allopurinol
c) Continue both
d) Discontinue both
e) Discontinue colchicine and continue allopurinol
Allopurinol may have protective effects over ischemic reperfusion injury and reduce infarct size. ... In
patients admitted with STEMI who are candidates of thrombolytic therapy, allopurinol is associated with
better 90-minute ST resolution, lower enzymatically determined infarct size, and in-hospital MACE.
HMG-CoA Reductase Inhibitors (Statins): Colchicine may enhance the myopathic (rhabdomyolysis) effect of
HMG-CoA Reductase Inhibitors (Statins). Colchicine may increase the serum concentration of HMG-CoA
Reductase Inhibitors (Statins). Risk C: Monitor therapy
If case about colchicine and statin: keep allopurinol and decrease dose of colchicine

31. Allopurinol dose should be titerated until what?


a) Max dose
b) Achieve target serum urate level

32. Patient has Chronic GERD


a) Refer for Endoscopy
b) Refer to rule out H. Pylori
c) Start PPI???
33. which of these reconstituted ABs should be stored in refrigerator and expires after one month?
a) Cefazolin inj
b) Cotrimoxazole
c) Clarithromycin
d) Clindamycin
e) Metronidazole (expiry 90 days)
Keep Refrigerated: All reconstituted antibiotics (EXCEPT Clarithromycin and SMX/TMP), all vaccines, eye
drops (latanoprost +/- timolo and trifluridine), teriparatide, sirolimus solution, tobramycin solution for
inhalation, insulin preparations, GLP-1 analogues, erythropoietin, interferon, omalizumab, denosumab,
Clindoxyl gel (clindamycin + benzoyl), biological DMARDs, and glatiramer.

34. In order to weigh a compound on a balance with 5% error & has sensitivity error of 6 mg.
What is the minimum weighable amount?
a) 30 mg
b) 12 mg
c) 6.3 mg
d) 120 mg
We can calculate the smallest quantity that can be weighed, on a balance of known sensitivity, to maintaina
desired level of accuracy.
This weight is referred to as the least weighable quantity (L.W.Q.). = (6 / 5 * 100) * 100% = 120 mg
35. Patient diabetic on metformin and take insulin after meals and insulin NPH in morning and
bedtime, his reading: Morning  12 – 14 mmol/l, After lunch  8 mmol/l, Before supper  8
mmol/l, Bedtime  6 mmol/l, At 3:00 am  3 mmol/l. What is the appropriate change that you
can make in his regimen?
a) Decrease insulin NPH at morning
b) Decrease insulin NPH at bedtime
c) Decrease insulin before supper
We should correlate hypoglycemia first so decrease NPH at bedtime is the suitable choice

36. The patient admits to having a drink every night and double this amount during weekends.
What do you tell him?
a) Alcohol consumption can cause hyperglycemia
b) Alcohol consumption can cause hypoglycemia
c) Alcohol consumption can increase the toxicity of some of his anti-diabetics
Lactic acidosis is a rare but serious metabolic complication that may occur due to metformin accumulation.
Caution patients taking metformin against acute or chronic alcohol intake that exceeds amounts normally
accepted as safe. Excessive alcohol intake potentiates the effect of metformin on lactate metabolism,
putting the patient at risk of lactic acidosis.
Acute intake of large amounts of alcohol is also a risk factor for hypoglycemia
Lactic acidosis is characterized by elevated blood lactate levels (>5 mmol/L), decreased blood pH, increased
anion gap and increased lactate/pyruvate ratio. It presents initially with nonspecific symptoms including
malaise, somnolence, and abdominal and respiratory distress. With greater severity, it may be associated
with hypothermia, hypotension and resistant bradyarrhythmia. Lactic acidosis occurs primarily in patients
with renal insufficiency, hepatic dysfunction, or other conditions involving hypoxemia, dehydration or sepsis.
Lactic acidosis may occur in patients who undergo radiologic studies with radiocontrast dye

37. If a patient is newly diagnosed with type I diabetes and taking insulin, what is the
honeymoon phase of insulin?
a) Give insulin in dose according patient body weight
b) Decrease insulin dose in initiation of therapy
c) Give oral medication in initiation of therapy
d) Requirement for insulin is decreased (definition)

38. What is your recommendation during the honeymoon period?


a) Decrease insulin dose
b) Increase insulin dose
c) Keep the same dose but with more BGS monitoring
This phenomenon is known as the honeymoon phase. Initiate insulin therapy judiciously in order to minimize
the risk of hypoglycemia and adjust the dose every 2–3 days according to blood glucose results.
Regular home monitoring allows patients to adjust the dose of insulin in response to abnormal blood
glucose levels, effects of diet and exercise, and changing blood glucose patterns
Other Ans is A. Here is to decrease insulin dose, not to monitor more. the trick in the question is when to
start insulin start at low dose and increase with low dose also with frequent monitoring. But when the
patient experience Insulin Honeymoon which may be experienced or not; at this time decrease the dose or
even no insulin.

39. 19 years old obese epileptic female pt on phenytoin have unprotected sex and use no
contraceptive, came to your pharmacy and wants the morning after pill. This second time this
month. She wants to use something long-term but is concerned about gaining weight. What
suitable option?
a) Combined oral contraceptive pills
b) Depoprogesterone injection
c) Copper-T intrauterine device IUD
The Canadian Paediatric Society recommends long-acting reversible methods of contraception (LARCs) as
first-line options for all young women (especially copper IUD and levonorgestrel IUS), regardless of risk of
contraceptive failure.

40. What is the problem with Nuvaring?


a) Insert it on the 5th day of menstruation
b) Can be re-inserted after 1-3 hours of removal without problems
c) If bleeding did not start after 3 days from having the ring for 3 weeks, do pregnancy test
d) Can expelled from vagina if there is straining
Special precautions for storage
 Prior to dispensing: 3 years, store in a refrigerator (2 °C - 8 °C).
 At the time of dispensing: The dispenser places a date of dispensing on the packaging. The product
should be inserted no later than 4 months from the date of dispensing, but in all cases prior to the
expiry date, whichever comes first.
 After dispensing: 4 months, store between 2°C - 30°C. May leave at room temperature for up to 4
months prior to insertion. Store in the original package in order to protect from light and moisture.
 Can be expelled from vagina while emptying bladder, especially during severe straining. If the ring is
accidentally expelled, it can be rinsed with cool to lukewarm (not hot) water, reinserted immediately
 Can be started at any time during your cycle. If start using it during the first 5 days of period, it will
work right away and there will be no need backup birth control. But if start on any other day of your
cycle, it takes 7 days to be protected from pregnancy.
41. An asthmatic patient with history of angina, he is taking BB, ASA, Symbicort (Budesonide/
Formeterol). He now has SOB, uncontrolled asthma. What he needs?
a) Pt is experiencing uncontrolled
asthma, increase asthma meds
b) Pt is experiencing uncontrolled
angina and is in need of another
medication
c) Change BB to ARB
d) Change BB to Amlodipine
e) Salbutamol
BBs is a worsening factor for asthma in this
case, so, we need to change it to a safer and
effective option which is amlodipine.

42. A hospitalized patient is admitted for acute exacerbation of COPD. His physician calls you
for your opinion regarding adding corticosteroid to his medications. Adding a steroid will:
a) Decrease mortality
b) Shorten the hospital stay
c) Improving quality of life
d) Improve lung function
Oral corticosteroids improve lung function and shorten length of hospital stay in all patients and reduce risk
of early relapse or hospitalization for subsequent acute exacerbations. They should be given within 30 days
of an acute exacerbation to reduce the risk of further exacerbations. Administration beyond 30 days is not
recommended. A 5-day course of oral prednisone 30–40 mg/day or equivalent is sufficient and has shown
outcomes comparable to a 10- to 14-day course. Tapering is unnecessary for oral corticosteroid courses that
last <2 weeks. There is no role for oral corticosteroid maintenance therapy for patients with COPD. There is
no advantage to using IV corticosteroids.

43. A patient has the following lab results: Normal ferritin, normal Hb, high MCV. What kind
of anemia is it?
a) Macrocytic-Hypochromic
b) Normocytic-Hyperchromic
c) Macrocytic-Normochromic
d) Microcytic anemia
Macrocytic anemia is a type of anemia that causes unusually large red blood cells. Deficiencies in vitamin B-
12 or folate often cause macrocytic anemia, so it is sometimes called vitamin deficiency anemia.
Hypochromic anemia is a generic term for any type of anemia in which the red blood cells are paler than
normal. (Hypo- refers to less, and chromic means chrome.)
Normochromic anemia is a form of anemia in which the concentration of hemoglobin in the red blood cells is
within the standard range, but there is an insufficient number of red blood cells.
Microcytic anemia is defined as the presence of small, often hypochromic, red blood cells in a peripheral
blood smear and is usually characterized by a low MCV (less than 83 micron 3). Iron deficiency is the most
common cause of microcytic anemia.
Hyperchromic anemia: an anemia with increase of hemoglobin in individual red blood cells and reduction in
the number of red blood cells.

44. The patient was diagnosed with pernicious anemia. Which supplement does she need?
a) Folic acid + Vit. B12
b) Vit. B12
c) Iron + Folic acid + Vit. B12
d) Iron
Megaloplastic anemia is B12 and Folic acid deficiency

45. Dr prescribed him Cyanocobalamin injection IM weekly for his pernicious anemia.
Manufacturer stopped production for a long time with no plan to be in the market in the near
future. What should you do:
a) Give the patient PO high dose cyanocobalamin
a) Order cyanocobalamin by SAP
b) Call manufacturer to ask him to reproduce cyanocobalamin
c) Wait till it come
Pernicious anemia: 100 mcg daily SC/IM × 1 wk; 200 mcg weekly SC/IM until Hb normalizes
Vitamin B12 deficiency: 30 mcg daily SC/IM × 5–10 days
Vitamin B12 has traditionally been given parenterally because deficiency is most often due to malabsorption,
and most cases of malabsorption are attributable to pernicious anemia with its lack of intrinsic factor.
High-dose oral vitamin B12 therapy is effective, feasible and cost-effective, but limitations include patient
adherence and the need for more attentive monitoring.

46. US patient, canadian resident. To get his prescription, is it legal to ship medication from
USA or not:
a) Legal ship in mail
b) In person with prescription and pharmacy packaging
c) Not allowed
d) Can be sent but the shipment must specify medication details
Importations of prescription drugs by Canadian residents are not permitted by mail or courier.
So as not to interrupt a course of treatment, Health Canada may use enforcement discretion to permit a
Canadians returning from abroad to bring with them on their person a single course of treatment or a 90-
day supply based on directions for use, whichever is less, of prescription drug.
47. A patient on Dabigatran will shift to Warfarin for renal impairment. How to shift?

48. Drug interactions regarding QT prolongation, which of the following interacts with
Citalopram?
a) Domperidone
b) HCTZ
c) Erythromycin
d) Amiodarone
According to lexicomb, avoid use of amiodarone with citalopram, while with domperidone you should adjust
dose, but with erythromycin, you should only monitor therapy.
49. Power of a study where number of patients was less than the NNT

50. A question regarding PER PROTOCOL analysis:


a) it is most widely used, dec power to detect difference
Intent to treat (ITT) analysis
 Means all patients who were enrolled and randomly allocated to treatment are
included in analysis and are analyzed in the groups to which they were
randomized. “Once randomized always analyzed”
 It is a statistical approach used for RCTs that taken into consideration all subjects
in treatment and control group including any non-adherence to the study protocol
(e.g. taking/administering treatment by the patient, dropout or protocol deviation).
It ignores noncompliance, withdrawal, protocol deviation, or anything happened
after randomization.
 Advantages: Avoid overestimation of drug effect, maintain sample size, reduce type I error & more
consistent with clinical practice.
 Disadvantages: susceptible to type II error & dilution of the drug effect.
Non-intent to treat analysis (Non-ITT): Aim to estimate the effect of treatment as delivered or as received (as
opposed to assigned) to account for non-adherence. Can be used as interim.
A per-protocol analysis: Opposite end of the spectrum from ITT analysis. It is an interpretation of randomized
clinical trial results that removes data from patients who didn't comply with the protocol. Imagine a trial
designed to test experimental drug A against standard treatment B. If some patients drop out of the trial before
investigators can measure the primary outcome, a per-protocol analysis wouldn't include their results with
patients who completed treatment. Its results represent the best-case treatment results that could be achieved if
the study sample were retained and remained compliant with treatment.

51. Lorazepam- patient is an alcoholic and wants an early refill because he’s going to the U.S.
for a week. He has two refills left, what do you do?
a) Call the doctor for early refill (if doctor have written interval)
b) Authorize one week that cover his trip??
c) Dispense one week but don’t bill it now

52. A child using Concerta for ADHD. He spends the weekend at his grandmother who calls
you wondering why her grandson who is just fine needs an amphetamine, and refuses to give
him the medication until you give her an explanation. You should:
a) Tell her he needs the drug to be calm
b) Tell her that he has ADHD and this is the ttt
c) Tell her that she needs to talk to his parents
d) Tell her that if he misses the doses it will be a huge problem.
53. Adult pt has ADHD, all are symptoms that he can get to confirm this diagnosis except?
a) Unable to listen
b) Unable to concentrate
c) Hyperactivity
d) Loss of appetite & Lethargy

54. 1st line Treatment recommendation for management of ADHD in adults with BD?
a) Bupropion
b) Atomoxetine
c) Methylphenidate

55. 8 months pregnant female has nausea and vomiting, she tried ginger 500mg but she couldn’t
tolerate the heart burn side effect. What is your recommendation?
a) Start ginger 250mg and come again after a week for follow up
b) Diclectin
c) Dimenhydrinate
d) Garlic at lower dose
Diclectin is currently the only prescription drug authorized in Canada for the treatment of nausea and
vomiting in pregnancy. 2-tab QHS PO + 1-tab QAM PO + 1-tab mid afternoon PO.
Since dimenhydrinate is recommended for augmenting pyridoxine treatment in pregnancy, it can be
considered for intermittent therapy of motion sickness in pregnant patients. Promethazine may also be
used.

56. Patient traveling to endemic area with malaria for 4 wk, when should he start prophylaxis?
a) When he arrives there
b) When he gets the symptoms
c) Take chloroquine starting 1 - 2 weeks before travelling
d) Apply a combination of sunscreen and DEET

57. How many tablets needed?


a) 9
b) 12
c) 18
2 * 250 * 2 before + 4 at distination + 4 at return = 18 if started 1 week before – 20 tabs if started 2 weeks

58. What the patient should do before taking the drug?


a) Pneumonia function
b) Ocular test
c) LFT
Chloroquine Adults: 2 tablets 500 mg phosphate salt once/wk PO. Adjust dose when ClCr <10 mL/min
phosphate Children: 8 mg/kg phosphate salt (5 mg/kg base) once/wk PO to a maximum of 500 mg.
Start 1–2 wk prior to exposure, continue during travel period and weekly for 4 wk after leaving the
endemic area
If it is not possible to initiate chloroquine 1–2 wk prior to exposure, a loading dose can be given to
rapidly achieve the effective levels:
 Loading dose, adults: 1000 mg phosphate salt taken in 2 divided doses, 6 h apart
 Loading dose, children: 16 mg/kg phosphate salt taken in 2 divided doses, 6 h apart
S.E: Common: nonallergic pruritus in African Canadians, N&V, headache, bitter taste.
Uncommon: hair depigmentation, skin eruptions, myopathy, reversible corneal opacity, partial
alopecia, blood dyscrasias. Rare: deafness, hypoglycemia (may be serious and life threatening),
nail and mucous membrane discoloration, photophobia, retinopathy.
Retinal toxicity a concern with continuous use of chloroquine, cumulative dose >100 g
chloroquine base. Baseline Screen retinal changes every 6–12 m if taking chloroquine for >5 y.
Avoid in patients with psoriasis as chloroquine may exacerbate the condition and avoid in
travellers with a history of epilepsy.
No pediatric formulation is available in Canada and drug has an exceptionally bitter taste.
Crushed tablets can be difficult to manipulate and store. The powder must be mixed with a sweet
food. Alternatively, a compounding pharmacist can prepare a flavoured suspension.
Suitable for travellers of all ages and for pregnant women.
Stomach upset may be reduced by taking these medications with food or glass of milk.

59. The same patient is traveling with 8 years kid for 8-hour flight and asks what can he do to
decrease the kid’s nausea & Motion sickness of flight?
a) Scopolamine patch (>10 yrs)
b) Dimenhydrinate (Gravol)
c) Non-sedating OTC antihistamines such as loratadine (Claritine)

60. Patient worried about his family, children and job and depressed all the day and has no
interest or pleasure in all and depressed about his future and worries if he dies what children
will do, what the assessment tool of his condition?
a) The 7-item Hamilton Depression Rating Scale (HAMD-7)
b) The Patient Health Questionnaire (PHQ-9)
c) The Mood Disorder Questionnaire (MDQ)
d) The Edinburgh Postnatal Depression Scale (EPDS)
The Patient Health Questionnaire (PHQ-9) is consisting of 9 questions that correspond to the DSM-5 criteria
for a major depressive episode.
The 7-item Hamilton Depression Rating Scale (HAMD-7) is a validated, brief, health-care professional
assessment designed to rate severity and remission.
The Mood Disorder Questionnaire (MDQ) is a useful screening instrument for manic or hypomanic symptoms.
The Edinburgh Postnatal Depression Scale (EPDS) is the most widely used and well-validated tool to screen
for depressive symptoms during pregnancy and postpartum period.
61. Naltrexone is used for?
a) Alcohol abstainence
b) Reduce alcohol craving
c) Reduce withdrawal symptoms
Naltrexone Naltrexone is an opioid antagonist, thought to decrease the euphoria related to endogenous
opioid release upon drinking alcohol, decreasing the amount of the neurotransmitter dopamine,
thus making drinking less pleasurable. Also reduces cravings for alcohol.
Treatment of choice in patients who wish to reduce use but not completely abstining.
Q. 50 mg PO once daily 1-hour befor drinking alcohol. Given potential risk of hepatotoxicity
Contraindicated with concurrent opioid therapy due to precipitation of opioid withdrawal.
Patient must be opioid-free for ≥ 7 days prior to initiation of treatment.
Use with caution if LFTs more than 5× the upper limit of normal

62. What is proper time to take Naltrexone to have a significant reduction in alcohol cravings
and intake?
a) 48 hrs after drinking alcohol
b) 72 hrs after drinking alcohol
c) One hour before drinking alcohol
d) 2 hours before drinking

Case about Naloxone kit


63. Before administering Naloxone for a patient who lost consciousness in your pharmacy:
a) You must confirm he used heroine (wrong)

64. Naloxone injection should be administered:


a) SC upper arm
b) SC lateral thigh
c) IM upper arm
d) IM lateral thigh

65. What is correct about Naloxone Kit?


a) Call 911 then give the dose
b) Inject Naloxone and call 911 if no response after 2 minutes
c) If no response after 10 min inject another injection
66. Naloxone intranasal spray use:
a) Maximum dose
b) Prime first
c) Apply when lying down

67. The antidote of benzodiazepines is:


a) Flumazenil
b) Aminophylline  antidote for
barbiturate
c) Protamine sulphate  antidote for
heparin
d) Idarucizumab  antidote for dabigatran

68. Female patient 30 years old; taking


contraceptive pills; she is a smoker and
alcoholic, suffers from generalized tonic-clonic
seizures, what maybe the cause?
a) Alcohol
b) Smoking
c) High carbs

69. Which AED to give?


a) Carbamazepine
b) phenytoin
c) Topiramate
d) Valproic acid
1st line: Carbamazepine Lamotrigine Levetiracetam
Phenytoin Valproic acid /divalproex
2nd line: Clobazam Topiramate

70. Which one can be used for Epilepsy


a) Vit E
b) Folic acid  used as supplement with phenytoin
c) Vit B12
d) ketogenic diet
71. Pharmacist was alone received a lamotrigine Rx and entered it as lamivudine and filled, tech
came from the break and released the medication to patient. How to prevent this error?
a) Flag each bag to make sure ot is not released without councelling
b) Hire more staff to make sire entering and checking done by different persons
c) Using tall man letter on the pharmacy system
d) Using tall man names on bottle’s label

72. According to ISMP, which one of these is the wrong abbreviation?


a) lamiVUDine - lamoTRIgine
b) HYDRalazine – hydROXYzine  HYDRalazine - hydrOXYzine
c) ARIPiprazole – RABEprazole
d) fentaNYL - SUFentanil

73. Pharmacy manager has a new team of interns, students and techs, applying non-maleficence
a) Immunize all team
b) Train all team
c) Insure his employees for injuries.

74. A pharmacy is in front of a busy clinic. The pharmacist offers free parking for patients
dispensing the medications from his pharmacy. One patient really needs the voucher, but she
hasn’t taken any medication in the last 18 months. What should the pharmacist response be?
a) Refuse and uphold justice
b) Agree and uphold fidelity.
c) Agree and uphold professionalism
d) Refuse, it is Illegal.

75. A 15 years old patient dispenses medication from your pharmacy. His father came to ask
you for an invoice for taxes. What should you do?
a) Give invoice with prices only without medication names
b) Give invoice with all the details
c) Refuse without consent

76. An obese patient taking Rizatripan + NSAID for migraines but only got partial relief. She
has had 6 recurrences of migraines without aura in the past 18 months which are causing her
stress and started to need days off from work. This time she went to the ER and they gave her a
Dihydroergotamine DHE injection and now she feels better. She comes to your pharmacy 12
hours later with a prescription of Naratriptan. What do you tell her?
a) Take it as soon as you notice aura.
b) Wait until 24 hours after DHE injection.
CTC. Due to the risk of serotonin syndrome, do not use within 24 h of a triptan.
77. She is eligible for receiving migraine prophylaxis because:
a) She reached threshold of migraines that is needed for prophylaxis
b) She exceeded the number of attacks per year
c) Because it’s affecting her daily life as she has to take days off
d) Because she had to take DHE
Consider prophylaxis if migraine attacks have a significant impact on the patient’s quality of life despite
appropriate use of abortive therapies, or if the frequency of attacks puts the patient at risk of medication-
overuse headache. There is little evidence on which to recommend an optimal duration of prophylaxis.
Guidelines suggest a trial of at least 2 months, following dose titration, before assessing benefit. Advise
patients to maintain a headache diary to monitor headache triggers, frequency and intensity, menstrual
cycle, use of preventive and abortive medications, and side effects. Successful prophylaxis is usually defined
as a ≥50% reduction in headache frequency or days with headache, though some patients may report
improved response to abortive therapy or decreased headache severity or duration. If there is no benefit
after a 2-month trial at the target/optimal dose, try a different medication. If prophylaxis is deemed
beneficial, continue for 6–12 months then consider tapering the dose to assess ongoing need. If headache
intensity and frequency increase, the dose can be increased to previously effective levels. Clinical guidelines
include recommendations for the following medications for migraine prophylaxis. Consider efficacy, adverse
effects, comorbid conditions (such as hypertension, depression, obesity), migraine severity and patient
preference when deciding which prophylactic agents to try

78. She got a prescription for Topiramate. This drug can help her with:
a) Decreasing her weight
b) Treating her migraine 9it is prophylactic not ttt)

79. What bad side effects could topiramate cause that might be an issue to this patient?
a) Teratogenic
b) Steven Johnson syndrome
c) Metabolic alkalosis (acidosis is right)
d) Interaction with NSAIDs
Serious skin reactions (Stevens-Johnson Syndrome [SJS] and Toxic Epidermal Necrolysis [TEN]) have been
reported in patients receiving. The majority of cases have occurred in patients concurrently taking other
medications that are known to be associated with SJS and TEN. There have also been several cases in
patients receiving monotherapy. The most frequently reported latency (half of cases where latency was
assessable) was 3 weeks to 4 months after initiating TOPAMAX therapy. It is recommended that patients be
informed about the signs of serious skin reactions. If SJS or TEN are suspected, use of TOPAMAX should be
discontinued.

80. Obese women on Metformin, Synthroid, ACEIs, she wants to be pregnant & comes asking
about folic acid. What is the appropriate dose of folic acid for her?
a) Folic acid 0.4 mg will be enough
b) Should increase her folic acid to 5 mg
c) Should increase her folic acid to 1 mg
Because she is not on any medications that can lead to neural tube, but if obese BMI> 35, she will take 5 mg
Although this is rare, if BMI is over 30, pt have 3 times the risk of a neural tube defect than a woman with a
BMI below 30.

81. If she becomes pregnant, which meds other than ACE-I should she cease:
a) Metformin
b) Synthroid

82. what should she use instead of the ceased drug?


a) Liothyronine
b) Propylthiouracil
c) Insulin
d) Glyburide

83. which one is considered a prescribing cascade?


a) Donepezil  Constipation  Senna
b) Tolterodine  Confusion  Risperidone
c) Roseglitazone  edema  furosemide
Prescribing cascade is defined as the situation in which a first drug administered to a patient causes adverse
event signs and symptoms, that are misinterpreted as a new condition, resulting in a new medication being
prescribed. The cascade may have multiple steps and differ in complexity and severity.

84. Teenager comes to your pharmacy with mild acne, not inflamed, open comedone and
closed comedones which drug can be used?
a) Glycolic acid 2%  Available without prescription.
b) Azaleic acid 15%  In Canada it is not officially indicated for use in acne.
c) Isotretinoin  for severe acne
d) Clindamycin 1%  Avoid using as monotherapy to limit bacterial resistance.

85. After 3 month he comes with very inflamed acne, before one month gave him benzoyl
peroxide + clindamycin in the morning and Adapalene 0.1% in the evening and no
improvement, what should you do?
a) Tell him to return to doctor to prescribe oral antibiotic
b) Tell him to return to doctor to presribe oral isotretinoin
c) Tell him to wait for 2-3 months
86. 6 year with OM, runny nose and pain, came with his mother to your pharmacy after 24 hr,
what u should give?
a) Azithromycin
b) Amoxicillin
c) Acetaminophen and follow up.

87. A child has sinusitis, Dr. Wrote a prescription for Cefprozil, what to counsel
a) Keep out of refrigerator
b) Take with or without food
c) Store at refrigerator
Cefprozil tablets and powder for suspension should be stored at room temperature (15–30°C). The powder
for suspension should be refrigerated (2–8°C) once reconstituted with water and discarded after 14 days.

88. Child took a dose of Cefprozil then he got hives after 1 day, his mother called the
pharmacist and told him that he got anaphylactic shock when he took 1st dose last year, what is
the possible reaction from the pharmacist
a) Give him diphenhydramine and ask to see his dr. (plus stop Cefprozil)
b) Nothing will happen
c) Refer to Emergency

89. Diabetic patient coming in to hospital, gets a prescription for: Apidra (insulin glulisine) 40
units before each meal, Lantus (insulin glargine) 40 units at bedtime & Metformin 500 mg BID.
What the problem in this prescription?
a) Dose of glulisine 40 units  should be 20 units after meals based on basal 40% & bolus 60%
b) Dose of Glargine 40 units
c) Metformin

90. Children with mild asthma persistent, his parents do not want CS, the physician asks you
about the benefit of leukotriene compared to corticosteroids in asthma; leukotrienes can?
a) Reduce bronchial remodeling
b) Decrease the need for SABA
c) Control Asthma symptoms

91. 68 yrs Patient with persistent atrial fibrilliation with cardioversion, hypertension,
dyslipidemia, Calculate CHADS2 score?
a) 0
b) 1
c) 2
d) 3
CHADS2 score Congestive heart failure 1, Hypertension 1, Age >75 years (female >65years) 1, Diabetes 1, prior Stroke 2
92. His EFV is 30%, and has renal failure, what to give to maintain rhythm control?
a) Amiodarone
b) Dronadarone
c) Sotalol
d) Digoxin
No dosage adjustment is required for patients with renal dysfunction, end-stage renal disease or dialysis.
Recommendations regarding dosage adjustment for patients with renal dysfunction, end-stage renal
disease or dialysis are based on intravenous data, and may not be representative of the oral formulation.

92. What anticoagulant should


you give as a prevention to this
patient?
a) Clopidogrel
b) Warfarin
c) Prausgrel
d) Ticagrelor

93. Girl 26 years was afraid from thrombosis as her sister 23 years had VTE (venous thrombo-
embolism) before, how to diagnose it?
a) Factor V Leiden  for stroke
b) D-Dimer tests  for Intermittent Claudication also
c) PT test  Measures deficiencies in factor II, VII, IX, X (2, 7, 9 and 10)

94. In order to to reduce the chances of further exacerbations of COPD, Pulmonary


rehabilitation is done at?
a) First 4 weeks.
b) After 2 months
c) After drug therapy
d) After 6 weeks
Although pulmonary rehabilitation is a limited resource, patients benefit in all stages of COPD. After an
acute exacerbation, recommend initiation of pulmonary rehabilitation within 4 weeks to reduce the chances
of further exacerbations.

95. What is initial symptoms of Heat stroke


a) Dry and hot skin
b) Edema
c) Muscle cramps
d) Diaphoresis
Heat-Related Illness Spectrum
 There are 2 types of heat stroke: classic and exertional. Hallmark symptoms of both include an
elevated body temperature of over 40°C (measured rectally as this is the most reliable method to
determine body temperature compared with oral or tympanic methods; and changes in mental
status. Older individuals with predisposing health risks who are exposed to poor environmental
conditions usually present with classic heat stroke. These patients differ from exertional heat
stroke sufferers in that they typically present with hot, dry skin, and the increase in core body
temperature is relatively less pronounced. This type of heat stroke can develop over several days.
 Exertional heat stroke, as its name depicts, usually occurs in younger, healthy individuals who have
participated in strenuous physical activity. Competitive athletes and military personnel are
population groups that typically develop this form of heat stroke, distinguished by the presence of
sweat and a more marked increase in core body temperature. Symptoms can occur rapidly, often
within a few hours of the activity.
Signs and symptoms: dehydration, CNS dysfunction (delirium, seizure, coma) and hot, dry skin.
Call 911—take patient to emergency room as soon as possible. In the interim (the intervening time):
 stop activity immediately and rest in an air-conditioned environment (or if not possible, a cool,
shaded area). Remove excessive clothing
 rehydrate with an oral rehydration solution containing glucose and sodium
 ensure good air circulation around patient
 cool body (on the way to hospital) with ice water towels or packs to the groin, neck, axillae and head

96. Rivaroxaban counselling?


a) Cause constipation
b) Take with or without food  Prefer take with food to minimize nausea side effect
c) Don’t Chew the tablet
d) Avoid antacids
For patients who are unable to swallow whole tablets, XARELTO tablets may be crushed and mixed with
applesauce immediately prior to use and administered orally. After the administration of a crushed
XARELTO 15 mg or 20 mg tablet, the dose should be immediately followed by food.
A crushed XARELTO tablet may be also administered via nasogastric (NG) tube. After confirming gastric
placement of the NG tube, the crushed tablet should be suspended in 50 mL of water and administered via
the NG tube after which it should be flushed with water. Because rivaroxaban absorption is dependent on
the site of drug release in the GI tract, avoid administration of XARELTO distal to the stomach as this can
result in reduced absorption and therefore reduced drug exposure. After the administration of a crushed
XARELTO 15 mg or 20 mg tablet, the dose should then be immediately followed by enteral feeding.
An in vitro compatibility study indicated that there is no adsorption of rivaroxaban from a water suspension
of a crushed XARELTO tablet to PVC or silicone nasogastric (NG) tubing.
No studies were conducted to support the crushing and administration of crushed XARELTO 2.5 mg tablets
and crushed ASA tablets together either as a mixture with applesauce or as a mixture administered via NG
tube

97. Many pop-ups appears in the computer system made the stuff don’t care and ignore the
pops, what to do?
a) Remove it
b) Make only pharmacist review it esp for new Rx
c) Escape for ph tech and assistant

98. A child with impetigo (S Aureus), what to give to treat him?


a) Oral antihistamine like Cetirizine
b) Topical antibiotics like Mupirocin.  Need Rx too OTC is polysporins
c) Topical antifungal like Clotrimazole
d) Topical antibiotics Fusidic acid  need Rx

99. If he has crusts, what should you do?


a) Clean with saline & warm compress
b) Clean with povidone iodine
c) Start oral cephalexin

100. Proton pump inhibitors may cause


a) Hypomagnesaemia
b) Hypocalcaemia
c) Hyperkalemia

101. Patients who are taking infliximab should be monitored during the first 2 hours for?
a) LFTs
b) blurred vision
c) Hypotension
d) CBC
102. Pregnant Female 21 years, come to you and said she heared about cannabis from her
friends for her pain, and she asks about it, what to tell her
a) TCH is high percentage in inhalation
b) You need a prescription to get cannabis
c) You cannot get cannabis before you are 25 years old
d) It has no significant effect on baby  contraindicated in pregnancy & breastfeeding
Recreational cannabis from 18 years old, while medical cannabis from 25 years old

103. Cognitive and motor abilities required to drive safely are negatively affected for up to how
many hours after consuming cannabis (weed).
a) 1 hour
b) 3 hours
c) 6 hours
d) 8 hours
No driving for 4 hours (inhalation), 6 hours (ingestion), 8 hours (post-euphoric effect), and impairment may
continue for 24 hours

104. Pharmacist receives recall notification from health Canada about something inside the
injection vial, what's the pharmacist action?
a) Call patients to bring it back and give them refund
b) Call patients and let them know, if they want bring it back or not.
c) Remove all stock from pharmacy
It is type I recall, so, give the patient the priority, so, we choose A
Other opinion, go with C as we have to ensure not to sell it for other pts.
Another version: Pharmacist receives recall notification from health Canada about misspelling
in label of inactive ingredient and lot no, what's the pharmacist action? Answer C

105. Which prescription to dispense


a) Testestrone 90-tab QD repeat 3 every 30 days
b) Fiorinal ® C ¼ 90-tab QD repeat 3 every 30 days
Testesterone OD ‫ شهور وفي‬٩ ‫ انو اتكتبت الروشته وفات عليها‬transfer ‫ حبه يتعمل وال ال‬09 ‫لي‬
N.B: fiorinal  butalbital+acetaminophen+codeine. If fiorinal 1/2 means codein 30 mg
Fiorinal 1/4 means codein 15 mg then only it is under narcotic preparation
But if codein 8 mg in fiorinal, then it will be under controlled 2
106. Caucasian boy 3 years with a previous history of treated scabies, lives with his family in
the basement, he is attending a day care and after 2 wks came with oozing vesicles and was
diagnosed with Varicella, what may be the cause?
a) Age
b) Living conditions
c) Her ethnicity
d) Attending Nursery school
The risk of becoming infected with the varicella-zoster virus that causes chickenpox is higher if you haven't
already had chickenpox or if you haven't had the chickenpox vaccine.
It's especially important for people who work in child care or school settings to be vaccinated.
Varicella virus vaccine is a live attenuated virus vaccine. Two doses of varicella virus vaccine are
recommended in healthy patients ≥12 months of age.

107. When will finasteride show effect in treating BPH?


a) 1 - 2 weeks
b) 4 - 6 weeks  this is for terazocin
c) 6 months

108. It is contraindicated with?


a) Pseudoephedrine
b) Testosterone
c) Doxazocin
d) Saw palmetto
A BPH should not use pseudoephedrine but finasteride can be used with testosterone

109. Onychomycosis candidiasis toenail, taking itraconazole, after 3 months no improvement,


what is the problem?
a) Itraconazole is not a good treatment.
b) Change the drug.
c) The patient needs further assessment
Clinical efficacy of onychomychosis, after 3–6 months of therapy is expressed as:
 Complete clinical cure = 0% nail plate involvement
 Clinical improvement = < 5–10% nail plate involvement
 Incomplete clinical response = > 10% nail plate involvement (reassessment is required)
110. Which of the following factors is most likely to contribute to national back order of a
medication (widespread shortage of a
pharmaceutical product)?
a) Quality control
b) No benefits because of price competition
c) Lot recall
d) Patent expiration
e) De-listing by provincial drug plans
f) Lack of raw material
g) Pricing
h) Changing of guidelines

111. A 30-year-old woman presents at your


pharmacy with complaints of painful and
swollen joints, fever, chest pain, hair loss, mouth
ulcers, swollen lymph nodes, feeling tired, and a
red rash, dr diagnosed him with SLE, what NON
pharmacological options to advise her?
a) Smoking cessation
b) Decrease salt and caffeinated beverages
c) Use moisturizer while going out in hot
weather

112. What the 1st choice?


a) Mefloquine
b) Hydroxychloroquine
c) Prednisone

113. After few months, the case


deteriorated, what is suitable for her?
a) High dose prednisone
b) Methotrexate

114. What to monitor when using this


therapy?
a) Ophthalmologic
b) HbA1C
c) CBC
d) ESR
Baseline assessment should include complete blood count (CBC) with differential and platelet counts,
hepatic enzymes, renal function tests and chest xray. Monitor hematology at least monthly, and hepatic
enzymes and renal function every 1–2 months

115. The pharmacist by error select Methotrexate 25 mg instead of 10 mg on central processing


unit CPU, then the tech completes the Rx upon the selection, what to do to avoid Entry error?
a) Educate technician
b) Remove 25 mg from CPU
c) Ask for 2 pt identifies
d) Make independent double check
a) Tall man letter
b) Alarm on system

116. SLE controlled, she wants to get pregnant, how much to wait after achieving remission?
a) After 1 month
b) After 3 months
c) After 6 months
Patients on methotrexate and mycophenolate are counselled to avoid pregnancy. Should be stopped 3
months (methotrexate) or 6 weeks (mycophenolate) prior to attempting conception.
Leflunomide is generally avoided in patients in whom future pregnancy is a possibility.
Cyclophosphamide may cause permanent infertility depending on the cumulative dose and the age of the
patient. Intake of synthetic gonadotropin releasing hormone (GnRH), oocyte cryopreservation or embryo
cryopreservation are possible fertility preservation methods.

117. What to give a lupus patient if getting pregnant to prevent preeclampsia?


a) 81 mg ASA
b) Azathioprine
c) Prednisone
Recommendations for the management of patients with SLE during pregnancy depend on the presence of
antiphospholipid antibodies with or without antiphospholipid antibody syndrome. Low-dose ASA is given
empirically to women with SLE for preeclampsia prevention and is safe in pregnancy starting around 12
weeks’ gestation with continuation to term.
Women with antiphospholipid antibody without overt antiphospholipid antibody syndrome may be treated
with ASA with a low-dose heparin (either unfractionated or low-molecular weight heparin). Women with
antiphospholipid syndrome are treated with prophylactic full dose heparin in addition to low-dose ASA, as
the combination improves live birth rate and prevents preeclampsia respectively.
Patients on methotrexate and mycophenolate are counselled to avoid pregnancy. These medications should
be stopped 3 months (methotrexate) or 6 weeks (mycophenolate) prior to attempting conception.
Recent small cohort studies showed no adverse pregnancy outcomes after preconception low-dose MTX
exposure in males or females. Women should still be switched to a low-risk medication e.g., azathioprine or
hydroxychloroquine, and observed for disease activity over 6 months prior to conception
118. Woman had a vertebral fracture and afraid to have another one so to avoid
a) Alendronate
b) Risedronate
c) Zoledronic acid

119. Euthanasia (intentionally ending a life to relieve pain & suffering), who one is most eligible?
a) Pt in a first stage colon cancer.
b) Pulmonary fibrosis pt not responding to any treatment
c) Major Depressive disorder pt

120. Patient of breast cancer taking tamoxifen and has depression, what not to give
a) Paroxetine
b) Fluvoxamine
c) Bupropion
d) Duloxetine
All SSRIs inhibit certain cytochrome P450 isoenzymes and can reduce the clearance of many drugs such as
clozapine, methadone, mexiletine, phenytoin, pimozide or propafenone, or decrease the enzymatic
conversion of a prodrug such as clopidogrel, codeine or tamoxifen to its active form.

121. SOAP analysis and they give in the question an “diagnosis” and asks about its meaning?
a) Assessment
b) Objective
c) Plan
d) Subjective
SOAP Meaning Example
SUBJECTIVE • What the patient reports Signs & symptoms, when they start
• Info from patient’s perspective Smoking history, angina symptoms,
patient medications
OBJECTIVE • What the provider reports/measures Monitoring parameters like Exercise
(Findings) • Diagnostics, Laboratory results tolerance, lipid profile, BP, HR,
blood glucose, Cholesterol level,
x-ray, SrCr (80)
ASSESSMENT Interpretation of subjective & objective information Working diagnosis
If writer is physician, assessment will be a disease state Drug interaction due to CYP450
or condition diagnosis & explain reason for diagnosis. NYHA,
The pharmacist SOAP notes, assessment will identify Framingham score
DTP and explains why DTP needs to be corrected.
PLAN • Action plan Discontinue Drug X and initiate
• Monitoring parameters Drug Y
122. calculation for sensitivity, specificities and Positive predictive value.
A study done on HIV test kit shows sensitivity of 0.68 and specificity of 0.97 interpret?
a) Negative result for HIV positive patient
b) Positive result for HIV positive patient
c) Negative result for HIV negative patient
https://www.aidsmap.com/about-hiv/sensitivity-and-specificity-hiv-tests
low sensitivity = false negative, so many results are negative but it could be false. So, the First part is
Negative results
High Specificity, measure the true negative rate.
High Specificity = True negative. Thus, in the previous question it is Negative Pt
accordingly, its low sensitivity and high specificity should mean Negative results to negative Pt,

123. Qs about female nurse -Which of the following needs Gloves to handle
a) Hydroxychloroquine
b) Pramipexole
c) Exemestane  in other version it is cyclosporine
d) Hydromorphine
Exemestane is a drug used to treat breast cancer in women who have gone through the menopause

124. Case about patient taking warfarin, INR has been stable for
a few months, he will dental procedure. What should we do?
a) Hold warfarin before a dental procedure
b) Continue warfarin before a dental procedure
c) Decrease the dose
Dental procedure low risk no D/C warfarin

125. If the doctor decides to hold warfarin, how long to hold


before the procedure?
a) 1 day
b) 2 days
c) 5 days
d) Don’t hold

126. After the procedure, the patient returns to warfarin, however, his INR was 5.2 without risk
of bleeding, what should we do?
a) Give oral vit. K
b) Omit dose and repeat INR before 2nd dose
c) Dec dose by 50%
d) Dec dose and follow up
127. Floroquiolones side effect
a) Blood glucose level alterations
b) Kidney failure
c) Rhabdomyolysis
Blood sugar disturbances, including high blood sugar and low blood sugar

128. Regular customer to your pharmacy on an insurance plan as follow, deductible of $100. He
paid 63.25 of it. Insurance co-payment is 10% after and they cover up to $10 dispensing fees. If
the med costs $87.50 including $15 dispensing fee how much do you collect from patient?
a) 41.85 $
b) 46.5 $
c) 48.87 $
d) 45.33 $
Soln: First, Calculate the remaining of deductible 100 - 63.25= 36.75 Fees = 15 - 10 = 5
The total price of Rx us 87.5 including both the fees and the deductible
So, 87.5 – 15 - 36.75 = 35.75 10% = 3.575 The total = 45.33

129. 50-year-old male smoker, has Cancer and taking cisplatin & dexamethasone. He has bad
history of montion sickness, he came complaining for severe NV. What is the risk for NV or
What could increase the induction of Nausea and vomiting?
a) Cisplatin therapy
b) Sleep depriviation
c) Age
d) Smoking
e) History of motion sickness
f) Gender
Potential Intravenous Chemotherapy Agents Oral Chemotherapy Agents
High Carmustine, Cisplatin, Mechlorethamine, Hexamethylmelamine, Procarbazine
(>90%) Streptozocin, Cyclophosphamide (>1500 mg/m2),
CTC: Although medication is the most likely cause of nausea and vomiting in a patient receiving cancer
treatment rule out other potential causes (e.g., fluid/electrolyte abnormalities, bowel obstruction, CNS or
hepatic metastases, infections, radiation therapy). Other drugs (e.g., opioids, digoxin, antibiotics) may cause
or exacerbate nausea and vomiting; therefore, a thorough medication history is essential.
Some chemotherapeutic agents are more likely to cause nausea and vomiting than others. It is important to
consider the emetogenic potential, the dose of the chemotherapy agents and the expected pattern of emesis
of the chemotherapy regimen when choosing antiemetics.
Patient-specific factors such as <50 years of age, female gender, and a history of motion sickness, nausea in
pregnancy, or chemotherapy-induced nausea and vomiting (CINV) may raise the risk of experiencing nausea
and vomiting; therefore, antiemetic regimens must be tailored to the individual patient.
130. Same Pt, the pt given ondansetron and aprepitant 125 mg in Day 1 and 80 mg on Day 2
and day 3, he also took dexamethasone, he was also on chlorpromazine but he suffered from
vomiting in Day 2 and Day 3. What to do for the next cycle?
a) Increase dose of aprepitant on day 2 and three
b) Increase dose of chlorpromazine
c) Change ondansetron to granisetron
d) Add prochlorperazine from the 1st day

131. What is the goal of therapy of the above patient?


a) Decrease nausea and decrease vomiting
b) Decrease nausea and stop vomiting
c) Stop nausea and decrease vomiting
d) Stop nausea and vomiting
Goals of Therapy
 Prevent or minimize acute (starting within 24 hours of chemotherapy), delayed (starting >24 hours
after chemotherapy) and anticipatory (starting before chemotherapy as a conditioned response)
nausea and vomiting to maintain quality of life, help patient adherence with active treatment and
avoid treatment delays
 Decrease incidence & severity of nausea & vomiting (once it has occurred) & maintain patient comfort
 Prevent complications, such as esophageal tears, dehydration, anorexia, malnutrition, weight loss,
pathological bone fractures, metabolic alkalosis, chloride and potassium depletion

132. what to give for anticipated nausea and vomiting?


a) Lorazepam
b) Cannabinol
c) Metoclopramide
133. Aprepitant effect on dexamethasone
a) Increase its plasma conc
b) Decrease its plasma conc
c) No effect

134. Patient was on Levothyroxine 0.15mg, he was admitted to the hospital and upon contacting
his physician they discovered that he should be on 0.5mg instead and his dose was increased a
few months ago; however; the patient had lost his prescription and the pharmacist just gave him
a repeat of what he originally had on his profile. Using a root cause analysis, where can you
identify where the error occurred
a) There is no 0.5mg formulation
b) There was no dialogue between the physician and the patient regarding the change
of his dose to a higher dose
c) Pharmacist did not double check the patient’s profile and never compared it to the
original hard copy originally received from the patient
d) Pharmacist did not double check to see what the patient was taking on his profile

135. Upon dispensing error, what may let you say u r using root cause analysis: (not in exam,
but worth reading)
a) Pharmacist mistake
b) Nurse mistake
c) System problem  as dispensing starts from there
d) Physician mistake

136. Pt travelling to the Dominican & wants a Twinrix vaccine, what to counsel?
a) Transmission of hepatitis B via sexual contact can be easily prevented by use of
barrier type contraception.
b) Transmission of hepatitis B via sexual contact is unusual.
c) Twinrix can prevent Guillain-Barre Syndrome
d) Hep A Respiratory droplets transmission

137. Twinrex rapid schedule


a) 0, 7, 21 days
b) 1, 3, 6 months
c) 1, 4, 12 weeks

138. Expected reaction after vaccination?


a) Allergic reaction
139. Pregnant and tonic clonic. A pregnant patient + partial seizure is going to start topiramate,
what is Side effect or DRP? I don't have the options
Possible increased risk of oral clefts if used during the 1st trimester.
May decrease efficacy of hormonal contraceptives; adjunctive nonhormonal birth
control is recommended.

140. Young university student patient with no insurance plan came to the pharmacy to take
plan b, she doesn't use contraception, she used plan b several times, doesn't want to use coc:
a) Recommend copper IUD
b) Pharmacist to tell her that plan B efficacy may be decreased
c) Evra patch
The Canadian Paediatric Society recommends long-acting reversible methods of contraception (LARCs) as
first-line options for all young women (especially copper IUD and levonorgestrel IUS), regardless of risk of
contraceptive failure. Safe for use in nulliparous adolescents.

141. How does plan B work?


a) Prevent implantation of ovum in uterine wall
b) Make environment hostile for the sperm
c) Change timing of menstrual cycle
Plan B is believed to act as an emergency contraceptive principally by preventing ovulation or fertilization
(by altering tubal transport of sperm and/or ova). In addition, it may inhibit implantation (by altering the
endometrium). It is not effective once the process of implantation has begun.

142. Difference between viral and bacterial


meningitis, if it is viral what increases?
a) CSF WBCs
b) CSF Glucose
c) CSF Protein
d) CSF lactate level

143. When two drugs are biosimilar, this means that both drugs are?
a) Same dose, strength & route
b) They are similar to generics & they should be considered equivelent to innovative brand
c) They need bioequivelent studies to show their equivelency to their innovative brand
d) The manufacture should have their own clinical studies that shows drug efficacy to
be interchangeable
e) Can be done on automatic substitution for a brand
f) Can be compared to the brand in clinical trials.
144. Healthy patient with increased Tg and LDL. What is drug of choice?
a) Statin
b) Fibrates
c) Ezetimibe

145. Pt taking statin, his dr calls to inform about colchicine for acute gout, what is appropriate?
a) Colchicine will increase statin levels
b) No interaction between both drugs
c) Colchicine will decrease statin levels
d) Statins will decrease colchicine levels
Colchicine increases HMG CoA reductase inhibitors levels; monitor for statin myotoxicity. (Rhabdomyolysis)

146. Which of the following you can not take with CYP3A4 inhibitor grapefruit?
a) Rosuvastatin
b) Atorvastatin
c) Solifenacin
d) Rivastigmine
Ok with grapefruit (Prava-Fluva-Rosuva)

147. Patient taking quinidine and pravastatin 10 mg. doctor ordered for clarithromycin. Which
problem is more serious?
a) QT prolongation with quinidine
b) Inhibition of CYP with pravastatin

148. Most common cause of traveller’s diarrhea?


a) V. Cholera
b) E. coli

149. Family of father, mother and a 6- or 9-years old child with sulfa allergy all have
traveller's diarrhea. What is the self care treatment?
a) Bismuth subsalicylate
b) Loperamide.
c) More fluid  if ORT choose it first
d) Azithromycin  other opinion said this is the answer

150. Questions about traveler diarrhea vaccines Dukoral?


Vaccines Contains the nontoxic B subunit of cholera toxin, which has significant homology with
Vibrio cholerae the toxin of ETEC and is approved in Canada for prevention of TD caused by ETEC.
whole cell/ Prevention of enterotoxigenic E. coli diarrhea:
recombinant Adults and children ≥2 y: Primary immunization: 2 doses PO; 2nd dose administered
cholera toxin B within 7–42 days after the 1st dose and at least 1 wk before reaching destination
subunit vaccine Booster: 1 dose every 3 months if the risk is continuous
Dukoral Taken orally on an empty stomach (1 h before or 1 h after eating or drinking)
S.E: Abdominal pain, diarrhea, nausea and vomiting.
May consider for prevention of TD in persons with chronic illnesses (e.g., HF, insulin-
dependent DM, IBD, chronic kidney disease), or in those with immune suppression

151. Old woman pt crying, scared that it could be a serious issue, she has pruritus from a
recurrent UTI & stress urinary incontinence, she is taking oxybutynin, SMX/TMP and now
she is taking nitrofurantoin. She also having depression, but well controlled on citalopram 40
mg. She is waking up early morning, what is her problem?
a) Recurrent UTI
b) Insomnia
c) Urinary incontinence

152. What to do?


a) Stop now use anti histamine and go to physician
b) Use anti histamine until u see ur family doctor*
c) It will go away
d) Go to emergency
e) Add trazodone
f) Increase the dose of Citalopram.

153. Mother asking about chewable herbal product for her kid’s fever. where to look?
a) Licensed natural health product database
b) CPS
c) Micromidex
d) Harret Lane

154. Trigeminal patient on carbamazepine, what to monitor?


a) Sodium
b) Potassium
c) LFTs  ↑ liver enzymes; transient neutropenia (monitor WBCs)
d) TSH
Na is important also as it causes inappropriate antidiuretic hormone release leading to hyponatremia. It is
more dangerous in oxcarbamazepine

155. Q about Erythropoeitin? Use if Hb ≤ 100 g/L


Darbepoetin alfa: Target Hb ≤120 g/L with increase limited to 10 g/L/2 wk, but: If excessive response,
decrease dose by 40%. If still excessive, hold dose until Hb falls.
Epoetin alfa: If no response to the maximum dose after 8 wk, discontinue. Target Hb ≤120 g/L. Do not
exceed target Hb. If Hb increased by more than 10 g/L per 2 wk, decrease dose by 25%.
Iron must be administered with ESA therapy to avoid depletion.

156. Cancer patient got febrile neutropenia, what to give?


a) Piperacillin/Tazobactam
b) Ciprofloxacin
c) Amox/Clavulanic
d) Vancomycin
Neutropenia and Varies with Prevention:
febrile traditional Q. Granulocyte colonystimulating factors (pegfilgrastim and filgrastim)
neutropenia cytotoxic in patients at high risk of febrile neutropenia where chemotherapy dosage
Neutrophils white chemotherapy reductions or delays should be avoided (i.e., patients who may expect a cure
blood cells are agent, regimen from their chemotherapy).
most affected by and individual S.E: Fever, arthralgias, splenomegaly, injection site reactions.
cancer risk factors Avoid administration within 24 h of chemotherapy (can worsen neutropenia)
chemotherapy (the risk of Antibiotics can be used instead, in select patients.
(within days to febrile Employ non pharmacologic measures: minimize risk of infection (wash hands
weeks) neutropenia is thoroughly and frequently, avoid large crowds and people with colds or flu,
the greater maintain good hygiene). Monitor for symptoms of infection
concern) Management:
Rapid recognition and response to potential infection required.
Standard management of febrile neutropenia includes empiric broad-spectrum
antibiotics, clinical assessment and supportive care.
Q. Double beta-lactam therapy = ceftazidime plus either piperacillin or
piperacillin/tazobactam. Cefepime, ceftazidime, imipenem/cilastatin or
meropenem.

157. Which one of the following is schedule I (need prescription)?


a) Oral potassium salt 
b) Nitroglycerin spray  II sublingual immediate release dosage forms
c) IV B12  II
d) Fusidic acid Cream
e) Epipen  II in pre-filled syringes for anaphylactic reactions to allergens
158. Patient with enlarged inflamed & painful prostate, has kidney problems. His Culture as
follow: Amoxicillin sen, Nitrofurantoin sen, SMX/TMP sen, What AB to choose
a) SMX/TMP * 3 days
b) Ciprofloxacin * 4 weeks
c) Indomethacin
d) Tamsulosin
Cases of acute interstitial nephritis secondary to fluoroquinolone use have been reported. Crystalluria with
renal failure is a very rare event caused by precipitation of the drug, usually in alkaline urine. Avoid in
patients with alkaline urine; ensure adequate hydration to prevent this occurrence.

159. Patient prescribed oral bisphosphonates refuses to take them when heard about the side
effects osteonecrosis of the jaw, he wants to take the injection which is not covered in his plan:
a) Send back to physician for a new prescription and pay for his uncovered injection
b) Tell her that Oral Bisphosphonates don’t cause osteonecrosis of the jaw

160. Pregnant patient with previous risk of DVT, what prophylactic agent should she use?
a) Enoxaparin
b) Warfarin
c) Fondaparinux

161. The reconstitution of an antibiotic by adding 101 ml of water yields a conc of 250mg/5 ml.
The pharmacist needed double conc, so he added powder of 2 bottles together. How much water
does he need to add to get double conc?
a) 101 ml
b) 202ml
c) 400 ml
d) 51 ml
C1V1 = C2V2  C2/C1 = V2/V1 = 2 2 V2 = V1 V2 = 101/2 = 51ml

162. Dawn phenomenon and Somogyi effect question, asking which insulin dose to adjust?
Treatment and prevention
 The only way to prevent the Somogyi effect is to keep blood sugar levels stable through effective
glucose management. Anyone who finds it hard to manage fluctuations in blood sugar levels should
speak to a doctor, who will help adjust their treatment plan.
Treatment options include:
 adjusting timing of insulin administration, lowering dose of insulin before bed, changing type of insulin
 eating a snack with the evening insulin dosage
 taking into account lifestyle factors, such as stress and exercise

163. Father & his daughter, she got


Mosquito bite, it become yellow,
crusty & itchy what to give?
a) Hydrocortisone 0.5%
cream
b) Fusidic acid
c) Mupirocin
d) Topical antihistamine

164. What non pharmacological


approach do u recommend
a) Cover with bandage
b) Use ZnO Moisturizure
c) Use DEET
d) Apply ice or a cool
compress

165. Then after a couple of days father returns to you as the bite is worsen, there is oozing and
pain, and said her child was scratching, the child said he didn't scratch. What to document?
a) Daughter said it isn’t itchy while father said it is itchy
b) Itching is the cause of ttt failure
c) TTT wasn’t effective *

166. what is the best recommendation?


a) Give diphenhydramine oral
b) Increase Hc concentration
c) Refer for more investigation
If the symptoms do not improve over 24–48 hours or if the symptoms are worse, see a health-care
professional.
167. A case of shingles the family household wise mother has active shingles, father who has
previously got chickenpox when he was child, a daughter that was immunized and a grand
mother is neither had chickenpox or was immunized, who is at risk in the family?
a) Her daughter may get shingles
b) Her mother may get chickenpox
c) Basically, father is at risk to get chickenpox again because he has had chicken pox before

168. Shingrix, what is true?


a) Safe for pregnant in 3rd trimester
b) Safe for elderly > 50 yrs
c) Safe for infants < 12 months
d) Shingrix is live attenuated vaccine
e) It is a Schedule 3

Vaccines, viral
varicella virus Prevention of varicella Avoid in pregnancy.
vaccine, live (chickenpox) Adverse effects: local pain, swelling, redness. A rash, resembling
attenuated Dosage: varicella zoster virus infection, may occur, although it is generally
Varilrix, ≥12 months: 2 doses minor and self-limited.
Varivax III SC. The second shot The need for booster doses is unclear, and still under study.
should be given at Do not administer the vaccine to immunocompromised patients
ages 4-6 years. (especially those with T-cell immunodeficiency).
Q. Prevention of herpes Avoid in pregnancy. Not recommended in pediatric patients.
zoster virus zoster (shingles) Adverse effects: local pain, swelling, redness. A rash, resembling
vaccine, live Dosage: varicella zoster virus infection, may occur, although it is generally
attenuated ≥ 60 y: 1 dose SC minor and self-limited.
Zostavax II Can be considered for The need for booster doses is unclear, and still under study.
those ≥ 50 y Do not administer the vaccine to immunocompromised patients
(especially those with T-cell immunodeficiency).
Can coadministered (in different site) with pneumococcal vaccine
Can be given at least 1 y following last episode of herpes zoster
Zostavax II should be stored refrigerated at a temperature of 2 to
8°C or colder until it is reconstituted for injection.
Discard if reconstituted vaccine is not used within 30 minutes.
zoster virus Prevention of herpes No data on use in pregnant women.
vaccine, nonlive zoster (shingles) Adverse effects include pain, redness and swelling at the injection
recombinant, Dosage: site; myalgia, fatigue, headache, shivering, fever, and nausea,
adjuvanted ≥50 y: 2 doses IM vomiting, diarrhea and/or abdominal pain.
Shingrix (0.5 mL administered The need for booster doses following the primary vaccination
at 0 and 2–6 months schedule has not been established.
IM) Limited data on use in immunocompromised adults.
Wait a minimum of 8 weeks after a perceived Zostavax to give
Shingrix. zostavax — 1 year — shingrix, shingrix — 8 weeks —
zostavax, shingles — 1 year — zostavax

169. Patient wants to know disease and problem about alfa trypsin deficiency, which references
a) CTC
b) Pharmacotherapy: physiopathology of disease
c) Pubmed plus
d) Cochran's

170. Pharmacist discovered a Prescription from 6 months that was forged, what should he do?
a) Use delay technique and then call the police
b) Report it to the college
c) It is too late to do anything
d) Review with the staff how to avoid this in the future

171. Pharmacist can do delegation for technician for


a) Counsel for nicotine Spray
b) Counsel for Smoking cessation
c) Counsel for Diabetic readings
d) Counsel for glucometer

172. which one can be used for smoking cessation in pregnancy?


a) Nicotine gum
b) Nicotine patch
c) Bupropion
d) Varenicline
In pregnancy 1st line is CBT, 2nd line is NRT (always use immediate release over continuous form i.e. gum,
lozenges, inhalers). Only use patch for smoking addiction

173. A case 67 years patient has AF with hypertension (170/100) and history of cardiogenic
stroke 2 month ago, why we couldn’t use alteplase?
a) History of stroke
b) High blood pressure (must be >185/100)
Absolute contraindications:
Pericarditis, previous intracranial hemorrhage; known malignant intracranial neoplasm, known cerebral vascular
lesion, ischemic stroke within 3 months except acute stroke within 3 h; suspected aortic dissection; active
bleeding or bleeding diathesis (excluding menses); significant closed head or facial trauma within 3 months.
Relative contraindications:
History of chronic severe, poorly controlled HTN, severe uncontrolled HTN (BP >180/110 mm Hg);
previous CVA more than 3 months prior or known intracerebral pathology not covered above;
traumatic or prolonged (>10 min) CPR or major surgery (<3 wk); noncompressible venous punctures; recent
(2–4 wk) internal bleeding; pregnancy; active peptic ulcer; current use of anticoagulants.

174. What is the criteria for a new drug to be recommended in formulary?


a) Clinical trials
b) Therapeutic Index

175. Which oral anti hyperglycemic agent requires a patient written consent?
a) Liraglutide
b) Rosiglitazone  Pioglitazone also
c) Empagliflozin
d) Saxagliptin

176. A management question about instruction of narcotics be available staff where the?
a) Pharmacy manager alone  check
b) Full-time pharmacy technician
c) Part-time pharmacist
d) Pharmacy assistant

177. Reference for discontinued medications?


a) CPS
b) Drug product database (online FDA approved drugs)
c) Lexi comp

178. Where to find data about use of Vit. D in cancer?


a) Medline
b) Onchologist
c) CTC
d) Primary journal

179. Best reference to review differences between ADRs of 2 drugs?


a) Micromedex
b) RxTx
c) Rxfiles  Preferred source for drug comparisons

180. All of the following are true about vaccine storage in lab fridge except?
a) Adjust the temp from 2-8C
b) Regular size fridge but only keep it in the door
c) Remove any frost over 1 cm from the fridge
d) Measure temprature twice daily
181. Reporting Adverse effects of a drug to all except?
a) Medeffect
b) Canada vigilance
c) CADTH
d) CMIRPS

182. If you give a drug dose and then after


two half-lives give another dose how
would you compare that to another drug
that was given one 1/2 life after the first
dose
a) 100%
b) 150%
c) 300%
d) 400%
The idea is that the drug decreases for each t1/2 with this rate 100  50  25  12,5. Etc
For 1st drug, we gave dose and at second half life at 25 you give new one 100 + 25 + 100=225
For 2nd drug 100 + 50 (first half life) = 150
Ratio = 225 / 150 * 100 = 150 %

183. Which of the following are false regarding Epipen injection?


a) Take off clothes before injection  Can be given over light clothes
b) There is some residual after injection (don’t use it)
c) Pull back the plunger until you see blood
d) If symptoms are still there after 10 min, inject another epipen.

184. An anticoagulants management pharmacy, a patient presented with INR 4.6, what is the
first thing to ask about?
a) Do you have any bleeding?
b) Have you had missed any dose?

185. 62 years female in palliative care, grade 4 cancer, taking opioid, has excrociating pain, he
wants to increase threshold of dose, what u have to do?
a) Increase threshold, risk of addiction minimal
b) No, increase the threshold
c) Give acetaminophen and ibuprofen
d) Increase threshold with secondary Dr opinion*
Metastatic cancer ... we should not be afraid of addiction as pain is always intolerable and where there is
pain, there is no overdose or addiction
186. 53 yr old male age came to your pharmacy with Prescription of C. trachomatis (he has
chlamydia-STI), his niece 12 yrs, she was outgoing, then one month, she came to pharmacy
with her grandfather, the pharmacist noticed she changed her personality (shy, catch her lower
abdomen), what the pharmacist has to do:
a) Report to child abuse
b) Refer to physician
c) Report to public health authority
d) Call her mom

187. Dr in hospital prescribe IV right medication, but pharmacist read conc of medication
wrong So the nurse gave IV wrong to the pt, what you have to do?
a) Page the Dr
b) Page the pharmacist
c) Page the nurse
d) Call pt

188. 2 chemo therapies that have the similar abbreviations DOXOrubicin & IDArubicin – error was
made writing the prescription, how to prevent errors?
a) Separate on shelf
b) Nurse to confirm with pharmacist before giving medication
c) Automated rx for specific tumours and cancer tx*
d) Double check

189. Frail elderly women. Which is the best injection site IM medication
a) Deltoid*
b) Gluteal
c) Leg muscle (Upper leg anterolateral thigh muscle)
d) Lower leg
Frail no deltoid but not sure B or C, I think c is better

190. Reporting Adverse effects of a


drug in hospital to?
a) Medeffect
b) Canada vigilance
c) CADTH
d) NSIR
191. Which anticoagulants no need to monitor?
a) Rivaroxaban
b) Heparin
c) Warfarin

192. Calculate the amount of Cacao butter required to make 6 suppositories, each containing
300mg MgSO4 if each suppository 2 gm of pure Cacao butter.
Where displacement value of MgSO4 = 2.5 or 1.4?
If Displacement volume of MgSO4 is 1.4. Answer is 10.716 gm of cocoa butter needed
If DV is 2.5 ............ Answer is 11.28gm of cocoa butter needed
If DV of MgSO4 is 1.4 ................ so each 1.4gm of Mgso4 will displace 1 gm of cocoa
butter///////////// 1.4gm...........1gm so 300mg ....... ? of cocoa butter = 214 mg ///////////// so
we need for each suppository of 2gm 1.786gm of cocoa butter to reach 2gm ///// so how
much for 6 suppositories ?? 1.786 X 6 = 10.716gm of cocoa butter
If DV of MgSO4 is 2.5 and i think this was the same in the exam ....... so 2.5 gm of MgSo4
displace 1 gm of cocoa butter so ............... 2.5 of Mgso4 ..equals.... 1gm cocoa Butter
...................... 300mg of MgSO4 ............? Cocoa = 120mg ////// so 2gm of suppository - 120mg
= 1.88 gm still needed of cocoa butter to make 2gm of suppository ////// so for 6 suppositories
we need 1.88g X 6 = 11.28gm ‫وده الرقم الصح اللى أنا كنت فاكره فى االمتحان فعال‬
‫يعنى ايه‬Displacement volume ??? ‫ يعنى لو‬DV of MgSO4 = 2.5 ‫ جرام من‬5.2 ‫ معناها كل‬MgSO4 ‫بتحطيهم‬
‫ ملجم‬099 ‫ طيب هو فى المسألة بيقوللك كل لبوسه فيها‬...... ‫ جم من زبدة الكاكاو‬1 ‫فى اللبوسه هيعملوا ازاحه وهيحلوا محل‬
‫ من‬MgSO4 ..... ‫ ملجم من‬159 ‫ملجم بتكافئ‬099 ‫ نعمل مقص يطلع كل‬..... ‫طيب دول يبقوا يساووا كام من زبدة الكاكاو‬
‫ ملجم من‬159 ‫ملجم من المغنيسيوم اللى بيعادلوا‬099 ‫ وأنا حاطط‬.... ‫ جم‬5 ‫ طيب اللبوسه اساسا‬////////////... ‫زبده الكاكاو‬
5999 ‫ يبقى عاوز أزود كام زبدة كاكاو ؟؟؟‬.... ‫زبدة الكاكاو‬mg - 120mg = 1880mg = 1.88gm of extra cocoa
butter...... //////////// 1.11 ‫ لبوسات يبقى‬6 ‫ طيب انا عاوز أعمل‬..... ‫طيب دى الكمي ه اللى أنا محتاجها فى كل لبوسه‬gm
X 6 = 11.28gm
‫ جم‬1.716 ‫ ملجم هيطلع‬512 ‫ جم بتوع اللبوسه بأطرح منهم الوزن المكافئ من زبدة الكاكاو اللى هو‬2
193. t1/2 is affected by:
a) Volume of distribution
b) Renal elimination
c) SR dosage form
d) Css
Other option. B not A ..................... T1/2=0.693/K ... K is elimination rate constant ... it's affected by renal
failure and so T1/2 .... While Vd is related to the the drug... i.e drug specific .... We all know that when
patient has a kidney failure, rate of elimination decreases then T1/2 of the drug will increase. Then dose
adjustment is needed to reduce the dose or increase interval according to the drug itself...

194. Patient with ASA toxicity and on patient profile is pco2 is 30 (normal is 32-34), Po2 is
normal, bicarbonate is 4 (6-7 normal) and PH is 7.3 (7.45-7.55 normal), so the patient is having?
a) Metabolic acidosis with respiratory compensation
b) Metabolic alkalosis with respiratory acidosis
To manage any acid base disturbance question, follow this sequence
1- check for PH Normal is 7.35-7.45
 If less than this range then acidosis
 If more .... alkalosis
2- check for HCO3 level (22-26 mmol)
 If more than 26 then metabolic alkalosis
 If less than 22 it’s metabolic acidosis
3- check for Pco2 (35-45 mmHg) we must memorize
these values.
 If more than 45 then lung is the cause of
acidosis and this case will be respiratory
acidosis.
 This happens in cases of respiratory
depression, COPD exacerbation or any case of reduced respiratory activity

195. Patient taking Oxyneo 30mg BID and for breakthrough pain Percocet 5mg/4hr PRN (he
usually takes 4 doses per day), patient can't afford ext. release oxycodone so shifted both IR and
ER to hydromorphone. Dr. wants to switch to Hydromorphone BID and morphine PRN. How
much does he need if “30 mg Morphine = 6 mg Hydromorphone = 20 mg Oxycodone”? But
choices included patch and IR tabs.
a) 6 mg HM BID + 4 mg q6h PRN
b) 9 mg HM BID + 1 mg q6h PRN
c) 12 mg HM BID + 2 mg q6h PRN
d) 12 mg HM BID + 4 mg q6h PRN
196. How to counsel when to take prn hydromorphone:
a) When all other tx fails
b) When feel the onset of pain
c) Regular basis

197. A patient with ADHD the doctor prescribed 10 mg Methylphenidate BID at 7am and at
3pm., (the question never said IR or ER). The child is well controlled but the mother told the
pharmacist that her son has insomnia. What is your best recommendation?
a) Take 2 doses in the morning (20 mg) at once
b) Switch to methylphenidate ER 2 tablets at noon
c) Switch to methylphenidate ER and take once at 7am and once at 3pm
d) Take one dose at 7 and the other at 12pm
e) Take just the morning dose
Generally, should be given before 4pm to avoid insomnia. The earlier the better

198. Same pt has depression taking citalopram what ADHD med would you give?
a) keep citalopram and add Atomoxetine
b) Change escitalopram to fluoxetine & add atomoxetine
c) Stop citalopram & add atomoxetine

199. Which eyedrop should be stored in the fridge?


a) Brinzolamide/Timolol
b) Latanoprost  (after dispensing can be at room temperature for 6 weeks)
c) Ciprofloxacin
d) Tobradex

200. A new drug for osteoporosis was studied in two groups. This medication shows 4.5% of
reduced risk of fracture in group -1 that received the drug and shows 9.5% of reduced risk of
fracture in group -2 that receive the placebo. on ther hand this drug shows 1.4% of development
of esophageal ulcer in group-1 where as 0.28% of esophageal ulcer in group -2. Calculate NNT?
a) 10
b) 20
c) 30
d) 40
NNT = Number needed to treat = 1/ARR = 1 / EER – CER = 1/9.5 – 4.5 = 1/5 = 0.2 * 100 = 20

201. Calculate NNH for above study?


a) 89
b) 95
c) 80
d) 50
NNH = 1 / ARI. ARI absolute risk increase = EER – CER = 1.4 – 0.28 = 1.12
NNH = 1/1.12 = 0.89 * 100 = 89

201. Female with hyperthyroidism, DM, HTN, Depression, she has bilateral pain when she
pushes her feet and edema, her medical profile includes levothyroxine, amlodipine, citalopram
what's reason of ankle edema?
a) From diseases
b) Levothyroxine
c) Amlodipine
d) Citalopram

202. Which one cannot be given with BBs?


a) Non-DHP  can cause bradycardia and cardiodepressant effect
b) ACEIs
c) ARBs

203. Pt on isotretinoin for acne scars, what should be monitored? Question might have “except”
a) AST, ALT
b) Dyslipidemia
c) Blood sugar lever
d) Creatinine level
Ans A. B&C also measured if patient is Hyperlipidemic or Hyperglycemic only i think not routine monitoring
Systemic Drug Therapy for Acne. Q. Retinoids Isotretinoin Accutane, Clarus, Epuris
It remains the most powerful anti-acne agent, with the majority of patients achieving clearing and sustained
remission, even in the most severe cases.
It is recommended as first-choice therapy for severe papulopustular or moderate nodular acne and for
nodular or conglabate acne for many reasons: clinical effectiveness, prevention of scarring and quick
improvement of a patient’s quality of life, including minimizing depression.
Avoid taking vitamin A as Oral isotretinoin is a natural metabolite of vitamin A.
0.5 mg/kg/day PO for the first month, increasing to 1 mg/kg/day as tolerated (with a goal cumulative dose of 120
– 150 mg/kg). Alternative lowdose regimen (particularly for treatment of resistant or quick-relapsing moderate
acne): 0.25–0.4 mg/kg/day.
A complete course of therapy consists of 12-16 weeks of Isotretinoin administration.
S.E: Teratogenicity. Common: mucocutaneous dryness, myalgia, arthralgia, photosensitivity, Headache. Rare:
hypertriglyceridemia, mood disorder, possibly suicide ideation, pseudotumor cerebri, erythema multiforme,
Stevens-Johnson syndrome, toxic epidermal necrolysis.
Blood potassium increased, blood alkaline phosphatase increased, blood bilirubin increased, blood urea
increased, elevated platelet counts, eosinophil count increased, false positive tuberculosis test, gamma-
glutamyltransferase abnormal, blood cholesterol increased, glucose urine present, haematocrit decreased, protein
urine, thrombocytopenia, WBC count decreased. Elevations in levels of serum creatine kinase (monitor)
Patients should be advised to use a skin-moisturizing ointment or cream and a lip balm from the start of treatment
as isotretinoin is likely to cause dryness of the skin and lips. When necessary a sun-protection product with high
protection factor of least SPF 15 should be used.
It is recommended that blood donation for transfusion purposes be deferred during therapy and for one month
after discontinuation of treatment.
Patients who experience tinnitus or hearing impairment should discontinue treatment and be referred for
specialized care for further evaluation.
Isotretinoin has been associated with inflammatory bowel disease (including regional ileitis, colitis and
hemorrhage) in patients without a prior history of intestinal disorders. Patients experiencing abdominal pain,
rectal bleeding or severe diarrhea should discontinue EPURIS immediately.
Tetracyclines: rare cases of benign intracranial hypertension (pseudotumor cerebri): allow 7 days washout after
stopping tetracyclines before starting isotretinoin.
The patient should have two negative pregnancy tests (β-hCG in urine or serum) before starting therapy with the
first pregnancy test conducted at initial assessment when the patient is qualified for therapy by the physician.
The patient then should have a second pregnancy test with a sensitivity of at least 25 mIU/mL with a negative
result, performed in a licensed laboratory, within 11 days prior to initiating therapy. The patient has had two or
three days of the next normal menstrual period before therapy is initiated.
Pregnancy test must be repeated monthly for pregnancy detection during treatment and at one month after
discontinuation of treatment. The dates and results of the pregnancy tests should be documented.
Treatment should be discontinued if the patient develops any of the following reactions: rash, especially if
associated with fever and/or malaise, conjunctivitis (red or inflamed eyes); blisters on legs, arms or face and/or
sores in mouth, throat, nose or eyes; peeling skin or other serious skin reactions.
The following tests are required before starting Isotretinoin, at first month, then as clinically indicated:
 Serum blood lipid determinations (under fasting conditions) should be performed before Isotretinoin is
given and then at intervals (one month after the start of therapy) until the lipid response to Isotretinoin is
established (which usually occurs within four weeks), and also at the end of treatment.
 Complete blood count and differential: for early detection of leukopenia, neutropenia,
thrombocytopenia and anemia.
 Liver function tests: Increases in about 15% of ALT, AST, ALP baseline levels have been reported.
Liver function tests should be monitored before treatment and at regular intervals during treatment (one
month after the start of treatment and at least three-month intervals thereafter) unless more frequent
monitoring clinically indicated.
 Blood glucose levels: all patients and in particular patients with known or suspected diabetes should have
periodic blood sugar determinations.
 Serious Adverse Event Warnings include psychiatric disorders (depression, psychosis and, rarely,
suicidal ideation, suicide attempts, suicide, and aggressive and/or violent behaviors). Monitor psychiatric
illness before dispensing isotretenoin.

204. pt with gonorrhea and chlamydia, how long should pt avoid sex?
a) 3 days
b) 1 week
c) 2 weeks
d) No need to avoid sex
Disease Management of Partner
Anogenital warts Likelihood of transmission and duration of infectivity unknown, but condoms associated
with decreased transmission of human papillomavirus
Bacterial vaginosis No treatment
Chlamydia and Refer all recent (<60 days) partners for testing and empiric treatment.
gonorrhea No sexual contact for patient or partner until 1 wk after initiation of treatment
Genital herpes Use condoms; be aware of asymptomatic shedding
Lymphogranuloma Refer all recent (<60 days) partners for testing and empiric treatment
venereum
Pelvic Refer all recent male partners (<60 days) for examination and treatment.
inflammatory Treat empirically for Chlamydia trachomatis and/or Neisseria gonorrhoeae.
disease No sexual contact for patient or partner until 1 wk after initiation of treatment
Syphilis, late latent Test all long-term partners and any of their children who were possibly exposed during
pregnancy
Syphilis, primary, Trace-back period for primary, secondary and early latent syphilis is 3 months, 6 months
secondary and and 1 y, respectively; refer all partners for testing.
early latent Refer all partners within previous 3 months for empiric treatment even if seronegative
Trichomoniasis Treat current sexual partner.
No sexual contact until patient and partner finished treatment and are asymptomatic
Vulvovaginal No treatment; consider treatment of sex partner(s) in women with recurrent infections
candidiasis

205. what vaccine type u should not give to pregnant?


a) Varicella zoster
b) Influenza
c) Pertusis vaccin
Some vaccines are not recommended during pregnancy, such as:
 Human papillomavirus (HPV) vaccine
 Measles, mumps, and rubella (MMR) vaccine
 Live influenza vaccine (nasal flu vaccine)
 Varicella (chicken pox) vaccine
 Certain travel vaccines: yellow fever, typhoid fever, and Japanese encephalitis
Note: these travel vaccines should generally not be given during pregnancy, unless your healthcare
provider determines that the benefits outweigh the risks.
Which vaccines should I get if I am pregnant?
CDC recommends that pregnant women get two vaccines during every pregnancy: the inactivated flu
vaccine (the injection, not the live nasal flu vaccine) and the Tdap (tetanus, diphtheria, and pertussis
(whooping cough) vaccine.
Flu vaccine
CDC recommends getting the flu vaccine if you are pregnant during flu season. While flu seasons vary in
their timing, CDC recommends getting vaccinated by the end of October, if possible. Getting vaccinated later
during flu season, though, can still be beneficial. Flu vaccines have been given to millions of pregnant
women over the years, and scientific evidence shows that it is safe. Getting the flu vaccine during pregnancy
is one of the best ways to protect yourself and your baby for several months after birth from flu-related
complications.
Tdap vaccine
Pregnant women are also encouraged to get the Tdap vaccine at any time during pregnancy, but optimally
between 27 and 36 weeks of each pregnancy, to protect yourself and your baby from pertussis, also known
as whooping cough. This vaccine is recommended during every pregnancy, regardless of how long it has
been since you previously received the Tdap vaccine. If you did not get a Tdap vaccine during your
pregnancy and have never gotten it, CDC recommends that you get the vaccine immediately after giving
birth.

206. Which vaccine can you NOT give to a person on a MAB Rituximab?
a) MMR
b) Flu vaccine
c) Pneumovax
Live vaccines available in Canada that are contraindicated in immunosuppressed IBD patients include
intranasal influenza, measles-mumps-rubella (MMR), smallpox, oral typhoid, yellow fever and varicella.
Live vaccines should not be given to patients using immunosuppressive therapy until 3 months after these
therapies are stopped.

207. A contraindication to giving a vaccine


a) Syncope after previous vaccine
b) Severe febrile illness
The only contraindication applicable to all vaccines is a history of a severe allergic reaction after a prior dose
of vaccine or to a vaccine constituent. Precautions are not contraindications, but are events or conditions to
be considered in determining if the benefits of the vaccine outweigh the risks.
1st option, give it and be ready with epinephrine and other resuscitation measures .... follow up for 30 min.
may stay in supine position for a while ... if reaction developed ... give Epipen.
Immunization with INFLUVAC® TETRA should be deferred in presence of febrile illness or acute infection.

208. Naïve pt, what u should not give?


a) Fentanyl patch
b) Morphine
c) Codeine
d) Meperidine

209. First order kinetics calculation?


519. women 165 cm and 61.5 Kg has herpes zoster and got a prescription of acyclovir 13mg/kg
5 times daily for 7 days, but dose of acyclovir calculated according patient ideal body weight
for pt has BMI > 30 using equation IBW= 45.5 + 2.3 kg for each inch over 5 feet, her creatinine
clearance is 29 ml/min. How many tablets (200mg tab) you should dispense to the patient
a) 150 tabs
b) 140 tabs
c) 200 tabs
BMI > 30 ‫ىف المسائل دى بتنىس وزن المريض الىل معىط ىف المسأله ونبدأ نحسب الوزن عىل المثاىل بس لما يكون‬
‫ ى‬..22 ‫يعن لما نحسب هنا هنالقيه‬
BMI < 30‫يعن ال ينطبق عليه الحالة هنا وبالتاىل ر ىأي نحسب عىل طول عىل وزن الشخص ألنه‬ ‫ى‬
So, 13mg X 61.5 X 5 times X 7 days / 200 each tablet = 140 tablet exactly

211. New refugees, old lady, have diabetic foot without osteomyelitis cellulitis, she needs
ambutation, according to their beliefs, son is the one makes decisions, who should give consent?
a) The old lady gives verbal consent
b) The old lady gives written consent to her son
c) Her son
d) Delegate the decision to the physician
Duration of therapy is based on clinical response; however, typical treatment courses for skin and soft tissue
infections range from 7 (mild) to 21 (severe) days, and the treatment of osteomyelitis may require 4–6
weeks of parenteral or several months of oral antimicrobial therapy.

212. Patient got cellulitis in his


arm, Suspected M.O could be
a) Strept. Pneumoniae
b) Staph. Aureus

213. Antibiotic of choice


a) Oral vancomycin
b) Oral cephalexin
c) IV Ampicillin
d) IV Cephalexin

214. Duration of ttt


a) 1 week
b) 2 weeks
c) 3 weeks
215. Ph Manager receievd a complaint that a pharmacist takes many sick leave, what to do?
a) Review the history of the sick leave first
b) Do a meeting with the complainer and the pharmacist

217. One question about which is the most evidence-based study?


a) Case control in cohort study  Nested Case Control Study
b) Case series study
c) Meta analysis
The Cochrane Library (highest evidence-based information) is a collection of databases in medicine and
other healthcare specialties provided by Cochrane and other organizations. At its core is the collection of
Cochrane Reviews, a database of systematic reviews and meta-analyses which summarize and interpret the
results of medical research.

218. a patient has irregular heart rate


a) If still continue after 48 hours go to see your physician
b) Refer patient to emergency room

219. How many ml of cefprozil and bactrim in each dose for a child weighing 36 pounds and
dose of ABs given in mg/kg/day?

220. Have an 8% 75g of coal tar and how many mls of coal tar solution to give 12.5%?
Answer 1:
The difference between the two conc. is 12.5 - 8 = 4.5%
4.5% ⇒ 4.5 mL in 100 g, so X in 75 g ⇒ X = 3.75 mL
Answer 2:
If 8 mL in 100 g, so X mL in 75 g ⇒ X = 6 mL
As 12.5 in 100 g, so X mL in 75 g ⇒ X = 9.375 mL.
So, the No. of mL required = 9.375 - 6 = 3.75 mL

221. Serum creatinine was given 136 mmol/ml, and creatinine clearance needed to be calculated
to pick a dose based on creatinine clearance from a table

222. Patient BSA 1.2 and given vanco 95mg/day, if the max dose per day for (adult) is 400/
BSA, what is the max dose u should give to this patient?
Answer 1 ----- 400 1.2 ----- X So, X = 480mg

223. What is the concentration in mg/100mL of a 1 in 10 000 solution of Epinephrine


1 in 10,000 means 1 gram (1000mg) of epinephrine in 10,000 mL
To get concentration per 100 mL, will divide both sides by 100
a concentration of 10mg/100mL = 10mg/dL
224. Calculation: Mark-up gross margin and selling price.

225. Calculate quantity of water add to 75 g of 8% v/w testosterone to get concentration 12.5%
v/w?
By applying law C1V1=C2V2 75*8%=12,5%* x2 X2= 48 grams
Now 8 gm --------- 100ml 48 gm --------- X X= 600 ml
To verify the result 75gm / 600 ml * 100 = 12,5 % The required strength

226. Testesteron gel prescription for 2.5gm daily of 2% for 30 days. You only have 1% and
they come in tubes each containing 2.5 g. you have 30 tubes.
How much testosterone powder do you have to add?
a) 300 mg
b) 765 mg
c) 780 mg
227. Calculation Aminoglycoside IV in a hospital where they approximate the dose to nearest
50 dose is 420mg stock 80mg/2ml how many ml will you take?
ANSWER:
Dose = 420 So be 400 to the nearest 50
80 mg --------- 2 ml 400 mg -------- X ml X = 400*2/80 = 10 ml

228. A vial containing 300 mg of Ceftriaxone Sodium for injection is to be reconstituted using
sterile water for injection to prepare a 4 ml of injectable solution. How much water for
injections that should be added to the powder? The displacement volume of Ceftriaxone sodium
is 0.03 ml/100mg? 3.91
ANSWER
Displacement volume = the space (volume) the dissolved powder will occupy in the solution. For ceftriaxone,
each 100mg will replace 0.03mL so 300mg will replace 0.09mL
So, to prepare a 4 mL solution, you will need to add 4 – 0.09 = 3.91 mL of water for injection

229. What true about sunscreen SPF 30


a) Sunscreen SPF 30 provide double protection of sunscreen SPF 15 (time not protection)
b) Physical sunscreen SPF 30 protect against UVB only (UVA too)

230. On the hospital discharge note, there’s a note that says “stop Naratriptan” on her profile,
mark it as:
a) Mark is as “discontinued”
b) Put it on hold
c) Written plan for patient

231. Vanco 1g q8hrs was given, level was subtherapeutic 4. Calculate the new dose but based
on pharmacokinetic parameters given, Vd, Ke and clearance

232. Dabigatran:
The capsules should never be broken, chewed, or opened to take the medicine. It may be
absorbing too fast if the capsules are opened, chewed, or broken. This can cause serious
bleeding. (in other version there is a question about Edoxaban)
Direct Inhibit both clot-bound and free thrombin lla.
Thrombin Patients >75 y, use with caution & consider reducing dose to 150 mg BID PO daily.
Inhibitors For patients >75 y with 1 or more risk factors for bleeding consider 110 mg BID PO.
Dabigatran, S.E: Bleeding, dyspepsia. Antidote: Idurucizumab
Pradaxa. Q. The capsule should NOT be chewed, broken, opened or crushed.
Use with caution with other drugs acting on Pgp e.g., quinidine.
Dabigatran is recommended to be discontinued 24 hours prior to endoscopic procedures,
irrespective of renal functional status in low bleeding risk procedures.
In high bleeding risk procedures or surgeries, dabigatran is recommended to be discontinued
48–72 hours prior in normal renal function and mild renal function (CrCl > 50 mL/minute).
In moderate renal impairment (CrCl of 30–49 mL/minute), discontinue 72–96 hours before
high-risk endoscopic procedure.
If severe renal impairment (CrCl < 29 mL/min), discontinue 96–144 hours before endoscopy.
Increased risk of bleeding with ClCr < 30 mL/min

233. 8 months pregnant patient has Heart failure & Edema, she is on Furosemide and gets
ototoxicity, what to give
a) Ethacrynic acid  it is more toxic
b) Ozolinone  Ozolinone is a loop diuretic which was never marketed.
To avoid ototoxicity rate of infusion IV equal or less than 4 mg/min
Thiazides cross the placenta. Possible fetal risks include jaundice, thrombocytopenia and other adverse
reactions that have occurred in adults. Increased concentrations of uric acid and creatinine have been found
in the amniotic fluid of pregnant women near term who are taking thiazides. There is a theoretical risk to
the fetus associated with plasma volume reduction induced by thiazide diuretics. A short course of diuretics
may be appropriate in patients with severe hypervolemia that is not relieved by rest or other measures
(reducing sodium intake and placement in a recumbent position). Therapy with a thiazide diuretic may be
indicated in pathological cardiac, nephritic or hepatic edema if the benefits outweigh the risk to the fetus.

234. When to refer the above pateint? If it’s getting worse in 2 weeks

235. Boy 4 years, got urinary incontinence 2 times per week, mother is worried, what do you
tell the mother?
a) Tell her this is normal for his age (must be 5 or above)
b) Needs to start on a drug
c) Refer to dr.
d) Explain the Enuresis

236. Stable Angina patient, HR= 60 BPM, BP= 169/100, his medications include EC-ASA,
bisoprolol, perindopril, his angina isn’t controlled, what to add??
a) Amlodipine (preferred than b due to heart block)
b) Diltiazem
c) Clopidogrel

237. GH is a 63-year-old male with history of ACS but no CHADS2 risk factors. He has aspirin
allergy. He got NSTEMI, what do you recommend for secondary prevention:
a) Aspirin
b) Dipyridamole
c) Clopidogrel
d) Warfarin
e) Rivaroxaban
ACS: Antiplatelet Drugs Aspirin, clopidogrel, prasugrel, ticagrelor, ticlopidine, and glycoprotein (GP) IIb/IIIa
inhibitors are examples of antiplatelet drugs. All patients are given aspirin 160 to 325 mg (not enteric-
coated), if not contraindicated (eg, life-threatening active bleeding), at presentation and 81 mg once a day
indefinitely thereafter. Chewing the first dose before swallowing quickens absorption. Aspirin reduces short-
and long-term mortality risk. If aspirin cannot be taken, clopidogrel 75 mg orally once a day or ticlopidine 250
mg orally twice a day may be used. Clopidogrel has largely replaced ticlopidine for routine use because
neutropenia is a risk with ticlopidine and white blood cell count must be monitored regularly.

238. GH came back after 2 weeks complaining of severe chest pain radiating to his left
shoulder, he used tadalafil 18hours before. He visits your pharmacy after diagnosed or
discharged form hospital because of angina, the prescription was NG SL to be used prn, patient
falls in the pharmacy. What will be your initial response?
a) Call 911
b) Call 911 & administer 2 crushed tablet of 80 mg ASA
c) Call 911 and administer nitroglycerin SL
d) Call 911, administer ASA 2 tablet 80 mg & NG
Systolic and diastolic blood pressure may be significantly reduced
following coadministration of nitrates and phosphodiesterase 5 inhibitors.
The manufacturers of sildenafil, tadalafil and vardenafil recommend that these drugs not be used in
combination with nitrates. In situations where nitrate use is required in a patient also receiving a
phosphodiesterase 5 inhibitor, effect on blood pressure can be reduced if there is sufficient time between
doses. Separate doses of nitrates and sildenafil and vardenafil by at least 24 hours. Allow 48 hours
between tadalafil administration and nitrates.

239. What the advantage of Clopidogrel than Ticagrelor


a) Used in NSTEMI
b) Secondary prevention of Stroke

240. Patient has a prescription of Metrazole (Metronidazole) 1-tab BID, he got Methotrexate by
mistake. What the signs he may have?
a) Muscle pain
b) Nausea
Oral overdose is often due to incorrect dosage and administration (e.g., daily rather than weekly
administration by patients). Symptoms include leukopenia, thrombocytopenia, anemia, pancytopenia, bone
marrow suppression, mucositis, stomatitis, oral ulceration, nausea, vomiting, GI ulceration and GI bleeding.
Symptoms of intrathecal overdose include headache, nausea and vomiting, seizure or convulsion and acute
toxic encephalopathy.
Recommended Management: Leucovorin calcium ( folinic acid) is used to counteract toxicity of inadvertent
overdosage of methotrexate. Administer as soon as possible. See leucovorin calcium monograph.
242. Patient on metronidazole for vaginitis, what to counsel
a) Use barrier contraception till end of treatment
b) Apply sunscreen for photosensitivity
c) Take on empty stomach (with or without food)

243. Mother for baby 18 months and has Lice, what is the correct Non pharmacological
intervention?
a) Head lice are extremely contagious and children who have head lice should be isolated.
b) Children be allowed to return to school after they have begun treatment with 24 hr
c) You must stuff all your child’s belongings in plastic bags, and put them in a freezer
No-nit” policy requiring children be free of nits before returning to school has not been effective in
mitigating outbreaks. Consequently, it is recommended that parents of an affected child be notified, and
that the child not be sent home early but receive treatment with an effective pediculicide that evening, and
return to school the next morning

244. what to give


a) Permethrin  (can be given> 2 months infants)
b) Tea tree oil
c) Isopropyl myristate/cyclomethicone
d) Dimethicone
245. Pt 70 years have Urinary tract infection, dr. gives SMX/TMP what’s the duration of ttt?
a) 3 days
b) 7 days
c) 5 days
Syndrome Infecting organism & Symptoms Firs-Line Second-Line
Acute Escherichia coli (80– 90%), SMX/TMP PO × 3 Fluoroquinolone
Uncomplicated Staphylococcus saprophyticus (5– 10%), days or (ciprofloxacin,
UTI (Cystitis) Klebsiella pneumoniae, Proteus mirabilis Trimethoprim PO levofloxacin,
Occurs in factors promoting infection include sexual × 3 day or norfloxacin), PO ×
females with intercourse, use of spermicide or diaphragm Nitrofurantoin PO 3 days
normal Symptoms include internal dysuria, hematuria, × 5 days or or
genitourinary frequency, suprapubic discomfort and urgency. Fosfomycin Cephalexin PO × 7
tract. Recurrences are common. tromethamine as days
Urine Culture: Generally, not recommended. single dose PO

246. Lady came with Folinic acid RX, the pharmacist told the technician that he will find it as?
a) Folic Acid
b) Leucovorin (MTX Antidote)
c) Folate
FOLINIC ACID (5‐formyl tetrahydrofolate) In contrast to folic acid, a synthetic form of folate, Folinic
acid is one of the forms of folate found naturally in foods. Leucovorin (brand name) is used to
reduce side effects of methotrexate because it is not affected by methotrexate in the same way
that folic acid is. Leucovorin is also used in combination with fluorouracil to treat cancer of the
colon. It is also used to treat megaloblastic anemia, which can occur during pregnancy

247. Patient take solifenacin what is worse his Parkinson's, what to do?
a) Take Mirabegron
Beta3-adrenergic agonists: considered as an alternative to antimuscarinics
Mirabegron, A β3 agonist that relaxes the detrusor smooth muscle and increases bladder capacity.
extended It is used for patients with overactive bladder.
release Effective in reducing the number of incontinence and micturition episodes.
25–50 mg once daily PO. Should not crushed or chewed.
S.E: Hypertension, nasopharyngitis, urinary tract infection, tachycardia, QTc prolongation is an
uncommon but significant adverse effect.
Should not be used in severe uncontrolled hypertension.
Coadministration with antimuscarinic agents may increase risk of urinary retention
Moderate inhibitor of CYP2D6 and weak inhibitor of Pgp.
May increase level of CYP2D6 substrates (desipramine, metoprolol), Pgp (digoxin, dabigatran).
In patients with severe renal impairment, moderate hepatic impairment or taking drugs
metabolized by CYP2D6 & with narrow therapeutic index, do not exceed 25 mg once daily PO.
248. 78 yr old Patient comes mild hearing loss, impacted earwax, he had it before, no pain, no
dizziness, tinnitus, what to give
a) Olive oil
b) Carbamide peroxide 1%
c) Refer
d) Mineral Oil

249. Technician do all except


a) Report errors to med effect
b) Compounding
c) Counsel on OTC medication
d) Doing patient history medication

250. Patient medication reconciliation? Eliminate it in entrance and save it for discharge

251. Technician dispensed 13 boxes of Sudafed desloratadine and pseudoephedrine (each box
has total > 6g) what to tell store manger
a) The pharmacist should be involved in sale process like this
b) Patient needs Rx for such high quantities
c) This can be use as illicit drug meth
d) Tell them that u have to be there by law

252. Patient has Parkinson and Psychosis, what is the antipsychotic you can recommend
a) Quetiapine
Treatment of Nonmotor Issues
Depression Common in PD, SSRIs and TCAs are mainstay of therapy for depression in patients with PD.
Psychosis Also common in PD, PD medications often need to be reduced or withdrawn because of
and dementia worsening of the patient's cognitive status.
Usually, anti-cholinergics are withdrawn first, followed by MAO-Bs, Amantadine, Dopamine
Agonists, and COM-T inhibitors, lastly leaving L-Dopa
With the exception of quetiapine and clozapine, all other antipsychotics should be avoided in PD
psychosis. Quetiapine has not shown consistent benefit in clinical trials, yet is often tried first to
avoid the blood monitoring requirements with clozapine use
Avoid olanzapine as it has not been shown to be effective and is poorly tolerated.
Cholinesterase inhibitors (donepezil, rivastigmine) have modest impact on improving dementia
Orthostatic Increase as the disease progresses. If conservative measures (↑ salt intake & hydration, avoiding
hypotension alcohol) not helpful, adding domperidone, midodrine and/or fludrocortisone can be considered.
Urinary Can be treated with anticholinergic drugs; options with less blood-brain penetration (e.g.,
urgency and darifenacin, trospium) or the newer nonanticholinergic agents (e.g., mirabegron), that are less
incontinence likely to cause confusion, are preferred.
Erectile dysfunction: Sildenafil, tadalafil and vardenafil can all be used
Constipation Limit drugs that have any anticholinergic effect. Ensure the patient has proper hydration with a
high fibre diet and exercise before considering treatments like PEG, lactulose or psyllium.
Sialorrhea OnabotulinumtoxinA is efficacious for the symptomatic control of sialorrhea

253. Systemic vs topical pseudoephedrine


a) Systemic cause tachycardia
b) Systemic cause tachyphylaxis more than topical
c) Topical is more effective
d) Systemic is more effective
Oral pseudoephedrine generally has weaker effect on nasal obstruction
Topical is faster and stronger effect

254. Lung Cancer Patient on stage 4 or 5 taking IV Morphine, dr. prescribes LMWH injection
for DVT, patient don’t want to take injection, what should you do?
a) Warfarin
b) Fondaparinux
c) Rivaroxaban
d) Dabigatran
Venous Traditional Routine prophylaxis with ASA, Q. Vitamin K antagonists e.g warfarin
thromboembolic cytotoxic warfarin or low molecular are used with caution as cancer patients
events drugs, weight heparin may be required are susceptible to wide fluctuations in
(DVT, PE) Bevacizumab, for patients receiving regimens INR.
(within weeks Tamoxifen, containing dexamethasone and Q. Low molecular weight heparins
to months) Sunitinib, thalidomide or its analogues. preferred for long-term anticoagulation.
Thalidomide &
Analogues

255. DVT risk factor


a) Hypertension
b) Smoking

256. Red eye case, Patient has Gritty, red and watery eye. what to do
a) Give eye prep  it is either viral or allergic
b) Give antibiotics  No ABX
c) Refer to doctor  After 48 hrs with no improvement

257. What to counsel patient about amiodarone?


a) Tell them 2 most important things
b) Relay ALL ADRs
c) Don’t counsel
258. female patient has diagnosed with 1st episode Psychosis, dr wants to start antipsychotic for
her, what to give?
a) Perphenazine
b) Risperidone
c) Haloperidol
d) Loxapine

259. After 2 months on drug, she got weight gain, and now she wants to change to a drug which
has less weight gain effect, what is the appropriate choice?
a) Olanzapine
b) Quitiapine
c) Ziprasidone
d) Clozapine

260. After 2 months later, her case is not controlled, dr diagnosed it as persistent psychosis?
what to give?
a) Clozapine
b) Aripiprazole
c) Olanzapine
d) Quitiapine

261. Duration of treatment?


a) 6 months
b) 2 months
c) 2 years
d) Indefinite
(as it is refractory) or at least 5 years if it mentioned the min ttt duration

261. Depressed lactated women take Citalopram 40 mg, she is afraid form side effect on baby,
what to tell her
a) Stop medication and use non pharm
b) There is no harm on baby
c) The medication is secreted in milk but no harm 5%
Also, escitalopram and Sertraline can be used in pregnancy and lactation)

262. Patient obese and has hypertension needs antipsychotic medication


a) Asenapine
b) Ziprasidone
Actually, for blood pressure A is better, for body wt B is better
263. During compounding what is true about using this apparatus
a) The measuring cylinder should remain under LAF
b) The electronic balance is more accurate than the ordinary one

264. A father for Male 15 years old son came to the pharmacy asking for his son’s medications
for a financial purpose concerning insurance
a) Disclose the requested info to him
b) Disclose only the financial info without any medication
c) Disclose the requested info to him and tell him to share this matter with his son

265. Diabetic patient suffering from sleep disturbance and decrease appetite, mood disturbance
and depression and feel burning sensation in his leg, what to give
a) Ibuprofen
b) Amitriptyline
st
1 line: TCAs and Gabapentinoids or SNRIs
2nd line: Topical Lidocaone or Tramadol
3rd Line: SR Opioids

266. Which of these can be delegated to the technician to be without independently the
pharmacist supervision?
a) Best possible med history (BPMH)
b) Med – Rec
c) Monitoring patient taking methadone
d) Educate about OTC med

267. Which med causes visual disturbances and greenish-yellow vision?


a) Digoxin
b) Amiodarone
c) Amitriptyline
d) Sildenafil  blue vision

268. Medication causes dose related SJS (Steven Johnson Syndrome)


a) Lamotrigine
b) Carbamazepine
c) Phenytoin
d) Valproic acid
Lamo>CBZ>Phenytoin
269. Patient was vaccinated, site of injection got swollen, hot and hardened, what is the type of
this reaction?
a) Guillain-Barre syndrome (immune system attacks nerves: weakness, tingling in extremities->paralysis)
b) Arthus reaction (hypersensitivity to intradermal injection of a vaccine into an animal)
c) Cellulitis
Arthus reaction typically occurs after intradermal injection (in contrast to our case, in which administration
was done intramuscularly) of an antigen in a primed host who has high levels of CF antibody. ... Arthus
reactions are complement dependent and neutrophil dependent.

270. Patient was started on a long-term opioid, what to do for constipation?


a) Wait till symptoms start to bother him (start concomitantly)
b) Initiate PEG
c) Initiate bulk laxative CI
Start stimulant, if not may go with PEG
1ST line stimulant: Senna or Bisacodyl 2nd line osmotic: PEG or Lactulose

271. Patient with DM, bilateral renal stenosis, Reynaud’s, his CrCL is 52, hypertensive. What to
give for hypertension?
a) Amlodipine
b) Beta Blocker
c) ACEI
Raynaud so no BB. Renal stenosis so no ACEIs.
If the patient has bilateral renal stenosis and he takes ACEIs, he will have a high opportunity to get Acute
renal failure. So why you challenge, go to the safe option you have

272. Cause of hemolytic anemia in case of taking SMX/TMP and the patient has G6P
dehydrogenase deficiency
Hemolysis may occur if sulfamethoxazole/trimethoprim is administered to individuals who are G6PD
deficient. This reaction is frequently dose-related.
Serum potassium should be monitored in patients at risk of hyperkalemia (e.g., those receiving higher doses
of sulfamethoxazole/trimethoprim, those with reduced renal function or elderly patients).

273. Technician dispensed incorrect form of Tacrolimus due to wrong order entry and the
pharmacist didn’t counsel as he was on break, how to reduce error?
a) Pharmacist ask patient about diagnosis
b) Independent double check
c) Leave a note near the desk about ‘look alike’ ‘sound alike’ drugs
d) Have extra staff so that you don’t have one person doing the whole process, etc.
e) Separate the stocks
f) Place an alert on Tacrolimus every 24hrs
‫القاعده األكبر شوف الغلطه ايه بالظبط و اختار ايه اللى يمنعها من اصلها غلطه فى اإلدخال يبقى أمنعها فى خطوه‬
‫اإلدخال نفسها غلطه فى التحضير يبقى أمنعها فى خطوه التحضير و هكذا‬

274. Doctor calls to ask about the antibiotic recommended in guidelines for a certain case
a) CADTH  Role 3: Evidence based best practices in drug prescribing and use
b) ISMP
c) For antimicrobials

275. Diabetic patient with CKD, cr. cl is 54, on metformin 500 BID, his A1c is 8, what to add?
a) Linagliptin
b) Gliclazide XR
c) Empagliflozin
276. Mother wants permethrin cr. For her child Treating scabies, which of the following is
INCORRECT counseling?
a) Treat bedmates (ttt symptomatic and asymptomatic household)
b) Child shouldn’t go to school till free of nits (children may go to school or daycare after the initial ttt)
c) Wash all bedlinens and dry in hot air
d) Only treat sexual partner

277. Patient with Celiac disease, she is using IUS.


Now, she diagnosed with anemia. What type?
a) Normocytic
b) Macrocytic
c) Microcytic (iron difficiency anemia)
Decreased iron absorption due to disorders such as celiac
disease, autoimmune atrophic gastritis and patients who have undergone gastric bypass surgery can also
result in iron deficiency anemia. Long term IV Iron is indicated due to compromised duodenal absorption
Celiac ID is going to be refractory to oral iron, note: folic acid and B12 supplements are equally important
278. What to give
a) Fumarate
b) Gluconate
c) Sulphate
(Polysaccharide-iron complex:150mg> Fumarate 99mg>Sulphate 60 mg>Gluconate 30 mg) in 300 mg salt

279. Cause of anemia


a) Use of estrogen
b) Heavy menstrual bleeding
Common causes: heavy menstruation, GI Bleeding, Chronic kidney failure, RA, Pregnancy, malabsorption

280. How long to continue iron


a) 1 months after HB level is normalized
b) 3 months after HB level is normalized (3-6 months)
c) 6 months after HB level is normalized

281. What to give?


a) Raloxifene
b) Alendronate
c) HRT
d) Teriparatide

282. Patient with one big, non-painful chancre sore, drug of choice?
a) Penicillin G (used in pregnancy and neurosyphilis)
b) Azithromycin
2.4 million units IM × single dose (consider a 2nd dose 1 wk later if patient is pregnant)

283. Pt has prescription for Azithromycin, what do you counsel them on?
a) Separate from dairy by 2h
b) Decrease sun exposure
c) Take w/ or w/o food

284. C. difficille outbreak in hospital, how to control?


a) Wear gowns and masks for people visiting patients
b) Use alcohol rubs (doesn’t kill c. difficile)
c) Isolate patient until taking first dose of antibiotic
Also Inform public health immediately
285. Technician role in hospital: Pharmaceutical calculation / Report overdose drug reach

286. CAP: Omeprazole: intervention / hypertension

287. Pre appointment anxiety: Dentist can prescribe


a) Amitriptyline
b) Lorazepam
c) Oxazepam
d) Propranolol
Lorazepam useful as an adjunct for the relief of excessive anxiety that might be present prior to surgical
interventions
Propranolol for social anxiety
Oxazepam is useful for the short-term symptomatic relief of excessive anxiety and tension in patients with
anxiety neurosis. Anxiety or tension associated with the stress of everyday life usually does not require
treatment with an anxiolytic. Oxazepam also may be of value in relieving the symptoms of acute alcoholic
withdrawal, including acute agitation and impending delirium tremens.

288. Cinacalcet. side effect, monitoring


Calcimimetics Increases the sensitivity of the Ca++ sensing receptor to circulating serum Ca++, thereby
reducing the secretion of PTH.
Cinacalcet Onset: PTH level reduced within 2– 6 h after a dose.
Recommended for the reduction of hypercalcemia in patients with parathyroid carcinoma and
for the reduction of significant hypercalcemia in patients with PH for whom
parathyroidectomy is not clinically appropriate or is CI.
Q. Indicated for treatment of 2ry hyperparathyroidism in patients with chronic kidney disease
who are on dialysis.
S.E: Nausea, vomiting, diarrhea, myalgia, hypocalcemia.
Inhibits CYP2D6; dose adjustments may be required for concomitant CYP2D6 metabolized
drugs with narrow therapeutic index, e.g., TCA as amytriptyline, flecainide.
Food increases bioavailability.
Monitor serum Ca++, serum PO4 and PTH. Calcium (Hypocalcemia), K (Hyperkalemia) and
Testosterone Levels
It is important to counsel patients to take this medication with meals to improve
bioavailability, to divide doses and titrate slowly every 2-4 weeks.
Serum calcium should be checked within 1 week of initiation and then monthly after
maintenance doses are reached.
Moderate to severe hepatic impairment increases drug concentrations by approximately 2- to
4-fold; dosage modification may be required.

289. Tiotropium. twice or once per day… is one deep inhalation enough to deliver full dose
LAMA No priming/ 2 inhalations of the powder content of Spiriva capsule once daily/Do not
take > 1 dose in 24 hrs
290. What is correct about clinical guidelines:
a) Updated every year
b) Uses only high evidence studies or It’s only evidence base
c) Group of recommendation by experts for treating specific condition

292. Diagnosis for hypertension: office based


Diabetes 130/80 Automated office based or home reading 135/85
Non automated office based 140/90 (white coat)

293. Patient on opioid developed hyperalgesia. how to explain this to the patient?
Opioid induced hyperalgesia is when the opioid intensifying the pain

294. Digoxin toxicity case- patient had QTC increase, bradycardia, etc.
I. Asking which symptom is attributed to the digoxin toxicity
II. Part of the case was about veracity and being honest
III. Other part was what to document about interaction
Digitalis toxicity  refer to ER
 Increased by renal failure (↓ excretion), hypokalemia or hyperkalemia (potentiates drug effect).
Hyperkalemia cause acute digitalis toxicity & hypokalemia cause chronic toxicity.
 Hypothyroidism, Hypercalcemia, hypoxia, renal failure & myocarditis are predisposing factors for
digitalis toxicities.
295. Sertraline in breast feeding
a) Counsel mother not to worry, etc.
b) Tell mother that her health in the most important
c) Tell mother to stop breastfeeding

296. How do you deal with a pharmacist that keeps calling out for sick day?
a) Call to see when they’re coming back.
b) It’s within the pharmacist’s right

296. Case about Women who was incontinent, was also given docusate. She was incontinent
pooping and peeing herself- why?

297. Patient without insurance, getting a med check for 30 dollars per 15 minutes – What
ethical principal. Veracity was the only one that made sense

298. Question about getting free samples from the manufacturer


a) You can sell it to a pt w/ a valid prescription as long as you only charge the pt a
dispensing fee
b) You can only give the samples to pharmacy personnel
Samples can be given free for patients, only dispensing fees can be counted

299. Pt on Sinemet and has nausea what do you do?


a) Give gravol PRN
b) Take w/ food to reduce nausea
c) Change dose
High protein diet interferes or slows down LD absorption

300. Vancomycin rapid infusion cause red man syndrome, what to do


a) Give diphenhydramine before infusion
b) Stop and restart infusion in 60 min
c) Give infusion over 60 minutes (more than 60 mins)
d) Infusion Reactions (red man syndrome)
Following rapid IV administration, a maculopapular or erythematous rash on the face, neck, chest and upper
extremities as well as pruritus, hypotension, angioedema, chest pain, wheezing, dyspnea, urticaria and
flushing may occur. The reaction usually begins a few minutes after infusion initiation but may not occur
until after the infusion is completed and generally resolves spontaneously over several hours after the
infusion is stopped. This anaphylactoid reaction, referred to as red man syndrome, occurs in response
to vancomycin-induced histamine release and is not a true hypersensitivity syndrome. The likelihood of this
reaction is proportional to the rate of infusion and therefore vancomycin should be infused over a period
of ≥60 minutes. Pretreatment with an antihistamine (hydroxyzine, diphenhydramine) may help prevent or
minimize a reaction. In patients who have experienced this reaction, lengthen the time of infusion for
subsequent doses. Monitor blood pressure during infusion. Concomitant administration of anesthetic agents
may increase the frequency of infusion-related events.

301. Which case can you use to make NNT study?


a) Migraine prophylaxis
b) Pregnancy
c) Fertility
As it’s unethical to do studies on pregnancy & fertility

302. Celecoxib case – Woman went to a different doctor to get Rx because her doctor did not
want her to take it because of risks. She went and got it from another doctor- walk in. Asked
pharmacist NOT to tell the doctor
a) Justice
b) Autonomy
c) Non-maleficence
If you didn’t tell the doctor you uphold autonomy, but violate non-maleficence

303. Mmol for TPN and molecular weight

304. Case control, retrospective, cohort, etc. Why wouldn’t you use this study to determine if
something is on the formulary?
a) Wasn’t compared to current therapy

305. SINEMET IR to change to SINEMET Controlled Release, what do you do?


a) Wait for 8 hours
b) Decrease intervals between doses
c) Decrease dose
When patients already receiving levodopa are switched to SINEMET®, levodopa must be discontinued for at
least 12 hours or more before SINEMET® is started. SINEMET® should be substituted at a dosage that will
provide approximately 20% of previous levodopa dosage.
When patients are receiving levodopa monotherapy or SINEMET® (levodopa and carbidopa), this
medication must be discontinued at least 8 hours before therapy with SINEMET® CR is started. Dosage with
SINEMET® CR 200/50 should be substituted at an amount that eventually provides approximately 10 to 30
percent more levodopa per day. The interval between doses should be prolonged by 30 to 50 percent. This
is because when Compared to Sinemet immediate-release formulation, bioavailability of Sinemet CR is 25–
30% lower and duration of action 25–30% longer.
Because entacapone enhances the bioavailability and therefore the central effects of levodopa, it may be
necessary to adjust the dosage of levodopa during the initial days to weeks of entacapone therapy in order
to reduce levodopa-related dopaminergic adverse reactions, e.g., dyskinesias, nausea, vomiting and
hallucinations. In some cases, it may be necessary to reduce the daily dosages of levodopa by about 10-30%.
This can be achieved through either reducing the dose of the levodopa preparation itself, or by extending
the interval between doses, according to the clinical condition of the patient.

306. Pregnant female 28-week suffering from asymptomatic bacteriuria, insomnia from urinary
incontinence & having sulfa allergy, she is asking about if she should be treated?
a) Wait and see
b) Should be treated for UTI
c) Give medication for insomnia

307. What is the most suitable treatment for her?


a) Nitrifurantoin 3 days
b) Smx/Trimethoprin 3 days
c) Amoxycillin 3 days
d) Fosfomycin 3 days
Management of Treat asymptomatic bacteriuria & symptomatic cystitis with 3 - 7 days course of amoxicillin
UTI during (if the organism is known to be susceptible), amoxicillin/ clavulanate, cephalexin, fosfomycin
Pregnancy tromethamine or nitrofurantoin with appropriate follow-up.
Nitrofurantoin is usually avoided near term because of the risk of inducing hemolytic anemia in
the fetus or newborn, especially in those with G6PD deficiency; however, this toxicity is rare.
Fosfomycin tromethamine may also be used in pregnancy when the organism is susceptible.
Ceftriaxone is the preferred empiric therapy for treating pyelonephritis in pregnancy. Step
down to oral therapy once the patient is stabilized and urine culture results are available.
Avoid trimethoprim and SMX/TMP in the 1st trimester of pregnancy.
Avoid SMX in the last 6 weeks of pregnancy & Avoid fluoroquinolones in pregnancy.
Breastfeeding fluoroquinolones (ciprofloxacin, levofloxacin, norfloxacin), nitrofurantoin and SMX/TMP,
Amoxicillin and cephalosporins are compatible with breastfeeding.

308. Post Menopause woman with hot flushes treated by


a) Venlafaxine
b) Duloxetine
c) Bupropion

309. Patient case w/ medcheck. Pt on ACE and ARB?


a) Discontinue the ARB
310. VITAMIN K STAT qs. Dr. wrote order for oral but we only have subq/IM/IV available.
Check if you can give one of the available formulations orally
Administration
At present only an injectable form of phytonadione is available commercially. It is approved for sc, im or iv
administration. The injectable formulation can be withdrawn from the vial and used orally. It can be given
undiluted or diluted in water or juice just prior to administration.
Oral absorption of phytonadione requires the presence of bile salts. Supplementation of bile salts should be
considered in patients with decreased bile secretion who receive phytonadione.
In older children and adults, im injection should be made in the upper outer quadrant of the buttocks. In
infants and young children, the anterolateral aspect of the thigh or the deltoid region is preferred.
When iv administration must be used, phytonadione should be injected very slowly, at a rate not exceeding
1 mg/minute.

311. Which of the following is incorrect regarding sterilization before giving an immunization?
Don’t remember options but they were all crap

312. What’s the syringe length for influenza Vaccine?


a) 3/4 inch
b) 1 to 1.5 inch
c) 5/8 inch
d) 1 inch -1¼ inch
B is correct: 5/8 if < 60 kg, 1 if > 60 kg, 1.5 if woman > 90 kg or man > 118 kg
NOVEMBER 2018
1. Eli, 55, was rushed to the hospital after being bitten by a dog on his leg. He was coming from
a pub on a Friday night after partying with his friends until the wee hours when this stray dog
chased after him. Eli was so drunk he wasn’t able to run fast from the canine. Upon arrival at
the emergency room, the wound was oozing with blood and he describedit painful. Eli is
allergic to penicillin. Which of the following organism is commonly cultured from confirmed
animal bite infection?
a) Streptococcus pyogenes
b) Staphylococcus aureus
c) Toxoplasma gondii
d) Pasteurella multocida

2. Following Eli’s case, what is the best therapy for him?


a) Amoxicillin/clavulanate
b) Cloxacillin
c) Metronidazole plus a second or third generation cephalosporin
d) Clindamycin plus doxycycline
 Human or animal bites (e.g., cat, dog): Treat with amoxicillin/ clavulanate.
 Pasteurella multocida is commonly present in animal bites, and resistance of this organism to cloxacillin,
cephalosporins and clindamycin is documented.
 Options for second‐line therapy of animal bites include combination therapy with either clindamycin or
metronidazole plus one of 2nd or 3rd generation cephalosporin, doxycycline, SMX/TMP or fluoroquinolone.
 Second‐line options for human bites include metronidazole combined with either ciprofloxacin or
levofloxacin, or monotherapy with moxifloxacin.
 Clindamycin is a reasonable agent for treating skin and soft tissue infections in penicillin‐allergic patients,
notwithstanding the increased risk of C. difficile infection and pseudomembranous colitis.
 Immunization against tetanus and rabies (where indicate) is advised.

3. What is the initial diagnosis for asthma?


a) Spirometry
b) Peak flow meter
Spirometer (preferable method of diagnosis): reduced expiratory
flow rates with reversibility.
Forced expiratory volume per second (FEV1 normal >80%): measures how much air a person can exhale
during a forced breath. Peak flow meter (home monitoring): used to diagnose asthma or to monitor patients
with severe asthma or poor perception of airway obstruction
Bronchoprovocation challenge test, using methanacholine or histamine if diagnosis is in doubt.
Blood eosinophil count (from CBC), FeNO (fractional exhaled nitric oxide), serum IgE and sputum eosinophils
count: used to predict treatment response to certain agents.
4. 14 to 17 days bleeding due to? Heavy Menstrual Bleeding
Abnormal uterine bleeding AUB that is characterized by prolonged (>7 days) or excessive (>80 mL) menstrual
blood flow that interferes with quality of life and may result in iron deficiency anemia.
Causes (PALM‐COEIN): Polyps, Adenomyosis, Leiomyomas, Malignancy, Coagulopathy, Ovulatory dysfunction,
Endometrial factors, Iatrogenic, Not classified (uterine scars from caesarean section).
Treatment: Combined hormonal contraceptives, Progestogen‐Only Contraceptives, Gonadotropin‐releasing
hormone (GnRH) agonists, such as leuprolide, goserelin and nafarelin, NSAIDs, Ulipristal & Mifepristone

5. Insomnia + anxiety (mild depression), why we should refer to dr.?


a) Symptoms more than 3 weeks (The red flag is suicidal ideation)

6. What to give?
a) Temazepam
b) Zopiclone
c) Quitapine
d) Alprazolam or antipsychotic  (high risk of rebound anxiety: high potency short t1/2)
In case of anxiety start CBT. BZD (Short term: Temazepam). SSRI (long term)
Most common BZD of choice in anxiety is Clonazepam, also Alprazolam or Lorazepam

7. Same patient gets an Rx of Sertraline, what is your counselling:


a) Take it with food to increase bioavailability
Alcohol can increase the nervous system side effects of sertraline such as dizziness, drowsiness, and difficulty
concentrating. Some people may also experience impairment in thinking and judgment.
CPS: ZOLOFT should be administered with food once daily preferably with evening meal, or, if administration
in the morning is desired, with breakfast. Swallow the capsule whole, do not divide, crush or chew.

8. Cyclosporine counseling:
a) Patient BUN/SCr, Mg++, and blood pressure require monitoring while on therapy.
b) A cytochrome P450 3A4 inhibitor, and P‐glycoprotein inhibitor. Avoid drinking grapefruit juice or
eating grapefruit while taking cyclosporine or cyclosporine (modified).
c) If you are taking cyclosporine (modified) to treat rheumatoid arthritis, it may take 4 to 8 weeks for
your symptoms to improve. Continue to take cyclosporine (modified) even if you feel well.
d) Do not stop taking cyclosporine (modified) without talking to your doctor. Your doctor may decrease
your dose gradually.
e) You may take oral cyclosporine with or without food, but take it the same way each time. Take the
medicine at the same time each day. Measure liquid medicine carefully.
f) You may become ill if you stop taking this medicine suddenly. You may want to take Cyclosporine
with some food if the medicine upsets your stomach.
g) Nephrotoxicity is a well‐known effect of cyclosporine that causes a reduction in glomerular filtration
rate through vasoconstriction of the afferent glomerular arterioles and may result in acute renal
failure. Isolated CsA‐induced hyperkalemia occurring through different mechanisms is also common
9. A physician prescribed potassium supplements 20 m. Eq TID to patient but this patient does
not want to take supplements. He asks the physician if he can eat bananas instead. If each large
banana has 602mg potassium. How many bananas should patient eat each day? (M. wt K = 39).
Answer:
20 m. Eq TID = 60 m. Eq per day

Mg = mEq * mol. Weight / valence = 60 * 39 / 1 = 2340 mg


Each banana contains 602 mg  No. of banana = 2340/6002 = 3.88

10. Brimonidine counselling?


a) If you wear contact lenses, remove them first, wait 15 minutes after using eye drops
to put them back into eyes.
b) If gel for rosacea, then apply for whole face not red spots only.
Rosacea: Apply small, pea‐sized amount to each of the 5 areas of the face (i.e., forehead, chin, nose, each
cheek) avoiding the eyes and eyelids, lips, mouth, and inside of the nose
Brimonidine, Suppress formation of aqueous humor & may ↑ uveoscleral outflow. High specificity for the
Alphagan P alpha2-receptor and can be used for variety of open-angle & angle-closure glaucomas.
Alphagan, preserved with purite not benzalkonium chloride. 1–2 drops Q12H or Q8H
S.E: Local allergic reaction, dry mouth, tachycardia, tremor, hypotension, headache.
Brimonidine has Lower incidence of ocular allergy when used chronically.
Vision may be blurry for a few minutes after you instill the eye drops. So, do not drive a car
or operate machinery until you know how this medication affects you
After first opening: 28 days. Do not store above 25C. Do not refrigerate.
C.I with MAO inhibitors.

11. Methotrexate, Vincristine intrathecal:


 Intrathecal methotrexate is given to prevent leukaemia cells entering the cerebrospinal fluid (CSF)
around the spine and brain. It is also used to treat leukaemia found in the CSF
 Vincristine, an antineoplastic agent, must never be injected intrathecally because of its devastating
neurotoxic effects, which are usually fatal.
Another version: Which you will make IV bag rather than syringe:
a) Methotrexate
b) Vincristine
c) Biologics (Adalimumab Sc. Infliximab IV, Etanercept Sc)
Vincristine is given through a vein by intravenous injection (IV push) or infusion (IV). There is no pill form.
Vincristine is a vesicant. A vesicant is a chemical that causes extensive tissue damage and blistering if it
escapes from the vein.
Dispense intravenous (IV) vinCRIStine in a minibag of a compatible solution (e.g., 25 mL for pediatric
patients and 50 mL for adults) and never dispense and/or administer the drug using a syringe.
Prohibit IV vinCRIStine in areas where intrathecal medications are administered and/or stored.
Confirm that any prescribed intrathecal medications have been administered before dispensing IV
vinCRIStine.
Drugs typically given by IV bag
 chemotherapy drugs such as doxorubicin, vincristine, cisplatin, and paclitaxel
 antibiotics such as vancomycin, meropenem, and gentamicin
 antifungal drugs such as micafungin and amphotericin
 pain medications such as hydromorphone and morphine
 drugs for low blood pressure such as dopamine, epinephrine, norepinephrine, and dobutamine
 immunoglobulin medications (IVIG)

12. Patient with osteoporosis who takes alendronate every Wednesday every morning, she
called doctor she forgot to take this morning what she should do?
a) Take it tonight
b) Take it tommorrow morning & continue as usual
If you are on a weekly schedule and miss a dose of this medicine, take it the next morning after you
remember. Resume your usual schedule taking the medicine on your chosen day the next week

13. Bisphosphonate monitoring in old age:


 Monitor renal function (serum creatinine and/or urea). Health Canada–approved product
monographs for bisphosphonates indicate restriction of use below a GFR of 30–35 mL/min, as
bisphosphonates rely on the kidney for excretion.
 Bone mineral density should be evaluated 1 to 2 years after initiating therapy and every two years or
more frequently in patients deemed high risk.
 Annual measurements of height, weight, serum calcium, 25‐ hydroxyvitamin D and assessment of
back pain for the development of compression fractures should be done.
 Alkaline phosphatase should be monitored at 6 to 12 weeks after the treatment initiation in patients
with a history of Paget disease and should be repeated at 6‐month to 12‐month intervals.

14. Tramadol causes seizures:


 Tramadol is a centrally‐acting analgesic indicated for moderate to severe pain.
 Seizures have been reported in patients receiving tramadol within the recommended dosage range.
 Concomitant use of tramadol hydrochloride increases the seizure risk in patients taking: SSRIs, TCAs,
MAOIs & neuroleptics. The use of tramadol with serotonergic medicines can increase risk of
serotonin syndrome.
 Tramadol is classified as a regular schedule l drug; so verbal + written rx+ refills or part fills.
However, like other narcotics, controlled, and targeted drugs it is monitored by the NMS so the
patient or the person picking it up must present a valid photo ID. So, if they asked watched by the
NMS =yes.
15. Which of the following organizations aim to enhance the safety of the medication use
system for Canadians?
a) ISMP (Institute for Safe Medication Practices Canada)
b) CMIRPS (Canadian Medication Incident Reporting and Prevention System)
c) CIHI (Canadian Institute for Health Information)
d) HC (Health Canada)
ISMP Canada's mandate includes analyzing medication incidents, making recommendations for the
prevention of harmful medication incidents, and facilitating quality improvement initiatives.
The purposes of the CMIRPS program are to:
 Coordinate the capture, analysis and dissemination of information on medication incidents;
 Enhance the safety of the medication use system for Canadians; and
 Support the effective use of resources through the reduction of potential or actual harm caused by
preventable medication incidents.

16. According to ISMP, what is the correct tall man letters that help draw attention to the
dissimilarities in look-alike drug below
a) PREDNIsone & PREDNIsolone
b) predniSONE & prednisoLONE

16. Medical reconciliation is supported by:


a) Med-effect
b) ISMP
c) BPMH
d) CADTH
Medication reconciliation is a formal process in which healthcare providers work together with patients,
families and care providers to ensure accurate and comprehensive medication information is communicated
consistently across transitions of care. Medication reconciliation requires a systematic and comprehensive
review of all the medications a patient is taking (known as a BPMH) to ensure that medications being
added, changed or discontinued are carefully evaluated. It is a component of medication management and
will inform and enable prescribers to make the most appropriate prescribing decisions for the patient.
ISMP Canada is the Intervention Lead for medication reconciliation in the successful Safer Healthcare Now!
network. ISMP supports Canadian teams in acute care, long term care, home care implements medication
reconciliation and optimize communication about medications across transitions of care.
A Best Possible Medication History (BPMH) is a history created using
1) a systematic process of interviewing the patient/family; and
2) a review of at least one other reliable source of information to obtain and verify all of a patient's
medication use (prescribed and non‐prescribed). Complete documentation includes drug name, dosage,
route and frequency. The BPMH is more comprehensive than a routine primary medication history which is
often a quick preliminary medication history which may not include multiple sources of information.
The BPMH is a 'snapshot' of the patient's actual medication use, which may be different from what is
contained in their records. This is why the patient involvement is vital.
17. A patient with papulopustular Rosacea on face but does not cover the whole face (maybe
eye), which dr. should you refer to:
a) Ophthalmologist
b) Dermatologist
c) Dentist
Another version: Psoriasis spreading to
other parts to body; whom to refer?

18. Dr. prescribed topical metronidazole,


however, it doesnot seem to be working
for him. Which oral medication should
this patient be started on?
a) Minocycline
b) Erythromycin
c) Amoxillin
d) Carithromycin
e) Bromonidine
Metronidazole 0.75%, 1% gel May need up to 12 wk of therapy to show pronounced improvement. Can be
used in combination with oral tetracyclines. There does not appear to be any significant difference in
efficacy among varying strengths and vehicles. If discontinued, relapse can occur.
Minocycline: 50–100 mg daily × 6–8 wk. Can be tried if tetracycline fails after 4–6 wk trial with good
adherence to therapy. No food restriction required. Useful for improving ocular rosacea. Recurrences
of rosacea may occur after treatment is discontinued.

19. Renal failure and primary treatment for Rosacea?


Patients with rosacea, particularly those with moderate‐to‐severe disease, have an increased risk of CKD
that is not completely explained by traditional risk factors. Patients with rosacea and their physicians should
be aware of this potential link with CKD. Careful monitoring of renal function and avoidance of long‐term
use of nephrotoxic drugs should be considered as part of integrated care for patients with rosacea,
particularly those older than 50 years.
Tetracycline is the only one that needs Dosage adjustment may be required in renal impairment, but mino
and doxy no renal adjust
Rosacea is an independent risk factor for CKD. High rosacea severity and old age further increased CKD risk
in patients with rosacea. Careful monitoring for CKD development should be included as part of integrated
care for patients with rosacea
CTC: There is increasing evidence that the presence of rosacea may be a predisposing factor or a marker for
systemic disease, as associations with conditions such as migraine, inflammatory bowel disease, depression
and cardiovascular disease, among others, have been reported.
20. All of the following are predisposing factor for gout except
a) Low Dose Aspirin
b) Hydrochlorothiazide
c) Niacin
d) Sedentary Lifestyle
e) Obesity
Drugs and Conditions Associated with Hyperuricemia and Gout
 Drugs: Alcohol, Cyclosporine, Cytotoxic chemotherapy, Diuretics (thiazide and loop), Ethambutol,
Interferon + ribavirin, Levodopa, Nicotinic acid (niacin), Pyrazinamide, Salicylates (low‐dose),
Tacrolimus, Teriparatide.
 Conditions: Excessive alcohol intake, Atherosclerosis, Chronic kidney, glomerular, interstitial renal
disease, Diabetes, Hyperlipidemia, Hypertension, Ischemic heart disease, Lead intoxication,
Metabolic syndrome, Myeloproliferative disorders and some cancers, Obesity, Urolithiasis history,
rarely genetic or acquired causes of uric acid overproduction.
 Dietary factors: excessive protein diet from red meats, organ meats & shellfish,  Purine intake
(shellfish, vegetables such as asparagus, cauliflower, spinach, beans, peas & mushrooms)

21. Which of the following are not considered first choice in the treatment of acute gout?
a) NSAIDs
b) Colchicine
c) Allopurinol
d) Prednisone
NSAIDs, colchicine or oral corticosteroids are appropriate first‐line options in therapy of acute gout.
Treatment should be initiated within the first 24 hours of acute gout attack onset. For acute attacks,
recombinant interleukin‐1 beta (IL‐1) receptor inhibitors such as anakinra and canakinumab may be
considered for use in patients who have failed or cannot take colchicine, NSAIDS or corticosteroids.

22. Lady with psoriasis had chest infection and taking Doxycycline, she uses prednisolone,
which one does not aggravate psoriasis symptoms?
a) Cold exposure
b) Trauma
c) Allergy
d) Pregnancy
e) Infection

23. A patient is being treated with terbinafine tablets for Onychomyoosis. Choose the correct
statement regarding terbinafine tablets
a) Treatment of the infection is only required for one month
b) Their main side effects are GIT side effects 5%.
c) It is okay to still keep breastfeeding while on terbinafine tablets (Secreted in brast milk)
d) Terbinafine tablets can be used in pregnant women
e) Store this medication in the fridge
Terbinafine counseling points:
 Avoid coffee, tea, cola, energy drinks or other sources of caffeine while taking this medication. The
body breaks down caffeine to get rid of it. Terbinafine (Lamisil) can decrease how fast the body gets
rid of caffeine. Taking caffeine along with terbinafine (Lamisil) can increase the risk of caffeine side
effects including jitteriness, headache, increased heartbeat, and other effects.
 Avoid exposure to sunlight or tanning beds. Lamivudine can make you sunburn more easily.
 Take terbinafine at the same time each day, either in the morning OR in the evening. You can take
terbinafine with or without food. If you forget to take your dose, take it as soon as you remember
that day. But, if it is nearly time (4 hours) until your next dose, just take next dose at the right time.
 You should avoid consuming alcoholic beverages while taking Lamisil. Daily alcohol use can increase
your risk of serious side effects.
 Terbinafine SE: Decreased appetite, Gastrointestinal symptoms (feeling of fullness abdominal
distension, dyspepsia, nausea, abdominal pain, diarrhoea), Rash, urticaria, Musculoskeletal reactions
(arthralgia, myalgia).
 Pregnancy: Foetal toxicity and fertility studies in animals suggest no adverse effects. Since clinical
experience in pregnant women is very limited, Terbinafine tablets should not be used during
pregnancy unless clinical condition of the woman requires treatment with oral Terbinafine tablets
and the potential benefits for the mother outweigh any potential risks for the foetus.
 Breast‐feeding: Terbinafine tablets are excreted in breast milk; mothers receiving oral treatment
with Terbinafine tablets should therefore not breast‐feed
 Terbinafine tablets are contraindicated for patients with chronic or active hepatic disease. Before
prescribing Terbinafine tablets, a liver function test should be performed and any pre‐existing liver
disease should be assessed

24. All are Pneumococcal vaccine indications except:


a) Pneumonia
b) AOM
c) Meningitis
d) Endocarditis

25. PRIS 30-year-old female on MTX for her rheumatoid arthritis. She took the influenza and
after pneumococcal vaccine 2 years ago and is coming today to ask for your recommendation
for vaccination years what would be the right thing to tell her?
a) Influenza vaccine can be taken now and every year
b) Take influenza and pneumococcal vaccine now
c) Take influenza vacaine now and pneumococcal vaccine in 3 years
MTX Patient won’t take Pneumovax 23 BECAUSE: persons who are immunocompromised, including
persons receiving immunosuppressive therapy may have a dimished immune response To Pneumovax 23.
They can take Prevnar 13 instead followed by Pneumovax after 1 year.

26. Records of narcotic destruction include Names of 2 witnesses, Date of destruction; Name of
drug; Strength per unit; Quantity, should be kept for?
a) 2 years
b) 5 years
c) 6 months
d) 12 months

27. Patient come with prescription and you realize it is forgery what you will do?
a) Tell the patient you don't have the medication and he can come tomorrow to tell police.
b) You tell the patient it is forgery and don’t dispense it
c) Tell the patient it is forgery and restrain the patient until call the police.
d) Send the patient to another pharmacy to dispense the medication.
e) Delay the prescription holder to call the police

28. Expired drug that does not need witness for destruction?
a) Ketamine
b) Nabilone
c) Butorphanol
d) Clonazepam
e) Olanzapine

29. Bar code scanning technology is important for?


 Bar code medication administration (BCMA) systems are electronic scanning systems that
intercept medication errors at the point of administration.
 When administering medications with BCMA, a nurse scans a bar code on the patient's wristband to
confirm that the patient is the right patient.
 Using a portable device nurses check bar coding on medication packaging against the bar code on
patient’s wrist band
 Ensures right patient, right drug, right route, right time, right dose, etc.
 From manufacturer to picking of drug
 Staff bar coding makes it possible to determine what drugs have been given to a patient and by
which staff member and time of administration

30. A police officer, who is investigating a car accident, wants to know the medication history
of a regular patient?
a) No, unless he has a warrant- violation of confidentiality
31. Hospital case of dispensing error- An Rx of Dalteparin & by error heparin dispensed due to
bad hand writing, what will you do:
a) Separate both drugs
b) Tall Man Letter
c) Unit dose
How to prevent Heparin errors in hospitals?
 Do not use error‐prone abbreviations ex. IU, U, etc
 If heparin dose is >999, use commas ex. 1,000 & 10,000
 Avoid < or > in orders and protocol and istead use greater than and less than.
 Employ computerized errors by prescriber
 Use an actual body weight to calculate the dose instead of estimated or verified weights.
 Obtain/ review lab tests before initiating and adjusting heparin dose
 Establish a protocol to reverse heparin effect.
 Apply measures to avoid mix up of different concentrations:
 Use only 1 or 2 concentrations and standardize orders to show available strength.
 Independent double check of all heparin orders including checking pump settings
 Use smart pumps for heparin
 Minimize heparin dispensing by nurses on the floor
 Use warning labels, Tall man letters, etc to prevent mix up with other medications and strenghths
 Employ bar code scanning technology.
 Develop a protocol to identify, treat and document heparin induced thrombocytopenia
 Employ anticoagulation clinical service
 May use LMWH when possible

32. Pharmacy technician and Pharmacy assistant roles in non-sterile compounding, steps are
given:
Personnel
Pharmacy  Responsible for the development, organization and supervision of all activities related to
manager compounding of non-sterile preparations in the pharmacy
Pharmacist or  Develops, organizes and oversees all activities related to compounding of non-sterile
pharmacy preparations in the pharmacy
technician  Ensures that personnel are fully trained and know policies and procedures
 Ensures that a risk assessment is performed for each preparation
 Ensures that appropriate facilities, equipment and references are available for use
 Ensures that the master formula and beyond-use date (BUD) are developed using
scientific references and are reviewed appropriately
 Ensures that a quality assurance program is in place
 Ensures that all records of decisions, activities and specifications are complete and
appropriately documented
 Compounds non-sterile preparations in accordance with approved formulas with
established policies and procedures
 Clearly documents decisions, completed activities and verifications before dispensing
(pharmacist) or releasing (pharmacy technician) a compounded product
 Ensures that all compounding standards and standards of practice have been met. This
includes understanding chemical and physical properties of ingredients, using
appropriate equipment, and performing necessary calculations
 The compounder must use professional judgement when deciding whether they have the
expertise to compound a specific product.
 A personnel training and assessment program is implemented.
Non-regulated Compounds non-sterile preparations under appropriate supervision in compliance with the
pharmacy requirements of the provincial/territorial pharmacy regulatory authority
personnel

33. Need to be prepared in laminar flow hood (3 options, answer is eye drop)
a) Venlafaxine
b) Tamsulosin
c) Trifluridine ophthalmic drops

34. A question on case control and cohort study

35. A question about composite endpoint?


Endpoint is the event/ measurement we are looking forex. Fracture, secondary MI, death.
Composite endpoint: is the one that include multiple events, ex: mortality, morbidity, hospitalization,
syndromes. There is usually a primary ex. Fatal MI + others ex. Non fatal MI
Advantages: high statistical power, reduce size/duration/ cost of studies (commonly used for new drug
registration

36. Where do I search for information about omega-3 containing products? CPS
Another version. Patient asking about Omega 3 if it is effective for prevention of stroke. Where
can the pharmacist check this information:
a) Reputable manufacturer for Omega 3
b) Heart and Stroke Foundation website
c) Primary literature

37. What is the test for pancreatitis?


a) Lipase  (Amylase was not in the choices)
b) ALT
c) AST
Another version:
Pancreatitis lab test shows no abscess or purulent discharge: Antibiotics are not required
In acute pancreatitis serum lipase and serum amylase are elevated however serum lipase are slightly more
sensitive in both major causes of pancreatitis gallstone and alcoholic associated acute pancreatitis.
Acute physiologic assessment and chronic health evaluation (apache) ii scores should be calculated on
admission and daily for the first 72 hours after admission. An apache ii score of 8 or higher at baseline or in
the first 72 hours is suggestive of severe acute pancreatitis and is predictive of a worse clinical course.

38. Dehydration self-prepared solution, what to add?


The "Simple Solution" ‐ Home made Oral Rehydration Salts (ORS) Recipe
 Six (6) level teaspoons of Sugar.
 Half (1/2) level teaspoon of Salt.
 One Litre of clean drinking or boiled water and then cooled ‐ 5 cupfuls
(each cup about 200 ml.)

39. A kid takes ethosuximide for seizure, what to monitor?


a) CBC
b) Blood pressure
c) Glucose
TC: It is only effective in treating absence seizures. Baseline and periodic assessment of hematologic, liver
and renal status should be undertaken

40. Parkinson disease with mild symptoms, what is the drug of choice?
a) Levodopa
b) Pramipexole
c) Entacapone
Dopamine Agonist: Bromocriptine, pramipexole and ropinirole are effective as monotherapy in the early
stages of the disease, and as adjunctive therapy with levodopa for patients with more advanced motor
complication. Rotigotine, available as a transdermal patch formulation, has been shown to be effective in
reducing Parkinson disease symptoms. Transdermal delivery results in stable plasma concentrations (which
avoids pulsatile stimulation of dopamine receptors), offers an alternative for patients wishing to reduce oral
medications and may result in improved adherence.
Based on Algorithm mild cases are treated by Selegiline and Rasagiline

41. Parkinson’s patient feels that somebody is on the door (hallucination after 10 years), what is
the possible diagnosis?
a) Paranoid schizophrenia
b) Lewy body demetia
About 20% of people with Parkinson's will develop some form of hallucinations (mainly auditory) or delusions
due to medications (high levels of dopamine in the brain). Risk factors include dementia, depression, sleep
disorders, impaired vision, PD medications, and advance stages of the disease. Hallucinations and delusions
are components of a condition called psychosis.
In the diagnostic criteria for PDPsy, it is considered that patients suffer from PD for at least more than 1 year
before psychosis develops. If this is not the case, there is an unsolved problem of an overlapping diagnosis
with Dementia with Lewy Bodies.
With the exception of quetiapine and clozapine, all other antipsychotics should be avoided in PD psychosis.
Quetiapine has not shown consistent benefit in clinical trials, yet is often tried first to avoid the blood
monitoring requirements with clozapine use

42. IM vaccines needle size: 25-gauge, one inch

43. Where to find a new drug comparative toxicity? CADTH

44. Fastest acting drug for migraine?


a) Sumatriptan SC
b) Rizatriptan sublingual
Subcutaneous sumatriptan was also the fastest acting, providing more people with pain relief within one
hour of treatment than any other route of administration.

45. Diabetic patient + neuropathic pain: tight glycemic control


TC: Pharmacologic Choices
 Optimal glycemic control is recommended for reducing the onset and progression of neuropathy.
 Anticonvulsants (gabapentin, pregabalin, valproate) and antidepressants (amitriptyline, duloxetine,
venlafaxine) may be used alone or in combination for symptomatic relief of peripheral neuropathic
pain. Few people have complete relief and therefore a 30–50% reduction in baseline pain, as
measured by a visual analogue scale of 0–10 with 10 being maximal pain, is considered a meaningful
response.
 Topical capsaicin and topical nitrate spray have also been studied. The use of opioids for neuropathic
pain is only for select patients after other options have failed due to the risks of tolerance, abuse,
dependency and addiction
46. For the safety of insulin in hospital which one is better?
a) Colored label
b) Triple check
c) Unit dose syringe???
d) Double checking

47. Most common organism causing meningitis in adults


a) N. meningitides
b) S. pneumonia

48. What is elevated in CSF fluid?


a) Glucose
b) WBC
c) Protein

49. What do you give for empiric treatment?


a) Vancomycin + ceftriaxone
b) Cefotaxime + Ampicillin  for infant < 1 month
c) Vancomycin + Ampicillin

50. Red eye + purulent discharge patient asks for polysporin eye drops
a) Give QID OTC
b) Refer to doctor  if not improved within 2 days

51. Shake well before use?


a) Pulmicort (Budesonide) turbohaler
b) Nitroglycerin
c) Epipen
d) Tobradex eye drops  Shake well before use. Store in an upright position at room temperature
52. Patient has GI bleeding, epiglottis pressure & had endoscopy what to give to prevent
variceal bleeding?
a) Propranolol
b) Ramipril
c) Hydralzine
d) Clonidine
Prevention of Variceal Bleeding
 1ry prophylaxis: NSBB (propranolol, nadolol) & carvedilol are valid first‐line treatments.
 2ry prophylaxis: combination of NSBB (propranolol or nadolol) with endoscopic variceal band ligation.

53. Patient on ALESSE missing 2 tablets and she is on Isotretinoin last evening she had
unprotected sexual activity. What do you recommend?
a) Take 2 tablets of Allesse & continue
b) Plan B
c) Refer
d) Start new pack

54. SE of pregabalin?
Pregabalin MOA: Binds to the α2-δ site, an auxiliary subunit of voltage-gated calcium channels in the
Lyrica CNS, inhibiting excitatory neurotransmitter release.
Pain relief accompanied by improvement in quality of life, sleep and function
Starting dose: 25–50 mg QHS PO; titrate slowly as tolerated to 300–450 mg/day (in 2
divided doses)
S.E: Sedation, dizziness, cognitive impairment, dry mouth, peripheral edema, weight gain.
May have enhanced CNS depressant effects when coadministered with CNS depressants.
May cause peripheral edema/weight gain when coadministered with thiazolidinediones
(pioglitazone, rosiglitazone).

55. a question in regard of consultation on latanoprost as a pharmacist?


a) Increase in eyelash number/length/thickness
b) Darkening of the eyelashes and eyelids
c) It can cause pigmentation on iris
d) keep it in refrigerator
It was a tricky question I can't remeber the rest but it was important
keep in refrigerator before open once is opened keep it out only for 4 to 6 weeks
xalatan  latanoprost + timolol (combination)  keep it out only 10 weeks

56. COPD risk factors?


 Exposure to tobacco smoke. People with asthma who smoke.
 Occupational exposure to dusts and chemicals. Exposure to fumes from burning fuel.
 Age. COPD develops slowly over years, so most people are at least 40 years old when symptoms begin.
 Genetics. The uncommon genetic disorder alpha‐1‐antitrypsin deficiency is the cause of some cases
of COPD. Other genetic factors likely make certain smokers more susceptible to the disease.

57. A patient with COPD who is taking ipratropium, what we should do next?
Check the tenchnique of using inhaler before adding or decreasing the drug we have to
check if the patient takes it or not

58. What's the role of corticostroid in COPD exacerbation?


a) Improve quality of life
b) Reduced the number of days for saying in hospital
Oral corticosteroids improve lung function and shorten length of hospital stay in all patients and reduce risk
of early relapse or hospitalization for subsequent acute exacerbations
Oral corticosteroids should be given within 30 days of an acute exacerbation to reduce the risk of further
exacerbations. Administration beyond 30 days is not recommended.
A 5‐day course of oral prednisone 30–40 mg/day or equivalent is sufficient and has shown outcomes
comparable to a 10‐ to 14‐day course. Tapering is unnecessary for oral CS courses that last <2 weeks.
There is no role for oral corticosteroid maintenance therapy for patients with COPD. There is no advantage
to using IV corticosteroids.

59. A patient with COPD, CHF and HIV, suffers from dyspnea and Doctor diagnosis
exacerbation, what we should give?
a) Levofloxacin
b) Ciprofloxacin
Empiric Antibiotic Therapy for Acute Exacerbation of COPD
Group Symptoms & risk factors Probable pathogen First choice antibiotic
Simple < 4 exacerbations in the past year M. catarrhalis  Amoxicillin
exacerbations Increased sputum purulence + at S. pneumonia  Doxycycline
(COPD least 1 of: Haemophilus spp.  SMX/TMP
without risk 1) Increased sputum volume All for 5–7 days
factors) 2) Worsening dyspnea
Complicated As in simple exacerbation + at M. catarrhalis Preferred:
exacerbations least 1 of: S. pneumonia  Amoxicillin/clavulanate
(COPD with 1) FEV1 < 50% predicted Haemophilus spp.  Cefuroxime axetil
risk factors) 2) ≥ 4 exacerbations per year Klebsiella spp. Both for 5 – 10 days
3) Ischemic heart disease Other gram-negative  Levofloxacin × 5 days
4) Use of home O2 pathogens Alternative:
5) Chronic oral corticosteroid Pseudomonas spp.  Azithromycin × 3 days
6) Antibiotic use in previous Higher probability of  Clarithromycin × 5–10
3 months betalactam resistance days
60. A mother went to a hiking trip with her two sons, 3 years and 9 months (all kids have
eczema). She is asking about insect repellent icaridin, what is true about it?
a) Protect against ticks, but not mosquitoes
b) Icaridin 20% for age < 6 months to 12 years
c) 10% use except face
d) Icaridin 10% for age < 6 months to 12 years
Icaridin Affect the insect's ability to detect the host by concealing attractants emitted by hosts or by
(also known changing the insect's ability to smell them. Effective against mosquitoes, ticks and black flies.
as picaridin) Duration of effect: 10% = 5 h for mosquitoes and 7 h for ticks.
20% = 7 h for mosquitoes and 8 h for ticks and black flies.
Available as spray, aerosol or towelette. No allergic reactions reported.
10 – 20% Apply directly to skin, avoid eye contact.
< 6 months: Not recommended. All ages ≥ 6 months: Up to 20%.
Reapply 10% after 5 h, up to QID. Reapply 20% after 7 h, up to BID
Low toxicity. Nonirritating to skin, but should be kept out of eyes and mouth.
If travelling to area with high risk of arthropodassociated disease, up to 10% may be
applied to children < 6 months.
No evidence that the use of icaridin by pregnant or breastfeeding women poses a health hazard
to unborn babies or breastfed infants/children

61. After returning from trip with few hour’s, they developed skin rash, the mother found insect
crawling on her 9 months bayby’s clothes & she took it before it inserts her mouth piece into
her son’s leg. she is worried that the rash is because of lyme disease, what is the best response
you will give as she is concerned about lyme disease?
a) Don’t worry as the ticks should be conducted on the skin with 72 hours
b) Don’t worry lyme disease is preventable as one dose of antibiotic can do the job
c) I understand your concern, the tick needs to be in the skin for 36 hrs, it is not lyme
disease as you took the tic out before the bite
d) It is exacerbation of eczema as you used chemical sunscreen
e) The symptoms are not for lyme disease but for poison ivy
Tick Removal:
 Use clean tweezers to grasp to head of the tick, pull perpendicularly to skin surface in consistent
motion.
 Wash the bite area with soap and water or alcohol‐based disinfectant.
 Try to save to the tick in a sealed container and record the date.

62. she asked for a treatment because of the resulting eczema. What is your advice about?
a) Topical corticosteroids
b) Mupirocin cream
c) Zno 15% cream
d) Refer to assess
63. Azithromycin drug interaction:
Prolongation of QT intervals, palpitations or cardiac arrhythmias have been reported with concomitant
administration of azithromycin and astemizole or terfenadine.
Concomitant administration of some macrolide antibiotics with P‐glycoprotein substrates, including digoxin,
has been reported to result in increased serum levels of the P‐glycoprotein substrate.

64. A patient who was on statin I think and the doctor prescribed a H. pylori regimen with
clarythromycin, you that ther is an interaction what you should do?
a) Call doctor and suggest a regimen without clarythromycin
b) Change the statin
If statin with clarithromycin  stop statin temporarily for 2 weeks
If ator with clarithromycin we have to decrease dose of atorvastatin by 20% so no play in clarithromyvin
If write in the choices decrease dose and monitor not decrease only, we have to use this choice or hold the
dose
If you will use it for chronic (long term) we have to change statin
In carbamazepine with clarithromycin  we do not have to play with both dose we have to give him both
and monitor
We cannot change antibiotic alone if they want to change all the regimen

65. What to monitor with statins?


a) Creatine kinase
b) AST
c) Creatinine clearance
d) Lipase
Check Liver function (ALT) at beseline & at least once at 3 months.
Check creatine phosphokinase levels CK if myalgia develops.

66. Heart failure patient. Feel tired when he walks or


climb some stairs. what NYHA stage is the patient?
a) Class I  No symptoms with ordinary activity
b) Class II  occur with ordinary activity
c) Class III  occur with less than ordinary activity
d) Class IV  occur at rest or with minimal activity.

67. His LVEF is <35% and patient taking perindopril


and Propranolol, what drug is suitable to add?
a) Spironolactone
b) Digoxin
Beneficial in Patients with severe HF (NYHA III‐IV and LVEF  35%) & In post MI patients with an LVEF  40%
and either HF symptoms or DM.
68. Drug interaction of perindopril (ACEI)
 Increased serum lithium levels and symptoms of lithium toxicity have been reported in patients
receiving concomitant lithium and ACE inhibitor therapy.
 DPP‐4 inhibitors (linagliptin, saxagliptin, sitagliptin) Patients taking concomitant DPP‐4 inhibitor
therapy may be at increased risk for angioedema.
 Tricyclic antidepressants / Antipsychotic / Anesthetics Concomitant use of certain anesthetics,
tricyclic antidepressants and antipsychotics with ACE inhibitors may result in further reduction of
blood pressure.

69. Handling drug shipment received in community pharmacy, how to handle?


a) Wear a single glove
b) Wear double gloves
c) Not wear gloves
d) No special handling requirements
e) Wear double gloves and mask
All drug shipments must be delivered unopened to the pharmacy or an area of the premises other than the
pharmacy if the storage of the drug shipment is temporary, safe and secure.

70. Anemia patient with low ferritin & high MCV what to give?
a) Vit B12
b) Iron
If High MCV + Low Hb → Folic acid +Vit B12. Low Ferritin+ Low MCV → Iron
 Macrocytic anemia: is a type of anemia that causes unusually large red blood cells. Deficiencies in
vitamin B‐12 or folate often cause macrocytic anemia, so it is sometimes called vitamin
deficiency anemia.
 Hypochromic anemia: is a generic term for any type of anemia in which the red blood cells are paler
than normal. (Hypo‐ refers to less, and chromic means chrome.)
 Normochromic anemia: is a form of anemia in which the concentration of hemoglobin in the red
blood cells is within the standard range, but there is an insufficient number of red blood cells.
 Microcytic anemia: is defined as the presence of small, often hypochromic, red blood cells in a
peripheral blood smear and is usually characterized by a low MCV (less than 83 micron 3). Iron
deficiency is the most common cause of microcytic anemia.
 Hyperchromic anemia: an anemia with increase of hemoglobin in individual red blood cells and
reduction in the number of red blood cells.

71. Patient removed ileum + severe pain, what to give? Fentanyl patch
If paralytic ileus is present or suspected, treatment with Durogesic DTrans should be stopped
Some patients with an ileostomy, short bowel with an ileostomy or continent fecal diversion may develop
a condition called High Output Stomas (or High Output Ileostomies), which results in severe watery
diarrhea stool often times liters per 24 hours. This leads to dehydration, fluid and electrolyte imbalance,
and difficulty with maintaining an ostomy pouching system, requirements for parenteral (intravenous)
fluid hydration, Emergency Room visits, and hospitalizations.
These patients may use opioids such as diphenoxylate and distilled opium tincture (DTO) at doses closely
monitored by their physician to manage this condition. These drugs can be life saving for the individual
patient. There is a growing concern that new regulations and laws being enacted are restricting and/or
prohibiting access to the prescription and coverage of opioids resulting in an increasing threat to their
quality of life and potentially increasing medical costs by needing to resort to the aforementioned
parenteral fluids, Emergency Room visits and inpatient hospitalizations. Opioids are usually the last line of
defense against this difficult problem. However, for some a low dose opioid is prescribed to better and
more effectively manage this non‐pain condition and restore the quality of life for the patient

72. A patient has over dosed on fentanyl, what could he be experienang?


a) Hypoventilation
b) Diarrhea
The manifestations of FENTORA overdosage are expected to be similar in nature to intravenous fentanyl
and other opioids, and are an extension of its pharmacological actions with the most serious significant
effect being respiratory depression.
Immediate Management
 Immediate management of opioid overdose includes removal of the FENTORA tablet, if still in the
mouth, ensuring a patent airway, physical and verbal stimulation of the patient, and assessment of
level of consciousness, as well as ventilatory and circulatory status.
 Provide ventilatory support, obtain intravenous access, and employ naloxone or other opioid
antagonists as clinically indicated. The duration of respiratory depression following overdose may be
longer than the effects of the opioid antagonist’s action (e.g., the half‐life of naloxone ranges from
30 to 81 minutes) and repeated administration may be necessary.

73. You are the pharmacy manager; a relief pharmacist was working and he dispensed to a
patient 5 boxes of fentanyl instead of 5 patches. The right amount was entered and put through
to the manufacturer. The patient cannot be reached at this point in time. What should you do?
a) Call the police to report what has happened
b) Call the insurance company to report the fraud
c) Call the relief pharmacist for disciplinary action
d) Call the doctor of the patient to report what has happened

74. Red flag for diarrhea and constipation?


Diarrhea Constipation
Blood or abnormal mucus in stool GI bleeding.
Extensive abdominal cramping or pain. Fever >38.5˚C Palpable abdominal mass.
Frail elderly & Young age (<2 y). Pregnancy. Unexplained iron-deficiency
Immunocompromised (e.g., HIV infection, immunosuppressants) anemia.
Persistent or chronic diarrhea. Persistent vomiting for >4 h. Unintentional weight loss.
Presence of chronic medical conditions (e.g., DM, HF, kidney dysfunction) Family history of colon cancer,
Recent use of antibiotics particularly those associated with Clostridium Symptom onset ≥ 50 years of
difficile colitis (e.g., clindamycin, ampicillin, cephalosporins) age.
Severe diarrhea (> 6 unformed stools per day for >48 h) Sudden/acute onset of change in
Signs or symptoms of debilitating dehydration bowel habits
Weight loss due to diarrhea. Worsening diarrhea.

75. Prophylaxis for bacterial peritonitis + amoxicillin allergy:


a) Fluoroquinolones (norfloxacin)
ttt in community: Cefotaxime, or Ceftriaxone +Albumin Infusion
ttt in Hospital Acquired: Piperacillin/Tazobactam or Meropenum/Vancomycin
2ry Prophylaxis: DOC is Norfloxacin, ciprofloxacin or TMP/SMX

76. A patient with pneumonia we want to switch from IV to oral which one is the most
important barrier?
a) Culture positive for pseudomonas
b) Diarrhea
c) Persistent cough
Another version: What to use in hospital acquired pneumonia?
 Piperacillin/Tazobactam,
 3rd generation cephalosporin,
 respiratory quinolones,
 carbapenem (if risk of resistance is high),
 addition of aminoglycoside (if risk of resistance is high)
Reasons PREVENTING an early iv‐oral Switch:
1) Iv course <48‐72 h
2) Persisting fever
3) Hemodynamically Stable
4) Dyspnea
5) Leucopenia/Neutropenia
6) No adequate drug level at infection site with oral ABX
7) No oral alternative
8) Nil by mouth (NPO)
9) Malabsorption (able to swallow, no nausea, vomiting or severe diarrhea)

77. Comparing weight gain with different antipsychotics:


a) RxTx
b) RxFiles
c) CTMA
78. During influenza outbreak, what is the best way to protect your staff
a) Taking pregnant staff away from patient’s
b) Staff with high risk household contacts should wear protective covering
c) Tell people that have not been vaccinated to use face mask
Measures include:
1) hand hygiene (soap and water) and/or alcohol‐based hand sanitizer
2) make sure all HCPs are vaccinated.
3) provide PPE such as facemasks, gloves, etc.
4) avoid sick HCPs from working (sick leave).
5) maintain physical distancing.
6) Recommend cough etiquette (in their sleeves and use tissues to cover nose and mouth when
coughing or sneezing and dispose of in waste bins)

79. A patient with open angle glaucoma, what we will treat her about treatment?
a) Not to develop closed angle glaucoma
b) to prevent macular degeneration
c) to maintain optic nerve and reserve its function
Goals of Therapy:
•Prevent, halt or slow progressive visual loss
•Preserve the structure and function of the optic nerve
•Eliminate pain and improve vision loss in acute forms
•Improve quality of life and functional vision

80. Vomiting for more than 3 days:


a) Refer to doctor
b) Give OTC drug
Refer if:
1) • Nausea or vomiting > 3 days without known cause
2) • Vomiting > 6 hours in a child
3) • Substantial weight loss
4) • Dehydration symptoms
5) • Fever
6) • Abdominal pain
7) • Blood in the vomit (hematemesis)
8) • Altered mental status
9) • Recent head trauma
10) • “Thunderclap” headache

81. What company can do after having NOC?


a) Release the drug in the market
82. A patient has a seizure; he is on valproic acid and he suffers from GI side effect what we
should do?
a) Change to divalproex
b) Change the medication route of administration
c) Decrease dose

83. What to do in dawn phenomenon?

Treatment options include:


 adjusting the timing of insulin administration
 lowering the dose of insulin before bed
 changing the type of insulin
 eating a snack with the evening insulin dosage
 taking into account lifestyle factors, such as stress and exercise

84. What electrolye disturbance with PPI?


a) Hypercalcemia
b) Hypomagnesemia
It has recently been recognized as a side effect of PPIs. Low magnesium levels may cause symptoms from
several systems, some of which being potentially serious, such as tetany, seizures and arrhythmias. It seems
that PPIs affect the gastrointestinal absorption of magnesium.

85. Pharmacist wants to delegate some his/her tasks to tech. Which is the biggest barrier?
a) Pharmacist attitude
b) Technician workspace
c) Some public legislation thing
d) Pharmacist availability to supervise
86. DM pt comes to pharmacy & he is in pre-contemplation stage, what to say at this stage?
a) Effect of high blood sugar on health
b) Different methods of tx
c) Goals of diabetic control
Precontemplation: The individual is not intending to change in the foreseeable future, usually measured as
the next 6 months. Contemplation. The individual is not prepared to take action at present, but is intending
to within the next 6 months.

87. Mother asking about diaper rash for her baby? What is non-pharmacological advice?
a) Don't use alcohol wipes
b) Talc powder should be used frequently
c) Expose the baby's skin for as long as possible
d) Increase the frequency of washing
o Air‐drying should be encouraged to diminish damaging effects of occlusion and maceration. Remove
diaper for as long as possible during cleansing, treatment and change
o Avoid powders (or use only with extreme caution) due to risk of inadvertent inhalation; if used, apply to
cotton puff or the hands and dab on to decrease risk of aspiration.
o The barrier‐absorptive base, zinc oxide,
in concentrations of 20–40%, is the
preferred barrier product for
treatment of diaper dermatitis (DD);
for prevention, lower concentrations
of zinc oxide or barrier‐only products
may be considered.
➢ Clean the area gently with a soft cloth
or hypoallergenic baby wipe after
urination or defecation.
➢ Rinsing with water and wiping gently is
sufficient to remove urine.
➢ Avoid diaper wipes containing
chemicals such as alcohol, fragrance,
lanolin, methylisothiazolinone or soap.
Wipes without sensitizers are as well‐
tolerated as water in the daily
cleansing of infants with atopic
dermatitis.
➢ Wipes should generally be
discontinued if skin is broken (decision
should be made on a case‐by case‐
basis).
88. Non-prescription pharmacologic treatment of diaper dermatitis:
a) 1% Hydroortisone in clotrimazole
b) 0.5% Hydrocortisone in Mupirocin.
c) Mupirocin 2% in a polyethylene glyol base
d) Nystatin cream
Hydrocortisone is a schedule I When sold in a concentration that provides 1% or less hydrocortisone in
preparations for topical use on the skin in children under 2 years of age.

89. Now, she took ZnO 25% for 2 days, but no improvement, what to do?
a) Refer to physician
b) Increase concentration to 40%
c) Use antifungal
d) Use hydrocortisone 1% cream

90. Pregnancy + N/V what is for self-care?


a) CTMA
b) E-CPS
c) Drugs used in pregnancy
d) Cochrane

91. Verapamil & digoxin interaction? What enzyme inhibition?


a) 2D6
b) 2C9
c) 2C19
d) P-glycoprotein
Potassium‐depleting corticosteroids and diuretics may be major contributing factors to digitalis toxicity.
Calcium, particularly if administered rapidly by the intravenous route, may produce serious arrhythmias in
digitalized patients.
Quinidine, verapamil, amiodarone, propafenone, indomethacin, itraconazole, alprazolam, and
spironolactone raise the serum digoxin concentration due to a reduction in clearance and/or in volume of
distribution of the drug, cause a rise in serum digoxin concentration, with the implication that digitalis
intoxication may result. This rise appears to be proportional to the dose.
Lexicomb: P‐glycoprotein/ABCB1 Inhibitors (Verapamil) may increase the serum concentration of P‐
glycoprotein/ABCB1 Substrates (Digoxin). P‐glycoprotein inhibitors may also enhance the distribution of p‐
glycoprotein substrates to specific cells/tissues/organs where p‐glycoprotein is present in large amounts
(e.g., brain, T‐lymphocytes, testes, etc.). Severity Moderate Reliability Rating Fair
92. Patient suffers from diabetic foot and ulcer, while doctor inspection he discovered that it
reaches to the bone. This is called?
a) Cellulitis
b) Osteomyelitis
c) Tendinitis
d) Ulcerative colitis
• Treat current infection and wound, Prevent recurrent infections/ulcer
• Maintain optimal glycemic control, Proper glycemic control
• Prevent complications (e.g. osteomyelitis, gangrene, amputation)
• About ¼ of skin ulceration will spread to deep tissue and/or bone
• About 10‐30% of diabetic foot ulcers progress to amputations
• Properly fitted/orthopedic footwear. Snug fit is recommended, not too small or too large
• Skin and nail care, avoid injury. Nail care with podiatrist/chiropodist if necessary
• Check water temperature (e.g. with elbow) before putting feet in
• Check shoes before putting them on. Do not walk barefoot. Insoles, padding
• Inspect feet daily for cuts/scratches/blisters
• Proper wound care (use of appropriate dressings)
• Assess moisture of diabetic foot ulcer: exudates → use hydrocolloid dressing to absorb exudate
(Aquacel®) versus low exudate → moist dressing (Allevyn®)
• Pressure relief with casts/crutches/braces
• Insufficient evidence to recommend a specific type of dressing
• One that mimics physiological moisture & environment is recommended
• Off‐loading to avoid pressure ‐ redistribution & relief of pressure

93. What maybe the responsible microorganism?


a) S. pneumonia
b) S. aureus
c) C. perfringens

94. Non pharmacological treatment?


a) Compress stocking
b) Wear open
c) Hot compress
d) Incision & drainage

95. Which is the proper treatment


a) Cefazolin (in normal osteomyelitis)
b) Cephalexin (in osteomyelitis in diabetic foot)
c) Cefamandole
d) Cefuroxime
Parenteral options: Carbapenem, Piperacillin/Tazobactam, 3rd Gen cephalosporin (cefotaxime,
ceftazidime, and ceftriaxone) plus clindamycin po/iv or metronidazole
Oral options: Amoxicillin/clavulanic acid, cephalexin, clindamycin, cloxacillin, doxycycline, Ciprofloxacin or
levofloxacin plus clindamycin or metronidazole

96. The duration of treatment is


a) 2 Weeks
b) 1 Month
c) 3 Month
d) 6 Month

97. What to council on imiquimod: treatment for anogenital warts?


a) Put dressing on it
b) Put it at night wash it tomorrow morning
c) First apply in clinic, refrigerate.
Immune Stimulate innate and acquired immune responses, which ultimately leads to inflammatory cell
Response infiltration within the field of drug application followed by apoptosis of diseased tissue.
Modifiers Self-applied  Apply QHS 3 times per wk with 1–2 days between treatments × up to 16 wk
Imiquimod Wash hands before and after treatment application, wash treatment area with soap and water
5% cream 6–10 h after application, earlier if skin reaction occurs.
3.75% cream Require 3 months for response.
S.E: Local irritation, pain, mild to moderate erythema, burning.
Do not use in pregnancy.

98. A patients with low TSH, he will experience all these symptoms except?
a) Cold intolarance
b) Weight loss
c) Diarrhea
d) Moist / oily skin
Hyperthyroidism: ↓ TSH, ↑ Free T3, Free T4. Thyroid scan (scin graphy), radioac ve iodine uptake (RAIU).
Hypothyroidism: ↑ TSH, an thyroid peroxidase (an ‐TPO), ↓Free T3, Free T4. Abnormal lipid profile.
 Hyperthyroidism symptoms: Excessive sweating. (wet skin), Heat intolerance, Increased bowel
movements. Tremor (usually fine shaking), Nervousness, agitation, anxiety, Rapid heart
rate, palpitations, irregular heart rate, Weight loss. Fatigue, weakness.
 Hyperthyroidism Signs: Weight loss, Hypotension, palpitation, can lead to atrial fibrillation, heat
intolerance, Anxiety, diarrhea. hyperreflexia; warm, moist skin; goitre or nodules

99. What she should take?


a) Methimazol
b) Levothyroxine
100. Now she got pregnant, what is the appropriate action?
a) Change methimazol to PTU
b) Continue the current medication
c) Increase dose by 50%
MMI preferred first‐line agent as it has long half life, especially in children, breastfeeding, except during the
1st trimester of pregnancy when PTU is preferred.

101. All of these are symptoms of hypoglycemia except?


a) Sweating
b) Hypotension
c) Shakiness
d) Palpitation
CTC: Early signs of hypoglycemia include sweating, hunger, nausea, shakiness, heart palpitations (heavy, fast
heartbeats), anxiety, feeling irritable, mood or behaviour changes, numb lips or tongue, and headache.
Mild to moderate hypoglycemia has autonomic symptoms: sweating, tremors, tachycardia, heart
palpitations (heavy, fast heartbeats), hunger, nausea, numb lips or tongue, headache and a general
sensation of weakness.
Severe hypoglycemia requires assistance in its recognition and/or treatment. Neuroglycopenic symptoms
such as confusion, anxiety, feeling irritable, altered behaviour, difficulty speaking and disorientation can
progress to seizures and coma that prevent the patient from appropriately treating hypoglycemic episode

102. Patient SBP + ascites, patient recovered from SBP, has ileectomy (removal of ileum), what
vitamin should be given IV?
a) Vit B12
b) Folate
c) Ascorbic acid

103. Same patient has diet problem, lose weight, uncontrolled DM, refer to: Dietician

104. Same patient has bile induced diarrhea, what to give:


a) Cholestyramine
b) Psylium
Bile acid binders are typically very effective at treating diarrhea associated with BAM. Some common bile
acid binders include: cholestyramine (Questran) colestipol (Colestid)

105. Patient with Toxoplasma Gondii+ sulfa allergy- a prescription (SMX/TMP) came and
given by a nurse, what the pharmacist should do?
a) Report a case report
b) Contact the doctor
CD4 <100 cells/mcL and positive Toxoplasma gondii serology
T. gondii The preferred prophylactic therapy is SMX/TMP, Atovaquone is an alternative
Encephalitis Stop 1ry prophylactic if CD4 >200 cells/mcL × ≥ 3 months.
Stop 2ry if CD4 >200 cells/mcL × ≥ 6 months.

106. Patient taking lithium, what counselling point for the patient
a) Avoid caffeine
b) Avoid salt
c) Take plenty water
These diet guidelines to keep lithium blood level stable:
 Drink 8 to 10 glasses of water or other liquids every day.
 Keep your salt intake about the same.
 Keep your caffeine intake about the same.
 Avoid alcoholic beverages.
 Take lithium with food or milk.
 Toxic levels may result when adding NSAIDs, ACEIs, ARBs and especially thiazide diuretics.

107. Thiazide cause? Hyperglycemia, hyperuricemia, hypercalcemia, hypokalemia

108. Cancer patient + calcium?


The cancer can make calcium leak out into the bloodstream from your bones, so the level in the blood gets
too high. The cancer might also affect the amount of calcium that your kidneys are able to get rid of.
Damaged areas of bone can release Ca into bloodstream if you have cancer that has spread to the bone.
Two bisphosphonate agents were approved by the US Food and Drug Administration for the treatment of
hypercalcemia of malignancy: pamidronate (Aredia) and zoledronic acid (Zometa).
Bisphosphonate therapy should be initiated as soon as hypercalcemia is detected, because it takes 2 to 4
days to lower the calcium level.

109. Patient with ascites on spironolactone, failed. What is the second line of treatment?
a) Furosemide
b) Metolazone
c) Amiloride
Algorithm: Spironolactone (Gynecomastia Eplerenone)  (switch to Amiloride) – Add Furosemide‐ Add
Metolazone‐ Paracentesis + Albumin

110. What to stop before colonoscopy incase of diverticulitis?


a) Metformin  stop before contrast media
b) Iron
c) PPI, Bismith and antibiotic  stop before Urea Breath test
Stop: Aspirin, NSAIDs, Iron, Clopidogrel (7 days before procedure), Ticlopidine (3 days before), Warfarin (5
days before procedure), In Diabetes, take half of your dose medication the day before the procedure
111. Case study, E=500, C=500, improved cognitive symptoms, decrease depression symptoms,
withdrawal due to side effects, Calculate ARR, NNT, OR, RR?

112. You receive the following prescription for prednisone: Prednisone 40mg po daily x 3 days
and reduce the dose by 2.5mg po every 3 days till completely stop. How many tablets of
prednisone 5mg do you need to fill this prescription?
ANSWER
Days 1 – 3 = 8 tablets/day x 3 = 24 tablets
Days 4 – 6 = 7.5 tablets/day x 3 = 22.5 tablets
Days 7 – 9 = 7 tablets/day x 3 = 21 tablets
Keep reducing by 1. 5 tablets till we reach 1.5 (½ tablet of 5mg per day x 3 days) this means 24
+ 22.5 + 21 + 19.5 + 18 + 16.5 + 15 + 13.5 + 12 + 10.5 + 9 + 7.5 + 6 + 4.5 + 3 + 1.5 = 204 tabs
(By the way, this regimen will take 48 days to complete. There are 16 steps in the regimen and
each one = 3 days)

113. Ca gluconate 3 grams, if each 1gram gives 4.5 m. Eq of Ca gluconate, how many m. moles
of Ca in the 3 grams?
Answer:
1 gm ------- 4.5 m. Eq 3 gm ------- X m. Eq X = 3*4.5/1 = 13.5 m. Eq
No. of m. mole = No. of m. Eq / Valency = 13.5 / 2 = 6.75 m. mole

114. TPN formulation contains 23% dextrose and 7% amino acids infusing at 145mL/h with fat
emulsion 25% 500 ml 3 times weekly by y-site infusion.
A) Calculate the average total daily kilo calories
B) Calculate the number of grams of protein daily
ANSWER
The total daily volume of TPN = 145 mL/hour x 24 hours = 3480 mL
Contents of dextrose = 3480 x (23/100) = 800.4 grams of carbohydrates x 3.4 kcal per gram =
2721 kcal
Contents of protein = 3480 x (7/100) = 243.6 grams of protein x 4 kcal per gram = 974.4 kcal
FAT is given separately (i.e. not a part of 3480mL/day) as a 500mL of 25% emulsion given
three times per week = 1500mL of emulsion per week.
The contents of fat in WEEKLY volume of emulsion = 1500 x (25/100) = 375-gram x 9 kcal
per gram = 3375 kcal/week
Daily caloric intake from fat = 3375 kcal/7 = 482.1 kcal
Total calories = 2721 + 974.4 + 482.1 = 4177.5 kcal
115. A patient who quit smoking and drink coffee execesivly,
started carbamazepine and now has headache, insomnia,
tremor and anxiety why?
a) The dose of gums is no enough
b) It's because of coffee
c) Adverse effect of carbamazepine
Smoking is inducer for CYP1A2 and caffeine is substrate. Also, smoker
enzymes inducer for coffee once pt stop smoking high quantities of
coffee in body may lead to toxic effect

116. A patient on carbamazepin and there were lots of titrating


up and he suffers from chronic kidney disease what is wrong?
a) Carbamazepin needs a loading dose
It should be titrated every week to reach the therapeutic the dose
I really can't remember i just know that in previous papers they missed chronic kidney disease
Notes: no loading dose with carbamazepine

117. A diabetic patient is using Humalog 50 Mix (50% Lispro and 50% NPH). He is taking 24
units of Humalog Mix before breakfast and before supper. He called today to get his Humalog
Mix. However, all of Humalog is back order. You've Humalog Lispro & Humalog Mix (25%
Lispro and 75 % NPH)
a) Give him 8 units of Humalog Lispro and 16 units of Humalog Mix "25:75 "
b) Give him 12 units of Humalog Lispro and 12 units of Humalog Mix "25:75 "
c) Give him 16 units of Humalog Lispro and 8 units of Humalog Mix "25:75
Break down the question so you won't get confused:
So if the patient needs 24 units mix (50/50) it means 12 units of each insulin needed.
Now we have a mix with (25/75) which means every 100 units contain 75 units NPH so if we want 12 units
NPH how much of mix? 100 x 12/75=16 units mix.
We already have one answer so go for A but if there were 2 answers with 16 units then also calculate the
lispro the same way: now we have 16 units mix (25/75) so it contains 25% lispro=> 16 units x 0.25= 4 units
but we need 12 units so 12 ‐ 4= 8 units lispro alone.

118. What to give for a patient who has a stroke 3.5hrs before?
a) Alteplase
b) warfarin
c) Aspirin 365
Stroke: 4.5 Hours. MI: 6 hours
119. For primary prevention of stroke which one is more important?
a) Weight loss
b) Blood pressure
c) LDL
The major problem in stroke is arrhythmia then hypertension

120. Female will start on isotretinoin, how many negative pregnancy tests should she has:
a) 1
b) 2
c) 3
d) 4
Female patients of childbearing potential must have had 2 negative urine or serum pregnancy tests with a
sensitivity of at least 25 mIU/mL before receiving the initial isotretinoin prescription. The first test (a
screening test) is obtained by the prescriber when the decision is made to pursue qualification of the
patient for isotretinoin. The second pregnancy test (a confirmation test) must be done in a CLIA‐certified
laboratory. The interval between the 2 tests must be at least 19 days.

121. Patient takes NG and asks about sildenafil for erectile dysfunction?
a) Tell him it is contraindicated with NG
b) Refer to physician to solve ED
Systolic and diastolic blood pressure may be significantly reduced following coadministration of nitrates and
phosphodiesterase 5 inhibitors.
The manufacturers of sildenafil, tadalafil and vardenafil recommend that these drugs not be used in
combination with nitrates. In situations where nitrate use is required in a patient also receiving a
phosphodiesterase 5 inhibitor, effect on blood pressure can be reduced if there is sufficient time between
doses. Separate doses of nitrates and sildenafil and vardenafil by at least 24 hours. Allow 48 hours
between tadalafil administration and nitrates.

122. Patient receives 1 gm Vancomycin IV, T ½ of Vancomycin is 3 days. After 3 days later, he
received 1 gm Vancomycin IV, the blood concentration was 15mmol/L what is the steady state
trough concentration of Vancomycin
Answer:
After 3 days (1 T1/2) the conc. was 15 m. mole/L.
So, at time of adminstration it was 30 m. mole/L.
123. Dilution calculation. The pharmacy carries one 45 grams tube of sulfur 30% cream. How
much diluent do you do you add to dilute this content of this tube to 7%?
ANSWER
We can use the equation C1V1 = C2V2
C1 = 30% V1 = 45 grams C2 = 7% V2 = total volume of the cream
AFTER DILUTION 30/100 x 45 = 7/100 V2
V2 = 15/0.07 = 192.8 gram (this is the total amount of the diluted cream)
How much diluent: 192.8 – 45 = 147.8 grams

124. Drug interaction: warfarin with fluvoxamine, fluvoxamine inhibits?


a) 2D6
b) 2C9
Numerous cytochrome p‐450 isozymes may be involved in the metabolism of warfarin, including CYP 2C9,
2C19, 2C8, 2C18, 1A2 and 3A4.
CYP 2C9 is likely to be the principal isozyme modulating anticoagulant activity in clinical use.
Fluvoxamine has the potential to inhibit CYP1A2, CYP2C9, CYP2C19, and CYP3A4 to a significant degree.
The coadministration of warfarin and fluvoxamine can result in an increase in the anticoagulant effect
of warfarin.

125. A girl 18-20 years taking (alesse, OCP) for acne, all things are normal except she has
heavy bleading during the second half of the cycle, what to do?
a) Increase progesterone

126. A physician has a prescription written by a friend physician, of Rivastigmin?


a) Fill it as it is
b) Complain to college of physicians
c) Refuse to fill it

127. A patients comes with cold store after 5 days (not sure) what will you say?
a) Oseltamivir
b) Zanamavir
c) There isn't any treatment that can shorten the improvement days
d) OTC solution (forget the name)

128. Same patient comes after 3 months for flu shot, zoster vaccine:
a) It is too soon for him
b) Check for Rx dose/accuracy
2 live vaccines can be taken either at the same day OR 1‐month part (4 weeks)
129. Arrange these drugs in sequence of narcotics(N), controlled(C), targeted
subs(G), prescriptions. Buprenorphine, lactulose, methylphenidate, diazepam,
clozapine, risperidone, tramadol
a) 1, 1, 1, 4
b) 1, 1, 3, 2
c) 2, 1, 2, 2
Buprenorphine (1N), lactulose (3), methylphenidate (1g), diazepam 1(c), clozapine 1, risperidone 1,
tramadol 1

130. JA 28-year-old female who is at the end of her first trimester presents with uncomplicated
UTI. As a child, she suffered from a penicillin allergy where she got hives all over her body.
What should be given to her to treat the UTI?
a) Trimethoprim
b) Amoxillin
c) Nitrofurantion
d) Cephalexin

131. Doctor wants to prescribe nitrofurantoin, what he should check?


a) CrCl
Anuria, oliguria, or significant impairment of renal function (creatinine clearance under 60 mL per minute or
clinically significant elevated serum creatinine) are contraindications to therapy with this drug.
Nitrofurantoin is contraindicated in patients with ClCr <30 mL/minute.
Reports suggest that exposure to nitrofurantoin in utero close to delivery may put newborns at risk for
hemolytic anemia, both in patients with G6PD deficiency and those who are not G6PD deficient.
Nitrofurantoin should be avoided in pregnant patients close to delivery. The clinician must weigh the
benefits and risks in each patient

132. Nurse takes urine sample from patient, it was turbid, and send it to the labs, the results
come positive: what is the causative organism?
a) E. coli

133. Ferrous gluconate TID, one tablet contains 300 mg Iron. How much elemental iron daily?
Answer:
Ferrous gluconate contains 11.7 % elemental iron.
3 Tabs/day = 900 mg Iron. Elemental iron = 900 * 11.7 % = 105.3 mg

134. GERD non pharmacological management:


a) Elevate head of the bed / 2 pillows
b) Eat 2 hours before bedtime  should be 3 hours
Nonpharmacologic Choices
 There is little evidence to support the effectiveness of lifestyle changes aside from weight loss and
elevating the head of the bed; however, lifestyle and dietary measures provide broad health‐care
benefits and carry no risk.
 Modify diet (avoid chocolate; caffeine; acidic citrus juices; large, fatty meals)
 Reduce body weight if BMI >30 kg/m2 or recent weight gain
 Avoid eating up to 3 hours before bedtime
 Avoid lying down after meals
 Elevate the head of the bed by 10–20 cm, particularly if nocturnal or laryngeal reflux symptoms are
present; this is best achieved by using a block under the legs to elevate the bed frame rather than
with pillows or a wedge
 Stop smoking, avoid alcohol, Avoid tight clothing

135. Pt. on pantoprazole, duration:


a) 2 months
b) 3 months

136. Pt. needs indefinite duration of pantoprazole, which disease?


a) Barret’s esophagus
b) H. pylori
c) Refractory GERD

137. Person has BPH and hypertension, so what is doc for him:
a) Prazocin
b) Tamsulosin
c) Finasteride
d) Terazocin

138. Hyperlipidemia 2.5 LDL + hypertention, patient takes losartan and atorvastatin, patient
asks the pharmacist for recommendation of heart failure/ attacks prevention:
a) Atorvastatin is enough
b) Other drugs have no evidence
c) Statin+ fibrates

139. Family members of a patient come to the pharmacy to return back his medication because
he does not take them (expired but not used sulbutamol, not expired not used enoxaparin, not
expired tables)
a) Return to manufacturer and take a credit
b) Discard salbutamol, return others to the shelf
c) Put only tablets on the shelf and discard others
d) Dispose all
Pharmacists must dispose all unused and expired medications returned from the public

140. Which can you return back to the fridge apart from required storage conditions
a) Latanoprost
b) Vaccine
c) Triflusal  Donot require fridge

141. Expired narcotics and Diazepam, how will you manage:


a) Dispose diazepam and return narcotics
b) Dispose narcotics and return diazepam
c) Wait for written authorization for return
d) Dispose after getting written authorization
If returned from patient, destroy If expired at pharmacy, return

142. Diabetic patient on metformin 500 mg BID to TID + pancreatitis, BMI= 28, HBA1C= 9%
Dr. wants to add a drug, what is your recommendation
a) Insulin
b) Acarbose
c) Saxagliptin
d) Liraglutide

143. Patient comes to your pharmacy, does not know English, he comes with an interpreter
a) Talk to the interpreter
b) Show gestures and hand movements
c) Talk to interpreter, tell him to talk to the patient so both will understand
d) Look at the patient & talk to the interpreter

144. Dr. wants to conduct a study & he prepared a consent and gave to you to revise- you
noticed that the side effects were not mentioned
a) Ask Dr. to add the side effects to the consent
b) Ask patient to search SE themselves
c) Tell patient orally
d) Tell other staff members
145. Pharmacy is busy, when technician enters Rxs, he makes mistakes:
a) Give another duty to the technician
b) Talk to the technician- meeting the pharmacy
c) Ask other pharmacist to enhance his work
d) Take disciplinary action

146. Patient is travelling for 2 weeks; he took 2 doses of the dukoral vaccine previously and
came to the pharmacist because he would like to get it again. He has an 18 months old baby and
his wife is breastfeeding. Choose the correct statement?
a) It is effective because it is broad spectrum against food and water bacteria & cholera
b) Wife should take 2 doses of the Dukoral
c) Child should only take 1 dose of the dukoral
d) If the mother takes the dukoral, the child would be protected through breastfeeding
Vaccines Contains the nontoxic B subunit of cholera toxin, which has significant homology with the
Vibrio cholerae toxin of ETEC and is approved in Canada for prevention of TD caused by ETEC.
whole cell/ Prevention of enterotoxigenic E. coli diarrhea:
recombinant Adults and children ≥2 y: Primary immunization: 2 doses PO; 2nd dose administered within
cholera toxin B 7–42 days after the 1st dose and at least 1 wk before reaching destination
subunit vaccine Booster: 1 dose every 3 months if the risk is continuous
Dukoral Taken orally on an empty stomach (1 h before or 1 h after eating or drinking)
S.E: Abdominal pain, diarrhea, nausea and vomiting.
May consider for prevention of TD in persons with chronic illnesses (e.g., HF, insulin-
dependent DM, IBD, chronic kidney disease), or in those with immune suppression
Store in the refrigerator at 2° to 8°C (35° to 46°F). Do not freeze. The vaccine can be stored
at room temperature (up to 25°C) for up to two weeks on one occasion only.

147. What could increase the risk of getting traveller's diarrhea?


a) Ranitidine
b) Echinacea
c) Garlic
Reassess the need for proton pump inhibitors (PPIs) and H2‐receptor antagonists (H2RAs) in individuals
travelling to areas with high rates of TD since these agents increase the risk of acquiring intestinal pathogens.

148. What could be given to reduce the diarrhea effects?


a) Milk
b) Orange juice
c) Coffee
d) Carbonated drinks
Oral Rehydration Therapy: Oral rehydration, especially in infants, pregnant women and the frail elderly, is
the cornerstone of all therapy. Commercially available sachets of oral rehydration therapy (ORT), e.g.,
Gastrolyte, Pedialyte, are dissolved in safe water and the resulting liquid is consumed until thirst is
quenched. The traveller should carry sufficient sachets to produce 2–4 litres of oral rehydration solution
(ORS). If ORS is unavailable, an emergency substitute can be prepared by adding one‐half teaspoonful (2.5
mL) of table salt and 6 teaspoonsful (30 mL) of sugar to 1 litre of safe water

149. Dukoral vaccine, left outside for 3 days what should you do?
a) Call the manufacture for advice on what to do?
b) Return back to fridge until expiry and call manufacture for credit loss
c) Discard of the vaccine as it is no longer effective
d) Just return it to the fridge
a) Ask who make this and take disciplinary action with him
Store in the refrigerator at 2° to 8°C (35° to 46°F). Do not freeze. The vaccine can be stored at room
temperature (up to 25°C) for up to two weeks on one occasion only. After mixing with the buffer solution
the vaccine should be consumed within 2 hours.

150. Lady had a bypass surgery and what's the pharmacist main concern?
a) Increase in gastric pH
b) Increase in gastric emptying
c) Increase in surface area of mucosal of stomach
Bypass surgery side effects: Bowel obstruction, dumping syndrome, which leads to diarrhea, flushing,
lightheadedness, nausea or vomiting, Gallstones, Hernias, Low blood sugar (hypoglycemia), Malnutrition,
Ulcers, Vomiting, Acid reflux

151. MG went to doctor for the second time during the last four months. He has crohn's disease.
He is taking oral corticosteroid and AZA. What is the most important goal of therapy for this
patient?
a) Avoid surgery
b) Complete cure
c) Prevent flare up
d) Improve quality of life
Corticosteroid: Remission induction of a flare up for 14 days. AZA for maintenance of remission

152. Patient on Metoprolol + Heparin (not sure) monitor:


a) Chest X ray
b) Na level
c) Platelet count
d) Glucose level
METOPROLOL should be administered cautiously to patients spontaneously hypoglycemic or diabetic
patients who are receiving insulin or oral hypoglycemic agents. β‐adrenergic receptor blockers,
including METOPROLOL, affect glucose metabolism and may mask the premonitory signs and symptoms of
acute hypoglycemia, such as tachycardia. In patients with insulin or non‐insulin dependent diabetes,
especially labile diabetes, or with a history of spontaneous hypoglycaemia, beta‐blockade may result in the
loss of diabetic control and delayed recovery from hypoglycaemia. The dose of insulin or oral hypoglycaemic
agent may need adjustment. Diabetic patients receiving METOPROLOL should be monitored to ensure that
diabetes control is maintained.

153. Hospital Pt. which medication has to be placed in basket, put a label
a) Ketamine

154. Pharmacist goes to a specialized care home for cognitive dysfunction, checks profile, what
is his concern:
a) Oxybutinin for over active bladder???
b) Mematine to decrease cognitive dysfunction
c) Terazosin for hypertrophy  anticholenergic may worsen cognetive dysfunction
d) Tamsulosin

155. 95 gm NaCl dissolved in 2 L. dextrose 5 % calculate m. Eq /L.


Answer:
95 gm ------- 2 L X gm ------- 1 L X = 1*95/2 = 47.5 gm
M.Wt = 58.5 Valency = 1
m. Eq = wt / M. wt m. Eq = 47.5/58.5 m. Eq = 0.8 m. Eq/L.
Or
2L. of 95 % NaCl, what is the m. Eq/L.
95 mg ------- 100 ml X mg ------- 1000 ml X = 1000*95/100 = 950 mg
M.Wt = 58.5 Valency = 1 m. Eq = 950 / 58.5 m. Eq = 16.24 m. Eq/L

156. For which of the following is consent needed?


a) Patients who are under 16 years old
b) Patients who are receiving treatment for psychiatric illness
c) Patients who have parkinson's disease
d) Patients who have urinary incontinence

157. Pt has depression, with low appetite and sexual dysfunction. What should be given to him?
a) Mirtazepine
b) Moclobemide
c) Citalopram
d) Venlafaxine
e) Amitriptylline
It has a lower rate of GI and sexual side effects but is associated with sedation and weight gain (increase
appetite). Most weight gains.
158. Roommate calls the pharmacist and tell him that her friend has not slept in 3 days and
shewas talking continuously
a) Tell them to call the pharmacist for consultation
b) Seek medical attention
c) Tell her not to worry and her roommate will be fine
d) Tell her that you cannothelp her asyou do not want to break confidentiality

159. This patient was then diagnosed with depression, what would be the best option for her?
a) Mirtazepine
b) Bupropion
c) Triazolam

160. Patient has urge incontinence, what should be given to her?


a) Estrogen
b) Tolteridone
c) Doxepine
d) Citalopram

161. A patient came with several symptoms to the pharmacist. What should the pharmacist ask
to eliminate UT infection?
a) Dryness of vagina
b) Discharge
c) Flank pain
d) Fishy odour
What Causes Flank Pain? If you have persistent pain, you should always consult your physician. However,
flank pain most commonly results from one of three causes: urinary tract infection (UTI), kidney stones, and
musculoskeletal problems like a muscle strain or pinched nerve

162. Which of these increases insulin's secretion?


a) Metformin
b) Meglinitide
c) Stagliptin
d) Thiazolidinediones
e) Saxagliptin

163. A mother of 17-year-old girl with longhair has tried permetherin 1% twice for removal of
lice with nosuccess. What could be the reason for this?
a) The hair was not soaked well
b) The girl did not leave it in for along period of time 10
164. What else could be given to this patient to treat her hair lice?
a) Tea tree oil
b) Lavender oil
c) Isopropyl myristate
d) Permetherin 1%

165. Which is the most effective in treating hot flashes?


a) Venlafaxine
b) fluoxetine
c) Bupropion
d) Mirtazepine

166. A patient was started on diabetic medication and the doctor wanted to monitor his
progress. What test should be done after one month of therapy?
a) Glycosilated hemoglobin
b) Post prandial blood gluose
c) Lipids
d) Lipase
e) Liver toxicity

167. This patient's creatinine clearance


is 15ml/min. Which medication could
he not take?
a) Metformin
b) Pioglitazone
c) Insulin
d) Giclazide

168. A patient was admitted to the


hospital two days ago. The nurse has
given him aripiprazole main instead of
rabeprazole. Who should be contacted
regarding this error?
a) Family of the patient
b) The patient's family doctor
c) The patient's hospital doctor
d) The communitypharmacist of this
patient
169. What side effects could this patient be experiencing?
a) Sedation
b) Diarrhea
c) Hyperglycemia
d) Tremors  main side effect is akathesia
e) EPS
The most common side effects of Aripiprazole are: feeling of restlessness (akathisia), drowsiness, shaking
(tremors), abnormal movement, sleep walking and eating while asleep (sleep‐related eating disorders

170. A patient has body aches, fever of 39C, sore throat, cough and runny nose. He has been
taking oxymetazoline for 1 week, acetaminophen and levothyroxine. What could this patient be
suffering from?
a) Influenza
b) Common cold
a) Sinusitis
c) Pharyngitis
d) Allergic rhinitis

171. What is the possible DTP?


a) The acetaminophen taken by the patient is the wrong medication for fever
b) Too much of the right medication (oxymetazoline)
c) Interaction between levothyroxine and oxymetazoline
172. A patient has 82 years old and BMI of 20kg/ m2. The patient has osteoarthritis, what
should be given to him?
a) Capsaicin cream
b) Didofenac and misoprostol
c) Aspirin
d) Morphine
e) Hyaluronic acid

173. She asked for an intrarticular injection for her arthritis; what should be given to her?
a) Hyaluronic acid
b) Fentanyl injection
c) Desmopressin aoetate
d) Septra injection
e) Meropenem injection

174. What is the least thing that can be used to help her?
a) Hydrotherapy
b) Weight loss
c) Knee brace
d) Insoles

175. A doctor is looking for amedication for one of his patients who has glaucoma. The patient
is currently on salbutamol and flovent. He also suffers from a sulpha allergy. What is the DOC?
a) Prostaglandin analogues
b) Carbonic anhydrase inhibitors
c) Beta blockers
d) Alpha andrenergic

176. Which class of medication cannot be used due to his allergy?


a) Carbonic anhydrase inhibitors
b) Prostaglandin analogues
c) Beta blockers
d) Alpha andrenergic

177. How to counsel patient in order to increase the absorption of eye drops?
a) Blink your eyes for a few moments
b) Separate 2-3 minutes between two drops 5-10 drops
a) Leave your eyelid closed
c) Put a cold compress on the eye
178. How to decrease the systemic absorption of eye drops?
a) Put your fingers on the lacrimal duct for 1 – 2mins.
General Counselling tips for Eye drops (Tobradex)
 Suspension eye drops should be shaken well before use
 Remove contact lenses prior to instilling drops & wait at least 15min after instilling drops to insert
contact lens
 Instil only 1 drop & close eye without squeezing or blinking for at least 1 min to allow absorption
 Use nasolacrimal duct occlusion by applying gentle pressure with finger across the bridge of the
nose for 2 minutes to prevent systemic absorption
 Wait 5‐10 min between each drop to improve absorption
 Discard bottle after 28 days after opening

179. 78-year-old patient, is admitted to the hospital and suffering from C difficle. He suffers
from hypothyroidism, and his creatinine dearance is 198mm/L He also had an infection 6
months ago, for which he took doxacillin. What could be the cause of his C diffiaile?
a) Age
b) Hypothyroidism
c) Cloxacillin
d) Renal disease
RISK FACTORS FOR CDI
 Age > 65 (Elderly)
 Immunosuppression
 Exposure to antibiotics in last 3 months
 Ampicillin, amoxicillin, broad spectrum penicillin
 Cephalosporin
 Fluoroquinolones
 Clindamycin
 GI surgery
 Contact with infected person
 Therapy with proton pump inhibitors, H2 receptor blockers (lowers stomach acidity)
 Duration of hospitalization
 Chemotherapy

180. A doctor is asking you regarding approved medication in Canada for PTSD, which
reference would you check?
a) Compendium of therapeutic choices
b) Compendium of Pharmaceuticals and Specialties
c) Medline
d) Remington
e) Martindale
181. Regarding financial ratios, what is correct (k-type)
a) To find the strongest point
b) To find the weakest point
c) To objectify.

182. Agrandmother gave by accident an overdose of acetaminophen to her grandson, what


would be suitable to do after this was discovered? (k-type)
a) Give N-acetylcysteine as soon as discovered
b) Report to doctor for hepatic check
c) Check the child as it could have caused CNS side effects

183. Regarding Age-related Macular Degeneration (AMD); choose the correct statement
a) Beta carotene vitamins should be given to all smokers to prevent AMD
a) Medication should not begiven as a preventative measure
b) Only treat those with wet AMD and not dry AMD
c) Vitamins should be given to everyone with AMD not for wet

184. Ajisa 54-year-old diabetic patient who is currently having a COPD exacerbation, what
should be given to him in order to control it?
a) Amoxiclav
b) Clindamycin
c) Cefazolin
d) Ciprofloxacin
Complicated Exacerbation if: ABX Use in 23 months, >4 AECOPD/year, IHD (CAD, Or HF), O2 Therapy and
immunocomprised

185. A patient suffers from restless leg syndrome; what can be given to him?
a) Pramipexole
b) Iron
c) Vitamin D
d) Morphine
e) Mepiridine

186. Methadone  8.1 gm Na Benzoate  1.15 gm Liquid qs  810 ml


What is the amount of Methadone & Na benzoate to prepare 900 ml?
Answer: Methadone = 8.1/810 * 900 = 9 gm
Na Benzoate = 1.15/810 * 900 = 1.27 gm

187. What is a risk factor for Parkison's disease?


a) Smoking
b) Gender
c) Repeated Head Trauma
d) Age
e) Alcohol
Other Risk factors: Family history, high fat diet.
There is evidence that Nicotine, caffeine and exercise have neuroprotective role

188. Benzydamine mouthwash counseling - all expect?


a) Dilute with warm water into a 1:1 solution
b) Don't eat hot foods directly after it
c) For pharyngitis & mucositis caused by radiation therapy
d) Swish in the mouth for as long as possible then swallow
189. A patient on digoxin 0.125mog, his serum levels were taken several days later and it came
to 5.9 mmol/L (normal 4-5mmmol/L). What is the best course of action to take for this patient?
a) Reduce the dose to 0.0625mag every other day
b) Withhold digoxin for 3 days then restart at 0.0625mog
c) Stop digoxin
d) Continue digoxin
e) Refer to emergency

190. Which of the following interacts with Linezolid? Moclobemide (Serotonin Syndrome)

191. Which of the following is a side effect of dabigatran?


a) Nephropathy
b) Skin nearosis
c) Dyspepsia
d) Tremors
e) Headaches
Direct Inhibit both clot-bound and free thrombin lla.
Thrombin Patients >75 y, use with caution & consider reducing dose to 150 mg BID PO daily.
Inhibitors For patients >75 y with 1 or more risk factors for bleeding consider 110 mg BID PO.
Dabigatran, S.E: Bleeding, dyspepsia. Antidote: Idurucizumab
Pradaxa. Q. The capsule should NOT be chewed, broken, opened or crushed.
Use with caution with other drugs acting on Pgp e.g., quinidine.
Dabigatran is recommended to be discontinued 24 hours prior to endoscopic procedures,
irrespective of renal functional status in low bleeding risk procedures.
In high bleeding risk procedures or surgeries, dabigatran is recommended to be discontinued
48–72 hours prior in normal renal function and mild renal function (CrCl > 50 mL/minute).
In moderate renal impairment (CrCl of 30–49 mL/minute), discontinue 72–96 hours before
high-risk endoscopic procedure.
If severe renal impairment (CrCl < 29 mL/min), discontinue 96–144 hours before endoscopy.
Increased risk of bleeding with ClCr < 30 mL/min

192. Patient, presented with a black circle on her thigh, what could be the cause of this?
a) Warfarin bruising
b) Prednisolone
c) Levodopa
d) Amiodarone
e) Levothyroxine
Overdose: Suspected or overt abnormal bleeding (e.g., appearance of blood in stools or urine, haematuria,
excessive menstrual bleeding, melena, petechiae, excessive bruising or persistent oozing from superficial
injuries, unexplained fall in hemoglobin) is a manifestation of excessive anticoagulation.
193. Patient who was taking venlafaxine, stopped it abruptly and went on to suffer from flu-ike
symptoms & agitation, what can bedone for this patient?
a) Do nothing, this will resolve on its own
b) Restart venlafaxine and taper down slowly
c) Put the patient on fluoxetine in order to reverse these symptoms
If your patient experiences significant discontinuation symptoms, resume the last prescribed venlafaxine
dosage, with a plan for a more gradual taper. Acute discontinuation syndrome also can be treated by
initiating fluoxetine, 10 to 20 mg/d; after symptoms resolve, fluoxetine can be tapered over 2 to 3 weeks.

194. What is the organism responsible for cellulitis


a) Staph. Auereus
b) P. aueriginosa
c) S viridians
d) M. catarrahlis
e) N. meningitides

195. All can be used to treat cellulitis, except


a) Cloxacillin
b) Clindamydin
c) Cephalexin
d) Norfloxacin
Cellulitis TTT are all C

196. Duration of treatment for cellulitis:


a) One week
b) Two weeks
c) One month
d) Three weeks

197. Apatient comes to ask your recommendation for a multivitamin, what could be the reason
this patient needs vitamin D?
a) Fat restricted diet
b) Diabetes Mellitus
c) Depression
d) Liver impairement

198. Which medication does allopurinol interact with?


a) Azathioprine
b) Colchicine
Allopurinol may increase serum concentrations of the active metabolite(s) of Azathiprine. More
specifically, allopurinol may increase mercaptopurine serum concentrations and promote formation of
active thioguanine nucleotides. Severity Major Reliability Rating Excellent
Consider further dose reduction or alternative therapies for patients with low or absent thiopurine S‐
methyl transferase (TPMT) activity receiving azathioprine and allopurinol.

199. A patient who is obese, suffers from hypertension as well as heart burn. What is your best
recommendation for him?
a) Refer to the physician to get a proton pump inhibitor
b) Recommend sodium alginate
c) Recommend antacids after food
d) Give him famotidine for his heart burn

200. What is some non pharmacological advice to give this patient?


a) Regular exercise
a) Weight loss
b) Lie down after eating
c) Walk after eating

201. Side effects of mepiridine


a) Seizures
b) Encephalopathy
c) Tremors
d) Skin necrosis  warfarin

202. A pregnant woman just had a C-section and the physician prescribed her meperidine Q4hr
for her pain. One day later, she told the nurse that she had restless legs and muscle twitches.
What is the reason for her symptoms?
a) The oxytocin delayed effect of the surgery
b) The delayed effect of the epidural after the surgery
c) Neurotoxiaity of meperidine
General and CNS: mood changes (such as euphoria, dysphoria), weakness, headache, agitation, tremor,
muscle twitches, severe convulsions, uncoordinated muscle movements, transient hallucinations and
disorientation, delirium or confusion, visual disturbances.

203. Clarithromycin is contraindicated with which of the following


a) Ketoconazole
Clarithromycin (CYP3A4 Substrates (High risk with Inhibitors) – Ketoconazole (Systemic) (CYP3A4 Inhibitors
(Strong))
204. What's the organism that could be present if the patient has CD4 < 200?
a) PJP
b) P. auereginosa
c) S viridans
CD4< 200: PJP CD4<100 T Gondii CD<50 MAC: Mycobacterium Avium complex

205. What can be used to treat her?


a) Cotrimoxazole
b) Levofloxacin
c) Amoxiclav
d) Dapsone

206. What to monitor with Zidovudine


a) CBC or WBC
b) Lipase
c) Thyroid
d) Lipids
e) Heart
Zidovudine S.E: Nausea, headache, malaise, fatigue, rash, myositis, myocarditis, anemia, leukopenia,
(AZT) hepatic steatosis, elevated liver enzymes, lactic acid and CK. Longterm use associated
pyrimidine with peripheral lipoatrophy. Available as an oral syrup.
analog Additive hemotoxicity with other agents, e.g., anemia with dapsone, foscarnet,
ganciclovir, pentamidine, ribavirin.
Pharmacologic antagonism with stavudine. Avoid combined use of AZT and ribavirin or
stavudine as are activated by the same intracellular pathways.

207. A pregnant woman presents with flu-like symptoms, he CD4 levels were at 500 cells/ml
and her viralload is 1200 cell s/ mL Why should this patient be treated?
a) Pregnancy
b) Her CD4 level
c) Her viral load
Initiation of ttt should be considered more urgent in pregnant woman, in patients with Symptomatic HIV
infection, and in asymptomatic patient wuth CD Count < 500 cells/ul

208. A child who takes 4 units of rapid acting insulin with lunch and 6 units of rapid acting
insulin with dinner. He is also on insulin glargine 15 units at bedtime. All of his levels are
within range except his bed time levels seem to be a little bit high and his mother is concerned
about this. What can be done to correct this?
a) Snack at bedtime
b) Lower the units of insulin glargine
c) Increase meal of suppertime
d) Increase the rapid acting dose given with supper

209. A patient came in with the following Rx for acyclovir: 2gm po Q12hr 2 doses XM: 6 doses
Refill x1. The patient is asking to have the whole Rx filled including the refills all at this time as
he does not want to come back again to get it. What is the strength and quantity of tablets to be
given to him?
a) 500mg, 96 tabs
b) 500mg, 48 tabs
c) 500mg, 26 tabs
d) 1000mg, 46 tabs
e) 1000mg, 18 tabs

210. Which drug does not cause hirustism?


a) Prednisone
b) Finasteride
c) Minoxidil
d) Phenytoin
e) Tadalafil
Many drugs can cause hirsutism: androgens, glucocorticosteroids, progestins, estrogen antagonists
(clomiphene, tamoxifen), minoxidil, cyclosporine, danazol, diazoxide, phenytoin, D‐penicillamine, and
interferon

211. All the following are used in Framingham risk score FRS, except?
a) Diabetes
b) Age
c) Gender
d) Family history
e) LDL
FRS involves age, smoking, diabetes, cholesterol level, systolic BP & HDL‐C. Double the FRS when there is a
family history of premature CV disease (modified FRS)

212. PR a 19-year-old male comes to your pharmacy to ask for your recommendation for his
comedonal, 1 mid non-inflammatory acne. What could you recommend to him as a pharmacist?
a) Azaleicacid
b) Gyolic acid
c) Tretinoin cream
d) Isotretinoin
e) Clindamycin topical
213. After 6 months, PR returned to the pharmacy and told the pharmacist that the doctor gave
him topical Cindoxyl (clindamydn and benzoyl peroxide). PR is frustrated that this cream has
not done anything for his acne, what do you advise him?
a) He should switch to an oral antibiotic
b) Advise him to wait for 2-3 months to see an improvement  if he mentioned a period
c) This treatment is inappropriate for him
d) He should be put on isotretinoin

214. After a few months, PRs father comes to your pharmacy and asks you to give him a list of
his son's profile for tax purposes. What is the most appropriate action to do in this case?
a) Refuse to give PRsfather any information
b) Tell PR’s father that you need PR’s consent to release this information
c) Give PR’s father the list of medication andcost
d) Give PR’s father only the cost of medicationsand notwhat PR takes

215. Apatient who has atopic dermatitis was on betamethasone cream. The patient's dermatitis
was not controlled on this cream. Thus, the doctor changed it to mometasone ointment. The
itching has resolved; however, the inflammation was not. What should the patient take?
a) Adapalene
b) Tacrolimus
c) Mometasone cream
d) Clobetasol
CTC: Calcineurin inhibitors, also referred to as
topical immune modulators, are a class of
medications designed to specifically block
calcineurin. They provide a targeted, specific anti‐
inflammatory mechanism in contrast to the wide‐
ranging effects of corticosteroids.

216. A patient experienced another episode


of C. difficile, what should be given to him?
a) Oral vancomycin
b) Oral metronidazole
c) IV vancomycin
d) Oral ciprofloxacin
e) Bismuth salicylate
Recurrent CDI or Relapse means re‐occurrence within 8 weeks of finishing therapy
First Recurrence: (2nd occurrence): Treat as initial episode
2nd Recurrence: (3rd or more Occurrences): Use Vancomycin po+ Saccharomyces Boulardii (Type of yeast
Probiotic) for 20 days Taper or pulse
217. Bupropion can NOT BE taken in all cases; except:
a) Seizures
b) Bulimia nervosa
a) Post myocardial infarction Post MI
c) Pregnancy
d) Anorexia nervosa

218. A patient who is taking a proton pump inhibitor, is in need of a calcium supplement; which
one can be recommended to him?
a) Calcium carbonate
b) Calcium gluconate
c) Calcium citrate
d) Calcium acetate
Because calcium citrate is not dependent on acid or pH for absorption, it may be the preferred calcium
supplement for PPI users. Calcium citrate supplements and calcium in natural products such as cheese and
milk will provide patients with greater bioavailability regardless of pH.

219. A patient, who is currently in


the process of quitting smoking, has
some episodes where theirs is a lot
of stress and he cannot control but to
smoke a cigarette or two during that
period. What can be done in this
case to reduce his smoking?
a) Behavioural management
b) Inhaler for fast nicotine
replacement
c) Gum as needed (if crening
mainly monming)

220. What to give OTC for the


treatment of shingles
a) Lidocaine
b) Calamine
221. What is true about rizatriptan wafer?
a) Dissolve in saliva and absorb through GI track
b) We should put sublingual
c) Can't remember others
For MAXALT RPD Wafers, administration with liquid is not necessary. The wafer is packaged in a blister
within an outer aluminum pouch. Patients should be instructed not to remove the blister from the outer
pouch until just prior to dosing. The blister pack should then be peeled open with dry hands and the wafer
placed on the tongue, where it will dissolve and be swallowed with the saliva

222. A patients was on rosavastatin 40 he gets myosititis and Doctor stop the med what we
should give?
a) Atorvastatin 40
b) Simvastatin 80
c) Ezetimibe
If no heart problem uses ezetimbe even if there is problem in myalgia if has heart of problem use low dose of
statin

223. Patient take morphine and he got Myoclonus so what is the appropriate thing to do?
a) Switch the dose of morphine to hydrocodone.
b) Reduce dose of morphine and give hydrocodone dose.
c) Give morphine and Benzos
Reducing the dose is the answer, Adding BZD is the last resort due to additive CNS Depressive effect.
Myoclonusis a neuroexcitatory SE due to higher opioid doses (M3G is excitatory). Note that Hydromorphone
carries the very same SE due to H3G

224. In case of myalgia who we should monitor? darken urine Ck wasn't the option

225. For hypertension the the salt intake should be below:


a) 2000mg
b) 1000mg
CTC: Consider sodium intake target of <2000 mg (88 mmol) per day.

226. A patient who has AF is on warfarin, which one asking about not true?
a) Having bracelet (used or diabetic)
b) The INR between 2-3
227. With regard of safety of oral solution which one is correct? it is not kcal
What is the best strategy to avoid administering oral medication via parenteral route?
a) Use amber syringe instead of transparent ones
b) Use a syringe that is incompatible with the injectable part
c) Put a label on metric and non-metric syringe with oral use only
d) Move all of oral solution to pharmacy form stock (this choice if we have Kcl in the
question)
e) Prepare all of oral solution in oral syring (no prepare wrong to say prepare (in children
we give syringe (ml) to be easy in administration and correct dose)
f) Put both pounds and kg on patient chart

228. A patient with frequent attack of Raynaud’s want to get medication, what we can give?
a) Felodipine
b) Propranolol
c) Atenolol
Can't remeber the rest, if nifedipine XL we have to choose it
TTT: Prazocin, CCBs (Amlodipine, Felodipine XL, Diltiazem), PDE5 (Sildenafil and Tadalafil)

229. A guy with CHF who has COPD as well and was hospitalized he feel pain in one leg what
is the problem (not sure it was pain or edema)
a) DVT
b) CHF getting worse
c) Exacerbation of COPD
Painful  DVT. Painless  CHF
DVT if swelling, unilateral Pain, Erythema, warmth, & redness without inflammation
CHF if Peripheral Edema‐ fatigue‐ dyspnea

230. A pregnant woman who has DVT what we should give?


a) Warfarin
b) Rivaroxaban
c) Enoxaparin
231. Which of the following statements is correct regarding DVT. Patient is taking rivaroxaban
20 mg once daily:
a) Should take it with food
b) Cause constipation
c) Tablets can be Chewed or swallowed (crushed and mixed with applesauce)
d) Avoid antacids (no relation mentioned)
e) Rivaroxaban can be taken if CrCl is less than 30
f) Rivaroxaban should be started a high dose in the first 3 weeks
g) Rivaroxaban should not be taken in liver dysfunction
h) Rivaroxaban is not indicated for initial treatment for DVT
Administer doses ≥15 mg with food; doses of 2.5 mg and 10 mg may be administered without regard to meals.
For nonvalvular atrial fibrillation, administer with the evening meal.
For patients who cannot swallow whole tablets, the tablets may be crushed and mixed with applesauce
immediately prior to use; immediately follow administration of the 15 mg and 20 mg tablets with food (2.5
mg and 10 mg tablets may be administered without regard to food).
Your doctor may give you the crushed XARELTO tablet also via a tube.
Missed doses:
Patients receiving 15 mg twice daily dosing who miss a dose should take a dose immediately to ensure 30
mg of rivaroxaban is administered per day (two 15 mg tablets may be taken together); resume therapy the
following day as previously taken. Patients receiving 2.5 mg twice daily who miss a dose should take a
single 2.5 mg dose at the next scheduled time; then resume therapy as usual. Patients receiving once‐daily
dosing who miss a dose should take a dose as soon as possible on the same day; resume therapy the
following day as previously taken.
SIDE EFFECTS
As XARELTO acts on the blood clotting system, most side effects are related to signs of bruising or bleeding.
In some cases, bleeding may not be obvious, such as unexplained swelling.
Patients treated with XARELTO may also experience the following side effects: nausea, vomiting, stomach
ache, constipation, diarrhea, indigestion, and decreased general strength and energy.
Monitoring and Laboratory Tests
Measuring PT using Neoplastin reagent, or Factor‐Xa assay using rivaroxaban ‐specific calibrators and
controls, may be useful to inform clinical decisions in these circumstances.
Although XARELTO therapy will lead to an elevated INR, depending on the timing of the measurement, the
INR is not a valid measure to assess the anticoagulant activity of XARELTO.
At recommended doses, XARELTO affects the measurement of the aPTT and Heptest. These tests are not
recommended for the assessment of the pharmacodynamic effects of XARELTO.
Contraindications
 Clinically significant active bleeding, including gastrointestinal bleeding
 conditions at increased risk of clinically significant bleeding, eg, recent cerebral infarction
(hemorrhagic or ischemic), active peptic ulcer disease with recent bleeding.
 Concomitant systemic treatment with strong inhibitors of both CYP 3A4 and P‐glycoprotein (P‐gp),
such as ketoconazole, itraconazole, posaconazole, or ritonavir.
 Concomitant treatment with any other anticoagulant, including
o unfractionated heparin (UFH), except at doses used to maintain a patent central venous or
arterial catheter, low molecular weight heparins (LMWH), such as enoxaparin and dalteparin.
o heparin derivatives, such as fondaparinux, and
o oral anticoagulants, such as warfarin, dabigatran, apixaban, edoxaban, except under
circumstances of switching therapy to or from XARELTO.
 Hepatic disease (including Child‐Pugh Class B and C) associated with coagulopathy, and having
clinically relevant bleeding risk.
 Pregnancy, Nursing women & Hypersensitivity to XARELTO or to any ingredient in the formulation.

232. A patient with Allergic rhinitis who does not get better with nasal corticostroid what we
should do next?
a) Nasal decongestant
b) Ipratropium
CTC: Intranasal ipratropium reduces the volume of watery nasal discharge. It may be used when rhinorrhea
is the only symptom or when rhinorrhea is refractory to topical INCS and/or antihistamines. This may be
especially helpful in some types of vasomotor rhinitis, e.g., “skier’s nose.” Intranasal ipratropium is
effective for rhinorrhea secondary to allergic rhinitis but not for another symptom

233. Which one needed to be shaken before use?


a) Budesonide turbuhaler
b) Fluticasone evohaler
c) Salbutamol MDI
d) Spiriva Respimat
Other options needed to be primed only

234. Cancer patient who is on opioids who has gas and hasn't have bowl movement for 3 days?
a) Refer to doctor
b) PEG
c) Docusate (Na and Ca)
1st line: Senna –Bisacodyl, 2nd line: PEG‐ Lactulose or naloxgol
Refer  if has family history of colon cancer
Constipation 5-HT3 Occur 1 or 2 days after cancer Suppositories and enemas are used with
(within hours to antagonists, therapy starts if the patient is prolonged, severe constipation, but they
days for 5-HT3 bortezomib, receiving mulitple days of therapy may introduce an infection risk in
antagonists) thalidomides with 5-HT3 antagonist antiemetic. neutropenic patients.
(within weeks to and analogues, Patient education and preventive Educate patient to discontinue laxatives
months for others) vinca alkaloids measures, including laxatives and before the onset of cancer therapy-
stool softeners, should considered induced diarrhea
235. patient with H pylori get the regimen for 10 days and now he is good, what we should do?
a) Continue PPI
b) Stop PPI
c) Change PPI to H2 blocker
d) Go to doctor to do assessment for the case

236. A patient with IBD wants to start etanercept, what you should warn patient?
a) Risk of infection
Rituximab  the only one does not cause infection (TB)
Anakinara  least cause risk of infection. Tofacetinib  most one cause HSV
Infliximab  the only one taken with methotrexate but the other with or without

237. A cancer patient who is on hydromorphine has nausea what you should give?
apparently, the nausea was because of hydromorphine (patients has constipation)
a) Metoclopramide
b) Domperidone
c) Dexamethasone
This is not CINV. Here from Hydromorphone and has constipation so prefer Prokinitic agent
Metoclopramide in cancer and opioid induce nausea. Dimenhydrinate  for motion sickness.
Domepridone  for anorexia nervosa. Here can not use dexamethasone alone

238. patient is suspected to have hepatitis A; doctor wants to rule out other hepatitis what he
should aks about?
a) Alcohol consumption
b) Eating outdoors
c) Sexual activity

239. What combination to avoid in treatment of HIV


a) Tenofovoir TDF and didanosine
b) Tenofovoir TDF and emtricitapin
c) Abacavir and lamivudine
d) Atazanavir and ritonavir

240. Which one can cause nausea?


a) Dutasteride
b) Meloxicam
Dyspepsia, nausea, vomiting, abdominal pain, constipation, flatulence, diarrhea
241. A cancer patient who was on opioid he stared doxazosin what we should say?
a) Doxazosin can exacerbate CNS suppression of opiod
b) It can cause dizziness
Doxazocin (Blood Pressure Lowering Agents) may enhance the hypotensive effect of opioid
Morphine (Systemic) (Hypotension‐Associated Agents)

242. which one can cause gingival hyperplasia


a) Nitroglycerin
b) Topiramate
c) Nifedipine
d) Phynetoin
e) Cyclosporin

245. what we can do?


a) Keep oral hygiene
b) For brushing; do up and down instead of left and right

246. A smoker patient 15 years old needs plan B


a) Dispense
b) Can't remember
If refuse to do dispense so  autonomy
If accept to do dispense and refuse to do council  beneficence

247. After a while he comes a doctor prescribed alesse and she wants to start on 5th day of her
cycle what you should say?
a) Use another method for first 7 days
Quick start is recommended to improve short term compliance and is Not associated with an increase in
unscheduled bleeding or other side effects. Back‐up contraception or abstinence should be used for the 1st 7
consecutive days of CHC use unless CHC was initiated on the first day of menses

248. After one month she came back and she said she forget to take her pill last night (it was
day 5) what will you say?
a) Discard this bach and start new one
b) Take one pill as soon as possible and continue the rest
Because she is calling last night so it means she loss 2 tablets. If calling in the same night take it as soon as
possible

249. Another question regarding OCP, I chose IUD but can't remeber what was the question
(looking for very effecient way for contraception)
250. which one cause erectile disfuction?
a) Hydrochlorthiazide

251. Homeopathic medicine? DIN-HM


Always look for a Natural Product Number (NPN) or Homeopathic Medicine Number (DIN‐HM) on the
product's label, which indicates that it has been approved by Health Canada. In order to be authorized for a
NPN, information supporting the safety and efficacy of a product must first be assessed by Health Canada

252. A patients has a family history of DVT and she was young what will you check?
a) Factor V leiden
b) Platlets
c) D-dimer
d) Modified ranking score  for stroke
If you are screening for DVT causes, then Factor V Leiden genetic test
If you are ruling out acute DVT, IT IS Dimer test
D Dimer is ONLY to R/O Acute VTE
Factor V Leiden screening is for young persons who unexpectedly endure A DVT/PE and do not have
apparent risk factors (unprovoked VTE)

253. A patient has low back pain for 3 days I think (completely forgot)
a) Do not stop
b) Avoid bed rest
c) Resume activity as soon as tolerated
d) Use acetaminophen with or without codeine.
e) All are true

254. Doctor prescribed NSAID for pain but he does not like take meds what will you suggest?
a) Hot pad
b) Cold compress
CTMA: The application of heat may provide short‐term relief in acute pain but there is insufficient evidence
to recommend the use of cold packs

255. Patient with GERD after eating pizza has abdominal pain and vomiting for 3 days (2-3
times/day) and he has seizure, takes Carbamazepine, he is seizure-free for 10 years, what is your
recommendation:
a) Refer to doctor for further assessment
b) Give him Ranitidine
c) Give him Omeprazole
CTMA: Further evaluation is required immediately if the GERD patient experiences any of the following
symptoms: Abdominal pain, Anemia, Chest pain, choking, vomiting, dysphagia, GI Bleeding, Odynophagia
and unintentional weight loss
Gastroesophageal reflux disease (GERD) can mimic epileptic seizure, and may be misdiagnosed as epilepsy.
On the other hand, GERD can be more commonly seen in children with neurological disorders such as
cerebral palsy (CP); this co‐incidence may complicate the management of patients by mimicking refractory
seizures.

256. A patients with osteoporosis and osteoarthritis I thinks he went swimming once a week his
work is located 2 km to his home what will you suggest?
a) Walk to her work
b) Increase swimming to 3 times a week
Exercise and Physiotherapy CTC
 Introduction of exercise at home and structured exercise under the guidance of a physiotherapist are
the key initial management strategies in OA.
 In knee OA, both resistance exercises and aerobic activities of different intensities were shown to
effectively reduce pain and improve function. However, optimal exercise programs should have 1 aim
and focus such as improving aerobic capacity or the strength of quadriceps muscles. Exercise is
associated with a reduction in functional limitation over 2 years. The benefit can be sustained in
some patients for 2–6 months after stopping the exercise regimen.
 Exercise programs can also reduce pain, improve physical function and reduce the need for total hip
replacement by 44% in patients with hip OA. Exercise programs can reduce pain and improve physical
function in hand OA.
 Yoga and tai chi may reduce pain and stiffness and improve physical function and quality of life for
patients with OA.
 Aquatic exercise can provide short‐term benefits for hip and knee OA pain. The benefits of swimming
can be similar to cycling exercises.
 Some patients may benefit from transcutaneous nerve stimulation (TENS); however, the
effectiveness of TENS has not been established.
 Acupuncture around an affected joint may offer temporary relief but the effect may not be sustained
or provide functional benefit, and a patient's response to acupuncture may depend on multiple
variables including site of application and the acupuncturist's style of application

257. A patients come and ask you about strontium for osteoporosis, where you should check?
a) Midline (because it is off label unapproved)
b) CPS
c) Pubmed
d) Canadian drug data base
e) Martindale
258. What is the side effect of anstrazole? Hot flash wasn't there
a) Anemia
b) Arthralgia
Another version: what is you concern about anastrazole? decreases bone density
Aromatase P450 Endometriosis patients tend to have high levels of Aromatase P450 expressed at their
inhibitors endometriums. Aromatase P450 inhibitors reduce levels of estrogen production from
Anastrazole sources other than the ovary, such as the adrenal gland.
Letrozole They can be used with OCs, GnRH analogues, or Progestins to reduce the risk of ovarian
cyst development.
S.E: hot flushes and sweating, osteoporosis, insomnia, body aches, loss of appetite.
Arthritis/arthralgia, joint pain/stiffness and hot flushes were reported very commonly
(≥10%), Common: (≥1%-<10%) asthenia, bone pain, myalgia, rash, N, diarrhea, headache

259. Which one is true about dermatitis?


a) Bath oils before bath can contain water better
b) Collodiol oatmeal is not as effective as bath oils in containing water
c) Tacrolimus is more effective than corticostroid
Colloidal oatmeal preparations contain starch and protein and are effective antipruritics. Addition of
oatmeal products may be soothing but does not promote increased water absorption. For dry skin, they are
not as effective as oils in trapping water to maintain hydration unless the oilated versions are used. Bathing
in colloidal oatmeal baths is useful when large body areas are involved
It has been thought that bath oils applied during or after bathing may help to reduce the rate of water loss
through the epidermis; however, a good‐quality RCT found that adding emollients to bath water did not
result in any significant clinical improvement in atopic dermatitis compared to bathing without emollients

260. elderly pt with history of earwax,


congested and hearing loss without pain
and tinnitus, what you recommend?
a) Refer
b) Olive oils 4-6 drops bid
c) Chlorbutol
Hearing loss, puruitis, fulness  ttt as he has
earwax and give carbamide
Hearing loss without anything  refer to know
what is wrong
See a health‐care provider (and do not syringe
the ears: Olive oil 4‐6 drops) if You have any of
the following: a rash in your ear, a history of
ear surgery, tubes in your ears, an ear
infection, received radiation treatment to the
area, sharp foreign objects in the ear canal,
You have any of the symptoms of a burst eardrum; these include: bleeding or discharge from the ear,
dizziness, pain in the ear, ringing in the ears & hearing loss

261. A patients who has sacabies and regweed allergy what we should give?
a) Crotamiton
b) Premethrin
c) Isopropyl myristate
Permethrin 5% is avoided in patients who are allergic to chrysanthemus/ragweed as it is whole body
Pyrethrin is CI in ragweed and chrysanthemum allergic patient
Crotamiton 10% is 2nd line alternative after Permethrin 5%

262. A patient with scabies what you should suggest?


a) Put everything in washing machine with hot water
b) Pruritis if increase with treatment, need more investigation
Counsel that itchiness (Pruritis) may persist or even increase over several weeks despite killing the mites and
is not by itself evidence of persistent infection: use Oral antihistamine or topical mild CS for pruritis
Wash bed linens and clothes in hot water and keep non washable items that are in direct skin contact in
sealable bags x 5‐7 days

263. A patient suffers from shingles comes with rash on her eye what you should suggest?
a) Trifluridine
b) Acyclovir oral used for 7 days in eyes
c) Refer to ophthalmologist for iv
corticostroid
Eye with herpes  refer to take prescription he
will take corticosteroid for pain & quality of life
If refer only in the choices we have to choose it
If refer + IV corticosterode is wrong because iv has
no use
Triflurodine will be a good choice if it is Herpes simplex virus infection not zoster

264. What do you suggest for papiloma virus vaccine?


a) It can be started for girls when they are 9
Vaccines for the Prevention of External Anogenital Warts
Q. Gardasil Prevention of external anogenital warts caused by HPV types 6 and 11.
9-valent human Females 9–45 y and males 9–26 y: 0.5 mL administered at 0, 2- and 6-months IM
papillomavirus Alternative 2-dose schedule in immunocompetent individuals 9–14 y: 0.5 mL administered
(types 6, 11, 16, at 0 and 6–12 months IM.
18, 31, 33, 45, If alternate dosing is required, the second dose should be administered at least 1 month after
52, 58) the first dose, and the third dose should be administered at least 3 months after the second
dose. All doses should be administered within a 1-year period.
recombinant S.E: Pain, swelling, erythema and pruritus at injection site, headache, fatigue, syncope,
vaccine lymphadenopathy, anaphylaxis. Not recommended in pregnancy, but used in breastfeeding.

265. A boy on methylphenidate ir BID at seven and 3 pm can't sleep what we should do?
a) IR bid take at 7 and 12
b) SR take at 12
c) SR take 7 and 3
Management of Common Adverse Effects of ADHD Treatment
Adverse Effect Monitoring Management
Appetite suppression Monitor for consistent appetite Maximize nutrition content when
Stimulants, suppression and changes in weight Q patient is not having symptoms of
atomoxetine, 2 wk for the first 2 months, then Q 6 appetite suppression (e.g., evenings,
bupropion months. before the morning dose of stimulant).
In children and adolescents, monitor Reduce portions & ↑ snacking times.
height as well. Consider nutritional meal supplements.
Consider drug holidays on weekends or
during vacations
Cardiovascular (HR, BP) Monitor BP and HR in the first 2 wk They should always be tapered; if
Stimulants  ↑ HR/BP. of starting a stimulant medication, stopped abruptly, they may cause a
Alpha-2 agonists  ↓ then Q 3 months. hypertensive crisis.
HR/BP. ECG not needed routinely if no If significant changes occur in BP, HR
TCAs  tachycardia. previous history of CV disorder. or ECG, discontinue and consider
consulting a cardiologist.
Psychiatric (anxiety, Monitor for difficulties falling asleep, Often worse upon initiation and
irritability, insomnia, tics) staying asleep and/or early morning resolves after 1–2 wk of therapy.
Stimulants, awakenings at 1 wk, then monthly for May need to lower the stimulant dose,
atomoxetine, the first 3 months, then Q 6 months. change time to earlier administration,
bupropion, Caregiver may use Sleep Disturbance add a more sedating medication at
venlafaxine Scale for Children or the Children’s bedtime or discontinue the offending
Sleep Habits Questionnaire to monitor stimulant. Minimize use of caffeine and
at home. other psychostimulants.
Limit stimulating activities (e.g., use of
electronic devices) in the evenings.
Overdosage Acute overdosage include: Dilated pupils, shallow rapid respiration, fever,
hyperpyrexia, chills, rhabdomyolysis, sweating, hyperactive tendon reflexes.
Benzodiazepines are first-line agents in amphetamine overdose for agitation,
movement disorders, seizures, tachycardia, and hypertension.
2nd line includes antipsychotics as chlorpromazine, ziprasidone or haloperidol.

266. A question regarding a person who takes both atomoXetine and methylphenidate
a) Take both of them verbally
b) Go to a doctor and get new prescription
c) Call the dr for verbal methylphenidate
267. A child now is on atomoxetin for one week and get headache, what will you consult?
a) Go to emergency
b) It is a transient side effect and will go away
c) Go to dr if it is bothersome
Common, usually transient: anorexia, insomnia, weight loss, irritability, dizziness, weepiness, headache,
abdominal pain. (monitor weight & appetite every 6 months)
Transient ‐ stop and re‐evaluate: “zombie‐like” effects, psychotic reactions (such as hallucinations),
agitation, tachycardia, hypertension, growth failure (Monitor growth suppression, record weight and height
at baseline and then every 3–6 months), rebound hyperactivity, leukopenia, blood dyscrasias.

269. A side effect of atomoxetin which is different by methylphenidate?


a) Not controlled not stimulant and cause sedation or somelance.
S.E: Suicidal ideation, sudden cardiac death, sedation, headache, rhinorrhea, abdominal pain, nausea small
increases in heart rate and blood pressure, liver toxicity, exacerbation of tics.

270. A patient is diagnosed for psychosis and he is stabilized on (not remember) how long he
should continue medication?
a) One year
b) 6 months
c) Reduce the dose

271. Which one is a sign of


withdrawal syndrome of opioids?
a) Diarrhea
b) Miosis
c) Seizure

272. When the sign of withdrawal of methadone is maximum?


a) 2 weeks
b) 3 days

273. There was a cancer patient who was on morphine and Doctor has increased the dose of
morphine and she suffers from myoclonus; her son came to pharmacy and need early refill you
look at his file and see some other early refills in the past
a) Call the doctor and increase the dose
b) Call the doctor and change to fentantyl
c) Because patient has pain give the meds
If happened myoclonus (twist the muscle), delirium, visual hallucination; call the dr to change opioid
If no myoclonus, call the dr to assess the case. Myoclonus is a sign of toxicity, so we need to change opioid
274. Which one can be taken by J-tube or Which one can be opened and sprinkled on food?
a) Something SR
b) Clindamycin
c) Dabigatran

275. A patient is amytripyilin 10 mg for pain he read side effects and desided not take it what
will you say?
a) The benefit of analgesic of amytripyilin outweigh of its side effects
b) The dangerous side effects just happen to rare people
c) It's a well tolerated medication and rare side effects can be managed

276. There were a study that the result was the people who are controlled does not need SMBG
so in which situation SMBG is good in this Population?
a) Patient as their personal interest
b) When they don't have adherence to meds
Sugar is controlled and patient is ok so give him device if patient ask
If patient on oral drug but controlled  to buy diabetic device depends on patient desire but if on insulin he
must have the device to determine his glucose level before insulin dose
Insulin
 If on insulin> 1time/day: (type 1 or 2): SMBG At least 3 times/day (both pre &postprandial)
 If insulin 1 time/day + other non insulin (type 2) SMBG at least once per day
If insulin or no insulin were used in type 2
 If glycemic targets are not met or the pt started on non insulin therapy associated with hypoglycemia
(secretagogues) SMBG >2 times per day until glycemic target is met followed by infrequent SMBG 2‐3
times/week
 If glycemic target met or the pt started on non insulin therapy NOT associated with hypoglycemia
(METFORMIN + OR – Incretin baes drugs or Pioglitazones): infrequent SMBG (2‐3 times / week is
adequate)
SMBG Tips: Fingertip > palm > forearm or thigh
 If pt hand is soiled: recommend minimal pressure to test the 2nd drop not the 1st to avoid false
elevated reading.
 In general, recommend against squeezing the fingertip, as squeezing can lead to inaccurate SMBG
reading due to sugar in the interstitial fluid rather than the blood
277. Patient’s serum cratinine has increased what was the reason?
a) Metfromin
b) Canagliflozin
Canagliflozin increases serum creatinine and decreases egfr in a dose dependent fashion. Should not be
initiated in patients with an eGFR <60 ml/min/1.73 m2, and should be discontinued when eGFR is below 45
The efficacy of canagliflozin is dependent on renal function, and efficacy is reduced in patients who have
moderate renal impairment and likely absent in patients with severe renal impairment.
In patients with an eGFR < 60 mL/min/1.73 m2 or CrCl < 60 mL/min, a higher incidence of adverse reactions
associated with volume depletion (e.g., postural dizziness, orthostatic hypotension, hypotension) was
reported, particularly with the 300 mg dose. In addition, in such patients more events of elevated potassium
and greater increases in serum creatinine and blood urea nitrogen (BUN) were reported.
Therefore, the canagliflozin dose should be limited to 100 mg once daily in patients with eGFR < 60
mL/min/1.73 m2 or CrCl < 60 mL/min and canagliflozin should not be used in patients with an eGFR < 45
mL/min/1.73 m2 or CrCl < 45 mL/min. Canagliflozin has not been studied in severe renal impairment (eGFR
< 30 mL/min/1.73 m2 or CrCl < 30 mL/min) or ESRD.
Monitoring of renal function is recommended as follows:
 Prior to initiation of canagliflozin and at least annually, thereafter.
 Prior to initiation of concomitant medicinal products that may reduce renal function and periodically
thereafter
 For renal function approaching moderate renal impairment, at least 2 times to 4 times per year. If
renal function falls persistently below eGFR 45 mL/min/1.73 m2 or CrCl < 45 mL/min, canagliflozin
treatment should be discontinued

278. Who is responsible for funding academic detailing?


a) Federal & provincial government
Academic detailing in Canada.
 The Canadian Agency for Drugs and Technologies in Health has (CADTH) created the Canadian
Optimal Medication Prescribing and Utilization Service (COMPUS), to work with federal, provincial,
and territorial ministries of health to identify and promote evidence based best practices in drug
prescription and utilization among healthcare providers and consumers.
 Funded by provincial agencies & Canadian Institute of Health Research.
 The Canadian Academic Detailing Collaboration (CADC) evaluating processes and outcomes of AD.
The majority of AD has targeted physician prescribing practices, and improvements observed in
physician prescribing of: Antibiotics, Benzodiazepines, NSAIDS.
 AD also targeted behaviors related to provision of preventive services or general management of
conditions commonly seen in general practice: Diabetes, Osteoporosis, Smoking cessation & Cancer
screening.
 Q. Who does not fund academic detailing? Manufacturer.
 Who does? Federal & provincial government
279. A patient is one efavirenz and....navir. Which one is correct?
a) Efavirenz inducer decrease methadone
It was really hard question you should know all the interactions of this medication
Methadone undergoes hepatic N‐demethylation by cytochrome P450 enzymes, principally CYP3A4, CYP2B6,
CYP2C19, and to a lesser extent by CYP2C9 and CYP2D6.
 Co‐administration of methadone with inducers of these enzymes may result in a morerapid
metabolism and potential for decreased effects of methadone, whereas administration with CYP
inhibitors may reduce metabolism and potentiate methadone’s effects.
 Although anti‐retroviral drugs such as efavirenz, nelfinavir, nevirapine, ritonavir, and lopinavir +
ritonavir combination are known to inhibit CYPs, they are shown to reduce the plasma levels of
methadone, possibly due to their CYP induction activity. Therefore, drugs administered
concomitantly with methadone should be evaluated for interaction potential; clinicians are advised
to evaluate individual response to drug therapy (see also Drug‐Herb Interactions, St. John’s Wort).

280. A patient is currently diagnosed with angina, hypertension, hyperlipidimia. Which of the
following is most appropriate?
a) Acebutelol
b) Lisinopril
c) Hydrochlorothiazide
d) Verapamil
e) Diltiazem

281. A 40 years old patient with suspected endometriosis based on her infertility and persistent
pelvic pain. Best therapy to treat her condition:
a) NSAID/opioids
b) Low dose OCs
c) GnRH analogs
As low‐dose COC therapy can be unlimited in duration and is associated with fewer adverse effects, it is
reasonable to consider GnRH agonist therapy only when low‐dose COC therapy ineffective or contraindicated

282. A cancer Pt decided to medical aided suicide due to unbearable agony of disease state. The
appointed treatment team including RPH decided to implement the AUTONOMY but
Pharmacist intern ask the preceptor to allow him to discuss with pt since he does NOT believe
in self-die. Now what will do by Preceptor?
a) It will break confidentiality if allowed
b) Allow intern and cautioned him not to challenge pt
c) Should not allow to keep Autonomy
d) Intern can discuss with patient as intern is a part of medical care
Pharmacist can’t assess medical suicide and can’t decide if he is eligible or not.
Usama said the decision should be voluntary by the patient
MAY 2018
1. A case about a patient who has MI and presents with coughing, chest pain, abdominal Pain
(epigastric pain), which other disease condition do you need to rule out?
a) Pancreatitis
b) PUD
c) GERD
d) FD
The following 3 questions establish whether the discomfort is considered nonanginal chest pain e.g GERD,
atypical angina or typical angina.
1. Is the discomfort substernal?
2. Are the symptoms precipitated by exertion?
3. Are the symptoms relieved within 10 minutes of rest?
Patients who respond “yes” to all 3 questions have typical angina. Patients who respond “yes” to 2 questions
have atypical angina. Patients who respond “yes” to 1 question or no questions have nonanginal chest pain.

2. KD is 49-year-old woman suffering from mild GERD symptoms from last two days. She
takes metformin 1000 mg BID and Sitagliptin 100mg OD, she has hypertension and on
ramipril, hydroclhorothiazide. Which of the following is appropriate action by the Pharmacist?
a) Recommend OTC Ranitidine 150mg BID
b) Recommend OTC famotidine 20 mg BID
c) Recommend calcium containing antacid as she is close to menopausal age
d) Refer

3. After using the recommended


drug for 4 weeks, she came to the
pharmacy complaining that her
symptoms still persistent, what to do
a) Give omeprazole
b) Refer the patient
c) Give another H2RA
d) Give antacids

4. Duration of PPI treatment for


GERD
a) 4 weeks
b) 8 weeks
c) 3 months
d) 6 months
5. In which of these conditions do you use PPI indefinitely
a) Barrets oesophagus
b) H. pylori
c) Refractory GERD  as referactory long term use not indefinitely

6. PPI side effects:


a) Hypomagnesaemia  also Osteoporosis, Vit B12 def., Risk of fall (Dizziness)
b) Hypocalcemia
Or; in nearest future what deficiency of vitamin will above patient have? Cyanocobalamin

7. What is the drug of choice for Zollinger Ellison syndrome?


a) Proton pump inhibitors
b) H2 receptor antagonists
Zollinger‐Ellison syndrome is a rare condition in which one or more tumors form in your pancreas or the
upper part of your small intestine (duodenum). These tumors, called gastrinomas, secrete large amounts of
the hormone gastrin, which causes your stomach to produce too much acid. The excess acid then leads to
peptic ulcers, as well as to diarrhea and other symptoms. The disease may occur at any time in life, but
people usually find out they're affected between ages 20 and 50.
Signs and symptoms of Zollinger‐Ellison syndrome may include: Abdominal pain, Diarrhea, Burning, aching,
gnawing or discomfort in your upper abdomen, Acid reflux and heartburn, Nausea and vomiting, Bleeding in
your digestive tract, Unintended weight loss & Decreased appetite.
Proton pump inhibitors are powerful drugs that reduce acid by blocking the action of the tiny "pumps"
within acid‐secreting cells. Commonly prescribed medications include lansoprazole (Prevacid), omeprazole
(Prilosec, Zegerid), pantoprazole
(Protonix), rabeprazole (Aciphex)
and esomeprazole
Octreotide (Sandostatin), a
medication similar to the hormone
somatostatin, may counteract the
effects of gastrin and be helpful for
some people.

8. Drugs of choice for CAP in


the ward?
9. Drug interaction between linezolide and morphine or Moclobemide  Serotonin syndrome
linezolid oral and morphine oral both increases affecting serotonin levels in the blood. Too much serotonin is
a potentially life‐threatening situation.
Serotonin syndrome: agitation, excitement, sweating, mental status change. Involuntary, twitching of
muscle, tremor, rigidity, myoclonus, diarrhea. Severe symptoms: hyperthermia, seizures, hyperreflexia,
thrombosis, delirium, respiratory arrythmia. Discontinue offending drug, supportive therapy, BZD,
dantrolene, Cyproheptadine

Case Study: Z.T. is a 26-year-old pregnant female diagnosed with CAP. She complains of fever,
chills, pleuritic chest pain, and nonproductive healthy and does not recall taking antibiotics in
the past year. Her only current medications are a prenatal vitamin and folic acid. She is in the
second trimester of her pregnancy.
10. The most appropriate antibiotic for treating Z. T`s pneumonia is:
a) Doxycycline 100 mg BID x 1 day, then 100 mg OD
b) Erythromycin 333 mg TID
c) Moxifloxacin 400 mg OD
d) Telithromycin 800 mg OD
Perform a chest radiograph in patients for whom CAP is suspected. In healthy pregnant women with no
recent antibiotic exposure, use of high‐dose amoxicillin is recommended.
Treat women with severe CAP with a beta‐lactam and a macrolide; erythromycin is the preferred macrolide
in pregnancy. Amoxicillin is the preferred beta‐lactam; ceftriaxone is an alternative
In most women, there will be clinical improvement within 48–72 hours, and therapy should not be changed
in the first 72 hours unless there is marked clinical deterioration. Fever should resolve in 2–4 days and cough
after 7–10 days. Radiologic abnormalities may persist for up to 6 weeks, so continued hospitalization is not
required to await radiologic improvement. Therapy is recommended for a minimum of 5 days for
uncomplicated CAP.

11. Z. T`s physician obtained a sputum culture. The lab result is Streptococcus pneumoniae
intermediate-level resistance to penicillin and resistance to tetracycline. Considering this
information, the most appropriate antibiotic choice is:
a) Amoxicillin high dose 1 g TID
b) Azithromycin 500 mg OD x 1 d, then 250 mg OD
c) Erythromycin 333 mg TID
d) Levofloxacin 750 mg OD
e) None of the above
Streptococcus Penicillin nonresistant (Minimum inhibitory concentration MIC <2 mg/L):
pneumoniae  Initial therapy: penicillin G, amoxicillin
 Alternatives: macrolide, PO cephalosporins (cefprozil, cefuroxime), IV cephalosporins
(cefuroxime, ceftriaxone, cefotaxime), clindamycin, doxycycline, respiratory
fluoroquinolones
Penicillin resistant (MIC ≥2 mg/L):
 Initial therapy: cefotaxime, ceftriaxone, respiratory fluoroquinolone (PO or IV)
 Alternatives: vancomycin, linezolid, H.D amoxicillin (3 g/day for penicillin MIC ≤4
mg/L)

12. CAP stepdown from IV PO, what to monitor post therapy change:
a) WBC.
b) Weekly Chest X-ray.
c) Weekly PFT.
d) Daily symptoms.
Switch patients from IV to oral antibiotics when the following criteria are met: GI tract is functioning
normally, hemodynamically stable; 2 temperature readings are normal, normalized white blood cell count;
subjective improvement in cough and shortness of breath; able to consume oral medications.
Other opinion A: as WBS must become normal to shift fro iv to oral

13. Case about CAP + Diabetes and will be treated outpatient?


a) Amoxclav. (High Dose)
b) Azithromycin IV
c) Erythromycin

14. Which of the following drugs should be prepared in a biological safety cabinet?
a) Gemcitabine

15. Instruction for Horizontal laminar flow hood:


a) Operate for 30 minutes before start.
b) Work 6 cm from edge.
c) Clean from side to side from backward.
d) Close the UV light when operating.
16. a case about a patient on levothyroxine in the morning, sildenafil in the evening, presents
with hypotension in the morning. What could be the reason for the morning hypotension
a) Use of sildenafil at night
b) Use of smaller cuff size for BP measurement
CVS: abnormal electrocardiogram, angina pectoris, arrhythmia, AV block, cardiac arrest, cardiomyopathy,
heart failure, hypertension, hypotension, palpitation, postural hypotension, myocardial ischemia, syncope,
tachycardia, varicose vein, vascular anomaly;
The manufacturers of sildenafil, tadalafil and vardenafil recommend that these drugs not be used in
combination with nitrates. In situations where nitrate use is required in a patient also receiving a
phosphodiesterase 5 inhibitor, effect on blood pressure can be reduced if there is sufficient time between
doses. Separate doses of nitrates and sildenafil and vardenafil by at least 24 hours. Allow 48 hours
between tadalafil administration and nitrates.

17. Which Of his medication will interact with iron


a) Levothyroxine
b) Sildenafil
Take morning on empty stomach. Absorption may be reduced by antacids, Ca salts, cholestyramine
(separate administration by at least 6 h), colestipol, iron salts.
T3 is not reported to interact with iron salts.
Monitor TSH levels to adjust initial dosage after 6 to 8 wks then as required or annually
One way to make a quick temporary dosage adjustment and allow the patient to use the levothyroxine
tablets they have on hand is to ask the patient to take 1 extra pill per week (if TSH is slightly above target)
or 1 fewer per week (if TSH is slightly below target).
Pediatrics: SYNTHROID Tablets may be given to infants and children who cannot swallow intact tablets by
crushing the tablet and suspending the freshly crushed tablet in a small amount of water (5 to 10 mL),
breast milk or non‐soybean‐based formula. The suspension can be given by spoon or dropper. Do not store
the suspension for any period of time. The crushed tablet may also be sprinkled over a small amount of food,
such as apple sauce. Foods or formula containing large amounts of soybean, fibre, or iron should not be
used for administering SYNTHROID.

18. How long do you have to wait when switching from an ACEI to ANRI?
a) 2days
b) 3days
When switching from an ACE inhibitor to sacubitril/valsartan, allow a washout period of 36 hours between
the two treatments. The washout period is not needed when switching from an ARB to sacubitril/valsartan.

19. Non-pharmacological therapy for a 6-years old boy with nocturnal enuresis?
a) Restrict day time fluid intake
b) Enuresis alarm
c) Desmopressin.
Enuresis alarms are effective for children ≥ 7 years of age when used properly for 3–4 months.
20. Which of the following will you suspect is a forged prescription
a) a prescription that was faxed from a patient’s place of work
b) a prescription in which the prescriber is not known to you

21. A patient on methotrexate 0.01mg per kg wants to take zostavax vaccine. What is your
recommendation?
a) Can take it safely
b) Wait for 3months after stopping methotrextae before taking Zostavax
Live vaccines should not be given to patients using immunosuppressive therapy until 3 months after these
therapies are stopped. While herpes zoster (zostavax) is also a live vaccine, it is not used to elicit a primary
immune response; therefore, it may be considered safe to administer to patients receiving low‐dose
immunosuppressive therapy: methotrexate ≤0.4 mg/kg/week, azathioprine ≤3.0 mg/kg/day, 6‐
mercaptopurine ≤1.5 mg/kg/day, prednisone <20 mg/day (or equivalent).

22. What to monitor while using methotrexate? Renal & hepatic functions, Chest X-Ray
Q. A folic acid antagonist that inhibits cytokine production and purine nucleotide biosynthesis,
Methotrexate leading to immunosuppressive and anti-inflammatory effects.
S.E: Nausea, flulike aches, headache, oral ulcers, bone marrow and liver toxicity, pneumonitis,
immunosuppression, lymphoma.
Alcohol restriction may minimize hepatotoxicity. NSAIDs may increase serum concentrations.
Take leucovorin (folinic acid) 1 mg OD on a different day than the methotrexate, separated by
≥24 hours to reduce severity of adverse effects.
Contraindicated in pregnancy and breastfeeding owing to its teratogenic and cytotoxic effects.
Recommend effective contraception. Discontinue 3 – 6 months prior to attempted conception.
Q. Baseline assessment should include complete blood count (CBC) with differential and
platelet counts, hepatic enzymes, renal function tests and chest xray. Monitor hematology at
least monthly, and hepatic enzymes and renal function every 1–2 months.

23. Most common SE of Methotrexate?


a) Nausea.
b) Pneumonitis
c) Dry cough
24. A patient using methotrexate for RA and is complaining about performing daily activities at
home. which of the following healthcare professional will you recommend?
a) Physiotherapist
b) Occupational therapist
Physio Outpatient and Assess, treat, prevent and manage injury, pain, disease or disorders.
therapists or hospital or Optimize client’s physical function, functional independence, mobility
Physical rehabilitation-based and quality of life. Educate clients and others about health, wellness,
Therapist practice fitness and self-management
(PT) Physiotherapy treatments can includ: Personalized exercise programs
designed to improve strength, range of motion, and function. Massage.
Joint mobilization and manipulation to reduce pain and stiffness.
Occupational Outpatient, hospital, Develop individual and group programs with people affected by illness,
Therapists rehabilitation-based injury, developmental disorders, emotional or psychological problems
(OT) practice, senior and ageing to maintain, restore or increase their ability to care for
home themselves and to engage in work, school or leisure
Develop and implement health promotion programs with individuals,
community groups and employers.
OTs may specialize in a specific age group or disability as those with
arthritis, coordination disorder, mental illness, or spinal cord injury.

25. Cancer patient with diabetes and high blood pressure and started on dexamethasone for
nausea and vomiting about 1 week now. After taking steroid dexamethasone for 1 week, which
of the following effect would be seen in the patient?
a) Increase in blood glucose
b) Hypoglycemia
c) Osteoporosis
d) Constipation
26. the pharmacist recommends hin Ondansetron, then he came constipated after 2 days, what is
the apprpriate action?
a) Refer to doctor.
b) Give laxative
As ondansetron is known to increase large bowel transit time, patients with signs of subacute intestinal
obstruction should be monitored following administration.
Ondansetron has a dose dependent QTc prolongation effect. For IV administration, the effect is expected to
be greater with a faster rate of infusion. Using the minimum effective dose and a slow rate of infusion
should always be favored. Avoid ondansetron in patients with congenital long QT syndrome.
Ondansetron should be administered with caution to patients who have or may develop prolongation of
QTc, including congestive heart failure, bradyarrhythmias or patients taking other medicinal products that
lead to either QT prolongation or electrolyte abnormalities. Hypokalemia, hypocalcemia, and
hypomagnesemia should be corrected prior to ondansetron administration

27. Why is aluminium phosphate or aluminium hydroxide added to vaccines?


a) To increase the body’s immune response to vaccines (immunogenicity enhancer)
b) To inactivate viruses and toxins  formaldehyde
c) To protect them against freeze-drying or heat  gelatin
Aluminum hydroxide, aluminum phosphate, and potassium aluminum sulfate (alum) are used as adjuvants
in vaccines designed to stimulate systemic immunity. Aluminum salts are used in DPT, pneumococcal
conjugate, hepatitis A, papilloma, anthrax, and rabies vaccines.

28. A case about a patient


with stable angina. which of
the following medications
will reduce the symptoms
of angina?
a) NTG SL
b) LA CCB
c) BB
d) LA NTG.

29. Non-pharmacological therapy for RLS due to varicose veins


a) Increase mental alertness
b) Increase physical activity
c) Compression Stock
d) Use scleratherapy
Non‐pharmacological therapy for other causes: Mental activity (cross words, suduku), avoid coffee and alcohol and
caffeine, take hot bath, stretch before sleep & decrease sleep deprivation. In patients with RLS and varicose veins,
consider sclerotherapy to improve RLS symptoms
30. after a period of time, patient symptoms became more frequent and bothersome, need daily
therapy, what drug should be given?
a) Pramipexole
b) Cyclobenzaprine
c) Levodopa

31. How to differentiate/Clarify use of pramipexole in RLS and dementia?


a) Add to the patient profile in system.

32. Which of the following patient will you refer? 7year old with anal itching
Refer all those with suspected pinworm infestation to a physician so the diagnosis can be confirmed. This is especially
important in pregnant women, children under 12, and those with renal or hepatic impairment

33. Lice case about a 3yr old child whose mother was apprehensive and worried about the
chemicals used to treat lice and its effects. What do you say to show empathy?
a) Tell the mother not to worry about the lice that you will treat the child
b) Lice are very common in children
c) Do not worry about the treatment, I’ll choose safe & effective option for your child
Sympathy is being emotional without help. Empathy is to be emotional and offer the best help.

34. the mother was also worried about the dog getting lice from the child. What to tell her?
a) Dogs do not carry human lice
Identify and examine potential human contacts to prevent a cycle of reinfection. For head lice, only those contacts
with live lice or nits within 1 cm of the scalp should be treated. Pets do not transmit human lice and should not be
treated. All contacts of a person with scabies, even if asymptomatic, require treatment. This includes sexual and close
personal or household contacts within the preceding month

35. Drug of choice for the child?


a) Permethrin 1%
b) Permethrin 5%
c) Pyrethrins/piperonyl butoxide
d) Dimeticone (dimethicone).

36. Head Lice Treatment Failures


a) Permethrin 5%
b) Pyrethrins/piperonyl butoxide
c) Topical ivermectin
In cases of head lice treatment failure:
• Switch to agent of different pharmacologic class
• Permethrin 5% cream applied to scalp and left on overnight
• Oral TMP TMP‐SMX with permethrin 1% or topical crotamiton 10% applied to scalp, left on for 24h (adults)
• Oral ivermectin 200mcg/kg repeated in 10 days or 400 mcg/kg repeated in 7 days
Unconventional treatments:
• Cetaphil cleanser applied to scalp to dry in place shown to be effective in one uncontrolled study
• No substantial evidence for hot air, hair styling gels, mayonnaise, vinegar, isopropyl alcohol, olive oil,
melted butter, petroleum jelly, acetomicellar complex, or tea tree oil

37. Patient takes 5mg prednisone tab. She is taking 10mg/day for the 1st week then dose
decreased by half tab every week. Calculate total tab required for 4 weeks.
1st week: 10 x 7 = 70 mg
2nd week: 7.5mg x 7 = 52 mg
3rd week: 5mg x 7 days= 35 mg
4th week: 2.5 mg x 7 day = 17.5 mg
Total =70+52+35+17.5 = 174.5 mg / 5mg per Tablet = 35 tablets

38. Drug of choice for a pregnant woman with bacterial vaginosis?


a) Oral metronidazole
b) Refer to family doctor
Pregnancy: Metronidazole 500mg PO BID x7d
Alternative: Clindamycin 300 mg PO BID x 7d  Avoid clindamycin Cream
Sexual partners: No need to be treated
Follow‐up: No follow‐up is necessary unless the patient is pregnant or has recurrent symptoms
If Fungal Pregnant: Topical azole antifungal then oral azole for at least 7 days

39. Which of the type of dementia will get worse when given antipsychotic?
a) Lewy body dementia
b) Vascular dementia

40. A patient called in to make enquiry about the rizatriptan she took 30mins ago but yet to see
any relief. What do you counsel?
a) Do not repeat if no relief at all
after 2hours

41. What to do in the next episode?


a) Change to another triptan 
(after 24hrs)
b) Add NSAIDs
c) Add Opioid.
Subcutaneous sumatriptan has the fastest
onset of action and remains the most efficacious triptan for a severe migraine attack
42. Case about a child with ADHD on atomoxetine. His Mother is concerned as the child
experience Hypertension without headache and suicidal ideation. What to monitor?
a) Suicidal thoughts
b) Addiction potential
c) Height and weight
Suicide‐related behaviour (suicide attempts and suicidal ideation) has been reported in patients treated with
atomoxetine. In double‐blind clinical trials, suicide‐related behaviours were uncommon, but more frequently
observed among children and adolescents treated with atomoxetine compared to those treated with
placebo, where there were no events. In adult double‐blind clinical trials, there was no difference in the
frequency of suicide related behaviour between atomoxetine and placebo. Patients who are being treated
for ADHD should be carefully monitored for the appearance or worsening of suicide‐related behaviour.

43. What is the appropriate action for this patient?


a) Refer to pediatric specialist in ADHD
b) Add clonidine
c) Stop medication and reassess
d) Add Guanfacine
Common, usually transient: anorexia, insomnia, weight loss, irritability, dizziness, weepiness, headache,
abdominal pain. (monitor weight & appetite every 6 months)
Transient ‐ stop and re‐evaluate: “zombie‐like” effects, psychotic reactions (such as hallucinations),
agitation, tachycardia, hypertension, growth failure (Monitor growth suppression, record weight and height
at baseline and then every 3–6 months), rebound hyperactivity, leukopenia, blood dyscrasias.

44. A case about a patient with anaemia. His MCV is High and Hb is low what do you give?
a) Vit B12 injection plus folic acid
b) Vit B12 injection (IM/SC)
c) Iron supplement
The first line of treatment for many people is correcting nutrient deficiencies. This can be done with
supplements or foods like spinach and red meat. You may be able to take supplements that include folate and
other B vitamins. You may also need vitamin B‐12 injections if you don't absorb oral vitamin B‐12 properly

45. What to do if there is a Shortage of B12 injection


a) Shift to high dose oral
b) Find alternative

46. Bacterial skin infection case. A child has boils with mild crusts. What is the drug of choice?
a) Topical mupirocin
b) Oral cephalexin
Impetigo: Mupirocin & Fusidic acid are considered the treatment of choice for patients with uncomplicated
impetigo localized to 2 or 3 small areas. Both are schedule I, prescription drugs & equally effective
47. the above patient came back and said the skin infection got better and it later became worse.
The mother thinks it’s because the daughter was Itching it but the daughter denied doing so.
what do you document in the patient profile?
a) Despite giving something for it, it got worse
b) Mother and daughter’s narrative do not correlate

48. Beneficienc question. which of the following cases will show beneficience?

49. Case about patient had influenza and was in bed for 6-days, not a smoker. Now he has DVT.
which of the following could have resulted in the DVT?
a) Influenza
b) Long period without mobilization
c) Smoking

50. Which of the following medication can be given with a JTUBE & Nasogastric tube
a) Dutasteride
b) Tamsulosin CR
c) Clindamycin capsule
A jejunostomy tube (J‐tube) is a soft, plastic tube placed through the skin of the abdomen into the
midsection of the small intestine. The tube delivers food and medicine until the person is healthy enough to
eat by mouth. A minority of medicines althougth formulated as liquids may not be suitable for tube
administration e.g. lansoprazole suspension.

51. As a pharmacist, what can you delegate to the pharmacy technician when you are busy?
a) Arrange Data entry, prescription verification, filling, pickup and product check
b) Take verbal narcotic prescription
c) Counsel on prescription medications
d) Counsel on OTC

52. During influenza outbreak, what is the best way to protect your staff
a) Taking pregnant staff away from patient’s
b) Staff with high risk household contacts should wear protective covering
c) Tell people that have not been vaccinated to use face mask

53. For Breakthrough influenza in LTC facility, how to protect asymptomatic residents:
a) Isolation
b) No visits
c) Give all oseltamivir as prophylaxis
d) All members should be infection free for 10 days.
Prevention of influenza
Post‐exposure prevention in individuals 1
year of age or older following contact with a
clinically diagnosed influenza case when
influenza virus is circulating in the
community. The appropriate use of Tamiflu
for prevention of influenza should be
determined on a case by case basis by the
circumstances and the population requiring
protection. In exceptional situations (e.g. in
case of a mismatch between the circulating
and vaccine virus strains, and a pandemic
situation) seasonal prevention could be
considered in individuals one year of age or
older.
Tamiflu is indicated for post‐exposure
prevention of influenza in infants less than 1
year of age during a pandemic influenza
outbreak

54. A case about a patient on duloxetine and was on plenty medications. Which medication
duloxetine interact with?
a) Aspirin
b) Naproxen
c) Amiodarone
Duloxetine is a CYP2D6 substrate, while amiodarone is a CYP2D6 inhibitor.

55. case about a patient who has


asthma and on symbicort and
salbutamol. Patient had influenza
72hours ago. what do you do?
a) Don’t treat as it is more
than 48hours
b) Give Zanamivir
c) Give Amantadine
d) Give Oseltamavir
Do not use Zanamivir in patients with
asthma or COPD due to risk of serious
bronchospasm
56. 68 yrs old female taking escitalopram, rosuvastatin, antacid question about which
combination of medication will cause QT prolongation
a) Moxifloxacin and ondasetron
b) Azithromycin + Citalopram
Despite B can cause QT prolongation, option A is more powerful.

57. patient with rhinorrhoea already used


ICS and not working. What do you give:
a) Ipratropium nasal spray
b) Xylometazoline nasal spray
c) Oral loratidine
Ipratropium may be used when rhinorrhea is the
only symptom or when rhinorrhea is refractory
to topical INCS and/or antihistamines.

58. which of the following is an opioid


withdrawal symptom
a) Epistaxis
b) Diarrhoea
c) Seizure
d) Miosis
Opioid withdrawal symptoms: dysphoric mood,
N&V Muscle aches, rhinorrhea, lacrimation,
young, insomnia and dilated pupil (mydriasis).
piloerection and sweating. Peak in 36‐72 hours

59. what is the treatment of Opioid toxicity symptoms?


a) Narcan kit (Naloxone IM)
b) Thiamine iv
c) Lorazepam

60. Nurse did an administration error of Vancomycin and infused it rapidly. which of these
effects the patient will experience?
a) Rigor (red man syndrome)
b) Hypotension
c) Seizure
d) Fever
Following rapid IV administration, a maculopapular or erythematous rash on the face, neck, chest and upper
extremities as well as pruritus, hypotension, angioedema, chest pain, wheezing, dyspnea, urticaria and
flushing may occur. The reaction usually begins a few minutes after infusion initiation but may not occur
until after the infusion is completed and generally resolves spontaneously over several hours after the
infusion is stopped. This anaphylactoid reaction, referred to as red man syndrome, occurs in response
to vancomycin‐induced histamine release and is not a true hypersensitivity syndrome.
The likelihood of this reaction is proportional to the rate of infusion and therefore vancomycin should be
infused over a period of ≥60 minutes. Pretreatment with an antihistamine (hydroxyzine, diphenhydramine)
may help prevent or minimize a reaction. In patients who have experienced this reaction, lengthen the time
of infusion for subsequent doses. Monitor blood pressure during infusion. Concomitant administration of
anesthetic agents may increase the frequency of infusion‐related events.

61. What is true about Sunscreen?


a) Should be re-applied after swimming immediately
b) Sunscreen SPF 30 provide double protection of sunscreen SPF 15
c) Physical sunscreen SPF 30 protect against UVB only
d) Apply DEET, and after 20 min apply sunscreen before outdoor activities.
e) Apply before going out by 60 min.

62. In hospital patient has received 1 gm Vancomycin after 2 hr the blood conc. was 35
m.mol/L. after 72 hr the conc. was 17 m.mol/L & he has received a second dose of 1 gm
Vancomycin. Peak conc. is 20 m.mol/L, trough 15 m.mol/L when should he take the third dose?
a) 1 day after the second dose
b) 2 days after the second dose
c) 3 days after the second dose
d) 5 days after the second dose
Answer:
Log C = log Cₒ - k*t /2.303 k = (2.303 / t) * log (Cₒ / C)
k = (2.303/70) *log (35/17) = 0.010 hr-1
Log C = log Cₒ - k*t /2.303 Log 35 = log Cₒ - (0.010*2) / 2.303
Log Cₒ = 1.544 + 0.008 = 1.552 ----------------- Cₒ = 35.7 m.mol/L
So, upon the second dose:
Cₒ = 35.7 + 17 (remains from the 1st dose) = 52.7 m.mol /L
C = 15 m.mol / L. (trough conc. at which the 3rd dose has to be taken)
k = 0.010 hr -1 Log C = log Cₒ - k*t /2.303 t = (2.303 / k) * log (Cₒ / C)
t = (2.303/0.010) * log (52.7/15) =125.679 hr = 5.23 days
So, the 3 rd dose has to be taken 5 days after the 2nd dose
63. Which of the following will cause weight gain?
a) Bupropion
b) Quietiapine
c) Topiramate
Quetiapine SE: Sedation, hyperglycemia, weight gain, QT prolongation, TSH

64. Which antipsychotic causes the least metabolic and weight gain side effects
a) Ziprasidone
b) Clozapine
c) Olanzapine
d) Quetiapine

65. A customer come to you and ask for mutivitamin and minerals information for protection
against AMD, what to counsel
a) Copper is given for anaemia caused by lutein
b) Omega 3 will help prevent disease progression
c) Vitamins and supplement will only help for dry and not wet
d) Lutein used as alternative of B carotene to reduce the risk of Lung cancer.
Why we add copper to the vitamin supplement we add to AMD to reduce Cu deficiency that may result from
Zn.

66. Patient with hyperkalemia 5.9.


What will you give?
a) Insulin plus glucose
b) Calcium gluconate
c) Sodium polystyrene
d) Lactulose
In severe hyperkalemia (K+ >6.5
mmol/L) or when significant or
advanced ECG changes are present
(loss of P waves or widening of QRS
complexes), continuous cardiac
monitoring should accompany
treatment. Give IV calcium promptly
and begin insulin. Initiate K+ removal simultaneously.
In less severe situations, K+ removal with or without redistribution agents (e.g., insulin) may be sufficient.
Estimate renal function and ongoing gain of K+ in extracellular fluid. Initiate treatment early and more
aggressively when renal failure is present or there is rapid and severe release of intracellular K+ (e.g.,
rhabdomyolysis, tumor lysis syndrome) than when there is slow or no input (e.g., hyperkalemia induced by
K+‐sparing diuretics).
67. All of the following are cold chain drugs except;
a) Latanoprost
b) Brimonidine eye drops
c) Dukoral Vaccine
d) Trifluridine eye drops

68. A case about a patient using antidepressant and now had psychosis. Which medication will
you add on?
a) Quetiapine
b) Clozapine

69. Diabetic patient has foot infection due to crash in the furniture what causative organism?
a) Clostridium perfringens
b) Streptococcus agalactiae
c) Staph aureus

70. Drug of choice for the patient above if he has penicillin allergy? Clindamycin

71. Cat bite is caused by which organism? pasteurella multocida

72. Drug of choice for dog / cat bite? Amoxicillin plus clavulanic acid
Cat bite ‐ AMoxi/clav 2nd: clinda or metro+ 2nd or 3rd generation cephalosporin OR SMX/TMP or FQ or
doxycycline.

73. Dukoral vaccine. What to counsel


a) Take with food
b) 2 doses to be taken before travel
c) Booster dose after 5years

74. How to minimise insulin errors in the hospital ward


a) Use pencilled syringes and label
b) Educate nurses on danger of giving wrong dose

75. What will you recommend for short term treatment for a patient unable to sleep and has
comorbid depression
a) Buspirone
b) Triazolam
c) Clonazepam
Insomnia as Side effect Usually diminishes after 1–2 wk of therapy. Daytime dosing. Consider short‐term use
of hypnotic (e.g., zopiclone) during initiation & Counsel on sleep hygiene
76. Narcotics destruction what is true
a) Need authorization before destruction
b) Must be witnessed by 2 pharmacists
c) Just thrash it in the bin
d) Records and quantity of disposal must be kept for 2 years

77. You receive a prescription for Tamsulosin for a female patient. When you inquire about the
indication, the patient explains that it is to facilitate the removal of kidney stones. Pharmacist
can find information about off-label indications in all of the following references EXCEPT:
a) PDR
b) AHFS
c) Micromedex
d) Clinical Pharmacology

78. which reference to search about Y-site drugs incompatibility?


a) PDR
b) AHFS
c) Micromedex
d) Clinical Pharmacology
A y‐site is an injection port on the side of an IV line

79. The best source for Drug comparison is? RX file


80. Somebody has a cancer and wants to know about the quality and safety of natural products
for skin. where to check?
a) Natural product database
b) Ask the doctor

81. Drug interaction between amiodarone and cyclosporine


a) Increase cyclosporine blood levels
b) Increase amiodarone blood levels
c) Decrease cyclosporine levels
d) Nothing will happen
Amiodarone is a CYP3A4 inhibitor, while cyclosporine is CYP3A4 substrate.

82. A female patient weight is 138 pound and 57 years old; her
serum creatinine level is 150. what is her Cr. Cl?
Answer: Weight in kg = 138/2.2 = 62.73
Cr. Cl (ml/min) = (140 – 57 x 62.73) / 72 x 150 = 0.48ml/min
Female C. Cl = 0.85 x 0.48 = 0.41ml/min

83. Patient hospitalized discharged and comes with prescription of perindopril and had had
angioedema before. What to do
a) Refuse to fill as he has had that side effect before
b) Fill it
c) Tell the doctor to change to candesartan
In case of a switch from ACE inhibitors to ARBs, it seems reasonable to stop ACE inhibitors and start ARBs
the following day at an equivalent dose

84. Arrythmia patient using phenytoin, what is appropriate counselling?


a) Don’t use chlorhexidine M.W since it stains
b) Take exactly full dose, Folic acid use, maintain Oral hygiene.
Drug‐Food Interactions
The presence of food may affect phenytoin absorption, but may also reduce its gastrointestinal side effects.
Taking phenytoin with food on a consistent basis may minimize any food‐associated fluctuations in
bioavailability. Phenytoin bioavailability can be significantly reduced in the presence of enteral nutrition
products. Tube feeds should be held for 2 hours before and after phenytoin doses
Excessive gingival overgrowth itself can also changes the gingival contour, impeding oral function and
speech and having an anti‐aesthetic effect. Additionally, it can also compromise effective oral hygiene and
may have negative implications for the systemic health of affected patients. Several approaches for the
treatment of gingival overgrowth have been proposed: basic periodontal therapy, including oral hygiene
instructions, prophylaxis, scaling and root planing, and continuous motivation as well as surgical therapy
(gingivectomy or a flap procedure), antiseptic mouthwashes, and change of medication
85. If doctor give divalproex instead of valproic acid, what is correct about it?
a) Monitor hepatic enzymes especially in the first 6 months
b) GI tolerated better with valproic acid
c) Can be used in pregnancy
Liver function should be measured before therapy and then periodically monitored during the first 6 months
of therapy, especially in those who seem most at risk, and those with a prior history of liver disease

86. A young girl notices white patch on her back, no disease condition except that she is using
OCP what can cause it?
a) Not using enough sunscreen
b) Photosensitive reaction to OCP
Other version: COC causes Cholasma‐‐‐‐‐‐‐ttt by hydroquinone.
Melasma (also known as chloasma faciei, or the mask of pregnancy when present in pregnant women) is a
tan or dark skin discoloration. It is thought to be caused by sun exposure, genetic predisposition, hormone
changes, skin irritation. Usually resolves after pregnancy. However, up to 30% of cases may be persistent
TTT: Broad‐spectrum sunscreen + Topical hydroquinone 2–4% applied BID to affected areas may be effective

87. which of the following has longer duration of action


a) Codeine
b) Hydromorphone
c) Oxycodone
d) Fentanyl patch

88. Patient has a fentenyl patch on, wants to undergo MRI. What will you ask the patient to do?
a) Remove the patch as might cause the skin to burn
b) Leave the patch in its site.
c) Change the patch site
Some transdermal patches containing aluminum or other metal in their nonadhesive backing shouldn’t be
worn during MRI because of skin burn risk.
Health care professionals should advise patients wearing medication patches about procedures for proper
removal and disposal before MRI and replacement afterwards.
Transdermal Patches That Must Be Removed Before MRI
Patch Comment
Clonidine Remove before MRI
Catapres-TTS Reapply same patch to another site afterwards
Diclofenac Remove before MRI. Reapply same patch afterward if <2 hours have elapsed since
Flector patch was removed. Apply new patch if longer
Estradiol Remove before MRI
Testosterone For reapplication after the procedure, follow same instructions as for patch falling off
Lidocaine and Tetracaine  Remove before MRI
Fentanyl Remove before MRI
Duragesic After procedure, apply a new patch to an alternate site
Methyl Salicylate OTC product
and Menthol Remove before MRI
Salonpas Replace with fresh patch after procedure
Nicotine To avoid confusion, recommend removing patch, regardless clear or opaque, before
MRI. Reapply same patch after procedure
Oxybutynin Removal before MRI is recommended because manufacturer can’t guarantee product
Selegiline doesn’t contain metal
Rivastigmine Consider removal before MRI
Patch hasn’t been studied in patients undergoing MRI
Rotigotine Remove before MRI or cardioversion
Scopolamine Remove before MRI. Place new patch on an alternate site following procedure

89. Side effect of cytarabine used for a cancer patient


a) Conjunctivitis
b) Ototoxicity
c) Hand-foot syndrome
Conjunctivitis Anthracyclines Artificial tears may have a Usually reversible upon discontinuation
(within days) Cytarabine dilutional effect. of offending agent.
(Dose > 2g/m2) For cytarabine, prednisolone 1% Artificial tears to provide local relief.
Fluoro or dexamethasone 0.1% eye drops Corticosteroid ophthalmic preparations
pyrimidines Q4–6H started before & continue to manage inflammation.
> 48 h post cytarabine. Ocular pain may require analgesics.
Ophthalmic diclofenac may also
have a role.

90. HIV case. patient with viral load 90,000 and CD4 580,
What is the reason to start treatment?
a) Viral load
b) CD4 count
Another version: When should treatment with antiretroviral
drugs be started in a patient who is HIV-positive?
a) As soon as possible
b) Once progression to AIDS occurs
c) At the time of detectable viral loads
d) When CD4 counts drop to less than 500 cells/mm3
 plasma HIV ribonucleic acid (RNA) level (viral load), used in conjunction with the CD4+ T‐lymphocyte count
(CD4 count), is the best prognostic marker for progression to AIDS and survival in untreated
patients.[11] Plasma viral load ranges vary according to the test employed. There is no “safe” level. The
most commonly used plasma HIV‐1 RNA assay has a quantitation limit of 40 copies/mL
 CD4 count and percentage are useful in determining where a patient lies in the continuum of HIV disease
and the need for specific intervention (see Table 1). Knowledge of the CD4 count can also help to narrow
the differential diagnosis in a symptomatic HIV‐infected patient. In adults, a CD4 count of 430–1360
cells/mcL (0.43–1.36 Giga/Litre or G/L) is considered normal in most laboratories
 HIV drug resistance should be assessed in all patients at baseline (as soon as possible after HIV is
diagnosed) and in cases of confirmed viral load rebound to >200 copies/mL while on treatment
 After infection, viral load levels are very high, but then your body fights back and it drops to much lower
levels. Over time though, usually over several years, the levels of virus increase again. It is usually high
(around 50,000‐100,000 copies/mL) by the time that your CD4 count drops to around 350 cells/mm3.
 After starting treatment, viral load falls quickly and CD4 counts rise slowly.
 If ART brings viral load down to less than 50 copies/mL, then treatment can last for many years

91. Tetanus needle size:


a) IM angle 90 - 22 to 25-gauge 5/8”
Subcutaneous (MMR, Zostavax) are only 5/8”
In IM: 5/8 In old or <60 Kg, 1‐1.5 inch in normal, 1.5” in obese
92. The most common pathogen of Spontaneous bacterial peritonitis
a) E. coli [40%]
b) K pneumoniae [7%]
c) Streptococcus pneumoniae [15%]

93. What is the drug of choice?


 Community‐acquired SBP: treat immediately with an empiric third‐generation cephalosporin (e.g.,
cefotaxime or ceftriaxone) for 5 days.
 Nosocomial‐acquired SBP: treat empirically with piperacillin/tazobactam or meropenem/ glycopeptide.
 Norfloxacin is th drug of choice for prophylaxis
 Laboratory tests: culture and polymorphonuclear (PMN) cell count of ascitic fluid; repeat after treatment
to ensure resolution of the infection.

94. Case about HIV patient with oral thrush and has difficulty in swallowing and now
developed a systemic infection. What to do
a) Nystatin
b) Amphotericin
c) Capsofungin
d) Fluconazole

95. Case about alcoholic and diabetic patient having pancreatitis, what to monitor
a) Amylase & Lipase
b) AST & ALT

96. What to give for diabetes control?


a) Insulin
b) Sitagliptin
c) Liraglutide
d) Metformin
Facts on Pancreatitis
 Symptoms: Acute and intense pain, epigastric abdominal pain radiating to the back, Right upper
quadrant pain, anorexia, and nausea
 Lab test: increased amylase and lipase (lipase is more sensetive), hypocalcaemia, glucose tolerance.
 Diet: Fat restricted diet & clear liquid diet (supply electrolyte and energy in a fluid that needs minimal
digestion.
 Causes of chronic: gall stones or heavy alcohol use, cystic fibrosis
 Treatment: IV fluid and pain control (and monitor heart, lung and kidney in ICU)
 HIV NRTI (Didanosine & Stavudine), (GLP‐1) Agonists, Dipeptidyl Peptidase‐4 DPP ‐ 4 Inhibitors,
Ezetimibe, Cholestyramine, Azathioprine, Colesevelam, Tigecycline, Pentamidine, Isoniazid, Valproic acid
 Pancreatitis is a side effect
97. Drug of choice for pancreatitis for the above patient
a) Piperacillin plus tazobactam
b) Patient do not need antibiotics
c) Amoxicillin
Supportive care with fluid, resuscitation, pain control and nutritional support
Prophylactic abx are NOT recommended regardless of type and severity

98. Patient with CINV on


dexamethasone plus aprepitant and
ondansetron. also given
prochlorperazine prn. Patient still
experiencing nausea, what to do?
a) Repeat prochloperazine
b) Give nabilone
c) Give metoclopramide

99. Patient brings prescription of elemental iron 100mg, you have in stock ferrous gluconate
300mg per 5ml and fumarate 150mg per 5ml. which would be the smallest volume of elemental
iron amongst them?
100 mg elemental iron/300 mg ferrous fumarate 33%  so, in 5ml there is 50mg elemental iron = 100mg/10ml
35 mg elemental iron/300 mg ferrous gluconate 12%  so, in 5ml there is 35mg elemental iron = 100mg/14ml
This means that small volume of ferrous fumarate contains much elemental iron

100. Patient has Gritty, red and watery eye. what to do


a) Give eye prep
b) Give antibiotics
c) Refer to doctor
Another version: Pt. with red eye what should you say when you gave him?
1) Cyclosporin or Loteprednol  See your doctor if not resolved in 3-5 days
2) Levocabastine  Shake well before use & Prime.

101. patient with cataract had surgery, doctor placed him on two inflammatory medication
(NSAIDs and Corticosteroid). what is the main purpose?
a) Stop inflammation
b) Stop lens from detaching
Note: After 30 days, you should dispense all drops again. Wait 3‐5 minutes between drops.
The goals of the perioperative prophylactic pharmacologic treatment are: Control inflammation, prevent
infection, maintain eye comfort and Promote early visual rehabilitation.
The goals of the postoperative assessment are: Detect intraocular infection in its early stages, detect
postoperative uveitis or intraocular pressure (IOP) elevation and detect other abnormalities in the
postoperative course

102. A study done on HIV test kit shows sensitivity of 0.68 and specificity of 0.97. Whats the
interpretation (can’t remember exact words used)
a) Positive results for HIV negative patient
b) Negative results for HIV postive patient
c) Positive for HIV positive patient
d) Negative results for HIV negative patient.

103. Side effect of bupropion?


a) Drowsiness
b) Agitation
c) Weight gain
d) Sundowning  ttt by trazodone.

104. Case about a patient with


Parkinson.
what medication to avoid?

105. Patient taking lithium, what


counselling point for the patient
a) Avoid caffeine
b) Avoid salt
c) Take plenty water
These diet guidelines to keep lithium blood level stable:
 Drink 8 to 10 glasses of water or other liquids every day.
 Keep your salt intake about the same. Keep your caffeine intake about the same.
 Avoid alcoholic beverages. Take lithium with food or milk.
 Toxic levels may result when adding NSAIDs, ACEIs, ARBs and especially thiazide diuretics.
106. A child using Concerta for ADHD. He spends the weekend at his grandmother who calls
you wondering why her grandson who is just fine needs an amphetamine, and refuses to give
him the medication until you give her an explanation. You should:
a) Tell her he needs the drug to be calm
b) Tell her that he has ADHD and this is the ttt
c) Tell her that she needs to talk to his parents
d) Tell her that if he misses the doses it will be a huge problem.

107. You are selling zopiclone. what do you counsel on?


a) Take 1-hr before sleep
b) Doesn’t cause drowsiness
Administer just before retiring at night, when there is sufficient time for a full night's sleep.
Advise patients of risk of next‐day impairment, even if feeling fully awake.
Allow at least 12 hours between bedtime dose and any activity requiring mental alertness, e.g., driving.
Duration of treatment varies from a few days to two weeks with a maximum, including the tapering off, of
four weeks.

108. Elderly pt. with insomnia, what to Give? Zopiclone


The lowest dose for the shortest duration is recommended. Treatment should rarely exceed 7–10 days. Re‐
evaluate the patient if treatment beyond 2–3 weeks is required

109. About influenza immunization


a) Children less than 2 should not be immunized
b) Will also works for pneumonia
c) Prevent Guillain-Barre Syndrome
d) Can be given to pregnant women

110. 3 yr old boy with persistent runny nose, congestion, fever for 2 weeks what will you do?
a) Give desloratadine
b) Give oxymetazoline
c) Give 1st gen. Antihistamine
d) Refer cos it’s more than 2 weeks
Common cold See a health‐care provider if any of these things happen:
 You have trouble breathing or You make strange sounds when you breathe
 Your throat is very sore
 You have a lung disease such as asthma, emphysema or chronic bronchitis
 You have a fever for more than 24 hours or Your cold or flu lasts for more than 7–10 days
Take a child to a health‐care provider if any of these things happen:
 They seem to have an earache
 They have a high fever (temperature above 39°C or 102°F)
 They seem very sleepy most of the time, seem very cranky or fussy most of the time
 They have rapid breathing or trouble breathing
 They have a cough that lasts for more than 10 days
 They have a skin rash
 They seem to be dehydrated (dry mouth, no urine output over 6 hours, crying without tears)

111. Pt. on sertraline for 4 weeks


and doctor noticed no
improvement in the case, what is
the appropriate action to do?
a) Shift to Venlafaxine.
b) Increase dose
c) Add CBT
d) Switch to quitiapine
Patients not responding to a 50 mg
dose may benefit from dose increases.
Dose changes should be made in steps
of 50 mg at intervals of at least one
week, up to a maximum of 200 mg/day.
Changes in dose should not be made more frequently than once per week given the 24‐hour elimination half
life of sertraline.
The onset of therapeutic effect may be seen within 7 days. However, longer periods are usually necessary to
demonstrate therapeutic response, especially in OCD

112. Patient is travelling to Malaria Zone in africa, what is the appropriate counselling,
a) Use Mefloquine (2 weeks before travel, during travel and 4 weeks after return)
b) Sleep in mosquito nets not effective
c) Wear in light color clothes and wear fitted clothes.
d) You can use DEET under 2 months of age as insect repellent.

113. Patient with thyroid dysfunction. When do we check levels of TSH? 


Initially check at 6‐8 weeks then annually (dose adjustment at 4‐6 weeks)
Throughout pregnancy, women on thyroid hormone replacement should check TSH Q4 wk in 1st trimester,
Q6 wk until ~32 wk gestation and 4–6 wk after any dosage adjustment.
During pregnancy, women typically require lower doses of antithyroid medication and often go into
remission. Monitoring is best done by measuring TSH, fT3 and fT4 every 6–8 weeks, and more often if a
change in clinical status or dose is made.
114. Case about a patient with uncontrolled glucose which wasn’t controlled with diet, he has
renal impairment, what to start?
a) Sitagliptin
b) Metformin
c) Gluburide
d) Empagliflozin  empa, cana for renal – empa, cana, lira for cv problems

115. First line in stroke patient who is in the hospital within 3hours of stroke (not
heamorrhagic)
a) Asa
b) Alteplase
c) Clopidogrel
116. Stroke secondary prevention
taking ASA but he can’t tolerate it,
what can be used?
a) Clopidogrel

117. Elderly Patient with HF and


using BB, what to add?
a) Sotalol
b) ACEI
c) ARBs
d) Digoxin

118. Question about VTE patient is old without uterus and on oral contraceptive, what is the
diagnostic test for DVT?
a) D dimer test
b) Factor V leiden (FVL)
VTE PPX 10‐14 days:
 Hip and knee replacement  DOACs (apixa or rivaroxaban are preferred for 3 months)
 Moderate Risk  Heparin
 High risk  Fondaparinux
 PROPHYLAXIS and TX LMWH
 LMWH and Heparins increase hyperkalemia when used with ACE

119. Case about a patient who smoke and is obese, had osteoarthritis. You counsel her about
smoking and she says she will like to stop but the smoking helps her cut down on food craving.
What stage is she?
a) Precontemplation
b) Contemplation
c) Preparation
d) Action
In another version  Patient is not ready to listen  so, it will be precontemplation

120. Women came with accidentally, 3 bowel movement, no vomiting, CBC count is 14K,
(severe if >15K), Cr Cl was ~70, C. Difficle = 8 (Moderate), what is the drug of choice?
a) Metronidazole 500 mg TID PO ˣ 10 days
b) Vancomycin 125 mg QID PO ˣ 10 days
c) Vancomycin 500 mg QID PO
d) Metronidazole 500 mg Q8H IV
121. How to know when the patient returns to normal or How to monitor that pt. is doing ok?
a) Start to eat and drink
b) Stop vomiting (patient wasn’t vomiting)
c) Stop therapy or pt is improving
d) Negative stool culture (most sensitive)
Diagnosis consists of detection of C. difficile toxin in the stool or presence of characteristic pseudo
membranous colitis on endoscopy. Stool culture is the most sensitive test but it is expensive and has a slow
turnaround time, so it is rarely used in clinical practice. Enzyme‐immunoassay testing for C. difficile toxins A
and B is rapid but less sensitive. Molecular testing with polymerase chain reaction (PCR) assays targeting
the toxin B gene has been developed and is highly sensitive; however, it is also expensive. There is limited
value in repeat testing during the same episode of diarrhea.

122. after a while, her husband came to you with a C. difficile infection, what do you
recommend for him?
a) Co-trimoxazole
b) Fidaxomycin
c) Vancomycin

123. Given lipid% amino acid 7% dextrose 20% at a rate of 62.5ml/hr and dextrose 4% at rate
42ml/hr, weight 65kg.
Calculate total protein/kg daily?
Calculate total carbohydrate daily (1g=3.4kcal)
Answer:
Protein is in amino acid,
rate 62.5 ml ------- 1 hr. X ml ------- 24 hr (day). X = 24*62.5/1 = 1500 ml
Amino acid conc. is 7 gm ------- 100 ml. Y gm ------- 1500 ml.
Y = 1500 * 7 / 100 = 105 gm 105gm / 65 kg = 1.6 gm/ Kg / Day
Carbohydrates are in Dextrose
62.5 ml ------- 1 hr 42 ml ------- 1 hr
X ml ----------- 24 hr X ml -------- 24 hr
= 1500ml = 1008 ml
20gm ------- 100 ml 4 gm ------- 100 ml
X gm --------1500ml X gm ------- 1008ml
= 300gm = 40.32gm
Total = 300+40.32 = 340.32g daily x (3.4 Kcal) = 1157 kcal/day
124. Asthma FEV less than 40. what type
a) Mild
b) Moderate
c) Severe

125. COPD risk factor


a) Alpha1-antitrypsin deficiency
b) Family history
c) Gender
d) Caucasian race
COPD Precipitating factors: cigarette smoking, heavy exposure to dusts and chemicals, air pollution, alpha1‐
antitrypsin deficiency, which has been associated with emphysema, liver disease, panniculitis

126. AOM CASE. Child is 6years old with penicillin allergy not attending daycare and no
antibiotics use in the the last one year. what to do
a) Give azithromycin
b) Acetaminophen and follow up
c) Send to doctor.
Another version: Acute otitis media child 3 year going to daycare. What is the first step? What
is the second step?  wait and watch for 24-48 hrs  treat with HD AMOXICILLIN (75 -
90mg/kg/day) if no improvement amoxiclav.

127. Doctor later placed the child on cefprozil. what to counsel?


a) Take with or without meal
b) It is photosensitive
c) Store in room temperature after reconstitution
Suspension should be refrigerated (2–8°C) once reconstituted with water and discarded after 14 days.

128. After taking cefprozil, the kid got hives, what should you do?
a) Send to doctor.
b) Refer to emergency
c) Give benadryl.
129. medication that can cause esophagitis
a) Doxycycline
b) SMX/TMP
 Antibiotics: tetracyclines, especially doxycycline. Other antibiotics that can cause esophagitis include
clindamycin, amoxicillin, metronidazole, ciprofloxacin, rifaximin, etc.
 (NSAIDs): aspirin and aceclofenac can lead to esophageal mucosal injury
 Bisphosphonates: alendronate, ibandronate. Risedronate seems to be slightly safer than alendronate
in terms of gastrointestinal side effects
 Ascorbic acid, Potassium chloride and ferrous sulfate
 Acetaminophen & Warfarin
 Chemotherapeutic regimens: dactinomycin, daunorubicin, bleomycin, methotrexate, 5‐fluorouracil,
cytarabine, and vincristine cause esophagitis perhaps due to oropharyngeal mucositis.
 Others: anti‐hypertensives, quinidine, glimepiride, tiropramide, pinaverium bromide, esomeprazole.

130. Patient is sexually active and developed ulcer. what is it?


a) Syphilis: single large painless ulcer
b) LGV: fussures, fistula then bacterial infection
c) Gonorrhea: cervicitis and urethritis
d) Trichomonas: vaginal symptoms (yellow discharge)
Syphilis m.o. is Treponema Pallidium. Test all sexual partners and their children and tx infected. Penicillin G
2.4 million units IM one dose Or Ceftriaxone and doxycycline.

131. which antidepressant is most appropriate for a 78yr old woman with history of falls?
a) Paroxetine
b) Amitriptyline
c) Duloxetine
d) Aripiprazole
Beers criteria: Describes the list potential inappropriate medications to elderly (drug to avoid in seniors or
reduce dose) developed by the American geriatric society (AGS).
Therapeutic Category / Drug(s) Recommendation Alternatives
Tertiary TCAs, alone or Avoid: Depression
in combination: Highly anticholinergic, sedating, SSRI, SNRI, Bupropion, Vilazodone,
 Amitriptyline and cause orthostatic hypotension; Trazodone, Mirtazapine
 Doxepin (> 6mg/day) the safety profile of low-dose Insomnia
 Imipramine doxepin (≤6 mg/day) is Trazodone, Ramelton
comparable to that of placebo. Migraine Prophylaxis
Antidepressants: Avoid in med with BPH. Propranolol, Topiramate, Divalproex.
 Paroxetine Caution advised due to Neuropathic Pain
anticholinergic effects, sedation Gabapentin, Duloxetine
and orthostatic hypotension For all indications a trial of
nortriptyline prior to use of
amitriptyline.
132. 65-year-old patient presents with 4 prescriptions. Total cost of Rx is $156. Senior’s
insurance deductible for the year is $100. For this transaction, what Patient pays?
Patient has deductible of $100. He paid 63.25 of it. Insurance co-payment is 10% after and they
cover up to $10 dispensing fees. If the med costs $87.50 including $15 dispensing fee how
much do you collect from patient? answer in some sources 49 other sources 45.33
Soln: First, Calculate the remaining of deductible
100 - 63.25= 36.75  Fees = 15 - 10 = 5 T he total price of Rx us 87.5 including both the
fees and the deductible  So, 87.5 -15-36.75 = 35.75 10% = 3.575 The total = 45.33

133. All of these should be avoided in Acute renal failure except?


a) NSAIDs
b) ACEI
c) Steroid

134. Patient with Epilepsy, levetricitam, lamotrigine and needs


contraceptive, what to give?
a) Plan B
b) Ulipristal
c) IUD
d) COC
IUD: Use within 7 days of unprotected intercourse as an emergency contraceptive.

135. Due to pharmacoeconomic studies, blood glucose testing is not necessary in type 2 DM pt, a
patient being managed on only dietary changes. In what case can you override this recommendation
a) Patient wants to motivate himself
b) Patient request (autonomy)

136. Patient worried about his family, children and job and depressed all the day and has no
interest or pleasure in all and depressed about his future and worries if he dies what children
will do, what the assessment tool of his condition?
a) The 7-item Hamilton Depression Rating Scale (HAMD-7)
b) The Patient Health Questionnaire (PHQ-9)
c) The Mood Disorder Questionnaire (MDQ)
The Patient Health Questionnaire (PHQ‐9) is consisting of 9 questions that correspond to the DSM‐5 criteria
for a major depressive episode. The 7‐item Hamilton Depression Rating Scale (HAMD‐7) is a validated, brief,
health‐care professional assessment designed to rate severity and remission.
Mood Disorder Questionnaire (MDQ) useful screening instrument for manic or hypomanic symptoms.
The Edinburgh Postnatal Depression Scale (EPDS) is the most widely used and well‐validated tool to screen
for depressive symptoms during pregnancy and postpartum period.
137. Heart failure patient. Feel tired when he walks or climb some stairs. what NYHA stage?
a) Class I  No symptoms with ordinary activity
b) Class II  Symptoms occur with ordinary activity
c) Class III  Symptoms occur with less than ordinary activity
d) Class IV  Symptoms occur at rest or with minimal activity.

138. which of the following patient will their HbA1C be consistent?


a) Pregnant patient
b) 66yr old man with end stage renal failure ESRD
c) A man receiving therapy for prostate cancer
d) Black american with sickle cell anaemia

139. What not to put on Label in hospital extemporaneous prep?


a) Pharmacy identification (name, address and telephone number of the compounder’s).
b) Drug identification (active ingredients, source, concentration, form, route of
administration, volume, solute, amount prepared).
c) Special precautions for disposal or destruction of the preparation;
d) Beyond-Use Dates BUD

140. Patient with colon surgery in severe pain


What do you recommend
a) SC hydromorphone
b) Other options were all oral medication
Another version: In case of acute pain which
dosage form of Morphine is used?
a) IV  if severe
b) IM
c) PO
d) SR
Oral morphine is a better alternative for acute pain;
an equipotent dose will be at least as effective as
codeine, with a more predictable adverse effect
profile. Morphine is available in several immediate‐
release dosage forms and sustained‐release
preparations but the longer‐acting formulations
should rarely be used to treat acute pain. Because
morphine's adverse effects are predictable, the dose
can be titrated to achieve pain‐free status.
141. Question about fentanyl disposal in a safe and effective way
a) Fold the sticky sides together and flushing them down the toilet
Ask patients to return used patches to the pharmacy when presenting for the next dispensing

142. UTI uncomplicated symptoms (internal dysuria,


hematuria, frequency, suprapubic discomfort and
urgency), what is the most causative organism?
a) E. Coli
b) S. Aureus

143. Diagnostic test that separates pyelonephritis from


UTI? Blood in the urine  in UTIs

144. Pyelonephritis ttt duration?


Mild: Fluoroquinolone (ciprofloxacin, levofloxacin, norfloxacin), PO × 7–14 days
severe: Aminoglycoside IV ± ampicillin IV for initial therapy; if appropriate, step down to oral therapy as in
mild to moderate infections in order to complete 10–14 days
IV to oral‐in 48 hrs for ttt of pyelonephritis

145. Definition of a biosimilar drug


 A biosimilar is a new, highly similar version of a biologic drug that comes to the Canadian market by
a different manufacturer after the patent for original product has expired.
 Biosimilars were previously called subsequent entry biologics (SEBs).
 Biosimilar are not identical but comparable to its reference biologic drug RBD

146. Patient taking warfarin has INR was 5.2 without risk of bleeding, what should we do?
a) Give oral vit. K
b) Omit dose and repeat INR before 2nd dose
c) Dec dose by 50%
d) Dec dose and follow up
147. Case about a patient taking plenty medication. Which medication can cause osteoporosis?
a) Corticosteroid
Medications associated with increased risk of fractures:
Aromatase inhibitors, anticoagulants (unfractionated and low molecular weight heparins), antiretroviral
therapy, cyclosporine, corticosteroid therapy (at least 3 months' cumulative therapy in the previous year at
a prednisone‐equivalent dose ≥7.5 mg daily), loop diuretics, proton pump inhibitors (PPIs), SSRIs,
thiazolidinediones, chemotherapy and high doses of vitamin A.

148. A patient using Brilinta (Ticagrelor) and hospital ran out of stock, what will you do?
a) Change to prasugrel
b) Change to clopidogrel
c) Change to ASA + Dipyridamole

149. Patient with recent NSTEMI and hypertension, hyperlipidemia, he has drug eluting stent,
how long should be taking clopidogrel + ASA from prophylaxis from heart problems
a) One year
b) One month
c) 3 months
d) 6 months

150. Patient has RA using biological modifiers Etanercept, what to monitor?


a) Recurrent infections (specially TB)
Biologic DMARDs for Rheumatoid Arthritis
TNF-alpha A recombinant monoclonal antibody that binds to TNF-α, thereby interfering with
Inhibitors TNFI endogenous TNF-α activity by blocking its interaction with cell surface receptors.
Adalimumab Improve symptoms in patients with moderate to severe RA who are unresponsive to
Certolizumab csDMARD therapy alone.
Etanercept (acts Given in combination with MTX in newly diagnosed patients.
against both TNF May be used as monotherapy in case of MTX contraindication
and ) Adalimumab: 40 mg Q2 wk SC.
Golimumab Certolizumab: 400 mg at wk 0, 2 and 4, then 200 mg Q2 wk SC. May give 400 mg Q4 wk
SC as maintenance dose
Etanercept: 25 mg twice weekly or 50 mg once weekly SC
Golimumab: 50 mg once monthly on same date each month SC. Effective in patients who
have failed therapy with other TNFi.
S.E: Injection-site reactions; infections (including TB); newonset psoriasis; increased risk
of lymphoma (children and adolescents), leukemia autoimmune phenomena.
Monitoring: Baseline CBC, LFTs, creatinine, hepatitis B and C serology, PPD and chest
x-ray to assess for latent TB. May consider screening for ANA.
Susceptibility to or presence of serious and/or recurrent infection; SLE, demyelinating
disease and heart failure are relative contraindications.
151. Coadministration of co-trimoxazole with sulfonylureas glyburide can result in?
b) Increase the risk of hypoglycemia.

152. Pt has acute gout and his renal function Cr Cl less than 25 and he is on allopurinol what do
you give to control pain
a) Colchicine – renal /hepatic
b) Naproxen – renal
c) Prednisone 10 mg
d) Indomethacin
Avoid colchicine in crcl < 30 ml/min: avoid acute & prophylaxis therapy

153. Patient got a rash from allopurinol what do you do:


a) Desensitize rash by oxypurinol if mild
b) Monitor rash
c) Stop medication
d) Give Febuxostat
Allopurinol should be discontinued at first appearance of skin rash or any sign of serious adverse reactions.
Skin rash may sometimes be followed by more severe hypersensitivity reactions such as exfoliative,
urticarial or purpuric lesions, as well as Stevens‐Johnson syndrome (a sometimes‐fatal form of erythema
multiforme) and, very rarely, generalized vasculitis, which may lead to irreversible hepatotoxicity and death.
Hypersensitivity reactions, frequently marked by fever, chills, lymphadenopathy, arthralgia and/or
eosinophilia, usually begin 2–4 weeks after start of therapy. In patients with cutaneous reactions
to allopurinol for whom there is no therapeutic alternative, consider desensitizing the patient to allopurinol.
After recovery from mild reactions, the patient may, if desired, be desensitized by reintroducing at a low
dose and gradually increasing. For example, an allopurinol suspension of 1 mg/5mL (200 μg/mL) can be
compounded and started at a dose of 50 μg and increased up to 100 mg over 28 days. If the rash
recurs, allopurinol should be discontinued permanently.

154. Allopurinol counselling:


 Take after meals to decrease GI upset.
 Take plenty of water (can cause urolithiasis).
Allopurinol increases the pharmacologic and toxic effects of thiopurines by inhibiting their metabolism
Reduce initial thiopurine dose to 25% or 33% of the recommended initial dose when given with allopurinol.

155. What gene causes SJS of Carbamazepine?


a) HLA-B*1502
b) HLA-B*5701  abacavir
Susceptibility may also increase the risk of developing SJS/TEN and other drug hypersensitivity reactions.
The strongest associations have been described for HLA‐B*5701 with the abacavir hypersensitivity
syndrome, HLA‐B*5801 with allopurinol‐induced severe cutaneous adverse reactions and HLA‐B*1502
with carbamazepine‐induced SJS/TEN. Health Canada recommend genetic screening for patients of Asian
ancestry before initiation of carbamazepine therapy, and avoidance of carbamazepine in patients who test
positive. In addition, advise the patient to enrol in the MedicAlert program.

156. HTN & Diabetes target?


a) < 130/80
b) < 140/90

157. Risedronate DR counselling?


a) Take with food
b) Take 30 minutes before food
Not with empty stomach like others bisphosphonate
and it can be administered at night as well

158. Chronic use of aminoglycoside in CKD. How could you adjust the dose?
a) keep dose and increase interval.
b) Decrease dose and interval
Concentration dependent antibiotics Time dependent antibiotics
Those antibiotics which eradicate pathogenic Those classes of antibiotics whose killing response is
bacteria by achieving high concentration at the dependent on time are termed as time dependent
site of binding are termed as concentration antibiotics.
dependent antibiotics. In renal disease decrease dose
In renal disease increase intervals of drug. Higher concentration of such drugs does not result in
These antibiotics show optimum response in greater killing of organism. “The inhibitory effect can be
killing bacteria(bactericidal) when their effective because their concentration exceeds the MIC for
concentration is either equal or greater than 10 the microorganism. Hence, these antibiotics are referred to
times above the MIC (minimum inhibitory as time-dependent antibiotics.
concentration) at the site of infection for certain For time-dependent drugs, the pharmacodynamic
target micro-organism. parameter can be simplified to the time that serum
The pharmacodynamic parameter of such concentrations remain above the MIC during the dosing
antibiotics can be simplified as a peak/MIC interval (t > MIC)”. They show optimum killing response
ratio. when the time that the drug remains above the MIC is
Examples: either equal or greater than 50% of the dosing interval.
Aminoglycosides, flouroquinolones & Examples: Different antibiotics whose response is time
Azithromycin are the classes of antibiotics dependent are penicillins, cephalosporins, carbapenems,
which show concentration dependent killing of monobactams), clindamycin, macrolides (erythromycin,
organism. clarithromycin), oxazolidinones (linezolid).

159. According to ISMP Tallman lettering, what is wrong:


a) CEFuroxime & cefIXIME
b) predniSONE & prednisoLONE
c) metFORMIN & metroNIDAZOLE
160. Dextromethorphan Toxicity: Feeling of flying, excitability, paranoia, hallucination and euphoria,
altered perception, visual disturbances, delusion, psychosis, mania, lethargy, ataxia, slurred speech,
sweating, hypertension, nystagmus, disorientation.

161. Rheumatoid Arthritis monitoring:


TC: Patients should be seen as frequently as every 1–3 months while their disease is active. Those with well‐
controlled disease or in remission can be followed at longer intervals (every 6–12 months). Adjust therapy
frequently until a patient reaches a target (remission or at least low disease activity), ideally by 3–6 months.
Consider major changes to therapy, such as adding or switching agents, if there is ongoing disease activity
after 3 months of maximal therapy. Minor adjustments, including the judicious use of oral, IM or
intraarticular corticosteroids may occur more frequently.

162. Finasteride effect in BPH appears in?


a) 6 months
b) 4 – 6 weeks  this is for alpha blockers

163. Tobramycin accidental exposure: vestibular (Ototoxic) & nephrotoxic

164. What do you monitor for olanzapine?


NMS – muscle rigidity, fever, high CK, high WBC. Risk factor  dehydration.
Treatment  D/C drug then Dantrolene +/‐ Bromocriptine
EPS: dystonia, parkinsonism, akathisia, tardive dyskinesia, tardive dystonia, tardive akathisia
Hyperlipidemia & Dysglycemia.

165. Whats non-pharm can help scabies in school going? Avoid skin to skin contact

166. what is the treatment of choice? Permethrin 5%. All close contacts should be treated.

167. Tramadol: schedule 1, regular prescription drug

168. Treatment of Endocarditis, MRSA; prosthetic valve:


a) vancomycin+ rifampin+ gentamicin for 2 wk total tx 6 wk
169. Treatment of Rosacea?
Metronidazole, it takes 3 months for
full effect.

170. If employee take too much of


breaks what to Do?
Discuss with him privately

171. How long should patient


usessuboxone 8 to 24 mg /day?
a) For minimum 12 months and
may consider life time.
SUBOXONE sublingual tablets should be
placed under the tongue until dissolved.
Altering the tablet to take it by routes other than the indicated sublingual route can lead to serious adverse
events including death. Do not cut, break, crush or chew SUBOXONE
Note: Throughout this guidance, the dosage refers to the buprenorphine content of the tablets. The ratio of
buprenorphine to naloxone in Suboxone® is 4:1.
Naloxone is present in Suboxone®to deter the abuse of buprenorphine by injection
The recommended starting dose is 8 mg SUBOXONE on Day 1, initiating with 4 mg and then an additional 4
mg dose may be administered depending on individual patient’s requirement. The suggested total dose
target for treatment on Day 1 is within the range of 8 and 12 mg

172. Pt. was on methadone, admitted to hospital and stopped methadone, when will the patient
experience maximum withdrawal symptoms of methadone?
a) 1 day
b) 2 days
c) 3 days
d) 5 days
The methadone withdrawal symptomstimeline goes as follows:
 Day 1: Withdrawal symptoms may start within 24 hours after the last dose of methadone depending on the
level of dependence. The most common withdrawal symptoms in this period include flu-like symptoms,
such as sweating, fevers, muscle aches, and chills.
 Days 2-10: The first few days (3-4 days most peak of symptoms) will likely be the most arduous portion of
the withdrawal process. At this point, the person must secure a proper support system, such as professionals
at a detox or treatment program to overcome the severe side of the symptoms. Common symptoms at this
time can include strong cravings, insomnia, paranoia, hallucinations, irritability, and anxiety.
 Days 11-21: For the majority, the worst of the symptoms will have dissipated after day 10. Some symptoms
may still be present, such as mood swings, cravings, depression, agitation, and feelings of fatigue. A support
system will help get you through the symptoms without wanting to relapse.
 Days 22 and beyond: For those on a significant dose of methadone (40mg or above), symptoms can persist
for weeks or months. The most common nagging symptoms can include cravings, fatigue, and depression.

173. Hand hygiene: which is not a part of hand hygiene?


a) Wash your hand with soap and antimicrobial and include jewelry.
https://www.publichealthontario.ca/en/eRepository/2010-12%20BP%20Hand%20Hygiene.pdf
Effective hand hygiene kills or removes transient bacteria on the skin and maintains good hand health.
There are two methods of killing/removing microorganisms on hands:
1. Hand sanitizing with a 70 to 90% alcohol‐based hand rub (ABHR) is the preferred method (when hands are
not visibly soiled) for cleaning hands.
Using easily‐accessible ABHR in health care settings take less time than traditional hand washing and has
been shown to be more effective than washing with soap (even using an antimicrobial soap) and water
when hands are not visibly soiled.
2. Hand washing with soap and running water must be performed when hands are visibly soiled.
The effectiveness of alcohol is inhibited by the presence of organic material. The mechanical action of
washing, rinsing and drying is the most important contributor to the removal of transient bacteria that
might be present.
If hands are visibly soiled and running water is not available, use a moistened towelette to remove the
visible soil, followed by ABHR.
ALCOHOL-BASED HAND RUB vs. SOAP AND WATER
Alcohol‐based hand rub (ABHR):
 ∙ preferred when hands are not visibly soiled
 ∙ should contain 70 – 90% alcohol
 ∙ takes less time than hand washing
 ∙ more effective than hand washing with soap and water when hands are not visiblysoiled
 ∙ mechanical rubbing action is important to kill transient bacteria
 ∙ less drying to hands than soap and water
Hand washing with soap and running water:
 ∙ preferred when hands are visibly soiled because alcohol is inhibited by organic matter
 ∙ mechanical action of washing, rinsing and drying removes most transient bacteria

175. what makes drugs a Therapeutic equivalent:


a) Drugs that treat the same disease but of different
classes (levo and amoxiclav)
Therapeutic equivalent: drugs of different active ingredients (same
class e.g. antihistamines), expected to have same therapeutic or
clinical response. Omeprazole and Esomeprazole

174. Overbid-use one’s authority to reject or cancelled, so it


can be can be?
a) Conscious objection
b) Paternalism.
Conscientious objection in health care is the refusal to perform a legal role or responsibility because of
moral or other personal beliefs. Most states have “conscience clauses” that describe the right of physicians
and other health care providers to refuse to provide services such as abortions.

176. Special Access Program “SAP”- What`s true:


a) Manufacturer makes the final decision
b) SAP authority certifies efficacy
c) The medication is provided free of charge
d) The medication is provided for only 3 months  6 months
e) The medications usually investigational drugs  drugs not approved for sale in canada

177. Treatment of Osteoporosis in pregnancy


a) Ca and Vit. D
b) Alendronate
c) Alendronate + vit D

178. Amiodarone monitoring:


Lung KL6, chest X-ray, AST, ALT, fundoscopy, TSH, Free T4, eye (Corneal deposition),
grey man syndrome.

179. Latanoprost S.E. iris pigmentation, Corneal deposition, Visual disturbance.

180. You have subs. A conc. is 4% to be added to subs B to have a final volume of 50 ml. After
mixing both subs. The conc. of subs. A in the final solution was 0.1 %, calculate the amount of
subs. B in Mg in each dose if you divided the total solution to 5 small doses. Conc. of subs B is
10% w/v.
Answer: C1*V1 = C2*V2. 4%*V1 = 0.1%*50.
V1 = 0.1%*50/4% = 1.25 ml which is the final volume of subs. A.
Volume of subs. B = 50 -1.25 = 48.75 ml

181. Least compliant contraceptive? Progesterone only pills (Minipill)


PROGESTIN‐ONLY CONTRACEPTIVE Norethindrone (Micronor ®)
Less effective for preventing pregnancy than COC. Failure rate 0.3 % vs 0.1% with COC if perfectly used. However, they
have non non‐contraceptive benefits such as:
 Decreased dysmenorrhea, bleeding, risk of endometrial cancer, increased protection against PID
 Must be taken on a tight regular schedule and thus are often not used (a missed dose is defined as
missed dose > 3 hours for oral pill)
 May be beneficial in patients with migraines, early postpartum (if not solely breastfeeding), cardiac
disease (CAD, MI), age >35 yrs & smoker (or possibly obese), history of DVT/PE, hypertriglyceridemia,
HTN with CV disease or age >35 years, lactating women where estrogen is contraindicated, hepatic
disease, diabetes with microvascular complications
 Given PO. Consistent use is required to ensure efficacy
 If a dose is missed by >3 hours, back back‐up contraception or 2 days is required Depo
Depo‐Medroxyprogesterone Acetate (DMPA) (Provera®
 Injection to be given within 5 days of onset of period every 3 months (10‐13 weeks)
 Interval between injection should not exceed 13 weeks
 Issue: delayed onset of fertility/menses after stopping the drug (up to 1 year)
 Black box warning: ↑risk of osteoporosis (or ↓BMD) thus ensures adequate calcium & vit D intake.
 Avoid using >2 years due to ↓BMD (use for shortest me and re re‐evaluate if COC may be used)
Levonorgestrel intrauterine system (Mirena®):
 Works for at least 5 years & Highly effective with low pregnancy rate
 Reduce menstrual blood loss (up to 1/3 of women do not have periods and bleeding decreased by up
90%), decrease fibroid growth, dysmenorrhea, endometriosis related pain
 Decreases risk of cancer of uterus. Normal cycle returns after after1‐3 months of removing device

182. COC missed dose


 1 pill in the first week
 > 2 OC-pills missed at any time
 7 or more hormone free days
 13 weeks or more between DPMA injections
 3 hours late for POC use back up for week

183. Can we use LABA in children less than 6 yrs


for asthma? No

184. Saquinavir SE:


Increased liver enzymes, GI upset, headache, QTc and PR
interval prolongation, hyperlipidemia.

185. Period of treatment of manic episode in bipolar disorders? 6 months


CTC: For manic episodes, the optimal duration of the second‐generation antipsychotic in a 2‐drug
combination has not been determined, although a randomized clinical trial suggests that treatment for 6
months yields the most favorable risk‐benefit ratio; decrease in relapse rate was similar to that achieved
with treatment for 12 months, and significant weight gain due to metabolic side effects did not occur within
6 months
186. Clinical trial report show that in the control group 338 patients out of 9332 patients died &
in the drug group 320 out of 9650 patients died, find RRR?
Answer:
Control = 338 / 9332 *100% = 3.62 %.
Drug = 320 / 9650 *100 % = 3.31 %
RRR = (3.62-3.31) / 3.62 * 100 % = 8.5 %

187. INR is increased by all of the following, except?


a) Azoles
b) Cimetidine
c) Clarithromycin
d) Phenytoin

188. Non pharm of Reynaud’s phenomenon


 Minimize cold exposure and avoid prescribing medications with vasoconstrictive potentials.
 Reassure patients that there are no complications and teach them warming exercises that may help
terminate an attack.
 Instruct patients to dress warmly and Encourage them to stop smoking & to avoid using vibrating tools.
 Acupuncture was ineffective in patients with secondary Raynaud phenomenon.

189. Reynaud’s disease ttt? Nifedipine XL

190. Acetaminophen Overdose: More than 4g.

191. Baby after 13 months with protien milk allergy & has enteropathy, what to give:
a) Iron fortified
b) Pasteurized whole cow milk
c) Cow milk
d) Soy milk
Soy milk NOT recommended for infants with cow's milk protein–induced enteropathy or enterocolitis, as 30–
60% will also be sensitive to soy. However, infants who have an immunoglobulin E–associated reaction to
cow's milk protein may tolerate soy formulas.

192. Montreal Cognitive Assessment (MoCA) or Mini-Mental State Examination (MMSE) are
used for
a) Alzehimer
b) Parkinson disease
c) OCD
193. What is dangerous when IV? Sterile water for injection
Sterile Water for Injection USP is a clear, colorless, odorless liquid. It is sterile, hypotonic, nonpyrogenic, and
contains no bacteriostatic or antimicrobial agents
Accidental Eliminate all 1,000 mL bags of sterile water (labeled for “injection,” “irrigation,” or
administration of an “inhalation”) from all areas outside of the pharmacy.
intravenous infusion Use alternatives to avoid the storage and use of 1,000 mL (1 liter) bags of sterile water
of sterile water for injection, irrigation, or inhalation in patient care areas.
Establish policy that 1,000 mL bags of sterile water can only be ordered by pharmacy.

194. Case about old man who refuse to take Pneumococcal vaccine & all family members want.
a) We should give all and leave the man to his autonomy.

195. Not a symptom of Alzehimer


a) Aphasia
b) Fatigue
c) Amnesia
d) Apraxia

196. Why we are bridiging Warfarin and LMWH: To maintain the anticoagulant effect.

197. Patient recently on levodopa what to monitor


a) Hypertension
b) TSH
c) Sr. Creatinine
 Frequent monitoring of BUN, creatinine levels, and hepatic function is necessary
 Good hepatic function is crucial in patients taking levodopa because it is where the drug is decarboxylated.
 Also, it is important to test for intraocular pressure in patients with glaucoma.
 Testing for peripheral neuropathy before and while on levodopa is also very important.
 Patients should also be regularly monitored for dyskinesia.

198. Parkinson patient taking levodopa


and carbidopa IR in the morning and SR
at night, he developed high muscular
rigidity & dyskinesia, how to avoid?
a) Shift from IR to CR
b) Add entacapone
c) Add Amantadine
d) Decrease 25% of dose
e) Shift both to SR
If both wearing off & dyskinesia, go with amantadine. If wearing off only B & E are right
199. Hepatitis B: Avoid Unprotected Sex.

200. Drugs in Freezer:


a) MMR
b) Polio
c) Botox
All varicella‐containing vaccines (VAR, Varivax; ZOS, Zostavax; and MMRV, ProQuad) should be stored
between between ‐58°F and +5°F (‐50°C and ‐15°C).
The measles, mumps, rubella vaccine (MMR) can be stored either in the freezer or the refrigerator. Storing
MMR in the freezer with MMRV may help prevent inadvertent storage of MMRV in the refrigerator.
No Live vaccine administration for less than 6 month or Immunocompromised
Use as soon as possible after reconstitution. Store reconstituted vaccine in the vaccine vial in a dark place at
2 to 8°C. Discard if not used within 8 hours. Before reconstitution, M‐M‐R II should be stored refrigerated at
a temperature of 2 to 8°C. The vaccine may also be stored in a freezer at temperatures above −50°C; if
subsequently transferred to a refrigerator, the vaccine may be placed back in the freezer.

201. Why we use BB after MI? to avoid Re-infarction.


After MI, patients are at substantial risk for subsequent major adverse cardiovascular events (MACE), such
as nonfatal MI, death or stroke. Secondary prevention with several classes of drugs independently reduces
incidence of MACE by approximately 25%
Cumulative relative risk reduction if all 4 drugs are used =75%: DAPT‐ACE‐BB‐Statin‐smoking cessation
BB in heart failure: decrease mortality. BB in stable angina: symptom relief

202. Raloxifene? agoinst on Bone and lipid and antagonist on breast and uterus.
Raloxifene prevents postmenopausal bone loss, increasing bone density by approximately 3% and reducing
new vertebral fractures by 30–40%.
Raloxifene is a selective estrogen receptor modulator (SERM); it acts as an estrogen antagonist in breast and
uterine tissue, but has estrogen‐like activity in bone and lipid metabolism. Like estrogen, it causes a
modestly increased risk of deep vein thrombosis and pulmonary embolism in postmenopausal women. It
also significantly reduces the relative risk of estrogen receptor–positive breast cancer by 76%. Raloxifene is
not associated with increased cardiovascular risk.

203. Bleomycin SE: Pulmonary toxicity. (pneumonitis, rarely pulmonary fibrosis)

204. Doxorubicin/Danurobocin SE: Cardiac toxicity. Ttt by Dexrazoxane

205. Treatment of anemia associated with cancer chemotherapy? Epoitin & Darbepoeitin
Epoetin alfa is a recombinant human erythropoietin with a relatively short half‐life that is typically given at
least 3 times per week. It may also be administered daily in the preoperative surgical setting where a more
rapid rise is desired. Darbepoetin alfa is a synthetic erythropoietin analogue with a longer half‐life that is
typically given weekly or biweekly, and monthly in some patients
206. Prevention of Neutropenia associated with cancer chemotherapy? Filgrastim.

207. Treatment of Xerostomia associated with cancer chemotherapy


Give Pilocarpine (Off label by the way)
Artificial saliva products can help to replace moisture and lubricate the mouth
May require up to 12 wk of treatment to show dry mouth symptom improvement.

208. Cyclophosphamide SE?


Alopecia & Hemorrhagic cystitis
(Use MESNA Prevent of high-dose cyclophosphamide-induced hemorrhagic cystitis.)

209. Capecitabine Black box?


Hand & foot syndrome (60%), Acute renal failure, Cardiotoxicity.

210. Baby 18 month with Constipation for 2 days?


a) Glycerin supp.
b) Refer
c) PEG
Infants Rectal disimpaction of infants can be achieved with pediatric glycerin suppositories.
< 1 Year Barley malt extract, corn syrup or sorbitol can be used as stool softeners
Use of enemas, Heavy mineral oil and stimulant laxatives are not recommended.
Children PEG (superior), lactulose or sorbitol are considered first-line agents for the treatment of
≥1 Year constipation whereas magnesium hydroxide is second line.
Heavy mineral oil is considered third line to avoid risk of lipoid pneumonia.
Senna and bisacodyl can be used as rescue medication when other agents have failed.
Disimpaction with enemas is recommended after diagnosis of impaction via rectal examination.
After disimpaction, maintenance therapy should be initiated for 1–2 months once the child has
>3 stools/week.
Red Flag  GI bleeding, Palpable abdominal mass.
Symptoms  Unexplained iron-deficiency anemia, Unintentional weight loss.
 Family history of colon cancer, Symptom onset ≥ 50 years of age.
 Sudden/acute onset of change in bowel habits
When to See  Children < 2 years old constipation >2 wks (or no bowel movement for 7 days) despite
Your Health- of use of laxative.
Care  You have not had a bowel movement for 7 days
Provider If  You are extremely uncomfortable because you are constipated
you are  You have pain in your rectum (back passage) or rectal bleeding
constipated  You have a fever, your belly hurts or you feel like throwing up
and:  Your bowel movements are thin as a pencil
 You have a problem with your bowels that lasts longer than 2 weeks or keeps coming
back
211. Pt due to post neuralgia from herpes simplex he has mouth ulcer
Benzydamine and rinse with water, Topical Lidocaine.

212. DOC for tonic clonic seizure?


Carbamazepine Lamotrigine Levetiracetam Oxcarbazepine Valproic Acid/Divalproex

213. Pt. with UTI uncomplicated, what is the organism? K. pneumonia.


E Coli (rod ‐ve) in both hospital or community (80% of outpatient)
Klebsiella pneumonia (rod ‐ve), Enterobacter, Proteus

214. UTI – Pregnant?


Amoxicillin (if the organism is known to be susceptible), amoxicillin/ clavulanate, cephalexin, fosfomycin
tromethamine or nitrofurantoin with appropriate follow‐up.

215. Amlodipine SE? Hypotension, flushing, peripheral edema.

216. ISMP abbreviations?


QOD. Every other day. Mistaken as “q.d.” (daily) or “q.i.d. (four times daily) if the “o” is poorly written

217. Allergic Rhinitis? No fever.


Symptoms (congestion, nasal itch, rhinorrhea, sneezing, postnasal drip, ocular), frequency and severity

218. BMI calculation = wt in kg / H in M * H in M

219. Primary literature? Cancer book.

220. Antidote for BDZ? Flumazenil

221. Opioid IR/CR conversion calculation


CR opioids can easily be converted to IR by crushing or biting the tablet.
http://nationalpaincentre.mcmaster.ca/opioid/cgop_b02_r08.html

222. Atorvastatin and Grapefruit interaction.


a) Reduce atorvastatin dose
b) Increase atrovastatin dose
c) No interaction

223. Roflumilast drug interaction?


 Fluvoxamine (Luvox, Solvay), an inhibitor of both CYP1A2 and CYP2C19, serum roflumilast increased.
 Strong CYP450 inducers (e.g., carbamazepine, phenobarbital, phenytoin, rifampin) may reduce
therapeutic effect of roflumilast and are not recommended in combination.
224. Q Fever m.o? Coxielia burnetti  ttt Doxycycline, FQ & Macrolide.
Q fever, also called query fever, is a bacterial infection caused by the bacteria Coxiella burnetii. The bacteria
are most commonly found in cattle, sheep, and goats around the world. Humans typically get Q fever when
they breathe in dust that was contaminated by infected animals
Q fever is treated with the antibiotic doxycycline. How long you take the medicine depends on whether or
not you have acute or chronic Q fever. For acute infections, antibiotic treatment lasts two to three weeks.
People who have chronic Q fever usually must take a combination of antibiotics for at least 18 months.

225. Ziprasidone causes? sudden cardiac death.


Epidemiological studies provide evidence that antipsychotics increase the risk of sudden cardiac death. ...
Among the antipsychotic drugs, ziprasidone is associated with the greatest QT prolongation.
Increased Mortality in Elderly Patients with Dementia: BLACK BOX

226. Doxepin SE?


Doxepin Because of its potent antihistaminic effects, it has been evaluated in very low dosages (3 mg and 6
mg vs. 150–300 mg [dosing for depression]) and is approved by Health Canada for sleep-
maintenance insomnia.
S.E: The most common side-effects to Doxepin 50mg Capsules are drowsiness, dry mouth and
constipation. Somnolence, sedation and nausea.
May cause dose dependent, next-day impairment of activities requiring alertness, including driving
a car, despite the patient feeling fully awake.
Metabolized by CYP2D6 and CYP2C19; inhibitors of these enzymes may increase exposure
to/toxicity of doxepin. Potential additive effects with other CNS depressants and/or alcohol.
Cimetidine and sertraline can significantly increase serum levels and/or toxicity of doxepin.
Should not be taken within 3 h of a meal to minimize drowsiness the next day.

227. How long can Capsaicin shows effect?


3-4 weeks to see effect, give trial for 8 weeks before contacting your doctor, wear a glove if
you feel irritation on the hand, don’t apply on broken skin or near mucous membrane.
This is usually a transient effect most prominent in the 1st wk of treatment; it diminishes or disappears with
continued use at the recommended dose. If applied less frequently than recommended or used
intermittently, the burning effect may persist
Full therapeutic response may not be apparent for 1–2 wk in arthritic disorders or 2–4 wk in neuralgias
Consider wearing gloves to apply capsaicin. Rub in well until no residue remains on the skin. If not wearing
gloves, wash hands with soap and warm water immediately after application. However, if capsaicin is used
on hands, refrain from washing hands for a minimum of 30 minutes. Heating pads should not be used and
hot baths or showers should be avoided immediately prior to or following application since warm water or
excessive sweating may intensify the localized burning sensation. Do not wrap or cover area with a
bandage. capsaicin is not a rubefacient. Although it is classified as a counterirritant, the therapeutic effect
of capsaicin involves both counterirritation and desensitization of pain receptors.
228. 1st line treatment for Stress urinary incontinence?
a) Pelvic floor muscle training (kegel exercise)
b) Conjugated estrogens, vaginal cream
c) Duloxetine
d) Oxybutynin
http://www.canadiancontinence.ca/pdfs/pelvicmuscleexercises.pdf

229. Prednisone Calculation: Dose to be given as an infusion 900mg, 50mg/hr initially then
increase by 50 mg/hr every 30 min until maximum 400mg/hr. how many hours needed?
Answer:
50 ml /hr means 25 ml/ 30 min  0....25 ....50....75....100...125....150....175...200
Total = 900  8*30 min = 4 hr if need 400 only

230. In order to weigh a compound on a balance with 5% accuracy & has sensitivity error of 6
mg. What is the minimum weighable amount?
a) 30 mg
b) 12 mg
c) 6.3 mg
d) 120 mg
We can calculate the smallest quantity that can be weighed, on a balance of known sensitivity,
to maintain a desired level of accuracy. This weight is referred to as the least weighable
quantity (L.W.Q.). = (6 / 5 * 100) * 100% = 120 mg

231. 6 years old child the Dr. ordered a loading dose of Phenytoin 20mg/Kg infused with a rate
of 0.5 mg/Kg/min. If you've it in 100 ml bag, what is the rate of pump that you must be adjusted
in ml/hr.?
Answer:
0.5 mg ------- 1 kg ------- 1 min 20 mg ------- 1 kg ------- X X = 1*20/0.5 = 40 min.
100 ml ------- 40 min Y ml ------- 60 min Y = 60*100/40 = 150 ml
So, rate = 150 ml/ hr

232. Dr. prescribed for a 7 years old boy 16 ml/hr. from the following formula
Na 40 m. mole/ L K 40 m. mole / L
then he wanted to give him 14ml/hr. of the formula. How many m. moles of K the boy will get
daily?
Answer:
Rate = 14ml / hr =14ml*24hr = 336 ml / day
40 m.mol ------- 1000 ml X m.mol ------- 336 ml
X =336*40/1000 = 13.44 m. mole / Day
233. Nonsedative antipsycotic
a) Aripiprazole
b) Ziprasidone
c) Lurasidone
d) Clozapine
Aripiprazole, brexpiprazole, Asenapine & palperidone are the
less sedative antipsychotics

234. Isotretinoin monitoring ? LFT and TG


Another version:
Pregnant lady wants to use Isotritinoin  have 2
negative pregnancy test and use barrier for 7 days.

235. 18-month-old child with diarrhea  refer


Red Flags: Pregnancy, Recent ABX use within 1 month (R/O c. Difficile), severe>6 bowel movements/day x
>48hrs, Fever, Vomiting, Abdominal cramps or pain (R/O SC. Difficile), Dehydration signs: sunken eyes & dry
mouth, Tarry stool, comorbidities: DM, CAD, CKD etc…, immunocompromised, chronic diarrhea

236. Intrinsict BB and CCB  not with structural heart abnormalities

237. When your selling phenytoin what do you counsel the patient on
a) Take with food
b) Avoid drinking alcohol
Acute alcohol intake may increase serum levels of phenytoin while chronic alcohol use may decrease them

238. Selenium: 0.24

239. Something about


adrenaline - anesthetia, epipen
Adrenaline is added to a local anesthetic to prolong its effect. However, caution is recommended when
adrenaline is used in body parts with end arteries, that is, arteries that are the only blood supply of that
particular organ, for example, fingers and toes
240. A clinical trial about 2 vitamins X and T may decrease the risk of MI the result is hazard
ratio HR 0.63. (CI 95%:0.3-0.8), so what does that mean:
a) T decrease risk of MI more than X
b) X decreases the risk of MI more than T
c) There is a trend that T may have a role in MI
d) There is no big difference between x and T in decreasing MI
When Hr is below 1 it means it decreasing the risk and if it above 1 it means increasing the risk
And 95 percent it is significant difference so D is wrong.
If HR was above 1 so it could cause Mi but it is below 1 so C is wrong and as HR between T and X is 0 ,63 it
means researcher saying he is 95 percent confident that drug X decrease Mi 63 percent more as compare to
Drug T and his confidence level is this can be between 30 to 80 perecent.
If range of confidence interval crosses 1, it's not significant. Range is already given 0.7‐1.5.
If range is before 1 (0.7‐0.9), the drug is more effective, if after 1 the placebo is more effective. If range was
0.7‐0.9 then ans would be B
• RR/HR/OR
 If Cl contains 1 (95% CI 0.7 – 1.2)  there is NO difference = Accept NULL hypothesis = P > 0.05
 If Cl does NOT contain 1 (95% CI 1.1 – 1.8 or 0.3 – 0.8)  There IS difference = Reject the null
hypothesis = P </= 0.05

241. 1:5000 what is strength in 60 ml?


1: 5000 means 1 gm in 5000 ml so, X in 60ml X = 60/5000 = 0.012gm

242. A patient with a RX and amp He is in a hurry. The tech counselled him

243. Flumist live attenuated influenza vaccine (LAIV)


a) Can be administered to < 2 years and older adults.
b) Not indicated for children < 2 y or adults > 59 y.
Who should not be vaccinated with the nasal spray flu vaccine? Some people should not get the nasal spray
influenza vaccine:
● Children younger than 2 years, Adults 50 years and older, Pregnant women
● People with a history of severe allergic reaction to any component of the vaccine
● Children 2 years through 17 years of age who are receiving ASA or salicylate‐containing medications.
● People with weakened immune systems (immunosuppression)
● Children 2 years through 4 years who have asthma or who have had a history of wheezing in the past
12 months.
● People who have taken influenza antiviral drugs within the previous 48 hours.
● People who care for severely immunocompromised persons who require a protected environment (or
otherwise avoid contact with those persons for 7 days after getting the nasal spray vaccine).
In addition, the following conditions are precautions to the use of the nasal spray influenza vaccine:
➢ Asthma in people aged 5 years and older.
➢ Other underlying medical conditions that can put people at higher risk of serious influenza
complications. These include conditions such as lung disease, heart disease (except isolated
hypertension), kidney disease (like diabetes), kidney or liver disorders, neurologic/neuromuscular, or
metabolic disorders. “People at High Risk of Developing Flu–Related Complications.”
➢ Moderate or severe acute illness with or without fever.
➢ Guillain‐Barré Syndrome within 6 weeks following a previous dose of influenza vaccine

244. Lactulose is schedule 3, Flonase (Fluticasone) is schedule 3

245. SE of ARNI use is low BP & angioedema (similar to ARBs)

246. Duloxetine SE: Very common: Headache, somnolence, nausea, dry mouth. Common: decreased
appetite, insomnia, agitation

247. Man came to pharmacy to check BP but when pharmacist check it low, patient say this was
the lowest about 100/80. This has never happened before what is the reason for this?
a) Sildenalfil had regularly for 3 months
b) Pharmacist did not put hand on hard surface below heart level
c) The measuring cuff was loose etc.

248. 14 years old girl come to your pharmacy with 40 years man and seems like they are in
harmony. She was asking about non pharm for her vaginal discharge, you realized that it is
Trichomonasis infection. What to do?
a) Refer to specialist
b) Report to child abuse
c) Consulting both of them about the infection transmission and using condoms
d) Call police
Other opinion said it is B as age below 16
249. Compounding that need sterile conditions:
a) Anti-microbial cream
b) Nasal drops
c) Eye drops
d) Otic or ear drops

250. Black pt with HTN, gout and Diabetes - What to give – ACEIs

251. Lyme disease test:


Screening for enzyme immunoassay (EIA): lacks specificity and its use as a stand‐alone test could yield false‐
positive results. If EIA is equivocal or positive, use confirmatory immunoblot (IB).
Lumbar puncture (LP): analysis of the CSF should be considered for patients with unilateral facial nerve
palsy. Synovial fluid analysis, ECG, Asymptomatic seroconversion.

252. Patient has onychomycosis (Tinea unguium) will refer all except:
a) Nail with dark colour
b) Thick nail
c) Pre-school age
When should you see a health‐care provider?
 If your toes or fingers become swollen, red, painful or drain pus. This may be a bacterial infection
that will need antibiotics.
 If you have a fungal infection that is not clearing up or you are getting other infections on your skin.
 To make sure the medicine you take by mouth is working and not causing any harm.
 If you develop any side effects from your medications such as rash, nausea, vomiting, fatigue,
abdominal pain or dark‐coloured urine.
Red Flags: <12 yrs old, pain swelling & redness (pain only happen when nail detaches), Psoriasis, Drugs such
as Tetracycline or chemotherapy

253. Testosterone is Sch. 1, Refill with interval with no transfer.

254. Study conducted in hospital, what is the criticism as it differs from community?
a) Sensitivity
b) Specificity
c) Positive likelihood ratio
d) Positive predictive value

255. Patient with Ascites and can’t tolerate spironolactone, what to give him?
a) Amiloride
b) Furosemide
c) Metolazone
Can be substituted for spironolactone if intolerable side effects develop.
256. A patient is currently using Digoxin for the maintenance of congestive heart failure
Condition. His current blood digoxin level was found to be 2.5 ng/ml. What should be
recommended for this patient?
a) Decrease the dose of Digoxin
b) Continue digoxin as prescribed
c) Skip digoxin for 2 days and restart at a reduced dose.
d) Discontinue digoxin and add antidote
Measure trough serum concentrations at least 8 h after administration and adjust the dose to maintain the
serum concentration between 0.6 and 1 nmol/L.
Digoxin toxicity is more commonly associated with serum digoxin concentrations greater than 2 nanogram/ml.
However, serum digoxin concentration should be interpreted in the clinical context. Toxicity may occur with
lower digoxin serum concentrations. In deciding whether a patient's symptoms are due to digoxin, the clinical
state together with the serum potassium level and thyroid function are important factors
Digoxin should be temporarily discontinued until the adverse reaction resolves.
Every effort should also be made to correct factors that may contribute to the adverse reaction (such as
electrolyte disturbances or concurrent medications). Once the adverse reaction has resolved, therapy
with digoxin may be reinstituted, following a careful reassessment of dose.
Withdrawal of digoxin may be all that is required to treat the adverse reaction. However, when the primary
manifestation of digoxin overdosage is a cardiac arrhythmia, additional therapy may be needed.
If the rhythm disturbance is a symptomatic bradyarrhythmia or heart block, consideration should be given to
the reversal of toxicity with DIGIBIND [Digoxin Immune Fab, the use of atropine, or the insertion of a
temporary cardiac pacemaker. However, asymptomatic bradycardia or heart block related to digoxin may
require only temporary withdrawal of the drug and cardiac monitoring of the patient.
If the rhythm disturbance is a ventricular arrhythmia, consideration should be given to the correction of
electrolyte disorders, particularly if hypokalemia (see Administration of Potassium) or hypomagnesemia is
present. DIGIBIND is a specific antidote for digoxin and may be used to reverse potentially life‐threatening
ventricular arrhythmias due to digoxin overdosage

257. Drug shortage and the pharmacist is keeping to his family member
a) Violate Justice
b) Violate veracity
c) Uphold beneficience

258. Patient in hospital is taking IV Cotrimoxazole (each ml contains 80 mg SMX / 16mg


TMP) 0.5 ml Q6hr. Dr. want to shift him to liquid (200 mg SMX / 40 mg TMP) in each 5 ml,
how many of liquid is needed to provide the equivalent dose as was taken IV.
Answer:
0.5 ml Q6hr = 0.5 * 4 = 2 ml /Day
SMX: 80 mg ------- 1 ml X mg ------- 2 ml X= 2*80/1 = 160 mg
Liquid 200 mg ------- 5 ml 160 mg ------- Y ml Y = 160*5/200 = 4 ml/Day
259. PJP prophylaxis in HIV +ve patient? sulfamethoxazole / trimethoprim

260. HF + Candesartan (Newly Rx) and taking Ferrous gluconate OD and catheter and takes
BB+ Statin+ Aspirin? Could be Ferrous gluconate dose

261. DVT pt and have needle phobia, what to give?


a) Rivaroxaban
b) Dalteparin
Pharmacokinetics: Absorption: Fondaparinux sodium administered by subcutaneous injection is rapidly and
completely absorbed (absolute bioavailability is 100%). Following a single subcutaneous dose of fondaparinux
sodium 2.5 mg in young male subjects, Cmax of 0.34 mg/L is reached in approximately 2 hours.

262. Stakeholder needs, system quality, finance, software program.

263. How to compare between NPH insulin and long acting insulin? Clinical guidelines

264. ISMP? Pantoprazole IV Q 12 H (This is the right option)

265. Hospital error (hydromorphone and morphine)? Remove from Ward stock or Separate.

266. Symogi effect? Decrease bed time insulin.

267. You observe shortage in narcotics, you knew that the reg. technician is taking from it, what
to do? terminate & inform the college.

268. Pharmacist is rude to the patient? Talk privately to the pharmacist.

269. Kid on Methylphenidate, his teacher claims his improvement but he is anorexic now, what
to do? Give big breakfast to the kid.
 Common, usually transient—continue therapeutic trial: anorexia, insomnia, weight loss, irritability,
dizziness, weepiness, headache, abdominal pain.
 Transient—stop and re‐evaluate: “zombie‐like” effects, psychotic reactions (such as hallucinations),
agitation, tachycardia, hypertension, growth failure, rebound hyperactivity, leukopenia, blood
dyscrasias.
 Monitor patient for suicidal thoughts/ideation; consider a change in treatment if concerns arise.
 Overdose symptoms—stop and retitrate: “glassy eyes,” insomnia, hyperactivity.
 Significant: sudden cardiac death reported; neurologic symptoms; exacerbation of tics; avoid in
patients with a history of cardiovascular conduction disturbances, hypertension, acute psychotic
episodes and hyperthyroidism.
 If seizures occur, or if frequency increases in patient with controlled epilepsy, stop and re‐evaluat
What to monitor?
 Monitor BP and heart rate at baseline and within 1–3 months at follow‐up.
 CNS stimulants are associated with the potential for abuse and dependence. Assess for this risk prior to
prescribing and monitor for signs of abuse and dependence while on therapy.
 Monitor for bipolar disorder and screen for manic symptoms.
 Monitor for growth in pediatric patients, including height and weight every 3–6 months.
 Capsule contents can be sprinkled on soft food such as applesauce, ice cream or yogourt

270. Post MI? Give high dose statin.


Statin high dose for plaque stabilization: Atorvastatin 80 mg, rosuvastatin 40 mg

271. Patient goes to hospital and start treatment for opioid withdrawal and the pt. dead, what to
do with his narcotics? If not a unit dose or used, you can return to the pharmacy

272. How to avoid a mistake that happened with a technician?


a) Double check strategy (Decrease confirmation bias)

273. How to ensure that the Verbal Rx was received correctly?


a) Repeat the names with Doctors

274. Patient is having Hip necrosis, He is on Allopurinol, prednisone, Methotrexate, NSAID.


Why he develops Hip necrosis? Prednisone.
Injuries, such as hip dislocation or fracture, can damage nearby blood vessels and reduce blood flow to bones.
Steroid use. Use of high‐dose corticosteroids, such as prednisone, is a common cause of avascular necrosis

275. Solution contain 5,000,000 unit diluted by 8 ml of water yield 500,000 unit / ml.how much
water needed to make it 125,000 unit/0.5 ml.
Answer:
500,000 U ------- 1 ml 5,000,000 U ------- X ml
X = 1*5,000,000/500,000 = 10 ml Amount of Substance = 10 - 8 = 2 ml.
125,000 U ------- 0.5 ml 5,000,000 U ------- X ml
X = 5,000,000*0.5/125,000 = 20 ml Amount of water = 20 - 2 = 18 ml

276. Newly diagnosed Diabetes seminar? Don’t educate on Insulin mixing

277. Nitroglycerin Spray? un-scheduled drug


(All NTG Except for sublingual immediate release dosage forms) Nitroglycerin sublingual immediate release
dosage forms schedule 2

278. ISMP? 15.0 (No trailing zero)


Methadone 8.1 gm Na Benzoate 1.15 gm Liquid qs 810 ml
279. What is the amount of Methadone & Na benzoate to prepare 900 ml?
Answer:
Methadone = 8.1/810 * 900 = 9 gm Na Benzoate = 1.15/810 * 900 = 1.27 gm

280. Topentadol? Opioid narcotic (No refills allowed, no verbal, no transfer, sales record for 2 years,
Partfill with with total amount quantity for each partfill and interval between fills)

281. Depression 1st episode, when to stop? 6-9 month

282. Aminoglycoside IV in a hospital where they approximate the dose to nearest 50. Dose is
420 mg, Stock 80 mg/2ml, how many ml will you take?
Answer:
Dose = 420 so be 400 to the nearest 50
80 mg -------- 2 ml 400 mg ------- X ml X = 400*2/80 = 10 ml

283. 67 yrs old man needs pneumococcal vaccine? Take one dose now & booster every 5 years.
Polysaccharide (pneumococcal 23-valent Conjugate (pneumococcal 13-valent conjugate
polysaccharide vaccine (PNEU-P-23) vaccine (PNEU-C-13)
Conjugate to protein carrier to enhance immunogenicity Recommended for routine infant immunization and in
Recommended for children ≥ 24 months and adults with those with immunocompromising conditions.
conditions that increase risk of invasive pneumococcal Q. Persons who are eligible for both conjugate and
disease (IPD), adults ≥65 y, smokers, residents of polysaccharide vaccines should receive conjugate
longterm care facilities, persons with alcoholism, vaccine first. However, those who have already
homeless persons and illicit drug users. received PNEU-P-23 may still receive PNEU-C-13, in
A booster dose recommended in those with asplenia, which case administer ≥1 y after PNEU-P-23.
sickle cell disease, hepatic cirrhosis, chronic kidney Approved for adults ≥50 years of age and advisable
disease or nephrotic syndrome, HIV infection, for patients ≥65 years of age on an individual basis.
immunosuppression. Adult Hematopoietic stem cell transplantation HSCT
Q. Give 8 wk after last dose of conjugate vaccine (if recipients: 3 doses (0.5 mL/dose) IM administered 4
eligible for both). wk apart, starting 3–9 months after transplant. No role
In Hematopoietic stem cell transplantation HSCT for booster dose.
recipient, give 6–12m after conjugate vaccine last dose. Adults with HIV infection and immunocompromising
Give after 5 y if ≥11 y of age when initially immunized. conditions: 0.5 mL IM once. No role for booster dose.
After 3 y if ≤10 y of age at time of initial immunization.

284. Pregnant lady needs castor oil, to follow non-maleficience, what to say? Avoid taking it

285. Surrogate end point? Microalbuminuria


Albuminuria is a biologically plausible surrogate endpoint for the progression of chronic kidney disease
286. Resistant psoriasis case on MTX, what to give next?

287. Trigeminal neuralgia? OXYcarbamazepine.

288. Drug X cause S.E in 50 patients out of 500 & Drug Y cause the same S.E in 25 out of 500,
what is NNT?
Answer:
X = 50/500 *100% = 10% Y = 25/500*100% = 5%
ARR=10-5 = 5 % NNT = 1/ARR* 100 = 1/5 * 100 = 20

289. calculation about displacement value?


A vial of lyophilized drug is labeled 10,000 units, to reconstitute, add 17 ml of Sterile Water for
Injection to obtain 500 units per ml. How many ml of SWFI must a pharmacist add if a 1000
u/ml concentration is needed by the nurse ?
a) 7ml
b) 8.5 ml.
c) 10 ml
d) 17 ml
Calculate the displacement vol of the drug
500u in 1 ml  10000 u in x ml so, X=10000*1/500= 20 ml
Displacement vol of drug= total vol – swfi  20 – 17 = 3 ml
Then calculate the vol of swfi to make the new conc.
1000 u in 1 ml  10000u in x ml
Total vol x= 10000*1/1000=10 ml
Swfi vol= total vol-displacement vol of the drug  10 – 3 = 7ml
290. patient wants to know the effectiveness of vitamin D in cancer, which reference will you
search for this information?
a) Reviewed articles about vitamin supplementation
b) Ask oncologist in cancer center
c) CTC
d) Primary journal

291. patient with atrial fibrillation and doesn’t want to use medications, what to do?
a) Leave him
b) Keep engaging him so that he can change his mind

292. A hypokalemic patient his K level is 2.3 m. mole/L so he is in need to K supplement. If the
maximum amount allowed per day is 3L given at the rate of infusion of 60m.mole/L. find the
rate of infusion in m. Mole/hr.
Answer:
60 m. Mole ------- 1L. X m. Mole ------- 3L.
X= 3*60/1 = 180 m. Mole
180 m. Mole ------ 24 hr. Y m. Mole ------- 1 hr.
Y = 1*180/24 = 7.5 m. mole / hr.

293. Narcotic: BZD Narcotic… Benzo…pre target…C4-0

294. Pt. with DM and LDL<2……???? Has lower risk for cardiovascular diseases, such as heart
attack and stroke. http://guidelines.diabetes.ca/cpg/chapter25

295. Numbers to know, maybe you will find in a tough calculation (75 min, 4.83)
November 2017

1. In community pharmacy, which is one need more expenses:


a) Inventory of the pharmacy
b) Rent & Salary of stuff
c) Tax expense
d) Facility maintenance cost
According to a report from the Texas Tech University School of Pharmacy: "In an average pharmacy, costs of
goods sold account for approximately 68% of total expenditures."

2. Student counsells patient and patient was confused. What do you do?
a) Practice with student before he counsels
b) Give the student material to read before counselling
c) Make it a policy to be present while student is counselling & interrupt whenever
they say something wrong (should be under pharmacist supervision)

3. Lady come to pharmacy for buying OTC product for cough and seasonal allergic problems.
While talking with Pharmacist she told that she has cardiac problem and takes medication. Now
what is the initiative pharmacist can take if he wants to prevent such events in the future?
a) Keep this type of product behind the counter so pharmacist can intervene
b) Attach a sign in the isle so patient knows he need to talk to pharmacist for advice
c) Don’t sale these types of products
d) Ask the front store staff not to recommend medicinal products

4. You heard that a front store employee is telling patient that ibuprofen is a good option for her
condition. What is the appropriate action pharmacist need to take?
a) Review about counselling policy will all staff
b) Tell the front store employee not to do such action
c) It’s an OTC product and not a concern for pharmacist

5. 60 years old patient is diagnosed with atrial fibrillation. His blood pressure reading is 135/85
but heart rate is 120-130. Patients is taking HCTZ for his hypertension. She is doing a general
surgery and she needs VTE prophylaxis. Also, she is on raloxifene and some other medications.
What inreases her risk of VTE?
a) Raloxifene  VTE risk similar to estrogen.
b) Amiodarone
c) NSAID
6. What’s her CHADS2 score?
a) 0
b) 1
c) 2
d) 3
CHADS2 score: Congestive heart failure 1, Hypertension 1, Age >75 years (female >65years) 1, Diabetes 1,
prior Stroke 2

7. What would you give her for VTE prophylaxis, what will be the treatment of choice?
a) Dabigatran
b) Rivoroxaban
c) Dalteparin
d) Warfarin
e) Clopidogrel
Dalteparin, enoxaparin, nadroparin and
tinzaparin are low molecular weight heparins
approved for both the treatment and
prophylaxis of VTE moderate risk.
Fondaparinux is an indirect factor Xa inhibitor
approved for the prophylaxis of VTE in high‐
risk orthopedic patients
Apixaban, dabigatran and rivaroxaban are
direct‐acting oral anticoagulants that can be
used for VTE prophylaxis following elective
total hip or total knee replacement surgery.

8. Doctor give patient Rivaroxaban as DVT prophylaxis after surgery. What is the monitoring
for Rivaroxaban?
a) Require professional monitoring for bleeding
b) Self monitoring of bleeding
c) Renal function
d) INR
Answer can be Glomerular Filtration rate instead of Bleeding monitoring
Measuring PT using Neoplastin reagent, or Factor‐Xa assay using rivaroxaban ‐specific calibrators and
controls, may be useful to inform clinical decisions in these circumstances.
Although XARELTO therapy will lead to an elevated INR, depending on the timing of the measurement, the
INR is not a valid measure to assess the anticoagulant activity of XARELTO.
At recommended doses, XARELTO affects the measurement of the aPTT and Heptest. These tests are not
recommended for the assessment of the pharmacodynamic effects of XARELTO.
9. The hospital is going to do research on VTE prophylaxis by looking at charts. What is the
most valuable info you can get from this research?
a) Find out bad abbreviations made when ordering VTE prophylaxis
b) Find out the best duration of stay for VTE patients
c) Something about lack of standardized care for VTE patients
Charts here can mean Case control study. Possibly they can correlate the risk of clots with the duration of
LMWH administration to find an association

10. Meningitis case. 14 years old child who has cochlear implants and had recurrent otitis media
in his early child hood, he is admitted to hospital with high fever, Pain and he was unconscious.
His history includes use of SABA PRN & Fluticasone/Salmeterol BID for asthma control, what
will be probable cause of patient’s meningitis Or What increases risk of meningitis?
a) Cochlear implants
b) Recurrent Infection of AOM
c) Corticosteroid uses for Asthma
d) Hypothyroidism
Some people say; the reason it's not bilateral cochlear implant because it's date back to 3 years ago.
Recently has recurrent otitis media is the answer. And also, they ask OSAMA: he said I surf a lot I think
cochlear implants. Misbah answer: recurrent otitis media
CTC ‐ Conditions increasing risk of invasive pneumococcal disease: chronic cerebral spinal fluid leak,
chronic neurologic condition that may impair clearance of secretions, cochlear implants, chronic cardiac or
pulmonary disease, diabetes mellitus, chronic kidney disease, nephrotic syndrome, chronic liver disease
(including hepatic cirrhosis due to any cause), asthma that required medical care in the preceding 12
months, sickle cell disease or other hemoglobinopathies, congenital immunodeficiencies, anatomic or
functional asplenia, immunocompromising therapy, HIV infection, hematopoietic stem cell transplant
recipient, malignant neoplasms, solid organ or islet transplant candidate or recipient.
The most common pathogens that causes bacterial meningitis are the same pathogens that colonize the
mucosal surface of the respiratory tract; they include Streptococcus pneumoniae, Neisseria meningitidis
and Haemophilus influenzae. During or after a respiratory viral illness, colonization rates in the
nasopharynx will be up to 40%, especially in young children.
Importantly as well, one should be very careful with the Q wording when it comes to risk factors versus
causation. So, for example, if the Q version is about RISK FACTOR (association), cochlear implants remain t
he most significant risk factor allowing access of pathogens to the brain, especially that this Q version
is straightforward since AOM is in that scenario a remote history.
However, for a Q version in which you've repeated recent AOM episodes, and the Q is asking clearly,
"what is the most likely CAUSE of the current meningitis? Then the answer will change to AOM, especially
That cochlear implant here appears in the background (remote history). I hope this is clear to everyone.
Flipping scenarios is a very common phenomenon on PEBC exams, and if the candidate is dragged by
previous experience related to similar questions or situations, they're very likely to make mistakes on
those questions. On the other hand, candidates who are going to tackle each and every question based on
their knowledge, understanding, and acquired skills will stand a much better chance.
11. What is likely cause of meningitis in this kid?
a) S. pneumonia
b) Neiserria meningitides
Children ≥1 month: S. pneumoniae, N. meningitidis, S. agalactiae (Group B streptococcus), Haemophilus
influenzae type b, E. coli (rare), L. monocytogenes (rare), Other Enterobacteriaceae

12. What is elevated in CSF fluid?


a) Protein
b) Glucose
c) WBC
Another version: What increase in viral
meningitis compared with bacterial meningitis?
a) WBCs
b) Protein
c) Lactate
d) Glucose

13. What do you give for empiric treatment?


a) Vancomycin + ceftriaxone
b) Cefotaxime + Ampicillin  for infant < 1 month
c) Vancomycin + Ampicillin

14. Laboratory culture test result for this patient has come back and its H. influenza that causing
the meningitis. But sensitivity results still aren’t back yet. What can we add to the existing
regimen as empiric therapy?
a) Penicillin
b) Azithromycin
c) Cefotaxime
d) Ampicillin
H influenza: beta lactamase negative: ampicillin, beta lactamase positive: ceftriaxone/cefotaxime

15. DZ 35 years old lady having skin problem identified as psoriasis. She smokes occasionally
and take 2-3 drinks in a week. She had an upper respiratory tract infection and took antibiotic
medication 2 weeks. What could be the probable reason for psoriasis exacerbation in this case?
a) Cigarette smoking
b) Infection
c) Use of antibiotics
d) Family history
16. He took hydrocortisone valerate but his psoriasis is still occasionally itchy. What is the
first thing you do?
a) Recommend something else
b) Ask how he has been using the hydrocortisone
Question the diagnosis if there is no relief of disease within 1–2 months of optimal therapy.

17. Lady with seizure and shes obese. Shes on a


contraceptive. Drinks alcohol 4 drinks per day on
weekends. Smokes twice per week. What are her
risk factors for seizures?
a) Smoking
b) Alcohol use
c) Obesity

18. What would you start her on for her seizures?


a) Divalproex
b) Phenytoin
c) CBZ
d) Leviteracetam  other options ↓ OC efficacy
All patients treated with antiepileptic drugs, irrespective of indication, should be monitored for signs of
suicidal ideation and behaviour and appropriate treatment should be considered. Patients (and caregivers of
patients) should be advised to seek medical advice should signs of suicidal ideation or behaviour emerge.

19. Which one can be used for Epilepsy


a) Vit E
b) Folic acid  used as supplement with phenytoin
c) Vit B12
d) ketogenic diet
Another version: Who else would you refer her to? Dietician
Potentially beneficial dietary interventions in the treatment of epilepsy include identifying and treating blood glucose
dysregulation, identifying and avoiding allergenic foods, and avoiding suspected triggering agents such as alcohol,
aspartame, and monosodium glutamate.
The ketogenic diet may be considered for severe, treatment‐resistant cases. The Atkins diet (very low in
carbohydrates) is a less restrictive type of ketogenic diet that may be effective in some cases.
Nutrients that may reduce seizure frequency include vitamin B6, magnesium, vitamin E, manganese, taurine, dimethyl
glycine, and omega‐3 fatty acids. Administration of thiamine may improve cognitive function in patients with epilepsy.
Supplementation with folic acid, vitamin B6, biotin, vitamin D, and L‐carnitine may be needed to prevent or treat
deficiencies resulting from the use of anticonvulsant drugs.
Vitamin K1 has been recommended near the end of pregnancy for women taking anticonvulsants.
Melatonin may reduce seizure frequency in some cases, and progesterone may be useful for women with cyclic
exacerbations of seizures.
20. Female patient complain that she become anxious and get panicked. She worked in a
company where she has to do a lot of presentations, face many clients and she feels like fleeing
form such situations. lately her situation is worsening even she went to emergency for chest
pain. Doctors couldn’t find any abnormality in her heart. Last week she got 8 panic attack and
now doctor wants to initiate a 6 weeks trial of anti anxiety medication. What is the best choice?
a) Sertraline
b) Sertraline + clonazepam
c) Venlafaxine
d) Venlafaxine + clonazepam
Augmenting the SSRI or SNRI with a brief course of a low‐dose benzodiazepine (no longer than 8 weeks, to
minimize risk of dependence/withdrawal) can increase adherence to medication and produce a more rapid
response than with antidepressants alone.
CTC: The SSRIs citalopram, escitalopram, fluoxetine, fluvoxamine, paroxetine and sertraline are all effective
in treating panic disorder with or without agoraphobia. In several meta‐analyses, SSRIs have been shown to
improve panic symptoms, avoidance behaviours associated with agoraphobia, depressive symptomatology
and general anxiety; however, no SSRI has proved superior to any other

21. She becomes pregnant and is stressed about her job and scared she’ll lose her job and
having hard time maintaining her expenses. She is asking pharmacist to help her with her
medication expenses and other issues. Which is the best option to look for help?
a) Social worker
b) Occupational therapist
c) Manufacturers program
d) Give discount from pharmacy

22. Patient on loratidine for her allergies. She has been using it for 2 days. She has chronic
allergies. Used loratidine before. What do you tell her?
a) Wait 2 weeks for effect
b) Add decongestant oral
c) Add topical decongestant
d) Recommend she goes to her doctor as she may become tolerant to loratidine and may
need to try another agent.

23. 1000 ml of 10% dextrose w/v dextrose was prepared. How many Kilocalorie will
this make knowing that 1 gram of dextrose give 3.4
Answer:
10 gm ------- 100 ml X gm ------- 1000 ml X = 1000*10/100 = 100 gm
1 gm ------- 3.4 Kcal 100 gm ------- X X = 100*3.4/1 = 340 Kcal
24. Something about annual performance meeting. What is not true.
a) Talk about performance
b) Work out personality conflicts of staff
c) Advise staff about business goals

25. Patient is prescribed with Tobramycin Eye droop with a concentration of 13.5 mg/mL.
You have a stock solution of 0.3% tobramycin. Now what volume of 40 mg/mL conc. you
will add with 5 ml of 0.3% stock solution?
a) 4 ml
b) 2 ml
c) 6 ml
d) 5 ml
X = 1 * 0.3 / 100 = 0.003 gm = 3mg So 0.3%=3mg/ml
By allegation method:

26. Testosterone gel prescription for 2.5 g daily of 2% for 30 days. You only have 1% and
they come in tubes each containing 2.5 g. You have 30 tubes. How much testosterone powder
do you have to add?
a) 300 mg
b) 765 mg
c) 780 mg
27. Prescription compound recipe. Drug 8%, BHT 0.001%, Something else qs Mitte: 60g.
How much of the BHT do you need?
a) 0.06 g
b) 0.6 mg
c) 0.06 mg
0.001gm ‐‐‐ 100gm X ‐‐‐‐‐ 60gm x = 60 * 0.001 / 100 = 0.0006mg = 0.06gm

28. What to do in case of RX forgery that has already been dispensed?


a) Reduce your stock of narcotics
b) Change the staff
c) Report it to OCS
If it is discovered months after being dispensed, call the local police and the regulatory authority.

29. What interacts with tamoxifen?


a. Venlafaxine
b. Paroxetine
CYP2D6, is the principal enzyme that converts tamoxifen into endoxifen, a form that is active in the
body. Some antidepressants are such strong inhibitors of CYP2D6 that women who take these drugs may
not benefit from tamoxifen. Paroxetine (Paxil), fluoxetine (Prozac), and bupropion (Wellbutrin) — are most
likely to inhibit CYP2D6 and interfere with tamoxifen treatment.
30. You’re taking phenelzine. What
CAN you eat with it?
a) Yogurt
b) Draft beer
c) Fava beans
d) Aged cheese

31. Verapamil is metabolized by


a) CYP2D6
b) CYP2C9
c) CYP3A4
d) p-glycoprotein
Verapamil increases digoxin levels by 50 –75% within 1 wk (monitor levels).
Inhibit the metabolism of carbamazepine, cyclosporine, lovastatin, simvastatin.
Increased sensitivity to the effects of lithium (neurotoxicity) has been reported during concomitant
verapamil‐lithium therapy with either no change or an increase in serum lithium levels.
In a small number of patients with hypertrophic cardiomyopathy, concomitant use of verapamil and
quinidine resulted in significant hypotension.
Monograph: Verapamil is extensively metabolised. In vitro metabolic studies indicate that verapamil is
metabolised by cytochrome P450, CYP3A4, CYP1A2, CYP2C8, CYP2C9 and CYP2C18. In healthy men, orally
administered verapamil hydrochloride undergoes extensive metabolism in the liver, with 12 metabolites
having been identified, most in only trace amounts. The major metabolites have been identified as various N
and O‐dealkylated products of verapamil. Of these metabolites, only norverapamil has any appreciable
pharmacological effect (approximately 20% that of the parent compound), which was observed in a study
with dogs.

32. An obese patient taking Rizatripan + NSAID for migraines but only got partial relief. She
has had 6 recurrences of migraines without aura in the past 18 months which are causing her
stress and started to need days off from work. This time she went to the ER and they gave her a
Dihydroergotamine DHE injection and now she feels better. she was discharged from the
hospital about 12 H ago. She is being prescribed with Rizatriptan as her 3rd triptan trial. What
will be your advice while dispensing the medication??
a) Take it as soon as you notice aura.
b) Wait until 24 hours after DHE injection.
c) Take 2 tablet of Rizatriptan when you get the attack
d) Don’t take the drug until 3 Days
CTC. Due to the risk of serotonin syndrome, do not use within 24 h of a triptan.
33. After a short trial of rizatriptan physician decided to start her on topiramate as migraine
prophylaxis. Prophylactic treatment for migraine is considered in which of the following
condition or She is eligible for receiving migraine prophylaxis because:
a) Patient is admitted to emergency department
b) Patient at risk of Medication Overuse Headache or Hampering patient quality of life
c) Dihydroergotamine has already use as last line
d) Obese patient need prophylaxis
e) She reached threshold of migraines that is needed for prophylaxis
f) She exceeded the number of attacks per year
MOH > 15 DAYS NSAIDS > 10 days Triptans per month
Consider prophylaxis if migraine attacks have a significant impact on the patient’s quality of life despite
appropriate use of abortive therapies, or if the frequency of attacks puts the patient at risk of
medicationoveruse headache. Guidelines suggest a trial of at least 2 months, following dose titration,
before assessing benefit. Advise patients to maintain a headache diary to monitor headache triggers,
frequency and intensity, menstrual cycle, use of preventive and abortive medications, and side effects.
Successful prophylaxis is usually defined as a ≥50% reduction in headache frequency or days with headache,
though some patients may report improved response to abortive therapy or decreased headache severity or
duration. If there is no benefit after a 2‐month trial at the target/optimal dose, try a different medication. If
prophylaxis is deemed beneficial, continue for 6–12 months then consider tapering dose to assess ongoing
need. If headache intensity and frequency increase, the dose can be increased to previously effective levels

34. What is the additional benefit patient will get using Topiramate?
a) Improvement in Nausea
b) Improvement in sleeping pattern
c) Weight loss
d) Appetite improvement

35. On the hospital discharge note, there’s a note that says “stop rizatriptan” (she was
taking rizatriptan before), on her profile, mark it as:
a. Mark is as “discontinued”
b. Put it on hold/log

36. What bad side effects could topiramate cause that might be an issue to this patient?
a) Teratogenic
b) Steven Johnson syndrome
c) Metabolic alkalosis
d) Interaction with NSAIDs
37. What auxiliary label to add to dexamethasone?
a) Take with food or milk
b) Can be taken in Systemic fungal infections
c) Increase dose for elderly patient
CTC: Oral formulations should be administered with or after meals to minimize GI upset.

38. Patient comes into clinic checkup. He is on some drugs and got a script last visit for
metolazone. Patient is nauseous, has hypocalcemia and hypophosphatemia. But the pharmacist
at the pharmacy screwed up and gave methotrexate and patient has been taking it biweekly for 4
weeks. What sign you will look for methotrexate toxicity?
a) Hypophosphatemia
b) Hypocalcemia
c) Nausea
d) Diarrhea
e) Abdominal pain
f) Renal failure
CTC: The most frequently reported adverse reactions include ulcerative stomatitis, leukopenia, nausea and
abdominal distress. Other frequently reported adverse effects are malaise, undue fatigue, chills and fever,
dizziness and decreased resistance to infection.

39. Which of the following should be done for the patient?


a) HbA1C
b) TSH
c) Platelets
CTC: Hematologic  Hemorrhage (1–10%), neutropenia (>10%), thrombocytopenia (>10%)

40. What needs to be refrigerated?


a) Clarithromycin suspension
b) Travaprost (2-25 degrees)
c) Timolol
d) Latanoprost
e) A product with erythromycin and benzoyl peroxide
Keep Refrigerated: All reconstituted antibiotics (EXCEPT Clarithromycin and SMX/TMP), all vaccines, eye
drops (latanoprost +/‐ timolo and trifluridine), teriparatide, sirolimus solution, tobramycin solution for
inhalation, insulin preparations, GLP‐1 analogues, erythropoietin, interferon, omalizumab, denosumab,
Clindoxyl gel (clindamycin + benzoyl), biological DMARDs, and glatiramer.
41. Patient with H pylori on Omeprazole, Clarith, Amox, and metronidazole 250 mg TID for 14
days. What’s wrong with that?
a) 14 days is too short
b) Metronidazole Should be 500 TID.
Other version: Metronidazole counseling.
a) Avoids sun
b) Take with food
c) Separate from antacids
Don't take more or less metronidazole than is recommended. Try to take the medicine at the same time
each day. The tablets can be taken with food or a glass of milk to prevent upset stomach. You should take
the extended‐release tablet on an empty stomach at least one hour before, or two hours after, a meal
Treatment Quadruple Therapy: FOR 14 DAYS
of PUD due  Recommended option (First line or prior treatment failure)  PPI (BID), bismuth
to subsalicylate (2 tabs QID), metronidazole (500 mg TID-QID), tetracycline (500 mg QID).
Helicobacter  Recommended option (First line)  PPI (BID), amoxicillin (1 g BID), metronidazole (500
pylori mg BID), clarithromycin (500 mg BID).
Infection Triple Therapy: FOR 14 DAYS
 Recommended option (Prior treatment failure only)  PPI (BID), amoxicillin (1 g BID),
levofloxacin (500 mg QD)
 Restricted option (First line) 
1. PPI (BID), amoxicillin (1 g BID), metronidazole (500 mg BID).
2. PPI (BID), metronidazole (500 mg BID), clarithromycin (500 mg BID).
3. PPI (BID), amoxicillin (1 g BID), clarithromycin (500 mg BID).

42. Pharmacist discovers that this was due to a data entry error by the technician, what to do?
a) Seminar for techs on sound alike and look alike meds
b) Make it a policy for techs to double check meds before data entry
c) Separate them on the shelf

43. Patient with C diff. She finished Maxiflox course already and taking Pantoprazole for
occasional heartburn after food. WBC is less than 15 and no abdominal pain. What’s the
severity of her infection?
a) Mild-moderate
b) Severe

44. What do you treat?


a) Vancomycin IV
b) Metronidazole 500 mg TID
c) Fidaxomicin
a. Rifampin
45. What is goal?
a) C. Diff negative in 10 days.

46. What is a risk factor that you can modify?


a) Discontinue pantoprazole
b) Aerobic exercise

47. She comes back in a month with recurrent infection. Also, mild/moderate. What do you give?
a) Metronidazole 500 mg TID.
b) Vancomycin
c) Rifaxamin
A 1st CDI relapse will occur in 10–25% of patients and should be managed based on its severity by repeating
the first line of therapy used
Severity Criteria Treatment
Mild Diarrhea plus any additional signs or Metronidazole 500 mg TID PO ˣ 10 days
moderate symptoms not meeting severe or complicated If unable to take metronidazole, vancomycin
disease criteria 125 mg QID PO ˣ 10 days.
WBC ≤15 x 10^9/L and a serum creatinine If no improvement with metronidazole in 5 –
(SCr) level of <133 mcmol/L 7 days, consider changing to vancomycin
Fidaxomicin 200mg PO BID x 10 days
Severe disease Serum albumin < 3 g/dL + 1 of the following: Vancomycin 125 mg QID PO ˣ 10 days or;
WBC ≥15 ˣ 10^9/L & Abdominal tenderness Fidaxomicin 200mg PO BID x 10 days
Severe and Any of the following attributable to CDI: Vancomycin 500 mg QID PO +
complicated Admission to ICU for CDI Metronidazole 500 mg Q8H IV +
disease Hypotension ± required use of vasopressors Vancomycin 500 mg in 500 mL saline QID
“Fulminant” Fever ≥38.5 °C. Mental state changes PR as enema
Ileus or significant abdominal distension Surgical consultation suggested
WBC ≥35 ˣ 10^9/L or < 2 ˣ 10^9/L
Serum lactate levels >2.2 mmol/L
End organ failure, e.g., renal failure
Recurrent Recurrent CDI within 8 wk of completion of Repeat metronidazole or vancomycin pulse
CDI therapy regimen. Consider FMT Fecal microbiota
transplantation after 3 recurrences.

48. Osteomyletis case. The patient’s culture shows MRSA. Patient is already on Vancomycin.
What do you give?
a) Linezolid
b) Metronidazole
c) Vancomycin
Vancomycin: If MRSA
Clindamycin or Linezolid: Only if Vancomycin intolerant patient or resistant strain
49. How long do you treat minimum?
a) 3 months
b) 4 months
c) 6 weeks
Duration of therapy:
 4 to 6 weeks IV antibiotics
 6 weeks in more severe initial presentation, slow resolution of systemic and local signs
 Exception enterobacteriaceae and G ‐ve organisms: may be treated with oral FQ (6‐8 weeks)
 Switch to oral antibiotics once patient stabilized i.e. labs normalized, decreased pain, no complications
(e. g. no necrosis).
 Medication requires good oral absorption. Ensure oral dose can be tolerated (e.g. GI upset)
 If Diabetic foot: minimum ttt is 6 weeks

50. Case about an infant with cough, congestion, and fever of 38 degrees for past 3 days.
Mom wants a chewable tablet for fever. Where do you look
a) Compendium of selfcare products
b) other
The Compendium of Self‐Care Products (CSCP) is the nonprescription companion to Patient Self‐Care—
Helping Your Patients Make Therapeutic Choices (PSC). It is an exclusive collection of information about
nonprescription products including drugs and natural health products, home tests medical aids and medical
devices

51. What do you do?


a) Nasal saline drops
b) Refer to doctor
52. Patient with long QT syndrome and doctor needs an antibiotic for UTI. Where do you look?
a) Medline
b) Canadian Cardiovascular Society.
c) CredibleMeds
CredibleMeds is an online database of independent
information regarding the safe use of medicines.
The database, launched in 2009, helps to detect
serious drug‐drug interactions, especially those
associated with QT prolongation or the potentially
lethal arrhythmia, torsades de pointes (TdP).
It also assists with measurement of the quality of
healthcare delivery for the Centers for Medicare and
Medicaid Services, and aids in the management of
patients with inherited channelopathies.
The overall goal of CredibleMeds is to support
efforts to improve the safe use of medicines

53. CL is suffering from facial pain, nasal congestion, rhinorrhea for last few days & diagnosed
with bacterial sinusitis. He is also suffering from cough which is really bothersome at night. His
medication profile includes Ramipril for Hypertension. What could be the reason for her Cough?
a) Post nasal drip syndrome
b) Medication ramipril induced  No, as she suffers from cough at night only
c) Exhaustion from infection
d) Reactive airways
Post‐nasal drip describes the feeling of mucus secretions moving down the back of the throat, often causing
cough. It's normal to swallow some of the mucus that's made in your nose and sinuses without even realising
it. But if the mucus becomes thicker or if there is more mucus than usual, you may get post‐nasal drip.

54. What is the drug of choice for her condition?


a) Moxifloxacin
b) Azithromycin
c) Erythromycin
d) SMX/TMP
CTC: Macrolides, SMX/TMP, cephalexin, cefadroxil and cefixime are not recommended for empiric therapy
of rhinosinusitis due to unpredictable/poor activity against S. pneumoniae and/or H. influenzae.
If a patient has received antibiotics (especially macrolides or fluoroquinolones) within the past 3 months,
choose a different antibiotic/antibiotic class to treat acute bacterial rhinosinusitis as there is a higher risk
of multidrug‐resistant S. pneumoniae. Levofloxacin and moxifloxacin have good coverage of the pathogens
involved; however, fluoroquinolones should be reserved for patients with severe, documented allergy to
beta‐lactams or patients who have failed first‐line antibiotic therapy.
55. Comes back 2 weeks later and she still has symptoms, she said she has severe myalgia and
it’s been a week with fever. What do you do?
a) Lots of rest and fluids
b) Refer to get an x ray done
c) Refer to doctor for antibiotics
d) Refer to doctor for some other reason
If myalgia suspect flu then refers to doctor
In this case: fluids + rest would make sense although it is like an alternative for referral
Note: prolonged rest >3 days predisposes the risk of DVT (just a reminder)

56. Patient who is 70 years old and has an infection. I can’t remember what it was. In the
past 5 years she’s only had TDAP vaccine and Flu vaccine. What vaccine does she need?
a) Varicella
b) Pneumococcal
c) Meningitis shot
CTC: conjugated meningococcal vaccine is now part of routine childhood or adolescent immunization
programs in some provinces and territories in Canada 4CMenB is recommended for patients considered to
be at high risk, such as those with functional or anatomic asplenia, sickle cell disease or complement
deficiency, or close contacts of cases of meningitis B, and for outbreak management for a targeted strain. In
otherwise healthy individuals, including infants, decisions regarding administration of 4CMenB should be
made on an individual basis after assessment of risk

57. Patient with paget’s disease of the bone comes in with a written Rx for 30 mg risedronate
once daily. Pharmacist dispensed 5 mg once daily. Why did this happen?
a) Pharmacist didn’t question the dose.
b) Pharmacist didn’t double check the indication

58. Pharmacist discovers that this happened because while the technician was on lunch break,
the pharmacist entered, filled, and bagged the prescription. When the patient came for pick up,
the technician handed the medication to the patient without any further interventions:
a) Making it a policy to check with each patient the indication if it’s a new medication
b) Ask the doctor to start using computerized prescription generator.
c) Triaging system

59. You are a hospital pharmacist & need to decide which medication needs to be made urgently.
a) Pamidronate (hypercalcemia)
b) Ceftriaxone (infection)
c) Eye drops using vancomycin
60. You need to come up with a system to make sure urgent prescriptions aren’t missed.
a) Send someone to pick them up or delivered on a regular interval
b) Desk clerk calls you whenever they need something urgently
Another version
What can be done in a hospital pharmacy to streamline delivery of urgent product?
a) Technician will triage which prescription is more important
b) Develop a policy regarding urgent medication where a stuff will call first on telephone
c) Stock urgent medication in Ward for quickness of service
d) GO pick up prescription in a regular interval to reduce building up prescription

61. LY is 30 years old pregnant lady who is at the end of her 1st trimester. She is allergic to
penicillin and recently identified with UTI. What is the best choice for her?
a) Nitrofurantoin
b) Trimethoprim
c) SMX/TMP
d) Amoxicillin

62. In which of the following case Med effect does not monitor A/E or What side effects can’t
be found in Canada Vigilance database?
a) Investigational drug
b) Herbal Medication
c) Drug in Phase 4 trial
d) Special access program drug
e) Vitamins and minerals
f) NHPs
The following health products marketed in Canada are collected by the program: prescription and non ‐
prescription medications; natural health products; biologics (includes biotechnology products, vaccines,
fractionated blood products, human blood and blood components, human cells, tissues & organs);
radiopharmaceuticals; and disinfectants and sanitizers with disinfectant claims.

63. Cystic fibrosis. Mom wants you to be an active part of her kid’s cystic fibrosis. What info
do you request? Cystic fibrosis isn’t even in the effing CTC……….
a) Vaccinations
b) URTI history
c) Vitamins and minerals
d) Family history of lipid disorders
Since CF are prone to respiratory tract infections = a gift question 😊
64. Patient taking OxyNeo 30 mg BID and Percocet 4 times per day. Wants to switch to something
Else. 30 mg Morphine = 6 mg Hydromorphone, 30 mg Morphine = 20 mg Oxycodone.
a) 6 mg HM BID + 4 mg q6h PRN
b) 9 mg HM BID + 1 mg q6h PRN
c) 12 mg HM BID + 2 mg q6h PRN
d) 12 mg HM BID + 4 mg q6h PRN
Another version:
Patient is taking Oxycodone HCl 30 mg QID & Percocet (Oxycodone/Paracetamol) 6 mg PRN
to control her pain approximately 4 times a day. Now doctor want to change him to different
opioid as per bellow conversion chart. What will be the recommendation of pharmacist?
a) 10 mg Hydromorphone QID and 4.5 mg Q6H PRN
b) 17 mg Hydromorphone TID
c) 15 mg Hydromorphone QID
d) 16 mg Hydromorphone TID & 5 mg Q6H PRN

65. What is NOT related to PIPEDA (Personal Information Protection & Electronic Documents
Act) rules?
a) It has data base of people who
identify the thefts.
b) Patient information can not be
used for publicity
c) Don’t disclose personal
information to anyone
d) To disclose personal information
to third party
e) Use the personal information
to be used in the pharmacy

66. What can be prescribed for lactating mother to increase lactation?


a) Domperidone
b) Prochlorperazine
c) Metoclopramide
d) Verapamil
CTC: Some mothers may not produce enough milk to maintain their baby's needs. A breastfeeding expert
should assess the problem and initiate counselling, relaxation techniques or mechanical expression.
If nonpharmacologic therapy is unsuccessful, treatment with domperidone 30 mg/day is an option. By
blocking D2 and D3 dopamine receptors in the pituitary gland, domperidone increases prolactin levels and
breast milk production. Fenugreek is a commonly recommended natural health product to increase breast
milk supply, although evidence of its efficacy is Minimal
67. A female patient came to the pharmacy with the following Medication profile; HCTZ for
Hypertension, Valproic acid for Seizure and pain, Short term NSAID general pain. Now,
Patient is pregnant and dr prescribed Diclectin for nausea and vomiting, but she is complaining
that nausea is still bothersome even after she is taking 1 or 2 pills regularly. What is the DTP?
a) Taking too little of the right drug
b) Drug – drug interaction
c) Not getting appropriate medication
d) Drug – disease interaction
CTC: Recommended Dose and Dosage Adjustment
 Two (2) Diclectin delayed release tablets at bedtime to control nausea and vomiting occurring in the
morning; additionally, one (1) delayed release tablet in the morning and one (1) delayed release
tablet mid‐afternoon to control symptoms throughout the day. The dosage schedule may be
individualized according to timing, duration, severity and frequency of the symptoms experienced by
the patient. Diclectin can be prescribed in any trimester of pregnancy.
 Diclectin is a delayed‐release formulation that works optimally when given 4 to 6 hours prior to
anticipated onset of symptoms. The delay in action may be prolonged when tablets are taken with food.
However, based on the available data, the above recommended dosage schedule should be followed
 Diclectin tablets being of a delayed release formulation should not be prescribed on an as needed
basis (prn). It is important that Diclectin is taken daily for optimal effect.
 A gradual tapering dose of Diclectin is recommended at the time of discontinuation to prevent a
sudden onset of symptoms

68. For the above patient, all are appropriate actions to suggest except
a) Add dimenhydrinate with Diclactin
b) Add diphenhydramine
c) Change HCTZ to Ramipril
d) Change Valproic acid to Carbamazepine
e) Switch NSAID to Acetaminophen

69. Which cascade in the elderly makes most sense?


a) Tolterodine − confusion − risperidone
b) Donepezil − constipation – senokot
c) Roseglitazone  edema  furosemide
d) Hydrochlorothiazide- Hyponatremia-Seizure
e) Mirtazapine- insomnia- Diazepine
Prescribing cascade is defined as the situation in which a first drug administered to a patient causes adverse
event signs and symptoms, that are misinterpreted as a new condition, resulting in a new medication being
prescribed. The cascade may have multiple steps and differ in complexity and severity.
70. What side effect does dabigatran cause?
a) Dyspepsia
b) Renal dysfunction
c) Myositis
d) Vomiting
S.E: Bleeding, dyspepsia. Also, Epistaxis is defined as bleeding from the nostril, nasal cavity, or nasopharynx.
Must not crush or chew. Antidote: Idurucizumab

71. What is the most possible cause of giving prophylaxis for Endocarditis when patient
is undergoing dental surgery?
a) Cardiomyopathy
b) Mitral valve stenosis
c) Prosthesis
d) Congenital Valvular defect
Cardiac Conditions Associated with the Highest Risk of Adverse Outcome from Endocarditis for which
Prophylaxis with Dental Procedures is Reasonable
‐Prosthetic cardiac valve or prosthetic material used for cardiac valve repair
‐Previous infective endocarditis
‐Congenital heart disease (CHD)
1) Unrepaired cyanotic CHD, including palliative shunts and conduits
2) Completely repaired congenital heart defect with prosthetic material or device, whether placed by
surgery or by catheter intervention, during the first 6 months after the procedure
3) Repaired CHD with residual defects at the site or adjacent to the site of a prosthetic patch or
prosthetic device
‐Cardiac transplantation recipients who develop cardiac valvulopathy

72. Patient with VTE prophylaxis. Medical team is assessing her. What ethical thing are
they upholding?
a) Paternalism
b) Veracity
c) Non maleficience

73. 70-year-old being injected for vaccine. She is scared. What do you tell her?
a) Reinforce that it will protect other people
b) Tell her you’re available to answer any question
c) Tell her that she has to take the vaccine as recommended by the physician for her safety.
d) Tell her that she has to receive the vaccine to protect the public
e) Inform her about benefits and risks of the vaccine
f) Do not give her the vaccine
74. Where do you inject if not enough muscle mass in elderly or What is the alternative site for
IM injection in a Male patient with low muscle mass?
a) 45-degree angle deltoid
b) 90-degree anterolateral thigh
c) 90-degree buttock

75. All are UTI symptoms except


a) Vertebrocostal angle pain -------------------
b) Suprapubic pain
c) Urgency
d) Hematiurea
e) Dysurea

76. Patient got a severe allergic reaction and inject one dose of EpiPen. 911 has been called for
him. His condition is improved but he is experiencing mild rash. What to do in such situation?
a) Give another injection within 5 minutes
b) Give 50 mg oral dimenhydrinate plus go to hospital in all cases
c) Give topical dimenhydrinate cream
d) Give 2 more dose of EpiPen 5 min interval

77. Which of the fallowing are false regarding EPI injection?


a) Take off clothes before injection
b) There is some residual after injection
c) Pull back the plunger until you see blood
d) If symptoms are still there after 10 min, inject anothr epipen.
Epinephrine is effective after 1 injection. However, symptoms may recur and further injections may be
required. Allergic reaction is a serious, acute, allergic reaction that may cause death. It has a sudden onset
and generally lasts less than 24 hours. Because a severe allergic reaction is a generalized reaction, a wide
variety of clinical signs and symptoms may be observed. Physicians may determine that a single dose of
epinephrine may not be enough to reverse the symptoms of a severe allergic reaction. As such, your
healthcare provider may prescribe more than one EpiPen® to have on hand.
Epinephrine can be re‐injected every 5 to 15 minutes until there is resolution of the severe allergic reaction
or signs of adrenaline excess (such as palpitations, tremor, uncomfortable apprehension, and anxiety).
After receiving epinephrine, you must be transported to hospital, for evaluation and a period of observation
of no less than 4 hours. This is because of the possibility of either a “biphasic” reaction (a second reaction)
or a prolonged reaction. The attending physician will consider such factors as the severity of the reaction,
your history and response, and the distance from the hospital to your home. Severe allergic reactions
typically follow a uniphasic course; however, 20% will be biphasic in nature. The second phase usually
occurs after an asymptomatic period of 1 to 8 hours, but may occur up to 38 hours (mean 10 hours) after the
initial reaction. About 1/3 of the second‐phase reactions are more severe, 1/3 are as severe, and 1/3 are
less severe. The second‐phase reactions can occur even following administration of corticosteroids. After
administration, you should seek medical attention immediately or go to the emergency room. For the next
48 hours, you must stay within close proximity to a healthcare facility or where you can call 911. Protracted
anaphylaxis, which is frequently associated with profound hypotension and sometimes lasts longer than 24
hours, is minimally responsive to aggressive therapy, and has a poor prognosis.

78. Epinephrine injection site


a) IM 90o lateral arm
b) IM 90o lateral thigh
c) SC 45o lateral arm
d) SC 45o lateral thigh
Epipen:
 Depends on the weight. IM Shot maybe through clothes
 Only one shot & not all the dose will come out
 After the shot must go to emergency as you might get another attack
 CTC: In most patients, epinephrine is effective after 1 injection. However, symptoms may recur and
further injections may be required to control the reaction. Epinephrine can be re‐injected every 5 to
15 minutes until there is resolution of the anaphylaxis or signs of adrenaline excess (such as
palpitations, tremor, uncomfortable apprehension and anxiety). EpiPen and EpiPen Jr are intended
for intramuscular use in the anterolateral aspect of the thigh, through clothing if necessary. Do not
inject into the buttock

79. Anticoag used with CrCl 60


a) Warfarin
b) Rivaroxaban
c) ASA
Dabigatran (dose adjustment if >80 years)
Rivaroxaban (dose adjustment for CrCl 30‐50, AVOID in CrCL <30)
Apixaban (dose adjust if > 80 yrs or weight < 60kg and AVOID in CrCL < 15)

80. Bar code reader system is installed in the pharmacy which scan product after filling the
prescription. what is the possible benefit of such measure?
a) Identify proper pack size OR Helps make sure you have the correct pack size
b) Reduce dosing error
c) Reduce Strength error OR Helps to make sure you have the correct strength
A DIN uniquely identifies the following product characteristics: manufacturer; product name; active
ingredient(s); strength(s) of active ingredient(s); pharmaceutical form; route of administration.
UPC: Universal Product Code: Consists of 12 numeric digits that are uniquely assigned to each trade item to
identify pack size of the same strength medication in a similar dosage form e.g., 100 versus 500 pill pack.
81. A list of controlled and narcotic drug, what can you do if they are all expired?
a) Send back controlled for money
b) Send back narcotics for money
c) Destroy them. Can’t send them back for refund

82. Which of the following doesn't require to send to OCS before destruction?
a) Tramadol
b) Oxycodone
c) Nabilone

83. 40-year-old pt has


sebaceous thing on his scalp
its scaly but not silvery. What
do you give?
a) Ketoconazole
b) Shampoo with
salicyclic acid
c) Shampoo with tar
d) Refer for other
treatment
Dandruff (Malassezia yeast) is a noninflammatory form with increased desquamation.
Dry, white scales scattered diffusely over scalp

84. Patient in hospital is taking IV Cotrimoxazole (each ml contains 80 mg SMX / 16mg TMP)
0.5 ml Q6hr. Dr. want to shift him to liquid (200 mg SMX / 40 mg TMP) in each 5 ml, how
many of liquid is needed to provide the equivalent dose as was taken IV.
Answer:
0.5 ml Q6hr = 0.5 * 4 = 2 ml /Day
SMT: 80 mg ------- 1 ml X mg ------- 2 ml X= 2*80/1 = 160 mg
Liquid 200 mg ------- 5 ml 160 mg ------- Y ml Y = 160*5/200 = 4 ml/Day

85. Patient taking oral contraceptives and has gout. She is experiencing acne problems. She was
not using any medication for her acne since last year. Now she has many papules & pustules
and pustules are erupting. What could aggravation of ACNE?
a) Levonorgestrel/Ethinyl Estradiol
b) NSAID
c) Allopurinol
d) Gender
86. What is your recommendation for her situation?
a) Benzoyl peroxide
b) Recommend her to doctor for oral antibiotic
c) Use face scrub for her acne
She has many papules & pustules and pustules are erupting.

87. A question about publication bias. What is the example of publication bias?
a) Primary literature fulfilling criteria of good studies
b) Peer reviewed journal as a good source
c) Ranking primary resource of Renowned Journals as superior
Publication bias is a type of bias that occurs in published academic research.
It occurs when the outcome of an experiment or research study influences the decision whether to publish or
otherwise distribute it. Putting a specific study first and rank it for viewers to read it first

88. What do you not monitor for liver dysfunction?


a) INR
b) GGT
c) AST
d) CRP
AST: To diagnose hepatocellular disease and assess disease progression (main indicator)
ALT: relatively lower than AST in persons with alcoholism
89. Pregnant lady work as a hospital pharmacist and dealing with cyctoxic drugs, what is right?
a) Use personal protective equipment with gloves
b) Double up on the gloves plus PPE
c) Change her room
Double gloving (wearing of inner & outer gloves) not recommended for routine use but may indicated for:
a) Some surgical procedures, e.g., orthopaedics, maxillofacial surgery
b) Chemotherapy and biotherapy administration, safe handling and disposal
c) Protocols for specific diseases, e.g., viral hemorrhagic fever.
d) Environments where gloves may be damaged or grossly contaminated during critical patient care
events, e.g., motor vehicle accident.

90. What electrolye disturbance with PPI?


a) Hypomagnesemia
b) Hypercalcemia

91. What is true about glucagon?


a) Can be used for beta blocker toxicity
b) Indicated for emergency treatment of severe hypoglycemia
c) Active when taken orally or parenterally
d) Induce acetylcholine
e) Improve appetite
GLUCAGON is a polypeptide hormone identical to human glucagon which is manufactured by recombinant
DNA technology and has the same molecular structure as animal sourced glucagon. Glucagon causes an
increase in blood glucose concentration. Glucagon acts only on liver glycogen, converting it to glucose..
Glucagon is inactive orally because it is destroyed in the GIT before it can be absorbed.
GLUCAGON (glucagon for injection, rDNA origin) should be given only if patients are unconscious or
unresponsive and unable to ingest oral glucose. After intramuscular injection, the patient will normally
respond within 10 minutes. If the patient does not respond within 10 minutes, intravenous glucose must be
administered as soon as an IV access can be established. Because glucagon is of little or no help in states of
starvation, adrenal insufficiency, or chronic hypoglycemia, intravenous glucose should be used for treatment
of hypoglycemia in those conditions. Can’t give in adrenal /pancreatic tumor

92. Patient stepped on nail and got osteomyelitis. What


lab will be different compared to healthy people?
a) Change in X-ray
b) Erthrocyte sedimentation rate
CBC and acute‐phase reactants (erythrocyte sedimentation
rate [ESR], C‐reactive protein [CRP]) as baseline, blood
cultures (positive for bacteria in 25–50% of cases).
X‐ray Not sufficiently sensitive to rule out osteomyelitis, but
important test to assess other etiologies of limb pain
93. Same patient, Culture show that he had gram positive cocci clusters, what caused his
osteomyletis?
a) S aureus
b) Streptococcus pyogens
c) P aeruginosa
Streptococcus Gram-positive cocci in chains (when cultivated in liquid media)
pyogenes Nonmotile, non-spore-forming, often encapsulated (capsule composed of hyaluronic acid)
S. aureus Gram-positive cocci in grape-like clusters. Nonmotile, non-spore-forming
Pseudomonas Gram-negative, rod-shaped, asporogenous, and monoflagellated bacterium that has an
aeruginosa incredible nutritional versatility. It is a rod about 1-5 µm long and 0.5-1.0 µm wide.

94. If he had penicillin allergy, which is the proper treatment without wound Debridement?
a) Cefazolin 10 – 14 days
b) Cefazolin 4 – 6 weeks
c) Gentamycin 10 – 14 days
d) Cefuroxime 4 – 6 weeks
These infections are monomicrobial; empiric coverage should include activity against S. aureus. Duration of
antibacterial therapy should be minimum of 4 ‐ 6 weeks (especially for vertebral osteomyelitis). If there is
adequate debridement, 10‐14 days treatment is often adequate.
In Neonates: In Children: In Adults:
Caused by: Caused by: Caused by:
S. Aureus S. aureus (MSSA & MRSA) S. aureus.
Group B streptococci Group A streptococci. Gram -ve enterics.
Gram negative enterics Kingella kingae (children <3 y) Sickle cell disease.
Empiric IV Antibacterials: Rare: H. influenzae, S. pneumoniae, Salmonella spp.
MRSA unlikely, cloxacillin gram -ve enterics, K. kingae.
+ cefotaxime (to cover gram Empiric IV Antibacterials:
negative enteric bacilli).  MRSA unlikely, cloxacillin or cefazolin.
MRSA possible,  MRSA possible, vancomycin.
vancomycin + cefotaxime Replace cefazolin with cefotaxime if not immunized against H.
or ceftriaxone. influenzae

95. Patient is prescribed with Sinemet tablet. Sig: 1.5 tablet 5 times for 12 weeks. Patient told
pharmacist that he will no be able to divide tablet into half. Now pharmacist will divide the
tablet for him. How many tablets needed to divide?
a) 250
b) 230
c) 210
d) 300
84 (12 WK) x 5 x 1 = 420 tabs x 0.5 = 210 OR 2.5 TAB * 84 DAYS = 210 TABS
96. What does not need shake well label?
a) Nitroglycerin SL spray pump
b) Clarithromycin suspension
c) Metered dose Inhaler
d) Nystatin suspension
Can be used until expiry, do not shake, need priming in 1st time or if not used for 14 days.

97. Some old lady and has trouble with her meds at home. All are options to do except?
a) Give her easy open lids
b) Give her blister pack

98. Case about gout. Allopurinol can be used in renal dysfunction: decrease maintenance dose
to 100 mg/day if ClCr is 10–20 mL/min; 100 mg Q2–3 days if ClCr <10 mL/min
CTC: Indications for the use of urate‐lowering therapy include 1. established diagnosis of gouty arthritis
along with either 2. tophus or tophi by clinical exam, 3. ≥2 attacks per year, 4. chronic kidney disease stage 2
or worse (GFR ≤89 mL/min/1.73 m2), or past urolithiasis
Renal impairment: decrease maintenance dose to 100 mg/day if ClCr is 10–20 mL/min; 100 mg Q2–3
days if ClCr <10 mL/min

99. Methadone, what does it interact with?


a) Quetiapine
b) Zopiclone
c) Omeprazole
Lexicomp: QT‐prolonging Agents (Highest Risk) may enhance the QTc‐prolonging effect of QUEtiapine.

100. Case about parkinsons. What can exacerbate it? Metoclopramide

101. What natural thing can slow progession?


a) CO Q-10
b) Exercise
c) High protein diet
Stress importance of staying active and having a regular exercise routine. To improve motor and nonmotor
symptoms, exercise program should include aerobic, strength, balance and gait training e.g., dance, tai chi, yoga.
Provide patient education via books, websites and local and national Parkinson societies (all available
through Parkinson Canada). Encourage awareness of the important roles of allied health professionals such
as speech, physical and occupational therapists, and home care as the disease becomes more advanced

102. Patient calculate their CrCl and they give you chart with dose of antivirals. Choose right
dose for patient with herpes zoster.

103. Trial use per protocol and didn’t use intention to treat. What is the problem?
a) Low power to detect differences between treatments
Intent to treat (ITT) analysis
 Means all patients who were enrolled and randomly allocated to treatment are included in analysis and
are analyzed in the groups to which they were randomized. “Once randomized always analyzed”
 It is a statistical approach used for RCTs that taken into consideration all subjects in treatment and
control group including any non-adherence to the study protocol (e.g. taking/administering treatment by
the patient, dropout or protocol deviation). It ignores noncompliance, withdrawal, protocol deviation, or
anything happened after randomization.
 Advantages: Avoid overestimation of drug effect, maintain sample size, reduce type I error & more
consistent with clinical practice.
 Disadvantages: susceptible to type II error & dilution of the drug effect.
Non-intent to treat analysis (Non-ITT): Aim to estimate the effect of treatment
as delivered or as received (as opposed to assigned) to account for non-adherence.
Can be used as interim.
A per-protocol analysis: Opposite end of the spectrum from ITT analysis. It is an
interpretation of randomized clinical trial results that removes data from patients
who didn't comply with the protocol. Imagine a trial designed to test experimental
drug A against standard treatment B. If some patients drop out of the trial before
investigators can measure the primary outcome, a per-protocol analysis wouldn't
include their results with patients who completed treatment. Its results represent
the best-case treatment results that could be achi

104. MD wants to know if a drug is marketed for sale in Canada yet. Where to check?
a) Health Canada drug database

105. Mom asking info about prevnar for child.


Prevnar 13® is a vaccine indicated in children 6 weeks through 17 years (prior to the 18th birthday) for active
immunization for the prevention of invasive disease caused by Streptococcus pneumoniae (AOM) and for
children 6 weeks through 5 years of age (prior to the 6th birthday) for the prevention of otitis media caused
by 7 of the 13 serotypes in the vaccine
Number of doses spaced apart dependent on age e.g. 2 ‐ 6 m, 7 ‐ 11m, 12 ‐ 23m, etc.
In adults 18 years of age and older, Prevnar 13® is indicated for active immunization for the prevention of
pneumonia and invasive disease caused by S. pneumoniae serotypes in the vaccine
Prevnar 13® does not protect against disease caused by S. pneumoniae serotypes that are not in the vaccine
106. A mother is afraid from vaccinating her child because of needle pain, what should you tell
her before injecting him?
a) Give diclofenac 12.5 supp.
b) Breastfeed him before and after vaccination (before, during, after vaccination)
c) Lay child supine before giving vaccine (hold you baby)
d) Cold compresses
e) Use topical anesthetic Lidocaine 10 -20 min before (20 to 60 minutes)
f) Give Acetaminophen 10-15 mg kg 30 min before vaccination
g) Tell him it is not painful
h) Tell nurse to hold him while he is in supine position

107. Something about vaccine infoemation. Public health units


Public Health Units
 Public health units must conduct cold chain incident and annual routine inspections.
 Contact your public health unit for assistance with vaccine storage and handling practices.
 Public health units are required by the Vaccine Storage and Handling Protocol under the Ontario
Public Health Standards, issued under the authority of the Health Protection and Promotion Act to
respond to reports of all cold chain incidents and to inspect premises, at least once annually, where
provincially funded vaccines are stored.
 The purpose of cold chain incident and routine (annual) inspections is to: Ensure the proper
management of vaccine inventories; Provide education strategies to minimize vaccine wastage;
Reduce provincially funded vaccine wastage; and Promote vaccine safety and effectiveness.
108. Lady with influenza. Had symptoms for a week. Comes in and asks what to do.
Tylenol is not working for her.
a) Refer to doctor
CTMA: See a health‐care provider if any of these things happen:
 You have trouble breathing or make strange sounds when you breathe
 Your throat is very sore
 You have a lung disease such as asthma, emphysema or chronic bronchitis
 You have a fever for more than 24 hours
 Your cold or flu lasts for more than 7–10 days

109. Comes back 2 days later and said MD told her she has viral influenza. What is true?
a) She is passed the point of contagion
b) Needs frequent hand washing
CTMA: How can you prevent colds and flus? There is no sure way to prevent colds and flus. A yearly flu shot
will help to cut your chance of getting the flu. You can also help protect yourself by washing your hands
often. Clean hands help to prevent colds and flus from spreading.
Always wash your hands: Before cooking or eating, before feeding a baby or child, before giving someone
medication & After wiping your nose
Follow these steps for proper handwashing:
 Wet your hands under running water.
 Using soap, scrub your hands for 20 seconds (the time it takes to sing Twinkle Twinkle Little Star).
 Rinse your hands under running water for 10 seconds.
 Dry your hands with a clean towel.
Or use Hand sanitizer, cough in tissue or sleeve, stay home if symptoms are present
110. What is the mechanism of action of plan B?
a) Toxicity to Sperm
b) Inhibition of endometrial Implantation of egg
c) Alter motility of Fertilized egg
d) Inhibition of fallopian tube implantation

111. What is the most likely reason plan B will not work for a patient?
a) Weight of the patient 80 Kg
b) Antibiotic use
c) Smoking

112. What is drug interaction?


a) Fenofibrate−omeprazole
b) Clarithromycin−lisinopril
c) Metronidazole−antacid
Omeprazole is a CYP 2C19 substrate. Coadministration with fenofibrate may increase the plasma
concentrations and risk of adverse effects of drugs that are substrates of CYP450 2C19, 2A6, and 2C9
isoenzymes. The proposed mechanism, based on in vitro data, is decreased clearance due to fenofibrate‐
mediated inhibition of CYP450 2C19, 2A6, and 2C9.

113. Something about issue with vaccines in a particular area. What could have caused this?
a) Cold chain got messed up
114. Which rx is written correctly?
a) Zopiclone 7.5mg po nightly prn
b) Other options had OD and IU in them

115. Proper tall man lettering:


predniSONE and predniSOLONE

116. What is glatiramer used for?


a) Multiple Sclerosis
b) AMD
c) Systemic Lupus Erythematous
d) Neuropathic pain
e) Giant cell Arteritis

117. ISMP list of drugs dangerous to adults Hydromophine, doxorubicin and some other drug

118. What do you do during the honeymoon phase? Reduce dose of insulin

119. TC is a female patient gave birth 4 weeks ago and wants recommendation form pharmacist
about best contraceptive method, she can use. What will be your recommendation as a pharmacist?
a) DMPA
b) Levonorgestrel IUD
c) COC
At 4 weeks if not breastfeeding then the choice is COC. At 6 weeks: if breastfeeding: 1ST line IUS and 2nd line
DMPA. Immediately/breastfeeding: IUS, Mini pill POP, Barriers method and spermicide

120. She is breastfeeding and needs med for milk supply but baby is feeding fine, lots of wet
diapers and regained weight. What is the appropriate action?
a) No need for a med
b) Give domperidone
c) Metoclopramide

121. What needs auxillary label take with or after meals?


a) Carbamazepine
b) Cloxacillin  Take 1 to 2 hrs brefore food to ↑ absorption
Carbamazepine should be taken with meals to lessen unwanted effects (eg, stomach upset, nausea,
vomiting). Carbamazepine extended‐release capsules do not need to be taken with meals unless they upset
your stomach. CTC: Ingestion of food has no significant influence on the rate and extent of absorption
regardless of the dosage form of TEGRETOL: Food might decrease GI Effect
122. Patient needs antipsychotic but has diabetes and insomnia and poor mood. What to give?
a) Quetiapine
b) Olanzapine
c) Risperidone
d) Aripiprazole  this ok but can cause insomnia

123. Something about where to check for an ADR? Medeffect

124. Rules for getting narcotic from another pharmacy?


a) Needs a written order and record kept

125. Consensus trial tests enalapril against placebo for heart


failure and measured CV mortality after 6 months in 243
patients. Enalapril group had 33 events, placebo had 55. What was ARR/RRR?
ARR = EER – CER = 55 – 33 = 22 RRR = CER – EER / CER = 22 / 33 = 0.7

126. Phamacy revenue decreased so manger asks pharmacists to do MED check for patients.
This patient has asthma and well controlled, she has 3 inhalers so she qualifies for meds check,
but has no DRP and did med check 2 months ago but because she is taking 3 medications you
have to do MED check to be paid by GOVT, what is the problem?
a) Paternalism
b) Beneficience
c) Professional integrity
d) Conflict of interest

127. Patient taking med from other doctor and doesn’t want family MD to know because family
MD doesn’t want him seeing other MDs. What are you upholding by not telling family MD?
a) Confidentiality
b) Autonomy

128. Patient has tried nonpharm for diabetes but her labs still missed but she still wants to try
diet and shit. What shows beneficence?
a) Tell her the info but in the end, it is her choice
b) Tell her she can try but make agreement to try pharm measures if it fails again

129. Patient is having thoughts of suicide and harm to self that have been getting worse. What
do you do to show beneficence?
a) Tell them that you will tell their MD because it is important
b) Tell them that this is problem and that they should tell their MD but leave it up to them
130. Tobramycin IV 400mg/24h was given to a patient. Peak concentration is required is
>20mg and trough concentration required is <0.5 mg. Post dose peak is 28 mg/L and after 10
hours is 7 mg/L. Find the T1/2 of the drug.
a) 5 hours
b) 9 hours
c) 10 hours
d) 15 hours
Solution:
ln C – ln Co - Kt ln 28 – ln 7 = K x 10
K = ln 28 – ln 7 / 10 = 0.139 T1/2 = 0.693 / 0.139 = 5hrs

131. In the above patient what is the correct dose?


a) 400 mg/24 hours
b) 400 mg/36 hours
c) 400 mg/48 hours
d) 400 mg/72 hours
400 mg / 24 hr ‐‐‐‐‐‐‐ 28 mg/ L X mg / 24 hr ‐‐‐‐‐‐‐ 20 mg / L X = 20*400 / 28 = 285.7 mg
285.7 mg ‐‐‐‐‐‐‐ 24 hr 400 mg ‐‐‐‐‐‐‐ Y Y = 400*24/285.7 = 33.6 hr.

132. Parents come to pharmacy informing that, they found some live lice and nits on their
child’s long hair. They heard there is an lice infestation in the school. 7 years old boy is allergic
to Chrysanthemum. What is the best treatment for him?
a) Permethrin 1%
b) Pyrethrin
c) Isopropyl myristate/Cyclomethicone
d) Crotamin 10%
133. after a period of time they came and said
that there is no improvement. What may be the
reason it is not working for the girl?
a) Not soaked the hair well enough
b) The girl did not leave it in for along
period of time 10
c) Didn’t use enough quantity

134. What is the next TX of choice?


a) Vinegar soaking
b) Tea tree oil
c) Pyrethrin shampoo
d) Hair comb to remove them
On its own, tea tree oil was the most effective treatment tested. Tea tree oil and peppermint appeared to be
most useful for repelling lice. Tea tree oil and lavender were also found to prevent some feeding by lice on
treated skin. Tea tree oil: apply to infested hair for 30 to 40 minutes

135. Mom is pregnant and got lice from her daughter, what to give?
a) Premethrin
b) Lindane
c) Dimethicone

136. Patient came back with drugs he didn’t use. Wants refund. What can you do?
a) Can give refund but can’t use returned meds
b) It is illegal to give refunds or credit
c) Take the medications and give store credit
d) Give a refund
e) Take the medications for re-sell

137. What drug can you use for Extended-


Spectrum Beta-Lactamase-ESBL-
Producing E. coli?
a) Carbapenem IV
b) Ciprofloxacin  If UTI
c) Nitrofurantoin

138. Patient came back with expired insulin. What can you do to prevent?
a) Check expiry before dispensing
b) Put in coloured labeled bins
139. What is the benefit having all pharmacy connected to know what was dispensed for who?
Prevent polypharmacy with elderly

140. Patient has pain when nurse comes to change his dressings. Is already taking regular
hydromorphone. What to give?
a) IM morphine
b) IM ketorolac
c) IR hydromorphone

141. Who to not worry about when giving pseudophedrine?


a) Hypothyroidism
b) Diabetes
Use with caution in CVD, diabetes, hyperthyroidism, prostatic hypertrophy and angle‐closure glaucoma.

142. What is not true about amiodarone? Need to avoid taking with acetaminophen

143. Patient has heart burn. Taking antacids but not providing relief long enough. Wants
something that will last 12 hours.
a) Give ranitidine OTC

144. Smoker bronchitis. Prescribed salbutamol and hycodan. What is DTP?


a) Smoking will decrease salbutamol effectiveness
b) Salbutamol has no indication
c) Hycodan not indicated
Each HYCODAN tablet or teaspoonful (5 mL) contains: Hydrocodone Bitartrate, USP 5 mg. Homatropine
Methylbromide, USP 1.5 mg. HYCODAN tablets also contain: calcium phosphate dibasic, colloidal silicon
dioxide, lactose, magnesium stearate, starch and stearic acid.

145. Some lady has atopic dermatitis and her infant in 13 months old. Wants to stop
breastfeeding. What formula to give? No effing clue what this one was. 13-month-old still
breastfeeding
a) Iron-fortified infant formulas
b) Hydrolized cow-based milk
c) Soy milk (hipster baby)
CTMA: Infants who cannot exclusively breastfeed and who are at high risk of allergy and atopic disease
should be given extensively hydrolyzed protein formula, which may delay or prevent occurrence

146. HPV VACC – when to vac girls?


a) Gardasil Females 9–45 y and males 9–26 y
Theresa and her teenage daughter, Becca, come in to discuss HPV vaccination because Becca has received a
letter from the school, where the vaccine will be offered. Both mother & daughter are hesitant. Theresa expresses
fear due to anecdotes she has read online about serious HPV vaccine side effects: “I know there is a chance of an
autoimmune reaction and that’s why it has been permanently banned in Japan.” Becca has heard from girls in the
older grade that the vaccine is painful. What is the best response to the mother?
a) Tell her the reports online are ridiculous, she should ignore them
b) Acknowledge that her daughter’s safety is of the utmost importance; tell her the rate of autoimmune
disorders following HPV vaccination were very low in clinical studies, with rates in the treatment group
no higher than placebo with total incidence below 3%
c) Explain the status of HPV vaccination in Japan and why those decisions were made there
d) Acknowledge that her daughter’s safety is of the utmost importance; explain that vaccine safety in
Canada is among the most rigorous in the world, and that the HPV vaccines have been thoroughly
tested and shown to be safe

147. Amitriptyline S. Es? Does not cause myosis (pupil constrict)


Amitriptyline causes mydriasis, seizures, blood pressure, and heart rate increase.

148. Sinusitis with penicillin allergy


Children Adult
Clinical Acute Rhinosinusitis (symptoms < 4 wk and ≤ 3 episodes/y)
Presentation Refer to ENT specialist if ≥4 episodes/y
Usual Pathogens Strep. Pneumoniae, M. catarrhalis, H. influenzae, Occasionally S. aureus, S. pyogenes,
Recommended First-line: First-line:
Empiric Therapy Standard-dose amoxicillin × 10 days or Amoxicillin × 5–7 days
high-dose amoxicillin × 10 days
Penicillin allergy: B-lactam allergy:
≤8 y: clindamycin + cefixime × 10 days Doxycycline 5–7 days
>8 y: doxycycline × 10 days
Severe infection or immunocompromised: Severe infection or immunocompromised:
amoxicillin/clavulanate (7:1) + amoxicillin/clavulanate (7:1) ± amoxicillin
amoxicillin × 10 days × 5–7 days
Nonsevere beta-lactam allergy: Nonsevere beta-lactam allergy:
ceftriaxone × 10 days Ceftriaxone × 5–7 days
Severe beta-lactam allergy / anaphylaxis: Severe b-lactam allergy/anaphylaxis:
levofloxacin × 10 days levofloxacin × 5 days
Failure of First Line Amoxicillin/clavulanate (7:1) ± Amoxicillin/clavulanate ± amoxicillin × 5–
Agents: amoxicillin × 10 days 10 days
(No improvement after Nonsevere beta-lactam allergy: Beta-lactam allergy:
7 days of antibiotic clindamycin + cefixime × 10 days or Levofloxacin or moxifloxacin × 5-10 days
therapy or clinical ceftriaxone × 10 days
deterioration or Severe beta-lactam allergy/anaphylaxis:
recurrence within 3 m)
levofloxacin × 10 days
Comments Consider resistant organisms, especially penicillin-resistant S. pneumoniae and
ampicillin-resistant H. influenzae
149. Palliative care with ulcer

150. Pneumonia outpatient,


Improvement can be detected by
all except
a) Fever
b) Spirometer
c) Level of conciousness
d) Chest X ray

151. pt diagnosed with Bacterial Vaginosis, what should be done?


a) Refer as it sexually transmitted disease
b) Do not refer, because it’s not a sexually transmitted disease
c) Report to health authority as it sexually transmitted disease
d) Refer although it is not sexually transmitted disease

152. Lady 32 married on EHS –dival

153. Pt has agitation, ppl watching while sleeping

154. Sotalol interaction with clarithromycin? Fatal QT prolongation X: Avoid combination


Lexi: QT‐prolonging Agents (Highest Risk) may enhance the QTc‐prolonging effect of Clarithromycin.

155. Female patient is pregnant in her first trimester suffering from severe nausea & vomiting
that she can't go to her work telling you that her doctor has already prescribed diclectin for her
she takes it as one tablet at night but still wakes up with severe nausea and vomiting in the
morning. What to tell her?
a) She's taking inappropriate medication
b) She needs to contact her doctor
c) She's taking too little of the right medication
Diclectin taken up to 4 tablets daily. 2‐tab QHS PO + 1‐tab QAM PO + 1‐tab mid afternoon PO.

156. LZ is 48 years old lady who is very concerned about her bone health. Her mother was
identified with osteoporosis few yeas ago and she is suffering a great deal since then. Based on
Which T score she will be identified as osteoporosis patient?
a) -1
b) -2.5
c) -3.5
d) -2.6
157. Patient came to pharmacy complains from white lesion in his mouth with 7 mm depth, it is
painful from 3 days, he has difficulty while eating but not severe. He is taking azathioprine.
Asking you what to do?
a) Refer to doctor
b) Do not use any medication. It will be healed
c) Give him topical anesthetic to
relief pain
d) Give him antiviral

158. RA - what do you monitor before


hydroxychloroquine?
a) Ophthalmologic
b) CBC
c) ESR

159. Meningitis vacc


Meningococcal type B conjugate vaccine (4CmenB) Local reactions:
Contains 4 antigenic components of serogroup B N. meningitidis. pain, erythema,
No data for infants <2 months or pregnant or breastfeeding women. induration.
Infants 2–5 months: 0.5 mL IM at 2, 4 and 6 months of age. Booster dose should be given Systemic reactions:
between 12–23 months of age. malaise, fever,
Unvaccinated infants 6–11 months: 3 doses (0.5 mL each) with an interval of ≥2 months irritability,
between the first and second dose. Third dose to be administered in second year of life ≥2 headache.
months after second dose Incidence of fever
Unvaccinated individuals 12 months–17 y: 2 doses (0.5 mL each) with an interval ≥2 months higher when used
between doses. The need for a booster dose has not been established with other vaccines
Meningococcal group B bivalent recombinant vaccine (rLP2086) Trumenba, contains 2 Increased risk of
antigenic components of serogroup B: rLP2086 subfamily A and B N. meningitidis hemolysis or low
Children >10 y and adults: 2 doses (0.5 mL each) with interval of 6 months between doses hemoglobin when
Individuals at increased risk of invasive meningococcal disease (including children >10 y administered to
and adults), e.g., asplenia, sickle cell disease, immunodeficiencies: 3 doses (0.5 mL each) patients being
with an interval of ≥1 month between the first and second dose and ≥4 months between the treated with
second and third dose eculizumab

160. It has come to your attention that front store manager sold two boxes (2x24) or Claritin PLUS
Pseudoephedrine/Ibuprofen combination products each containing 15 tablets. What is your action?
a) Pharmacist intervention is needed while selling this product
b) Do not keep this product in pharmacy
c) Tell the front store manager that this is a precursor to amphetamines
d) Hang a sign in the isles saying to talk to the pharmacist
e) It is your legal responsibility to cash out the patient
161. Erythromycin counselling? Refrigerate at 2° to 8° C

162. An old lady has Dementia, her daughter asks you not to tell her mother about the medication
because if she knows, she will not take it. What is the best sentence to describe veracity?
a) You do as the daughter asked from you
b) You give the patient the reading material and talk to the patient (Not remember but all
about reading material

163. A patient had a stroke with dysphagia was taking phenytoin tablet what to give him instead
due to dysphagia
a) Give him phenytoin suspension
b) Give him phenytoin chewable tablets
c) Continue giving him tablets

164. which drug causes Fast HR


a) Metoprolol
b) Felodipine
c) Diltiazam
d) Enalapril

165. Elderly uncomplicated UTI

166. What is the Monitoring of Rheumatoid arthritis?


 Active 1-3 month
 Target achived 6 months
 CBC, LFT, RFT and ESR
167. Hep C testing guidelines

168. 79-year-old woman with all the diseases. What supplement should she get
a) L-carnitine
b) Vitamin D

169. Academic Detailing is an important tool to optimize the use of medications supported by
a) CMIRPS (Canadian Medication Incident Reporting and Prevention System)
b) CIHI (Canadian Institute of Health Information)
c) CADTH (Canadian Agency for Drugs and Technologies in Health)
d) Health Canada
Academic detailing programs in Canada have followed some (if not all) of the principles of academic detailing
to foster improved clinical decision‐making. In 2003, the Canadian Academic Detailing Collaboration (CADC)
was developed by 6 provinces (NS, ON, MB, SK, AB and BC) to represent the academic detailers of Canada.
With the support of the Canadian Agency for Drugs and Technologies in Health (CADTH), representatives from
each academic detailing program meet monthly (online) to share experiences in academic detailing.

170. A patient heard about a new vitamin on a TV show and asked you about it. You told her
that the vitamin does not have any evidence. She accepted your advice and said thank you.
What did you follow?
a) Fidelity
b) Beneficience
c) Verasity
d) Justice
171. What is need special handling
a) Pamidronate tablet
b) Valacyclovir
c) Exemestane
d) Darbepoetin

172. HIV patient visited to Bangkok in previous years and she got therapy for infection in
hospital. She wants to know the reason for HIV infection. You asked her to check your
reference in detail to find out the reason behind this. Which ethical principal you demonstrate?
a) Veracity
b) Justice
c) Beneficence
a) Fidelity

173. There was another long question about Atrial fibrillation and same which anticoagulant
you will start and the crcl is. 60 min/ml
b) ASA
c) Clopidogrel
d) Warfarin
e) Rivaroxaban
Stop ASA and start clopidogrel??? I think i picked this

174. Manufacturer has sent you a notification about drug recall for a specific drug due to change
the label print while there is no change will be done to the ingredients. What is the appropriate
action?
a) Call patients asking them to return the medication that they have to the pharmacy and
return all medication on shelf to manufacturer
b) Return only sealed vials on shelf while continue using vials for dispensing.
c) Call the whole saler to purchase more stock of the drug to cover expected drug shortage
d) Recall the currecnt stock either they are open or sealed and no need to contact
patients

175. Long case for pancreatitis and use acarbose for diabetes, what is trigger pancreatitis?
a) Smoking
b) Alcohol
c) Acarbose
176. What is the most sensitive test for pancreatitis?
a) Amylase
b) Lipase
c) ALT
d) AST
Acute pancreatitis Acute pancreatitis is acute inflammation (Non-infectious) of the pancreas (sometimes,
adjacent tissues). The most common triggers are gallstones and alcohol intake.
The severity of acute pancreatitis is classified as mild, moderately severe, or severe based
on the presence of local complications and transient or persistent organ failure
Causes
Disorders: hypertriglyceridemia, alcoholism & bulimia nervosa.
Drug induced: EtoH, CHC (Estrogen/ EE  ↑ TG), Anti-HIV medictions (didanosine,
PIs), isotretinoin ↑ TG, incretin-based drugs DPP - 4 Inhibitors (Alogliptin, Linagliptin,
Saxagliptin & Sitagliptin), (GLP-1) Agonists (Dulaglutide, Semaglutide, Liraglutide &
Lixisenatide
In acute pancreatitis serum lipase and serum amylase are elevated however serum lipase
are slightly more sensitive in both major causes of pancreatitis gallstone and alcoholic
associated acute pancreatitis.
Acute Physiologic Assessment and Chronic Health Evaluation (APACHE) II Scores
should be calculated on admission and daily for the first 72 hours after admission.
An APACHE II Score of 8 or higher at baseline or in the first 72 hours is suggestive of
severe acute pancreatitis and is predictive of a worse clinical course.
Supportive care with fluid resuscitation, pain control and nutritional support
Prophylactic abx are NOT recommended regardless of type and severity

177. Codeine effect decrease by?


a) Pramipexole
b) Paroxetine  Paroxetine inhibits CYP2D6. Codeine needs that to activate

178. Technician received call for verbal order for Doxycycline & made error by writing the
drug name wrong but he discovered it from the dose & how many to take it per day & then you
called the physician & corrected it, what to prevent this error?
a) Only pharmacist receive the call
b) Technician read it back to prescriber over the phone or Repeat order after doctor
c) Tech do it under pharmacist supervision
d) Take the order only if the pharmacy is quiet
e) Pharmacist has to double check

179. Nitrofurantoin? Take with Food  increases bioavailability by 40%


CTC: Nitrofurantoin is administered orally and all dosage forms should be taken with food or milk to
minimize gastric upset. Administration with food may also enhance absorption
180. Dr wants to know a new interaction/new side effect about Ezetimibe, what to check.
a) Rx files
b) Medline
c) Micromedix
d) Checking clinical trials

181. pneumococcal conjugate vaccine


Polysaccharide (pneumococcal 23-valent Conjugate (pneumococcal 13-valent conjugate
polysaccharide vaccine (PNEU-P-23) vaccine (PNEU-C-13)
Conjugate to protein carrier to enhance immunogenicity Recommended for routine infant immunization and in
Recommended for children ≥ 24 months and adults with those with immunocompromising conditions.
conditions that increase risk of invasive pneumococcal Q. Persons who are eligible for both conjugate and
disease (IPD), adults ≥65 y, smokers, residents of polysaccharide vaccines should receive conjugate
longterm care facilities, persons with alcoholism, vaccine first. However, those who have already
homeless persons and illicit drug users. received PNEU-P-23 may still receive PNEU-C-13, in
A booster dose recommended in those with asplenia, which case administer ≥1 y after PNEU-P-23.
sickle cell disease, hepatic cirrhosis, chronic kidney Approved for adults ≥50 years of age and advisable
disease or nephrotic syndrome, HIV infection, for patients ≥65 years of age on an individual basis.
immunosuppression. Adult Hematopoietic stem cell transplantation HSCT
Q. Give 8 wk after last dose of conjugate vaccine (if recipients: 3 doses (0.5 mL/dose) IM administered 4
eligible for both). wk apart, starting 3–9 months after transplant. No role
In Hematopoietic stem cell transplantation HSCT for booster dose.
recipient, give 6–12m after conjugate vaccine last dose. Adults with HIV infection and immunocompromising
Give after 5 y if ≥11 y of age when initially immunized. conditions: 0.5 mL IM once. No role for booster dose.
After 3 y if ≤10 y of age at time of initial immunization.

182. Donepezil will interact with what


a) Ramipril
b) Bisoprolol
Additive bradycardic effects when combined with BBs or calcium channel blockers; few reports of actual
interactions.

183. To monitor effectiveness of furosemide through fluid loss with cardiac edema or ascites,
what should be monitored?
a) Fluid input fluid output
b) Glomerular filteration rate
c) Patient weight
d) ECG
184. Occupational Safety and Health Administration (OSHA) recommends?
a) Recommendation and information on surface contamination, worker contamination
and risk assessment.
b) Recommends prepackaging of oral drugs, crushing and splitting of cytotoxic drugs.
c) Recommends appropriate handling precautions in spill management.
d) All of the above

185. Patient experiences sudden urges to go the washroom for past few months she is
embarrassed to go out in public. She denies leakage upon cough & laughing. She has 3 kids
who are now teenage. She feels strees. Which type of incontinence?
a) Stress
b) Urge
c) Mixed
d) Frequency

186. what should be monitored before initiation of Methotrexate?


a) LFT
b) CBC
c) Chest X-ray
CTC: Baseline CBC, LFTs, albumin, creatinine, hepatitis B and C serology, chest x‐ray; repeat CBC, LFTs,
albumin and creatinine monthly × 3 months, then Q1–3 months. Consider HIV screening in high‐risk patients

187. Which of the following option is correct If you dilute 5% stock solution into 0.0125%?
a) 2.5 ml in 1000 ml
b) 5 ml in 2 Liter
c) 1 ml in 1 liter
d) 1.5 ml in 1500 mL
Dilution factor is 400

188. Drugs arranged accr to wt gain I chose e-cps

189. Hyperthyroidism? Muscle weakness or tinnitus


Sx: symptoms such as weight loss, palpitations, diarrhea, heat intolerance and anxiety

190. Empathy lice question? I can understand your concerns

191. There was one more no appropriate way to deal with consciential denial of pharmacist
denying to dispense?
a) Not to explain patients your moral grounds
192. What is a good communication strategy for preventing med error?
a) Phone doctor’s office and talk to him directly
b) Fax doctor’s office.

193. Brimonidine in ttt erythromatous rosacea counselling?


a) If you wear contact lenses, remove them first, wait 15 minutes after using eye drops to
put them back into eyes.
b) If gel for rosacea, then apply for whole face not red spots only.
c) Stain on clothes
d) Get onset of action 30 min after application
e) Apply only on papules
Alpha2-adrenergic Apply small pea-sized amount to each of the 5 areas of face (forehead, chin, nose, each
Agonists cheek) avoiding the eyes and eyelids, lips, mouth & inside of nose.
Brimonidine tartrate S.E: Erythema, flushing, skin burning sensation, contact dermatitis, headache.
0.33% gel applied CV effects may occur due to systemic absorption if applied to damaged skin or
once daily accidentally ingested orally.
Keep out of reach of children.
May start to reduce redness within 30 min with peak effect at 3 h.
Wash hands immediately after applying.
Rebound redness/flushing can occur on withdrawal.

194. Which drug not to use in automated counting machine?


a) Atomoxetine
b) Anastrazole

195. Which injection is fatal if given IV?


a) Nacl 0.9%
b) Dextrose
c) Sterile water for injection  Causes Hemolysis

196. Narcotic forgery. Patient come with prescription; you realize it is forgery what you will do?
a) Tell the patient you don't have the medication and he can come tomorrow to tell the
police (Delay technique).
b) You tell the patient it is forgery and don’t dispense it
c) Tell the patient it is forgery and restrain the patient until call the police.
d) Send the patient to another pharmacy to dispense the medication
197. Questions of spondylitis?
 Spondylitis is one of the most common causes of chronic backache and pain in the neck, making it
very difficult for a patient to live a normal, pain‐free life.
 Spondylitis is a condition caused by an inflammation of the vertebrae of the spinal cord. When the
vertebral joints become inflamed, the vertebrae may grow or fuse together, leading to a rigid spine.
It’s a form of osteoarthritis that affects the spine. The pain may not be felt initially, but gradually
becomes worse as the condition progresses. Usually, spondylitis starts in the lower back, proceeding
upwards towards the spine and leading to chronic back ache. What is Spondylitis?
 What causes Spondylitis? Genetics appear to be the most common cause of this condition. Research
shows that over 90 percent of people with this condition carry a particular gene called the human
leukocyte antigen B27 (HLA‐B27). That explains why somebody with a family history of spondylitis is
more likely to develop this condition than someone who doesn’t.
 What are the common symptoms of spondylitis? Loss of coordination or normal reflexes, Stiff neck
or back, Tingling or numbness in the affected area, Difficulty walking and moving around Severe
chronic pain and discomfort are the most obvious symptoms of Spondylitis.
 Identification of the patient’s symptoms while taking history is the first step towards diagnosis. The
doctor will also do a physical examination and request image tests – X‐ray, MRI, CT scans – to find
out the underlying cause of pain and see which part of the spine is most affected. How can
Spondylitis be diagnosed?
 What treatment options are there for this condition? The main aim of any treatment option for this
disease is to reduce pain and stiffness caused by the inflammation of the vertebrae.
 AO treatment Even if the process of degeneration has started or has been ongoing for many of years,
realignment of the vertebrae to their proper position can slow or the stop the process from
continuing and allow for repair phase and the best possible range of motion.
 What lifestyle changes can help provide relief from the symptoms of spondylitis? People with
spondylitis can lead normal lives with a few lifestyle changes using pain management techniques
advised by their doctors. Smoking and tobacco are believed to aggravate the condition; therefore,
patients are recommended to quit smoking to relieve them from pain.
 Other measures that should be practiced include: Sleeping on a firm mattress with the back straight
to avoid fusion of vertebrae. Not sleeping with a large pillow under the neck to avoid fusion of
vertebrae near the neck. Not propping up legs as it may cause fusion of joints in the knee region.
Maintaining a good posture without any slumping or slouching. Avoiding activities with sudden
impacts such as jumping, etc. Hot baths and showers also provide relief.

198. Meningitis with H. influenza, which antibiotic?


a) Penicillin G
b) Cefotaxime
CTC: S pneumonia/ N Meningitidis: Penicillin G
H Influenza/Listeria Monocytogenes: Ampicillin
Group B streptococcus: Penicillin
199. Hospital acquired
pneumonia main organism is
pseudomonas???

200. What not to monitor after


etanercept?
a) Srcr
b) Infusion reaction
c) Infection
Monitoring: Baseline CBC, LFTs, creatinine, hepatitis B and C serology, PPD and chest x‐ray to assess for
latent TB. May consider screening for ANA.

201. case epilepsy a pregnant patient and taking a medication (that was teratogenic), what is the
apropriate action?
a) Change medication
b) Leave her on her medication
c) Stop medication

202. What drug of treat nephrolithiasis?


a) Norfloxacin
b) Tamsolusin
Prednisolone in combination with nifedipine or tamsulosin, prednisolone is proven to facilitate spontaneous
passage of a ureteral stone in several small prospective studies. Only a short course of therapy (5‐10 d)
should be administered.

203. What about mom breastfeeding 4-6 instead of 6-8 now is worried

204. Diabetes, clcr 15 ml/min, hypertension also, what to give


a) Metformin
b) Exenatide
c) Canagliflozin
d) Liragliptine
In patients with type 2 diabetes already initiated on treatment for diabetic nephropathy, the use of
INVOKANA 100 mg can be continued in patients with an eGFR <30 mL/min/1.73 m2. INVOKANA 100 mg
should be discontinued if dialysis is initiated

205. What to monitor? 24 hours urine checkup


CTC: Response to INVOKANA treatment should be monitored by periodic measurements of blood glucose
and HbA1c levels. Due to its mechanism of action, patients taking INVOKANA will test +ve for glucose in
their urine.

206. Confidence interval question –0.71

207. Patient is taking warfarin as per following schedule


SAT, MON, WED, FRI 6 mg daily. SUN, TUES, THUR 2 mg daily
Now Dr. wants to decrease the dose by 20%. Which is correct? (Idea is over all decrease
of dose by 20%, total dose/week= 30 mg now choose answer which contain overall 80%
of 30 mg= 24 mg)
a) SAT, MON, TUE 10 mg BID
b) SUN, WED, THUR 2.5mg BID
c) SAT, SUN, MON, TUES, WED, THU 4 mg every day
d) SAT, SUN, WED 5 MG BID & MON, TUES, FRI 3 mg

208. Amiodarone is added to the regimen due to atrial fibrillation. Patient is already on Warfarin.
What will be the consequence?
a) Adjust dose of Warfarin as amiodarone will increase warfarin
b) Adjust dose of Warfarin as amiodarone will decrease warfarin
c) No drug therapy problem
d) Takes 3−7 days for amiodarone affect
e) Warfarin and amiodarone need a dosage adjustment.
Amiodarone is a potent inhibitor of the enzymes responsible for warfarin metabolism. Decreased metabolism
of warfarin leads to higher plasma concentrations and an increased risk of bleeding complications. When
initiating amiodarone in a patient receiving warfarin, the dose of warfarin should be decreased and more
frequent monitoring of the INR should occur.
An empiric warfarin dosage reduction of 30% to 50% at the initiation of amiodarone might be considered,
with close monitoring of anticoagulant effect to further refine optimal dosing.

209. Patient is taking Warfarin. After adding a new drug therapy INR increases to 4.5 but
patient is not experiencing any bleeding. What is the best course of action?
a) Give oral vitamin K
b) Give parenteral Vitamin K
c) Stop warfarin for 5 days
d) Stop warfarin for 7 days
210. 28 months old baby having nasal congestion, Rhinorrhea, Myalgia, Mother is using OTC
acetaminophen, but very mild improvement has been seen. Now mother called a nearby
pharmacy asking if there any suitable chewable dosage form of medication available, what is
the best source to check this information?
a) Sick kid’s pediatric guideline
b) CTMA
c) CTC

211. Patient with Cellulitis & MRSA is


identified as causative agent. What is the
appropriate treatment option?
a) Cefotaxime IV
b) Levofloxacin PO
c) Amoxicillin PO
d) Linezolid IV

212. What is the Duration of treatment for


cellulitis?
a) 10 days
b) 3 weeks
c) 5 days
d) 2 weeks

213. 48 years old lady came to hospital with multiple symptoms i.e. fever, cough, arthralgia,
myalgia etc. Her history reveals that she was a Sex worker also elicit drug users. Her lab. test
came up with Viral Load of 19000 copy/ml and CD4 is 190 and she is diagnosed as HIV patient.
Given above condition what will be the drug Regimen for her along with Darunavir/Ritonavir?
a) Lamivudine/Abacavir
b) Lamivudine/Tenofovir
c) Didanosin/Stavudine
d) Stavudine/Raltegravir

214. Now doctor want to screen for Hepatitis B infection. What will be the recommendation
according to HIV treatment guideline for screening of Hepatitis B?
a) Screening only Patient with risky Sexual behaviour
b) Screen patient who has low CD4 count
c) Screening only Patient who is an elicit drug user
d) Screen for HEP B in all patient with HIV
215. Patient was found yelling to people and buses with confusing behaviour near a bus stop.
He was nearly unconscious. After admitting to hospital his condition is identified as hepatic
encephalopathy. Patient has history of alcoholism and has Liver cirrhosis. What is the
appropriate treatment for the patient?
a) Lactulose
b) Rifamixin
c) Rehydration
d) Oral Corticosteroid

216. What will be the indication of patient is recovering


after the treatment has been initiated?
a) Urination will resume and become normal
b) Pain will be controlled
c) Patient will regain his consciousness gradually
d) Bowel movement will be normal

217. How to prophylaxis of SBP?


a) Azithromycin
b) Bactrim SMX/TMP
c) Metronidazole
d) Clindamycin
Use prophylactic antibiotics after a patient has experienced 1 episode of SBP or in patients with renal
impairment or severe liver disease (Child‐Pugh score >9) whose ascites protein level is <15 g/L.2 A patient
with 1 episode of SBP has a 69% chance of recurrence within 1 year. Treatment with sulfamethoxazole /
trimethoprim, norfloxacin or ciprofloxacin decreases the rate of SBP recurrence, and, although it does not
improve survival, is very cost effective.6 Based on the best available evidence norfloxacin is considered the
drug of choice. Start prophylactic antibiotics after the completion of intravenous antibiotic therapy for the
acute episode and continue until resolution of ascites, liver transplantation or death. Patients with an episode
of SBP should be referred and considered for possible liver transplantation if eligible

218. Patient is initiated on Levodopa recently. What is the Monitoring of Levodopa?


a) Hypertension
b) TSH
c) Sr. Creatinine
 Frequent monitoring of BUN, creatinine levels, and hepatic function is necessary for individuals taking
Levodopa. Good hepatic function is crucial in patients taking levodopa because it is where the drug is
decarboxylated.
 Also, it is important to test for intraocular pressure in patients with glaucoma. Testing for peripheral
neuropathy before and while on levodopa is also very important. Patients should also be regularly
monitored for dyskinesia.
 Patients need to be observed for psychotic behavior and hallucinations when on dopaminergic
medications. Confusion and excessive dreaming can be accompanied with hallucinations. Thus, patients
with a history of past psychiatric disorders should not be treated with levodopa.
 As an extra precaution, patients should be monitored for melanoma. It is not known yet whether the
risk of melanoma is increased due to levodopa use or Parkinson disease.

219. Patient is prescribed with expensive biologics which will not be covered by his insurance
where to hook up?
a) Manufacturer program
b) Give drug in credit
c) Tell the doctor to go back to previous regimen
d) Social worker

220. HM is an obese (BMI 30) female patient diagnosed with Type-II Diabetes. She has
hypercholesterolemia and other complications. Doctors recommends non-pharmacological
measures for her. Which is the best strategy for her?
a) Physiotherapist for weight reduction and physical balance
b) Dietician for food adjustment
c) Social worker
d) Joining health club

221. She tried nonpharmacologic approach for a month, but her sugar level is not controlled &
HbA1C 8.1%. Now her doctor, pharmacist, dietician agreed to start pharmacologic treatment,
but she keeps on insisting to continue non-pharmacologic approach for another few days. What
is the best response from a pharmacist?
a) Agree with physician and tell her to take medication for future disease
b) Agree with patient’s opinion
c) Let the physician figure out the problem
d) Advice non-pharmacologic for her

222. Doctor decided to start HM on Drug therapy, what is her best option for treatment?
a) Metformin
b) Exenatide
c) Gliclazide
d) Sitagliptin
223. Which is not sign of hypoglycaemia?
a) Hunger
b) Sweating
c) Fruity smell (diabetic keto acidosis)
d) Tremor
Mild to moderate hypoglycemia has autonomic symptoms: sweating, tremors, tachycardia, heart
palpitations (heavy, fast heartbeats), hunger, nausea, numb lips or tongue, headache and a general
sensation of weakness.
Severe hypoglycemia requires assistance in its recognition and/or treatment. Neuroglycopenic symptoms
such as confusion, anxiety, feeling irritable, altered behaviour, difficulty speaking and disorientation can
progress to seizures and coma that prevent patient from appropriately treating the hypoglycemic episode.

224. 308. Dronedarone contraindication


a) Hypertension
b) Heart failure
c) CAD
d) Renal failure
Dronedarone is contraindicated in patients with:
● Permanent atrial fibrillation of any duration in which sinus rhythm cannot be restored and attempts
to restore it are no longer considered by the attending physician
● History of, or current heart failure, regardless of NYHA functional class
● Left ventricular systolic dysfunction: Second‐ or third‐degree atrio‐ventricular (AV) block, complete
bundle branch block, distal block, sinus node dysfunction, atrial conduction defects or sick sinus
syndrome (except when used in conjunction with a functioning pacemaker

225. Patient asking about missing pills and


with contraceptives. Know the rules for
what to do if you skip in week 1 vs week
2/3.

226. Patient has a viral infection. Patient is


symptomatic. Doctor says no antimicrobial
therapy is necessary. How do you advice?
a) Tell patient that the infection will
resolve on its own
b) Recommend therapies for patient’s
symptoms
227. SM is a 31 years old male works as a data entry operator. He has asthma problem and
takes SABA 2 puff QID PRN and Fluticasone/Salmeterol combination 1puff BID. Last week he
missed 1 day in office because he could not sleep at night for his asthma exacerbation. He lives
in a small basement in the downtown area. Now a day his asthma is quit bothering him. What
could be the reason associated with his condition?
a) Pattern of work
b) Living condition
c) SM has a week immunity

228. What is the goal of treatment in his case?


a) Reduce side effects of asthma medication
b) Decrease his work place absence by improving his condition
c) Reduce excessive use of asthma medication

229. What is the next treatment option for SM?


a) Increase the dose of Fluticasone/Salmeterol
b) Add Ipratropium
c) Avoid exacerbating factors
d) Add oral corticosteroid
230. What is true about salmeterol & fluticasone Diskus?
a) You need to shake it before use
b) Deeply Inhale the medication in one inhalation
c) Clean the device every 6 months
d) Store it in refrigerator
CTC: How to use your ADVAIR DISKUS properly:
a. Sliding the lever of your DISKUS opens a small hole in the mouthpiece and unwraps a dose ready for
you to inhale it. When you close the DISKUS, the lever automatically moves back to its original position
ready for your next dose when you need it. The outer case protects your DISKUS when it is not in use.
b. Open: To open your DISKUS hold the outer case in one hand and put the thumb of your other hand on
the thumb grip. Push the thumb grip away from you, until you hear it click into place.
c. Slide: Hold your DISKUS with the mouthpiece towards you. Slide the lever away until you hear another
click. Your DISKUS is now ready to use. Every time the lever is pushed back a dose is made available for
inhaling. This is shown by the dose counter. Do not play with the lever as this releases doses which will
be wasted.
d. Exhale: Hold the DISKUS away from your mouth. Breathe out as far as is comfortable. Remember—
never exhale into your DISKUS.
e. Inhale: Before you start to inhale the dose, read through this section carefully. Once you have fully
exhaled, place the mouthpiece to your mouth and close your lips around it. Breathe in steadily and
deeply through your mouth until a full breath is taken. Remove the DISKUS from your mouth. Hold
your breath for 10 seconds or as long as is comfortable. Breathe out slowly.
f. Close: To close your DISKUS, place your thumb in the thumb grip, and slide it back until you hear a
click. The lever is now automatically reset for your next use. The counter on the DISKUS indicates how
many doses are remaining.
g. Rinse out your mouth, gargle with water after each dose. Do not swallow the water.

231. Pharmacy intern is counselling about a drug, but patient seems to be confused and came
to you to clarify. What can you do as a pharmacist to improve intern’s performance?
a) Give him more resource to study before counselling
b) Observe him counselling and intervene if anything confusing
c) Practise counselling with him
d) Tell him to study before counselling

232. Patient saw on TV that a new study revels a significant drug-drug interaction between two
of the drugs she is taking currently. Now she is telling you about this and worried about it. What
issue you will check regarding this study?
a) Is this study is conducted on same ethnic population patient belongs
b) Is it a placebo-controlled study?
c) Does the interaction is final endpoint of the study
234. What is true about composite endpoint of a study EXCEPT?
a) Lots of question is answered
b) Rare parameters can be included in the composite endpoint
c) Lots of factor are considered in the composite endpoint
d) Outcome is very simple

235. Baby sitter administer 2 teaspoons full of Amoxicillin suspension instead of 2 ml per dose.
Parents are very worried about this condition. What is the appropriate action for parents?
a) Take to child to emergency
b) Watch for Diarrhea and vomiting for next 48 hours
c) Call poison control centre
d) Nothing will happen
CTC: If you think you have taken too much AMOXICILLIN, contact your healthcare professional, hospital
emergency department or regional poison control center immediately, even if there are no symptoms

236. What option will uphold Doctors autonomy?


a) Formulary restriction
b) Pre-authorized dispensing
c) Prescribing authority

237. Hospital authority did not include a drug into formulary due to the Lack of study. Which
Ethical principle is signified by the action?
a) Beneficence
b) Non-maleficence
c) Justice
d) Veracity

238. A study is conducted on 500 patients and found significant difference between placebo and
drug. and what could be the limitation of study?
a) Study is poorly conducted
b) Long term effect of drug not studied
c) Not published in reputed journal

239. Which is the best source to look for comparative study between a newly approved drug A
and drug B which is already existing in the market?
a) Monograph of Product A
b) Practice guideline
c) Medline plus
d) Primary study on this drug
240. All is true regarding HPV vaccine EXCEPT?
a) Reduce cervical screening
b) Given by SC
c) Can be given female patient 9 years
Q. Gardasil Prevention of external anogenital warts caused by HPV types 6 and 11.
9-valent human Females 9–45 y and males 9–26 y: 0.5 mL administered at 0, 2- and 6-months IM
papillomavirus Alternative 2-dose schedule in immunocompetent individuals 9–14 y: 0.5 mL administered
(types 6, 11, 16, at 0 and 6–12 months IM.
18, 31, 33, 45, If alternate dosing is required, the second dose should be administered at least 1 month after
52, 58) the first dose, and the third dose should be administered at least 3 months after the second
recombinant dose. All doses should be administered within a 1-year period.
vaccine S.E: Pain, swelling, erythema and pruritus at injection site, headache, fatigue, syncope,
lymphadenopathy, anaphylaxis.
Not recommended in pregnancy, but used in breastfeeding.

241. Patient went abroad and bought a combination of natural medication. What should you
check for these products OR how do u know if they are licensed in Canada?
a) NPN
b) DIN HM
c) DIN
NPN: Plant, vitamins, amino acid, omega 3, magnesium, probiotic.
HM: A basic belief behind homeopathy is “like cures like.” In other words, something that brings on
symptoms in a healthy person can ‐‐ in a very small dose ‐‐ treat an illness with similar symptoms. This is
meant to trigger the body’s natural defenses. Treatments for other ailments are made from poison ivy,
white arsenic, crushed whole bees, and an herb called arnica.

242. 4 years Kid with night time incontinence who was never potty trained. Pees twice a
week. What do you give?
a) Reassure her that no problem with her son, Normal for his age, give nothing
b) Give her desmopressin
c) Give her oxybutynin

243. What is the mesh size of preparing a sterile preparation aseptically?


a) 2.2 micron
b) 0.22 micron
c) 1.6 micron
d) 0.33 micron
244. What drug should be prescribed if a patient undergoes electronic cardioversion?
a) Warfarin
b) Ticagrelor
c) Prasugrel
Patients with AF or atrial flutter lasting ≥ 48 hours should be anticoagulated for at least 3 weeks prior to
electrical cardioversion and maintained for a minimum of 4 weeks after.
NCBI: With the increased use of DOACs in routine clinical practice, several practical issues have emerged,
including considerations for cardioversion. Cardioversion restores sinus rhythm in AF and can improve
cardiac symptoms, but is associated with increased thromboembolic risks. While warfarin can decrease
these risks, its use is associated with delayed procedures due to its slow onset of action. By contrast, DOACs
act rapidly, have a short half‐life and predictable pharmacokinetics. Two randomised clinical studies, X‐VERT
and ENSURE‐AF, have demonstrated the efficacy and safety of rivaroxaban and edoxaban, respectively, in
this setting. Compared with warfarin or other VKAs, both have been found to reduce the time between
administration and cardioversion and may facilitate more predictability in planning cardioversions. Results
from ongoing and future studies will add further data to inform optimal use of these agents.

245. Mother wants permethrin cr. For her


child Treating scabies, which of the
following is INCORRECT counseling?
a. Treat bedmates
b. Child shouldn’t go to school till
free of nits
c. Wash all bedlinens and dry in hot air
d. Only treat sexual partner
CTMA: Nonpharmacologic Therapy
Clothes and linens should be washed in soap and hot water (60°C) and machine dried using the hot cycle, dry
cleaned, or stored in plastic bags for 5–7 days. All surfaces, rugs, furniture and unwastable items should be
vacuumed. Avoid body contact with others until completion of treatment and follow up. Children may return
to school the day after treatment is completed. Fingernails should be closely trimmed to prevent skin injury
resulting from excessive scratching.

246. All are used in the treatment of hot flush except


a) Estrogen
b) Gabapentin
c) Venlafaxine
d) Sertraline
SSRI: Escitalopram, fluoxetine and paroxetine
CTC: Not all women are candidates for HT or care to consider HT as a therapeutic option. A mild to moderate
reduction in the frequency and/or severity of hot flashes has been shown with some SSRIs & SNRIs (e.g.,
citalopram, desvenlafaxine, escitalopram, paroxetine, venlafaxine)
247. female Patient 29 years old from asian heritage, she is taking oral contraceptive and
diagnosed with in Generalised tonic-clonic seizure. What is the drug of choice?
a) Valproic acid
b) Carbamazepine
c) Lamotrigine
d) Levetiracetam
Valproic acid has least interaction (valproic acid is DOC in normal Condition)

248. What could be the reason for nation wide shortage of a product?
a) Disruption in cold chain management
b) Quality control requirement increased
c) Manufacturer Price competition
d) Inadequate ordering of Raw material by Whole seller

249. Manufacturer want to host a program for their new cough suppressant products. What is
the best course of action a pharmacist can follow to uphold ethical & professional values of
pharmacy practice?
a) Pharmacy will not accept any financial compensation from manufacturer
b) Only evidence-based product will be show cased in the program
c) Company can’t recommend any product directly
d) Pharmacy assistant will organise the event with compensation from pharmacy

250. Physician samples can be dispensed in which of the following situation?


a) Dispensing the sample with only taking the dispensing fee
b) Dispense the Sample with no charge at all
c) Only pharmacy stuff can use this
d) It is not legal to dispense Physician sample

251. Which preparation is needed to be prepared under laminar air flow?


a) Preparation of a Parenteral solution
b) Isotonic Saline
c) Vasopressin syringe of 20 units planned for direct IV injection for heart surgery
d) Preparation of 50 ml of ceftriaxone to be given as infusion for a hospitalized child to
control his pneumonia
e) Preparation of eye drop form to be used after eye surgery
252. 50 years old diabetic patient is diagnosed with Hypertension. He has a solitary kidney with
renal artery stenosis, which is the drug of choice?
a) Hydrochlorothiazide
b) Ramipril
c) Metoprolol
d) Amlodipine
According to guidelines ACEI are first choice in solitary kidney due to slow progression of CKD. But in bilateral
artery stenosis it’s not first choice and if we use need to monitor bcz now there is risk of acute renal failure.
Avoid ACEIs or ARBs in patients with bilateral renal artery stenosis or unilateral disease with a solitary kidney.

253. What is the Prescription requirements for Modafinil?


a) Written & verbal prescription with refill
b) Written prescription only
c) Written and Verbal prescription with 1 refill only
d) Written and Verbal prescription with no refill

254. What is the Prescription requirements for Clobazam?


a) Written and verbal prescription with no refill
b) Only written prescription
c) Written and verbal prescription with refill within 1 year
d) Written and verbal prescription with refill if interval is mentioned

255. Pharmacist wants to return some expired products to manufacturer. List of drugs given below
Testosterone, Phenobarbital, Lorazepam, Diazepam. What are the regulations regarding return?
a) Phenobarbital returned with Normal drug product
b) Lorazepam diazepam with normal products
c) With prior written authorization

256. The following are roles of Public Health Agency of Canada, EXCEPT:
a) Provides vaccination schedules for both adults & infants
b) Protects Canadian from preventable health risks related to food and zoonotic disease.
c) Primarily involved in treatment of chronic diseases causing burden to Health Care
system. The role of PHAC is just control and prevention but NOT treatment.
d) Prevents and controls infectious diseases
The role of the Public Health Agency of Canada is to:
 Promote health; Prepare for and respond to public health emergencies;
 Prevent and control chronic diseases and injuries; Prevent and control infectious diseases;
 Serve as a central point for sharing Canada's expertise with the rest of the world;
 Apply international research and development to Canada's public health programs; and
 Strengthen intergovernmental collaboration on public health and facilitate approaches to public health
257. Pharmacist entered a prescription of a wrong medication. What to do to prevent this error?
a) Tall man letter on bottle
b) Check with patient the indication
c) Ask the technician to double check this specific medication upon entry
d) Print a paper and place next to computer with look-alike sound alike

258. 4 years old child is presented with runny nose, nasal congestion & malaise. He is taking
loratadine for 2 days but there is no significant improvement. What can you suggest?
a) Continues loratadine for a week (CTMA)
b) Add oral decongestant
c) Go to doctor
d) Add topical decongestant
(Add topical or oral decongestant as PRN if congestion is there. We can add anti cholinergic i.e. Ipratropium
if runny nose there). CTMA: Intranasal ipratropium is effective for rhinorrhea secondary
to allergic rhinitis but not for other symptoms. Antihistamine plus Decongestant > 6 YRS OLD
Because antihistamines may have only a modest effect on nasal congestion, antihistamines and
decongestants are often combined. Some patients may respond to this combination when corticosteroids
have failed or when either medication alone does not provide adequate relief of nasal symptoms
Decongestants are not recommended for use in children under 6 years of age. In those children, intranasal
saline drops or spray may be used to clear nasal passages before eating or sleeping.

259. Few days later parent comes up with complain about a night time cough with burking
sound. What is the recommendation of pharmacist?
a) Tell the parents to observe the patient for a week
b) Wait till 48 hours and no improvement see a doctor
c) Give him Decongestant
d) Refer to doctor

Questions on rosacae. Patient is suffering from mild to moderate rosacea


260. What is considered as a trigger for rosacea?
a) Alcohol
SHESHAA - Spicy food ‐ Heat ‐ Emotional stress ‐ Sunlight ‐ Hot beverages ‐ Alcohol ‐ Application of CS.

261. You can advise him with which of the following nonphamacolog action?
a) Avoid alcoholic beverage
b) Use astringests for cooling
c) Avoid smoking
d) Sun exposure is good for your case
e) Avoid exposure to cold weather
262. What the best topical medication used to treat rosacea?
a) Tretinoin
b) Corticosteroids
c) Clindamycin
d) Metronidazole
e) Azelic acid

263. Dr prescribed topical metronidazole; however, it does not seem to be working for him.
Which oral medication should this patient be started on?
a) Erythromycin
b) Clindamycin
c) Keep this tx and add minocycline
d) Stop metronidazole and give isotretinoin
e) Stop metronidazole and give tetracycline

264. A new exciting drug coming out. As pharmacy manager, what will influence whether or
not you will include it in the inventory?
a) Local demand
b) Price of acquisition
c) Safety profile
265. Flu case what to tell the patient?
a) Having bedrest & fluids
b) To go to emergency now
c) Not to take any medication

266. This patient is working in long term care facility what to tell her
a) Avoid going to work till she's completely symptoms free
b) She can go to work after 2 days of being afebrile
c) She can go to work after one week

267. Which antibiotic is associated with Gallbladder Pseudolithiasis?


a) Vancomycin
b) Ciprofloxacin
c) Tetracycline
d) Ceftriaxone
Development of sludge or stones in the gallbladders due to increase bilirubin & usually reversible upon
discontinuation of Ceftriaxone

268. In a pt. Taking dexamethasone, u should advise him


a) To take it with food
b) Avoid sun exposure due to photosensitivity
c) Avoid taking caffeine with it

269. What to monitor in patient taking amiodarone?


a) Ocular
b) Hepatic function
c) Blood glucose
d) Platelet
e) Renal function

270. A case about hypertensive pt suffering from raynaud’s disease, what's the DOC for her?
a) Amlodipine
b) Hydrochlorothiazid
c) ACEI

271. Company withdraws a drug you can do all except


a) Find out patients who are already using this drug
b) Post this info. to the patient in pharmacy
c) Prepare all the stock to give it back
272. When 2 drugs are biosimilar, this means that
a) They are interchangeable in the formulary
b) Having the same dose & dosage form
To be authorized for sale in Canada as a biosimilar, a drug must meet a detailed set of criteria from Health
Canada (for example, similar biochemical structure; similar pharmacokinetic and pharmacodynamic
characteristics) and must demonstrate safety and efficacy for each indication; in certain situations, it is
possible to extrapolate therapeutic similarity from one indication to another indication.

273. Who require consent in a clinical trial?


a) Female of child bearing age
b) Children less than 16 years old
c) Patients with hepatic cirrhosis

274. The formulary decided to refuse a drug for the lack of evidence; so, this is upholding?
a) Justice
b) Protecting vulnerable
c) Non-malficience
d) Fidelity

275. For safety purpose which of the following require written RX


a) Phenoparbital
b) Loperamide
c) Ketamine

276. Upon receiving a shipment of narcotic medication pharmacist has document it in Narcotic
& controlled register. All of the following information is needed by the pharmacist EXCEPT.
a) Invoice number
b) The date of receiving
c) The amount of narcotics
d) The name of supplier
A pharmacist, upon receipt of a narcotic from a licensed dealer, shall forthwith enter in a book, register or
other record maintained for such purposes, the following:
1. The name and quantity of the narcotic received;
2. The date the narcotic was received; and
3. The name and address of the person from whom the narcotic was received.
277. Which need dose adjustment in patients with renal impairment?
a) Cotrimoxazole
b) Moxifloxacin
c) Cefixime
Cotrimoxazole: Dosage adjustment is recommended when sulfamethoxazole/trimethoprim is used in
patients with reduced renal function to avoid drug accumulation
Moxifloxacin: No dosage adjustment is required for moxifloxacin in renal impairment.
Cefixime: should be used with particular care in the presence of severely impaired renal function. Dose
modification is recommended for patients with moderate or severe renal impairment (i.e., creatinine
clearance of <40 mL/min

278. Drug is being studied to maintain remission in Lupus patients. Didn’t exclude those with
active disease. Lupus p=0.04 on antinuclear antibodies in 30 patients. Best next step for
conducting follow-up trial?
a) More patients
b) Measure clinical endpoints (remission attainment)
c) Exclude those with active SLE

279. Which requires nurse to wear gloves


a) Prednisone
b) Cyclosporine
c) Aspirin
d) Clopidogrel

280. What NOT to monitor with olanzapine?


a) CBC
b) Weight
c) Blood glucose

281. Which of the following can't be given in nasogastric tube?


a) Clindamycin Suspension
b) Diltiazim controlled release
c) EC ASA

282. Question about company wants to pay for a meeting in the pharmacy, what to do to keep
integrity of the pharmacy?
a) Talk about products of the company only with evidence
b) Not to talk about any of the company's product in the meeting
c) Refuse to take any money from the company
283. Prazosin all except
a) It may cause hypotension
b) It may cause nasal congestion
c) Take it at bedtime to minimize side effect
d) Metabolized by kidney
Studies indicate that Hypovase is extensively metabolized, primarily by demethylation and conjugation, and
excreted mainly via bile and faeces. Renal blood flow and glomerular filtration rate are not impaired by long
term oral administration and thus Hypovase can be used with safety in hypertensive patients with impaired
renal function.

284. Folinic acid is? Leucovirin

285. What is the reason for documentation in hospital pharmacy, all except?
a) For patient interest
b) Do not depend on staff cognitive function
c) For reimbursement

286. Doctor is starting a patient who is 60 kg on Isotretinoin, he wants to start at 0.5 mg/kg/day
for 4 weeks. After the 4 weeks he wants to increase the dose to 1 mg/kg/day. The total
cumulative dose is 120-150 mg/kg. If he starts with 30 mg daily for 4 weeks, what is the next
step?
a) 30mg BID for 3 months
b) 30mg BID 4 months
c) 30mg BID 5 months
d) 60 mg BID for 3 months
e) 60 mg BID 5 months
In general, patients initially should receive 0.5 mg/kg body weight daily for a period of two to four weeks,
when their responsiveness to the drug will usually be apparent.
Maintenance dose should be adjusted between 0.1 and 1 mg/kg body weight daily and, in exceptional
instances, up to 2 mg/kg body weight daily, depending upon individual patient response and tolerance to
the drug. A complete course of therapy consists of 12‐16 weeks of administration.

287. Female Patient has shingles. Her husband had chickenpox when he was child. Her mother
and her daughter had not chickenpox before, what you should tell her
a) Her husband may get chickenpox again
b) Her daughter may get shingles
c) Her mother may get chickenpox
288. Dr decided to decrease dose of warfarin 20%. Right now, the patient taking 30 mg per
week. Which of the following regimen you will select fo this patient?
a) 3.5 mg daily
b) 4mg 6 days and Sunday off
c) 4 mg daily

289. Treatment A is 44 of 56 quitting of smoking. Treatment B is 29 of 71 quitting smoking.


What is odd ratio of treatement A to B?
a) 1
b) 1.2
c) 1.9
d) 0.23
OR = (A/C)/(B/D) = AD/BC = odds of exposure among cases/Odds of exposure among control
= 44 * 71 / 56 * 29 = 3124 / 1624 = 1.9

290. Mother is getting Nystatin cream for her breast candidiasis and her baby is getting Nystatin
oral suspension for his oral candidiasis, which of the following is considered a good advice for
the mother?
a) Use a bottle to feed your baby during the time of treatment
b) Use a dropper or cotton swab to apply the Nystatin drops into your baby’s mouth
c) Apply the Nystatin cream to the infected area just before feeding your baby
d) Put nystatin drops in his mouth then breastfeed him to ensure swallow it

291. A kid feel from the bike and has a wound which is excoriated, a braided, inflamed, and
free of dirt or object. After cleaning the wound, which one would apply?
a) Put the bandage on it
b) Keep the wound open to allow the wound to heal
c) Apply topical antibiotics
d) Refer him to the physician
Nonpharmacologic Therapy
 Clean the wound with soap and drinkable tap water to remove dirt and debris from the wound area. If
possible, use running water or water under gentle pressure to clean the wound. A squirt bottle containing
water or a large syringe with no needle can also be used. Do not use hydrogen peroxide or rubbing
alcohol directly on the wound as they can be irritating and interfere with healing.
 Stop bleeding by applying gentle pressure to the wound using a sterile gauze or clean dressing. Apply
pressure for 10 minutes (15 minutes if the person is taking blood thinners) and if the bleeding does not
stop, refer the patient for further medical treatment. Remove any pieces of dirt or other material (such as
glass, metal or gravel) from the wound by rubbing the area gently with a clean gauze pad or using a pair
of tweezers that have been cleaned with rubbing alcohol.
 If a dressing is used, it should be changed daily, or more often if it appears dirty or damp. The dressing
can be removed after 48 hours if the wound is healing well. When changing the dressing, monitor for
signs of infection such as: red, puffy areas around the wound that are tender to touch red streaks coming
from the wound throbbing pain in the wound area pus (creamy yellowish-grey fluid) in the wound.
Other signs of infection include fever, chills, or tender lumps or swelling in the armpit, groin or neck.

292. Pt with seizure asked which of the following to take


a) Selenium
b) Folic Acid
c) Vitamin E
At the time of seizures evidence for systemic selenium deficiency could be documented. Our findings support
the hypothesis that presence of selenium depletion in the brain amongst patients with epilepsy constitutes
an important triggering factor for the origin of intractable seizures and subsequent neuronal damage.
women of childbearing potential who take AEDs should receive continuous folic acid supplementation (at
least 1 mg/day; some experts recommend 5 mg/day) to potentially reduce the risk of teratogenic effects
associatedwith AEDs

293. Wants to decrease risk of seizure in future. Who to refer?


a) Physiotherapist for exercise
b) Dietitian for better diet
Certain types of diets, including the ketogenic diet and the Modified Atkins Diet, may reduce seizure
frequency in some patients. However, these treatments require strict adherence to a high‐fat, low‐
carbohydrate diet and are not practical for most adults with epilepsy
MAY 2017
1. 48-year lady, work as a teacher & newly
diagnosed with Parkinson, all of the
following symptoms are associated with
Parkinson's disease? EXCEPT
a) Difficulty in speaking (Slurred speech)
b) Xerostomia
c) Urinary retention
d) Diarrhea

2. She is experiencing poor hand writing and she fall down two times falls 2 times and
changed her full-time job with part time job, what do you recommend?
a) Start treatment if it affects to daily living of life.
b) Start treatment Immediately
c) Start treatment when you get older so we delay side effects

3. what is the treatment for that case?


a) Pramipexol
b) Selegiline
c) Levodopa
d) nothing yet
Bromocriptine, pramipexole and
ropinirole are effective as monotherapy in
the early stages of the disease, and as
adjunctive therapy with levodopa for
patients with more advanced motor
complications

4. Where to go for speaking problem?


a) Speech / Language Pathologists
b) Occupational Therapists (OT)
c) Physiotherapists

5. Which of the following test are recommended prior to initiating bisphosphonate therapy
for Osteoporosis?
a) Dental status work
b) Lumbar X-ray
c) CBC
d) Platelets
6. What is not side effect of bisphosphonate?
a) Osteonecrosis of Jaw (ONJ)
b) Head femoral necrosis
c) They cause atypical femoral shaft fracture
d) Hypothermia
Febrile reactions, nausea, vomiting, flulike syndrome, local infusion site reactions, mild hypomagnesemia,
hypokalemia and hypophosphatemia. Osteonecrosis of the jaw (need good oral hygiene).

7. SJ 60year old patient is suffering from diabetes type 2 & taking oral antidiabetic since
2years. Now, he is experiencing low blood glucose level in early morning before breakfast
and midnight. Which of the following antidiabetic drug is responsible for this symptom?
a) Exenatide
b) Metformin
c) Meglitinide repaglinide
d) Pioglitazone
Repaglinide is capable of inducing hypoglycemia. Proper patient selection, dosage, and instructions to
the patient are important to avoid hypoglycemic episodes. Hepatic insufficiency may cause
elevated repaglinide levels in the blood and may also diminish gluconeogenic capacity, both of which
increase the risk of serious hypoglycemic reactions.
The risk of hypoglycemia is increased when pioglitazone is used in combination with insulin or a
sulfonylurea. Hypoglycemia in pioglitazone monotherapy is rare.

8. SM 45year old patient is discharged from hospital and pharmacist is doing medication
reconciliation prior to dispense new medication. During reconciliation, pharmacist found one
medication is missing, which was taken by patient regularly prior to admit in hospital.
Pharmacist delay dispense and contact to hospital physician regarding this concern. Which
ethics demonstrate by pharmacist?
a) Veracity
b) Fidelity
c) Justice
d) Autonomy

9. A wife frighten to give her husband anticancer med named Temozolamide because of
safty on her own body, what to advise her?
a) Take it with glove
b) No worries do it, its safe to touch
c) Wash hands and table and any surface with alcohol sparay
d) Wash hands with soap before and after
CTC: TEMODAL Capsules must not be opened or chewed, but are to be swallowed whole with a glass of
water. If a capsule becomes damaged, avoid contact of the powder contents with skin or mucous
membrane. In the case of accidental contact with skin or mucous membrane, flush with water.
10. Patient is looking for least expensive medication for his treatment. Which reference is
appropriate to find it out?
a) CDR
b) PMPRB
c) Drug product database
d) Health Canada
CADTH Common Drug Review (CDR)
 Reviews drugs and makes reimbursement recommendations to Canada's federal, provincial, and
territorial public drug plans, with the exception of Quebec, to guide their drug funding decisions.
 Conducting reviews of the clinical, cost-effectiveness (pharmacoeconomic analysis), and patient
evidence for drugs and providing formulary listing recommendation.
 Gives options for drug prices to compare between.

11. Physician prescribe morphine 30mg. Pharmacist did a dispensing error & gave wrong
dose morphine 60mg, which has score on it (scored tablet). Patient realize he received wrong
drug after reaching at home. He called pharmacy & ask suggestion. What do you suggest?
a) Identify error, Apologise, recollect wrong drug, dispense right dose & right quantity
b) Identify error, Apologise, dispense new drug
c) Identify error, Apologise, ask patient to take half tablet of morphine 60mg

12. According to ISMP, which may cause least error?


a) Digoxin 0.625mg PO QD
b) Levothyroxin 25mcg PO Daily
c) HCTZ 12.5mg PO daily

13. Patient is using Biological response modifier since last 6months. what do you check in
patient after 6months.? EXCEPT
a) Renal function
b) Tuberculosis Infection
c) Injection site reaction
d) LFT & CBC

SL, 15year old male is coming into your pharmacy to get recommendations for acne. He has
no any allergies, no medical conditions. His acne is mild, no inflammatory, no Papules or
pustules present, only comedogenic. But it causes him to concern.
14. Which initial medication do you recommend to his mild acne?
a) Azelaic acid
b) Glycolic acid
c) Adapalene
d) Tretinoin
15. After 6m, SL returns & mentioned, one month ago, his physician has prescribed
clindamycin & benzoyl peroxide topical gel combination use QD & Azelaic acid apply BID.
But he is not finding any
improvement. What do you
recommend for this condition?
a) Ask him to visit his physician
for reassessment
b) Inform him to continue
therapy as it will take 2-
3months to see improvement
c) Inform him to stop using this
medication
d) Recommend him to use oral
antibiotic as it is moderate acne

16. After 1 month, SL’s father come into pharmacy & ask about his son’s medication list for
tax purposes. What is your action?
a) Give him, as he is his father
b) Ask about medication, if he knows then give it
c) Ask him about verbal consent
d) Refuse to give to protect privacy & confidentiality, without any consent

17. Influenza scenario, what will pharmacist educate to infected patient to prevent spreading
infection to others?
a) Patients should wash their hands frequently to prevent viral infection

18. Influenza vaccine for pt taking warfarin.


Lexicomp: Vaccines may enhance the anticoagulant effect of Vitamin K Antagonists. However, most
available evidence suggests no meaningful change in anticoagulant effect.

19. Patient has egg allergy (baked food anaphylaxis reaction-hives). He wants influenza shot.
How do you give it?
a) Give dose under under physician supervision
b) Flu-shot cannot be given
c) can take it with antihistamine
Another version: Which of these patients is appropriate to receive vaccines in pharmacy?
a) Pregnant patient wants LAIV
b) Guy with egg allergy needs flu vaccine (if non severe, can be in pharmacy)
c) Cancer chemotherapy and wants varicella
d) Woman has a history of fainting with vaccines wants hep B
20. Which is true about vaccines:
a) Influenza vaccine may cause Guillain-Barré Syndrome (GBS)
b) May cause autism

21. Patient experiences fever, chest pain, cough during night time & hoarse sound. What do
you record this patient?
a) Refer him to doctor to to rule out bacterial pneumonia
b) Fluid and bed rest
c) Go to dr for codeine prescription
d) Dextromethorphan
22. Advice for a man with low back pain? Continue activity
Most of the time, back pain gets better in 2–4 weeks with very little treatment. Treatment usually consists of:
 Returning to your usual activities as soon as you can
 Reducing the amount of time, you rest in bed since it may slow the healing process
 Taking over-the-counter pain medications & Applying heat
If symptoms don't improve within 3–4 weeks, your doctor may send you to a therapist for special exercises.

23. Child gets pinworm infection. What is appropriate suggestion to his parent?
a) Treat child & all other family householders even if asymptomatic
b) Treat child & all other symptomatic family members
c) All close friend need treatment

24. Question about DVT or PE. Patient has leg edema and erythema. He diagnosed with
DVT. What do you recommend?
a) Wear compression stockings
b) Elevate legs to reduce swellings
Other version: Risk of DVT: immobility
CTC: Deep Vein Thrombosis Non pharm
 Rest if symptoms warrant (reduces pain and swelling), but evidence suggests early ambulation is safe
and may actually be preferred, as resolution of pain and swelling may be faster.
 Elevating the swollen limb while resting may hasten resolution of edema.
 NSAIDs for pain are effective but may increase the risk of bleeding, especially when used with
anticoagulants. Start with acetaminophen, but opioids may be required for a few days.
Elastic Stockings and Compression Bandages
 The post-thrombotic syndrome (PTS) is a clinically important and frequent complication of DVT developing
in as many as 60% of patients after proximal DVT. The most prominent symptoms are chronic swelling and
pain, discomfort when walking, and skin discoloration. Precise prediction of which patients will develop
PTS is not possible although some data support a higher probability for those with iliofemoral DVT and
recurrent DVT. It was thought that graduated compression stockings that apply an ankle pressure of 30–
40 mmHg and a lower pressure higher up the leg reduced the risk of PTS following DVT.
 Graduated compression stockings can improve edema and pain in acute stage of DVT. They can also
relieve symptoms in patients who develop PTS but they do not prevent the development of PTS. Use of
compression stockings is not appropriate when there is pre-existing peripheral vascular disease.

25. The Patient had total knee replacement surgury? What anti coagulant should be used?
a) Warfarin
b) Dabigatran
c) Fondaparinux
d) Rivaroxaban
LMWHs can all treat and prevent VTE. UFH is an effective prophylaxis only for moderately risky procedures.
Fondaparinux is used to prevent VTE in high risk orthopedic patients.
Vitamin K antagonists are used at doses specific to a target INR of 2-3.
Dabigatran, Apixaban, and Rivaroxaban are new oral anti-coagulants that can be used for VTE prophylaxis
after elective total hip/knee replacement surgery. Dabigatran and Rivaroxaban are superior to Apixaban.
26. Patient is going to inject Enoxaparin (dalteparin sod.) SC injection. What would you
recommend to patient prior to injecting?
a) Prior to injecting needle, pulled up plunger to remove any trapped air bubbles
b) Prior to injecting, do not remove any trapped air bubbbles because of exact dose
c) Inject it to 45’ angle in thigh
Administered by subcutaneous injection at 450 for the prevention of venous thromboembolic disease,
treatment of deep vein thrombosis, treatment of unstable angina and non-Q-wave myocardial infarction
and treatment of acute ST-segment Elevation Myocardial Infarction.
Instructions for self-injection
 When at home, there is nothing for you to prepare. The syringe is pre-filled with the exact
amount of drug required. Do not press on the plunger prior to injection.
 Solution should be inspected visually for clarity, particulate matter, precipitation, discolouration,
and leakage prior to administration. Do not use if solution shows haziness, particulate matter,
discolouration or leakage.
 The recommended site for injection is into the fat of the lower abdomen. This should be at least 5
centimeters away from your belly button and out towards your sides.
 Prior to injection, wash your hands and cleanse (do not rub) the selected site for injection with an
alcohol swab. Select a different site of the lower abdomen for each injection.
 Remove the needle cover by pulling it straight off the syringe. If adjusting the dose is required,
the dose adjustment must be done prior to injecting.
 NOTE: To avoid the loss of drug when using the 30 and 40 mg pre-filled syringes, do not expel the
air bubble from the syringe before the injection.
 Sit or lie down in a comfortable position and gather a fold of skin with your thumb and forefinger.
 Then holding the syringe at a right angle to the skin folded between your thumb and forefinger,
insert the needle as far as it will go. Hold the skin fold throughout the injection process. Once the
needle has been inserted, the syringe should not be moved. Push the plunger to inject. Be sure the
syringe is empty and the plunger is pushed all the way down before removing the syringe.
 Remove the needle at a right angle, by pulling it straight out. A protective sleeve will
automatically cover the needle.
 NOTE: The safety system allowing release of the protective sleeve can only be activated when the
syringe has been emptied by pressing the plunger all the way down.
 You can now let go of the skin fold and apply light pressure to the skin at the injection site for
several seconds with an alcohol swab. This action will help lessen any oozing or bleeding. Do not
rub the injection site.
 You should then safely dispose of the syringe and needle with its protective sleeve, so they remain
out of reach of children.

27. Patient is using LMWH (enoxaparin) during hospital stay for DVT. Now, pt had renal
impairment and Dr. wants him to switch on warfarin. How would you switch it?
a) Continue LMWH for 5 days and start warfarin together than stop LMWH 5 days after
starting warfarin
b) Stop enoxaparin and start warfarin when INR in therapeutic range
27. What is minimum duration of therapy for
DVT?
a) 3 months
b) 6 months
c) 9 months

28. He got dalteparin. CrCl 29 needs anticoagulant


a) Discontinue LMWH and start Rivaroxaban
b) Aspirin
c) Continue LMWH for 5 days and start warfarin
d) discontinue LMWH and start warfarin
We use UFH Instead (EXPECTED)

29. A case of mania, Patient has bipolar depression and takes divalproex, CBZ. Her
roommate call pharmacy and said she is acting strangely, experiences excessive moodiness,
and she didn’t sleep for 3 nights?
a) Medical attention is required
b) She needs to go to the emergency
c) Needs to see her doctor to assess???

30. What to give for immediate calminess?


a) IM Lorazepm, for sleep
b) Olanzapine
c) Lamotrigine
31. When that bipolar girl gets depression. What we give?
a) Mirtazapine
b) Quetiapine
c) Lithium.
1st line drugs 2nd line drugs 3rd line drugs
Quetiapine, lamotrigine, lithium, Divalproex, adjunctive SSRI or atypical antipsychotic,
lurasidone, adjunctive lamotrigine, bupropion, ECT, cariprazine, SNRI, MAOI, IV
lithium or divalproex plus lurasidone olanzapine plus fluoxetine ketamine, rTMS,
stimulants, light therapy.
Not recommended:
Monotherapy with antidepressants, gabapentin, aripiprazole or ziprasidone, adjunctive ziprasidone or
levetiracetam, lamotrigine+folic acid, mifepristone.

32. What to give for hiccups? IV Chlorpromazine


Dopamine All act as a dopamine antagonist or by enhancing gastric emptying.
Antagonists Q. Chlorpromazine historically has been the drug of choice for persistent hiccups.
Chlorpromazine Parenteral: 25–50 mg IV over 30–60 min.
Oral: 25–50 mg TID – QID PO × 2–3 days
S.E: Anticholinergic effects, extrapyramidal effects, hypotension, sedation.
Additive sedative effects with CNS depressants, including alcohol.

33. What is least criteria during inventory check?


a) Drug indication and duration of therapy

34. Venlafaxine to paroxetine or


citalopram to venlafaxine, how
to do?
35. Laminar airflow hood work criteria?
a) Work within 6inches (15cm) inside the flow hood

36. Whats not to prepare in horizontal laminar airflowhood


a) TPN large volume parenteral
b) Nasal solutions or antibacterial topical cream
c) Chemotherapy

37. Which one needs to be handled with gloves and prepared in vertical laminar hood?
a) Infliximab
b) Etanercept
c) Docetaxel, paclitaxel & etoposide

38. Nurse prepare insulin wrong a lot. What do you recommend to prevent insulin dosing
dispensing error in hospital?
a) Dispense from centralized pharmacy area through pharmacist only
b) Prepare unit dose system in pharmacy

39. What is prophylaxis of spontaneous bacterial peritonitis


a) Cotrimoxazole  Norfloxacin is the drug of choice for prophylaxis
b) Metronidazole
c) Meropenem  For nosocomial-acquired SBP
d) Cefotaxime or Ceftriaxone  Used for Community-acquired SBP

40. Patient with BPH and take terazosin, dr wants to give sildenafil 25 mg, what is
pharmacist concern?
a) Interaction of sildenafil and terazosin
b) Sildenafil dose is low
Lexi: These classes of agents can be used together with caution for erectile dysfunction. If tadalafil is
used for benign prostatic hyperplasia, discontinue any alpha 1-blocker at least 1 day prior to initiation.
41. Patient 63 years old with Atrial fibrillation, HF, Hypothyroidism. What is the drug of
choice for control of heart rhythm?
a) Digoxin
b) Amiodarone
c) Sotalol
d) Propafenone

42. Which test is required prior to dispense amiodarone, except?


a) Liver transaminases
b) Chest X-ray
c) TSH
d) Renal failure
Monitor LFT transaminases and Serum TSH Q 6m, eye exam, Chest X-Ray at baseline & annually

43. Amiodarone should not be used with which drug? Verapamil


Amiodarone should be used with caution in patients receiving β-receptor blocking agents (e.g.,
propranolol, a CYP3A4 inhibitor) or calcium channel antagonists (e.g., verapamil, a CYP3A4 substrate,
and diltiazem, a CYP3A4 inhibitor) because of the possible potentiation of bradycardia, sinus arrest, and
AV block; if necessary, amiodarone can continue to be used after insertion of a pacemaker in patients
with severe bradycardia or sinus arrest.
Simvastatin and atorvastatin  myopathy/rhabdomyolysis.
Trazodone, Loratadine  QT interval prolongation and torsade de pointes

44. Dronedarone is contraindicated in this patient beacause of?


a) Renal impairment
b) Hypothyroidism
c) HF

45. Which drug should be taken in the morning?


a) Amiloride
b) Amiodarone
c) Atenolol
When prescribed once daily, diuretics should generally be taken in the morning. When prescribed twice
daily or more, the last dose should be taken before 4:00 p.m. to avoid diuresis during the night.

46. Alcoholic Patient has refill for Temazepam 15mg PO QHS PRN total Qty. 14 tablets. His
last fill was 6 days ago. Today he come to pick up his early refill He wanted 28 tablets more.
due to planning for USA trip for 4 days. What is your decision?
a) Contact to his doctor regarding early refill
b) Refuse to give and give him copy of Rx???
c) Give him 14caps and document cause of early refill
d) Give him 28caps
47. New pharmacy owner is going to arrange childhood vaccination seminar for pneumococcal
vaccine to parents (mom-dad). In these medical conditions, you can recommend this vaccine;
EXCEPT in?
a) Acute otitis media
b) Meningitis
c) CAP
d) Infective endocarditis

48. What do you check prior to dispense Isotretinoin? Psychiatric illness


Systemic Drug Therapy for Acne. Q. Retinoids Isotretinoin Accutane, Clarus, Epuris
It remains the most powerful anti-acne agent, with the majority of patients achieving clearing and sustained
remission, even in the most severe cases.
It is recommended as first-choice therapy for severe papulopustular or moderate nodular acne and for
nodular or conglabate acne for many reasons: clinical effectiveness, prevention of scarring and quick
improvement of a patient’s quality of life, including minimizing depression.
Avoid taking vitamin A as Oral isotretinoin is a natural metabolite of vitamin A.
0.5 mg/kg/day PO for the first month, increasing to 1 mg/kg/day as tolerated (with a goal cumulative dose of 120
– 150 mg/kg). Alternative lowdose regimen (particularly for treatment of resistant or quick-relapsing moderate
acne): 0.25–0.4 mg/kg/day.
A complete course of therapy consists of 12-16 weeks of Isotretinoin administration.
S.E: Teratogenicity. Common: mucocutaneous dryness, myalgia, arthralgia, photosensitivity, Headache. Rare:
hypertriglyceridemia, mood disorder, possibly suicide ideation, pseudotumor cerebri, erythema multiforme,
Stevens-Johnson syndrome, toxic epidermal necrolysis.
Blood potassium increased, blood alkaline phosphatase increased, blood bilirubin increased, blood urea
increased, elevated platelet counts, eosinophil count increased, false positive tuberculosis test, gamma-
glutamyltransferase abnormal, blood cholesterol increased, glucose urine present, haematocrit decreased, protein
urine, thrombocytopenia, WBC count decreased. Elevations in levels of serum creatine kinase CPK (monitor)
Patients should be advised to use a skin-moisturizing ointment or cream and a lip balm from the start of treatment
as isotretinoin is likely to cause dryness of the skin and lips. When necessary a sun-protection product with high
protection factor of least SPF 15 should be used.
It is recommended that blood donation for transfusion purposes be deferred during therapy with EPURIS and for
one month after discontinuation of treatment.
Patients who experience tinnitus or hearing impairment should discontinue EPURIS treatment and be referred for
specialized care for further evaluation.
Isotretinoin has been associated with inflammatory bowel disease (including regional ileitis, colitis and
hemorrhage) in patients without a prior history of intestinal disorders. Patients experiencing abdominal pain,
rectal bleeding or severe diarrhea should discontinue EPURIS immediately.
Tetracyclines: rare cases of benign intracranial hypertension (pseudotumor cerebri): allow 7 days washout after
stopping tetracyclines before starting isotretinoin.
The patient should have two negative pregnancy tests (β-hCG in urine or serum) before starting therapy with the
first pregnancy test conducted at initial assessment when the patient is qualified for therapy by the physician.
The patient then should have a second pregnancy test with a sensitivity of at least 25 mIU/mL with a negative
result, performed in a licensed laboratory, within 11 days prior to initiating therapy. The patient has had two or
three days of the next normal menstrual period before therapy is initiated.
Pregnancy test must be repeated monthly for pregnancy detection during treatment and at one month after
discontinuation of treatment. The dates and results of the pregnancy tests should be documented.
Females must not become pregnant while taking Isotretinoin or for at least one month after its discontinuation.
Treatment should be discontinued if the patient develops any of the following reactions: rash, especially if
associated with fever and/or malaise, conjunctivitis (red or inflamed eyes); blisters on legs, arms or face and/or
sores in mouth, throat, nose or eyes; peeling skin or other serious skin reactions.
The following tests are required before starting Isotretinoin, at first month, then as clinically indicated:
 Serum blood lipid determinations (under fasting conditions) should be performed before Isotretinoin is
given and then at intervals (one month after the start of therapy) until the lipid response to Isotretinoin is
established (which usually occurs within four weeks), and also at the end of treatment.
 Complete blood count and differential: for early detection of leukopenia, neutropenia,
thrombocytopenia and anemia.
 Liver function tests: Increases in about 15% of ALT, AST, ALP baseline levels have been reported.
Liver function tests should be monitored before treatment and at regular intervals during treatment (one
month after the start of treatment and at least three-month intervals thereafter) unless more frequent
monitoring clinically indicated.
 Blood glucose levels: all patients and in particular patients with known or suspected diabetes should have
periodic blood sugar determinations.
 Serious Adverse Event Warnings include psychiatric disorders (depression, psychosis and, rarely,
suicidal ideation, suicide attempts, suicide, and aggressive and/or violent behaviors). Monitor psychiatric
illness before dispensing isotretenoin.

49. What is contraindicated with isotretinoin?


a) Vitamin A
b) Tetracycline
c) Grapefruit juice
CTC: Isotretinoin is also contraindicated in the following conditions: breastfeeding women, hepatic and
renal insufficiency, hypervitaminosis A, patients with excessively elevated blood lipid values, patients
taking tetracyclines

50. There is not any standard guideline available for non-sterile compounding in community
pharmacy. For storage condition, what do you recommend?
a) Store at room temperature for 7 days
b) Use it for 3 months
c) Do not use it more than 6 months
NAPRA GUIDELINES:
 A storage procedure must be established that is consistent with any requirements of the
applicable pharmacy regulatory authority
 Active and inactive ingredients must be stored according to manufacturers’ recommendations, in
a manner that prevents cross-contamination
 Each finished product must be stored according to the requirements outlined in its Master
Formulation Record

51. patient traveling to endemic area with malaria for 4 wk, when should start prophylaxis?
a) When he arrives there or gets the symptoms
b) Take chloroquine starting 1 - 2 weeks before travelling
c) Apply a combination of sunscreen and DEET
52. How many 500 mg tablets are needed?
a) 12
b) 9
c) 18

53. What the patient should do before taking the drug?


a) Pneumonia function
b) Ocular test
c) LFT

54. The same patient is traveling with 8 years kid for 8-hour flight and asks what can he do
to decrease the kid’s nausea & Motion sickness of flight?
a) Scopolamine patch
b) Dimenhydrinate (Gravol)
c) Non-sedating OTC antihistamines such as loratadine (Claritine)

55. patient has BPH, HTN, Diabetes type2 and experiencing increase in prostate size. Which
drug is required for treatment?
a) Finasteride
b) Tamsulosin
c) Doxazosin

56. How long it takes to see improvement in symptoms?


a) 6 months
b) 3 months

57. What is finasteride side effect?


a) Erectile dysfunction
b) Hypotension
c) Flushing
d) Diarrhea

58. Which drug is contraindicated for this patient?


a) Pseudoephedrine
b) Saw palmetto
c) St Johns Wort
Risk Factors:
 Caffeine intake, smoking, fluid intake, high intensity physical activities.
 BPH, bowel problems (constipation, fecal impaction). Morbid obesity (notably abdominal).
 Medications (alcohol, alpha-agonists, alpha-antagonists, anticholinergics, cholinergic agonists,
diuretics, psychotropics, sedative hypnotics, sympatholytics)
 Elderly, Female gender, Pregnancy (or post partum). Restricted mobility/environmental issues
59. As a pharmcy manager, which drug you handle wearing gloves if broken or leaking?
a) Dutasteride (Child bearing age person shouldn’t handle finasteride tabs)

60. Drug should handle gently before inj


a) Erythropoietin
b) Muromonab
c) Letrozole

61. GI bledding Patient had endoscopy what to give to prevent variceal bleeding?
a) Propranolol
b) Ramipril
c) Hydralzine
d) Clonidine
Prevention of Variceal Bleeding
 1ry prophylaxis: NSBB (propranolol, nadolol) & carvedilol are valid first-line treatments.
 2ry prophylaxis: combination of NSBB (propranolol, nadolol) with endoscopic variceal band ligation.

62. C. Difficile scenario, no febrile, 8 unformed stools/day, no vomiting. He takes antibiotic


for infection few days ago and diagnose with C. Difficile. What’s appropriate statement?
a) She does not require treatment
b) She has mild-mod CDAD and requires antimicrobial treatment
c) She has severe CDAD and requires antimicrobial treatment
d) She has severe, complicated cdad and requires antimicrobial treatment

63. What is initial goal for first 24-hr?


a) Prevent spread & Hospitalization
b) Correction of electrolyte imbalances
c) C. Diff free in 10 days (treat for 10-14 days, goal is to eradicate C Diff)
d) Rehydrated & > 3 BM/day in 48h
64. What is initial treatment?
a) Metronidazole 500mg TID PO
b) Vancomycin 500 mg IV

65. Now, he experiences second occurrence. What drug is initiated?


a) Metronidazole 500mg TID PO
b) Vancomycin 500 mg IV

66. After medical check, dr figured that the Patient taking pantoprazole 40mg for acid reflux.
So, what action is required?
a) Stop pantoprazole
b) Continue pantoprazole
c) Change to H2RA

67. What is the difference between sinusitis and viral rhinitis?


a) Difficulty sleeping
b) Face pain
c) Fullness of ear
d) Symptoms > 1 weak
Sinusitis Viral rhinitis
Adults: persistent symptoms of URTI without improvement for at Characterized by a sore throat
least 10 days or worsening after initial improvement within 10 days, usually resolving within a few days,
with both nasal congestion/purulent discharge and facial pain, followed by nasal congestion,
with or without fever, maxillary toothache or facial swelling. rhinorrhea, sneezing and cough.
Nonspecific concurrent symptoms include headache, halitosis Nasal discharge can sometimes be
(unpleasant mouth odour), hyposmia (reduced ability to smell and to purulent and mistaken for bacterial
detect odours)/anosmia (loss of the sense of smell), ear pain/pressure, sinus infection.
fatigue and cough Fever is infrequent in adults but
Children: symptoms of acute bacterial rhinosinusitis are similar in common in children.
children but often also include irritability, lethargy, prolonged cough,
vomiting occurs in association with gagging on mucus.

68. Allergic rhinitis, dr. has prescribed mometasone nasal spray. Explain device technique?
a) Spray in one nostril and close another nose meanwhile (vice versa)
b) Use saline to remove thick mucus before using the spray
CTC: Shake container well before each use. Patients should be instructed on
the correct method of use, which is to blow the nose, then insert the nozzle
carefully into the nostril, compress the opposite nostril and actuate the
spray while inspiring through the nose, with the mouth closed
69. A patient has granisetron and dimenhydrinate prn for CINV with lots of other med. what
the first thing you get from Rx?
a) Ondansetron for acute chemotherapy induced N & V
b) Prochlorperazine for chemotherapy induced N & V
c) Dimenhydrinate don’t work for acute & delayed chemotherapy N & V

70. Ondansetron side effect? Headache and constipation


Very common: Headache. Common: Sensation of warmth or flushing, constipation. Local burning
sensation following insertion of suppositories. Local IV injection site reactions
Rare: QTc prolongation (including Torsade de pointes)

71. BJ Psycho Patient is smoker used to smoke half of packet every day and currently on
clozapine. Come for routine chek up at clinic following lab test value.
White blood cell 12000 (5000-11000), Neutrophil count 1.7 (normal 2.8-7.7)
RBC normal, Eosinophil less, Lymphocytes 0. what abnormality patient face?
a) WBC
b) Neutrophills
c) Lymphocyetes
d) Red blood cells
Need for regular (weekly for first 6 mo) blood monitoring WBC. Agranulocytosis is a side effect of
clozapine, is an acute condition involving a severe and dangerous leukopenia (lowered white blood cell
count), most commonly of neutrophils, and thus causing a neutropenia in the circulating blood.

72. Least symptom of clozapine


a) EPS
b) Myocarditis
c) Hypersalivation
d) Agranulocytosis

73. Late friday evening patient come for 2 days advance of clozapine until doctor return on
monday. He slept lately on Friday night and missed his blood work. There are no more
refills. What is your action?
a) Give refill & inform him to come back on Monday with blood work for follow up
b) Give 2days supply and follow up
c) Inform him to contact hospital emergency clinic
d) Refuse to dispense (don’t need it)
There was a huge debate on this question but I can say to you: No blood, no drug this is the big rule of
clozapine. Besides, we must know for how long this patient is using this drug as there is a pattern in the
analysis of CBC starting weekly then can be monthly or by wider intervals.
Now we must know the compete scenario for this patient. By other words if he in the beginning of the
therapy don't give and let him wait till Monday and we will consider this a case of missing doses.
If he is using it for a long time and his lab works is good, we can give an emergency supply for 2 days.
Finally, if no more details in the question, apply the rule of “No blood, No drug”.
74. Now BJ quite smoking what is your recommendation to change clozapine dose
a) Increase clozapine dose as decrease hepatic enzymes.
b) Decrease clozapine dose as decreae hepatic enzymes
c) Increase clozapine dose as increase hepatic enzymes
d) Decrease clozapine dose as increase hepatic enzymes
e) No change in clozapine
Inducers of CYP1A2 or CYP3A4 such as carbamazepine, phenytoin, rifampin or cigarette smoking may
reduce clozapine levels.

75. 54-year lady is diagnosed with HIV & depression. She was work as a prostitute in her
early 30-40 age, she also a drug abuser. Now 53 yo and not using any drug, what drug is
good for naive patient like her?
a) Zidovudine/lamivudine
b) Abacavir/lamivudine
c) Tenofovir/lamivudine
d) Didanosine/lamivudine
Combination antiretroviral therapy (cART) is the standard
of care and is defined as combination of at least 3 active
antiretroviral drugs.
Preferred N[t]RTI options are emtricitabine/tenofovir
(available as a fixed-dose combination),
lamivudine/tenofovir or abacavir/lamivudine (only for
patients who are HLAB* 5701 negative; available as a fixed-dose combination).

76. What is true regarding starting HIV cART treatment?


a) Viral load 5000 copies per ml
b) CD 4 500 or less
c) All patient irrespective to viral load or CD4 after confirm diagnosis
Antiretroviral therapy is recommended in all patients, with the possible exception of those consistently
maintaining both a CD4 count >500 cells/μL and a viral load <1000 copies/mL while off therapy.
77. As per current HIV guideline, which HIV infected patient is eligible to check hepatitis C?
a) All HIV infected patient
b) All HIV infected to those who have drug addiction history
c) Only for whom born after 1994
d) Only for those born b/w 1945 to 1994

78. Dr. prescribed darunavir / ritonavir (protease inhibitor) for her. Which of the following
combination therapy would you add for this patient?
a) Atazanavir/ritonavir
b) Emtricitbine/tenofovir
c) Lamivudine/raltegravir

79. She comes to ask for the reason of HIV, had a motorcycle accident in Thailand 8 y ago,
is it the reason? What aligns with pharmacist answer?
a) Veracity
b) Paternalism
c) Autonomy
d) Justice

80. Someone with signs of ketoacidosis, what to give IV:


a) Repaglinid
b) Aspart
c) Regular
d) Glargine
81. The duration of NPH can be stored in room temperature (after opening vial)?
a) 4weeks
b) 6weeks
c) 8 weeks
The standard recommendation from all the insulin manufacturers is that a vial of insulin you are
using can be kept at room temperature for up to 28 days.

82. A case 67 years patient has AF with hypertension (170/100) and history of cardiogenic
stroke 2 month ago, why we couldn’t use alteplase?
a) History of stroke
b) High blood pressure
Absolute contraindications:
Pericarditis, previous intracranial hemorrhage; known malignant intracranial neoplasm, known
cerebral vascular lesion, ischemic stroke within 3 months except acute stroke within 3 h; suspected
aortic dissection; active bleeding or bleeding diathesis (excluding menses); significant closed head or
facial trauma within 3 months.
Relative contraindications:
History of chronic severe, poorly controlled HTN, severe uncontrolled HTN (BP >180/110 mm Hg);
previous CVA more than 3 months prior or known intracerebral pathology not covered above;
traumatic or prolonged (>10 min) CPR or major surgery (<3 wk); noncompressible venous
punctures; recent (2–4 wk) internal bleeding; pregnancy; active peptic ulcer; current use of
anticoagulants.

83. He is on unhealthy diet but


doesn’t smoke. Which intervention
is the most appropriate?
a) Recommend DASH diet
b) Recommend alcohol
reduction
c) Low sodium
d) Wt loss & exercise

84. He has difficulty in swallowing.


Which healthcare professionals do you
refer patient?
a) Occupational therapist
b) Physiotherapist
c) Dietician
d) Chiropractor
85. The patient was taking aspirin prior
to stroke. Now, Dr. wants to recommend
other agent for secondary prevention.
Which medication would you suggest?
a) ASA/dipyridamole
b) Warfarin
c) Clopidogrel

86. What to give for hypertension?


a) ACEI + diuretic
b) ACEI + BB
c) Amlodipine

87. Lady is suffering from urinary tract infection. She is allergic to penicillin (Hives). She is
end of her first trimester. What is DOC?
e) Nitrofurantoin
a) Trimethoprim
b) Amoxicillin
c) SMX/TMP

88. Which of the following anti-epileptic drugs are associated with Weight loss?
a) Carbamazepine
b) Topiramate
c) Phenytoin
d) Gabapentin

89. Case about patient who has BPH, and taking drug therapy for treatment. He forgets
anything and his family name. which of his med contribute his symptoms?
a) Tolteridine
b) Doxazosin
c) Finasteride
Risk Factors for dementia
 Non modifiable: Age > 65, Family history, Genetics, Females > males (evidence is not strong)
 Modifiable: Vascular risk factors, Atrial fibrillation, hypertension, CHD, diabetes, obesity, smoking
 Others: History of head trauma, Lower socioeconomic class, Education, Depression & Alcohol
Medication history is important to rule out drug-induced cognitive impairment. Anticholinergic side effects
of medications can lead to cognitive impairment. Q. A few examples of drug classes commonly associated
with anticholinergic effects are:
 Antiemetics/antivertigo agents, e.g., dimenhydrinate, promethazine, scopolamine
 Antihistamines, e.g., diphenhydramine, hydroxyzine
 Antimuscarinics, e.g., darifenacin, fesoterodine, oxybutynin, solifenacin, tolterodine
 Antipsychotics, e.g., chlorpromazine, clozapine, olanzapine
 TCAs, e.g., amitriptyline, clomipramine, desipramine, doxepin, imipramine, nortriptyline, paroxetine
90. Dr diagnosed him with dementia. He takes anticholinestrase inhibitor as a drug of
therapy. What is goal for treatment?
a) Reduce repetitive questioning
Reduction in repetitive questioning is a common treatment goal. Monitoring is done by MMSE, an annual
decline of <2 points while on drug therapy indicates a beneficial effect and warrants continuation

91. What test to do before starting donepezil?


a) Methacholine challenge test
b) Kidney functions
c) Liver functions
d) Cardiac test ECG

92. A woman with hypothyroidism, what could be decrease?


a) TSH
b) fT4
c) T3

93. She was on daily levothyroxin. Now


she is pregnant, what appropriate action?
a) Increase dose by 50 %
b) Add T3
c) Reduce dose
d) Continue and no change

94. MP is a 63-year-old man with an active stage of herpes zoster (shingles). His shingles
symptoms started with unusual skin sensation, such as pain, itching, tingling and burning and
then painful rash appeared on the right side of torso and extended to front and back after 4 to
5 days. What is the correct statement regarding transmission of shingles to others?
a) Shingles is highly contagious and the virus spread through sneezing, coughing or
casual contact.
b) Shingles is not contagious because it causes by the same virus that cause chickenpox.
c) Shingles is highly contagious and the virus spread by direct contact with fluid
from the blister.
d) A person with shingles rash only can pass virus to children and pregnant women.
CDC: Cover the rash. Avoid touching or scratching the rash. Wash your hands often.
Avoid contact with the following people until your rash crusts:
o pregnant women who have never had chickenpox or the chickenpox vaccine;
o premature or low birth weight infants; and
o people with weakened immune systems, such as people receiving immunosuppressive medications
or undergoing chemotherapy, organ transplant recipients, and people with human immuno
deficiency virus (HIV) infection.

95. Dr. prescribed antiviral (acyclovir) therapy. But pain is still same. Now, which drug is
required for treatment?
a) Amitryptyline
b) Tramadol
c) Morphine
d) Codeine
Oral antivirals (acyclovir, famciclovir, valacyclovir) are most effective if started within 72 hours of the
onset of the rash. Even if the rash has not appeared, the sudden occurrence of severe, acute neuropathic
pain unilaterally in the forehead or thoracic area in an individual over 60 years of age is reasonable
cause to initiate antiviral drug therapy, since these agents are safe and well tolerated.
Amitriptyline Initial: 10–25 mg QHS po, Increase by 10–25 mg daily at weekly intervals, until pain relief
or side effects.

96. Shingles risk factors


a) Age > 65
b) Encounter with someone
with herpes zoster
c) Encounter with someone
with herpes simplex
You can only get shingles if you had
chickenpox in the past. At the time
you had chicken pox the virus was
stored in in nerve cells in the spine.
The virus remains in resting phase for
years. Stress, illness and age >50 can
cause it to become active.
97. Nurse did a dispensing error and gave wrong medication aripiprazole to patient. He got
wrong drug for 3 days. Which side effect may experience by patient?
a) Sedation
Other CNS Effects: Akathisia- Tremors- dizziness

98. Patient has diabetes type 2. He experiences hypoglycemia at mid night (3AM -
3.3mmol/L) and hyperglycemia in early morning before breakfast (FBG - High) -somogyi
effect. What is your recommendation?
a) Decrease the dose of bedtime insulin NPH

99. 25months old child diagnosed with recurrent AOM. He got first 1st occurrence 11months
ago and 2nd occurrence 2months ago. Patient’s mother is coming with high dose amoxicillin
prescription. Which drug is most appropriate for this patient?
a) Amoxicillin-clavulanate
b) Cloxacilin
c) Azithromycin.
d) Cefprozil
Alternatives if Alternatives if Treatment
Characteristics Therapeutic Tips First choice Treatment Failure Failure Noted on Day 10-
Noted on Day 3 28
0-6 weeks Investigate for Refer to N/A N/A
bacteremia. emergency for
AOM often due to further
gram-negative assessment; fever
bacteria may be related to
sepsis.
Age 6 weeks to 6 Treat most cases of Standard-dose High Dose High Dose
months AOM with antibiotic amoxicillin or amoxicillin / amoxicillin/clavulanate
for 10 days HD amoxicillin clavulanate or × 10 days or cefprozil
Age ≥ 6 months Consider deferring cefprozil or × 10 days or
with no risk treatment to see if cefuroxime axetil or cefuroxime axetil × 10
factors (no AOM resolves in 24- ceftriaxone IM/IV × days or ceftriaxone
frequent bouts of 48 h only if follow- 3 days IM/IV × 3 days
AOM and no up can be ensured Consider Consider
antibiotics in and if antibacterial tympanocentesis tympanocentesis
previous 3 months) therapy can be
Age ≥ 6 months initiated if High Dose
with risk factors symptoms worsen Amoxicillin
(received <2 y: treat for 10
antibiotics in days
previous 3 m) ≥2 y: treat for 5 days
Any Age Verify AOM High Dose Ceftriaxone IM/IV; 3 High Dose
Frequent AOM Treat AOM episode amoxicillin / days. amoxicillin/clavulanate
for ≥10 days clavulanate Consider × 10 days or
Consider conjugated tympanocentesis. Cefprozil × 10 days or
pneumococcal cefuroxime axetil × 10
vaccine if <5 y days or ceftriaxone
Give influenza IM/IV × 3 days
vaccine yearly, all Consider
ages tympanocentesis

100. What antibiotic shouldnot be given?


a) Cloxacilin
b) Azithromycin.
c) Cefprozil

101. Patient has HTN, Diabetes and acute exacerbation of COPD, he discharged from
hospital. Dr. Prescribed oral prednisone for 1week therapy. What is rational?
a) To improve lung function
Other Therapeutic outcomes: decrease hospital stay and increase recover time, decrease the risk of
subsequent exacerbation and hospitalization

102. He is on lots of medication and also a beta blocker. He can’t stand, his symptoms like
coldness of extremities, what is the cause?
a) BB
Precipitating factors and triggers:
 Cigarette smoking, heavy exposure to dusts and chemicals, air pollution, Age > 40 years old.
 Q. alpha1-antitrypsin deficiency, which has been associated with emphysema, liver disease, panniculitis
 An increase in oxidative stress in airways plays an important role in the pathogenesis of COPD.
 Drug may worsen disease or response to therapy: Beta-blockers, ACE inhibitors (causes dry cough)

103. What to add for controling his HTN as he has renal problem too? Nifedipine XL

104. Which antibiotic is required for his case?


a) Amoxi/Clav
b) Ciprofloxacin
c) Amoxicillin
Group Symptoms & risk factors Probable pathogen First choice antibiotic
Simple < 4 exacerbations in the past M. catarrhalis  Amoxicillin
exacerbations year S. pneumonia  Doxycycline
(COPD Increased sputum purulence + at Haemophilus spp.  SMX/TMP
without risk least 1 of: All for 5–7 days
factors) 1) Increased sputum
volume
2) Worsening dyspnea
Complicated As in simple exacerbation + at M. catarrhalis Preferred:
exacerbations least 1 of: S. pneumonia  Amoxicillin/clavulanate
(COPD with 1) FEV1 < 50% predicted Haemophilus spp.  Cefuroxime axetil
risk factors) 2) ≥ 4 exacerbations per Klebsiella spp. Both for 5 – 10 days
year Other gram-negative  Levofloxacin × 5 days
3) Ischemic heart disease pathogens Alternative:
4) Use of home O2 Pseudomonas spp.  Azithromycin × 3 days
5) Chronic oral Higher probability of  Clarithromycin × 5–10
corticosteroid betalactam resistance days
6) Antibiotic use in
previous 3 months

105. You are in a busy night shift in the pharmacy and you noticed one of the technicians is
doing a lot of data entry mistakes, what is the best action
a) Identify error, take him aside and tell him about his mistakes and how he should
handle them
b) Appoint another technician with him to enter drugs in system
c) Make a more capable technician take his place and assign him to a less demanding
job.
d) You enter the Rx during that shift

106. Patient gets capsaicin cream. What do you educate?


a) Burning sensation will go away with regular use
b) Use warm pack to increase absorption
c) Effect in a week
d) If no effect, stop in week

107. Arrange these drugs in sequence of narcotics(N), controlled(C), targeted subs(G),


prescriptions. Buprenorphine, lactulose, methylphenidate, diazepam, clozapine, risperidone,
tramadol
a) 1, 1, 1, 4
b) 1, 1, 3, 2
c) 2, 1, 2, 2

108. pt injected Twinrix and got allergy on his right hand. Now epipen should be injected at?
a) Right hand IM
b) Right hand SC
c) Left hand IM  Contralateral Arm
d) Left hand SC
109. Epinephrine question after allergic reaction. Now, patient is conscious but experiencing
itching after epinephrine injection all other symptoms resolved. What do you recommend?
a) Give diphenhydramine orally

110. Which drug is not required refrigerator storage condition?


a) Trifluridine
b) Cyclosporine
c) Latanoprost
CPS: Cyclosporine Oral Solution USP should be stored and dispensed in the original container. Store
between 15 and 30°C, preferably not below 20°C for prolonged periods, as it contains oily components of
natural origin which tend to solidify at low temperatures. Do not store in the refrigerator and protect
from freezing.

111. Travoprost side effect? Increases brown coloration of iris & thickness of eyelashes

112. Which drug is not required to SHAKE WELL BEFORE USE?


a) Dexamethasone/Tobramycin Drop
b) Symbicort Turbuhaler  needs priming
c) Nepafenac eye drops
d) Pyrantel pamoate solution
e) Fluticasone nasal spray
f) Betoxolol eye gel

113. Goal of therapy for Glucoma?


a) Preserve optic nerve function
b) Prevent from progressing to closed angle glucoma
c) Prevent spontaneous detachment of optic nerve
d) Prevent ARMD

114. Patient has glucoma. Which drug is contraindicated?


a) Corticosteroids (Increase IOP)
Type of Open-Angle Glaucoma Angle-Closure Glaucoma
Glucoma Acute (emergency) Chronic
Acquired, Elevated IOP (the most important and only Female gender, Advanced Advanced age,
Primary modifiable risk factor), Advanced age, age, Positive personal Positive family
Black & Hispanic ethnicity (Mexican history (in the contralateral history, Hyperopia
ancestry), Positive family history, Myopia eye), Positive family history,
(Nearsightedness), Vascular diseases such as Hyperopia (farsightedness),
migraine, hypertension or nocturnal White, Inuit & Chinese
hypotension, Type 2DM ethnicity
Acquired, Blunt or penetrating trauma, Previous Proliferative diabetic
Secondary intraocular surgery, Previous intraocular retinopathy and central
inflammation, Corticosteroid use, retinal vein occlusion
Pseudoexfoliation syndrome, Pigment (neovascular angleclosure
dispersion syndrome glaucoma)
Congenital Positive family history

115. Sensitivity error calculation Error 5%? 90 mg

116. Allegation method calculation?


a) 48.75ml

117. Patient has diabetes. He takes metformin 850mg BID, gliclazide BID. He takes rum,
wine with dinner daily. But he doubles the drink in weekend. What patient my experience?
a) Experience acute hypoglycemia in weekend

118. all of these are symptoms of hypoglycemia except?


a) Sweating
b) Hypotension
c) Shakiness
d) Palpitation

119. You are putting on an education session for nurses in the perioperative ward of
the hospital. What information is important to include in this presentation:
a) Cost of the newest medications used for this indication
b) Rationale behind providing-operative pain control
c) How to recognize a patient’s abuse potential
d) Mechanism of action of the newest pain control medication on the market.
120. Patient has severe pyelonephritis, with chills, fever, flank pain. Which drug is initiated
after discharge from hospital?
a) Amoxi/clav
b) SMX/TMP
c) Levofloxacin
d) Gentamycin
Mild to E. coli (90%), P. mirabilis & K. pneumoniae Fluoroquinolone Amoxi/clav PO or
moderate (5%) (ciprofloxacin, SMX/TMP PO or
pyelonephritis Occurs in women who experience recurrent levofloxacin, Trimethoprim PO
OR uncomplicated UTIs but at lower frequency norfloxacin), all for 10-14 days
Acute than cystitis. PO × 7–14 days
Nonobstructive Classic presentation includes fever, N&V,
Pyelonephritis flank pain with or without associated irritative
Severe urinary symptoms. Aminoglycoside IV Fluoroquinolone
pyelonephritis Patients who present with UTI with only lower ± ampicillin IV for IV × 10–14 days
urinary tract symptoms or asymptomatic initial therapy; or
bacteriuria occasionally have associated occult if appropriate, step 3rd generation
renal infection. down to oral cephalosporin IV ±
Bacteremic infection occurs most frequently in therapy as in mild aminoglycoside IV
diabetic women or women >65 y. to moderate × 10–14 days or
Urine Culture: always recommended. infections in order Carbapenem IV ×
Consider blood cultures. to complete 10–14 7–14 days for
days ESBL-producing
organisms

121. Which reference is recommended to find out drug drug interaction?


a) Micromedex
b) RxFiles
Micromedex facilitates evidence-based decision making in the areas of drug selection, disease
and condition management, toxicology, and alternative medicine. It even addresses specific
questions, such as dosing and IV compatibility.

122. Least weight causing antidepresant. Where to look for?


a) eCPS
b) Drug Product database
c) RxFiles  Preferred source for drug comparisons

123. Doctor intern ask you about hypovolemia in hyponatremia. Where and how to search?
a) Micromedex type hypovolemia
b) Pubmed type hypovolemia then clinical data
c) Pubmed type hypovolemia in hyponatremia
d) Google scholar search management of hypovolemia in hyponatremia
Another version: Intern asking about price of insulin injection. Where to look for?
124. NNT Calculation - drug-126(live -33) and placebo-127(live -56)

125. Calculation for Relative risk reduction (RRR = ARR/CER)

126. Calculation answer 4ml diluent and 1ml stock

Two Calculations of TPN


127. A person with serum 64ml/hour rate of D/W 20% and 64ml/Hour of D/W 5% with
NaCl 0.9% If each 1 g dextrose give 10 cal how much is total calory intake in a DAY
Answer
D/W 20% = 64 * 24 = 1536ml
20 ------- 100 X -------- 1536 X = 1536*20 / 100 = 307.2gm
D/W 5% = 64 * 24 = 1536ml
5 ------- 100 X -------- 1536 X = 1536*5 / 100 = 76.8gm
Total dextrose = 307.2 + 76.8 = 384gm * 10 cal = 3840 calories

128. Another one something same but with AA serum and need the result by g/kg/day

129. Patient has renal dysfunction (Crcl <30ml/min)? Dabigatran increases risk of bleeding

130. Antidote of dabigatran? Idarucizumab

131. Dabigatran counselling? Don’t crush chew swallow with food


CPS: Before you start taking PRADAXA and regularly after, your doctor will test your kidney function.
PRADAXA should be taken with a full glass of water and can be taken with or without food. If PRADAXA
upsets your stomach, take it with meals or within 30 minutes after meals. If PRADAXA still upsets your
stomach, consult your doctor or pharmacist. It is important to continue taking PRADAXA as prescribed.
Swallow the capsule(s) whole. Do not chew or open the capsule. Do not sprinkle the pellets on food or
mix with liquids.

132. Initial goal of antidepressant therapy in 2-4 week?


a) To improve sleep
All somatic symptoms are expected to improve earlier than the mood ie first month. Insomnia can be
refractory to improvement and this is a different point

133. Inserted catheter gram negative organism?


a) S. epidermidis
b) E. coli
Indwelling catheter is a tube inserted into urethra:
 coli (50%), P. mirabilis (20%), Enterococcus faecalis (10%), Pseudomonas aeruginosa, Providencia
stuartii, Citrobacter spp., Enterobacter spp., Serratia spp., group B streptococcus
Central-venous-catheter-related bloodstream infections (CRBSIs):
 Gram-negative bacteria include Escherichia coli, Klebsiella pneumoniae, Pseudomonas
aeruginosa, Enterobacter aerogenes and cloacae, although a vast array of organisms can be
responsible. Infections caused by Pseudomonas and Klebsiella species occur more commonly in
patients with serious underlying diseases (e.g., extensive trauma, burns, or malignancy)
 Gram-positive organisms: Staphylococcus epidermidis is the most common gram-positive
organism isolated from blood (~ 30% of isolates) and accounts for the majority of infections that
are associated with an intravascular catheter. Staphylococcus aureus also causes a significant
number of bloodstream and intravascular catheter infections and the incidence of clostridial and
streptococcal infections has increased

134. A patient is admitted to the hospital and was put on IV infusion. After few days, she
developed infection caused by coagulase (-) gram (+) microorganism from her blood culture.
Which of the following is the pathogen?
a) Pneumococcus
b) Strep pneumonia
c) Staph epidermidis
d) Strep group B
Coagulase (-) gram (+) – S. epidermidis; Coagulase (+) and gram + ve is S. aureus

135. cellulitis infection with maceration, erythema and foot swelling and had maceration
between toes and he is prediabetes, what can be the cause of developing cellulitis?
a) Tinea pedis
b) He is prediabetes
Contributing factors including edema, stasis dermatitis and interdigital tinea pedis should be identified
and treated in all patients

136. Cellulitis organism? Caused by S. Aureus or beta-hemolytic streptococci


137. If that case with edema what the non pharmacological measures?
a) Elevate his leg
b) Soak in saline water

138. What is drug of choice for Cellulitis?


a) Cephalexin PO
b) Cefazolin IV

139. How to apply fentanyl patch? https://medlineplus.gov/druginfo/meds/a601202.html


1. Clean the area where you plan to apply the patch with clear water and pat completely dry. Do not
use any soaps, lotions, alcohols, or oils.
2. Tear open the pouch containing the fentanyl patch along the dotted line, starting at the slit.
Remove the patch from the pouch and peel off both parts of the protective liner from the back of
the patch. Try not to touch the sticky side of the patch.
3. Immediately press sticky side of the patch onto the chosen area of skin with the palm of your hand.
4. Press the patch firmly for at least 30 seconds. Be sure that the patch sticks well to your skin,
especially around the edges.
5. If the patch does not stick well or comes loose after it is applied, tape the edges to your skin with
first aid tape. If the patch still does not stick well, you may cover it with Bioclusive or Tegaderm
brand see-through dressings. Do not cover the patch with any other type of bandage or tape.
6. If a patch falls off before it is time to remove it, dispose of the patch properly and apply a new
patch. Leave the new patch in place for 72 hours.
7. When you are finished applying the patch, wash your hands with water right away.
8. When it is time to change your patch, peel off old patch, apply a new patch to a different skin area.
9. After you remove your patch, fold it in half with the sticky sides together and flush it down a toilet.

140. Hospital pharmacy carry diclofenac/misoprostol combination. What is appropriate


action to prevent harm?
a) Avoid it from staff who have ASA induced asthma
b) Avoid it from pregnant staff

141. A hospital wants to know the difference of cost for two medication for treatment of
disease, where to check?
a) CADTH CDR
b) Health Canada
c) CTC

142. Physician prescribed drug X to patient, which is recalled by manufacturer after few days
(Drug recall type 1). Manufacturer inform pharmacist to recall drug. What is pharmacist action?
a) Inform patient & physician about recall drug X and inform patient to see his Dr.
Type 1: serious adverse health consequences or death, alert the patient
Type 2: temporary health consequences or where the probability of serious adverse health consequences
is remote, do not alert patient
Type 3: use or exposure to a product is not likely to cause any adverse health consequences
143. There was one question if acetaminophen recalled. What is not to be done??
a) Contact the patients
b) Take off the drugs from shelf
c) Put note in pharmacy or put posters at counter
d) Note the pack size of drug recalled. ⁠

144. Pharmacist received compounding prescription. But he is short of few ingredients to


prepare it. What is pharmacist next step?
a) Contact to nearest institution to get remaining ingredients

145. Calculation of Meq and mmol. How to convert mEq to mmol?


 mEq/L = mmol/L x valence mEq/L = (mg/dl x 10 x valence) ÷ mol wt

146. MP’s physician diagnosed microcytic hypochromic anemia (Hb 89g/L) and ask the
pharmacist to suggest an alternative form of oral iron to provide an equivalent amount of
elemental iron as FeSO4.7H2O 300mg tid. Molecular wt or atomic wt are:
Ferrous gluconate (FeC12H22O14) = 446.
Ferrous Sulpahte heptahydrate (FeSo4.7H2O) = 277.9 Iron (Fe) = 55.8.
What is the equivalent iron dosage to be administered daily as the gluconate salt?
First, calculate the total amount of elemental iron in FeSO4.7H2O 300mg tid 300 mg tid = 900mg daily
MW of ferrous sulphate heptahydrate = 277.9 277.9 = 55.8gm of iron
900mg would be = 900*55.8/277.9 = 180.7mg elemental iron per day
So, you calculate the amount of ferrous gluconate which will give him 180.7mg elemental iron
MW of ferrous gluconate is 446 446gm contains 55.8gm elemental iron
So, how much will contain 180.7mg = 180.7*446/55.8 = 1444mg ferrous gluconate

147. Nurse made dispensing error in hospital. who will contacted first by pharmacist after
identifying error? Hospital physician  It is better to inform head department of the nurse

148. Which medication doesn't require any prior authorization for destruction of expired med
from office of controlled substances & do not need witness?
a) Clozapine
b) BDZ.
c) Methadone
d) Methylphenidate (Ritalin)

149. After purchasing narcotics from wholesaler, pharmacist have to return the signed receipt
to wholesaler within how many days? With in 5 days
Requirements regarding Narcotic Invoices
 On receipt of the narcotics, purchases must be recorded on the Narcotic Purchase Record
 Pharmacists should be aware that any narcotic order placed through a computer or remote input
device requires the pharmacist to return the signed receipt to the Licenced Dealer in 5 days.
 If receipts are not received by Licenced Dealer, further orders can be withheld from the pharmacy.
150. Patient has kidney dysfunction
and receiving vancomycin 1g Q12H
in hospital. What dosage adjustment
is needed?
a) 1g IV Q24H (keep dose and
increase interval)
b) 1g Q12H
c) 1.5g Q24H
d) 2g Q24H
The reference range for vancomycin
trough levels is 10-20 µg/mL (15-20
µg/mL for complicated infections). The
reference range for vancomycin peak
levels is 25-50 µg/m
There was another question of
vancomycin peak level? I chose
vancomycin peak levels never used
for therapeutic montitoring. It is
vancomycin trough that is used

151. Person 100kg is coming with


concern to lose his weight in
pharmacy. What is your first
appropriate question?
a) Ask weight and height to measure BMI to provide appropriate guidance
b) If he has tried anything else so far (if the height is written and he is not obese)
c) Obesity comes with lots of problems do you have any specific problem which would
like to discuss apart from meal replacement.

152. Patient takes warfarin - sun tue thurs sat - 6mg,


mon wed fri - 4mg. Now, his INR values are come
up with higher than usual range constantly. How to
manage this situation?
a) Mon tue wed thurs fri - 3mg, sat - skip
warfarin dose
Another version
Warfarin Blister pack question. Patient on warf 4mg
5 days of the week and 2 mg 2 days of the week. All
warfarin 2 mg tabs throughout the Bp. After InR
results weekly dose needs to be reduced by 20%.
How would you choose regimen so that patient can keep using 2 mg tabs? I chose warfarin 4
mg 6 days if the week no warfarin on 7th day 4 mg in 3days and 2 mg in 4 days
A patient comes with antibiotic X. Pt is also on warfarin and his INR is 2.3. Antibiotic X
interacts with warfarin. If Friday night and you can’t contact the doctor. What do you do?
a) Tell pt to take half dose of warfarin until they see the doctor
b) Follow up on Monday (give the antibiotic and followup)
c) Fax physician’s office and they will see it on Monday
d) Fax lab and tell them about the interaction

153. Dose of valacyclovir for recurrent


anogenital warts
a) 500 mg or 1000 mg, once-daily
dosing for 2 -12 months
b) 1000 mg PO TID for 7 days  this
is for herpes zoster

154. Patient has stable angina. Which drug


is required to reduce mortality and improve
quality of life?
a) Nitroglycerine sublingual spray

155. patient taking pantoprazole for 3 years. Symptoms free for last two years. What to do?
a) Step down
b) Refer for again endoscopy
c) Stop therapy
d) Tapering

156. Now, she gets second occurrence. What is


next appropriate therapy?
a) Reinitiate PPI for another 4-8weeks
b) Recommend to go for endoscopy
c) Add H2blocker
d) Taper regimen
157. What to give to get ready for endoscopy?
a) PEG3350
b) Electrolytes
c) Magnesium sulfate (Epsom Salts)

158. After endoscopy, Dr. diagnose Heart failure. Initial treatment for HF? ACE inhibitor

159. She developed edema, and Cr. cl < 60, She has allergy to furosemide, what to give
a) HCTZ
b) Bumetanide
c) Ethacrynic acid

160. What do you check for this patient on daily basis? Weight

161. There is a shortage of levofloxacin in hospital and notified by manufacturer. Or IV


levofloxacin is backordered What is an appropriate alternative for respiratory Infections.
How to manage this situation?
a) IV moxifloxacin
b) IV ciprofloxacin
c) IV metronidazole

162. A new drug has entered the formulary which is the brand of infliximab, what is true?
a) It is automatically substituted for infliximab
b) It has the same active ingredient and route of administration

163. There is increase in prescription for drug A (ACEI) in community pharmacy, what could
be the reason?
a) A shortage of the other ACEI drugs.
b) The drug company is giving physicians samples of drug A
c) Reps. detailing to docs

164. New clinic opened and there is a high demand in drug X. As a pharmacy manager, you
want to buy large stock of drug X, what is most important consider?
a) The demand of drug x in the local area
b) Side effects of drug X
c) Having updated knowledge about mechanism and pharmacology of drug X

165. A cancer Patient taking bisacodyl for her constipation. She is having one bowl movement
daily, but in the last two days she had no bowl movments. what will you do if you know that
she increased the dose of bisacodyl from one tab to 5 tabs over one year?
a) Refer to her physician for assessment of her case
b) Continue same medication and increase fiber intake
c) Switch to another laxative (don’t remember which)
d) Dietary fibres and check for improvement after 3 days
Options include:
 Continue and optimize nondrug
measures (eg, senna 2 tablets at
bedtime to a maximum of eight
tablets/day), or
 Try a combination of stimulant (senna
or bisacodyl) and osmotic laxatives
(lactulose or PEG 3350).

166. Sertraline counselling?


Sertraline can be taken with food or on an empty stomach. It is best to give the medicine in the morning.
If two doses a day are required, it is best given in the morning and the early afternoon. If your child
misses a dose of the medication it can be given as soon as you remember.

167. Patient with diabetes and take metformin and sitagliptin, HbA1c is 7.1 what we can do?
a) Add glyburide

168. A case for otitis media. 26 months baby


that had otitis in 11 months and 2 months ago.
And was treated with high dose amoxicillin.
Her mother comes and you counsel for?
a) It takes 2‐3 days after taking
medication to get rid of fever
b) Use cold compresses on the ear to help
with the pain

169. All the following antibiotics can be used for the baby except
a) Cloxacillin & Cephalexin
b) Amoxi/clav
c) Azithromycin

170. Which registered technician can't do but only pharmacist can do in hospital pharmacy?
a) Preparing sterile products using aseptic techniques
b) Collect best medication history
c) Calculation for compounding
d) Report adverse effects of drug to Hospital
171. Nurse from care house LTC calls to tell you that they don’t dispense medication to the
patient unless it’s in blister pack, what will you do to uphold patient autonomy?
a) Call the patient and ask him if he prefers his meds in a blister pack
b) Re do the medication in a blister pack and send it

172. What infection that technician should not do compunding or near compunding?
a) Herpes labialis
b) Blepharitis
c) Aphthous ulcer
d) Otitis media
e) Hepatitis

173. What is the treatment for cradle cap in a 2-month-old child?


a) Massage baby’s scalp with olive oil before washing with non-medicated shampoo
b) Shampoo with 0.5% zinc pyritheone
c) Hydrocortisone 0.5%
If scales do not loosen easily, mineral oil or petroleum jelly. Last resort: Mild CS Cream or antifungal sh.

174. Case about focal seizure, patient is positive human leukocyte antigen (HLA) HLA-
B*1502, What is the treatment?
a) Carbamazepine
b) Oxcarbazepine
c) Levetiracetam
HLA Association with Drug-Induced Adverse Reactions
 Abacavir Hypersensitivity and HLA-B∗57:01 (Skin)
 Carbamazepine Oxcarbazepine Hypersensitivity and HLA-B∗15:02, HLA-B∗15:11, HLA-A∗31:01 (Skin)
 Allopurinol Hypersensitivity and HLA-B∗58:01 (Skin)
 Dapsone Hypersensitivity and HLA-B∗13:01 (Skin)
 Amoxicillin-Clavulanate-Induced DILI and HLA Haplotypes
 Flucloxacillin-Induced DILI and HLA-B∗57:01 Association
 Antithyroid Drug-Induced Agranulocytosis and HLA-B∗38:02-HLA-DRB1∗08:03 Haplotype

175. Patient taking Oxcarbazepine for seizure, what may be beneficial for seizure to take?
a) Omga 3 FA
b) Folic acid
c) L- carnitine
d) Vitamin E
Potentially beneficial dietary interventions in the treatment of epilepsy include identifying and treating
blood glucose dysregulation, identifying and avoiding allergenic foods, and avoiding suspected triggering
agents such as alcohol, aspartame, and monosodium glutamate.
The ketogenic diet may be considered for severe, treatment-resistant cases. The Atkins diet (very low in
carbohydrates) is a less restrictive type of ketogenic diet that may be effective in some cases.
Nutrients that may reduce seizure frequency include vitamin B6, magnesium, vitamin E, manganese,
taurine, dimethylglycine, and omega-3 fatty acids.
Administration of thiamine may improve cognitive function in patients with epilepsy.
Supplementation with folic acid, vitamin B6, biotin, vitamin D, and L-carnitine may be needed to prevent
or treat deficiencies resulting from the use of anticonvulsant drugs.
Vitamin K1 has been recommended near the end of pregnancy for women taking anticonvulsants.
Melatonin may reduce seizure frequency in some cases, and progesterone may be useful for women
with cyclic exacerbations of seizures.

176. Question regarding tonic clonic seizures, all are first line except?
a) Carbamazepine
b) Lamotrigine
c) Phenytoin
d) Levetiracetam
Tonic-clonic (“grand mal”) seizures: 1st line 2nd line or add on
can begin as a focal seizure (secondarily carbamazepine brivaracetam
generalized) or can involve the entire brain lamotrigine clobazam
at onset (primarily generalized), fairly levetiracetam lacosamide
uniform sequence of motor features (tonic oxcarbazepine perampanel
and clonic phases), impaired consciousness, duration of 1–2 minutes, valproic phenytoin
postictal stupor, confusion and headache acid/divalproex topiramate

177. Hospital management wants the pharmacist to take an action to decrease the ISMP
errors, what pharmacist should do?
a) Schedule seminars for physicians and nurses to educate
b) Print the ISMP’s list of dangerous abbreviations and place it in every department
c) Audit and give feedback to physicians on their usage of dangerous abbreviations???

178. What is correct regarding insulin pens?


a) Prime every time before use
b) Not suitable for patients with impaired sites
c) Good for mixing different types of insulin

179. Which of these do not need Sale record?


a) Dextropropoxyphene
b) Buprenorphine patch
c) Butorphanol
d) Diazepam

180. What is the percentage of alcohol in hand sanitizer?


a) 70%
b) 95%
For alcohol-based hand sanitizers, the Food and Drug Administration (FDA) recommends a concentration
of 60% to 95% ethanol or isopropanol, the concentration range of greatest germicidal efficacy.
181. Woman use triphasic COC and going for
one-week vacation, so want to delay menses for
one week
a) Leave last week in the 1st pack & start
new pack

182. Patient with ASA toxicity and on patient


profile is pco2 is 30 (normal is 32-34), Po2 is
normal, bicarbonate is 4(6-7 normal) and PH is
7.3 (7.45-7.55 normal), so the patient is having?
a) Metabolic acidosis with respiratory
compensation
b) Metabolic alkalosis with respiratory
acidosis

183. A 15 years old patient came with a ciprofloxacin prescription for 3 days as she
experienced traveler’s diarrhea before. She wants to go Honduras and took 2 doses of
dukoral 3 years, why will the pharmacist refuse to dispense the ciprofloxacin
a) There is a ciprofloxacin resistance in this area
b) For prophylaxis ciprofloxacin has to be started 2 days before travel and every day
during the trip
c) The safety of ciprofloxacin is not well established under 18 years old
The safety and efficacy of ciprofloxacin hydrochloride tablets and ciprofloxacin injection in children and
adolescents (under the age of 18 years), pregnant and nursing women has not yet been established.
Damage to juvenile weight-bearing joints and lameness were observed both in rat and dog studies but
not in weaned piglets. Histopathological examination of the weight-bearing joints in immature dogs
revealed permanent lesions of the cartilage.

184. Organism responsible for


Intra-abdominal infections?
a) Bacteroides fragilis
b) E. coli
c) S. aureus

185. In pharmacy they wrongly


give amlodipine & nimodipine,
what is the cause of this error?
a) Look-alike/Sound-alike
(LASA) Drug Names
186. Statin myopathy happen with
a) High dose
b) Long term use
c) Flouroquinolones
d) acute or chronic renal problem
Patient Factor Concomitant Medications Statin Properties
 Advanced age (>80 y)  Amiodarone  High dose statin
 Alcohol abuse  Azole antifungals  High bioavailability
 Chronic renal insufficiency (itraconazole,  Limited protein
 Excessive grapefruit juice intake ketoconazole) binding
 Frailty, small body frame  Calcium channel blockers  High lipophilicity
 Gender (women > men) (nondihydropyridines)  Potential for drug-
 Hepatic dysfunction  Cyclosporine drug interactions
 Hypothyroidism (untreated)  Fibrates (particularly metabolized by
 Inherited myopathies gemfibrozil) cytochrome P450
 Intercurrent infections  HIV protease inhibitors pathways,
 Perioperative periods  Macrolides especially 3A4 and
 Vigorous exercise (clarithromycin, 2C9
 Vitamin D deficiency erythromycin)
 Nicotinic acid (rarely)

ADHD scenario 5 questions


187. Mom came with 8-year-old kid, wants ADHD med but not stimulant, what first line?
a) Atomoxetine
b) Bupropion
c) Clonidine

188. It doesn’t work, she now wants a stimulant. Doctor prescribes Methylphenidate
extended - release 36 mg. What is the issue?
a) Methylphenidate is not a first-line stimulant for ADHD
b) Methylphenidate is not to be used in patients under 12
c) Methylphenidate dose is too high for initiation (trial is from 3-4 weeks, continue for
6-12 months if works, start at 18mg)
Initial: 0.3 mg/kg/day po
Usual: 0.15–1 mg/kg/day po or 10–60 mg/day po in 1–3 divided doses
Concerta XL may not be indicated in all children with ADHD syndrome. Lower doses of short-acting
methylphenidate formulations may be considered sufficient to treat patients new to methylphenidate.
Careful dose titration by the physician in charge is required in order to avoid unnecessarily high doses of
methylphenidate. The recommended starting dose of Concerta XL for patients who are not currently
taking methylphenidate, or for patients who are on stimulants other than methylphenidate, is 18 mg
once daily.
The recommended dose of Concerta XL for patients who are currently taking methylphenidate three
times daily at doses of 15 to 45 mg/day
189. You inform mother that methylphenidate can cause stunted growth. She grows
anxious and says “I don’t want my child to not grow normally!”
a) Inform her that it is not a big deal
b) Tell her you understand her concern and that you will work together to monitor
and minimize the risk
c) Inform her that in clinical trials, patients on the medication only were 2 cm shorter
than the placebo group

190. What is the reason for Drug holiday with stimulants: To assess if they still have ADHD
Weaning the medication for a 2- to 3-week period once a year (usually in the summer months) may
provide an opportunity to reassess ADHD-related behaviours and to confirm whether the stimulant is still
required for the next school term.
Extended drug holidays, e.g., several months over summer holidays, are generally not recommended in
children with moderate to severe ADHD symptoms who are doing well on the medication.

191. Pharmacist doesn’t like giving plan B because of religious reasons. Your moral values
go against dispensing plan B. Your refusal is conflicting with which ethical value
a) Justice
b) Beneficence
c) Veracity
d) Fidelity
e) Non-maleficence

192. 125 kg woman on phenytoin for seizures comes in for plan b & donot like condoms.
This is second time in the last 6 weeks she takes it. What counselling
a) Wont work as well d/t phenytoin
b) Menses will occur in 5-7 days
c) It is not safe method
d) Efficacy of plan b is lower during time
Another version
26-year-old lady on phenytoin. What to counsel?
a) Oral contraceptives increase effectiveness of Phenytoin
b) Weekly monitoring to achieve dose
7 to 10 days may be required to achieve therapeutic blood levels with DILANTIN and changes in dosage
(increase or decrease) should not be carried out at intervals shorter than 7 to 10 days.

194. What contraception to give for long-term use?


a) Only levonogestraol based pills
b) Evra patch
c) Levonorgestrel IUD
d) Norgestimate 350
e) EE35/Norgestimate250
195. BL is a 65-year-old male who has recently been diagnosed with typical chronic stable
angina. He only has angina if he takes the stairs up to his fourth-floor apartment too quickly.
Otherwise, the symptoms are not significantly affecting his daily activities. He also has type
2 diabetes and hypertension. He does not smoke and has no known allergies. His father died
of a heart attack at age 68. BL"s medications include metformin 500 mg BID and
hydrochlorothiazide 12.5 mg daily.
According to the above information, how many CHADS2 score does BL have?
a) Two
b) Three
c) Four
CHADS2 score  Congestive heart failure 1, Hypertension 1, Age >75 years (female >65years) 1, Diabetes
1, prior Stroke 2

196. Cat bite, what antibiotic? Amoxi/clav


 Human or animal bites (e.g., cat, dog): Treat with amoxicillin/ clavulanate.
 Pasteurella multocida is commonly present in animal bites, and resistance of this organism to
cloxacillin, cephalosporins and clindamycin is documented.
 Options for second-line therapy of animal bites include combination therapy with either
clindamycin or metronidazole plus one of a second- or third-generation cephalosporin,
doxycycline, sulfamethoxazole/ trimethoprim or a fluoroquinolone.
 Second-line options for human bites include metronidazole combined with either ciprofloxacin or
levofloxacin, or monotherapy with moxifloxacin.

196. Question about needle size and


diameter suitable for a kid

197. The function of Aluminium


phosphate in Twinrix vaccine?
a) Stability
b) Preservative
c) Inhance immune action

198. Physician asked about new evidence base treatment of a disease. Where to look?
a) Cochrane library

199. Pregnant women for morning sickness. She is in her 2nd trimestre feel nausea and asked
for non pharmacological therapy for nausa in pregnancy, where to look?
a) CTMA
b) Briggs

200. 3 questions on abbreviations/safety (look over ISMP list of unsafe abbreviations)


ISMP tallman lettering? DOPamine and DOBUTamine
201. This q came but different numbers. The label of a dry powder for oral suspension states
that when 111 mL of water are added to the powder, 150 mL of a suspension containing
250mg/5mL ampicillin are prepared. How many ml of water should be used to prepare a
suspension which would contain a dose of 8 mg/kg per 5 mL. to be given to a child weighing
60 lbs using same total amount of drug as of 150 ml of 250mg/5ml suspension.
250mg = 5ml (since that’s the strength) Total volume = 150ml
So, total weight would be 150*250/5 = 7500mg of ampicillin contained in the bottle
Adding 111ml to form 150ml  means, 7500mg ampicillin is occupying 150 -111 = 39ml
Weight of child = 60lbs = 60/2.2 = 27.3kg
The dose is 8mg/kg, so total amount of drug per 5ml would be: 8*27.3 = 218.4mg/5ml
So, we know the total drug in the bottle is 7500mg
We know that we need a dose of 218.4mg/5ml
So, the question is, if 218.4mg = 5ml 7500mg = ??ml. So, 7500*5/218.4 = 171. 7mls
This is the total volume of liquid in the bottle after mixing. However, we know that 39ml is occupied by
the ampicillin powder So how much water are we going to add???
Subtract: 171.7 - 39 = 132.7ml Answer

202. This q came but different numbers. If the suggested continuous infusion rate of a drug is
10 mg/kg/hr q8h, how many ampoules of the drug would be needed daily for a 100 kg
patient if each 25 mL ampoule has a concentration of 10 mg/mL?
First find out how much drug the patient will need 10mg/kg/hr
He weighs 100kgs So that is going to be 1000mg/hr= 24000mg/Day 24000 /3 = 8000 q8h
Now, each 25ml ampoule = 10mg/ml
Therefore, 25ml would be 25*10 = 250mg (in each ampoule). We need 24,000mg daily.
So, how many ampoules? 24000/250 = 96 ampoules (Answer)

203. You have 20 ml ampoule having a concentration of 0.75%, and you need to prepare 250
ml solution having a concentration of 0.125%, using 20 ml ampoules so what is the volume
of liquid needed to dilute the solution:
C1V1 = C2V2  0.75 x V1 = 250 x 0.125  V1 = 250 x 0.125 / 0.75=41.67ml
So, amount of liquid needed to add is = 250 – 41.67 = 208.34ml
OR
100ml contain 0.125mg. So, 250 ml contains = 0.312
100ml contains 0.75. So, 20ml contains= 0.15gm
0.15gm present in 20ml. So, 0.312gm will be in = 0.312 x 20 / 0.15 = 41.67
So, amount of liquid needed to add is = 250 – 41.67 = 208.34ml

204. Mark up percentage and gross margin?


The acquisition cost of tacrolimus is 2.15 $/ gm. & the mark-up is 15 % & the dispensing fee
is 11 $ what would be the price for 30 gms of tacrolimus.
So cost is $2.15/ g and we need the cost for 30  $2.15/ g x 30 g= $64.5
then markup is %15 so we have to divide the cost by (1-0.15)  64.5/ (1-0.15) = $75.88 and now we add
the dispensing fee of $11 so $75.88+$11= $86.88
205. Case with some medications and one of them an ACEIs with K 6.4 mmol/L. which
Antibiotic is not good for his infection?
a) Cotrimoxazole as it gives Hyperkalemia too
Penicillin G potassium also increases K
level

206. A father wants something for


his baby’s shiny rash on his leg. He
uses petrolatum for each time
changing baby’s diaper and now
with 2 days trial zinc oxide rashes
are better but not gone completely.
Whats your counseling?
a) Continue this treatment
b) Give him clotrimazole
c) Give him hydrocortisone
d) Refer

207. A medication has 100% renal


elimination. Half life affected by?
a) Vd
b) Dose
c) AUC
d) Tmax

208. Patches should remove perior MRI. Why?


a) May have some metal in some of them and burn skin
b) Affect on quality of picture

209. Patient took his medications in the morning and afternoon, he got that the names on vial
label are for different patient and he is now so angry. What is the first step?
a) Call the other patient not to take medicine
b) Ask if he took med for further assessment
c) Call manager
Assess the current patient for ADEs and call the other patient who might have picked up the current
patient's meds to avoid ingestion of the meds.
210. Patient has CAP in hospital and Dr. wants to go from IV to oral, what to monitor after
step down therapy?
a) Self monitoring daily
b) Weekly pulmonary function test
c) Weekly chest X-ray
d) Lung measures capacity
Switch patients from IV to oral antibiotics when
the following criteria are met: GI tract is
functioning normally, hemodynamically stable;
2 temperature readings are normal, normalized
white blood cell count; subjective improvement
in cough and shortness of breath; able to
consume oral medications.

210. Female patient is 29 years old, 100 lb, has


serum creatinine 135%, calculate ClCr?
Answer:
wt = 100/2.2 = 45.45kg
CrCl = (140 – 29) * 45.45 / 72*135%
= 5044.95 / 97.2 = 51.903 * .085
= 44.118 = 0.44ml/min

211. Advice for chlorohexidine?


a) Cause teeth staining with continuous use
b) Not to take flouride paste at the same time with it and have 30 min after
Frequently reported side effects of this drug: Mouth irritation, Change in taste & Staining of mouth,
teeth, or fillings
Dosing: oral rinse: Swish for 30 seconds with 15 mL (one capful) of undiluted oral rinse after toothbrushing,
then expectorate; repeat twice daily (morning and evening). Therapy should be initiated immediately
following a dental prophylaxis. Patient should be reevaluated and given a dental prophylaxis at intervals no
longer than every 6 months.

212. Sodium Alginate use in formulations? Stability

213. Why we use dopamine agonist instead of levodopa?


a) Less wearing off
b) Less somnolence

214. Long case of Psoriasis trigger?


a) Smoking
b) Infection
215. Aerochamber counseling, which of the following is correct?
a) Breath normally
b) you may take multiple doses at once
c) when you hear whistling it means your taking it correctly
d) Don’t rinse even if steroid
e) Spray the puff then take 5-6 normal breaths

216. Case use ACE and Metoprolol. HR 80, what is problem?


a) Decrease Metoprolol

217. What to use as regular usage?


a) Condom
b) DMPA
c) EE 60mcg + dosepirinone

218. Which one pharmacist can return?


a) Spiriva (inhalation)
b) Idoxuridine
c) Infliximab

219. I had a question about a 6 months baby crying and drooling with no signs of infection
so what should u give the patient:
a) Benzocaine gel before feeding the baby solid food
b) Use APAP/ibuprofen alternating
c) A cold frozen clean piece of cloth
d) Wait and no problem
CTMA: Local measures can help minimize a child's discomfort during tooth eruption. The child may chew
and bite on a frozen face cloth or cooled teether. A safe teether, cooled in the refrigerator before use,
can be very effective in reducing the local symptoms. It should not have any small parts that could break
off and cause the child to choke. Canadian Dental Association recommends rubbing the back of a small,
cold spoon on the gum. Do not place anything in the child's mouth that could be a choking hazard. Avoid
long-term contact with very cold items. Teething biscuits are not recommended because sugar content.

220. what is the wrong label?


a) Cefuroxime axetil with food
b) Alendronate in lunch box

221. Most important conflict between a staff pharmacist and pharmacy owner?
a) Conflict due to financial obligation⁠⁠
b) Conflict of interest
Owner = profit-oriented Manager = patient-centered Ex: It's possible to see the following ethical dilemma
Pharmacy owner = "more clinical services $ regardless of care" VS. Pharmacy manager = "more patient-
centered care regardless of $"
222. Girls come with prescription but her insurance doesn’t cover something. Pharmacy rule
to charge $30 for 15 min counsult, what will align with pharmacy doing,
a) Autonomy
b) Justice
c) Conflict of interest
d) Paternalism

223. A pharmaceutical company that makes non-Rx cough and cold medicine wants to pay a
pharmacy to hold a patient education workshop. What best way to reduce conflict of interest?
a) The pharmacy cannot accept payment for the workshop
b) The pharmacist in charge of the event cannot accept personal compensation
c) None of the company’s medications can be specifically showcased during the event
d) Only company’s products with evidence-based efficacy can be presented during the event

224. Pharmacist doesn’t treat well with staff; manager should do what?
a) Nothing, it is their problem
b) Ask both staff come and tell whats happen
c) Talk to him, about a specific tech who told the problem
d) Talk with him in private about general concern

225. Whats the reason choosing clopidogrel?


a) Alteplase
b) PCI

226. Which med if expired or had problem can return to wholesaler and asked for credit?
a) Hydromorphone
b) Diazepam
c) Lactulose

227. What to monitor in raloxifen?


a) Hot flushes
b) CBC

228. Which is so dangerous if mistakenly use as IV?


a) Distilled water
b) D/W 10%
c) Nacl 0.45%
d) NaCl 0.9%
Direct injection of water into the bloodstream would also result in an osmotic effect in blood cells. Blood
cells, being mostly saline, would draw water, causing them to expand rapidly to the point of cell
membrane rupture. This would result in the contents of the cells spilling into the plasma. Extensive
damage to a large percentage of body cells may result in death.
229. Amikacin is to be dosed as per canadian guidelines for 98 kg person according to
adjusted body weight calculation if there is 30% body weight difference ibw and abw (abw=
50+ 2.3 (inches >5 inches). This person is to be dosed 15 mg/kg. Crcl is to be used to
determine the dosing interval. If crcl > 60 then every 24 hrs and crcl 40-60 ml/min then 36
hrs. 1150 mg every 36 hrs

230. Side effect of mirtazapine?


a) Sedation
b) Daytime somnolence

231. If mother 26 y came to you her child in neonatal care unit and she want to take flu
vaccine to protect him and she sent breastfeeding milk to him
a) She can take nasal or injection vaccine
b) She can take trivalent or tetravalent but not nasal
c) She takes quadrivalent only
FluMist is not absorbed systemically by the mother following intranasal administration and
breastfeeding is not expected to result in exposure of the child to FluMist. Flumist is from 2-59 and CI in
pregnancy & breastfeeding

232. If patient had Rx of divalproex and he can't swallow and want suspension
a) Make divaloproex suspension and gave to him
b) Call dr to give equal doses of valproic acid syrup
c) Call dr to give equal dose of phenytoin chewable tablets
d) Give pt tablet to crush by crusher at home

233. For CAP, how long ttt should be using amoxycillin?


 For patients who are well enough to be treated on an ambulatory basis, a minimum of 5 days of
antibiotic therapy is required.
 Patients who are hospitalized, who respond to treatment within 48 hours and who have no
complications may be treated for 5–10 days.
 Specific etiologies may require longer treatment, such as 21 days for severe legionnaires’ disease,
14 days for bacteremic aerobic gram-negative bacilli pneumonia and up to 21 days for pneumonia
caused by P. aeruginosa.
 Empyema requires drainage and treatment for 14 days or longer. Prolonged therapy is necessary
when a lung abscess complicates pneumonia. Antibiotics are given IV until patient become afebrile
for 72 hours and then orally until the cavity has closed, a process that may take 12–16 weeks.

234. Pregnant lady hypertension and other conditions, comes in with Rx for Diclectin.
Ramipril, fluoxetine and lorazepam. What to do?
a) Switch Ramipril to labelatol
b) Switch fluoxetine and lorazepam to CBT
c) Switch fluoxetine to paroxetine
235. Which vaccine can’t be taken in pregnant lady with gluten allergy?
a) Varicella  LAV
b) Infleunza
c) Pneumococcal

236. Patient taking immunosupressant Methotrexate with infliximab, can’t take what vaccine?
a) Heapatitis A
b) Hepatitis B
c) MMR
d) Zostavax II
Live vaccines available in Canada that are contraindicated in immunosuppressed IBD patients include
intranasal influenza, measles-mumps-rubella (MMR), smallpox, oral typhoid, yellow fever and varicella.
While herpes zoster is also a live vaccine, it is not used to elicit a primary immune response; so, it may be
considered safe to administer to patients receiving low-dose immunosuppressive therapy: methotrexate
≤0.4 mg/kg/week, azathioprine ≤3.0 mg/kg/day, 6-mercaptopurine ≤1.5 mg/kg/day, prednisone <20
mg/day (or equivalent).

237. What not to give pt came home from hospital after MI?
a) Vericella zoster
b) Pneumonia
c) Tetanus booster
d) Diphtheria booster
The question can be for a pt <50 yrs and shingles is one of the options

238. In a hospital pharmacy, it is observed that the prescription error has increased recently.
Which of the following factors mostly contribute to the increased dispensing error?
a) Inexperience or lack of training of the pharmacists and technicians
b) Inefficient light
c) In appropriate working environment
d) Increased workload of the staff
Increased workload is the most common cause for increased errors made by pharmacy staff. You can
definitely say A is wrong if you pay attention to the word "recently " in the question, which indicates,
these errors weren't common in the past (same staff was working). Besides, if anyone has tried to apply
for a job at the hospital pharmacy will know how hard it is to get in because they only hire experienced
and qualified pharmacists and technicians

239. Pramipexole side effect


a) Constipation
b) Sweating
c) Tinnitus
CTMA: Pramipexole may cause nausea, constipation, sleepiness, dizziness, dream abnormalities, amnesia
(memory loss), fatigue, muscle weakness, restlessness, weight decrease, including decreased appetite,
weight increased, hiccups, accidental injury, confusion, increase in cholesterol, aggressive behavior
240. A lifeguard who worked 5 years outside in the sun, got lesions in his back, irregular
edges, not defined and mottled. which one can be developed?
a) Erythema
b) Edema
c) Hyperpigmentation
d) Melanoma skin cancer (malignant melanoma)
Melanoma appears as a hyperpigmented skin bump that resembles a mole, but has a more irregular
appearance. Sun exposure to UV light (or from tanning beds) is a major cause of melanoma.

241. Upon dispensing error, what may let you say u r using root cause analysis:
a) Pharmacist mistake
b) Nurse Mistake
c) System Problem
d) Physician Mistake
Root‐cause analysis Understanding why an event occurred is the key to learning from mistakes and
developing effective recommendations to prevent the same error from occurring twice. Root‐cause
analysis is a technique for undertaking a systematic investigation that looks beyond the individuals
concerned and seeks to understand the causes and environmental context in which the incident
happened. The process involves data collection, cause charting, root cause identification, as well as
generating and implementing recommendations

Dry eyes ques.


242. Eye Case; female patient with eye allergy, inflammation and redness not AMD and on a
bunch of meds. She had many problems, parkinson, allergic rhinitis, eczema, read a lot and
use contraceptives. What do you ask the patient to evaluate if he needs to be referred?
a) Do you wear contact lenses?
b) Is there discharge?
c) What med is causing this condition?

243. How many factors cause her eye


dryness?
a) 1
b) 2
c) 3
d) 4
e) 5

244. What to recommend for her?


a) Saline
b) Artificial tears
c) Cyclosporine
245. One question was like that he was drinking daily beer and all stuff so which drug u can't
give or most contraindicated
a) Metformin
b) Amitriptyline
c) Paroxitine

246. Uncontrolled HTN may lead to all except


a) PAD
b) Tricuspid valve dysfunction
c) Hypertrophy
d) Stroke
e) Retinopathy

247. Warfarin with drug X. Potential interaction?


a) Dispense
b) Decrease warfarin half till see doctor
c) Fax a note to doctor
d) Give patient a note to take to the doctor

248. Infant dehydration symptoms, except?


a) Sunken eyes
b) Polyurea
c) Dry mouth

249. Sumatriptan subcu? Fastest onset

250. Question about side effect of lurasidone?


 Lurasidone must be taken with food at least 350 calories.
 S.E: Akathisia, anxiety, EPS, insomnia, nausea/vomiting, somnolence.
 Monitor patient for lurasidone associated NMS and discontinue if NMS is suspected.
 May potentiate antihypertensive drug effects.

251. What is the expended Scope of pharmacist? Acedamic education

252. RR question with 55 and 35 patients or something like that answer 40.5%

253. What complication of herpes zoster, In elderly? Encephalitis


An infection by herpes zoster virus is a common and important cause of encephalitis. Herpes zoster virus
encephalitis if not treated promptly can result in significant morbidity and mortality.

254. N95 masks are used for serious diseases like TB, SARS, influenza etc
People with chronic respiratory, cardiac, or other medical conditions that make breathing difficult should
check with their health care provider before using an N95 respirator because the N95 respirator can
make it more difficult for the wearer to breathe. Some models have exhalation valves that can make
breathing out easier and help reduce heat build-up. Note that N95 respirators with exhalation valves
should not be used when sterile conditions are needed. All FDA-cleared N95 respirators are labeled as
"single-use," disposable devices. If your respirator is damaged or soiled, or if breathing becomes difficult,
you should remove the respirator, discard it properly, and replace it with a new one. To safely discard
your N95 respirator, place it in a plastic bag and put it in the trash. Wash your hands after handling the
used respirator. N95 respirators are not designed for children or people with facial hair. Because a proper
fit cannot be achieved on children and people with facial hair, the N95 respirator may not provide full
protection.

255. What happens if endocarditis left untreated


a) High BP
b) HIV patient
c) TB patient
Complications include heart failure, periannular abscess, mycotic aneurysm and glomerulo nephritis.
Emboli to extracardiac sites such as the brain, lung or kidney may also occur.

256. Pharmacist has severe allergy with penicillin (hives), what he can do?
a) Inventoty count
b) Counselling
c) Product check

257. One que about pharmacist best outcome...something...and one option was by providing
health records of patients.

258. patient got angioedema from enalapril; now his dr want to switch to ARB, how to switch?
a) Stop, wait & switch.
b) Switch directly
c) Continue with small dose.

259. I had SLE related whole scenario


a. A 53-year-old man presents at your pharmacy with complaints of painful and swollen
joints, fever, chest pain, hair loss, mouth
ulcers, swollen lymph nodes, feeling tired, and
a red rash, dr diagnosed him with SLE, what
NON pharmacological options to advise him?
a) Smoking cessation
b) Decrease salt and caffeinated beverages
c) Use moisturizer while going out in hot
weather
b. What initial treatment to give orally?
a) Hydroxychloroquine
b) High dose corticosteroid
c) MTX

c. She came back with a joint attack &


Flare ups; now what to give?
a) Methotrexate
b) Sulfasalazine
c) Mycophenolate

d. What test to monitor Methotrexate?


a) Chest X-ray
b) TB
c) CK
d) TSH
e) Eye examination
f) Skin rash.
Baseline assessment should include complete blood count (CBC) with differential and platelet counts,
hepatic enzymes, renal function tests and chest xray. Monitor hematology at least monthly, and hepatic
enzymes and renal function every 1–2 months.

260. Which need docs intervention?


a) Hydromorphone 2mg q4h prn - 20 tabs
b) Adavair (Fluticasone/salmeterol) 1 puff bid for 30 days
c) Atrovent  bcz we still need puff quantities

261. Question about paralysis and respiratory depression


a) Buprenorphine
b) Ketamine
c) Midazolam
d) Rocuronium
e) Propofol
Rocuronium is NMJ blocker. Causes
paralysis.
Used to relax the muscles. It works by
blocking the signals between your
nerves and your muscles.
Rocuronium is given before general
anesthesia in preparing you for
surgery. Rocuronium helps to keep
your body still during surgery
262. Pt had pernicious anemia begins monthly injection of B12. She sees great improvement
but the medication becomes indefinitely backordered. What do you advice her?
a) Order online pharmacy
b) Take high dose oral B12
c) Wait till you get B12 IV
d) Go across the border to the the US to continue treatment

263. Cancer patient taking Methadone, his doctor is out of town, the available doctor is not
registered to prescribe Methadone, what to do as a pharm.?
a) Do not give him as you are not allowed to
b) Fill prescription expecting doctor will sign it when he returns
c) Call doctor and get verbal prescription
d) Ask available doctor to register himself for one dose to prescribe Methadone
In the past, physicians required exemptions to prescribe methadone, but As of May 2018, physicians do
NOT require exemptions to prescribe methadone.

264. Which does not require sterility


a) Large parenterals
b) Nasal drops
c) Topical antibiotics
d) Otic preps

265. Prophylaxis antibiotic for cancer patient will have hysterectomy or Colorectal surgery;
a) Ceftriaxone
b) Vancomycin
c) Clindamycin + tobramycin
d) Cefazolin + metronidazole
Antibiotic Prophylaxis to Prevent Surgical Site Infections
Surgery Common Pathogen Recommended Antimicrobials*
Cardiothoracic Staphylococcus aureus, Cefazolin, cefuroxime sodium
coagulase-negative staphylococci (Zinacef), or vancomycin
Gastrointestinal Enteric gram-negative bacteria, Cefoxitin, cefotetan, ampicillin /
anaerobes, sulbactam, or cefazolin plus
enterococci metronidazole
Gynecologic (vaginal, Enteric gram-negative bacteria, Cefoxitin, cefotetan, cefazolin, or
abdominal, or laparoscopic group B streptococci, ampicillin/sulbactam
hysterectomy) enterococci, anaerobes
Orthopedic S. aureus, Cefazolin, cefuroxime sodium, or
coagulase-negative staphylococci vancomycin
Vascular S. aureus, Cefazolin or vancomycin
coagulase-negative staphylococci,
enteric gram-negative bacilli
266. Which rx needs priority
a) Topiramte for pt has uncontrolled epilepsy
b) NPH insulin
c) Metronidazole for C. Difficle pt
d) Rivaroxaban for DVT pt
Anything for maintenance therapy can wait. Acute medications like pain medications and antibiotics for
severe cases (c. difficile for example) should always have priority. Don't be tricked by medications with
immediate actions. It's the condition of the patient, that is important here.
Why not insulin? Because it is used as a substitution therapy. Can they wait? Yes, why not
Why not Rivaroxaban for dvt? Long duration of action and for prevention. If it was streptokinase instead,
then it should be dispensed immediately.
Topiramate? Maintenance therapy. Nobody takes topiramate to treat a symptom right way
So why metronidazole for c. Difficile? 1) it's an antibiotic 2) c. Difficile can kill. We want to start the
patient as soon as possible on antibiotics even if it's a mild case

267. COPD patient with acute bronchitis comes with new prescription for ipratropium nasal
spray. He is taking salbutamol inhaler and advair. He told you he has been diagnosed with
bronchitis. What is the drug related problem in this case?
a) Call dr and recommend him antibiotic
b) Call Dr And Suggest Step Up Therapy
c) Wrong Dosage Form Given
d) Wrong Drug Given
Wrong dosage form given. He should get ipratropium inhaler not nasal spray

268. Pharmacist is starting program to help patients with dementia. Best way to get word out?
a) Pamphlets in drug bags
b) Presentation to local caregivers
c) Note to physicians
d) Call caregiver
e) Put pampheletes in prescriptions

269. While injecting what should be taken care of?


a) Radial nerve
b) Axillary nerve

270. Prescription for antibiotic suspension. Total rx was something which was rounded up to
5.25 gm. Of drug. In the dry powder when u add 127 ml water it gave 50 mg/ml conc. Of
total 150 ml. Father wants 350mg/5 ml as he was concerned about large volume to be given
to the child. How you ll prepare?
150 -127 = 23ml displacement. That 23ml was 50mg/ml x 150ml = 7.5g of powder.
Cross multiply: 7.5g/23ml = 5.25g/X. X = 16.1Ml displaced with 5.25g of powder.
For 350mg/5ml = 70 mg/ml. Solve for mL
so, ml/70mg X 52500mg = 75ml. Then 75ml – 16ml = 59mL to add.
271. Cost effective sum. I can’t remember the question. It was using 5.5mg permL vials to
make dilute solution, And 200 mcg something. How many vials to use so that none gets
wasted. Ca phosphate mEq given...something...what was ans?
6.75

272. Obese patient comes in asking for meal replacement. First question to ask him?
a) Have you tried other stuff (e.g. exercise)?
b) Let me calculate your BMI
c) Let’s discuss your weight reduction goal
d) Weight is a known health concern. Do you have any specific health problems that are
prompting you to lose weight?

273. Wt loss how much is good


a) 0.5 to 1 kg per week
b) 2 to 5 kg per week
c) 5% per month
d) 10% per month

274. One Q about a patient who has HTN & pain. In his prescription nifedipine, Amoxiclav,
Acetaminophen & Statin. What is true
a) He is not adherant
b) He is not getting what he needs

275. They gave when he did his last refill, Nifedipine 2month ago, Statin 1 month ago. Also,
emcitabine/tenofovir drug patient taking. Wat supplement to give?
a) Folic acid.
b) Iron.
276. What needs to be swirled?
a) Benzadyzamine also swirl. Do not swallow
b) Nystatin
c) Tantum oral rinse (pharexia)
Nystatin: Shake oral suspension well prior to use. Swish and gargle; retain in the mouth for as long as
possible prior to swallowing.

277. Which one has no benefites in Ascites patients?


a) Lactulose  For Hepatic Encephalopathy
b) Furosemide
c) Spironolactone
d) Paracentesis

278. Clarithromycin interaction with?


a) Allopurinol
b) Metoprolol
c) Citalopram  Qt prolongation

279. What is best statement about biosafety lab for cancer sterile preps?
a) Spill management kits should be there
b) Pregnant workers should wear personal protective equipment

280. What is monitoring for ethosuximide patient?


a) WBC
b) Slit Eye Check
c) CBCs

281. Phenytoin calculation. 2 ml per hr. 19 fm of 1% 31 gm 5%

282. Rash with abacavir right??


Abacavir S.E: Hypersensitivity reactions in patients with genetic predisposition (can be severe).
Guanosine The patient must stop therapy and not be rechallenged.
analog Low potential for drug interactions. Available as an oral solution.
Screen patients for the presence of HLA-B*5701 before starting; do not use if patient is
positive for HLA-B*5701.
Monitor closely early on for hypersensitivity reaction.

283. Patient dead, a family member came to return his unused medications (different drugs &
Narcotics), which one is the apprrpriate action to take as a pharmacist?
a) Where he buys return there to destroy all
b) Discard narcotics, return others to the shelf
c) Put only tablets on the shelf and discard others
284. Public washroom in pharmacy case?

285. A pharmacy assistant is caught injecting insulin for his diabetes in the pharmacy. What
is best for manager to make this risk-free work environment?
a) Send Pharmacy assistant to private area to inject
b) Ask him to do injection in lunch room
c) Ask him to do injection during lunch time
d) Ask him to inject when pharmacy is less busy

286. What is a main criterion for a new drug to be recommended in formulary?


a) Evidence from clinical trials
b) Therapeutic index

287. 4 ml diluents - 1 ml stock solution 100 mg. Pt got 50 mg instead of 25 and call you
asking if he can use half dose, what will u say?
a) No and dispense right one

288. Selenium calculation question. you have 0.5 micromole/L, your stock is 0.25
micromole/L. How much of 40 mcg/ml of selenium would you add if you want to make
482L (M wt of selenium is 79)?
a) 0.4
b) 0.3
c) 0.34
d) 0.24

289. Pyrantel pamoate claculation? yes 5 tabs


Pyrantel Acts as a depolarizing, neuromuscular-blocking agent, causing release of acetylcholine and
pamoate inhibition of cholinesterase, leading to paralysis of the worm.
The paralyzed worm releases its hold on the intestinal tract and is expelled.
Adults and children >1 y: 11 mg/kg (base) single dose PO; repeat in 2 wk
Maximum: 1g (base)
11 mg/kg base equals 31.9 mg/kg pyrantel pamoate.
Liquid form (50 mg/mL base) should be shaken well before use.
Tablets available as 125 mg pyrantel base.
S.E: Anorexia, nausea, vomiting, cramps, diarrhea, headache, dizziness, drowsiness, Red
color urine, feces.
Pyrantel pamoate and piperazine have antagonistic effects; avoid combination.
Avoid in the 1st trimester of pregnancy. Caution in hepatic impairment.
290. A doctor suspects that a patient may have angina and recommends treatment. What
is the MOST important to rule out of the differential diagnosis when diagnosing angina.
a) GERD
b) Abdominal Aortic Aneurism
c) Pancreatitis
d) PUD
Abdominal aortic diameter ≥ 3 cm typically constitutes an abdominal aortic aneurysm.
The cause is multifactorial, but atherosclerosis is often involved.
Most aneurysms grow slowly (~10%/year) without causing symptoms, and most are found incidentally.
Risk of rupture is proportional to the size of the aneurysm.
The following 3 questions establish whether the discomfort is considered nonanginal chest pain e.g
GERD, atypical angina or typical angina.
1. Is the discomfort substernal?
2. Are the symptoms precipitated by exertion?
3. Are the symptoms relieved within 10 minutes of rest?
Patients who respond “yes” to all 3 questions have typical angina. Patients who respond
“yes” to 2 questions have atypical angina. Patients who respond “yes” to 1 question or no
questions have nonanginal chest pain.

291. Nitrofurantoin what is true?


a) Food increases absorption
b) Take with food because of bitter after taste
c) Take on empty stomach
d) Take with food because of Diarrhea
Nitrofurantoin is administered orally and all dosage forms should be taken with food or milk to minimize
gastric upset. Administration with food may also enhance absorption.
Gastrointestinal side effects: Diarrhea, dyspepsia, abdominal pain, constipation, emesis and pseudo
membranous colitis (including that due to C. difficile) have occurred.

292. Black color spot on thigh is due to which drug?


a) Warfarin
b) Prednisolone
c) Levodopa
d) Amiodarone
e) Levothyroxine.

293. Can only return to wholesaler without information?


a) Tranylcypromine MOAi
b) Controlled drugs
c) Narcotic drugs

294. Dr lose this narco thing what u can dispense with prescription? CNS stimulant
295. After NOC? Drug go to real world NOC---DIN---PMPRB
What does notice of compliance allow company to start doing
a) Trials
b) Marketing a new drug

296. Cost benefits or utility?

297. After accident. Empiric treatment gentamycin plus metro

298. Which of these requires a record of both purchase and sale


a) Butorphanol
b) Buprenorphine
c) Testosterone
d) Rizatriptan

299. ISMP label error two questions today & yesterday paper has also two questions

300. How much should be distance between two injections in the same muscle?
a) 1 inch
b) 2.0 Inch
c) 0.5 inch
d) 3.0 inch
Multiple injections given in the same extremity should be separated by a minimum of 1".
Insert needle at a 45° angle into fatty tissue of the anterolateral thigh. Make sure you pinch up on
subcutaneous tissue to prevent injection into the muscle.

301. Prednisone Calculation: Dose to be given as an infusion 900mg, 50mg/hr initially then
increase by 50 mg/hr every 30 min until maximum 400mg/hr. how many hours needed?
Answer:
50 ml /hr means 25 ml/ 30 min  0....25 ....50....75....100...125....150....175...200
Total = 900  8*30 min = 4 hr if need 400 only

302. The pharmacy use a syringe with both degree ml and kg. The pharmacist found that
there is a lot of mistakes done by nurses injecting oral morphine as IV using oral syringes.
what to do to eliminate this error?
a) Purchase oral syringes that doesn’t fit in IV lines
b) Label oral solutions as ORAL USE ONLY
c) Move all of oral solution to pharmacy form stock
d) Prepare all of oral solution in oral syringe
e) Put both pounds and kg on patient chart
Unintended Do not stock bulk oral solutions of medications on patient care units.
intravenous Use only oral syringes that are distinctly marked “Oral Use Only.”
administration of When ISO 80369 compliant syringes (e.g., ENFit) are used for administration of oral
oral medications liquid medications, always highlight on the pharmacy label, or affix an auxiliary label,
“For Oral Use Only” on the syringe.
Ensure that the oral/enteral syringes used do not connect to any type of parenteral tubing
used within the organization.

303. What source for drug trial?


a) Medline
Secondary References:
Cochrane Review à Recommendations and disease specific consensus guidelines
PubMed/Medline à Abstracts from multiple journals and some full texts
Primary sources (original material ie clinical studies) are published in journals or accessed using
PUBMED, MEDLINE, EMBASE, IPA (secondary sources)
Journals are primary but become secondary when they are review articles.

304. Registered pharmacy technician (RPhT) and Pharmcy Assistant (PA) roles of care

305. Non-sterile compounding expiry?


a) 14 days in room temp (for water containing liquid preps- should be cold temp)
b) 30 days in cold (for water containg topical preps – doesn’t need to be cold)
c) 3 months or earliest expiry
d) 6 months or earliest expiry (for non-aq formulations)
What is the difference between expiry dates and beyond-use dates?
The manufacturer or distributor gives an expiry date to a drug product based on known stability data. It
indicates the expected timeframe in which a drug product meets the therapeutic and stability
requirements based on the published monograph or literature.
Beyond-use dates, on the other hand, provide the date after which a compounded preparation shall not
be used and are determined from the date when the preparation is compounded. Compounders provide
the beyond-use date (based on the manufacturer’s stability information and the literature with respect
to stability, compatibility, and degradation of ingredients) to limit patient use of the compounded
preparation. All compounded preparations must contain a beyond-use date.
306. Instead of rabeprazole, patient got aripiprazole in hospital. Who to tell?
a) Family
b) Family doctor
c) Hospital doctor

307. What SE the patient may experience?


a) Sedation
b) Diarrhea
c) Hyperglycemia

308. There is question related third party insurance oppose prescriber autonomy. What
insurance company policy messes with doctor’s autonomy the most
a) Plan maximums
b) Forced generic substitution
c) Therapeutic interchange
d) Something wrong
e) Therapeutic interchangeability

309. Treatment of Perianal fistulae in crohn’s disease? Metronidazole


CTC: Short courses (2–4 weeks) of metronidazole and/or ciprofloxacin are useful for the treatment of
patients with CD and perianal fistulae or isolated colonic CD. Metronidazole may cause a disulfiram-like
reaction in some patients if alcohol is ingested, and neuropathy may occur with long-term use.

310. Which of the following causes the least metabolic ADRs?


a) Ziprasidone
b) Clozapine
c) Quetiapine
d) Risperidone

311. Chicken pox vaccine daughter got? Who should avoid?


Chicken pox vaccine to daughter........mother at home.... Who to avoid?

312. Community acquire pneumonia stepdown, what to monitor post therapy change
a) WBC daily
b) Weekly CXR
c) Weekly PFT
d) Daily symptoms (subjective)

313. Framingham risk score FRS EXCEPT?


a) Family history
b) Age
c) Smoking
d) LDL
e) Diabetes
FRS involves age, smoking, diabetes, cholesterol level, systolic BP & HDL-C. Double the FRS when there is
a family history of premature CV disease (modified FRS)

314. What has a drug interaction with ACE Inhibitor


a) Bactrim (trimethoprim) (enhances hyperkalemic effect of ACE - monitor)

315. Syncope episode


a) Decrease bisoprolol
b) Go to doc and get ECG in next 2 weeks
c) Send to ER for assessment
CTC: Treatment is directed at the cause of syncope. Treat any reversible causes and taper and stop drugs
that might cause syncope, where possible. Refer patients with syncope secondary to bradycardia (asystole
or complete heart block) for a permanent pacemaker. Refer patients with suspected or diagnosed
ventricular tachycardia, an abnormal ECG (including signs of old infarction, conduction system disease and
genetic arrhythmias) and all patients with structural heart disease to a cardiologist, preferably an
electrophysiologist. The following addresses treatment of syndromes of orthostatic intolerance.

316. What appropriate monitoring before starting hydroxychloroquine


a) Ophthalmic exam
b) CBC

317. What is weekly dose for Actonel DR (Risedronate DR)?


a) 35 mg weekly
b) 150 mg weekly
c) 7 mg daily
Residronate In Paget: 30 mg OD for 2 months. In OP: 35 once Weekly, 150 mg once per month PO
Alendronate in Paget: 40 mg daily for 6 months. In OP: 10 mg daily PO or 70 mg once weekly PO

318. Pharmacist get needle injury during vaccinating to patient. What is appropriate?
a) Wash hands with soap and water
If you pierce or puncture your skin with a used needle, follow this first aid advice immediately:
encourage the wound to bleed, ideally by holding it under running water. wash the wound using running
water and plenty of soap. do not scrub the wound while you're washing it.

319. Atrial fibrillation, emergency been a week?


a) Amiodarone IV
b) Metoprolol IV
c) Digoxin IV
d) Electrical cardioversion within 6 hours
320. Bronchitis and patient had cream coloured sputum, cough for 2 weeks and normal
temp. Doctor assessed and said no need for antimicrobial therapy. Why?
a) Cream coloured sputum
b) Less than 4 weeks cough
When cough is present for <3 weeks, with or without sputum production, it is consistent with the
diagnosis of acute bronchitis. Acute bronchitis should be differentiated from the common cold, acute
exacerbation of COPD, asthma and community-acquired pneumonia.
A nonbacterial cause is present in >90% of uncomplicated acute bronchitis. Acute bronchitis is generally
self-limited and symptoms usually resolve in 10–14 days, but cough can last for up to 8 weeks.

321. Maintenance cardioversion


a) Mexiletene  Rarely used as monotherapy.
b) Propafenone
c) Sotalol
Immediate Therapy for Sustained VT or VF. If a patient with sustained monomorphic VT is unstable (e.g.,
has hypotension, angina, heart failure or marked symptoms), cardioversion is effective and safe
Flecainide and propafenone should not be used in patients with coronary artery disease or structural
heart disease. Weak beta-blocking effect. Use with care, if at all, in patients with LV dysfunction,
especially prior MI. Active metabolites accumulate in rapid metabolizers. Monitor QRS duration
sotalol is a betablocker. May be especially effective in exercise-related arrhythmias. Likely more
effective than other drugs in suppressing inducibility of VT.

322. Stroke and dysphagia; refer to


a) Speech pathologist
b) Occupational Therapist
c) Physiotherapist
d) Dietitian
The speech-language pathologist (SLP) is
responsible for the diagnosis of swallowing
problems and determining a course of
treatment.

323. Stroke guy was on ASA. What to give for anti-platelet?


a) Increase ASA to 325
b) Switch to clopidogrel
c) Add ticagrelor

324. Post MI (PCI no fibrinolytics) pateint, on rosuvastatin, lansoprazole and some other
stuff. Started on ticagrelor 90 BID. Why would we wanna switch to clopidogrel?
a) No fibrinolytics
b) PPI interaction
c) Dyspnea
d) Interaction with rosuvastatin
Ticagrelor is a direct, reversible inhibitor of the P2Y12 receptor. It is more effective in acute coronary
syndrome when compared to clopidogrel. Ticagrelor is endorsed as first-line therapy in current
guidelines. Bleeding is increased among patients treated medically or with PCI when compared to
clopidogrel. Dyspnea may occur in approximately 14% of patients treated with ticagrelor.
Lexi: Ticagrelor may increase the serum concentration of Rosuvastatin. Ticagrelor may decrease renal
function, leading to increased rosuvastatin concentrations

325. Which would contradict federal law


a) Authorizing Refill for clonazepam
b) Adjusting dose of Ramipril based on BP

326. What affects half-life in a a drug that’s completely renally excreted?


a) SR formulation
b) Volume of Distribution
c) Dose
For a drug that is excreted solely by glomerular filtration, the elimination halflife may change markedly
in accordance with the binding affinity of the drug for plasma proteins
"Drug administration by any other route than the intravenous route."
After extravascular drug administration, the rate and extent of drug absorption, from its site of
administration to the systemic circulation, determine the onset and the intensity of drug effect. The
duration of effect is mainly determined by the dose, the half-life and to a variable degree by the
absorption rate (e.g. slow-release formulations).
Initially, after an extravascular administration, the entire drug is at the site of absorption and none has
yet reached the systemic circulation. Therefore, the rate of absorption is high and there is no elimination
yet. As long as the rate of absorption exceeds that of elimination, the plasma concentration rises
(absorption phase). The maximum concentration (Cmax) is reached when the rate of disposition matches
the rate of absorption. Thereafter, the rate of disposition exceeds the rate of absorption and the plasma
concentration declines (disposition phase).
The factors affecting the plasma concentration-time profile after extravascular administration are:
1-Dose: Lower doses usually produce a proportional decrease in plasma concentration at all times.
2-Bioavailability: lower bioavailability results in a proportional decrease in plasma concentration.
3-Rate of absorption: Slower rate of absorption delays and reduces the magnitude of the peak of plasma
concentration.
4-Volume of distribution: Larger Vd is responsible for both a lower peak concentration and a longer
elimination half-life, delaying the peak approach.
5-Clearance: CL mainly affects the disposition phase; however, an increase in clearance also results in a
faster approach to a lower peak concentration.
Apparent half-life (t1/2): In some cases, such as for controlled-release preparations, the rate of decline of
the drug plasma concentration is not due to elimination alone. Other factors such as absorption rate or
distribution rate influence plasma concentration decay. In such conditions, the observed half-life is called
apparent half-life.

327. Homeopathic meds have an ID number


a) DIN
b) DIN-HM (HM = homeopathic medicine)
c) NPN
328. Which of these has evidence for stroke
a) Perindopril/indapamide
b) Candesartan
c) Ramipril and Beta blocker
d) CCB
There is evidence that coadministration of angiotensin converting enzyme (ACE) inhibitors, such as
the perindopril erbumine component of COVERSYL PLUS LD/COVERSYL PLUS/COVERSYL PLUS HD, or
of angiotensin receptor antagonists (ARBs) with aliskiren increases the risk of hypotension, syncope,
stroke, hyperkalemia and deterioration of renal function, including renal failure, in patients with
diabetes mellitus (type 1 or type 2) and/or moderate to severe renal impairment (GFR <60 mL/min/1.73
m2). Therefore, the use of COVERSYL PLUS LD/COVERSYL PLUS/COVERSYL PLUS HD in combination with
aliskiren-containing drugs is contraindicated in these patients

329. Raynaud’s and on a lot of meds. Allopurinol 100 (15 y), amitriptyline 10 (5y) for
neuralgia, atorva 10 (12y), citalopram 20, metoprolol 50 bid (titrated over last 6 months),
other shit. He has cold extremities and still feels neuropathic pain. Has had a gout attack in
last 2 years. All labs and vitals normal. What is likely causing his cold extremities?
a) Metoprolol
b) Atorva
c) Allopurinol

330. He did not find relief from neuralgia. What is the drug therapy problem?
a) Non-compliance due to fear of side effects.
b) Insufficient dose for diabetic neuralgia.
c) Incorrect first line therapy for diabetic neuralgia.
d) Over dose of Amitriptyline.
Tips: Amitriptyline Initial: 10–25 mg QHS po, Increase by 10–25 mg daily at weekly intervals, until pain
relief or side effects.

331. Which of the following do not require preservatives


a) Large volume parenterals
b) Ear drops

332. Pilot on lorazepam. Refilling early, what to do?


a) Call his boss
b) Address issue with patient
c) Say nothing
d) Advise not to take it within 6 hours of flying (if he came in asking for something
before flight)
333. Forgery. What do to?
a) Come back later (have the police called for then)
b) Give it back to him and tell him out of stock
c) Give it back to him and tell him it’s a forgery
d) Restrain patient and call police

334. Pharmacist is treating techs like shit


a) Publicly humiliate him (basically)
b) Have him come up with self-solutions to adress the issues with the techs
c) Address the attitude in general rather than specific incidents

335. Which of these would increase risk of bleeds with warfarin


a) Rifampin
b) Carbamazepine
c) Fluvoxamine
Selective Serotonin Reuptake Inhibitors may enhance the anticoagulant effect of Vitamin K Antagonists.
Severity Moderate Reliability Rating Excellent
Patient Management: Monitor for increased therapeutic/toxic effects of oral anticoagulants if a selective
serotonin reuptake inhibitor (SSRI) is initiated/dose increased, or decreased effects if an SSRI is
discontinued/dose decreased.

336. What to prescribe for daily angina symptoms?


a) Bisoprolol
b) Nitro spray
c) Ramipril

337. Inflectra is the “something” of Infliximab. What does this mean


a) Therapeutically and pharmaceutically equivalent
b) Identical formulation, dose and dosage form
c) It can be substituted for Remicade
Infliximab is a chimeric monoclonal antibody biologic drug that works against tumor necrosis factor
alpha and is used to treat autoimmune diseases. Trade name: Remicade, Remsima, Inflectra

338. Regarding his gout attacks, what med might you increase?
a) Allopurinol.
b) Metoprolol
c) Atorvastatin
Starting dose: 100 mg daily PO. Maximal decrease in uric acid occurs within 1-3 weeks.
Usual: 300 mg daily PO titrated to urate levels; Maximum: 800 mg daily PO
To improve tolerability, divide doses >300 mg to 2–3 times/day.
The minimum effective dose is 100–200 mg daily. Allopurinol is better tolerated when taken with meals.
Renal impairment: decrease maintenance dose to 100 mg/day if ClCr is 10–20 mL/min; 100 mg Q2–3
days if ClCr <10 mL/min
339. Patient taking Lithium and candesartan, taking 2 cups of coffee per day and high
sodium diet & 1 cup alcohol per day, smoke 1 to 2 cigarettes per day. What is the DTP:
a) Candesartan interaction with lithium
b) Candesartan interact with high sodium diet
c) Alcohol can’t be taken with Candesartan
d) Smoking is prohibited with Candesartan
Lexi: ARB II may increase the serum concentration of Lithium & ACE, loop and thiazide too

340. Which symptoms that patient may suffer with Lithium


a) Hirsutism
b) Hand Tremors
c) Weight Gain
d) Arthralgia

341. What will increase lithium toxicity with this patient


a) Diet
b) Caffeine
c) Renal Failure
d) Alcohol
e) Smoking
CTC: Lithium is CI in renal failure
Caffeine: may decrease the serum concentration of lithium by increasing renal clearance. May require
more frequent serum monitoring if patient is increasing or decreasing caffeine intake.
Sodium: alterations in sodium intake can affect serum lithium concentrations. Sodium and lithium are
both excreted and absorbed by the kidneys. Therefore, ingestion of large amounts of sodium can lead to
increased excretion of both ions and subtherapeutic lithium levels. Conversely, if sodium intake is
restricted, lithium and sodium will be reabsorbed, leading to higher serum lithium levels and a risk
of lithium toxicity. This can pose therapeutic issues for patients who are following salt-restricted diets
(e.g., congestive heart failure, hypertension, etc.).

344. How do you treat Lithium Overdose?


Early symptoms of chronic lithium toxicity can usually be treated by reduction or cessation of dosage of
the drug and resumption of treatment at a lower dose after 24–48 hours.
Whole bowel irrigation with a polyethylene glycol solution may help minimize lithium absorption,
particularly of sustained-release formulations.
Activated charcoal does not adsorb lithium but may be of value if multiple drug ingestion is suspected.
Measure serum lithium levels every 4–6 hours until the serum lithiumlevel is below 2 mmol/L.
Maintain fluid and electrolyte balance. Maintain urine output and sodium excretion by administering iv
sodium chloride 0.9%. Administration of large amounts of sodium in the absence of sodium depletion has
not been shown to be successful in accelerating lithium excretion.
Sodium bicarbonate, mannitol, loop and thiazide diuretics, carbonic anhydrase inhibitors or
phosphodiesterase inhibitors are not recommended.
Hemodialysis is indicated in the following settings: potentially toxic exposures in patients with renal
failure; neurologic dysfunction including altered mental status; acute ingestion with serum lithium
concentration of ≥4 mmol/L; acute-on-chronic or chronic overdose with serum lithium concentration
≥2.5 mmol/L; patients who may not be able to tolerate sodium repletion. Peritoneal dialysis is not
recommended.

Amoxicillin Allergy
Candidate: Hello Dr. ,if a pregnant patient comes to pharmacy with rash all over the arms and legs after
taking amoxicillin for 3 days,no swelling no SOB ,just rash, what’s the best action here?
answer
First Fact: A true amoxicillin allergy will cause an immediate onset (usually within an hour of a dose) of
rash/hives, swelling, shortness of breath and even anaphylaxis. This is called an IgE-mediated reaction. On
the other hand, maculopapular rashes (red patches on skin surface) with amoxicillin are common and do
NOT represent an absolute contraindication for future use.
1) Amoxicillin rashes are skin changes that can develop while on amoxicillin as a reaction to this medicine.
2) The reactions can be immediate (within 1-2 hours) or delayed (after several hours or days).
3) Assess for red flags = itchiness, feeling unwell, fever, skin blisters (hives = raised bumps or patches that
are itchy), and difficulty breathing (informative questions).
If yes to any, see your doctor ASAP since this is a serious allergic reaction.
Assess the symptoms = severity (how bad), swelling, location, onset, etc. (informative questions).
Overall, sudden onset (within 2 hours), highly pruritic rash, hives, or SOB are indicators to STOP amoxicillin
and see your doctor.
4) Reassure = these rashes are common and do NOT necessarily mean that you are allergic to the
medication (amoxicillin). Similarly, this reaction does NOT necessarily warrant stopping amoxicillin or
switching to other antibiotics. Otherwise, if your doctor determines that you have a penicillin allergy (or
you're likely allergic) discontinuing the drug is the first step in treatment. Take photos of the rash to show
your doctor.
5) This rash (likely nonallergic maculopapular rash = widespread, small, flat pink/red circles/spots and little
red bumps) is expected to go away once you complete taking the medicine. However, this might take
several days (most likely) or even weeks to go away.
6) Offer oral antihistamines (e.g., cetirizine) for itch.
Offer a topical steroid (e.g., hydrocortisone 1% cream) to help with itch and redness.
7) Offer nondrug measures (colloidal oatmeal, moisturizer, cool cloth, etc.)
If you develop hives, talk to your family doctor should to consider referral to allergy testing to R/O
penicillin allergy.
NOVEMBER 2016
1. A pregnant diabetic woman come to you complaining that she has vaginal itching, what is the
first question that you will ask this woman?
a) Is there any discharge?
b) What is your blood glucose reading?
c) For how long do you have these symptoms?

2. What increase risk of these symptoms?


a) She is pregnant
b) She is diabetic
c) Vaginal douching
d) All of them
Risk factors for bacterial vaginosis: Intrauterine devices (IUD), Sexual intercourse with new or multiple partners,
Lack of lactobacilli & Douching. Diabetic is alarm (red flag) for vaginosis in pregnancy if no vaginal douching

3. After that she came and show you the result of examination. She has Gardnerella vaginalis.
What is the most appropriate treatment for this woman?
a) Metronidazole ovules.
b) Metronidazole tablets.
c) Clindamycin cream
Bacterial vaginosis in pregnancy: oral metronidazole 500 mg orally twice daily for 7 days or clindamycin 300
mg orally twice daily for 7 days. Topical agents are not recommended. Vulvovaginal candidiasis in pregnancy:
topical azole antifungals are safe and effective in pregnancy at least ttt 7 days

4. A mother of 6 years old child came to you and she was annoyed because his son wet his bed
every day. What will you say to her?
a) This normal in all children in his age.
b) Obstructive sleep apnea is a common cause
c) 80 percent getting dry without treatment
d) Parent support is critical for successful treatment.
e) Most of children with urinary incontinence require refer to specialist.
CTC: Without treatment, 15% of affected children are expected to become dry each year.
Q. In cases of daytime incontinence,
 Advise parents to refrain from humiliating or punishing the child and to support the child’s efforts with
positive reinforcement.
 Avoid excessive intake of fluids within 2 hours of bedtime & empty bladder before going to bed.
 Encourage the child to avoid deferral of micturition and to do bladder training exercises.
Q. Children with enuresis:
 Enuresis alarms are effective for children ≥ 7 years of age when used properly for 3–4 months.
 More effective in children with a normal urine output and a small or normal bladder capacity.
 Complementary therapies as hypnosis, chiropractic, faradization or homeopathy have no effect.
 Transcutaneous parasacral electrical nerve stimulation, functional magnetic stimulation, laser
acupuncture or limiting milk intake to avoid hypercalciuria may improve response and decrease relapse

5. She asked you about non pharmacological treatment for his son:
a) Fluid restriction 2 hrs before sleeping
b) Enuresis alarm.
c) Cognitive treatment
CTC: Therapeutic Tips
 Predictors of positive treatment outcome include motivated child, supportive family, age over 10 years
 Predictors of treatment failure include developmental delay, low self‐esteem, a history of behaviour
problems or multiple wettings at night, frequent daytime voiding, parental intolerance or annoyance,
and unstable family dynamics.
 The cause of most cases of daytime incontinence is uncovered by noninvasive investigations (history,
physical exam, urinalysis, urine culture and ultrasound of kidney and bladder).
 Relative to desmopressin, enuresis alarms are superior in that once the child achieves dryness, there is
less chance of relapse. The effects of desmopressin are immediate, whereas enuresis alarms take
longer to reduce frequency of bedwetting.

6. What should laboratory test she should do for her son?


a) Urinalysis
b) Serum electrolytes.
c) Serum creatinine
d) Serum albumin.
Investigations
 History with attention to: family history of urinary incontinence, bowel function; constipation, pattern
of wetting, history of UTIs or urologic surgery and psychological status of child.
 Physical examination with attention to: perineal sensation and reflexes, sphincter tone, genitalia,
particularly the urethral meatus, possible occult spinal dysraphism, direct observation of voiding.
 Other: diary to record voiding pattern
and/or bowel movements, urinalysis &
urine culture

7. What is the most appropriate treatment


for this child?
a) Mirabegron
b) Oxybutynin (day time enuresis)
c) Toltoridone (day time enuresis)
d) Desmopressin
8. pt. with scabies and has ragweed allergy what is the appropriate treatment:
a) Crotamiton
b) Isopropyl myrstate
c) Permethrin
d) Pyrethrin

9. What is the information you will tell the pt. upon


counselling?
a) You can use zinc oxide for purities.
b) You can use benzocaine for purities
c) Put clothes in hot water and keep
unwashable items in sealed bags for 5-7 days.
d) Extended pruritus after treatment need further
assessment.

10. A woman come to you complaining that she has trouble hearing. she told you that she
previously treated and removed excessive wax from her ear. What should you tell her?
a) Clean your ear using a clean cloth.
b) You can use olive oil to remove wax (To be used as prophylaxis to avoid recurrence)
c) Use chlorbutol drops 2-3 times daily for 3-5 days
d) Refer her to a doctor.
11. A pt. presented with fatigue and upon assessment found that left ventricular ejection fraction
was 35% his current medication includes metoprolol, what medication he should start:
a) Digoxin
b) Ramipril
c) Valsartan
d) Continue current metoprolol therapy.
Ramipril  Ejection fraction is less than 40

12. What medication will decrease cardiovascular risk except?


a) Perindopril
b) Atenolol
c) Valsartan
d) Bumetanide
Beta Blockers used in HF are only Carvedilol and Bisoprolol and Metoprolol
Bumetanide is loop diuretic as substituent for Furosemide

13. After he took treatment his symptoms (edema-fatigue-dyspnea) still persist. What should
you add?
a) Spironolactone
b) Furosemide
c) Amlodipine

14. A patient, who is asthmatic and had previous MI and he had CABG from about one year.
He presented with symptoms of stable angina; his current medications are Metoprolol & Ec-
ASA 81 mg once daily. What drug need to be optimized
a) EC-ASA
b) Metoprolol
c) Atorvastatin
d) Amlodipine
He is asthmatic and if we
optimized metoprolol, it would
increase risk of asthma.
15. After that he bring a prescription with nitrates. What is the action of nitrates?
a) Increase arterial resistance with no effect on cardiac output.
b) Decrease arterial resistance and increase cardiac output
c) Decrease arterial resistance with no effect on cardiac output
d) Increase arterial resistance and increase cardiac output
CTC: Nitrates are primarily venodilators. As a consequence, they reduce cardiac preload and, with it,
myocardial oxygen demand.
Although organic nitrates can dilate both arteries and veins, venous dilation predominates when these drugs
are given at normal therapeutic doses. Venous dilation reduces venous pressure and decreases ventricular
preload. This reduces ventricular wall stress and oxygen demand by the heart, thereby enhancing the oxygen
supply/demand ratio. A reduction in preload (reduced diastolic wall stress) also helps to improve
subendocardial blood flow, which is often compromised in coronary artery disease. Mild coronary dilation or
reversal of coronary vasospasm will further enhance the oxygen supply/demand ratio and diminish the anginal
pain. Coronary dilation occurs primarily in the large epicardial vessels, which diminishes the likelihood of
coronary vascular steal. Systemic arterial dilation reduces afterload, which can enhance cardiac output while
at the same time reducing ventricular wall stress and oxygen demand. At high concentrations, excessive
systemic vasodilation may lead to hypotension and a baroreceptor reflex that produces tachycardia. When
this occurs, the beneficial effects on the oxygen supply/demand ratio are partially offset. Furthermore,
tachycardia, by reducing the duration of diastole, decreases the time available for coronary perfusion, most of
which occurs during diastole (click here for more details).

16. A 35 years old woman come to you and told you that she worries
because she has family history of AMD. She said that she heard about a
formula contain (zinc, copper, vitamin C, vitamin E, B-carotene) that will
help her in prophylaxis. she is not smoker. What will you tell her?
a) She didn’t need to take anything
b) She can take this formula as it is
c) She can take this formula without B-carotene
d) She can take this formula but replace B-carotene with lutein.
NO, we do NOT start. As long as the patient is asymptomatic, there's NO evidence for AREDS. You then need
to advise the patient about lifestyle changes that can help reduce their risk, whenever applicable.
CTC: Amsler Grid Instructions for the patient:
 Use adequate lighting and wear the glasses you would normally use to read.
 Cover 1 eye at a time and test the other.
 With the open eye, stare at the centre dot while considering the following questions:
 Is any corner or border of the grid not visible?
 Are there any broken or missing lines?
 Are any of the lines blurry, curved or wavy?
 Repeat the test 2–3 times per week. Tell your eye‐care professional if you answer yes to any of the
questions.
17. What combination to avoid in treatment of HIV
a) Tenofovoir TDF and didanosine TD= Adverse drug rx
b) Tenofovoir TDF and emtricitapin
c) Abacavir and lamivudine
d) Atazanavir and ritonavir

18. A patient with HIV, he also has hypertension and hyperlipidemia. His current medication
includes atorvastatin 10 mg, amlodipine 10mg & tenofovir + efavirenz. What should be done?
a) Increase dose of statin and keep amlodipine
b) Increase dose of statin and increase dose of amlodipine
c) Keep dose of statin and keep amlodipine
d) Keep dose of statin and increase amlodipine.
Efavirenz is a CYP3A4 inducer → ↑ amlodipine and sta n metabolism.
Lexi: Efavirenz may decrease the serum concentration of AtorvaSTATin.
Efavirenz may decrease the serum concentration of Calcium Channel Blockers

19. What should he monitor for HIV?


a) Liver ultrasound
b) Genotype
c) TSH
CTC: liver (ALT, AST, bilirubin, INR), renal (creatinine, estimated glomerular filtration rate [eGFR], serum
phosphate, urinalysis, urine albumin‐to‐creatinine ratio [UACR], urine protein‐to‐creatinine ratio [UPCR])
FOR AN ARV REGIMEN, A CLINICIAN WOULD MONITOR THE FOLLOWING:
Rash (especially initially), GI upset such as NVD (especially initially as likely transient).
Lipid battery (ARV drugs can mess up with cholesterol) and lypodystrophy, BMD (DXA).
Kidney function (GFR) is important for safety of ARV regimens, especially tenofovir disoproxil fumarate.
Liver function (ALT) and symptoms of hepatotoxicity (NV, dark‐colored urine, fatigue) for ARV drugs that are
nasty to the liver, especially nevirapine and efavirenz.
CNS SEs for ARV drugs with neurotoxocity such as evafirenz, especially if INH is required for LTBI, CBC with
diff., HGB, iron markers,etc. for ARV drugs with nasty blood toxicity (bone marrow suppression) as zidovudine.
ECG (QT prolongation),
DDIs (PLEASE review the most relevant ones from your Book, especially those related to ritonavir and
cobcistat when combined with common meds such as statins & antihypertensive drugs)
NOTES:
Genetic testing (HLA‐B*5701) that predispose patients to abacavir (ABC) hypersensitivity reaction is ordered
BEFORE starting an ARV. Genotyping testing is ordered only AFTER the viral load remained persistently
elevated above 200 copies/mL on an ARV regimen.
Please remember:
ADRs are PART OF YOUR MONITORING PLAN, and In case of serious life‐threatening events (e.g., hyper
sensitivity reaction due to ABC, symptomatic hepatotoxicity, or severe dermatological reactions) recommend
immediate D/C of ALL ARV drugs and reinitiate an alternative regimen WITHOUT overlapping toxicity.
20. A patient will travel to Mozambique after few weeks. he has a history of seizure. His current
medication includes CBZ, he asked you about non pharmacological treatment to protect against
mosquito bites what should you tell him
a) Apply DEET to all your body before any outdoor activities (X Not all your body)
b) Wear long dark sleeves before outdoor activities.
c) Sleep under fine mosquito net
d) Apply DEET, and after 20 min apply sunscreen before outdoor activities.
Wear clothes covering exposed skin with long sleeve white colour ok but dark one is wrong
Sleep in an air‐conditioned or screened room.
Concurrent application of sunscreen with the insect repellent N, N‐diethyl‐meta‐toluamide (DEET) may lower
SPF but the efficacy of DEET appears to be maintained. If application of DEET is delayed after the sunscreen
has been applied, the sunscreen product maintains more of its original SPF.

21. After that he come and bring Rx for mefloquine. What will you do?
a) Tell him to take it in the day he will leave.
b) Tell him to continue take it for two weeks after he returns
c) Call doctor and discuss with him to change mefloquine to another drug
Mefloquin is Contraindicated in patients
with a history of seizures, depression,
generalized anxiety disorder, psychosis,
schizophrenia or other psychiatric
disorders, self‐endangering behaviour,
suicide attempts or suicidal ideations.

22. A35 years old patient presented with pleuritis. After assessment it is shown that he has
meningitis. What is the causative M.O for meningitis?
a) Staph Aureus
b) S. Pneumonia
c) N. Gonorrhea
d) M.Cattaralis.
Pleurisy, also known as pleuritis, is inflammation of the membranes that surround the lungs and line the chest
cavity (pleurae). This can result in a sharp chest pain while breathing.

23. What is the empiric treatment?


a) Vancomycin+Gentamicin
b) Vancomycin+Ceftriaxone
c) Ampicillin+Gentamicin
24. What treatment should start before empiric treatment
a) Dexamethasone
b) Acetaminophen
Patients frequently experience poor outcomes due to a brisk inflammatory response that causes neuronal
damage. Limiting this inflammation by administration of corticosteroids before or with the first dose of
antimicrobials. CTC: Pleuritis: NSAID – Dexamethasone‐ Methotrexate (refractory)

25. Upon examination CSF what indicate infection


a) Decrease lactate
b) Increase glucose
c) Increase protein
d) Decrease white blood cells.

26. What vaccine should this patient receive after treatment


a) Meningococcal vaccine
b) Pneumonia vaccine
c) Flu vaccine
d) Varicella vaccine
There's NO point or need for meningitis vaccine. It is a completely WRONG answer.
when meningitis vaccine is indicated? (ONLY if mandated by a travel such as Hajj to Mecca or as part of
routine vaccination in infancy / childhood) Keyword analysis AND
Facts:
1) Keyword: Pleuritis (translate) = hematogenous meningitis from lungs = Strep pneumonia primary suspect.
2) Meningitis vaccines are generally NOT indicated for meningitis immunocompetent patients (unless you've a
compelling need such as asplenia, HIV, etc.)

27. Another of child who diagnosed with ADHD. Came to you and complaining that his child
has insomnia that make him unconcentrated in school. His current medication is
methylphenidate 7.00 am and 3.00 pm what should you told her
a) All stimulant cause insomnia.
b) Contact doctor to change the
medication
c) Switch drug to long acting
formulation
d) Keep taking the drug with Change
timing to 7.00 and 12 noon
28. After that she came again and told you that her child complaining from headache what you
will tell her
a) Headache is transient side effect and Will disappear upon continue using the drug
b) Her child will tolerate headache by using drug
c) If headache is bothersome. She can contact her son's doctor
d) Switch to long acting medication which has fewer side effects
Common, usually transient: anorexia, insomnia, weight loss, irritability, dizziness, weepiness, headache,
abdominal pain. (monitor weight & appetite every 6 months)
Transient ‐ stop and re‐evaluate: “zombie‐like” effects, psychotic reactions (such as hallucinations), agitation,
tachycardia, hypertension, growth failure (Monitor growth suppression, record weight and height at baseline
and then every 3–6 months), rebound hyperactivity, leukopenia, blood dyscrasias.

29. After that she got a prescription with atomoxetine. She asked you about the difference
between atomoxetine and methylphenidate. What will you tell her?
a) Atomoxetine is non stimulant and may cause somnolence
b) Both are stimulant and cause insomnia
c) Methylphenidate is stimulant and has more side effect than atomoxetine

30. After that she told you that she will travel and want to transfer atomoxetine and
methylphenidate to pharmacy where she will go what will you told her?
a) Both are not permitted to be transferred.
b) Both are permitted to be transferred once
c) You can transfer atomoxetine but methylphenidate needs a new written RX
d) You can transfer atomoxetine and call doctor for new RX of methylphenidate.
Because it can be verbal
In fact, the patient is the one who will call the doctor, so the clinic phones in a prescription to the new
pharmacy since the patient has to indicate which pharmacy in the new residence they will deal with.

31. A woman presented with flu like symptoms, nausea, insomnia and anxiety. The patient
didn't take his medication because he can’t pay. What drug this patient was talking?
a) Morphine
b) Diazepam
c) Alcohol
d) Fluvoxamine  Most nauseating SSRIs
SE: Agitation, nervousness, anxiety, insomnia, somnolence, tremor, headache, dizziness, anorexia
Palpitations/ tachycardia, Abdominal pain, constipation, diarrhoea, dry mouth, dyspepsia, nausea, vomiting
Hyperhidrosis, Sweating, Asthenia, malaise
32. What can you out do to help this patient getting his medication?
a) Suggest to hook patient up with social services program (>1 DRUG)
b) Search for the patient for a manufacturer plane (only 1 drug)
c) Provide him with samples from doctor
d) Ask doctor to change drug to less expensive one
If very expensive → manufactuer plan. Acute → sample from a Dr. Brand → change to generic

33. Carbamazepine cause Rash. That is my develop to SJS, what gene should be ensure that it is
not +Ve before giving CBZ: (see in may 17)
a) HLA_B*1502
b) G-6-PD
c) Gynotype
HLA Association with Drug‐Induced Adverse Reactions
➢ Abacavir Hypersensitivity and HLA‐B∗57:01 (Skin)
➢ Carbamazepine and Oxcarbazepine Hypersensitivity and HLA‐B∗15:02, HLA‐B∗15:11
➢ Allopurinol Hypersensitivity and HLA‐B∗58:01 (Skin)
➢ Dapsone Hypersensitivity and HLA‐B∗13:01 (Skin)
➢ Amoxicillin‐Clavulanate‐Induced DILI and HLA Haplotypes
➢ Flucloxacillin‐Induced DILI and HLA‐B∗57:01 Association
➢ Antithyroid Drug‐Induced Agranulocytosis and HLA‐B∗38:02‐HLA‐DRB1∗08:03 Haplotype

34. Pregnant women got DVT what treatment should she start
a) Warfarin
b) Clopidogrel
c) Rivaroxiban
d) Enoxaparin

35. Pt with atrial fibrillation started taking amiodarone from two months, what should monitor
a) Glucose
b) Hypertension
c) Neurotoxicity
d) Corneal deposition
Neurotoxocity (eg, ataxia, peripheral neuropathy such as numbness). Corneal microdeposits are almost
universal in amiodarone patients and can be seen in almost every patient after 6 mo on amiodarone. They're
mostly benign and do NOT necessitate dose adjustment unless symptomatic (uncommon).

36. Test to detect DVT?


a) D-dimer → (for claudication also)
b) Factor V leiden  for stroke
c) Fibrin
37. A patient got myalgia. He is taking rosuvastatin 40 mg. He stops his medication. He has
previous MI and he has hypertension. Which drug could he restart?
a) Atorvastatin 40 mg
b) Ezetimibe
c) Gemfibrozil 600 mg
Atorvastatin: 10 to 20 is moderate – 40 to 80 is high. Rosuvastatin: 5‐10 is moderate – 20‐ 40 is high
In case of CVD like MI  I can not use ezetimbe alone as it considers week even if i have problem with statin.
In this case, i have to choose low dose then titrate. So, here just change
if the question without any CV problem and has a problem with

38) What should he monitor for myalgia?


a) Weight
b) Length
c) Urine discoloration → cause dark urine

39) Patient is taking statin and warfarin what should he do?????


a) Take them together.
b) Short term increase in statin and no change in warfarin
c) Short term decrease in warfarin and no change in statin
d) Long term decrease in warfarin and no change in statin
HMG‐CoA Reductase Inhibitors (Statins) may enhance the anticoagulant effect of Vitamin K Antagonists.
HMG‐CoA Reductase Inhibitors (Statins) Interacting Members Fluvastatin*; Lovastatin*; Pitavastatin;
Pravastatin*; Red Yeast Rice; Rosuvastatin*; Simvastatin* Exceptions AtorvaSTATin

40) You are planning for a seminar for seniors with pervious stroke to talk about life style
changes which health care professional you will invite?
a) Dietician
b) Chiropractor
c) Occupational therapist
41) We want to make a seminar for diabetes, what will you do in order to invite patient?
a) Put a promotion paper in the dispensing area
b) Make pamphlets and put in medication
c) Call patient who are taking oral hypoglycaemic medications
d) Call patients who are dispensing sugar kits

42) What specialist will you invite for this seminar?


a) Physiotherapist
b) Diabetic educator
c) Podiatrist
d) Registered nurse

43. Which of these BBs is LEAST appropriate for a patient with typical chronic stable angina?
a) Atenolol
b) Metoprolol
c) Bisoprolol
d) Acebutolol selective with ISA
Acebutalol because it is used only in case of bradycardia. A, B, C are selective.

44. In the day of the event there is a workload, who will you take with to help you?
a) A technician
b) Manager of the pharmacy
c) Nurse

45) A diabetic patient taking insulin NPH before breakfast and after supper. He also takes
regular insulin before meals he wakes up with hyperglycaemia. He is diagnosed to have dawn
phenomenon what should be done:
a) Decrease after supper dose NPH
b) Increase before breakfast dose of NPH
c) Increase NPH after supper dose
d) Increase before supper dose of regular insulin
Dawn  increase NPH after supper

46) A diabetic patient is taking oral hypoglycemic medication he found that blood glucose level
elevated before and after meals what is the best insulin regimen, he should start with
a) Insulin NPH after supper
b) Insulin glargine 10 units before bed time)
c) Insulin NPH before breakfast and after supper and regular insulin before each meal
Can not begin with two insulin. Bed time basal  should be long acting
Glargine is Long acting, 1st line to be ADDED when oral fail to control
47) What goal of treatment after starting therapy?
a) HA1C <7% after two months x 3 months
b) Randomized blood glucose level 12 mmhg
c) Post prandial blood glucose level 12-16
mmgh x
d) Fasting blood glucose level is 4-7 mmgh

48. Patient has psychosis. He is on haloperidol.


Today, he comes with Rx of Resperidone.
When you council him, you noticed that he cannot stand for long time and he bends his legs
many times. What is the effect that the patient suffers from?
a) Rigidity
b) Akathesia
c) Acute dystonia
d) Tardive dyskinesia
Akathisia is a movement disorder that makes it hard for you to stay still. It causes an urge to move that you
can't control. You might need to fidget all the time, walk in place, or cross and uncross your legs.
Usually, akathisia is a side effect of antipsychotic drugs

49. A patient is diagnosed for psychosis and he is stabilized on (not remember) how long he
should continue medication or What is the treatment duration for this episode?
a) 6 months
b) At least 1 year
c) Indefinitely
N.B. there was not 2 years or 5 years in the choices
Depression  first episode one year
Continue maintenance pharmacotherapy for at least 1–2 years for first‐episode patients who achieve
symptom remission and functional recovery. Longer treatment (2–5 years) may be required for individuals
with a long duration of untreated psychosis, more severe illness, slower response, substance abuse and
history of suicidal or aggressive behaviour

50. What is contraindicated for flu vaccine (quadrivalent, live attenuated (LAIV) FluMist)?
a) Patient < 2 years
b) Immunocompromised patients
c) Patient who have egg allergy
d) Patient who took another vaccine from 3 months
51. Patient traveled to Africa. He got diarrhea, what could be the causative organism?
a) Rotavirus
b) Legionella monocytogenesis
c) S. Saprophyticus
d) Salmonella typhi
Bacterial: Ecoli‐salmonella‐shigella >>>> Viral: Norovirus‐Rotavirus

52. After that he wants to get vaccinated before travelling again. He will take all these vaccines
except
a) Typhoid vaccine
b) Dukoral vaccine
c) Hepatitis B vaccine
d) Cholera vaccine

53. What is true about human papilloma virus vaccine


a) It is used to protect against breast cancer
b) It can be given to females from 9 years old

54. A patient travelled in a trip. What infection he could get after eating unclear food
a) Hepatitis A infection
b) Tetanus infection
c) Varicella infection

55. Doctor wants to rule out other types of hepatitis infections. What doctor will ask about?
a) Alcohol consumption
b) Eating outdoors
c) Sexual activity

56. A woman has her lab test as follow: TSH 20 (normal 0.4 – 4.5) what symptom she is
suffering from
a) Weight loss
b) Oily skin
c) Cold intolerance
d) Palpitation

57. After that she wants to get pregnant. What would you advise her?
a) Take multivitamins
b) Start propylthiouracil before getting pregnant & switch to methimazole after 1st trimester
c) Increase Levothyroxine by 25-50% or take 2 extra tablets for 2 days/week
58. A woman is complaining from rapid heart rate. She has lost weight about 12 kilograms &
has low TSH. What treatment should she start?
a) Levothyroxine
b) Methimazole

60. What drug interact with clarithromycin


a) Tobramycin
b) Quetiapine
Lexi: QUEtiapine may enhance the QTc‐prolonging effect of QT‐prolonging Strong CYP3A4 Inhibitors
(Moderate Risk). QT‐prolonging Strong CYP3A4 Inhibitors (Moderate Risk) may increase the serum
concentration of QUEtiapine.

61. What is the ordinary label of Azithromycin suspension


a) Take with or without food
b) Take before breakfast
c) Separate from food with one hour before or two hours after taking drug
Oral suspension and tablet azithromycin → with or without food
Capsule of azithromycin → one hr before meal or 2 hrs a er meal

62. Case of Testosterone: Patient diagnosed with hypogonadism, can’t tolerate exercise, the
doctor gave him testosterone patch but he suffered from allergic reaction in the site of the patch,
so the doctor prescribes oral testosterone. what to counsel him:
a) Take it with food for better bioavailability
b) Since he had allergy to patch, he can’t use oral
c) Take on empty stomach
Oral. To ensure absorption, Restandol Testocaps must be taken with a normal meal, if necessary, with a
little fluid, and be swallowed whole without chewing. It is preferable that half of the daily dose be taken in
the morning and the other half in the evening. If an uneven number of capsules is taken daily, the greater
part should be taken in the morning.

63. What is the benefit from taking testosterone?


a) It can prevent prostatic cancer
b) Protect bones from losing mineral density
Testosterone therapy may also be indicated in osteoporosis due to androgenic deficiency
HRT for men only on Hypogonadism
Restandol Testocaps is used in adult men for testosterone replacement to treat various health problems
caused by a lack of testosterone (male hypogonadism). This should be confirmed by two separate blood
testosterone measurements and also include clinical symptoms such as impotence, infertility, low sex drive,
tiredness, depressive moods, bone loss caused by low hormone levels.
Restandol Testocaps are used if your body does not make enough testosterone of its own, for example:
impotence caused by hormonal disorders after castration or a similar problem called eunuchoidism
decreased sex drive and decreased mental and physical activity certain types of infertility because you don’t
make enough sperm It may also be used to treat a type of bone disease (osteoporosis) caused by having too
little testosterone

64. What is the most S.E. of anastrozole


a) Dizziness
b) Allergy
c) Anemia
d) Arthralgia
Hot flushes are not in the answer

65. What to monitor when using anastrazole: Bone mass density


As anastrozole lowers circulating estrogen levels it may cause a reduction in bone mineral density with a
possible consequent increased risk of fracture.
Women with osteoporosis or at risk of osteoporosis, should have their bone mineral density formally
assessed at the commencement of treatment and at regular intervals thereafter.
Treatment or prophylaxis for osteoporosis should be initiated as appropriate and carefully monitored. The
use of specific treatments, e.g., bisphosphonates, may stop further bone mineral loss caused by anastrozole
in postmenopausal women and could be considered
Anastrazole is part of the risk factors in CAROC Score for Osteoporosis

66. Patient complaining from lower back pain. What is the non-pharmacological treatment
could help him?
a) Bed rest
b) Increase physical activity

67. What could you advise this patient to relief the pain
a) Hot-pads
b) Apply Ice-cubes
Back pain hot but cold pads in sport injuries
Acute treatment is best summarized by the RICE protocol:
Rest the injured part.
Ice: Wrap an ice bag, cold pack or package of frozen peas in a damp, thin cloth and apply to the injured area
for 15–20 minutes at a time, at least QID for the first 48 hours (or longer if swelling continues).
Compress with an elastic bandage if there is swelling such as in an ankle sprain.
Elevation: Try to elevate the injured part above the heart.
68. Which drug cause steven-Johnson syndrome
a) Fluoxetine
b) Trazadone
c) Phenelzine
d) Lamotrigine

69. MB female was diagnosed with osteoporosis. She has O.A. in her knee. She broke her arm
when she was young. She works as hair dresser and her work locates 2 km from her home. sHe
goes for swimming in weekend. She takes one glass of wine at weekend, and smokes 1 pack of
cigarettes daily. What is the risk factor of osteoporosis?
a) Previous fracture (young age<40 so not counted)
b) Smoking
c) Alcohol
d) Osteoarthritis

70. She wants to change his life style & her work is located 2 km to his home. What change will
help her to improve her symptoms?
a) Decrease alcohol intake
b) Walk to work everyday
c) Control risk of falls at home
d) Increase swimming activity 3 times weekly
CTC: Nonpharmacologic Choices Recommended for everyone:
 Regular exercise (especially impact type)
 Fall prevention: minimize hazards for falling in the home (e.g., remove throw rugs, install grab bars
in bathrooms, ensure adequate lighting), assess drugs implicated in falls such as benzodiazepines
and other psychotropics, improve strength and balance
 Smoking cessation
 Dietary measures: encourage adequate protein, calcium and vitamin D intake, avoid excessive
alcohol (>2 drinks/day) and caffeine (>4 cups of coffee per day or equivalent)
71. After that she told you that she heard about strontium which is beneficial for her symptoms.
When you searched you found that its indication in OP is not approved. What reference you
used to find information about strontium
a) Compendium of pharmaceuticals and specialities
b) Micromedex
c) Pub-med
There was not Martindale in choices
Strontium ranelate is not available or approved for osteoporosis in Canada. However, some patients obtain
strontium ranelate from Europe. Strontium is widely promoted for treatment of osteoporosis. In Europe,
strontium ranelate is available as a prescription drug. In the United States, strontium is a dietary
supplement available as the carbonate, chloride, citrate, gluconate, and sulfate salts.

72. Which drug interacts with coffee and increases its toxicity
a) Clarithromycin
b) Penicillin
c) Ciprofloxacin
Lexi: CYP1A2 Inhibitors (ciprofloxacin) may increase the serum concentration of Caffeine and Caffeine
Containing Products

73. What type of enzyme that verapamil inhibits


a) 2D6
b) 2C9
c) 2C19
d) P-glycoprotein
DDI with Digoxin

74. Question about dutasteride (memorize all the side effects)


Decreased libido and semen volume, erectile dysfunction, headaches, dry skin.
CTC: The most common adverse reactions reported in subjects receiving dutasteride were impotence,
decreased libido, breast disorders (including breast enlargement and tenderness), and ejaculation disorders

75. What to council a patient using chlorhexidine


a) Keep in mouth for a while before swallowing
b) It could stain teeth by prolonged use
c) It should be diluted before use
d) You could eat or drink directly after using
76. Patient is taking PPI. What should be monitored for
a) Hip fracture
b) Osteonecrosis (Dexamethasone)
c) Gastric bleeding (NSAIDs)

77. A patient with a cold sore. He come to you and told you that he has a lesion from 5 days and
it began to crust from about one day. He asks you about something to accelerate healing of the
lesion. What you will tell him
a) He can take oral antiviral
b) He can take ducosanol (72 hours)
c) He can use ibuprufen
d) Nothing to accelerate lesion healing

78. A diabetic patient came to you complaining that after wearing a new shoe he found a red
lesion on his leg. This lesion began to worsen and being infected. What could be the causative
micro-organism of the infection?
a) S. Viridians
b) P. Aeruginosa
c) Chamilobacter
Most Likely Pathogen involved in Diabetic Foot Infection: Staphylococcus aureus (MSSA or MRSA), beta‐
hemolytic streptococci (group A or B most common), Streptococcus pyogenes (group A streptococcus),
Pseudomonas aeruginosa, Gram‐positive bacteria including enterococci, Gram‐negative bacteria and
anaerobic bacteria
Macerated foot Tissues are excessively moist and macerated from soaking the foot Pseudomonas
aeruginosa may be the predominant pathogen.

79. What is the empiric treatment


of this infection?
a) Amox. /clav.
b) Clindamycin
c) Ampicillin
d) Ceftriaxone
80. A skateboarding teenager fell on his knees. He came to you in the pharmacy and show you
his knee which was red & bruise. After you examine it you found that the wound is clear from
any debris. What should be done?
a) Clean the wound with 70% alcohol and leave it
b) Leave the wound to be dried in air
c) Clean, then, cover the wound with film dressing
d) Clean the wound, then, cover with fiber gauze dressing.
Self‐treatment of minor wounds includes the following steps:
Cleanse the wound: Remove dirt and debris from the wound as soon as possible to prevent infection and
promote healing. Once debris has been removed, carefully wash the wound with water. Drinkable tap water
is as effective for wound cleaning as saline or purified water, with no increased risk of infection or decrease
in wound healing
Stop the bleeding: Apply a clean dressing or gauze to the wound area for 10 minutes. If the bleeding does
not stop within 10 minutes (or 15 minutes if anticoagulated), the patient should seek emergency medical
attention. Monitor those on anticoagulants for up to 15 minutes, as the clotting time will be longer, and
refer if the bleeding does not stop within that time period. Visible pieces of dirt or other foreign material
that remain after irrigation can be gently picked out of the wound with tweezers that have been cleaned
with rubbing alcohol, or by brushing gently with clean gauze.
Apply a dressing: Choose a dressing that maintains a moist wound bed (see Dressings) to protect the wound
from possible infection and improve the healing process. Minor cuts, paper cuts or skin cracks can be closed
using tissue adhesives or a liquid bandage. Larger cuts, where the edges won't stay together, the edges are
jagged or the wound is deep, may need stitches.
Débridement: The removal of foreign material such as dead or contaminated tissue from the wound should
be performed only by an appropriately trained health‐care practitioner and under sterile conditions

81. You are in the pharmacy & have many medications unused. You want to return them to the
manufacturer. Upon contacting manufacturer, he said that he will take any medication except
that needs to be refrigerated. What medication the manufacturer will not accept?
a) Latanoprost eye drops
b) Chloramphenicol tablets
c) Vancomycin

82. What from the following represent a prescription cascade?


a) HCTZ → hypocalcaemia → ca++
b) Meloxicam → abdominal pain → PPI.
c) Mirtazapine → insomnia → lorazepam. (cause sedation)
d) Donepezil → constipation → Laxative. (cause diarrhea)
Meloxicam: Gastrointestinal effects GI bleeding, ulceration or perforation, which can be fatal, has been
reported with all NSAIDs at anytime during treatment, with or without warning symptoms or a previous
history of serious GI events. The risk of GI bleeding, ulceration or perforation is higher with increasing NSAID
doses, in patients with a history of ulcer, particularly if complicated with haemorrhage or perforation, and in
the elderly. These patients should commence treatment on the lowest dose available. Combination therapy
with protective agents (e.g. misoprostol or proton pump inhibitors) should be considered for these patients.

83. Patient has rosacea on


Metronidazole cream has only 50%
improvement what you give:
a) Oral tetracycline
b) Oral isotretinoin
What trigger Rosacea? CTC:
Sunlight, Heat, Hot beverages, Spicy
foods, vinegar, Alcohol, Use of
astringents (alcohol‐ or acetone‐based
products), Emotional stress.
Medications: calcium channel blockers,
niacin, nicotinic acid, nitrates, topical
corticosteroids, sildenafil.

84. A patient has shingles and she is complaining of rash and severe pain. She has Rx
amitriptyline 10 mg. After you dispense the medication. She came again to you and told you
that when she read the medication leaflet at home, she found that it has many side effects and
she do not want to take it. What should you tell her?
a) The benefit you get from relieving pain outweigh the side effects of the drug.
b) These side effects are usually tolerated by most patients.
c) Side effects not happen with all most patients.
d) This dose is very low to cause these side effects.

85. After a time she came again to you. She told you that the symptoms are controlled but she
still has pain. What could be used locally for the pain?
a) Calamine.
b) Lidocaine.
c) Antibiotic.
Post herpetic topical options: 1st line Lidocaine 2nd Line capsaicine

86. Now, there is ocular involvement for 5 days what to give:


a) Analgesics only
b) Oral Antiviral
c) Trifluridine eye drop
d) Refer to emergency for IV methyl prednisone
87. Patient with CAP. He is taking IV. Therapy doctor wants to stepdown him to oral
medication. What is not suitable cause to stepdown this patient?
a) Diarrhea
b) Persistent cough
c) +ve sputum culture upon admission.
d) Chest x-ray showed consolidation upon admission.
Switch patients from IV to oral antibiotics when the following criteria are met: GI tract is functioning
normally, hemodynamically stable; 2 temperature readings are normal, normalized white blood cell count;
subjective improvement in cough and shortness of breath; able to consume oral medications.

88. After that doctor want to discharge this patient. All of the following are monitored for
discharging expect
a) Pulse.
b) Spirometer rate.
c) Temperature.
d) Sputum production.

89. Patient with RLS. He has peripheral vascular disease; he drinks one cup of coffee per day.
What is the risk factor for RLS (I think he is not heavy smoker)?
a) PVD.
b) Coffee. (he drinks within the range)
90. We do a blood test and understand that his Hb is low what we should examine in addition (I
really can't remember)
a) B12
b) Iron index
CBC, electrolytes, BUN, creatinine, fasting glucose, serum iron, ferritin and iron saturation
The most common microcytic and hypochromic anemias are iron deficiency anemia and thalassemia trait.

91. What is the non-pharmacological


advice for him?
a) Stop coffee.
b) Increase activity.
c) Have a best rest.

92. What is the pharmacological


treatment for this patient?
a) Pramipexole.
b) Bromocriptine.

93. FM is 62 years old female. She


has cystitis. What is the possible
microorganism?
a) Shigella.
b) Klebsiella pneumonia
c) Gonorrhea.

94. What is the suitable treatment?


Syndrome Infecting organism & Symptoms Firs-Line Second-Line
Acute Escherichia coli (80– 90%), SMX/TMP PO × 3 Fluoroquinolone
Uncomplicated Staphylococcus saprophyticus (5– 10%), days or (ciprofloxacin,
UTI (Cystitis) Klebsiella pneumoniae, Proteus mirabilis Trimethoprim PO levofloxacin,
Occurs in factors promoting infection include sexual × 3 day or norfloxacin), PO ×
females with intercourse, use of spermicide or diaphragm Nitrofurantoin PO 3 days
normal Symptoms include internal dysuria, hematuria, × 5 days or or
genitourinary frequency, suprapubic discomfort and urgency. Fosfomycin Cephalexin PO × 7
tract. Recurrences are common. tromethamine as days
Urine Culture: Generally, not recommended. single dose PO

95. Post antibiotic effect is? The suppression of bacterial growth that persists after short exposition of
organism to antimicrobial’. The Post-Antibiotic Effect (PAE) shows the capacity of an antimicrobial drug
to inhibit the growth of bacteria after removal of the drug from the culture”
96. Patient with GERD after eating pizza has abdominal pain and vomiting for 3 days (2-3
times/day) and he has seizure, takes Carbamazepine, he is seizure-free for 10 years, what is
your recommendation:
a) Refer to doctor for further assessment
b) Refer to get PPI Omeprazole
a) Famotidine OTC
b) Sodium alginate
c) Daily magnesium

97. The doctor diagnosed him with H. Pylori and gave him:
a) Dexlansoprazole
b) Amoxicillin for 10 days
c) Clarithromycin 500 bid for 10 days
Treatment Eradication of Helicobacter pylori:
of PUD due Quadruple Therapy: FOR 14 DAYS
to ● Recommended option (First line or prior treatment failure) → PPI (BID), bismuth
Helicobacter subsalicylate (2 tabs QID), metronidazole (500 mg TID-QID), tetracycline (500 mg QID).
pylori ● Recommended option (First line) → PPI (BID), amoxicillin (1 g BID), metronidazole
Infection (500 mg BID), clarithromycin (500 mg BID).
Triple Therapy: FOR 14 DAYS
● Recommended option (Prior treatment failure only) → PPI (BID), amoxicillin (1 g BID),
levofloxacin (500 mg QD)
● Restricted option (First line) →
1. PPI (BID), amoxicillin (1 g BID), metronidazole (500 mg BID).
2. PPI (BID), metronidazole (500 mg BID), clarithromycin (500 mg BID).
3. PPI (BID), amoxicillin (1 g BID), clarithromycin (500 mg BID).

98. He can’t pay for Dexlansoprazole, and he needs something less expensive:
a) Give pantoprazole and inform the doctor first
99. He is good now, what is your concern about PPI:
a) He can stop it after seeing his doctor
b) He can stop it now
c) He can’t stop it and he need a long-term treatment because of h. pylori.
If patient come before 10 days of treatment and feeling good i have to tell him you have to continue on
medication from 10‐14. After 14 days if the patient is feeling good i can tell him you can stop the medication
after seeing the doctor in‐order to perform the test to confirm the eradication go H. pylori from 2‐4 if give
result continue to 8to 4 to include the first duration 2 to 4 weeks

100. He was smoking 10 cigarettes a day & now he starts smoking cessation with nicotine gums
(7 gums/day 2mg) and he is suffering now nausea, tremor, vomiting, irritability, restlessness
and difficulty in sleeping. What could be the reason of these symptoms?
a) Toxicity of carbamazepine
a) Nicotine withdrawal because of low dose of gums or Subclinical dose of nicotine
leads to nicotine withdrawal.
b) Caffeine toxicity
CARING: Craving, Agitation / Anxiety, Restlessness, Insomnia, Nervousness, Gain wt.
CTMA: Initial dose: 10–12 pieces/day PO; may increase to 20 pieces/day if needed

101. What to give for trigeminal neuralgia?


a) Oxcarbamazepine
b) Local lidocaine
c) Carbamazepine

102. Concerning the D-D interaction between Carbamazepine and Clarithromycin what is your
recommendation:
a) Give him the treatment and monitor carbamazepine toxicity
b) Call the doctor to change to another alternative (Azithromycin or FQs)
c) Hold the carbamazepine during clarithromycin treatment
d) Call the doctor to decrease clarithromycin dose to 500mg die
Clarithromycin CYP3A4 Inhibitor + carbamazepine → ↑ carbamazepine levels
Lexi: CarBAMazepine may increase serum concentrations of the active metabolite(s) of Clarithromycin.
Clarithromycin may increase the serum concentration of CarBAMazepine. CarBAMazepine may decrease the
serum concentration of Clarithromycin.

103. Patient used Nasonex for rhinorrhea and no effect: Ipratropium


Intranasal anticholinergics
Ipratropium Intranasal ipratropium reduces the volume of watery nasal discharge. It may be used when
Atrovent rhinorrhea is the only symptom or when rhinorrhea is refractory to topical INCS and/or
Nasal Spray antihistamines. Optimal dose based on specific patient's symptoms and response
0.03%: 2 sprays This may be especially helpful in some types of vasomotor rhinitis, e.g., “skier’s nose.”
per nostril BID- S.E: Nosebleeds, nasal dryness, dry mouth or throat, headache.
TID Avoid accidental release of nasal spray into eyes.

104. Insulin can’t be mixed: Glargine CTC: Appearance: clear…. Do not mix with other insulins.

105. What to stop before colonoscopy incase of diverticulitis?


a) Metformin → stop before contrast media
b) Iron
c) PPI, Bismuth and antibiotic → stop before Urea Breath test

106. 46 yrs old patient who is Diabetic, hypertensive, Dyslepidemic, alcoholic, heavy smoker
and has Dyspepsia, he is at risk of CVD due to all EXCEPT:
a) Smoking
b) Alcohol
c) Hypertension
d) Hyperlypedimia
e) Diabetes

107. How much is the limit amount of sodium in hypertensive patient: less than 2000 mg/day

108. Case of cancer: Patient has cancer takes Cisplatin, has hypertension, Diabetes, depression
(takes Bupropion). The doctor prescribes for him drugs for nausea and vomiting:
a) Aprepitant
b) Dexamethasone
c) Ondansetron
d) Prochloroperazine

109. What is your concern:


a) Aprepitant + bupropion
b) Ondansetron + bupropion  causes serotonergic syndrome
c) Dexamethasone may cause hyperglycemia  right also, but B is more dangerous

110. He has constipation why: Ondansetron.


Ondansetron: Headache – Constipation. If said diarrhea  choose aprepitant
111. No bowel movement for 3 days without any abdominal cramps what you give:
a) Psyllium
b) Bisacodyl
c) PEG
d) Refer
Constipation in cancer patient → give senna and biscodyl
Constipation with family history of cancer → refer

112. He has now nausea after eating not relief on prochloperazine what to give:
a) Metoclopramide
b) Olanzapine
c) Restart aprepitant
d) Nabilone

113. 26-year-old has unprotected intercourse 3 days ago, not using contraceptive, has migraine
with aura and she has a problem with compliance to medications, what is your concern:
a) Give her plan B  Approved for use up to 72 hours after unprotected sex
b) She comes too late after her relationship
c) Don’t give her plan B
d) Refer

114. She needs contraceptive what you give her:


a) COC pills
b) COC patch
c) Intrauterine device
d) Progesterone oral
115. Patient comes to emergency with diversion at 8 a.m. but he supposed to come to your
pharmacy at 9 a.m. to take his daily dose of Methadone, the doctor in the emergency didn’t let
him go because of his severe case of diversion so he decides to prescribe for him Methadone to
take it at the hospital, what he should do:
a) Get a temporary licence to prescribe methadone
b) Ask doctor to apply for exemption temporarily OBSOLETE
In the past, physicians required exemptions to prescribe methadone, but As of May 2018, physicians do NOT
require exemptions to prescribe methadone.

116. The nurse gave an idea that he shouldn’t take any opioid for his pain since he is on
methadone, so she violates:
a) Autonomy
b) Beneficence

117. Methadone withdrawal symptoms peak after?


a) 2-3 days
b) 2 weeks
c) 7 days

118. Interaction with Methadone except


a) Levofloxacine (Qt)
b) Quetiapine (Qt Highest Risk, Avoid combination)
c) Alesse
d) Zopiclone (CNS Depressant enhance same effect of Methadone)
If no except in question use quetiapine
Methadone is a substrate of CYP2D6 & CYP2C19
CYP2D6 Inhibitors CYP2C19 inducers CYP2C19 inhibitors
Amiodarone Quinidine • Carbamazepine • PPI’s (not pantoprazole)
Buproprion Fluoxetine Paroxetine • Phenytoin • Voriconazole
Sertraline • Primidone • Fluconazole
Diphenydramine Dimenhydrinate • Phenobarbital • Fluvoxamine
Chloroquine Terbinafine • Fluoxetine

119. 6-year-old with pinworm: Refer


CTMA: Refer all those with suspected pinworm infestation to a physician so the diagnosis can be confirmed.
This is especially important in pregnant women, children under 12, and those with renal or hepatic
impairment. Pyrental Pamoate is Schedule II

120. Patient with blocked ear: Olive oil


4–6 drops in the ear canal BID for up to 4 days or 3 drops in the affected ear QHS for 3–4 nights
121. Patient with UTI, doctor prescribes Nitrofurantoin, what should the pharmacist ask about:
a) Renal impairment
Anuria, oliguria, or significant impairment of renal function (creatinine clearance under 60 mL per minute or
clinically significant elevated serum creatinine) are contraindications to therapy with this drug.

122. What to council on imiquimod: treatment for anogenital warts


a) Apply at night and wash it in the morning by soap and water
b) Put dressing on it
Immune Stimulate innate and acquired immune responses, which ultimately leads to inflammatory
Response cell infiltration within the field of drug application followed by apoptosis of diseased tissue.
Modifiers Self-applied  Apply QHS 3 times per wk with 1–2 days between treatments × up to 16 wk
Imiquimod Wash hands before and after treatment application, wash treatment area with soap and water
5% cream 6–10 h after application, earlier if skin reaction occurs. Require 3 months for response.
3.75% cream S.E: Local irritation, pain, mild to moderate erythema, burning. Do not use in pregnancy.

123. A hospital will begin to buy commercial Pot. Chloride instead of preparing it, the
commercial product will make difference in all the following EXCEPT:
a) Waste amount of Pot. Chl.
b) Fulfilling need of Pot. Chl.
c) Total amount used of Pot. Chl.
The question has been written in different way in other exam, as what are the advantages of
buying commercial KCL and the answer is to reduce the error

124. 84- Patient with PJP and has glucose-6 phosphate dehydrogenase deficiency. What oral
drug he should take instead of cotrimoxazole.
a) Pentamidin.
b) Atovaquone.
c) Clindamycin.
CD4 <200 cells/mcL or thrush (Pneumocystis jirovecii pneumonia) Primary and secondary prophylaxis
Q. PCP  Preferred prophylactic therapy for PCP is SMX/TMP.
Oral candidiasis  Alternatives: dapsone PO, atovaquone PO or monthly inhaled pentamidine.
 Stop prophylactic if CD4 > 200 cells/mcL × ≥ 3 months
 Treatment of Oral candidiasis with azole antifungal agents.

125. Bupropion increased dose cause: Seizure

126. Pt. With MI and has certain symptoms, he is also beginning to take TPA, why is it contra
indicated?
a) Cerebral haemorrhage
b) Pericarditis
127. What drug need at least 10 days washout period to switch to another drug:
a) Meclobamide to fluoxetine (Meclobemide is 2-5 days to switch to any Antidepressant)
b) Fluoxetine to citalopram
c) Phenelzine to citalopram
d) Citalopram to paroxitine
Citalopram should not be given to patients receiving Monoamine Oxidase Inhibitors (MAOIs) including
selegiline in daily doses exceeding 10mg/day. Citalopram should not be given for fourteen days after
discontinuation of an irreversible MAOI or for the time specified after discontinuation of a reversible MAOI
(RIMA) as stated in the prescribing text of the RIMA. MAOIs should not be introduced for seven days after
discontinuation of citalopram

128. You are in province that allowed the pharmacist to do refills for chronic medications, he
run out of his medication and has no remaining refills, his doctor is out of town and he takes an
appointment after 1 week with another doctor, what you will do:
a) Give him 1-week refill
b) Call the new doctor to give you a new prescription
c) Send him to an emergency clinic to bring a new Rx.

129. Which one cause gingival hyperplasia:


a) Cyclosporine
b) Topiramate
c) Nitroglycerine
Drug‐induced gingival overgrowth is a side effect associated principally with 3 types of drugs:
anticonvulsant (phenytoin), immunosuppressant (cyclosporine A), and various calcium channel blockers
(nifedipine, verapamil, diltiazem).

130. What to do:


a) Decrease dose of drug
b) Apply proper oral hygiene in a circular motion not linear
c) Massage the gum using a brush in a linear motion

131. Patient has tremors what is the cause:


a) Hypertension
b) Hypoglycemia
c) Insomnia

132. What to give him:


a) Propranolol
b) Baclofen
c) Glucagon inj.
133. Least drug causing sexual dysfunction?
a) Spironolactone
b) Digoxin
c) HCTZ
d) Atenolol
e) Ramipril

134. All these drugs are high alert by ISMP, except?


a) ASA
b) Digoxin
c) Warfarin
d) Alteplase
e) Cisplatine
f) Hydromorphone
135. Patient has refill for lorazepam wants to travel for few weeks and needs to transfer one
refill to the pharmacy there, what you should tell him:
a) Lorazepam can be transferred only once

136. Side effect of flouroquinolones: Glucose abnormality


If there is neurotoxic i will choose it. Flouro cause neurotoxicity and phototoxcitiy and glucose abnormality

137. Tobramycin S.E. Vestibular toxicity


Ototoxicity (auditory and/or vestibular), nephrotoxicity, neuromuscular paralysis

138. Technician prepared a wrong concentration of MTX what to do:


a) Place a note about MTX different concentration
b) Do a meeting
Sometimes it's an issue related to just one employee; however, the wording of the question can be "you've
been told" or "it was raised to your attention", which means it is an allegation rather than a confirmed fact. In
this case, you review policies and procedures with the whole staff. However, if the pharmacy manager saw the
tech counseling, they should intervene on the spot. Another example would be the front store manager saw
the tech counseling patients a few times or saw them helping patient select OTC meds a few times and
informed the pharmacy manager, in this case the manager can meet first with all people involved in the event
to confirm the facts, then initiate disciplinary actions, as deemed necessary, in a one‐on‐one meeting with the
employee for whom the issue was confirmed. The same applies to errors, etc. Here too, it depends on the
objective of the meeting. So, if the manager is going for a disciplinary action because of repetitive and/or
negligence, then s/he should meet with whoever is involved. If the manager is reviewing policies or going RCA,
then s/he should be meeting with the whole staff (or all people involved).
Accidental daily Use a weekly dosage regimen default for oral methotrexate in electronic systems when
dosing of oral medication orders are entered.
methotrexate MTX Require a hard stop verification of an appropriate oncologic indication for all daily oral
intended for weekly methotrexate orders.
administration Double-check all printed medication lists & discharge instructions to ensure that they
indicate correct dosage regimen for oral MTX prior to providing them to the patient.
Provide specific patient and/or family education for all oral MTX discharge orders.

139. A patient is on valproate sodium and he suffers from GI side effect, what we should do?
a) Change to divalproex (SAME DOSE)
b) Change the medicaton

140. What can the technician do? Transfer tramadol

141. Girl forget 1 pill of COC? Take as soon as possible

142. Finasteride ¼ pill? Veracity (Alopecia 1 mg and BPH 5 mg)


143. Technician counsel a patient, what to do?
a) Take desciplinary action.
b) Advice him not to counsel again

144. Sildenafil counselling? Effect finish within 4 hrs


CTC: Half‐life of 3‐5 hours. The maximum recommended dosing frequency is once per day.
To be taken as needed approximately 30 – 60 minutes before sexual activity. However, SILDENAFIL may be
taken anywhere from 0.5 hour to 4 hours before sexual activity.
The manufacturers of sildenafil, tadalafil and vardenafil recommend that these drugs not be used in
combination with nitrates. In situations where nitrate use is required in a patient also receiving a
phosphodiesterase 5 inhibitor, effect on blood pressure can be reduced if there is sufficient time between
doses. Separate doses of nitrates and sildenafil and vardenafil by at least 24 hours. Allow 48 hours
between tadalafil administration and nitrates.

145. Donepezil gives effect in?


a) 2 weeks
b) 2 months
c) 3 months
d) 1 year
It is the only cholinesterase inhibitor approved
for all disease severities
The initial daily dose (5 mg) of donepezil is usually taken at night, but can be taken in the morning if sleep
disturbances occur. After 4 weeks, try increasing to the target dose of 10 mg/day. Adjust dose after 4 wk
Donepezil was effective in 3‐ to 6‐month trials in patients with mild to moderate Alzheimer disease (MMSE
score of 10– 26).

146. Man called and asked you if you have oxycodone in your stock what you should say to
him:
a) Do you have an Rx?
b) Give me your name and number and I will check and call you back

147. A pharmacy manager did a physcial count of morphine, he found 30 tabs more in
inventory. What can be the cause for this?
a) You entered Rx on system but didnt count them
b) You dispensed Oxycodone instead
c) Error due to proceeding pescription on system
d) You prepared 30 tabs but patient didn’t take it & you forget to cancel on the system.
Another version: Error in the stock of narcotics: Check sales purchase on the system

148. Computer system order (same as in other exams)


149. Tamoxifen DDI with paroxetine? It is a CYP 2D6 substrate
Inhibitors of CYP 2D6: Amiodarone Buproprion Fluoxetine Paroxetine Sertraline Diphenydramine
Dimenhydrinate Chloroquine Quinidine Terbinafine.
They will decrease tamoxifen metabolism and prevent it from converting to its active form, so no effect.

150. A physician prescribed potassium supplements 20mEq TID to patient but this patient does
not want to take supplements. He asks the physician if he can eat bananas instead. If each large
banana has 602 potassium. How many bananas should the patient eat each day? (Mwt K+=39)
Answer:
20 m. Eq TID = 60 m. Eq per day
m.Eq = M. Wt * Valency = 39 * 1 = 39 mg Each day = 60mEq = 60*39= 2340 mg
Each banana contains 602 mg No. of banana = 2340/6002 = 3.88 = 4 bananas / day

151. Addison disease, you have a chart with equivalence to methyl prednisone so methyl
prednisone was 4 and prednisone was 5 in the chart and there was dexamethasone (the chart
included their glucocorticoid and mineralocorticoid levels) and he was taking methyl
prednisone 15 mg BID and he wanted to change it to prednisone so what the suitable dose
would you give?
Answer:
Methyl Prednisone 4 : Prednisone 5
15 * 2 = X → X = 15 * 2 * 5 / 4 = 37.5 mg Daily

152. Patient is prescribed with Sinemet tablet. Sig: 1.5 tablet 5 times for 12 weeks. Patient told
pharmacist that he will no be able to divide tablet into half. Now pharmacist will divide the
tablet for him. How many tablets needed to divide?
a) 250
b) 230
c) 210
d) 300
84 (12 WK) x 5 x 1 = 420 tabs x 0.5 = 210 OR 2.5 TAB * 84 DAYS = 210 TABS

153. Patient in hospital is taking IV Cotrimoxazole (each ml contains 80 mg SMT / 16mg TMP)
0.5 ml Q6hr. Dr. want to shift him to liquid (200 mg STM / 40 mg TMP) in each 5 ml, how
many of liquid is needed to provide the equivalent dose as was taken IV.
Answer:
0.5 ml Q6hr = 0.5 * 4 = 2 ml /Day
SMT: 80 mg ------- 1 ml X mg ------- 2 ml X= 2*80/1 = 160 mg
Liquid: 200 mg ------- 5 ml 160 mg ------- Y ml Y = 160*5/200 = 4 ml/Day
154. Crcl calculation? and which stage renal failure

155. Standard liter of TPN is 500 ml Dextrose 45%- and 500-ml amino acids 12%. Solution is
infusing at 160 mL/h with 230 mL of fat emulsion 20% infused separately daily.
A) Calculate total kilo calories per day
B) Calculate the total grams of protein daily
C) Calculate the % of calories obtained from the fat daily
ANSWER:
A) The total daily TPN solution = 160mL per hour x 24 hours = 3840 mL
Each 1 liter of TPN contains 500mL of 45% dextrose i.e. each liter contains (500mL x 45)/100
= 225 grams of dextrose
Each 1 liter of TPN contains 500mL of 12% amino acid solution i.e. each liter contains (500mL
x 12)/100 = 60 grams of protein
The above quantities are per 1 liter only but the patient needs 3840 mL = 3.84 Liters
Amount of carbohydrates in daily TPN volume = 225g/L x 3.84 L = 864 grams of
carbohydrates DAILY x 3.4 kcal per gram = 2937.6 kcal per day from carbohydrates.
Amount of protein in daily TPN volume = 60 grams/L x 3.84 L = 230.4 grams of protein
DAILY x 4 kcal per gram = 921.6 kcal from protein
Fats is given on the side as 230mL of 20% fat emulsion daily Quantity of fat in this dose = 230
x (20/200) = 46 grams of fats daily x 9 kcal per gram = 414 kcal from fat
Total kcal = 2937.6 (carbohydrates) + 921.6 (protein) + 414 (fat) = 4273.2 kcal per day
B) Grams of protein daily = 230.4 grams
C) % of kcal from fat = 414/4273.2 = 9.6%

156. Toxic dose of Acetaminophen? 4gm = 500mg x 8

157. High dose Amoxicillin for a child by mistakes what to do?


a) Watch 48hrs for diarrhoea
158. Case about Flu like symptoms and insomnia?? DRP??? Answered before
Due to withdrawal symptoms of venlafaxine due to sudden stop

159. How to ttt? Start at high dose" same previous dose" and then taper slowly or Use
Fluoxetine one dose

160. Which of the following shows withdrawal symptoms after 24 hrs & increase with 3-5 hrs?
a) Diazepam
b) Flurazepam
c) Lorazepam
d) Clonazepam
e) Chlordiazepoxide

161. Do interchangeable substitution and then document and fax to doctor after. After 2 months
feels better want to stop although still have one more refill " 30 days".
a) Discontinue only after doctor permission

162. What is important concern for interchangeability?


a) High therapeutic index
b) Low therapeutic index
c) Large AUC
d) Small AUC
e) Short t ½

163. Alchohol addiction … will suffer from? Seizure


CTC: Alcohol Withdrawal
Approximately two‐thirds of patients with mild to moderate withdrawal symptoms can be managed with
supportive measures and monitoring. Pharmacologic treatment is always required for moderate to severe
alcohol withdrawal and may be required for about one‐third of patients with mild to moderate symptoms.
Most cases respond well to benzodiazepines, as they reduce the hyperactivity of GABA receptors that
occurs during withdrawal. A benzodiazepine protocol, with dosing based on the patient’s CIWA‐Ar score, can
be used in the primary care setting to alleviate withdrawal symptoms. Patients should be counselled on the
risk of respiratory depression if alcohol relapse occurs during (or shortly after) benzodiazepine treatment due
to its synergistic effect. For dosing information. Larger doses of benzodiazepines and/or addition of a different
GABA‐inhibitory drug such as phenobarbital may be required for resistant alcohol withdrawal (<5% of cases)
and patients will require monitoring in a critical care area
164. Refer to whom? Social worker

165. Manager of a pharmacy putting schedule program, is concerned with all EXCEPT
a) Workers are treated equally
b) Schedule is template from month to month
c) Schedule can be predictable for future???
d) They feel committed

166. Which drug requires shaking


a) Pulmicort (Budesonide) turbohaler
b) Tobradex eye drops
c) Atrovent (ipratropium) priming only
ATROVENT HFA Inhalation Aerosol is a solution aerosol that does not require shaking. However, as with any
other metered dose inhaler, some coordination is required between actuating the canister and inhaling the
medication.

167. Drug only to return if not to be refrigerated? Clindamycin

168. Patient given CPZ chlorpromazine IM 50mg for intractable hiccups. What is the issue?
a) High dose
b) Wrong indication
c) Wrong dosage (should be IV not IM)
1st line CPZ: 25‐50 mg PO TID‐QID X 2‐3 days OR IV 25‐50mg over 30‐60 mins (IV is the route which has best
evidence). 2nd line: Haloperidol. 3rd line: Metoclopramide or Baclofen (intractable hiccups=>1month)
Persistent 2‐30 days/ intractable>30 days

169. A study compares amlodipine and verapamil " cohort study"


170. Case CHF takes B.B. after 2 weeks progress to edema and dysnea. what to add for his
medication? Furosemide

171. Patient got arrthymia and doctor Rx Moxifloxacin?


a) QTc prolongation due to moxifloxacin
CTC: QTc interval prolongation, torsades de pointes, palpitations, cardiac arrest, ventricular arrhythmia and
ventricular tachycardia; aortic aneurysm and dissection.

172. COPD case role of cortisone in hospital?


a) Improve quality of life
b) Shorten the stay in hospital
Oral corticosteroids 1. improve lung function and 2. shorten length of hospital stay in all patients and3.
reduce risk of early relapse or hospitalization for subsequent acute exacerbations.

173. A patient with COPD and CHF and HIV suffers from dyspnea and Doctor diagnosis
Exacerbation, what we should give?
a) Levofloxacin
b) Ciprofloxacin
Beta‐lactam/beta‐lactamase inhibitor (e.g., amoxicillin/clavulanate), Second generation cephalosporin or
fluoroquinolone

174. Cancer patient … hypercalcemia? Pamidronate Also, Zolidronic acid ‐ clodronate


Cancer patient DVT prophylaxis? Enoxaparine
Malignancy and Pregnancy LMWT

175. Pt. Taking Warfarin, needs a pain-killer, what to take?


a) ASA
b) Ibuprofen
c) Acetaminophen
d) COX 2 naproxen
176. Sulfa allergy pt. what to take??
a) Furosemide
b) Celecoxib
c) Sulfasalazine
The 3 are CI in sulfa allergy

177. Pt. With TIA taking ASA few days ago he fell down, what is the best for him?
a) Increase dose of ASA
b) Change to Dipyridamole
c) Stop ASA and take Clopidogrel

178. Insulin how to avoid dispensing error? Clear unit syringe


 Use needles with only units on it (avoid ml+U Needles)
 Store only U100 Insulin and U100 syringues or pens in patients care units
 Let central pharmacy prep all IV Bolus insulin
 Develop hospital wide standards strengths for unsulin infusion to be used in all patient care units
 double check dose before administration
 avoid correction/sliding scale insulin to treat hyperglycaemia

179. Donepzil 10mg, what is wrong? High starting dose


The initial daily dose (5 mg) of donepezil is usually taken at night, but can be taken in the morning if sleep
disturbances occur.
CTC: Adults: The recommended initial dose of DONEPEZIL is 5 mg taken once daily. Therapy with the 5‐mg
dose should be maintained for 4‐6 weeks before considering a dose increase, in order to avoid or decrease the
incidence of the most common adverse reactions to the drug and to allow plasma levels to reach steady state.
Based on clinical judgement, the 10‐mg daily dose may be considered following 4‐6 weeks of treatment at 5
mg/day. The maximum recommended dose is 10 mg taken once daily.

180. Phenytoin causes all EXCEPT:


a) Gingival hyperplasia
b) Osteomalase
c) Pulmonary fibrosis
d) Hirsutism (hypertrichosis)
e) Increase liver enzymes

181. Kidney failure pt with COPD needs ttt for Influenza? Oseltamavir

182. New device that talks, promote for whom?? Insulin pt. " who progress eye problem"
183. Pregnant, what not to stop? Sertraline
CTC: If pharmacologic therapy is indicated, citalopram, escitalopram and sertraline are first‐line options

184. Who to refer to doctor? NA


Patient worsen depression symptoms last two weeks after starting
antidepressant last month

185. Breastfeeding mother recurrence candida what to give?


a) Fluconazole oral
b) Clotrimazole vaginal tablet
Recurrence for pregnant/breastfeeding: (>4 episodes/year)
Vaginal azoles 500 mg x 10‐14 days then once monthly for 6 months
Remember topical here is better than oral (opposite to bacterial)

186. Five days of period with Alesse Rx what to say?


a) Take a backup for 7 days
To promote adherence, should be day 1 period to start

187. Role of pharmacy assistant and reg. technician what each one can and can't do?

188. Iron deficiency anaemia what to monitor? Serum ferritin

189. Drug cause nausea for pt. takes hydromorphone?


a) Dutasteride
b) Meloxicam

190. A 66 -year-old patient with atrial


fibrillation and stroke, calculate
CHADS2 score:
a) 1
b) 2
c) 3
d) 4.

What to give: Warfarin

191. Fire in the pharmacy what is the policy? Go to assembly point


192. Rx Rosuvastatin for patient taking warfarin? Long term decrease in warfarin dose
Lexi: HMG‐CoA Reductase Inhibitors (Statins) may enhance the anticoagulant effect of Vitamin K Antagonists.
Fluvastatin*; Lovastatin*; Pitavastatin; Pravastatin*; Red Yeast Rice; Rosuvastatin*; Simvastatin* Exceptions
AtorvaSTATin

193. Warfarin monitor? Platelet, kidney " no INR in choices " NA

194. For primary prevention of stroke which one is more important?


a) Weight loss
b) Blood pressure
c) LDL

195. Acetaminophene toxicity what to do? N. acetyl cysteine

196. C. difficle prevention method in hospital? Visitors wash their hands

197. Hyperkalemia? I.V. glucose + insulin


If K <6.5 (asymptomatic): 1st line IV Bolus insulin + IV Glucose. 2nd line Nebulized Salbutamol
If >6.5 (symptomatic: N/V, Muscles weakness, EKG abnormalities) → Calcium gluconate IV + (ASYMPTOMATIC)

198. Female is going to use Accutane, what shouldn't you advise her?
a) To plan birth while using drug.
b) Not to get pregnant after stopping within one year “one month”
c) Can take it with other acne treatment “not with Vit A and tetracyclines”
d) No breast feeding during the treatment
Contraindicated in the following conditions:
 Pregnancy. Breastfeeding women,
 Hepatic and renal insufficiency,
 Hypervitaminosis A,
 Patients with excessively elevated blood lipid values,
 Patients taking tetracyclines

199. Fosfomycin go shortage?


a) Search alternative with doctor
b) Assess and stock
c) Keep for ICU
200. All are found on drug label EXCEPT: name of the pharmacist

201. Woman came with a prescription for Rizatriptan Wafer. Which of the following is a true
statement about Rizatriptan Wafer?
a) It is absorbed from the buccal cavity
b) Co-administration with alcohol is contraindicated
c) It is used for migraine with nausea
d) It is absorbed faster than Rizatriptan tablets
e) It is contraindicated with people who have difficulty swallowing
Dissolve orally in saliva and then swallow and absorb from stomach
CTC: The recommended single adult dose is 5 mg. The maximum recommended single dose is 10 mg. There is
evidence that the 10 mg dose may provide a greater effect than the 5 mg dose. The choice of dose should
therefore be made on an individual basis, weighing the possible benefit of the 10 mg dose with the potential
risk for increased adverse events.
For MAXALT RPD Wafers, administration with liquid is not necessary. The wafer is packaged in a blister within
an outer aluminum pouch. Patients should be instructed not to remove the blister from the outer pouch until
just prior to dosing. The blister pack should then be peeled open with dry hands and the wafer placed on the
tongue, where it will dissolve and be swallowed with the saliva. It is indicated for: acute treatment of migraine
attacks with or without aura in adults.

202. Pregnant woman needs help? Midwife

203. Pt. Taking Atorvastatin, Losartan, also eats a lot of fruit and steamed vegetables. Now she
has muscle pain which may be due to
a) Dose of Atorvastatin
b) Drinking Grapefruit  as she is on diet, and drinking fresh fruit juices.
“also, Losartan causes muscle pain 1%”
Patient- and Statin-Related Factors That May Potentiate Myotoxic Effects
Patient Factor Concomitant Medications Statin Properties
 Advanced age (>80 y)  Amiodarone  High dose statin
 Alcohol abuse  Azole antifungals  High bioavailability
 Chronic renal insufficiency (itraconazole, ketoconazole)  Limited protein
 Excessive grapefruit juice intake  Calcium channel blockers binding
 Frailty, small body frame (nondihydropyridines)  High lipophilicity
 Gender (women > men)  Cyclosporine  Potential for drug-
 Hepatic dysfunction  Fibrates (particularly drug interactions
 Hypothyroidism (untreated) gemfibrozil) metabolized by
 Inherited myopathies  HIV protease inhibitors cytochrome P450
 Intercurrent infections  Macrolides (clarithromycin, pathways, especially
 Perioperative periods erythromycin) 3A4 and 2C9
 Vigorous exercise  Nicotinic acid (rarely)
 Vitamin D deficiency

204. You advise the above patient all EXCEPT:


a) Eat banana which is rich in Potassium
b) Continue on her diet

204. A teenager taking OC, her parents came to the pharmacy to ask if their daughter is taking
OC, the pharmacist feels 2 conflicting principals
a) Veracity and Beneficence
b) Veracity and Autonomy
c) Confidentiality and Autonomy

206. pt don't speak English come with interpreter what is the most important while counselling?
a) Talk to the interpreter
b) Show gestures and hand movements
c) Talk to interpreter, tell him to talk to the patient so both will understand
d) Look at the patient & talk to the interpreter
There is an answer where the 3 of them exchange information, this is the one
207. which one Can be taken in oesophagus tube or J-tube?
a) Clindamycin tablets
b) Something which was SR
c) Dabigatran
d) Omeprazole

208. Cyclosporine not to be used with? Phenytoin


Lexi: Phenytoin may increase the metabolism of CycloSPORINE (Systemic).

209. Metronidazole interacts with? Gliclazide x NA


Metronidazole Interactions Antabuse (disulfiram) Anticoagulants (blood thinners) such as Coumadin (warfarin)
Dilantin (phenytoin) Hismanal (astemizole) Lithobid (lithium) Phenobarbital (Luminal and Solfoton) Tagamet
(cimetidine) Vitamins.

210. A prescription calls for 220 mg anhydrous zinc sulfate. However, your pharmacy has zinc
sulfate heptahydrate. To receive the equivalent amount of elemental zinc, how many milligrams
of zinc sulfate heptahydrate (7.H2O) would be patient need to take?
(MW: zinc 65, ZnSO4 161, H2O 18).
a) 123 mg
b) 220 mg
c) 300 mg
d) 392 mg
ZnSo4 = 161 ZnSo4 7H2O = 287
161 contains 220mg of Zn So, 287 will contain = 287 x 220 / 161 = 392.13

211. Which Rx written perfectly?

212. A lady come to buy a cough suppressant syrup contain codeine, you saw her
yesterday buying the same drug from other pharmacy you are working in what to do?
a) Ask her about her cough and what happen for the yesterday bottle

213. Altruism Question

214. Jaundice what to monitor? Bilirubin

215. History of cancer ttt for O.P.? Raloxifene

216. Admission to hospital what to write to do good reconciliation?


Note that he takes pramipexole for restless leg (Drug+indication)
217. Patient with osteoporosis who takes alendronate every Wednesday every morning, she
Called doctor she forgot to take this morning what she should do?
a) Take it tonight
b) Take it tomorrow morning and continue as schedule
CTC: Patients should be instructed that if they miss a dose of ALENDRONATE 70 mg once weekly, they
should take one dose on the morning after they remember. They should not take two doses on the same
day but should return to taking one dose once a week, as originally scheduled on their chosen day.
ALENDRONATE should only be taken upon arising for the day.

218. Finasteride 5mg written for prostate but actually used for alopecia because insurance
don't pay for this indication what to do? Don't dispense

219. Pt lost her medication and she forgot where did she put it and came to the pharmacy to
ask about new spare refill what to till her as the insurance will not pay for lost drug? You
should pay for the medication

220. Not given intrathecal? Vincristine→ KILL instantly


ERROR BEST PRACTICE
VinCRIStine and Should never be administered intrathecally because they can be neurologically fatal.
other vinca alkaloids Dispense vinCRIStine and other vinca alkaloids in a minibag of a compatible solution
inadvertently and not in a syringe. An effective prevention strategy that reduces the risk of
administered by the inadvertently administering vinca alkaloids via the intrathecal route is to dilute the drug
intrathecal route in a minibag that contains a volume that is too large for intrathecal administration (e.g.,
25 mL for pediatric patients and 50 mL for adult patients).

221. Kidney pt. on dialysis? Refer to assess his need of water-soluble vitamins

222. Cipro interact with phenytoin cause? Seizure


Lexi: Ciprofloxacin (Systemic) may diminish the therapeutic effect of Phenytoin. Ciprofloxacin (Systemic)
may decrease the serum concentration of Phenytoin

223. Pt with one kidney need drug for hypertension? CCB (Not ACE: Ramipril)

224. Smoker taking CBZ now he admits to hospital so stopped smoking as smoking in
forbidden what to do?
a) Decrease the dose of CBZ
If clozapine: It has been estimated that three‐quarters of patients who take clozapine are cigarette smokers.
If a patient's smoking status changes, one should consider checking a clozapine blood level. If a patient
suddenly stops smoking, their clozapine serum could elevate to a toxic level, leading to serious side effects.
It is well documented that cigarette smoke can induce cytochrome P450 (CYP) isoenzymes, specifically
CYP1A1, CYP1A2, and CYP2E1. Because clozapine is primarily metabolized by CYP1A2 (approximately
70%), smoking can induce clozapine metabolism and abruptly stopping smoking can increase clozapine levels.

225. Cephalexin interaction?


a) Azithromycin
b) Fibro
Cephalexin Interactions Cephalexin may increase blood levels of the type 2 diabetes drug metformin. This
could increase your risk for side effects. Cephalexin may build up in the blood when combined with the
medication probenecid (Benemid, Probalan), which is used to treat gout
Cephalosporins may enhance the anticoagulant effect of warfarin

226. a diabetic patient type 2 who takes metformin TID and the doctor decided to start insulin
for him which regime is correct for initiating?
a) Glargine 10 units before bedtime
b) NPH 30 units before breakfast and bedtime
c) 5 units rapid before each meal.
Generally, insulin is added to existing antihyperglycemic therapy when targets are not being met.
Insulin should be initiated immediately in individuals with metabolic decompensation and/or symptomatic
hyperglycemia.
Daily bedtime of basal insulin is prescribed at a dose of 0.1–0.2 units/kg of NPH insulin or long‐acting
analogue.
Alternatively, an empiric dose of 5–10 units of intermediate (NPH) or long‐acting insulin (glargine or
detemir) may be initially chosen depending on whether the patient is lean or obese. Subsequently, the dose
is adjusted to achieve a fasting glucose <7 mmol/L.

227. Latanoprost counseling? Eye lashes elongation or coloring pigmentation.


Latanoprost keep it at refrigerator till first use the it can be kept at room temp. for one month.
Chloramphenicol should be kept at refrigerator because it doesn't contain preservative.

228. Which one can be crushed (not chewed)?


a) Dabigatran
b) Rivaroxaban
c) Omeprazole
d) Finasteride

229. Diabetic and obese patient takes:


a) Metformin
b) Glyburide
c) Acarbose
d) Sitagliptin
230. HBA1c high before and after food blood glucose level high 10, 12 option type of insulin
with times NA

231. U/A what to give? ACE or BB

232. Which of RA treatment takes longest duration?


a) Methotrexate  6-8 weeks
b) Gold  3-6 months
c) Azathioprine  2-3 months
d) NSAIDs
e) Sulfasalazine  2-3 months

233. 91 days OCP?


a) To reduce the pain
b) To reduce the cycle
CTC: Use of extended cycles (with planned hormone‐free intervals) or continuous use of CHCs, such as pills,
rings or patches, without a hormone‐free interval may provide patients with relief from severe dysmenorrhea,
heavy flow or socially undesirable flow.[24] This can be achieved with products marketed for extended use (84
active tablets and 7 placebo or ultra‐low estrogen tablets) or by the use of any pill, patch or ring in a
continuous regimen (users take no break in between packages). Hormone‐free intervals between cycles of any
length should not exceed 7 days.
Compared with a 28‐day cycle, extended cycles or continuous use of CHCs results in fewer bleeding days,
decreased likelihood of side effects (e.g., pelvic pain, headache, bloating, swelling, tenderness) and improved
symptoms of endometriosis and polycystic ovary syndrome. Extended cycles may also be associated with
lower pregnancy rates.[25] One of the side effects or disadvantages of extended/continuous use is irregular,
unscheduled bleeding. If an inadvertent pregnancy occurs (due to several missed pills or forgotten change of
patch or ring), a woman may not realize she is pregnant without the amenorrhea that might otherwise alert a
cyclic contraception user that she could be pregnant.
A 2018 large cohort study showed that low‐dose continuous or extended cycle use of CHCs is associated with a
small increased risk of venous thromboembolism (VTE) when compared with cyclic COC use, which is unlikely
to be clinically significant

234. Aripiprazole? Akathisia- Tremors-Sedation

235. Floconazole and clopidogrel interaction


Lexi: CYP2C19 Inhibitors (Fluconazole) may decrease serum concentrations of the active metabolite(s) of
Clopidogrel.
236. Pregnant, Morning sickness using Diclectin, Dr ask if he could add Dimenhydrinate, your
response is
a) NO
b) Yes, Safely
c) Yes, but only intermittently
d) Yes, although not enough trials
CTC: Since dimenhydrinate is recommended for augmenting pyridoxine treatment in pregnancy, it can be
considered for intermittent therapy of motion sickness in pregnant patients. Promethazine may also be used.

237. Pt. With osteoarithritis taking Acetaminophen, came to your pharmacy saying that lately
he feels pain, swelling in other areas and other symptoms (Rheumatoid arthritis symptoms)
what would you advise him?
a) To see his doctor for disease assessment
b) To see his doctor to increase Acetaminophen
c) To add Ibuprofen
d) To increase Acetaminophen dose
“Tylenol Arthritis Pain Extended Relief Caplets: Adults and children over 12 years of age: 2 sustained release
caplets (1300 mg) every 8 hours, not to exceed 6 caplets (4000 mg) in 24 hours. Swallow each caplet whole
with water on an empty stomach. Do not crush, chew or dissolve the caplet.”

238. 298. Pt. Taking Warfarin, has black stools, you should ask him all EXCEPT:
a) About his diet
b) About other drugs he is taking
c) Protamine diet
d) Iron
e) Bruising regimen

239. Low back pain, what you recommended? Keep active – hot pads

240. Ismp questions

241. Public health agency? Prevent chronic disease

242. Pregnant patient should not take which medication? Candesartan

243. A case with post MI: Aspirin + ticar


244. Letrozole side effect?
Aromatase Endometriosis patients tend to have high levels of Aromatase P450 expressed at their
P450 endometriums. Aromatase P450 inhibitors reduce levels of estrogen production from sources other
inhibitors than the ovary, such as the adrenal gland.
Anastrazole Can be used with OCs, GnRH analogues, or Progestins to reduce risk of ovarian cyst development
Letrozole S.E: hot flushes and sweating, osteoporosis, insomnia, body aches, loss of appetite.

245. A patient who has AF is on warfarin which one is not true?


a) Having bracelet
b) The INR between 2-3
c) Can't remember the rest

246. For the safety of insulin in hospital which one is better?


a) Color coding
b) Triple check
c) Separate from shelves

247. In regard of safety of oral solution which one is correct?


a) Move all of oral solution to pharmacy form stock
b) Prepare all of oral solution in oral syringe and label “ORAL USE ONLY”
c) Put both pounds and kg on patient chart
d) Use a syringe with both degree ml and
Unintended Do not stock bulk oral solutions of medications on patient care units.
intravenous Use only oral syringes that are distinctly marked “Oral Use Only.”
administration of When ISO 80369 compliant syringes (e.g., ENFit) are used for administration of oral
oral medications liquid medications, always highlight on the pharmacy label, or affix an auxiliary label,
“For Oral Use Only” on the syringe. Ensure that the oral/enteral syringes used do not
connect to any type of parenteral tubing used within the organization.

248. A patient with frequent attack of Raynaud’s want to get medication what we can give
a) Felodipine XL
b) Propranolol

249. A guy with CHF who has COPD as well and was hospitalized he feel pain in one leg what
is the problem (not sure it was pain or edema)
a) DVT
b) CHF getting worse
c) Exacerbation of COPD
250. Rivaroxaban counselling?
a) Cause constipation
b) Take with or without food
c) Don’t crush or chew
d) Avoid antacids
CTC: Swallow the tablet preferably with water. Try to take the tablet at the same time every day to help you to
remember it.
If you have trouble swallowing the tablet whole, talk to your doctor about other ways to take it.
The tablets may be crushed and mixed with applesauce. Take it right away after you have mixed it. A crushed
2.5 mg or 10 mg tablet can be taken with or without food. Eat food right after taking a crushed 15 mg or 20 mg
tablet. Your doctor may give you the crushed XARELTO tablet also via a tube.

251. patient with Allergic rhinitis who does not get better with nasal corticosteroid what we
should do next?
a) Nasal decongestant
b) Ipratropium

252. Which one needed to be shaken before use?


a) Budesonide turbuhaler
b) Fluticasone evohaler
c) Salbutamol MDI
d) Spiriva Respimat
Other options needed to be primed only

253. A patient who was on statin I think and the doctor prescribed a H. pylori regimen with
clarythromycin, you that ther is an interaction what you should do?
a) Call doctor and suggest a regimen without clarithromycin
b) Change the statin

254. A cancer patient who was on opioid & he started doxazosin, what we should say?
a) Doxazosin can exacerbate cns suppression of opiod (hypotensive effect not CNS)
b) Doxazosin can cause dizziness
Lexi: Blood Pressure Lowering Agents may enhance the hypotensive effect of Hypotension‐Associated Agents.
Although the concomitant use of two or more drugs that may lower blood pressure (either as a therapeutic
intention or as an adverse effect) is often clinically appropriate, use of such combinations often substantially
increases the risk for hypotension. Monitor patients closely for additive hypotensive effects if two or more of
these agents are combined.

255. A smoker patient 15 years old needs plan B


a) Dispense
256. After a while She comes a doctor prescribed alesse and she wants to start on 5th day of her
Cycle what you should say? Use another method for first 7 days

257. After one month she came back and she said she forget to take her pill last night (it was
day 5) what will you say?
a) Discard this Batch and start new one
b) Take one pill as soon as possible and continue the rest
Don’t forget: If this case had sex accident: recommend EC

258. question about rabeprazole (totally forgot)

259. Which one is true about dermatitis?


a) Bath oils before bath can contain water barrier
b) Collodiol oatmeal is not as effective as bath oils in containing water
c) Tacrolimus is more effective than corticosteroid

260. A patient with pneumonia, dr wants to switch from IV to oral which one is the most
important barrier?
a) Culture positive for pseudomonas
b) Diarrheas
c) Persistent cough

261. Which one is a sign of withdrawal syndrome of opioids


a) Diarrhea
b) Meiosis
c) Seizure

262. A patient who quit smoking and drink coffee excessively started carbamazepine and now
has insomnia tremor and anxiety why?
a) The dose of gums is now enough
b) It's because of coffee
c) Adverse effect of carbamazepine

263. There was a cancer patient who was on morphine and Doctor has increased the dose of
Morphine and she suffer from myoclonus her son came to pharmacy and need early refill you
look at his file and see some other early refills in the past NA
a) Call the doctor and increase the dose
b) Call the doctor and change to fentanyl
c) Because patient has pain give the meds
The right answer which is not provided here is dispense and ask patient to go to see Dr. for reassessment.
264. a question about orthostatic hypotension (not sure) and I put levodopa

265. Which one is the fastest medication for migraine?


a) SC sumatriptan
b) Oral zolmitriptan
c) Sumatriptan nasal

266. If a patient takes zolmitriptan or rizatriptan can't remember and has improved, when he can
take the second dose?
a) 2 hours
b) Half an hour
c) One hour

267. When you are counselling the patient about the drug use, you found that this drug is used
also for treatment of other conditions. What will you do?
a) Educate patient about the approved indication of the drug.
b) Put a note on the patient file that it is used for RLS.
c) Let the patient know about the other indications of the drug.

268. A patient on carbamazepine and there were lots of titrating up and he suffers from chronic
Kidney disease what is wrong? NA
a) Carbamazepine needs a loading dose
b) It should be titrated every week to reach the therapeutic dose
I really can't remember i just know that in previous papers they missed chronic kidney disease

269. There were a study that the result was the people who are controlled does not need Self-
monitoring of blood glucose (SMBG), so in which situation SMBG is good in this Population?
a) Patient as their personal interest
b) When they don't have adherence to meds
Copy from past exam

270. Diabetic patient, his serum creatinine has increased what was the reason?
a) Metformin
b) Canagliflozin

271. A diabetic patient, 55-year-old who his blood pressure was .... he takes ......
and you reviewing his profile, what is your concern?
a) A lack of drug for his cardiovascular protection (statins weren't in his profile meds)
There were some options that they could be correct but I can't remember
272. He suffers from erectile dysfunction and he asked about sildenafil
a) It is less effective in diabetic patient
b) It has at least 4 hours duration
c) It can cause neuropathy

273. Which on is the cause of neural damage?


a) Diabetic patient who are insulin dependent
Long‐term diabetes mellitus may lead to complications that involve the small blood vessels
(microangiopathy), large blood vessels (macroangiopathy) and nerves (neuropathy) of multiple organs and
systems. Intensive treatment regimens control blood glucose more effectively than conventional regimens
and reduce the risk of long‐term diabetic microvascular complications (retinopathy, nephropathy,
neuropathy).

274. KW is a diabetes patient with CrCl 28 ml/min comes to the pharmacy with symptoms of
shallow breathing, discomfort and decreased appetite. Upon investigation you knew that he
takes NPH 25 Units BID and metformin for diabetes and citalopram for depression, he also uses
garlic to reduce his cholesterol. What possibly caused these symptoms?
a) High dose NPH
b) Citalopram
c) Metformin
d) Garlic
Lactic acidosis may rarely occur with metformin accumulation. It is a medical emergency and requires
prompt discontinuation of metformin and treatment in a hospital setting. Metformin should not be used in
patients with a history of lactic acidosis.
Lactic acidosis is characterized by elevated blood lactate levels (>5 mmol/L), decreased blood pH, increased
anion gap and increased lactate/pyruvate ratio. It presents initially with nonspecific symptoms including
malaise, somnolence, and abdominal and respiratory distress. With greater severity, it may be associated
with hypothermia, hypotension and resistant bradyarrhythmia. The frequency of metformin‐associated
lactic acidosis is not known, but in data taken from trials, it is estimated to be <4 cases per 100 000 patient‐
years.
Lactic acidosis occurs primarily in patients with renal insufficiency, hepatic dysfunction, or other conditions
involving hypoxemia, dehydration or sepsis. Lactic acidosis may occur in patients who undergo radiologic
studies with radiocontrast dye.
Metformin use in patients with diabetes and heart failure was previously thought to increase the risk of
lactic acidosis. However, newer data suggests metformin does not increase the risk in this population, and is
associated with better cardiovascular outcomes than other antihyperglycemic therapies

275. Who is responsible for academic detailing?


a) Funding is created by province
b) Funding is created by pharmaceutical companies

276. Pt. Taking terbutaline inh. And using it a lot, if pharm. Fail to advise him to see his doctor
to add corticosteroid inh. To his medication, then he mostly violated?
a) Beneficence
b) Non-malfecience
c) Veracity
d) Autonomy
e) Paternalism

277. Pt. Using turbohaler, you tell him all EXCEPT:


a) Not to exceed dose
b) It should be activated just prior to breathing in
c) It leaves an after taste
d) You can use a spacer

278. Theophylline dose should be decreased in all EXCEPT:


a) Elderly
b) Neonates
c) Pregnancy
d) Smokers
e) HME inducers
Heavy smokers metabolizing Theophylline twice as fast as non‐smokers

279. An old man staying with his daughter and grandchildren and need to be vaccinated, but he
is afraid to, his daughter is worried about her kids, what is the best to be done?
a) Put him in a nursing home
b) Vaccine all family members
c) Give him Zanimavir
280. To mix drugs, you can find it in
a) USPDI
b) CPS
c) Remington

281. Patient taking Phenazopyridine, what auxiliary label should be added?


a) It discolour urine red
The patient should be advised to take phenazopyridine with or following food or after eating a snack to
reduce stomach upset.
The patients should be aware that phenazopyridine causes a reddish orange discoloration of the urine and
feces, and may stain clothing. Phenazopyridine may cause discoloration of body fluids and staining of
contact lenses has been reported. There have been reports of teeth discoloration when the product has been
broken or held in the mouth prior to swallowing Patients should be instructed to take phenazopyridine for
only 2 days

282. Which of the following has highest side effects of these corticosteroids?
a) Betamethasone  (most potent and has the highest SE)
b) Triamcinolone (medium)
c) Hydrocortisone (low)
d) Prednisolone (oral)
e) Dexamethasone (oral)

283. All are true about Amiodarone EXCEPT:


a) May cause increase in the thyroid hormone
b) Photosensitive
c) Its effect takes 3-7 days “response generally requires at least 1 week, usually 2 or more.”
d) It regulates the sinus rhythm

284. Female patient taking OC, heard that if she missed taking it daily it increases risk of
contraception failure with which of the following agents this most likely to happen?
a) Progestin only
b) Estrogen only
c) High estrogen and low progestin
d) Triphasic
e) Monophasic
285. Female with a fungal inf. wants to switch to easier dosing drug, which is best for her?
a) Ketoconazole (Nizoral) used for seborrhea
b) Nystatin (used for oral thrush)
c) Fluconazole (Diflucan) once dosing only
d) Miconazole (used for candidias but require 4 weeks BID)
e) Itraconazole (used for onychomycosis)

286. Pt. taking digoxin, furosemide and has Rx for enalapril, what is your recommendation?
a) Dispense it as is
b) Change Enalapril
c) Decrease dose of Enalapril
d) Stop Furosemide
e) Change Furosemide
Now your concern in digoxin therapy to avoid hypokalemia which leads to digitalis toxicity
Enalapril will cause hyperkalemia. Furosemide will cause hypokalemia
So net result moderate potassium level. No problem to be coadminstered with digoxin

287. All the following do not cause cerebral haemorrhage EXCEPT:


a) ASA
b) Clopidogrel
c) Heparin
d) TPA (Tiaprofenic Acid, Surgam)

288. Which of the following shows false ovulation timing?


a) Dannazol (Suppress FSH and LH, both of them)
b) Progestin (because it will suppress LH which is the ovulation indicator)
c) Estrogens (suppress the FSH which means NO OVULATION)

289. Pt. With high Triglycerides, which of the following is best for him?
a) Lovastatin
b) Gemfibrozil [LOPID]
c) Niacin
d) Colestipol

290. Iso sorbide dinitrate cause all EXCEPT:


a) Syncope
b) Hypotension
c) Palpitation
d) Bradycardia
e) Headache
“Headache, reflex tachycardia, paradoxical increase of anginal pain, palpitations and Syncope due to nitrate
vasodilatation, although rare, has been reported.

291. Raloxifen “Evista” causes all EXCEPT:


a) Decrease lipid
b) Increase bone resorption” reduce bone resorption”
c) No breast cancer
d) No endometrial cancer
e) Alleviate Vasomotor symptoms

292. Raloxifen is different from oestrogen in all EXCEPT:


a) Used only for menopause
b) No breast cancer
c) No endometrial cancer
d) Alleviate hot flushes “raloxifen aggravates flushes”

293. Diabetic patients using strips, what is true about it?


a) Can be used after expiry date
b) Can be read after 3 days
c) Should be checked visually before use
d) Should be used regularly

294. Pt. taking Gentamicin, by measuring his blood level it was low half hour before taking
dose, and low ½ hour after taking dose (4 mmol/L) the effective level is 8 mmol/L, what to do?
a) Increase dose and keep interval
b) Decrease dose and keep interval
c) Keep dose and increase interval
d) Increase dose and increase interval
Following i.v. or i.m. administration, 2 or 3 times daily, the peak concentration, measured 30 minutes to
1 hour after administration, is expected to be in the range of 4 to 6 µg/mL. With once daily administration,
transient, high peak concentrations can be anticipated.
With all regimens, the dosages should be adjusted to avoid prolonged concentrations above 10 to 12 µg/mL.
Through levels above 2 µg/mL, measured just before the next dose, should also be avoided “

295. All the following are true about Repaglinide:


a) It is short acting
b) Taken just prior to meals
c) If you miss a meal do not take it
d) It increases insulin secretion
296. Pregnant Wmn, fifth month, she has heartburn after eating, she also drinks alcohol and
smokes. She is 35years old, you suspect her to have GERD for all EXCEPT
a) Pregnant in 5th month
b) Heartburn after eating
c) Her lifestyle
d) She feels it after laying down
e) Her age

297. Which of the following is most effective in relieving symptoms of GERD and healing
ulcers?
a) H2-Blockers
b) PPI
c) Pro-kinetic
d) Antacids
e) Sucralfate
MAY 2016

1. LVEF is <35% and patient taking Ramipril


and Propranolol, what drug is suitable to add?
a) Spironolactone
b) Digoxin
Beneficial in Patients with severe HF (NYHA III‐IV and
LVEF < 35%) & In post MI patients with an LVEF ≤ 40%
and either HF symptoms or DM.

2. Pt. Taking Alendronate, you advise him all


EXCEPT
a) Take it on empty stomach
b) Do not eat anything until 30 min
c) Take it with fruit juice
Serum calcium and symptoms of hypocalcemia should be monitored during therapy with ALENDRONATE.

3. He took it every Sunday, calls as she missed her dose at noon, what you should tell her:
a) Take it right now
b) Skip the dose and take the other dose as scheduled.
c) Take this dose tomorrow morning and the next dose on Sunday as per schedule.

4. A patient calls you, he had migraine, he has taken rizatriptan half an hour ago, it has no
effect, what do you suggest?
a) Use maximum 4 tabs in 24 hr
b) Take another one
c) Do not take another dose, it will be ineffective.
d) Take subcutaneous after 2 hours.
e) Take naratriptan after 2 hours.
CTC: Second dose not likely to be effective if first dose provided no relief. Do not use 2 different triptans in
the same 24‐hour period. Do not use a triptan and dihydroergotamine (DHE) in the same 24‐hour period

5. Pt on sumatriptan, came with a prescription. What drug has interaction with sumatriptan?
a) Citalopram (SSRI) Triptans: Caution with SSRIs or SNRIs (increased risk of serotonin syndrome).

6. Pregabalin prescription needs what?


a) Written and verbal refill
b) No verbal
7. Parkinson disease is in early stage and 48 years old; all of the following symptoms are
associated with Parkinson's disease? EXCEPT
a) Difficulty in speaking (Slurred speech)
b) Xerostomia
c) Urinary retention
d) Diarrhea
Q. Nonmotor features of PD:
 Dementia, psychosis
 autonomic dysfunction (excessive sweating &
salivation, constipation, bladder frequency
/urgency, orthostasis) become more disabling
features as disease progresses, difficulty in
speaking and swallowing.
Q. Motor features of PD:
 Akinesia: loss or impairment of the power of voluntary movement.
 Bradykinesia: slow movement.
 Dyskinesia: repetitive involuntary movement of tongue, limbs, hands, and trunk.
Remember: Prampixole SE: constipation

8. What do you recommend?


a) Selegiline
b) Pramipexole
c) Levodopoa
d) No nead of medication because of
adverse effect
CTC: Bromocriptine, pramipexole and ropinirole are effective as monotherapy in the early stages of the
disease, and as adjunctive therapy with levodopa for patients with more advanced motor complications

9. How we can increase safety use of insulin in hospital, or what is the best way to minimize
insulin related errors?
a) Double check prescription and preparation
b) Use colored coded Bins to every type of insulin.
c) Give only from pharmacy, not from ward stock (strongest in general)
d) Use different colored baskets
e) Tall man lettering
f) Using cartridge (insulin pen)
g) Using template (computerized insulin order template)
10. Advantage of pneumococcal vaccine in children is that it can be used for all except?
a) Meningitis
b) Otitis media
c) Pneumonia
d) Peritonitis
e) Bronchitis
f) Tonsillitis
g) Septicaemia
Monograph: 4.1 Therapeutic indications Active immunisation for the prevention of invasive disease,
pneumonia and acute otitis media caused by Streptococcus pneumoniae in infants, children and adolescents
from 6 weeks to 17 years of age. Active immunisation for the prevention of invasive disease and pneumonia
caused by Streptococcus pneumoniae in adults ≥18 years of age and the elderly. See sections 4.4 and 5.1 for
information on protection against specific pneumococcal serotypes. The use of Prevenar 13 should be
determined on the basis of official recommendations taking into consideration the risk of invasive disease and
pneumonia in different age groups, underlying comorbidities as well as the variability of serotype
epidemiology in different geographical areas.

11. Which drug has myotoxic effect: Gemfibrozil


Drug‐induced myopathies: HMG‐CoA reductase inhibitors (statins), fibrates, alcohol, D‐penicillamine,
interferon‐α, procainamide, Zidovudine, germanium, Colchicine, vincristine, Chloroquine, hydroxychloroquine,
quinacrine, perhexiline, amiodarone, Emetine, ipecac, Corticosteroids.

12. Myopathy monitoring in physical examination:


a) Weight
b) Length
Muscle weakness in thighs, shoulders, arms and legs
Featured snippet from the web on physical examination, many myopathy patients, especially those with
acquired myopathies, demonstrate symmetrical muscle weakness in a proximal to distal gradient. Sensation
is intact, and deep tendon reflexes are preserved unless there is severe weakness. Creatine is the test

13. After 24 hr of acetaminophen toxicity (Max dose 4g.), what is the marker of toxicity?
a) Increase in ALT and AST
b) Increase in WBC
c) Decrease creatinine clearance
AST/ALT should be monitored up to 48 hrs after an acute ingestion.
MAX hepatic necrosis appears 2‐5 days following the overdose
Signs: hypoglycemia, inc prothrombin time, inc bilirubin, ALT, AST
14. First choice of nonpharmacologic treatment for PTSD: CBT
CTC: Featured snippet from the web on
physical examination, many myopathy
patients, especially those with acquired
myopathies, demonstrate symmetrical
muscle weakness in a proximal to distal
gradient. Sensation is intact, and deep
tendon reflexes are preserved unless
there is severe weakness.

15. If nonpharmacologic actions


does not affect what we should do?
a) Paroxetine
b) Lithium
c) Venlafaxine
CTC: SSRIs and SNRIs have been shown
to reduce the severity of all the
symptom clusters of PTSDs.
Fluoxetine, paroxetine, sertraline and venlafaxine are first‐line options.

16. A patient with chronic alcohol dependence. What’s different between alcohol and opioid
withdrawal?
a) Alcohol withdrawal takes longer
b) Opioid withdrawal longer than alcohol
c) Alcohol withdrawal increases mortaility (HARDER)
Alcohol withdrawal can cause seizures, deficiency in vit B1, K, Mg, PO4, HTN, dehydration, infections, liver
disease, increase INR, insomnia & irritability up to 72 hrs.
TTT of severe; more than 50mg diazepam iv in the 1st hr, or more than 200mg in the 1st 3 hrs &/or
Phenobarbital in resistant withdrawal.
17. Which one is the most appropriate for
referring this patient?
a) Dietitian
b) Physiotherapist
c) Social worker

18. What kind of substance abuse does not


need special medication
a) Opioid
b) Alcohol
c) Cocaine

19. Obese patient stayed at hospital because of foot injury. He had MI 2 years ago and has
diabetes type 2, he is a heavy smoker. What’s the most appropriate action after leaving the
hospital?
a) Home nurse for foot dressing
b) Consult with dr for smoking cessation
c) Physiotherapist

20. 67-year-old lady use acetaminophen 650 mg TID asked topical ointment for her OA
a) Menthol
b) Diclofenac
c) Methylsalicylate
d) Capscasin
CTC: If NSAID therapy is desired, begin
with topical rather than oral agents in
persons ≥75 years of age. Monitor for
serious adverse effects that may occur in
the elderly, including GI upset. Counsel
patients on common side effects
including dry skin and local application
site reactions.
21. She had an injecting to her knee. What is the name of medication?
a) Hyaluronic acid  or CSs
b) Lidocaine
c) Adalimumab

22. A female patient, smoker for 25 years


with DVT and takes medications. What to
give prophylaxis for DVT before
hysterectomy?
a) Rivaroxaban
b) Dabigatran
c) Warfarin
d) Deltaparin

23. Dr started on warfarin. What to Counsel?


a) Use contraception while on warfarin
CTC: COUMADIN (warfarin sodium) is contraindicated in pregnancy because the drug passes through the
placental barrier and may cause fatal hemorrhage to the fetus in utero. Women of childbearing potential must
take precautions not to become pregnant while on COUMADIN therapy. Furthermore, there have been
reports of birth malformations in children born to mothers who have been treated with warfarin during
pregnancy. If the patient becomes pregnant while taking this drug, she should be apprised of the potential
risks to the fetus, and the possibility of termination of the pregnancy should be discussed in the light of those
risks.

24. Drug that increase INR with warfarin


a) Fluconazole all azoles
b) St. John wart
c) Rifampin
d) Alcohol

25. Upon discharge the first advice to the patient:


a) Compression stocking
b) Stop smoking
c) Go to physiotherapy for mobility

26. Patient is in early stage of dementia come to your pharmacy, best approach to him? NA
a) Speak loudly
b) Ask him to come with his family or his caregiver
c) Speak in which no decision making
27. Patient come to ER with non-responsive depression the diagnosis is withdrawal syndrome.
Which drug causes?
a) Bupropion
b) Venlafaxine
c) Fluoxetine
Fluoxetine & bupropion = less withdrawal symptoms
Paroxetine & venlafaxine have short t1/2 so they have rapid withdrawal symptoms

28. She didn’t refill her prescription because of economic issue, what should you do?
a) Ask manufacture for help program
b) Ask her doctor to subsidize her prescription by free sample
c) Help her to find society help
d) Ask dr to prescribe affordable drug
Another version, Sintinab expensive: call manufacture to see if have lower price for special pt.

29. Patient has depression. Ask a drug that does not cause weight gain
a) Bupropion
b) Paroxetine
c) Venlafaxine
d) Citalopram

30. A 18-year-old girl usually comes to your pharmacy by her family. Today she came alone
and she asked 2 bottles of ipeca syrup with a basket of junk food. what should you do?
a) Call her parents
b) Ask her why she needs ipeca syrup
c) Sell her what she needs without doing anything
d) Refuse to give her ipeca and said out of inventor

31. Counselling about ciprofloxacin, which one is not true?


a) Drink coffee with it
b) CYP1A2 Inhibitor
c) Take with a lot of water
Caffeine + Ciprofloxacin → ↓ caffeine elimina on
FYI: The combination of ergotamine and caffeine is used to prevent and treat migraine headaches.

32. Among the following, which one requires a prompt medical referral while taking CHC?
a) Severe Abdominal Pain
b) Diarrhea
c) Chloasma
d) Headache Relieved by Acetaminophen
e) Irregular Bleeding

Another version, a young girl has tanned in tanning salon, comes with spot on her neck and
upper back, she has no medical issue, only using contraception, what could be the cause?
a) Photosensitivity due to COC
Some people who take oral contraceptives may become more sensitive to sunlight than they are normally.
When you begin taking this medicine, avoid too much sun and do not use a sunlamp until you see how you
react to the sun, especially if you tend to burn easily. If you have a severe reaction, check with your doctor.

33. What causes neural tube defect?


a) Valsartan
b) Phenytoin
Another version, In which condition do you have to give folic acid to prevent
neural tube defects? Diabetes

34. Which condition makes a pregnant woman to take folic acid before
conception after delivery and postpartum? Being on epileptic drugs
Moderate risk women of neural tube defect NTD includes:
➢ Maternal or paternal personal or family history of other folate‐sensitive congenital
anomalies (limited to specific anomalies for cardiac, limb, cleft palate, urinary tract,
congenital hydrocephaly).
➢ Maternal or paternal family history of NTD in a first‐ or second‐degree relative.
➢ Maternal diabetes; maternal kidney dialysis.
➢ Maternal use of folate‐inhibiting drugs (carbamazepine, cholestyramine,
metformin, methotrexate, phenobarbital, phenytoin, primidone, sulfasalazine,
triamterene, trimethoprim, valproic acid).
➢ Maternal GI malabsorption conditions e.g., Crohn disease, active celiac disease, gastric bypass surgery.
High risk includes: Maternal or paternal personal NTD history or a previous NTD pregnancy
34. Patient taking morphine 15mg q4h. Wants long-acting. Equivalency dosing: Codeine
200mg; Morphine 30mg; Hydromorphone 6mg; Oxycodone 20mg.
a) Codeine 400mg q12h
b) Morphine 30mg q12h
c) Hydromorphone 6mg q12h
d) Oxycodone 60mg q12h
Answer: 90 mg morphine= 18 mg hydrocodone X 0.66= 12 mg (6mg q12h)

35. Patient with AF, congested heart and hypertension, what’s the DOC for decreasing rhythm?
a) Amiodarone
b) Propranolol
c) Digoxin
Another version: Amiodarone monitor? Vision (Amiodarone cause neurotoxicity in 2 months)

36. Which one must be shaken before use?


a) Fluticasone spray
b) Dexamethasone drop
c) Miacalcin (calcitonin-salmon)  prime
d) Salbutamol inhaler  shake, prime 1st time and if not used for 5 days
e) Imitrex (sumatriptan nasal spray)  no prime, no shake
f) NG spray  do not shake, need priming in 1st time or if not used for 14 days.
g) Stadol (Butorphanol Nasal Spray, 10 mg/mL)  prime, no shaking
h) Livostin spray (levocabastine Nasal Spray)  should be shaken before each
application. Before using the pump delivery system for the first time, the pump
reservoir should be filled up by priming until a fine spray is delivered.

37. Symbicort turbohaler counselling?


a) Shake before use
b) Inhale deeply and strongly
For Inhaler Techniques: https://www.lung.ca/lung‐health/get‐help

38. A patient with non-inflammatory acne and she is on benzoyl peroxide but still has acne, her
physician switched to topical erythro/benzyl peroxide BID, but didn’t feel any improvement.
The patient also has depression and taking fluoxetine, what possible cause of this ttt failure?
a) Too much of a right medication
b) Drug drug interaction
c) Treatment is not correct from the beginning
d) Drug disease interaction.
CTC: Infrequent: acne, alopecia, contact
dermatitis, eczema, maculopapular rash, skin
discoloration, skin ulcer, vesiculobullous
rash. Rare: furunculosis, herpes zoster,
hirsutism, petechial rash, psoriasis, purpuric
rash, pustular rash, seborrhea.

39. Patient needs to know what’s the particular herbal medication efficacy for some diseases.
what would you say?
a) Herbal remedies are not useful
b) Start to search to see if that herbal drug has any evidence in primary literature

40. Patient on rosuvastatin for some problem, doctor prescribed clarithromycin. What would
you suggest?
a) Stop rosuvastatin, keep his new medication
b) Try another antibiotic
Lexi: Clarithromycin may increase the serum concentration of Rosuvastatin.

41. Doctor wants to know 3A4 interaction with atorvastatin and grapefruit juice, what is the
best reference?
a) CPS  Also, Lexicomp
b) RxFiles
c) Martindale

42. All are correct about Pravastatin EXCEPT?


a) May be used during pregnancy (All statins are CI in pregnancy, only cholestyramine and
colestipol are used)
b) Can drink grapefruit juice (Prava, Fluva, Rasuvastatin are ok with grapefruit)
43. Patient using nasonex (mometasone) 3y/o & older, still getting rhinorrhea, what to give?
a) Ipratropium 0.03% (+ in Corticosteroid intolerant Allergic rhinitis)
b) Intranasal vasoconstrictor
c) Intranasal saline
d) Decongestant

44. An old lady lives alone, has recently hear loss and fills her ear are congested, no tinnitus,
and she does not use any medication. What to do?
a) Syringing with cold water
b) Empty the ears manually by a curette
c) Refer to doctor (hearing loss: red flag, hearing trouble: instill one to two drops of olive/ mineral oil)
Another version: A patient with feeling of fullness, hearing loss & external otitis what do you
recommend? (Acute otitis externa)
a) Acidifying agents (Acetic acid 2%)
b) Al acetate
c) Antibiotic polysporin
d) Need for RX eardrops
In hearing loss, we refer if it is unexplained. But if due to otitis externa or ear wax, we can treat it
Algorithm: Topical ABX OTC: Acidic solns &/or Otic drops Polysporin + Systemic analgesic
Topical ABX RX: Acidic solns &/or Otic drops: FQS monotherapy (Cipro, moxi and ofloxacin) or FQs‐CS
combination + systemic analgesic 7‐10 days (Preferred Option)

45. Before start using antibiotic ear drops, what your recommendation?
a) Wash your ear to remove wax x
Method of administration for otic use only. Instruct the patients to shake the bottle well before use. The
suspension should be warmed by holding the bottle in the hand for several minutes to avoid dizziness, which
may result from the instillation of a cold suspension. The patient should lie with the affected ear upward, and
then the drops should be instilled pulling several times on the aurical. For patients with acute otitis media with
tympanostomy tubes, the tragus should be pumped 5 times by pushing inward to facilitate penetration of the
drops into the middle ear. This position should be maintained for around 5 minutes to facilitate penetration of
the drops in the ear. Repeat, if necessary, for the opposite ear. To prevent contamination of the dropper tip in
order to limit bacterial risks, care should be taken not to touch the auricle or the external ear canal and
surrounding areas, or other surfaces with the dropper tip of the bottle. Keep the bottle tightly closed when
not in use. Keep the bottle until the completion of the treatment.

46. What causes danger if injected I.V?


a) Sterile water for inj.
b) Dextrose 10 %
c) NaCl 0.45 %
d) Saline 0.9 %
Intravenous administration of Sterile Water for Injection without a solute may result in hemolysis.
Another version: Which is more harmful if injected intravenous:
a) Dextrose 10%
b) Sodium chloride 0.3%
c) Sodium chloride 3%
d) Sodium chloride 0.9%
Hypotonic is more dangerous

47. A patient who is a teacher and mother of 3 children, has incontinence, she is stressed lately
& embarrassed about a situation, she says that she has no leakage of urine with cough, sneeze,
or exercise. What is the type of incontinence?
a) Stress.
b) Urge.
c) Mixed

48. What should be the treatment:


a) Oxybutynin
b) Duloxetine (for stress)
c) Imipramine (not used)
d) Desmopressin (children for
noct.)

49. Who would you refer this patient


for:
a) Urologist
b) Occupational therapy
c) Social worker

50. Tests need to be done:


a) Urinalysis
b) Electrolytes
CTC: Investigations:
 Cough test (to detect stress incontinence)
 Urinalysis (to rule out infection, glucosuria, hematuria)
 Renal function tests (urea, serum creatinine, glucose and calcium) if renal dysfunction suspected or
polyuria present
 Urodynamic testing may be used to measure detrusor compliance and contractility, postvoid residual
volume, and bladder capacity
51. A 4-year-old child with bedwetting for one or two nights, he has nothing else, what to tell
his mom?
a) Positive parenteral support crucial for successful treatment
b) Child apnea causes urinary incontinence
c) Without treatment, child becomes continent by 20% each year
d) 20% of nocturnal enuresis can be decrease by patient care
< 5 years do nothing reassure

52. Nonpharmacological option:


a) Fluid restriction (before
bedtime)
b) Enuresis alarm (3-4mo to
work)
c) Do nothing

53. Pharmacological treatment


should be:
a) Desmopressin >.5 yrs
b) Imipramine
c) Duloxetine
d) Refer
e) Oxybutynin

54. A patient has COPD, why doctor gives CS inhaler?


a) To improve quality of life
b) Improve FEV1 "lung function" → + reduce hospital stay
Combination ICS monotherapy NOT recommended in COPD.
Inhaled ICS + LABA combinations are more effective synergistically, and may slow progression in
Corticosteroids moderate/severe COPD.
and Long-Acting If the patient has severe COPD with repeated exacerbations, recommend ICS/LABA +
Bronchodilators Tiotropium – this improves bronchodilation, lung deflation, reduces frequency/severity,
improves health, and reduces hospitalization.
Use of corticosteroids within 30 days of the exacerbations of COPD as a 5‐day course, 30‐40mg/day

55. A COPD patient wants to travel to USA he has so many problems, but his COPD is well
controlled, wants to have a safe and good trip, comes with a prescription dr wrote antibiotic for
him, why?
a) For prophylaxis against exacerbation (not prophylaxix)
Antibiotics Long-term macrolide therapy can help to reduce the number of patients experiencing
exacerbations as well as the frequency of exacerbations experienced by each patient.
Macrolides: Azithromycin may be most effective at preventing exacerbations requiring
treatment with both antibiotics and corticosteroids. Patients who are older and who have
milder COPD may be more likely to respond to long-term azithromycin therapy.
Current smokers appear to receive no benefit from long-term azithromycin. Use of
azithromycin in current smokers should be carefully considered
Cephalosporins: cefprozil & cefuroxime axetil. Tetracyclines: doxycycline
Fluoroquinolones: ciprofloxacin, levofloxacin, moxifloxacin
Sulfonamide Combinations: sulfamethoxazole/ trimethoprim

56. A pharmacist harm herself during vaccine application, what do first? wash hands
Needle stick injury prevention
➢ Infection control guidelines are designed by Canadian Centre for Occupational Health and Safety
CCOHS to protect providers and patients from exposure to diseases spread by blood and bodily fluids.
➢ Providers should treat blood & other bodily fluids of ALL patients as a potential source of infection.
The following precautions should be taken:
➢ Wear gloves if likely to touch body substances or mucous membranes
➢ Wear eye protection and face mask if likely to be splashed
➢ Wear lab coat or gown if likely to be soiled
➢ Place needles in sharps containers (do NOT recap)
➢ Remove gloves using aseptic technique and wash hand immediately
➢ Place soiled articles in plastic bag for disposal. Place soiled linen in laundry bag
➢ If you pierce or puncture your skin with a used needle, follow this first aid advice immediately:
encourage the wound to bleed, ideally by holding it under running water. wash the wound using
running water and plenty of soap. do not scrub the wound while you're washing it.

57. Patient on Capcetabine and Ramipril. Finished course of Ciprofloxacin 2 days ago. Patient
now has erythematous on palms and feet (hand foot syndrome). Cause of this due to?
a) Capecitabine
b) Ciprflox
c) Ramipril
Hand-foot syndrome (HFS) Apply moisturizers at least TID. Dose interruptions and reduction
(erythema, edema, pain, peeling of Avoid prolonged heat exposure. result in greatest benefit.
skin on fingers and toes) (weeks to Urea-based emollients preferred. Manage promptly with
months) Capecitabine, liposomal Avoid irritation and friction from symptomatic treatments and
doxorubicin, fluorouracil ill-fitting shoes & clothing. analgesics.
Hand-foot-skin reaction (HFSR) Vitamin B6 150–200 mg/day may Discontinue therapy if severe or
(same as above + hyperkeratosis at be prescribed (evidence poor). recurrent.
pressure points) (weeks to months) Limit prolonged pressure to Wound care for desquamation and
Axitinib, dabrafenib, pazopanib, weight-bearing areas and treat ulcerations.
regorafenib, sorafenib, sunitinib existing calluses & hyperkeratosis
58. What to suggest for capecitabine induced Mouth erythema and ulcer?
a) Mouth wash by normal saline
b) Mouth wash by benzydamine
c) Mouth wash by nystatin
d) Use chlorohexidine MW (avoid alcohol)- the question is all except
Mucositis Cryotherapy (ice chips) prevents mucositis or reduces its severity in patients receiving bolus
(general, painful treatments of fluorouracil or high dose melphalan.
erythema or Ice & other cold foods are contraindicated with oxaliplatin containing regimens (e.g., folfox).
localized lesions) Dexamethasone oral rinses for the first 2–4 cycles of everolimus treatment reduces the
(within days to incidence and severity of mucositis. Evidence supports the use of low-level laser therapy to
weeks) prevent mucositis in stem cell transplant settings, but it is used in limited centres.
Severity varies Employ nonpharmacologic measures: basic oral hygiene and the use of a clean, soft-bristle
with agent and toothbrush that is replaced on a regular basis, judicious flossing and use of bland M.W for mild
individual, symptoms such as soda and salt rinses or alcohol-free MW (may irritate)
Afatinib, Combination mouth washes usually are institution-specific and contain varying proportions of
Cabozantinib, lidocaine, nystatin diphenhydramine, antacids, and dexamethasone.
Everolimus, Q. Benzydamine M.W: Can be diluted (1:1) with lukewarm water to reduce irritation
Ibrutinib, Itraconazole solution & Capsules, 200 mg daily PO × 7–14 days. Capsules less effective due
Lenvatinib, to variable absorption.
Palbociclib, Nystatin: 5–10 mL TID PO. Rinse, then spit or swallow (if extends to throat or esophagus).
Regorafenib Chilling nystatin may improve palatability, but avoid in patients receiving oxaliplatin
(pharyngeal neuropathies can be triggered by cold).
Fluconazole:100-200mg daily PO×7-14 days. Esophageal mucositis: ↑ to 200–400 mg daily PO
Doxepin 0.5% oral rinse may be effective in relieving pain associated with oral mucositis.
Systemic analgesics (e.g., opioids) for severe mucositis

59. Methadone dose was 100 mg / week is


that ok? Typical methadone dose to
reduce cravings & withdrawal symptoms
between 60-120mg daily.

60. Pharmacist wants to delegate some his/her tasks to tech. Which is the biggest barrier?
a) Pharmacist attitude
b) Technician workspace
c) Pharmacist availability to supervise
d) Technician not well trained
e) College regulations that some duties must be done by Pharm.

61. Which one can be stepped down for oral antibiotic, after 48 hours?
a) Pyelonephritis
b) Osteomyelitis
62. In pyelonephritis which one can’t be used after initiating step down TX?
a) Nitrofurantoin
b) Cotrimoxazole
c) Fluoroquinolone
Nitrofurantoin should not be administered to patients with acute bacterial pyelonephritis as nitrofurantoin
does not reach therapeutic concentrations in the upper urinary tract, and bacteremia often accompanies this
disease. CI in anuria, oliguria, or significant impairment of renal function (creatinine clearance under 60 ml per
minute or clinically significant elevated serum creatinine) are contraindications to therapy with this drug.
Mild to E. coli (90%), P. mirabilis & K. pneumoniae (5%) Fluoroquinolone Amoxi/clav PO or
moderate Occurs in women who experience recurrent (ciprofloxacin, SMX/TMP PO or
pyelonephritis uncomplicated UTIs but at lower frequency than levofloxacin, Trimethoprim PO
OR Acute cystitis. norfloxacin), all for 10-14 days
Nonobstructive Classic presentation includes fever, N&V, flank PO × 7–14 days
Pyelonephritis pain with or without associated irritative urinary
Severe symptoms. Aminoglycoside Fluoroquinolone IV
pyelonephritis Patients who present with UTI with only lower IV ± ampicillin IV × 10–14 days or
urinary tract symptoms or asymptomatic for initial therapy; 3rd generation
bacteriuria occasionally have associated occult if appropriate, step cephalosporin IV ±
renal infection. down to oral aminoglycoside IV
Bacteremic infection occurs most frequently in therapy as in mild × 10–14 days or
diabetic women or women >65 y. to moderate Carbapenem IV ×
Urine Culture: always recommended. infections in order 7–14 days for
Consider blood cultures. to complete 10–14 ESBL-producing
days organisms
63. First side effect of atomoxetine that you should monitor?
a) Eye
b) Tremor
c) Suicidal thoughts
 ADHD and its related co‐morbidities may be associated with increased risk of suicidal ideation and/or
behaviour. Rigorous clinical monitoring for suicidal ideation or other indicators of potential for
suicidal behaviour is advised in patients of all ages. This includes monitoring for agitation‐type of
emotional and behavioural changes, and clinical worsening.
 Families and caregivers of pediatric patients being treated with atomoxetine hydrochloride should be
alerted about the need to monitor patients for the emergence of agitation, anxiety, panic attacks,
hostility, irritability, hypomania or mania, unusual changes in behaviour, and other symptoms, as well
as the emergence of suicidality particularly after starting treatment or changing the dose. Such
symptoms should be reported immediately to healthcare providers. Such monitoring should include
daily observation by families and caregivers.
 Growth and development should be monitored during treatment with atomoxetine. Patients
requiring long‐term therapy should be monitored and consideration should be given to dose reduction
or interrupting therapy in patients who are not growing or gaining weight satisfactorily.
 Atomoxetine hydrochloride should be used with caution in patients with congenital long QT syndrome,
acquired long QT syndrome (for example, due to concomitant use of a drug that may prolong the QT),
or a family history of QT prolongation. Patients should be screened for pre‐existing or underlying
cardiovascular or cerebrovascular conditions before initiation of treatment with atomoxetine
hydrochloride and monitored for new conditions of the heart or brain during the course of treatment.

64. Schizophrenic patient who is


obese, has hyperlipidemia,
insomnia, dementia and
hyperglycemia, fear of weight
gain, which one is drug of
choice?
a) Quetiapine.
b) Risperidone.
c) Aripiprazole.
(Avoid ROCQ with metabolic disorders:
Olanzapine / clozapine: ++++
Resperidone / Quetiapine: +++)

65. Most inappropriate advice for a red eye with pruritus & discharge?
a) Occlusive eye patch
b) At least 4 times gramicidin
c) Put compress
d) Give antibiotic for 48 hours, if no improvement, refer
An eyepatch is a small patch that is worn in front of one eye. It is often worn by people to cover a lost or
injured eye (or after surgery), but it also has a therapeutic use in children for the treatment of amblyopia (See
orthoptics and vision therapy). Eyepatches used to block light while sleeping are referred to as a sleep mask.

66. Patient smokes pack a day for (I think 30 years). Has HTN. Best non-pharm to control BP?
a) DASH diet
b) Smoking cessation
All individuals should be advised about a healthy lifestyle to prevent or control hypertension and CVD:
 Weight loss of 4 kg or more if overweight (target body mass index: 18.5–24.9 kg/m2; waist
circumference <102 cm in men and <88 cm in women).
 Healthy diet—high in fresh fruits, vegetables, soluble fibre and low‐fat dairy products, low in saturated
fats and sodium, e.g., DASH
 Sodium intake target of <2000 mg (88 mmol) per day.
 Increase dietary potassium intake (e.g., fruit and vegetable component of DASH eating plan) if the
patient is not at risk of hyperkalemia. Risk factors include renin‐angiotensin inhibitors or other agents
that can increase potassium, chronic kidney disease and serum potassium >4.5 mmol/L.
 Regular, moderate intensity cardiorespiratory physical activity for 30–60 minutes on most days.
 Low‐risk alcohol consumption (0–2 drinks/day, <9 drinks/week for women and <14 drinks/week for
men). Smoke‐free environment.
67. A study has done in particular period of time, now
about a drug. what kind of study it is?
a) Cohort
b) Case control
c) Cross sectional

68. What is a nested case control study


a) A cohort study in a case control study
b) A case control in a cohort study
c) A randomized controlled study

69. Pt with asthma and takes medication for parkinsonism he got flu and wants to take a
medication for his symptoms what is the most suitable drug that could be used
a) Amantadine
b) Oseltamivir
c) Zanamivir (CI in Asthma and COPD)

70. Female 39 y came to you she told you that she is play sport, taking multivitamins, non
smoker, she is fear bec. she has a family history of macular degeneration she told you that she
heared there a combination use for prophylaxis of macular degeneration that contains vit C, E,
zinc, B. carotene, copper
a) Take this combination as it is
b) Take this combination without B carotin
c) Take this combination, replace B- carotene with leutine
d) Don’t take any thing

71. Which one is true regarding insulin mixing


a) Detemir
b) Glargine
c) Regular and NPH
Long acting can’t be mixed Detemir& Glargine (slow depot)

72. A patient has insomnia and has no appetite (depression symptoms) what’s the reason you
refer him to doctor?
a) He has these symptoms for more than 3 weeks
PHQ9 is over the last 2 weeks

73. A patient when went through chemotherapy, he has intense nausea. What should be given
added on to next cycle of treatment? Lorazepam  For anticipatory nausea
74. Where to search compatibility of parenteral drug:
a) Martindale
b) Micromedex (has a link to Trissel’s
c) AHFS

Drug X is administered in IV form of NaCl solution where we can find its compatibility in this
solution?
a) Manufacture monograph (If the q is about reconstitution of a parenteral product)
b) CPS
c) TC

75. A patient has rosacea, what nonpharmacological action do you consider?


a) Don’t drink alcohol (no spicy food, Alcohol cause flushing)
b) Use astringents for cooling

76. Dr prescribed topical metronidazol for 12 weeks with pt improvement about 50%, then the
case worsened and new red spots
appeared. Dr prescribed oral
tetracycline for 6 months. what
the ttt instruction?
a) Wrong ttt
b) Continue metronidazole
for 6 months.
c) Stop metronidazole & start
oral tetracycline for 6 mo.
d) Continue metronidazole
& add oral tetracycline.

77. Which one is associated with


ONJ (osteonecrosis of the jaw)
a) Oral bisphosphonates
b) IV bisphosphonates
c) Someone starting
bisphosphonates

78. What is DOC in fibromyalgia? TCA


For chronic neuropathic pain 1st line TCA → For depression TCA is a 2nd line
But for depression + neuropathic pain it's not TCA → It's SNRI duloxe ne it's 1st line for depression
Fibromyalgia pain + depression it's duloxetine
CTC: duloxetine and pregabalin are the only 2 prescription medications with official Health Canada indications
for the treatment of fibromyalgia; both drugs have comparable efficacy and tolerability at the recommended
doses. Low doses of amitriptyline at bedtime (e.g., starting at 5 mg and progressing slowly, every 2–3 weeks,
to a maximum of 50 mg) can improve sleep and reduce pain and fatigue.

79. Which one needs sales record?


a) Butorphanol
b) Buprenorphine Patch

80. Patient 66-year-old with atrial fibrillation, HTN in range just had non-cardiogenic stroke
and on aspirin. Calculate CHADS2 score?
a) 1
b) 2
c) 3
d) 4

81. What to give as a prophylaxis? (WRONG Question)


a) Add clopidogrel to ASA.
b) Warfarin + ASA + clopidogrel.
c) Switch aspirin to clopidogrel.
d) Switch aspirin to dipyridamole.
Tips: For patients who were taking ASA prior to
their stroke, consider other antiplatelet agents,
such as clopidogrel 75 mg once daily or
ASA/dipyridamole 25/200 mg twice daily

82. Patient on dabigatran, he has a renal


impairment, what’s the problem?
a) Dabigatran has renal clearance
b) Do not chew and crush and also enteric coated.
c) Increase risk of bleeding with creatinine clearance 22 ml/min
CTC: Patients with severe renal impairment (eCrCl <30 mL/min)
When removing a capsule from the blister, please note the following instructions:
 Tear off one individual blister from the blister card along the perforated line
 Peel off the backing foil and remove the capsule
 The capsule should not be pushed through the blister foil
 PRADAXA should be taken with a full glass of water and can be taken with or without food. If PRADAXA
upsets your stomach, take it with meals or within 30 minutes after meals. If PRADAXA still upsets your
stomach, consult your doctor or pharmacist. It is important to continue taking PRADAXA as prescribed.
 Swallow the capsule(s) whole. Do not chew or open the capsule. Do not sprinkle the pellets on food or
mix with liquids.
83. Patient on rosuvastatin has myalgia,
physician stops statin, what’s the next thing to
do?
a) Simvastatin 80mg
b) Atorvastatin 40mg
c) Ezetimibe
d) Gemfrobzil

84. What to monitor considering the myalgia?


a) Urine discoloration
b) Skin
c) Creatinine kinase (CK MM)
Statin induced myopathy, monitor:
 Pain/myalgia
 CK
 SCr (GFR)
 Electrolytes
Urine discoloration is a helper for the patient to
identify a red flag related to rhabdomyolysis
secondary to statin, however it is NOT clinically
specific to kidney damage and WILL NOT BE MONITORED BY CLINICIANS

85. What is the ezetimibe side effect? Arthralgia


CTC: Common: back pain, arthralgia, diarrhea, fatigue, abdominal pain, dizziness, headache.
Rare: myopathy, rhabdomyolysis, hepatitis, acute pancreatitis, thrombocytopenia.

86. A patient asks for a new treatment for diabetes mellitus, where we should look?
a) Guideline
b) Pubmed (primary)

87. 25 years old was sexually assaulted, has rx for plan b. But didn't tell her doctor about the
assault as she believe it's not important to inform him, what’s the appropriate action?
a) Dispense rx as it is (respect autonomy)
b) Tell her it is the pharmacist responsibility; it is obligatory to report any sexual assault
c) Give her list of sites that she can contact if she is sexually assaulted
d) Inform the police

88. A skateboarding teenager fell on his knees. He comes in the pharmacy with his knees
excoriated, red, swelling, but not infected & no pus. There was no dirt or debris in wound. After
appropriate irrigation of the wound, what is the best action the pharmacist should do?
a) Give topical antibiotics
b) Cover the wound with an appropriate dressing
c) Refer him to the physician to check if there is an infection in the wound
d) Recommend tetanus injection.
CTMA: Self‐treatment of minor wounds includes the following steps:
Cleanse the wound: Remove dirt and debris from the wound as soon as possible to prevent infection and
promote healing. Once debris has been removed, carefully wash the wound with water. Drinkable tap water is
as effective for wound cleaning as saline or purified water, with no increased risk of infection or decrease
in wound healing
Stop the bleeding: Apply a clean dressing or gauze to the wound area for 10 minutes. If the bleeding does not
stop within 10 minutes (or 15 minutes if anticoagulated), the patient should seek emergency medical
attention. Monitor those on anticoagulants for up to 15 minutes, as the clotting time will be longer, and refer
if the bleeding does not stop within that time period. Visible pieces of dirt or other foreign material that
remain after irrigation can be gently picked out of the wound with tweezers that have been cleaned with
rubbing alcohol, or by brushing gently with clean gauze. These steps protect the wound from infection and
tissue destruction and help the wound to heal faster
Apply a dressing: Choose a dressing that maintains a moist wound bed (see Dressings) to protect
the wound from possible infection and improve the healing process. Minor cuts, paper cuts or skin cracks can
be closed using tissue adhesives or a liquid bandage. Larger cuts, where the edges won't stay together, the
edges are jagged or the wound is deep, may need stitches.
Débridement: The removal of foreign material such as dead or contaminated tissue from the wound should be
performed only by an appropriately trained health‐care practitioner and under sterile conditions

89. When you don’t refer? If bleeding stops when you put pressure
CTMA: The initial assessment of a wound determines if it can be self‐treated or requires referral for further
treatment. Superficial wounds that are small and accompanied by limited bleeding are suitable for self‐
management. Patients require further assessment and/or treatment in the following situations: wounds that
continue to bleed after 10 minutes of pressure; deep puncture wounds; gaping wounds; wounds that expose
fat, muscle or bone; wounds from animal bites; wounds with visible foreign material or dirt; wounds causing
severe pain; wounds in patients with underlying medical conditions or drug therapy that put them at risk of
delayed healing (not healed by 3 weeks) or infection
(see Wound Complications); and large, complicated
or chronic wounds

90. Pt which is alcoholic, then he discovered


that he has viral hepatitis B, what could be
the cause of getting this infection?
a) The pt is alcoholic
b) The pt is sexually active
c) SE of corticosteroid
91. What should be your advice to this pt? Stop alcohol

92. Hepatitis B vaccine Side effect:


a) Multiple sclerosis.
b) GBS
c) Encephalitis

93. Modafinil schedule? Schedule I prescription drug. Written, Verbal, Refills allowed
94. A mother came to your pharmacy; her Child has nits & lice in class the school. He is
allergic to ragweed. What is the first action?
a) Give vacation for the child till treatment
b) Give prophylactic treatment for all children in class
c) Prevent head to head contact among children
d) Treatment when you see nits.
CTC: Treatment for a lice infestation should not usually be initiated unless there is a clear diagnosis
(live lice visualized).

95. What will you recommend for him?


a) Tea tree oil
b) Permethrin 1% cream rinse (CI with ragweed allergy)
c) White vinegar → only for nit removal
d) Pyrethrine/Piperonyl butoxide
Another version: Scabies ragweed allergy? Crotamiton
96. Given RESULTZ Isopropyl myristate. Which is true?
a) Contraindicated with ragweed
b) Can cause seizures
c) Apply to dry hair for 10 mins (after 10' you rinse with warm water)
d) Apply, massage hair, then rinse
e) Leave it for 30 min, allow to dry

97. How do you know lice treatment worked?


a) No live lice after 24 hours
b) No pruritus after 24 hours

98. Nitroglycerin is contraindicated in?


a) Crcl < 60. (WRONG ANSWER)
CTC: Nitroglycerin is contraindicated in:
• Patients with known hypersensitivity to nitroglycerin or any of the excipients, or with previous idiosyncratic
reaction to organic nitrates
• Patients with severe anemia; closed angle glaucoma; increased intracranial pressure;
• Patients with acute circulatory failure (cardiogenic shock, severe hypovolemia or severe hypotension);
• Patients with HF (aortic or mitral stenosis, constrictive pericarditis, hypertrophic obstructive cardiomyopathy)
• Patients with myocardial infarction.
• With phosphodiesterase type 5 (PDE5) inhibitors.

99. Child given Phenytoin 100mg TID (I think chewable/infatabs) for epilepsy first time, what
is not appropriate to tell the doctor?
a) Phenytoin chewable cause more gingival hyperplasia than suspension (both the
same)
b) Make dose BID to avoid school time dosing (but Dr has to change it not pharmacist)
c) Give with food to decrease GI irritation (with, without or during meals)
NOT appropriate to tell the Dr. = chewable phenytoin causes more ADRs than other oral forms (NOT logical;
this is a SUPERGIFT Q, so for Canadian grads, they laugh and say "Thank you PEBC". Means there's NOTHING
NEW here!) This is an IMPORTANT Q, and any mistakes in similar questions is serious.
100. Phenytoin gingival hyperplasia most with
a) Chewable tablets
b) Capsules
c) Suspension
d) Parenteral solution
Chewable phenytoin tablets might be possibly more linked to gingival hyperplasia due to a relation to the
salivary level of phenytoin. This is a HISTORICAL Q and a MARGINAL point.

101. Monitoring for phenytoin


a) Hemoglobin (CBC)
b) Urinalysis
c) Kidney function
Phenytoin serum level determinations may be necessary to achieve optimal dosage adjustments
CBC, liver function, vitamin D status and suicidality should be monitored with chronic use.
Additional monitoring for IV use: continuous cardiac monitoring (rate, rhythm, blood pressure). Blood
pressure and pulse should be monitored every 15 minutes for 1 hour after administration.
Phenytoin may impair mental and/or physical abilities required for performance of hazardous tasks such as
operating machinery or driving a motor vehicle.
Serum concentrations of phenytoin may be difficult to interpret in renal failure or hepatic failure. Monitoring
of free (unbound) concentrations or adjustment to allow interpretation is recommended
Rapid IV injection may lead to hypotension, arrhythmias, seizures and cardiorespiratory arrest. These effects
are generally not associated with oral overdose of phenytoin alone.
Management of phenytoin overdose is mainly supportive. Monitor respiratory and circulatory function and
employ appropriate supportive measures. In addition, monitor ECG, phenytoin serum levels and blood
glucose.

102. Onychomycosis: patient with toenail infection taking itraconazole for 12 weeks, her nail
still discolored and coming off (or somewhat). treatment failure because:
a) Wrong choice of drug
b) Short duration of
treatment
c) Wait a bit it’ll get better
Onychomycosis: fungal infection;
can be either toenail or fingernail;
we use itra only if yeast infection &
fingernail, if toenail we only use
terbinafine which is strongest. It
can take upto 1 year to go back to
normal.
103. Patient diagnosed with dementia & started donepezil 10 mg, he has allergy to TMP, what’s
drug related problem?
a) Donepezil dose too high
b) Patient allergy to HTCZ
It is the only cholinesterase inhibitor approved for
all disease severities
The initial daily dose (5 mg) of donepezil is usually
taken at night, but can be taken in the morning if
sleep disturbances occur. After 4 weeks, try
increasing to the target dose of 10 mg/day. Adjust
dose after 4 wk
Donepezil was effective in 3‐ to 6‐month trials in
patients with mild to moderate Alzheimer disease
(MMSE score of 10– 26).
Treatment should be initiated and supervised by a physician experienced in the diagnosis and treatment of
Alzheimer's dementia. Diagnosis should be made according to accepted guidelines (e.g. DSM IV, ICD 10).
Therapy with donepezil should only be started if a caregiver is available who will regularly monitor drug intake
for the patient. Maintenance treatment can be continued for as long as a therapeutic benefit for the patient
exists. Therefore, the clinical benefit of donepezil should be reassessed on a regular basis. Discontinuation
should be considered when evidence of a therapeutic effect is no longer present. Individual response to
donepezil cannot be predicted.

104. When you called the nurse, she pissed about calling because of this issue. You tell her you
are upholding which ethic?
a) Fidelity
b) Non-maleficence
c) Beneficence

105. Student counseling on diabetic med, but patient found it confusing. What is the best action
to take?
a) Give student extra material to read
b) Go over with student counseling prior to dispensing
c) Review the information with student before counseling a patient
All 3 answers apply, but we have to pick one answer.
Education is a weak approach in general and people keep forgetting, depending on frequency of counseling,
the topic or the type of therapy.
The most appropriate option here which NOT only fulfills beneficence and nonmaleficence but also the role of
the preceptor is to attend the counseling session and to intervene whenever needed to either complement or
correct any counseling points/tips.
106. Where to search for most recent evidence-based medicine:
a) Drug Product Database.
b) Systemic review of randomized trials
c) Cochrane library → highest level of evidence‐based medicine
d) Pharmacy letter
Primary literature = systematic review

107. Getting an antiviral for prophylaxis or Influenza vaccine in long term care facility, what
the most important thing you should consider?
a) Agent that does not meet renal adjustment
b) Strain of virus
c) Cost of drug
d) Dosing schedule
e) Allergies.
https://www.cdc.gov/flu/professionals/infectioncontrol/ltc-facility-
guidance.htm#:~:text=CDC%20recommends%20antiviral%20chemoprophylaxis%20for,was
%20identified%20on%20affected%20units.
Prevention and Control of Influenza in the Long-Term Care Setting
108. HIV case: HIV patient with opportunistic infection with CD4 count less than 200,
Pneumocystis jiroveci Pneumonia, treatment: Cotrimoxazole for 21 days
CD4 <200 cells/mcL or thrush (Pneumocystis jirovecii pneumonia) Primary and secondary prophylaxis
Q. PCP  Preferred prophylactic therapy for PCP is SMX/TMP.
Oral candidiasis  Alternatives: dapsone PO, atovaquone PO or monthly inhaled pentamidine.
 Stop prophylactic if CD4 > 200 cells/mcL × ≥ 3 months
 Treatment of Oral candidiasis with azole antifungal agents.

109. Given Azithromycin once per week, what is this for?


a) MAC prophylaxis
CD4 <50 cells/mcL: Mycobacterium avium complex (MAC)
The preferred prophylactic therapy is a macrolide (azithromycin once weekly or clarithromycin BID).
Rifabutin is an alternative
Stop 1ry prophylactic if CD4 >100 cells/mcL × ≥3 months and stop 2ry if CD4 >100 cells/mcL × ≥6 months

110. Which drug regimen is not right for HIV:


Regimen including abacavir, can’t be used with HLA‐B positive. (check HIV regimens).
Combination of drugs that cannot be used together.
Abacavir (need to complete HLA‐B*5701 testing prior to starting to avoid rash) + interaction with alcohol
(Remember Carbamazepine CI HLAB 15:02)
111. Anastrozole side effect? Arthralgia, Hot flushes
Aromatase Endometriosis patients tend to have high levels of Aromatase P450 expressed at their
P450 inhibitors endometriums. Aromatase P450 inhibitors reduce levels of estrogen production from
Anastrazole sources other than the ovary, such as the adrenal gland.
Letrozole They can be used with OCs, GnRH analogues, or Progestins to reduce the risk of ovarian
cyst development.
S.E: hot flushes and sweating, osteoporosis, insomnia, body aches, loss of appetite.
As anastrozole lowers circulating estrogen levels it may cause a reduction in bone mineral
density with a possible consequent increased risk of fracture.
Women with osteoporosis or at risk of osteoporosis, should have their bone mineral density
formally assessed at the commencement of treatment and at regular intervals thereafter.
Treatment or prophylaxis for osteoporosis should be initiated as appropriate and carefully
monitored. The use of specific treatments, e.g., bisphosphonates, may stop further bone
mineral loss caused by anastrozole in postmenopausal women and could be considered.

112. Black pt with diabetes, Metformin 500 TID, Gliclazide 40 po, after some time HbA1C 7?
a) Increase gliclazide
b) Add insulin (always when A1C is above 9)
The total daily dose may vary from 40‐320mg. The dose should be adjusted according to the individual's
response, commencing with 40‐80mg daily and increasing until adequate control is achieved. A single dose
should not exceed 160mg. When higher doses are required, gliclazide should be taken twice daily and
according to the main meals of the day.
In obese patients or those not showing adequate response to gliclazide alone, additional therapy may be
required

113. Patient with insomnia and anxiety?


a) Temazepam
b) Zopiclone
Some benzodiazepines have demonstrated efficacy
in the treatment of anxiety disorders, although they
generally do not help with comorbidities like
depression. They have also been shown to be useful
in relieving acute anxiety, agitation and panic
attacks at the beginning of treatment while waiting
for antidepressant treatment to take effect, as well
as to mitigate anxiety and agitation flare‐ups that
may occur when beginning to take antidepressant.
Benzodiazepines remain a second‐line choice in the
treatment of anxiety disorders because of the risk of
abuse, sedation, cognitive impairment, dependence
and withdrawal syndrome, and falls associated with
their use.
It is generally recommended to use them as a short‐term measure during the first few weeks of treatment
only, if possible. However, benzodiazepines are sometimes continued for a few months if they are effective
and well tolerated in the absence of antidepressant response, until an effective treatment is found.
In the elderly, benzodiazepines should be used with caution because, due to age‐related pharmacokinetic
changes, older patients are more likely to experience adverse effects, such as psychomotor and cognitive
impairment, and present a higher risk of falls and fractures. Benzodiazepines are not recommended in the
presence of a comorbid substance use disorder or a history of one.

114. Lithium with ACEIs?


a) Change to lamotrigine
b) Monitor closely for lithium toxicity
Interactions which increase lithium concentrations. Co‐administration of the following drugs with lithium
may lead to increased lithium concentrations and a risk of toxicity:
 Any drug which may cause renal impairment has the potential to cause lithium levels to rise, thereby
causing toxicity. If the use of the drug is unavoidable, carefully monitor lithium blood level and adapt
dosage as necessary.
 Antibiotics (metronidazole, tetracyclines, co‐trimoxazole, trimethoprim), N.B. Toxic symptoms may also
occur at low or normal levels when used in conjunction with co‐trimoxazole or trimethoprim. Lithium
toxicity has been reported on isolated occasions in patients receiving spectinomycin.
 Non‐steroidal anti‐inflammatory drugs (including selective cyclooxygenase (COX) II inhibitors); monitor
serum lithium concentrations more frequently if NSAID therapy is initiated or discontinued.
 Drugs affecting the renin angiotensin system (ACE inhibitors, Angiotensin II receptor antagonists).
 Diuretics (including herbal preparations). In addition to the effects noted above, thiazide diuretics
show a paradoxical antidiuretic effect resulting in possible water retention and lithium intoxication.
Loop diuretics (furosemide and bumetanide, and etacrynic acid) seem less likely to cause lithium
retention, although caution is warranted.
 Drugs affecting electrolyte balance, e.g. steroids, may alter lithium excretion and should be avoided.

115. Which of the following is associated with the highest rate of hyponatremia or Which
causes SIADH (Syndrome of inappropriate ADH)??
a) Fluoxetine (SSRI)
b) Mirtazapine
c) Moclobemide
d) Bupropion
Also, SSRI, TCA, SNRI, Chloropromazine, CBZ → C/I with ADH t e.g: desmopressin
CTC: When choosing an antidepressant, it is important to take several factors into account. SSRIs are less likely
than tricyclic antidepressants (TCAs) to cause anticholinergic side effects or to worsen orthostatic
hypotension, which are common and problematic in this population. An increased risk of hyponatremia/SIADH
(syndrome of inappropriate antidiuretic hormone secretion), possibly potentiated by concurrent thiazide use,
has been reported in elderly patients taking SSRIs. Monitoring of electrolytes is recommended in this setting
because of the effect of hyponatremia on cognitive function.
116. Post Herpetic Neuralgia with acute pain, what is the first line treatment?
a) TCA → amitriptyline is the preferred agent
b) Opioid
c) Lidocaine.
Tricyclic antidepressants (TCAs), gabapentinoids such as gabapentin and pregabalin, and SNRI antidepressants
(duloxetine, venlafaxine) are considered first‐line agents. When initiating pharmacotherapy, initial follow‐up
should be arranged in about 2 weeks and regularly thereafter for monitoring
Tramadol and topical lidocaine can be considered as second line. Combination therapy can be helpful.

117. Ampoules has 0.5ml with 10Units. How many ampoules need to added to 500ml of D5W
if u want 60ml/hr at rate of 4U/min?
a) 8 amp
b) 20 amp
c) 24 amp
4U / MIN = 4 * 60 = 240U/HR. 10U ‐‐‐‐‐‐‐‐‐‐ 0.5ML 240U ‐‐‐‐‐‐‐‐‐‐‐‐ X
X = 240 * 0.5 / 10 = 12ML/.5ML = 24 ampoules

118. TSH is high what symptoms you expect? cold intolerance (hypothyroidism)

119. Inventory in pharmacy department why is UPC code and DIN is better than just DIN?
a) Compare stock
b) No. of pack size
c) No. of tablets
d) Same medication different package size
Remember: What's the advantage of bar code system when scanning medication used in the
dispensing work flow?
a) Helps make sure you have the correct pack size
b) Helps to make sure you have the correct strength
Pack size does NOT matter for accuracy or safety.
Pack size is an advantage of bar code scanners when you do inventory or stock counts

120. Cancer associated thrombosis? LMWH (malignancy or pregnancy: LMWTH)


Venous Traditional Routine prophylaxis with ASA, Q. Vitamin K antagonists e.g warfarin
thromboembolic cytotoxic warfarin or low molecular weight are used with caution as cancer
events (deep vein drugs, heparin may be required for patients are susceptible to wide
thrombosis, bevacizumab, patients receiving regimens fluctuations in INR.
pulmonary tamoxifen, containing dexamethasone and Q. Low molecular weight heparins
embolism) (within sunitinib, thalidomide or its analogues. preferred for long-term
weeks to months) thalidomide and anticoagulation.
analogues
121. Patient with asthma on use fluticasone/salmeterol & salbutamol for 3 years, dr. changed
him to formoterol/budesonide, he has a history of infection. Now he is going to Florida and he
got a prescription for azithromycin, what is the cause for choosing azithromycin as best drug
option:
a) History of infection
b) Long term use of corticosteroid
c) Everything is okay and functions are good.
Answer: long‐term low‐dose azithromycin (exacerbation prophylaxis): is applicable ONLY for SEVERE COPD or
asthma.

122. Which counselling do you give?


a) Use salbutamol for rescue treatment with Symbicort
b) Use high dose of symbicort to to prevent worsening of asthma
c) Replace salbutamol
d) Stop salbutamol (no need for salbutamol)

123. In regard of symbicort which one is correct


a) Need propellent
b) Use spacer to increase lung absorption
c) Use deep strong inhalation

124. Which one does not cause hyperkalemia or hypokalemia?


a) Acetaminophen
Hyperkalemia Hypokalemia
Drug-induced: ACEIs, ARBs, BBs, aliskiren, Drug-induced: aminoglycosides, amphotericin B,
aminocaproic acid, cyclosporine, digoxin overdose, antipseudomonal penicillins, beta2-agonists, caffeine,
drospirenone, heparin, K+ supplements, K+ sparing foscarnet, insulin, laxatives, licorice, long-term
diuretics, ketoconazole, NSAIDs, penicillin G corticosteroid therapy, loop and thiazide diuretics,
potassium, trimethoprim or TMP/SMX, theophylline, tocolytic agents.
pentamidine, succinylcholine, tacrolimus, alfalfa and . Diarrhea, vomiting
nettle. . Inadequate dietary intake
. Renal failure, hyporeninemic hypoaldosteronism. . Familial history (Bartter or Gitelman syndrome)
. Diabetes, adrenal insufficiency. . Mineralocorticoid excess, e.g., primary aldosteronism
. Familial history of hyperkalemia . Renovascular disease, Metabolic alkalosis
. Acidosis, Massive transfusions . Osmotic diuresis (diabetes), Hypomagnesemia
. Crush injury, trauma, hemolysis, tumor lysis. . Increased sweat loss, Dialysis/plasmapheresis
125. A patient with diabetes type 1 taking glargine and glulisine, dyslipidemic taking
atorvastatin 80 mg. she comes with the following Rx: metformin 500 mg t.i.d., glargine 40 units
at bedtime, glulisine 40 units before meals and atorvastatin 80 mg.
What is the first thing that pull your attention in this Rx:
a) Dose of metformin
b) Too high dose of glulisine (120 U total: should be 20U ‐1/2 Glargine dose only, 6‐8 each meal)
c) Too high dose of glargine
d) Too high dose of atorvastatin
NCBI: The ideal final total bolus insulin amount (the sum of all meal bolus doses) should be about half the
basal dosing. Calculation of starting bolus dosing can be done as in the basal plus regimen, either 4 to 6 units
per meal or 0.1 U/kg/d. Alternatively, if the patient is on 60 units of long‐acting analog and BGs are well above
goal range, the prescriber could consider about 20 units of bolus dose (60 U divided by 3 meals) if the patient
eats 3 routine meals a day with at least 30 g of carbohydrates, and physical activity levels are fairly consistent.
If the patient eats the most carbohydrates at lunchtime, consider more bolus at lunch (ie, 18 U of bolus for
breakfast and dinner and 24 U of bolus for lunch coverage). Patients need to separate the time between the
bolus doses, usually a minimum of 4 hours apart, to avoid insulin stacking, which is a common reason for
hypoglycemia. Insulin stacking occurs when additional quick or rapid insulin is injected when the previous
insulin is still in the body or when there is insulin on board. Typically, bolus analogs stay in the body for about
4 to 6 hours, thus necessitating separation of the doses at least 4 hours apart. Patients sometimes inject more
bolus insulin after high postprandial readings, which can result in insulin stacking. In some cases, the patient
may misunderstand and take mealtime insulin at a scheduled time instead of at the time of the meal.

126. A patient taking NPH 10 units before bedtime and rapid acting 4 units before each meal,
he called you and told you that he took 4 units NPH instead of rapid acting and asked what he
should do right now:
a) Just take 6 units of NPH now
b) Take 4 units of rapid acting and do not take the remaining dose of NPH.
Answer: Administer the bolus since still before dinner then later administer the balance of
the basal

127. Diabetic patient wants med for ED given sildenafil. Best Counselling?
a) Sildenafil less effective in diabetics
b) May last 4 hours
c) Decrease night vision → impairment in vision, sensetivity to light

128. Side effect of corticosteroid? Mood change


129. In hospital clostridium difficile is spread, what should you do?
a) Visitors wash their hands with alcohol. First measure: Always wash hands
b) Visitors put grab
c) Patients not be visited for 24 hr
d) Stool examination of other patients

130. Another question what is the treatment for CD? Oral vancomycin (first line)

131. IV drug abuser patient has developed endocarditis, what is the most causative organism of
endocarditis?
a) Staph aureus (40%)
b) Strep. viridans (20%)
c) Entero (10%)
d) S. bovis

132. Empiric treatment:


a) Ciprofloxacin
b) Vancomycin
c) Ceftriaxone

133. When cultured the causative organism Enterococcus faecalis, Treatment:


a) Vancomycin.
b) Ceftriaxone.
c) Gentamicin
134. You will add:
a) Ampicillin
b) Ceftriaxone
c) Gentamicin

135. Duration of treatment:


a) 1-2 weeks
b) 4-6 weeks
c) 6-8 weeks

136. Pt. has Pharyngitis 1 month before & took ampicillin 500 mg for 7 days, now he has dental
surgery tomorrow, his doctor
should use which of the
following for prophylaxis?
a) Ampicillin
b) Amoxicillin
c) Clindamycin

137. Monitoring for


clindamycin:
a) Cardiomegaly
b) Spleenomegaly
Routine blood examinations should be done during concomitant therapy with primaquine to monitor potential
hematologic toxicities.
Periodic liver and kidney function tests and blood counts should be performed during prolonged therapy when
treating patients with severe liver disease.
As with all antibiotics, perform culture and sensitivity studies in conjunction with drug therapy.
Efficacy of clindamycin should be closely monitored in patients using concomitant St. John’s wort, a CYP3A4
inducer.
Hepatic/Biliary/Pancreatic
In patients with moderate to severe liver disease, prolongation of the half‐life of clindamycin has been found.
However, it was postulated from studies that when given every eight hours, accumulation
of clindamycin should rarely occur. Therefore, dosage reduction in liver disease is not generally considered
necessary. Periodic liver enzyme determinations should be made when treating patients with severe liver
disease.

138. Post exposure prophylaxis for meningitis: all can be given as a single dose, except?
a) Rifampin
b) Ceftriaxone
c) Ciprofloxacin
When should I give rifampicin?
● When used to prevent meningococcal infection, rifampicin is given for 2 days only.
● Give it twice each day, once in the morning and once in the evening. Ideally, these times are 10–12
hours apart, for example some time between 7 am and 8 am, and between 7 pm and 8 pm.
● You need to give rifampicin to your child when
their stomach is empty. Give it about 30 minutes
before a meal or 2 hours after a meal.
● Rifampin dosing post exposure prophylaxis:
Haemophilus influenzae type b exposure:
children≥1 month: 20 mg/kg PO once
daily×4days; maximum600 mg/day‐
N. meningitidis exposure: Children<1 month:5
mg/kg Q12HPO × 2days; maximum600 mg/day
Children≥1 month:10 mg/kgQ12H PO× 2 days;
maximum 600 mg/day

139. Nurse couldn't find folinic acid; she called the pharmacist & asked him where could she
find it under; Folinic acid is present in the market under the name of:
a) Leucovorin → Ca salt of folic acid
Counteract methotrexate (which is anti folic acid), can be taken for megaloplastic anemia, it can cause toxcity
with Leucovorin/flurouracil; must be under physician’s; el folic acid antagonist mesh hat2assar fih 3ashan
mo5talef shwayya. C/I: prencious or megaloplastic anemia
Side Note: u take folic acid ma3 el methotrexate kol youm except the day u take methotrexate

140. A new prescription for carbamazepine with the dose equals to that titrated for 3 weeks,
what do you do: Start low and go slow.
A low initial daily dosage with a gradual increase in dosage is advised. Dosage should be adjusted to the needs
of the individual patient. pmsCARBAMAZEPINE CR should be taken with meals whenever possible.
Monograph: It is advised that with all formulations of Tegretol, a gradually increasing dosage scheme is used
and this should be adjusted to suit the needs of the individual patient.
Tegretol should be taken in a number of divided doses although initially 100‐200mg once or twice daily is
recommended. This may be followed by a slow increase until the best response is obtained, often 800‐1200mg
daily. In some instances, 1600mg or even 2000mg daily may be necessary.

141. Patient has quit smoking. She drinks 3-4 large cup of coffee and he is using carbamazepine
she suffers from tremor what is the cause?
a) Increase side effect of carbamazepine
b) Side effect of caffeine (Smoking 1A2 inhibitor: reduced caffeine effect, when quit
smoking, excessive caffeine is not broken and causes tremors)
Another version: Clarythomycin with carbamazepine? Change Clarythromycin
Coadministration of CYP3A4 inhibitors as clarithromycin may increase carbamazepine plasma concentrations
and induce adverse reactions
142. Pt taking infliximab for arthritis. What we should monitor after 8 hr of injection?
a) Hypotension
b) Electrolytes
Infliximab for Injection has been associated with hypersensitivity reactions that vary in their time of onset.
Hypersensitivity reactions, which include urticaria, dyspnea, and/or bronchospasm, laryngeal edema and
hypotension, have occurred during or within 2 hours of infliximab for Injection infusion.
Infliximab has been associated with acute infusion‐related reactions, including anaphylactic shock, and
delayed hypersensitivity reactions. Acute infusion reactions including anaphylactic reactions may develop
during (within seconds) or within a few hours following infusion. If acute infusion reactions occur, the infusion
must be interrupted immediately. Emergency equipment, such as adrenaline, antihistamines, corticosteroids
and an artificial airway must be available. Patients may be pre‐treated with e.g., an antihistamine,
hydrocortisone and/or paracetamol to prevent mild and transient effects.

143. Automated dispensing system in hospital? Minimize error


Advantages of automated dispensing systems Disadvantages of automated dispensing systems
Returned medications can be easily credited by scanning The bar-code check of the filled cassette is very
the patient ID label and the medication. time consuming.
A high degree of accuracy for filled medications can be Because of the system’s high degree of accuracy, it
achieved. The system restocks itself. may soon be impossible to eliminate the pharmacist
Reduce medication errors check of machine-filled medications.
Using robotics expired drugs, a removed automatically Manual picks would still require a pharmacist’s
when the robotic arm reads the medication’s bar code check.

144. Patient has deductible of $100. He paid 63.25 of it. Insurance co-payment is 10% after and
they cover up to $10 dispensing fees. If the med costs $87.50 including $15 dispensing fee how
much do you collect from patient? answer in some sources 49 other sources 45.33
a) 8.25
b) 41.83
c) 45.33
d) 50.83
Soln: First, Calculate the remaining of deductible
100 - 63.25= 36.75 → Fees = 15 - 10 = 5 →The total price of Rx us 87.5 including both the
fees and the deductible → So, 87.5 -15-36.75 = 35.75 10% = 3.575 The total = 45.33

145. Academic detailing by pharmacists provides a service to physicians by:


a) Educating on improved prescribing legibility.
b) Advising on optimal patient interviewing techniques.
c) Recommending strategies to avoid medication wastage.
d) Providing current information on best prescribing practices.
e) Promoting the use of physician samples given to patients.
146. Azithromycin S. E? photosensitivity
Medications that cause photosensitivity
➢ Thiazides and Related Diuretics: Chlorthalidone, Hydrochlorothiazide, Metolazone
➢ Tetracyclines: Doxycycline, Minocycline
➢ Photodynamic Therapy PDT: Verteporfin, Visudyne
➢ Fluoroquinolones: Ciprofloxacin Levofloxacin Moxifloxacin
➢ Glycylcyclines: Tigecycline
➢ Sulphonamides: sulfamethoxazole/ trimethoprim
➢ St. John's wort (Hypericum perforatum)
➢ Antipsychotics: Chlorpromazine, Methotrimeprazine, Loxapine, Perphenazine, Zuclopenthixol
➢ Tretinoin Isotretinoin Coal tar
➢ Macrolides: Azithromycin Clarithromycin

147. A prescription is for 120 g of 15% urea cream. The pharmacist has 20% urea cream to be
mixed with an appropriate base. How much base is needed to compound this prescription?
30 gm
15 g 100
15 g ??? = 75 + 25 base = 100 "that we have %"

we aded 25 to get 100


we need? to get 120 = 30g

148. A patient with hypertension takes HCTZ and he is diabetic taking metformin and
glyburide, taking also statin, he is suffering from sexual dysfunction. What could be the cause?
a) HCTZ
b) Statins.
c) Metformin.
d) Glyburide.
Drugs Associated with Erectile Dysfunction, Delayed Ejaculation and Hypoactive Sexual Desire Disorder
Erectile 5-alpha reductase inhibitors acetazolamide alcohol (acute) alcohol (chronic) alpha-blockers
dysfunction anti-androgens barbiturates carbamazepine cimetidine clofibrate clonidine digoxin GnRH
analogues ketoconazole lithium MAOIs metoclopramide opioids phenothiazines phenytoin
spironolactone SSRIs thiazide diuretics tricyclic antidepressants
Delayed 5-alpha reductase inhibitors alpha-blockers
ejaculation antipsychotics SSRIs
Hypoactive 5-alpha reductase inhibitors alcohol (chronic) anti-androgens barbiturates BBs carbamazepine
sexual desire GnRH analogues ketoconazole MAOIs opioids phenytoin spironolactone SSRIs TCAs
149. Patient with HbA1c (?) and blood pressure 130/90 and LDL 3 (LDL target < 2). On
gliclazide 30mg, HCTZ 12.5mg. What to do?
a) Increase dose of gliclizide
b) Add rosuvastatin
c) Add basal insulin
d) Increase dose of antihypertensive/change (can’t remember)

150. Stroke patient. Patient BP 170/100; ischemic stroke diagnosed 2 months ago, patient on
EC-ASA. Alteplase CI because?
a) Blood pressure
b) ASA
c) Stroke
THROMBOLYTICS Absolute contraindications:
Alteplase (serine protease by recombinant DNA Pericarditis, previous intracranial hemorrhage; known
technology. Very short half-life (5 to 30 minute) malignant intracranial neoplasm, known cerebral vascular
Tenecteplase (longer half life) lesion, ischemic stroke within 3 months except acute
Reteplase (genetically engineered, smaller derivative stroke within 3 h; suspected aortic dissection; active
of recombinant Tpa) bleeding or bleeding diathesis (excluding menses);
MOA: low affinity for free plasminogen but a very significant closed head or facial trauma within 3 months.
high affinity for plasminogen bound to fibrin, act Relative contraindications:
directly/indirectly to convert plasminogen to plasmin, History of chronic severe, poorly controlled HTN, severe
uncontrolled HTN (BP >180/110 mm Hg); previous CVA
which, in turn, cleaves fibrin, thus lysing thrombi.
more than 3 months prior or known intracerebral
Give early in STEMI within 6 hours for maximum pathology not covered above; traumatic or prolonged
reduction of mortality/morbidity. (>10 min) CPR or major surgery (<3 wk);
Benefit reduced after 6 hours. noncompressible venous punctures; recent (2–4 wk)
S. E: Bleeding (fatal). Orolingual angioedema. internal bleeding; pregnancy; active peptic ulcer; current
The greatest absolute benefit is seen where the risk of use of anticoagulants.
mortality is highest (anterior MI, previous MI).

151. Past stroke/ TIA what to give for hypertensive


a) ACEI + thiazide

152. Trigmenal Neuralgia. What to use? Oxcarbazapine


1st line; Carbamazepine (more studied,so better) & Oxcarbazapine (less SE), el etnen BID sustained release law
ebtadet CBZ w 3amal effect bs SE not tolarated, u can convert to oxcarbazapine (causes hyponatremia), or
reduces the dose & add baclofen (muscle relaxant), or add other drugs (phenytoin, gabapentin, pregablin,
valproic acid, clonazepam) last resort

153. Lady came to you in the pharmacy asking about plan B after 2 days of unprotected
intercourse she was 80 kg and she told you that she used it about 3 times in the previous year,
She is taking amoxicillin now, what makes you advise her that it may not work with her
a) She took it multiple times previously
b) She is 80 kg weight
c) It has been long period after the intercourse
d) Amoxicillin use.
No interactions between amoxicillin and Plan B. AMOXICLLIN may affect the gut flora, leading to lower
estrogen reabsorption and reduced efficacy of combined oral estrogen/progesterone contraceptives.
Health Canada advisory regarding reduced effectiveness in women weighing 75– 80 kg and lack of
effectiveness in women weighing ≥80 kg, but further evidence is required. Recommend regardless of BMI.

154. Which of the following statements relating to emergency contraception is TRUE?


a) The contraceptive effectiveness of Plan B, an emergency contraceptive product, is equal
across the first 72 hours following unprotected intercourse. (95% effective if taken
within 24 hours of protected sex)
b) Copper Intrauterine Devices (IUDs) can be used for emergency contraception and
studies have shown that they are often more effective than hormonal contraceptives.
c) Emergency contraceptive pills can abort an established pregnancy if taken within 72
hours of embryo implantation or Dislodges egg from endometrium (stop egg release,
fertilization and implantation)
d) Hormonal emergency contraception has been shown to greatly increase the risk of VTE
(venous thromboembolism) and therefore should not be used in women who are already
at risk of this condition. (levonorgestrel = progesterone/ no estrogen= no VTE risk)
e) You should have your menses within 3 weeks
(Expect next period on time or possibly early. Exception: patients using Plan B® more than once a month,
menstrual changes can occur early or later than expected with lighter or heavier menstrual periods)
How effective is the IUD at
preventing pregnancy?
The emergency IUD is the
most effective method
of emergency
contraception - less than 1%
of women who use the
IUD get pregnant.
It's more effective than
the emergency pill at
preventing pregnancy after
unprotected sex
155. Patient has first gout attack. Renal function was 29ml/min. What to give?
a) Colchicine
b) Febuxostat (not in acute gout attack)
c) Prednisone 10 mg PO
Colchicine is contraindicated in patients with renal
or hepatic impairment who are also taking CYP3A4
or Pgp inhibitors. MAX dose 6mg, 1.2mg el awel
ba3den 0.6 to control SE.
Caution is advised when corticosteroids are
used in patients with renal insufficiency, acute
glomerulonephritis and chronic nephritis.

156. Patient infused Vancomycin IV 15min. Get red-man syndrome. Next initial action?
a) Report as allergy on profile
b) Give over at least 60 mins next time (diphenhydramine + wait then re-initiate)
c) Report to Canada vigilance program
TC: Infusion Reactions (red man syndrome)
Following rapid IV administration, a maculopapular or erythematous rash on the face, neck, chest and upper
extremities as well as pruritus, hypotension, angioedema, chest pain, wheezing, dyspnea, urticaria and
flushing may occur. The reaction usually begins a few minutes after infusion initiation but may not occur until
after the infusion is completed and generally resolves spontaneously over several hours after the infusion is
stopped. This anaphylactoid reaction, referred to as red man syndrome, occurs in response to vancomycin‐
induced histamine release and is not a true hypersensitivity syndrome. The likelihood of this reaction is
proportional to the rate of infusion and therefore vancomycin should be infused over a period of ≥60 minutes.
Pretreatment with an antihistamine (hydroxyzine, diphenhydramine) may help prevent or minimize a reaction.
In patients who have experienced this reaction, lengthen the time of infusion for subsequent doses. Monitor
blood pressure during infusion. Concomitant administration of anesthetic agents may increase the frequency
of infusion‐related events.

157. Patient with CAP on IV treatment nurse wants to go P.O. What is a suitable reason not to
switch to P.O.?
a) Diarrhea
b) Persistant cough
c) Chest Xray showed +ve consolidation
d) S. pneumonia positive
When to switch to P.O.: GIT Is Functioning Normal
E.G: No Vomiting, Diarrhea. Hemodynamically Stable
E.G: BP. 2 Temp Readings R Normal (Less Than 37.5)
Over A Period Of 16hrs. Able to Consume Oral
Medication. Normalized While Blood Cell Count.
Subjective Improving in Cough & SOB.
158. Buddy has chronic alcoholism. Has hypertension, diabetes, insomnia & osteoarthritis.
Started on digoxin, also on APAP and HCTZ. Started feeling nausea, diarrhea and tremors.
What caused the toxicity except OR all cause tremors except?
a) Alcohol use (Didn’t say he drank alcohol jus he has chornic alcoholism)
b) Digoxin excessive slowing of pulse, arrythmeia, AV block
c) HCTZ
d) Acetaminophen
Another version: Patient got tremor might due to which condition?
a) Hypertension
b) Diabetes
Conditions associated with tremors: MS, stroke, neurodegenerative diseases (that damage or destory parts of
the brain), alcohol abuse/withdrawal, mercury poisoning, thyroid & liver failure.
Drugs associated with tremors: CS
Chronic alcoholism = drinking large volumes of ETOH daily.
We agreed that chronic ETOH is a CYP 2E1 INDUCER & can increase acetaminophen toxicity by increasing the
yield of its oxidative metabolite (NAPQI)
HCTZ can increase the risk of digoxin toxicity by inducing hypokalemia
N/V is a shared overdose sign in both toxicities
What causes tremors to develop?
Tremors can be caused by a variety of things, including: prescription medications, diseases, injuries & caffeine
The most common causes of tremors are: muscle fatigue, ingesting too much caffeine, stress, aging, low
blood sugar levels
Medical conditions that can cause tremors include: stroke, traumatic brain injury, Parkinson’s disease, which is
a degenerative disease caused by loss of dopamine‐producing brain cells, multiple sclerosis, which is a
condition in which your immune system attacks your brain and spinal cord, alcoholism, hyperthyroidism,
which is a condition in which your body produces too much thyroid hormone
Treatment:
 Beta‐blockers (Propranolol) are usually used to treat high blood pressure or heart disease. However,
they have been shown to reduce tremors in some people.
 Tranquilizers, such as alprazolam (Xanax), may relieve tremors that are triggered by anxiety.
 Anti‐seizure medications are sometimes prescribed for people who can’t take beta‐blockers or who
have tremors that are not helped by beta‐blockers.

160. SJS is caused by? Lamotrigine


SASPAN (Sulfonamide AB, Allupurinol, pencillin, anti‐epileptics, NSAID)
161. SG Pregnant lady admitted in hospital for labour
pain and given meperidine and gone trough c-section.
(No information given about oxycodone use and
epidural injection). After birth she is on bed rest for 3
days and develop creepy crawly and nagging
sensation in legs, doctor identified as symptops of
RLS. Q what is the cause of RSL in this patient?
a) Meperidine
b) Pregnancy
c) Bed rest for three days.
d) Oxycodone before labore
e) Epidural injection
Notes:
1) one case, we have "nagging sensation and a crawling feeling
in the legs" = RLS, and the #1 risk is rest and sleep. Although
pregnancy is a risk factor for RLS, here, the woman already gave birth.

2) in the other case, we have "restless legs and muscle twitches" = myoclonus = drug overdose, and here, the
suspect is meperidine due to its neuroexcitatory metabolite (normeperidine).
Importantly, RECALL that in RLS, one does NOT see any cramps, spasms, tremor or twitches; it's merely an
"awful feeling of a vibration in the legs".

162. Now doctor want to start treatment for RLS what is your recommendation?
a) Pramipexole (dopamine agonist)
b) Lamotrigin
c) Carbamazepine
d) Amytriptilyne

163. Doctor started therapy of RLS as pharmacist where you document this therapy.
a) In ward chart
b) In computerized pt file that pt started on RLS treatement
c) Talk to nurse
d) Patient profile in ward.
164. After discharge what non pharmacological treatment you suggest to this patient
a) Increase physical activity (exercise moderately)
b) Hot bath
c) Drink coffee (avoid caffeine)
d) Bed rest.
Nonpharmacologic Choices. While pharmacologic therapy is needed for moderate to severe symptoms,
nonpharmacologic measures may be useful in milder cases:
● Engage in mental alertness activities (playing cards or video games or doing crossword
puzzles) to reduce symptoms during times of boredom.
● Abstain from alcohol, caffeine and nicotine.
● Take hot baths, stretch and exercise moderately.
● Discontinue medications that may be contributing to symptoms, e.g., antidepressants,
antipsychotics, dopamine‐blocking antiemetics and sedating antihistamines.
● Minimize aggravating factors such as sleep deprivation.
● In patients with RLS and varicose veins, consider sclerotherapy to improve RLS symptoms.

165. Best place to look which antipsychotic causes least weight gain?
a) RxFiles
b) e-CPS

166. Risk of torsades de pointes or Which one cause arrhythmia? Hypokalemia


Drugs associated with prolonged QTc interval/torsades de pointes, such as: amiodarone, azithromycin,
clarithromycin, domperidone, erythromycin, haloperidol, methadone, pimozide, quinine, sotalol, ziprasidone.
Additional risk factors for torsade de pointes in the general population include:
● female gender; age 65 years or older; family history of sudden cardiac death at <50 years;
● baseline prolongation of the QT/QTc interval;
● presence of genetic variants affecting cardiac ion channels or regulatory proteins;
● cardiac disease (e.g., MI, left ventricular hypertrophy, cardiomyopathy, conduction system disease);
history of arrhythmias (especially ventricular arrhythmias, atrial fibrillation, or recent conversion from
atrial fibrillation); bradycardia (<50 beats/minute);
● acute neurological events (e.g., intracranial or subarachnoid haemorrhage, stroke, intracranial trauma);
● nutritional deficits (e.g., eating disorders, extreme diets);
● diabetes mellitus; autonomic neuropathy.
The most common causes of acquired long QT syndrome are medications and electrolyte disorders
(eg, hypokalemia, hypomagnesemia).

167. Which provincial insurance plan thing takes away doctors’ autonomy most?
a) Therapeutic formulary/ Therapeutic interchangeability
b) Max day of supply that they pay for
c) Don’t pay vacation supply over some x days
168. which one needs urgent referral
a) Cough with ACEi (type B reaction)
b) Orthostatic hypotension with Domperidone
Domperidone is a proposed treatment of orthostatic hypotension (OH) in Parkinson's disease (PD).

169. Patient on PPI – patient at risk of


a) Hip fracture hip, wrist or spine
b) ONJ Osteonecrosis of the jaw associated with bisphosphonates
Monograph: PPIs, especially if used in high doses and over long durations (>1 year), may modestly increase the
risk of hip, wrist and spine fracture, predominantly in the elderly or in presence of other recognised risk
factors. Observational studies suggest that proton pump inhibitors may increase the overall risk of fracture by
10–40%. Some of this increase may be due to other risk factors. Patients at risk of osteoporosis should receive
care according to current clinical guidelines and they should have an adequate intake of vit D and calcium.

170. Patient hyponatremic – risk of? Seizure


Excessive drinking of water during exercise can cause hyponatremia (blood sodium <135 mmol/L).
Signs of hyponatremia include: weight/fluid gain, mental confusion & general weakness.
Hyponatremia can lead to seizures, coma and even death. Recreational athletes have been identified as being
at risk of over‐hydration and hyponatremia more than competitive athletes. To prevent over‐hydrating during
an event, recreational athletes are encouraged to drink only enough water to quench thirst.

171. Which study shows surrogate endpoint? which is effect of specific ttt
a) Decrease incidence of AIDS using HIV drugs
b) Decrease of AF using Dabigatran (may be this, not sure though)

172. Make 1L of 0.25%w/v. How mg many in 30ml?


0.25g → 100 ml ? → 30 ml = 0.075g X 1000 = 75mg
173. Want 144mmol/L of Na (or NaCl?). Have 110mmol/L. How much of 3%NaCL needs to be
added to 500ml? (0.9%NaCl has 154mmol/L)

174. Patient injecting SC Dalteparin.


a) Prior to injecting needle, pulled up plunger to remove any trapped air bubbles
b) Prior to injecting, do not remove any trapped air bubbbles because of exact dose
c) Inject it to 45’ angle in thigh
Getting Gather your supplies: heparin, needles, syringes, alcohol wipes, medicine record, and container.
Ready If you have a pre-filled syringe, make sure you have the right medicine at the right dose.
Do not remove the air bubbles unless you have too much medicine in the syringe.
Filling the Wash your hands with soap and water, and dry them well.
Syringe Check the heparin bottle label. Make sure it is the right medicine and that it has not expired.
If it has a plastic cover, take it off. Roll the bottle between your hands to mix it. Do not shake it.
Wipe the top of the bottle with an alcohol wipe. Let it dry. Do not blow on it.
Know the dose of heparin you want. Take the cap off the needle, being careful not to touch the needle to keep it sterile.
Pull back the plunger of the syringe to put as much air in the syringe as the dose of medicine you want.
Put the needle into and through the rubber top of the heparin bottle. Push the plunger so the air goes into the bottle.
Keep the needle in the bottle and turn the bottle upside down.
With the needle tip in liquid, pull back on the plunger to get the right dose into the syringe.
Check the syringe for air bubbles. If there are bubbles, hold both the bottle and syringe in one hand, and tap the syringe
with your other hand. The bubbles will float to the top. Push the bubbles back into the heparin bottle, then pull back to
get the right dose.
When there are no bubbles, take the syringe out of the bottle. Put the syringe down carefully so the needle does not
touch anything. If you are not going to give the shot right away, carefully put the cover over the needle. If the needle
bends, do not straighten it. Get a new syringe.
Giving the Wash your hands with soap and water. Dry them well.
Shot Choose where to give the shot. Keep a chart of places you have used, so you do not put the heparin in the same place all
the time. Ask your provider for a chart.
Keep your shots 1 inch (2.5 centimeters) away from scars and 2 inches (5 centimeters) away from your navel. Do not
put a shot in a spot that is bruised, swollen, or tender.
The site you choose for the injection should be clean and dry. If your skin is visibly dirty, clean it with soap and water.
Or use an alcohol wipe. Allow the skin to dry before giving the shot.
The heparin needs to go into the fat layer under the skin.
Pinch the skin and put the needle in at a 45º angle.
Push the needle all the way into the skin. Let go of the pinched skin. Inject the heparin slowly and steadily until it is all
in.
After all the medicine is in, leave the needle in for 5 seconds. Pull the needle out at the same angle it went in. Put the
syringe down and press the shot site with a piece of gauze for a few seconds. Do not rub. If it bleeds or oozes, hold it
longer.
Throw away the needle and syringe in a safe hard container (sharps container). Close the container, and keep it safely
away from children and animals. Never reuse needles or syringes.
Write down the date, time, and place on the body where you put the injection.

175. Patient took hydromorphone caused nausea and got morphine instead now.
a) Tell patient to bring back hydromorphone to pharmacy to destroy
176. Patient given CPZ chlorpromazine IM 50mg for intractable hiccups. What is the issue?
a) High dose
b) Wrong indication
c) Wrong dosage form (should be IV or oral- not IM)
CPZ 1st Gen antipsychotic, intractable hiccups more than a month, persistent hiccups upto a month. DOC: IV
CPZ over 30'‐ 1hr
Parenteral: 25–50 mg IV over 30–60 min. Oral: 25–50 mgTID– QID PO × 2–3 days

177. All types of dosage form can be crushed and put in nasogastric tube except:
a) Suspension
b) Crushed EC-coated (or sustained release)
c) Powder of gelatin capsule
d) Film coated
Nasogastric tube is a tube through the nose; u can give film & sugar‐coated tabs
Enteral Nutrition (EN), tube feeding, is given via different types of tubes. One type of tube feeding can be
given via a tube placed down through the nose into the stomach or bowel, known as Nasoenteric Feeding
and includes naso gastric (NG), naso duodenal and naso jejunal (NJ) feeding.

178. Best way to prevent or handle forgery?


a) Give only few tabs (or whatever) until you can confirm the Rx
b) Report to OCP and local police
c) Keep less inventory
d) Forgot other option
6 steps;
1‐use delay tactic
2‐dont physically restrain the suspect
3‐notify the police
4‐optional: fax a copy of the forgery report to OCP
5‐preserve the evidence (retain the RX if possible)
6‐advise the physcian

179. Doctor handwrites .5mg Clonazepam. Patient given 5mg. What is the most appropriate
reasoning for the error?
a) Missing leading zero/naked decimal point → ISMP should be 0.5mg
b) Doctor handwrites
c) Not know about the dosage (Clonazepam almost rare/never given as 5mg??)
180. Patient having Bipolar I disorder on Topiramate?
a) Change to Lithium
Bipolar I: history of 1 clear manic episode with/without episodes
hypomania or depression
Bipolar II: history of hypomanic & major depression disorder
DOC for both: Li. Topiramate can’t be used

181. Isotretinoin giving 30mg OD for first month then increases


to 1mg/kg/day. Patient weighs 60kg. What to give after first
month? Cumulative dose 140-150mg/kg
a) 30mg BID for 4 months
b) 30mg BID for 5 months
c) 60mg BID for 4 months
d) 60mg BID for 5 months
Minimum cumulative dose = 120mg * 60Kg =7200 mg
taken dose = 30mg/daily *28 days = 840 mg
So, the dose should be taken = 7200 ‐ 840 = 6360 mg
If patient takes 30 mg BID = 30mg *2 = 60 mg Daily
No. of days = 6360 / 60 = 106 days = 3.5 months
As this is the minimum cumulative dose. So, the dose should be used for more than 3.5 month the answer is
30 mg BID for 4 months
Monograph: Isotretinoin therapy should be started at a dose of 0.5 mg/kg daily. The therapeutic response to
isotretinoin and some of the adverse effects are dose‐related and vary between patients. This necessitates
individual dosage adjustment during therapy. For most patients, the dose ranges from 0.5‐1.0 mg/kg per day.
Long‐term remission and relapse rates are more closely related to the total dose administered than to either
duration of treatment or daily dose. It has been shown that no substantial additional benefit is to be expected
beyond a cumulative treatment dose of 120‐150 mg/kg. The duration of treatment will depend on the
individual daily dose. A treatment course of 16‐24 weeks is normally sufficient to achieve remission.
In the majority of patients, complete clearing of the acne is obtained with a single treatment course. In the
event of a definite relapse a further course of isotretinoin therapy may be considered using the same daily
dose and cumulative treatment dose. As further improvement of the acne can be observed up to 8 weeks
after discontinuation of treatment, a further course of treatment should not be considered until at least this
period has elapsed.
CTMA: Conventional dosing of isotretinoin is 0.5–2 mg/kg/day for 12–16 weeks. Initiating with a low dose and
titrating will minimize transient exacerbation of acne at the start of therapy. Some patients experience a
relapse of acne after the 1st course of isotretinoin: a 2nd course may be used starting at least 8 weeks after
the end of the 1st course (as acne may continue to improve during this time). Studies have shown that other
dosing regimens (low‐dose, intermittent) may also be effective.
182. Pharmacist notices better drug for patient. Calls doc didn’t pick up or return fax. What to
do to improve collaborative something with doc
a) Switch drug and send doc a note about the change
b) Meet doc and create a communication rapport and something else
c) Give patient note to take to doc

183. A mother of a child 5 years old, came to you after her child took influenza vaccine by 2
days and told you that her child is experiencing oculorespiratory symptoms and he is not eating
well in for 5 days and he got pale, what is the reason for these symptoms to occur?
a) The child is dehydrated.
b) He is small.
c) He has allergy from the vaccine.
d) It is a side effect of the vaccine.
CTC: Oculorespiratory syndrome (ORS) was initially identified during the 2000–2001 influenza season and has
been observed, though rarely, in subsequent seasons. It is characterized by the presence of bilateral red eyes
plus 1 or more respiratory symptoms (cough, wheeze, chest tightness, difficulty breathing, sore throat), with
or without facial swelling, that occurs within 24 hours of receiving influenza vaccination and resolves within 48
hours of symptom onset. ORS is not considered an allergic reaction and is not a contraindication to
vaccination. Individuals having experienced ORS without mild to moderate lower respiratory symptoms can
safely receive influenza vaccine in subsequent seasons. Those who have had ORS with lower respiratory
symptoms should be referred to the local Medical Officer of Health and assessed prior to vaccination.

184. ARMD patient smoker wants VitC, E, Cu, Zn and VitA


a) Give but without VitA
b) Give but replace VitA with Lutein
ARMD: Age related macular degenration (eye condition), 2 types dry (white yellow spots in the central retina)
or wet (severe vision loss), if he's not a smoker, u give all of them 3ady.
Beta‐carotene–containing formulations are no longer recommended for prevention of AMD
progression because of an increased risk of lung cancer. Lutein and zeaxanthin may be a suitable replacement
for beta‐carotene in the original AREDS
formulation.

185. Women with depression


symptoms. You refer because?
a) Worsening symptoms for
past 3 weeks
b) She had the same symptoms 2
years ago
186. Effectiveness of nicotine gum?
a) 5%
b) 25%
c) 45%

187. A patient taking nicotine replacement therapy for smoking


cessation. What is the side effect of nicotine overdose?
a) Diarrhea (GI disturbances)
b) Irritability.
c) Insomnia

188. Patient being well controlled with oral meds for T2D. When should he be given self-
monitoring blood glucose machine in T2D?
a) If insurance covers
b) Patient non-adherent to meds
c) Patient request (more to this option with specific wording)
Self monitoring For patients treated with insulin, individualize self-monitoring strategies and take both pre-
of blood glucose and postprandial measurements.
(SMBG) A minimum of 3 measurements per day should be performed for those on basal-bolus
insulin regimens.
No benefits for those who are being managed on oral antihyperglycemics or lifestyle alone.
For medications that carry a higher risk of hypoglycemia (e.g., glyburide, insulin), or in
patients with risk factors for hypoglycemia (e.g., advanced age, renal dysfunction, multiple
comorbidities, hypoglycemia unawareness), SMBG is useful to ensure safety.
189. Pregnant patient has had NV. Over the past few days gotten really bad to point where she
couldn’t go work. What to do?
a) Call doc and request for Rx for Diclectin
b) Refer to emergency
c) Forgot other options
1st: Diclectin 4tabs daily; 2 HS, 1 AM, 1 afternoon (adjust acc to severity)

190. Pantoprazole interaction acts with?


a) Clopidogrel
b) Levothyroxine
c) Rilpivirine
Co‐administration is contraindicated due to significant decreases in rilpivirine exposure and loss of
therapeutic effect
191. Patient with asymptomatic hyperkalemia.
Tried diet restriction didn’t work. How to
treat?
a) Sodium Polyesterene
b) Insulin with glucose

192. Most cause of drug shortage?


a) Generic price increase
b) Quality assurance requirements
increase #1
c) Doctors are not prescribing particular
drug
d) Recall of some batches
e) Not profit to mfg
f) Increase demand with not enough
supply
g) Raw material shortage #2

193. Manufacturer tells you iron sucrose production decreases; how would you conserve stock?
a) Automatically sub for iron dextran on orders
b) Take whatever is the on the wards and put on pharmacy shelves
c) Call other pharmacies/hospitals nearby and obtain their stock

194. A patient with HPT, Raynaud, DM, solitary kidney, what is the treatment for HPT?
a) ACE
b) HCTZ
c) Amlodipine
d) Metoprolol

195. Osteomyelitis case: Patient with uncontrolled diabetes, leg swelling and inflamed red spot
and Wet. (macerated) what could be the causative organism of these symptoms:
a) Pseudomonas aeruginosa.
In previous case what is the ttt of case
a- Amox/clav
b- Ceftriaxone
c- Pipracillin/ Tazobactam
d- Penicillin
Empiric treatment targeting Pseudomonas aeruginosa is generally unnecessary unless risk factors present, e.g.,
history of foot soaking, severe or chronic infection
196. Treatment of Folliculitis:
a) Ceftazidime inj (if osteo-pseudomonas)
b) Ciprofloxacin oral (if Pseudomonas folliculitis)
c) Cefazolin (is diabetic foot –staph infection)
Fluoroquinolones play little role in treating common bacterial skin infections unless gram‐negative organisms
are suspected. They may play a limited role in the treatment of community acquired MRSA infections,
depending on susceptibility patterns. Ciprofloxacin can be used to treat Pseudomonas or so called “hot‐tub”
folliculitis in select patients with extensive or symptomatic involvement, keeping in mind the typically self‐
limited course in untreated patients.
Mupirocin’s mechanism of action is not shared by other topical antibiotics and thus resistance has not been an
issue to date with the exception of mupirocin‐resistant MRSA strains. Resistance rates to mupirocin in MRSA
are increasing and linked with prior exposure to the drug. Topical Mupirocin is particularly useful for localized
impetigo and folliculitis caused by S. aureus.
197. Patient with sinusitis the pt has penicillin allergy, taking clarithromycin 500 mg b.i.d. for
10 days. There is no improvement, what is your concern about the Rx
a) There is stronger treatment than clarithromycin
b) Too long duration.
c) Wrong TTT
d) Wrong duration
e) Too small dose
In case of B‐lactam allergy:
Doxycycline 5–7 days, Macrloides:
Poor activity against H. influenzae.
Significant macrolide resistance in
S. pneumoniae and Streptococcus
pyogenes.

198. What is the adult duration of treatment of sinusitis?


Antibiotic therapy should be reserved for those patients with bacterial sinusitis as defined by history, including
duration of symptoms >10 days, and physical examination. When an antibiotic is indicated, amoxicillin remains
the antibiotic of first choice for acute bacterial sinusitis
If first‐line therapy has failed, choose a different antibiotic and treat for 5–10 days with the new agent
Therapeutic Tips:
● Most episodes of acute rhinosinusitis resolve without antibiotic therapy.
● In adults with uncomplicated acute rhinosinusitis, a treatment duration of 5–7 days is recommended.
● For children, data on shorter durations of therapy are lacking; therefore, it is still recommended to
treat for 10 days.
● Inform patients that improvement may take up to 7 days and complete resolution of symptoms may
take up to 14 days.
● If the patient deteriorates at any time, reassess for acute complications, other diagnoses and/or
adherence to treatment.
● Routine follow‐up on completion of therapy in asymptomatic patients is not required.
● If a patient has received antibiotics (especially macrolides or fluoroquinolones) within the past 3
months, choose a different antibiotic/antibiotic class to treat acute bacterial rhinosinusitis as there is a
higher risk of multidrug‐resistant S. pneumoniae.
Refer to ear, nose and throat (ENT) specialist if a patient:
1‐ fails second‐line therapy
2‐ experiences 4 or more episodes of bacterial rhinosinusitis per year
3‐ has anatomic anomalies
4‐ develops complications
199. Acute otitis media: patient with AOM, after treatment, a lot of discharge, and deafness
because of the discharge, what is the appropriate:
a) Refer to physician
b) Give ciprofloxacin
CTC‐ Referral:
● for treatment failures or recurrences unresponsive to therapy, consider referral to an ENT specialist
who can obtain middle ear fluid for culture to identify the pathogen involved and its antibiotic
susceptibility profile
● refer children with frequent, recurrent episodes of AOM (≥3 episodes in 6 months or ≥4 episodes in 12
months) to an ENT specialist for consideration of myringotomy and tympanostomy tubes
● refer children with recurrent episodes of AOM for an audiology assessment to monitor for any
conductive hearing loss.

200. Patient with recurrent acute otitis media:


Drug of choice:
a) High dose amoxicillin/clavulanate
b) Ceftriaxone for 3 days
c) Cefprozil

201. A patient after an accident what to give as


empiric antibiotic:
a) Piperacillin/tazobactam
b) Cefazolin + gentamicin

201. Woman with metastatic cancer takes chemotherapy and dexamethasone plus ondansetron
for chemotherapy induced nausea and vomiting. She feels nauseated many days to the extent
that she can’t drive to the center of the chemotherapy (Anticipated Nausea). What do give her:
a) Diphenhydramine
b) Domperidone
c) Metoclopramide
d) Lorazepam.

202. After the 2nd chemotherapy dose, she got severe neutropenia and fever what to give her:
a) Piperacillin/tazobactam (broad spectrum antibiotic)
b) Ceftriaxone
c) Ciprofloxacin
d) Vancomycin
203. What is to monitor with ciprofloxacin
a) Vestibular function
b) Ocular function
c) Creatinine clearance
d) ALT and AST
Hepatobiliary system
Cases of hepatic necrosis and life‐threatening hepatic failure have been reported with ciprofloxacin. In the
event of any signs and symptoms of hepatic disease (such as anorexia, jaundice, dark urine, pruritus, or tender
abdomen), treatment should be discontinued.
SE: Increase in transaminases, Increased bilirubin

204. Patient taking tobramycin, what to monitor:


a) Vestibular function (aminoglycosides: ototoxicity)
b) Hepatotoxicity
c) Renal function

205. Dose adjustment with renal failure, all except:


a) Tetracycline.
b) Clarithromycin.
c) Linezolid (used for MRSA instead of Vancomycin in case of renal failure)

206. Peripheral venous catheters can help in all these except?


a) Multiple injection for many drugs be given same time
b) Help people doing activities like hokey, tennis
c) Helpful for people who need drugs for short time less then a week

207. Upon discharge of a patient with


CAP, what to monitor except:
a) Heart rate
b) Respiratory rate
c) Oxygen saturation
d) White blood cells.
e) chest X-ray

208. When do we step down antibiotic in CAP:


a) When white blood cell within normal range.
b) Self-monitoring of body function, temperature, and the patient is getting well.
209. Pt. got MI and got hospitalized and everything is good now, all can be monitored after
discharge except
a) Troponin (first 72 hours only)
b) Creatinine kinase
c) Recurrent chest pain
d) Heart rate.
e) CBC
f) Epoxidase enzyme

210. You discovered that Dukoral vaccine left out of refrigerator for 3 days, what you will do.
a) Call the manufacture.
b) Just return it to the fridge
c) Discard it in a safe way.
d) See what is the cause of this to happen
e) Ask who make this and take disciplinary action with him
Store in the refrigerator at 2° to 8°C (35° to 46°F). Do not freeze. The vaccine can be
stored at room temperature (up to 25°C) for up to two weeks on one occasion only. After
mixing with the buffer solution the vaccine should be consumed within 2 hours.

211. Preferred dose delivery in hospital:


a) Unit dose system
b) Porter delivery system
c) Ward stock system

212. Question about a study done on a new pain killer that the physician thinks it's great, but
when u check the statistical data of the study done on this medication compared to iboprofen
600 mg tid, u find that the p-value > 0.05 and the 95 % CI is 0.7 - 1.5, so u conclude that
a) There drug is more effective than iboprfen
b) The NSAID is similar in effect to the new medication
c) There's a significant difference between the two medication
d) The medication is ineffective
e) There's no significance between them
If range of confidence interval crosses 1, it's not
significant. Range is already given 0.7‐1.5.
If range is before 1 (0.7‐0.9), the drug is more
effective, if after 1 the placebo is more effective.
If range was 0.7‐0.9 then ans would be B
213. Vancomycin calculation, A drug is given at the dose of 1g IV Q12H to a patient. The
desired therapeutic level of the drug in the blood is 15 – 20 mg/L. When blood drawn just
before the next dose, the concentration was found 10.57 mg/L. What is the appropriate dose of
the drug so that the desired therapeutic level is maintained?
a) 1125 mg IV Q12H
b) 1250 mg IV Q12H
c) 1500 mg IV Q12H
d) 1750 mg IV Q12H
e) 2500 mg IV Q12H

214. Calculation: gentamicin and gentamicin sulfate. 1 mg = 590 mcg gentamicin sulfate, you
want a solution 0.3%. How much in 5 ml.
a) 24.5 mg.

215. College of Pharmacy is governed by:


a) Provincial regulatory authority (check relation between the two).

216. The patient taking risedronate 5 mg, by mistake the pharmacist dispensed risedronate 30
mg. Error is caused by: No checking the indication
a) No reviewing the prescription.
b) Tallman letter
c) Handwriting prescription

217. Black hypertensive patient taking HCTZ, what to give:


a) ACEIs. (no ACEI in black Americans)

218. Atypical antipsychotic has a metabolic disorder 3 times in the first year, what to monitor:
a) Glucose.
b) Lipid profile.
c) Weight gain.

219. A patient with otitis externa, taking polymyxin plus gramicidin, what to counsel:
a) Use at least 4 times per day.
b) Use 2 days more after treatment.
When administering topical treatment, enough liquid to fill the canal (3–4 drops) should be instilled 3–4 times
daily (most products except fluoroquinolones, Twice‐daily dosing.).
Monitoring of Therapy:
Symptoms should be significantly reduced within 48–72 hours of therapy and completely resolved in one week
for most patients. Occasionally up to 14 days of treatment is needed. Follow up with the patient in 3–5 days to
ensure symptoms are improving and at the end of treatment to ensure resolution. If symptoms worsen or do
not resolve, consider the following: medication reaction (contact dermatitis); development of a
superinfection; incorrect diagnosis; improper or infrequent use of ear drops; inadequate penetration of topical
agents due to debris or narrowing of the canal; immunosuppression or malignant otitis externa; or organism
notsusceptible to topical agent selected. Assessment for further treatment will be required.

220. A 24-year- old woman (height 64 inches, weight 160 lb) with T1DM is very conscientious
about her image and is deeply concerned about gaining weight. Her A1C is 8.2%, and her
physician keeps increasing her insulin doses. Other laboratory values are normal except for
LDL, which is 140 mg/dL. She takes no drugs except insulin and an occasional antibiotic for
urinary tract infections. She confesses to you that she doesn’t take her full insulin doses and
asks if there is anything else, she can do to help improve control and lose weight. Which one of
the following is best to add to this patient’s regimen for its
insulin sparing and weight reduction benefits?
a) Sitagliptin 100 mg daily
b) Colesevelam titrated up to 3.75 gm daily
c) Dapagliflozin 5 mg daily
d) Liraglutide titrated up to 1.8 mg daily

221. A question about AeroChamber and vented syringe use.

222. What is the drug therapy problem between bupropion and


fluvoxamine.
a) It does not cause serotonergic syndrome.
Bupropion may enhance the adverse or toxic effect of fluvoxamine LEXI

223. A girl called you in the pharmacy tell you about her friend having insomnia for 3 days and
started seeing things moving towards her, what to tell her:
a) Seek medical attention.

224. You hold a meeting in your pharmacy about diabetic patient self monitoring, what would
you talk about:
a) Diabetic device and strips use.

225. Who can help you in that day: Nurse from the government.
226. A lady got white spots in her upper back she made a tan but all her body stained except
these spots, what is the possible cause of this spots
a) Psoriasis
b) Tinea versicolour (Multiple white pink to brown macules with an overlying fine
scale.) – Yeast infection
c) Acne
d) Rash
Pityriasis versicolor is an infection of the stratum corneum of the skin where sebaceous glands are present,
especially the upper trunk. Since the term tinea refers to diseases caused by dermatophytes, the preferred
term for this infection, which is caused by yeast (and not dermatophytes), is pityriasis (meaning scaling).
Malassezia species (formerly called Pityrosporum orbiculare or Pityrosporum ovale) normally colonize the skin
but cause an opportunistic infection in association with hereditary factors, immunodeficiency, malnutrition,
oily skin, hyperhidrosisTre or use of corticosteroids or oral contraceptives. It affects about 3% of the general
population and occurs most commonly in postpubertal adults and in warm, humid climates.
The term versicolor denotes a variety of colours or changing colours.
The most common presentation is multiple white to reddish‐brown macules that may coalesce to form large
patches of various colours ranging from white to tan. A fine scale is apparent when scratched. The lesions tend
to be darker than the surrounding skin in fair‐skinned patients and lighter in dark‐skinned patients. This is
primarily a cosmetic problem where the lesions do not tan along with the surrounding normal skin.
Recurrence rates are as high as 60–80%. It is not considered contagious and is not due to poor hygiene.
Treatment: topical azoles (clotrimazole, ketoconazole, miconazole) and selenium sulfide 2.5% suspension.

227. All of these drugs require increase the interval in renal failure except
a) Aminoglycosides
b) Flouroquinolones
c) Azithromycin
d) Penecillin

228. Pt came to you he complains form white lesion in his mouth with 7 mm depth, it is a
painful from 3 days, asking you what to do?
a) Refer to dr
b) Don’t use any it will be healed
c) Give him topical anesthetic to relief pain
d) Give him antiviral
Red Flags: Fever (systemic illness): more than 5 sors, greater than 1 cm in diameter, for more than 14 days
lesion(s), on throat, on the roof of mouth or on the gums, UC/CD (IBD colitis Pt.), Immunocompromised (HIV):
REFER (may be the question stem missed one of these flags)
229. Pt came to you with inflamed eye with discharge what could you advise him
a) Cover it to prevent spread of infection (never cover the eye unless surgery or loss)
b) Take AB for 48 hrs then go to Dr

230. Pt with CD4 count < 200 what should he take ttt for prophylaxis
a) Toxoplasma
b) T.B
c) PJP

231. This pt has sulfa allergy what should he take


a) SMX/TMP
b) Pentamidine
c) Atovaquine
Treatment of choice is 21 days of oral or IV sulfamethoxazole/ trimethoprim (SMX/TMP, cotrimoxazole) (see
Table 6). For severe disease in patients intolerant to SMX/TMP, IV pentamidine may be given for 21 days. Oral
dapsone plus trimethoprim is better tolerated than SMX/TMP, but is suitable only for mild or moderate
disease (there is no IV formulation). Other alternatives include oral atovaquone (for mild to moderate cases)
and clindamycin plus primaquine. The G6PD level should be checked prior to therapy with primaquine or
dapsone. In patients with severe PCP, the addition of oral prednisone decreases morbidity and the side effects
of SMX/TMP (rash, hyperkalemia). Patients with PaO2 <70 mm Hg on room air or an alveolar‐arterial O2
gradient >35 mm Hg should receive adjunctive prednisone 40 mg twice daily for 5 days, then 20 mg twice daily
for 5 days, then 20 mg daily to complete 21 days of treatment.
232. Pt CD4 count <50 what should he got
prophylaxis from
a) Toxoplasma
b) T.B
c) Pneumonia
d) Mycobacterium avium

233. Lady go to the dr complaining from rigidity in her hand while she was driving the dr check
his medical condition to see what will be the possible cause of thes symptomss he found that
she had DM, insomnia, HTN and she drinks alcohol. Which of these diseases could cause the
syptoms?
a) HTN
b) DM
c) ALC (Chronic alcoholism and alcohol-induced neuropathy are the causes
and/or possible overdrinking.)
d) Insomnia

234. In previous case after that she came to you, told you that in the previous 3 wks her status
was worsening, she can’t sleep. you referred her to the dr bec.
a) The Sxs have worsen
b) She can’t sleep
c) She looks like pale

235. A husband came to you, told you that his wife which is a five-week pregnancy has
vomiting, in the previous 2 days she can’t go to work, what would be the most ethical action he
will take
a) Told him that this is normal in first 3 months
b) Give him OTC for vomiting
c) Call the dr to send you a declictin Rx by fax
d) Refer to ER
Since dimenhydrinate is recommended for augmenting pyridoxine treatment in pregnancy, it can be
considered for intermittent therapy of motion sickness in pregnant patients. Promethazine may also be used.

236. Pt has anemia, he found that MCV is high what would be the cause of anemia, the pt is
vegetarian
a) Iron deficiency
b) Megaloplastic anemia
237. What will the treatment?
a) Vit. B12
b) Erythropiotine
c) Folic acid
d) Iron IV

238. Which of the following condition require vit D supplement?


a) Kidney disease
b) Fat restricted diet
c) Diabetes militus
d) Depression

239. Which of the following is wrong about using varenicline for smoking cessation?
a) It should be taken with food to avoid nauseating side effect
b) It is possible to smoke in the first days
c) It can be used in uncontrolled HTN
d) It should be started 150 mg once daily the 150 mg twice daily
0.5 mg daily PO for 3 days then BID for 4 days then 0.5–1 mg BID PO for 12 wk. Patient should quit smoking 1–
2 wk after starting varenicline. If patient is still smoking 4 wk after starting, reassess therapy. Can be continued
for an additional 12 wk if patient has benefited. If 1 mg BID is not tolerated, can reduce to 0.5 mg BID. No
tapering necessary when discontinuing

240. There is a large outbreak of influenza in your province. The commercially available
oseltamivir powder for oral suspension is unavailable from the manufacturer due to increased
demand. You receive a prescription for oseltamivir suspension for a 15 kg, 13-month-old child.
What dose would you give to treat a 15 kg child to treat influenza?
a) 7.5 mg BID
b) 15 mg BID
c) 30 mg BID (≥1 y and ≤15 kg: 30 mg BID PO × 5 days = treatment)
d) 45 mg BID
Recommended Dose—Treatment of Influenza
241. Oseltamavir should be monitored with?
a) Elderly patient
b) Patient with seizure
People with the flu, particularly children and adolescents, may be at an increased risk of seizures, confusion,
delirium, hallucinations, agitation, anxiety or other abnormal behaviour early during their illness. These
events may occur shortly after beginning TAMIFLU or may occur when flu is not treated. These events are
uncommon but may result in self‐injury to the patient, sometimes fatal. Therefore, patients should be
observed for signs of unusual behaviour and a healthcare professional should be contacted immediately if
the patient shows any signs of unusual behaviour.

242. Vaginal fishy smell


Bacterial Characterized by fishy odour, grey or milky, thin, creamy or The most common cause of
vaginosis copious discharge. vaginitis.
No inflammation or Pruritus The drug of choice PO/pv
Fishy odour often intensified after addition of 10% (KOH). Clindamycin or Po/pv
Vaginal pH >4.5 (5-6), +ve "Whiff" test, -ve PMN metronidazole

243. Carbamezapine causes SJS – which gene is involved?


a) HLA-B*1502
b) G6PD

244. Causes least weight gain?


a) Aripiprazole
b) Quetiapine
c) Risperidone
d) Clozapine

245. What is true about Tacrolimus? “for atopic dermatitis”


a) Used for long intermittent
b) Apply bandage over it
Immunosuppressant, for the prophylaxis of organ rejection in patients receiving allogeneic liver or kidney
transplants, Tacrolimus has been used in combination with azathioprine.

246. What is true about Cyclophosphamide? “anti-neoplastic for malignant lymphoma”


a) Used topically for Psoriasis W
b) Used in severe cases of RA W

247. With Methotrexate you monitor? “folate antagonist anti metabolite”


a) Renal function
b) Hepatic function
c) Ocular function It might cause transient blindness

248. Pt. Taking Terbutaline inhaler (short acting) and Corticosteroid inhaler, then she came for
an early refill of terbutaline “bricanyl” by asking her you found she is using it a lot what should
you do?
a) See her doctor as soon as possible
b) Revise inhaler technique with her
c) Advise her against using terbutaline a lot

249. Lactulose used in cirrhosis to? Decrease NH3 in blood

250. What is true about Zaleplon? “Hypnotic”


a) Short t1/2
b) Better effect than Benzodiazepines
c) It is a Benzodiazepine

251. Pharmacy manager wants to increase the pharmacist duties, the pharmacist concerned
about the patients more than other duties without hiring more pharm. What to do?
a) Recruit pharmacist
b) Give pharmacists training
c) Reduce other duties done by pharmacists

252. What is common between BDZ & alcohol withdrawal?


a) Same symptoms
b) Seizures
c) Thiamine used in treatment “for alcohol Withdrawal Only”

253. Pt. With type 2 Diabetes recently feel pale and fainted she is taking Glyburide. You ask her
about all EXCEPT:
a) Any changes in her diet
b) Other drugs she is taking
c) Any increase in exercise

254. Pharm. Invited to give lecture on psoriasis to physician, what is important to talk about?
a) New non-prescription ways of treatment
b) New prescription drugs and how they work
c) What happened over the last 10 years?
255. Erythropoiesis used in anaemia caused by chronic renal failure, should monitor
a) BP
b) Haemoglobin
c) Parathyroid hormone
Darbepoetin alfa: S.E:
A synthetic erythropoietin analogue with a longer half-life that is typically Hyper & hypotension, headache,
given weekly or biweekly, and monthly in some patients. thrombosis, N &V, diarrhea,
MOE: constipation, arthralgia, myalgia,
Bind to erythropoietin receptor on erythroid progenitor cells, stimulating chest pain, arrhythmia, edema,
RBC production and differentiation. dyspnea, cough.
-Chronic renal failure: 0.45 mcg/kg weekly SC/IV, then increase by 25% Target Hb ≤120 g/L with increase
monthly, if no response; decrease by 25% as Hb approaches 120 g/L limited to 10 g/L/2 wk, but:
-Cancer chemotherapy (endogenous erythropoietin level ≤200 units/L):  Q. If excessive response,
2.25 mcg/kg weekly SC/IV. If inadequate response after 6 wk, increase to decrease dose by 40%.
4.5 mcg/kg weekly SC/IV.  If still excessive, hold
dose until Hb falls.
Q. Epoetin alfa: S.E:
A recombinant human erythropoietin with a relatively short half-life that is Hypertension, headache, seizures,
typically given at least 3 times per week. thrombosis, nausea, vomiting,
Usually given by SC. May be given IV if access already established. diarrhea, arthralgia, chest pain,
-Chronic renal failure: Initial: 50–100 units/kg 3 times weekly SC/IV, then edema, cough.
increase by 25% every 4–8 wk to maximum 300 units/kg/dose to achieve Hb Increases risk of deep venous
of 120 g/L, decrease dose by 25% thrombosis and other thrombotic
-HIV, on antiretrovirals (endogenous erythropoietin level ≤500 units/L): complications in spine surgery
Initial: 100 units/kg 3 times weekly SC/IV, then increase by 50 units/kg/dose patients. Pure RBC aplasia (rare).
every 4–8 wk to maximum 300 units/kg/dose In patients with renal failure,
-Cancer chemotherapy (endogenous erythropoietin level ≤200 units/L): achievement of higher Hb targets
Initial: 50 units/kg 3 times weekly SC/IV, or 40 000 units weekly SC/IV, is not associated with better
then increase by 50 units/kg/dose every 8 wk to maximum 300 units/kg/dose outcomes.
-Chronic hepatitis C, on ribavirin: 40 000 units weekly SC/IV Survival was worse in patients
-Surgery: 600 units/kg SC/IV 21, 14 and 7 days before surgery and then on treated to higher Hb targets (>120
the day of surgery g/L) with erythropoietin as
Q. If no response to the maximum dose after 8 wk, discontinue. compared with placebo in patients
Target Hb ≤120 g/L. Do not exceed target Hb. with head and neck cancer and in
If Hb increased by more than 10 g/L per 2 wk, decrease dose by 25%. those with breast cancer.

256. What is true about Metronidazole tab?


a) Put it in amber glass
b) Don’t use with alcohol
c) Expires after one year
257. Pt. Having candidiasis, what not advise him? Partner should be treated or see a doctor
Vulvovaginal candidiasis VVC
Caused by Candida albicans
Q. Characterized by inflammation, Stinging/burning, pruritus of vulva and vaginal
areas, odourless white, clumpy, curdy “Cottage cheese” discharges.
Vaginal pH <4.5 (normal), -ve "Whiff" test, +ve PMN.
Risk factors include Chemical irritants (amtiseptics, soaps, deodorants,), DM,
excessive refined carbohydrates diet, HIV/AIDS, antibiotics, chemotherapy, CS,
hormone therapy, oral contraceptives, tamoxifen, Menses, Pregnancy, Stress, synthetic undergarments, Tight-
fitting clothes. contraceptives (e.g. spermicide diaphragms u, sponges, intrauterine devices), douching. age
between 30 & 40.
Treatment is unnecessary in asymptomatic cases. Use prophylactic topical or oral azoles at start of antibiotic
treatment in women prone to recurrent VVC secondary to antibiotic use.
Recurrent vulvovaginal candidiasis (≥ 4 episodes/year) requires investigation and referral
No treatment for sex partner; but consider treatment in women with recurrent infections
Fluconazole, oral: Uncomplicated infection: 150 mg PO × single dose
Recurrent infection (≥4 episodes/y): 150 mg every 72 h × 3 doses then weekly for 6 m.
S.E: Headache, nausea, abdominal pain, diarrhea, dyspepsia, dizziness. C.I in pregnancy.
If patient is prone to VVC and requires a course of antibiotics, consider prescribing prophylactic topical or oral
antifungals until antibiotic course is completed.
Itraconazole, oral: Recurrent infection (≥4 episodes/y), maintenance treatment (following induction treatment
with topical azole x 10–14 days): 200–400 mg monthly PO × 6 m.
Clotrimazole, Vaginal tablet and cream:
Recurrent infection (≥4 episodes/y): extend treatment period to 10– 14 days then maintain with clotrimazole
500 mg vaginal tablets once monthly for at least 6 months.
Menstruation is not an indication to stop treatment.
Q. Safe for use in pregnancy; 7–14 days treatment period may be necessary.
Follow up if symptoms persist despite treatment or recur within 2 months of onset.
Miconazole, vaginal ovules or cream, also available in combination with external cream. Ovules contain
hydrogenated vegetable oil and mineral oil, which may decrease effectiveness of condoms or diaphragms.
Terconazole 0.4% vaginal cream Uncomplicated infection: 1 applicatorful (5 g) QHS PV × 7 days
S.E: local hypersensitivity, very rare cases of anaphylaxis & toxic epidermal necrolysis TEN
Advise patients to discontinue treatment if symptoms of allergic reaction appear
Boric acid vaginal capsules: reserved for treatment of recurrent or non-albicans VVC.
Recurrent infection (≥4 episodes/y): 300–600 mg gelatin capsules daily PV × 14 days then maintain with 300
mg capsule PV × 5 days/month beginning the first day of menstrual cycle; continue for at least 6 months.
Useful in treatment of VVC in immunocompromised host.
S.E: Local irritation, vaginal burning; more pronounced with higher dose. 300 mg capsules are less irritating
than 600 mg capsules. Contraindicated in pregnancy.

258. Eye drops containing Methyl paraben, Sodium Edetate, ascorbic acid, what is true?
a) Methyl paraben used as stabilizer
b) Sodium edetate used as…
c) Ascorbic acid used as antioxidant.
259. For a solution to be sterile it should be free of?
a) Microorganism
b) Particle matter
c) Pyrogens

260. Marijuana regulations are done according to.


a) Federal rules “through health Canada”
b) Provincial rules

261. Which of the following are symptoms of Tardive Dyskinesia?


a) Nystagmus
b) Involuntery movement “spastic, repetitive movements of the face, limbs, and torso”
Extrapyramidal side effects Prevention is key—use SGAs first-line.
(EPS; dystonia, parkinsonism, akathisia, tardive Q. If EPS occurs, first reduce dose; consider switch
dyskinesia, tardive dystonia, tardive akathisia) to SGA if on FGA.
Prophylactic use of anticholinergics (benztropine,
procyclidine, trihexyphenidyl) is not recommended
even with FGAs, & should be used only on short-term
basis to treat parkinsonism associated with FGAs.
Anticholinergics not recommended with SGAs
For akathisia:
If dose reduction is not effective, BBs propranolol
Assessment: rating scales such as Simpson-Angus Scale, 10–120 mg/day are the treatment of choice with
Barnes Akathisia Scale or ESRS are useful to assess EPS monitoring for hypotension.
and the Abnormal Involuntary Movement Scale or the Benzodiazepines also provide symptom relief.
ESRS is used to assess TD Anticholinergics are ineffective
Monitoring: baseline assessment in antipsychotic-naïve For acute dystonia (acute torticollis, oculogyric
firstepisode patients, crisis): benztropine or diphenhydramine IM, followed
In multiple-episode patients when initiating a new by reduction in dose or switch to SGA
antipsychotic, and in firstepisode and multiple-episode For tardive dyskinesia:
patients whenever dosage of antipsychotic is changed; There is no evidence-based treatment— prevention is
assess weekly for 2–4 wk or until EPS resolves. key. Use SGAs first-line.
In stable patients, assess for TD every 6 months or more Antiparkinsonian medications are not effective and
often in patients at higher risk (on FGAs, erratic may worsen symptoms.
medication adherence or intermittent treatment, female, If TD occurs, suggest consultation with a psychiatrist.
age > 55, diagnosis of an affective disorder, substance Consider switching to an SGA.
abuse, diabetes) For persistent, severe TD, consider clozapine trial

262. Pt has anxiety now he has insomnia, which medication can he used?
a) Sertraline “Zoloft” antidepressant
b) Nefazodone “serzone” antidepressant
c) Bupropion “Zyban” smoking cessation aid
263. Pregnant woman came to you with migraine, what do you recommend for her?
a) Ergotamine
b) Sumatriptan
c) Acetaminophen
d) NSAIDs

264. After delivery, her doctor gave her Propranolol as prophylaxis for migraine, what should
be monitored?
a) Severity of attacks
b) How often attacks come
c) HR

265. Pt. had migraine took Sumatriptan, his migraine was relieved but returned again, so what
should he do?
a) Stop it and switch to Ergotamine
b) Take another dose “after 2 hours max 200mg/24h”

266. 6 years old child has red spots, dry rashes, swelling on lots of body areas. You advise his
mother to see a doctor, meanwhile what you could give her to relief the pain?
a) HC 0.5%
b) Oatmeal bath
c) Cold pads

267. What is true about topical corticosteroid?


a) Nasal drops are more swallowed than nasal spray
b) When nasal drops are swallowed, they cause systemic side effects

268. Pt. Taking clindamycin, what do you advise her?


a) It should be refragirated
b) If diarrhea occurred, take loperamide till you see your doctor

269. How could you tell that this is a good journal?


a) Testimonies
b) Peer review
c) Case studies

270. Patient ask you for information about nutritional stuff, what to advise him to get it from?
a) Detritions
b) Health Canada web site
c) Local public health office

271. Where to find knowledge about availability of vaccine for out breaking virus
a) Health Canada Website
b) Medline

272. Medroxyprogesterone “Provera” may cause


a) Acne
b) Breast cancer
c) Edema

273. Pregnant woman come to you asking for OTC product as she has infection with grey
discharge and she suspects a candida infection, by asking her you found it is her first time to get
this inf., you recommend her to see a doctor immediately because
a) It is her first time to get infection
b) It does not look like candida
c) Nystatin is not given to pregnant woman “oral is not given and shouldn’t use applicator
with local cream”

274. Pt. Who is not working right now, his doctor switched him from Erythromicin to
Levofloxacin, what is his concern?
a) Timing of doses
b) Frequency of doses
c) High price

275. Pharmacy manager saw technician giving exempted Codeine to the patient, he interfered
because
a) Pharmacist should document sales by himself
b) Pharmacist should interfere with the patient by himself
c) Pharmacist should interfere with narcotic matters by himself

276. Rosiglitazone you should monitor


Blood sugar level. Liver function. Fluid retention

277. Pt. With hypothyroidism taking corticosteroids which may cause?


a) Bruising on elbow
b) Puffy face
c) Slurred speech
278. You noticed an increase in sales of antidiarrheal and you got similar feedback of another
pharmacy what you should do?
a) Report (coz it might be an outbreak)
b) Investigate with patients and another pharmacy

279. Vit D deficiency occur in all following cases


a) Elderly
b) Household
c) Sun block users

280. Pt. Taking Zidovudine, what should be monitored? “Retrovir” antiretroviral agent
Hematologic toxicities appear to be related to pre‐treatment bone marrow reserve and to dose and duration
of therapy. In patients with poor bone marrow reserve, particularly in patients with advanced symptomatic
HIV disease, frequent monitoring of hematologic indices is recommended to detect serious anemia or
granulocytopenia. In patients who experience hematologic toxicity, reduction in hemoglobin may occur as
early as 2 to 4 weeks, and granulocytopenia usually occurs after 6 to 8 weeks.
Patients treated with zidovudine should be under close clinical observation to manage potential
opportunistic infections associated with HIV disease. Prompt recognition of infection or toxicities and
appropriate management is required.

281. Baby's soy-milk formula is used in case of?


a) Sucrose intolerance
b) Allergy to cow milk protein
c) Lactose intolerance
Soy Protein Free of cow's milk protein and lactose. They are iron-fortified and are designed to meet the
Isolate– nutritional needs of term-born infants.
Based They are recommended for term infants with galactosemia or congenital lactase deficiency.
Formulas They can be used as a supplement to breastfeeding for infants of mothers who follow vegetarian
diet or for infants whose mothers wish to feed them nonanimal protein-based formula.
They are not recommended for infants with cow's milk protein–induced enteropathy or
enterocolitis, as 30–60% will also be sensitive to soy. However, infants who have an
immunoglobulin E–associated reaction to cow's milk protein may tolerate soy formulas.

282. When changing from Sinemet to Sinemet CR, what is true?


a) Wait for 8 hrs before starting CR
b) Increase dose interval by 30-50%
c) Decrease dose by 30%
“Combined therapy with levodopa and carbidopa reduces the amount of levodopa required for optimum
therapeutic benefit by about 75 to 80%, permits an earlier response to therapy, and also reduces the
incidence of nausea, vomiting and cardiac arrhythmias.”
283. Pt. Coming to your pharmacy to get her BP reading, she is taking Lisinopril you found her
BP 165/125 what should you do?
a) Send her to emergency directly
b) Increase dose of Lisinopril
c) Tell her to see her doctor

284. Health care plan make sure that


a) Prices of medication in pharmacies are the same
b) Dispensing fees are the same

285. Factors affecting passage of drugs to milk


a) Molecular wt.
b) Pka
c) Partition coefficient
Factors controlling passage of drugs into breast milk
Lipid solubility Lipid soluble drugs pass more freely in the breast milk
Molecular weight Low molecular weight drugs are more likely to get transferred to breast milk than
high molecular weight
Degree of ionization Nonionized form of drugs are more likely to be transferred into breast milk.
Ph of the plasma and Weakly alkaline drugs tend to be concentrated in milk.
milk: Weakly acidic drugs don't enter the milk to a significant extent and tend to be
concentrated in plasma.
Plasma protein Highly plasma protein‐bound drugs pass less into milk. Anti diabetic
binding of drugs
Drug concentration in Transfer of drug from mother’s blood to milk is passive and is low with drugs that
maternal serum have large volume of distribution (Vd).
Short half life (t ½).

286. New formulary was discovered and found to treat and decrease morbidity due to certain
disease, before producing this medication they should know?
a) Prevalence of disease
b) Type of morbidity
c) Cost of medication

287. Cephalosprins label


a) Not taken with milk or antacids
b) Finish course
c) Keep in a refrigerator
288. How to assist improvement of lifestyle of a patient
a) By decreasing his BP
b) By increasing his ability to do more work
c) By well being

289. Pt with renal failure needs to take a diuretic, what to take?


a) Furosemide
b) Thiazide W
c) Spironolactone
Spironolactone could be used in Ascitis

290. Benzodiazepines bottle falls down on the floor, what is the right action?
a) Collect tablets in presence of witness and discard
b) Keep in a locked place
c) Add amount to sales registration book

291. Amoxicillin + Clavulanate susp label


a) Shake well
b) Avoid excessive temp
c) Keep in refrigerator “should be used within 7-10 days”

292. All cause bronchospasm, EXCEPT:


a) B-Blockers
b) Neostigmine
c) Bethanechol
d) Epinephrine

293. N-acetyl cysteine is used in acetaminophen toxicity to? “NAC prevents the Liver damage that
leads to death after Acetaminophen overdose (to be effective NAC must be administered within 12 hours of
Acetaminophen poisoning (due to NAC stimulating Glutathione production which functions as an
Antioxidant and protects the Liver from Acetaminophen ‐induced oxidative damage)”

294. Pt. On Morphine, has constipation although keeping good diet and taking milk magnesia,
what to advise him
a) Increase dose of milk of Magnesia (Osmotic)
b) Use senna daily (stimulant)
c) Use Bisacodyl supp. (stimulant)
d) Use Docusate PR (Stool Softener)
e) Use Psyllium (fibers)
295. Antibiotics used to eradicate H. pylori are all EXCEPT:
a) Bismuth subsalicylate
b) Clarithromycin
c) Tetracycline
d) Ciprofloxacin
e) Amoxicillin

296. Pt. has got 3 episodes of depression, being treated with antidepressant, what should be
done?
a) Take it indefinitely
b) Take it for one year
c) Take it for 2 years
“one episode 1 year, two episode 2 years”

297. Which of the following cause drug diversion during surgery room?
a) Fentanyl
b) Nitrous oxide
c) Morphine
d) Meperidine

298. Pharmacy manager wants to change his computer system, what is the correct order of his
action?
a) Calculate his budget or cost of project
b) Find out the capability of existing machines
c) Find out the vendors requirements
d) Find out if the software is capable of fulfilling these function

299. Pt. Having nasal dryness due to using O2 treatment of COPD and wants OTC product, the
pharmacist advice him to use oil-based ointment, the manager overheard him and since this is
contraindicated with this patient, what should the manager do?
a) Go directly correct this to patient a pharmacist
b) Talk to pharmacist aside and tell him to go back and correct himself to pt.
c) Talk to pharmacist aside and tell him to call patient next day and correct himself
d) Keep pharm. Dignity and talk to him after patient goes away
e) Leave pt. Take ointment and when returns back follow up with him as he mentions he is
still not feeling well, then tell him to try water base oint.
300. All are included in GMP in a hospital EXCEPT
a) Cleaning floor
b) Sanitary conditions
c) Keeping narcotics

301. Cost of things added afterwards is called?


a) Incremental cost. “the increase or decrease in costs as a result of one more or one less
unit of output”
b) Acquisition cost

302. Pt. With rx for Diazepam and he wants to transfer it to your pharmacy, what is right?
a) Call the other pharmacy to transfer prescription
b) Ask him to see a doctor for a new prescription
c) Ask him to go to other pharmacy

303. Cancer patient taking Methadone, his doctor is out of town, the available doctor is not
registered to prescribe Methadone, what to do as a pharm.?
a) Do not give him as you are not allowed to
b) Fill prescription expecting doctor will sign it when he returns
c) Call doctor and get verbal prescription
d) Ask available doctor to register himself temporarily to prescribe Methadone

304. Ph. Saw technician giving exempted Codeine to pt. What is the right thing to do?
a) Interrupt him and continue with patient, then talk to him after pt. leaves.

305. Manager of hospital pharmacy wants to talk about ways to be aware of dispensing errors,
he would invite all of the following EXCEPT
a) Nurse
b) Patients
c) Pharmacists
d) Risk management representative
e) Good practice management

306. After delivery, her doctor gave her Propranolol as prophylaxis for migraine, what should
be monitored?
d) Severity of attacks
e) How often attacks come
f) HR
307. Clarithromycin can cause prolongation of QT if taken with which of the following?
a) Fexofenadine
b) Cetirizine
c) Ketoconazole
Clarithromycin: shake well and take with food. Do not fridge. Interacts with sotalol cause QT prolongation
As with other macrolides, hepatic dysfunction, including increased liver enzymes, and hepatocellular and/or
cholestatic hepatitis, with or without jaundice, has been infrequently reported with clarithromycin. Allergic
reactions ranging from urticaria and mild skin eruptions to anaphylaxis and Stevens‐Johnson Syndrome/
toxic epidermal necrolysis have occurred with orally administered clarithromycin.

308. Pt. Used to have migraine, but lately its severity is increasing so now she is using
Acetaminophen 1000 mg daily for 3 months, it is not relieved, what to conclude?
a) Drug induced headache “used twice/week max. great risk of rebound headache”

309. For the above patient you advise her all EXCEPT
a) See her doctor as soon as possible
b) Stop taking acetaminophen  taper
tapering and stopping the offending agent(s), and starting a prophylactic medication such as amitriptyline

310. Nurse in a hosp. Has morphine 10 mg on her floor, but she also wants to have morphine
2mg as well, she asked the pharmacy manager, in this case the pharmacy manager would
consider all the following except
a) Effectiveness of morphine 2 mg
b) High risk of error of having two conc. On the ward
c) High risk of error in calculating doses from 10 mg
d) Wastes coming out of 10 mg

311. Hospitals work according to federal rules to be eligible for?


a) Tax free
b) Part of money

312. All of the following are taken on empty stomach EXCEPT


a) Cloxacillin (1hr before or 2hr after meals)
b) Azythromicin “with or without food”
c) Zafirlukast (1hr before or 2hr after meals)

313. Fosinopril and Lisinopril are equal in which of the following?


a) Both cause dry cough & angioedema
b) Both are prodrug (Lisinopril is not)
c) Contraindicated in bilateral renal artery stenosis
d) Both cause hyperkalemia (enalapril doesn't cause hyperkalemia)
e) Direct vasodilator (no they both work on ACE inhibition)
f) Their use with thiazide reduces their effect (increase their effect)
g) Serum accumulation in renal impairment (they are not Lisinopril is 100% excreted by
kidney while fosinopril is excreted by urine and feces 50/50%)

314. Pt. Taking Ramipril should advise him all except


a) Might experience dry cough
b) Might experience edema
c) Should eat pot rich food
d) Should restrict salt food

315. The most addictive agent among the following


a) Prochloropromazine
b) Diphenhydramine
c) Meperidine (pethidine)

316. Pt. Having dry cough, using moclobamide” MAO “, which one is best for him?
a) Oxymetazoline
b) Phenylephrine, acetaminophen
c) Pseudoephedrine
d) Dextromethorphan

317. Pt. has depression and insomnia, which anti-depressant can he use?
a) Fluxetine
b) Phenelzine
c) Amitriptyline
d) Bupropion
e) Moclobemide

318. Which of the following agents is used in bipolar depression can cause Leukopenia?
a) Carbamazepine
b) Lamotrigine
c) Lithium
319. All the following are symptoms of tension headache EXCEPT?
a) Fever and neck stiffness
b) Nausea and vomiting
c) Aggravated by physical activity
Clinical Feature Tension-type Headache Migraine
Quality Pressing (nonpulsating) Throbbing/pulsating (at least part of the time)
Severity Mild to moderate Usually moderate to severe, although can be mild
Location Bilateral Usually unilateral (can be bilateral, especially in children)
Frequency Episodic (<15 days/month) or chronic (≥15 days/month)
Duration 30 min to 7 days 4 –72 h
physical activity Not aggravated Aggravated by physical activity
Associated No nausea/vomiting. At least one of the following:
symptoms Anorexia may occur. 1. Nausea and/or vomiting
Photophobia or phonophobia 2. Photophobia and phonophobia
but not both 3. May occur with or without aura (usually visual)
Preventive Cognitive behavioural therapy Many patients get relief by lying down in a dark, quiet
Therapy Relaxation training room and applying a cold cloth or ice pack to the head.
Biofeedback Falling asleep often provides relief as well
BBs without ISA (e.g., propranolol, nadolol, metoprolol).
For disabling and/or chronic TCAs (e.g., amitriptyline, nortriptyline).
tension-type headache, Ca channel blockers (e.g., flunarizine, verapamil).
pharmacologic prophylaxis Serotonin (5-HT2) receptor antagonists (e.g., pizotifen),
may be considered (e.g., valproic acid/divalproex sodium, topiramate, candesartan,
amitriptyline, mirtazapine, lisinopril, gabapentin.
nortriptyline, venlafaxine). NSAIDs (e.g., naproxen sodium 550 mg twice daily for 1
week per month for menstrual migraine prophylaxis
Natural Health Products e.g: Butterbur 75 mg. CO-Q10
(300 mg/day) & Melatonin 3 mg.
Drug therapy Simple analgesics (e.g., ASA, -Simple analgesics (acetaminophen, ASA) or NSAIDs.
acetaminophen) or NSAIDs - Triptans or 5-HT1B/1D receptor agonists (almotriptan,
(e.g., ibuprofen, naproxen eletriptan, frovatriptan, naratriptan, rizatriptan, sumatriptan
sodium) will often alleviate - Ergotamine Derivatives Nasal or injectable dihydroergot
tension-type headaches. amine (DHE) effective for migraine.
Amitriptyline and nortriptyline Oral ergot has limited efficacy and excessive side effects.
are effective in reducing -Adjunctive antiemetics e.g., metoclopramide,
headache frequency and chlorpromazine, prochlorperazine, ketorolac,
severity dexamethasone or opioids (not first-line therapy)
-Butalbital (with ASA, caffeine and/or codeine)

319. All the following can be transferred EXCEPT


a) Clonazepam” rivotril”
b) Mazindol “samorex, anti obesity not approved in canada”” amphetamine like action”
c) Alprazolam” zanax”
d) Chlordiazepoxide “benzodiazepine”
e) Cyclobenzaprine “flexeril, muscle relaxant”

320. Pt. With breast cancer, her doctor wrote a prescription of Meperidine “Demerol” and
indomethacin, what is your recommendation?
a) Increase Meperidine and decrease Indomethacin
b) Increase Indomethacin and decrease Meperidine
c) Change to Morphine
d) Increase meperidine and stop indomethacin
e) Increase Indomethacin and stop Meperidine

321. Now the above patient is in the hospital because she is worried, pt is taking morphine,
what is the best dosage form?
a) IV
b) IM
c) SC
d) Oral

322. Warfarin will cause hepatic problems in all the following cases EXCEPT?
a) Change in the diet
b) Drugs that inhibit HME
c) Thrombocytopenia
d) Neutropenia

323. Pt. Taking Lugol solution you advise him that


a) Taken with food
b) External use only
c) It stains
d) Need good ventilation

324. A lady with post-herpetic neuralgia pain, now she has a prescription for Amitriptyline
“indicated in this condition” and Tylenol #3, what should you advise her?
a) Recommend Tylenol #1 instead of #3
b) Tell her that Amitriptyline is the best for her
If Amitriptyline causes side effects, she should stop it and continue with Tylenol #3. If Amitriptyline causes
side effects, she should go to a DR. clinic
325. This lady came to you with a prescription for Percocet” oxycodone” you noticed it is her
2nd one in 2 weeks from a one-hour clinic, she is not seeing her family doctor, she finished 50
tabs in 2 weeks, what should you advise her?
a) To see her family doctor and discuss her problem and find a suitable ttt.
b) To use capsacin instead
c) To take Pecocet as it is good for her, but not use it too much
d) Call the doctor who wrote the prescription ad tell him she is taking too much

326. Pt. Taking Salbutamol Inh., you should advise him all EXCEPT:
a) How to use turbohaler
b) Not to exceed dose
c) Shake it well
d) Use it PRN

327. Pt. Using diskhaler, you tell him all EXCEPT:


a) Not to exceed dose
b) To activate it by sliding it back and forth
c) Every time you use it the no. of doses decrease
d) It has a dose counter
e) You should clean mouth piece with wet cloth

328. Pt. Taking drug in a wrong way, pharm counselled him many times, now pharm doesn’t
want to counsel him he mostly violates
a) Beneficence
b) Non-maleficience
c) Veracity
d) Autonomy
e) Paternalism

329. Cancer patient does not want to take her medication due to side effect, her doctor asks
pharm. To tell her that that medication is multivitamin, what is the doctor seeking for
a) Beneficence
b) Non-malfecience
c) Veracity
d) Autonomy
e) Paternalism
330. All are true about influenza vaccine EXCEPT:
a) Infection begins from December till March
b) Given to elderly without regards to co-morbidity
c) If given as split can cause more side effects
d) Can be given with other vaccine at the same time

331. Unapproved use of drugs is found in


a) CPS
b) Medline
c) Medics drugdex
"Most of the references except CPS"

332. Pt. Ask you about Ginseng, where can you get credible knowledge about it?
a) CPS
b) Another Ph.
c) Pharmacist letter
d) Detritions
Note: Ginseng is an herbal medicine, you can find herbal drug in following references:
 Laurence book of herbal drug
 Patient self‐care
 Compendium of nonprescription drug
 Herbs everyday foe health care professionals

333. For a medical brochure to be more credible, it should contain knowledge from?
a) Reviews from 50 sources
b) Manufacturer knowledge
c) Reputable journals
d) Double blind case studies
NOVEMBER 2015

1. Patient JG admitted in hospital with Mild stroke of subcranial hemorrhage, had perivous MI,
BP= 140/95, HR: 90, LDL 3.9, TG 2.2, HDL 0.9. Current medication ASA 81 mg, Nifedipine
50 mg, Bisoprolol 5 mg. What is the best drug suggest JG for lipid lowering therapy?
a) High dose Atorvastatin
b) Low dose Atorvastatin
c) High dose fenofibrate
d) Low dose fenofibrate
e) High dose niacin
Starting a statin early after MI decreases both early and late adverse events. Guidelines suggest
documenting the cholesterol profile within 24 hours of infarction, as LDL cholesterol may be falsely low for
up to 4 weeks postinfarction. Evidence suggests that intensive treatment with higher‐dose statins (e.g.,
atorvastatin 80 mg/day or rosuvastatin 40 mg/day) early after MI confers additional benefits compared
with moderate dosages. The benefit of statins extends beyond lowering of LDL cholesterol levels alone as
they contribute to plaque stabilization by preventing future acute coronary syndromes. Current Canadian
Cardiovascular Society guidelines advocate a minimum target LDL‐C of <2 mmol/L

2. What is your recommendation for controlling blood pressure?


a) Increase Bisoprolol dose
b) Change nifedipine to perindopril
c) Change aspirin to clopidogrel
d) Terazocin
ACE inhibitors (the king) should be considered in all post‐MI patients

3. Doctor prescribed Metoprolol for controlling blood pressure and for primary prevention of
stroke, what is best goal to add betablocker?
a) Decrease risk stroke attack
b) Reduce recurrent infarction
c) Reduce risk of acute coronary syndrome
Beta‐Blockers: In the absence of contraindications, consider this class of medication in all patients after
STEMI or NSTEMI, as they have been proven to reduce MACE postinfarct. In particular, beta‐blockers play a
role in the reduction of reinfarction and prevention of ventricular arrhythmia in the early phase after
infarction. Beta‐blockers should be started within a few days of infarction and the dose titrated to
maximally tolerated doses with heart rate not >70 bpm. However, IV beta‐blockers should be used
cautiously in the early acute phase, as they may contribute to hemodynamic instability in patients suffering
large infarcts. Avoid betablockers in patients with hypotension, bradycardia and active heart failure.
Patients with reduced LV function derive significant benefit from beta‐blockers, but should be monitored
closely, especially in the postinfarction period. This benefit extends to those already treated with ACEIs.
If tolerated, beta‐blockers should be continued indefinitely, especially in patients with reduced LV function;
however, they can be stopped at follow‐up in patients with normal LV function if fatigue or other side
effects are encountered

4. MJ Admitted in emergency department for complaining NSTEMI and having high blood
pressrure, Diabetes and BPH, on following drugs. Terazocine, Metformin, ASA.
What is drug you adding for this person
a) Perindopril
b) Amlodipin
c) Tamsulosin
d) Valsartan
e) Clonidine

5. Now MJ went to Percutaneous coronary intervention PCI with drug stent, what is your
recommendation for prevention of stroke
a) ASA
b) ASA/dipyridamol
c) Warfarin
d) ASA + ticagrelor
e) Clopidogrel
The thienopyridine clopidogrel continues to be commonly used after PCI. In MI patients receiving clopidogrel
after PCI, a dose of 150 mg daily for the first 6 days has been shown to be superior and should be
considered. A daily dose of 75 mg should be used after the initial 6 days of high‐dose therapy or for those
patients deemed not to require the initial high‐dose therapy. For patients with ASA allergy, chronic
clopidogrel therapy may be a substitute.

6. NP a Male patient was admitted to emergency department after a car accident. He had
penetrating trauma in his left leg and got a femur fracture. What should be given for prophylaxis
of osteomyelitis?
a) Cefazolin + gentamycin
b) Cefazoline and metronidazole
c) Cephalexin
d) Amoxycillin
e) Ticarcillin
Antibiotics for open fracture:
For grade I open fractures: 1st generation cephalosporin cefazolin IV X 3days to cover gram +ve organisms,
use clindamycin or FQ IV for true penicillin‐allergic patients, add vancomycin if MRSA suspected
For grade II and III open fractures: cefazolin IV plus gram ‐ve coverage + or ‐ anaerobic coverage i.e. add an
aminoglycoside or use a 3rd generation cephalosporin (e.g ceftriaxone) + or ‐ metronidazole or clindamycin

7. Same patient after some time he was being transferred to Long term care facility. What is oral
therapy to initiate in this patient to decrease risk of DVT?
a) ASA
b) Warfarin
c) Dalteparin
d) Clopidogrel
e) Rivoroxaban
Apixaban, dabigatran and rivaroxaban are direct‐acting oral anticoagulants that can be used for VTE
prophylaxis following elective total hip or total knee replacement surgery.

8. NP started on oral warfarin therapy, as pharmacist you recommend oral therapy for at least?
a) 1 month
b) 3 months
c) 9 months
d) 12 months
The recommended duration of oral anticoagulation for treatment of DVT/PE is dependent on the risk of
recurrence. Treatment of a first episode in a patient with a transient risk factor should continue for just 3
months.
9. NP on Dabigatran for DVT and Now NP discharge from hospital, what do you recommend?
a) Discontinued dabigatran and start Rivoroxaban
b) Continued dabigatran for 5 day after discharge and start Warfarin
c) Stop dabigatran and start warfarin
d) Continued dabigatran and start rivoroxaban
e) No need to change dabigatran
Because it is already oral anticoagulant and self‐monitoring

10. NP on Warfarin therapy and doctor want to discontinue oral therapy for knee replacement
surgery. What is step down therapy?
a) Discontinue Warfarin
b) Discontinue heparin and start rivoroxaban
c) Discontinue Warfarin before 5 days and start enoxaparin
Perioperative management of anticoagulation
11. SG Pregnant lady admitted in hospital for labor pain and given meperidine and gone trough
c-section. (No information given about oxycodone use and epidural injection). After birth she is
on bed rest for 3 days and develop creepy crawly and nagging sensation in legs, doctor
identified as symptoms of RLS. What is the cause of RSL in this patient?
a) Meperidine
b) Pregnancy
c) Bed rest for three days.
d) Oxycodone before labor
e) Epidural injection

12. Now doctor want to start treatment for RLS what is your recommendation?
a) Pramipexole
b) Lamotrigine
c) Carbamazepine
d) Amytriptilyne
Antiepileptic drugs such as Carbamazepine and Valproic acid, Amantadine, Baclofen and Clonidine, these
drugs should be reserved for patients who develop tolerance to recommended first‐line agents.

13. Doctor started therapy of RLS as pharmacist where you document this therapy.
a) In ward chart
b) In computerized pt file that pt started on RLS treatement
c) Talk to neurse
d) Patient profile in ward.

14. After discharge what non pharmacological treatment you suggest to this patient
a) Increase physical activity
b) Hot bath
c) Drink coffe
d) Bed rest

15. Mother of child 28-month-old called your pharmacy for complaining of ear pain and mild
fever. Recurrent AOM 1 month, 23rd month and 25th
month. Previously treated with high dose amoxicillin.
What you will not recommend to this child?
a) Cefprozil
b) Cloxacillin
c) Clindamycin
d) Azithromycin
e) Amoxycilline clavulanate
16. She is asking for something she can (give her 2-year son) as chewable tablet for pain and
fever. Which reference you will check?
a) CPS
b) Remington
c) Martinedale
d) Sick kids’ drugs handbook of formulary
e) Therapeutics of choice

17. You refer to doctor for antibiotic


treatment, what is your recommendation?
a) You can’t give ASA and Ibuprofen
together.
b) Use cold compress.
c) Fever and pain will subside within
48-72 hours of Tx

18. You need to prepare 50 gm 5% onintemnt from 3% ointment and 10% ointment.
3% 7parts

5%

10% 2parts
3% = 7/9 * 50 = 38.89gm 10% = 2/9 * 50 = 11.11gm

19. Health Canada issues Notice of compliance NOC apply following can sponsor legally start
a) Can start clinical trials on animals
b) Can start phase II clinical trail
c) Can market drug in Canada
d) Manufacture can apply for patent

20. BJ Psycho Patient is smoker used to smoke half of packet every day, currently on clozapine.
Come for routine check up at clinic following lab test value. WBC 12000 (5000 -11000),
Neutrophil count 1.7 (normal 2.8-7.7), RBC normal, Eosinophil less, Lymphocytes 0, what
abnormality patient face?
a) WBC
b) Neutrophils
c) Lymphocyetes
d) Red blood cells
21. Now BJ decides to quit smoking what is your recommendation to change clozapine dose
a) Increase clozapine dose as decrease hepatic enzymes
b) Decrease clozapine dose as decrease hepatic enzymes
c) Increase clozapine dose as increase hepatic enzymes
d) Decrease clozapine dose as increase hepatic enzymes
e) No change in clozapine
Smoking can induce clozapine metabolism and abruptly stopping smoking can increase clozapine levels.
It is well documented that cigarette smoke can induce cytochrome P450 (CYP) isoenzymes, specifically
CYP1A1, CYP1A2, and CYP2E1. Because clozapine is primarily metabolized by CYP1A2 (approximately 70%),
smoking can induce clozapine metabolism and abruptly stopping smoking canincrease clozapine levels.

22. What side effect can be developed least?


a) Myocarditis
b) EPS
c) Hypersalivation
d) Agranulocytosis

23. Late Friday evening patient come for 2 days


advance of clozapine until doctor return on
Monday. He slept lately on Friday night and
missed his blood work. There are no more refills. What is your action?
a) Give refill and inform him to come back on Monday with blood work for follow up
b) Give 2-days' supply and follow up
c) Inform him to contact hospital emergency clinic
d) Refuse to dispense (don’t need it)
The cautionary note is that if you've a similar version in which the patient is missing lab repeatedly without
a clear reason, you can tell the patient the rule of, "NO blood, NO drug".
In other words (to understand the "NO blood, NO drug" rule), before dispensing continuing clozapine
prescriptions, the pharmacist should verify that the patient has done their WBC and differential tests (to get
ANC) by checking with the registry online (they fax blood work to pharmacies).
If the blood work has NOT been done within the required time frame (as determined by the physician; eg,
every 2 weeks), pharmacists should NOT dispense clozapine (i.e., “NO blood, NO drug”).
Another version/permutation of the very same Q (I guess you shared it previously) is that if you've fever or
signs of infection, do NOT dispense more doses until seeing the Dr. and getting their CBC and differential
counts (ANC) assessed. Finally, a couple of concluding separate notes about clozapine use is that:
1) Pharmacists should NOT dispense the first clozapine prescription until confirmation has been received
from the registry that it is safe to start clozapine.
Same recommendation applies to patients restarting clozapine after a period of being off the medication
Similarly, a patient must NOT be switched from one brand of clozapine to another by a pharmacist UNLESS
s/he obtains a new, registry‐specific patient registration form completed by the prescriber.
24. DG depression patient on citalopram for second episode, she had this episode before 2 years
ago. Now patient diagnosed with adult ADHD, she is irritable, talkative and sleep late night.
What are adult symptoms of ADHD
a) Inability to listen
b) Hyper activity
c) Insomnia
d) Talkative

25. Which Non-pharmacological is the best for DG’s ADHD?


a) Relaxation
b) CBT
c) Electroimpulsion
Behavioural therapies are designed to minimize negative behaviours and promote positive ones by teaching
parents (and sometimes teachers) techniques to improve a child's behaviour. Behavioural therapies play an
important role in improving social and family interactions, self‐esteem and the common behaviours seen in
ADHD. Behavioural strategies that included sleep hygiene practices reduced the severity of parent‐ and
teacher‐reported ADHD symptoms and sleep problems at 3 and 6 months postintervention

26. What is drug therapy you will sugest?


a) Keep escitalopram and add Atomoxetine
b) Decrease citalopram gradually till discontinued and start Atomoxetine
c) Stop citalopram and start Atomoxetine
d) Stop citalopram and start desamphetamine
Atomoxetine (NE reuptake inhibitor) should also be considered for those with ADHD and comorbid
substance‐abuse disorder or depression. Contraindications to atomoxetine include hypersensitivity to
atomoxetine, narrow angle glaucoma, history of severe cardiac or vascular disorders, pheochromocytoma,
and concurrent use with an MAOI.
+ Antidepressants may benefit patients with comorbid conditions such as depression, anxiety, enuresis or tic
disorders

27. BPH patient with hypertension. His Current medication: Atorvastatin 40 mg QD,
Metoprolol 50 mg BID, Terazosin AM. What is the appropriate action to control hypertension?
a) Add Tamsulosin
b) Increase Atorvastatin dose
c) Increase dose of metoprolol.
Initial: 50 mg/day. Usual: 100–200 mg/day. Maximum: 400 mg/day
Give regular formulations BID po; SR formulations once daily po
28. He Wants to stop terazosin as he doesn’t like to take many medications. What to advise
him?
a) Stop terazosin when symptoms subside
b) Don’t stop terazosin as therapy is indefinite
c) Stop when 50% reduction in PSA
d) Stop after 3 months
e) Stop after 6 months
Maximal response seen in weeks (4 ‐ 6 weeks).
Therapeutic Tips
Patients with minimal symptoms that do not interfere with their normal activities and are not associated
with the complications of BPH should be managed by active surveillance and regular follow‐up.
Patients starting to develop progressive symptoms or who are moderately inconvenienced or bothered by
them are candidates for pharmacologic intervention.
Continue drug therapy indefinitely, since symptoms recur when medication is stopped.
Complicating factors or unexpected (or lack of) response to any intervention are indications for urologic
consultation.
Avoid decongestants and other drugs with alpha‐adrenergic activity because they can stimulate smooth
muscle in the bladder neck and prostate, reduce the benefit of alpha‐adrenergic receptor antagonists and
increase bladder outlet obstruction.
Drugs with antimuscarinic activity may reduce detrusor contractility. Although they may be less problematic
than previously thought in patients with symptoms of bladder outlet obstruction, these agents should be
used with caution.

29. Taking terazosin at 8 am every morning. Had complaint regarding Dizziness in day time.
What you ll recommend? Change Terazosin dose to bed time
Adverse events from terazosin and doxazosin may be reduced by taking them at bedtime.

30. A lady come to your pharmacy for prescription of diclectin for N&V in pregnancy, she is
taking topiramate for seizure, you recommend adding dimenhydrinate. What Drug therapy
problem you will rectify?
a) Topiramate to valproic acid
b) ACE inhibitor to labetalol
c) Rivoroxaban to warfarin
she is a pregnant woman and ACEIs is CIs with her so we have to convert it to labetalol

31. Which of the following drug can cause embryopathy in pregnant woman.
a) Lithium
b) Warfarin
c) Phenytoin
COUMADIN (warfarin sodium) is contraindicated in pregnancy because the drug passes through the
placental barrier and may cause fatal hemorrhage to the fetus in utero.
Embryopathy characterized by nasal hypoplasia with or without stippled epiphyses (chondrodysplasia
punctata) has been reported in pregnant women exposed to warfarin during the first trimester.
Central nervous system abnormalities also have been reported, including dorsal midline dysplasia
characterized by agenesis of the corpus callosum, Dandy‐Walker malformation, and midline cerebellar
atrophy. Ventral midline dysplasia, characterized by optic atrophy, and eye abnormalities have been
observed. Mental retardation, blindness, and other central nervous system abnormalities have been
reported in association with second and third trimester exposure.
Although rare, teratogenic reports following in utero exposure to warfarin include urinary tract anomalies
such as single kidney, asplenia, anencephaly, spina bifida, cranial nerve palsy, hydrocephalus, cardiac
defects and congenital heart disease, polydactyly, deformities of toes, diaphragmatic hernia, corneal
leukoma, cleft palate, cleft lip, schizencephaly, and microcephaly.
Spontaneous abortion and still birth are known to occur and a higher risk of fetal mortality is associated
with the use of warfarin. Low birth weight and growth retardation have also been reported.

32. MK admitted to hospital and having mechanical valve problem & maintain on respirator. He
has pyelonephritis, doctor order cotrimoxazole. What is causative organism that doctor
prescribed for?
a) E coli
b) Straphylococcus
c) Ligonella
d) PJP
Mild to E. coli (90%), P. mirabilis & K. pneumoniae (5%) Fluoroquinolone Amoxi/clav PO or
moderate Occurs in women who experience recurrent (ciprofloxacin, SMX/TMP PO or
pyelonephritis uncomplicated UTIs but at lower frequency than levofloxacin, Trimethoprim PO
OR Acute cystitis. norfloxacin), all for 10-14 days
Nonobstructive Classic presentation includes fever, N&V, flank PO × 7–14 days
Pyelonephritis pain with or without associated irritative urinary
Severe symptoms. Aminoglycoside Fluoroquinolone IV
pyelonephritis Patients who present with UTI with only lower IV ± ampicillin IV × 10–14 days or
urinary tract symptoms or asymptomatic for initial therapy; 3rd generation
bacteriuria occasionally have associated occult if appropriate, step cephalosporin IV ±
renal infection. down to oral aminoglycoside IV
Bacteremic infection occurs most frequently in therapy as in mild × 10–14 days or
diabetic women or women >65 y. to moderate Carbapenem IV ×
Urine Culture: always recommended. infections in order 7–14 days for
Consider blood cultures. to complete 10–14 ESBL-producing
days organisms
33. Patient in hospital is taking IV Cotrimoxazole (each ml contains 80 mg SMT / 16mg TMP)
0.5 ml Q6hr. Dr. want to shift him to liquid (200 mg STM / 40 mg TMP) in each 5 ml, how
many of liquid is needed to provide the equivalent dose as was taken IV.
Answer: 0.5 ml Q6hr = 0.5 * 4 = 2 ml /Day
SMT: 80 mg -------- 1 ml X mg ------- 2 ml X= 2*80/1 = 160 mg
Liquid: 200 mg ------- 5 ml 160 mg ------- Y ml Y = 160*5/200 = 4 ml/Day

34. CJ 165-pound patient admitted to hospital with CAP, diabetes, ischemia and doctor
admitted in hospital for pneumonia treatment, as pharmacist what you recommend as initial
empirical treatment?
a) Amoxicillin
b) Cephalexin
c) Clindamycin
d) Cefotaxime
e) Levofloxacin
Empiric Therapy
 For CAP of moderate severity, there is no difference between beta-lactam alone, macrolide and beta-
lactam, or fluoroquinolone therapy. A systematic review showed no difference between macrolides &
fluoroquinolones but fewer adverse events with clarithromycin than erythromycin. Another review
showed no difference between macrolides and fluoroquinolones for death.
 For outpatients, amoxicillin as first choice or either amoxicillin/clavulanate or doxycycline as second
choice.
 For severe pneumonia, 3rd generation cephalosporin (ceftriaxone or cefotaxime) in combination with
 clarithromycin is a rational empiric regimen. However, it should be noted that macrolide antibiotics
(e.g., azithromycin, clarithromycin) and fluoroquinolones may cause QT interval prolongation and
caution is advised.
 For inpatients: Systematic review evidence for inpatients showed no difference between macrolides
and fluoroquinolones for death and no difference for death whether or not atypical coverage was
used. In order to reduce increasing fluoroquinolone resistance and prevent adverse events (e.g., QT
interval prolongation), use of a respiratory fluoroquinolone should be reserved for when cephalosporins
or penicillins cannot be used.
 Piperacillin/tazobactam should be used only for severe pneumonia or in patients at high risk for
resistant pathogens, e.g., P. aeruginosa.
Duration of Antibiotic Therapy
 For patients who are well enough to be treated on an ambulatory basis, a minimum of 5 days of
antibiotic therapy is required.
 Patients who are hospitalized, who respond to treatment within 48 hours and who have no complications
may be treated for 5–10 days.
Specific etiologies may require longer treatment, such as:
 21 days for severe legionnaires’ disease & pneumonia caused by P. aeruginosa.
 14 days for bacteremic aerobic gram-negative bacilli pneumonia & Empyema that requires drainage.
 Prolonged therapy is necessary when a lung abscess complicates pneumonia.
35. Culture test order by doctor and results suggested MRSA and doctor started IV Vancomycin
15 mg/kg as iv infusion. Round dose to nearest 250 mg. Infusion rate is follow:
If dose is 1gm infusion rate is 250 mg/hour. If dose is 1.1gm-1.5 gm infusion rate is 300
mg/hour. If dose is 1.6 -2 gm infusion rate is 350 mg/hour. How much time it takes for CJ to
finish his infusion.
a) 1 hour
b) 2 hours
c) 3 hours
d) 4 hours
e) 6 hours
15 mg * 165/ 2.2 = 1125 mg Round it to nearest 250 = 1000 mg
If rate is 250 mg per hour So, you need 4 hours

36. Vancomycin trough level


a) 0-5 mg/dl
b) 5-10 mg/dl
c) 10-20 mg/dl
d) 25-30 mg/dl
Peak: 25–40 mg/L. Trough: 15–20 mg/L.

37. In hospital ward nurse give mistakenly dose of vancomycin in 5 min, what is immediate
adverse reaction will happen in CJ.
a) Hyponatremia
b) Hypokelemia
c) Hypotention  red man syndrome

38. Patient having sexual dysfunction, high blood pressure and diagnosed with depression what
is best drug for him?
a) Fluoxetine
b) Venlaflexin
c) Bupropion
d) Buspirone

39. Now, he feels guilty & try to suicide, no sleep. Which antidepressant to give?
a) Venlafaxine
b) Mirtazapine (depression +insomnia)
c) Tricyclic antidepressants
40. In order to weigh a compound that that has 95% accuracy on a balance that has sensitivity
error of 4.5 mg. What is the minimum weighable amount?
a) 6 mg
b) 9 mg
c) 90 mg
d) 120 mg
Sensitivity = Weight * error 6 = Weight * 5% Weight = 6*100/5 = 120mg

41. It is the month of September and there is a sudden increase in the sale of ipecac syrup
mostly kids. What should be done to avoid abuse of ipecac syrup as a pharmacy manager?
a) Restrict sale
b) Make sure all can get council for ipecac
c) Set age limit for ipecac sale
d) Set maximum limit on amount of ipecac to be sold
e) Move ipecac to behind the counter so interaction can be done during sale

42. Pt have current medication: Butalbital/ASA/caffeine/codein, Clonazepam, Methylphenidate,


Nifedipine, Lactulose, Nitroglycerine. How many narocitc, control drugs, targeted drug and
prescription drugs
a) 4-0-1-2
b) 1-1-1-2
c) 1-1-1-1 → NG and lactulose non-Rx

43. Side effect of ciprofloxacin


a) Hepatotoxicity
b) Skin pigmentation
c) Phototoxicity
Ciprofloxacin has been shown to cause photosensitivity reactions. Patients taking ciprofloxacin should be
advised to avoid direct exposure to either extensive sunlight or UV irradiation during treatment

44. UTI scenario. Pregnant lady in her 24th week with penicillin allergy. She got hives with
penicillin use in past. What to give?
a) Nitrofurantoin
b) Trimethoprim
c) Cephalexin
45. What drug is not used in dementia?
a) Resperidone
b) Rivastigmine
c) Galantamine
d) Amantadine (in Parkinson)

46. AE is scheduled to undergo bariatric surgery that will result in a bypass of the duodenum,
proximal jejunum and all of her stomach except for the cardia. Her surgeon has asked AE to
discuss any necessary modifications to her medication regimen with her pharmacist. Which of
the following physiologic factors would be expected to increase following AE's gastric bypass
surgery?
a) Gastric pH (fewer pumps)
b) Surface area of the gastric mucosa
c) Gastric emptying time (dumping)
d) Enterohepatic recirculation

47. Which of the following references is the most useful for determining appropriate post-
surgical modifications to AE's medication regimen?
a) RxFiles
b) Common Drug Review
c) Remington: The Science and Practice of Pharmacy
d) Compendium of Pharmaceuticals and Specialties

48. Which of the following alternatives is the most appropriate recommendation for the
pharmacist to suggest for switching AE's osteoporosis therapy?
a) Teriparatide
b) Etidronate
c) Denosumab
d) Raloxifene

49. 35 yr. old man and his family going for vacation to island for 3 weeks. He took 2 doses of
dukoral vaccine 3 years ago and his wife never had
Dukoral before, as a pharmacist what would you
suggest for traveler diarrhea prophylaxis
a) Father needs Dukoral 1 shot
b) Mother need 2 shot as she never vaccinated
c) Child need 1 dose.
50. What could increase the risk of getting traveler's diarrhea?
a) Ranitidine
b) Echinacea
c) Garlic
Reassess the need for proton pump inhibitors and H2‐receptor antagonists in individuals travelling to areas
with high rates of travellers’ diarrhea since these agents increase the risk of acquiring intestinal pathogens
because they inhibit the HCL production.

51. What could be given to reduce the effects?


a) Milk
b) Orange juice
c) Coffee
d) Carbonated drinks
Dairy products, alcohol, caffeine, prune juice, orange juice and apple juice should be avoided.
But when it comes to stomach distress, many people view a cup of flat soda as just what the doctor ordered.
The quick and popular remedy — usually in the form of cola, ginger ale or clear sodas — is said to help settle
the stomach with its slight fizz and replenish fluids and glucose lost by vomiting and diarrhea

52. Dukoral vaccine, left outside for 3 days, what should you do?
a) Call the manufacture for advice on what to do
b) Return back to fridge until expiry and call manufacture for credit loss
c) Discard of the vaccine as it is no longer effective
Store at 2 to 8°C. DO NOT FREEZE.
The vaccine can be stored at room temperature (up to 25°C) for up to two weeks on one occasion only. After
mixing with the buffer solution the vaccine should be consumed within 2 hours. The sodium hydrogen
carbonate sachet may be stored separately at room temperature (up to 25°C).

53. During vacation children


complained diarrhea, father
called pharmacist for antibiotic
for child, AS pharmacist what
will you recommend?
a) SMX TMP
b) Ciprofloxacin
c) Azithromycin
d) Cephalexin
e) Amoxicillin
54. How to manage a vaccine in the fridge, to keep
it at its proper temperature
a) Keep it in a bar size fridge, just for
pharmaceuticals
b) Adjust the temp from 0-8C
c) Regular size fridge but only keep it in the
door
d) Remove any frost from the fridge
e) Store full bottles of water on empty
shelves and on the door.

55. AC got shingles and after one month get prescription Amitriptyline 10 mg for pain and
pharmacist deliver RX to her home, AC read drug information and decided to not take
medication due to S/E. What pharmacist counseling on phone?
a) This is low dose you may not experience this SE
b) Major side effects are manageable
c) If you have any see your doctor
Another version: A patient has shingles she is complain rash and fever.shs has a prescription of
Amitriptyline 10 mg so you dispense medication and then she came back from home and saying
this drug has alot of sides effect so I don't want to take what you tell her.
a) Explain side effect verses benefit
b) Side effect is well tolerated
c) Side effect not happened all almost pt.
d) Dose is very small to cause side effects.
First, 2 answers can be omitted: C and D.
C because its is NOT a professional response since there's no guarantee this SE won't occur in this patient +
this response does NOT address the patient's concern.
D because some side effects are dose‐dependent but others NOT, and since the Q stem did NOT specify
which SE the patient had a concern with, then we CANNOT pick this answer.
Now, we are left with 2 answers, answers A and B. To pick the right answer, one needs the exact wording of
the Q stem because it'll contain accurate keywords that would guide the correct answer.
Although acute neuropathic pain control is NOT mentioned, this answer seems to be more comprehensive
and addresses the patient's concern by explaining the SEs and clarifying the pain control benefit.
Importantly, Answer B might be correct in other scenarios, depending on the Q stem wording.
The patient tells you: "I don't want to use the medication."
Would your professional answer be, "if SEs occur, see your doctor.”?
Your issue is that you're trying your best to rationalize systematically each and every answer which is a
double‐edged sword and will NOT help with those questions.
Always, rely on YOUR OWN UNDERSTANDING to find the answer that fits the scenario and details the most.
This means that there's NO way to find correlations and prepare in advance for those questions
56. Which parenteral product to be prepared in safety hood or biological cabinet?
a) Pamidronate
b) Infliximab
c) Etanercept
d) TPN
e) Docetaxel

57. Following preparation are prepared in horizontal LAF, Except?


a) TPN
b) Iron preparation
c) Saline solutions
d) Chemotherapy infusion (vertical LAF)

58. 49 yr old patient come for regular checkup, COPD exacerbation last year 2 times and this
year 2 times, he had Pneumonia and influenza vaccination before 2 years ago. AS pharmacist
what will you recommend?
a) Vaccinate for influenza and pneumonia
b) Vaccinate for influenza only
c) pneumococcal vaccine every year
d) Take influenza vaccine now and pneumococcal vaccine in 3 years

59. Depot medroxyprogesterone, what to monitor?


a) Calcium
b) Hight
c) Weight loss
Use of depot medroxyprogesterone acetate intramuscular (DMPA‐IM) reduces serum oestrogen levels and is
associated with significant loss of BMD due to the known effect of oestrogen deficiency on the bone
remodelling system. Bone loss is greater with increasing duration of use; however, BMD appears to increase
after DMPA‐IM is discontinued and ovarian oestrogen production increases.
This loss of BMD is of particular concern during adolescence and early adulthood, a critical period of bone
accretion. It is unknown if use of DMPA‐IM by younger women will reduce peak bone mass and increase the
risk for fracture in later life i.e. after menopause.
A study to assess the BMD effects of DMPA‐IM (Depo‐Provera) in adolescent females showed that its use
was associated with a statistically significant decline in BMD from baseline. After discontinuing DMPA‐IM in
adolescents, return of mean BMD to baseline values required 1.2 years at the lumbar spine, 4.6 years at the
total hip and at least 3.4 years at the femoral neck. However, in some participants, BMD did not fully return
to baseline during follow‐up and the long‐term outcome is not known in this group. In adolescents, Depo‐
Provera may be used, but only after other methods of contraception have been discussed with the patients
and considered to be unsuitable or unacceptable. Adequate intake of calcium and Vitamin D, whether from
the diet or from supplements, is important for bone health in women of all ages.
60. Pharmacist administer vaccine to pt, what should be the most important thing to monitor?
a) Record any post vaccination allergy in pt. record
b) Update pt. immunization record
c) Record any past allergy

61. What is with doctor and pharmacist


a) Change drug when DDI
b) Change refill

62. Doctor is busy and not replying any suggested changes by pharmacist as change drug from
sedative to less sedative?
a) Call doctor
b) Give written recommendation to patient and talk to doctor in next visit
c) Take note that doctor not following disciplinary
d) Ask patient to call doctor

63. Medical reconciliation, what is best source for best medical information?
a) Family doctor
b) Community pharmacy
c) Pt close relative
d) Hospital previously admitted
Reliable source of information: Community pharmacy, Drug vials, Family physician, Patient and family
interview, Insurance claims data & Hospital data (from prior visit).

64. Same case, what is best to write for drug in pt record


a) Dosage form
b) Dose, & route of admin
c) Pharmacy name
d) Last refill
Medication Reconcilliation (MedRec)
 Process of creating the most accurate list of all possible medications a patient is currently taking
(Drug name, Indication, Dose, Frequency, Route)
 Completed ideally within 24 hours from admission to hospital & transfer.
 Ensure medication history is consistent across all transitions of care

65. New Clinical pharmacist in hospital start new therapy, what he will do to manage
therapeutic effect?
a) Follow other HCP
b) Frequent ward visit
66. Medical intern gone in hospital and has given MTX for 15mg/ day instead of weekly, this
error was happened during generation of Computerized medication entry, how will you prevent
future error?
a) Use automated system input for MTX weekly basis rather than daily in future
b) Train hospital staff
c) Send the intern for the further training and education

67. Inventory in pharmacy department why is UPC code and DIN is better than just DIN?
a) Compare stock
b) No. of pack size
c) No. of tablets
d) Same medication different package size

68. Inventory reconciliation improved by except


a) Tag every item when stock in shelf
b) Inventory when stock put on shelf
c) Calculating by invoice
d) Good sold record

69. Pharmacy manager want to put new rules for narcotic reconciliation to reduce error related
to narcotic. What would be appropriate step?
a) Assign pharmacy person for regular interval inventory
b) Pharmacist do reconcile when Rx fill
c) Delegate work to a volunteer
d) Do narcotics check annually
e) Computer generated sales record will be kept with prescription record

70. Nurse order morphine injection for break through pain in emergency. On duty Pharmacy
technician received RX, what is her best action in very busy pharmacy?
a) Fill as written
b) Tell nurse that we don’t have time
c) Prioritize and talk to pharmacy staff (Triage= prioritize)
d) Deliver to ward when it is ready.

71. Vaccination to child, discussed options and let her decide


a) Fidelity
b) Autonomy
c) Actimy
d) Paternalism
72. Pharmacist belief that he cannot fill OCs
a) Send to another pharmacy

73. JG your customer, in your pharmacy one angry man shout on her and she screamed, she
looks afraid, you are watching these, what is your action?
a) Call police
b) Call JG for stand aside and asked incident in private
c) Interrupt them and say JG you need any assistance
d) Call that man get out from pharmacy
e) Not your business

74. MK 160 lb RX amoxycillin 15 mg/kg in 100 ml D5W bag which contain 10 ml extra to
nominal volume, you have 5% amoxycillin in your hand. How many ml you will add to 100 ml
bag and withdrawn from the bag?

75. PJP present if the patient has CD4 < 200, what prophylaxis in HIV +ve patient?
a) Sulfamethoxazole / trimethoprim

76. HIV scenario. CD4 = 586 and Viral load was 97000/cell. What is a reason to start anti-
retroviral Tx?
a) Viral load
b) CD4 count
CD4 count and percentage are useful in determining where a patient lies in the continuum of HIV disease
and the need for specific intervention. Knowledge of the CD4 count can also help to narrow the differential
diagnosis in a symptomatic HIV infected patient. In adults, a CD4 count of 430–1360 cells/mcL (0.43–1.36
Giga/Litre or G/L) is considered normal in most laboratories.

77. Patient is on zidovudine. which of the following you will monitor?


a) Lipase
b) Creatinine
c) Potassium
d) CBC
Monitor: CBC, Hgb
Anemia: (Hgb <7.5 g/dL or decline >25% from baseline) discontinue drug until recovery of marrow evident
Neutropenia: (granulocyte <750 cells/mm³ or decline >50% from baseline) discontinue drug until recovery of
marrow evident
Haematological Adverse Reactions: Anaemia (usually not observed before six weeks of Zidovudine therapy
but occasionally occurring earlier), neutropenia (usually not observed before four weeks therapy but
sometimes occurring earlier) and leucopenia (usually secondary to neutropenia) can be expected to occur in
patients receiving Zidovudine. These occurred more frequently at higher dosages (1200‐1500 mg/day) and in
patients with poor bone marrow reserve prior to treatment, particularly with advanced HIV disease.
Haematological parameters should be carefully monitored. For patients with advanced symptomatic HIV
disease it is generally recommended that blood tests are performed at least every two weeks for the first
three months of therapy and at least monthly thereafter. Depending on the overall condition of the patient,
blood tests may be performed less often, for example every 1 to 3 months.
If the haemoglobin level falls to between 7.5 g/dl (4.65 mmol/l) and 9 g/dl (5.59 mmol/l) or the neutrophil
count falls to between 0.75 x 109/l and 1.0 x 109/l, the daily dosage may be reduced until there is evidence
of marrow recovery; alternatively, recovery may be enhanced by brief (2‐4 weeks) interruption of
Zidovudine therapy. Marrow recovery is usually observed within 2 weeks after which time Zidovudine
therapy at a reduced dosage may be reinstituted. In patients with significant anaemia, dosage adjustments
do not necessarily eliminate the need for transfusions

78. Question for mefloquine and calculate tablet required and for children half for child
Start at least 1 wk prior to exposure and continue weekly for 4 wk after leaving the endemic area
➢ Adults: 250 mg base (1 tablet) once/wk PO
➢ Children: 5–9 kg: 31.25 mg base (one‐eighth tablet) once/wk PO
➢ Children: 10–19 kg: 62.5 mg base (one‐quarter tablet) once/wk PO
➢ Children: 20–29 kg: 125 mg base (one‐half tablet) once/wk PO
➢ Children: 30–45 kg: 187.5 mg base (three‐quarter tablet) once/wk PO
➢ Children: >45 kg: adult dose

79. What can cause phlebitis in IV infusion, when given peripheral rather than central IV line
a) Iron
b) Lipid
c) Na+
d) Ascorbic Acid
e) Trace elemnets
Phlebitis. Phlebitis is inflammation of a vein. It is usually associated with acidic or alkaline solutions or
solutions that have a high osmolarity. Phlebitis can also occur as a result of vein trauma during insertion,
use of an inappropriate I.V. catheter size for the vein, or prolonged use of the same I.V.

80. Drug X has new side effects and Doctor call for reference, SE is not mentioned in drug
monograph
a) eCPS
b) TC
c) Martindale
d) Remington
e) Manufacturer information staff

81. Doctor want Ref for treatment of with evidence? Clinical practice guideline
82. Doctor ask for therapy comparison
a) Cps
b) TC
c) CPG
d) Drug facts and comparison

83. In a Clinical pharmacy, pharmacy manager wants to increase 20-50% clinical practice and
clinical coordinator talked him regarding work overload, what is not relevant to check job
description? (not clear)
a) Talk to coordinator to resolve issue
b) Check hours and duty of other pharmacist staff and transfer to clinical department
c) Decrease irrelevant task in pharmacy
d) Update job duties & description of coordinator

84. What is public funded and privately done,


a) Health care in clinic
b) Catheter in clinic also do vision correction laser treatment
c) Cosmetic for sinusitis in cosmetic clinic

85. You have pharmacy in province where, within 3 days prescription filling allowed for
antibiotics. A mother come with child after 6 days for prescription for her child and asked
medicine and said she didn’t get time to fill. What will you do ethically?
a) Refill as written
b) Say No I can’t give
c) Tell her to go walking clinic and get prescription
d) Give her few advanced drugs and say see doctor and come back
e) Call doctor

86. Which drug therapy can be injected in Rheumatoid Arthritis?


a) Hyaluronic acid
b) Fentanyl injection
c) Desmopressin aoetate
d) Septrainjection
e) Meropenem injection
Another version:
Patient on hemodialysis. Want to try something natural for osteoarthritis: Hyaluronic acid
87. What is best source for comparing treatment options after hospitalization (not clear)
a) Rx files
b) Common drug review
c) Martindale
d) CPS

88. What is not primary source


a) Pharmacoeconomics
b) Case report
c) Review article (2ry literature)
d) Stability study
e) Clinical trials
Primary literature: source: peer reviewed Journal Examples: Research articles, pharmacoeconomic studies,
conference papers/posters, thesis (if includes an original research, if it includes a review of primary
literature then it is secondary), patency and case reports.
Secondary literature: Examples of database: MEDLINE, PUBMED, Iowa Drug Information Service (IDIS),
International pharmacy Abstracts (IPA), Scopus, Cochrane library, and CINHAL. Search using key words
Examples: REVIEW ARTICLES (literature review, case review, case series) meta‐analysis, clinical guidelines,
abstracting sources (like MedWatch), newsletters or bulletins and Journals specialized in publishing
literature review
Tertiary literature: Examples: Textbooks, compendia, clinical guidelines, review articles, Pharmacist letters,
online resources such as Medline Plus, Micromedex, etc.

89. Importance of Demographics in clinical trial


a) Clinical trial bias, author for review
b) Can apply statistics
c) Improve author conclusion for different group
Demographics are the characteristics of certain population‐ Demographics should be balanced between
treatment and control group (to minimize bias). P value is usually reported for each category.

90. Patent Medical Pricing Review Board PMPRB role?


a) For generic and patent application
b) Price unit of patented medication
c) Unit dose price for Mfg

91. Medical necessary requirement and sources governed by


a) Health Canada
b) Private sector
c) Province and territory
92. Mother come for recommendation; her child has type I DM his Blood Glucose reading are
as follow: Morning before breakfast 5.2 mmol/L, After breakfast 8.2, Super before 6.2, Super
after 5.9, night 9.9. Child is on Glulisin before every breakfast and glargine 6 unit at bed time,
what is your recommendation
a) Increase Supper glulisine
b) Decrease supper glulisine
c) Take full dinner
d) Light snakes berofe bed time
e) Exercise before bed time

93. Mother came to pharmacy. Her son is diabetic. She had readings for her son’s sugar levels.
She is worried about bedtime sugar levels. Son is using Insulin glargine 14 U night, Insulin
Glulisine 4U (brkfst) 4U (Lunch) 6U (supper). Sugar levels: Before brkfst- After brkfst 7.8
Before lunch - btwn 4-7 After lunch– 7.4, Before supper - btwn 4-7 After supper 5.8 Bedtime
3.9. What you will do as a pharmacist?
a) Add betime snack
b) Reduce dose of bedtime glargine
c) Reduce dose of suppertime glulisine

94. What may cause shortage of drug in country?


a) Generic price increase
b) Quality assurance requirements increase
c) Doctors are not prescribing particular drug
d) Recall of some batches
e) Not profit to mfg

95. Doctor order RX cough syrup 900 ml Dextromethorphan 8.1 gm Phenylephrine 0.81 gm
QS simple syrup 800 ml
Pharmacy technician started preparation and prepared 800 ml of syrup. As a pharmacist how
will you rectify this error?
a) Add 100 ml syrup
b) Add gm DMP and PE

96. Performance review in pharmacy for pharmacy assistants. What is correct?


a) Consider those things only which were indicated at the starting of the year

97. Record keeping for narcotic prescription? 2 years


98. Benzydamine (Tantum) mouthwash directions except:
a) Can swirl before swallow
b) 1:1 dilute
c) Use when needed
d) Food apart
e) Avoid spicy and hot meal
CTMA: For painful ulcers, benzydamine 0.15% topical solution containing ethanol 10% could be used as a
rinse every 3 hours on as‐needed basis. The solution should not be swallowed, and ingestion of food or hot
liquids while the mouth is numb should be avoided to minimize the risk of burning the mouth or biting the
tongue or cheeks.
Monograph: ADULTS AND ELDERLY: Rinse or gargle with 15 ml (approximately 1 tablespoonful) every 1½ to
3 hours as required for pain relief. The solution should be expelled from the mouth after use.
Benzydamine 0.15% w/v Mouthwash should generally be used undiluted, but if 'stinging' occurs the rinse
may be diluted with water. Uninterrupted ttt should not exceed 7 days, except under medical supervision.
4.4 Special warnings and precautions for use
Benzydamine use is not advisable in patients with hypersensitivity to acetylsalicylic acid or other NSAIDs
Caution should be exercised in patients suffering from or with a previous history of bronchial asthma.
Generally be used undiluted, but if 'stinging' occurs the rinse may be diluted with water.
Avoid contact with eyes.
Oroeze/Benzydamine 0.15% w/v Mouthwash contains methyl parahydroxybenzoate which may cause
allergic reactions (possibly delayed). It also contains propylene glycol which may cause skin irritation.
This medicinal product contains 10.26 vol % ethanol (alcohol), i.e. up to 1215 mg per dose, equivalent to 100
ml beer, maximum 71ml wine per dose. Harmful for those suffering from alcoholism. To be taken into
account in pregnant or breast‐feeding women, children and high‐risk groups such as patients with liver
disease, or epilepsy.

99. Citalopram sudden stop Side effects? Suicidal ideation


Antidepressant Discontinuation Syndrome
Rapid discontinuation or dose reduction of most antidepressants may be associated with discontinuation
syndrome. This syndrome is often reported with SSRIs, most likely because these drugs are the most
frequently prescribed, but any antidepressant taken for 6 weeks or more may be associated with
discontinuation‐related symptoms. Patients taking agents with a shorter half‐life (e.g., paroxetine,
venlafaxine) are at greatest risk of discontinuation‐emergent symptoms, which include anxiety, crying,
headache, increased dreaming, insomnia, irritability, myoclonus, nausea, electric shocks, tremor, flulike
symptoms, imbalance and sensory disturbance.
Patients experience somatic, neurologic and psychological symptoms attributed to a rapid decrease in the
availability of 5‐HT within 1–7 days of stopping the drug. If untreated, symptoms typically last between 3
days and 3 weeks, but may occasionally persist for several months.
Inform patients that discontinuation syndrome may occur if they abruptly stop or reduce the dose of their
medication. Taper antidepressant doses gradually by approximately 25% per week and monitor for a re‐
emergence of depressive symptoms. Fluoxetine has a long halflife and can be tapered more rapidly than
other SSRIs.
Reassure patients who experience discontinuation syndrome that the condition is not serious or life‐
threatening and severe symptoms will usually resolve in 3 days or less. The syndrome can be reversed by
restarting the antidepressant and tapering the dose more slowly.
Alternatively, if a slow taper is poorly tolerated, substitute with 1 dose of fluoxetine 10–20 mg PO. If
discontinuation‐emergent symptoms have not resolved after several days, a second dose of fluoxetine 20
mg may be taken if necessary. Consider an herbal product containing ginger if drug interactions or adverse
effects limit use of other possible antinausea medications. There are some evidence that cognitive or
mindfulness‐based therapies combined with tapering may lessen the discontinuation‐emergent effects.
Another novel approach used in the Netherlands involves the use of extemporaneously packaged pills in
doses decreasing by increments as small as 0.5 mg, to be used over several weeks.

100. Pt with COPD exacerbation, Doctor prescribed Azithromycin for 7 days and prednisolone PO
7 days. Reg. medications: Pulmicort (budesonide) 2 puff BID, Combivent Inhaler (Ipratropium
/Albuterol) 2 puff BID prn. Pharmacist dispense 1-month supply of prednisolone by mistake,
Patient come back after one month to your pharmacy. What is the main action pharmacist take?
a) Talked to patient that mistake happened and notify doctor
b) Notify health Canada
c) Make report and give to patient
d) Make report and discussed to all staff members in pharmacy

102. What is wrong about corticosteroid use?


a) Tapering is necessary even within 2 wk use of corticosteroid

103. What to give as antibiotic? Amox/Clav

104. Same above case what symptom can happen


a) Bruise
b) Myalgia
c) Hypotention
A "steroid withdrawal syndrome", apparently without associated with adrenal insufficiency, may also occur
following abrupt withdrawal of glucocorticoids. This syndrome causes symptoms such as anorexia, nausea,
vomiting, lethargy, headache, fever, joint pain, desquamation, myalgia, weight loss and / or hypotension.
These effects are believed to be due to the sudden change in glucocorticosteroid concentration rather than
to low corticosteroid levels.

105. What is oral treatment of Rosacea


a) Metronidazole
b) Doxycycline (Add on topical metronidazole)
c) Clindamycine
d) Ciprofloxacin
106. Patient having profile of arrythmia and hypertension. and on azathioprine. Dr added
allopurinol for gout, what interaction will happen
a) Azathioprine and allopurinol
b) Some other hypertensive and allopurinol
6‐mercaptopurine and azathioprine: Azathioprine is metabolized to 6‐mercaptopurine which is inactivated
by the action of xanthine oxidase. When 6‐mercaptopurine or azathioprine is given concurrently with
Allopurinol, only one‐quarter of the usual dose of 6‐mercaptopurine or azathioprine should be given because
inhibition of xanthine oxidase will prolong their activity.

107. Spironolactone and eplerenone, what is distinct side effect?


a) Hypercalcemia
b) Gynecomastia (Remember: in case of gynecomastia, shift to Amiloride)
c) Dehydration
d) Dizziness

108. Black ethnic Canadian have blood pressure BP 149/95 HR 89, What is drug of choice
a) Perindopril
b) Amlodipine (CCBs and Diuretics are effective in black americans)
c) Candesartan
d) Tamsulosin
Long‐acting dihydropyridine CCBs can be used as first‐line agents. Short‐acting formulations of these agents
(nifedipine) have caused an increase in cardiovascular events in randomized controlled trials and should not
be used. Elderly pts with isolated systolic hypertension & black patients is particularly responsive to CCBs
109. NYHA (III) Pt on spironolactone, Dr prescribed amiodarone, which causes?
a) Hyperkalemia
b) Hypernatremia
c) Hypercalcemia
d) Hypokalemia
e) Hypermagnesemia
Since antiarrhythmic drugs may be ineffective or may be arrhythmogenic in patients with hypokalemia, any
potassium or magnesium deficiency should be corrected before instituting and during Cordarone therapy.
Use caution when coadministering Cordarone with drugs which may induce hypokalemia and/or
hypomagnesemia.

110. pt. with HTN, taking perindopril for long time, has osteoporosis & taking residronate,
meloxicam. 2 weeks before she was diagnosed with gastro enteritis and doctor prescribed her
esomeprazole, chances of fall been increased and before fall she is feeling headache and
dizziness. What increases the risk of fall?
a) Esomeprazole (sure, not ACE)
b) Perindopril
c) Meloxicam
d) Residronate

111. Drug cause stimulate insulin secretion in body


a) Biaguanide
b) Ezetimibe
c) Thiazolidonediones
d) Meglitinide
e) Acetazolamide

112. Pt on ASA low dose, how will you avoid or decrease GI Side Effects?
a) Used with GI protectant
b) Entric Coated
c) Changed to Dipyridamol
d) Use H2RA
e) Eradicate H pylori
Prevention of Consider using a gastroprotective agent for all patients on chronic ASA or NSAID therapy who
PUD during have risk factors for PUD:
ASA or NSAID 1) > 65 years of age
Therapy 2) Use of high-dose or multiple NSAIDs
3) Concomitant use of corticosteroids, antiplatelet agents (e.g., clopidogrel), anticoagulants
(e.g., warfarin, new oral anticoagulants) or SSRIs.
4) Severe comorbidity, e.g., HF, COPD, chronic renal or hepatic disease, malignancy
5) History of gastric or duodenal ulcer or upper GI bleeding
Accepted gastroprotective strategies include once-daily PPIs, misoprostol 800 µg daily (in 4
divided doses) and substitution of a traditional NSAID with a COX-2 inhibitor.
Eradication of H. pylori prior to initiation of ASA or NSAID therapy may reduce the risk of
symptomatic ulcers.
Treatment of  When appropriate, stop ASA or the NSAID. Treat with standard-dose PPIs.
PUD during  Treatment may be stopped 8 weeks after discontinuation of ASA or NSAIDs.
ASA or NSAID  If NSAIDs and ASA cannot be discontinued, continue PPI therapy concomitantly.
Therapy  H2RAs and misoprostol  Less effective alternatives.

113. A patient who is taking a proton pump inhibitor, is in need of a calcium supplement; which
one can be recommended to him?
a) Calcium carbonate
b) Calcium gluconate
c) Calcium citrate
d) Calcium acetate

114. Dr wants to know 3A4 interaction with atorvastatin and grapefruit juice what reference you
will refer?
a) eCPS
b) Remington
c) Merk Mannual
d) Martindale

115. SE of fentanyl patch


a) Aneurism
b) Hypoventilation (in monograph)

116. Cyclosporine drug interaction with following durg (Remember both SE: gingival
hyperplasia)
a) Amlodipine
b) Erythromycin
c) Phenytoin
All inducers of CYP3A4 and/or P‐glycoprotein are expected to decrease cyclosporin levels.
Examples of medicinal products that decrease cyclosporin levels are:
Barbiturates, carbamazepine, oxcarbazepine, phenytoin, nafcillin, intravenous sulfadimidine; probucol;
orlistat; Hypericum perforatum (St. John's Wort); ticlopidine, sulfinpyrazone, terbinafine, bosentan.
117. Pt pregnant on metronidazole oral for bacterial vaginitis, what to counsel this pt.
a) Use barrier method when do sexual activity (no treatment for sexual partner in
bacterial vaginosis, not considered STD- in Trichomoniasis, avoid sexual contact)
b) No alcohol in option (Disulfiram action)
c) Avoid antacid with it (if you are taking antacids containing aluminum, calcium,
magnesium or sodium bicarbonate, or zinc supplements, take them 1 to 2 hours before or
1 to 2 hours after bismuth, metronidazole, and tetracycline)
d) Take empty stomach (during or after meal to avoid GI side effects)

118. Progesterone S. Es?


a) Wt gain (Monograph, weight increase or decease)
b) Menstrual irregularities

119. Estrogen + progesterone? Decrease chance endometrial cancer

120. COCs cause breakthrough bleed at half of cycle, what is your recommendation?
a) Increase EE, Decrease Progestin (first half of cycle)
b) Decrease EE, Increase Progestin
c) No change in EE, Increase Progestin
d) Decrease EE, Decrease Progestin
e) Progestin only pills.

121. Atopic dermatitis, dr prescribed


dexamethasone for 3 months, not
effective now dr changed to clobetasol
relive some but pain is persistent.
What is next therapy?
a) Methotrexate
b) Tacrolimus
c) Mometasone
d) Emollient
e) Betamethasone
122. A patient is being treated with terbinafine tablets for Onychomycosis. Choose the correct
statement regarding terbinafine tablets
a) Treatment of the infection is only required for one month (Regarding onychomycosis of
the toe nails, a 12-week treatment is usually sufficient, although a few patients with poor
nail outgrow may require a longer treatment duration (6 months or longer).
b) Their main side effects are GIT side effects (Abdominal distension, dyspepsia, nausea,
abdominal pain, diarrhoea.)
c) It is okay to still keep breastfeeding while on terbinafine tablets (C/I)
d) Terbinafine tablets can be used in pregnant women (C/I)
e) Store this medication in the fridge (Store in original package to protect from light)
Terbinafine S.E: GIT upset, headache, sensory loss of smell& taste, heavy disturbance, hepatotoxicity

123. What is not use in diabetes?


a) Echinacea (prevent and relieve symptoms of URTI, not used in diabetes)
b) Ginseng (interacts with metformin- If question is what is contraindicated with metformin)

124. Linezolide CI with?


a) Amitryptiline
b) Amlodipine
c) Nitrazepam
d) Moclobemide (serotonin syndrome)
Contraindicated if within 2 wk of an MAOI.

125. RS diagnosed with cataract in both eyes. Doctor order surgery for cataract in right eye after
1 week and on left eye after 8 weeks to right eye. Patient come with rx of Moxifloxacin solution
one drop OU BID, Ketorolac suspension one drop OU BID, Dexamethasone one drop OU BID.
Why doctor prescribed above medications to RS
a) For infection & inflammation
b) For decrease intraocular pressure (no glaucoma medications mentioned here)
c) For decrease red eyes

126. After 8 week there is another surgery in second eye. What to counsel?
a) Discard old eye drops and buy new ones
b) Separate 3-5 min after one drop of each medication to increase absorption
Any changes to postoperative ophthalmic medications should be discussed with the treating ophthalmologist.
Initiate topical antibacterials immediately following surgery rather than waiting until 1st postoperative day
Advise patients to separate the administration of different eye drops by a period of at least 5 minutes. If there
is a contraindication to systemic absorption of the medication, counsel the patient to close the eye and, while
trying to avoid touching the operated eye, put pressure on the inner canthus for 30–60 seconds after instilling
drops. This is done to reduce the transfer of ophthalmic medication to the nasal and/or oral mucosa where it
may be absorbed systemically.
In all other patients, simply closing the eye for 30–60 seconds may be sufficient to maximize ophthalmic
absorption.
Any worsening of vision, floaters or eye redness, especially in the 1st postoperative week, should be
considered endophthalmitis until proven otherwise and requires urgent assessment by an ophthalmologist.
Treatment with many medications in this setting is for a limited course; therefore, any unused ophthalmic
medication should be disposed of properly.
Patients having clear corneal cataract surgery should take all their usual medications (including
anticoagulants and antiplatelet agents) on the day of surgery except for some diabetic medications.
Due to the risk of hypoglycemia, insulin and insulin secretagogue (sulfonylureas, meglitinides) doses may be
modified or the medication may be held the day before or on the day of surgery. Metformin may be held the
day before and on the day of surgery due to the risk of lactic acidosis. If medication doses and/or regimens
are modified, more frequent glucose monitoring may be required.
Monograph moxifloxacin eye drops Method of administration
For ocular use only. Not for injection. Moxivig 0.5%w/v eye drops, solution should not be injected
subconjunctivally or introduced directly into the anterior chamber of the eye.
To prevent contamination of the dropper tip and solution, care must be taken not to touch the eyelids,
surrounding areas or other surfaces with the dropper tip of the bottle.
In order to prevent the drops from being absorbed via the nasal mucosa, particularly in new‐born infants or
children, the nasolacrimal ducts should be held closed for 2 to 3 minutes with the fingers after administering
the drops. After cap is removed, if tamper evident snap collar is loose, remove before using the product.
If more than one topical ophthalmic medicinal product is being used, the medicinal products must be
administered at least 5 minutes apart. Eye ointments should be administered last.

127. After she did the surgery in her eye, she still has many eye drops left in home, she said so
to pharmacist. What to do?
a) She has to buy new eye dps
b) Hold for her till she needs them
c) Give her pred only & use other two she has home
d) Give her other two & continue on pred she has home

128. What is counseling to RS for instilling


eye drops?
a) Shake well before use all eye drops
b) Put 1-2 min after one drop for better
absoption
c) Pinch nasal duct for 1-2 min to
reduce systemic absorption

129. Camphor and menthol when mixed together will produce? Eutectic mixture
Eutectic system is a mixture or solution which the ingredients solidify or liquefy simultaneously. Menthol is
able to form liquid eutectic at room temperature with camphor in the ratio of 8:2, 7:3, 6:4 and 5:5
whereas menthol and borneol in the ratio of 8:2 and 7:3, menthol and WS‐3 in the ratio of 6:4 and 1:1.

130. For what purpose can it be used? Pruritus


Menthol 0.25–3% or camphor 0.25–0.5% in a light, nonperfumed lotion are commonly used in dermatology.
These agents elicit a cooling sensation on the skin, which reduces the sensation of itch. Patients reporting
reduction in itching with application of cool compresses may benefit from topical therapies containing these
agents. Menthol may be preferred as it is less toxic if systemically absorbed.

131. A lady came to your pharmacy with her 2-month-old child. Her child was having Diaper
dermatitis. What is your recommendation?
a) Use cotton cloth diapers instead of commercial diaper
b) Allow to air dry properly between changing the diapers
Air‐drying should be encouraged to diminish damaging effects of occlusion and maceration. Remove diaper
for as long as possible during cleansing, treatment and changes.
Avoid practices that may cause chapping or burns e.g., drying area with a hair dryer, exposure to infrared
lamps.
Use incontinence products with absorbent cores and breathable covers (e.g., most commercial disposable
products) to increase aeration and hinder Candida albicans survival.
Absorptives, antifungals and anti‐inflammatories are reviewed in Pharmacologic Therapy.

132. Non-prescription pharmacologic treatment of diaper dermatitis:


a) 1% Hydrocortisone in clotrimazole
b) 0.5% Hydrocortisone in Mupirocin.
c) Mupirocin 2% in a polyethylene glycol base
d) Nystatin cream

133. Same mother asked about vaccination but she was worried regarding several issues she
read on internet sources regarding vaccination. What could be her concern?
a) Presence of thiomersal in several vaccines
b) Occurrence of Gullian- Barre syndrome due to several vaccines
c) Occurrence of autism with use of MMR vaccine

134. What should be done as a pharmacist if she doesn’t want to vaccinate her child?
a) Tell her about risk of not giving vaccination to child but at the end follow her
decision (AUTONOMY)
b) Tell her that public health agency wants all of the Canadian children vaccinated
c) Tell her that she is putting public health to a risk by this decision
135. Methadone scenario. A huge prescription. Several medications were given. Methadone was
prescribed for pain management and the dose was 100mg/ml. What should be your concern in
this prescription?
a) Methadone dose was too high

136. What should be the pharmacist’s first concern while receiving methadone prescription?
a) Exemption of the prescriber (obsolete question)

137. Same person came to your clinic for methadone earlier than his prescription carries time as
he used more than prescribed. He was not able to tolerate the pain so he used more dose. What
should be done for doing beneficence to the patient?
a) Call the physician for early authorization

138. You are a hospital pharmacist. Your pharmacy is busy and you have lots of prescription
pending. Nurse called and requested immediate supply of morphine for a patient going through
extreme pain. You have shortage of staff today. What you will act with beneficence?
a) Tell nurse that pharmacy is busy and can’t dispense with priority
b) Dispense morphine immediately and send one of the pharmacy staff to nursing unit to
deliver it
c) Ask nurse to send someone voluntarily to pick up morphine

139. Cost of product $2 and retail price was $ 2.49. what is right regarding this.?
a) Gross margin is 0.49$
b) Mark up is 49%
c) Margin is 20% of retail price

140. Some sum. answer was 140 ml

141. How much hydrocortisone 0.5% cream and hydrocortisone powder are required to prepare
a 135g 2.5 % hydrocortisone cream?
0.5% 97.5
2%
100% 2
Each part = 135/99.5 = 1.357
From the HC powder we will need: 2 parts x 1.357 each = 2.7
From the 0.5% HC cream we will need: 97.5 parts x 1.357 each = 132.3 (or simply 135 – 2.7 =
132.3)
142. SUM on TPN: Protein dose to female. Answer was either 106g/day or 1.6 gm/kg/day (not
sure) Carbohydrate 5% and something else. Calaculate total calories.

143. Meperidine was given to patient. He developed signs like twitching in leg and seizure.
a) Due to active metabolite of meperidine

144. 36-month-old child. Had nasal congestion and fatigue. Fever 38ºC. what you will give for
congestion? Nasal saline
The guidelines for treatment of allergic rhinitis in children are similar to those for adults; however, many of
the allergic rhinitis studies exclude children <12 years of age. Health‐care practitioners must ensure they
select the correct dosage, ensure proper administration and minimize adverse effects.
Most second‐generation antihistamines are now available in pediatric formulations for children >6 months
and are generally preferred over first‐generation agents due to improved adverse effect profiles.
Intranasal corticosteroids are also effective and are considered safe in children >2 years of age, depending
on the formulation. Intranasal budesonide and mometasone have not shown growth suppression with
prolonged use at recommended doses. Intranasal beclomethasone, fluticasone propionate and
triamcinolone has been shown to reduce growth velocity by 0.2–0.9 cm per year within the 1st year of
treatment. Longer‐term studies have not been conducted. If intranasal corticosteroids are used, use the
lowest possible dose, monitor growth and use other therapies (e.g., antihistamines) to minimize the dose of
corticosteroid required for symptom control.
Decongestants are not recommended for use in children under 6 years of age. In those children, intranasal
saline drops or spray may be used to clear nasal passages before eating or sleeping.

145. Got symptoms of Croup. And fever 39ºC (Symptoms were given. Croup was not written
there). APAP (paracetamol) and Ibuprofen were given within several hours by mother. What
you will counsel first? Do not use APAP and IBU concomitantly
All options were relevant, we need to prioritize the correct ones
Some clinicians recommend alternating acetaminophen and ibuprofen administration to reduce fever;
however, there is insufficient evidence to support this as a routine practice and it is not recommended.
While alternating or combining acetaminophen and ibuprofen may result in a greater period of time
without fever, the clinical benefit of this difference is uncertain. It is important to note that no difference
was found in patient discomfort in the only 2 trials that assessed it. This practice has not been shown to be
either safe or more effective in improving discomfort than a single antipyretic. In addition, potential risks of
prescribing 2 antipyretics may include parental confusion and dosing errors with associated toxicity.

146. RX for Citalopram X 2 months, Clonazepam X 2 week. Patient has anxiety problem.
Missed flight last week due to anxiety. What you will counsel as pharmacist?
a) Take citalopram on daily basis and clonazepam PRN
147. KT is a 23-yo female who presents to the ambulatory clinic this morning complaining of
pain and burning on urination with an increased need to urinate. She has not seen a physician
recently, except to get prescription for birth control pills. She is anxious to be seen as she has
just started a new job and she needs to get to work. If her doctor diagnosed as non- complicated
bladder (cystitis) infection. What is initial therapy?
a) Cotrimoxazole 7 days
b) Nitrofurantoin 3 days
c) Ciprofloxacin 5 days
d) Cotrimoxazole 3 days

148. KT returns to the clinic 10 weeks later with signs and symptoms of another UTI. The
physician asks your opinion on treating her again with another course of TMP/SMX 1 DS tablet
tid for 3 days. What is your appropriate reply?
a) That this is probably a reinfection, and therefore, TMP/SMX is appropriate, although you
recommend treatment for 14 days.
b) That it is a relapse due to a resistant organism and would recommend treatment with a
fluoroquinolone.
c) That this is a reinfection with Escherichia coli and a second course of treatment with
3 days of TMP/SMX should be initiated.
d) That KA may be a complicated UTI and should be admitted to the hospital for proper
workup and IV antibiotics.

149. Prescriber gave XYZ drug. Pharmacist thinks YYZ is better. He called doctor but the
doctor did not pick up the call. How to act with beneficence?
a) Do substitution and call the doctor later
b) Give written details to patient so he can consult his physician
c) Tell patient to call doctor

150. Unapproved indication (most authenticated source?)


a) e-CPS
b) MEDLINE

151. Drug and grapefruit interaction. (Most authenticated source?)


a) CPS
b) The merck manual
c) Martindale

152. Rare SE. in animals? Case report. Rare diseases  case control
153. PTSD. Which SSRI is best?
a) eCPS
b) CTC
c) Rx Files

154. LAY language? eCPS

155. Doxycycline? With food


The capsules should be swallowed with plenty of fluid in either the resting or standing position and well
before going to bed for the night to reduce the likelihood of oesophageal irritation and ulceration.
If gastric irritation occurs, it is recommended that Doxycycline Capsules be given with food or milk. Studies
indicate that absorption of doxycycline is not notably influenced by simultaneous ingestion of food or milk.

156. Animal bite question? Amox/ CLAV

157. Meningitis question. Pseudomonas positive.


a) Ceftazidime
b) Piperacillin/Tazobactam
c) Etrapenam

158. Patient 46 years old male. Long-term


diabetes mellitus, having constipation from
last yr. Started with 1 tablet of bisacodyl and
over a year increased up to 5 per day. What is
risk factor for constipation?
a) Age
b) Gender
c) Diabetes mellitus

159. What to give for his constipation?


a) Senna
b) Docusate
They can continue on the stimulant laxative,
but there will be a risk of "lazy bowel".
An alternative would be prucalopride.
160. Drug A and Drug B for dyslipidemia. What to check for effectiveness?
a) Reduction in TG
b) Reduction in LDL
c) Reduction in CV risk
For RCT purposes, this depends on the endpoint, whether "true" or "surrogate".
Otherwise, for therapeutic monitoring, it's LDL, Apo‐B, or non‐HDL.

161. Tobramycin (Peak trough) (learn peak trough … 6 questions on it)


Tobramycin IV 400 mg/24 hr was given to a patient, peak conc. is > 20mg & rough conc.
required is < 0.5 mg. post dose peak was 28 mg/L.& after 10 hr. was 7 mg/L. find the T1/2 &
the correct dose.
Answer:
Log C = Log Cₒ ‐ (k*t /2.303) Log 7 = Log 28 ‐ (10K /2.303) k=0.14
T1/2= 0.693 / K = 0.693 / 0.14 = 4.99 hr (5 hr.)
400 mg / 24 hr ‐‐‐‐‐‐‐ 28 mg/ L X mg / 24 hr ‐‐‐‐‐‐‐ 20 mg / L X = 20*400 / 28 = 285.7 mg
285.7 mg ‐‐‐‐‐‐‐ 24 hr 400mg ‐‐‐‐‐‐‐ Y Y = 400*24/285.7 = 33.6 hr.

162. Dementia patient. Physician already ordered CBC and electrolyte, which of the following
you will not order for diagnosis of dementia?
a) Folic acid
b) Vit b12
Dementia
Careful history with attention to memory impairment and potentially reversible causes, e.g., medications,
vitamin B12 deficiency, hypothyroidism, depression. Cognitive impairment can be assessed using the
Montreal Cognitive Assessment (MoCA) or Mini‐Mental State Examination (MMSE); functional disability is
measured with tools such as the Disability Assessment for Dementia (DAD) or the Functional Assessment
Staging Tool (FAST). Medication history is important to rule out drug‐induced cognitive impairment.
Anticholinergic side effects of medications can lead to cognitive impairment. Having numerous prescribers, a
frequent occurrence in elderly patients, is a risk factor for polypharmacy with anticholinergic agents. The
Anticholinergic Cognitive Burden Scale was created to assess the risk of cognitive impairment associated
with the cumulative use of anticholinergic agents.
A few examples of drug classes commonly associated with anticholinergic effects are:
 antiemetics/antivertigo agents, e.g., dimenhydrinate, promethazine, scopolamine
 antihistamines, e.g., diphenhydramine, hydroxyzine
 antimuscarinics, e.g., darifenacin, fesoterodine, oxybutynin, solifenacin, tolterodine
 antipsychotics, e.g., chlorpromazine, clozapine, olanzapine
 tricyclic antidepressants, e.g., amitriptyline, clomipramine, desipramine, doxepin, imipramine,
nortriptyline, paroxetine
An association between the long‐term use of benzodiazepines and dementia incidence has been
demonstrated; this may be explained by the anticholinergic effects of many benzodiazepines.
Physical examination to identify the cause, which is rarely reversible.
Other tests:
Laboratory tests: CBC, electrolytes, kidney function, TSH, vitamin B12, calcium, blood glucose.
In order to assess for prodromal dementia or “very early Alzheimer disease without dementia,” researchers
have attempted to use plasma and cerebrospinal fluid biomarkers (e.g., amyloid beta, tau protein);
however, the results from these studies are rarely applicable outside of the research setting.
Neuroimaging (usually CT head scan) if: <60 years of age, new onset & rapid progression, post‐head injury,
focal or lateralizing signs, history of cancer, use of anticoagulants, early urinary incontinence and gait
disorder & unusual cognitive symptoms

163. Pharmacist want to increase awareness for new dementia program in his pharmacy. Which
of the following is best measure?
a) Call the local caregivers of elderly patient: Simple and direct
b) Give letter to physician
c) Give brochures with all prescription dispensing
d) Give advertisements in local news papers

164. Aripiprazole counselling? Sedation

165. Patient is heavy smoker. Started bupropion therapy. After one week came back. Could not
control the urge and smoked several cigarettes in stressful condition. What will you do as she
had uncontrollable craving?
a) Add Nicotine gum PRN (not in combination with bupropion)
b) Stress management (CBT): Answer
c) Add varenicline (superior in men only, question in women)
Despite concerns over additive side effects, such as nausea, headache or dyspepsia, when combining
varenicline with NRT, the combination was found to be well tolerated in 2 separates studies. These 2 studies
yielded contradictory results. One study found the combination to be superior to varenicline alone for
cessation rates; the other found no benefit.
Further studies are needed to assess long‐term efficacy and safety of the combination.
The combination of varenicline and bupropion was studied in a group of people who were unable to reduce
their smoking by at least 50% after 1 week of NRT. Compared with varenicline alone, subjects taking
varenicline plus bupropion were more likely to be smokefree at 8–11 weeks. The combination was
statistically superior in men, but not in women.
Smokers with high nicotine dependence were also more likely to be successful on the combination. This
study was of short duration, but suggests that the combination is safe and may be effective for select
smokers. Further studies are needed to better define the role of this combination.
Bupropion combined with NRT has been studied with mixed results. One study found higher quit rates in the
combination group, but the difference was not statistically significant.
Another study found no difference among bupropion monotherapy, NRT monotherapy or the combination.
A third study in patients with schizophrenia found the combination of bupropion and high‐dose NRT had
greater abstinence rates while using bupropion and highdose NRT compared with placebo and high‐dose
NRT, but relapse rates were high and no difference in cessation rates was found at 1 year. This study was
small and may have lacked statistical power to detect a true difference. Based on the currently available
evidence, the combination of bupropion and NRT cannot be recommended for routine use.

166. Glatiramer storage? Fridge


Store in the original package in order to protect from light. Store in a refrigerator (2°C to 8°C). Do not freeze.
If the pre‐filled syringes cannot be stored in a refrigerator, they can be stored between 15°C and 25°C, once,
for up to one month. After this one‐month period, if the glatiramer acetate pre‐filled syringes have not been
used and are still in their original packaging, they must be returned to storage in a refrigerator (2°C to 8°C).

167. Which of the following cause weight loss? Topiramate

168. Female 125 kg and 26 yr old. Frequent condom breakage. Came for plan B for 2nd time in
past 6 weeks. Difficulty in intercourse. Taking phenytoin from past 4 years. What to counsel
first at this moment
a) Phenytoin reduces effect of LNG
b) Plan B effect reduces with repetitive use
c) If you vomit within 2-3 hrs repeat the dose
The ‘morning‐after’ pill Levonelle (containing levonorgestrel) ‐ you will need a pill that contains 3 mg
levonorgestrel, to take as soon as possible after sex. This is twice the amount of levonorgestrel that women
who don’t take epilepsy medicines usually take Intrauterine device (IUD, also known as a coil) ‐ guidelines
suggest that this could be better than Levonelle at stopping you getting pregnant

169. She wants some ongoing contraceptive. What to give?


a) Spermicide
b) LNG-IUS
c) COC (EE 35 ug-XX)
d) COC lesser
A single dose of levonorgestrel 1.5 mg used within 24 hours of unprotected intercourse prevents 95% of
expected pregnancies. Efficacy is highest if treatment is provided within 24 hours; it can be taken up to 5
days after unprotected intercourse, though the effectiveness declines with increasing delay between
unprotected intercourse and treatment initiation. Unprotected intercourse is defined as no contraceptive
method used, condom breakage, more than 2 OC‐pills missed any time during the cycle, 1 pill missed in the
first week, more than 7‐day pill‐free interval, more than 13‐week interval between DMPA injections or
ejaculation on external genitalia.
The efficacy of levonorgestrel may be reduced with increased body weight or BMI. In March 2014, Health
Canada issued an advisory to inform women and health‐care providers that levonorgestrel ECs are less
effective in women weighing 75–80 kg and ineffective in women weighing more than 80 kg. In May 2014,
the Society of Obstetricians and Gynaecologists of Canada (SOGC) responded to Health Canada's advisory
and concluded that, until further evidence is available, women who do not have access to or do not wish to
use alternative EC methods (such as copper IUDs) should not be discouraged from using levonorgestrel‐only
EC, as it may still provide some benefit. In July 2014, the European Medicines Agency (EMA) completed an
extensive review of levonorgestrel‐only ECs and determined there are limited and inconclusive data on the
effect of high body weight/high BMI on the contraceptive efficacy of levonorgestrel.
Levonorgestrel EC has a good safety record. Side effects include nausea, vomiting, dizziness and fatigue.
Emergency contraception has no effect on an established postimplantation pregnancy.
For patients taking a hepatic enzyme–inducing medication, it is preferable to take a nonhormonal EC, e.g.,
copper IUD. For women unable or unwilling to use the copper IUD, it is an option to take a total of 3 mg (2 ×
1.5 mg tablets) levonorgestrel as a single dose as soon as possible after unprotected intercourse.
Hormonal contraception can be started within 24 hours of levonorgestrel EC use. Backup contraception will
be needed for the first 7 days of hormonal contraception.

170. According to expanded scope of pharmacist, pharmacist can do substitution of therapy, and
physician are so skeptical about that. What will you do to reduce this confusion/doubt of
physician?
Get involved in local physician meetings to talk about new scope of pharmacist and to let
them know about how pharmacist can help better in circle of care.

171. Error happened – medication error, what will you do?


a) Record details in pt. record so he can talk to the physician
b) Give him written summary of error and tell him to take along in physician visit

172. T1DM patient with HTN. Doc want to give antihypertensive. Which of the following
medication require careful monitoring?
a) HCTZ (hyperglycemia)

173. There was a question related to asthma emergency. Related to methylprednisolone

174. Patient was having Urinary incontinence. Several drugs. which was responsible?
I think it was dimenhydrinate
175. Some question related to paroxetine for depression

176. Pt. is taking ACE inhibitors, furosemide and low dose Metoprolol and patient comes for
refill of metoprolol, as a pharmacist what would you monitor?
a) Muscles pain
b) Nystagmus
c) Depression
Fatigue, bradycardia, decreased exercise capacity, headache, impotence, vivid dreams.
Less common: hyperglycemia, depression, heart failure, heart block.
Fewer noncardiac effects due to cardioselectivity.

177. What is NOT a goal in Rheumatoid arthritis patient? To prevent cartilage damage
Goals of Therapy
 Fully control signs and symptoms of the disease, including pain, stiffness and fatigue
 Halt radiographic progression and joint damage
 Maintain physical function and work capacity and maximize quality of life
 Obtain rapid clinical improvement with a goal of 50% improvement within 3 months and ideally
clinical remission.
If remission is not possible, at a minimum the target is low disease activity within 6 months
Remission can be defined using multiple composite disease activity measures. In general, remission means
the absence of disease activity as assessed by a clinician (swollen and tender joints) and the patient (global
assessment of disease activity) as well as laboratory results (CRP and/or ESR). A commonly used tool for
assessing disease activity is the DAS28. When using DAS28, remission is defined as a score of <2.6 and low
disease activity as a score of <3.2. These scores may be used to guide therapy choices and adjustments.

178. Child of certain small age may be 2-3 year. His weight was 7.5 kg. Was prescribed APAP
liquid formulation for certain condition. His Elderly grand mom gave him tablespoon full
instead of teaspoon full. For certain days. Don’t remember the exact last part of the question...
what is most appropriate regarding this situation?
a) Main SE of APAP is CNS depression
b) Give N-acetylcystine for 150mg/kg dose
c) Send to doctor for hepatic check up
179. A patient had symptom of GERD after eating spicy food. From last 7 days. When to refer?
a) Difficulty in swallowing
GERD symptoms range from mild to severe. The severity of symptoms and esophageal mucosal injury
correlate with the total time the esophageal mucosa is in direct contact with acid (at pH <4) per 24‐hour
period.
Mild symptoms do not interfere with daily activity and are usually of low intensity, short duration, not
nocturnal, infrequent (<3 times weekly) and without major complications. Severe symptoms regularly
interfere with daily activities and are usually of high intensity, persistent (>6 months), nocturnal, frequent
and often associated with complications. Dysphagia, defined as difficulty in swallowing (experienced
anywhere from the mouth to the stomach), is an ominous symptom that necessitates endoscopic evaluation.
In contrast, the more common GERD symptom of globus (a continuous feeling of a lump in the throat that
does not interfere with swallowing) is a benign occurrence in acid reflux, does not require investigation and
usually responds to effective acid reduction
Upper endoscopy is not required in typical GERD. Consider endoscopy in patients with:
“alarm features” suggesting an upper GI malignancy: dysphagia (especially for solids), weight loss (>5%),
epigastric mass, anemia or GI bleeding refractory GERD: heartburn and reflux symptoms despite optimal
therapy with twice‐daily PPI 30 minutes before meals for 4–8 weeks.
Other causes of esophagitis: eosinophilic, “pill” or infectious esophagitis
Barrett esophagus (BE): endoscopic screening is recommended in men with chronic (>5 years) and/or
frequent (≥once/week) symptoms of GERD and 2 or more of the following risk factors: >50 years of age,
Caucasian race, presence of central obesity (waist circumference >102 cm or waist‐hip ratio >0.9), current or
past history of smoking, and a confirmed family history of BE or esophageal adenocarcinoma (EAC) in a first‐
degree relative; screening in women is not generally recommended

180. What you will do for this patient?


a) Refer him to physician
b) Give famotidine
Trivial to Mild Gastroesophageal Reflux Disease
Most people with mild symptoms do not seek medical attention and will obtain symptomatic relief with
antacids, alginates or histamine H2‐receptor antagonists (H2RAs). Additional therapy becomes necessary if
these agents fail to sufficiently control symptoms or GERD severity increases

181. Woman with Urge incontinence. Age 50 yr. What you will give? Tolterodine
Nonpharmacologic Choices
When secondary causes have been ruled out or treated, initiate behavioural therapy:
Pelvic floor muscle training (PFMT): although primarily prescribed for stress UI, PFMT may also benefit urge
UI. PFMT consists of performing 12–20 muscle contractions, held for 10 seconds each, 3–5 times per day;
may take 6–8 weeks to see results.
Bladder training (timed voiding): consists of maintaining a voiding schedule that is gradually increased to a
reasonable interval with minimal incontinence episodes, e.g., 1‐hour voiding intervals that are increased
weekly until voiding every 2–3 hours with minimal incontinence.
Biofeedback training: stimulation that teaches the patient to isolate and control pelvic floor muscles
Pharmacologic Choices
Antimuscarinics (darifenacin, fesoterodine, oxybutynin, solifenacin, tolterodine, trospium) are considered
second‐line treatment for urge incontinence when nonpharmacologic options fail to provide adequate
symptomatic relief. These medications have been shown to increase bladder capacity, improve urge
symptoms, enhance quality of life and reduce incontinence episodes up to 50%. However, the clinical
significance in some patients may be small. They can be combined with nonpharmacologic therapies for
added benefit

182. Symptoms of honeymoon phase. What you


will advise?
a) Keep the insulin dose normal and
increase glucose monitoring

183. D.M. level was given. 3 am 5.6 mmol/l, 8


am 11.1 mmol/l What is this?
a) Dawn phenomena (no hypoglycemia at
2 to 3 am- so no Somogyi)
If the blood sugar level is low at 2 a.m. to 3 a.m.,
suspect the Somogyi effect.
If the blood sugar level is normal or high at 2 a.m. to 3
a.m., it's likely the dawn phenomenon.

184. What is DOC in fibromyalgia? TCA


For chronic neuropathic pain 1st line TCA → For depression TCA is a 2nd line
But for depression + neuropathic pain it's not TCA → It's SNRI duloxe ne it's 1st line for depression
Fibromyalgia pain + depression it's duloxetine

185. Gout and hypertension. Which is the drug of choice for gout attack?
Colchicine (Avoid NSAIDs and glucocorticoids)

186. Rash with allopurinol? Shift to Febuxostat

187. A Boy came with prescription for non-inflammatory acne. which is the tx?
a) Topical retinoids
188. What to counsel? Avoid Vitamin A supplements (i think)
Once daily at bedtime to avoid photosensitivity – irritating
Therapeutic Tips of Acne: CTC
To prevent the development of new lesions, topical acne therapies should be applied as frequently as
prescribed to the affected zones rather than to the lesions alone.
Since many acne treatments are irritating, the skin‐care regimen should be gentle; avoid astringents and
abrasives.
Topical retinoids are effective and recommended as first‐line therapy for most patients, but they are often
underutilized due to patients having difficulty with initial irritation. It is important to emphasize techniques
to minimize irritation such as starting with less frequent application (e.g., every 2 days or less) or shorter
contact times (e.g., apply for 2 hours daily then wash off) and slowly increasing as tolerated. Regular use of
a moisturizer can also minimize dryness and irritation.
Topical antibiotic/benzoyl peroxide combinations have short shelf lives: adhere to expiry dates.
Each time an agent is added or changed, allow several weeks of treatment before assessing effectiveness.
Acne is a disease that usually lasts for years. Once control has been achieved, the treatment regimen may be
simplified but some ongoing suppressive therapy may be required. The exception to this is oral isotretinoin,
which can induce prolonged remission

189. After a month came back as there was no improvement with (clindamycin + Benzoyl
peroxide + Adapalene).
a) Tell him that it will take 2-3 months for current condition to improve

190. Now on isotretinoin? Check for suicidal behavior


191. Patient with patches on hand & elbow (Psoriasis). Was on betamethasone valerate, no
improvement. What to give?
a) Tacrolimus (I chose this, i think it was wrong)
Pharmacologic Choices
Choosing appropriate therapy from a number of effective topical and systemic agents can be challenging.
Topical agents such as medium‐high potency corticosteroids, vitamin D derivatives, tazarotene, coal tar
and anthralin are used alone or in combination.
Frequent use of nonmedicated moisturizers in addition to using other therapies, will help restore the barrier
function of the skin.
Phototherapy is typically used only if topical therapies are ineffective on their own. If required, consult a
dermatologist who is familiar with phototherapy and the various systemic therapies. The tars used with UVB
are either crude coal tar 2% or coal tar distillate 10% with or without salicylic acid. UVA therapy following
topical or systemic administration of the photosensitizing agent psoralens (PUVA) is used less frequently
than in the past due to concerns about long‐term adverse effects and because of the increasing use of
NBUVB therapy, which has improved the risk/benefit profile. Phototherapy may be combined with other
topical or systemic therapies.
Systemic therapy with acitretin, cyclosporine, methotrexate or biologic response modifiers is considered
for moderate to severe psoriasis of the body or extremities that is not responsive to topical therapy or
phototherapy.

192. Rosacea: need PO TX. What to give? 1st topical metronidazole then adds oral
Doxycycline/tetracycline/Minocycline
Topical calcineurin inhibitors are a treatment option for patients with limited psoriatic manifestations.
Tacrolimus ointment and pimecrolimus cream are derivatives of cyclosporin that inhibit calcineurin, thereby
blocking the synthesis of inflammatory cytokines and further activation of T cells that contribute to
psoriasis. They are not approved for the treatment of psoriasis, but are considered useful for treating thin
skin or fold areas, despite minimal published data on use or effectiveness. They are reserved as an option for
recalcitrant plaques of the face, genitals and intertriginous areas, where use of potent topical
corticosteroids or irritating agents are of concern.
193. Lice Scenario: pt used permethrin cream twice. Still found live nits and lice in hair.
What is a reason for TX failure?
a) Did not soak hair well in the cream

194. What is the next TX of choice?


a) Vinegar soaking
b) Tea tree oil
c) Pyrethrin shampoo (permethrin 5% cream)
d) Hair comb to remove them
Treatment Failure of head lice treatment may be due to resistance, misdiagnosis, lack of adherence, or
failure of re-infestation.
head lice Q. Treatment with permethrin may fail if hair is not thoroughly soaked. Two bottles are often
needed for thick or long hair.
 If the lice/scabies are truly resistant, switching to another class may be helpful.
 One regimen has been Permethrin 5% cream overnight on scalp.
 Suggested Oral Sulfamethoxazole-Trimethoprim in combination with Permethrin
 Topical crotamiton 10% applied to scalp and left on for 24 hours in adults.
 Oral ivermectin 200 mcg/kg repeated in 10 days or an alternative regimen of 400
mcg/kg repeated in 7 days.
Ivermectin is not approved for use in children weighing <15 kg.
Topical ivermectin 0.5% effective for treatment of head lice, but is not available in Canada.
Topical ivermectin 1% is available in Canada for the treatment of rosacea but it has not been
studied in the treatment of lice or scabies.

195. Endocarditis: Doctor want to do tooth extraction. pt is on piperacillin/Tazobactam. What to


give for prophylaxis?
a) PO clindamycin 600 mg
196. STI. Non gonococcal Uveitis. What should be given to pt?
I think Doxycycline was in option. I don’t remember other options
https://www.ncbi.nlm.nih.gov/books/NBK535411/
Pharmacologic Choices
A test of cure for chlamydia is recommended when the preferred treatment (azithromycin or doxycycline) is
not used, when symptoms persist, when reinfection is suspected, when therapeutic adherence is questioned,
or when the patient is a prepubertal child or pregnant woman.
Canadian guidelines recommend repeat screening of all individuals with chlamydia infection 6 months post‐
treatment.
A test of cure for gonorrhea is recommended when preferred treatment is not used, when symptoms persist,
when reinfection is suspected, when therapeutic adherence is questioned, when previous gonorrhea
treatment failed, when drug resistance is suspected, or when the patient is a child or pregnant woman.
Gonococcal infections of the pharynx are more difficult to treat; a test of cure is also recommended for all
pharyngeal infections.
Canadian guidelines recommend repeat screening of all individuals with gonococcal infection 6 months post‐
treatment. Fluoroquinolones are no longer recommended for treating gonococcal infections in Canada and
should be considered as an alternative treatment only if fluoroquinolone susceptibility is demonstrated or if
local fluoroquinolone resistance is under 5% and a test of cure can be performed. The preferred treatment
regimen for anogenital gonococcal infections is a combination of a third‐generation cephalosporin
(ceftriaxone or cefixime) with azithromycin. In patients with cephalosporin‐resistant gonorrhea or a history
of anaphylactic reaction to penicillin or allergy to cephalosporins, the recommended alternative treatment
regimen is a combination of gentamicin with azithromycin, or gentamicin with doxycycline if there is
evidence of macrolide‐resistant infection or anaphylactic reaction to macrolides.

197. Plan B question; girl came for plan B and pharmacist believes that not to dispense plan B.
How to do beneficence to the pt?
a) Tell her that I cannot dispense Plan B and direct her to nearby pharmacy

198. An error in hospital. Where to report? NSIR

199. KCL solution. Frequently administered concentrated solution to pt by nurse in hospital.


What to suggest? https://www.ismp-canada.org/download/cjhp/cjhp0209.pdf
a) Keep KCL in lock and key with narcotics
b) Label KCL as concentrated electrolyte
c) Dispense premixed pt specific dose from pharmacy

200. There was question on LASA (look alike, sound alike) drugs error?
visual double check method
201. A pharmacy intern dispensed 5 packs of fentanyl patch 5 each. It was found out after some
days. Tried to call the patient but could not reach him. What is appropriate to do?Call police
a) Report the intern to OCS
b) Call insurance company to talk about error
c) Call physician
d) Report forgery

202. What metaanalysis is the best? Combination of many RCTs

203. Patient was prescribed 3 grams of calcium gluconate. 1 gram of calcium is equal to 4.5
mEq. How much would this patient take of mEq of calcium
a) 6.75
b) 13.5
c) 22.5
d) 14
e) 3.5

204. Recently in Canada a system is added which connects all pharmacy with a centralized pt
profile. Why it was added. (most appropriate ans)
a) To reduce polypharmacy by elderly pt (i chose this)

205. Why you will do documentation of compounding? Inventory checking for compounding

206. The hospital wants to input new system and as a part of this they want to prepare a BPMH
for all patients in their hospital. Which is unbiased source to get BPMH?
a) Physician
b) Patient
c) Community pharmacy
d) Hospital pharmacy

207. In hospital new policy. Pharmacist panel regarding intervention of pharmacist in BPMH
documentation. What should not be considered?
a) Remuneration of something cognitive services (don’t remember exactly)

208. 65 yr old bachelor had leg injury. Was getting discharged from hospital. Needed dressing
every day. What is first priority for the patient?
a) Home care nurse
b) OT
c) Physiotherapist
209. Which of the following service is publicly funded and privately delivered?
a) Dermatological plastic surgery
b) Cataract surgery in clinic where laser system is available
c) Physiotherapy in hospital

210. Norfloxacin? empty stomach


Ensure adequate hydration is maintained to prevent crystalluria, which has been reported with the use of
fluoroquinolones (rare).
Ciprofloxacin: take with or without food (food decreases GI adverse effects); do not take within 2 hours
before or 6 hours after calcium (>800 mg) intake; do not crush or chew extended‐release tablets.
Levofloxacin: may be taken with or without food; do not take within 2 hours of iron, zinc or antacids
containing calcium, aluminum or other metal cations.
Moxifloxacin: may be taken with or without food; do not crush or chew. Take medication, 4 hours before or
8 hours after iron, zinc or antacids containing calcium, aluminum or other metal cations.
Norfloxacin: take on an empty stomach (at least 2 hours before or after ingestion of dairy product[s] or a
meal)

211. An old lady known to pharmacist is in the pharmacy today. Pharmacist saw that a young
fellow has grabbed her collar in front aisle and shouting on her and telling her to open her bag.
What you will do?
a) Call police because store environment is disturbed
b) Do nothing its none of your business
c) Approach there and ask if he can be helpful to resolve the conflict

212. A probation officer came for asking some details of an inmate regarding the usage of
drugs. What would you will do to help them in investigation?
a) Tell him you would only release info after receiving consent from the inmate

213. What you’ll dispense first as you got 5 prescription together?


a) NPH insulin
b) Metronidazole for C. Difficle pt
c) Rivaroxaban for DVT pt
d) Topiramate

214. Patient new comer to Canada, came to learn English, came to the pharmacy today to pick
up his antibiotic, how to council:
a) Using pictures
b) Speak then stop repetitively to make the pt nod by his head that he understands
c) Consider his silence is a sign of understanding
215. Clerical activity was too much of a load on a single pharmacist as they were increased
from 20 to 50%. He complained for work load. What you will NOT do as pharmacy manager?
a) Update clerical activities to his current job description (will not solve the problem)
b) Take his suggestion on his role and modifications

216. When to add a drug in formulary even if it is 80% more expensive.


a) If it is available in different dosage form and strengths

217. What you will see for addition of a drug in formulary? Therapeutic index

218. Thyroid disease, what is wrong when using levothyroxine?


a) Requirement of medication decrease during pregnancy

219. After what time you will check TSH level


a) after every 6-8 wk of therapy

220. Patient come with facial and jaw paralysis, he had stroke attack it was mild (right sided
subcortical ischemic stroke), he had some heart condition & ASA allergy, what to choose?
a) Clopidogrel
b) Warfarin

221. What will you measure baseline for myositis during statin treatment?
a) AST
b) CK & dark urine

222. A lady was 72 yr, height 160cm/weight 52kg, having osteoarthritis, what Won’t help her
a) Wt loss (strongest answer)
b) Hydrotherapy (minimally useful for knee or hip OA)
c) Walking
d) Knee brace
e) Foot insole

223. she has ASA allergy, ready for operation


and needed something for pain, what to give
a) Diclofenac gel
b) Capsaicin
224. Which scale to be used to check post stroke quality of life?
a) Apache
b) Child pugh
c) Modified rankin score
APACHE II APACHE II is a severity-of-disease classification system, one of several ICU scoring
FOR ICU systems.
APACHE II score = acute physiology score + age points + chronic health points.
Minimum score = 0; maximum score = 71. Increasing score is associated with increasing
risk of hospital death. Choose worst value in the past 24 hours.
TIMI The thrombolysis in myocardial infarction TIMI Risk Score for UA/NSTEMI estimates
FOR ACS/MI mortality for patients with unstable angina and non-ST elevation myocardial infarction (MI).
FRAMINGHAM The Framingham Risk Score is a gender-specific algorithm used to estimate the 10-year
CVS RISK OF 10 cardiovascular risk of an individual.
YR
NYHA
FOR HF

CHADS2 CHADS2 score (congestive heart failure, hypertension, age ≥75 years, diabetes mellitus,
FOR AF stroke [double weight]). Assess risk of stroke in atrial fibrillation
CHILD PUGH The Child-Pugh score is a system for assessing the prognosis including the required strength
FOR LIVER of treatment and necessity of liver transplant of chronic liver disease, primarily cirrhosis. It
DISEASES provides a forecast of the increasing severity of liver disease and expected survival rate.
MODIFIED Commonly used scale for measuring the degree of disability or dependence in the daily
RANKIN activities of people who have suffered a stroke or other causes of neurological disability.
FOR STROKE
DUKE CRITERIA FOR ENDOCARDITIS
CIOM/RUCAM FOR PREDICT LIVER DAMAGE BY MEDICATION
Child APAT score Auditory Processing Abilities Test (APAT)
The APAT is a nationally-standardized, norm-reference battery which determines
a child's risk of Auditory Processing Disorder (APD), and identifies specific strengths and
weaknesses. It also documents improvement in auditory processing skills, as a result of
therapeutic interventions.

225. The drug is excreted by kidney, Digoxin dose 0.125, its level when the withdrawal 1 hr
after injection is 2.5 ng/L. (Drug is first kinetic by infusion). What is the reason?
a) Prolonged t1/2 due to kidney failure
b) Measured the concentration early before the drug being distributed
Measure trough serum concentrations at least 6 h after administration and adjust the dose to maintain the
serum concentration between 0.6 and 1 nmol/L. The idea of digoxin does not measure its serum level
directly after dose, so it is calculated after waiting for about 6 hours for after the dose.
226. If you know that normal level is <2, what to do?
a) Nothing to do because it is early to measure the serum level
b) D/C for 3 days and restart at 0.625
c) Change to 0.0625 every otherday
d) Decrease the dose to 0.625
Individualize the dosage based on the patient’s age, weight, renal function and concomitant drugs; the
usual range is 0.0625–0.25 mg daily. Given the narrow therapeutic index of digoxin, pay particular attention
to identifying and preventing potential drug interactions.
Digoxin
 Narrow therapeutic index drug. Predominantly renally eliminated (dose 0.12 mg to 0.25 mg), hepatically
metabolized into active metabolite.
 Prolonged distribution phase (6-12 hours). Gastrointestinal metabolism
Significant drug interactions:
 Quinidine, verapamil & amiodarone and hypokalemia-causing drugs like furosemide, thiazides,
corticosteroids, macrolides and tetracycline.
 Patients using amiodarone medication should reduce 30 % - 50% of their digoxin dose.
Digoxin dosing is guided by patient’s cardiac and renal function and disease state being treated.
Since digoxin is available in several different dosage forms, the bioavailability (F) must also be considered.
IV = 1; capsule = 0.9; tablets = 0.7; elixir = 0.8
Digoxin Oral Trough sample Heart Failure: Dosage should be stable for 5–7
and Post-load: at least 6 h after 0.6–1.3 nmol/L. days in patients with normal
IV last dose of loading regimen renal function.
(IV or PO). Atrial Fibrillation: Half-life 35–40 h.
For periodic monitoring, 1–2 nmol/L. Time to steady-state prolonged
sample just before next dose. in patients with ↓ renal function.
For suspected toxicity, Levels taken within 6 h of dose
sample anytime. may be artificially elevated.

227. What decrease mortality in patient with stable angina


a) Statin
b) Clopidogrel
c) BB
d) CCB

228. This patient was frequently using more nitroglycerin, what to give now
a) Amlodipine
b) Felodipine
Dihydropyridines exert their effects primarily by arterial dilation. Amlodipine, felodipine and nifedipine have
been effective in stable angina, though felodipine does not have Health Canada approval for this indication.
229. AMD, a customer come to you and ask for mutivitamin and minerals information for
protection against AMD, what to counsel (BEST)
a) Vitamin and mineral not recommended in asymptomatic patient
b) They will help only wet, not dry
c) Copper is added to decrease risk of anemia

230. Patient with DVT, given LMWH in hospital, doctor wanted to change to step up therapy
after LMWH, what you will suggest? Continue LMWH for 5 days & then give warfarin
Given for 5 days or until INR is greater than 2 for 1‐2 days.

231. How long (minimum) duration for wararin


a) 1 month
b) 3 months
c) 6 months
d) 12 months

232. What is true about warfarin (too many long big option)
a) Cotrimoxazole increase INR with warfarin
233. A question on herbal and warfarin (check)

234. What is drug of choice for BP in pregnancy? Nifedipine Xl

235. Side effect of Cotrimoxazole? Hyponatremia/ Hyperkalemia


236. Pt. is taking Metformin, his HbA1C % 8.2, he is overweight (Make sure not Obese) and he
wants cheaper alternative?
a) Gliclazide

237. HbA1c %, what is the most appropriate condition for glycemic control measured by
glycosylated heamoglobin OR Which one of these HBA1C result is consistent?
a) Pregnant woman in her First trimester
b) African-american female with sickle cell anemia
c) Heart condition
d) Chemotherapeutics for prostate cancer
e) Man 66 yrs has ESRD (End Stage Renal Disease)
Q. Factors that can affect A1C
Factor Increased A1C Decreased A1C
Erythropoiesis Iron defeciency. Use of erythropoietin, iron or B12
B12 defeciency Reticulocytosis.
Decreased erythropoiesis Chronic liver disease
Altered hemoglobin Fetal hemoglobin. Hemoglobinopathies. Methemoglobin
Variable change in A1C Genetic determinants
Altered glycation Alcoholism Ingestion of aspirin, vitamin C or vitamin E
Chronic renal failure Hemoglobinopathies
Decreased erythrocyte pH Increased erythrocyte pH
Erythrocyte destruction Increased erythrocyte lifespan: Decreased erythrocyte lifespan:
Assays Splenectomy Chronic renal failure Hemoglobinopathies
Splenomegaly Rheumatoid arthritis
Antiretrovirals Ribavirin Dapsone

238. Raynaud's phenomena repeated scenario, what does worsen the condition? BB

239. Patients comes to fill prednisolone RX and by mistake you have dispensed 1 month of
supply instead of 1 week, and after 1 month pt. comes for refill, at this time you have
discovered your mistakes of 1 month supply instead of 1 week , what will be a result of Adrenal
Insufficiency?
a) Tinnitus
b) Diarrhea
c) Muscle weakness
d) Somnolence
Symptoms of adrenal insufficiency (Addison Syndrome) are (most common): Fatigue & Muscle weakness,
Loss of appetitie, Weight loss, Nausea and vomiting. Abdominal pain
Other symptoms (less common): Hypotension, dizziness or fainting, Hypoglycemia, Craving salty foods,
Irritability and depression
Lab Abnormalities: hyponatremia, hyperkalemia and anemia
240. How to avoid this mistake in future?
a) At the time of counselling you will explain and counsel about medication and its use
b) Follow up

241. How will you increase Collaborative teamwork in hospital? EXCEPT


a) Be shadow of physician
b) Send pharmacist to pt. care unit
c) Rotate shift of pharmacist frequently
The question is about increasing collaboration between pharmacy and other hospital departments, so this
answer will increase collaboration inside the pharmacy

242. What is the true Tallman lettering?


a) DOPAMine, DOBUTAMine.
b) DOPamine, DOBUTamine.
c) DOpamine, DObutamine

243. An asthma patient. Having glaucoma. What is contraindicated?


a) Timolol
b) Latonoprost
c) Acetazolamide

244. Patient in ICU, what score used?


a) APACHE II
b) Modified rankin scale
c) Child Pugh
d) Child APAT score

245. COPD pt. with acute exacerbation in the ICU, what to give?
a) IV corticosteroid
b) Salmeterol

246. He needs antibiotic, what to give?


a) Amx/clav
b) Doxcycline
c) Levofloxacin
247. You are the pharmacy manager; a relief pharmacist was working and he dispensed to a
patient 5 boxes of fentanyl instead of 5 patches. The right amount was entered and put through
to the manufacturer. The patient cannot be reached at this point in time. What should you do?
a) Call the police to report what has happened
b) Call the insurance company to report the fraud
c) Call the relief pharmacist for disciplinary action
d) Call the doctor of the patient to report what has happened
It is a waste of narcotic. Ask dr. For rx or report to ocs within 10 days

248. What Pharmacist would recommend above pt for topical pain relive
a) Add lidocaine for Jaw pain

249. What is the concern to give for a home


of cognitive dysfunction elderly patient?
a) Memantine for parkinson’s disease
b) Terazosin for hypertrophy
c) Oxybutynin for overactive bladder
(Anticholinergics)
d) Tamsulosin

250. For which of the following is consent needed?


a) Patients who are under 16 years old
b) Patients who are receiving treatment for psychiatric illness
c) Patients who have Parkinson’s disease
d) Patients who have urinary incontinence

251. Patient has depression, with low appetite and sexual dysfunction. What should be given to
him?
a) Mirtazepine
b) Moclobemide
c) Citalopram
d) Venlafaxine
e) Amitriptylline

252. A patient is on the following list of medication


Oxycodone SR 10mg; 1-tab BID (N)
Oxycodone IR 5mg, 1-tab PRN(N)
Nabilone 1mg 1-tab QD(N)
Pregablin 75mg BID
Ramipril 2.5mg QD
Lorazepam 1mg QHS
The doctor recently added methadone to this patient list of medication. Including
methadone(N), list the number of his medications in the order of narcotic: controlled: targeted
and prescription drugs
a) 4,0,1,2
b) 3,0,1,2
c) 4,1,1,1
d) 5,2,0,1
e) 3,2,2,2

253. Side effects of meperidine


a) Seizures
b) Encephalopathy
c) Tremors
d) Skin necrosis

254. A pregnant woman just had a C-section and the physician prescribed her meperidine Q4hr
for her pain. One day later, she told the nurse that she had restless legs and muscle twitches.
What is the reason for her symptoms?
a) The oxytocin delayed effect of the surgery
b) The delayed effect of the epidural after the surgery
c) Neurotoxicity of meperidine
Due to its adverse effect profile, meperidine (pethidine) should not be used for pain that will last more than
3 hours (Meperidine is not considered a first‐line option in the treatment of acute pain and should not be
given for pain that is expected to last more than 3 hours where morphine is a better choice. Limit its use to
short‐term (i.e., 24–48 hours) due to the accumulation of normeperidine, a neurotoxic metabolite that can
cause seizures in some patients and CNS effects such as tremors, hyperreflexia, hallucinations. Avoid
using meperidine in patients with renal failure or liver disease and in those who have received MAOIs in the
past 14 days. In some hospitals, meperidine has been removed from the formulary because of these
concerns.

255. Doctor prescribed (they gave as mg/mL and mL number – calculation gave 5250mg) and
the father wanted the solution as 350mg/ 5mL. for the solution to be made, the pharmacist
checked compatibility and solubility and it was 127mL of distilled water to make up 150mL of
drug X 50mg/mL. What is the amount of distilled water to be used?
a) 47mL
b) 84mL
c) 97mL
d) 107mL
ANOTHER VERSION
Prescription for antibiotic suspension. Total rx was something which was rounded up to 5.25
gm. Of drug. In the dry powder, when u add 127 ml water it gave 50 mg/ml conc. Of total 150
ml. Father wants 350mg/5 ml as he was concerned about large volume to be govien to the child.
How you ll prepare.?
a) 44 ml
b) 59 ml
c) 84 ml
d) 107 ml
50mg/ml (150 ml) (when added 127 ml) so total 7.5 gm in 150 ml. Here 23 ml was occupied by 7.5 g powder.
Now rx was for 5.25 gm. 7.5 gm = 23 ml 5.52 gm =? = 16.01 ml
So, if u ll add 54 ml water it will produce 70 ml of suspension which is 5.25gm/70 ml (so 75 mg in 1 ml) or
nearly 375 mg/ 5 ml
150 ‐127 = 23ml displacement. That 23ml was 50mg/ml x 150ml = 7.5g of powder.
Cross multiply: 7.5g/23ml = 5.25g/X. X = 16.1Ml displaced with 5.25g of powder.
For 350mg/5ml = 70 mg/ml. Solve for mL
so, ml/70mg X 52500mg = 75ml. Then 75ml – 16ml = 59mL to add.

256. What can be given to increase the absorption of medication on an empty stomach?
a) Enteric coated medication
b) Extended release medication

257. RJ is a 48-year-old man who suffers from diabetes and hypertension. Recently, he has been
suffering from constipation. What could be the cause of the constipation?
a) Diabetes
b) Age (65 only)
c) Gender
d) Hypertension

258. Side effect of co-trimoxazole


a) Hyperkalemia
b) Hyponatremia
c) Thyroid abnormality
d) Hyperglycemia
e) Hyper triglycerides
259. Side effect of fluoroquinolones?
a) Tremors
b) Photosensitivity
c) Diarrhea
d) Constipation
e) Skin necrosis

260. Which drug does not cause hirustism?


a) Prednisone
b) Finasteride
c) Minoxidil
d) Phenytoin
e) Tadalafil

261. What to give OTC for the treatment of shingles


a) Lidocaine
b) Calamine
c) Capsaicin
d) Methylsalicylate
e) Menthol

262. A mother who came in asking about vaccines for her child, she seems very skeptical and is
not sure whether to vaccinate her child or no. what can you tell her?
a) I am aware of the recent links between autism and MMR vaccine but no reason to worry
b) All kids should be vaccinated to avoid any later complications and early mortality
c) No need to vaccinate the kids, they will be ok
The Right version:
A mother tells the pharmacist that she is very reluctant to give her child the routine vaccination
recommended for him as per the Canadian National Advisory committee on immunization
guidelines because of her religious belief that good spirits will leave her kid if he receives man-
made vaccines to protect him against natural disease. What is the appropriate advice by the
pharmacist?
a) Canada is a free country and vaccination is optional
b) If she opts not to immunize her child, she might compromise the child's and the
society's health and safety
c) If she opts not to immunize her child, she will have to sign a waiver of liability at the
provincial health authority
d) Vaccination is mandatory according to Canada constitution in order to ensure herd
immunity
263. A post-MI patient, what should be given to him?
a) Low dose atorvastatin
b) High dose atorvastatin
c) Niacin
d) Fibrates

264. Dose to be given as an infusion 900mg, 50mg/hr initially then increase by 50 mg/hr every
30 min until maximum 400mg/hr. how many hours needed.
a) 2.5
b) 3
c) 3.5
d) 4

265. Rifampin is available as 300mg capsules. Calculate the total number of capsules needed
for this prescription. The doses are the following: Adults → 300mg BID, Children more than
1kg → 10mg/Kg. Rx: adults: 4 adults: 20 children who weight 20 Kg, 10 children who weight
30 Kg
a) 36
b) 146
c) 166
d) 206
e) 306

266. What technique followed in the pharmacy before beginning a non-aseptic compounding:
a) Wash ur hands with soap for 30 min.
b) Wash ur hands with soap containing antimicrobial material
c) Wash ur hands with very hot water
d) Use a brush to clean the areas under ur nails.

267. Technician working in a pharmacy told u that there 4 pts waiting outside waiting for their
RX to be filled, which 2 start first?
a) Levothyroxine IV
b) (medication IV) for STEMI pt
c) (Iron med. IV) for pt going for hemodialysis
268. Pt with fever, congested nose, body aches, sore throat, cough. He had these symptoms
from 7 days. And he started taking corticosteroid since symptoms appeared but no effect. What
condition does he have?
a) Common Cold
b) Influenza
c) Sinusitis

269. Sound like/look alike medications…what to do:


a) Verify the indication with each pt.
b) Say the name of the medication with loud voice
c) Double check before dispensing

270. What will cause QT prolongation with clarithromycin:


a) Ketoconazole
b) Cetirizine

271. A Mother for a 4 yr old child came asking about vaccination for her child, wt 2 say?
a) Multi concurrent vaccines can overwhelm his immunity

272. A mother of a child 5 years old, came to you after her child took influenza vaccine by 2
days and told you that her child is experiencing oculo-respiratory symptoms and he is not eating
well in and he got pale, what is the reason for these symptoms to occur?
a) The child is dehydrated.
b) He is small.
c) He has allergy from the vaccine.
d) It is a side effect of the vaccine.

273. FM is a 23-yo female who presents to your pharmacy in a grocery store chain on 11/20.
She asks the pharmacy technician if the pharmacist can recommend something for what she
thinks is a cold. As you approach FW, your notice that she is visibly shaking with a heavy
sweater wrapped around her. She tells you that yesterday she started feeling “terrible.” You
ascertain that her symptoms include myalgias, arthralgias, fatigue, cough, runny nose, and a
stuffy nose. She says that she is not really sneezing, and she doesn’t know for certain if she’s
been running fever because she does not have a thermometer at home, although she states that at
work today her temperature was 100.7 F. She wants you to recommend something to make her
feel better. She just started a new job as a preschool teacher and does not have very much sick
time saved up. FM wants to know how long she will have to endure this misery. The average
duration for influenza infection in adults is:
a) 3 wks to 5 wks
b) 5 to10 days
c) 4 to 7 days
d) 1 to 21 days

274. During a review of her medications, C.B. a 73 years Old female tells the pharmacist that
she takes levothyroxine every day in the morning with breakfast along with other medications
that include calcium supplement for osteoporosis and lansoprazole for gastric protection. She
has been doing this since she started taking calcium 6 months ago. The pharmacist contacted
the physician’s office to inquire about her TSH. The latest TSH was conducted 3 weeks ago
and it was within normal range. No recent changes to levothyroxine dose were made in the last
6 months. What recommendation should the pharmacist give C.B.?
a) Switch the calcium salt she takes to a less interactive one
b) Discontinue calcium therapy and start vitamin D 1000 units daily
c) Take levothyroxine at least 30 minutes before breakfast
d) Ensure that calcium is taken at least 6 hours after levothyroxine
e) Recommend nothing

Answer: B- Bring issue to Children's Aid society

ANSWER: B- Staff salary


MAY 2015

1. Which of the following causes dose related Steven-Johnson syndrome?


a) Carbamazepine
b) Phenytoin
c) Lamotrigine
d) Valproic acid
e) Phenobarbital
Another version: Which causes steven Johnson if titrated fast? Lamotrigine.

2. A pregnant lady at 36-week has UTI, penicillin and ASA allergy what to give?
a) Amoxicillin
b) Fosfomycin
c) Nitrofurantoin
d) Azithromycin
e) SMX/TMP
Nitrofurantoin is usually avoided near term because of the risk of inducing hemolytic anemia in
the fetus or newborn, especially in those with G6PD deficiency; however, this toxicity is rare.

3. Which one is the true Tall man lettering?


a) VINcristine  VINblastine
b) vinCRITINE  vinBLASTIN
c) VINCRISTine  VINBLASTine
d) vinCRIStine  vinBLAStine

4. Pt. Take cortisone for log time, what do u tell him?


a) Take care it may cause HTN, try to reduce salt intake
b) Something not in lay language like glucocorticoid

5. Ph. tech dispensed diamicron gliclazide 30 mg 4 tabs am instead of diovan valsartan 80 mg 1


tab, What will patient feel?
a) Increase pulse rate
b) Decrease pulse rate
c) Decrease blood glucose
d) Increase systolic
e) Increase diastolic

6. How to decrease error? Show the patient drug before dispense


7. 15-month child diarrhea and vomiting for past 36 hours
a) Wait 2-3 days
b) Go to Dr.
c) Give over counter medication

8. The main objective for Common Drug Review


a) Provides access to non-marketed drugs for practitioners
b) Providing recommendation for formulary listing
c) Control price of brand
d) Monitor the marketed health products in Canada
The Common Drug Review (CDR), at the Canadian Agency for Drugs and Technologies in Health (CADTH), is a
pan-Canadian process for conducting objective, rigorous reviews of the clinical, cost-effectiveness, and
patient evidence for drugs. CDR also provides formulary listing recommendations to Canada’s publicly
funded drug plans (except Quebec).
The objectives of CDR are to:
• reduce duplication of reviews by jurisdictions
• provide equal access to timely, evidence-based information and expert advice
• consolidate the submission filing process for pharmaceutical manufacturers.

9. What is NOT related to PIPEDA (Personal Information Protection & Electronic Documents
Act) rules?
a) Disclose personal information to anyone
b) To disclose personal information to third party
c) Use the personal information to be used in the pharmacy
d) To disclose personal information to Regulatory Authority
There are a number of requirements to comply with the law. Organizations covered by PIPEDA must
generally obtain an individual's consent when they collect, use or disclose that individual's personal
information. People have the right to access their personal information held by an organization. They also
have the right to challenge its accuracy.
Personal information can only be used for the purposes for which it was collected. If an organization is going
to use it for another purpose, they must obtain consent again. Personal information must be protected by
appropriate safeguards.
What is personal information?
Under PIPEDA, personal information includes any factual or subjective information, recorded or not, about
an identifiable individual. This includes information in any form, such as:
• age, name, ID numbers, income, ethnic origin, or blood type;
• opinions, evaluations, comments, social status, or disciplinary actions; and
• employee files, credit records, loan records, medical records, existence of a dispute between a
consumer and a merchant, intentions (for example, to acquire goods or services, or change jobs).
What is not covered by PIPEDA?
There are some instances where PIPEDA does not apply. Some examples include:
• Personal information handled by federal government organizations listed under the Privacy Act
• Provincial or territorial governments and their agents
• Business contact information such as an employee’s name, title, business address, telephone number
or email addresses that is collected, used or disclosed solely for the purpose of communicating with
that person in relation to their employment or profession
• An individual's collection, use or disclosure of personal information strictly for personal purposes
(e.g. personal greeting card list)
• An organization's collection, use or disclosure of personal information solely for journalistic, artistic
or literary purposes
Unless they are engaging in commercial activities that are not central to their mandate and involve personal
information, PIPEDA does not generally apply to:
• not-for-profit and charity groups; or
• political parties and associations.
Municipalities, universities, schools, and hospitals are generally covered by provincial laws. PIPEDA may
apply in certain situations.

10. 1000 ml of 10% dextrose w/v dextrose was prepared. How many Kilocalorie will this make
knowing that 1 gram of dextrose gives 3.4 Kilocalories?
Answer:
10 gm ------- 100 ml X gm ------- 1000 ml X = 1000*10/100 = 100 gm
1 gm ------- 3.4 Kilocalories 100 gm ------- X X = 100*3.4/1 = 340 Kilocalories

11. pt. travelling to Madagascar so expect all except:


a) Hep. A
b) Hep. B
c) Amebiasis
d) Giardiasis
The Q rephrased: "ALL of the following infections are transferred by the orofecal route, EXCEPT:"

12. A lady with atopic dermatitis, gave birth. 4 weeks ago, and breast feeding and no allergic or
medical condition for the baby and she is receiving aid from the community and wants a
reliable method of contraception:
a) COC
b) DMPA
c) Nuvaring
d) IUD
e) Diaphragm with spermicide
Contraceptive Choices in the Postpartum Period
Breastfeeding does not provide reliable contraceptive efficacy without a backup barrier method or
alternative, as lactational amenorrhea is difficult to maintain (depends on the mother ensuring consistent,
exclusive breastfeeding, with no supplemental food or fluids given; additionally, menses must not have
returned and the baby must be <6 months of age).
Barrier methods and spermicide can provide lubrication to the hypoestrogenic vagina but are not as
effective for contraception as other methods.
The Canadian Contraception Consensus guidelines recommend progestin-only methods of contraception in
postpartum mothers regardless of breastfeeding status. Progestin-based methods are recommended during
breastfeeding due to their lower associated risk of thromboembolism in the puerperium (i.e., the first 6
weeks postpartum) compared with COCs, and their neutral effect on milk supply and establishment of
breastfeeding. These methods can be introduced immediately after delivery. Oral formulations must be
taken every day at the same time, without missing a pill, to minimize spotting and maintain contraceptive
efficacy. Women who are breastfeeding or those with risk factors for VTE should avoid use of COCs in the
first 6 weeks postpartum due to an increased risk of thrombosis. In women who are not breastfeeding, COCs
may be started 3 weeks postpartum.
Although expulsion and perforation rates are higher for LNG-IUS when inserted immediately after delivery
compared to 6 weeks postpartum, immediate insertion after delivery, either vaginally or via cesarean
section may be considered.
An IUD can also be inserted immediately after a first- or second trimester abortion. Immediate insertion
appears to be safe and effective and provides convenience. As well, it provides assurance of contraception
postpartum, ensures the patient is not currently pregnant, and decreases the risk of repeat unintended
pregnancies and abortions. Ensure good fundal placement; this can be difficult in the immediate postpartum
state with the larger uterine cavity.

13. Which is case you can't take bismuth subsalicylate:


a) Diabetes
b) Age
c) Renal failure
d) Hepatic failure
e) GERD
4.3 Contraindications
Hypersensitivity to any of the ingredients, aspirin or other nonsteroidal antiinflammatory drugs and patients
with renal disorders. Pregnancy and breastfeeding.
4.4 Special warnings and precautions for use
There is a possible association between salicylates and Reye's syndrome when given to children. Reye's
syndrome is a very rare disease which affects the brain and liver and can be fatal. For this reason, salicylates
should not be given to children aged under 16 years, unless specifically indicated (e.g. Kawasaki's disease).
Do not take with aspirin. Do not take if suffering from gout.
If symptoms are severe or persist for more than 2 days, talk to your pharmacist or doctor.
Do not exceed the stated dose. Keep all medicines out of the reach of children.

14. Methadone  8.1 gm. Na Benzoate  1.15 gm Liquid qs  810 ml


What is the amount of Methadone & Na benzoate to prepare 900 ml?
Answer:
Methadone = 8.1/810 * 900 = 9 gm Na Benzoate = 1.15/810 * 900 = 1.27 gm

15. Patient coming in counselling area tell pharmacist he had noticed that the front shop
employee works a part-time at his restaurant and he is afraid that he will know about his
medication. What to tell him?
a) All medication profiles are confidential and front shop employees cannot have access
to it.
b) All medication profiles are confidential and only pharmacist staff can access it
Does Front shop employees have any access to patient files? NO

16. Travelling to area of malaria so dr prescribed mefloquine, what to tell her?


a) Take 1 day before travel
b) Take on day of travel
c) Take 2 weeks after back
d) Take first thing in the morning on empty stomach on the day of travelling
e) 2 weeks before & 2 weeks after (10 days before, during travel, then 4 weeks after)
For malaria prophylaxis the stated dose of Lariam should be given once weekly, always on the same day.
In order to ensure, before arrival in endemic area, that Lariam administration is well tolerated, it is
recommended to start chemoprophylaxis with Lariam 10 days before departure (i.e. first intake 10 days
before departure and 2nd intake 3 days before departure). Subsequent doses should be taken once a week
(on a fixed day).
Treatment should be continued for 4 weeks after leaving a malarious area (minimum treatment period 6
weeks). The maximum recommended duration of administration of Lariam is 12 months.
The recommended chemoprophylactic dose of Lariam is approximately 5 mg/kg bodyweight once weekly.
17. Mefloquine is contraindicated with?
Contraindicated in patients with a history of seizures, depression, generalized anxiety disorder, psychosis,
schizophrenia or other psychiatric disorders, self-endangering behaviour, suicide attempts or suicidal
ideations.

18. Mum & child got candida infection what is true:


a) Feed the baby with bottle
b) Use cotton swab with nystatin with it on z mouth of z baby
c) It is same to put clotrimazole & feed z baby
d) Put nystatin in his mouth then breast feed him to ensure he swallow it
Nystatin oral suspension- Use and counseling:
 For oral thrush and intestinal moniliasis
 Treatment option for mild uncomplicated disease preferably after meals or after breastfeeding
 Do not eat or drink anything after using Nystatin oral suspension in order to increase contact time
with the mouth surface
 For treatment of infections of the oral cavity, the medication should be dropped directly on the tongue
by means of calibrated dropper and retained in the mouth for as long as possible before swallowing
 For treatment of intestinal infections, may be dropped directly on the tongue by means of the
calibrated dropper, or may be mixed with milk, lukewarm formula, or other non-acid vehicles, or
incorporated in honey, jelly or peanut butter.
How is thrush treated?
Thrush is one condition that you are better off not treating on your own. If you suspect you and your baby have
it, see your doctor or midwife. You and your baby will both need to be treated at the same time, even if only
one of you is showing symptoms.
According to Canadian Paediatric Society your baby will likely be prescribed an anti-fungal liquid medication
that will be given four times a day for two weeks. You may be prescribed an antifungal medication, usually
micozanole or nystatin, to put on the infection. Micozanole is thought to be more effective than nystatin.
Gentian violet, previously recommended as a natural remedy for thrush in babies, should not be used. Health
Canada warns that products containing gentian violet may increase the risk of cancer and not safe.
If you have ductal thrush you will need to be treated with an antifungal medicine. As well as taking medicine,
there are other things you can do to keep thrush at bay:
 Clean all toys, soothers, and breast pump parts in boiling water after each use, to avoid re-infecting
yourself or your baby.
 Avoid using breast pads with a plastic barrier, as they can encourage yeast growth.
 Wash your hands often, especially after applying medication and changing your baby's diaper.
 Use a separate towel for everyone in the family and change it daily if possible.
 Wash your and your baby's clothes at the highest temperature.
 Use only plain water to wash your nipples, and dry them thoroughly afterwards, as thrush flourishes
in damp conditions.
 To ease deep breast pain, you can take a mild painkiller such as acetaminophen until the worst is
over and your treatment begins working.
You may want to add probiotics to your diet to help the friendly bacteria that suppress thrush to grow again
in your digestive tract. You can take probiotics in pill form or by eating live natural yogurt, but don't rely on
them to fight your thrush infection.
Can I still breastfeed when my baby or I have thrush?
Yes. Thrush shouldn't stop you breastfeeding your baby, and you can carry on while you're both being
treated. If you expressed milk during the time that you or your baby had thrush, it is a good idea to throw it
away, as it may re-infect your baby.

19. Federal funding is utilized for the reimbursement of drug purchases of (k type)
a) First nations
b) Veterans
c) Financial contingent

20. What is the lab test for hypercalcemia?


a) Albumin
b) INR
c) TSH
d) PTH
Investigations
 History and physical examination with special attention to: onset and duration of symptoms
(anorexia, constipation, nausea, vomiting, altered mental status, drowsiness, malaise, hyporeflexia,
polydipsia, polyuria, bone pain and muscle weakness are symptoms usually associated with acute
hypercalcemia); chronic hypercalcemia is usually asymptomatic
Laboratory evaluation:
 serum ionized calcium (preferred where available) or serum calcium and albumin; corrected Ca++
value (mmol/L) = (0.02 × [40 − measured albumin g/L]) + measured Ca++ serum intact parathyroid
hormone (PTH); perform this test before giving bisphosphonates, as they can alter serum PTH levels
 serum parathyroid hormone-related peptide (PTHrP) and 1,25-dihydroxyvitamin D level if PTH low
 24-hour urine collection to measure calcium-to-creatinine ratio (if familial hypocalciuric
hypercalcemia or milk-alkali syndrome is suspected)
 serum phosphate, alkaline phosphatase, total protein, serum creatinine and urea

21. Treatment of hypercalcemia?


a) Pamidronate
b) HCT
c) Hydrocortisone

22. Rheumatoid Arthritis case, woman has 2 joint pain in morning in her distal meta (fingers)
she had HTN, when to start therapy for RA
a) Best early
23. Which one is suitable for her? (No MTX in options)
a) Hydrochloroquine
b) Naproxen
c) Ibuprofen

24. Initial baseline monitoring with DMARDs or MTX in RA treatment?


a) Fever
b) Alopecia
c) Serum folic acid
d) Chest X ray

25. Child 4 years old with pinworm and anal itching while other family members have no
symptoms. Father is wondering whether all family members should be treated because of the
cost of the medication:
a) Tell him all family members should be treated
b) Child only needs to be treated
c) Child can be treated with 2 doses and the rest of the family with a single dose
d) Give all the family except grandma
e) All of the bed linen should be discarded & use new sheets
Advise patient regarding:
Proper use of the drug, expected results, management of side effects and need for any repeat
The need to treat infected family members or close household contacts at the same time (unless there is a
contraindication)
- Adjunctive nonpharmacologic measures and proper hygiene
- The mostly innocuous nature of a pinworm infection
- Visiting a health-care practitioner if symptoms recur

26. Dr. prescribed pyrantel pamoate (OTC) for pinworms, how to determine the dose?
a) Weight
b) Age
c) Gender
Pyrantel Acts as a depolarizing, neuromuscular-blocking agent, causing release of acetylcholine and
pamoate inhibition of cholinesterase, leading to paralysis of the worm.
The paralyzed worm releases its hold on the intestinal tract and is expelled.
Adults and children >1 y: 11 mg/kg (base) single dose PO; repeat in 2 wk Maximum: 1g (base)
11 mg/kg base equals 31.9 mg/kg pyrantel pamoate.
Liquid form (50 mg/mL base) should be shaken well before use.
Tablets available as 125 mg pyrantel base.
S.E: Anorexia, nausea, vomiting, cramps, diarrhea, headache, dizziness, drowsiness, Red color
urine, feces. Pyrantel pamoate and piperazine have antagonistic effects; avoid combination.
Avoid in the 1st trimester of pregnancy. Caution in hepatic impairment.

27. If pyrantel pamoate fails what to give next


a) Mebendazole  First-line agent for the treatment of pinworms
b) Metronidazole
c) Fluconazole
A confirmed diagnosis by an appropriate health-care practitioner is recommended before pharmacologic
treatment is initiated to minimize unnecessary exposure to potential adverse effects of the anthelmintics.
Pinworm infections respond readily to drug therapy but reinfection is common.
Treatment failure is usually due to reinfection.
Treatment with the anthelmintics mebendazole (prescription only) or pyrantel pamoate (nonprescription)
results in a high cure rate with minimal side effects.
Albendazole is also an effective treatment but is available only through Health Canada’s Special Access
Program (SAP). Since these anthelmintics do not reliably kill pinworm eggs, it is prudent to repeat treatment
2 weeks later.
Since reinfection occurs easily, prevention (e.g., handwashing) should always be discussed at the time of
treatment. Since pinworms can be passed from one person to another, some experts suggest treating all
close contacts of a person with pinworms even if they are asymptomatic to prevent reinfection and spread
of pinworms. In households where more than 1 member is infected or where repeated, symptomatic
infections occur, this is especially important. The use of laxatives to facilitate removal of pinworms after
anthelmintic therapy is not necessary.
28. Case about bulimia nervosa. Which
one is a non-Pharmacological aspect to be
applied on the patient? CBT

29. What to give her?


a) Mirtazpine
b) Fluoxtine (1st line)
c) Sertraline
Sertraline, citalopram, escitalopram is DOC in
pregnancy

30. Then her case is not controlled what will happen from that
a) Esophagitis (in case of vomiting)
b) Pancreatitis (in case of using laxative)
c) Hepatitis
d) Gastric ulcer
With repetitive vomiting, the esophageal epithelium suffers repeated abnormal exposure to acidic gastric
contents and microtrauma. Consequences of this can include esophagitis, esophageal erosions and ulcers,
Barrett's esophagus and bleeding
31. What is the expected period to see effect from medication?
If therapy is effective, continue for at least 6 months, and preferably 1 year.

32. What decrease the efficacy of treatment: Purging after treatment

33. How to increase the efficacy of treatment?


a) Regulate medication with purging time
b) Use Ondansetron
c) Use Prokinetic it decreases abdominal cramps and vomiting and diarrhea
In Bulimia Nervosa:
1) Avoid QT-prolonging drugs due to the risk of arrhythmias; eg, TCAs, domperidone (prokinetic), etc.
2) Avoid drugs that can cause weight gain or enhance appetite; eg, mirtazapine, paroxetine, or olanzapine,
etc. since these drugs may increase appetite and affect compliance and even worse can increase the vicious
cycle of restriction, binging and purging.
3) Avoid drugs excreted by the kidney such as lithium since toxicity might be enhanced due to recurrent
dehydration and electrolyte disturbances.
4) Avoid drugs that lower seizure threshold; eg, bupropion (whether fie mood or smoking cessation) and
tramadol, etc. since they increase the risk of seizures.
5) Avoid any narrow therapeutic index drug due to the potential impulsivity in bulimic patient to reduce the
risk of suicidal overdoses.
Finally, an answer to that question (to reduce medication loss through purging) would be:
1) Ondansetron, or 2) Administer the medication once daily at bedtime.
Therapeutic Tips
Purging can reduce or prevent drug absorption. When pharmacotherapy is not effective, ask the
patient about the timing of both purging behaviour and dose administration.
A temporary worsening of binge and purge behaviour often occurs during psychological treatment, e.g.,
when a long-suppressed traumatic event such as sexual abuse is uncovered and addressed, or with
significant life stress. This does not indicate a worsening in the patient's overall condition.
Treatment with more than 1 antidepressant at the same time is not recommended—there are
no proven advantages and there is a potential to increase adverse effects and cost.
Response to medication will vary from patient to patient—a trial of several agents may be needed to find
one that is effective. When an antidepressant is effective, continue for 6–12 months.
When discontinuing antidepressants, taper the dose gradually to prevent the development of antidepressant
discontinuation syndrome. For more information see Antidepressant Discontinuation Syndrome in Depression.
Treatment of psychiatric comorbidity is necessary for long-term cure.

34. In dispensing a prescription by law the pharmacist is responsible on all except


a) Accuracy
b) Efficacy
c) Appropriateness

35- In order to weigh a compound that that has 5% accuracy on a balance that has sensitivity
error of 4.5 mg. What is the minimum weighable amount?
a) 4.5mg
b) 9mg
c) 45mg
d) 90mg
e) 180mg
Sensitivity = Weight * error 4.5 = Weight * 5% Weight = 4.5*100/5 = 90mg

36. Least concern when using tertiary reference:


a) Personal bias
b) Accuracy

37. Patient have chronic liver disease, what score used? see in nov. 15
a) APACHE II
b) Modified rankin scale
c) Child Pugh
d) Child APAT score
The Child-Pugh score is a system
for assessing the prognosis —
including the required strength of
treatment and necessity of liver
transplant — of chronic liver
disease, primarily cirrhosis. It
provides a forecast of the
increasing severity of your liver
disease and your expected survival
rate
38. Patient had a lot of chest pain and pain due to fibromyalgia is uncontrolled. He is on many
medications, Bisoprolol 5mg daily, Amitriptyline 10mg prn. What may be the possible reason
for his symptom?
a) Bisoprolol is too low (5mg daily)

39. What DTP of amitriptyline?


a) Too low dose of a correct drug
b) Not taken PRN
there is some evidence supporting the efficacy of amitriptyline 25 mg for the short-term treatment of FM
symptoms, although better studies are needed to specify the magnitude of the effect and its clinical
relevance. There is no evidence supporting the use of higher doses of amitriptyline or using it for periods
longer than 8 weeks

40. After trial of TCA and


gabapentin no improvement; so,
what will u add to her med?
a) Fluoxetine
b) Duloxetine
c) Nortriptyline
d) Tylenol

41. What to monitor to consider that


treatment is successful in this
patient?
a) Return to work
b) Eliminate pain
c) Sleep 8 hours a night
d) Resolution of tingling.

42. Same case of Fibromyalgia, patient


want to take Soya supplements where to
look at the efficiency of soya in
fibromyalgia;
a) PSC
b) Licensed NHPD database
c) Natural products comprehensive database
43. Sinemet Levodopa/Carbidopa monitor all except?
a) Blood glucose
b) Appetite
c) Urinary incontinence
Commonly, levels of blood urea nitrogen, creatinine, and uric acid are lower during administration of
'Sinemet' than with levodopa. Transient abnormalities include elevated levels of blood urea, AST (SGOT),
ALT (SGPT), LDH, bilirubin, and alkaline phosphatase.
Decreased haemoglobin, haematocrit, elevated serum glucose and white blood cells, bacteria and blood in
the urine have been reported.
Positive Coombs' tests have been reported, both with 'Sinemet' and levodopa alone.
'Sinemet' may cause a false positive result when a dipstick is used to test for urinary ketone; and this
reaction is not altered by boiling the urine. The use of glucose oxidase methods may give false negative
results for glycosuria.
Dopamine Dysregulation Syndrome (DDS) is an addictive disorder resulting in excessive use of the product
seen in some patients treated with carbidopa/ levodopa. Before initiation of treatment, patients and
caregivers should be warned of the potential risk of developing DDS
Urogenital: urinary retention, urinary incontinence, priapism.

44. Which of these is the goal for treatment


of acute gout?
a) Reduce inflammation
b) Reduce pain
c) Reduce serum uric acid

45. He is hypertensive and what do you give


him to treat gout?
a) Indomethacin
b) Colchicine
c) Corticosteroid

46. How to treat HTN in pt with gout:


Use of losartan and calcium channel blockers
as treatment for hypertension reduced the risk
of gout.
Diuretics, other angiotensin receptor blockers and
angiotensin-converting enzyme inhibitors and β-
blockers increased the risk.
47. Doctor asks for unapproved use of drug, which reference can you use to find this
information?
a) USP-Di Vol.1
b) Merck manual
c) Micromedex
d) Medline
e) CPS

48. RRR Calculation with 20 event and 12 controlled and calculate NNT?
ARR = 20 – 12 = 8 NNT = 1 / ARR * 100 = 100/8 = 12.5

49. In same study before for amlodipine and metoprolol, given numbers to calculate RRR? not
specifying what control and what not answer was 10.4

50. Calculation of NNT, answer was 18 then?


a) 18 patients needed to show adverse outcome
b) 18 patients needed to show (positive) outcome
c) Overall RRR 35 % drug vs placebo

51. Baby 13 months, her mom wants to wean him:


a) Iron fortified formula
b) Pastuerized cow milk
c) Cow milk-based formula
Whole 3.25% cow's milk (not partly skimmed or skim) is appropriate for infants after 9-12 months of age
who eating iron rich solid foods at most meals & continues to be recommended for the second year of life

52. A lot of hypertensive patient have treatment failure; pharmacist is requested to do


modification on treatment. Who should give him permission?
a) Patient
b) Nurse
c) Family doctor
d) Patient’s wife

53. Pharmacist explain what benefit and risk of adding new drug to patient, and let him decide
after new data given, what ethic principle is upholding here?
a) Autonomy
b) Beneficience
c) Justice
54. Patient came to pharmacy with Rx for prednisone, she says her doctor prescribed it for
increasing egg ovulation hope getting a child, what first question you would ask her? (Pretest)
a) Did your Dr tell you about risk and benefits of using it during pregnancy?
b) Did your Dr tell you about risk of this medicine on fetus?
c) Did your Dr look for a safe alternative, as this medicine has no safe study on this
indication? (NO VALID INDICATION)

55. She will start with high dose of prednisone for 6 months, so what should u counsel her?
a) Photosensitivity may occur so be cautious
b) Restrict the salt because it may cause peripheral edema
c) Prednisone may increase her blood pressure (the patient was already hypertensive)

56. Goal of dementia except


a) Delay long term homes
b) Reverse cognitive impairment
c) Postponed the care giver
Goals of Therapy
- Alter the natural disease progression to meet patient’s and caregiver’s goals
- Treat cognitive, behavioral and psychological symptoms
- Alleviate caregiver burden & Minimize medication side effects

57. Champix Vareniciline start dose 150 once for 3 days then 150 bid what is DTP?
a) Dose too high.

58. Pt 25 yr gets acute attacks of asthma since she was 7 yrs old, she is on Salbutamol and
Fluticasone. Her asthma was exacerbated due to OTC:
a) Ibuprofen
b) Acetaminophen
59. Now she has a severe attack and presented at ICU, what to treat it her with?
a) IV methyl prednisone
b) Switch salbutamol to formeterol
c) Add tiotropium inhaler
d) Switch salbutamol and fluticasone to salmeterol and fluticasone
e) Give oral corticosteroid

60. What to give to prevent future attacks at home


a) IV methyl prednisone
b) Switch salbutamol to formeterol
c) Add tiotropium inhaler
d) Switch salbutamol and fluticasone to salmeterol and fluticasone
e) Give oral corticosteroid
f) Avoid triggers
There is NO need to make any changes since the acute attack was drug-induced.
It's enough to avoid the offending agents.
61. Parkinsonism is related to all except
a) Family history
b) Gender
c) Influenza
d) Age
e) Head trauma
(Risk of developing PD is twice as high in men than
women, but women have a higher mortality
rate and faster progression of the disease.)

62. Furosemide side effects, which is correct:


a) Hypoglycemia (hyperglycemia, hyperuricemia)
b) Hypomagnesemia (hypokalemia, hypocalcemia)
c) Hyperkalemia
d) Hypercalcemia

63. What's false about nicotine withdrawal:


a) Irritability
b) Trouble sleeping
c) Hungry appetite
d) Lack of concentration
e) Nausea and vomiting
Dysphoric or depressed mood, irritability, anxiety, difficulty concentrating, restlessness, increased
appetite/weight gain, GI symptoms, headaches and insomnia.
Symptoms generally peak 24–72 h after the last cigarette and subside after about 2 wk.
Cravings can continue for years, and are probably related to behavioural and psychological aspects of
nicotine addiction

64. Patient is heavy smoker and know that smoking


harm him, he tries to quit and failed so, what stage he
is on?
a) Precontemplation
b) Contemplation
c) Preparation
d) Action

65. He heard about new drug for smoking cessation


so where to look in (i cannot remember)
66. Gum in smoking cessation success %.
a) 1-5%
b) 2-20%
c) 3-40%
d) 4-55%

67. Pt on warfarin, dr prescribed leviteracetam but pharmacist


give levofloxacin instead by mistake, so what to see?
a) Monitor bleeding
Seizures Sedation Agitation Headaches (tons of answers apply)

68. Pharmacist Initial action


a) Document
b) Refer to ER
c) Disciplinary action with tech
Depends on the reaction, but referral is likely; however, the most important action you need to take first is
to ASSESS the patient.

69. What to do to prevent such error?


a) Pt review
b) Decrease space bet ph and tech preparing area
c) Space on shelf
d) Put label
e) Review stock bot with label
CANNOT be answered unless we know the error stage, especially that no universal answers were given.
This must have been clear to you.

70. Drug shortage due to;


a) New Quality Assurance regulations
b) Price competition
c) Wholesalers ordering too much
d) Sale representatives visiting the Physician of the area frequently causing them to
prescribe the medication.
e) Advertisement of the medication at media and customers wanted to purchase the drug.

71. This is the first episode for RL and his symptoms have resolved with pramipexole. what is
the recommended duration for therapy?
a) 6 months
b) 1 year
Lifelong treatment. However, can other answers apply, the answer is always "YES", depending on the
question details and objectives.

72. He can't keep sitting on chair, which side effect that RL suffers from
a) Akathisia
b) Acute dystonia
c) Tardive dyskinesia
d) Late dystonia
RLS often presents as discomfort in the legs that patients describe as creeping, crawling, pulling, or itching;
movement typically relieves this discomfort. Feelings of akathisia also have been described as an inner
restlessness and a need to get up and move to relieve the tension

73. Hepatic patient with no intracellular involvement and was diagnosed with cholestatic
jaundice what to monitor?
a) Albumin
b) ALP (Alkaline phosphatase)
c) Bilirubin
d) AST, ALT

74. Pt taking 10 units NPH breakfast & bedtime & 4 units regular before each meal he mixed &
before dinner took 4 units NPH instead, so what to till patient?
a) Skip tonights doses
b) Take 6 NPH & regular
c) Take 4 NPH & regular insulin
d) Skip regular & take 6 NPH
Take regular insulin and the remaining of NPH

75. Patient taking Insulin NPH before sleep, and regular insulin prior each meal. His glucose
was as follow: Fasting 12-13, Pre dinner 5-7, Pre lunch 5-6.5, at 3:00 am 3-3.5 mmol. What
advice to tell?
a) Shift NPH at night and give regular one instead
b) Give NPH before meal
c) Decrease NPH dose at bed time (somogyi)

76. Long term adverse effect of amiodarone:


a) Bruising
b) Corneal deposition
c) Skin
d) GI
Late Onset ADR: gray man syndrome or bluish discoloration of the skin due to deposition of amiodarone
underneath the skin. (up to 15 years)

77. Amiodarone auxiliary label? Put sun screen bcz its photosensitive
Skin and subcutaneous tissue disorders (see section 4.8)
Patients should be instructed to avoid exposure to sun and to use protective measures during therapy as
patients taking Amiodarone tablets can become unduly sensitive to sunlight, which may persist after several
months of discontinuation of Amiodarone tablets. In most cases symptoms are limited to tingling, burning
and erythema of sun-exposed skin but severe phototoxic reactions with blistering may be seen.
Eye disorders (see section 4.8)
If blurred or decreased vision occurs, complete ophthalmologic examination including fundoscopy should be
promptly performed. Appearance of optic neuropathy and/or optic neuritis requires amiodarone withdrawal
due to the potential progression to blindness. Unless blurred or decreased vision occurs, opthamological
examination is recommended annually.

78. Which one of the following is not performed for a patient on Amiodarone?
a) Renal Function CrCl.
b) LFTs
c) Chest X-Rays
d) Eye Examination
e) TSH
Monitor transaminases and thyroid function Q6 months, CXR annually.

79. What not given with amiodarone?


a) Metoprolol
b) Ramipril
Amiodarone should be used with caution in patients receiving ß-receptor blocking agents (e.g., propranolol,
a CYP3A4 inhibitor) because of the possible potentiation of bradycardia, sinus arrest, and AV block. If
necessary, amiodarone can continue to be used after insertion of a pacemaker in patients with severe
bradycardia or sinus arrest.

80. Treatment of hyperphosphatemia:


a) Calcium carbonate
b) Insulin
c) Sodium polystyrene
If serum phosphate is progressively or
persistently elevated despite a low phosphate
diet (800–1000 mg phosphate/day), start
therapy with a low dose of calcium-containing
phosphate binder (calcium carbonate
or calcium citrate) if hypercalcemia is not present.
The maximum safe dose of calcium-based phosphate binders with regards to vascular calcification is not
known.
If hypercalcemia develops, reduce the dose of calcium-containing phosphate binders and vitamin D
analogues if used. If hyperphosphatemia is still present, the patient may be changed to a non-calcium
containing phosphate binder, such as lanthanum, sevelamer carbonate or sevelamer hydrochloride. The
carbonate salt of sevelamer may help to neutralize uremia-induced metabolic acidosis.

81. Injection drug was recalled from hospital in specific region in province which is just
distributed to hospitals or med recall in special area due to
a) Aseptic technique
b) Adverse effect
c) Issue with cold chain transfer
d) Manufacturing error
e) Unable to meet stability testing

82. BDZ withdrawal symptoms after 2-3 days:


a) Lorazepam
b) Diazepam
c) Alprazolam
d) Oxazepam
SA-BZD: Withdrawal begins within 6-8 hours, peaks around the 2nd day, and begins to improve on the 4th or
5th day.
LA-BZD: Withdrawal may NOT develop for >1 week, peaks during the 2nd week, and improves during the
4th or 5th week.
The Q rephrased is: "Which of the following BZD will result in the fastest withdrawal?"
83. Depression case with sexual dysfunction, hyperlipidemia and was taking Metoprolol,
Atorvastatin, ASA, Sildenafil, Gliclazide, what to give him for depression treatment?
a) Bupropion
b) Mirtazepine
c) Venlafaxine
d) Paroxetine

84. What worsening his case? Propranolol/ Metoprolol

85. Pt come from vacation; what other things pharmacist needs to know to exclude other
hepatitis than hepatitis B?
a) Marijuana use
b) Sexual activiy

86. Alcohol withdrawal case, one of them on what is the difference between alc withdrawal &
opioid withdrawal?
a) Opoid take long time
b) Alcohol more withdrawal effect
c) Opoid more withdrawal effect
d) Alcohol is deadly (Mortality 5%)

87. Case on an alcoholic patient admitted to the


hospital. He is having withdrawal symptoms,
what is a symptom that this patient may
experience
a) Seizures
b) Hallucinations
c) Pinpoint Pupil
88. Upon discharge from the hospital, who would you recommend for this patient
a) Social worker
b) Nutritionist
c) Occupational therapist
d) Physiotherapist

89. Non pharmacological options for alcohol withdrawal


Several psychosocial treatment options are
available, as psychoeducation, relapse
prevention training, trauma therapy,
cognitive behavioural therapy,
motivational enhancement therapy,
interpersonal therapy, network support
(e.g., Alcoholics Anonymous, 12-step), and
group and family therapies.
The combination of motivational
interviewing and cognitive behavioral
therapy is effective in patients with
comorbid depression and anxiety, which is
a common presentation in alcohol use disorder.

90. To reduce dispensing errors with KCl which one of the following must be followed in a
hospital where there are repeated errors by the nursing staff
a) Train the nursing staff
b) Ask nursing staff to call the Pharmacist while dispensing
c) Take the concentrates away from the nursing room and should be in the custody of
the Pharmacist. (Remove It from Ward)
d) Send premixed bags to the ward, put concentrated KCL label

91. Study done between a drug and placebo, the 95% CI 0.7-1.5 so you conclude?
a) Placebo more effective
b) Drug more effective
c) There is a significance
d) No significance
If range of confidence interval crosses 1, it's not significant. Range is already given 0.7-1.5.
If range is before 1 (0.7-0.9), the drug is more effective, if after 1 the placebo is more effective.
If range was 0.7-0.9 then ans would be B
92. Patient with cold sores, after 5 days of feeling of tingling and rash and patient wants
something to decrease duration of symptoms:
a) Refer to give acyclovir oral
b) Refer to give valcyclovir
c) Give docosanol-nothing you can give can accelerate the healing
d) No treatment will reduce the symptoms after this period  More than 3 days- do nothing
e) Topical acyclovir

93. Which is true of cold sores?


a) Organism causes is HSV-1
b) Shedding time …. Days
The development of cold sores is primarily related to 2 herpes simplex viruses (HSVs): herpes simplex-1 (HSV-
1) and herpes simplex-2 (HSV-2). HSV-1, which is most commonly transmitted via saliva, causes the majority
of oral herpes infections; HSV-2, which is present in genital secretions, causes the majority of genital herpes
infections. However, orogenital contact may cause a primary infection of either type in either the oral or
genital region. Both HSV-1 and HSV-2 can cause primary or recurrent infections. When symptomatic, and
involving the oral region, a 1st infection is known as primary herpes gingivostomatitis. Although not
inevitable, 20–40% of patients who experience a primary herpes infection develop subsequent recurrent
herpes infections caused by reactivation of HSV that remains latent in neural ganglion cells.
Recurrences in the oral region most commonly affect the vermilion border of the lips and are known as
herpes labialis, cold sores or fever blisters. Less commonly, recurrences appear on the hard palate,
chin or oral mucosa.

94. New drug in the market where to find its storage conditions: manufacturer leaflet

93. Patient had HTN, Dyspepsia, he had heart burn after dinner last night, he has no problem in
swallowing. but his heart burn radiates to his arm and chest. What is not appropriate to ask?
a) What`s your previous GERD drugs
b) What`s your DM drugs
c) H. Pylori no role in GERD.

94. What the appropriate action to do?


a) Refer
b) Give him Ranititdine before bed time

95. What to council for Pantoprazole xpt


a) Don't chew or crush
b) Need long term ttt
c) Should be spaced from antiacid
d) Taken 30 min before breakfast
96. Ciprofloxacin in 2 questions: Causes dysglycemia
And in the other Q Tendinitis

97. Patient gets tired when he climbs a flight of stairs and walking few minutes in the sidewalk
next to him home, does not get SOB when wearing his cloths and does not feel tired when
resting. which NYHA stage?
a) I
b) II
c) III
d) IV

98. What do you recommend?


a) Give salbutamol inh.
b) Sorry can’t remember the rest mcq

99. what to monitor in the same pt. of HF (he can monitor @ home): weight
100. Question on Lithium, what should be increased while a patient is on Lithium. Which
advice can you tell to the patient who is on Lithium:
a) Avoid caffeine
b) Avoid salt
c) Take plenty water
Take lithium with food or milk. Drink 8 to 10 glasses of water or other liquids every day.
Keep your salt intake about the same. Keep your caffeine intake about the same.
Avoid alcoholic beverages.
Toxic levels may result when adding NSAIDs, ACEIs, ARBs and especially thiazide diuretics.

101. All of the following is true about lithium except


a) Kidney fx test should be assessed prior to therapy
b) Thyroid fx test should be assessed prior to therapy
c) Pregnancy test should be assessed prior to therapy
d) Acute manic episode Treatment reduction in mania within 2days (2-4 weeks to show
efficacy)
Lithium therapy should not be used during pregnancy, especially during the first trimester, unless considered
essential. There is epidemiological evidence that it may be harmful to the foetus in human pregnancy.
Lithium crosses the placental barrier. An increase in cardiac and other abnormalities, especially Ebstein
anomaly, are reported. Therefore, a pre-natal diagnosis such as ultrasound and electrocardiogram
examination is strongly recommended. In certain cases where a severe risk to the patient could exist if
treatment were stopped, lithium has been continued during pregnancy. Studies in animals have shown
reproductive toxicity.
If it is considered essential to maintain lithium treatment during pregnancy, serum lithium levels should be
closely monitored and measured frequently since renal function changes gradually during pregnancy and
suddenly at parturition. Dosage adjustments are required. It is recommended that lithium be discontinued
shortly before delivery and reinitiated a few days post-partum.
Neonates may show signs of lithium toxicity necessitating fluid therapy in the neonatal period. Neonates
born with low serum lithium concentrations may have a flaccid appearance that returns to normal without
any treatment.
102. What is the laboratory sign of hypothyroidism?
a) Low TSH
b) Low free T4
c) Low T3
d) Low Antithyroid Peroxidase

103. Very Low TSH levels in monitoring of a hypothyrodism patient, what should be done:
a) Decrease dose of levothyroxine
b) Increase dose of levothyroxine,
c) Stop levothyroxine for some time then re-initiate

104. A long scenario on a patient who has been losing weight and his TSH values are given,
which show that he has Hyperthyroidism, which one of the following should he be using
a) Levothyroxin
b) Triiodothyronin
c) Methimazole

105. A physician prescribed a fortified eye drops for a patient Tobramycin 13.5mg /ml gtt ou
twice daily. How much of the 40 mg/ml stock solution should be added to 5ml of 0.3% to get
the desired concentration?
Answer:
0.3 % means 0.3 gm ------- 100 ml
X gm ------- 1 ml X=1*0.3/100=0.003 gm = 3mg So 0.3%=3mg/ml
By allegation method:
40mg/ml 10.5 = X

13.5mg/ml X = 5*10.5/26.5 = 1.98 ml

3mg/ml 26.5 = 5ml

106. long case, cr. cl 23mmol/min/1.72


m2...chronic kidney disease, which stage?
a) Stage 1
b) Stage 2
c) Stage3
d) Stage 4
e) Stage 5
107. A lady very swollen hand, erythema, fever, do not sleep, missed 2 days of work; dr
prescribed antibiotic to?
a) Decrease swelling in 24 h
b) Improve sleep
c) Decrease absence from work

108. Pt with renal artery stenosis, newly diagnosed diabetes, he has HTN but he does not want
to start pharmacotherapy.
a) Agree with pt
b) Support Dr decision to Start medications
c) Give Herbal product
d) Non-pharmacological counselling
Missing info. A1c? DOD, he tries diet changes for 3 months or no

109. What to give for HTN?


a) Amlodipine
b) ACEIs
c) BB

110. What to give for diabetes?


a) Insulin
b) Metformin
c) Sitagliptin

111. What to give to pregnant with DVT?


a) Enoxaparin (pregnancy or malignancy, LMWH)
b) Warfarin
c) Dabigatran
d) ASA

112. Mother breastfeeding and wants to increase her lactation what to give:
a) Bromocriptine
b) Metoclopramide
c) Domperidone (dopamine antagonist, increase prolactin, no BBB)
d) Pramipexole

113. A 27 years old female would like to get some natural remedy to help her lose weight. Her
body weight 217 pounds and her height are 5 feet and 2 inches. What is her Body Mass Index?
ANSWER
BMI = MASS (Kg) / Hieght (m)2
Mass = 217 pounds/2.2 = 98.6 Height = (5 x 30.5) + (2 x 2.54) = 157.58 cm = 1.5758m
BMI = 98.6 / (1.5758)2 = 98.6/2.483 = 39.7 Kg/m2

114. Most common cause of gingival hyperplasia:


a) Phenytoin,
b) Valproic acid,
c) Carbamazepine

115. What to do
a) Decrease dose
b) Change drug
c) Use proper oral hygiene
d) Use teeth brush right
e) Use mouth wash /wk
Plaque control is the gold standard of treatment for gingivitis induced by dental plaque (see Oral Hygiene,
Dental Plaque and Caries). To prevent progression to periodontitis and even eliminate gingivitis, encourage
patients with gingivitis to adhere to the oral hygiene regimen recommended by their dentist.
Evidence suggests that toothbrushing in addition to flossing regularly reduces gingivitis.
Destruction of supragingival plaque through frequent professional cleaning and good oral hygiene has been
associated with a beneficial effect on the subgingival bacterial population in moderately deep pockets. In
addition, dental interventions such as scaling and root planing have been shown to reactivate protective
antibodies. If plaque control cannot be achieved manually (e.g., due to lack of dexterity) or in patients who
are systemically compromised or postoperative, topical antimicrobial products may be used as an adjunct to
regular plaque-control measures. Chlorhexidine is effective for the treatment of mild gingivitis.

116. Woman has itchy in her vagina, malodorous &


grey secretion, that is first time for her. Why you
would refer her to Physician?
a) Because its different than C. Albicans
b) Because its her first time
c) Because it could be STD
d) Use OCP
e) Use A.B
The decision to recommend treatment for vulvovaginal candidiasis must be made with care.
Other forms of vaginitis or sexually transmitted diseases (which can have similar symptoms) and allergic
or adverse reactions must be ruled out.
Further evaluation may be required if the patient:
- Is prepubertal—vulvovaginal candidiasis is not common in this group and should be assessed
- Presents with vaginal symptoms for the 1st time
- Has an underlying illness such as diabetes
- Is pregnant
- Has a recurrence of vulvovaginal candidiasis within 2 months of the last episode (complicated cases
may require alternative drug therapy)
- Is immunosuppressed
- Is at risk of an STI, e.g., history of unprotected intercourse, multiple partners, casual sexual
Asymptomatic BV, reassure.
For VVC: Refer if you've any red flags (mentioned fo2). Offer treatment if otherwise.
For BV, Refer any symptomatic BV: That being said, the key is to perform the right assessment in order to
make the correct preliminary diagnosis.
117. Pharmacy technician counsels pt. while you are on phone call & the pt take the medication
& left, what is the appropriate response?
a) Advice to Counsel under supervision
b) Report the technician board about him
c) Advice the tech to council only OTC drugs
d) Document the refusal of pt. to counselling
e) Advice the tech not to do it again in the future

118. In a hospital pharmacy, there is an understaffing in pharmacists and technicians, as a


manager you can do:
a) Automate some of the processes
b) Reduce working hours in weekend
c) Hire student and intern and pay them.
d) Decrease overlaps between shifts

119. Hepatitis A prevention:


a) Immunoglobulin
b) Lamivudine
c) Ribavirin
Pooled immunoglobulins with activity against hepatitis A for Postexposure prophylaxis for hepatitis A within
2 wk of exposure (if exposure is continuous, repeat in 5 months).

120. Reasons for documenting error in hospital pharmacy, all except?


a) For patient interest
b) Do not depend on staff cognitive function
c) For reimbursement
d) To ISMP to penalize pharmacist
(ISMP will NOT penalize pharmacists)

121. Drug abuser with severe cellulitis (swelling, erythematous), what is defense mechanism of
skin protection? (N/A)
a) Alveolar macrophage
b) Sebaceous gland secretion
c) Moist and damp conditions
d) Decrease blood supply to the tissue
e) Availability of bacterial nutrients
122. What antibiotic to give for I.V.
a) Cloxacillin
b) Norfloxacin
c) Ceftazidime
d) Gentamicin
e) Cotrimoxazole

123. How do you know AB gave effect?


a) No fever after 7 days
b) Culture give negative results

124. Aseptic technique prep contamination due


a) Object
b) Personal
c) Technique

125. About CPS (K-TYPE)


a) Contain (7aga sa7)
b) Drugs in canada and america
c) Conversion iv to oral

126. Androgen prescription, where to look for regulations;


a) Health Canada
b) Targeted substances act
c) Controlled drugs & substance act

127. A mother with watery


eyes, foreign body sensation &
pain for 2 days because her
child put his finger in her eyes
what to give:
a) Polymoxin B
b) Refer
Foreign body = trauma, refer to
ophthalmologist to rule out any
injury
128. Hospital pharmacy, why to put policy & regulations?
a) To be eligible for 3rd party funds
b) To ensure quality assurance

129. Pharmacist works for a family health practice and patient with restless leg syndrome
patient has diabetes, start to make hemodialysis, he developed tremors. Doctor asks you to
determine possible other causes of tremor. What may be a contributing disease state?
a) Diabetes
b) Kidney disease
c) Hypertension
d) Gout
Cause of tremors: electrolyte imbalance due to hemodialysis

130. Which vitamins would you give to above patient undergoing hemodialysis:
a) Water soluble vitamins plus vit b complex
b) Minerals
c) Fat soluble vit ADEK
d) Refer to physician to assess his need
Removed by dialysis and cannot build up again by the body
The specialist should determine their need (here it's the most appropriate answer; however, another
applicable answer is water soluble vitamins)

130. He will do hemodialysis, which one of the following must be avoided


a) CBZ
b) Valproic Acid
c) Gabapantin
d) Phenytoin

131. Therapeutic decision was made on unwell designated study on new drug after conducting
trial on small group, what ethic is VIOLATED?
a) Autonomy
b) Non maleficence
c) Veracity
d) Justice

132. Fish contains high amount of:


a) Phosphorous
b) Potassium
133. Lady want to be pregnant soon asking for recommendation for vitamin; what to give her?
a) Multivitamins
b) Multivitamins with iron
c) 1mg folic acid
d) 5mg folic acid
e) Fat soluble vitamins ADEK

134. when to start her vitamins for


pregnancy
a) 1-month after pregnancy
b) First day of pregnancy
c) Continue at end of pregnancy

135. All of the following are true for the Calcitonin salmon spray, except?
a) It cannot be used in Shellfish allergy (right answer- no allergy with shellfish)
b) Calcium and vitamin D are recommended with calcitonin therapy.
c) It is used in alternate dose.
d) Prime when use
One spray (200 IU) once a day
administered intranasally, alternating
nostrils daily into one nostril only. You
should switch between each nostril every
time you use Sandoz Calcitonin NS.
Your physician may prescribe calcium and
vitamin D together with Sandoz Calcitonin
NS to help retard the progressive loss of
bone mass. Upon, first use only, the pump
must be primed. The product should be
allowed to reach room temperature before
priming
https://www.accessdata.fda.gov/drugsatfda_d
ocs/label/2017/020313s036lbl.pdf
Store the unopened medicine in a
refrigerator between 2 and 8 degrees C
(36- and 46-degrees F). Do not freeze.
Once opened store at room temperature between 15 and 30 degrees C (59- and 86-degrees F) in an upright
position for up to 35 days. Throw away any unused medicine after the expiration date.
136. Pt aboriginal has believed that hearing something bad or hearing about harm can happen to
her, what is conflicting with the pharmacist when he is trying to counsel her?
a) Confidentiality
b) Autonomy
c) Veracity
d) Justice

137. Which one is NOT C.I to patient


with Sulfa allergy?
a) Celecoxib
b) Furosemide
c) Sulfasalazine
d) Nitrofurantoin

138. Very complicated patient, had


glaucoma, asthma, sulfa allergy,
doctor wants to start on combo of
dorzolamide and timolol. Drug
disease interaction?
a) Timolol and asthma
(contraindication)
b) Dorzolamide and allergy

139. Ca gluconate 3 grams, if each


1gram gives 4.5 m. Eq of Ca
gluconate, how many m. moles of Ca
in the 3 grams?
Answer: 1 gm ------- 4.5 m. Eq 3 gm ------- X m. Eq X = 3*4.5/1 = 13.5 m. Eq
No. of m. mole = No. of m. Eq / Valency = 13.5 / 2 = 6.75 m. mole

140. A woman is diagnosed with breast cancer due to a gene inherited in the family. she is
living with her mother, her son 16 years old and her daughter 13 years old. she was worried
about her daughter getting breast cancer in the future and was asking you whether she should
tell her daughter to get her tested for this gene. According to autonomy of the daughter what
would your advice her to do:
a) Share the decision with her grandmother
b) Share decision with her brother
c) Tell her daughter to be the decision maker to decide
d) The mother is the decision maker and she should decide for her daughter
Another version:

141. Which needs shaking before use?


a) Nitroglycerin
b) Epipen
c) Tobradex

142. Case of old lady in care home what to do for her


a) Send her medication to home at easily open vial
b) Send her medication in blister back
c) Let her come to take her medication
d) Let her son to take her medication

143. Home care facility asks your pharmacy to apply blister packs for a patient, which of these
will comply the autonomy principle?
a) Ask pt if he want that or something like according his desire

144. 45-year woman obese, height? weight? (BMI = 35) not smoking but parents drink caffeine
4 cups eat too much diabetic so diagnosed for lipids what make u start medications?
a) Age
b) Gender
c) Obesity
d) Diabetes
e) Family history of smoking

145. Hyperkalemia, asymptomatic, ttt with?


a) Na polysterene
b) Insulin with glucose

146. What increases the credibility of Meta-Analysis?


a) Case control studies are included
b) Reviewed by an author before being published
147. Study followed during 5 years between amlodipine and metoprolol, chose pt. by chance.
what study is that?
a) Cohort
b) RCT
c) Case control

148. Patient had cancer; his doctor changed the anticancer drug because it is less effective than
new drug. Unfortunately, new drug costs 10.000 $. Which one of the following actions that you
shall take as a Pharmacist to help that patient?
a) Arrange SAP to him
b) Tell a doctor to return to old drug
c) See if there is manufacture support program or Talk to the manufacturer regarding
the discount of the cancer patient who needs medication for his treatment.
d) Do nothing excuse the patient
e) Ask the Pharmacy for a favor free of cost.

149. Child long hair, treated 2 times with permethrin and still found live nits and lice in hair,
tell mother what is reason?
a) Hair not soaked well

150. what to give:


a) Lindane
b) Pyrethrins/Piperonyl Butoxide
c) Oral clindamycin
Treatment Failure of head lice treatment may be due to resistance, misdiagnosis, lack of adherence, or
failure of re-infestation.
head lice Treatment with permethrin may fail if hair is not thoroughly soaked. Two bottles are often
needed for thick or long hair.
 If the lice/scabies are truly resistant, switching to another class may be helpful.
 One regimen has been Permethrin 5% cream overnight on scalp.
 Suggested Oral Sulfamethoxazole-Trimethoprim in combination with Permethrin
 Topical crotamiton 10% applied to scalp and left on for 24 hours in adults.
 Oral ivermectin 200 mcg/kg repeated in 10 days or an alternative regimen of 400
mcg/kg repeated in 7 days.
Ivermectin is not approved for use in children weighing <15 kg.
Topical ivermectin 0.5% effective for treatment of head lice, but is not available in Canada.
Topical ivermectin 1% is available in Canada for the treatment of rosacea but it has not been
studied in the treatment of lice or scabies.

151. Same case, mum is pregnant & got lice, what to give? permethrin
152. Pharmacy-community project aim at the following except or New pharmacy service will
care for all except
a) Changing the organization or Physical appearance of the pharmacy
b) To be eligible for third party funding or offer from third part insurance
c) Patient care
d) Local medical approval

153. 15 Kg patient taking medication with strength 20 % w/v & rate is 4ml / hr. what is the rate
of infusion per day in gm / Kg.
Answer: 20 gm ------- 100 ml X gm ------- 4 ml X = 4*20/100 = 0.8 gm
So, 4 ml / hr. = 0.8 gm / hr. = 0.8 *24 gm / day = 19.2 gm / day
So, 19.2 gm /Day /15 kg = 19.2/15 gm / Kg / Day = 1.28 gm / Kg / Day

154. Depressed pt discontinued his drug & got withdrawal symptoms, which drug causes it?
a) Bupropion
b) Venlafaxine
c) Fluoxetin
Fluoxetine & bupropion = less withdrawal symptoms
Paroxetine & venlavexine have short t1/2 so they have rapid withdrawal symptoms

155. How do you manage it?


a) Give higher dose of venlafaxine
b) Give a lower dose of venlafaxine
c) Tell him to go home and do non-pharm.

156. Patient gets capsaicin cream. What is true?


a) Burning sensation will go away with regular use
b) Use warm pack to increase absorption
c) Effect in a week
d) If no effect, stop in week
Topical therapies may also be tried as an adjunct to systemic agents where pain relief is not adequate. Both
agents should be applied 3–4 times daily. Maximal effect can take up to 2 weeks for topical NSAIDs and 4
weeks for topical capsaicin. Unfortunately, the tingling and burning sensation caused by capsaicin often
prevents an adequate trial of this medication.
- Pain relief may take up to 2 wk with daily use. Maximum effect can take up to 4 wk.
- Apply with gloves and wash hands thoroughly after application to avoid irritation of other areas.
- Tingling/burning usually decreases within 72 h with repeated use; if effect is bothersome, use lower
concentration or pretreat with topical lidocaine or EMLA cream.
- Do not apply near mucous membranes or on broken skin.
- Do not cover with tight or occlusive dressing. Do not place heating devices (e.g., hot water bottle,
heating pad) on skin after applying product.
157. Pt on HCTZ 50mg and amlodipine 5mg, got hypokalemia. what is DTP?
a) Taking medications that he do’to need for it
b) DDI
c) Taking too low dose
d) Taking too high dose
Diuretics can cause hypokalemia that may be associated with adverse cardiovascular outcomes. Consider
alternative first-line agents in those with or strongly predisposed to a serious arrhythmia, e.g., prolonged QT
syndrome. Consider using a combination product to minimize the risk of hypokalemia (hydrochlorothiazide
plus, a potassium-sparing diuretic—spironolactone, amiloride or triamterene). Reserve the use of high
doses (e.g., >25 mg/day of hydrochlorothiazide) for patients with resistant hypertension unresponsive to
treatment with multiple drugs or secondary to renal impairment. Consider using a loop diuretic in patients
with renal impairment. Diuretics may worsen dysglycemia, although cardiovascular outcomes in patients
with diabetes who are treated with diuretics are similar to those treated with ACE inhibitors.

158. 60 years old male + black + hypertension + DM and taking hydrochlorothiazide. His BP is
elevated so how to control it:
a) Take ACEI (renoprotective)
b) Take BB
c) Add Spironolactone
d) Switch HCTZ to furosemide (Frusemide is not antihypertensive drug, only for edema)
159. Which of the following antidepressants is associated with the highest rate of
hyponatremia?
a) Fluoxetine (SSRI)
b) Mirtazapine
c) Moclobemide
d) Bupropion
Hyponatremia is a symptom of SIADH caused by SSRI & can cause seizures, can be treated by hypertonic
solution. Also, SSRI, TCA, SNRI, Chloropromazine, CBZ  C/I with ADH ttt e.g: desmopressin

160. What to do if the cold chain of vaccines was broken?


a) Vaccines are affected by heat more than cold
b) Varicella should be stored between 2 & 8C
c) Report to public health.

161. Differentiation between methadone and buprenorphine/naloxone


a) Not need DR attempted
b) Can be given parentral
c) You don’t need an exemption for methadone.
d) Give less respiratory distress (low risk of toxicity and death)

162. Methadone prescription Methadone 30 mg once


daily for 7 doses dispensed in Tang juice
What is the problem with this Rx?
a) No mls mentioned per mg to dispense
b) No carrying on details specified
c) No beginning and ending dates
163. Upon calling Dr., he said that initial dose for patient is 100 mg. what is DTP?
a) Initial dose is high (20-30 mg initially then increase to 60-120 mg)
b) Initial dose is low

164. now everything is corrected, as a pharmacist what first thing you will check? (obsolete)
a) See if his Doctor is exempted
b) See if a pharmacy authorized to dispense it

165. A woman is admitted to the ER for Opioid withdrawal because of missed methadone doses
over the past few weeks. What could she be experiencing?
a) Epistaxis
b) Miosis
c) Seizures
d) Dry mouth
e) Diarrhea

166. Patient addict opioids and he use Methadone, where he will be honest.
a) If he said, I stole a dose
b) If he decreases the addicted amount
c) If he said, I’m hiding a dose
d) If he said, I will go to be treated at the hospital

167. he missed some of methadone doses he came now to a pharmacy for early refill, what is
your action? >3days treatment is withheld until Doctor adjust the dose
a) Call Doctor to authorize to give him refill
b) Talk to him about risk and benefit and let him decide to take or not take refill
c) Explain him risk and benefit of his miss, and refuse to give refill (refuse to dispense
and notify the prescriber)

168. When the methadone patient will reach


the maximum withdrawal symptoms?
a) 1 Day
b) 2 Days
c) 3 Days
169. To reduce the errors relevant to Methadone, what needs to be done?
(obsolete question since the time at which methadone was compounded)
a) Put Methadone in a separate container which is identifiable
b) Put with labels on it
c) Colored fluorescent prominent markings/labels.
http://www.cphm.ca/uploaded/web/PracticeResources/Safety-Quick-Guide-MMT.pdf

170. Long case about stroke patient attend to hospital in 3 hrs. with his wife, he is semi-
paralysed, who should agree to give alteplase?
a) The pt.
b) His wife

171. Dr. ask the ph. to prepare Alteplase if she is good candidate but 1hr later dr said that she is
not good candidate why?
a) As time zone passed
b) He follows patrenalism as he did not ask the concent of his daughter
c) After he stablize what to give (asa clopidogrel)
d) Because he has hemorahgic stroke

172. Pharmacy manager did a random narcotic count check, discovered discrepancy between
computer & actual records by 1 bottle (50 tabs), what is the best appropriate action;
a) Notify health canada
b) Check records to verify or Compare actual sales & purchase records vs computer
c) Call last pt took medication & check with him
d) Call whole saler to make sure from quantity
e) Report to or call the police
f) Take disciplinary action with pharmacy staff
173. Pharmacist and physician working together to put brochure for primary health care givers
about influenza outbreak. What is the most important to include in??
a) Guidelines
b) Pt information

174. Girl with mild acne, what to give?


a) Azelaic acid
b) Erythromycin
c) Tretinoin (Rx drug)
Pharmacist dispense glycolic acid OTC while dr. prescribes Tretinoin

175. She is worried about the results of medicines that he is been using for Acne, what should
be your concern being a Pharmacist
a) Tell patient nothing can be done
b) Tell patient that it takes 2-3 months for the medicine to show its effects.
176. she used benzoyl peroxide for 4 months no effect. She is experiencing severely inflamed
face and is taking multivitamins and paroxetine for depression. The doctor then gave him
benzoyl peroxide/clindamycin but no improvement. What is the DTP?
a) Drug drug interaction
b) Inappropriate medication
c) Too high dose (don't remember the options)
d) Drug disease interaction

177. she started a 40 mg QD Accutane (Isotretinoin), what to check before?


a) Pregnancy
b) Weight as she was 80 kg

178. Which combination require pharmacist intervention?


a) Isotretinoin + Tetracycline (increase intracranial pressure)

179. Pharmacist got a Rx of medication to dispense a medication (1mg./dose) and it should be


(.... /day), he noted and dispensed and pt was transferred to ICU. Hospital will make committee,
who will be called?
a) Nurse who gave med
b) Pharmacist who dispensed
c) Physician who wrote rx
d) Member from pharmacy management
180. Patient is stressed because her mother had a hip fracture and she had to take care of her.
Her profile shows pantoprazole, calcium carbonate and vitamin D. What is your concern?
a) Needs to start bisphosphonates
b) change calcium carbonate to calcium citrate

181. pt 85 year start dabigatran, what should you monitor before start Dabigatran
a) Self monitor bleeding
b) Kidney function
c) Liver function
d) Aptt

182. Giving Sc Implant of dabigatran to patient in home, what advice to tell?


a) If oozing happens, just rub it by hand
b) If there is hardness after injection just rub to increase the absorption

183. Dabigatran can be added to nasogastric tube or not?


The capsule should NOT be chewed, broken, opened or crushed.

184. Patient taking dabigatran what to tell him?


a) Avoid leafy vegetables
b) Do not crush, swallow as a whole.
Patients taking older anticoagulants, such as warfarin, were required to avoid eating foods that were high
in vitamin K, such as cabbage, spring onions, broccoli, fermented soy products, Brussels sprouts, and kale
and other leafy greens. This is because warfarin inhibits the clotting process by reducing the amount of
vitamin K in the blood. Pradaxa, on the other hand, impedes clotting using a completely different
mechanism of action—it inhibits thrombin, an enzyme that enables platelets to stick together to form blood
clots. Because Pradaxa's blood-thinning action isn't related to vitamin levels in the blood, people taking the
drug aren't required to avoid certain types of foods. Pradaxa patients can enjoy the potentially life-saving
benefits of an anticoagulant while eating healthy green vegetables.

185. An 85 old lady, living on her own, only have controlled HTN. Her daughters brought her
to the ER because she is not feeling well since a couple of days, she is hearing that her husband
is running after her at her home! She does not want to stay in the hospital. Then she agreed to
stay, After ISA she is better, a team from hospital should advise about next step, the team is:
a) The daughter and the social worker
b) The daughter and nurse and occupational therapist
c) The lady and nurse and social worker
186. The easiest dosage form for preparation an interchangeable drug is:
a) Suspension
b) Solution
c) sustained release tablets
d) IV

187. Therapeutic Interchangeability by a Pharmacist is due to what of the following


a) Pharmacologically same
b) Therapeutically similar effects

188. Beta carotene supplements, which to use in terms of making a decision:


a) Use smoking status to decide if going to use beta carotene.
The increased risk of lung cancer in smokers taking beta-carotene supplements reported in earlier studies
and in AREDS prompted the recommendation to avoid beta-carotene in smokers and former smokers and
led to the marketing of a supplement formulation without beta-carotene.

189. You are planning to do a seminar to


seniors with previous stroke, what Health
Care Professional you’ll invite to speak?
a) Dietician
b) Occupational Therapist
c) Social Worker
d) Chiropractor
e) Physiotherapist

190. One of seniors asks you about


which affect risk of stroke the
most, she is Smoking, age 84years,
diabetes, high blood pressure &
Hypothyroidism. All are risk
factors; except? (K-type)
a) Age
b) Hypothyroidism
c) Hypertension
Risk factors (hypertension, smoking,
diabetes mellitus, dyslipidemia,
excessive alcohol intake, high body
mass index, low exercise, family history of vascular disease or hemostatic disorders)
191. Pt with Addison disease, attended to hospital comatose with ketoacidosis & what to give?
a) Prednisone IV
b) Dexamethasone po
c) Fluticasone intranasal

192. Another question realted to the case: they brought a chart with equivalence to methyl
prednisone, so methylprednisone was 4 and prednisone was 5 in the chart and there was
dexamethasone (the chart included their glucocorticoid and mineralocorticoid levels) and he
was taking mehtylprednisone 15 mg BID and he wanted to change it to prednisone so what dose
would you give?
a) 20 mg BID
b) 40 mg BID
c) Change to a different corticosteroid

193. Caution when handling or Which should be avoided to be directly in contact?


a) Hydroxyurea
b) Digoxin
c) Allupurinol
d) Methotrexate

194. Which drug will appear in breast milk? (Alkaline- low Mwt- free- high lipophilic EE)
a) High molecular wt
b) Low lipid solubility

195. Tylenol # 3 (acetaminophen 300 mg + codeine 30 mg + caffeine 15 mg), regulations:


a) Written only
b) Verbal and written
c) Can be transferred

196. Tylenol 3 mitte cc uc, all of the following right except


a) Call police for forgery
b) Call clinic to verify rx
c) Ask pt why he saw his dr
d) Call clinic to verify physician
Another version: tylenol 3 and Rx (cc mitt dict) you will do xpt
a) Call Dr office to verify the direction
b) Call Dr office to verify the amount
c) Call Dr office to verify the Rx 4-call police about forgy Rx
d) Ask patient about what Dr tell him about uses
Always call physician to verify.
197. The pharmacist does medical
reconciliation to?
a) Avoid allergy
b) Decrease ADR
c) Continuity of care

198. What needs a prescription to be


dispensed or Which one of the following is
schedule I?
a) Vit B 12 parentral
b) Mupirocin 2%
c) Fusidic Acid
d) Oral potassium salt
e) Nitroglycerin spray
f) Epipen

199. which will least to cause a dispensing error:


a) QD for everyday
b) QID
c) .5
d) ml

200. Patient with a recent MI and needs to quit smoking, which statement is true:
a) Bupropion can be safely used in his case
b) Use of NRT immediately MI is unacceptable

201. 15 years old girl, smoker and had her last period 14 days ago, she came to your pharmacy
and asked about plan-B after 3 days of sexual intercourse. What is the right action?
a) Dispense plan-B
b) Refuse to dispense
c) Advise her to see a gynecologist
d) Tell her there is no effect after 3 days

202. After that, she came again with prescription for COCs what is your advice?
a) You must use barrier for first few days (7-day pack up with combined)
b) It has a risk of breast cancer
c) Combined Hormonal therapy is not suitable for your age
d) Call the doctor before you dispense to confirm
203. The repeated Plan B question and the pharmacist did not want to dispense. What to do:
a) Give her this time only
b) Send her to the doctor
c) Send her to nearby pharmacy

204. Pt had severe back pain, was on oxycodone and BDZ and many refills, coming to ask for
Tylenol 3, what to do.
For Tylenol 3 and all other narcotics you need to verify with the prescriber because the day supply has to be
clear. Usually for acute pain you would see these directions: take 1-2 tabs q4-6h prn but again it has to be
clearly indicated by the prescriber

205. Child 18 month (weight 11 kg) has otitis media, he took amoxicillin 2 months before now,
what you give him?
a) Amoxi / clav. X 10 days in younger than 2 years
b) Azithromycin.
c) Amoxicillin
206. Now, he has a 3rd episode, doctor
prescribed amoxicillin (certain dose was
written but when I calculated /11 kg, I
found it was 40 mg/kg which considered
less than correct which 90 mg/kg for his
2nd episode). What is the DTP or what is
your concern about this Rx??
a) Low dose of Amoxicillin
b) Short Duration of treatment
c) No need for Antibiotic

207. Otitis Externa, what is the causative agent?


a) S. Pneumonia
b) P. aeruginosa (20-60%)
c) S. aureus (10–70%)
Pathogens responsible for infection of the middle ear (Streptococcus pneumoniae, Haemophilus influenzae,
or Moraxella catarrhalis) are uncommonly found in cultures of the external auditory canal when the
tympanic membrane is intact. The epithelium absorbs moisture from the environment.

208. Which statement is true about insulin pen:


a) Can be mixed
b) Not for visually impaired patients
c) Should be primed each time before use
Priming means removing air bubbles from the needle, and ensures that the needle is open and working.
The pen must be primed before each injection. To prime the insulin pen, turn the dosage knob to the 2 units
indicator. With the pen pointing upward, push the knob all the way.

209. When to give an ACEI and ARB


together (obsolete)
a) Diabetes without nephropathy
b) HF
Two indications for combination therapy with
ACE inhibitors and ARBs are prominently cited in
the literature: heart failure and CKD with
proteinuria
210. Rivastigmine which of the following is true
a) Reassess in at least 3 months
b) Take on empty stomach  Take with breakfast and dinner
c) No drug interaction (non reported, not metabolized by CYP450)
d) It is proven to be the best treatment for alzheimers dementia
The cholinesterase inhibitors are the mainstays of treatment for cognitive and functional symptoms, and
may also have a role in managing behavioural and psychological symptoms, especially in mild to moderate
dementia. Donepezil, rivastigmine and galantamine have distinct structures and modes of action. In the
absence of blinded, controlled, head to-head trials, all 3 agents seem to be of equal efficacy, and thus the
most cost-effective agent is often used.

211. What is the goal of therapy: increase time to long term care
Goals of Therapy
- Alter the natural disease progression to meet patient’s and caregiver’s goals
- Treat cognitive, behavioral and psychological symptoms
- Alleviate caregiver burden
- Minimize medication side effects
212. Which interacts with SSRIs: tramadol = Serotonin syndrome

213. 19 yo started on escitalopram, what do you monitor? Suicidal ideation.

214. Patient with Low HDL and high TG, what to give?
a) Fenofibrate  More potent in lowering TG and increasing HDL
b) Niacin

215. Carbamazepine started at high dose, what do you recommend?


a) Decrease dose and titrate up weekly.
b) Start with loading dose
c) Use SR formulations
Start low go slow to avoid dose dependant side effect. Carbamazepine is a substrate and an inducer of CYP
3A4. it metabolizes itself (inducer of its own metabolite)
Carbamazepine advantage over phenytoin due to easier dosing due to linear pk
Adults: It is advised that with all formulations of Tegretol, a gradually increasing dosage scheme is used and
this should be adjusted to suit the needs of the individual patient.
Tegretol should be taken in a number of divided doses although initially 100-200mg once or twice daily is
recommended. This may be followed by a slow increase until the best response is obtained, often 800-
1200mg daily. In some instances, 1600mg or even 2000mg daily may be necessary.

216. Pregnant patient, what’s the DTP to change the meds? (pre-ecelpmsia)
a) Change the ACEi to methyldopa.

217. Patient had ONE kidney, what do you give for BP?
a) Amlodipine
b) ACEi

218. Now she's concerned about diabetes and wanted weight loss, what do you give her?
a) Metformin (cause her creatinine was >30 so it's fine)
219. A person on phenytoin and came in to the hospital because of seizure, on a cardiac
medication, diabetic and there are given values concerning his status, phenytoin level is below
the appropriate serum level, why did he get seizure?
a) Too low dose of phenytoin
b) Needs dual medication
c) Switch to CBZ

220. They changed him to 460 mg phenytoin po, after 6 days they measured his serum levels,
and they were in the range, why is this not reliable?
a) Should measure 2 hours post dose
b) Should measure 6 hours post dose
c) He's taking an oral medication
d) The periodic serum levels need to be monitored due to Saturated kinetics

221. Patient is being placed on 10mg Atorvastatin; however, she is very reluctant to starting the
statin therapy because her Dad has been using statins and had really bad muscle aches and
myopathy, what is the initial most appropriate approach the pharmacist will do?
a) Acknowledge her reluctance to taking the statin
b) Evaluate her chances of getting myopathy
c) Tell her its a rare side effect and have no worries
d) Tell her it is a dose related side effect and she should not worry

222. All true about pneumococcal vaccine except?


a) Taken every 5 to 10 years
b) Infleunza vaccine is enhanced if taken with pneumococcal vaccine
c) Take > 65years
d) Pneumococcal vaccine doesn’t protect against Endocarditis

223. Levothyroxine toxicity will cause?


a) Weight gain
b) Hypotension
c) Diarrhea
d) Constipation

224. What's true about Gardasil


a) Take on 6-month duration
b) Females 9-25 y
c) All types of papiloma
d) Treat anal cancer
Gardasil Prevention of external anogenital warts caused by HPV types 6 and 11.
9-valent human Females 9–45 y and males 9–26 y: 0.5 mL administered at 0, 2- and 6-months IM
papillomavirus Alternative 2-dose schedule in immunocompetent individuals 9–14 y: 0.5 mL administered
(types 6, 11, 16, at 0 and 6–12 months IM.
18, 31, 33, 45, If alternate dosing is required, the second dose should be administered at least 1 month after
52, 58) the first dose, and the third dose should be administered at least 3 months after the second
recombinant dose. All doses should be administered within a 1-year period.
vaccine S.E: Pain, swelling, erythema and pruritus at injection site, headache, fatigue, syncope,
lymphadenopathy, anaphylaxis. Not recommended in pregnancy, but used in breastfeeding.

225. Labeling requirement of baclofen


a) May cause drowsiness or dizziness
b) Avoid Grape fruit
c) Can be stopped abruptly
d) Avoid caffeine containing beverage
Baclofen can be taken with caffeine and it has nothing to do with CYP3A4, the dose should be reduced
slowly when the drug is discontinued (over a period of approximately 1-2 weeks).

226. A lady come taking a cough med contain codeine and you saw here yesterday in another
pharmacy taking same med so? ASK FIRST
a) Call police
b) Refer her to dr
c) Asses her cough
d) Do not sell here

227. There was a direct cross multiplication question where u get the answer in gms and he
provided us with the specific gravity to convert it to ml

228. t1/2 is affected by:


a) Volume of distribution
b) Renal elimination
c) SR dosage form
d) Css
229. 31 years old male with meningitis case & culture shows gram +ve bacteria. What is the
suspected microorganism?
a) S. Pneumonia
b) H. Influenza
c) N. Gonorrhea

230. A question on a patient with


Meningitis with the values of
Lactic acid, Protein, Glucose and
WBCs, which one of the following
is true (Q was which one will
remain constant)
a) A reduced Lactic Acid
b) A lowered Protein
c) Increased Glucose
d) Increased WBC

231. What is the DOC for treatment?


a) Ceftriaxone + Vancomycin
b) Ampicillin + Ceftriaxone
c) Ampicillin + Vancomycin

232. What to give as add on therapy?


a) Normal Saline
b) Dexamethasone
c) Dextrose
d) Acetaminophen
The rationale behind adjunctive therapy with dexamethasone is that, despite adequate antibiotic
treatment, patients frequently experience poor outcomes due to a brisk inflammatory response that causes
neuronal damage. Limiting the inflammation by administration of corticosteroids could potentially decrease
neurologic sequelae such as hearing loss and disability. If corticosteroids are administered, they should be
initiated either before or with the first dose of antimicrobials.
A theoretical concern regarding the use of dexamethasone in meningitis is the potential for delayed
sterilization of the CSF (due to altered CSF drug penetration) in individuals treated with vancomycin for
resistant S. pneumoniae. There are no large clinical studies to answer this question. As stated above,
strongly consider the addition of rifampin to the antibacterial regimen if resistant pneumococci are isolated
in a patient who has received dexamethasone.
233. What the legislation for Tramadol?
a) Written Rx only with refills
b) Written & Verbal Rx without refills
c) Written & Verbal Rx with refills & Part fill

234. What the legislation for Testosterone?


a) Written Rx only with refills
b) Written & Verbal Rx without refills
c) Written & Verbal Rx with refills
d) Written & Verbal Rx with refills with specified time interval

235. The best source of information that presents a gateway to immunization information
including vaccine safety, immunization schedules, is:
a) Canadian Public Health Association
b) Public Health Agency of Canada
c) Canada Immunization Research Center
d) Canada Health Act

236. Patient comes with prescription and you realize it is forgery what you will do?
a) Tell the patient it is forgery and don’t dispense it & return the Rx to him.
b) Tell the patient it is forgery and restrain the patient until call the police.
c) Send the patient to another pharmacy to dispense the medication.
d) Give him advance of 2 tab. & ask him to come next day & call the police.
e) Adopt the delaying tactics and ask the Customer to come after a few hours and in
the meanwhile call the police.

237. Diabetic patient on Metformin and Gliclazide and drinking 2 cups of alcohol per day and 4
cups on the weekend what is the correct statement
a) Alcohol is contraindicated with Metformin
b) Drinking 2 cups of alcohol daily with Gliclazide is accepted.
c) Excess alcohol will cause hyperglycemia.
d) Alcohol is contraindicated with Gliclazide
e) Patient will experience hypoglycemia during the weekend.
Another version: He is now using high toxicity of methanol what is therapy for him? Ethanol
Alcohol may be incorporated into a diabetes meal plan, provided there are no other contraindications.
Alcohol consumption may mask symptoms of hypoglycemia, reduce hepatic glucose production and increase
ketones. However, when used in moderation it can be safe and even beneficial from a cardiovascular
perspective. Alcohol consumption should be limited to ≤2 standard drinks per day and <10 drinks per week in
nonpregnant, nonlactating females with diabetes and ≤3 standard drinks per day and <15 drinks per week in
men. One standard drink is defined as 10g of alcohol which will be contained in 341 mL of 5% alcohol beer,
43 mL of 40% alcohol spirits or 142 mL of 12 % alcohol wine.
Increased physical activity and reduced or no food intake can increase the risk of hypoglycemia with alcohol
ingestion. In those with type 1 diabetes, moderate consumption of alcohol with or a few hours after the
evening meal can result in hypoglycemia the following morning or up to 24 hours later.
Those using insulin or insulin secretagogues (e.g., meglitinides, sulfonylureas) should be informed of the risk
of delayed hypoglycemia resulting from alcohol consumption with or after the previous evening meal and
advised of measures to prevent hypoglycemia including:
 consuming alcohol with food, eating a carbohydrate-containing snack before bed, adjusting insulin
and monitoring blood glucose.
 Inform patients who use metformin that consuming alcohol within the suggested limits is unlikely to
be problematic, but that acute or chronic ingestion of larger quantities of alcohol can contribute to
the development of lactic acidosis.
 Alcohol contains 29 kJ or 7 cal/g and can therefore contribute to weight gain. Alcohol containing
medications are unlikely to contribute to poor blood glucose management or weight gain when used
in moderation.

238. Patient with signs of allergic rhinitis, watery eyes, nasal discharge & green respiratory
discharge for 3 weeks she tried taking Loratadine but it was not effective, why would you refer
this patient?
a) Due to the constant sneezing and itching for more than 2 weeks
b) Green discharge
c) Due to the bilateral headache, a rare symptom that had never occurred before.
d) Loratidine was ineffective
e) Due to the frequency as it had occurred twice the same month
During the assessment, identify duration, frequency
and severity of symptoms as well as precipitating
factors and allergens, occupational exposure, and
response to current and previous therapy.
When recommending treatment, consider also the
effectiveness and adverse effects of the treatment
alternatives, patient preference and cost.
Consider the need for prescription therapy or referral
for allergy testing if the patient has already tried
appropriate nonprescription therapy for 2 weeks
without an adequate response, or if the allergen
responsible for symptoms cannot be readily identified.
Also refer patients for further assessment if they have signs or symptoms that are unilateral or are not
usually associated with allergic rhinitis (e.g., fever, facial pain, loss of smell or taste, recurrent epistaxis,
purulent nasal or ocular secretions, postnasal drip with or without rhinorrhea) or symptoms suggesting
complications such as asthma.
239. Patient taking nasal mometasone spray for
Allergic rhinitis, what’s true?
a) Take each nostril per day
b) It must take saline to remove secretion

240. Allergic Rhinitis+ Common Cold, why


you choose oxymetazoline? Decongestants
(Fast acting 15 min- long acting, up to 12
hours)
241. Which drug is known with its class side effect that causes hepatic cholestatic dysfunction?
a) Penicillin
b) Aminoglycosides
c) Estrogen (oral contraceptives)
d) Nitrates
The causes of cholestasis are divided into two groups:
Within the liver
Causes include acute hepatitis, alcoholic liver disease, primary biliary cholangitis with inflammation and
scarring of the bile ducts, cirrhosis due to viral hepatitis B or C (also with inflammation and scarring of the
bile ducts), certain drugs (for example, amoxicillin/clavulanate, chlorpromazine, azathioprine, and oral
contraceptives), hormonal effects on bile flow during pregnancy (a condition called cholestasis of
pregnancy), and cancer that has spread to the liver.
Outside the liver
Causes include a stone in a bile duct, narrowing (stricture) of a bile duct, cancer of a bile duct, cancer of the
pancreas, and inflammation of the pancreas (pancreatitis).

242. Azithromycin caps auxiliary label.


a) With or without food
b) Separate for dairy product, Juice & Antacids
c) Take without food
d) Immediately after food
e) 1 hour before meal or 2hrs after meal
Azithromycin capsules should be given as a single daily dose. In common with many other antibiotics
Azithromycin Capsules should be taken at least 1 hour before or 2 hours after food.
Children and adolescents with a body weight above 45 kg, adults and the elderly:
The total dose of azithromycin is 1500 mg, which should be given over three days (500 mg once daily).
In the case of uncomplicated genital infections due to Chlamydia trachomatis, the dose is 1000 mg as a
single oral dose. For susceptible Neisseria gonorrhoeae the recommended dose is 1000 mg or 2000 mg of
azithromycin in combination with 250 mg or 500 mg ceftriaxone according to local clinical treatment
guidelines. For patients who are allergic to penicillin and/or cephalosporins, prescribers should consult local
treatment guidelines

243. Regarding Chlorhexidine what is true?


a) May stain teeth if used for long term
b) Interact with Antacid
c) Contraindicated in old age
d) Available in oral and injection form
e) Swish and swallow.
244. A diabetic patient is using Humalog 50 Mix (50% Lispro and 50% NPH). He is taking 24
units of Humalog Mix before breakfast and before supper. He called today to get his Humalog
Mix. However, all of Humalog is back order. You've Humalog Lispro & Humalog Mix (25%
Lispro and 75 % NPH)
a) Give him 8 units of Humalog Lispro and 16 units of Humalog Mix "25:75 "
b) Give him 12 units of Humalog Lispro and 12 units of Humalog Mix "25:75 "
c) Give him 16 units of Humalog Lispro and 8 units of Humalog Mix "25:75 "
Break down the question so you won't get confused:
So, if the patient needs 24 units mix (50/50) it means 12 units of each insulin needed.
Now we have a mix with (25/75) which means every 100 units contain 75 units NPH
so, if we want 12 units NPH how much of mix? 100 x 12/75 = 16 units mix.
We already have one answer so go for A but if there were 2 answers with 16 units then also calculate the
lispro the same way: now we have 16 units mix (25/75)
so, it contains 25% lispro  16 units x 0.25 = 4 units but we need 12 units so 12 ‐ 4= 8 units lispro alone.

245. A diabetic patient type I. in the honeymoon period, what to advise him:
a) Keep Insulin dose because it's a short period time phenomenon
b) Keep Insulin dose but increase the frequency of glucose level testing
c) Decrease the dose of Insulin
d) Increase the dose of Insulin

246. What do you counsel on a patient newly diagnosed with dementia starting a cholinesterase
inhibitor?
a) If you fail one medication in this class you will probably fail all medications within the
class
b) Ginko Biloba will enhance the efficacy of this medication
c) Decrease the Repetition of question is a good goal of treatment
It is important to define target symptoms prior to treatment initiation. Effectiveness is often considered to
be either improvement or no deterioration of target symptoms. In untreated patients with mild to moderate
dementia, a decline of 2–4 points per year on the MMSE scale is expected; therefore, an annual decline of
less than 2 points while on drug therapy would typically indicate a beneficial effect. However, despite its
widespread use in clinical research, the MMSE is now recognized as a poor measure of treatment response
in individual patients.
An emerging monitoring strategy is to set individualized treatment goals with the patient/caregiver prior to
treatment and then measure if these goals were attained at regular intervals. Reduction in repetitive
questioning is a common treatment goal in patients with mild-moderate Alzheimer disease and usually
corresponds to a generally positive treatment effect. Monitor treatment effects 2 weeks after initiating
therapy or increasing dosage and then periodically thereafter based on feasibility.
247. When we check the efficacy of
Dementia treatment
a) After 3 months then every year
b) After 2 weeks then every 3
months (2 weeks after initiation, 3
months to kick in, start low and go
slow)
c) Every 6 months
d) Every year

248. Pregabalin may be used in all cases Except


a) Renal Impairment (75% excreted renally, caution)
b) Hepatic disease
c) Blood disorders

249. You have a prescription with Menthol and camphor, which type of mixture is that
a) Liquid mixture
b) Eutectic mixture
c) Ionic Mixture
d) Liquid Crystalline Mixture

250. This mixture used for


a) Impetigo
b) Back pain
c) Herpes Zoster
d) Pruritus

251. When you allow the technician to make this compounding


a) If he made this compounding before
b) If he did this compounding in the field of the pharmacist's vision
Any supervision, choose it
Must be accountable and whoever delegated (pharmacist) will be held responsible for outcomes.

252. Why do you keep the record of the compounding?


a) To know the cost of the compounding
b) To be easy to make it again when asked for refill
c) To determine the stocks of the ingredient
d) Reliability of reproducibility when preparing
253. Pneumocystis Jirovecii Pneumonia (PJP) has been reported to be a leading cause of death
in HIV-infected infants. Which of the following could be used as prophylaxis against PJP?
a) Amoxicillin
b) Folic acid
c) Vitamin E
d) Co-Trimoxazole

254. Side Effects of Sevelamer tablet


(treatment of hyperphosphatemia)
a) Nausea  Heartburn, bloating, gas.
b) Insomnia
c) Ascites
d) Neuropathic pain

255. All below references can be used for Drug-Drug interaction EXCEPT
a) CPS
b) Lexicomp
c) Remington
d) TC

256. Patient suffers from diabetic foot and ulcer, while doctor inspection he discovered that it
reaches to the bone, this called?
a) Cellulitis
b) Osteomyelitis
c) Tendinitis
d) Ulcerative colitis

257. The duration of treatment is


a) 2 Weeks
b) 1 Month  4 – 6 weeks
c) 3 Month

258. Which is the proper treatment


a) Cefazolin
b) Cephalexin
c) Cefamandole
d) Cefuroxime
Empiric antibiotic regimens MUST ALWAYS COVER Staphylococcus aureus.
Afterwards, you tailor according to culture results from specimens (preferably of bone).
259. Oral contraceptives are affected by all of the following EXCEPT:
a) Erythromycin
b) Clonidine
c) Phenobarbital
d) Gabapentin (clonidine, Gabapentin not affect OCP)

260. Which of the following injections should be refrigerated?


a) Diphenhydramine suspension
b) Sumatriptan
c) Depo-Provera
d) Glatiramer
Advise patients that the recommended storage condition for Glatiramer is refrigeration (36-46o F /2-8o C),
although Glatiramer can be stored at room temperature (59-86o F /15-30o C) for up to one month.
Glatiramer should not be exposed to higher temperatures or intense light.

261. Endocarditis case patient took ticarcillin before, he has prosethic valve now his Dr need to
give him prophylaxis for dental surgery. what is correct one?
a) He does not need prophylaxis
b) Amox/clav 2 gm
c) Clindamycin 600 mg
d) Gentamycin
Answer #2: bring ticarcillin dose closer to dental
procedure

262. Complication of endocarditis


a) Hypertension crisis
b) Heart Failure
c) Liver Failure
d) Kidney Failure

263. Which medication should be stopped before IV contrast media?


a) Metformin (pre-operative, before MRI, before Die)
b) Acarbose
c) Pioglitazone
d) Gliclazide
Intravascular administration of iodinated contrast agents may lead to contrast induced nephropathy,
resulting in metformin accumulation and an increased risk of lactic acidosis. Competact should be
discontinued prior to or at the time of the imaging procedure and not restarted until at least 48 hours after,
provided that renal function has been re-evaluated and found to be stable.
264. A 10-week pregnant lady came to you in the pharmacy and said she has burning & itching
in the vaginal area and it’s the first time for her to experience these symptoms and she said she
has a white-grey discharge and malodorous. Why would you refer this patient? (K-TYPE)
a) Because this is the first time for her
b) A pregnant woman can take intravaginal medication
c) Her symptoms are different from those of candidiasis

265. Patient has migraine she is seeking for something fast to relief her symptoms:
a) Naproxen
b) Naratriptan (second dose after 4 hs)
c) Zolmitriptan (second dose after 2 hs)
d) Subcutaneous Sumatriptan (second dose after 1 h)

266. A patient has shingles and she was admitted to the hospital due to her severe and acute
pain. Her pain is now controlled. You delivered the medication to her as prescribed
Amitriptyline 10 mg QHS. When she went home and read the leaflet, she decided not to take
the medication due to its side effects. What should the pharmacist tell her?
a) The pain relief provided by this medication will outweigh any side effects.
b) The common side effects usually tolerated by most patients and can be managed.
c) This is a very low dose to cause these side effects
d) If you experience any side effects go right away to the doctor
e) Serious side effects do not happen to everyone

267. Old man coming out of hospital, can be given all vaccines, EXCEPT:
a) Varicella
b) Pneumonia
c) Influenza
d) Diphtheria
e) Tetanus

268. A week ago, you dispensed Nitrofurantoin to a patient. Today she came to the pharmacy
and told you that she got stomatitis. You immediately call the doctor to confirm whether the
similar side effects have ever been reported to him by other patients for the same indication, at
which the doctor denies, which of the following action needs to be taken.
a) Report to Med effect (health Canada) (S. effects)
b) Report to Institution of Safe Medication Practice (ERROR)
c) Tell patient that it will subside in a few days.
d) Call doctor to change the medication.
e) Immediately report the same to ISMP.
269. You are a hospital pharmacist. You discovered that one of patients had an order for Losec
and technician who prepared prescription misinterpreted it as Lasix. However, the pharmacist
who was there in that shift is on vacation today. The patient has been taking the wrong
medication for three days so far including this day. Who is the first person you should contact?
a) The physician who wrote the prescription.
b) The pharmacist in charge of that shift
c) The technician who prepared the prescription
d) The patient’s family
e) The nurse on the patient care unit

270. What do you monitor for this patient? (Hyper GLUC)


a) Glucose
b) Lipase
c) Uric acid
Overdose can cause massive diuresis resulting in dehydration, volume depletion and electrolyte disturbances
with consequent hypotension and cardiac toxicity. The clinical picture in acute or chronic overdose depends
primarily on the extent and consequences of electrolyte and fluid loss, e.g. hypovolaemia, dehydration,
haemoconcentration, cardiac arrhythmias due to excessive diuresis. Symptoms of these disturbances include
severe hypotension (progressing to shock), acute renal failure, thrombosis, delirious states, flaccid paralysis,
apathy and confusion. High doses have the potential to cause transient deafness and may precipitate gout
(disturbed uric acid secretion).
Management
 Benefits of gastric decontamination are uncertain. In patients presenting within 1 hour of ingestion,
consider activated charcoal (50g for adults: 1g/kg for children)
 Observe for a minimum of 4 hours - monitor pulse and blood pressure.
 Treat hypotension and dehydration with appropriate IV fluids
 Monitor urinary output and serum electrolytes (including chloride and bicarbonate). Correct
electrolyte imbalances. Monitor 12 lead ECG in patients with significant electrolyte disturbances

271. Drug X / Drug Y / Drug Z, Ratio is 1:2:2 and he total quantity mitte 150g; If Con. of the
total mix is 0.1%, what is the Con. of X?
30:60:60, So X= 20%, So Conc.= 20%*0.1%= 0.02%

272. Why we give Vitamin D with Ca?


a) Increase the intestinal passive absorption of Ca
b) Increase the intestinal active absorption of Ca
c) Improve bone resorption
Calcium can only reach its full bone-building potential if your body has enough vitamin D. Calcium and vitamin
D work together to protect your bones—calcium helps build and maintain bones, while vitamin D helps your
body effectively absorb calcium.
273. Patient is taking phenytoin & must take Combined Oral Contraceptive:
a) Ethinyestradiol 50 μg / Norethindrone 0.5 mg (+ progestin)
b) Ethinyestradiol 35 μg /Norethindrone 0.5 mg
c) Progesterone only
d) Spermicide
Clinically significant interactions occur with the use of antiepileptic medications (e.g., carbamazepine,
eslicarbazepine, oxcarbazepine, phenobarbital, phenytoin, primidone) and the antibiotics rifabutin and
rifampicin, which are known to induce hepatic enzymes. It is advisable to recommend a contraceptive
method that will not be affected by enzyme inducers (e.g., depot medroxyprogesterone acetate injectable,
copper intrauterine device, levonorgestrel intrauterine system, barrier contraception) when taking a
concomitant interacting medication.

274. Seizure patient & he is taking Valproic & suffer from intolerable GIT S.E, what to do?
a) Lower the dose of Valproic
b) Change Valproic to its salt
c) Change to Ethosuximide
d) Add Ethosuximide
Divalproex sodium= GI tolerability may be better than with valproic acid.

275. To determine the safety of taking insulin in hospitals and avoid errors, what do you
recommend? (Unit Dose)
a) Use ml unit in insulin syringe
b) Use the patient home insulin
c) Put insulin beside patient bed to decrease the time to take insulin
d) Use appropriate syringe unit that is clear enough to the patient and nurse

276. 39-year-old female on Escitalopram & started to feel anxiety, shortage of sleep, inability to
listen, recently diagnosed with ADHD, what is the most prominent symptom of ADHD?
a) Anxiety
b) Inability to listen
c) Insomnia
277. What is the best course of action?
a) Keep Escitalopram & add Atomoxetine
b) Change Escitalopram to Fluoxetine & add Methylphenidate
c) Stop Escitalopram & add Atomoxetine
d) Increase the dose of Escitalopram to manage ADHA & Depression
Atomoxetine, a norepinephrine reuptake inhibitor, is recommended as a second-line agent for the treatment
of children ≥6 years of age, adolescents and adults with ADHD. It is not classified as a stimulant and is not a
controlled substance. The efficacy and tolerability of atomoxetine have been studied in several well-
designed trials.
RCTs confirm that after 6–12 weeks of treatment, atomoxetine reduces core ADHD symptoms by at least 25–
30% in 60–70% of individuals. The efficacy of atomoxetine approaches that of stimulants, although it may
take 3–4 weeks to see its beneficial effects. While some guidelines list it as a first-line option, the available
evidence supports a role in therapy for those who have either not responded to or not tolerated an
adequate trial of stimulant medications. It should also be considered for those with ADHD and
comorbid substance-abuse disorder or depression.
Contraindications to atomoxetine include hypersensitivity to atomoxetine, narrow angle glaucoma, history
of severe cardiac or vascular disorders, pheochromocytoma, and concurrent use with an MAOI.

278. Patient wants to know the effectiveness of Vitamin D in cancer, which reference will you
search for this information
a) Reviewed article about vitamin supplementation
b) Ask oncologist in cancer center
c) Therapeutic choices
d) Primary journal

279. What is true about chemotherapy induced nausea and vomiting treatment?
a) Dimenhydrinate can’t be used
b) Granisetron can cause headache & Constipation (headache: very common,
constipation: very common, diarrhea: common)
c) Dexamethasone should be used after 2 days of chemotherapy
Remember: Dexamethasone SE: mood change,
Aprepitant and netupitant inhibit CYP3A4 and
interact with corticosteroids, requiring a
decrease in the antiemetic dose of
dexamethasone when used concomitantly.
280. Why progesterone is added to OCT?
a) Decrease cervical cancer
b) Decrease breast cancer
c) Decrease endometrial cancer

281. Patient on his profile: Oxycodone,


Methamphetamine, Lorazepam, Lactulose,
Amitriptyline, Losartan, Atorvastatin, so the
number of narcotics, controlled, targeted
subs. & regular medication taken
respectively is:
a) 1:1:1:4
b) 2:2:1:4
c) 1:0:2:4
Oxycodone: narcotic- Lorazepam: controlled- Methamphetamine: targeted- others: regular

282. Patient on his profile: Barbital + Acetaminophen + Codeine, Oxycodone, Morphine,


Diazepam, Captopril & Metoprolol, then his physician adds to him Methadone, So the number
of Narcotic: Controlled: Targeted: Regular medication taken respectively is?
a) 4:0:1:2
b) 3:1:1:2
c) 4:1:0:2
d) 3:0:2:2
Narcotics:4- controlled:0 – targeted: diazepam- regular: captopril, metoprolol

283. A physician called you to ask about the newest treatment for multiple sclerosis. What is the
most suitable method for the pharmacist to get this information?
a) Pub Med
b) E-therapeutics
c) E-cps
d) Clinical practice guidelines
e) Primary Literature

284. A physician called you to ask about the usage of drug X in pregnant woman, in which
reference do you look for this information?
a) Mother risk (OTC)
b) CPS
c) TC
NOTE: MotheRisk does NOT exist any more
285. What is the most appropriate non-pharmacological advice to give to a baby's mother her
son suffers from diaper rash?
a) Use antifungal (complicated)
b) Exposure his skin to air to dry it or Aeration for baby skin
c) Use talc powder frequently
d) Increase frequency of washing
Avoid, cornstarch, also no alcoh. Wipes, acid ph cleanser
CTMA: Air-drying should be encouraged to diminish damaging effects of occlusion and maceration. Remove
diaper for as long as possible during cleansing, treatment and changes.

286. Non-prescription pharmacologic treatment of diaper dermatitis:


a) 1% Hydrocortisone in petrolatum
b) 0.5% Hydrocortisone in Mupirocin
c) Mupirocin 2% in a polyethylene glycol base
d) Topical hydrocortisone 0.5%
Hydrocortisone is a schedule I When sold in a concentration that provides 1% or less hydrocortisone in
preparations for topical use on the skin in children under 2 years of age.
Hydrocortisone 0.5% and 1% are available without a prescription; however, according to NAPRA,
hydrocortisone 1% is restricted to prescription when used in those <2 y

287. 65-year-old patient of your pharmacy presents with 4 prescriptions. Total cost of Rx is
$156. Senior’s insurance deductible for the year is $100. For this transaction. Patient pays?
$100. Patient has deductible of $100. He paid 63.25 of it. Insurance co-payment is 10% after
and they cover up to $10 dispensing fees. If the med costs $87.50 including $15 dispensing fee
how much do you collect from patient? answer in some sources 49 other sources 45.33
Soln: First, Calculate the remaining of deductible
100 - 63.25= 36.75  Fees = 15 - 10 = 5  The total price of Rx us 87.5 including both the
fees and the deductible  So, 87.5 -15-36.75 = 35.75 10% = 3.575 The total = 45.33

288. What is the odd ratio for a trial done on 2 products A & B in smoking cessation, the result
was as follow:
Product A: 44 Of 56 stop smoking (44*71/29*56)
Product B: 29 Of 71 stop smoking
a) 1.9
b) 0.053

289. Which of the following switching between antidepressants require a washout period of 10
days?
a) Switching from Venlafaxine to Escitalopram
b) Switching from Phenelzine (MAOI) to Escitalopram (2 week)
290. Sign of toxicity of Dextromethorphan is:
a) Delirium
b) Bradycardia (hyperthermia+ histamine release+ hypertension+ tachycardia)
c) Bronchospasm
Dextromethorphan, the active metabolite of dextromethorphan, causes a variety of physiological effects via
several different mechanisms. Like ketamine and phencyclidine, dextrorphan blocks NMDA receptors
resulting in hallucinations, euphoria, dissociation, agitations, and coma.
Dextrorphan binds serotonin receptors, potentially leading to serotonin syndrome (seizures, muscle rigidity,
autonomic instability, rhabdomyolysis). This activity can occur in the settings of overdose and standard
doses when combining dextromethorphan with SSRIs, SNRIs, MAOIs, cocaine, TCAs or other serotonergic
agents. Dextromethorphan also blocks the reuptake of peripheral adrenergic neurotransmitters, causing
hypertension, tachycardia, mydriasis, and diaphoresis. Dextrorphan does not bind mu or delta opioid
receptors but does bind sigma opioid receptors leading to antitussive effects.
Overdose Symptoms:
These include nausea and vomiting, CNS depression, dizziness, dysarthria (slurred speech), myoclonus,
nystagmus, somnolence (drowsiness), tremor, excitation, mental confusion, psychotic disorder (psychosis),
and respiratory depression.
Management:
Treatment of overdose should be symptomatic and supportive. Gastric lavage may be of use. Naloxone has
been used successfully as a specific antagonist to dextromethorphan toxicity in children.

291. A patient asked the pharmacist to meet her outside the pharmacy to discuss with her
something about her medical case, the pharmacist refuse so he follows?
a) Professional act
b) Justice
c) Fidelity

292. A pharmacist received a recall for product X, his mother used this product so he decided to
keep all the stock of product X that he has for his mother, what ethics does he violate:
a) Beneficence
b) Justice
c) Fidelity

293. A pharmacist in a province which prevent dispensing the Rx after 3 days of its writing
date, A mother of a 2 years old boy came with a Rx wrote from 5 days, what is the sentences
that the pharmacist can tell the lady to show his sympathy with her?
a) Sorry, I Can't dispense the Rx
b) I can offer you a delivery to the physician office to change the date of the Rx
c) I have a child and know your responsibilities and duties toward your kids
Note: Empathy- different answer
294. Which one may cause hyperglycemia?
a) Removing TPN while using Insulin
b) Giving 50 U of Insulin instead of 5 U
c) Giving Long acting Insulin without using Regular Insulin

295. In hospital patient has received 1 gm Vancomycin after


4 days the blood conc. was 17 m.mol/L was 35 m.mol/L after
injection & he has received a second dose of 500 gm
Vancomycin. Peak conc. is 20 m.mol/L, trough 15 m.mol/L
when should he take the third dose?
a) 1 day after the second dose
b) 2 days after the second dose
c) 3 days after the second dose
d) 5 days after the second dose

296. What will happen if we increase the dose of a product X


a) Increase the Rate of Elimination
b) Increase the Volume of Distribution
c) Increase Peak Plasma Concentration – Answer: Cmax
Dose = Vd x Cp, CL= Vd x k

297. Pharmacist concerned about not receiving the appropriate bonus after the annual review, in
his manager's opinion he was not performing well, while in his opinion he was doing five
clinical checks per week, which has increased to eight, which is a 60% improvement, what is an
appropriate criticism for that?
a) Pharmacist did not improve significantly over last year
b) Pharmacist was unfairly assessed
c) Objective was not measurable
d) There were no criteria set for measuring the objective or Lack of specific
performance crieteria assessment
e) Lack of Patients input feedback

298. Which one of the below can’t be used with Naïve Opioid patient?
a) Fentanyl Patch
b) Fentanyl Injection
c) Hydromorphone
d) Meperidine
e) Morphine
299. Which of these Formula is an exempted Narcotic?
a) Codeine 3.3 mg + 5 ml of Solvent
b) Codeine 8 mg + Substance X " non narcotic "
c) Methadone + Phenylephrine + Pseudoephedrine
d) Codeine 3.3 mg + 5 ml of Solvent + 2 substances " non narcotic "

300. Patient is on Morphine IV 15 mg Q4Hr., Dr wants to switch him to SR Tab. form if you
know that the equivalent dose is: Morphine 30 mg, Oxycodone 6 mg, Codeine 200 mg,
Hydromorphone 100 mg. what will be the suitable choice:
a) 300 mg Codeine Q12 Hr.
b) 60 mg Morphine Q 12 Hr.
c) 15 mg Oxycodone Q 12 Hr.
d) 100 mg Hydromorphone Q 12 Hr.
Morphine dose = 15 x 6= 90 mg = 18 mg oxycodone x 0.66= 12 mg daily (6 mg Q 12 hr)
= 300 mg hydromorphone x 0.66= 200 mg daily (100 mg
hydromorphone Q 12 hr)

301. What to monitor with Gabapentin?


a) Renal function
b) Liver function
c) CBC
Monograph: Gabapentin is removed from plasma by haemodialysis. Dosage adjustment in patients with
compromised renal function or undergoing haemodialysis is recommended

302. Aspirin toxicity with the level given of PH 7.2, Co2, and HCO3 all are less than normal.
what is the Expected problem?
a) Respiratory Acidosis
b) Respiratory Alkalosis
c) Metabolic Acidosis
d) Metabolic Alkalosis
e) Mixed condition
https://www.youtube.com/watch?v=5UDbuUm8abY
Cause of death in Aspirin toxicity: cardiac arrest due
to pulmonary edema.
pH: Low = acidemia
CO2: Low (respiratory alkalosis)
HCO3: Low (metabolic acidosis)
Apply "SMORE" on pH and CO2 = S (same) = M
(metabolic acidosis); look at HCO3 to confirm acidosis,
then look at anion gap, if provided.
303. Diabetic Patient with some heart problem, will need help from?
a) Social Worker
b) Diabetic Educator
c) Orthopedic
d) Dietician
e) Occupational therapist

304. Which is the most drug may cause Hepatotoxicity (do LFT)?
a) Valproic acid
b) Topiramate
c) Phenytoin
d) Gabapentin
Liver function should be measured before and then periodically monitored during the first 6 months of
therapy, especially in those who seem most at risk, and those with a prior history of liver disease.
Amongst usual investigations, tests which reflect protein synthesis, particularly prothrombin rate, are most
relevant.
Confirmation of an abnormally low prothrombin rate, particularly in association with other biological
abnormalities (significant decrease in fibrinogen and coagulation factors; increased bilirubin level and
raised transaminases) requires cessation of Convulex therapy.
As a matter of precaution and in case they are taken concomitantly salicylates should also be discontinued
since they employ the same metabolic pathway.
As with most antiepileptic drugs, increased liver enzymes are common, particularly at the beginning of
therapy; they are also transient.
More extensive biological investigations (including prothrombin rate) are recommended in these patients; a
reduction in dosage may be considered when appropriate and tests should be repeated as necessary.

305. What is true about Atopic Dermatitis (Pruritus)


a) Oil baths before taking a shower to prevent dehydration
b) Topical Pimecrolimus is more effective than corticosteroids
c) Oatmeal bath not equal effective as pre oil bath
d) Loratadine better than Diphenhydramine
Topical calcineurin inhibitors (TCIs) such as tacrolimus and pimecrolimus are analogues of cyclosporine and
have anti-inflammatory effects. They are highly effective in improving dermatitis and pruritus. Tacrolimus
was effective in the treatment of corticosteroid-resistant allergic contact dermatitis in 1 study and as
effective as a moderate-potency corticosteroid in another. Tacrolimus and pimecrolimus have been shown
to be effective for treatment of atopic dermatitis in children and adults. There is some evidence that
tacrolimus may have a greater effect than pimecrolimus over time. Topical calcineurin inhibitors may not be
as effective as moderate- to high-potency topical corticosteroids in the treatment of atopic dermatitis. Use
of TCI therapy 2–3 times weekly between atopic dermatitis flares is recommended on flare-prone areas to
help prevent relapses and decrease the amount of topical corticosteroid needed.
Colloidal oatmeal preparations contain starch and protein and are effective antipruritics. Addition of
oatmeal products may be soothing but does not promote increased water absorption. For dry skin, they are
not as effective as oils in trapping water to maintain hydration unless the oilated versions are used. Bathing
in colloidal oatmeal baths is useful when large body areas are involved.
Oil baths, if added at the beginning of the bath, they may prevent rather than enhance hydration and
thus, should be added near the end of the bath to trap water in the skin.

306. All should be taken on empty stomach EXCEPT:


a) Indinavir
b) Cefuroxime Axetil
c) Azithromycin (with or without food)
d) Levothyroxine
e) Residronate

307. Which substance that may be biologically interact with Quinolone? check
a) Erythromycin
b) Calcium
c) Warfarin
d) Digoxin
Digoxin + FQ = QT prolongation - - > biological DDI (most relevant pharmacodynamic DDI)
Calcium + FQ = physical binding - - > physical binding (pharmacokinetic)
Erythromycin + FQ = QT as well
Erythromycin and FQ work synergistically but this additive interaction is on the bacteria NOT directly related
to the body.

308. Hypertensive, Hyperlipidemic, Diabetic, Obese patient got chest pain, dyspnea &
diagnosed as Pulmonary Embolism, what is the main cause for his case:
a) Hypertension
b) Hyperlipidemia
c) Diabetes
d) Obesity

309. SW is 9 years old. He takes methylphenidate for ADHD. Today he came to your pharmacy
and told you that he wants to stop his medication because it makes him drowsy at school and he
found difficulty to fall sleep at night, furthermore he fells embarrassed when he takes the
medication at school, what is your advice to SW
a) Suggest changing to ER formula
b) Tell him don not be embarrassed
Psychiatric (anxiety, Monitor for difficulties falling asleep, Often worse upon initiation and
irritability, insomnia, tics) staying asleep and/or early morning resolves after 1–2 wk of therapy.
Stimulants, awakenings at 1 wk, then monthly for May need to lower the stimulant dose,
atomoxetine, the first 3 months, then Q 6 months. change time to earlier administration,
bupropion, Caregiver may use Sleep Disturbance add a more sedating medication at
venlafaxine Scale for Children or the Children’s bedtime or discontinue the offending
Sleep Habits Questionnaire to monitor stimulant. Minimize use of caffeine and
at home. other psychostimulants.
Limit stimulating activities (e.g., use of
electronic devices) in the evenings.

310. Which one not used in Colonoscopy in case of diverticulitis


a) NSAIDs
b) Ferrous Gluconate

311. Which preparation doesn't contain preservatives:


a) Total Parenteral Nutrition
b) Eye drops
c) Ear drops
d) Nasal spray

312. Pt taking Oxycodone and doctor want to change it to Fentanyl patch what is the right dose:
a) Fentanyl 25 mcg/24hr
b) Fentanyl 25 mcg/72hr
c) Fentanyl 25 mcg/48hr

313. Which RX need pharmacist intervention: (Azithromycin: concentration dependent)


a) Azithromycin orally once a day for 5 days
b) Azithromycin tid / 5 days
c) Azithromycin qd / 3 days
d) Azithromycin orally as a single dose (2gm in Gonorrhea)
In uncomplicated Chlamydia trachomatis urethritis and cervicitis, the dose is 1000 mg as a single oral dose.
For all other indications the dose is 1500 mg, to be administered as 500 mg per day for three consecutive
days. As an alternative the same total dose (1500 mg) can also be administered over a period of five days
with 500 mg on the first day and 250 mg on the second to the fifth day.

314. Metformin is Contraindicated at which Cr. Cl level


a) 20 ml / min
b) 30 ml / min (not recommended)
c) 60 ml / min (caution)
GFR should be assessed before treatment initiation and regularly thereafter, see section 4.2. Metformin is
contraindicated in patients with GFR <30 mL/min and should be temporarily discontinued in the presence of
conditions that alter renal function.
Decreased renal function in elderly patients is frequent and asymptomatic. Special caution should be
exercised in situations where renal function may become impaired, for example when initiating
antihypertensive therapy or diuretic therapy and when starting treatment with a NSAID.

315. A 65 yrs old male was diagnosed with MI,


what are the markers of ACS, all EXCEPT
a) Troponin II
b) CK
c) Serum Creatinine
d) Myoglobin

316. Terazosin is used in all EXCEPT:


a) Prostate Hypertrophy
b) Prostate Hyperplasia
Terazosin tablets are indicated for:
• The treatment of mild to moderate hypertension
• The symptomatic treatment of urinary obstruction
caused by benign prostatic hyperplasia (BPH).
317. Patient with Pre cataract surgery in her left eye then will do it in right eye after 8 weeks,
doctor prescribe for her 3 eye drops one was prednisolone and other 2 were moxifloxacin,
sundilac (anti-inflammatory) TID for 7 days then: What the best action to do:
a) Dispense all drops
b) Too high a dose
c) Too long duration
Anti-inflammatory agents: These medications are used for a few weeks postoperatively to reduce
inflammation and the risk of developing cystoid macular edema. A Cochrane review of low-quality
evidence demonstrated that ophthalmic NSAIDs alone or in combination with ophthalmic corticosteroids
may be more effective in controlling inflammation and preventing cystoid macular edema when compared
with ophthalmic corticosteroids alone. Anti-inflammatory agents may contribute to a faster visual recovery.

318. What was the reason for prescribing these eye drops?
a) Prevent infection and control inflammation
b) To decrease IOP
c) Decrease inflammation

319. After she did the surgery in her eye, she still has many drops left at home. What to do
a) She has to buy new eye drops
b) Hold for her till she needs them
c) Give her prednisolone only & use other two she has at home
d) Give her other two & continue on prednisolone she has at home

320. When you counsel patient what is true to increase absorption of the eye drops?
a) Blink the eye quickly for 1-2 min
b) Leave the eyelids closed and the finger pressing gently for 2 min
c) Separate between each drop from each drug from 3-5 min (false: minimum five min.)
d) Put cold pack for 2 min on closed eye
Therapeutic Tips
Any changes to postoperative ophthalmic medications should be discussed with the treating ophthalmologist.
Initiate topical antibacterials immediately following surgery rather than waiting until the 1st
postoperative day.
Advise patients to separate the administration of different eye drops by a period of at least 5 minutes. If there
is a contraindication to systemic absorption of the medication, counsel the patient to close the eye and, while
trying to avoid touching the operated eye, put pressure on the inner canthus for 30–60 seconds after instilling
drops. This is done to reduce the transfer of ophthalmic medication to the nasal and/or oral mucosa where it
may be absorbed systemically.
In all other patients, simply closing the eye for 30–60 seconds may be sufficient to maximize ophthalmic
absorption.
Any worsening of vision, floaters or eye redness, especially in the 1st postoperative week, should be
considered endophthalmitis until proven otherwise and requires urgent assessment by an ophthalmologist.
Treatment with many medications in this setting is for a limited course; therefore, any unused
ophthalmic medication should be disposed of properly.
Patients having clear corneal cataract surgery should take all their usual medications (including
anticoagulants and antiplatelet agents) on the day of surgery except for some diabetic medications. Due to
the risk of hypoglycemia, insulin and insulin secretagogue (sulfonylureas, meglitinides) doses may be
modified or the medication may be held the day before or on the day of surgery. Metformin may be held the
day before and on the day of surgery due to the risk of lactic acidosis. If medication doses and/or regimens
are modified, more frequent glucose monitoring may be required.

321. Which eye drop will cause Iris Pigmentation?


a) Latanoprost
b) Dorzolamide
c) Timolol
d) Brimonidine
e) Pilocarpine

322. Vancomycin S.E in infusion made flushing what to tell patient about that?
a) This S.E or rash through systemic absorption

323. NuvaRing all except:


a) Out of fridge 3 days
b) It needs back up for first use
c) If left for more than 3 weeks it need pregnancy test
Prior to dispensing to the user, store refrigerated 2-8°C (36-46°F). After dispensing to the user, NuvaRing can
be stored for up to 4 months at 25°C (77°F); excursions permitted to 15-30°C (59-86°F) [see USP Controlled
Room Temperature].

324. 3 years child has constipation


for 1 week, he will be referred to
doctor if?
a) Has Abdominal pain
b) His stool forms not
consistent
325. Which one of the following requires immediate doctor consultation?
a) 7-year-old has anal itching
b) 3 years child with constipation for 10 days
c) 3-month child who have fever (to rule out meningitis)
d) 9-month child with Diahrria

326. A lady who has burnt her fingers and


wants to relieve pain immediately, which
one the following should she be using
a) Acetaminophen
b) Ibuprofen
c) Lidocaine Gel
d) Methyl salicylate
Usual doses of nonprescription analgesics (e.g.,
acetaminophen, ibuprofen) are often sufficient to
control pain in minor burns.
ASA should be avoided in superficial partial-thickness or deep partial-thickness burns because platelet
inhibition poses a risk of bleeding in the presence of open wounds. Deeper burns with open areas often
require opioids for adequate pain management.
Local anesthetics are a common ingredient in topical products marketed for the relief of minor pain and
itching associated with superficial burns. There is no evidence to substantiate their effectiveness. Lidocaine
and benzocaine commonly cause contact dermatitis, while pramoxine has low sensitizing potential. There is
a risk of systemic absorption if local anesthetics are applied to blistered or large areas of skin. Camphor,
menthol and phenol are counterirritants thought to have a cooling effect on the burned area and to provide
some relief from itching. However, there is no evidence available for the use of counterirritants in the
treatment of minor burns.
Another version: Topical ttt for neuropathic pain: lidocaine
327. A mother is afraid from vaccinating her child because of needle pain, what should you tell
her before injecting him?
a) Give diclofenac 12.5 supp.
b) Breastfeed him before and after vaccination (before, during, after vaccination)
c) Lay child supine before giving vaccine (hold you baby)
d) Cold compresses
e) Use topical anesthetic Lidocaine 10 -20 min before (20 to 60 minutes)
f) Give Acetaminophen 10-15 mg kg 30 min before vaccination
g) Tell him it is not painful
h) Tell nurse to hold him while he is in upright position

328. which capsule can you crush and give through J-tube (jejunum)
a) Clindamycin 300 mg
b) Dutasteride 0.5 mg
c) Tamsulosin CR
d) Diltiazem CD
329. Woman came back with a prescription for Rizatriptan Wafer. Which of the following is a
true statement about Rizatriptan Wafer?
a) It is absorbed from the buccal cavity  from stomach, swallow
b) Co-administration with alcohol is contraindicated
c) It is used for migraine with nausea
d) It is absorbed faster than Rizatriptan tablets
e) It is contraindicated with people who have difficulty swallowing

330. Early carbamazepine toxic signs and symptoms


a) Petechial hemorrhage
b) Weight loss
c) Diarrhea
d) Blurred vision
Signs and symptoms of carbamazepine toxicity may include
the following: Drowsiness. Slurred speech. Ataxia.
Hallucinations. Nausea, vomiting. Tremors. Seizures.
Oliguria.

331. Which one of the following is recommended for


ACS as the initial DOC
a) Clopidogrel
b) Ticlopidine
c) ASA
d) Warfarin
332. Patient diagnosed with dementia treated with Donepezil. When we can monitor the
improvement? (3-6 months to kick in)
a) 2 weeks
b) 3 - 6 month
c) 1 month
d) 24 months
Donepezil was effective in 3-6-month trials in patients with mild to moderate Alzheimer's disease (MMSE
scores 10 to 26).
Monitor treatment effects 2 weeks after initiating therapy or increasing dosage, then every 3 months.

333. MI patient released from hospital which of the following is not recommended after his
release
a) Isosorbide dinitrate
b) Furosemide
c) Lisinopril
d) ASA
334. Hypertension complication will affect all of the following EXCEPT?
a) Ventricle enlargement
b) PAD
c) Tricuspid valve dysfunction
d) Hypertrophy
e) Stroke
f) Retinopathy

335. Testosterone 1% gel? to improve mood

336. Patient received wrong medication in hospital. Who to contact first?


a) Patient
b) Pharmacist
c) Physician
d) Nurse

337. Hormonal replacement therapy: decrease bone loss

338. Advantages of bivalirudin over other anticoagulants?


a) Safer bleeding profile than other anticoagulants.
Argatroban and danaparoid are approved for treatment of heparin-induced thrombocytopenia
(HIT) in Canada. Limited data indicate that bivalirudin and fondaparinux may also be effective.

339. Which one decrease the concentration of tamoxifen:


a) Fluovoxamine
b) Paroxetine
c) Citalopram
Paroxetine & Fluxetine are strong inh. Of CYP2D6

340. Peripheral venous catheters can help in all of these except:


a) Multiple injections for many drugs be given same time
b) Help people doing activities like hockey, tennis….
c) Helpful for people who need drugs for short time less than a week

341. Which of these needs washing out as 10 days?


a) Fluoxetine to sertraline
b) Phenlzine to venlafexine
c) Venlafexine to citalopram (6 – 8)
342. HbA1c %, what is the most appropriate condition for glycemic control measured by
glycosylated heamoglobin OR Which one of these HBA1C result is consistent?
a) Pregnant woman in her First trimester
b) African-american female with sickle cell anemia
c) Heart condition
d) Chemotherapeutics for prostate cancer
e) Man 66 yrs has ESRD (End Stage Renal Disease)

343. Patient with Dandruff and Seborrhea taking hydrocortisone for 2 weeks and no
improvement. What’s your recommendation?
a) Continue on hydrocortisone for another 2 weeks
b) Stop hydrocortisone and start Ketoconazole
c) Refer to dermatologist
d) Use a cool air humidifier
e) Add Salicylic acid
f) Add coal tar
Topical Antifungal: first line treatment (ketoconazole alone)

344. SSRI is contraindicated with? St. john wort


345. Child 8 yrs. has no dyspnea, no sputum, just he has cough and he have atopic dermatitis.
what to do?
a) Give him OTC Dextromethorphan
b) Refer to Dr (as atopic dermatitis may be a symptom of asthma)

346. New ofloxacin what study need to be done for?


a) Double blind placebo
b) Randomized comparative trial

347. Renal patient has a new Rx of Norfloxacin what you have to do.
a) Keep dose and decrease the interval
b) Keep interval and decrease dose
c) Decrease interval or decrease the dose
d) Keep interval and Keep the dose
e) Keep dose and increase interval

348. Patient had refills in basket in your pharmacy you are MANAGER in, you noticed it has
been 2 weeks no one picked it, then you knew the patient has died. After that you requested the
insurance company not to bill this patient. then you returned the medicines to shelf. What ethic
you are doing here?
a) Autonomy
b) Veracity
c) Fidelity

349. Pharmacist knew that specific drug will be short for a while, so he kept all remaining
amount to his mother who uses same, what ethics he is VIOLATED?
a) Veracity
b) Non malificence
c) Justice

350. Measurement of Cushing disease by using ?


a) Prednisone
b) Trimicinolone
c) Dexamethasone
Dexamethasone for diagnosis of Cushing syndrome

http://www.endocrinesurgery.net.au/cushings-diagnosis/

351. You have subs. A conc is 4% to be added to subs B to have à final volume of 50 ml. After
mixing both subs. The conc of subs. A in the final solution was 0.1 %, Calculate the amount of
subs. B in Mg in each dose if you divided the total solution to 5 small doses. Conc of subs B is
10% w/v.
Answer : C1*V1 = C2*V2 4%*V1 = 0.1%*50
V1 = 0.1%*50/4% = 1.25 ml which is the final volume of subs. A
Volume of subs. B = 50 -1.25 = 48.75 ml
10 gm ------- 100 ml X gm ------- 48.75 ml
X = 48.75 * 10/100 = 4.875 gm 4.875mg /5 doses = 0.975 gm in each dose.

352. Which of these situations will show Autonomy?


a) Explain what benefits and risks of drug
b) Discuss with patient about risk and benefit in confidential area
c) Discuss with patient why he is not eligible for taking certain medicine

353. 6 years old (pt weight 100 lb) child the Dr. ordered a loading dose of Phenytoin 20mg/Kg
infused with a rate of 0.5 mg/Kg/min. If you've it in 100 ml bag. what is the rate of pump that
you must be adjusted in ml/hr.?
Answer:
0.5 mg ------- 1 kg ------- 1 min 20 mg ------- 1 kg ------- X X = 1*20/0.5 = 40 min.
100 ml ------- 40 min Y ml ------- 60 min Y = 60*100/40 = 150 ml
So, rate = 150 ml/ hr
354. Patient in hospital is taking IV Cotrimoxazole (each ml contains 80 mg SMT / 16mg TMP)
0.5 ml Q6hr. Dr. want to shift him to liquid (200 mg STM / 40 mg TMP) in each 5 ml, how
many of liquid is needed to provide the equivalent dose as was taken IV.
Answer:
0.5 ml Q6hr = 0.5 * 4 = 2 ml /Day
SMT: 80 mg ------- 1 ml X mg ------- 2 ml X= 2*80/1 = 160 mg
Liquid 200 mg ------- 5 ml 160 mg ------- Y ml Y = 160*5/200 = 4 ml/Day

355. Patient on Valproic acid, he took (total amount 1300 mg divided as Tid) for Absence
seizure, he is stabilized on it, but he had some diarrhea from it, what best action for him:
a) Change to Ethosuximide
b) Make it Bid
c) Decrease the dose by 20 %
(Divalproex, if not, then dose reduction, if not, then ethosuximide.)
Childhood Absence Epilepsy
For childhood absence epilepsy, ethosuximide and valproic acid are similarly effective in preventing
seizures, and both medications are superior to lamotrigine. Ethosuximide is associated with lower rates
of attention difficulties and fewer behavioural problems than valproic acid and is usually the drug of first
choice in childhood absence epilepsy.[38] However, ethosuximide is not effective in preventing other seizure
types such as generalized tonic-clonic seizures, and should not be used as monotherapy in children with
multiple seizure types.
Gastrointestinal disorders:
Very common: nausea
Common: vomiting, gingival disorder (mainly gingival hyperplasia), stomatitis, gastralgia, diarrhoea
The above adverse events frequently occur at the start of treatment, but they usually disappear after a few
days without discontinuing treatment. These problems can usually be overcome by taking Epilim with or
after food.
Uncommon: pancreatitis, sometimes lethal
Dosage
Dosage should start at 600 mg daily increasing by 200 mg at three-day intervals until control is achieved.
This is generally within the dosage range 1000 – 2000 mg per day, i.e. 20 – 30 mg/kg/day body weight.
Where adequate control is not achieved within this range the dose may be further increased to 2500 mg per
day.

355. Methotrexate error happened instead of dispensing one dose weekly given one daily, how
this could be prevented
a) Implement new system alarm for Dr to alert him for Mtx weekly regime
356. Error happened in giving hydromorphone instead of Sterile water, that was in patient care.
How that could be prevented?
a) By putting note in care unit for nurse
b) By putting colored labels on each bottle
c) By using opaque bottle for one and transparent for other one

357. Which one of these is best for drug distribution?


a) Unit dose for individual
b) Decentralized distribution

358. New system in a pharmacy enable pharmacist to see a picture of Rx in refilling on the
patient`s profile. This technique will help in?
a) Decrease the error continuation
b) Notice any dose changes happening

359. Patient had two oxycodone one is SR and other IR; he is elder and he uses senna 2 tablet
and docusate. Still not enough for his constipation, what to do next?
a) Giving him MgOH
b) Give him enema
c) Increase senna dose

360. COMPUS, what does it do


The Canadian Agency for Drugs and Technologies in Health has (CADTH) created the
Canadian Optimal Medication Prescribing and Utilization Service (COMPUS), to work
with federal, provincial, and territorial ministries of health to identify and promote
evidence based best practices in drug prescription and utilization among healthcare
providers and consumers.
361. Pharmacy assistant inform a manager that pharmacist had bad behavior with the
technicians, as a manager what is considered good feedback way to pharmacist?
a) Discipline action
b) Let himself specify concerns for technicians
c) Emphasize of necessity respect the technician staff
d) Ask the Pharmacist in person to suggest the ways to improve the behavior and
resolve the conflict.

362. Patient has DM using Metformin, other symptoms but she now has low saditary and gain
some weight. what action could be best to add on her medicine?
a) Gliclazide
b) Acarbose
c) Sitagliptin

363. Patient on Clopidogrel, ASA, Atorvastatin 80 mg, he took diclofenac / misopristol before
admission to hospital. What would be problem here?
a) Continue Diclofenac

364. Which could make him diarrhea?


a) Atorvastatin
b) Misoprostol (EE)

365. He now complaining from muscle pain what would be the reason?
a) Interaction between Atorvastatin and other drugs
b) High dose of atorvastatin (myopathy- rhabdomyolysis)

366. What action as a pharmacist will do? Report this case to Hospital to review the reason

367. This Myalgia patient what best to consult?


a) Emergency
b) Physician within 48 hrs
c) Cardiologist
d) Tell her Dr about that in her next scheduled visit

368. A pregnant patient wants to ask about the side effects of Vanlafaxine from a pharmacist,
which one of the following is the best suitable resource relevant to the scenario
a) Merck Manual
b) Rx files
c) Mother risk
369. Child with croup, what to tell his parents?
a) No worry croup symptom will be finished with 48 hrs
Viruses are the most common cause, particularly parainfluenza virus types 1 and 3. Influenza A and B,
adenovirus, respiratory syncytial virus (RSV), metapneumovirus, coronavirus & mycoplasma have also
isolated.
Sounds like seal/barky cough with rhinorrhea & fever. Stridor, chest tightness, and respiratory distress.
Symptoms of croup resolve mostly within two days but can persist up to one week.
Risk factors include: Ages 6 months - 3 years old & Male gender
Dexamethasone: Mainstay of croup therapy. 0.15–0.6 mg/kg PO/IM/IV once.

370. A 42-year-old male presented to a clinic with a complaint of twitching in his legs. He
reported nightly discomfort for the past two years. He noted that it occurred once he was in a
semi-reclined supine position watching television in bed for a period of time. The discomfort
was described as “pressure” without pain or paresthesia, occurring more frequently in his right
leg. The presenting complaint sometimes caused him to wake up at night and roll over in bed.
Relieving factors included shifting positions and “shaking out the leg.”. Triggers that linked
with increase in RLS symptoms could be all of the below EXCEPT
a) Caffeine
b) Periods of inactivity
c) Stretch your legs before bedtime.
d) Long-distance flights
e) Immobilization, such as a cast
371. All of the below are non-
pharmacological treatment for RLS and
may help EXCEPT
a) Try hot baths and massages.
b) Moderate, regular exercise.
c) Try relaxation techniques, such as
meditation or yoga.
d) Establish good sleep hygiene
e) Complete Bed rest

372. What is the Drug of Choice (DOC) to treat RLS (Restless Leg Syndrome)?
a) Reminyl (Galantamine)
b) Mirapex (Pramipexole)
c) Cymbalta (Duloxetine)
d) Elavil (Amitriptyline)
e) Effexor (Venlafaxine)

373. Now he is in hospital what to be notified in his record?


a) Dose change
b) Patient refuse taking medicine
c) Efficacy

374. Levothyroxine 0.1mg once daily. other medications like atorvastatin 40 mg, clopidogrel.
Taken to elder patient (65 yrs) she is complaining from diarrhea, what DTP?
a) High dose of atorvastatin
b) Interaction between atorvastatin and clopidogrel
c) High dose of levothyroxine (this true answer as 0.1 mg is too high for elder pt)
Adults
Initially 100 micrograms daily, preferably taken before breakfast or the first meal of the day. Adjust at three
to four-week intervals by 50 micrograms until normal metabolism is steadily maintained. The final daily
dose may be up to 100 to 200 micrograms.
Elderly: As for patients aged over 50 years.
For patients over 50 years, initially, it is not advisable to exceed 50 micrograms daily. In this condition, the
daily dose may be increased by 50 micrograms at intervals of every 3-4 weeks, until stable thyroxine levels
are attained. The final daily dose may be up to 50 to 200 micrograms.
Patients over 50 years with cardiac disease
Where there is cardiac disease, 25 micrograms daily or 50 micrograms on alternate days is more suitable. In
this condition, the daily dose may be increased by 25 micrograms at intervals of every 4 weeks, until stable
thyroxine levels are attained. The final daily dose may be up to 50 to 200 micrograms.
375. Which of the following is used prospectively to prevent a medication error?
a) Failure Mode and Effect Analysis (FMEA)
b) Root cause Analysis (RCA)
c) Medication Incident report
Failure Mode and Effects Analysis (FMEA) is a structured approach to discovering potential failures that may
exist within the design of a product or process. Failure modes are the ways in which a process can fail.
Effects are the ways that these failures can lead to waste, defects or harmful outcomes for the customer.

376. New drug, you don`t know whether its approved or not. Where to know this info?
a) Health Canada Med effect
b) Drug product data base

377. Who to report to if unusual side effects or not mentioned by manufacture are observed?
a) Canada Vigilance

378. What to monitor in Ziduvodine?


a) CBC
b) Lipase

379. What`s true about Provincial Regulatory Act role?


a) Its related to Health Canada
b) Not to be mandated in the future
c) It can be changed for coping occupation changes
The College can introduce changes effective immediately without the need to pass parliamentary bills; eg,
COVID-19 practice guidelines updates.

380. Provincial Regulatory Act for?


a) Provide practical guidelines for the practice
b) For patient care interest

381. What`s true about provincial drug policy?


a) The formularies in province are made by
national formulary system

382. Patient had osteoporosis taking Ca supplement,


other medicines she is 60 yr. Dr prescribed for
diphenhydramine, what’s your concern?
a) This drug will increase risk of falling
383. Mother has toe nail fungal infection, what is best choice?
a) Oral Ketoconazole
b) Oral Terbinafine
Efinaconazole (benefit/risk). AVOID systemic or defer to after deliver.
Choices during Pregnancy
Since fungal nail infections are generally nonurgent problems and require many months of systemic
treatment, it is prudent to delay treatment until after pregnancy. If drug treatment must be implemented,
topical local therapy is preferred over oral systemic therapies.
Topical antifungals all display limited, if any, systemic absorption so is theoretically considered safe in
pregnancy; consider their use only when benefits outweigh potential risks.
In animal studies of systemic antifungals, teratogenic effects have been observed with use in pregnancy.
Fluconazole, used in high doses (>400 mg daily) for a prolonged period of time during the 1st trimester of
pregnancy, may be associated with fetal malformations (craniofacial, skeletal and cardiac malformations).
Large epidemiological studies of fluconazole for use in pregnancy have mixed results; more data
are needed to confirm the risks of fluconazole when used for onychomycosis.
Although experience with itraconazole use in pregnancy is limited, prospective studies with first-trimester
treatments for an average of 12 days and a maximum of 90 days, revealed no increased rate of major
malformations. However, rates of spontaneous and induced abortion were higher in the exposed group.
Larger studies are warranted to confirm these data. Animal reproduction studies with terbinafine do not
suggest any embryotoxicity, but human pregnancy data are lacking.
Choices during Breastfeeding
Systemic treatment of the mother with itraconazole or terbinafine should be avoided during breastfeeding
since these drugs accumulate in breast milk. It is not known whether fluconazole is compatible with
breastfeeding when used for many weeks in the treatment of OM. Short-term (2–3 wk) treatment with
fluconazole is acceptable in nursing mothers for other conditions. The breastfed infant is expected to receive
>15% of the maternal dose of fluconazole with a half-life in milk of 30 hours.

384. All these are non-pharmacological advices about nail fungal infection, EXCEPT?
a) Avoid bare foot
b) wear socks
c) Put antifungal powder
Persons with OM should be treated for any associated tinea pedis. Prevention and nondrug strategies include:
- Wearing footwear and socks that minimize humidity
- Drying feet and interdigital spaces thoroughly after washing
- Using footwear to avoid fungal transmission from shared public spaces (i.e., avoid being barefoot)
such as swimming pools and nail salons
- Keeping nails clean and cut short
- Avoiding sharing nail clippers or footwear
- Preventing further trauma to toenails; wearing nonrestrictive footwear or fitted orthotics may be helpful
- Discarding old, infected footwear
- Wearing rubber gloves to protect fingernails if hands are immersed in water for long periods of time
- Applying emollients on cracked skin to reduce further entry points for fungus
- Controlling chronic health conditions, such as diabetes mellitus or peripheral vascular disease
385. What to tell patient taking zopiclone
a) Avoid taking grapefruit  zopiclone is a substrate of CYP3A4
d) Can be taken in patients with myasthenia gravis
e) Safe in impairment of respiratory function (e.g., significant sleep apnea syndrome).
Grapefruit and grapefruit juice should be avoided when taking Zopiclone Orion. Grapefruit may increase the
effect of Zopiclone Orion. If you are pregnant or breast-feeding, think you may be pregnant or are planning
to have a baby, ask your doctor or pharmacist for advice before taking this medicine

386. Hospital Committee (P&TC) accept new drug (D) which is same pharmacology class of
existed drugs (A, B, C). But new drug (D) is more expensive by 80% than old drugs. What of
these options would be a reason for this committee to accept adding new Drug?
a) BCZ it gives slight improvement in surrogate
b) New drug has more available concentrations and doses
c) Manufacturer offered to hospital discount on new drug
d) Give them some details about seizure from written pamphlet
Cheaper (discount) to beat other deals since same pharmacology

387. Benzocaine cream/ointment/gel 0.1% ratio is 1:2:2 and total qty mitte 150g what is percent
of benzocaine?
a) 20.0%
b) 0.03%
c) 0.02%  Total 5 so 0.1 /5 = 0.02
d) 0.2%
e) 0.3%

388. Patient has cerebral ischemia, he has also asthma (he never said anything related to heart
problem), he wants antithrombotic drug what to give?
a) Aspirin (not true bcz he has asthma)
b) Clopidogrel
c) Warfarin

389. Patient is going to USA for 4 days, he had Rx Lorazepam. Last fill was 6 days ago with
amount 14 pills his Sig was One pill PRN, he came to you asking for double refill (2*14) because
of his travel. What will be your action if you also know that he has history of alcoholism?
a) Refuse and give him a copy of his Rx
b) Give him & document early refill
c) Call Physician to ask him
d) Give him 14 tabs (one refill) only
Call physician to authorize an early refill (as long as there's an interval set by the physician; it has to be
honored by the pharmacist)

390. Female had a drug filled from your pharmacy she had filled as 90 tablets (as 3 refills), but
she misplaced it and then she had new Rx from Doctor, her insurance company refuse to pay for
misplaced drugs, what will be your action?
a) Give her some of pills in advance of refill
b) Tell her she must pay for this new Rx

391. A drug has been recalled (Oral Contraceptives) and it has been observed that by mistake
the company has sent the Placebos instead of the actual drug, what should be The Pharmacist’s
first initiative taken at the Pharmacy.
a) Tell Drs nearby about this problem
b) Inform the staff about it
c) Do nothing, just stock the medication.
d) Call the Customers who had taken the Placebo medication to be replaced by Oral
Contraceptives.
e) Report to ISMP

392. Patient taking phenytoin for seizure, she doesn`t like condom, she wants emergency
contraceptive pill. what advice to give? (double the dose)
a) It can be taken for daily use
b) There would be some spotting in first use

393. Now he uses phenytoin tid, he has stomach pain. What good option for him then?
a) Give him Solution
b) Make it Once daily
c) Make it Bid
Dose and salt must be known

394. What is true about phenytoin? Given with food to decrease GIT S. E

395. New immigrant to Canada m she doesn’t speak English well, her child (9 years old) speaks
English well, he had fallen in school and had contractions in his leg. he has seizure and he
thought that some spirit is moving on him, you noticed that his mother is not comfortable while
you speak to her child, what least action would be appropriate for this case?
a) Tell him what risk & benefit of using anti-seizure medicine
b) Slowly talking to him
c) Seizure has no relation to spirit thoughts he has
d) Show them some visual pictured about seizure
e) Give them some details about seizure from written pamphlet
You don't have to speak slowly, this is NOT the accurate version of the Q

396. New medicine to be in your pharmacy, as a manager what first thing to consider?
a) Price or offer given
b) Need of this medicine among your patients

397. You are a manager, and knew there are many various culture differences in your
customers, what will be your action?
a) Seek analysis each of culture for clients
b) Analyze social economic demography of your neighborhood

398. Calculation. You will add some of (A) 3 % to another one, then the final volume is 250 ml
and final concentration is 10%, how many mls of diluent needed to add?
I do not remember the exact numbers but the idea here was like following;
Q1C1=Q2C2 then Q1 100 ml So, the diluents needed (250 -100 = 150)
this is a trick as there was in option 100 ml (don`t be quick and choose it as he asked about
diluent to be added) 150 ml (true)

399. Patient had renal failure taking atorvastatin, what is important to measure as baseline?
a) Liver enzymes
b) CK

400. Woman 60 yrs old, she has urinary incontinence she wants something oral, what to give?
a) Estrogen (Local or vaginal, not oral)
b) Tolteridine

401. Something about which of these medications increase Qtc?


a) Levothyroxine
b) Quetiapine (remember methadone + Quetiapine = increase QT prolonagation)

402. Elder woman has Osteoporosis; she falls many times, what other specialist other than
Physiotherapist you will recommend?
a) Occupational therapist
b) Nutritionist
403. In Seminar or lecture for elder women in Long term residence, what topic you will focus
on?
a) New devices or walkers for helping them not to get fall
b) Importance of having care givers or someone to look after them

404. Patient taking antihyperthyroidism, what advice to tell?


a) Shift methimazole to PTU before pregnancy
b) Keep taking Methimazole before pregnancy (CI in preganacy- PTU in first
trimester then continue methimazole)
c) Decrease dose of anti-hyper thyroidism

405. Patient has Raynaud’s & neuralgia, is on Amitriptyline 10mg PRN & on Metoprolol 200
BID for 1 year (high dose) now patient complains that Raynaud’s getting worse and still
complaining of neuralgia. What wrong with this regimen
a) Neuralgia worsen Raynaud’s
b) High dose Metoprolol (maximum dose: 300 mg daily)
c) Amitriptyline used for long time
d) High dose Amitriptyline
Hypertension
Initially 100mg daily. This may be increased, if necessary, to 200mg daily in single or divided doses.
Combination therapy with a diuretic or vasodilator may also be considered to further reduce blood pressure.
Metoprolol may be administered with benefit both to previously untreated patients with hypertension and
to those in whom the response to previous therapy is inadequate. In the latter type of patient, the previous
therapy may be continued and metoprolol added in to the regime with adjustment of the previous therapy if
necessary.
Angina
Usually 50-100mg two or three times daily. In general, a significant improvement in exercise tolerance and
reduction of anginal attacks may be expected with a dose of 50-100mg twice daily.
Cardiac arrhythmias 50mg two or three times daily is usually sufficient. If necessary, the dose may be
increased to 300mg daily in divided doses.
Following the treatment of an acute arrhythmia with metoprolol tartrate injection, continuation therapy
with metoprolol tablets should be initiated 4-6 hours later. The initial oral dose should not exceed 50mg
twice daily.
Myocardial infarction - early intervention
In order to achieve optimal benefits from intravenous metoprolol, suitable patients should present within 12
hours of the onset of chest pain. Therapy should commence with 5mg iv every 2 minutes to a maximum of
15mg total as determined by blood pressure and heart rate. The second or third dose should not be given if
the systolic blood pressure is less than 90mmHg, the heart rate is less than 40 beats/minute and the P-Q
time is greater than 0.26 seconds, or if there is any aggravation of dyspnoea or cold sweating. Orally,
therapy should commence 15 minutes after the injection with 50mg every 6 hours for 48 hours. Patients who
fail to tolerate the full i.v. dose should be given half the suggested oral dose.
Maintenance
The usual maintenance dose is 200mg daily given in divided doses. The treatment should be continued for at
least 3 months.
Thyrotoxicosis: 50mg four times daily. Dose should be reduced as euthyroid state is achieved.
Prophylaxis of migraine: 100-200mg daily in divided doses (morning and evening).

406. What the correct action? Give him XL Nifedipine (CCB – 1st line in Raynaud's)

407. Dog bite which causes lesion which Antibiotic appropriate?


a) Cephalexin
b) Cefadroxil
c) Mupirocin
d) Amox/Clav

408. Clostridium difficle has occurred in patient, he taking antibiotic Ceftraixone. He also
taking atorvastatin. what could happen?
a) Drug-Drug interaction – Lexicomp, no interaction

409. What is NOT true regarding C. difficle treatment (like he means one these is not a sign of
treatment of clostridium)?
a) Stool test done weekly
b) Fever will decrease in 3 or 4 days
c) Other options I don’t remember

410. Malaria Pt, travelling to Mexico after 2 weeks, he has Rx of chloroquine what to advice?
a) Combine with DEET and Sunburn is effective for him
b) Start medicine tomorrow every week till you come back
c) Use anti mosquito from dusk and
dawn

411. Which one is micro complication of


Diabetes?
a) Diabetic leg infection
b) Coronary artery problems
c) Peripheral vein problem
412. Pt had atorvastatin 80 mg took it in Hospital he had myalgia where to report this problem?
a) Hospital which he took this Rx from

413. Now what to advice him?


a) Discuss this matter with your next scheduled visit
b) Discuss this matter with your cardiologist
c) See physician within 48 hrs

414. Assessment in SOAP like?


a) Drug addition to his old profile
b) Patient symptoms
c) Drug interaction due to CYP 450 (the problem)
The assessment includes a list and brief summary of the patient’s active issues along with a description of
any significant changes since the last time you took notes.

415. All of the following can be used for therapeutic drugs for disease issue EXCEPT?
a) Remington (mainly about chemistry, stability properties)
b) CPS

416. Patient taking medication, his Dr does not know what best for him suitable to his disease.
where best to find the info?
a) Applied therapeutic book
b) Other books not seem to be relative (don’t remember)

417. Forgot one pill of OC what to tell? Take it as soon then continue regularly

418. New off-label where to find? Pubmed

419. What is the DRP is the following.


a) Allupurinol with ASA.
b) Allupurinol with Warfarin DDI
c) Allupurinol with Azithromycine
6-mercaptopurine and azathioprine: Azathioprine is metabolised to 6-mercaptopurine which is inactivated
by the action of xanthine oxidase. When 6-mercaptopurine or azathioprine is given concurrently with
Allopurinol, only one-quarter of the usual dose of 6-mercaptopurine or azathioprine should be given because
inhibition of xanthine oxidase will prolong their activity.
Salicylates and uricosuric agents: Oxipurinol, the major metabolite of allopurinol and itself therapeutically
active, is excreted by the kidney in a similar way to urate. Hence, drugs with uricosuric activity such as
probenecid or large doses of salicylate may accelerate the excretion of oxipurinol. This may decrease the
therapeutic activity of Allopurinol, but the significance needs to be assessed in each case.
Coumarin anticoagulants
There have been rare reports of increased effect of warfarin and other coumarin anticoagulants when co-
administered with allopurinol, therefore, all patients receiving anticoagulants must be carefully monitored

420. What is the Primary Prevention of


Hypertension
a) Reducing the BP to 125/85
b) Reducing Calorie intake
c) Weight Loss

421. A patient has been using Metformin 500 mg BID for a while but there seems to be no
improvement in his blood sugar control, which one of the following should be included with
current medication.
a) Insulin
b) Glyburide
c) Pioglitazone
d) Roziglitazone

422. Which of the following causes weight Gain?


a) Estrogen
b) Progesterone
c) Topiramate
Weight increased or decreased with progesterone

423. When rizatriptan can be reused after having used once earlier?
a) 8 hours after the first administration if it is ineffective
b) 2 hours after administration of the first dose if it is effective

424. All are Parkinson’s symptoms


Except
a) Tinnitus
b) Akathisia
c) Dystonia
d) Hypotension
425. A question on the fact that a Pharmacist got a needle pricked while participating on a
procedure on Hepatitis C patient at a clinic. What should be the first action that ought to be
taken by him after the prick?
a) Take antibiotic right away
b) Wash hands with a detergent or soap
c) Do nothing at all.

426. Ondansetron SE
a) Confusion
b) Dizziness
c) Fast Heartbeat
d) Fever
e) Headache (Very common: Headache & constipation)
f) Shortness of Breath

427. Mother coming to you in the pharmacy with her child and upon checking him you
suspected conjunctivitis, you should tell her:
a) It is contagious
b) It is viral conjunctivitis treated by oral antiviral
c) It is better to see the doctor to better assess his condition
When should you seek more medical advice?
 You have pain or severe redness in your eyes
 Your vision is altered—you can't see as well as usual
 You have had this problem before
 You have a disease (such as diabetes) along with your eye problem
 You have used a nonprescription treatment for 48 hours or more but the condition has not improved
 The condition gets worse with treatment
 Without treatment, the condition has lasted longer than 48 hours
428. A patient who has had his heart valve replaced, which one of the following he should be
having once the valves don’t work
a) Lungs
b) Heart Failure
c) Infection

429. Which one of the following should be avoided with Levothyroxine?


a) Vit D
b) Vit A
c) Calcium

430. How long does it take for the Mood Symptoms to get better while on SSRIs?
a) 2 weeks
b) 1 week
c) 4-6 weeks

431. A long scenario on Hypertension patient 145/90, which one of the following should be
used
a) BB
b) Tamsolusin
c) Clonidine

432. A case on traveler’s diahorrea the person has had Dukoral, does her wife need?
(if pregnant no)

433. Does the child need? he is 18 months, but breastfed? (No, more than 2 years Can start)

434. Which antibiotic for child


a) Erythromycin
b) Azithromycin

435. Ear pull method for installing kids, what is given for pain relief? Ibuprofen

436. What are the side effects you should monitor in a patient taking Raloxifene?
a) Hot flashes
b) Hypercalcemia
c) Uterine bleeding
d) VTE
e) MI
437. Patient with Splenectomy, more susceptible to which M.O:
a) Streptococcus Pneumonia
b) Staph. Aureus
c) Influenza Virus
d) Neisseria Meningitis
What is this patient’s risk of infection after splenectomy?
Elective splenectomy is indicated in the management of certain medical conditions. Patients who have
undergone splenectomy are at risk of overwhelming postsplenectomy infection, which is characterized by
sepsis, meningitis or both, and carries a fatality rate of 50%–70%. These episodes occur in patients who have
had splenectomy at a rate of 0.2%–0.5% per year, with a lifetime risk of about 5%. S. pneumoniae is the
most common pathogen (> 50%), followed by other encapsulated bacteria such as H. influenzae and N.
meningitidis, and less commonly by gram-negative organisms such as E. coli, Salmonella and Pseudomonas.
What immunizations are recommended before and after elective splenectomy?
To reduce the risk of overwhelming postsplenectomy infection and other invasive bacterial infections,
vaccinations should be administered at least two weeks before elective splenectomy. Canadian
immunization guidelines recommend the 23-valent pneumococcal polysaccharide vaccine (Pneu-P-23), the
conjugate H. influenzae type B vaccine, and both the meningococcal C conjugate (Men-C-C) and
polysaccharide (Men-P-ACYW135) vaccines. More durable protection against serogroup C meningococcal
disease may be achieved by giving Men-C-C two weeks before vaccination with Men-P-ACYW135. Booster
vaccinations are recommended for adults five years after the initial vaccination with both Men-P-ACYW135
and Pneu-P-23, and continuing every five years thereafter with Men-P-ACYW135.5 Complete
recommendations are outlined in the 2006 Canadian Immunization Guide
What investigations and management are appropriate if fever develops postsplenectomy?
Fever in a patient who has undergone splenectomy is a medical emergency and requires prompt assessment
and management. For all patients presenting with fever after splenectomy, blood cultures should be done. A
lumbar puncture should be performed if meningitis is suspected. Expert opinion recommends that empiric
antibiotic therapy includes an intravenous third-generation cephalosporin (e.g., ceftriaxone or cefotaxime),
combined with vancomycin to ensure coverage of drug-resistant pneumococci. Despite a lack of clinical
evidence, some experts also recommend a “pill-in-pocket” approach with patient self-administration of
amoxicillin or amoxicillin–clavulanate at the onset of febrile illness and instructions to the patient to present
for urgent medical assessment. Lifelong antibiotic prophylaxis in adults is controversial; however, daily
penicillin has been suggested for patients with ongoing immune or hematologic disease.
What other advice can be offered to reduce the patient’s risk of postsplenectomy infection?
Patient awareness and education is paramount in preventing infections after splenectomy. Experts suggest
routine use of medical alert bracelets and laminated information cards by patients with asplenia. Patients
should also be educated on the risks of contracting fatal infections from animal bites (e.g., Capnocytophaga
canimorsus from dogs) or from exposure during travel (e.g., malaria, babesiosis), and about the need for
prompt medical assessment in these situations.
438. What should be given for prophylaxis
a) Cefazolin
b) Cephalexin
c) Ceftazidine
d) Ciprofloxacin
https://www.publichealth.hscni.net/sites/default/files/factsheet_for_health_professionals.pdf

439. Death of patient (Bronchospasm) due to Dimenhydrinate

440. Which one of the following shall be most appropriate to use for the rate control?
a) Beta Blockers
b) Digoxin
c) Amiodarone (rhythm control)
d) Propafenone

441. A long scenario on a manic patient, should be referred?


a) Psychiatrist
b) Social Worker
c) Pharmacy that works on the weekends and has delivery services.

442. A long scenario on a patient who has been prescribed acne medication and his father wants
to know about it, what needs to be taken into account.
a) Patient’s Autonomy
b) Patient’s Confidentiality
c) Veracity
443. A long question on a patient who volunteers for a clinical trial so which ethical aspect has
been followed for the patients.
a) Veracity
b) Autonomy.

444. Which on of the following is not a Primary Journal


a) Med effect
b) Pub med
c) Review Articles (2ry literature)
d) Merck Manual has no references

445. A long scenario on a patient who has half of a tablet with him and is from USA he travels
to Canada and asks a doctor about the medication identification, the doctor has a similar tablet
at his clinic, which one of the following resources can help them both identify the medicine.
a) E-cps (((for Canada only)))
b) Martindale
c) PDR
d) Rx Files

446. You are a Pharmacy manager, you observe that in the queue there is an old lady, behind
her there is someone who has put his hand on her arm and she is speaking loudly and
immediately puts her hand in the handbag, what you would do in the situation.
a) Ignore the situation
b) Call 911
c) Approach the lady and ask if everything is OK.
d) Ask Technician to look what is happening.

447. Academic detailing. What is NOT true


a) Updates doctors
b) Government Funded
c) Pharma Companies funds them

448. Patient wants to share his profile with other healthcare professionals what would you do as
a Pharmacist
a) Provide the patient a detailed note of whatever treatment he underwent to facilitate
him sharing it with other healthcare professionals.
b) Refuse the patient and ask him to request his doctor
449. If a Pharmacist wishes to improve the system records for the patients which one of the
following is the best step adopted by the Pharmacist for the best interest of the patient in the
community Pharmacy.
a) Incorporating the entire details of the patient medication and sequence of details
that the patient underwent during the course of treatment.

450. You want to prepare 50 gms of 2.5 % Cream and the creams that you have in hand are
Cream A 5% and Cream B 1%, what quantity of both Cream A and Cream b shall be required
to make 50 gms of cream with a Final potency of 2.5 %. Allegation Method.
a) 10 gms of Cream A and 40 gms of Cream B
b) 12 gms of Cream A and 18 gms of Cream B
c) 18.75 gms of Cream A and 31.25 gms of Cream B T
d) 15 gms of Cream A and 35 gms of Cream B

451. Dilution calculation, simple one answer 12 mgs

452. A long calculation on warfarin dose and regimen given like from Monday to Wednesday
the patient receives X mgs, then from Wednesday to Friday he receives X mgs, and then on
Saturday and Sunday he receives X mgs. Which one of the following should he be receiving if he
had missed a few doses?
I chose 2.5 mgs PRN. Not sure that was the one that sounded reasonable.

453. Calcium MEqs calculation not sure I got 23??? A fluke, not sure right or wrong.

454. Laminar Flow hood prevents or is suitable for which one of the following (DUMMY)
a) Powder bags of drugs
b) Liquid injectable vials
c) Dry Powder injections

455. Etanercept?
a) Should be refrigerated.
b) Injected IM  25 mg twice weekly or 50 mg once weekly SC

456. The new manager hired by the Pharmacy owner, which of the following the new Pharmacy
Owner can NOT do.
a) Anything relating to the financial resource, it’s the jurisdiction of the Pharmacy
owner only to play around with the financial resource.

457. Cost Effective Analysis? Pertains to Units.


458. Crohn treatment all stage

459. A Pharmacist was on leave and there has been an important issue that cropped up in his
absence and its week end, how would you prefer informing the Pharmacist
a) E mail him
b) Put a note on his table.
c) Write a personal letter to him in detail and put it in his personal Mailbox in Pharmacy.

460. A Pharmacist opens a new Pharmacy in a new area, which of the following shall be the
first aspect that he needs to be taking into account.
a) The flow of Customers in the area
b) The Demographic aspects of the area including the diversity of the population living
in the area.

461. Calc about % accuracy. giving false + ve, false -ve, true +ve & true -ve

462. New ofloxacin what study need to be done to test its efficacy in CAP?
a) Randomized placebo control
b) Cohort Study
c) Comparative study

463. Therapeutic decision made on unwell designated study on new drug, what ethic is violated?
a) Veracity
b) Non maleficence
464. Patient had HTN, Dyspepsia. What is not appropriate Question?
a) What`s your previous GERD drugs
b) What`s your DM drugs

465. Which one of these anti-thrombotic for a patient has atrial fibrillation, which one has direct
effect on thrombin?
a) Clopidogrel
b) Warfarin
c) Dabigatran
d) Rivaroxaban

466. Patient came to pharmacy with Rx for prednisone, she says her doctor prescribed it for
increasing egg ovulation hope getting a child, what first question you would ask her?
a) Did your Dr tell you about risk and benefits of using it during pregnancy?
b) Did your Dr tell you about risk of this medicine on fetus?
c) Did your Dr look for a safe alternative, as this medicine has no safe study on this
indication?

467. What should you monitor with Lasix use?


a) Liver function
b) Allergy
c) Renal function

468. Patient with peanut allergy, what she can’t take:


a) Progestrone
b) Prometrium/ Also isotretinoin
c) Topical estrogen
d) Levonorgestrel
PROMETRIUM® (micronized progesterone) is indicated for: women with an intact uterus as an adjunct to
postmenopausal estrogen replacement therapy to significantly reduce the risk of endometrial hyperplasia
and carcinoma.
CONTRAINDICATIONS
• hypersensitivity to this drug, soya, peanut or to any ingredient in the formulation of the capsule.
• Liver dysfunction or disease as long as liver function tests have failed to return to normal;
• personal history of known or suspected estrogen-dependent or progestin-dependent malignant
neoplasia (e.g. breast cancer or endometrial cancer)
• endometrial hyperplasia; undiagnosed abnormal genital bleeding;
• known or suspected pregnancy;
• classical migraine;
• active or past history of arterial thromboembolic disease (e.g. stroke, myocardial infarction, coronary
heart disease)
• active or past history of confirmed venous thromboembolism (such as deep venous thrombosis or
pulmonary embolism) or active thrombophlebitis;
• partial or complete loss of vision due to ophthalmic vascular disease.

469. RA Patient tried Hydroxychloroquine but there was no response, why?


a) Dose used may need to increase
b) Hydroxychloroquine should be used for 3-6 month to see a response
c) Effectiveness of monotherapy with Hydroxychloroquine is limited
d) Start loading dose for few days will improve the response
Used in combination with other DMARDs or as monotherapy for mild to moderate RA without poor
prognostic factors.

470. 65 y patient in hospital has infection taking ceftriaxone and clindamycin. After 2 days hey
discovered she has UTI & prescribe ciprofloxacin for 7 days although that she was
asymptomatic. What is DTP?
a) Taking medication without indication
b) Needs Additional Drug Therapy
c) Taking wrong medication
d) Too much or too little from the right medication
NOVEMBER 2014
1. Hypertensive patient with glaucoma, sulfa allergy and asthma, what is the drug of choice for
Glaucoma?
a) Prostaglandin analogue (Latanoprost) first line
b) Carbonic Anhydrase Inhibitors (CAIs) (Dorzolamide) caution with sulfa allergy
c) Beta blocker (Timolol) contraindicated with asthma
d) Alpha adrenergic (Brimonidine)
e) Anticholinergic (Pilocarpine)

2. Patient has sulfa allergy and glaucoma, and ASTHMA on a bunch of drugs, what is the main
interaction that would concern the pharmacist?
a) Timolol/Asthma (Bronchospasm)
3. Timolol eye drops counseling? Nov 2012
a) Discard once opened after 1 month
b) Once you open it you need to refrigerate
c) Shake before use

4. Which class of medication can’t use because of cross sensitivity with the sulfa allergy?
a) Prostaglandin analogue
b) Carbonic Anhydrase Inhibitors (CAIs) Dorzolamide
c) Beta blocker
d) Alpha adrenergic
e) Anticholinergic
5. What is the first line agent for open angle glaucoma? Prostaglandin analogue
a) Acetazolamide
b) Carbonic anhydrase inhibitors
c) Timolol
d) Pilocarpine
e) Apraclonidine

6. When you counsel patient what is true to increase absorption of the eye drops?
a) Blink the eye quickly for 1-2 min (no blinking or squeezing)
b) Leave the eyelids closed and the finger pressing gently for 2 min
c) Separate between each drop from 3-5 min (minimum 5 minutes if different drugs, and 1
min if the same drug)
d) Put cold pack for 2 min on closed eye

7. When you counsel patient what is true to decrease the systemic absorption of the eye drops?
a) Keep your head tilted down towards the floor
b) Separate1-2 min between each eye drop
c) Firmly pressing the eyelid for at least 1 minute
d) Put your hands on the lacrimal duct for 2 to 3 min following instillation

8. A child hit his mother in her eyes 2 days ago, now she suffering from burning and irritation
in her eyes, pain and she feel that there is a foreign body inside, what is your recommendation?
a) Give her analgesic for her pain
b) Tell her to use cool pack on her eyes.
c) Advise her with antihistamine eye drops.
d) Refer to physician for assessment. (red flag, trauma, foreign body- to role out injury)

9. Female 32 y, 73 Kg, height 5 ' 6, waist circumference <100, her work is 2 km away from
home, she is working hard to save money to go to Caribbean in her annual vacation after 2
month, she comes in to your pharmacy & wants to try Orlistat to lose weight quickly. Her
physical activity is minimal. What would you advise her?
a) She is good candidate to Orlistat
b) Advice to adjust her diet
c) Advise her to walk while she goes to work
d) Pay a registration to join a health club
Height= 5 feet, 6 inch = 1.67 m, BMI= 26.17 m2
10. While technician adding a Levetiracetam RX to patient data on the computer, he wrote it by
mistake Levofloxacin at computer system. How the manage the error in the future?
a) Put alarm on the computer
b) Put label on all look alike sound alike
c) Check the medication with the original prescription (always check the original
prescription, not the dispensing record)
d) Check the stock bottle with the bar code
All patients treated with antiepileptic drugs, irrespective of indication, should be monitored for signs of
suicidal ideation and behavior and appropriate treatment should be considered.
There have been reports of decreased levetiracetam concentration during pregnancy. This decrease is more
pronounced during the third trimester (up to 60% of baseline concentration before pregnancy). It is
recommended that clinical response should be monitored carefully in women receiving levetiracetam
treatment during pregnancy, and determination of changes in plasma concentrations should be considered
to ensure that adequate seizure control is maintained throughout pregnancy.
Levetiracetam is rapidly and almost completely absorbed after oral administration. The extent of absorption
of levetiracetam was not altered by food, but the rate of absorption was slightly reduced.
11. Patient with Hypothyroidism, he was
complaining of fatigue, constipation, oily skin and
acne, all are signs of uncontrolled hypothyroidism
EXCEPT:
a) Constipation
b) Fatigue
c) Oily skin
d) Weight gain
e) Cold intolerance

12. Metronidazole work on CYP 2C9 enzyme so it


interacts with? N/A
a) Gliclazide
b) Haloperidol
c) Furosemide
d) Propranolol

13. Cyclosporine interact with


a) Phenytoin
b) Ampicillin
c) Donepezil
d) Alendronate
Drugs that decrease ciclosporin levels
All inducers of CYP3A4 and/or P-glycoprotein are expected to decrease ciclosporin levels. Examples of
drugs that decrease ciclosporin levels are:
Barbiturates, carbamazepine, oxcarbazepine, phenytoin; nafcillin, intravenous sulfadimidine, probucol,
orlistat, hypericum perforatum (St. John's wort), ticlopidine, sulfinpyrazone, terbinafine, bosentan.
Products containing Hypericum perforatum (St John´s Wort) must not be used concomitantly with Neoral
due to the risk of decreased blood levels of ciclosporin and thereby reduced effect.
Rifampicin induces ciclosporin intestinal and liver metabolism. Ciclosporin doses may need to be increased 3-
to 5-fold during co-administration.
Octreotide decreases oral absorption of ciclosporin and a 50% increase in the ciclosporin dose or a switch to
intravenous administration could be necessary.

14. Which of these is Contraindicated with Clarithromycin?


a) Duloxetine
b) Montelukast
c) Itraconazole  increase QT prolongation
d) Hydrochlorothiazide
Itraconazole appears to be capable of significantly increasing clarithromycin serum concentrations. Limited
available information suggests a possible bidirectional interaction between the drugs, presumably through
their effects on CYP3A4 activity

15. Patient was taking oxycodone and the doctor want to change it to Fentanyl patch what is the
right dose:
a) Fentanyl BID
b) Fentanyl 25 mcg/12hr
c) Fentanyl 25 mcg/24hr
d) Fentanyl 25 mcg/48hr
e) Fentanyl 25 mcg/72hr

16. Which one of the below can’t be used with Naïve opioid patient?
a) Fentanyl Patch
b) Fentanyl injection
c) Hydromorphone
d) Morphine
e) Codeine

17. What NOT to tell about fentanyl patch?


a) Do not apply the new patch to the same place as the last one (true)
b) Apply first aid tape only to the edges of the patch if it falls off right away after
applying https://www.youtube.com/watch?v=x8sOrXljT9I
c) Do not use the patch if the pouch seal is broken (true)
d) Press the patch with the palm of your hand and hold there for at least 30 seconds (true)
Special precautions for storage
Store in the original pouch, in order to protect from light.
This medicinal product does not require any special temperature storage conditions.
Nature and contents of container
Each patch is packed in a heat-sealed pouch. The pouch material is a lamination of polyethylene
terephthalate (PET), low density polyethylene (LDPE), aluminium foil, adhesive and acrylonitrile film.
Durogesic DTrans is supplied in cartons containing 3, 4, 5, 8, 10, 16, 20 or 30 individually packed patches.
Not all pack sizes may be marketed.
Instructions for disposal:
Used patches should be folded so that the adhesive side of the patch adheres to itself and then they should
be safely discarded. Any unused medicinal product or waste material should be disposed of in accordance
with local requirements.
18. There is a significant risk associated with drug diversion in hospitals specially with
a) Meperidine
b) Codeine
c) Fentanyl
d) Tramadol

19. Patient was prescribed Omeprazole, which is sparingly soluble in alcohol, what is the best
method to prepare the medication:
a) Solution
b) Suspension
c) Dry powder
d) Elixir
In misbah
A patient is using ranitidine and this drug is sparingly soluble in alcohol. Which of the
following dosage form is good to prepare?
a) Elixir
b) Oral solution
c) Suspension
d) Cream
Ans: B Tips: CPS Monograph. Zantac ® Oral solution (containing 7.5% w/v alcohol)

20. What is true when patient on Omeprazole (PPI) & taking calcium, iron, other medications?
a) Omeprazole increase the absorption of Iron.
b) Omeprazole decrease the absorption of iron.
c) Omeprazole has no effect on absorption of iron
d) Omeprazole is contraindicated with iron
By reducing stomach acid, omeprazole may reduce the absorption of iron and make ferrous sulfate less
effective in treating your condition. Also, decreases absorption of Vit B12- Remember: Vit C increases
absorption of iron

21. Patient with Hypertension and mild heartburn (GERD) symptoms after eating and BMI
26.1, what is your recommendation?
a) Give him Famotidine
b) Advice with Sodium Alginate
c) Recommend Magnesium Hydroxide antacids
d) Refer him to doctor to prescribe PPI
22. What is your advice to the same patient regarding non-pharmacological tips for (GERD)
a) Weight loss (not obese)
b) Do regular exercise
c) Small snacks before sleeping
d) Lie down after eating
There is little evidence to support the effectiveness of lifestyle changes aside from weight loss and elevating
the head of the bed; however, lifestyle and dietary measures provide broad health-care benefits and carry
no risk.
- Modify diet (avoid chocolate; caffeine; acidic citrus juices; large, fatty meals)
- Reduce body weight if BMI >30 kg/m2 or recent weight gain
- Avoid eating up to 3 hours before bedtime
- Avoid lying down after meals
- Elevate the head of the bed by 10–20 cm, particularly if nocturnal or laryngeal reflux symptoms are
present; this is best achieved by using a block under the legs to elevate the bed frame rather than
with pillows or a wedge
- Stop smoking, avoid alcohol & Avoid tight clothing

23. When to refer the same GERD patient all EXCEPT


a) If he feels burning sensation more than 3 days/week
b) If patient has difficulty in swallowing
c) When patient wakeup at night with heartburn
d) When Nonpharmacological treatment fail

GERD symptoms range from mild to severe. Severity of symptoms and esophageal mucosal injury correlate
with the total time the esophageal mucosa is in direct contact with acid (at pH <4) per 24-hour period.
Mild symptoms do not interfere with daily activity and are usually of low intensity, short duration, not
nocturnal, infrequent (<3 times weekly) and without major complications.
Severe symptoms regularly interfere with daily activities and are usually of high intensity, persistent (>6
months), nocturnal, frequent and often associated with complications.
Dysphagia, defined as difficulty in swallowing (experienced anywhere from the mouth to the stomach), is an
ominous symptom that necessitates endoscopic evaluation. In contrast, the more common GERD symptom
of globus (a continuous feeling of a lump in the throat that does not interfere with swallowing) is a benign
occurrence in acid reflux, does not require investigation and usually responds to effective acid reduction.

24. Obese Pt with GERD and his lifestyle includes 1-2 alcohol every day. What is your
recommendation
a) Minimize alcohol intake
b) Reduce body weight
c) Avoid alcohol
d) Do upper endoscopy

25. In which case we can recommend to step down from IV to oral after 24 hrs?
a) Meningitis
b) Endocarditis
c) Osteomyelitis
d) Pyelonephritis

26. Patient with severe pyelonephritis, with chills, fever, flank pain, what will be the DOC
a) Norfloxacin
b) Levofloxacin
c) Clotimoxazole
d) Gentamicin (inpatient pyelonephritis)
27. 80 years old lady having insomnia with sleeping problems, her life style include: having a
walk after her early dinner at 5 pm and then return home and relax by listening to music, doing
some aerobic and then watch TV in bed, she drinks decaffeinated tea and does not have alcohol
except a cherry drink: What is the reason for her Insomnia?
a) Leisure activity
b) Evening walk
c) Watching TV at bed time
d) Coffee intake
e) Alcohol Consumption
Stimulus-Control Therapy
 Stimulus-control therapy is based on the concept that sleep is a conditioned response to temporal
(bedtime) and environmental cues. The focus of stimulus-control therapy is to eliminate maladaptive
behaviours, with the overall goal of associating the bedroom with sleep. These measures
complement and are often included in, general sleep hygiene recommendations.
Recommendations include:
- Go to bed only when tired. Avoid napping during the day.
- Use the bedroom only for sleep and intimacy.
- Get up at the same time in the morning regardless of sleep duration.
- If unable to sleep after 15–20 minutes, get out of bed and go to another room to read in a dimly lit
environment. Avoid watching television or using a computer screen as the light emitted from such
devices may have an arousing effect. Return to bed when feeling sleepy.

28. What do you advise to improve her sleep hygiene?


a) Get off the bed if she couldn’t sleep after 20 min
b) Change all bed sheets regularly
c) Schedule a late evening walk
d) Sleep every day at a different time
General sleep hygiene education focuses on behavioural and environmental factors that precede sleep and
that may interfere with sleep. Stimulus control measures (above) are often included in sleep hygiene
recommendations.
Personal habits:
 Fix a bedtime and an awakening time
 Avoid caffeine 4–6 hours before bedtime and minimize total daily intake
 Avoid nicotine near bedtime and upon awakening at night
 Avoid alcohol 4–6 hours before bedtime
 Avoid heavy, spicy or sugary foods 4–6 hours before bedtime
 Exercise regularly, but not within 2 hours of bedtime
Sleeping environment:
 Use comfortable bedding
 Find a comfortable temperature setting for sleeping and keep the room well ventilated
 Block out all distracting noise
Getting ready for bed:
 Try a light snack before bed such as warm milk and foods high in the amino acid tryptophan (e.g.,
bananas)
 Try relaxation techniques before bed. Don't take your worries to bed
 Establish a pre-sleep ritual such as a warm bath or a few minutes of reading
 Get into your favourite sleeping position

29. What would be the best pharmacotherapy recommendation?


a) Lorazepam
b) Zopiclone
c) Citalopram
d) Diphenhydramine
Q. Although not a benzodiazepine, the cyclopyrrolone zopiclone acts at the benzodiazepine receptor
Zopiclone and has similar therapeutic and adverse effects
Adults: Initial dose: 3.75 mg HS PO. If needed, titrate to 5 –7.5 mg HS PO. Maximum 7.5 mg/day.
Geriatrics: 3.75 mg HS PO. Increase to 5 mg once daily if needed and tolerated.
Maximum 5 mg/day in elderly, patients with hepatic, renal impairment, or taking a strong CYP3A4
inhibitor.
The lowest dose for shortest duration is recommended. Treatment should rarely exceed 7–10 days.
Re-evaluate the patient if treatment beyond 2–3 weeks is required.
Due to rapid onset, administer just before retiring at night when there is sufficient time for a full
night's sleep. Use lowest effective dose. Use only when full night’s sleep is possible.
Advise patients of risk of next-day impairment, even if feeling fully awake.
Allow at least 12 hours between bedtime dose and any activity requiring mental alertness, e.g.,
driving. Advise patients to wait ≥12 h before driving or operating machinery.
S.E: Impaired cognitive function and, rarely, anterograde amnesia and transient global amnesia.
Bitter/metallic taste. May cause dosedependent, next-day impairment of activities requiring
alertness, including driving a car, despite the patient feeling fully awake.
Minimal additive effects with low doses of alcohol.
CYP3A4 inhibitors (e.g., clarithromycin, itraconazole, ritonavir) may increase its plasma levels.
CYP3A4 inducers (carbamazepine, phenytoin, St. John’s wort) decrease zopiclone plasma levels.
Monitor patient for increased or reduced response and adjust dose accordingly.
Does not accumulate; tolerance may be delayed and rebound insomnia may be reduced.

30. Which class could be used to control the anticipatory nausea & vomiting in cancer patient?
a) Benzodiazepines
b) Anticholinergic
c) Serotonin antagonist
d) Corticosteroids
Benzodiazepines provide useful antianxiety, amnesic and sedating effects. Lorazepam and alprazolam are
the most commonly used, and have been studied in cases of anticipatory nausea. They are usually used in
combination with other antiemetics.
Cannabinoids: Nabilone is of limited use because it is available only as an oral formulation and is associated
with several side effects including dry mouth, dizziness, drowsiness, mood alterations, hallucinations,
delusions, tachycardia and hypotension. Higher doses are associated with increased risk of side effects. It is
generally used in refractory nausea and vomiting or in combination with other antiemetics.

31. Cancer patient taking Granisetron and still have nausea and vomiting, what is your
recommendation to break through the nausea and vomiting
a) Use Ondansetron because have better effect than Granisetron
b) Use Dimenhydrnate for the nausea and vomiting
c) Give Prochlorperazine PRN for breakthrough
d) Use another dose of Granisetron

32. A patient diagnosed with breast cancer, and her lab test shows Hypercalcemia (lab test was
3.4 and normal levels were given) what is the DOC for Hypercalcemia?
a) Pamidronate / Clodronate/ Zoledronic acid
b) Insulin
c) Corticosteroids
d) Na polystyrene
33. What is the lab test she has to do for Hypercalcemia investigation?
a) Albumin
b) Serum creatinine
c) CK
d) Amylase
Laboratory evaluation:
 serum ionized calcium (preferred where available) or serum calcium and albumin; corrected Ca++
value (mmol/L) = (0.02 × [40 − measured albumin g/L]) + measured Ca++
 serum intact parathyroid hormone (PTH); perform this test before giving bisphosphonates, as they
can alter serum PTH levels
 serum parathyroid hormone-related peptide (PTHrP) and 1,25-dihydroxyvitamin D level if PTH low
 24-hour urine collection to measure calcium-to-creatinine ratio (if familial hypocalciuric
hypercalcemia or milk-alkali syndrome is suspected)
 serum phosphate, alkaline phosphatase, total protein, serum creatinine and urea

34. A patient with Tonic-Clonic seizure, physician prescribed Phenytoin 300 mg PO per day for
1 month, what is the most INAPPROPRIATE action the pharmacist can do:
a) Talk to the patient about phenytoin side effects
b) Call the physician to change the dose to 100 mg TID
c) Ask the patient if the doctor is willing to measure the serum phenytoin
d) Let patient know about phenytoin drug interaction
e) Caution the patient about alcohol use with phenytoin
35. Why Carbamazepine is preferred over Phenytoin, because carbamazepine
a) Has no active metabolites
b) Follows saturated metabolism (not sure)
c) Has better cosmetic side effects
d) Less GI side effects
e) Less CNS side effects
Answer: Cosmetically does NOT result in gum overgrowth

36. Dose independent side effect of carbamazepine


a) Rash (SJS due to 15:02 gene)
b) Angioedema
c) Loss of muscle coordination
d) Feet or lower legs (oedema)

37. Which drug may cause lymphoma in children less than 6 years
a) Methotrexate
b) Infliximab (risk of infections and malignancy)
c) Sulfasalazine
d) Corticosteroid
Malignancies, some fatal, have been reported among children, adolescents and young adults (up to 22 years
of age) treated with TNF-blocking agents (initiation of therapy ≤ 18 years of age), including Remicade in the
post-marketing setting. Approximately half the cases were lymphomas. The other cases represented a
variety of different malignancies & included rare malignancies usually associated with immunosuppression.
A risk for the development of malignancies in patients treated with TNF-blockers cannot be excluded.

38. A patient administered to the hospital with diarrhea, he has a history of Crohn's disease, for
10 years, the goal of treatment includes all the following except:
a) Avoid surgery*
b) Relieve symptoms
c) Induce remission
d) Prevent recurrence
e) Avoid drug side effects

39. Patient with crohn disease currently on corticosteroid, what is the DOC if he has refractory
Crohn’s Fistula
a) Cyclosporine
b) Budesonide
c) Infliximab
d) Sulfasalazine
Infliximab and adalimumab are effective in inducing and maintaining remission in patients with moderate
to severe CD and in inducing closure of perianal fistulae. Combining infliximab with azathioprine is more
effective than infliximab monotherapy, suggesting that initial dual therapy may be preferable for high-risk
patients, particularly those who are naïve to prior treatment with biologics and immunomodulators. It is
unclear whether these findings can be extrapolated to the other biologic agents or to those with prior
exposure to immunomodulator therapies.
Vedolizumab is effective at inducing and maintaining remission in patients with moderate to severe CD,
including patients who failed prior TNF-alpha antagonist therapies. The efficacy of vedolizumab for
treatment of perianal fistulae or extraintestinal manifestations of CD is uncertain. The benefit of
combination therapy of vedolizumab with immunomodulators also remains unclear. The time-to
effectiveness for vedolizumab is variable, with the most rapid onset as early as 2 weeks in TNF-alpha
antagonist-naïve patients and as late as 10 weeks in TNF-alpha antagonist-experienced patients.
Monograph: Fistulising Crohn's disease
Patients with fistulising Crohn's disease with acute suppurative fistulas must not initiate Remicade therapy
until a source for possible infection, specifically abscess, has been excluded

40. What can be used for maintenance to maintain remission in patients with
Crohn’s disease
a) Sulfasalazine
b) Prednisone
c) Azathioprine
d) Budesonide
41. What are the correct tall man letters that help draw attention to the dissimilarities in look-
alike drug below?
a) PREDNIsone & PREDNIsolone
b) predniSONE & prednisoLONE
c) predniSOne & predniSOLOne

42. To check whether compounded product is already available in market. Q Tallman letter
which one is correct
a) DOPAmine and DOBUTamine
b) DOPamine and DOBUTAmine
c) DOPamine and DOBUtamine
d) DOPamine and DOBUTamine

43. Where to search for Tall Man lettering


a) CIHI
b) ISMP
c) CADTH
d) Health Canada
44. What drug may cause QT prolongation
a) Phenelzine
b) Fluoxetine
c) Sotalol
d) Heloperidol
Sotalol can cause serious ventricular arrhythmias, primarily Torsade de Pointes (TdP) type ventricular
tachycardia, a polymorphic ventricular tachycardia associated with QT interval prolongation. QT interval
prolongation is directly related to the plasma level of sotalol.

45. 45 years old teacher with parkinsonism, she was embarrassed as she falls many times and
now, she is working part time instead of full time: What is your advice for this patient
a) Start treatment to avoid delaying of disease progression
b) Do not start treatment because of the side effects of the medication
e) Start the treatment as quality of life is affected
c) Change her work

46. What is the DOC for this patient?


a) Pramipexole
b) Selegilin
c) Levodopa
d) Domperidone
(Avoid levodopa in younger patients due to development of motor complications)

47. What may worsen Parkinsonism case?


a) Domperidone (used in nausea +
Parkinson)
b) Risperidone
c) Metoclopramide (pass BBB)
d) Erythromycin
48. Patient newly diagnosed with Alzheimer’s and start Donepezil 10mg, what is true
a) If any of the medication fail, the rest will fail
b) Ginko biloba may enhance the effect of donepezil
c) We consider the improvement upon decrease in the repetitive question (gold
standard)
d) Start with high dose and titrate
Reduction in repetitive questioning is a common treatment goal in patients with mild-moderate Alzheimer
disease and usually corresponds to a generally positive treatment effect. Monitor treatment effects 2 weeks
after initiating therapy or increasing dosage and then periodically thereafter based on feasibility.

49. After one month, the patient developed severe nausea, vomiting and diarrhea. What is your
appropriate suggestion?
a) Reduce the dose of donepezil
b) Stop the medication temporarily and start a later
c) Change to another class
d) Change to another cholinesterase inhibitor.
Q. Selective and have greater affinity for AchEi in brain than periphery.
Donepezil Reduces the hydrolysis of acetylcholine, increasing the amount available in the synaptic cleft.
Piperidine- Donepezil was effective in 3- to 6-month trials in patients with mild to moderate Alzheimer
based disease (MMSE score of 10 – 26). It was also effective in moderate to severe Alzheimer disease
Centrally (MMSE 0 – 17), and is the only cholinesterase inhibitor approved for all disease severities
active Initial daily dose 5mg, taken at night. Can be taken in the morning if sleep disturbances occur
reversible, non After 4 weeks, try increasing to the target dose of 10 mg/day. Adjust dose after 4 wk
competitive. S.E: Cholinesterase inhibitors: theoretically, these agents may lower seizure threshold, increase
the risk of GI ulceration or bleeding, or exacerbate COPD or asthma.
Donepezil: >10%: headache, nausea, diarrhea. <10%: vomiting, anorexia, fatigue, sleep
disturbance, syncope, muscle cramps, urinary frequency. Bradycardia (uncommon), heart block
(uncommon), rhabdomyolysis (uncommon), neuroleptic malignant syndrome (uncommon).
Cholinesterase inhibitors: theoretical concern regarding antagonistic effect of combined therapy
with cholinesterase inhibitors and drugs with anticholinergic activity.
Additive bradycardia when combined with BBs or CCBs; few reports of actual interactions.
Toxicity may be increased by inhibitors of CYP2D6 or CYP3A4 such as paroxetine,
erythromycin, prednisone, grapefruit juice. Effectiveness may be reduced by inducers of
CYP2D6 or CYP3A4 such as carbamazepine, phenytoin, rifampin.
Patients should be carefully monitored for muscle pain, tenderness or weakness and darkened
urine, particularly if accompanied by malaise or fever. Blood creatine phosphokinase (CPK)
levels should be assessed in patients experiencing these symptoms.
Close monitoring for adverse effects in patients with hepatic disease, reanal diseases & geriatrics
being treated with donepezil hydrochloride is therefore recommended.

50. What is important to tell the patient about Donepezil? 3-6 months to kick in
a) It will show effect after 3- 6 months at least
51. Which is a true about Alzheimer treatment
a) Completely eliminate the symptoms
b) All symptoms will stop worsen
c) It will prevent the disease from progressing
d) Don’t give drugs till symptoms interfere with daily activity & more bothersome
(Parkinson's)
It is important to define target symptoms prior to treatment initiation. Effectiveness is often considered to
be either improvement or no deterioration of target symptoms. In untreated patients with mild to moderate
dementia, a decline of 2–4 points per year on the MMSE scale is expected; therefore, an annual decline of
less than 2 points while on drug therapy would typically indicate a beneficial effect. However, despite its
widespread use in clinical research, the MMSE is now recognized as a poor measure of
treatment response in individual patients.
Goals of Therapy
- Alter the natural disease progression to meet patient’s and caregiver’s goals
- Treat cognitive, behavioural and psychological symptoms
- Alleviate caregiver burden
- Minimize medication side effects

52. Patient was on warfarin for 5 years and now the doctor prescribed Rosuvastatin. what to do
with the warfarin dose:
a) Short term increase in warfarin dose
b) Short term decrease in warfarin dose
c) No change is expected
d) Long term increase in warfarin dose
e) Long term decrease in warfarin dose
DDI: Warfarin + Fluvastatin or simvastatin

53. Which of the following is correct when taking warfarin & ciprofloxacin together
a) Decrease Cipro bioavailability by 50%
b) Enhance the effect of warfarin & appropriate laboratory tests should be routinely
monitored
c) No changes in international normalized ratio (INR)
d) Warfarin side effects that require immediate medical attention

54. Female Pt. takes warfarin at 9 am, 2 pm, 7pm & 9pm. She forgot the morning dose and call
you at 2 pm. What to tell her?
a) Skip the missed dose & manage remaining regimen at the usual time
b) Refer for dose reschedule
c) Take double dose right now
d) The dose should be taken as soon as possible and delay the next dose
Warfarin
If you're taking warfarin and you miss one of your doses, you should skip the dose you missed and wait to
take your next scheduled dose as normal. Don't take a double dose to make up for the one you missed.
If you accidentally take a dose that was much higher than recommended, contact your anticoagulant clinic
or GP for advice.
Newer anticoagulants
If you're taking apixaban or dabigatran twice a day and you miss one of your doses, you should take it as
soon as you remember if it's still more than 6 hours until your next scheduled dose. If it's less than 6 hours
until your next dose, skip the dose you missed and take the next scheduled dose as normal.
If you accidentally take a double dose, skip your next scheduled dose and take the following dose the next
day as scheduled.
If you're taking rivaroxaban once a day and you miss one of your doses, you should take it as soon as you
remember if it's still more than 12 hours until your next scheduled dose. If it's less than 12 hours until your
next dose, skip the dose you missed and take the next scheduled dose as normal.
If you accidentally take a double dose, take your next dose the next day as scheduled.

55. What is true about the Gabapentin:


a) Gabapentin will cause Heart disease
b) Hepatic disease may happen with Gabapentin
c) Gabapentin need dosage adjustment in renal disease.

56. Gabapentin, which of the following is true or What to monitor with Gabapentin?
a) CrCl (Renal) need renal dose adjustment
b) Liver function as it causes hepatic dysfunction
c) CBC
d) It is a Sodium channel blocker
e) t 1/2 is long enough to allow once daily dosing

57. Patient with depression and agitation on citalopram and the doctor prescribed Lorazepam
1mg TID, what is true:
a) Patient stop citalopram and take lorazepam
b) Patient should take lorazepam for 3 weeks with citalopram till the citalopram gives effect.
c) You can’t give lorazepam and citalopram together.
d) You have to taper for lorazepam before stopping.

58. 15 years old has ADHD that was previously controlled on methylphenidate stopped taking
his medications, his mother asks you to speak to him regarding his medication. During the
interview, he told you that he cannot sleep and gets embarrassed by taking pills at school and is
too tired to focus. What is your appropriate response?
a) Recommend that he exercise at night to help him sleep better
b) Offer to help him to deal with his symptoms
c) Suggest that he switch to a long acting formulation
d) Tell him he must take his medication
Long-Acting Stimulants
The CADDRA 2018 guidelines recommend long-acting stimulants as first-line therapy for ADHD. Long acting
formulations of mixed salts amphetamine (Adderall XR), methylphenidate (Biphentin and Concerta) and
lisdexamfetamine (Vyvanse) have a duration of action of 8–14 hours and are as effective as appropriately
dosed shorter-acting stimulants.
Advantages of these long-acting products include single daily dosing, potential for improved adherence,
avoidance of the need for medication administration at school, decreased abuse potential and decreased
risk of rebound hyperactivity.
Patients with ADHD are at higher risk of impulsive behaviour and substance use. Treating ADHD with
stimulants can decrease the risk of substance abuse.
Answer analysis:
c) Although LA-stimulant will help avoid social stigma, it is expected to worsen sleep.
a) Excercise at night is expected to cause insomnia.
b) Nothing is given to explain how the pharmacist will help the patient with his symptoms.
d) "must take" is clearly against autonomy.

59. The mother is afraid that her child may be addicted with the use of ADHD medications what
you will tell her?
a) ADHD medication will decrease the chance of abuse when he became older
b) ADHD medication is not addictive
c) Decrease the dose in the first 6 month
d) Stop medication and go to specialist

60. All are true about Methylphenidate EXCEPT


a) Should be given cautiously to patients with a history of drug dependence or alcoholism
b) Should not be used in children under six years
c) Improve Athlete performance (amphetamine misuse in sports)
d) Improve Concentration & academic performance
e) Improve Impulsive behaviour

61. What is your role as a pharmacist if you see that this patient needs treatment change
a) Fax the doctor to get him to authorize changing patient medication.
b) Tell patient to go to the ER to get a prescription for the new medication
c) Suggest that mother ask for appointment with his doctor & discuss pharmacist
recommendations
d) Call the doctor and tell him which med to switch the patient to.
62. NS, is 16 yrs old girl suffering from acne, taking Ovral 21 (active ingredient given) &
Ibuprofen. She has not been complaining of acne for the last year. Now her acne is pustular &
inflamed. What are the risk factors that aggravates her acne?
a) Age
b) Gender
c) Ovral (OC)
d) Ibuprofen
Each LO/OVRAL tablet, contains 0.3 mg of norgestrel (dl-13-beta-ethyl-17-alpha-ethinyl-17- beta-
hydroxygon-4-en-3-one), a totally synthetic progestogen, and 0.03 mg of ethinyl estradiol, (19-nor-17α-
pregna-1,3,5 (10)-trien-20-yne-3,17-diol).
63. Person with acne and is experiencing severely inflamed face and is taking multivitamins and
other medications was on benzoyl peroxide but wasn't working for him, so doctor changed him
to erythromycin and retinoid, what to worry about?
a) Drug-drug interaction
b) Inappropriate medication (should use isotretinoin 1st line– because Severe inflamed)
c) Too high dose
The tetracycline antibiotics (doxycycline, minocycline and tetracycline) are effective first-line options when
systemic antibiotics are indicated. Because minocycline does not appear to be more effective than other
systemic therapies, and is associated with an increased risk of drug-induced lupus or hepatitis, some experts
recommend the use of doxycycline or tetracycline over minocycline.
Due to concerns about development of bacterial resistance, use of other effective antibiotics (such as
macrolides and sulfamethoxazole/ trimethoprim) should be reserved for those patients in whom
tetracyclines are contraindicated, not tolerated or not effective.
Evidence regarding duration of use of antibiotics is lacking; recommendations are based on expert opinion.
Allow at least 6 weeks to see if an antibiotic is effective. Bacterial resistance can be reduced by using
oral antibiotics in combination with benzoyl peroxide. When possible, limit duration of oral antibiotic
therapy to 3 months while maintaining topical therapy. If acne relapses despite appropriate topical therapy,
consider alternative systemic therapy. The major concern regarding long-term oral antibiotic therapy
remains bacterial resistance; guidelines recommend that systemic antibiotics should not be used as
monotherapy.
64. Side effects of isotretinoin?
a) Conjunctivitis
b) Ankle edema

65. What to monitor with Isotretinoin?


a) Pulmonary Fibrosis
b) Psychiatric disorder
c) Test for Iron level
Systemic Drug Therapy for Acne. Q. Retinoids Isotretinoin Accutane, Clarus, Epuris
It remains the most powerful anti-acne agent, with the majority of patients achieving clearing and sustained
remission, even in the most severe cases.
It is recommended as first-choice therapy for severe papulopustular or moderate nodular acne and for
nodular or conglabate acne for many reasons: clinical effectiveness, prevention of scarring and quick
improvement of a patient’s quality of life, including minimizing depression.
Avoid taking vitamin A as Oral isotretinoin is a natural metabolite of vitamin A.
0.5 mg/kg/day PO for the first month, increasing to 1 mg/kg/day as tolerated (with a goal cumulative dose of 120
– 150 mg/kg). Alternative lowdose regimen (particularly for treatment of resistant or quick-relapsing moderate
acne): 0.25–0.4 mg/kg/day.
A complete course of therapy consists of 12-16 weeks of Isotretinoin administration.
S.E: Teratogenicity. Common: mucocutaneous dryness, myalgia, arthralgia, photosensitivity, Headache. Rare:
hypertriglyceridemia, mood disorder, possibly suicide ideation, pseudotumor cerebri, erythema multiforme,
Stevens-Johnson syndrome, toxic epidermal necrolysis.
Blood potassium increased, blood alkaline phosphatase increased, blood bilirubin increased, blood urea
increased, elevated platelet counts, eosinophil count increased, false positive tuberculosis test, gamma-
glutamyltransferase abnormal, blood cholesterol increased, glucose urine present, haematocrit decreased, protein
urine, thrombocytopenia, WBC count decreased. Elevations in levels of serum creatine kinase (monitor)
Patients should be advised to use a skin-moisturizing ointment or cream and a lip balm from the start of treatment
as isotretinoin is likely to cause dryness of the skin and lips. When necessary a sun-protection product with high
protection factor of least SPF 15 should be used.
It is recommended that blood donation for transfusion purposes be deferred during therapy and for one month
after discontinuation of treatment.
Patients who experience tinnitus or hearing impairment should discontinue treatment and be referred for
specialized care for further evaluation.
Isotretinoin has been associated with inflammatory bowel disease (including regional ileitis, colitis and
hemorrhage) in patients without a prior history of intestinal disorders. Patients experiencing abdominal pain,
rectal bleeding or severe diarrhea should discontinue EPURIS immediately.
Tetracyclines: rare cases of benign intracranial hypertension (pseudotumor cerebri): allow 7 days washout after
stopping tetracyclines before starting isotretinoin.
The patient should have two negative pregnancy tests (β-hCG in urine or serum) before starting therapy with the
first pregnancy test conducted at initial assessment when the patient is qualified for therapy by the physician.
The patient then should have a second pregnancy test with a sensitivity of at least 25 mIU/mL with a negative
result, performed in a licensed laboratory, within 11 days prior to initiating therapy. The patient has had two or
three days of the next normal menstrual period before therapy is initiated.
Pregnancy test must be repeated monthly for pregnancy detection during treatment and at one month after
discontinuation of treatment. The dates and results of the pregnancy tests should be documented.
Treatment should be discontinued if the patient develops any of the following reactions: rash, especially if
associated with fever and/or malaise, conjunctivitis (red or inflamed eyes); blisters on legs, arms or face and/or
sores in mouth, throat, nose or eyes; peeling skin or other serious skin reactions.
The following tests are required before starting Isotretinoin, at first month, then as clinically indicated:
 Serum blood lipid determinations (under fasting conditions) should be performed before Isotretinoin is
given and then at intervals (one month after the start of therapy) until the lipid response to Isotretinoin is
established (which usually occurs within four weeks), and also at the end of treatment.
 Complete blood count and differential: for early detection of leukopenia, neutropenia,
thrombocytopenia and anemia.
 Liver function tests: Increases in about 15% of ALT, AST, ALP baseline levels have been reported.
Liver function tests should be monitored before treatment and at regular intervals during treatment (one
month after the start of treatment and at least three-month intervals thereafter) unless more frequent
monitoring clinically indicated.
 Blood glucose levels: all patients and in particular patients with known or suspected diabetes should have
periodic blood sugar determinations.
 Serious Adverse Event Warnings include psychiatric disorders (depression, psychosis and, rarely,
suicidal ideation, suicide attempts, suicide, and aggressive and/or violent behaviors). Monitor psychiatric
illness before dispensing isotretenoin.

66. For acne case, after how long should you switch to other therapy
a) Over 2-4 weeks
b) 2-3 months
c) 4-6 moths
d) 1-2 weeks
Consider after ineffective topical trial of 2-3 months OR moderate-severe acne
Parameter Timeframe/Degree of change Actions
Short-term Effectiveness Endpoints (Acne resolution/control)
Lesion count Decrease by 10–25% within 4–8 wk, with control, or If endpoints not
more than a 50% decrease within 2–4 months. achieved, consider
Comedones Resolve by 3–4 months. further therapy.
Inflammatory lesions Resolve within a few wk.
Anxiety or depression Achieve control or improvement within 2–4 months.
Long-term Effectiveness Endpoints
Progression of severity No progression of severity If endpoints not
Recurrent episodes Lengthening of acne-free periods throughout therapy. achieved, consider
Scarring or No further scarring or pigmentation throughout further therapy.
pigmentation therapy.
Safety Endpoints (treatment side effects)
For each nondrug or drug measure initiated, list the side effect (safety endpoint) most likely to occur, the
degree to which it might be tolerated, if at all, and within what timeframe it might be expected.
Indicate how the side effect would impact therapy, i.e., continue and monitor, continue and treat side
effect, continue but decrease dose, or discontinue therapy and choose alternative.

67. What increases INR with warfarin?


a) High salt diet
b) Grape Fruit products
c) Smoking
d) Binge alcohol consumption = consumption of excessive alcohol amount in short period of time.
Acute ingestion of a large amount of alcohol may inhibit the metabolism of warfarin and increase INR.
Conversely, chronic heavy alcohol intake may induce the metabolism of warfarin. Moderate alcohol intake
can be permitted.
It takes most people 1 to 2 hours to process one standard drink. That means after 3 standard drinks
you should wait at least 3 hours before driving. After you stop drinking, the alcohol levels in your system can
continue to rise for up to 3 hours. Nothing can speed up the absorption of alcohol by your body.
Interactions with food and food supplements
Individual case reports suggest a possible interaction between warfarin and cranberry juice, in most cases
leading to an increase in INR or bleeding event. Patients should be advised to avoid cranberry products.
Increased supervision and INR monitoring should be considered for any patient taking warfarin and regular
cranberry juice.
Limited evidence suggests that grapefruit juice may cause a modest rise in INR in patients taking warfarin.
Certain foods such as liver, broccoli, Brussels sprouts and green leafy vegetables contain large amounts of
vitamin K. Sudden changes in diet can potentially affect control of anticoagulation. Patients should be
informed of the need to seek medical advice before undertaking any major changes in diet.
Many other food supplements have a theoretical effect on warfarin; however, most of these interactions are
not proven. Patients should generally avoid taking any food supplements whilst taking warfarin and should
be told to advise their doctor if they are taking any, as more frequent monitoring is advisable.

68. What is true regarding Allopurinol when treating gout


a) Titrate Allopurinol till reach desired serum uric acid
b) Titrate Allopurinol till serum uric acid is decreased by 50%
c) Raise Allopurinol till reach the maximum dose
d) Start with high dose and decrease dose after 2 weeks
Merck Manual: The preferred first line ULT (urate lowering therapy) is allopurinol, including for those with
moderate-to-severe chronic kidney disease. A low starting dose of allopurinol (≤100 mg/day or lower in the
case of chronic kidney disease) or febuxostat (≤40 mg/day) should be used. In addition, a treat-to-target
management strategy should be employed, with titration of ULT guided by serial serum urate
measurements, targeting a serum urate of <6 mg/dL. Concomitant anti-inflammatory prophylaxis therapy is
strongly recommended when initiating ULT for a duration of at least three to six months. For patients of
Southeastern Asian descent and for African-American patients, testing for the HLA-B*5801 allele is
conditionally recommended prior to starting allopurinol. Colchicine, nonsteroidal anti-inflammatory drugs,
or glucocorticoids are strongly recommended for the management of gout flares
Pharmacologic Choices
Urate-lowering therapy includes xanthine oxidase inhibitors and uricosurics (see Table 2). Indications for
the use of urate-lowering therapy includes established diagnosis of gouty arthritis along with either tophus
or tophi by clinical exam, ≥2 attacks per year, chronic kidney disease stage 2 or worse (GFR ≤89 mL/min/1.73
m2), or past urolithiasis. The optimal time to initiate urate-lowering therapy remains controversial. One
view is that the 1st attack is a late event in the gouty diathesis; even if further attacks do not occur, it
cannot be assumed that renal damage will not. The other view is that because recurrence may be delayed
for many years and chronic tophaceous gout develop only in a minority, therapy can be delayed until
recurrence or detection of tophi.
A treat-to-target approach has been recommended by ACR and EULAR. The minimum serum urate
target is <360 μmol/L, the saturation point of monosodium urate in the extracellular fluid. A lower serum
urate target (<300 μmol/L) is recommended in patients with more severe gout (e.g., patients with
tophaceous gout) to facilitate the improvement of signs and symptoms through the dissolution of urate
crystals. Once the urate crystals have dissolved, a maintenance serum target should be <360 μmol/L to
avoid the development of new tophi.
During the intercritical period, nonessential prescription medications that induce hyperuricemia should be
discontinued. Patients on low-dose ASA should continue therapy if used for prevention of cardiovascular
disease However, initiation or dose-adjustment of urate-lowering therapy may be important to prevent
recurrent gouty attacks with low-dose ASA. Thiazide diuretics can be continued in patients who
have difficult-to-control hypertension or worsening of hypertension with discontinuation.

69. Statin Monitoring, all EXCEPT


a) Arthralgia (SE of ezetimibe)/
b) Serum Creatinine
c) Creatinine Kinase
d) Liver function
Case reports have suggested an association between all statins and cognitive impairment (e.g., memory
loss, confusion). There has been a variable time to onset (1 day to years) and symptoms have generally
resolved within a median of 3 weeks after statin discontinuation. A systematic review and meta-analysis of
high-quality trials was unable to show that statins alter cognition in the short term. However, there was
evidence suggesting long-term statin treatment reduced the incidence of dementia.
Measure lipid levels and liver enzymes when starting statin therapy. Unless there is a concern regarding
elevated triglyceride levels, lipid levels may be drawn in the nonfasting state. Lipid levels may be repeated 4
weeks after starting treatment or following a dose change to asses treatment response.
Repeat liver enzyme testing after 3 months or if liver toxicity suspected. If symptoms of myopathy develop,
measure creatine phosphokinase levels.

70. 25-year-old male with meningitis case. What is the meningitis microorganism?
a) S Pneumonia (gram positive)
b) H influenza (gram negative)
c) N gonorrhea

71. What is the DOC for meningitis?


a) Ampicillin + Cefotaxime
b) Ceftriaxone + Vancomycin
c) Vancomycin + Ampicillin + Cefotaxime
d) IV Dexamethasone
72. What to give as add on therapy?
a) IV normal Saline
b) Dexamethasone IV
c) Acetaminophen
d) Dextrose

73. Patient with Pneumonia, confusion & fever was admitted to the hospital with PSI score 114,
why we start IV treatment with this patient: SEVERLY ILL or HIGH RISK OF DEATH
a) Patient is severely ill and need IV medication as oral medication won’t be effective
in his case
b) Because type 4 needs IV intervention PSI score more than 114
c) He can’t take oral medication because he is confused.
d) We should start IV as he has high fever
Route of administration
Antibiotics should be administered by oral route for outpatients. For inpatients, endo-venous treatment
should be switched to oral administration as clinical stability is obtained
Starting treatment- in case of septic shock
Antibiotic treatment should be started within 1 h after the diagnosis of pneumonia in case of septic shock,
because this reduces mortality, while the recommendations in the other categories of patients have a low
level of evidence
Starting treatment- in absence of septic shock
The recommendation to start antibiotics within 4 h from the diagnosis of pneumonia in the absence of septic
shock had low level of evidence; nevertheless a study on the implementation of the bundles of pneumonia
management revealed that early administration of antibiotics is one of the more consistent interventions
leading to reduced 30-day mortality [30-day mortality IP 22/250 (8.8%) vs. 253/1862 (13.6%), adjusted OR
0.59, 95% CI 0.95 to 0.37, p = 0.030] [29], confirming the need of an early start of adequate antibiotic
treatment for pneumonia.

74. Which medication is contraindicated with IV contrast Media?


a) Glyburide
b) Insulin
c) Metformin
d) Acarbose
Biguanides First choice for patients with a new and uncomplicated diagnosis of T2DM
Metformin 500–2500 mg / day PO divided BID or TID. Take with meals.
Reduce hepatic gluconeogenesis, lower glucose absorption and enhance insulin-mediated glucose
lowers uptake. ↓ uric acid. Effective treatment of polycystic ovary syndrome POS.
HbA1c by S.E: Nausea, diarrhea, abdominal discomfort, anorexia, metallic taste, lactic acidosis if hepatic,
approximate renal disease, CHF & alcohol (warn patient).
1%. Alcohol potentiates its hypoglycemic effect. No weight gains.
C.I in hepatic and renal impairment, previous lactic acidosis & pregnancy.
Can potentiate the hypoglycemic effects of insulin and sulfonylureas.
To avoid GI SEs, start low & titrate up q2 – 4 wk.
↓ vit B12 (cyanocobalamin) absorption thus can cause megaloblastic anemia.
Withhold prior to radio contrast media as they may deteriorate renal function.

75. What if the patient doing contrast media forgot and took the Metformin what he should do:
a) Monitor electrolytes (risk of lactic acidosis)
b) Take carbohydrate meal
c) Avoid high salt diet
The most serious adverse event that has been observed during metformin use is lactic acidosis, which is
characterized by an elevated blood lactate concentration (>5 mmol/L), decreased blood pH (<7.35) and
electrolyte disturbances with an increased anion gap (1,6–9).
76. What Rx need the pharmacy intervention?
a) Aspin 81mg per day
b) Fluconazole 150 mg Stat
c) Digoxin 0.125 mg TID
d) Azithromycin tid / 5 days
e) Azithromycin qd / 3 days
f) Azithromycin orally as a single dose
AZITHROMYCIN: Adults and children over 10 years:
Rapid oral loading: 750-1500micrograms (0.75mg-1.5mg) as a single dose. If a greater risk or less urgency
eg the elderly, the oral loading dose should be given in divided doses 6 hours apart, assessing clinical
response, before giving each additional dose.
Slow oral loading: 250-750micrograms (0.25mg-0.75mg) should be given daily for 1 week, followed by
appropriate maintenance dose. A clinical response should be seen within one week.
DIGOXIN: Individualize the dosage based on the patient’s age, weight, renal function and concomitant
drugs; the usual range is 0.0625–0.25 mg daily.

77. Diabetic patient, what is the least to monitor?


a) Hypertension
b) ALP
c) Serum Creatinine
d) Creatinine Albumin Ratio
e) HbA1c

78. Aspirin toxicity with the level given of PH 7.2, Co2, and HCO3 all are less than normal.
what is the Expected problem?
a) Respiratory acidosis followed by decompensated metabolic alkalosis.
b) Metabolic acidosis compensated by respiratory alkalosis
c) Respiratory alkalosis compensated by Metabolic acidosis
d) Metabolic acidosis compensated by respiratory acidosis
PH<7.25 so acidosis, HCO3 low so metabolic acidosis- body compensation: decrease CO2 (CO2 low) so
respiratory alkalosis

79. 55 y patient, FEV1(50%) what is the measurement needed to confirm the COPD diagnosis?
a) Pulse oximetry
b) Lung morphology (destruction of alveoli)
c) Alpha 1 antitrypsin
d) Inhaled drugs
INVESTIGATIONS:
A standardized approach (i.e., an anatomic diagnostic protocol [ADP]) will lead to identifiable cause(s) of
cough in most patients.
More detailed testing (e.g., CT scan, bronchoscopy) should be reserved for those with no overt cause
detected on initial evaluation and cough refractory to empiric therapeutic trials for the most common
etiologies with clinical suspicion for other less-common causes.
History and physical exam, with special attention to:
duration of cough (variation in cough over the day), history of smoking (tobacco, cannabis) and medication
use (ACE inhibitor), signs and symptoms of asthma, upper airway symptoms, GERD examination of nasal
cavities, oropharynx and otic canals, respiratory and cardiac systems occupational and environmental
causes, e.g., nonspecific irritants or allergens personal or family history of atopy/asthma
Objective measurements useful in the ADP:
- chest x-ray (CXR)
- spirometry to evaluate for asthma (pre-/post-bronchodilator) and COPD
- challenge testing (e.g., methacholine) to detect airway hyperresponsiveness; however, this can be
false-positive for asthma in up to 20% of cases in the investigation of cough

80. Acute Gout Case, what is the goal of therapy


a) Reduce inflammation
b) Reduce pain
c) Reduce serum uric acid

81. All are considered a risk factors for gout EXCEPT


a) Diet
b) Gender
c) Smoking
d) Alcohol
Drugs and Conditions Associated with Hyperuricemia and Gout
 Drugs: Alcohol, Cyclosporine, Cytotoxic chemotherapy, Diuretics (thiazide and loop), Ethambutol,
Interferon + ribavirin, Levodopa, Nicotinic acid (niacin), Pyrazinamide, Salicylates (low-dose),
Tacrolimus, Teriparatide.
 Conditions: Excessive alcohol intake, Atherosclerosis, Chronic kidney, glomerular, interstitial renal
disease, Diabetes, Hyperlipidemia, Hypertension, Ischemic heart disease, Lead intoxication,
Metabolic syndrome, Myeloproliferative disorders and some cancers, Obesity, Urolithiasis history,
rarely genetic or acquired causes of uric acid overproduction.
 Dietary factors: excessive protein diet from red meats, organ meats & shellfish,  Purine intake
(shellfish, vegetables such as asparagus, cauliflower, spinach, beans, peas & mushrooms)

82. Which drug may raise uric acid levels and lead to gout attacks
a) Carbamazepine
b) Sulfinpyrazone
c) Hydrochlorothiazide
d) Amiodarone

83. Which drug may be used for acute gout attack in hypertensive patient?
a) Allopurinol
b) Naproxen
c) Colchicine
d) Celecoxib

84. Cellulitis case, what is the microorganism


a) Moraxella Catarrhalis
b) Group A Streptococcus  predominant if non purulent cellulitis
c) S. aureus  predominant if purulent cellulitis
d) Pseudomonas aeruginosa

85. All can treat Cellulitis EXCEPT:


a) Clindamycin
b) Cephalexin
c) Cloxacillin
d) Norfloxacin

86. Cellulitis, Duration of treatment


a) 2 days
b) 5 days
c) 10 days
d) 4 weeks
87. 68 Patient his profile includes Levothyroxine, renal problem, Cloxacillin 6 month ago,
Zopiclone PRN went to hospital and diagnosed with C difficile, physician prescribe IV
Vancomycin. What is the reason of the C difficile diarrhea?
a) Age
b) Cloxacillin
c) Hypothyroidism
d) Renal problem
Risk factors for CDI
 Age > 65 (Elderly). Obesity. Enteral feeding. Immunosuppression. GI surgery. Chemotherapy
 Antibiotics use: Ampicillin, amoxicillin, broad spectrum penicillins, Cephalosporins, fluoroquinolones,
Clindamycin. Therapy with PPIs, H2RAs (lowers stomach acidity)
 Duration of hospitalization & Contact with infected person

88. For that patient with C difficile, what is your advice now
a) Monitor Electrolyte
b) Add Metronidazole
c) Change IV Vancomycin to PO
d) Change to Bismuth subsalicylate
Severity Criteria Treatment
Mild Diarrhea plus any additional signs or Vancomycin 125 mg QID PO ˣ 10 days.
moderate symptoms not meeting severe or complicated Fidaxomicin 200mg PO BID x 10 days
disease criteria Metronidazole 500 mg TID PO ˣ 10 days
WBC ≤15 x 10^9/L and a serum creatinine
(SCr) level of <133 mcmol/L
Severe disease Serum albumin < 3 g/dL + 1 of the following: Vancomycin 125 mg QID PO ˣ 10 days or;
WBC ≥15 ˣ 10^9/L & Abdominal tenderness Fidaxomicin 200mg PO BID x 10 days
Severe and Any of the following attributable to CDI: Vancomycin 500 mg QID PO +
complicated Admission to ICU for CDI Metronidazole 500 mg Q8H IV +
disease Hypotension ± required use of vasopressors Vancomycin 500 mg in 500 mL saline QID
“Fulminant” Fever ≥38.5 °C. Mental state changes PR as enema
Ileus or significant abdominal distension Surgical consultation suggested
WBC ≥35 ˣ 10^9/L or < 2 ˣ 10^9/L
Serum lactate levels >2.2 mmol/L
End organ failure, e.g., renal failure
Recurrent Recurrent CDI within 8 wk of completion of Repeat metronidazole or vancomycin pulse
CDI therapy regimen. Consider FMT Fecal microbiota
transplantation after 3 recurrences.
89. Patient on Bupropion SR 150 mg TID at 7am, 2 pm and 9 pm, and called at 12 noon and
told the pharmacist that he missed the morning dose, what should you recommend
a) Skip dose & take half tablet as soon as you remember and continue your normal schedule
b) Refer to his physician to reassess the dosing schedule.
c) Skip the missed dose and give the next dose as scheduled
d) Give the missed dose now and give the next dose as scheduled
Recommended Dose and Dosage Adjustment
The usual recommended dose of sustained-release bupropion hydrochloride is 100 to 150 mg/day given
once daily. As with all antidepressants, the full antidepressant effect of BUPROPION SR may not be evident
until several weeks of treatment. In patients who are not responding to a dose of 150 mg/day the dose may
be increased up to a maximum of 300 mg/day. Dose increases should occur at intervals of at least 1 week. In
order to minimize the risk of seizure, single doses of BUPROPION SR must not exceed 150 mg. Doses
of BUPROPION SR greater than 150 mg/day should be administered BID preferably with at least 8 hours
between successive doses.
Missed Dose
BUPROPION SR should be taken at the same time each day and no more than the recommended dose should
be taken each day. If the normal administration time has been missed, the dose should be skipped and
administration resumed at the normal administration time of the following day.
Administration
Patients should be advised to swallow BUPROPION SR tablets whole with fluids, and NOT to chew, divide,
crush or otherwise tamper with the tablets in any way that might affect the release rate of bupropion.
Monitoring
Rigorous clinical monitoring for suicidal ideation or other indicators of potential for suicidal behaviour is
advised in patients of all ages given an antidepressant drug. This includes monitoring for agitation-type
emotional and behavioural changes.
All patients with hepatic impairment should be closely monitored for possible adverse effects (e.g. insomnia,
dry mouth, seizures) that could indicate high drug or metabolite levels
Renal Impairment: Bupropion is extensively metabolized in the liver to active metabolites, which are largely
further metabolised before being excreted by the kidneys. Bupropion hydrochloride sustained-release
tablets treatment of patients with renal impairment should be initiated at a reduced dosage regimen, as
metabolites may accumulate in such patients to a greater extent than usual. The patient should be closely
monitored for possible adverse effects (e.g. insomnia, dry mouth, seizures) that could indicate high drug or
metabolite levels.

90. Patient taking alendronate every Wednesday morning and he missed the dose this morning
and he called you at 5 pm and asked you what to do
a) Refer to physician to advice regarding new schedule
b) Take the missed dose tomorrow morning, then return to original schedule
c) Take the missed dose tomorrow morning and rescheduled
d) Take the dose once you remember and continue as normal scheduled
91. Patient with CHF, Amiodarone was prescribed, it is contradicted with
a) Carvedilol
b) Verapamil
c) Lisinopril
d) Furosemide
Amiodarone should be used with caution in patients receiving β-receptor blocking agents (e.g., propranolol,
a CYP3A4 inhibitor) or calcium channel antagonists (e.g., verapamil, a CYP3A4 substrate, and diltiazem, a
CYP3A4 inhibitor) because of the possible potentiation of bradycardia, sinus arrest, and AV block; if
necessary, amiodarone can continue to be used after insertion of a pacemaker in patients with severe
bradycardia or sinus arrest

92. What drug may decrease post MI mortality in patient with Heart Failure
a) Carvedilol/ Bisoprolol
b) Hydrochlorothiazide
c) Digoxin
d) Verapamil

93. All drug may be used post MI prophylaxis EXCEPT


a) ASA
b) Statin
c) ACE inhibitor
d) Diltiazem should be given for future MI prevention

94. Patient had a knee surgery done recently; you are concerned to avoid what?
a) Cellulitis
b) DVT
c) Infection
d) RLS
95. Which is the most drug may cause Hepatotoxicity (or CI in hepatic dysfunction)
a) Valproic acid
b) Gabapentin
c) Topiramate
Topiramate monograph Decreased hepatic function
In hepatically-impaired patients, topiramate should be administered with caution as the clearance of
topiramate may be decreased.
96. 75 y old lady with Osteoarthritis in the back and knee, what is the non-pharm
recommendation:
a) Foot insoles
b) Advise her to go to senior aerobic classes
c) Knee brace
d) Put bars at home to decrease falling

Note that the best evidence of benefits with orthotics is for reducing foot pain in people with RA and lower
extremity OA.

97. She is undergoing a knee surgery, she wants something to relieve her acute pain before the
surgery, what is your most evident recommendation:
a) Capsaicin Cream
b) Diclofenac oral + Misoprostol (Arthotec)
c) Hyaluronic Injection
d) Corticosteroids Injection
e) Glucosamine and Chondroitin

98. Same lady had been prescribed Celebrex 200 mg by her doctor, what to monitor
a) Hypertension
b) GIT side effects
c) Renal function
d) Myocardial Infarction
Patients with history of MI or stroke, serious heart disease–related chest pain or serious heart disease such
as HF should not use COX-2 inhibitors. In patients at risk for ulcer complications (e.g., age >65, previous GI
bleed, comorbid medical conditions, low-dose ASA or anticoagulant use), consider using celecoxib with a PPI
1st followed by celecoxib alone and lastly a nonselective NSAID plus a PPI for gastrointestinal protection.
99. Same lady is taking acetaminophen 650mg 2 tablets QID, what is the Drug Therapy
Problem (DTP)?
a) GIT Side effects
b) Bleeding after surgery
c) High dose of Acetaminophen (>4 gm)
d) Acetaminophen in not suitable for her age

100. When using Capsaicin cream to relieve joint pain & improve symptoms, it may take up to
a) 30 min
b) 1-2 weeks
c) 3-4 weeks
d) 2 days

101. Patient with Osteoporosis. Hairdresser - standing all day. Two glasses of alcohol a week,
smokes 1/2 a pack a day. And she has knee
osteoarthritis but it doesn't hurt her. Swims
twice a week and works 2 km away from her
house. What's the recommended non-pharm:
a) Walk to work
b) Swim increase to 3 times a week
c) Avoid hazards to decrease risk of
falling in the house

102. What is her risk


factor for osteoporosis?
a) Smoking
b) Alcohol
c) Osteoarthritis

103. What may help in


osteoarthritis treatment?
a) Weight loss
b) Acupuncture
c) Knee brace
d) Aerobic exercise
104. Patient taking Lithium and candesartan, taking 2 cups of coffee per day and high sodium
diet & 1 cup alcohol per day, smoke 1 to 2 cigarettes per day. What is the DTP:
a) Candesartan interaction with lithium
b) Candesartan interact with high sodium diet
c) Alcohol can’t be taken with Candesartan
d) Smoking is prohibited with Candesartan

105. Which symptoms that patient may suffer with Lithium


a) Hirsutism
b) Hand Tremors
c) Weight Gain
d) Arthralgia

106. What will increase lithium toxicity with this patient


a) Diet
b) Caffeine
c) Renal Failure
d) Alcohol
e) Smoking

107. Pioglitazone contraindicated in


a) Renal Failure
b) Asthma
c) Heart Failure
d) Hypertension

108. HPT cause all end Organ damage all except:


a) Retinopathy
b) Cerebrovascular accident
c) Ventricular hypertrophy
d) Sexual dysfunction
e) Renal impairment

109. CARN is Canadian Adverse Reaction Newsletter published by


a) PUB-med
b) ISMP
c) MedEffect
d) Canadian pharmacy association
Health Product Info Watch (MedEffect™ Canada):
 A monthly publication intended primarily for healthcare professionals.
 Provides factual information on serious or unexpected side effects or adverse reactions suspected of
being associated with health products, such as prescription and non-prescription medications,
natural health products, biologically derived products, and medical devices.
 The Health Product InfoWatch replaced Health Canada's quarterly Canadian Adverse Reaction
Newsletter (CARN) in 2015.

110. In “SOAP” Smoking


history is related to
a) Subjective
b) Objective
c) Assessment
d) Plan

111. What is related to (Assessment) in SOAP for patient with cardio problems?
a) Framingham Score
b) Angina Symptoms
c) Exercise tolerance
d) Diet regime
Angina symptoms (critical assessment/conclusion by the HCP)
112. In SOAP notes what comes in “objective”?
a) Lipid profile
b) Allergy
c) Symptoms

113. What will present in the P part of “SOAP”?


a) Allergies
b) History of smoking
c) Blood test
d) Chest X-ray
e) Initiation of antifungal therapy

114. Patient on specific medications for 6 months and now he had no refill, his pharmacy is in a
province where pharmacist regulation allows him to do an extra refill, what will pharmacist do?
a) Do an extra refill for him and dispense
b) Contact doctor to ask extra refill
c) Ask patient to visit his doctor for follow up
d) Take a written consent from the patient and dispense
Renew and dispense (then inform the prescriber)

115. What is considered the highest level of Evidence Based Medicine:


a) Medline
b) Cochrane library
c) Meta-analysis
d) Pharmacy letter
116. Which option you will follow to best advice for patients
a) Follow RCT (Randomized Clinical Trial)
b) Follow most EBM (Evidence Based Medicine)
c) Manufacturer leaflet
d) News letters

117. What is NOT related to PIPEDA (Personal Information Protection & Electronic
Documents Act) rules
a) Don’t disclose personal information to anyone
b) To disclose personal information to third party
c) Use the personal information to be used in the pharmacy
d) To disclose personal information to Regulatory Authority
PIPEDA in brief
There are a number of requirements to comply with the law. Organizations covered by PIPEDA must
generally obtain an individual's consent when they collect, use or disclose that individual's personal
information. People have the right to access their personal information held by an organization. They also
have the right to challenge its accuracy.
Personal information can only be used for the purposes for which it was collected. If an organization is going
to use it for another purpose, they must obtain consent again. Personal information must be protected by
appropriate safeguards.
How the Act applies
PIPEDA applies to private-sector organizations across Canada that collect, use or disclose personal
information in the course of a commercial activity.
The law defines a commercial activity as any particular transaction, act, or conduct, or any regular course of
conduct that is of a commercial character, including the selling, bartering or leasing of donor, membership
or other fundraising lists.
What is personal information?
Under PIPEDA, personal information includes any factual or subjective information, recorded or not, about
an identifiable individual. This includes information in any form, such as:
age, name, ID numbers, income, ethnic origin, or blood type;
opinions, evaluations, comments, social status, or disciplinary actions; and
employee files, credit records, loan records, medical records, existence of a dispute between a consumer and
a merchant, intentions (for example, to acquire goods or services, or change jobs).
What is not covered by PIPEDA?
There are some instances where PIPEDA does not apply. Some examples include:
Personal information handled by federal government organizations listed under the Privacy Act
Provincial or territorial governments and their agents
Business contact information such as an employee’s name, title, business address, telephone number or
email addresses that is collected, used or disclosed solely for the purpose of communicating with that person
in relation to their employment or profession
An individual's collection, use or disclosure of personal information strictly for personal purposes (e.g.
personal greeting card list)
An organization's collection, use or disclosure of personal information solely for journalistic, artistic or
literary purposes
Unless they are engaging in commercial activities that are not central to their mandate and involve personal
information, PIPEDA does not generally apply to: not-for-profit and charity groups; or political parties and
associations.
Municipalities, universities, schools, and hospitals are generally covered by provincial laws. PIPEDA may
apply in certain situations.

118. Academic Detailing is an important tool to optimize the use of medications supported by
a) CMIRPS (Canadian Medication Incident Reporting and Prevention System)
b) CIHI (Canadian Institute of Health Information)
c) CADTH (Canadian Agency for Drugs and Technologies in Health)
d) Health Canada
Academic detailing across Canada
The academic detailing programs in Canada have followed some (if not all) of the principles of academic
detailing to foster improved clinical decision-making. In 2003, the Canadian Academic Detailing
Collaboration (CADC) was developed by 6 provinces (NS, ON, MB, SK, AB and BC) to represent the academic
detailers of Canada. With the support of the Canadian Agency for Drugs and Technologies in Health
(CADTH), representatives from each academic detailing program meet monthly (online) to share experiences
in academic detailing. CADC and CADTH have worked together to develop and disseminate key messages of
CADTH's evidence reviews to prescribers.
The mission of CADC is to:
1) promote the development and visibility of academic detailing in Canada,
2) collaborate in developing and disseminating evidence-informed interventions to optimize practice and
3) facilitate research and evaluation of academic detailing initiatives on health outcomes in Canada.
Since rigorous evidence of the real-world impacts of academic detailing in Canadian settings is lacking, the
CADC provinces formed the Academic Detailing Evaluation Partnership Team (ADEPT) in 2008 to evaluate
the impact of detailing on physician prescribing.

119. Patient coming to Canada from USA in a visit and he is going back in 2 days. He is out of
his hypertension medication Perindopril & Amlodipine. His US doctor faxed his RX. What is
true sentence if you will dispense 2-day supply
a) It is illegal to dispense
b) It is illegal but ethical to support Non-maleficent
c) It is legal but unethical to dispense
d) It is illegal and unethical
120. A pharmaceutical company that makes non-Rx cough and cold medicine wants to pay a
pharmacy to hold a patient education workshop. What is the best way to reduce conflict of
interest?
a) The pharmacy cannot accept payment for the workshop
b) The pharmacist in charge of the event cannot accept personal compensation
c) None of the company’s medications can be specifically showcased during the event
d) Only the company’s products with evidence-based efficacy can be presented during the
event

121. You are a pharmacist in charge for small size hospital, you got to know that there is an
international shortage in Fosphenytoin. What is the appropriate action?
a) Call the manufacturer to order all stock they have
b) Change all patients to Phenytoin
c) Keep the available stock for ICU use
d) Check for suitable alternative

122. Pharmacist give vaccine shot according to which one of the 5 main principles in the
Canada Health Act
a) Public Administration
b) Universality
c) Portability
d) Accessibility
e) Comprehensiveness
The Canada Health Act (1984) lists the conditions that provincial/territorial health insurance plans must
respect in order to receive federal cash contributions. The five conditions listed in the act are public
administration, accessibility, comprehensiveness, universality and portability.
CNA believes that these conditions are essential to Canada’s health care system:
 Public administration means that provincial insurance programs must be publicly accountable for the
funds they spend. Provincial governments determine the extent and amount of coverage of insured
services.
 Moreover, management of provincial health insurance plans must be carried out by a not-for-profit
authority, which can be part of government or an arm’s-length agency.
 Accessibility means that Canadians must have reasonable access to insured services without charge
or paying user fees.
 Comprehensiveness means that provincial health insurance programs must include all medically
necessary
 services. The Canada Health Act defines comprehensiveness, broadly, to include medically necessary
services
 “for the purpose of maintaining health, preventing disease, or diagnosing or treating an injury,
illness or disability.”
 Universality means that provincial health insurance programs must insure Canadians for all
medically necessary hospital and physician care. The condition also means that Canadians do not
have to pay an insurance premium in order to be covered through provincial health insurance.
 Portability means that Canadians are covered by a provincial insurance plan during short absences
from that province

123. When to look for dispensing regulation for Androgen


a) Narcotic Regulation
b) Controlled Drugs and Substances Act
c) Food and Drugs Act
d) Prescription Monitoring Act

124. What is true regarding Standards of Practice for Schedule II


a) No need for pharmacist intervention
b) Require professional intervention from the pharmacist at point of sale
c) May require intervention by the pharmacist to confirm the selection made by patient
d) Need a physician RX

125. Pharmacy manager at hospital want to buy a new computer system, what is the most
important to the least consideration: 1- Financial. 2- Stake holder needs. 3- Quality of system 4-
Try software program
a) 2 - 4 - 1 - 3
b) 2 - 3 - 1 - 4
c) 1 - 2 - 4 - 3
d) 3 - 2 - 4 – 1

126. All should follow the Narcotic Destruction Regulation EXCEPT


a) Ketamine
b) Pentazocine
c) Nabilone
d) Olanzapine

127. What may improve the quality of the meta analysis


a) To be retrospective
b) To be open and unspecified
c) Homogenous
d) To conclude many RCT
128. The main objective for Common Drug Review
a) Provides access to non-marketed drugs for practitioners
b) Providing recommendation for formulary listing
c) Control price of brand
d) Monitor the marketed health products in Canada

129. Pharmacy is holding a seminar about hypertension for the elderly, what to use in the
presentation in order to make sure patients understood the message?
a) Hand out leaflets
b) Use a slides and diagrams in the presentation
c) Place a microphone in front at the end of the presentation and have them come up to ask
questions
d) Give them handouts of the presentation

130. A lady asking about compression stockings. What is correct advice to this patient?
a) Compression stockings should be worn all day and taken off at night
b) Buy one size bigger so patient can feel comfortable
c) Put on stockings in the evening before you go to bed
d) Replace your stockings every 12 months
Elastic compression stockings are also known as surgical or support stockings or hose. In the simplest form,
support stockings are intended to relieve tired, aching legs and prevent swelling of feet, ankles and legs.
Individuals who stand or sit for long periods of time or older individuals with compromised venous return
are ideal candidates for these stockings. Compression stockings may be prescribed for medical conditions
such as varicose veins, lymphedema, venous eczema and ulceration, deep vein thrombosis, and post-
thrombotic syndrome.
Elastic compression stockings decrease superficial venous pressure, increase the upward flow in unoccluded
deep and superficial veins, and raise local interstitial pressure. Compression of the leg also prevents some
edema. Compression stockings are designed to give graduated support with the most pressure exerted at
the ankle, less at the calf and the least at the thigh. They provide varying compression, ranging from 12–60
mm Hg at the ankle. No single standard classification of compression is used. The most common is low or
class 1 of less than 20 mm Hg, medium or class 2 of 20–30 mm Hg, and high or class 3 of >30 mm Hg.
Proper fit is essential to ensure comfort and effectiveness. Ill-fitting stockings may be the reason an
individual discontinues wearing them. Some stockings, especially ones with high compression, may be
esthetically unappealing for some wearers who, as a result, choose not to wear them. The most common
reasons for not being able to wear compression stockings include skin damage (especially seeping, open
wounds), allergy to any of the stocking components, extensive leg edema or a malformed leg, a history of
peripheral artery disease, and marked impairment of sensation in the leg.
A correct fit requires accurate measurements of the nonedematous leg first thing in the morning. To ensure
effective compression, the stocking should be washed and dried according to the manufacturer’s directions
and replaced every 2–3 months. They are intended to be removed at night and put on in the morning before
beginning daily activities. Manual dexterity is needed to put on or remove the stockings, and devices are
available to assist with this.
Compression stockings are available as pantyhose or socks (above or below the knee), with open or closed
toes and in various colours.
The required compression and affected leg area should be the starting point in selection.
Antiembolism stockings are worn by the nonambulatory individual to prevent venous emboli caused by
inactivity. They provide less support and are not suitable for ambulatory individuals.
Nontherapeutic elastic stocking is widely available and often used by air travelers or by people who feel
they have “tired legs.” Precise measurements are not needed and the compression is usually uniform.

131. Which vitamins would you give to a patient undergoing hemodialysis 3 times per week?
a) Refer to physician
b) Vit D
c) Fat-soluble vitamin
d) Water-soluble vitamins

132. Female patient diagnosed with Bacterial Vaginosis, what should be done?
a) Refer as it sexually transmitted disease
b) Do not refer, because it’s not a sexually transmitted disease
c) Report to health authority as it sexually transmitted disease
d) Refer although it is not sexually transmitted disease

133. For the same female patient with Bacterial Vaginosis, what is correct
a) Poorly controlled diabetes considered as predisposing factor
b) Sexually transmitted and can be prevented by practising safer sex
c) Treatment of male partners is not indicated
d) Associated with an increased risk of HIV

134. A case about hypertensive patient COPD and acute exacerbation need antibiotic?
a) Levofloxacin
b) Clarithromycin
c) Amox / Clavulanic
d) Ceftriaxone

135. Female asking a pharmacist if her husband got a sexual transmitted disease, what to do?
a) Tell her to ask her husband
b) Go and ask his doctor
c) Go and do HIV test
d) Take a protective measure
136. Chlorhexidine mouth wash, what do you counsel
a) Might stain your teeth
b) Separate it from fluoride containing toothpaste (both)
c) Can cause Dizziness & Headache
d) Can cause Dysphagia

137. When to give an ACEI and ARB together? (obsolete)


a) Heart Failure
b) Post Myocardial infarction
c) Cerebrovascular haemorrhage
d) Diabetic nephropathy

138. Patient with left ventricular hypertrophy, what cannot be used:


a) Hydralazine
b) Enalapril
c) Amlodipine
d) Thiazide

139. Which drug cause urine incontinence


a) Furosemide
b) Levodopa/carbidopa
c) Meperidine
d) PEG
140. The most effective in reducing the frequency and severity of hot flashes is
a) Gabapentin
b) Venlafaxine
c) Amitriptyline
d) Fluoxetine

141. M.O found in parental solution; Culture shows -ve coagulase gram +ve cocci
a) Staph Epidermis
b) Strept. Pneumonia (-ve Catalase)
c) Staph Aureus (+ve coagulase)
d) Strept Pyogen (-ve Catalase)
Staphylococcus epidermidis which is known as a coagulase-negative and Gram-positive Staphylococcus, is
one of the five significant microorganisms that are located on human skin and mucosal surfaces with the
ability of causing nosocomial infections due to the wide usage of medical implants and devices

142. Pt on 40 mg Atorvastatin & Levofloxacin suffer from myotoxicity. This may be due to
a) High statin dose
b) Use of levofloxacin along with Atorvastatin
c) Using Atorvastatin for long period
d) Side effects of statin and it will be controlled by time

143. Nitrofurantoin preferred with food to


a) Prevent GIT side effects
b) Decrease incidence of diarrhea
c) Increase bioavailability by 40 %
144. Pt has depression and need to use antidepressant that not affect his sexual dysfunction what
to give
a) Mirtazapine or Bupropion
b) Sertraline
c) Venlafaxine
d) Fluoxetine

145. Mirtazapine SE
a) Somnolence at daytime
b) Anxiety
c) Weight loss
d) Dry mouth

146. The long-term use of corticosteroids (6 months) may cause all except
a) Photosensitivity
b) Mood changes
c) Hyperglycemia
d) Osteoporosis
What side effects can corticosteroids cause?
Side effects depend on the dose of medication you receive and may include:
 Elevated pressure in the eyes (glaucoma)
 Fluid retention, causing swelling in your lower legs
 High blood pressure
 Problems with mood swings, memory and behavior and other psychological effects, such as
confusion or delirium
 Weight gain, with fat deposits in your abdomen, face and the back of your neck
 When taking oral corticosteroids longer term, you may experience:
 Clouding of the lens in one or both eyes (cataracts)
 High blood sugar, which can trigger or worsen diabetes
 Increased risk of infections, especially with common bacterial, viral and fungal microorganisms
 Thinning bones (osteoporosis) and fractures
 Suppressed adrenal gland hormone production, which may result in a variety of signs and symptoms,
including severe fatigue, loss of appetite, nausea and muscle weakness
 Thin skin, bruising and slower wound healing
Side effects of inhaled corticosteroids
When using inhaled corticosteroids, some of the drug may deposit in your mouth and throat instead of
making it to your lungs. This can cause: Fungal infection in the mouth (oral thrush) & Hoarseness.
If you gargle and rinse your mouth with water — don't swallow — after each puff on your corticosteroid
inhaler, you may be able to avoid mouth and throat irritation. Some researchers have speculated that
inhaled corticosteroid drugs may slow growth rates in children who use them for asthma.
Side effects of topical corticosteroids
 Topical corticosteroids can lead to thin skin, red skin lesions and acne.
Side effects of injected corticosteroids
 Injected corticosteroids can cause temporary side effects near the site of the injection, including skin
thinning, loss of color in the skin, and intense pain — also known as post-injection flare. Other signs
and symptoms may include facial flushing, insomnia and high blood sugar. Doctors usually limit
corticosteroid injections to three or four a year, depending on each patient's situation.
Reduce your risk of corticosteroid side effects
 To get the most benefit from corticosteroid medications with the least amount of risk:
 Try lower doses or intermittent dosing. Newer forms of corticosteroids come in various strengths and
lengths of action. Ask your doctor about using low-dose, short-term medications or taking oral
corticosteroids every other day instead of daily.
 Switch to nonoral forms of corticosteroids. Inhaled corticosteroids for asthma, for example, reach
lung surfaces directly, reducing the rest of your body's exposure to them and leading to fewer side
effects.
 Make healthy choices during therapy. When you're taking corticosteroid medications for a long time,
talk with your doctor about ways to minimize side effects. Eat a healthy diet and participate in
activities that help you maintain a healthy weight and strengthen bones and muscles.
 Consider taking calcium and vitamin D supplements. Long-term corticosteroid therapy may cause
thinning bones (osteoporosis). Talk with your doctor about taking calcium and vitamin D
supplements to help protect your bones.
 Take care when discontinuing therapy. If you take oral corticosteroids for a long time, your adrenal
glands may produce less of their natural steroid hormones. To give your adrenal glands time to
recover this function, your doctor may reduce your dosage gradually. If the dosage is reduced too
quickly, your adrenal glands may not have time to recover and you may experience fatigue, body
aches and lightheadedness.
 Wear a medical alert bracelet. This or similar identification is recommended if you've been using
corticosteroids for a long time.
 Get regular checkups. If you're taking long-term corticosteroid therapy, see your doctor regularly to
check for side effects

147. Alcoholic patient has an


attitude problem and hallucinating
with hepatic encephalopathy, so
what will be elevated in his lab
work:
a) Serum ammonia
b) Alkaline phosphatase
c) Creatinine Kinase
d) LFT
148. Patient with Hepatic Encephalopathy on Lactulose, what is the indicator of improvement
a) Gastrointestinal bleeding control
b) Become conscious
c) Decrease AST and ALT
d) Weight loss
149. He is now moving to another province in Canada. What can the pharmacist advice about
his medication?
a) He can only refill in the residing province
b) He can get lactulose in another province as it is OTC medicine (however,
prescribing and monitoring by the specialist is recommended)
c) He needs to get a new prescription in the new province
Lactulose is schedule III

150. Lactulose was prescribed for the above patient, so how long he has to take it:
a) Till he regains his consciousness
b) Till his liver functions improve
c) Till he quits alcohol
Until normal mentation is restored

151. Pregnant women got Gonorrhea what is the DOC


a) Ceftriaxone
b) Amoxicillin
c) Cefixime
Untreated gonococcal infections may lead to endometritis and pelvic sepsis in the mother and ophthalmia
neonatorum and systemic infection in the newborn.
Screen all pregnant women at 1st prenatal visit. Ceftriaxone is the preferred antibiotic (see Table 7).
Patients should abstain from sexual intercourse for 7 days after initiation of adequate treatment and until
all sexual partners have been treated. Patients and ongoing sexual partners should have a follow-up test of
cure 3–4 weeks after completing therapy. Always treat for chlamydia in confirmed gonococcal infections
.

152. Pneumococcal vaccine will prevent all EXCEPT:


a) Endocarditis
b) Diabetes
c) Meningitis
d) Otitis Media

153. Lady with psoriasis had chest infection and taking Doxycycline, she uses prednisolone.
what trigger Psoriasis symptoms:
a) Cold Weather
b) Infection
c) Prednisolone
d) Doxycycline use
154. The DOC for Psoriasis is
a) Methotrexate
b) Sulfasalazine
c) 6-MP
d) Azathioprine
Psoriasis is a common inflammatory disorder affecting the skin that has a genetic component and several
triggers (eg, trauma, infection, certain drugs).
The most common skin findings are usually well-circumscribed, erythematous papules and plaques covered
with silvery scales in plaque psoriasis, but lesions differ between the other less common subtypes of
psoriasis.
Psoriatic arthritis develops in 5 to 30% of patients and can cause joint destruction and disability.
Diagnose based on the appearance and distribution of lesions.
Use topical treatments (eg, emollients, salicylic acid, coal tar preparations, anthralin, corticosteroids,
vitamin D3 analogs, calcineurin inhibitors, tazarotene), particularly for mild disease.
Use ultraviolet (UV) light therapy, usually for moderate or severe psoriasis.
For extensive psoriasis, use systemic treatments, such as Acitretin (oral retinoid), immunomodulatory
(biologic) agents, methotrexate, cyclosporine, retinoids, and/or other immunosuppressants.

155. What is true about Atopic Dermatitis (Pruritus)


a) Oil baths before taking a shower to prevent dehydration
b) Topical Pimecrolimus is more effective than corticosteroids
c) Oatmeal bath not equal effective as pre oil bath
d) Loratidine better than diphenhydramine

156. 48 y Technician pharmacist living in basement, she was absent from work as she had
asthma, she used Symbicort 2 buffs BID, what is the goal of Asthma treatment
a) Decrease absence from work
b) Prevent using of oral prednisone
c) Decrease exercise induced asthma
d) Decrease night time asthma attack
Goals of Therapy
- Prevent asthma-related mortality
- Prevent exacerbations
- Maintain asthma control:
maintain normal activity levels, e.g., avoid absence from work or school, maintain ability to
exercise without limitations
prevent daytime (e.g., cough, wheeze, dyspnea; goal: ≤twice per week) and nocturnal
symptoms (e.g., night waking; goal: none)
- prevent need for reliever therapy (goal: ≤twice per week)
- Provide optimal pharmacotherapy and avoid adverse effects
157. What may trigger her Asthma symptoms
a) Occupation
b) Current residence
c) Age
d) Mal-nutrition

158. What is your current recommendation for that patient


a) Increase the dose of Symbicort Inhaler to 8 puffs / day
b) Add Tiotropium
c) Put a plan for her asthma
d) Advice Symbicort before exercise
For those willing to use an ICS/LABA as reliever therapy (only currently indicated for budesonide /
formoterol), the asthma action plan can be a bit simpler. The plan may include the use of the ICS/LABA as
required (up to 8 puffs/day) or adjusting the controller dose aggressively, the latter not having as much
evidence and being rarely done.

159. All are true about Montelukast uses EXCEPT


a) It decreases CS use
b) Used with patient refuse ICS
c) For rescue asthma
d) Exercise induced asthma

160. Patient on Corticosteroid, physician will start to withdraw the corticosteroids, the concerns
related to corticosteroids withdrawal is:
a) Cushing Syndrome
b) Stomach irritation
c) Fluid retention
d) Adrenal insufficiency

161. When patient using corticosteroids, the expected side effects


a) Puffy face
b) Bruising of the skin (purpura)
c) Slow gait
d) Weight loss

162. Patient taking mixed insulin (25 % lispro + 75% Neutral Protamine Hagedorn -NPH) 32
Unit morning before breakfast and 24 unit with supper. Patient admitted to the hospital and
there is shortage of mixed insulin that patient takes, so what can be given as an alternative for
the morning dose:
a) 18-unit Lispro and 6 Unit NPH
b) 24 Unit Lispro and 8-unit NPH
c) 6-unit Lispro and 18 Unit NPH
d) 8-unit Lispro and 24-unit NPH
32U= 8 U short acting lispro + 24 U NPH

163. HIV patient on Clotimoxazole, and got Pneumocystis Jiroveci Pneumonia (PJP) infection
and have sulfa allergy, what is the alternative
a) Trimethoprim
b) Cephalexin
c) Atovaquone
d) Pentamidine

164. Post MI hypertensive patient, while patient is discarded from the hospital what to advice
for smoking cessation
a) Bupropion
b) Nicotine gum (1st line)
c) Varenicline
d) No need as MI will encourage patient for smoking cessation
Smoking cessation programs and cardiac rehabilitation referral should be offered early post-MI to all
motivated patients.
Use of the nicotine replacement strategies bupropion or varenicline can be safely offered in the early post-
MI period

165. COPD patient on Sulbutamol, Formetrol & Tiotropium. What is your recommendation?
a) Sulbutamol
b) Tiotropium
c) Formetrol
d) Roflumilast to be add on
Phosphodiesterase 4 (PDE4) Inhibitors
Roflumilast is an oral medication that suppresses the release of inflammatory mediators through inhibition
of cyclic AMP breakdown. It is indicated as add-on therapy with bronchodilators for the maintenance
treatment of severe COPD associated with chronic bronchitis (history of chronic cough and sputum) in adult
patients with a history of frequent exacerbations.
Roflumilast has demonstrated a significant improvement in prebronchodilator FEV1 (48–80 mL increase)
along with a small reduction in exacerbations requiring systemic corticosteroids (8%), even for patients who
were also taking a LABA. A Cochrane review found that patients were 23% less likely to experience an
exacerbation over the study period (≤1 year) with an NNT of 20; however, quality of life and symptoms
scores were not greatly improved.
Data support addition of roflumilast to foundation bronchodilatory therapy to reduce exacerbations, either
before or in addition to introduction of ICS/LABA, particularly in patients with
frequent or severe exacerbations.
Roflumilast may be considered for add-on therapy to existing triple therapy (ICS/LAMA/LABA or LAMA +
ICS/LABA) for people with COPD who have had at least 1 exacerbation in the past year.
Methylxanthines

166. What should be avoided for patient on taking Roflumilast


a) Tiotropium
b) Omeprazole
c) Candesartan
d) Metoprolol

167. A drug with NPN what is true


a) Should be sold in pharmacy only
b) Need pharmacist intervention before dispensing
c) Can be found in self-selection area only in pharmacy
d) Could be found in any retail outlet
What are natural health products?
‘Natural health products’ is a term used to describe herbs and other supplements made from natural sources
that are sold to prevent sickness or promote health. There are many different types, including: vitamins and
minerals; herbal remedies; homeopathic medicines; traditional medicines (traditional Chinese medicine,
traditional East Indian [Ayurvedic] medicine, etc); probiotics; and amino acids and essential fatty acids.
You can buy natural health products in pharmacies and other stores without a prescription, or you can get
them from a care provider.
Are these products regulated?
Health Canada has a Natural Health Products Directorate that regulates products for sale in Canada. Check
the label. If a product has a Drug Identification Number (DIN) or a Natural Product Number (NPN) or Drug
Identification Number –Homeopathic Medicine (DIN-HM), it has been assessed by Health Canada.
The Natural Health Products Directorate of Health Canada regulates products that have a wide margin of
safety. It is working to develop appropriate paediatric labelling to help guide parents.

168. New treatment for Multiple Sclerosis (MS) where to find


a) CPS
b) e-CPS
c) PubMed
d) Merck Index
169. What is the most important thing pharmacist can do to enhance patient compliance
a) Check the pharmacy report about late refill
b) Asking patient family member & care giver about patient adherence
c) Let pharmacy staff to ensure refilling RX on time
d) Technician to send refill few days before due date

170. Patient on Bupropion 150 mg at 7 am, 2 pm, 9 pm. Missed one dose what to do
a) Refer to his physician to reassess the schedules
b) Skip the dose and continue as normal
c) Take 2 doses once he remembers

171. All should be taken on empty stomach EXCEPT


a) Indinavir
b) Cefuroxime Axetil
c) Azithromycin
d) Residronate
After oral administration cefuroxime axetil is absorbed from the gastrointestinal tract and rapidly
hydrolysed in the intestinal mucosa and blood to release cefuroxime into the circulation. Optimum
absorption occurs when it is administered shortly after a meal.
Following administration of cefuroxime axetil tablets peak serum levels (2.9 μg/mL for a 125 mg dose, 4.4
μg/mL for a 250 mg dose, 7.7 μg/mL for a 500 mg dose and 13.6 μg/mL for a 1000 mg dose) occur
approximately 2.4 hours after dosing when taken with food. The pharmacokinetics of cefuroxime is linear
over the oral dosage range of 125 to 1000 mg. No accumulation of cefuroxime occurred following repeat
oral doses of 250 to 500 mg.

172. Mom comes with her 7-y daughter complaining that she tried Nix (Permethrin 1%) twice
for her daughter who have long hair and she implement pharmacist advice but it doesn’t work,
what is the most likely reason for not working?
a) She didn’t soak well hair for enough time
b) Not enough quantity was used (needs 2 bottles for long hair)
c) She didn't dry clean the bed sheets
d) She has a long hair

173. She came back after 2 weeks and ask for your recommendation as teacher found live lice
once again
a) Lindane
b) Tea tree oil
c) Permethrin
d) Isopropyl Myristate
Failure of permethrin treatment (KDR) can be immediate or delayed as explained:
1) Immediate (on 1st application) if you've lived lice 12-24 hours after correctly applying permethrin.
2) Delayed (after 2nd application) if live lice were found after a technically correct 2nd application that is 7-
10 apart from a technically correct first application.
"Delayed kdr" means already 2 applications of permethrin 7-10 days apart. If this occurs, switch to physical
agents.
1st July--》 1st application Permethrin
10th July --》 2nd application permethrin
11th july found live lice delayed knock down resistance -------》 move to physical agents
Treatment failure of head lice may be due to resistance to topical agents with a neurotoxic mode of
action, including permethrin and pyrethrin/piperonyl butoxide.[23] In contrast, resistance to those products
with a physical mode of action (isopropyl myristate 50%/cyclomethicone 50% and dimeticone) is unlikely
to develop.
Other explanations for treatment failure include misdiagnosis, lack of adherence or reinfestation. Treatment
with permethrin may fail if hair is not thoroughly soaked. Two bottles are often needed for thick or long
hair. In cases of resistance, switching to a different pharmacologic class may be helpful, although resistance
to permethrin may cross over to pyrethrins and other pyrethroids. The following have been advocated:
permethrin 5% cream applied to scalp and left on for several hours or overnight; oral sulfamethoxazole/
trimethoprim in combination with permethrin 1%; topical crotamiton 10% applied to scalp and left on for 24
hours in adults; oral ivermectin 200 mcg/kg repeated in 10 days or an alternative regimen of 400 mcg/kg
repeated in 7 days. Ivermectin is not approved for use in children weighing <15 kg.
Topical ivermectin 0.5% is effective for treatment of head lice, but is not available in Canada. Topical
ivermectin 1% is available in Canada for the treatment of rosacea but it has not been studied in the
treatment of lice or scabies.

174. Mom discovered that she has lice but she is 2 months pregnant, what do you recommend
for her?
a) Permethrin
b) Soak head in vinegar
c) Pyrethrin
d) Refer

175. Physician asking the pharmacist about the latest guidelines' treatment for a Osteoporosis,
where would the pharmacist find it?
a) e-Therapeutics
b) CPS
c) Merck Manual
d) Clinical Guidelines
e-Therapeutics + is now RxTx. New look. New Name. Same Great Content
We’ve redesigned and rebranded e-Therapeutics+ with the Canadian health care provider in mind.
RxTx includes a powerful search engine designed to get you the drug and therapeutic information, along
with the clinical decision support tools you need in fewer steps. You will have faster access to the same
great content with an improved, streamlined design.
 New navigation and term suggestions make searching faster
 Comprehensive full document searches across all drug and therapeutic content
 Plus, many more new features to help you find what you need quicker and easier
 RxTx—Designed to save you time and improve health outcomes for your patients.

176. A child 25 kg needs to take 2 mg/kg daily elemental iron of ferrous sulphate. The only
formulation you have in stock is 300 mg/5ml, which contains 20 mg/ml of elemental iron, how
many ml would be needed for a 30-day supply?
a) 60
b) 75
c) 90
d) 120

177. Selenium Calculation: you have 0.5 micromole/L,


your stock is 0.25 micromole/L. How much of 40
mcg/ml of selenium would you add if you want to
make 482L (M wt of selenium is 79)?
a) 0.4
b) 0.3
c) 0.34
d) 0.24

178. A physician prescribed for a patient a fortified


eye drops Tobramycin 13.5mg /ml gtt ou twice daily.
How much of the 40 mg/ml stock solution should be
added to 5ml of 0.3% to get the desired concentration?
a) 1 ml
b) 1.5 ml
c) 2 ml

179. The Questions about stats and a study is done on a drug X and placebo, and drug X
showed negative superiority results (haga keda), so what does that mean?
a) There is no difference between drug and placebo
b) The drug was inferior to the placebo
c) Placebo inferior to the drug
180. Long case, about patient on so
many meds, and recently was
admitted to the hospital because he
is drinking too much alcohol, now
he is cleared up which symptom
would indicate alcohol withdrawal
symptoms
a) Sweating
b) Mydriasis
c) Vivid dreams
d) Tremor

181. For alcohol withdrawal, which


medication should he be given?
a) Clonidine
b) Venlafaxine
c) Diazepam
d) Suboxone

182. What may lead to decrease


volume of distribution (VD)?
a) Depression
b) Diabetes
c) Liver Impairments
d) Fat in Diet

183. What may increase bioavailability of a 100 % renal excreted drug?


a) Volume of distribution (VD)
b) Low PH
c) Increase Hepatic clearance
d) Low protein diet

184. Patient with runny nose & congestion for 2 days his profile includes HT+ BPH + GERD.
What is your recommendation?
a) Loratidine
b) Normal saline
c) Refer to physician
d) Fluid & rest
e) Pseudoephedrine
185. Where we can check for Drug-Drug interaction
a) Clinical Guidelines
b) Micromedex (very comprehensive)
c) Nominum index
d) Remington

186. 6 y child with primary nocturnal enuresis (3-4 bed-wetting / week), what non-
pharmacological advice may help
a) Caffeinated beverage restriction
b) Decrease Na intake
c) Restricted activity before sleep

187. 6 y child with primary nocturnal enuresis (3-4 bed-wetting / week), goal of treatment with
Desmopressin is
a) Decrease sphincter contraction at night
b) Decrease urine production
c) To reduce the number of wet nights to once / week
d) Complete dry nights within 1 month
188. In order to weigh a compound on a balance with 5% error & has sensitivity error of 6 mg.
What is the maximum weighable amount?
a) 30 mg
b) 12 mg
c) 6.3 mg
d) 120 mg
Error= Sensitivity/wt. – wt= 6/0.05= 120 mg

189. A physician prescribed potassium supplements 20mEq TID to patient but this patient does
not want to take supplements. He asks the physician if he can eat bananas instead. If each large
banana has 602 potassium. How many bananas should the patient eat each day? (M. wt K+=39)
Answer:
20 m. Eq TID = 60 m. Eq per day m. Eq = M. Wt * Valency = 39 * 1 = 39 mg
Each day = 60mEq = 60*39= 2340 mg
Each banana contains 602 mg No. of banana = 2340/6002 = 3.88

190. What is used to assess the extent of heart failure


a) FEV
b) NYHA
c) ECG
d) CHAD
191. With which condition Dronedarone is Contraindicated
a) GIT problems
b) Renal Failure
c) Heart Failure
d) Asthma
Patients with history of, or current heart failure or left ventricular systolic dysfunction
Dronedarone is contraindicated in patients in unstable hemodynamic conditions, with history of, or current
heart failure or left ventricular systolic dysfunction.
Patients should be carefully evaluated for symptoms of Congestive Heart Failure. There have been
spontaneously reported events of new or worsening heart failure during treatment with dronedarone.
Patients should be advised to consult a physician if they develop or experience signs or symptoms of heart
failure, such as weight gain,
dependent oedema, or
increased dyspnoea.
If heart failure develops,
treatment with dronedarone
should be discontinued.
Patients should be followed
for the development of left
ventricular systolic
dysfunction during
treatment.
If left ventricular systolic
dysfunction develops,
treatment with dronedarone
should be discontinued.
192. What is true about plan B
a) Provide protection from HIV/AIDS & STDs/STIs
b) Don’t prevents a fertilized egg from attaching to the uterus
c) Some women may experience spotting after taking plan B
d) If period is 3 days late after plan B, it is possible that there is a pregnancy

193. Side Effect of Fluoroquinolones


a) Ascites
b) Hypoglycemia
c) Ankle edema
d) Osteoporosis
Dysglycaemia: As with all quinolones, disturbances in blood glucose, including both hypoglycaemia and
hyperglycaemia have been reported, usually in diabetic patients receiving concomitant treatment with an
oral hypoglycaemic agent (e.g., glibenclamide) or with insulin. Cases of hypoglycaemic coma have been
reported. In diabetic patients, careful monitoring of blood glucose is recommended.

194. Where you can find list of lactose-free medications


a) PubMed
b) Merck Index
c) Martindale
d) CPS (not in a list though)

195. Patient on Ibuprofen & levothyroxine what to tell him


a) Call doctor if you feel any shortness of breath
b) See improvement in 1 week
c) Separate between Ibuprofen 4-6 hours
d) Levothyroxine should be administered as a single daily dose, preferably after breakfast
NSAIDs: False low plasma concentrations have been observed with concurrent anti-inflammatory treatment
such as phenylbutazone or acetylsalicylic acid and levothyroxine therapy.

196. Calculate the NNT if absolute reduction is 8% & relative reduction is 40%
a) 1.25
b) 2.5
c) 5
d) 12.5
197. Doctor need a recommendation on RLS (Restless Leg Syndrome), where to search
a) PSC
b) Clinical Guidelines
c) CPS
d) Ident-A-Drug

198. Oral dose with 85% bioavailability of 500 mg IV, how many oral mg should give the same
IV Bioavailability
a) 425 mg
b) 585 mg
c) 575 mg
d) 525 mg

199. Calculate the t ½ of Vancomycin, first dose given at 10 am and second dose taken at 10 pm
(12 hours total time)
a) 12 h
b) 22 h
c) 24 h
d) 6 h

200. A researcher hypothesized that black cohosh is beneficial with osteoporosis women and he
followed them over time using data available from 1995 to 2005. Which study design best
describes this study?
a) Cohort
b) Case control
c) Case report
d) Case series

201. A lady on Tri-Cyclen 21 (Triphasic) and she


is travelling for 2 weeks and she want to skip her
menses while she is travelling, what is your
recommendation?
a) Start a new pack after the old pack
immediately
b) Start a new pack for 2 weeks and stop
c) Continue with the 3rd week of a new
pack after the old pack
d) Start a new pack on the 2nd week of old
pack
Another version.
A girl uses oral contraceptive pills. She doesn’t want menses for 1 week. What to tell her?
a) Start 1st row of second pack on 22th day of first pack
b) Start 2nd row of second pack on 22th day of first pack
c) Start 3rd row of second pack on 22th day of first pack
d) Start 4th row of second pack on 22th day of first pack

202. Post Myocardial infraction Patient have a RX Nifedipine, you call doctor to change it,
which ethical principal you follow
a) Autonomy
b) Justice
c) Beneficence
d) Nonmaleficence

203. Patient taking Nifedipine and candesartan develop ankle edema, what may increase the risk
for swelling?
a) ASA
b) Diet
c) Ibuprofen
d) Acetaminophen

204. A lady with panic disorder (scares of flying). She is on Clonazepam & citalopram what to
say?
a) Benzodiazepines will not provide benefits of speed of response and overall response
b) Clonazepam long-term use is not recommended it can be taken PRN for acute
anxiety
c) Benzodiazepine is not approved for the treatment of Panic Disorder (PD) disease
d) Combination may offer higher levels of panic symptoms compared to the monotherapy
First-Line Agents: The SSRIs citalopram, escitalopram, fluoxetine, fluvoxamine, paroxetine and sertraline are
all effective in treating panic disorder with or without agoraphobia. In several meta-analyses, SSRIs have
been shown to improve panic symptoms, avoidance behaviours associated with agoraphobia, depressive
symptomatology and general anxiety; however, no SSRI has proved superior to any other.
Venlafaxine, an SNRI, has also demonstrated efficacy in reducing the severity of panic disorder
symptoms and is associated with a response rate similar to SSRIs.
Second-Line Agents: There is clinical evidence supporting the use of TCAs in the treatment of panic disorder,
and imipramine and clomipramine have been shown to reduce the frequency and severity of panic attacks
and symptoms of agoraphobia. There is, however, much less clinical evidence for the use of desipramine
or nortriptyline in panic disorder. Despite having efficacy similar to SSRIs, TCAs remain second-line
agents because of their significant adverse effects profile and the risk of toxicity in overdose.
Mirtazapine has demonstrated efficacy in open-label trials and in a randomized controlled study.
Benzodiazepines are a second-line option in the treatment of panic disorder, and alprazolam,
clonazepam, lorazepam and diazepam have been studied for this indication.
Studies have also been conducted with alprazolam and clonazepam as an adjuvant treatment with an
antidepressant. However, since these drugs are associated with a risk of abuse, dependence and withdrawal
syndrome, as well as a risk of falls and CNS adverse effects, they are generally not used for long-term
treatment of panic disorder. Their use is mainly limited to the first few weeks of treatment when rapid relief
of anxiety or panic attacks is necessary, or to reduce the exacerbation of anxiety and agitation that may be
present at the beginning of antidepressant treatment.
Clonazepam and lorazepam are generally the preferred agents, and while alprazolam and diazepam have a
faster onset of action, they present a greater risk of abuse. In addition, with its short elimination half-life,
alprazolam is associated with increased anxiety between doses and a greater withdrawal syndrome.

205. Patient with rheumatoid arthritis undergoing hip arthroplasty, educate patient about
a) Muscle exercise
b) Steps to reduce DVT risk
c) How to prevent risk of fracture
d) Dash Diet

206. Which of the following considered as Osteoporosis risk factor


a) Overweight
b) Smoking
c) Low thyroid hormone level
d) High protein diet

207. Cipro (CIPROFLOXACIN) will increase the level or effect of all the below EXCEPT
a) Lovastatin
b) Pravastatin
c) Atorvastatin
d) Simvastatin
Products metabolised by cytochrome P450: Pravastatin is not metabolised to a clinically significant extent
by the cytochrome P450 system. This is why products that are metabolised by, or inhibitors of, the
cytochrome P450 system can be added to a stable regimen of pravastatin without causing significant
changes in the plasma levels of pravastatin as have been seen with other statins. The absence of a
significant pharmacokinetic interaction with pravastatin has been specifically demonstrated for several
products, particularly those that are substrates/inhibitors of CYP3A4 e.g. diltiazem, verapamil, itraconazole,
ketoconazole, protease inhibitors, grapefruit juice and CYP2C9 inhibitors (e.g. fluconazole).
In one of the two interaction studies with pravastatin and erythromycin a statistically significant increase in
pravastatin AUC (70%) and Cmax (121%) was observed. In a similar study with clarithromycin a statistically
significant increase in AUC (110%) and Cmax (127%) was observed. Although these changes were minor,
caution should be exercised when associating pravastatin with erythromycin or clarithromycin.
Other products: In interaction studies, no statistically significant differences in bioavailability were observed
when pravastatin was administered with acetylsalicylic acid, antacids (when given one hour prior to
pravastatin), nicotinic acid or probucol.

208. Which need witness during destruction


a) Chlorzoxazone
b) Lorazepam
c) Tramadol
d) Zolpidem
Is Chlorzoxazone a controlled substance?
Chlorzoxazone is used in the treatment of muscle spasm and belongs to the drug class skeletal muscle
relaxants. FDA has not classified the drug for risk during pregnancy. Chlorzoxazone 500 mg is not
a controlled substance under the Controlled Substances Act (CSA).

209. D.O.C for Chlamydia trachomatis (Uncomplicated Chlamydia Infection) N/A


a) Tetracycline
b) Erythromycin
c) Metronidazole
d) SMX-TMP
210. Patient on Metformin & Gliclazide. Drinks 2 cup of alcohol daily and double on weekend.
Find main danger action?
a) Alcohol C.I with Gliclazide
b) Alcohol C.I. With Metformin
c) Alcohol will cause acute hypoglycemia on weekend
d) Alcohol increase hepatic glucose

211. What is the most suitable action for pharmacist stay alone in his pharmacy and have strong
belief against dispensing contraception if a lady asked for OC?
a) Refuse to dispense
b) Forget his belief and dispense OC
c) Send patient to other pharmacy
d) Tell her it is not available

212. Which case we can check after 48 hours (false question: de escalation from IV to oral after
48 hrs is with pyelonephritis)
a) Hyperthyroidism
b) Pyelonephritis
c) Cellulitis
d) Atrial fibrillation

213. How to increase pharmacy profit or how to improve profitability:


a) Increase sale
b) Increase inventory
c) Increase mark up
d) Decrease expenditure
e) Pharmacy
f) Increase local advertisement
g) Decrease staff
h) Increase working hours

214. Pt have symptoms of overactive bladder taking oxybutynin, stopped after 3 months why?
a) Frequent urination increased
b) Symptoms controlled
c) Incontinence still there
If the question: stopped after 3 months, what is your concern? Frequency urination increased.
Drugs (see table Drugs Used to Treat Incontinence) should supplement, not replace, behavioral changes. The
most commonly used are oxybutynin and tolterodine; both are anticholinergic and antimuscarinic and are
available in extended-release forms that can be taken orally once a day. Oxybutynin is available as a skin
patch that is changed twice a week as well as topical gels that are applied to the skin daily.
Newer drugs with anticholinergic and antimuscarinic properties include solifenacin and darifenacin, which
are taken orally once a day, and trospium, which is taken once or twice a day. Drugs may be required to
suppress urgency symptoms due to detrusor overactivity (hyperactivity) with impaired contractility. Drugs
with a rapid onset of action (eg, immediate-release oxybutynin) can be used prophylactically if incontinence
occurs at predictable times. Combinations of drugs may increase both efficacy and adverse effects, possibly
limiting this approach in older patients. OnabotulinumtoxinA is administered via cystoscopic injection into
the detrusor muscle and is useful in treating urge incontinence refractory to other treatments in patients
with neurologic causes (eg, multiple sclerosis, spinal cord dysfunction).

215. Dementia patient has been started his Initial treatment medication, what to council?
a) Use Gingko Biloba, it may help more
b) If one drug doesn’t work then none will work from the same class
c) Goal is to decrease repetition of questions
216. A lady who came from Asia & she is on Ethambutol+ Rifampin + Pyrazinamide, what
should be monitored
a) ECG with Pyrazinamide
b) Liver function with Rifampin
c) Serum uric acid Rifampin
d) Ophthalmologic examination with Ethambutol

217. Same lady had neurological disorders, this may be linked to


a) Pyrazinamide
b) Malnutrition
c) Ethambutol
218. Pharmacy and Therapeutics Committee (P&TC) refuses to consider a new drug because of
the lack of safety trials on it. They are following
a) Confidentiality
b) Non-male efficiency
c) Autonomy
d) Paternalism

219. Acetaminophen toxicity in child (K-type)


a) Refer for hepatic check
b) Cause CNS effect
c) Give N-acetyl cysteine

220. 5 years Child with diarrhea for the last 36 hours/ what to do
a) Loperamide
b) Bismuth subsalicylate
c) Refer
d) ORS (Oral Rehydration Solution)
Severe diarrhea requires fluid and electrolyte replacement to correct dehydration, electrolyte imbalance,
and acidosis. Parenteral fluids containing sodium chloride, potassium chloride, and glucose are generally
required. Salts to counteract acidosis (sodium lactate, acetate, bicarbonate) may be indicated if serum
bicarbonate is < 15 mEq/L (< 15 mmol/L). An oral glucose-electrolyte solution can be given if diarrhea is not
severe and nausea and vomiting are minimal. Oral and parenteral fluids are sometimes given
simultaneously when water and electrolytes must be replaced in massive amounts (eg, in cholera).

221. Physician prescribed Meperidine for


Pregnant lady just had a C-section to
control her pain. After 3 day she told the
nurse about restless legs & that she had
muscle twitches. What is the reason for
her symptoms?
a) The oxytocin delayed effect of the
surgery
b) The delayed effect of the epidural
after the surgery
c) Neurotoxicity of Mepiridine-
muscle twitches
d) Long bed resting (3 days) cause of
RLS

222. A patient coming to ask for multivitamin, what is the main reason that he may need vit. D?
a) Depression
b) Diabetes Mellitus
c) Liver impairment (activation of vitamin D)
d) Low fat in diet
What causes a vitamin D deficiency? A deficiency in vitamin D can result from inadequate exposure to
sunlight, inefficient production in the skin, not enough vitamin D in your diet, and health conditions that can
affect it including, gastrointestinal disorders, renal diseases, and liver diseases
Vitamin D is a prohormone with several active metabolites that act as hormones. Vitamin D is metabolized
by the liver to 25(OH)D (calcifediol, calcidiol, 25-hydroxycholecalciferol, or 25-hydroxyvitamin D), which is
then converted by the kidneys to 1,25-dihydroxyvitamin D (1,25-dihydroxycholecalciferol, calcitriol, or active
vitamin D hormone). 25(OH)D, the major circulating form, has some metabolic activity, but 1,25-
dihydroxyvitamin D is the most metabolically active. The conversion to 1,25-dihydroxyvitamin D is regulated
by its own concentration, parathyroid hormone (PTH), and serum concentrations of calcium and phosphate.
223. Patient refuse to take his statin RX and Pharmacist told the doctor. Which ethical principal
pharmacist will uphold? N/A
a) Paternalism
b) Autonomy
c) Veracity
d) Justice

224. Doctor wants to give strontium for osteoporosis. Where to look for?
a) CPS
b) Di caprio pharmacotherapy
c) Martindale
d) Pub med

225. Which antacid could be used


with renal impairment patient (CrCl <
30)?
a) Phosphate formulation
b) Calcium Carbonate
formulation
c) Magnesium-containing antacid
d) Aluminium-containing antacid

226. Community acquired pneumonia patients with Risk Class IV (PSI score is 119)
hospitalized for treatment. Why would you give IV? Severely ill or high mortality
a) High absorption
b) Patient with class IV should take IV
c) IV is faster that PO
d) Oral is less effective in such patient with high mortality rate

227. Patient on sildenafil 25mg and Terazosin. Why do you contact the doctor?
a) To increase Sildenafil dose to 50 mg
b) Serious drug-drug problem
c) To add PPI for GIT side effects
d) To space at least 4 hours

228. Pt taking Efavirenz / Emtricitabine / Tenofovir + SMX-TMP. You can council about all
EXCEPT?
a) Take all with or without food (Tenofovir is taken with food)
b) You can take all anticancer with SMX-TMP
c) Nausea may continue for days – weeks, but resolves over time with good adherence
d) Take before bedtime to minimize side effects
The recommended dose of Tenofovir disoproxil for the treatment of HIV or for the treatment of chronic
hepatitis B is 245 mg (one tablet) once daily taken orally with food.

229. Poor patient comes to your pharmacy to fill a Rx of his son and after that he asked for
salbutamol inhaler for himself, but he doesn’t have a RX
a) Advise him to apply for social welfare
b) Say no to him
c) Dispense 1 inhaler as emergency refill & record on his sons’ insurance
d) Try to get a free sample for him
Another version:
Patient with asthma does not have an insurance, his kid does and his mother is responsible for
it, he asks if you can give him his medication and charge it on the child insurance
a) You refuse to dispense it*
b) Dispese it as once' emergency case
c) You have to talk to the mother first
d) You dispense it

230. Which is common side effect for Finasteride


a) Decrease INR
b) Increase TG
c) Impotence
d) Prostatitis

231. Which group induce cholestatic jaundice as side effect?


a) Penicillin
b) Estrogen
c) Macrolides
d) Cephalosporin

232. Native valve Endocarditis caused by


coagulase-negative or positive
a) Beta-hemolytic
b) Streptococci
c) Staphylococcus
d) S. Epidermis
233. Which one of the following may be especially important in the above patient monitoring?
a) Zopiclone for sleep & mood changes
b) FeSo4 for serum ferritin & saturated Iron and blood Iron
c) Amlodipine 10mg QD & renal failure

234. Pt. has asthma drink alcohol and smokes.


What makes his condition worse?
a) Smoking
b) Alcohol
c) Regular exercise
d) Using a dehumidifier

235. Preparation that not need preservative is?


a) Ear drops
b) Large volume parenteral (TBN)
c) Eye drops
d) Nasal spray

236. Hair dresser lady with osteoporosis. Her workplace is 2 km from home. What is your
advice for her regarding osteoporosis?
a) Swimming is important to bone health
b) Take complete rest
c) Try to walk to work, Weight-bearing exercises is important
d) Avoid overweight, Obesity considered as risk factor
237. Doctor wants an advice for Hypertension patient
a) Salt restriction may not be effective
b) DASH diet has more successful evidence
c) Weight-loss program is critical
d) Long term effect of low-carbohydrate diet is very beneficial
Nonpharmacologic Choices
- All individuals should be advised about a healthy lifestyle to prevent or control hypertension and
cardiovascular disease.
- Weight loss of 4 kg or more if overweight (target BMI: 18.5–24.9 kg/m2; waist circumference <102
cm in men and <88 cm in women).
- Healthy diet high in fresh fruits, vegetables, soluble fibre and low-fat dairy products, low in saturated
fats and sodium, e.g., DASH eating plan in Your Guide to Lowering Pressure on the NIH website.
- Consider sodium intake target of <2000 mg (88 mmol) per day.
- Increase dietary potassium intake (e.g., fruit and vegetable component of DASH eating plan) if the
patient is not at risk of hyperkalemia. Risk factors include renin-angiotensin inhibitors or other
agents that can increase potassium, chronic kidney disease and serum potassium >4.5 mmol/L.
- Regular, moderate-intensity cardiorespiratory physical activity for 30–60 minutes on most days.
- Low-risk alcohol consumption (0–2 drinks/day or ≤10 drinks/week for women; 0–3 drinks/day or ≤15
drinks/week for men).
- Smoke-free environment.

238. Pt has no bowel moments for 3 days. His profile may include
a) Levothyroxine
b) Diltiazem
c) Amlodipine
d) Esomeprazole
Monograph: Constipation, dyspepsia, gastric pain, nausea

239. Pt comes to ER with toxicity of anticholinergic herb, he may suffer from all EXCEPT:
a) Miosis
b) Dry mouth
c) Blurred vision
d) Hyperpyrexia
Remember: Atropine eye drops cause mydriasis

240. All are correct regarding diabetic foot council EXCEPT?


a) Check your feet every day
b) Wash your feet daily with warm water
c) Spend more time resting to decrease heel pain
d) Do not cross your legs when you are sitting
Prevention of Diabetic Foot Infections
Prevention of diabetic foot infections involves local and systemic factors. Interventions include care of
the feet, callus paring, glucose control and treatment of medical comorbidities, including smoking
cessation. Correcting glucose control may help prevent progression of diabetic complications, aid the
resolution of any existing infection and promote wound healing.
Advise patients with diabetes who have peripheral neuropathy to inspect the feet and shoes daily, have a
foot examination by a health-care provider at least 4 times per year, avoid walking barefoot and use
appropriately fitted footwear with soft protective insoles. Advise patients to have regular nail care
(cutting nails straight across) and avoid the application of heat or chemicals to the feet. Individuals who
smoke should be encouraged to stop smoking.
To prevent infection, promote healing of noninfected ulcers (see Nonpharmacologic Choices: Wound
Care). An open ulcer on the foot of a person with diabetes does not require antimicrobial treatment
unless the ulcer is clinically infected (erythema, edema, purulent discharge, new or intensified pain,
systemic signs such as fever, chills, rigors or dysglycemia). In addition to contributing to the increasing
rates of antimicrobial resistance, the use of these agents for clinically noninfected wounds could lead to
adverse outcomes such as allergic reactions, antibiotic-associated diarrhea, Clostridium difficile–
associated disease, taste perversion and painful peripheral neuropathy (with metronidazole).
Consult with a surgeon who has experience in foot and ankle surgery if corrective surgery for treatment
of morphologic abnormalities is being considered.
Therapeutic Tips
- Examine an ulcer using an instrument such
as forceps. If bone is palpated at the base
of a diabetic foot or ankle ulcer,
osteomyelitis is present unless proven
otherwise.
- Foot baths are not recommended because
they may lead to maceration of the skin.
- In the absence of a previous history of foot
ulcer, open wound or current systemic
signs of infection, a red, warm, swollen
foot may be a sign of early Charcot
arthropathy.
- Biopsy or aspiration of joints in Charcot
arthropathy could create risk of iatrogenic
infection; if systemic signs of infection,
such as fever, chills, rigors or poor glucose
control, are absent, consider the risk
versus benefit of invasive tests that could
introduce infection.
241. Patient with Estimated Glomerular Filtration Rate 40 mL/min had intitated treatment for
albuminuria. Monitoring Frequency should be repeated in
a) 4-6 weeks
b) 2-3 months
c) 2-4 weeks
Microalbuminuria is albumin excretion persistently between 30 and 300 mg/day (20 to 200 micrograms
[mcg]/min); lesser amounts are considered within the range of normal, and amounts > 300 mg/day (> 200
mcg/min) are considered overt proteinuria. Use of the urine albumin/urine creatinine ratio is a reliable and
more convenient screening test because it avoids timed urine specimens and correlates well with 24-hour
values. A value > 30 mg/g (> 0.03 mg/mg) suggests microalbuminuria. The reliability of the test is best when
a midmorning specimen is used, vigorous exercise is avoided before the test, because vigorous exercise can
cause transient dipstick positivity for protein, and unusual creatinine production (in cachectic or very
muscular patients) is not present. Microalbuminuria can occur in all of the following:
 Diabetes mellitus & Hypertension
 Renal allograft dysfunction
 Preeclampsia
 Urinary tract infection & Chronic kidney disease
Microalbuminuria is an early stage of diabetic kidney disease in both type 1 and 2 diabetes; the progression
of renal disease is more predictable in type 1 than 2 disease. Microalbuminuria is a risk factor for
cardiovascular disorders and early cardiovascular mortality independent of diabetes or hypertension.
Sulfosalicylic acid (SSA) test strips can be used to detect protein other than albumin (eg, immunoglobulins in
multiple myeloma) when dipstick urine tests are negative; urine supernatant mixed with SSA becomes turbid
if protein is present. The test is semiquantitative with a scale of 0 (no turbidity) to 4+ (flocculent
precipitates). Readings are falsely elevated by radiopaque contrast agents.
Ketones spill into urine with ketonemia, but use of test strips to measure urinary ketones is no longer widely
recommended because they measure only acetoacetic acid and acetone, not beta-hydroxybutyric acid. Thus,
a false-negative result is possible even without an exogenous cause (eg, vitamin C, phenazopyridine, N-
acetylcysteine); direct measurement of serum ketones is more accurate. Ketonuria is caused by endocrine
and metabolic disorders and does not reflect renal dysfunction.

242. What is the most frequent side effects with Baclofen?


a) Weight gain
b) Abnormal hepatic function
c) Palpitation
d) Somnolence & Drowsiness  do not stop abruptly (very commen)

243. In clinical trials, double blind crossover means?


a) A group doesn’t know but examiner does
b) Both groups take same medication all times
c) Both groups take different medication at different times
d) At one point all subjects switch from an active substance to a placebo or vice versa.
244. What is the meaning of P<0.005?
a) Values are insignificant
b) Values are significant
c) Values have no certainty
d) Values are highly significant

245. Which can cause hyperglycemia?


a) High alcohol consumption before dinner
b) Long acting insulin without short acting during the dinner
c) Exercise before breakfast
d) 0.1 ml of 100 units instead of 1 ml of Insulin

246. Pt using Clotrimazole as antifungal. What NOT recommended in recurrence?


a) Make sure shoes fit correctly
b) Change to Tolnaftate
c) Avoid walking barefoot
d) Keep feet clean, cool and dry
247. PT with swelling leg (edema) come to pharmacy & wants to sit right now. He may be on?
a) Nifedipine
b) Ramipril
c) Metformin
d) Omeprazole

248. Patient doesn’t want to take a medication after you tried to convince him many times, what
principle to follow?
a) Confidentiality
b) Beneficence
c) Autonomy
d) Justice

249. NR is 50 years old male. He was diagnosed with depression. His medical history includes
sexual dysfunction, Dyslipidemia and hypertension. His profile includes Atorvastatin,
Proparanolol, hydrochlorothiazide& Sildenafil. What is DOC for his case
a) Mirtazapine
b) Bupropion
c) Fluoxetine
d) Citalopram

250. Which drug worse his depression?


a) Sildenafil
b) Propranolol
c) Atorvastatin

251. Now NR suffer from an acute gout attack what is the drug of choice in his case
a) Indomethacin  not in hypertension or kidney failure
b) Acetaminophen  not used in Gout
c) Colchicine  close monitoring due to atorvastatin
d) Corticosteroid  not in hypertension or dyslipidemia

252. A patient has severe Raynaud’s and Angina pectoris. All of the following medications will
benefit his angina, EXCEPT:
a) Felodipine
b) Diltiazem
c) Nadolol
d) Nitroglycerin
All the given meds will benefit his angina (wording issue), but NOT all of them will fit his/her condition since
BB would worsen primary Raynaud's syndrome.

SUPERGIFT Q! (5-second type)


253. What causes Torsade de pointes?
a) Hypokalemia (treat with magnesium)

254. To measure the effectiveness of spironolactone therapy in ascitis measure:


a) Weight  improvement after 2 weeks wt loss 0.5 – 1 kg /day] from 1—1.5 kg
b) Abdominal growth
c) Electrolytes
d) Creatinine

255. All are lab tests for anemia except:


a) Hematocrit
b) Albumin bound ferric----
c) MCV

DIAGNOSIS
 Complete blood count (CBC), serum iron, iron-binding capacity, serum ferritin, transferrin saturation,
reticulocyte count, red cell distribution width (RDW), and a peripheral blood smear
 Rarely bone marrow examination
Iron deficiency anemia is suspected in patients with chronic blood loss or microcytic anemia, particularly if
pica is present. In such patients, a CBC, serum iron and iron-binding capacity, and serum ferritin and
reticulocyte count are obtained (see table Typical Serum Values for Iron, Iron-Binding Capacity, Ferritin, and
Transferrin Saturation).
Iron and iron-binding capacity (and transferrin saturation) are measured because their relationship is
important. Various tests exist; the range of normal values relates to the test used and varies from
laboratory to laboratory. In general, normal serum iron is 75 to 150 mcg/dL (13 to 27 micromol/L) for men
and 60 to 140 mcg/dL (11 to 25 micromol/L) for women; total iron-binding capacity is 250 to 450 mcg/dL (45
to 81 micromol/L) and transferrin saturation is 20 to 50%. Serum iron level is low in iron deficiency and in
many chronic diseases and is elevated in hemolytic disorders and in iron-overload syndromes. The iron-
binding capacity increases in iron deficiency, while the transferrin saturation decreases.
Serum ferritin levels closely correlate with total body iron stores. The range of normal in most laboratories is
30 to 300 ng/mL (67.4 to 674.1 pmol/L), and the mean is 88 ng/mL (197.7 pmol/L) in men and 49 ng/mL
(110.1 pmol/L) in women. Low levels (< 12 ng/mL (27 pmol/L)) are specific for iron deficiency. However,
ferritin is an acute-phase reactant, and levels increase in inflammatory and infectious disorders (eg,
hepatitis), and neoplastic disorders (especially acute leukemia, Hodgkin lymphoma, and gastrointestinal
tract tumors). In these settings, a serum ferritin up to 100 ng/mL remains compatible with iron deficiency.
The reticulocyte count is low in iron deficiency. The peripheral smear generally reveals hypochromic red cells
with significant anisopoikilocytosis, which is reflected in a high red cell distribution width (RDW).
The most sensitive and specific criterion for iron-deficient erythropoiesis is absent bone marrow stores of
iron, although a bone marrow examination is rarely needed.

256. All will interact with ciprofloxacin Except


a) Phenytoin
b) Caffeine
c) Ferrous sulfate
d) Statin
Effects of other products on ciprofloxacin:
Drugs known to prolong QT interval
Ciprofloxacin, like other fluoroquinolones, should be used with caution in patients receiving drugs known to
prolong QT interval (e.g. Class IA and III antiarrhythmics, tricyclic antidepressants, macrolides, antipsychotics)
Chelation Complex Formation
The simultaneous administration of ciprofloxacin (oral) and multivalent cation-containing drugs and mineral
supplements (e.g. calcium, magnesium, aluminium, iron), polymeric phosphate binders (e.g. sevelamer or
lanthanum carbonate), sucralfate or antacids, and highly buffered drugs (e.g. didanosine tablets) containing
magnesium, aluminium, or calcium reduces the absorption of ciprofloxacin. Consequently, ciprofloxacin
should be administered either 1-2 hours before or at least 4 hours after these preparations. The restriction
does not apply to antacids belonging to the class of H2 receptor blockers.
Food and Dairy Products
Dietary calcium as part of a meal does not significantly affect absorption. However, the concurrent
administration of dairy products or mineral-fortified drinks alone (e.g. milk, yoghurt, calcium-fortified
orange juice) with ciprofloxacin should be avoided because absorption of ciprofloxacin may be reduced.
Probenecid
Probenecid interferes with renal secretion of ciprofloxacin. Co-administration of probenecid and
ciprofloxacin increases ciprofloxacin serum concentrations.
Metoclopramide
Metoclopramide accelerates the absorption of ciprofloxacin (oral) resulting in a shorter time to reach
maximum plasma concentrations. No effect was seen on the bioavailability of ciprofloxacin.
Omeprazole
Concomitant administration of ciprofloxacin and omeprazole containing medicinal products results in a
slight reduction of Cmax and AUC of ciprofloxacin.
Agomelatine
In clinical studies, it was demonstrated that fluvoxamine, as a strong inhibitor of the CYP450 1A2 isoenzyme,
markedly inhibits the metabolism of agomelatine resulting in a 60-fold increase of agomelatine exposure.
Although no clinical data are available for a possible interaction with ciprofloxacin, a moderate inhibitor of
CYP450 1A2, similar effects can be expected upon concomitant administration.
Tizanidine
Tizanidine must not be administered together with ciprofloxacin (see section 4.3). In a clinical study with
healthy subjects, there was an increase in serum tizanidine concentration (Cmax increase: 7-fold, range: 4 to
21-fold; AUC increase: 10-fold, range: 6 to 24-fold) when given concomitantly with ciprofloxacin. Increased
serum tizanidine concentration is associated with a potentiated hypotensive and sedative effect.
Methotrexate
Renal tubular transport of methotrexate may be inhibited by concomitant administration of ciprofloxacin,
potentially leading to increased plasma levels of methotrexate and increased risk of methotrexate-
associated toxic reactions. The concomitant use is not recommended (see section 4.4).
Theophylline
Concurrent administration of ciprofloxacin and theophylline can cause an undesirable increase in serum
theophylline concentration. This can lead to theophylline-induced side effects that may rarely be life
threatening or fatal. During the combination, serum theophylline concentrations should be checked and the
theophylline dose reduced as necessary (see section 4.4).
Other xanthine derivatives
On concurrent administration of ciprofloxacin and caffeine or pentoxifylline (oxpentifylline), raised serum
concentrations of these xanthine derivatives were reported.
Phenytoin
Simultaneous administration of ciprofloxacin and phenytoin may result in increased or reduced serum levels
of phenytoin such that monitoring of drug levels is recommended.
Cyclosporin
A transient rise in the concentration of serum creatinine was observed when ciprofloxacin and cyclosporin
containing medicinal products were administered simultaneously. Therefore, it is frequently (twice a week)
necessary to control the serum creatinine concentrations in these patients.
Vitamin K antagonists
Simultaneous administration of ciprofloxacin with a vitamin K antagonist may augment its anti-coagulant
effects. The risk may vary with the underlying infection, age and general status of the patient so that the
contribution of ciprofloxacinto the increase in INR (international normalised ratio) is difficult to assess. The
INR should be monitored frequently during and shortly after co-administration of ciprofloxacin with a
vitamin K antagonist (e.g., warfarin, acenocoumarol, phenprocoumon, or fluindione).
Duloxetine
In clinical studies, it was demonstrated that concomitant use of duloxetine with strong inhibitors of the
CYP450 1A2 isozyme such as fluvoxamine, may result in an increase of AUC and Cmax of duloxetine.
Although no clinical data are available on a possible interaction with ciprofloxacin, similar effects can be
expected upon concomitant administration (see section 4.4).
Ropinirole
It was shown in a clinical study that concomitant use of ropinirole with ciprofloxacin, a moderate inhibitor of
the CYP450 1A2 isozyme, results in an increase of Cmax and AUC of ropinirole by 60% and 84%, respectively.
Monitoring of ropinirole-related side effects and dose adjustment as appropriate is recommended during
and shortly after co-administration with ciprofloxacin (see section 4.4).
Lidocaine
It was demonstrated in healthy subjects that concomitant use of lidocaine containing medicinal products
with ciprofloxacin, a moderate inhibitor of CYP450 1A2 isozyme, reduces clearance of intravenous lidocaine
by 22%. Although lidocaine treatment was well tolerated, a possible interaction with ciprofloxacin
associated with side effects may occur upon concomitant administration.
Clozapine
Following concomitant administration of 250 mg ciprofloxacin with clozapine for 7 days, serum
concentrations of clozapine and N-desmethylclozapine were increased by 29% and 31%, respectively. Clinical
surveillance and appropriate adjustment of clozapine dosage during and shortly after coadministration with
ciprofloxacin are advised (see section 4.4).
Sildenafil
Cmax and AUC of sildenafil were increased approximately twofold in healthy subjects after an oral dose of
50 mg given concomitantly with 500 mg ciprofloxacin. Therefore, caution should be used prescribing
ciprofloxacin concomitantly with sildenafil taking into consideration the risks and the benefits.
Zolpidem
Co-administration of ciprofloxacin may increase blood levels of zolpidem, concurrent use is not
recommended.

257. Which cause pseudo biliary lathis:


a) Amoxicillin
b) Ceftriaxone
c) Tobramycin
d) Vancomycin
Biliary pseudolithiasis is a rare ADR of ceftriaxone where the antimicrobial complexes with Ca and looks like
gallstones.
Biliary lithiasis
When shadows are observed on sonograms, consideration should be given to the possibility of precipitates
of calcium ceftriaxone. Shadows, which have been mistaken for gallstones, have been detected on
sonograms of the gallbladder and have been observed more frequently at ceftriaxone doses of 1 g per day
and above. Caution should be particularly considered in the paediatric population. Such precipitates
disappear after discontinuation of ceftriaxone therapy. Rarely precipitates of calcium ceftriaxone have been
associated with symptoms. In symptomatic cases, conservative nonsurgical management is recommended
and discontinuation of ceftriaxone treatment should be considered by the physician based on specific benefit
risk assessment.
Biliary stasis
Cases of pancreatitis, possibly of biliary obstruction aetiology, have been reported in patients treated with
Ceftriaxone. Most patients presented with risk factors for biliary stasis and biliary sludge e.g. preceding
major therapy, severe illness and total parenteral nutrition. A trigger or cofactor of Ceftriaxone-related
biliary precipitation cannot be ruled out.
Renal lithiasis
Cases of renal lithiasis have been reported, which is reversible upon discontinuation of ceftriaxone. In
symptomatic cases, sonography should be performed. Use in patients with history of renal lithiasis or with
hypercalciuria should be considered by the physician based on specific benefit risk assessment.

258. AB is an immigrant from south Asia. She is malnourished and she has pulmonary and
extra pulmonary tuberculosis and she has fever. She is currently taking INH, rafampin,
ethambutol, pyrazinamide and pyridoxine for 9 months. Why is she more susceptible of getting
neuropathy?
a) She is malnourished
b) She is a female
c) She has extra pulmonary TB
d) She has fever

259. Antibiotic used for prophylaxis in appendectomy


a) Cefatriaxone
b) Cefazidine
c) Ciprofloxacin
d) Cefuroxime
e) Cefazolin
Merck Manual: Appendectomy should be preceded by IV antibiotics. Third-generation cephalosporins are
preferred. For nonperforated appendicitis, no further antibiotics are required. If the appendix is perforated,
antibiotics should be continued until the patient’s temperature and white blood cell count have normalized
or continued for a fixed course, according to the surgeon’s preference. If surgery is impossible, antibiotics—
although not curative—markedly improve the survival rate. Although several studies of nonoperative
management of appendicitis (ie, using antibiotics alone) have shown high rates of resolution during the
initial hospitalization, a significant number of patients have a recurrence and require appendectomy during
the following year. Thus, appendectomy is still recommended.
260. Immunization to be given every 10 years are: (more than one correct answer was given)
a) Tetanus
b) Influenza vaccine
c) Hepatitis B vaccine
261. Clarithromycin auxillary label?
I. Shake well
II. Take full course
III. Keep in fridge
 Nitrofurantoin to be taken with food increase absorption 45% May 2011
 Sertraline to be taken with food as this will increase its effect* 40% (Nov2011)
 Cefuroxim to be taken with food (Nov2011) Nov 2010
 Zeprasidon to be taken with food. Testesterone to be taken with food
 Clarithromycin to be taken with food

262. Which one needs adjustment in renal failure


a) Cotrimoxazole (all sulfas)
b) Moxiflox
c) Doxyxycline
d) Azithromycin (clarithro)
e) Clindamycin
Renal impairment: The elimination half-life of trimethoprim is increased by a factor of 1.5-3.0 when the
creatinine clearance is less than 10 mL/minute. When the creatinine clearance falls below 30 mL/min the
dosage of Co-Trimoxazole should be reduced.
Hepatic impairment: Caution should be exercised when treating patients with severe hepatic parenchymal
damage as there may be changes in the absorption and biotransformation of trimethoprim and
sulfamethoxazole.
Elderly patients: In elderly patients, a slight reduction in renal clearance of sulfamethoxazole but not
trimethoprim has been observed.

263. Fluoroquinolone class effect


a) Alopecia
b) Glycemic abnormalities
c) Nephrotoxity
d) Thrombocytopenia (all antibiotics)
e) Rhabdomyolosis

264. To make an elderly pt who has difficulty falling asleep to sleep quickly without hangover
a) Diazepam
b) Flurazepam
c) Oxazepam
d) Zopiclone & Temazepam
e) Triazolam
265. Pt received IV dose of drug X then blood samples were taken & drug blood conc were
At 10 00 60 mmol/L, at 22 00 45 mmol/L. Calculate T1/2 of drug x
a) 18
b) 24
c) 29
lin C=lin Cₒ-kt lin 45=lin 60-k X 12 k= 0.02397 t½ = 0693/0.02397=28.9

266. The pore size of hepa filter used for strelization of a parentral drug is?
a) 0.22mm
b) 0.33mm
Hepa filter has a pore size of 0.33 microns. Whereas filters used in aspectic techniques have a pore size of
0.22 microns

267. The easiest dosage form for preparation an interchangeable drug is:
a) Suspension
b) Solution
c) sustained release tablets
d) IV

268. When to disclose patient information or When does the pharmacist have right to break
confidentiality?
a) Patient intent to harm himself
b) Third party requires information
c) Patient has cognitive impairment
d) Pt under the legal age
e) HIV patient

269. An HIV patient got infected by having sex with a drug abuser, so; what to tell his wife:
I. Tell her how he got the infection
II. She has to test for HIV
III. She has to discuss the matter with her husband

270. Girl was raped 2 days, she has a Rx for Plan B, she didn’t tell the physcian that she was
raped as she thought it was none of his business so what the pharmacist has to do:
I. Dispense it
II. Give her the no. of the hot line for sexually assaulted women
III. Report the incident to the police as the is obligate
Dispense + contacts of helpline (if she wishes).
271. Which reference u will use if you want to know the latest guidelines published and
protocol
management in certain disease:
a) e-CPS
b) AHFS
c) Medline (pubmed)

272. Vincristine side effects include:


a) Peripheral Neuropathy + not injected
epidural
b) Kidney toxicity
c) Neuropathy
d) Liver damage
e) Nephrotoxicity

273. 70 yrs old lady taking medication for cancer, since yesterday her body temp has been
raised above normal. What is appropriate action?
a) Give acetaminophen
b) Non pharm
c) Send to emergency stat
d) Tell her it is normal, it will resolve without medication
Fever> 38° C in a patient with neutropenia is an emergency. Evaluation should include immediate chest x-ray
and cultures of blood, sputum, urine, stool, and any suspect skin lesions. Examination includes possible
abscess sites (eg, skin, ears, sinuses, peri-rectal area), skin and mucosa for presence of herpetic lesions,
retina for vascular lesions suggestive of infectious emboli, and catheter sites. Rectal examination and use of
a rectal thermometer should be avoided.
Febrile neutropenic patients should receive broad-spectrum antibiotics chosen on the basis of the most likely
organism.
Monotherapy with an IV broad-spectrum antipseudomonal beta-lactam (ceftazidime, imipenem/ cilastatin,
meropenem or piperacillin/ tazobactam) is suitable for the treatment of febrile neutropenic episodes
In neutropenic patients with a suspected bacterial infection, continue antibacterial therapy until patients
are no longer neutropenic (neutrophils ≥0.5 × 109/L) and are afebrile for ≥48 hours.
Granulocyte (G-CSF) and granulocyte-macrophage colony-stimulating factors (GM-CSFs) (e.g., filgrastim,
pegfilgrastim) may decrease the incidence and duration of neutropenia after chemotherapy.

274. What is DOC of anticipatory nausea and vomiting?


a) Lorazepam.
b) Dexamethasone
c) Nabilone
275. Inventory turnover rate ABC pharmacy ANS AVERAGE TOR 10

276. What is most contamination of laminar flow hood?


a) Human
b) Sterile product
c) Disinfectant
d) Gloves

277. All these lab tests will monitor for Alzheimer patient, Except.
a) Folic Acid
b) Vitamin B12
c) Mini Mental State Exam
d) Cognitive Impairment
e) Echo Encephalogram
Investigations
 Careful history with attention to memory impairment and potentially reversible causes, e.g.,
medications, vitamin B12 deficiency, hypothyroidism, depression.
 Q. Cognitive impairment can be assessed using the Montreal Cognitive Assessment (MoCA) or Mini-
Mental State Examination (MMSE).
 Q. Functional disability is measured with tools such as the Disability Assessment for Dementia (DAD)
or the Functional Assessment Staging Tool (FAST).
Medication history is important to rule out drug-induced cognitive impairment. Anticholinergic side effects
of medications can lead to cognitive impairment. Q. A few examples of drug classes commonly associated
with anticholinergic effects are:
 Antiemetics/antivertigo agents, e.g., dimenhydrinate, promethazine, scopolamine
 Antihistamines, e.g., diphenhydramine, hydroxyzine
 Antimuscarinics, e.g., darifenacin, fesoterodine, oxybutynin, solifenacin, tolterodine
 Antipsychotics, e.g., chlorpromazine, clozapine, olanzapine
 TCAs, e.g., amitriptyline, clomipramine, desipramine, doxepin, imipramine, nortriptyline, paroxetine
Laboratory tests: Q. CBC, electrolytes, kidney function, TSH, vitamin B12, calcium, blood glucose.
In order to assess for prodromal dementia or “very early Alzheimer disease without dementia,” researchers
have attempted to use plasma and cerebrospinal fluid biomarkers (e.g., amyloid beta, tau protein);
however, the results from these studies are rarely applicable outside of the research setting.
Neuroimaging (usually CT head scan) if: < 60 years of age, new onset, rapid progression, post-head injury,
focal or lateralizing signs, history of cancer, use of anticoagulants, early urinary incontinence and gait
disorder, unusual cognitive symptoms.

278. After three months above patient face side effects as edema, wt gain and leg swollen,
what drug would cause these side effects?
a) Glyburide
b) Metformin
c) Insulin
d) Pioglitazone
e) Acarbose
Pioglitazone Monograph:
Pioglitazone can cause fluid retention, which may exacerbate or precipitate heart failure. When treating
patients who have at least one risk factor for development of congestive heart failure (e.g. prior myocardial
infarction or symptomatic coronary artery disease or the elderly), physicians should start with the lowest
available dose and increase the dose gradually. Patients should be observed for signs and symptoms of
heart failure, weight gain or oedema; particularly those with reduced cardiac reserve.
There have been post-marketing cases of cardiac failure reported when pioglitazone was used in
combination with insulin or in patients with a history of cardiac failure. Patients should be observed for
signs and symptoms of heart failure, weight gain and oedema when pioglitazone is used in combination
with insulin. Since insulin and pioglitazone are both associated with fluid retention, concomitant
administration may increase the risk of oedema.
Post marketing cases of peripheral oedema and cardiac failure have also been reported in patients with
concomitant use of pioglitazone and nonsteroidal anti-inflammatory drugs, including selective COX-2
inhibitors. Pioglitazone should be discontinued if any deterioration in cardiac status occurs.
A cardiovascular outcome study of pioglitazone has been performed in patients under 75 years with type 2
diabetes mellitus and pre-existing major macrovascular disease. Pioglitazone or placebo was added to
existing antidiabetic and cardiovascular therapy for up to 3.5 years. This study showed an increase in
reports of heart failure; however, this did not lead to an increase in mortality in this study.
279. Lady wants to go carribian, her wt is
73 kg height 5feet 6inches, one of her
friends told her to take orlistat tab for wt
reduction. what is following statement is
true
a) She is not eligible to take orlistat

280. Management of ascites patient is


using spironolactone, but it is not effective,
what you will add for relieve syndrome
a) Add furosemide
High plasma aldosterone levels in patients with ascites result in sodium and fluid retention; thus,
spironolactone (an aldosterone antagonist) is the diuretic of choice. The clearance of spironolactone and its
active metabolites are impaired in advanced cirrhosis. Full therapeutic diuretic effect can take up to 2 weeks
to be observed; adjust dose slowly in this setting.
There is evidence that starting with a combination of sprionolactone and furosemide may enhance diuresis
versus spironolactone alone. In addition, furosemide helps to control serum potassium levels. Metolazone
can be added if ascites is refractory to spironolactone and furosemide. The combination of furosemide and
metolazone can produce
profound diuresis, causing
volume depletion and
electrolyte abnormalities,
e.g., hypochloremic
metabolic alkalosis,
hypokalemia. Start with low
doses and titrate up.
Amiloride can be
substituted for
spironolactone if intolerable
side effects develop.
281. What virus transmission through fecal-oral
a) Hep-A
b) Hep-B
c) Hep-C

282. Glargine SE
a) Wt. gain
b) Lipodystrophy
c) Hyperglycemia

283. Patient had tinea pedis – having recurrent infections – counselling should include
a) Use tolfonate as prophylaxis
b) Treat for at least two weeks
c) Switch to tofonate powder
d) Keep using clotrimazole
Advise patient to:
1. Finish the recommended course of treatment to prevent recurrence, even though symptoms may
improve before the treatment course is complete.
2. Dry the feet last after showering or bathing and use a clean towel every day, to prevent
autoinoculation.
3. Prevent transmission to others by not going barefoot around the home or in public areas until the
infection is cured.

284. All can be used in treatment of hot flashes except


a) Estrogen
b) Progesterone
c) Raloxifene SE: hot flushes
d) Venlafaxine

285. 3 months infant his mother came to your pharmacy she told you that she has just
switched her baby from breast feeding to formula instead. Now, she is worry because he
did not defecate from 36 hr what you should do
a) Do not warrant treatment with mineral oil
b) Warrant treatment with mineral oil
c) warrant treatment with prune juice
d) Warrant treatment with glycerine supp.
e) Refer to doctor
286. RS is an elderly patient 75 years old who presented to your pharmacy complaining of
constipation. She usually has one bowel movement daily. Now she has bowel movement every
2-3 days. She also had MI 7 years ago and hypertension. What is your best advice for her?
a) Cascara
b) Mineral oil
c) PEG 3350
d) Sodium sulfate
e) Magnesium hydroxide
Elderly
Treatment of the older adult is often complicated by comorbidities, cognitive impairment & polypharmacy.
The prevalence of constipation increases from one-quarter of the female population at 65 years of age to
one-third by 85. The incidence of constipation is slightly lower in the elderly male population. There is a
paucity of evidence-based recommendations in the management of constipation in the elderly.
Management should be tailored to each individual's needs and expectations regardless of age or place of
residence.
Functional abilities related to mobility, following instructions, communicating needs, eating,
drinking and cognitive status must be assessed.
Fluid intake should target 1500–2000 mL daily unless fluid restrictions are imposed as in those
with heart failure. Low fluid consumption with bulk-forming laxatives can exacerbate
constipation. Dietary fibre should be targeted at 25–30 g daily, which may allow discontinuation
of laxatives and may increase the senior's well-being.
Exercise may be performed to patient's capacity; pelvic tilt, trunk rotation and leg lifts are recommended for
bedridden patients.
Medication review should rule out polypharmacy and drug-induced constipation.
Incidence of cognitive impairment, fall rate, delirium, urinary incontinence and depression have been found
higher in a study of nursing home residents with opioid-induced constipation.
Opioid-induced constipation is a risk in patients with a high burden of anticholinergic medications and
high/chronic opioid doses. Caution should be applied to avoid these medications and, if required, to keep
doses to the lowest possible effective dose. If using opioids, a laxative should be started at the beginning of
therapy.
Clients have an incrementally higher risk of constipation with each advancing state of renal failure. The level
of renal impairment must be determined prior to using laxatives. The use of saline laxatives is
contraindicated in renal and heart failure. Saline osmotics have been linked to increased death, secondary
to congestive heart failure. Limitations for use include possible multiple electrolyte abnormalities such as
hypermagnesemia, hyperphosphatemia, hypocalcemia and hypokalemia.
When used, oral saline laxatives (e.g., sodium phosphate) should be administered with sufficient water to
prevent dehydration.
Magnesium citrate is generally reserved for bowel cleansing and should be used with caution in renal
impairment. PEG, without electrolytes or lactulose, is safe and effective for use in seniors suffering from
acute or chronic constipation.
A trial comparing PEG 4000 to lactulose in clients over 70 years of age with chronic constipation showed PEG
did not affect nutrition and had few side effects; this clinical tolerance enhanced compliance and
contributed to the efficacy of PEG versus lactulose.
Also, lactulose is cleared during hemodialysis by 83.6% requiring dosage supplementation.
Stimulant laxatives (e.g., senna, bisacodyl) may cause severe cramping and electrolyte losses when used
long term.
Encourage institutions to establish an interdisciplinary team approach to prevent and manage
constipation.

287. Drug will be recall from the market due to its adverse effect on stomach, the physician
called you and asked you to order quantity from this drug because he see that stomach
bleeding is a minor effect what you should do
a) Stoke a big quantity as the doctor requested
b) Do not sell or stoke this drug since the time you received the recall order
c) Call the suppliers and order all quantity they have

288. Group A Streptococcus throat DOC & goal of treatment?


Goals of Therapy:
Provide symptomatic relief, shorten duration of symptoms & Limit spread of
GAS to close contacts.
Prevent nonsuppurative complications, e.g., Acute Rheumatic Fever ARF.
Prevent suppurative complications, e.g., peritonsillar abscess (quinsy); acute
otitis media; acute sinusitis; cervical lymphadenitis; very rare: bacteremia,
meningitis, necrotizing fasciitis.
Pharmacologic Choices
Q. 10 days of penicillins (i.e., penicillin V or amoxicillin) remains the gold standard for therapeutic duration,
especially in patients at increased risk of ARF. Short courses (i.e., ≤7 days) of penicillins are not
recommended.
Shorter courses of non-penicillin antibiotics not routinely recommended except for azithromycin.
Q. For adults, - ve RADT or - ve throat culture is sufficient to withhold antibiotics. For children & adolescents,
antibiotics can be withheld 24–48 hours while awaiting results of throat culture (if RADT - ve or not
available).

289. Used in local treatment of neuropathic pain


a) Lidocaine
b) Capsicum
Chronic Neuropathic Pain
The Canadian Pain Society and other American and European guidelines provide recommendations for the
treatment of neuropathic pain based on RCTs. There are strong recommendations for use of tricyclic
antidepressants (TCAs), gabapentinoids (gabapentin and pregabalin) and SNRIs (duloxetine and
venlafaxine).
There are weak recommendations for use of topical lidocaine, topical capsaicin, tramadol, strong opioids
and botulinum toxin (topical agents and botulinum toxin are recommended only for peripheral neuropathic
pain).
There is no evidence to support or refute a useful role for oral NSAIDs in neuropathic pain. Various
interventional treatments, such as epidural steroid injections, are offered by chronic pain clinics, although
current evidence to support the efficacy of such treatments for neuropathic pain is lacking.

290. A patient was using lorazepam for generalized anxiety disorder. Doctor changed
lorazepam to buspirone. What is pharmacist appropriate counselling to the patient.
a) Buspirone takes 3-5 weeks to show its optimum effect.
Buspirone has shown comparable efficacy to benzodiazepines with lower sedative potential and a very
low risk of abuse. Like antidepressants, it has a slow onset of action of a few weeks. However, few
studies have compared the effectiveness of buspirone to antidepressants. As clinical experience
demonstrates efficacy that is sometimes unsatisfactory, buspirone is infrequently used in practice.
291. Patient is taking Granisetron, dexamethasone. If not controlled, what should we add:
a) Dexamethasone to be changed to prednisone
b) Option with diphenhydramine
c) Option with diphenyhdramine
d) Prochlorperazine as add on

292. A mother for a one-year-old child presented to you at the pharmacy and said that her child
has fever. She said his temperature is 39 and that he does not have seizures at night. You should
recommend all of the following, EXCEPT:
a) Give him acetaminophen not more than 5 doses per day
b) Remove excess clothing
c) If you’re going to sponge, do this 30 min after antipyretic (more effective in first 30 min)
d) Ensure a good fluid intake
e) Wake him up at night to give him a dose

293. What is the gold standard to measure


body temperature for this child?
a) Axillary thermometer
b) Rectal thermometer
c) Oral thermometer
d) Tympanic thermometer
e) Temporal thermometer
294. What is a true statement to tell for the mother?
a) Most of elevated body temperature is due to bacterial infection
b) Axillary temperatures are usually lower than rectal and oral ones.
c) Water immersion might be needed if fever reached 39.5
d) The risk of high fever might lead to tremors and twitching
Rectal temperatures are higher than oral temperatures by 0.6°C
Axillary temperatures are lower than oral temperatures by 0.5–1°C

295. A patient was treated for depression and took antidepressant then she developed into
mania. Her doctor diagnosed her with bipolar disorder. What should the doctor switch her to?
a) Switch to lamotrigine
b) Switch to carbamazepine
c) Stop antidepressant and start lithium
d) Continue antidepressant and lithium together

296. A hospital pharmacist got order for a parenteral, he realized he doesn’t have all materials?
a) Obtain stock from another institute
b) Order from manufacture and wait till it arrives
c) Call dr and ask him to change the order
d) Fill order with what u have and monitor the patient
297. What is contraindication of Plan-B contraceptives:
a) Migraine with aura
b) History of bleeding
c) Pregnancy
4.3 Contraindications
Hypersensitivity to the active substance or to any of the excipients listed in section 6.1.
4.4 Special warnings and precautions for use
Emergency contraception is an occasional method. It should in no instance replace a regular
contraceptive method.
Emergency contraception does not prevent a pregnancy in every instance. If there is uncertainty about
the timing of the unprotected intercourse or if the woman has had unprotected intercourse more than
72 hours earlier in the same menstrual cycle, conception may have occurred. Treatment with
levonorgestrel following the second act of intercourse may therefore be ineffective in preventing
pregnancy. If menstrual periods are delayed by more than 5 days or abnormal bleeding occurs at the
expected date of menstrual periods or pregnancy is suspected for any other reason, pregnancy should
be excluded.
If pregnancy occurs after treatment with levonorgestrel, the possibility of an ectopic pregnancy should be
considered. The absolute risk of ectopic pregnancy is likely to be low, as levonorgestrel prevents ovulation
and fertilisation. Ectopic pregnancy may continue, despite the occurrence of uterine bleeding.
Therefore, levonorgestrel is not recommended for patients who are at risk of ectopic pregnancy (previous
history of salpingitis or of ectopic pregnancy).
Levonorgestrel is not recommended in patients with severe hepatic dysfunction.
Severe malabsorption syndromes, such as Crohn's disease, might impair the efficacy of levonorgestrel.
This medicinal product contains lactose monohydrate. Patients with rare hereditary problems of galactose
intolerance, the Lapp lactase deficiency or glucose-galactose malabsorption should not take this medicine.
After levonorgestrel intake, menstrual periods are usually normal and occur at the expected date. They can
sometimes occur earlier or later than expected by a few days. Women should be advised to make a medical
appointment to initiate or adopt a method of regular contraception. If no withdrawal bleed occurs in the
next pill-free period following the use of levonorgestrel after regular hormonal contraception, pregnancy
should be ruled out.
Repeated administration within a menstrual cycle is not advisable because of the possibility of disturbance
of the cycle.
Limited and inconclusive data suggest that there may be reduced efficacy of Levonorgestrel 1.5mg tablets
with increasing body weight or body mass index (BMI) (see section 5.1). In all women, emergency
contraception should be taken as soon as possible after unprotected intercourse, regardless of the woman's
body weight or BMI.
Levonorgestrel is not as effective as a conventional regular method of contraception and is suitable only as
an emergency measure. Women who present for repeated courses of emergency contraception should be
advised to consider long-term methods of contraception.
Use of emergency contraception does not replace the necessary precautions against sexually transmitted
diseases.
298. Pt. newly married and don’t want to take OC daily and needs to be pregnant once she
stops the contraceptive. She is obese (weight is not given), what to recommend:
a) OC pills
b) IUS (levonorgestrel)
c) Combined contraceptives patch
d) Nuva-ring
e) Medroxy-progesterone injection
299. A child took 5 times the dose of amoxicillin suspension that he is supposed to get, what
isn’t a sign of anaphylaxis that require emergency:
a) Urticarial
b) Macupapular rash
c) Shortness of breath

300. What should be done:


a) Refer to Dr.
b) Refer to ER
c) Monitor the baby
Monitor the child (how old, etc. would also help guide the answer).
With antibiotic overdoses, monitoring is usually enough.
Poison control can also be consulted by the parent for reassurance.
If someone has overdosed and has serious symptoms such as passing out or trouble breathing, call 911.
Otherwise, call a poison control center right away. Canada residents can call a provincial poison control
center. Symptoms of overdose may include: severe vomiting, persistent diarrhea, a severe decrease in the
amount of urine, or seizures.

301. Patient is bringing methadone prescription from unauthorized doctor. Her family doctor on
vacation for three weeks, what is appropriate action of pharmacist? (obsolete)
a) Refuse to fill prescription
b) Phone to her family doctor and request prescription after he arrive
c) Tell new doctor to register temporarily
d) Right away fill her prescription
In the past, physicians required exemptions to prescribe methadone, but As of May 2018, physicians do NOT
require exemptions to prescribe methadone.

302. Drug is recalled you should do all except


a) Post the recalling display in the pharmacy
b) Contact patients to whom the drug was dispensed
c) Look for the batch size and number which is affected

303. BG is asking for the emergency contra plan-B®, what will you ask her?
a) How many partners do you have?
b) Did you take oral contraceptive before?
c) When was your last menses?
304. The physician asks you about your recommendation for her hyperkalemia?
a) IV fluids
b) Salbutamol
c) Calcium gluconate
d) Na polystyrene sulfonate (mild hyperkalemia)
e) Insulin
https://www.merckmanuals.com/professional/endocrine-and-metabolic-disorders/electrolyte-
disorders/hyperkalemia?query=hyperkalemia%20treatment

305. Later on, the nurse admitted that she has mistakenly given the patient 320 mg BID
instead of 160 mg BID, she told the medical staff. Which the first action you would ensure
that it is initially done?
a) Ensure the nurse apologizes for the patient’s family member
b) Contact the manager to ensure the nurse got a disciplinary action
c) Ensure the nurse knows how to treat hyperkalemia
d) Document that incident. Transfer the nurse to another department
e) Long bed resting (3 days)
306. A patient was given Novolin ge (30/70) BID instead of Novolin ge NPH 10 Units BID.
What is the most potential side effect?
a) Postprandial hypoglycemia
b) Fasting hypoglycemia
c) Nocturnal hypoglycemia
d) Dawn phenomenon

307. To avoid the above mistake what should you do?


a) Double check while dispensing
b) Computer alert
c) Make the patient check his insulin before leaving the pharmacy
d) Put different concentration of insulin in different places

308. A pharmacist is working on a Friday shift that has been very overloaded. He’s
speaking to the pharmacy manager expressing how hectic and busy the day was; and that
there was a backlog in the prescriptions. He’s asking the pharmacist manager whether it’s
possible that he leaves early today and goes back to his family for the weekend. The
manager was very understanding, and he agreed. But things get harder when the physician
in a nearby clinic called the manager and said that he has 17 patients with meningitis that he
wants to send to the pharmacy. What is the most appropriate response from the manager?
a) Tell the pharmacist that you should know your legal and ethical responsibilities and
shouldn’t leave until all the work is done. (workload)
b) Get additional staff to help the pharmacist with the anticipated and expected work
load and ask the pharmacist to stay as late as possible *
c) Tell the physician to send those patients to another pharmacy that is 40 km away
d) Tell the pharmacist these patients have meningitis and their treatment should not be
delayed

309. A patient is taking alendronate weekly every Wednesday morning. She called you on
Wednesday afternoon 5 pm, saying that she missed her pill this morning and asking what to
do. What is your most appropriate response?
a) Advise her to take the pill now and take the next pill next Wednesday the same time
b) Advise her to take the pill tomorrow morning and reschedule her medication every
Thursday
c) Advise her to take the pill tomorrow morning and continue as her schedule every
Wednesday
d) Advise her to take the pill at bed-time on empty stomach
e) Advise to take it at supper
310. All can be used in sunburns except?
a) NSAIDS
b) Lidocaine
c) Topical corticosteroids
d) Cold compresses
e) Oatmeal
Further exposure should be avoided until sunburn has completely subsided. Cold tap-water compresses and
oral NSAIDs help relieve symptoms, as may topical treatments (eg, aloe vera, other water-based lotions).
Petrolatum-based products such as petroleum jelly should be avoided in severe sunburns. Topical
corticosteroids are no more effective than cool compresses. Blistered areas should be managed similarly to
other partial-thickness burns (see Initial wound care), with sterile dressings and silver sulfadiazine.
Ointments or lotions containing local anesthetics (eg, benzocaine) or diphenhydramine typically should be
avoided because of the risk of allergic contact dermatitis.
Early treatment of extensive, severe sunburn with a systemic corticosteroid (eg, oral prednisone 20 to 30 mg
2 times a day for 4 days for adults or adolescents) may decrease the discomfort, but this use is controversial.
311. Patient is suffering Post-Traumatic Stress Disorder. What do you recommend?
a) SSRI
b) Gabapentin
c) SNRI
d) Amitriptyline
e) BDZ
312. Elderly female is confused about taking her many medications. You can do all of the
following except?
a) Prepare her medications in a blister pack
b) Go visit her at home to organize her medications
c) Ask her what she is taking
d) Review her medication history in the pharmacy
All given steps are applicable.
However, home visits are meant to perform a medication review not to organize meds.

313. A preparation of 5% stock soln & you want to make .0125% so you use;
a) 5ml in 2 liters
b) 5ml in 1 liter
c) 2.5 ml in 2.5 liters

314. You has a stock solution of 10% w/v. You are required to dilute it as 1:5 by adding
suitable diluents. The total final volume of solution is 5 ml. What is the appropriate volume of
diluents and stock solution you need to make the required solution?
a) 4 ml diluents and 1 ml stock solution
b) 4.5 ml diluents and 0.5 ml stock solution
c) 4.9 ml diluents and o.1 ml stock solution
Answer: 10 % Soln. diluted as 1:5 become 2 %
C1*V1 = C2*V2 2%*5=10%*V2 V2=2%*5/10%=1ml
So, 1 ml stock solution & 4 ml (rest of 5 ml needed) diluent
Final solution: 2% means
2 gm ----- 100 ml X gm ----- 5 ml X=5*2 /100 = 0.1 gm = 100 mg

315. Question was on find out accuracy and table of true positive, true negative, false negative
and false positive values have been given.

316. Drug X 5000 000IU when we add 23 ml of solvent its conc. becomes 200 000 IU/ml. How
much solvent should be added to make conc. 125 000 IU/0.5 ml (this type of question came
forgot the values)

317. An experiment done on patient drug 841 out of it 284 got side effect and on placebo 822
patient got 386 patients. Which of the following is right? (Values are approx)
a) NNT is 9 %
b) NNT is 11%
c) ARR is 12 %
d) RRR is 13%
318. A prescription contains drug x as follows (patient asked for dispensing refill too at same
time). Dose is 2gm Q 12 hr X 2 doses Mitte: 6 doses. Refill: 1. How may capsule would you
dispense?
a) 500 mg 36 capsules
b) 500 mg 48 capsules
c) 500 mg 72 capsules
d) 1000 mg 19 capsules

319. Pt received 1 gm vancomycin IV. T½of vancomycin is 3 days. 3 days later he received 500
mg IV. The blood conc was 15mmol/L. what is the steady state trough conc. of vancomycin
a) 30 mmol/L
b) 15 mmol/L
c) 45 mmol/L

320. Reporting Adverse effects of a drug in hospital to?


a) Medeffect
b) CADTH
c) NSIR

321. Most causative organism in UTI (no E. Coli in the option)


a) Klebsiella
b) S. aureus
c) Neisseria
d) Enterococci

322. What do you monitor in Enoxaparin?


a) INR
b) aPPT
c) PT
LMWHs are typically given subcutaneously in a standard weight-based dose (eg, enoxaparin 1.5 mg/kg
subcutaneously once a day or 1 mg/kg subcutaneously every 12 hours or dalteparin 200 units/kg
subcutaneously once a day). Patients with renal insufficiency may be treated with UFH or with reduced
doses of LMWH.
Monitoring is not reliable because LMWHs do not significantly prolong the results of global tests of
coagulation. Furthermore, they have a predictable dose response, and there is no clear relationship between
the anticoagulant effect of LMWH and bleeding.
Treatment is continued until full anticoagulation is achieved with warfarin (typically about 5 days).
Transition to the oral drugs rivaroxaban or apixaban can be done at any time with no overlap. Transition
to edoxaban or dabigatran requires at least 5 days of LMWH treatment, but no overlap is needed.
323. EpiPen storage
EpiPen® and EpiPen Jr® Auto-Injectors and Epinephrine Injection, USP should be stored in the carrier tube
provided at a temperature of 20-25 °C (68-77 °F); however, temperature excursions between 15-30 ºC (59-86
ºF) are permitted.EpiPen®, EpiPen Jr®, and Epinephrine Injection, USP Auto-Injectors should not be stored in
refrigerators or in a vehicle's glove box.1,2 EpiPen® Auto-Injectors and their authorized generic auto-
injectors should not be exposed to extreme heat or cold and should be protected from light.

324. What is true about Epipen


a) Can be injected through clothes
325. A lady with post-herpetic neuralgia pain, now she has a prescription for Amitriptyline
“indicated in this condition” and Tylenol #3, what should you advise her?
a) Recommend Tylenol #1 instead of #3
b) Tell her that Amitriptyline is the best for her
c) If Amitriptyline causes side effects, she should stop it and continue with Tylenol #3
d) If Amitriptyline causes side effects, she should go to a DR. clinic

326. Patient having uncontrolled angina, already on 12 hr nitrate patch, you CANNOT suggest:
a) 24-hour nitrate patches
b) Metoprolol
c) Amlodipine
d) Verapamil
24-hour nitrate (angina uncontrolled)

327. What is counselling of Etanercept?


a) Should be refrigerated.
b) Injected IM
Should be stored refrigerated at 2 to 8°C. DO NOT FREEZE. Keep the product in the original carton to protect
from light until the time of use. Do not shake. Keep in a safe place out of the reach of children.
May be transferred to room temperature storage (≤27°C) for a period not to exceed 60 days. Once
transferred to room temperature storage, it must be used within 60 days. Protect from direct sunlight,
sources of heat, and humidity.
TNF-alpha A recombinant monoclonal antibody that binds to TNF-α, thereby interfering with
Inhibitors TNFI endogenous TNF-α activity by blocking its interaction with cell surface receptors.
Adalimumab Improve symptoms in patients with moderate to severe RA who are unresponsive to
Certolizumab csDMARD therapy alone. Given in combination with MTX in newly diagnosed patients.
Etanercept (acts May be used as monotherapy in case of MTX contraindication
against both TNF Adalimumab: 40 mg Q2 wk SC.
and ) Certolizumab: 400 mg at wk 0, 2 and 4, then 200 mg Q2 wk SC. May give 400 mg Q4 wk
Golimumab SC as maintenance dose
Etanercept: 25 mg twice weekly or 50 mg once weekly SC
Golimumab: 50 mg once monthly on same date each month SC. Effective in patients who
have failed therapy with other TNFi.
S.E: Injection-site reactions; infections (including TB); newonset psoriasis; increased risk
of lymphoma (children and adolescents), leukemia autoimmune phenomena.
Monitoring: Baseline CBC, LFTs, creatinine, hepatitis B and C serology, PPD and chest
x-ray to assess for latent TB. May consider screening for ANA.
Susceptibility to or presence of serious and/or recurrent infection; SLE, demyelinating
disease and heart failure are relative contraindications.
328. Oral contraceptive is safe to given with which anti seizure medication
a) Gabapentin
b) CBZ
c) Oxcarbazepine
d) Phenytoin
There is an increased risk of combined oral contraceptive (COC) failure in women taking enzyme-inducing
AEDs. Ideally the COC should contain ≥ 50 mcg ethinyl estradiol; however, no COC products currently
available in Canada contain >35 mcg.
Some experts recommend combining 2 COCs that contain an identical progestogen to achieve 50–60 mcg
ethinyl estradiol.
In addition to hormonal methods, the use of a barrier method (condoms) is recommended for increased
contraceptive effect and to reduce the risk of sexually transmitted infections; however, barrier methods
should not replace more effective methods of contraception.
Q. Enzyme-inducing AEDs. Carbamazepine Eslicarbazepine Oxcarbazepine
Reduced efficacy of hepatically Perampanel (8 mg daily or higher)
metabolized drugs (e.g., oral Phenobarbital Phenytoin Primidone
contraceptives) Topiramate (200 mg daily or higher)
Rufinamide
Nonenzyme-inducing AEDs. Brivaracetam Clobazam Ethosuximide
Efficacy of hepatically metabolized Levetiracetam Lacosamide Lamotrigine
drugs not likely to be affected Gabapentin Vigabatrin Valproic Acid
Contraceptive methods likely to be Combined contraceptive patch
less effective in women taking Combined oral contraceptive
enzyme-inducing AEDs Progestin implant
Progestin-only oral contraceptive
Vaginal ring
Q. Contraceptive methods not likely Barrier contraception (less effective than other methods)
to be affected by enzyme-inducing Depot progesterone
AEDs Intrauterine device (hormonal or nonhormonal)

329. What is right about Modafinil (Schedule I prescription drug.)


a) Written only
b) Written and verbal only
c) Written and verbal with part fill

330. A patient brought back a product which was dispensed yesterday and it was expired. What
to do to prevent it
a) Before dispensing check the expiry
b) Ask the patient to check the expiry always before receiving it
c) Do the inventory check every 6 months
d) Train the cashier to check the expiry
331. Q counselling on testosterone how to take, side effect, indication (is it can be given in male
patient with hypogonadism diagnosed with osteoporosis)
Indications and Testim 1% (testosterone gel) is indicated for testosterone replacement therapy in adult
Clinical Use males for conditions associated with a deficiency or absence of endogenous testosterone.
Testim 1% (testosterone gel) should not be used to treat non-specific symptoms
suggestive of hypogonadism if testosterone deficiency has not been demonstrated and if
other etiologies responsible for the symptoms have not been
excluded. Testosterone deficiency should be clearly demonstrated by clinical features
and confirmed by two separate validated biochemical assays (morning testosterone)
before initiating therapy with any testosterone replacement, including Testim 1%
treatment.
The recommended starting dose of Testim 1% is 5 g of gel (one tube) containing 50 mg
of testosterone applied once daily (preferably in the morning) to clean, dry intact skin of
the shoulders and/or upper arms (area of application should be limited to the area that
will be covered by the patient's short sleeve t-shirt).
If a dose is missed, this dose should be taken only if the next scheduled dose is more
than 12 hours away. The missed dose should not be taken if the next scheduled dose is
less than 12 hours away. Resume a regular dosing schedule as soon as possible.
The physician or health care professional should advise patients of the following:
 Testim 1% should not be applied to the scrotum.
 Testim 1% should be applied daily to clean dry skin.
 Avoid application of topical testosterone products to sunburned areas of the body.
 In order to maintain serum testosterone levels in the normal range, the sites of
application should not be washed for at least two hours after application of Testim 1%.
 Men with known or suspected prostate or breast cancer should not use Testim 1%.
Adverse Application site erythema, Increased PSA, Increased hematocrit, Increased hemoglobin.
Reactions Headache NOS, Insomnia
Contraindications  Not indicated for use in women.
 Pregnant and nursing women should avoid skin contact with Testim 1%
application sites on men. Testosterone may cause fetal harm.
 Virilization has been reported in children who were secondarily exposed
to testosterone gel, including Testim 1%.
 Children should avoid contact with unwashed or unclothed application sites in
men using Testim 1%.
Monitoring and The patient should be monitored (including serum testosterone levels) on a regular basis
Laboratory Tests to ensure adequate response to treatment.
The following laboratory tests, performed routinely, are recommended to ensure that
adverse experience is detected and addressed:
 hemoglobin and hematocrit levels should be checked periodically (to detect
polycythemia);
 liver function tests; to detect hepatoxicity associated with the use of 17-alpha-alkylated
androgens (e.g. methyltestosterone);
 prostate specific antigen (PSA), Digital Rectal Examination (DRE), especially if the
patient presents with progressive difficulty with urination or a change in voiding habits;
 lipid profile, total cholesterol, LDL, HDL, and triglycerides;
 diabetics should be followed carefully and the insulin or oral hypoglycemic dosage
adjusted accordingly
Drug-Drug In diabetic patients, the metabolic effects of androgens may decrease blood glucose and,
Interactions therefore, insulin requirements.
The concurrent administration of testosterone with ACTH or corticosteroids may
enhance edema formation; thus, these drugs should be administered cautiously
particularly in patients with cardiac, renal or hepatic disease.
Androgens may increase sensitivity to oral anticoagulants. Dosage of the anticoagulant
may require reduction in order to maintain satisfactory therapeutic
hypoprothrombinemia.
It was found that some herbal products (e.g. St. John’s wort) which are available as
over-the-counter (OTC) products might interfere with steroid metabolism and therefore
may decrease plasma testosterone levels.
Androgens may decrease levels of thyroxine-binding globulin, resulting in decreased
total T4 serum levels and increased resin uptake of T3 and T4. Free thyroid hormone
levels remain unchanged, however, and there is no clinical evidence of thyroid
dysfunction.

332. Patient going to South Carolina, what vaccine does not require?
a) Influenza
b) Hep B
c) Cholera

333. What is wrong about compounding


a) Mechanical balance better than electronic balance in weighing powder

334. Patient is on metronidazole gel getting partial response but also suffered from side effect of
dry skin, what to recommend?
a) Change the medication
b) Decrease the no of applications
c) Switch to the metronidazole cream
Reduce application # (temporarily)

335. A regular patient of your pharmacy having hypertension asked you about pseudoephedrine
product is not available in the aisle from where he takes it frequently now not available. What to
do to prevent this kind of error?
a) Conduct a seminar on drug-disease interaction
b) Update OTC profile on every patient
c) Bring these kinds of product behind the counter
336. Where do check about which causes more hypoglycaemia NPH or regular insulin
a) TC
b) Clinical guidelines
c) RxFiles

337. Patient having severe disease and his medication is several dollars; he has not much money
to pay. How pharmacist can help?
a) Check that if drug is in SAP programme
b) Give the medication from the pharmacy donations
c) Give the medication and tell him to pay when he gets welfare money later
d) Ask the manufacture if they have something in their plan for this drug

338. A patient came to the pharmacy telling that computer printer beaten her badly last night
(psychic patient). She is having bruises in her back. Dr prescribed a medication for her but don’t
want take the medication. What to do?
a) Tell her risk vs benefit
b) Respect her decision

339. she is happy with pharmacist counselling and ready to take medication. She says that
pharmacist should counsel her after his shift too, pharmacist would not accept in following what?
a) Justice
b) Autonomy
c) Professional boundaries
d) Veracity

340. A diabetic patient is on methadone programme. She missed several doses and now she is in
hospital. What sentence proves the veracity?
a) Patient tell the Dr. that how many dosses she missed
b) Social worker tells the patient’s parents that she is in hospital
Telling the truth about missed doses (We need to see ALL other answers too, as it's very risky this way.)

341. In above patient what to monitor before restarting methadone


a) Glucose
b) Liver enzyme
Blood sugar is a universal monitoring parameter for any diabetic patient.

342. Police wants to know about medication history of drinking driver from the pharmacist.
a) Ask for written consent
b) Give to him
343. Handling which of these medications with care?
a) Hydroxyurea
b) Digoxin
c) Allopurinol

344. A lady came to your pharmacy. she has greyish vaginal discharge without itching, had this
symptom never before what should you do
a) Give her miconazole vaginal cream
b) Refer her to physician

345. A lady going for trip for 2 months, smoker 32-year-old, taking contraceptive. She does not
want to have period during the trip what to suggest?
a) Change OCP to DMPA
b) Tell her to take OCP continuous without break
c) Give her norethindrone until she returns
Back-to-back CHC without breaks

346. Pt taking prednisolone, having edema, what to consider except


a) Fluid retention
b) Avoidance of Cushing
c) Avoidance of Addison

347. Patient increases opioid dose by herself and get pain relief
a) Offer to talk with her physician about her dose and assessment

348. Digoxin and levothyroxine interaction? Before taking levothyroxine, tell your doctor if you also
use digoxin. You may need dose adjustments or special tests in order to safely take both medications
together. This combination may cause a decrease in digoxin levels.

349. Digoxin and Verapamil interaction? You may need dose adjustments or special tests in order to
safely take both medications together. This combination may increase digoxin levels. You should notify your
doctor if you have symptoms of nausea, loss of appetite, visual changes, slow pulse, or irregular heartbeats.

350. Pharmacist to enhance the application of expansion of scope of practice which approved by
regularity authority and resisted by physicians:
a) Lobby patients to pressure physicians to accept it
b) Send written letters to physicians explaining the new changes
c) Attend local meetings of physicians in your area to explain changes and address
their concerns.
MAY 2014
1. The destruction of expired narcotics (opened bottle, spoiled, returned from patient) must be
witnessed by
a) Pharmacist + Technician
b) Pharmacist + Practitioner
c) Technician + 2 Witness
d) Pharmacy Manager
e) Pharmacist + Lawyer
The destruction must take place in the presence of a witness who should be either another pharmacist or a
practitioner.

2. All are correct about Benzodiazepine Rx EXCEPT


a) Rx should be one year from the date written
b) Transfer of this prescription is permitted once
c) Sales record is required
d) Written, Faxed or verbal Rx are accepted
e) Refill are permitted via written, faxed or verbal Rx

3. What the legislation for acetaminophen + 30 mg codeine + Caffeine: (verbal narcotic)


a) Written Rx only with refills
b) Verbal Rx only without refills
c) Verbal & written Rx with refills
d) Verbal & written with no refills

4. Records for narcotic destructions including the name, strength per unit and quantity of the
substance destroyed must be kept for
a) 1 year
b) 2 years
c) 5 years
d) 7 years
e) 10 years
Authorization is not required to destroy benzodiazepines and other targeted substances. However,
witnessed records including the name, strength per unit and quantity of the targeted substance destroyed
must be kept on site for two years.
Documents pertaining to the destruction must be kept on site for a period of two years from the date of
destruction.
5. Any loss (including breakage) or theft of narcotic or controlled drugs must be reported to
Office of Controlled Substances within:
a) 1 days
b) 2 days
c) 5 days
d) 10 day
e) 1 week

6. Tylenol 3 Legal requirements on refills, verbal?


a) Can be transferred only once
b) Can have refills with a verbal order
c) Only written or faxed
d) No refills allowed only part fills

7. Legal prescription requirements for modafinil (controlled)


a) Verbal and written
b) Only written
Modafinil is an effective treatment for (ADHD) but it did not receive FDA approval due to reported cases of
Stevens–Johnson syndrome. FDA approved it for treatment of narcolepsy. In Canada it is Schedule 1 (normal
RX regulation, Verbal and written)

8. Patients who are taking Infliximab should be monitored during the first 2 hours during
infusion for
a) FBC (Full Blood Count)  FBC & LFTs done with Methotrexate and Leflunomide
b) LFTs (Liver Function Tests)
c) WCC (White Cell Count)
d) Chest pain, fever, hypotension
Visual symptoms and assessing for blurred vision with Chloroquine and Hydroxychloroquine
REMICADE has been associated with hypersensitivity reactions that vary in their time of onset. Most
hypersensitivity reactions, which include urticaria, dyspnea, and/or hypotension, have occurred during or
within 2 hours of infliximab infusion

9. What is a nested Case-control study design?


a) Case-control in a cohort study
b) Cross-sectional study
c) Cohort study in case control study
d) Randomized controlled study
e) Meta-analysis study
10. Lady came with Folinic acid RX, the pharmacist told the technician that he will find it as?
a) Folic Acid
b) Leucovorin
c) Folate
FOLINIC ACID (5‐formyl tetrahydrofolate) In contrast to folic acid, a synthetic form of folate, Folinic acid is
one of the forms of folate found naturally in foods. Leucovorin (brand name) is used to reduce side effects of
methotrexate because it is not affected by methotrexate in the same way that folic acid is.
Leucovorin is also used in combination with fluorouracil to treat cancer of the colon. It is also used to treat
megaloblastic anemia, which can occur during pregnancy

11. Parenteral medication recall from all hospitals in a single city (special geographic) due to
problem
a) Production mistake
b) Cold chain transportation
c) Manufacture aseptic technique
d) Product instability
e) Adverse effect
This problem should be reported to local public health agency

12. The best source of information that presents a gateway to immunization information
including vaccine safety, immunization schedule is?
a) Canadian Public Health Association
b) Institute of Health Research
c) Public Health Agency of Canada
d) Canada Immunization Research Center
e) Canada Health Act
Health Canada: A variety of information on immunization, including immunization schedules for children
and recommendations for travel vaccines.
Public Health Agency of Canada: A gateway to immunization information including vaccine safety,
influenza/flu shot information, immunization schedules, the Canadian Immunization Guide, and Flu Watch.
Also, Sexual Health and Sexually Transmitted Infections

13. Patient come to you with Pilocarpine RX for xerostomia (Dry mouth) associated with use of
psychoactive agents, what you will do.
a) Tell her it is an off-label indication, but it will help her case
b) Inform her that It’s toxic and should not be use
c) Call her doctor to change the medication
d) Dispense without informing her about that effect
Therapeutic indications
Alleviation of symptoms of salivary gland hypofunction in patients with severe xerostomia following
irradiation for head and neck cancer.
Treatment of symptoms of dry mouth and dry eyes in patients with Sjögren's syndrome.
Method of administration
For head and neck cancer patients:
The recommended initial dose for adults is 1 tablet of 5 mg three times daily. The maximal therapeutic
effect is normally obtained after 4 to 8 weeks of therapy. For patients who have not responded sufficiently
after 4 weeks and who tolerate the dose of 5 mg three times daily, doses of up to a maximum of 30 mg daily
may be considered. However, higher daily doses are probably accompanied by an increase in drug‐related
adverse effects. Therapy should be discontinued if no improvement in xerostomia is noted after 2 to 3
months of therapy.
For Sjögren's syndrome patients:
The recommended dose for adults is one tablet of 5 mg four times daily. For patients who have not
responded sufficiently to a dosage of 5 mg four times daily and who tolerate this dosage, increasing the
dose up to a maximum of 30 mg daily, divided over the day, may be considered. Therapy should be
discontinued if no improvement in the symptoms of dry mouth and dry eyes is noted after 2 to 3 months.

14. All the below risk factors could be


linked with Parkinsonism EXCEPT?
a) Age
b) Gender
c) Influenza
d) Family history
e) Head Trauma

15. Technician counselled medication behind the counter, what you have to do as a pharmacist?
a) Terminate the pharmacy technician
b) Make sure that he discusses the side effect with the patient
c) Interrupt the conversation and take over
d) Allow him to council under your supervision
e) Take disciplinary action

16. Patient come with prescription and you realize it is forgery what you will do?
a) Tell the patient you do not have the medication and he can come tomorrow to tell
the police.
b) You tell the patient it is forgery and don’t dispense it
c) Tell the patient it is forgery and restrain the patient until call the police.
d) Send the patient to another pharmacy to dispense the medication.
17. A 42-year-old male presented to a clinic with a complaint of twitching in his legs. He
reported nightly discomfort for the past two years. He noted that it occurred once he was in a
semi-reclined supine position watching television in bed for a period. The discomfort was
described as “pressure” without pain or paresthesia, occurring more frequently in his right leg.
The presenting complaint sometimes caused him to wake up at night and roll over in bed.
Relieving factors included shifting positions and “shaking out the leg.”
What is the Drug of Choice (DOC) to treat RLS (Restless Leg Syndrome)?
a) Reminyl (Galantamine)
b) Mirapex (Pramipexole)
c) Cymbalta (Duloxetine)
d) Elavil (Amitriptyline)
e) Effexor (Venlafaxine )
Restless legs syndrome is a sensorimotor disorder characterized by an irresistible urge to move the legs,
arms, or, less commonly, other body parts, usually accompanied by paresthesias (eg, creeping or crawling
sensations) and sometimes pain in the upper or lower extremities; symptoms are more prominent when
patients are inactive or recline and peak in severity around bedtime. To relieve symptoms, patients move
the affected extremity by stretching, kicking, or walking. As a result, they have difficulty falling asleep,
repeated nocturnal awakenings, or both. Symptoms may be worsened by stress. Episodes may occur
occasionally, causing few problems, or several times a week.

18. All of the below are non-pharmacological treatment for RLS and may help EXCEPT
a) Try baths and massages.
b) Moderate, regular exercise.
c) Try relaxation techniques, such as meditation or yoga.
d) Establish good sleep hygiene
e) Complete Bed rest
Engage in mental alertness activities (playing cards, video games or doing crossword puzzles) to reduce
symptoms during times of boredom. Abstain from alcohol, caffeine and nicotine. Take hot baths, stretch and
exercise moderately. Discontinue medications that may be contributing to symptoms, e.g., antidepressants,
antipsychotics, dopamine‐blocking antiemetics and sedating antihistamines.
Minimize aggravating factors such as sleep deprivation. In patients with RLS and
varicose veins, consider sclerotherapy to improve RLS

19. Triggers that linked with increase in RLS symptoms could be all of
the below EXCEPT
a) Caffeine
b) Periods of inactivity
c) Stretch your legs before bedtime
d) Long-distance flights
e) Immobilization such as a cast
20. Pharmacist works for a family health practice and patient with restless leg syndrome patient
has diabetes, start to make hemodialysis, he developed tremors. Doctor asks you to determine
possible other causes of tremor. What may be a contributing disease state?
a) Diabetes
b) Kidney disease (due to electrolyte imbalance)
c) Hypertension
d) Gout

21. Weight loss is a common Anticonvulsant Side Effects that linked with
a) Divalproex sodium
b) Lamotrigine
c) Carbamazepine
d) Topiramate

22. Patient using Lamotrigine for a long time, what’s the most significant side effect
a) Insomnia
b) Weight gain
c) Weight loss
d) Blurred vision
Patients with uncontrolled epilepsy should not drive or handle potentially dangerous machinery. During
clinical trials, common adverse effects included dizziness, ataxia, drowsiness, diplopia and blurred vision.
Patients should be advised to refrain from activities requiring mental alertness or physical coordination until
they are sure that LAMICTAL does not affect them adversely.
Blurred vision=>15.5%
Insomnia=>5.6%

23. A pharmacist taking so many sick days (abusing the rules), asked if can take a specific
weekend off, but pharmacy manger said NO, on that Sunday the pharmacist calls pharmacy and
tell that he is sick, what is the proper action of pharmacy manager
a) Give disciplinary notice
b) Do nothing, it is his right to take sick leave
c) Ask the pharmacist when he will come back to work
d) Call his Doctor to make sure that he is sick
e) Ask pharmacy staff about his vacation plans
24. You as Pharmacist manager has video tapes of pharmacist taking money from registry, so
you will
a) Call provincial regular authority
b) Take him away from register
c) Give him written notice
d) Confirm with Technician before any action
e) Guiding the pharmacist with discipline

25. Patient has a new RX for Nitroglycerin patch, he came to you as he feels it is not working,
what to do
a) Advice the patient to keep patch 24 hours
b) Ask the patient how he is taking Nitroglycerin
c) Call the doctor to add Amlodipine
d) Tell him to continue the current medication
e) Add B blocker
It is recommended that the patch is applied to healthy, undamaged, relatively crease free and hairless skin.
The best places to apply Deponit patches are the easily reached, fairly static areas at the front or side of the
chest. However, Deponit patches may also be applied to the upper arm, thigh, abdomen or shoulder. Skin
care products should not be used before applying the patch. The replacement patch should be applied to a
new area of skin. Allow several days to elapse before applying a fresh patch to the same area of skin.
Tolerance may occur during chronic nitrate therapy. To avoid development of tolerance, the GTN patch
should remain on the skin only for about 12‐14 hours, to ensure a nitrate free interval of 10‐12 hours.
Additional anti‐anginal therapy with drugs not containing nitro compounds should be considered for the
nitrate‐free interval if required.
As with any nitrate therapy, treatment with these patches should not be stopped abruptly. If the patient is
being changed to another type of treatment, the two should overlap.

26. What is preferred to be taken in the morning


a) Amiodarone
b) Amitriptyline
c) Amphotericin B
d) Amlodipine
e) Amiloride Diuretic

27. What is the correct counselling for a patient getting started with Insulin
a) Insert pen tip into skin at a 45º angle
b) Buttock and thigh are the best site with fast absorption
c) Check insulin flow (prime each time)
d) With shorter needles (≥ 5mm), you may need to gently lift the skin before injection
.

28. A pharmacist dispenses 5mg while the RX was 0.5 mg, What the cause of this error?
a) Use of trailing Zero
b) Use of nonstandard abbreviation
c) Lack of training
d) Work overload
e) Lack of leading zero

29. Patient has diabetes, what you will tell him.


a) Decrease alcohol intake to dink 1-2 drinks/week
b) The lacquer is preferable than wine
c) Calculate your diet calories
d) Do not take alcohol at the lunch
30. What is NOT important to be counselled with Diabetic patient by pharmacist
a) Lifestyle modifications
b) Decrease alcohol intake to drink 1-2 drinks/week (per day not week)
c) Self-monitor their blood glucose
d) Exercise and physical activity
e) Try to avoid injury
Because this is the normal guidelines for also non‐diabetic patients

31. A 75-y hypertensive patient with 165/95, what is the most appropriate initial medication
a) ACEI
b) B Blocker
c) CC Blocker
d) Furosemide (for ISH)
e) Amlodipine
https://www.hypertension.ca/en/professional/chep/therapy/hypertension-without-compelling-indications
Initial therapy should be monotherapy with a thiazide/thiazide‐like diuretic (Grade A); a beta‐blocker (in
patients younger than 60 years of age, Grade B); an ACE inhibitor (in nonblack patients, Grade B); a long‐
acting CCB (Grade B) or an ARB (Grade B). If there are adverse effects, another drug from this group should
be substituted. Hypokalemia should be avoided in patients treated with thiazide/thiazide‐like diuretic
monotherapy (Grade C).

32. Diabetic patient on Metformin and Gliclazide and drinking 2 cups of alcohol per day and 4
cups on the weekend what is the correct statement
a) Alcohol is contraindicated with Metformin
b) Drinking 2 cups of alcohol daily with Gliclazide is accepted.
c) Excess alcohol will cause hyperglycemia.
d) Alcohol should be avoided with Gliclazide
Or He will suffer hypoglycemia in the weekend

33. The most important concern when a drug enters the formulary is
a) Therapeutic Index
b) National adverse effects
c) Cost
d) Efficacy

34. Vaccines refrigerator has been disrupted for 3 days, your role as a pharmacist is to do what?
a) You should penalize the technician who is responsible for Vaccines
b) The pharmacist continues to sell the vaccines
c) Search for the reason that led to the malfunction
d) Develop a way to keep the vaccines in a safe place
Separate the affected vaccine from other vaccine supplies and label it as “DO NOT USE” to ensure that the
vaccine is not administered. Store the affected vaccine under appropriate cold chain conditions until its
integrity is determined.

34. You discovered that Dukoral vaccine left out of refrigerator for 3 days, what you will do.
a) Return to the manufacture.
b) Just return it to the fridge??
c) Discard it in a safe way.
d) Ask who make this and take disciplinary action with him
Dukoral vaccine may be stored at room temperature for up to 2 weeks on one occasion only

35. Pt has end stage breast cancer; she is taking morphine & NSAIDs what would you tell her?
a) Cancer patient do not get addicted to opioids.
b) Addiction is not a concern for cancer patient.
c) No need to combine NSAIDs with opioid for enhancing the analgesia

36. Patients with cancer pain who take opioids for > 2 month may start to become tolerant
Her daughter return telling you that she still feels sever pain and has been taking more doses
than prescribed what to tell her?
a) Since her pain is uncontrolled it is OK to take more than prescribed dose
b) Call the doctor and report the abuse
c) Advise her to try natural pain killer
d) Call the doctor to reassess her dose

37. Benzodiazepine withdrawal symptoms after 2-3 days is noticed with


a) Diazepam
b) Oxazepam
c) Alprazolam
d) Clonazepam

38. Patients took by mistake 4 tablets Valsartan 80 mg instead of Diamicron 30 mg tablet so


what is expected to happen?
a) Increase pulse rate
b) Decrease pulse rate
c) Decrease blood glucose.
d) Increase systolic
e) Increase diastolic
SUPERGIFT Q!
39. Patient with Levothyroxine toxicity may have
a) Weight gain
b) Hypotension
c) Nervousness
d) Bradycardia

40. What is the only first line medication available for ADHD that can have a verbal refill?
a) Methylphenidate
b) Dextroamphetamine
c) Atomoxetine
d) Perindopril

41. Mother to child with ADHD, she is afraid that her child may be addicted with the use of
medication what you will tell her?
a) ADHD medication will decrease the chance of abuse when he became older
b) ADHD medication is not addictive
c) Decrease the dose in the first 6 month
d) Stop medication and go to specialist

42. DOC in treatment of ADHD?


a) Methylphenidate
b) Gabapentin
c) Donepezil
d) Methyl hydrate

43. ADHD goal of therapy are all of the following except


a) Improve Athlete performance
b) Improve Concentration & academic performance
c) Improve Impulsive behaviour

44. Adolescent with ADHD that was previously controlled on methylphenidate stopped taking
his medications. His mother asks you to speak to him regarding his medication. During the
interview, he told you that he cannot sleep and gets embarrassed by taking pills at school and is
too tired to focus. What is your appropriate response?
a) Recommend that he exercise at night to help him sleep better
b) Offer to help him with his symptoms
c) Suggest that he switch to a long acting formulation
d) Tell him he must take his medication
45. What is your appropriate action (for patient above)?
a) Fax the doctor to get him to authorize dispensing of the new medication.
b) Tell patient to go to the ER to get a new prescription (not that serious)
c) Suggest that he make an appointment and discuss the pharmacist recommendations
with his doctor
d) Leave a message with the nurse saying which med the patient should be switched to (still
telling the doctor what to do)
e) Call the doctor and tell him which med to switch the patient to (Never tell the doctor
what to do)

46. A volunteer in RCT, which ethical principal will conflict with concept of administering
placebo to him?
a) Autonomy
b) Justice
c) Veracity
The answer is VERACITY. This is because patients do not know that they are getting placebo (although they
have consented to be deceived in the trial). Thus, concealed treatment (meaning they do not know what
they are getting) = lying = violating veracity.

47. Which medication should be stopped before IV contrast media?


a) Metformin
b) Acarbose
c) Pioglitazone
d) Gliclazide

48. Very old woman come to Canada with her son and she doesn't speak English but her son
can speak English and she will make surgery who will sign the consent?
a) Her son has to sign the consent from for his mother
b) Family doctor has to sign the consent
c) Patient herself (the son can help translate)
d) Patient and her son should sign the consent together

49. Female patient with asthma will go to Caribbean for vacation, she wants to take her
vaccines, she will take all EXCEPT?
a) Diphtheria, Tetanus
b) Hepatitis A, B
c) Pneumonia vaccine
d) Influenza
Td (CAN BE TURNED INTO A CORRECT ANSWER)
50. What interacts with Co-trimoxazole?
a) Metformin
b) Glipizide
c) HCTZ
d) Bisoprolol
If both comes choose HCTZ.
COTRIMOXAZOLE increases levels of
GLIPIZIDE by plasma protein binding
competition. Significant interaction
possible, monitor closely.
Diuretics (thiazides): in elderly patients concurrently receiving diuretics, mainly thiazides, there appears to
be an increased risk of thrombocytopenia with or without purpura.

51. Dentist prescribed Lorazepam for his patient, according to his scope of practice which
indication related to his choice of treatment.
a) Insomnia
b) Panic attack
c) Prior surgery anxiety
d) No valid indication

52. Hepatitis C patient need his medication, he lives in area away from his pharmacy, he needs
a refill and pharmacy do not have enough in stock and patient need it and cannot come and get
it later. It is going to be the weekend, what should the pharmacist do?
a) Mail the medication and cover the expenses of mailing himself and ask patient call for his
refills earlier next time
b) Mail the medication and ask the patient to pay the expenses
c) Ask the patient to go to another pharmacy
d) Call the nearest pharmacy to confirm medication availability then transfer the refill
e) Tell the patient to wait until Monday

53. Patient called you in the pharmacy, he is regular customer and need a drug, but you don’t
have stock from it, what you will do? Assess the emergency for his drug need

54. 15 years old girl, smoker and had her last period 14 days ago, she came to your pharmacy
and asked about plan-B after 3 days of sexual intercourse, what is the right action
a) Dispense plan-B & inform her about low efficacy
b) Refuse to dispense
c) Advise her to see a gynaecologist
d) Tell her there is no effect after 3 days
55. After that, she came again with prescription for COC what is your advice
a) You must use barrier for first few days (7 day pack up with combined)
b) It has a risk of breast cancer
c) Combined Hormonal therapy is not suitable for your age
d) Call the doctor before you dispense to confirm

56. Patient from USA want to buy an order of pseudoephedrine 60 mg quantity of 1000 tablets,
the pharmacist will refuse due to
a) It is not allowed to sell more than 3 gm in a package
b) Pseudoephedrine is a precursor for Methamphetamine
c) There are no Rx with the customer
d) It is Schedule II and it is the pharmacist rights to refuse to sell it
A new national law will restrict sales of pseudoephedrine and ephedrine...to limit diversion to crystal meth
labs. (Crystal meth means illicit methamphetamine hydrochloride)

57. Patient is travelling to USA, and wants to buy 1000 tab of pseudoephedrine 60mg and bring
it to his family and friends for the upcoming flu and cold season. All are ethical concerns for the
pharmacist except
a) Potential for drug abuse
b) Pseudoephedrine has many drug-drug interactions and side effects
c) Can only travel with medications for personal use only
d) There is No pharmacist-patient intervention with the other family members
Travel with meds for personal use (this is NOT my concern as a pharmacist)

58. Patient taking two much pseudoephedrine, what to do


a) Computer system to include OTC drugs in patient profile

59. Patient male 86 years, has atrial fibrillation, he is Diabetic and his profile includes
medication for hypertension, BPH, GERD and Dementia. Calculate the CHADS2 score?
a) 1
b) 2
c) 3
d) 4

60. Few days his heartbeat increased and now he cannot tolerate, he visited the ER
a) Dabigatran
b) ASA
c) Clopidogrel
d) Warfarin
61. When will he set for cardioversion?
Cardioversion is a medical procedure by which an abnormally fast heart rate (tachycardia) or cardiac
arrhythmia is converted to a normal rhythm using electricity or drugs. Synchronized electrical cardioversion
uses a therapeutic dose of electric current to the heart at a specific moment in the cardiac cycle.

62. Technician decides by himself to order patient medication and stock it so that the problem
of shortage does not happen again. Medication was very expensive, what is the correct action of
the pharmacist?
a) Return the medication.
b) Do something to the technician like disciplinary action
c) Review policy of ordering with all staff
d) Refer to pharmacy Manager (and let him/her decide)
e) Accept the order.

63. Which antidepressant need to taper off for minimum 10 days to switch medication?
a) Fluoxetine
b) Mirtazapine
c) Phenelzine
d) Venlafaxine
Paroxetine and Venlafaxine have the most discontinue effect appear in all antidepressants

64. Patient using Venlafaxine and doctor wants to switch to SSRI (Citalopram). What is true?
a) Stop Venlafaxine and start Citalopram immediately
b) Tapering Venlafaxine for 6-8 week then start Citalopram
c) Tapering Venlafaxine for 1-2 weeks then start Citalopram
d) This switch is not appropriate

65. Patient is prescribed meloxicam for muscle aches. Doctor suspects that he has osteoarthritis.
Current medications are acetaminophen, simvastatin, fluoxetine. What is drug therapy problem?
a) Interaction between meloxicam and simvastatin
b) Interaction between meloxicam and acetaminophen
c) Interaction between meloxicam and fluoxetine
d) Arthralgia with simvastatin
e) Myalgia with fluoxetine

66. Patient with ascites and takes Spironolactone, what should we add:
a) Furosemide
b) Metolazone
c) HCTZ
67. How to monitor Furosemide effectiveness through fluid loss with cardiac edema or ascites
a) Fluid input, fluid output
b) Glomerular Filtration Rate (GFR)
c) Patient weight
d) ECG

68. What is your recommendation for patient using Metronidazole gel for rosacea and his skin
becomes irritated or dry?
a) Advice the patient to discontinue
b) Switch from gel to cream
c) Stop the Metronidazole immediately
d) Refer the case to a dermatologist
Another correct option: Apply less of the preparation or use it less often until the irritation has settled

69. Pt with signs of allergic rhinitis, watery eyes, nasal discharge & green respiratory discharge
for 3 wks, she tried taking Loratadine but it was not effective, why would you refer this patient?
a) Symptom for more than 2 weeks
b) Green discharge
c) This is a complicated case and should be referred
d) Loratadine was ineffective

70. Menopause woman


complains from vaginitis &
dryness what form of
estrogen she can use.
a) Intravaginal
estrogen therapy
b) Estrogen Patch
c) Estrogen injection
d) Estrogen Tablet
Used to treat the symptoms of
genitourinary syndrome of
menopause.
It may be considered in patients
experiencing vaginal atrophy
due to menopause.
Reduces the incidence of
recurrent urinary tract infections
in postmenopausal women.
71. Which drug is known with its class side effect that cause hepatic cholestatic dysfunction?
a) Penicillin
b) Aminoglycosides
c) Estrogen
d) Nitrates

70. Pat with community acquired pneumonia and diabetes admitted in a hospital ward. What the
drug of choice for his condition?
a) B-lactam+ Macrolide
b) Amoxicillin/Clavulanate
For inpatients: Systematic review evidence for inpatients showed no difference between macrolides and
fluoroquinolones for death and no difference for death whether or not atypical coverage was used. In order to
reduce increasing fluoroquinolone resistance and prevent adverse events (e.g., QT interval prolongation), use of
a respiratory fluoroquinolone should be reserved for when cephalosporins or penicillins cannot be used.

71. His doctor decided that he does not need to stay in the hospital during treatment, what the
drug of choice for his condition?
a) Levofloxacin
b) Imipenem
c) Erythromycin
d) Doxycycline
For outpatients, amoxicillin as first choice or either amoxicillin/clavulanate or doxycycline as second choice.

72. A pharmacist in a community hospital is trying to increase the teamwork between the
pharmacy staff and other health care professionals in the area which of the following will not
contribute to achieving this end.
a) Send the pharmacist more often to the patient care area
b) Pay particular attention to the importance of communication
c) Facilitate inter-professional collaboration and understanding through training initiatives
d) Plan a disciplinary seminar lunch
Main objective of disciplinary seminar to strengthen Inter‐disciplinary team communication. Which is more
important in health care settings as the complex nature and demands of the health care work environment
requires the expertise and knowledge of differing individuals or specialists who can work together to solve
multifaceted and complex patient care problems. Furthermore, teamwork in healthcare is often the norm
and not the exception as there is a need to solve complex patient problems on a daily basis.

73. Hospital pharmacy manager wants to enhance co-operation between pharmacy staff and
physicians. Do all except:
a) Assign a mentor for new staff joining the pharmacy
b) Take staff in patients rounds
c) Assign tasks to staff in different hospital departments
d) Attend interdisciplinary lunch meetings
e) Let technicians attend grand medical rounds

74. Pharmacist want to do training for the technician’s staff. All will be of benefit except?
a) Hold regular staff meetings to discuss new initiatives
b) Attend a medical conference
c) Develop written job descriptions & update as necessary
d) Give techs the opportunity to expand their role for example, do glucose meter demos

75. Azithromycin caps Auxiliary label.


a) With or without food
b) Separate for dairy product
c) Take without food
d) Immediately after food
e) 1 hour before meal or 2hrs after meal
Azithromycin tablets and powder for oral suspension can be taken with or without food.

76. A regular client at your pharmacy called you on Saturday evening and she said that she ran
out of her medication and her doctor is on vacation and she asks you for a 2-day advanced until
she sees her doctor on Monday morning her profile is as follows,
 Amlodipine (90 tabs) once daily - last refill was 3 months ago and has no refills left.
 Enalapril (90 tabs) once daily - last refill was 3 months ago and has no refills left
 Paroxetine (90 tabs) once daily- last refill was 3 months ago and has no refills left.
 Zopiclone (30 tabs) once before sleep- last refill was 2 weeks ago and has no refills left.
Using your professional judgment as a pharmacist and your ethical legal principle what is the
most appropriate action you should do?
a) Advance her with two-day supply for all her medication
b) Advance her a two-day supply for all her medication except Zopiclone
c) Advance her a two-day supply of all her medication except Zopiclone and Paroxetine
d) Advance her to go to a walk-in clinic and get an authorized prescription
e) Advance her to go the nearest emergency department to get an authorized prescription

77. According to definition of drug interchangeability, generic should meet all, EXCEPT
a) Must have same amount of same or similar ingredients in same or similar dosage form
b) Comparable cost
c) Comparable pharmacokinetic
d) Same clinically significant formulation characteristics
e) To be administered by the same way as drug prescribed
78. Child with recurrent otitis media had taken Amoxicillin, two months before, what is the
most suitable medication for his current case?
a) Amoxicillin/Clavulanate
b) Cefuroxime
c) Ceftriaxone
d) Cloxacillin
e) Azithromycin

79. For the same case, you as a pharmacist what is the most important counsel for the mother?
a) Tell her that pain and fever may go away within 24-72hr after initiating antibiotics
b) Call the doctor and tell him to give her prophylactic treatment
c) Tell her to use cold compresses to relieve the pain and will not aggravate the fever
d) Tel her to measure the temperature every 4 hours

80. Child with AOM, he is 25mo old, when was 12-month-old used amoxicillin. Now presents
with otitis media. He was treated with amoxicillin. You will give him All except
a) Amoxicillin/Clavulanate
b) Cefuroxime
c) Cloxacillin

81. Child has asthma, and he comes and his asthma is getting worse, he takes SABA, if you do
not recommend going to the doctor to get ICS, you violate what?
a) Beneficence
b) Autonomy
c) Veracity
d) Justice

82. Long case about 7 y child with primary nocturnal enuresis (bed-wetting). What is the DOC?
a) Finasteride
b) Metoclopramide
c) Minirin (Desmopressin)
d) Chloral hydrate

83. How we can know that child improved, if


a) Totally dry nights
b) Dry night become around 25 %
c) Dry night become around 50 %
d) Dry night become around 75 %
e) Wet night become once/week
Desmopressin, an analogue of human antidiuretic hormone (ADH), decreases urine production when given
at bedtime and reduces the number of wet nights in 75% of children, with complete cessation in about 50%
of those who respond. Desmopressin is used when a rapid response is required. If successful, consider a 1‐
week interruption every 3 months to see if treatment is no longer needed. Desmopressin may be most
effective in older children with a normal bladder capacity but with a large urine output.
Patients with high urine output and reduced bladder capacity may require combination treatment
with desmopressin and a smooth muscle relaxant. If cost is a concern, reserve desmopressin for special
occasions such as overnight visits or camp. The risk of overhydration and hyponatremia associated
with desmopressin necessitates limiting fluid intake within 1 hour of going to bed to <250 mL for children
<12 years of age and <500 mL for children ≥12 years of age.
Desmopressin nasal spray is no longer indicated for the management of primary enuresis. Compared with
oral formulations, it is associated with a higher incidence of hyponatremia, which may result in seizures and
death.

84. After 3 month the mother came to tell you that the doctor stops the Minirin
a) The child is good now and the medication gave good result
b) The medication not good
c) It needs drug-holiday period
d) Due to side effect
If successful, consider a 1‐week interruption every 3 months to see if treatment is no longer needed.

85. Pharmacist call the doctor to tell him about something error in the prescription and the nurse
become nervous from this and the pharmacist explain to her that the doctor must do this due to
a) Non maleficence
b) Conflict of interest
c) Veracity
d) Autonomy

86. Cause of peripheral edema in hypertension patient taking Ibuprofen and Docusate sodium?
a) Docusate sodium
b) Ibuprofen
c) Drug interaction between Ibuprofen & Docusate

87. Patient that was controlled on their medications (furosemide 40mg and ACEI) and then
suddenly developed pedal edema. What was the cause?
a) Ibuprofen
b) Glucosamine
Pedal edema. This happens when fluid gathers in your feet and lower legs. It's more common if you're older
or pregnant. It can make it harder to move around in part because you may not have as much feeling in your
feet.
88. How do you fix the edema issue after stopping the offending agent?
a) Restrict Fluids
b) Double dose of furosemide for 3 days
c) Increase his ACEI
d) Add Hydrochlorothiazide
Management of Resistant Edema
Sodium restriction
1‐ emphasize adherence to ≤88 mmol/day (2 g Na+ or 5 g NaCl)
2‐ assess adherence to sodium restriction with 24‐hour urine sodium assessment
Fluid restriction (especially with hyponatremia): emphasize adherence to recommended fluid intake, which
may be ≤1 L/day if hyponatremia significant
Bed rest
1. assume supine position and elevate legs (increases glomerular filtration rate)
2. Ensure adherence to prescribed diuretic regime (dose and frequency)
Dosing of furosemide
a‐ increase the dose
incrementally until diuresis is
achieved, by doubling the dose
every 3–5 days
b‐ secretion of the diuretic into
the tubular lumen of the nephron
can be impaired in severe
hypoalbuminemia (albumin <20
g/L) or decreased renal perfusion
states such as renal failure or HF,
resulting in ineffective diuresis
with initial doses
c‐ if unable to maintain an
acceptable duration of effective
diuresis, increase the frequency
of dosing to 2–3 times per day
d‐ double the dose if switching
from IV to oral route
e‐ consider IV dosing if the GI
tract is edematous or in
decreased cardiac output states
that can lead to impaired
absorption of oral drug
f‐ consider continuous IV infusion
g‐ monitor clinical status
carefully, including weight, to ensure daily weight loss does not exceed 1 kg
h‐ monitor for and manage electrolyte disturbances and prerenal azotemia
89. Taking Furosemide may cause
a) Hypomagnesemia
b) Hypercalcemia
c) Hyperkalemia

90. Asthmatic patient has profile with medications for asthma, what may worsen his case:
a) Ibuprofen (or Advil)
b) ACE
c) Metformin
d) Lorazepam

91. In case in emergency for the same patient what is the DOC?
a) β2-agonist
b) Salbutamol
c) IV methyl prednisolone
d) Leukotriene Antagonist

92. Same patient left the hospital, what is the most important discharge treatment plan?
a) Salbutamol inhaler
b) Oral prednisone
c) IV methyl prednisolone
d) Leukotriene Antagonist
Avoid triggers

93. Hypertension may affect the below organs damage EXCEPT


a) Ventricular hypertrophy
b) Kidney
c) Sexual dysfunction
d) Neuropathy
94. In dispensing a prescription by law, the pharmacist is responsible for all except?
a) Accuracy
b) Drug Interaction
c) Interchangeability
d) Efficacy
e) Counseling

95. Patient has hemorrhagic stroke and came to the hospital. What’s right concerning Alteplase
a) He can take Alteplase immediately
b) He can take Alteplase within 3 hours
c) He can take Alteplase if he is under 75 y
d) He is not a candidate for Alteplase  for Ischemic Stroke

96. In the Above case, how does the doctor show paternalism?
a) Not asking his wife or the patient’s opinion
b) Following protocol

97. Conscious patient has 84 years and he arrived to the hospital with his wife. He has acute
myocardial infarction from 3hrs, and after 4hrs from patient arrival to the hospital, doctor
decide to give Alteplase, the decision is wrong because?
a) Patient age is over 75 y
b) Alteplase is Expensive
c) He had passed the Alteplase effective time
d) Alteplase is not approved for Acute MI
It is MI case not stroke. Thrombolytic therapy administered early in
the course of STEMI substantially reduces morbidity and mortality, particularly if the patient presents within
6 hours of symptom onset, Primary PCI is preferred in patients over the age of 75 because of a higher risk of
intracranial hemorrhage and higher overall early mortality seen with thrombolytic

98. We consider the doctor break the autonomy for the patient
a) If Doctor began proceedings without the opinion of the wife
b) If Doctor began proceedings without the opinion of patient family doctor
c) If Doctor began proceedings without the opinion of the wife and the patient
d) If Doctor began proceedings without the opinion of the patient

99. We consider the doctor follow paternalism in which case


a) If he did not ask the patient
b) If he did not ask the patient and his wife.
c) If he did not ask the patient wife.
100. Patient is stressed because her mother is hypertensive and had a fracture. She had to take
care of her, her mother’s profile shows Pantoprazole, Calcium carbonate and Vit D what is your
concern as a pharmacist.
a) Advice patient to start bisphosphonates
b) Change calcium carbonate to calcium citrate
c) Vit D is a fat-soluble vitamin
d) Advise her to start physiotherapy
A vertebral compression fracture, hip fracture or more than 1 fragility fracture after the age of 50 should be
considered virtually diagnostic of osteoporosis irrespective of BMD testing. A single fragility fracture (e.g.,
wrist) in this age group should be considered a sign of osteoporosis until proven otherwise. Test these
individuals with bone densitometry if available; they are still candidates for therapy if bone densitometry is
not available.

101. When to give ACEI and ARBs together? obsolete


a) If ACEI is not effective
b) Only in refractory Heart failure
c) If blood pressure is more than 165/95
d) In case of Post Myocardial Infarction

102. Patient has mild to moderate acne, on Azelaic acid and benzoyl peroxide, what the DTP
a) Risk of drug interaction
b) Increase side effects (2 X peelers)
c) Inappropriate medication

103. A person with acne and is experiencing severely inflamed face and is taking multivitamins,
Omega 3 and PPI was on benzoyl peroxide but wasn’t working for him, so doctor changed him
to erythromycin and Retinoid topical. What is the DTP
a) Drug-drug interaction
b) Inappropriate medication (severely inflamed)
c) Too high dose
d) Side effects

104. Same patient stopped all his acne medication because they were not working and started a
40 mg QD Accutane (Isotretinoin) what you check
a) Drug-drug interaction  Accutane with vitamin A in the multivitamin preparation
b) Too high dose
c) Inappropriate medication
d) Is cost covered by insurance or not
105. Side effects to Isotretinoin
a) Constipation
b) Conjunctivitis
c) Hypotension
d) Dyspepsia
Systemic Drug Therapy for Acne. Q. Retinoids Isotretinoin Accutane, Clarus, Epuris
It remains the most powerful anti-acne agent, with the majority of patients achieving clearing and sustained
remission, even in the most severe cases.
It is recommended as first-choice therapy for severe papulopustular or moderate nodular acne and for
nodular or conglabate acne for many reasons: clinical effectiveness, prevention of scarring and quick
improvement of a patient’s quality of life, including minimizing depression.
Avoid taking vitamin A as Oral isotretinoin is a natural metabolite of vitamin A.
0.5 mg/kg/day PO for the first month, increasing to 1 mg/kg/day as tolerated (with a goal cumulative dose of 120
– 150 mg/kg). Alternative lowdose regimen (particularly for treatment of resistant or quick-relapsing moderate
acne): 0.25–0.4 mg/kg/day.
A complete course of therapy consists of 12-16 weeks of Isotretinoin administration.
S.E: Teratogenicity. Common: mucocutaneous dryness, myalgia, arthralgia, photosensitivity, Headache. Rare:
hypertriglyceridemia, mood disorder, possibly suicide ideation, pseudotumor cerebri, erythema multiforme,
Stevens-Johnson syndrome, toxic epidermal necrolysis.
Blood potassium increased, blood alkaline phosphatase increased, blood bilirubin increased, blood urea
increased, elevated platelet counts, eosinophil count increased, false positive tuberculosis test, gamma-
glutamyltransferase abnormal, blood cholesterol increased, glucose urine present, haematocrit decreased, protein
urine, thrombocytopenia, WBC count decreased. Elevations in levels of serum creatine kinase CPK (monitor)
Patients should be advised to use a skin-moisturizing ointment or cream and a lip balm from the start of treatment
as isotretinoin is likely to cause dryness of the skin and lips. When necessary a sun-protection product with high
protection factor of least SPF 15 should be used.
It is recommended that blood donation for transfusion purposes be deferred during therapy with EPURIS and for
one month after discontinuation of treatment.
Patients who experience tinnitus or hearing impairment should discontinue EPURIS treatment and be referred for
specialized care for further evaluation.
Isotretinoin has been associated with inflammatory bowel disease (including regional ileitis, colitis and
hemorrhage) in patients without a prior history of intestinal disorders. Patients experiencing abdominal pain,
rectal bleeding or severe diarrhea should discontinue EPURIS immediately.
Tetracyclines: rare cases of benign intracranial hypertension (pseudotumor cerebri): allow 7 days washout after
stopping tetracyclines before starting isotretinoin.
The patient should have two negative pregnancy tests (β-hCG in urine or serum) before starting therapy with the
first pregnancy test conducted at initial assessment when the patient is qualified for therapy by the physician.
The patient then should have a second pregnancy test with a sensitivity of at least 25 mIU/mL with a negative
result, performed in a licensed laboratory, within 11 days prior to initiating therapy. The patient has had two or
three days of the next normal menstrual period before therapy is initiated.
Pregnancy test must be repeated monthly for pregnancy detection during treatment and at one month after
discontinuation of treatment. The dates and results of the pregnancy tests should be documented.
Treatment should be discontinued if the patient develops any of the following reactions: rash, especially if
associated with fever and/or malaise, conjunctivitis (red or inflamed eyes); blisters on legs, arms or face and/or
sores in mouth, throat, nose or eyes; peeling skin or other serious skin reactions.
The following tests are required before starting Isotretinoin, at first month, then as clinically indicated:
 Serum blood lipid determinations (under fasting conditions) should be performed before Isotretinoin is
given and then at intervals (one month after the start of therapy) until the lipid response to Isotretinoin is
established (which usually occurs within four weeks), and also at the end of treatment.
 Complete blood count and differential: for early detection of leukopenia, neutropenia,
thrombocytopenia and anemia.
 Liver function tests: Increases in about 15% of ALT, AST, ALP baseline levels have been reported.
 Blood glucose levels: all patients and in particular patients with known or suspected diabetes should have
periodic blood sugar determinations.
 Serious Adverse Event Warnings include psychiatric disorders (depression, psychosis and, rarely,
suicidal ideation, suicide attempts, suicide, and aggressive and/or violent behaviors). Monitor psychiatric
illness before dispensing isotretenoin.

106. Doctor is starting patient, who is 60kg, on Isotretinoin. Patient took 0.5mg/kg/day for 4
weeks. After the 4 weeks, doctor wants to increase it to 1mg/kg/day. The total cumulative dose
is 120-150mg/kg. If he starts with 30mg daily for 4 weeks, what is the next step:
a) 30mg bid x 3 months
b) 30mg bid x 4 months
c) 30mg bid x 5 months
d) 60mg bid x 3 months
e) 60mg bid x 4 months
Minimum cumulative dose = 120mg*60Kg=7200 mg
taken dose = 30mg/daily *28 days = 840 mg
So, the dose should be taken = 7200 - 840 = 6360 mg
If patient takes 30 mg BID = 30mg *2 = 60 mg Daily
No. of days = 6360 / 60 = 106 days = 3.5 months
As this is the minimum cumulative dose, So, the dose should be used for more than 3.5 month
the answer is 30 mg BID for 4 months

107. Effects of Rivastigmine on common symptomatology of Alzheimer's disease


a) Reassess at least within 3 months
b) Reassess at least within 6 months
c) Reassess at least within 12 months
d) Reassess at least within 24 months
Rivastigmine Inhibits non specific butyrylcholinesterase and reversible acetylcholinesterase or centrally
oral, selective arylcarbamate AchEi. Has short half life 2 hours, but able to inhibit AchEi to 10 hours.
transdermal Because of slow dissociation of carbamate enzyme, it is referred as pseudo-irreversible AchEi.
patch Initiated at 1.5 mg BID and increased to the minimum effective dose of 3 mg BID after 30 days.
The maximum dose is 6 mg BID PO. Not metabolized by cytochrome P450 system.
Oral: Adjust dose monthly; take with breakfast and dinner.
Patch:1 Exelon-5 patch daily; if well tolerated, increase to Exelon-10 patch after at least 4 wk.
If switching from oral rivastigmine, use Exelon-5 for patients taking <3 mg BID, and Exelon-10
for patients taking 3–6 mg BID
S.E: >10%: headache, dizziness, N/V, diarrhea, abdominal pain, anorexia. <10%: insomnia,
fatigue, syncope, dyspepsia, weight loss, UTI, rhinitis. Heart block, delirium & seizures are rare.
Patients should be monitored for symptoms of active or occult GI bleeding, especially those at
increased risk for developing ulcers, e.g., history of ulcer or receiving concurrent NSAIDS.
Patients may lose weight while taking cholinesterase inhibitors, including rivastigmine.
Therefore, the patient's weight should be monitored during therapy with EXELON PATCH.
Considering the possibility of an additive extra-pyramidal effect, the concomitant use of
metoclopramide and rivastigmine is not recommended.
Additive effects leading to bradycardia (which may result in syncope) have been reported with
the combined use of various beta-blockers (including atenolol) and rivastigmine.

108. Dose to be given as an infusion 900mg, 50mg/hr initially then increase by 50 mg/hr every
30 min until maximum 400mg/hr. how many hours needed
a) 3
b) 3.5
c) 4
Answer: 50 mg/hr for the first hour then add 50 mg every 30 min until reach 400mg/hr
1hr ½ ½ ½ ½ ½ ½ = total 4hr
So, 50 ----------- 50------50----50----50----- 50----50----- total 400mg/hr
109. Patient being treated for hepatitis B using Peginterfeon Alfa-2b . Why would you make
sure that he would not skip his dose?
a) To avoid opposing interferon from being developed
b) Missing dose may cause drug resistance & treatment failure
c) To avoid side effect
d) Missing dose will cost the patient a lot

110. Regarding Chlorhexidine what is true?


a) May stain teeth if used for long term
b) Interact with Antacid
c) Contraindicated in old age
d) Available in oral and injection form

111. A menopause woman has UTI, she uses Hormone Replacement Therapy (HRT), what is
the purpose?
a) CVD
b) UTI
c) Decrease bone loss
d) MI

112. Acetaminophen toxic dose


a) 325 mg *12 tabs
b) 500 mg *8 tabs
c) 500 mg *12 tabs (6000)

113. The most effective antidote when given within 8 hours of ingesting Acetaminophen is
a) N-acetylcysteine (NAC)
b) Activated Charcoal
c) Gastric lavage

114. A 50-year-old man presents in comatose state to the emergency department. His family has
noticed progressive confusion. History is significant for cirrhosis and alcoholism. His heart rate
is 112 bpm and BP is 105/66 mmHg. He is jaundiced and lethargic, is oriented to person and
place but not date, and has moderate ascites. Stool examination is positive for occult blood,
What is the main cause for that case?
a) Acute kidney failure
b) Hepatic encephalopathy
c) Cystitis
d) Stroke
e) Reye's Syndrome
115. How to measure this case improvement
a) FBC (Full Blood Count)
b) LFTs (Liver Function Tests)
c) When patient become conscious (due to decrease of ammonia circulating in plasma)
d) WCC (White Cell Count)

116. When you say the treatment is effective?


a) Weight decreased
b) Ability to eat increase
c) Improve cognitive function

117. Patient on specific medications for 6 months and now he had no refill, his pharmacy is in a
province where pharmacist regulation allows him to do an extra refill, what will pharmacist do?
a) Do an extra refill for him and dispense
b) Contact doctor to ask extra refill
c) Ask patient to visit his doctor for follow up
d) Take a written consent from the patient and dispense

118. What is the true Tallman lettering?


a) DOPAMine, DOBUTAMine.
b) DOPamine, DOBUTamine.
c) DObamine, Dobutamine

119. A woman with UTI has fever and chills taking OC what is the causative microorganism.
a) Escherichia coli
b) Pseudomonas Aeruginosa.
c) Moraxella catarrhalis

120. A pregnant woman with UTI, what to give her


a) Amoxicillin
b) Cefixime
c) Ciprofloxacin
d) Nitrofurantoin

121. Patient was diagnosed with UTI, which is the probable microorganism.
a) Klebsiella pneumonia
b) Streptococcus
c) Staph Aureus
d) Legionella
E. coli (80–90%), S. Saprophyticus (5–10%), K. pneumonia, Proteus mirabilis
122. Which is the most common side effect for Champix?
a) Hypertension
b) Nausea (affecting about three out of 10 people)
c) Peptic ulcer
d) Muscle cramps

123. Selenium you have 0.5 micromole/L, your stock is 0.25 micromole/L. How much of 40
mcg/ml of selenium would you add if you
want to make 482L (M wt of selenium 79)
a) 0.4
b) 0.3
c) 0.34
d) 0.24

124. You got the following Rx, what will be the final amount of mometasone cream?
10% Drug X 1% Drug Y 0.5% Tretinoin cream aa Mometasone cream M: 60 gm
Answer:
Drug X = 10 % * 60 gm = 6 gm Drug Y = 1 % * 60 gm = 0.6 gm
Remaining = 60 - 6 - 0.6 = 53.4 gm
Tretinoin cream = Mometasone cream Mometasone cream = 53.4 gm / 2 = 26.7 gm

125. Preparation of 5% stock solution, you want to make 0.0125% so you use.
a) 5 ml in 2 liters
b) 5 ml in 1 liter
c) 2.5 ml in 2.5 liter

126. A physician prescribed a fortified eye drops for a patient Tobramycin 13.5mg /ml gtt ou
twice daily. How much of the 40 mg/ml stock solution should be added to 5ml of 0.3% to get
the desired concentration?
Answer:
0.3 % means 0.3 gm ‐‐‐‐‐‐‐ 100 ml X gm ‐‐‐‐‐‐‐ 1 ml
X = 1*0.3/100 = 0.003 gm = 3mg So 0.3% = 3 mg / ml
by allegation method:

X = 5 * 10.5 / 26.5 = 1.98 ml


127. Patient went to her doctor and he found her T-score to be -3. She has a family history of
breast cancer. She is post-menopausal for 6 years what else would you need to know to assess
her fracture risk
a) Low trauma fracture
b) Vitamin D and vitamin C intake
T score: Less than ‐2.5 indicate osteoporosis. Substantial loss of bone density means a much higher risk of
having a fracture with minimal trauma. If k‐type will be both

128. What is the most suitable treatment?


a) Teriparatide
b) Raloxifene
c) Estrogen
d) Etidronate

129. Goal of therapy for osteoporosis?


a) Quality of bones
b) Increase BMD
c) Improve quality of Life
d) Quantity of bones
e) Prevent fractures

130. Patient is receiving insulin and his blood glucose level are well controlled his morning
level is 5.6 mmol and his night glucose level is 6.3m.mol, what may be the cause for his 3am
glucose level to be 11.3 mmol?
a) Gestational diabetes
b) Post prandial effect
c) Somogyi effect
d) Down phenomenon
If the blood sugar level is high or normal at 3am, it is the dawn phenomenon (a rise in blood sugar as a
person's body prepares to wake up). If the blood sugar level is low at 3am, it is the Somogyi effect.

131. A diabetic patient is using Humalog 25 Mix (25% Humalog lispiro and 75% Humulin N).
He is taking 40 units of Humalog Mix before breakfast and 30 units before supper. He called
today to get his Humalog Mix. However, all of Humalog is back order. What is your best
recommendation as a pharmacist to this patient as it’s the time for his breakfast dose?
a) Give him 10 units of Humalog lispro and 30 units of Humulin N
132. What is correct about horizontal laminar flow?
b) It is the most effective technique
c) Must not interfere with the airflow behind the object
d) Can place non-sterile object into the hood

133. Which of the following will require being prepared under the laminar flow hood?
a) Vasopressin syringe of 20 units planned for direct IV injection for heart surgery
b) Preparation of 50ml of ceftriaxone to be given as infusion for a hospitalized child to
control his pneumonia
c) Preparation of eye drop form to be used after eye surgery
d) Preparation of a drug solution used as enema

134. Contamination of laminar flow done by? Operator hands

135. In case of Dementia, patient should monitor all EXCEPT?


a) Vit B12
b) Folic acid
c) MMI (Mild Memory Impairment)
d) TSH
e) Electroencephalogram
CBC, electrolytes, kidney function, TSH, vitamin B12, folate (optional), calcium, blood glucose.

136. Patient diagnosed with dementia treated with Donepezil. When we can monitor the
improvement?
a) 2 weeks
b) 3 - 6 months
c) 1 month
d) 24 months
Donepezil was effective in 3‐6‐month trials in patients with mild to moderate Alzheimer's disease (MMSE
scores 10 to 26). Define and monitor target symptoms. Effectiveness is considered to be either improvement
or no change in target symptoms. Monitor treatment effects 2 weeks after initiating therapy or increasing
dosage, then every 3 months.

137. Trazodone is used for Dementia with agitation. Also used for sun downing

138. What do you counsel on a patient newly diagnosed with dementia starting a cholinesterase
inhibitor?
a) If you fail one medication in this class you will probably fail all medications within class
b) Ginkgo biloba will enhance the efficacy of this medication
c) Decrease the Repetition of question is a good goal of treatment
139. K.J 63-y patient with sulfa allergy and dementia came with a new prescription donepezil
10 mg/day. He is talking hydrochlorothiazide and ACEI. What is the problem in this case?
a) Initial dose of Donepezil is high
b) Drug interaction between Donepezil with hydrochlorothiazide No Interaction
c) Donepezil dose should start with 20 mg/day
d) Donepezil not recommended over 60 y

140. Which eye drop may cause cross allergy for the same patient?
a) Dorzolamide
b) Timolol
c) Naphazoline Hydrochloride
d) Latanaprost

141. A 65 years old asthmatic patient has sulfa allergy and taking Timolol for glaucoma, what is
the main interaction that would concern the pharmacist?
a) Timolol/Asthma
b) Timolol/ Sulpha allergy
c) Timolol and old age
d) Timolol not the drug of choice in Glaucoma

142. A provincial drug plan not approving a chemotherapeutic drug because of lack of clinical
trials on the drug, they follow which ethical principal?
a) Non maleficence
b) Conflict of interest
c) Veracity
d) Autonomy

143. Thiazide increase what


a) LDL
b) TG
c) HDL & TG
d) HDL
e) LDL and TG
Indapamide is the only thiazide diuretic that has not been shown to raise cholesterol levels.
144. Circumstances when confidential patient information may be disclosed to third parties
include all EXCEPT
a) Release of the information is required by Rules of Court
b) Inspector authorized under The Pharmacy Act, 1996
c) Health care professionals for bona fide medical
d) First degree family member
e) A police officer presents a warrant

145. Canada Health Act essential services in the hospital are funded by
a) Fully funded by federal
b) Federal, provincial, and insurance and our taxes
c) Funded by Federal & Tax
d) Fully funded by provincial only

146. Patient using Tetrahydrozoline eye drops now for his dry and irritated eye. Frequent use
over a long period of time may aggravate the problems to cause
a) Corneal ulcer
b) Iritis
c) Rebound hyperemia
d) Retinopathy

147. What is the cause of this case?


a) Tetrahydrozoline (like nasal decongestant, not more than 3 days, rebound
congestion- sympathomimetic effect, vasoconstriction)
b) Zinc sulphate
c) Benzalkonium chloride
d) Sodium cromoglycate

148. Levetiracetam was given instead of Levofloxacin by mistake and patient on warfarin and
has bleeding, as a pharmacist. What you have to do?
a) Monitor bleeding
b) Give Vit K injection
c) Stop Warfarin
d) Send to emergency to assess
Levetiracetam (1000 mg bid) did not influence the pharmacokinetics of R and S warfarin (2.5 mg, 5 mg or 7.5
mg daily). Prothrombin time was not affected by levetiracetam. Coadministration of warfarin did not affect
the pharmacokinetics of levetiracetam. Blood methotrexate and levetiracetam levels should be carefully
monitored in patients treated concomitantly with the two drugs.
All patients treated with antiepileptic drugs, irrespective of indication, should be monitored for signs of
suicidal ideation and behavior and appropriate treatment should be considered. Patients (and caregivers of
patients) should be advised to seek medical advice should signs of suicidal ideation or behavior emerge.

149. While technician adding a Levetiracetam Rx to patient data on the computer, he wrote it by
mistake Levofloxacin at computer system. Before you dispense to the patient, how you can
prevent the error?
a) Check stock bottle with patient vial
b) Check patient vial with original Rx
c) Double check for vial by Pharmacist & technician
d) Check stock bottle with patient profile
There is a completely different version of this Q in which this answer is NOT available.

150. In the above case, how to prevent this error in the future
a) Put label on each one
b) Space on shelf
c) Tallman lettering
d) Double check

151. The most powerful evidence-based studies are


a) Cohort
b) Case review
c) Case series
d) Case control

152. 1-year child with fever, what is the best way to measure his temperature?
a) Rectal
b) Oral
c) Forehead
d) Axillary

153. For the same child, what is correct statement of the following?
a) Axillary temperature is higher than the oral temperature
b) Rectal temperature is higher than the oral temperature
c) Oral temperature is higher than the rectal temperature
d) Rectal temperature is the same as oral temperature
A rectal temperature is 0.5°F (0.3°C) to 1°F (0.6°C) higher than an oral temperature.
An axillary temperature is usually 0.5°F (0.3°C) to 1°F (0.6°C) lower than an oral temperature.
154. Hepatic patient with no intracellular involvement and was diagnosed with cholestatic
jaundice what to monitor? ALP test performed to diagnose liver or bone disease.
a) Albumin Normal range is 44 to 147 IU/L, Higher‐than‐normal
b) ALP (Alkaline phosphatase) levels observed in Hepatitis & Biliary obstruction
c) Bilirubin
d) AST & ALT

155. Patient bringing methadone prescription from unauthorized doctor, her family doctor on
vacation for three weeks, what is appropriate action of pharmacist?
a) Refuse to fill the RX
b) Tell new doctor to register temporarily
c) Phone to her family doctor and request prescription
d) Fill the RX

156. An 85-year-old patient complain about taking him one hour to fall asleep and he also has
osteoarthritis and other diseases and he wanted a medication to help him initiate sleep without
hangover in the morning, what is your best recommendation?
a) Oxazepam
b) Zopiclone
c) Flurazepam
d) Lorazepam
TOT: Temazepam, Oxazepam, Triazolam => No hangover

157. Patient with asthma. She is taking salbutamol and


fluticasone. Patient has an exacerbation of asthma.
What do you do?
a) Initiate PO prednisone
b) Replace salbutamol with Salmeterol
c) Something about Tiotropium
d) Change to mometasone
e) Replace salbutamol and fluticasone with discus (fluticasone + Salmeterol)

158. Uncontrolled asthmatic patient come with prescription include handihaler (Spiriva). What
will you do as a pharmacist?
a) Prime it many times in front of him
b) Advise him to wash his mouse after each use
c) Ensure that he shake the handihaler before use
d) Use demo to show him how to use it
Avo haler and inhaler: Shake. CONST D  don’t shake ‫ﻛﻮ ﺴﺖ دى‬
C: Calcitonin, O: Otrivin, N: Nitroglycerin, S: Spiriva, Stadol, T: Turbohaler, D: Diskus
159. A patient is taking digoxin; all can put him at risk for digoxin toxicity EXCEPT?
a) Hepatic impairment
b) Hypokalemia
c) Renal impairment
d) Atrial fibrillation

160. Digoxin side effects? Visual disturbance


Anorexia, nausea, vomiting, visual disturbances, fatigue, dizziness, confusion, delirium; cardiac arrhythmia.
Noncardiovascular adverse effects do not always precede potentially fatal arrhythmias.

161. Framingham score depend on all EXCEPT?


a) Systolic blood pressure
b) Total cholesterol
c) Smoker
d) Family history also C-reactive protein
e) HDL cholesterol

162. You have a stock solute of 10% W/V you are required to dilute it as 1:5 by adding suitable
diluent, the total final volume of solution is 5 ml. what is the appropriate volume of diluents and
stock solution you need to make the required solute
a) 4ml diluent and 1ml stock solution
b) 4.5ml diluent and 0.5 ml stock solution
c) 4.9 ml diluent and 0.1 ml stock solution
d) 1ml diluent and 4 ml stock solution
163. Patient receive 1 gm Vancomycin IV, T ½ of Vancomycin is 3 days. After 3 days later, he
received 1 gm Vancomycin IV, the blood concentration was 15mmol/L what is the steady state
trough concentration of Vancomycin
a) 15 mmol/l
b) 30 mmol/l
c) 45 mmol/l
d) 60 mmol/l
e) 90 mmol/l
If same dose double concentration, if different dose multiplies conc x1

164. Patient has Endocarditis, which organ will be affected


a) Heart
b) Liver
c) Kidney
d) Teeth

165. As a preventive measure, people with heart valve abnormalities, artificial valves, or
congenital heart defects are given antibiotics as a prophylaxis against Endocarditis for all
EXCEPT
a) Placement of dental implants
b) Removal of tonsils or adenoids
c) Left ventricular hypertrophy (LVH)
d) Lung surgery
e) Open-heart surgery
EXCEPT: LVH
- CABG

166. What is the true Tallman lettering?


a) VINCRIstine  VINblastine
b) vinCRIStine  vinBLAStine‫ﻛﺮﻳﺲ ﺑﻼﺱ‬
c) VINCRIstine  VINBLAstine

167. SOAP in pharmaceutical care is


a) Subjective, Opportunity, Assessment, Plan
b) Subjective, Objective, Action, Plan
c) Subjective, Opportunity, Action, Process
d) Subjective, Objective, Assessment, Plan
168. What could be considered as P in the SOAP?
a) Initiate the treatment
b) Ask about Patient symptoms/side effects
c) Do blood work
d) Patient history

169. Using SOAP note, the patient medications should be listed under:
a) Subjective
b) Objective
c) Assessment
d) Plan

170. What could be considered as O in the SOAP?


a) Initiate the treatment
b) Ask about Patient symptoms/side effects
c) Lipid profile (LDL)
d) Patient history
S: Smoking history, angina symptoms, patient medications
O: Monitoring parameters like Exercise tolerance, lipid profile, lab test, blood glucose, Blood pressure,
heart rate, Cholesterol level, Vital signs
A: NYHA, Framingham score, Interaction between two drugs
P: Initiation therapy, new medication

171. When there is a dispensing error in the hospital, the hospital will do report to
a) ISMP (Institute for Safe Medication Practices Canada)
b) NSIR (National System for Incident Reporting)
c) Health Canada
d) CADTH (Canadian Agency for Drugs and Technologies in Health)

172. 15-month-old child and has meningitis, and they did spinal fluid test.
What do you suggest for this child?
a) Ceftriaxone + Vancomycin (CV)
b) Vancomycin + Aminoglycoside + Ceftriaxone (VAC)
c) Aminoglycoside + Ceftriaxone (AC)
d) Aminoglycoside + Vancomycin
173. How we know that, the child gets better, if there is decrease in
a) Lactate
b) Glucose
c) Red Blood cells
d) White Blood cells

174. 34-year-old otherwise healthy patient


with acute community acquired bacterial
meningitis, What would his CSF reveal?
a) Increased glucose in his CSF
b) Increased protein Also increase WBC
c) Decreased white blood cells
d) Decreased red blood cells

175. What is the most likely causative organism for infection? Culture revealed gram +ve cocci
a) Strep pneumonia (gram + cocci in meningitis)
b) Hem. Influenza (gram -)
c) N. meningitis (gram -)
d) Listeria monocyte (gram + baccillis rods)
e) Staph aureus (gram + cocci but wouldn’t be in meningitis)

176. What is the best empiric therapy for this patient until culture sensitivity tests are drawn?
a) Pipracillin/Tazobacatam
b) Ceftriaxone and vancomycin
c) Ampicillin and ceftriaxone

177. What is the cornerstone test in the prognostic evaluation of cirrhotic pts in the hospital?
a) Framingham test score
b) Child-Pugh score
c) CHAD test
d) PSI

178. Which prescription wrote in right way (less danger)?


a) Vitamin D 1000 IU daily
b) Digoxin 0.5mg po daily
c) Timolol QD
179. Your advice for patient with blood pressure 175/110? Super gift question
a) Visit your doctor
b) Go to ER (Emergency Room)
c) Sleep well and take anxiolytic
d) Control your salt intake
e) Stop smoking

180. Patient profile with some medication, where is the drug interaction between
a) Viagra + Co-trimoxazole
b) Omeprazole + Clopidogrel
c) Omeprazole + Viagra
d) Metronidazole + Calcium
PPI + Clopidogrel & Antacid + ticlopidine

181. BPH Patient takes testosterone; his case is worsening, as he cannot ride the Bicycle
What the doctor will do?
a) Decrease testosterone
b) Stop testosterone
c) Increase testosterone
d) Prescribe 5α-reductase inhibitors (5-ARIs)

182. Effect of Finasteride in the benign prostatic hyperplasia (BPH) will be after
a) 1 Week
b) 2 Weeks
c) 4 Weeks
d) 3 months
e) 6 months

183. Patient taking Propylthiouracil (PTU) for hyperthyroidism. Why she takes this medication?
a) To prevent the transformation T3 to T4
b) Prevent the absorption of thyroid hormone
c) Inhibit the synthesis of thyroid hormone
d) To prevent the transformation T4 to T3

184. When to monitor Thyroid Stimulating Hormone (TSH)


a) 2 weeks
b) 6 weeks
c) 3 months
d) 6 months
Thyroid function should be assessed every 6‐8 weeks until the patient is euthyroid and then rechecked
annually, aiming to maintain T4 and TSH within the normal range

185. Patient painting, and he is tired when he walks few steps or when he climbs the stairs, he is
classified according to NYHA score as
a) NYHA I
b) NYHA II
c) NYHA III
d) NYHA IV

186. Patient then develops a rash on his arm and started experiencing acute pain on the site. He
was prescribed Acyclovir orally and started his therapy, what may help with his symptoms?
a) Topical acyclovir will help prevent him from developing neuralgia
b) Topical acyclovir will treat the rash and increase healing time
c) Tell him to avoid contact with anyone who have not had chicken pox before
d) Give him a standard dose opioid for his acute pain
To decrease pain, give opioid or tramadol

187. What should you tell him?


a) In the future stay away from anyone who has not had chicken pox yet
b) It can be transmitted through the air
c) It’s contagious before the appearance of lesions
d) You may still experience pain after the blisters heal (Post Herpetic Neuralgia)

188. Patient has active stage of herpes zoster virus (Shingles) which is NOT suitable choice.
a) Acyclovir
b) Capsaicin cream (not in active stage, but used in post herpetic neuralgia)
c) Calamine lotion
d) Take a cool bath
e) Topical antibiotics

189. Medications treat post herpetic neuralgia pain may include


a) Standard dose opioid
b) Acetaminophen
c) 200 Ibuprofen
d) Celecoxib
Management of postherpetic neuralgia can be particularly difficult.
Treatments include gabapentin, pregabalin, TCAs, topical capsaicin or lidocaine ointment & botulinum toxin
injection. Opioid analgesics may be necessary. Intrathecal methylprednisolone may be of benefit.
190. What’s the role of capsaicin cream? Counteract
Post‐herpetic neuralgia (PHN) occurs as a complication of acute herpes zoster. The capsaicin 8 % (w/w)
dermal patch (Qutenza®) is a transient receptor potential vanilloid agonist that increases intracellular
calcium concentration, triggering calcium‐dependent protease enzymes to cause cytoskeletal breakdown.
This action is thought to lead to loss of cellular integrity and defunctionalization of nociceptor epidermal
nerve fibers. The capsaicin 8 % patch is indicated in the US for the treatment of neuropathic pain associated
with PHN. In pivotal, randomized, doubleblind, multicenter trials in patients with PHN, a single 60‐minute
application of the capsaicin 8 % dermal patch reduced mean numeric pain rating scale scores between
baseline and weeks 2–8 to a significantly greater extent than the low‐dose comparator patch (capsaicin 0.04
% w/w). The capsaicin 8 % (w/w) dermal patch was found to be safe and generally well‐tolerated. Most
commonly reported side effects were local, comprising dermal irritation, erythema, and pain at the site of
application. These effects were transient and mild to moderate in severity. Transient patch application‐
related pain was not a barrier to use and was managed with local cooling or oral analgesics in nearly all
cases. Thus, the capsaicin 8 % dermal patch appears to have a good safety profile, is well tolerated, and
offers effective topical therapy in some patients with PHN. This review aims to characterize the use of the
capsaicin 8 % dermal patch in the treatment of PHN.

191. Pregabalin may be used in all cases except


a) Hemodialysis
b) Hepatic disease
c) Blood disorders
50% of dose is removed by haemodialysis in the first four hours

192. Patient has cold sore & after 5 days she comes to the pharmacy for something to accelerate
the healing, what is your advice to accelerate the healing?
a) Don’t touch the sore and wash your hands frequently
b) Nothing can accelerate the healing after this period
c) Use sunscreen on lips
d) Change all towels, toothbrushes

193. You have a prescription with Menthol and camphor, which type of mixture is that
a) Liquid mixture
b) Eutectic mixture
c) Ionic Mixture
d) Liquid Crystalline Mixture

194. This mixture used for


a) Impetigo
b) Back pain
c) HerpesZoster
d) Pruritus
195. Patient consumes alcohol heavily, and he want to quit, where he will be honest.
a) If he said, I stole a dose
b) If he decreases the amount that he drinks
c) If he said, l am hiding a dose
d) If he said, I will go to be treated at the hospital

196. Where you can find the document for the medication not available in Canada?
a) CPS
b) eCPS
c) Martindale
d) TC

197. Reference for patient counselling:


a) e-CPS
b) Martindale

198. Which reference that’s not referenced


a) CPS
b) Merck mannual
c) TC
d) Remington
e) Mertindale

199. Nitroglycerin side effect all except?


f) Dizziness
g) Bradycardia
h) Mode alteration
i) Reflex Tachycardia
CVS: hypotension, reflex tachycardia, increase of anginal pain, palpitations and bradycardia. Syncope due to
vasodilation. CNS: headache, weakness, dizziness, apprehension and restlessness.

200. With the expanded scope, some provinces allow pharmacist to renew prescriptions, which
one of the following Canada health acts does this show
a) Affordability
b) Accessibility
c) Portability
d) Universality
201. Health Canada Act principles all are false except The five Canada Health Act principles provide
for:
a) Accountability Public Administration: The provincial and territorial
plans must be administered and operated on a
b) Publicity nonprofit basis by a public authority accountable to
the provincial or territorial government.
c) Universality Comprehensiveness: The provincial and territorial
plans must insure all medically necessary services
d) Affordability provided by hospitals, medical practitioners and
dentists working within a hospital setting.
Universality: The provincial and territorial plans
must entitle all insured persons to health insurance
202. Sertraline Drug interaction with? coverage on uniform terms and conditions.
Accessibility: The provincial and territorial plans
a) NSAIDs must provide all insured persons reasonable access to
medically necessary hospital and physician services
b) Tramadol without financial or other barriers.
Portability: The provincial and territorial plans must
c) BB cover all insured persons when they move to another
province or territory within Canada and when they
d) ACE travel abroad. The provinces and territories have
some limits on coverage for services provided outside
Canada, and may require prior approval for non-
emergency services delivered outside their
203. Sertraline counselling? jurisdiction.

a) Take with food to increase its bioavailability


b) Your mood will get better in 2 weeks

204. 19 years old patient taking escitalopram, what to monitor


a) Insomnia
b) Suicidal ideation
c) Hypertension
d) Ankle edema
Headache is most also nausea 22.7% > insomnia 10.9% and dry mouth 10.9 > dizziness10.1% diarrhea 10.1%.
It can also cause QT prolongation and remember SSRI causes hyponatremia (SAIDH disease) and sexual dys

205. Legal requirements for Tramadol? Tramadol is monitored opioid but not narcotic
It is schedule 1 in NAPRA with no special requirements

206. Female Patient want to buy herbal medication through the internet instead of physician Rx,
what you can tell her
a) Do not listen to the counsel of her Doctor
b) Check the price through many websites before you buy
c) Advise her to listen to her doctor
Home <135/85
207. Blood pressure targets in treated diabetic patients
General patient population <140/90
is
SBP <140
a) <125/75
Isolated systolic hypertension SBP <150 (if
b) <130/95 80 y or older)
c) <140/ 90 Diabetes mellitus Or Kidney
d) <130/80 <130/80
disease
208. Blood pressure targets in Isolated Systolic Hypertension is
a) <125
b) <130
c) < 140
d) < 150

209. Dose of hypertensive drug could be reviewed after


a) 1 week
b) 4-8 weeks
c) 2 weeks
d) 3 weeks

210. Obese Patient with high blood pressure what you will advise this patient?
a) Decrease Salt intake to reduce Sodium to reach 100mmol/day
b) He has to do aerobics for 30 min/day
c) Tell him to decrease his potassium less than 500 micromoles
d) Alcohol consumption should be no more than 3 alcoholic drinks per day
To decrease blood pressure, consider reducing sodium intake to 2g (5g of salt or 87mmol of sodium)/day.

211. Patient on Metformin 1500 mg po daily, Glyburide 2.5 mg TID also he has high blood
sugar at morning, what you have to do?
a) Increase Metformin
b) Increase Glyburide  Glyburide until 15mg/Day
c) Add another oral
d) Add insulin

212. This patient in addition, was taking SMX/TMP and starting feeling weak, dizzy and a
bunch of other hypoglycemic side effects. What is the DTP?
a) Sulfonylurea and SMX/TMP

213. When someone travel to a place where eating and drinking in locations associated with
poor sanitation and poor hygiene There is increase risk to which disease
a) Yellow fever
b) Salmonella
c) Rabies
d) Tick-borne encephalitis
e) Hepatitis A
214. Renal patient has a new Rx of aminoglycoside what you have to do.
a) Keep dose and increase the interval
b) Keep interval and decrease dose
c) Decrease interval OR decrease the dose
d) Keep interval and Keep the dose
For regular form  keep the dose and increase intervals
For Extended intervals form (Q24hr)  decrease dose and keep interval
Creatine clearance < 40–50 mL/min: Extended‐Interval (Once‐Daily)
Dosing of Amikacin is not recommended in Adults

215. Ciprofloxacin, Ofloxacin dose in Adult Patients with Renal Impairment


a) Keep interval and decrease dose

216. Levofloxacin Dose in Adult Patients with Renal Impairment


a) Increase interval and decrease dose
Aminoglycosides, flouroquinolones & Azithromycin are the classes of antibiotics which show concentration
dependent killing of organism. In renal disease increase intervals of drug & Keep the same dose

217. Norfloxacin Dose in Adult Patients with Renal Impairment


a) Increase interval and keep dose
No dosage adjustment is required for moxifloxacin in renal impairment.
Clarithromycin is principally excreted by the liver and kidney. In patients with a combination of hepatic (mild
to moderate) and renal impairments or in the presence of severe renal impairment, decreased dosage of
Clarithromycin or prolonged dosing intervals might be appropriate

218. Levofloxacin affected by:


b) Ca+
c) Mg
d) K+

219. Patient was diagnosed with Rheumatoid Arthritis and he will start Leflunomide and taking
SSRI and Atorvastatin. What is the correct action by pharmacist?
a) Avoid Leflunomide use with SSRI
b) Call doctor to highlight that SSRI may increase the effect and side effects of statin
c) Call doctor to highlight drug interaction between Atorvastatin and Leflunomide
d) Decrease the dose of SSRI before starting Leflunomide
Concomitant use of other agents known to induce hepatotoxicity may potentiate the risk of liver injury
associated with Leflunomide
220. Monitoring patients taking statin by
a) CBC
b) CK (Creatinine Kinase)
c) Myopathy
Check Liver function (ALT) at beseline & at least once at 3 months. Check creatine phosphokinase levels CK if
myalgia develops. Monitor for adverse effects in patients with moderate/severe renal impairment (<60
mL/min). Monit or patients for signs of cognitive impairment and treatment modified if appropriate.

221. Pneumocystis Jirovecii Pneumonia (PJP) has been reported to be a leading cause of death
in HIV-infected infants. Which of the following could be used as prophylaxis against PJP?
a) Amoxicillin
b) Folic acid
c) Vit E
d) Co-trimoxazole

222. Osteonecrosis of the jaw (ONJ) has been increasingly suspected to be a potential
complication of
a) Oral Bisphosphonate
b) IV Bisphosphonate for one week
c) Not taking enough Ca and Vit. D
d) Teriparatide
e) Raloxifene
Osteonecrosis of the jaw (ONJ) is a recognized risk in patients receiving high‐dose monthly IV
bisphosphonate therapy as part of the treatment of several cancers

223. You discovered drug problem with new Rx for one of your customers at your pharmacy,
you told the patient that you should call the doctor. After your call, the doctor insisted to give
the medication for patient, what you have to do.
a) Don’t dispense
b) Dispense the medication
c) Don’t dispense and call another doctor to discuss
d) Dispense medication after you document what happened between you, pt & doctor

224. Which of the following organizations aim to enhance the safety of the medication use
system for Canadians?
a) ISMP (Institute for Safe Medication Practices Canada)
b) CMIRPS (Canadian Medication Incident Reporting and Prevention System)
c) CIHI (Canadian Institute for Health Information)
d) HC (Health Canada)
225. Patient come to your pharmacy and tell you he wants share his medical information with
another doctor outside the country, what you will do?
a) Refuse
b) Ask for a consent from his doctor
c) Ask for approval from his family doctor first
d) Accept without a consent from the doctor

226. There is a common hospital dispensing error between Morphine oral & IV, how you can
avoid?
a) Create a label for the Morphine oral only
b) Space (one in the pharmacy, the other in the ward)
c) Make a label for the IV only
d) Initiate training for pharmacy staff

227. Xerostomia is associated with use of


a) ACEI
b) Metformin
c) Antipsychotic
d) Aminoglycoside
Also, Clonidine, Claritine

228. Main reason for dispensing error in community pharmacy


a) Noise
b) Lack of training
c) Work load
d) Lightening system

229. Why is it important to perform medication reconciliation, all are important EXCEPT?
a) To create a complete list of the medications the patient takes on regular basis
b) To reduce adverse drug events due to drug discrepancies
c) To review total cost of received therapy
d) To ensure the patient is receiving appropriate therapy

230. What is NOT related to PIPEDA (Personal Information Protection & Electronic
Documents Act) rules
a) Don’t disclose personal information to anyone
b) To disclose personal information to third party
c) Use the personal information to be used in the pharmacy
As a consumer, you should be aware that there is nothing in the Personal Information Protection and
Electronic Documents Act (PIPEDA) that prevents organizations from outsourcing the processing of data.
However, organizations must take all reasonable steps to protect that information from unauthorized uses
and disclosures while it is in the hands of the third‐party processor

231. Side Effects of Renvela tablet (Sevelamer Carbonate)


a) Nausea 25% N&V
b) Insomnia
c) Ascites
d) Neuropathic pain
Sevelamer is used to treat high blood levels of phosphorus in patients with kidney disease who are on
dialysis. It binds with phosphorus that you get from food in your diet and prevents it from being absorbed
into your blood stream.

232. Which of the following references can be used for IV incompatibility?


a) Drug Facts and Comparison
b) Micromedex
c) PDR (Physicians' Desk Reference)
d) AHSF American Society of Hospital Pharmacists

233. All can be used for Drug-Drug interaction EXCEPT


a) CPS
b) Lexicomp
c) Remington (used for compounding)
d) TC

234. What is the most reliable reference to check the best regime for
diabetic patient taking 2 different types of insulin?
a) eCPS
b) Clinical Practice Guidelines
c) Martindale

235. Patient is asking you what medications are available in Canada for treatment of urinary
incontinence:
a) Therapeutic choices for minor ailments
b) CPS
Written in the chapter of urinary incontinence in PSC for available products consult incontinence products
devices in compendium of self‐care products (products found in the pink papers therapeutic guide)
236. Where you can check solvent stability of IV Diphenhydramine
a) Manufacturer
b) Martindale
c) CPS
d) The King Guide

237. Comparison between drug A & Drug B where Drug A Drug B


done at ER (Emergency Room). Cardiac events 15 300
Calculate the number will benefit from the drug A
Death events 35 25
compared to B when treating 1000 patients
Calculate ARR & NNT for each drug?

238. Upon receiving notification of drug recall, pharmacist may do all the following EXCEPT:
a) The pharmacist should communicate the information about recall with pharmacy staff
b) Print copy of the recall notifications and post it in a place where patient can see it
c) Technicians should check available stock and remove the recalled product off the shelf
d) Pharmacist should select and stock an interchangeable alternative to the recalled drug
e) Pharmacist may contact health practitioners to suggest an alternative therapy

239. First-line triple therapy for Eradication of Helicobacter Pylori consists of any PPI plus
Clarithromycin and amoxicillin/Metronidazole administered for how long
a) Once daily for 10 days
b) BID for 1 week
c) Once daily for 2 weeks
d) BID for 10-14 days

240. Treatment of Helicobacter Pylori for 10 days with Omeprazole BID + Clarithromycin 250
mg BID + Metronidazole 250 mg BID, what is correct statement
a) High frequency of PPI, it should be once daily
b) Low dose of Metronidazole, it should be 500 mg
c) 10-day treatment is short, it should be 21 days
d) Low dose of Clarithromycin, it should be 500 mg

241. Pt above is unable to tolerate his medication regimen due to side effects. Since they are
expensive, he comes back asking for a refund. What is the most appropriate response?
a) It is illegal to give a refund for dispensed medications
b) Tell him to send the medications back to the manufacturer and ask for a refund
c) The pharmacy can give him a refund and return the medications to stock
d) The pharmacy can give him a refund but cannot return the medications to stock
242. A patient at a Hospital accidentally received an incorrect narcotic drug (Hydromorphone
instead of morphine) what is the correct action
a) Train all pharmacy staff to be attention
b) Use florescent label & physically separate
c) Double check before dispense
d) Disciplinary Action and Warning for all staff
http://www.ismp.org/newsletters/acutecare/articles/20040701.asp
I read the attachment ISMP the first solution after limited access is to separate them so it depends in the
case. If they are injections in the hospital and they are in the same drawer so separate them if they are in
the pharmacy and separated already then you can use tall man letters “HYDROmorphone – morphine”.
Therefore, it depends on the case scenario, which will be very clear in the exam

243. Seven years old boy coming with diarrhea


for 2 days, what is the correct action?
a) Advice child mother to call the family
doctor
b) Refer to ER
c) Advice to increase the fluid intake
(fluids or ORS)
d) Dispense OTC antidiarrheal
244. What is the side effect linked with Dextromethorphan addiction?
a) Emesis
b) Sedation
c) Visual Hallucination
d) Headache

245. Patient heard about a new vitamin from TV show, and you told him that this vitamin has
no evidence. He accepted your recommendation and did not buy the vitamin. You followed:
a) Fidelity
b) Beneficence
c) Veracity
d) Justice

246. A lady came to your pharmacy to ask about herbal product, you can search using
a) DIN DIN (Drug Identification Number
b) NOC NPN (Natural Product Number)
NOC (Notice of Compliance)
c) NPN VMD (Veterinary Medicines Directorate)
d) VMD

247. Patient suffer from diabetic foot and ulcer, while doctor inspection he discovered that it
reaches to the bone. This called?
a) Cellulitis
b) Osteomyelitis
c) Tendinitis

248. Which is the proper treatment


a) Cefazolin IV, first generation
b) Cephalexin (oral option) for several months
c) Cefamandole 2nd generation
d) Cefuroxime 2nd generation

249. The duration of treatment is


a) 2 – 3 weeks
b) 4 – 6 weeks (parenteral)
c) 3 months
d) 6 months
250. Which medication with auxiliary label that to avoid excessive caffeine intake?
a) Cotrimoxazole
b) Ciprofloxacin
http://www.drugs.com/caffeine.html
Other xanthine derivatives: On concurrent administration of ciprofloxacin and caffeine or pentoxifylline
(oxpentifylline), raised serum concentrations of these xanthine derivatives were reported.

251. Patient taking Phenytoin along with chronic medications (HTN, etc.) and he developed a
seizure. They show you his lab results and Phenytoin levels were in range last week and when
admitted to the hospital they are no longer in range. The only difference to his medication list is
that he was started on Cipro 3 days ago. What caused his Phenytoin levels to drop?
a) Missed a dose
b) Cipro interaction with Phenytoin
c) Renal function
Cipro altered serum levels of the antiepileptic drugs Phenytoin and carbamazepine (increased and
decreased) have been reported in patients receiving concomitant ciprofloxacin
Simultaneous administration of ciprofloxacin and phenytoin may result in increased or reduced serum levels
of phenytoin such that monitoring of drug levels is recommended.

252. What antibiotic may increase the risk of tendon rupture & tendinitis?
a) Azithromycin
b) Ciprofloxacin
c) Gentamycin
d) Cefixime

253. Patient having many medications was not diabetic, but recently developed hyperglycemia,
what medication caused this
a) Ciprofloxacin
b) Ibuprofen
c) ACEIs

254. Patient with Hypercalcemia 3 mmol/L could be treated with (the question might be with all
except insulin: used in hyperkalemia)
a) Insulin
b) Pamidronate
c) Diuretics
d) Cinacalcet
Cinacalcet is for hypercalcemia due to parathyroidcanerand in case of renal failure
https://www.merckmanuals.com/professional/endocrine‐and‐metabolic‐disorders/electrolyte‐
disorders/hypercalcemia?query=hypercalcemia
255. Monitor of Hypercalcemia
a) Albumin
b) TSH
c) ECG
Laboratory evaluation:
 serum ionized calcium (preferred where available) or serum calcium and albumin; corrected Ca++
value (mmol/L) = (0.02 × [40 − measured albumin g/L]) + measured Ca++
 serum intact parathyroid hormone (PTH); perform this test before giving bisphosphonates, as they
can alter serum PTH levels
 serum parathyroid hormone‐related peptide (PTHrP) and 1,25‐dihydroxyvitamin D level if PTH low
 24‐hour urine collection to measure calcium‐to‐creatinine ratio (if familial hypocalciuric
hypercalcemia or milk‐alkali syndrome is suspected)
 serum phosphate, alkaline phosphatase, total protein, serum creatinine and urea

256. A Woman who are planning to get pregnant should ideally start taking folic acid
a) Three months prior to conception
b) From the first day once pregnancy confirmed
c) During the second trimester
d) During the Third trimester

257. Physician wrote a refills Rx eye drops for his sister’s dog and she came to your pharmacy
with the prescription, as the veterinarian is not available, what do you do?
a) Give her one refill as emergency
b) Deal with this situation according to your professional judgement
c) Send her to veterinarian clinic
d) Tell her to ask the doctor to write the prescription on her name
258. While their disease is active,
Rheumatoid Arthritis patients should
be seen as frequently as every
a) 2-4 Weeks
b) 1–3 months
c) 4-6 Month
d) 6-12 Month (this is for RA patients
if well controlled disease or in
remission)
Patients should be seen as frequently as
every 1–3 months while their disease is
active

259. Oral Contraceptives are affected by all of the following EXCEPT


a) Erythromycin
b) Clonidine
c) Phenobarbital
d) Gabapentin
Valproic, Gabapentin and Lamotrigine ‫اﻟﺴﺖ ﺗﺤﺐ اﻟﻔﺠﻞ‬

260. Patient with Trigeminal Neuralgia (TN) what is the drug of choice?
a) Oxazepam
b) Gabapentin
c) Naproxen
d) Oxycodone
e) Vit. B6
Trigeminal neuralgia is severe paroxysmal, lancinating facial pain due to a disorder of the 5th cranial nerve.
Diagnosis is clinical. Treatment is usually with carbamazepine or gabapentin; sometimes surgery is required.

261. Adverse effects of Gabapentin


a) Renal
b) Hepatic
Gabapentin SE: vision changes, weight gain, somnolence, nausea and/or vomiting, dizziness, ataxia, fatigue,
nystagmus and tremor ‫ﺟﺎب ﺑ ﺘ ﺗﺨﻦ واﺗﻌﻤﺶ واﺗﺮﻋﺶ وﺟﺎﻟﻮ وﺟﻊ ﻄﻨﻪ و ﻨﺎم ﻛﺘ‬

262. Canada Vigilance online database contains information about


a) Regulatory Requirements for Advertising
b) Clinic and hospital data
c) Drug approved recently
d) Suspected drug adverse reactions
263. Patient’s FEV1< 40, what does this mean?
a) Mild
b) Moderate
c) Severe
d) Very severe

264. 54 y Diabetic patient with acute exacerbations of COPD, which antibiotics should be used
a) Cephalexin
b) Ciprofloxacin
c) TMP/SMX  Sulfa drugs will potentiate hypoglycemia
d) Azithromycin
Simple Exacerbations (COPD without risk factors) Amoxicillin, doxycycline, TMP/SMX, or extended spectrum
macrolides
Complicated (COPD with frequent exacerbations): Amoxicillin/clavulanate, 2nd gen. cephalosporin or
Fluoroquinolones)
Among the antimicrobials investigated: ciprofloxacin, clarithromycin, levofloxacin, metronidazole, and
Sulfamethoxazole‐trimethoprim were significantly associated with hypoglycemia

265. Patient with COPD and is allergic to penicillin (rash when he was 7) he had comorbid
conditions, which parenteral drug should be given to him.
a) Pipracillin/Tazobacatam
b) Ceftriaxone plus Azithromycin
c) Doxycycline-TMP/SMX- 2nd or extended spectrum macrolides

266. He went back to his home what he should be use


a) Oral prednisolone 5 days
b) Oral prednisolone 2 weeks
c) IV prednisolone 5 days
d) IV prednisolone 2 weeks
Oral corticosteroids improve lung function and shorten length of hospital stay in all patients and reduce risk
of early relapse or hospitalization for subsequent acute exacerbations.
They should be given within 30 days of an acute exacerbation to reduce the risk of further exacerbations.
Administration beyond 30 days is not recommended. A 5‐day course of oral prednisone 30–40 mg/day or
equivalent is sufficient and has shown outcomes comparable to a 10‐ to 14‐day course. Tapering is
unnecessary for oral corticosteroid courses that last <2 weeks. There is no role for oral corticosteroid
maintenance therapy for patients with COPD. There is no advantage to using IV corticosteroids.
267. What could be used as add-on therapy with bronchodilators for maintenance treatment of
severe COPD associated with chronic bronchitis
a) Montelukast
b) Zafirlukast
c) Roflumilast
d) Ipratropium
e) Oxygen Therapy
Roflumilast (Phosphodiesterase 4 Inhibitors) is an oral long‐acting medication that suppresses the release of
inflammatory mediators. It is indicated as add‐on therapy with bronchodilators for maintenance treatment
of severe COPD associated with chronic bronchitis. Data support addition of Roflumilast to foundation
bronchodilatory therapy to reduce exacerbations, either before or in addition to introduction of ICS/LABA.
Roflumilast has demonstrated a significant improvement in prebronchodilator FEV1 (48–80 mL increase)
along with a small reduction in exacerbations requiring systemic CSs (8%), even for patients who were also
taking a LABA. Roflumilast may be considered for add‐on therapy to existing triple therapy (ICS/LAMA/LABA
or LAMA + ICS/LABA) for people with COPD who have had at least 1 exacerbation in the past year.

268. High-risk patient on warfarin and he is going to do surgery, the surgeon advice that he
should be 5 days warfarin-free prior to surgery, what is the correct recommendation?
a) Switch Warfarin to ASA
b) Switch Warfarin to Clopidogrel
c) Switch Warfarin to Dabigatran
d) Switch Warfarin to Enoxaparin (Low molecular weight heparins - LMWHs)
"High‐risk" = recent stroke or VTE, mechanical heart valve, etc.?!
Enoxaparin (bridging)
For high risk of thrombosis (including risk associated with preoperative and postoperative):
i. Discontinue warfarin at least 5 days prior to surgery. Therefore, give last dose on day 6 to achieve 5
warfarin‐free days if day of surgery = day 0. Unless target INR is 3.0 (range 2.5 to 3.5) stop 6 days
prior. Therefore, give last dose on day 7 to achieve 6 warfarin‐free days if day of surgery = day 0.
ii. If indicated, give therapeutic dose of LMWH on day 4, day 3, and day 2 in consultation with a
haematologist at the closest referral centre/major hospital or thrombosis clinic.
iii. Last dose of LMWH is generally not given any later than 24 hours before the procedure. Exact timing
of the last LMWH dose will depend on the type and dose of LMWH and risk of thrombosis.
iv. Check INR the day before procedure to ensure it is below the goal INR (< 1.5 for most procedures). If
INR is higher than goal INR, discuss with physician performing the procedure.
v. In the absence of an indicated procedure‐specific thromboprophylactic regimen, start prophylactic
dose LMWH 12 – 24 hours after surgery/procedure provided hemostasis is assured. Consider
escalation to therapeutic dose of LMWH starting postoperative day 3, if there are no bleeding
concerns. Discuss dose and timing of postoperative LMWH with surgeon.
vi. Restart warfarin at preoperative dose as soon as hemostasis is assured and only after epidural
catheters are removed post‐surgery.
vii. Continue LMWH until the INR is in therapeutic range.
269. What to monitor if patient is on enoxaparin
a) aPTT
b) INR
c) CK
d) Self-monitored

270. An officer came to your pharmacy asking for prisoner medical information, what is your
action?
a) Disclose the information as prisoner lost his confidentiality
b) Refuse unless you have a written consent from the prisoner
c) Disclose as the officer represent the regulatory authority

271. According to NAPRA Schedule, all considered as Schedule II EXCEPT


a) IV Dimenhydrinate
b) IV Cyanocobalamine
c) Intranasal mometasone  schedule I
d) Buccal nitroglycerine
https://napra.ca/national‐drug‐schedules?keywords=mometasone&schedule=

272. Which one of these is schedule I:


a) Oral potassium salt
b) Nitroglycerin spray
c) IV B12
d) Terbinafine Cream
e) Epipen
Potassium salt injection is schedule I, but oral potassium salt is schedule II

273. A patient with “osteoarthritis” and likes to swim 3times/week. She works as a hairdresser,
which is 2 km away from home. She smokes half a pack of cigarettes per day. She felt once
when she was young (7 years old) and broke her arm. She takes 1 cup of alcohol on weekends.
What is her risk factor for developing “osteoporosis”? (She had both conditions)
a) Smoking
b) Alcohol
c) Fracture history
d) Osteoarthritis
OA isn't a risk factor for OP, but RA. Smoking induce estrogen which have anabolic effect on bone
274. Osteoarthritis patient taking Celebrex 200 mg, what to monitor?
a) GI Bleeding
b) Leukotriene
c) LDL
d) MI (Myocardial Infarction)

275. Patient with menopause, vasomotor symptoms, hot flashes etc. She also has urinary
incontinence and insomnia, blood pressure 150/90, What treatment would be most effective for
her vasomotor symptoms:
a) Venlafaxine
b) Estrogen
c) Raloxifene
Vasomotor symptoms have a significant impact on quality of life. Intolerance to heat, flushing
and perspiration are uncomfortable and embarrassing. Hot flashes or night sweats can lead to
sleepless nights, fatigue and decreased productivity at work

276. What do you monitor to check improvement?


a) Sleep
b) Urinary incontinence

277. What would be the most important measure of success for this patient’s therapy?
a) Decrease in incontinence
b) Decrease in insomnia
c) Access her BMD
d) Access her blood pressure
e) Decrease hot flushes
276) Hot flashes (if insomnia is related which is likely, then insomnia)

277) Decrease in hot flashes

There's a bit of a "disconnect" between 276) and 277)

278. Patient with four migraine attacks per


month, with severe nausea and aura, she is
also taking combined oral contraceptive,
what indicates necessity for prophylactic
therapy?
a) Aura
b) Number of attacks

279. What is a DTP you notice in this case:


a) Possible Drug Disease interaction with
oral contraceptive
b) Contraindicated with migraine with aura

280. What the best recommendation for her case


a) Sumatriptan 100 mg tab then the second dose after 4hrs
b) Sumatriptan spray then the second dose within 4hrs
c) Zolmitriptan spray then the second dose within 2hrs from the first dose
d) Sumatriptan tablet
281. Patient had a lot of chest pain and pain due to fibromyalgia is uncontrolled and is on many
medications, medications were Bisoprolol 5mg daily, Amitriptyline 10mg prn. What may be the
possible reason for his symptom?
a) Bisoprolol is too low (5mg daily)
May be for chest pain (initial dose 5mg and usual 10mg and max 20mg)

282. Which one of his medications may at a suboptimal level?


a) Amitriptyline suboptimal 10mg daily (still pain)
I think prn is the problem causing fibromyalgia

283. Patient has hypertension, Raynaud’s, cold intolerance, with diabetes and has fibromyalgia
Which med is likely causing a therapeutic problem?
a) Metformin 1g bid (7yrs)
b) Amitriptyline 10mg (5 yrs.)
c) Bisoprolol 50mg (Tapered over past 6 months)  DM and Raynaud’s

284. Patient blood pressure is still high, what would you add on? Nifedipine XL

285. Patient got headache for two weeks? Refer to physician


Red Flags for Serious Headache
 Severe/abrupt onset (“worst headache ever”)
 Onset in middle age or older (>40 years)
 Neurologic signs: stiff neck, focal signs, reduced consciousness
 Systemic signs: appears ill, fever, nausea/vomiting (not explained by migraine or systemic illness)
 Significant change in pattern of headaches: increased frequency and/or progressive severity
 Nocturnal occurrence or on awakening in the morning; if patient consistently has headache on
awakening or is awakened by a headache, brain tumor is a possible cause; however, migraines
sometimes occur on awakening
 Onset with exercise or sexual activity (may be benign or serious)
286. Patient taking APAP (Acetaminophen and codeine) regularly for headaches and it is not
helping. Doctor suspects rebound headache. What would confirm that it is?
a) Using APAP 2-3 times a week
b) Waking up with a headache
c) Aura happening before headache
Overview. Medication overuse headaches or rebound headaches are caused by regular, long‐term use of
medication to treat headaches, such as migraines. Pain relievers offer relief for occasional headaches. But if
you take them more than a couple of days a week, they may trigger medication overuse headaches
Medication‐overuse Headache: CTMA
Medication‐overuse headache (MOH) is an under‐recognized condition that may occur in patients who
suffer from migraines (primarily) or tension‐type headaches. Recognition and treatment of MOH may lead
to long‐term improvement in headache relief and quality of life for many patients. Frequent use of
analgesics or other acute migraine medications for ≥3 months can lead to MOH (headache present on ≥15
days per month) in patients with migraine/tensiontype headaches. MOH can occur in association with
simple analgesics (e.g., acetaminophen, ASA) or, less commonly, with NSAIDs. It is more common with
combination products containing barbiturates, caffeine and/or opioids. Overuse of ergotamine or triptans
and withdrawal from substances such as caffeine, opioids and estrogen have also been implicated.
Treatment of medication‐overuse headache (MOH) involves discontinuation of the drug(s) that are
implicated, relief of withdrawal symptoms, use of migraine‐specific medications (adhering to recommended
limits in frequency of use) to treat recurrent headaches, and initiation of prophylactic therapy (e.g.,
divalproex sodium, propranolol, topiramate, tricyclic antidepressants). The causative agents are usually
stopped abruptly or sometimes tapered, while titrating prophylactic therapy. Advise patients not to
discontinue high doses of butalbital‐containing analgesics abruptly, since seizures may occur on withdrawal.
Ideally, refer these patients to a neurologist/headache specialist. Primary care healthcare practitioners can
play a key role in preventing and managing MOH. Monitor the patient's use of prescription and
nonprescription medications, counsel on appropriate use of antimigraine medications and provide support
to patients who are withdrawing from medications.
To prevent the development of MOH when treating primary headache disorders such as migraine or tension‐
type headache, use simple analgesics (e.g., acetaminophen, ASA) <15 days per month and combination
analgesics or opioids <10 days per month. Frequent use (≥10 days per month) of ergot or triptan medications
can also result in MOH. Headache associated with overuse of analgesics tends to resemble a tension‐type
headache (migraine headaches can also be superimposed) whereas triptan overuse manifests as increased
migraine frequency.

287. What would you recommend for this patient?


a) Taper APAP
b) Give them just acetaminophen and tell them only to take it when its needed
c) Tell them to use celecoxib when needed
d) Patient is advised you to discontinue the pain medications gradually
288. Patient profile with a lot of medication and he has Headache, when you can say that
medication accused of Headache?
a) If the patient wakeup and has headache
b) If there is noise
c) When he does exercise and activity
d) Migraine + aura
But if he says he wakeup from headache, its cancer is the reason

289. A woman using COC and she is suffering from headache that bothersome. She is using
acetaminophen plus codeine almost every day either to treat the attack or to prevent it. She got
nausea, vomiting and other bothersome symptoms. She complains that this headache is
interfering with her work, which of the following will describe that she is suffering from drug-
induced headache?
a) She is having 2- 3 attacks per week
b) She is using acetaminophen more often (>15 days)
c) She is experiencing aura symptoms
d) It’s due to her COC

290. What will be your best recommendation for her to overcome her drug-induced headache?
a) Ask her physician to taper the drug gradually
b) Ask her physician to change her to morphine then taper it gradually
c) Ask her to switch to naproxen prn
d) Ask her to take only acetaminophen tablets to control her headache.

291. 25-year-old woman, heterosexual, sexually active, came to the pharmacy complaining of a
greyish white discharge, bad odour, but very itchy area and bothersome. She is very impressed
for these symptoms to tell her partner. She was diagnosed for bacterial vaginosis experienced
such symptoms.
a) She is asking about the possible
source of her infection.
b) Gardenella vaginalis not sexually
transmitted disease

292. What is the recommended treatment


for her case?
a) Metronidazole cream Po or cream
+- clindamycin cream
293. A power outage happened due to heavy rain that water covered the power room. As a
pharmacy manager, what is the first important thing to do?
a) Call the power company to ask when the power will be back
b) Access a temporarily re-storage for the fridge & vaccines
c) Don’t do anything
d) Ask the technician to call the customer whose Rx are scheduled for soon refill
e) Ask for the night guard to be present during the power outage

294. A poor man lost his job, previous addict, with no insurance. He cannot pay for his
medications. To help him, you can do all of the following except:
a) Ask the physician to give him samples
b) Call the manufacture to provide samples
c) Offer him medications for free
d) Provide him with samples you got from a nearby physician

295. Patient started therapy on allopurinol when should her uric acid levels be measured?
a) 3 days
b) 7 days
c) 4 weeks
d) 8 weeks
e) 3 months
Maximal decreases of uric acid serum concentration in 1–3 weeks.
A patient should be started on 1 of the xanthine oxidase inhibitors and the dose should be titrated upward
every 2–5 weeks to an appropriate maximum dose in order to achieve the target serum urate level. Once the
serum urate target is reached, urate levels should be checked every 6 months.
296. Patient on amoxicillin, what isn’t a sign of anaphylaxis that require emergency:
a) Urticaria
b) Macupapular rash
c) Shortness of breath
d) Pharyngo-edema

297. Rx: Hydromorphone Mitte:cc Uc diet. What can you do?


a) Hold it as it’s clearly a fraud
b) Call the walk-in clinic to confirm the dose
c) Call the walk-in clinic to confirm direction
d) Call doctor to confirm prescription
e) Call the police to confirm fraud
Side effects of ZIDOVUDINE
298. What to monitor for a patient taking Zidovudine? ‐Haematological toxicity (Neutropenia, granulocytopenia mostly
after 6 weeks), Bone marrow suppression
a) CBC ‐Pancreatitis (monitor lipase) sensitization reaction
‐Change in skin, nail pigmentation
b) Serum Lipase ‐Increase in lactic acid level may cause lactic acidosis, sever
c) Serum Creatinine hepatomegaly and stenosis
‐Myopathy fat redisposition resulting in central obesity
‫ﺯﻳﺪﺍﻥ ﺑﻴﻠﻌﺐ ﻣﺎﺗﺶ ﻓﻲ ﻗﻨﺎﺓ ﺳﻰ ﺑﻰ ﺳﻰ‬

299. A patient has a history of seizures and is on fentanyl patches and naproxen BID prn. He can’t
tolerate morphine or hydromorphone and still in pain at night. What is the drug therapy problem?
a) Fentanyl will decrease his seizure control
b) Lack of control on acute pain
c) Fentanyl is not used for acute pain

300. What do you do?


a) Give him naproxen for breakthrough pain
b) Add Tylenol
c) Take off old patch and add new one
d) Give him morphine

299) DTP: missing rescue opioid for BTP (wearing off) = lack of control on "pain flare" (BTP).

300) Any IR painkiller for BTP = Naproxen for BTP

301. Academic detailing all except


a) Funded by pharmaceutical companies
b) Updates doctors
c) Government Funded
302. What is true about medical service in Canada
a) Doesn’t cover pharmaceutical medications

303. Define minor trauma fracture


a) Can have fracture from minor traumas

304. Patient with low bone density and in menopause,


what to do? Increase bone density to prevent fractures

305. When to monitor HbA1C? Every three months

306. What is true about Terazosin? Titrate dose slowly

307. Aboriginal patient has believed that hearing something bad can happen to him, what is
conflicting with the pharmacist when he is trying to council her
a) Confidentiality
b) Autonomy
c) Veracity
d) Justice
It is veracity because pharmacist will not know how to tell her side effects, he has to hide that from her and
not tell her truth

308. Patient is seeing two separate doctors; his family doctor and a specialist and tells the
pharmacist that he does not want one of his family doctors to know that he is seeing the other
doctor because his family doctor does not approve of seeing the other doctor and asks you not
to say anything. Which of the following are you upholding?
a) Veracity
b) Confidentiality
c) Autonomy
d) Justice
e) Nonmaleficence

309. Patient goes to two different doctors and is being prescribed different medications from
both of them. Doctor 1 calls you to check up on the profile. The patient tells the pharmacist that
he does not want you to tell Doctor 1 about Doctor 2 because Doctor 1 does not want the patient
to see any other doctors. If the pharmacist does not tell doctor 1 about doctor 2, what is he
upholding?
a) Confidentiality
b) Autonomy
310. You can report all the following to Canada Vigilance except
a) Investigational drugs

311. Latanaprost
a) Change iris color

312. Patient is fasting “like in Ramadan” and he didn’t take any anti-diabetic drugs, his blood
glucose level is high, what you will give him
a) Add NPH
b) Low dose Metformin

313. Step down therapy may be implemented in which of the following conditions
a) Pyelonephritis
b) Endocarditis
c) Meningitis

313. You discovered a narcotic steeling, when to report? Within 10 days

314. Patient is taking Ramipril, his prescription expired and his doctor was outside the town,
and he will see him after two days, what you should do
a) Advance him with two days’ supply

315. What to do with first time candidiasis female patient? Refer to physician

316. A patient with ASA overdose; his blood pH is 7.2, which of the following is true for him?
a) Respiratory alkalosis
b) Metabolic acidosis
c) Metabolic alkalosis
d) Respiratory acidosis
Normal arterial blood pH is restricted to a very narrow
range of 7.35 to 7.45. A person who has a blood pH below
7.35 is considered to be in acidosis (actually, “physiological
acidosis,” because blood is not truly acidic until its pH drops
below 7), and a continuous blood pH below 7.0 can be
fatal.
Salicylate poisoning can cause vomiting, tinnitus, confusion, hyperthermia, respiratory alkalosis, metabolic
acidosis, and multiple organ failure. Diagnosis is clinical, supplemented by measurement of the anion gap,
arterial blood gases, and serum salicylate levels. Treatment is with activated charcoal and alkaline diuresis
or hemodialysis.
317. Metabolic acidosis caused by all of the following except
a) Acetaminophen
b) Thiazides also furosemide cause metabolic alkalosis
Metabolic Alkalosis Medication
Carbonic Anhydrase Inhibitors. Acids. Potassium‐Sparing Diuretics. Angiotensin‐Converting Enzyme
Inhibitors. Potassium Supplements. Fluid Replacements. Corticosteroids. Nonsteroidal Anti‐inflammatory
Agents.

318. Patient has profile with medication, and you have to know the interaction?
Verapamil + Cardiac glycosides (digoxin)
Verapamil may increase the plasma concentrations of digitoxin and digoxin. Verapamil has been shown to
increase the serum concentration of digoxin and caution should be exercised with regard to digitalis toxicity.
The digitalis level should be determined and the glycoside dose reduced, if required.

319. Which of the following is a non-


pharmacological measure for controlling
heart failure
Salt restriction

320. Patient come to hospital he has rash and


his BP 80/40, what happened with him?
a) Thrombosis
b) Hepatic, renal problem
c) Bacterial problem Septic shock
https://www.merckmanuals.com/professional/critical‐care‐medicine/sepsis‐and‐septic‐shock/sepsis‐and‐septic‐
shock?query=septic%20shock
LITHIUM SE
Cardiac arrhythmia, hypotension, ECG changes
Renal dysfunction, polyuria, thirst, Headache, slurred speech, blurred vision,
321. Lithium causes all except: anesthesia of skin, seizures, Anorexia, nausea, vomiting, diarrhea, weight gain
a) Hirsutism Diabetes insipidus - General fatigue, Thyroid, parathyroid abnormalities,
thyroid dysfunction, Hypo, hyper glycaemia, diabetes insipidus
b) Hand tremors
All except LIVER
c) Thyroid dysfunction
Skin and subcutaneous tissue disorders: Allergic rash, exacerbation of psoriasis, acneiform eruptions,
alopecia, acne, papular skin disorder, folliculitis, pruritus, rash.

322. Drug used 2 g BID, 2 dose, and 1 refill how much he will get? 48000 = 96 tablets

323. 500mg and put into dextrose where to check? Drug manufacture monograph
324. Which statin can be taken with Phenytoin?
e) Rosuvastatin
f) Atorvastatin
g) Lovastatin
h) Simvastatin
i) Fluvastatin
Rosuvastatin and pravastatin no interaction with Phenytoin
DDI Phenytoin + ator/ fluva/ Simvastatin as per monograph

325. Patient was given Phenytoin 0800. At 1000, levels were 60. At 2200, levels were 45, what
is T ½?
d) 6
e) 12
f) 24
g) 29
log C=log Cₒ‐kt/2.303 log 45=log 60‐k X 12/ 2.303
k= 0.02397 t½ = 0693/0.02397=28.9 hr

326. Patient had back pain. Her job is stocking shelves. She has started swimming. What would
you tell her to do?
c) Tell her to continue her daily activities as long as it’s tolerable
d) Use a heating pad at night
e) Tell her to tell her to ask her manage
for a position where she is sitting
f) Tell her to stop swimming
327. Given lipid%, amino acid 7%, dextrose 20% at a rate of 62.5ml/hr and dextrose 4% at rate
42ml/hr, weight 65kg
Calculate total protein/kg daily?
Calculate total carbohydrate daily (1g=3.4kcal)
62.5 ‐‐‐‐ 1hr x ‐‐‐‐ 24hr x = 1500ml/day
Amino acid = protein
7 gm‐‐‐‐‐‐‐‐‐ 100 ml y‐‐‐‐‐‐‐‐1500 ml (same rate as dextrose 20%) y = 105gm
Y = 105gm / 65 kg (wt) = 1.6g protein/kg daily
Dextrose
62.5 ml‐‐‐‐‐‐ 1 hr 42 ml‐‐‐‐‐‐‐‐‐‐‐‐‐ 1 hr
x‐‐‐‐‐‐‐‐‐‐‐‐24 hr X‐‐‐‐‐‐‐‐‐‐‐‐‐‐ 24 hr
= 1500ml = 1008 ml
20g‐‐‐‐‐‐‐‐100ml 4g‐‐‐‐‐‐‐‐‐‐‐‐ 100 ml
X ‐‐‐‐‐‐‐1500ml X‐‐‐‐‐‐‐‐‐‐‐‐‐1008
= 300gm = 40.32gm
300+40.32 = 340.32g daily x (3.4 Kcal) = 1157 kcal/day (total carbohydrate daily)

328. You are the manager of a pharmacy and want to start vaccinating patients in your
pharmacy. What is the order of events in which you would make this happen?
I. Arrange for a physician to come give the vaccines
II. See the need for the vaccines in the clientele
III. Arrange the proper storage of the vaccines
IV. Order a supply of the vaccines
II, III, IV, I

329. Chemotherapy woman just was diagnosed with stage 2 breast cancer. She currently is
taking ondansetron and dexamethasone for acute nausea but she experiences nausea prior to her
chemotherapy when she drives by, what do you give her?
a) Lorazepam
b) Aprepitant
Lorazepam and alprazolam are the most commonly used, and have been studied in cases of anticipatory
nausea. They are usually used in combination with other antiemetics.

330. She just received her chemotherapy session and got a fever with Neutropenia. The doctor
wants to start her on empiric therapy without testing. What should be given?
a) Cefazolin
b) Vancomycin
c) Ciprofloxacin
d) Metronidazole
e) Piperacillin/Tazobactam
331. You are a pharmacist who has been asked to be part of a clinical trial. You are overseeing
the patients while they sign their consent forms to be part of the trial. As you read the form they
are signing, you notice that a major side effect is not included. How would you show non-
maleficence?
a) Tell the patients they should withdraw now
b) Let the trial researchers aware of the situation and ask them to revise consent form
c) Tell the patient are you sure you wanna join maybe you should google side effects

332. A breastfeeding woman and wants something to increase her breast milk she says that she
is breastfeeding, but her baby is not gaining any weight. She wants something that will increase
her breast milk.
a) Bromocriptine
b) Domperidone
c) Patient does not need medications
If the baby is gaining weight, then there's NO need for any meds.

333. Major side effect of capecitabine for colon cancer?


a) Increased infections
b) Skin and nail pigmentation
c) Liver cirrhosis
Capecitabine (Xeloda) serious side effects
‐Acute renal failure due to dehydration and may be fatal
‐Cardiotoxicity include MI, angina, may lead to sudden death
‐Hand and foot syndrome (painful swelling, erythema of hands and feet)
‐Nausea, Vomiting and diarrhea that may lead to dehydration
‐Stomatitis
‐Hematological problems (neutropenia, thrombocytopenia)

334. (Same patient as above) The patient was


prescribed Cipro for UTI 2 days ago and he
developed some weird rash or something on the
soles of his hands. What could be causing it?
a) Ciprofloxacin
b) Capecitabine
c) Tamsulosin
d) Ramipril
Hand-foot syndrome (HFS) Apply moisturizers at least TID. Dose interruptions and reduction result in
(erythema, edema, pain, peeling of Avoid prolonged heat exposure. greatest benefit.
skin on fingers and toes) (weeks to Urea-based emollients preferred. Manage promptly with symptomatic
months) Capecitabine, liposomal Avoid irritation and friction from treatments and analgesics.
doxorubicin, fluorouracil ill-fitting shoes & clothing. Discontinue therapy if severe or
Hand-foot-skin reaction (HFSR) Vitamin B6 150–200 mg/day may recurrent.
(same as above + hyperkeratosis at be prescribed (evidence poor). Wound care for desquamation and
pressure points) (weeks to months) Limit prolonged pressure to ulcerations.
Axitinib, dabrafenib, pazopanib, weight-bearing areas and treat
regorafenib, sorafenib, sunitinib existing calluses & hyperkeratosis

335. You are trying to collect a medication history from a patient to make a Best Possible
Medication History (BPMH). Which of the following is the most reliable source?
a) Patient
b) Patients community pharmacy
c) Patients family
d) Patients written list
Medication reconciliation beneficial for both patients & health care systems. At transitions of care, it
increases patient safety through the reduction of medication errors, as well as through reduction of
potential and actual adverse drug events. It also decreases health care utilization by reducing hospital visits,
emergency department visits, and hospital readmissions related to adverse drug events.
A key step in the medication reconciliation process is obtaining a best possible medication history (BPMH),
which involves interviewing the patient or a caregiver to obtain a list of the patient’s home medications, and
then verifying this information against at least one other reliable source, such as the patient’s medication
vials or the community pharmacy record. https://www.ncbi.nlm.nih.gov/pmc/articles/PMC5931072/

336. Licensed pharmacy technician, what do you allow her to do?


a) BPMH (best possible medication history)
b) Switch dosage form for a script
c) Sign for package slips for drug G

337. A Pharmacist wants to delegate some of the other pharmacist work in an ambulatory
pharmacy to the registered technicians which of the following would be a considered a barrier.
a) Pharmacist attitude
b) Not enough space for technicians to work
c) Provincial regulations

338. Who makes the infectious disease guidelines in Canada?


a) Something with the words “infectious disease” in the answer
Association for Professionals in Infection Control (APIC): Guidelines, policies and standards that Infection
Prevention and Control professionals may use to support their own documentation and best practices.
339. Young female is going away for 6 weeks on a trip and will have sex with her boyfriend,
and she wants something that will cover her the whole trip so that there is no chance of
pregnancy. Patient is currently taking Alesse-21 (Levonorgestrel-ethinyl estradiol)
a) Recommend the patient change to Alesse-28 and then take a 14-day break then start up
again
b) Recommend that the patient take 2 Alesse-21 packs, and she should skip the placebo
week of the first pack and immediately start the second pack
(no placebo pills in Alesse-21, so start a new strip back to back)
The ALESSE 21 consists of 21 pink tablets containing active ingredients. ALESSE 28 consists of 21 pink tablets
containing active ingredients, followed by 7 days of white tablets containing placebo.

340. 18-year-old girl taking COC but keeps forgetting to take her pills so what would you
recommend for her to take? She’s got allergies to latex because she’s a slut
a) Another COC
b) DEPO-PROVERA
c) IUD
d) Evra patch (weekly)
Condom with spermicide (allergy to latex)
Use in Adolescents (12‐18 years) DEPO‐PROVERA is only indicated when other contraceptive methods are
considered unsuitable or unacceptable, due to unknown long‐term effects of bone loss associated with
DEPO‐PROVERA during the critical period of bone accretion

341. Which of the following injections should be refrigerated?


a) Diphenhydramine suspension
b) Sumatriptan
c) Depo-Provera
d) Glatiramer
The pre‐filled syringes of Glatiramer should be refrigerated immediately upon receipt (2‐8°C). Do not freeze.

342. Which of the following must be kept in the fridge before dispensing?
a) Accutane
b) Biaxin suspension (macrolide)
c) Epipen
d) None
Biaxin suspension (macrolide): Store granules for suspension between 15 and 25°C in a tightly closed bottle.
Protect from light. After reconstitution, store between (15 and 25°C), use within 14 days. Do not refrigerate
Epipen: Always store Epipen in the carrier tube with the blue safety release on until you need to use it. Store
at 25°C; excursions permitted to 15‐30°C. Do not refrigerate. Protect from light. Periodically check to make
sure the solution in the auto‐injector is not brown in color. Replace if it is discolored or contains a precipitate
343. 7 years old patient has atopic dermatitis. Patient tried hydrocortisone 1% but only had
partial response. Family doctor wants to start him on Pimecrolimus but is not aware of risk of
cancer associated with the drug. What should you tell the kid’s mother when she comes in to fill
the prescription?
a) Fill prescription and don’t mention risk of cancer If it is used for short term it is ok
b) Tell mother to take her son to visit a dermatologist If less than 2 years or after calcineurin
treatment failure
c) Call family doctor and recommend switching patient to mometasone
Pimecrolimus (Elidel) immunomodulating agent (topical cream) used in the treatment of atopic dermatitis
(eczema). (FDA) announced that Elidel packaging would be required to carry a black box warning regarding
the potential increased risk of lymph node or skin cancer, as for the similar drug tacrolimus

343. Patient is a lifeguard works in the sun, he noticed on his back elevated lesion, with
irregular borders and black something? What could it be?
a) Melanoma
b) Fungal infection
c) Hyperpigmentation
Melanoma Skin Cancer appears to be related to intense and intermittent sun exposure in childhood and
adolescence. It is the rarest type of cancer, but is responsible for the majority of skin cancer deaths. Risk is
increased in blond or red‐headed individuals who have skin that tans poorly and burns easily, those with a
large number of moles, chronic exposure to the sun or those with a past history of sunburns as a child. It
appears as a flat brown or black spot (commonly in a mole or other dark spot) with irregular edges that can
grow larger if left untreated

344. Where should Zostavax II be stored?


a) Fridge
b) Deep freezer
c) Freezer in a small fridge
d) Freezer door of a fridge

zoster virus Prevention of herpes zoster (shingles)


vaccine, live Dosage: ≥ 60 y: 1 dose SC. Can be considered for those ≥ 50 y
attenuated Adverse effects: local pain, swelling, redness. A rash, resembling varicella zoster virus
Zostavax II infection, may occur, although it is generally minor and self-limited.
The need for booster doses is unclear, and still under study.
Avoid in pregnancy. Not recommended in pediatric patients. Do not administer the vaccine to
immunocompromised patients (especially those with T-cell immunodeficiency).
Can coadministered (in different site) with pneumococcal vaccine
Can be given at least 1 y following last episode of herpes zoster
Storage and Stability
During shipment, to ensure that there is no loss of potency, ZOSTAVAX II must be maintained
at a temperature between −50°C and +8°C. Use of dry ice may subject ZOSTAVAX II to
temperatures colder than −50°C.
ZOSTAVAX II SHOULD BE STORED REFRIGERATED at a temperature of 2 to 8°C or
colder until it is reconstituted for injection. The diluent should be stored separately at room
temperature (20 to 25°C) or in the refrigerator (2 to 8°C).
Before reconstitution, protect from light.
Discard if reconstituted vaccine is not used within 30 minutes.
Do not freeze the reconstituted vaccine.

345. Phenytoin dose for a child of 20mg/kg put in 100 ml NS to be infused at rate of
0.5mg/kg/hr. What is the infusion rate?
0.5 mg ------- 1 kg ------- 1 min 20 mg ------- 1 kg ------- X
X = 1*20/0.5 = 40 min.
100 ml -------- 40 min Y ml ----- 60 min Y = 60*100/40 = 150 ml So rate = 150 ml/ hr

346. Psychotic patient is a smoker, doctor wrote olanzapine, and she had a rash on her arm that
she said her printer attacking her while she slept. What is the most important counselling point?
a) Smoking cessation
b) Use motivational counselling to stress the importance of adherence

347. The same patient above was hospitalized for emergency surgery, had not smoked for 5
days. What is your primary concern?
a) DVT prophylaxis
b) Hyperglycaemia
c) Decreased induction of 1A2

348. What is the most significant barrier between physician-pharmacist interactions?


a) Lack of accountability of pharmacists for patient outcomes
b) Lack of training of pharmacists
c) Pharmacists constantly calling doctor for clarifications

349. Patient with Alzheimer's disease is showing early signs of cognitive dysfunction. What is
the most appropriate method of counselling?
a) Speak in child-like terms
b) Counsel in a way that minimizes decision making
c) Ask that a spouse or caregiver be present during counseling
d) Speak loudly and slowly to improve understanding
350. Patient on opioids for cancer pain, needing increasing doses, and experiences myoclonus.
Her family is concerned about opioid dependence. What is your main response?
a) She won't be on opioids for long
b) Cognitive dysfunction will occur before dependence
c) Using NSAIDS adjunctively will just delay dependence
d) Dependence is not a concern in end of life care

351. Same patient sent her son in for her morphine refill. You notice that it is early, and the
intervals between fills are getting shorter. Son states her pain is increasing
a) Call the doctor to recommend fentanyl patch
b) Fill the script since she is in pain right now
c) Call doctor and discuss increasing her dose of morphine

352. Best way to ensure patient safety and efficacy of drug distribution in hospital setting
a) Unit dose and bar coding for each patient
b) Centralized pharmacy with ward stock
c) Decentralized pharmacies in every unit with ward stocking

353. Which medication causes neural tube defects if taken while pregnancy?
Maternal use of folate-inhibiting medications (carbamazepine, cholestyramine,
metformin, methotrexate, phenobarbital, phenytoin, primidone, sulfasalazine,
triamterene, trimethoprim, valproic acid).

354. Middle-aged man complains of recent bad mood, fatigue, muscle weakness, decreased
libido. Doctor diagnoses him with late onset hypogonadism. What will be your
recommendations as a pharmacist?
a) Finasteride
b) Tadalafil
c) Testosterone
d) Tamsulosin

355. Doctor prescribes testosterone 1% gel, what is the most important reason for treating him
a) Improve Mood
b) Prevent Benign prostatic hyperplasia
c) Prevent fractures
d) Prevent Rhabdomyolosis
e) Increase testicular size
ANDROGEL should NOT be used to improve body composition, bone and muscle mass, increase lean body
mass and decrease total fat mass. Serious long‐term deleterious health issues may arise.
ANDROGEL has not been shown to be safe and effective for the enhancement of athletic performance.
Because of potential risk of serious adverse health effects, this drug should not be used for such purpose.
If testosterone deficiency has not been established, testosterone replacement therapy should not be used for
the treatment of sexual dysfunction.
Testosterone replacement therapy is not a treatment for male infertility. Children and women should avoid
contact with unwashed or unclothed application site(s) of men using testosterone gel.
Gels are flammable. Following application of ANDROGEL (testosterone gel), allow gel to dry completely
before smoking or going near an open flame.
Oral testosterone no longer recommended as it causes severe liver damage and high risk of CV disease

356. Patient above has been using the gel properly and got a reaction of blistered skin, doctor
switched him to oral testosterone. What is the most important counselling point?
a) Taking oral testosterone with food helps increase bioavailability
b) Effects of oral testosterone can still be transferred through skin to skin contact
c) Oral testosterone is more difficult to titrate than the cream
d) If oral testosterone fails, there is no other option
Testosterone is Sch. 1, Refill with interval with no transfer.
Oral. To ensure absorption, Restandol Testocaps must be taken with a normal meal, if necessary, with little
fluid, and be swallowed whole without chewing. It is preferable that half of the daily dose be taken in the
morning and the other half in the evening. If an uneven number of capsules is taken daily, the greater part
should be taken in the morning.
Testosterone therapy may also be indicated in osteoporosis due to androgenic deficiency
HRT for men only on Hypogonadism

357. Same patient above has been taking medication for a while and states improvement in all
his symptoms, except now he is unable to get an erection. He states that he has the drive and
desire but he just cannot get it up. What do you advice?
a) Increase dose of testosterone
b) Decrease dose of testosterone
c) Add PDE-5 inhibitor
d) Add desipramine
Patient have the desire but no erection, so, PDE‐5 inhibitor is effective such as sildenafil

358. You have recently been asked to look up the current guidelines, but do not know where to
look for newly published guidelines. A doctor had asked you a question and you were
embarrassed you did not know the answer. Where would you look to keep yourself updated?
a) e-therapeutics  RxTx now
b) Medline
c) Attend an upcoming continuing education program
359. A woman is admitted to the ER for opioid withdrawal because of missed methadone doses
over the past few weeks. What could she be experiencing?
a) Epistaxis
b) Miosis
c) Seizures
d) Dry mouth
e) Diarrhea  Mydriasis, piloerection skin, sweating, insomnia

360. What is the best example of veracity?


a) She tells the doctors exactly how many doses she missed
b) The social worker contacts her family and tells them that she's been hospitalized

361. What is true about optimizing the efficacy of nitrofurantoin


a) Food increases bioavailability
b) Food decreases the chance of diarrhea
c) Avoid iron and Magnesium
d) Take on empty stomach (not too sure if this was the last option)
Not recommended for >65 years
Licorice may increase urinary excretion of nitrofurantoin while deglycyrrhizinated licorice may increase
bioavailability of nitrofurantoin.
Nitrofurantoin is administered orally and all dosage forms should be taken with food or milk to minimize
gastric upset. Administration with food may also enhance absorption.
The most frequent adverse events with nitrofurantoin use are nausea and flatulence. Other GI effects
include vomiting, diarrhea, anorexia, dyspepsia, constipation and abdominal pain. These effects are likely
dose‐related and can be minimized by reducing the dose or administering the drug with food or milk.

362. A new diagnosed female patient with rheumatoid arthritis has multiple joints affected. The
doctor wants to start her on DMARD therapy, when to start DMARDs therapy
a) Once diagnosed
b) When other therapy fails

363. Which of the following is true?


a) Avoid giving corticosteroids during initiation of DMARD
b) Initiate treatment as soon as possible to increase chance of treatment success
c) Delay therapy as much as possible to decrease adverse effects
d) Avoid giving leflunomide if anti something factor is negative
e) Something about his-CRP levels
364. Patient later switched to a biological DMARD. She has concerned that people at work are
getting sick and does not want to be affected. The doctor prescribed one dose of influenza
vaccine now, and another dose later. What is the drug therapy problem?
a) Too much of the right drug
b) Too little of the right drug
c) Drug-disease interaction
d) Drug-drug interaction
INFLIXIMAB decreases effects of Influvac (INFLUENZA VIRUS VACCINE) by pharmacodynamic antagonism.
Immunosuppressant also increase risk of infection with concomitant live vaccines. High likelihood serious or
life‐threatening interaction. Contraindicated unless benefits outweigh risks and no alternatives available
There is no convincing evidence that vaccine effectiveness is reduced. Ideally, vaccinations should be up to
date prior to initiating therapy, particularly in the case of live vaccines. If the patient has already
commenced biologic therapy, most inactive vaccines are recommended, if indicated (e.g., influenza,
pneumococcal)
The inactivated influenza vaccine or flu shot is given by injection, usually as single dose. Children under 9
years of age who have never had a seasonal influenza vaccine need two doses. The second dose of vaccine is
important to raise their level of protection and should be given 4 weeks after the first dose.
http://www.cdc.gov/flu/professionals/vaccination/vax-summary.htm
365. What is the common side effects associated with use of Biologic Response Modifiers
(Biologic DMARDs)?
a) Fever
b) Alopecia
c) Tachypnea
d) Leucopenia

366. Patient is starting Etanercept for rheumatoid arthritis


a) Must be kept at room temperature before injection
b) Inject into abdomen
c) Onset is 3-6 months
d) May cause serious infection leading to hospitalization
Should be stored refrigerated at 2 to 8°C. DO NOT FREEZE. Keep the product in the original carton to protect
from light until the time of use. Do not shake. Keep in a safe place out of the reach of children.
Before injection, allow ENBREL to reach room temperature (approximately 15 to 30 minutes). Do not remove
the needle cap while allowing the prefilled syringe or SureClick autoinjector to reach room temperature.
Prior to administration, visually inspect the solution for particulate matter and discolouration. There may be
small white particles of protein in the solution. This is not unusual for proteinaceous solutions. The solution
should not be used if discoloured or cloudy, or if foreign particulate matter is present.
Reconstituted solutions of ENBREL prepared with the supplied Sterile Bacteriostatic Water for Injection, USP
(0.9% benzyl alcohol) must be refrigerated in the original vial for not more than 14 days at 2 to 8°C, with
overall room temperature exposure of less than 12 hours during storage and handling/usage. DO NOT
FREEZE. Discard reconstituted solution after 14 days.
Serious infections leading to hospitalization or death, including sepsis, tuberculosis (TB), invasive fungal and
other opportunistic infections, have been observed with the use of TNF blocking agents including ENBREL.
Cases of TB may be due to reactivation of latent TB infection or to new infection.
Before starting treatment with ENBREL, all patients should be evaluated for both active and inactive
(‘latent') TB. If inactive (‘latent') TB is diagnosed, treatment for latent TB should be started with anti‐TB
therapy before the initiation of ENBREL. Lymphoma and other malignancies, some fatal, have been reported
in children and adolescent patients treated with TNF‐blockers
367. You are a hospital pharmacist and there is a patient who just had cardiac surgery. He is on
methadone and stable on a certain dose for months now. He did not take his daily dose of
methadone today as he was told not to take any fluids before his surgery. His surgeon comes to
you asking what you can do for him (obsolete)
a) Calculate the same dose of acetaminophen and codeine to give him the same pain relief
b) Tell him to call his family to pick up his daily dose for him
c) Tell the doctor to contact Health Canada and get a temporary exemption

368. Which of the following medications need special counselling for the prevention of
application toxicity
a) Hydroxyurea capsules
b) Digoxin
c) Salbutamol liquid
d) F-something dressing
Hydroxyurea capsules can cause severe decrease in blood cells

369. A pharmaceutical company that makes non-Rx cough and cold medicine wants to pay a
pharmacy to hold a patient education workshop. What is the best way to reduce conflict of
interest?
a) The pharmacy cannot accept payment for the workshop
b) The pharmacist in charge of the event cannot accept personal compensation for it
c) None of the company’s medications can be specifically showcased during the event
d) Only the company’s products with evidence-based efficacy can be presented during the
event

370. Which of the following medications would interact with Sotalol?


a) Clarithromycin
b) Omeprazole
c) Ramipril
Other drugs prolonging the QT‐interval: Sotacor should be given with extreme caution in conjunction with
other drugs known to prolong the QT‐interval such as phenothiazines, tricyclic antidepressants, terfenadine
and astemizole. Other drugs that have been associated with an increased risk for torsades de pointes
include erythromycin IV, halofantrine, pentamidine, and quinolone antibiotics.

371. Which is the best way to counsel a patient on a new inhaler?


a) Use visuals and diagrams
b) Suggest resources like online videos they can watch
c) Use a placebo inhaler to demonstrate the proper technique
d) Make the patient show you how to use the inhaler, after counselling him
Demo inhaler with placebo inhalations (that's what we do in real practice)
Asking the patient to use the inhaler is an appropriate action; however, it's meant to confirm understanding
and that the patient is clear on the inhalation technique.

372. What interacts with tamoxifen? SSRIs 2D6 inhibitors

373. Which is a contraindication to Propafenone?


a) HTN
b) Heart failure
Use of antiarrhythmic agents in recent myocardial infarction patients with asymptomatic ventricular
arrhythmias is associated with increased risk of mortality or nonfatal cardiac arrest as evidenced in the
Cardiac Arrhythmia Suppression Trial (CAST)
Special warnings and precautions for use
The weak negative inotropic effect of Arythmol may assume importance in patients predisposed to cardiac
failure.
In common with other anti‐arrhythmic drugs, Arythmol has been shown to alter sensitivity and pacing
threshold. In patients with pacemakers, appropriate adjustments may be required.
There is potential for conversion of paroxysmal atrial fibrillation to atrial flutter with accompanying 2:1
conduction block or 1:1 conduction.
Because of the beta‐blocking effect, care should be exercised in the treatment of patients with obstructive
airways disease e.g., asthma.
As with some other class Ic anti‐arrhythmic agents, patients with significant structural heart disease may be
predisposed to serious adverse events. Therefore, propafenone is contraindicated in these patients.
A Brugada syndrome may be unmasked or Brugada like electrocardiogram (ECG) changes may be provoked
after exposure to propafenone in previously asymptomatic carriers of the syndrome. After initiating therapy
with propafenone, as ECG should be performed to rule out changes suggestive of Brugada syndrome.
It is essential that each patient given Arythmol be evaluated electrocardiographically and clinically prior to
and during therapy to determine whether the response to Arythmol supports continued treatment.

374. Complication of endocarditis


a) Hypertension crisis
b) Heart Failure
c) Liver Failure
d) Kidney Failure
Untreated endocarditis can damage heart valves and permanently destroy your heart's inner lining causing
heart failure

375. Which of the following is a high-risk medication?


a) 0.9% NaCl and 3% dextrose
b) 10% dextrose
c) 0.3% NaCl
D5 0.3% NaCl‐ is a hypertonic saline solution that are used in critical care settings to help in haemorrhagic
shock

376. Patient took Amoxicillin for something. A few days after he finished his course he comes
back complaining of severe diarrhea and stomach upset. You tell him:
a) Take loperamide. Max 8 tabs
b) Go to the doctor you may have C. difficile
c) This is normal just drink fluids and it will pass

377. The most common pathogen in an intra-abdominal infection


a) S. aureas  found in the human respiratory tract and on the skin
b) B. fragilis  part of the normal flora of the human colon
c) S. viridians  found in the human respiratory tract and on the skin

378. A 45-year-old male has been


hospitalized after being in a motor
vehicle accident and has now developed
hospital-acquired pneumonia. What
would you start as empiric therapy?
a) Piperacillin/Tazobactam
b) Ceftriaxone
379. Which antibiotic is associated with Gallbladder Pseudolithiasis?
a) Vancomycin
b) Ciprofloxacin
c) Tetracycline
d) Ceftriaxone
Development of sludge or stones in the gallbladders due to increase bilirubin & usually reversible upon
discontinuation of Ceftriaxone

380. A new drug has been developed called Newfloxacin. How would you test its efficacy in
CAP?
a) Randomized placebo-controlled trial
b) Cohort study
c) Randomized Comparative study
Randomized Controlled Trial ("RCT"), the mainstay of experimental medical studies, normally used in testing
new drugs

381. Patient comes into the hospital with a STEMI. His medications include low dose ASA, an
ACE, a STATIN, NTG spray prn and diltiazem. Doctor would like to start him on metoprolol.
All are appropriate except:
a) Metoprolol should be titrated to a resting HR of 50-60 bpm
b) Patient should continue taking diltiazem

382. Patient above is a smoker. The doctor would like to initiate him on smoking cessation
therapy. Which of the following is true?
a) NRT patch is a better and safer option for a post MI patient
b) Bupropion is safe in post MI patients

383. What is the most common side effect of varenicline?


a) Nausea
b) Photosensitivity

384. A patient comes in complaining of urinary incontinence at night. Which of the following
medications could be contributing to this problem and can be switched to the morning?
a) Lisinopril
b) Metoprolol
c) Amiloride
Mirabegron (Myrbetriq) used to treat certain types of urinary incontinence. It relaxes the bladder muscle
and can increase the amount of urine your bladder can hold.
385. 100 mL IV bag with 10mL overfill solution. Wanted to add a specific amount of drug
(forgot the actual details.but it was easy to calculate and it came out to 60 mL of drug). The
question asked how much do you remove from the original bag to add this new drug (something
like that). The trick was to add 60 mL (amount of drug you need to add) to the 10 mL overfill
solution so you end up with 70mL. How much you take out of the original bag.

386. In the above case there is another question, according for the result what you will counsel
the patient?
a) Encourage the patient to take the vaccine because
b) Avoid the vaccine because the RRR is (difficult word)

387. 18 years female has mild acne and she takes hormonal medication what will worsen the
case

388. Question about patient has Peripheral artery disease

389. Patient has profile with nausea and hypertension, all this medication does this except?
Acetaminophen

390. Odd ratio. How to calculate it?


Odds ratio OR: https://www.youtube.com/watch?v=5zPSD_e_N04 watch this video
Measures association with exposure and outcome.
The odds represent the number of patients in a given group with an event divided by number of patients in the
same group without it.
Or; The OR represents the odds that an outcome will occur given a particular exposure, compared to the odds of
the outcome occurring in the absence of that exposure.
OR = (A/C)/(B/D) = AD/BC = odds of exposure among cases/Odds of exposure among control
A= number of exposed cases B=Number of exposed NON cases
C=Number of unexposed cases D= Number of unexposed non case
Tips. Always divide experiment by control (goes down in table)
Interpretation: Degree of association between disease and exposure
 OR > 1 Exposure is associated with higher odds of disease, OR the risk of the exposed group is greater
than risk in unexposed group.
 OR = 1 No association between exposure and disease, OR no association or the risk is equal in the two
experiment hands
 OR < 1 Exposure is associated with lower odds of disease, OR the risk of the exposed group is lower
than risk in unexposed group.
391. Case about patients quit smoking

392. Case about patient has fear from zoster and you have to calculate the NNT (537)

393. What is a barrier to patient centered care?

394. Meningitis case. 14 years old child who has cochlear implants and had recurrent otitis
media in his early child hood, he is admitted to hospital with high fever, Pain and he was
unconscious. His history includes use of SABA PRN & Fluticasone/Salmeterol BID for asthma
control, what will be probable cause of patient’s meningitis Or What increases risk of
meningitis?
a) Cochlear implants
b) Recurrent Infection of AOM
c) Corticosteroid uses for Asthma
d) Hypothyroidism
Importantly as well, one should be very careful with the Q wording when it comes to risk factors versus
causation.
So, for example, if the Q version is about a RISK FACTOR (association), cochlear implants remain the most
significant risk factor allowing access of pathogens to the brain, especially that this Q version is
straightforward since AOM is in that scenario a remote history.
However, for a Q version in which you've repeated recent AOM episodes, and the Q is asking clearly, "what
is the most likely CAUSE of the current meningitis? Then the answer will change to AOM, especially that the
cochlear implant here appears in the background (the remote history).
I hope this is clear to everyone.
Flipping scenarios is a very common phenomenon on PEBC exams, and if the candidate is dragged by
previous experience related to similar questions or situations, they're very likely to make mistakes on those
questions. On the other hand, candidates who are going to tackle each and every question based on their
knowledge, understanding, and acquired skills will stand a much better chance

395. What is likely cause of meningitis in this kid?


a) S. pneumonia
b) Neiserria meningitides

396. What is elevated in CSF fluid?


a) Protein
b) Glucose
c) WBC
397. MT, a registered pharmacy technician, has worked in a community pharmacy for 5 years.
He is well-trained and welcomes new learning opportunities. His manager now wants to assign
a new project related to the preparation of sterile products in the pharmacy. Which of the
following steps should be taken first with the technician?
a) Explore educational opportunities to prepare the technician for the project.
b) Provide an overview of the project goals and the intended role of the technician.
c) Discuss suggested strategies to effectively accomplish the project.
d) Negotiate a timeframe for the completion of the project.
B >> A >> C >> D

398. George, a 64-year-old male has been experiencing persistent low mood, sleeping difficulty,
severe anorexia and suicidal thoughts. Doctor diagnosed George’s condition as major
depression. Currently, he is on Hydrochlorothiazide/Candesartan 12.5 mg/35 mg daily and
Metformin 500 mg bid, Acetyl salicylic acid (ASA) 81 mg daily. Doctor is considering
prescribing an antidepressant. Which of the following drug requires precaution to George?
a) Sertraline
b) Bupropion
c) Venlafaxine
d) Citalopram
Bupropion (it has an anorexic effect).
Note that there's a combination naltrexone‐bupropion marketed for weight control.
Vanlafaxine also can possibly result in anorexia.
The ideal choice here would be fluoxetine.

399. Which of the following study is demographic causality?


a) Cross sectional
b) Cohort
c) Case control
d) Cross observational
What's exactly meant by "demographic causality"?!
What's the meaning of "cross‐observational"?!
It is difficult to derive causal relationships from cross‐sectional analysis or any observational studies.
The best way to prove a causal relationship is through RCTs.
Causal risk factors can be drawn from cohort studies; however, confounding factors and multiple limitations
make the conclusion about causal risk factors NOT reliable enough.
The are some methodologies that can strengthen causal inferences from cohort studies.
NOVEMBER 2013

1. Case about bulimia nervosa, what to give her:


a) Mirtazapine
b) Fluoxetine
c) Sertraline
SSRIs (fluoxetine), venlafaxine or trazodone.
Trazodone can be useful in patients with insomnia
associated with bulimia nervosa.
Due to toxicity and poor adherence, bupropion has been
associated with an increased risk of seizures in patients
with eating disorders

2. Then her case is not controlled, what will she


then suffer from:
a) Pancreatitis- in case of excessive use of laxatives
b) Esophagitis- in case of vomiting
Most physical symptoms and complications of bulimia nervosa result from purging. Self-induced vomiting
may lead to erosion of dental enamel of the front teeth, painless parotid (salivary) gland enlargement, and
an inflamed esophagus. Physical signs include: Swollen parotid glands, Scars on the knuckles (from
repeatedly inducing vomiting by using fingers to trigger gag reflex) & Dental erosion

3. What is the expected period to see effect from medication?


a) 12 months
b) 2-3 months
c) 6 months
Improvement in 2-3 m, but duration of treatment is 6-12m. Continue for at least 6 m, and preferably 1 year.

4. Treatment fail why? Purging can reduce or prevent drug absorption.


Purging can reduce or prevent drug absorption. When pharmacotherapy is not effective, ask the patient
about the timing of both purging behaviour and dose administration.
A temporary worsening of binge and purge behaviour often occurs during psychological treatment, e.g.,
when a long-suppressed traumatic event such as sexual abuse is uncovered and addressed, or with
significant life stress. This does not indicate a worsening in the patient's overall condition.
Treatment with more than 1 antidepressant at the same time is not recommended—there are no
proven advantages and there are potential to increase adverse effects and cost.
Response to medication will vary from patient to patient. When an antidepressant is effective, continue for
6–12 months. When discontinuing antidepressants, taper the dose gradually to prevent the development of
discontinuation syndrome. Treatment of psychiatric comorbidity is necessary for long-term cure.
5. Which medication should be stopped before IV contrast media?
a) Metformin
b) Acarbose
c) Pioglitazone
d) Gliclazide
e) Meglitinides
To avoid the possibilities of acute renal failure
that may increase the risk of lactic acidosis.

6. A woman came to your pharmacy and


asked you about evidence-based proved
non-prescription medication for
premenstrual syndrome. What is your
most appropriate recommendation?
a) Zinc
b) Calcium
c) Black cohosh
d) Primroses oil
e) NSAID
Premenstrual syndrome (PMS) is characterized by irritability, anxiety, emotional lability, depression, edema,
breast pain, and headaches, occurring during the 7 to 10 days before and usually ending a few hours after
onset of menses. Diagnosis is clinical, often based on the patient’s daily recording of symptoms. Treatment
is symptomatic and includes diet, drugs, and counseling.
The cause of PMS is unclear. Possible causes or contributing factors include:
Multiple endocrine factors (eg, hypoglycemia, changes in carbohydrate metabolism, hyperprolactinemia,
fluctuations in levels of circulating estrogen and progesterone, abnormal responses to estrogen and
progesterone, excess aldosterone or antidiuretic hormone [ADH])
A genetic predisposition. Possibly magnesium and calcium deficiencies
Estrogen and progesterone can cause transitory fluid retention, as can excess aldosterone or ADH.
Serotonin deficiency is thought to contribute because women who are most affected by PMS have lower
serotonin levels and because SSRIs, which increase serotonin, sometimes relieve symptoms of PMS.
Symptoms and Signs
Type and intensity of symptoms of PMS vary from woman to woman and from cycle to cycle. Symptoms last
a few hours to ≥ 10 days, usually ending when menses begins. Symptoms may become more severe during
stress or perimenopause. In perimenopausal women, symptoms may persist until after menses.
The most common symptoms are irritability, anxiety, agitation, anger, insomnia, difficulty concentrating,
lethargy, depression, and severe fatigue. Fluid retention causes edema, transient weight gain, and breast
fullness and pain. Pelvic heaviness or pressure and backache may occur. Some women, particularly younger
ones, have dysmenorrhea when menses begins. Other nonspecific symptoms may include headache, vertigo,
paresthesias of the extremities, syncope, palpitations, constipation, nausea, vomiting, and changes in
appetite. Acne and neurodermatitis may also occur.
Existing skin disorders may worsen, as may respiratory problems (eg, allergies, infection) and eye problems
(eg, visual disturbances, conjunctivitis).

7. In order to weigh a compound that that has 5% accuracy on a balance that has sensitivity
error of 4.5 mg. What is the maximum weighable amount?
a) 9mg
b) 45mg
c) 90mg
d) 180mg

8. Patient with HIV taking Tenofovir


300mg, lamivudine, BID 7am and 7pm
Lopinavir / ritonavir at 7 pm, and
levothyroxine qam. When they did a lab
test, they found that his triglycerides were
high. What the reason is for this.
a) Tenofovir
b) Lopinavir/ritonavir ‫ريتا طبخت لوبيا راحت الدهون عليت‬
c) Interaction of levothyroxine with Lopinavir/ritonavir (no interaction)
d) Interaction between Tenofovir and Lopinavir/retonavir. (would cause renal problems)
Hypertriglyceridemia is a side effect, protease inhibitors cause this
All PI cause hyperlipidemia except atazanavir

9. Patient called you at 3:00 pm and told you that this morning he was vomiting because he
drank a lot of alcohol last night, so he skipped his medications as he woke up late this morning.
He is asking you what to do next?
a) Take all the morning medications now and at 7pm with take Lopinavir/retonavir
b) Take only lamivudine and Tenofovir now and take the Lopinavir/retonavir with
other antiviral with supper
c) Skip all of today’s medications since he drank a lot of alcohol
d) Skip the morning medications and take the evening medications

10. Ferrous gluconate TID, one tablet contains 300 mg Iron. How much elemental iron daily?
Answer:
Ferrous gluconate contains 11.7 % elemental iron.
3 Tabs/day = 900 mg Iron.
Elemental iron = 900 * 11.7 % = 105.3 mg
11. According to NAPRA, Iron is Schedule:
a) I
b) II
c) III
d) Unscheduled
e) Schedule F
Iron with more than 30mg elemental iron per solid dose is schedule II, Equal or < 30mg is unscheduled

12. A 10-week pregnant lady came to you in the pharmacy and said she has burning and itching
in the vaginal area and it’s the first time for her to experience these symptoms and she said she
has a white-grey discharge and malodorous. Why would you refer this patient? K type
a) Because this is the first time for her
b) Her symptoms are different from those of candidiasis (Bacterial Vaginosis)
c) A pregnant woman can take intravaginal medication
When to refer: first episode, fever, pelvic pain, odour, under 12, pregnant, diabetes, two infections in less
than 2 months, Immunocompromised
Diagnostic Parameters Candidiasis Trichomoniasis Bacterial Vaginosis
Signs Pruritus + + –
/symptoms
Odour – + + (fishy)
Discharge white, clumpy & off-white or Grey or milky, thin, copious
curdy yellow, frothy
Inflammation + + –
Treatment Azoles Metronidazole Metronidazole or Clindamycin
(Oral or Vaginal) (Systemic ttt preferred than cream)

13. A skateboarding teenager fell on his knees. He presented to you in the pharmacy with his
knees excrociated, red, and something else. There was no dirt or debris in the wound. After
appropriate irrigation of the wound, what is the best action the pharmacist should do?
a) Give topical antibiotics
b) Cover the wound with an
appropriate dressing
c) Refer him to the physician to check
if there is an infection in the wound
d) Irrigate with saline (irrigation has
been already done)
e) Recommend tetanus injection
14. A patient with ADHD the doctor prescribed 10 mg Methylphenidate BID at 7am and at
3pm., (the question never said IR or ER). The child is well controlled but the mother told the
pharmacist that her son has insomnia. What is your best recommendation?
a) Take 2 doses in the morning (20 mg) at once
b) Switch to methylphenidate ER 2 tablets at noon
c) Switch to methylphenidate ER and take once at 7am and once at 3pm
d) Take one dose at 7 and the other at 12pm
e) Take just the morning dose
Generally, should be given before 4pm to avoid insomnia. The earlier the better

15. The doctor added Atomoxetine to this patient’s medications. One week later, mother came
and said that her child is complaining of headache for the past week. What is your best advice?
a) Advise her to see the physician if the headache is bothersome and persistent
b) This is transient side effect and should not be a concern
c) Atomoxetine does not cause headache (it does)
d) Advise her to go to the nearest emergency (not an emergency it’s a common side effect)

16. What is the difference between Methylphenidate and Atomoxetine?


Atomoxetine is not a stimulant drug so it will cause somnolence. Methylphenidate is a stimulant.

17. Same mother came back with a prescription for Rizatriptan wafer. Which of the following is
a true statement about Rizatriptan Wafer?
a) It is absorbed from the buccal cavity (its swallowed orally)
b) Co-administration with alcohol is contraindicated. (no interactions stated)
c) It is used for migraine with nausea (It causes nausea, it’s indicated for migraines with
or without aura)
d) It is absorbed faster than Rizatriptan tablets
e) It is contraindicated with people who have difficulty swallowing (it dissolves in the saliva
so very easy to swallow)
The bioavailability and Cmax of Rizatriptan were similar following administration of tablets and wafers, but
the rate of absorption is somewhat slower with wafers.

18. The mother will move to another town and wants to transfer her son’s prescriptions. What is
your best response? The son’s prescriptions had some refills left.
a) Transfer Atomoxetine but Methylphenidate cannot be transferred and needs a new
written prescription only.
b) Transfer Atomoxetine but Methylphenidate cannot be transferred and needs a new
verbal prescription
c) He needs written Rx for both Atomoxetine and methylphenidate
19. Patient has migraine she is seeking for something fast to relief her symptoms
a) Naproxen
b) Naratriptan
c) Zolmitriptan
d) Subcutaneous Sumatriptan
Subcutaneous Sumatriptan has the fastest onset of action and remains the most efficacious triptan for a
severe migraine attack. It is also useful in an acute cluster headache.

20. A patient came to your pharmacy complaining that his eyes are itchy and red for the past
month. He also said that he has been using eye drops all this period to treat his eye. You suspect
hyperemia. Which of the following medications may cause his symptoms?
a) Tetrahydrozoline decongestant constricts blood vessels in eye making them red
b) Nedocromil treats hyperemia
c) Polymixin B
d) Levocabastin treats hyperemia

21. A pregnant woman had vomiting 8 times today and she is not drinking well and does not
want to take any medications. Her husband calls you at the pharmacy asking what to do.
a) See or call your physician to prescribe
Diclectin
b) Have some rest
c) Eat small snacks
d) Go to the emergency
severe+ no fluids+ needs electrolyte balance, so emergency
Red flags: The following findings are of particular concern:
Abdominal pain, Signs of dehydration (eg, orthostatic
hypotension, tachycardia), Fever, Bloody or bilious emesis,
No fetal motion or heart sounds, Abnormal neurologic examination & Persistent or worsening symptoms

22. MB presented to your pharmacy. He said he had intermittent diarrhea for the past 10 days
and that he has been tired the last month. What is the most appropriate advice you should give
to MB?
a) Advise him to take loperamide
b) Advise him to have some bed rest and drink fluids
c) Advise him to visit his physician
d) Advise him to visit the nearest emergency department
e) Bismuth subsalicylate
Inflammatory Bowel Diseases: Inflammatory bowel diseases (IBD) such as Crohn's disease and ulcerative
colitis can cause intermittent, chronic diarrhea. Diarrhea, stomach pain, rectal bleeding, fever, and weight
loss may occur for weeks or months on end, often resolving for a period of time before recurring.
23. A 4 months baby was just weaned and was started on the formula, his normal bowel
movement was 3/day, now he doesn’t have any bowel movement in the last 36 hours, the baby
shows no symptoms, what should you do?
a) Refer to doctor
b) Wait a little bit longer
c) Give prune juice
d) Give infant mineral oil
e) Give infant glycerin suppository

24. A child is on amoxicillin 2mL once daily and his baby sitter accidentally gave him 10mL.
The child does not appear in any discomfort or pain. What should they do?
a) Take him to emergency
b) Observe the child for any diarrhea for 48 hours
c) Give him activated charcoal
You move based on what you see.
So, for example, if the child is vomiting, in distress and looks miserable (toxic appearance), send to ER.
If the child is too young (a few months old) and a a large dose was given, then call Poison Control Centre to
triage the parent.
If it's like this scenario, then the pharmacist has to manage the situation professionally as a health
practitioner.
25. Upon initiating Isotretinoin, a baseline measurement should be taken for which of the
following:
a) Liver
b) Glucose
c) Electrolytes
d) TG
e) Serum creatinine
Isotretinoin is nearly always effective, but use is limited by adverse effects, including dryness of conjunctivae
and mucosae of the genitals, chapped lips, arthralgias, depression, elevated lipid levels, and the risk of birth
defects if treatment occurs during pregnancy. Hydration with water followed by petrolatum application
usually alleviates mucosal and cutaneous dryness. Arthralgias (mostly of large joints or the lower back)
occur in about 15% of patients. Increased risk of depression and suicide is much publicized but probably rare.
It is not clear whether risk of new or worsened inflammatory bowel disease (Crohn disease and ulcerative
colitis) is increased.
CBC, liver function, triglyceride, and cholesterol levels should be determined before treatment. Each should
be reassessed at 4 wk and, unless abnormalities are noted, need not be repeated until the end of treatment.
Triglycerides rarely increase to a level at which the drug should be stopped. Liver function is seldom
affected. Because isotretinoin is teratogenic, women of childbearing age are told that they are required to
use 2 methods of contraception for 1 mo before treatment, during treatment, and for at least 1 mo after
stopping treatment. Pregnancy tests should be done before beginning therapy and monthly until 1 mo after
therapy stops.

26. Which of the following should be monitored on the next doctor visit?
a) Liver
b) CBC
c) TG
d) Glucose

27. Which of the following should be used with Isotretinoin? Lip balm
Patients should be advised to use a skin-moisturizing ointment or cream and a lip balm from the start of
treatment as Isotretinoin is likely to cause dryness of the skin and lips.

28. Patient with pneumonia PSI is 114


admitted to the hospital. Why Patient
admitted?
a) He is severely ill
b) Score is 114
c) He cannot take oral therapy
PSI score is ≤90, treat as outpatient. PSI score is ≥91,
treat in hospital.
29. Best treatment for this patient in ICU is IV therapy, what is the empiric treatment
a) Doxycycline
b) Moxifloxacin
c) Ceftriaxone plus clarithromycin
d) Amoxicilline clavulanate plus clarithromycin
Empiric Therapy
➢ For CAP of moderate severity, there is no difference between beta-lactam alone, macrolide and beta-
lactam, or fluoroquinolone therapy. A systematic review showed no difference between macrolides &
fluoroquinolones but fewer adverse events with clarithromycin than erythromycin. Another review
showed no difference between macrolides and fluoroquinolones for death.
➢ For outpatients, amoxicillin as first choice or either amoxicillin/clavulanate or doxycycline as 2nd choice.
➢ For severe pneumonia, 3rd generation cephalosporin (ceftriaxone or cefotaxime) in combination with
clarithromycin is a rational empiric regimen. However, it should be noted that macrolide antibiotics
(e.g., azithromycin, clarithromycin) and FQs may cause QT interval prolongation and caution is advised.
➢ For inpatients: Systematic review evidence for inpatients showed no difference between macrolides
and fluoroquinolones for death and no difference for death whether or not atypical coverage was
used. In order to reduce increasing fluoroquinolone resistance and prevent adverse events (e.g., QT
interval prolongation), use of a respiratory fluoroquinolone should be reserved for when cephalosporins
or penicillins cannot be used.
➢ Piperacillin/tazobactam should be used only for severe pneumonia or in patients at high risk for
resistant pathogens, e.g., P. aeruginosa.

30. A female patient diagnosed with metastatic breast cancer and started on chemotherapy. In
addition, she took dexamethasone and granisetron as a prophylaxis before the chemotherapy.
However, she still feels nauseated after her dose. Which of the following sentences is true?
a) Granisetron is more effective than ondansetron. Have same efficacy
b) Prochlorperazine can be taken as prn for her breakthrough symptoms
c) Diphenhydramine can be used as regular rather than prn → for motion sickness not CINV
d) Add dimenhydrinate to the existing regimen no
e) Continue on the dexamethasone only no
Also, metoclopramide, haloperidol, olanzapine, lorazepam, alprazolam can be used for breakthrough CINV
Olanzapine antagonizes several neurotransmitter receptors including dopamine and 5-HT receptors and has
shown some activity in acute, delayed and breakthrough nausea and vomiting.

31. What is Medication Reconciliation


a) Collecting patient history from different resources
b) Preventing medication interaction
Medication reconciliation is a formal process in which healthcare providers work together with patients,
families and care providers to ensure accurate and comprehensive medication information is communicated
consistently across transitions of care.
32. Patient admitted to the hospital due to a suicidal attempt. They found out that it was due to
Clomipramine and this medication stopped during his hospital stay. Then upon his discharge.
The patient took all his pre- admission prescriptions except the clomipramine Afterwards the
patient went to his community pharmacy and got his prescription medications that he got from
the hospital. Then when the patient went home, he continued taking his clomipramine pills that
he had at home. How could you have prevented this error?
a) Do a medication reconciliation upon discharge
b) Supply a written information with this prescription
c) Do a call back service to follow-up
d) Community pharmacist to check his medical profile when dispensing new prescription
e) Ask the patient
As the medical reconciliation already done for that patient, once he goes to the community pharmacy, now
this is the role of the pharmacist to revise with the patient what to take and what to stop.

33. A regular client at your pharmacy called you on Saturday evening (not midnight) and she
said that she ran out of her medications and the doctor is on vacation and she asks you for a 2-
day advance until she sees her doctor on Monday morning. Her profile is as follows (in a table)
90 Amlodipine - last refill was 3 months ago and has no refills left
90 Ramipril - last refill was 3 months ago and has no refills left
90 Paroxetine - last refill was 3 months ago and has no refills left
Zopiclone – last refill was 2 weeks ago and has no refills left
Using your professional judgment as a pharmacist and your ethical/legal principles, what is the
most appropriate action you should do?
a) Advance her a two-day supply of all her medications
b) Advance her a two-day supply of all her medications except Zopiclone
c) Advance her a two-day supply of all her medications except Zopiclone and paroxetine
d) Advise her to go to a walk-in clinic and get an authorized prescription
e) Advise her to go the nearest emergency department to get an authorized prescription

34. A patient with depression and was controlled over fluoxetine. She was complaining that
since she started medication, she has trouble sleeping and has insomnia. The physician is asking
about your recommendation, as he wants to solve this problem without using sedative
medication at night. What is your best recommendation?
a) Switch to another agent in the same group preferably Sertraline (both 16% insomnia)
Insomnia Usually diminishes after 1–2 wk of therapy
Venlafaxine (17%), Daytime dosing
bupropion (16%), fluoxetine (16%), sertraline (16%), Consider short-term use of hypnotic (e.g.,
fluvoxamine (14%), paroxetine (13%), zopiclone) during initiation
duloxetine (11%) Counsel on sleep hygiene
35. What is the minimum time until you see a significant improvement in her case?
a) 1 week
b) 2 weeks
c) 4 weeks
d) 3months
e) 6 months
Clinical improvement with SSRIs is gradual, occurring over
6–12 weeks.
2-4 weeks: Improvement in sleep and appetite ...and for
mood improvement after 6 weeks

36. She went into mania. Her doctor diagnosed her with bipolar disorder. What to do
a) Switch to lamotrigine
b) Switch to carbamazepine
c) Stop antidepressant and start lithium
d) Continue antidepressant and lithium together
People with bipolar shouldn’t be given antidepressants

37. A physician prescribed potassium supplements 20 m. Eq TID to patient but this patient does
not want to take supplements. He asks the physician if he can eat bananas instead. If each large
banana has 602 potassium. How many bananas should the patient eat each day? (M. wt K = 39).
Answer:
20 m. Eq TID = 60 m. Eq per day m. Eq = M. Wt * Valency = 39 * 1 = 39 mg
Each day = 60mEq = 60*39= 2340 mg
Each banana contains 602 mg No. of banana = 2340/6002 = 3.88
38. A physician prescribed a fortified eye drops for a patient. Tobramycin 13.5 mg/ml Gtt i ou
twice daily. How much of the 40 mg/ml stock solution should be added to 5 ml of 0.3% to get
the desired conc.?
a) 1 ml
40mg/ml 10.5/37=X X=1.98ml
b) 1.5 ml 13.5mg/ml
c) 0.5 ml 3mg/ml 26.5/37=5ml
d) 2 ml
e) 2.5 ml

39. MK is a regular client at your pharmacy. He is always having Epipen at home. He presented
today with a new Rx for epinephrine as he ran out of his epipen. Two days later, he came to the
pharmacy returning the epipen as he noticed that it has only three more months to expire. What
is the most appropriate action for the pharmacist?
a) Return the medication, dispose it in a safe way and accept the credit loss
b) Send it back to the manufacturer and tell him it was defective
c) Donate it to poor people
d) Refuse to return it.
e) Let the staff check the expiry dates every 6 months

40. What is true about EpiPen injection counselling?


a) You can even inject it on top of your clothes
b) You should keep it refrigerated
c) Shake well before injection
d) Take the 1st dose if improvement
occurs, don’t seek medical attention
e) If improvement occurs with the 1st
dose, wait up to 6 months to use the
second dose
Do not freeze and do not refrigerate it.

41. Patient having cellulitis, what is the


causative microorganism of cellulites?
a) Group A, Streptococcus B-
hemolytic GAS B
b) H-influenza
c) Enterococci
42. Cellulitis can be treated by all of the following except?
a) Cephalexin
b) Cloxacillin
c) Norfloxacin quinolones play little role in skin infections
d) Clindamycin
e) Amoxicillin/clavulanate

43.What is the duration of treatment of cellulitis?


a) 5 days if purulent cellulitis (predominant pathogen, staph aureus)
b) 3 days
c) 10 days (5-10 days, non-purulent cellulitis, GAS)
d) 21 days

44. Hospitals follow the federal standards and principles in order to:
a) Get federal money from the Canada Assistance Plan
b) Get their full share of the federal transfers for the hospital services

45. A patient took Novo Linge (30/70) BID instead of NoVo Linge NPH 10 Units BID. What is
the potential side effect? Given instead of intermediate
a) Postprandial hypoglycemia
b) Fasting hypoglycemia
c) Nocturnal hypoglycemia
d) Dawn phenomenon

46. To avoid the above mistake, what should you do? (according to the story)
a) Double check while dispensing
b) Computer alert
c) Make the patient check his insulin before leaving the pharmacy
d) Put different concentration of insulin in different places

47. Which of the following textbooks does NOT have references?


a) CPS
b) Remington
c) Therapeutic choices
d) Micoromedex
48. A study done on 500-2000 patients and showed no significant difference between new Drug
X and placebo. However, another recent study showed a statistically significant difference
between Drug X and placebo when it was investigated on 20,000 patients. What is the most
appropriate reason?
a) Type I error of the recent study
b) The small sample size of the old study
c) Type II error of the recent study
d) Different sample size

49. A pharmacist in a community hospital is trying to increase the teamwork between the
pharmacy staff and other health care professionals (HCPs) in the area. Which of the following
will NOT contribute to achieving this endeavor?
a) Make a mentor system to integrate new staff
b) Plan a disciplinary seminar lunch
c) Do reports about conflict resolution to HCPs.
d) Send the pharmacist more often to the patient care area

50. A community pharmacy was sold to a new owner and he hired a new manager. The owner
does not want to work in the dispensary but wants to keep the financial decisions for him. All of
the following tasks can be done by the manager, EXCEPT:
a) Hiring staff
b) Signing contract with the nursing home
c) Increase salaries
d) Purchasing medications
e) Inventory
Owner are responsible for financial decisions and contracts

51. Which task will be sole for the new owner?


a) Discuss for hiring new staff
b) Increase staff salary
c) Overview front merchandise

52. What is the main area can the owner work in to increase his profitability
a) Increase opening hours
b) Increase advertising in the local area
c) Decrease pharmacy staff
d) Delegate some of the pharmacy work to technicians
53. The reason for dispensing errors related to environmental conditions in a community
pharmacy is:
a) Low light in the pharmacy
b) The climate inside the pharmacy
c) Work load
d) Noise
e) Untrained staff

54. Camphor and menthol mixed with a base. What kind of mixture does camphor and menthol
make?
a) Eutectic mixture
b) Geometric
c) Colloidal
Menthol and camphor, both solids at room temperature, form a eutectic that is a liquid at room
temperature. This mixture is used for pruritus

55. A patient had gonorrhea. What do you recommend? Cefixime + azithromycin


55. The physician decided to give her ciprofloxacin as he suspects chlamydia infection as well.
What is your recommendation?
a) Add Ceftriaxone
b) Add Azithromycin preferred treatment for Chlamydia
c) Continue on ciprofloxacin

57. Which one of the following medications causes weight loss?


a) Topiramate
b) Carbamazepine
c) Phenytoin
d) Lamotrigine
e) Gabapentin

58. A patient has shingles and she was admitted to the hospital due to her severe and acute pain.
Her pain is now controlled. You delivered the medication to her as prescribed Amitriptyline 10
mg QHS. When she went home and read the leaflet, she decided not to take the medication due
to its side effects. What contributed to this problem?
a) No dialogue between the pharmacist and the patient
b) The physician did not give the patient sufficient information about the medication.
c) The leaflet contains detailed information

59. What should the pharmacist tell her?


a) The pain relief provided by this medication will outweigh any side effects
b) The common side effects usually tolerated by most patients and can be managed
c) This is a very low dose to cause these side effects
d) If you experience any side effects go right away to the doctor
e) Serious side effects donot happen to everyone

60. What is the concern of the following Rx? Ferrous sulfate iii hs, Sildenafil 25, Terazosin &
Omeprazole?
a) Terazosin and sildenafil → Severe hypotension
b) Sildenafil and hypertension

61. Several errors reported from the patient care area for KCL injection, as a pharmacy manager
what should you do?
a) Pharmacist prepare the infusion
b) Lock it in a cupboard and keep the key with one nurse in the patient care area
c) Put the label on the bin where you store the injections
d) Remove it completely from the ward and keep it in the pharmacy
62. In hospital pharmacy, wastage of KCL is due to
a) No site for storage
b) Remaining are thrown in garbage
c) Freshly prepared then used
d) Doctor order wrong quantity

63. Pt on 40mg atorvastatin, 500 mg clarithromycin new Rx for 10 days, what should he do?
a) Dispense the prescription
b) Stop the statin temporarily for 2 weeks
c) Change to levofloxacin
d) Reduce the dose of clarithromycin to 250

64. Doctor asks for unapproved use of drug, which reference


a) Micromedex
b) Pharmacological clinical data
c) Medline
d) CPS
e) TC

65. Old man coming out of hospital, can be given all vaccines, except
a) Varicella
b) Pneumonia
c) Influenza
d) Diphtheria
e) DPT
Correct if this old man is immunocompromised

66. Patient was recently diagnosed with bad cholesterol, what is the pharmacist first concern?
a) Patient knowledge about cholesterol
b) Start treatment immediately
c) Reduce diet and give high fiber food
Nondrug measures first. I'm not sure what "knowledge about cholesterol" means in this particular context.

67. Patient always busy, works 12-16 hours daily, smoke 1-2 packs daily, came back from work
by riding a bike, she loves to drink a cup of wine at night, what to advise her except:
a) Prepare healthy cooked meal and take it to work
b) Stop smoking
c) Reduce her alcohol intake (1-2 drinks per day)
68. What drug can a pharmacist return to whole sale:
a) Tranylcypromine → Nonselective irreversible MAOI
b) Narcotic
c) Methylphenidate
d) Benzodiazepines

69. Patient takes nasal spray which may cause systemic absorption, what is true:
a) Take each one separately 10 minutes apart
b) Take some in the morning while others at night
c) Put your finger on tear duct (for eye drop)
Eye drops yes but nasal drops spacing

70. Which eye drop stains iris? Latanaprost


S.E: Conjunctival hyperemia, foreign body sensation, burning, itching, stinging, headache, ↑ iris
pigmentation and eyelash length, flulike symptoms. Store in fridge, 6 w in room temperature

71. Diabetes, Renal failure, HT, what is the goal of Blood Pressure
a) 140/90
b) 130/80
c) 125/75

72. Diabetic patient with acute exacerbation of COPD, which antibiotic is used.
a) Cephalexin
b) SMX/TMP
c) Ciprofloxacin
d) Azithromycin
Group Symptoms & risk factors Probable pathogen First choice antibiotic
Simple < 4 exacerbations in the past year M. catarrhalis ➢ Amoxicillin
exacerbations Increased sputum purulence + at S. pneumonia ➢ Doxycycline
(COPD least 1 of: Haemophilus spp. ➢ SMX/TMP
without risk 1) Increased sputum volume All for 5–7 days
factors) 2) Worsening dyspnea
Complicated As in simple exacerbation + at M. catarrhalis Preferred:
exacerbations least 1 of: S. pneumonia ➢ Amoxicillin/clavulanate
(COPD with 1) FEV1 < 50% predicted Haemophilus spp. ➢ Cefuroxime axetil
risk factors) 2) ≥ 4 exacerbations per year Klebsiella spp. Both for 5 – 10 days
3) Ischemic heart disease Other gram-negative ➢ Levofloxacin × 5 days
4) Use of home O2 pathogens Alternative:
5) Chronic oral corticosteroid Pseudomonas spp. ➢ Azithromycin × 3 days
6) Antibiotic use in previous Higher probability of ➢ Clarithromycin × 5–10
3 months betalactam resistance days
73. Diabetic patient has COPD; renal dysfunction, lost consciousness, hospitalized, severe
emphysema, smoke 1 pack of cigarettes a day, shortness of breath, chest x-ray bilateral lobe
classified IV >115. What makes this patient require IV antibiotic treatment immediately?
a) Loss of consciousness, won’t be able to swallow
b) Chest x-ray
c) Classified patient as IV

74. What prevent this patient for getting better? Smoking

75. What empiric treatment in this case


a) Cefuroxime
b) Cotrimoxazole
c) Levofloxacin
d) Amoxicillin

76. Old man, hospitalized, HIV, what not to advise K-type


a) Not to take antacids with HIV medication
b) Not take HIV medication together
c) Not take HIV medication with SMZ/TMP

77. His wife ask if this HIV due to STD & if she is infected or no, what you recommend: k type
a) Say yes, he got it through sexual transmission
b) Tell her to ask your husband
c) Tell her to go and do HIV test

78. A patient had an acute gout attack and was treated, after that the doctor wants to initiate
allopurinol
a) Titrate till you reach the target dose of allopurinol
b) Titrate dose till you reach the target serum uric acid level
Starting dose: 100 mg daily PO. Maximal decrease in uric acid occurs within 1-3 weeks.
Usual: 300 mg daily PO titrated to urate levels; Maximum: 800 mg daily PO

79. Which drug is not used in acute gout:


a) Naproxen
b) Sulfinpyrazone is rarely used for treating gout
c) Colchicine
d) Narcotic
e) Prednisone
Uricosurics
Probenecid is recommended as an alternative first-line urate-lowering therapy, although it is available
only through Health Canada's Special Access Programme.
Other uricosuric agents that can be used include fenofibrate and losartan. Although fenofibrate and
losartan are not officially approved for gout prophylaxis, preliminary evidence suggests that either
fenofibrate or losartan may provide a small decrease in uric acid levels when used as adjunctive therapy in
patients with gout and dyslipidemia or hypertension.
While officially approved as a uricosuric, sulfinpyrazone is rarely used for treating gout as it requires
monitoring for blood dyscrasias and has other noteworthy adverse effects including increased risk of
bleeding and skin rashes.
Benzbromarone is a uricosuric agent available through Health Canada's Special Access Program; it may be
used in patients not responsive to conventional uricosuric therapy.

80. A man came (no work) wants to take inhaler for his son, divorced, no insurance, can’t pay
for the medication, what to do
a) Call the mother to take permission to dispense
b) Refuse to dispense, dispense for son only
c) Give him non pharm advise to improve asthma

81. Chicken pox, what to tell the mother


a) It is herpes zoster (varicella zoster)
b) It is contagious before appearance of lesions
c) It is passive immunity
d) You may still experience pain after the blisters heal
Chickenpox is caused by the varicella-zoster virus (human herpesvirus type 3); chickenpox is the acute
invasive phase of the infection, and herpes zoster (shingles) represents reactivation of the latent phase.

82. Treatment of anticipated N and V


a) Serotonin antagonist
b) Phenothiazine
c) Benzodiazepines
d) Corticosteroids

83. Treatment in chemotherapy (choice of agent)


depend on all except
e) Dose of drug
f) Type of drug
g) Route of administration
h) Site of tumor
84. Patient takes 5mg prednisone tab. She is taking 10mg/day for the 1st week then dose
decreased by half tab every week. Calculate total tab required
Answer: 1stweek: 10 x 7 = 70 mg
2nd week: 7.5mg x 7 = 52mg
3rd week= 5mg x 7 days= 35mg
4th week= 2.5 mg x 7 day = 17.5
Total = 175 / 5= 35 tablets

85. 300 mg Rifampin provided in market 5mg/5ml adult dose =300 mg BID. child >1kg 10 mg/
kg 4 adults, 20 children wt 20kg 10 children wt 30kg? 146 Capsules

86. Pt came with prescription for codeine tab, you do not have enough, what should you do:
a) Loan the remaining quantity from another pharmacy & refund it within 48hr
b) Buy from another pharmacy by verbal or written order
c) Buy from another pharmacy by written order and keep it in the sale record
Must be written (by fax)

87. In which reference you can look for dose adjustment in case of renal failure
a) AHFS
b) Martindale

88. Drug will be recall from the market due to its adverse effect on stomach, the physician
called you and asked you to order quantity from this drug because he see that stomach bleeding
is a minor effect what you should do
a) Stoke a big quantity as the doctor requested
b) Do not sell or stoke this drug since the time you received the recall order
c) Call the suppliers and order all quantity they have

89. LN has schizophrenia he is on Haloperidol, today he comes with prescription of Risperidone


instead of haloperidol. He asks for a chair because he cannot stand for a long time. You noted
that while he is setting, he stands many times and bends his back. Before changing haloperidol,
what doctor should assess?
a) Extra pyramidal effect (Akathisia)
b) Anticholinergic effect
Extrapyramidal side effects Prevention is key—use SGAs first-line.
(EPS; dystonia, parkinsonism, akathisia, tardive Q. If EPS occurs, first reduce dose; consider switch
dyskinesia, tardive dystonia, tardive akathisia) to SGA if on FGA.
Prophylactic use of anticholinergics (benztropine,
procyclidine, trihexyphenidyl) is not recommended
even with FGAs, & should be used only on short-term
basis to treat parkinsonism associated with FGAs.
Anticholinergics not recommended with SGAs
For akathisia:
If dose reduction is not effective, BBs propranolol
10–120 mg/day are the treatment of choice with
monitoring for hypotension.
Assessment: rating scales such as Simpson-Angus Scale, Benzodiazepines also provide symptom relief.
Barnes Akathisia Scale or ESRS are useful to assess EPS Anticholinergics are ineffective
and the Abnormal Involuntary Movement Scale or the For acute dystonia (acute torticollis, oculogyric
ESRS is used to assess TD crisis): benztropine or diphenhydramine IM, followed
Monitoring: baseline assessment in antipsychotic-naïve by reduction in dose or switch to SGA
firstepisode patients, For tardive dyskinesia:
In multiple-episode patients when initiating a new There is no evidence-based treatment— prevention is
antipsychotic, and in firstepisode and multiple-episode key. Use SGAs first-line.
patients whenever dosage of antipsychotic is changed; Antiparkinsonian medications are not effective and
assess weekly for 2–4 wk or until EPS resolves. may worsen symptoms.
In stable patients, assess for TD every 6 months or more If TD occurs, suggest consultation with a psychiatrist.
often in patients at higher risk (on FGAs, erratic Consider switching to an SGA.
medication adherence or intermittent treatment, female, For persistent, severe TD, consider clozapine trial
age > 55, diagnosis of an affective disorder, substance
abuse, diabetes)

90. This is the first episode for LN and his


symptoms have resolved with Risperidone,
what is the recommended duration for therapy
a) 6 months
b) 2 years
c) 1 year

91. Which side effect that LN suffers from


a) Akathisia
b) Acute dystonia
c) Tardive dyskinesia

92. Patient has nasal congestion, itchy eyes, water eyes, sneezing, and he started feeling
shortness of breath. Usually his symptoms are only nasal congestion every year around this
time. He wants something that will not cause drowsiness. what do u recommend?
a) Pseudoephedrine
b) Topical decongestants
c) Fexofenadine
d) Refer to the doctor
Consider the need for prescription therapy or referral for allergy testing if the patient has already tried
appropriate nonprescription therapy for 2 weeks without an adequate response, or if the allergen
responsible for symptoms cannot be readily identified. Also refer patients for further assessment if they
have signs or symptoms that are unilateral or are not usually associated with allergic rhinitis (e.g., fever,
facial pain, loss of smell or taste, recurrent epistaxis, purulent nasal or ocular secretions, postnasal drip with
or without rhinorrhea) or symptoms suggesting complications such as asthma.

93. What Q pharmacist should not ask to differentiate


between allergic rhinitis and common cold?
a) Is there any nasal discharge? In both
b) Do you have fever
c) For how long do you have symptoms

94. You will refer her, if she has


a) Dyspnea (S.O.B)
b) Headache
c) Symptoms for more 5 days

95. GM has rheumatoid arthritis suffers from severe symptoms. She took Hydroxychloroquine
200mg BID for 6 months, but now it is ineffective, so she went to her doctor who changed it to
methotrexate 15 mg injection weekly. GM takes CACO3 daily. What is the cause of
ineffectiveness of Hydroxychloroquine?
a) CaCO3
b) Low dose
c) Short course
d) Inappropriate ttt for severe cases
Hydroxychloroquine can also control symptoms of
mild RA. Funduscopic examination should be done
and visual fields should be assessed before and every
12 months during treatment. The drug should be
stopped if no improvement occurs after 9 months

96. What is main concern about using


methotrexate
a) Alopecia
b) Myelosuppression
c) Take on empty stomach to increase its
absorption
97. All of the following should be monitored with methotrexate except
a) Liver enzymes
b) CBC
c) TSH
Methotrexate is a folate antagonist with immunosuppressive effects at high dose. It is anti-inflammatory at
doses used in RA. It is very effective & has relatively rapid onset (clinical benefit often within 3 to 4 weeks).
Methotrexate should be used with caution, if at all, in patients with hepatic dysfunction or renal failure.
Alcohol should be avoided. Supplemental folate, 1 mg orally once/day, reduces likelihood of adverse effects.
Complete blood count (CBC), aspartate aminotransferase (AST), alanine aminotransferase (ALT), and
albumin and creatinine level should be determined about every 8 weeks. When used early in the course of
RA, efficacy may equal the biologic agents. Rarely, a liver biopsy is needed if liver test findings are
persistently twice the upper limit of normal or more and the patient needs to continue to use methotrexate.
Severe relapses of arthritis can occur after withdrawal of methotrexate. Paradoxically, rheumatoid nodules
may enlarge with methotrexate therapy.

98. If a dispensing error has happened in methotrexate dose as a result GM has taken
methotrexate daily instead of weekly for 1 week what should the pharmacist do?
a) Withhold apologizing because it is a ground for legal action
b) Report to health Canada
c) Report to the provincial authority
d) Offer to pay the cost of GM treatment
e) Report to ISMP

99. What is not important to talk in with this patient? Name of pharmacist who did the error

100. The most drug diversion in intensive care unit is? Fentanyl patches
Pseudoephedrine in community pharmacy

101. Drug X is insoluble in water and sparingly soluble in alcohol, what is the best dosage form
for formulation drug X
a) Elixir
b) Solution
c) Suspension
d) Paper powder

102. You are a pharmacy manger, the clinical counsellor told you that the pharmacist has
increased his duties by 30% to 50%. You can do all except:
a) Delete unnecessary activities which do not add benefit to the pharmacy
b) Offer to increase his salary
c) Do another job description includes his new activities
A change in technician and pharmacist job descriptions must be made with the involvement of both staff
and Human Resources Department guidance.

103. 1 week ago, you dispensed nitrofurantoin


to patient. Today she came to the pharmacy
and told you that she got stomatitis. Her
doctor and dentist did not find any cause for
her stomatitis. What to do?
a) Report to Medeffect (health Canada)
b) Report to ISMP

104. MG is a regular patient for 3 years. His medication profile includes Ramipril, ticlopidine &
oxycodone he has no refills for any of them. He told you that he forgot to go to his physician
because he thought that he has enough quantity of medications. His doctor is away until next
Monday& he has discovered the he does not have enough oxycodone. What is your best action?
a) Send him to a walk-in clinic to bring a prescription
b) Advance him some tablets till he brings authority from his physician on Monday
c) Try to call an emergency physician

105. The cost of necessary physician services is:


a) Fully insured & funded from federal provincial and territorial
b) Paid cash or by private insurance
c) Fully insured & funded from federal, provincial and territorial and may paid cash
or by private insurance
d) Funded from provincial tax
Fully insured by Federal, Provincial, and Territorial plans/funds

106. LO is 3 months infant. He was diagnosed with meningitis. The culture showed that is
G+ve streptococci. What is the microorganism
a) Staphylococcus aureus
b) Streptococcus pyogenes
c) Streptococcus pneumonia
d) Neisseria meningitis

107. What is the recommended antibiotic?


a) Gentamicin
b) Amikacin
c) Ceftriaxone
108. 45year-old male patient diagnosed with having DVT recently. Undergoing general surgery
and has HTN taking ACEIs and levothyroxine, what is the appropriate medicine for him as
thromboprophylaxis
a) Warfarin
b) Daltaparin
c) Heparin
d) Clopidogrel

109. Person has BPH and hypertension. What is most suitable treatment?
a) Prazosin Not used in BPH
b) Tamsulosin (selective)
c) Finasteride
d) Terazosin
Avoidance of anticholinergics, sympathomimetics, and opioids
Use of alpha-adrenergic blockers (eg, terazosin, doxazosin, tamsulosin, alfuzosin, silodosin), 5 alpha-
reductase inhibitors (finasteride, dutasteride), or the phosphodiesterase type 5 inhibitor tadalafil, especially
if there is concomitant erectile dysfunction
Transurethral resection of the prostate or an alternative procedure

110. A pregnant woman just had a C-section and the physician prescribed her meperidine Q4hr
for her pain. One day later, she told the nurse that she had restless legs and muscle twitches.
What is the reason for her symptoms?
a) The oxytocin delayed effect of the surgery
b) The delayed effect of the epidural after the surgery
c) Neurotoxicity of meperidine
d) Long bed resting (3 days)

111. A patient has asthma and takes Salbutamol and Fluticasone. She is coming early for her
salbutamol. Her profile includes:
Salbutamol 2 puffs before exercise … Filled 10, 20, 50, and 70 days ago.
Fluticasone 2 puffs BID … Filled 10, 20, 50, and 70 days ago.
(Dose can increase to four puffs BID if there is a cold).
What is a sign that her asthma is uncontrolled?
a) Using salbutamol daily 15min before exercise
b) Waking up at night twice with a cough
c) High dose of fluticasone
d) Peak flow meter FEV >90%
e) Salbutamol too much
112. What recommendation would you give to her doctor?
a) Add Salmeterol
b) Add ipratropium
c) Oral prednisone
d) Add terbutaline

113. Which drug requires special handling for the patient?


a) Digoxin
b) Hydroxyurea
c) Hydrochlorothiazide

114. You are a hospital pharmacist. You discovered that one of patients had an order for Losec
and technician who prepared prescription misinterpreted it as Lasix. However, the pharmacist
who was there in that shift is on vacation today. The patient has been taking the wrong
medication for three days so far including this day. Who is the first person you should contact?
a) The physician who wrote the prescription.
b) The pharmacist in charge of that shift
c) The technician who prepared the prescription
d) The patient’s family
e) The nurse on the patient care unit

115. Benzocaine cream/ointment/gel 0.1% ratio is 1:2:2 and total qty mitte 150g what is percent
of benzocaine?
a) 20.0%
b) 0.03%
c) 0.02% → Total 5 so 0.1 /5 = 0.02
d) 0.2%
e) 0.3%

116. You have compound A which is 20% w/w salicylic acid, you also have compound B which
is 10% w/v salicylic acid. You want to prepare a 2% salicylic acid solution in which you end up
using 1% of each of the above compounds (A and B). You will use
a) 5ml of compound a and 10ml of compound B
b) 4ml if compound A and 8ml of compound B
Answer:
Compound A: 20 mg -------- 100 mg 1 mg ------ X mg X= 1*100/20 = 5 mg of compund A
Compound B: 10 mg -------- 100 ml 1 mg ------ Y ml Y = 1*100/10 =10 ml of Compound B

117. What is a nested case control study


a) A cohort study in a case control study
b) A case control in a cohort study
c) A randomized controlled study

118. Arthritis patient needs topical treatment, What the best option for her
a) Capsaicin
b) Diclofenac

119. She then wants to try something natural for her knee arthritis? Hyaluronic acid

120. What is the counselling on insulin pen


a) Draw regular insulin first then NPH
b) Draw NPH then regular after

121. Energy drink is being sold in your pharmacy; the manufactures do not recommend this
product for patients under 18. As a pharmacist, what is the best thing to tell the manager?
a) Put a caution signage near the drinks to educate the customers
b) Restrict to adults’ only above age
c) Do not stock this product entirely

122. How to manage a vaccine in the fridge, to keep it at its proper temperature
a) Keep it in a bar size fridge, just for pharmaceuticals
b) Adjust the temp from 0-8C
c) Regular size fridge but only keep it in the door
d) Remove any frost from the fridge 1cm and above
123. Patient is travelling to USA, and wants to buy 1000 tab of pseudoephedrine 60mg and
bring it to his family and friends for the upcoming flu and cold season. All are ethical concerns
for the pharmacist except
a) Potential for drug abuse
b) Pseudoephedrine has many drug-drug interactions
c) Can only travel with medications for personal use only
d) There is no pharmacist-patient intervention with the other family members

124. Uncontrolled HTN may lead to all except


a) PAD
b) Tricuspid valve dysfunction
c) Hypertrophy
d) Stroke
e) Retinopathy

125. Domperidone has been questioned to cause infertility, where to look for:
a) Micromedex
b) Drugs in Pregnancy and Lactation
c) Hansten and Horn Drug Interactions Assessment and Management
d) Briggs

126. A pharmacy employee has noticed that his co-worker has been abusing his sick days lately,
as a manager of the pharmacy what is the best approach to go about this situation.
a) Tell him according to human resources it is his right and we can’t do anything about it
b) Tell him sick-days is part of the workers right and I cannot interfere
c) Monitor and review the recent sick days report and if you notice a trend in workers
abusing or taking too many sick days, then talk to the accused employee about the
situation
d) Leave a letter in the accused worker’s letter box informing him of the accusation

127. Patient was on Levothyroxine 0.15mg, he was admitted to the hospital and upon contacting
his physician they discovered that he should be on 0.5mg instead and his dose was increased a
few months ago. However, the patient had lost his prescription and the pharmacist just gave him
a repeat of what he originally had on his profile. Using a root and cause analysis, where can you
identify where the error occurred
a) There is no 0.5mg formulation
b) There was no dialogue between the physician and the patient regarding the change
of his dose to a higher dose
c) Pharmacist did not double check the patient’s profile and never compared it to the
original hard copy originally received from the patient
d) Pharmacist did not double check to see what the patient was taking on his profile

128. Diabetic patient on metformin and glyburide. 2 cups of alcohol per day and doubles it on
the weekend
a) Alcohol should be avoided with glyburide
b) Alcohol is contraindicated with metformin
c) Excess alcohol will cause hypoglycemia
Metformin monograph: acute or chronic EXCESSIVE alcohol intake is CI
Glyburide monograph: Avoid Alcohol

129. Mom comes in saying she tried permethrin twice for her child who have long hair, what is
the most likely reason for it not working
a) She didn’t leave it in long enough
b) Not enough quantity
Treatment failure of head lice with permethrin may fail if hair is not thoroughly soaked. Two bottles are
often needed for thick or long hair.

130. What do you recommend for her child to use?


a) Lindane
b) Tea tree oil
c) Permethrin 5%
d) Permethrin 1%

131. Mom discovered that she has lice, and she is 2 months pregnant, what do you recommend
for her?
a) Permethrin
b) Soak head in vinegar

132. Which one of these is schedule I:


a) Terbinafine cream
b) Nitroglycerin spray
c) IV B12
d) Dimenhydrinate IV
e) K salt
133. Woman with UTI, has fever and chills, and taking OC, What causative microorganism
a) Escherichia coli
b) Pseudomonas aeruginosa

134. What the DOC


a) Nitrofurantoin
b) Cotrimoxazole
c) Norfloxacin

135. What is the defense mechanism that


prevents UTI in females?
a) Short urethra (Risk factor for Cystitis)
b) Low urinary urea concentration
c) Low urinary PH

136. Glucose ketone kit availability which reference do you find it.
a) CPS
b) Drug info hand book
c) PSC = CTMA now

137. Pharmaceutical care for all except?


a) Cure the disease
b) Prevent the disease
c) Delay the progression of the disease & patients’ symptoms
d) Prevent the drug related problems

138. Why we give vitD with Ca?


a) Increase the intestinal passive absorption of Ca
b) Increase the intestinal active absorption of Ca
c) Improve bone resorption

139. What will increase gastric bleeding with NSAIDs?


a) Quinolone
b) Prednisone → Also SSRIs

140. Patient have hyperkalemia which medication cause it


a) Aliskiren → Also ACEI and ARBs
b) Furosemide
c) Atorvastatin
Direct Renin Prevents renin from converting angiotensinogen to angiotensin I. Thus, acts earlier in the
Inhibitors renin–angiotensin–aldosterone system than ACE inhibitors or ARBs and effective as ARBs,
Aliskiren ACE inhibitors & thiazides. The drug has a long duration of action and lowers BP to the same
extent as drugs from other antihypertensive classes.
Aliskiren should be used as an add-on agent after all first-line therapies have been tried.
S.E: Diarrhea. The incidence of dry cough and hyperkalemia is low compared with ACEIs.
Avoid combining with an ACEIs or ARB in patients with significant renal impairment.
Grapefruit juice may reduce serum concentrations.
Contraindicated in pregnancy, caution when prescribing to women of childbearing potential
May take 4 wk to realize maximum antihypertensive effect.
Effect on cardiovascular outcomes not yet established.
Limited data in patients with greater than moderate renal dysfunction.

141. Patient has history for respiratory infection & going to travel & taking fluticasone, doctor
prescribe for her azithromycin 2 tablet first day & then one tab daily, why as a pharmacist you
think doctor prescribed for her antibiotic:
a) Because she has history of respiratory infection

142. What you will Counsel her about azithromycin


a) You have to separate between the two tablets of the first day.
b) Space between the tablet & any other dairy product.
Azithromycin tablets can be taken with or without food.

143. What do you tell her about fluticasone turbohaler?


a) Clean it with wet cloth
b) Take a deep inside breath to reach your lung
c) Shake it every time before use

144. Patient taking phenytoin, has to take COC:


a) Estrogen50/estradiol
b) Estrogen35/estradiol
c) Progesterone only
d) Spermicide

145. A woman wants OC, she do not like


condoms has seizure and she is on phenytoin
she is also obese what do you recommend?
a) Spermicide
b) Micronor → Progestin-only
c) Levonorgestrel
d) COC
146. Patient is travelling to Mozambique, doctor prescribed to him Mefloquine, what is the right
dose? Take it one week before and until 4 weeks after coming back
Mefloquine Used to prevent malaria in travellers going to areas reporting chloroquine-resistant P. falciparum
Administer with a meal and with at least 240 mL of water
Start at least 1 wk prior to exposure and continue weekly for 4 wk after leaving the endemic area
Adults: 250 mg base (1 tablet) once/wk PO
Children: 5–9 kg: 31.25 mg base (one-eighth tablet) once/wk PO
Children: 10–19 kg: 62.5 mg base (one-quarter tablet) once/wk PO
Children: 20–29 kg: 125 mg base (one-half tablet) once/wk PO
Children: 30–45 kg: 187.5 mg base (three-quarter tablet) once/wk PO
Children: >45 kg: adult dose
If it is not possible to initiate mefloquine 1 wk prior to exposure, a loading dose can be given to
rapidly achieve the effective levels:
Loading dose, >45 kg: 250 mg base daily PO for 3 days, then 250 mg base weekly thereafter
S.E: Common: dizziness, nausea, vomiting, diarrhea, headaches, sinus bradycardia, nightmares,
insomnia, mood alteration, anxiety, irritability, hair loss, skin rash.
Rare: permanent dizziness, vertigo, tinnitus, and loss of balance; seizures; psychosis; thrombotic
thrombocytopenia purpura.
Contraindicated in patients with a history of seizures, depression, generalized anxiety disorder,
psychosis, schizophrenia or other psychiatric disorders, self-endangering behaviour, suicide
attempts or suicidal ideations.
Contraindicated in patients who are hypersensitive to mefloquine or structurally related
compounds, e.g., chloroquine, quinidine, quinine.
Loading dose is associated with an increased risk of depression.
When possible, mefloquine may be started 4 wk prior to departure to allow time to assess for
adverse effects prior to travel since 70% of severe adverse reactions occur within first 3 doses.
Drug of choice for pregnant women who require prophylaxis in chloroquine resistant area.
147. What else you can council him to do
a) Put DEET all over the body even the covered area
b) Put the sun block 20 min after DEET
c) Wear black long sleeves
d) Sleep in mosquito net
Nonpharmacologic Choices: Malaria transmission by the anopheline mosquito mainly occurs between dusk
and dawn. The following measures optimize protection during this time:
Use insect repellents containing diethyltoluamide (DEET) before outdoor activity during the main hours of
malarial transmission. DEET has been rarely associated with neurologic side effects in children exposed to
high concentrations (>35%) and prolonged use.
The American Academy of Pediatrics now supports the use of DEET 30% in children as young as 2 months of
age. In standard formulations, DEET 30% is effective for 4–6 hours.
Picaridin (icaridin) 20% is comparable to DEET across the majority of mosquito vectors, but lasts for 8 hours
and has greater acceptability due to less odour and greasiness.
para-Menthane-3,8-diol (PMD) is a synthetic derivative of oil of lemon eucalyptus that is effective for <2
hours at concentrations of 10%. PMD is not recommended for children <3 years of age and should be
considered a second-line option when DEET or picaridin cannot be used.
Use bed nets, preferably impregnated with permethrin. Use mosquito coils, aerosolized insecticides or
electrically operated insecticide generators containing pyrethroids.
Wear clothes covering exposed skin, weather permitting. Permethrin-impregnated clothing adds an
additional measure of protection. Sleep in an air-conditioned or screened room if possible.

148. A mother is afraid from vaccinating her child because of needle pain, what should you tell
her before injecting him?
a) Give diclofenac 12.5 supp.
b) Breastfeed him before and after vaccination (before, during, after vaccination)
c) Lay child supine before giving vaccine (hold you baby)
d) Cold compresses
e) Use topical anesthetic Lidocaine 10 -20 min before (20 to 60 minutes)
f) Give Acetaminophen 10-15 mg kg 30 min before vaccination
g) Tell him it is not painful
h) Tell nurse to hold him while he is in upright position

149. Fragile fracture is?


a) Fracture due to minimal or simple trauma
b) Fracture associated with osteoporosis
pharmacologic therapy of osteoporosis is recommended for high-risk individuals and may be considered in
individuals at moderate risk. A hip fracture or a vertebral fracture occurring after menopause or age 50
automatically places a patient in a high-risk category irrespective of BMD, unless the fracture occurred due
to a high trauma accident (e.g., car accident, falling down 3 or more steps). In CAROC, corticosteroid therapy
or a fragility fracture at a site other than hip or spine moves a patient to a higher risk category than that
indicated by BMD and age. Two such fractures place a patient in a high-risk category. A fragility fracture is
one that occurs with low trauma (e.g., a fall from a standing height or less). An individual in the “low
risk” category is not likely to receive clinically meaningful fracture prevention benefit from pharmacologic
therapy

150. Doctor prescribe off label drug, what you can tell the patient:
a) Off label come after post surveillance of the drug
b) Health Canada does not give NOC for OFF label indication
NA (both INCORRECT). Off-label rely on small studies, expert consensus, or seeding trials.

151. Drug that cause SIADH? Paroxetine


Drugs and SIADH
Many drugs cause fluid retention which can result
in dilutional hyponatremia. Some drugs
trigger vasopressin release and/or potentiate the
renal effect of endogenous vasopressin; some
have a direct vasopressin-like effect on the
kidneys (eg, oxytocin, desmopressin).
There is some debate as to whether drug causes of
euvolemic hyponatremia that do not directly
involve vasopressin or its receptors should be
considered SIADH, but most authorities include
drugs as causes.
Many drugs have been linked to SIADH, but members of five drug classes are most often implicated:
analgesics (particularly opioids and nonsteroidal anti-inflammatory drugs), antiseizure drugs (particularly
carbamazepine), antidepressants (particularly certain selective serotonin reuptake inhibitors, venlafaxine),
antipsychotic drugs, and cytotoxic drugs (particularly cyclophosphamide and vincristine). Not all drugs in each
class seem equally causative

152. Deficiency in sodium cause? Seizures

153. Treated with? Hypertonic saline


The syndrome of inappropriate ADH (vasopressin) secretion is defined as less than maximally dilute urine in
the presence of serum hypo-osmolality, in patients with normal adrenal, thyroid, renal, hepatic, and cardiac
function who do not have hypotension, volume depletion, or other physiologic causes
of vasopressin secretion. SIADH is associated with myriad disorders. Hyponatremia is the result, and
symptoms are those of hyponatremia. Diagnosis is by measurement of serum and urine osmolality and
electrolytes. Treatment is with water restriction, sometimes with oral or intravenous sodium chloride, and
rarely with vasopressin receptor antagonist drugs such as conivaptan or tolvaptan.
Pathophysiology
Vasopressin is a hormone produced by the posterior pituitary to help control fluid homeostasis. The
hormone increases water reabsorption in the distal nephron, producing a concentrated urine and diluted
plasma. Vasopressin release is stimulated by any of the following:
➢ Increased plasma osmolality
➢ Decreased blood volume
➢ Decreased blood pressure
➢ Stress
➢ Certain drugs
Low plasma osmolality inhibits vasopressin secretion, allowing the kidneys to produce dilute urine.
Vasopressin release is inappropriate in the presence of normal or low plasma osmolality and normal or high
blood volume and blood pressure. In such cases, the water inappropriately retained by the kidneys
ultimately causes euvolemic (dilutional) hyponatremia, in which total body sodium and thus extracellular
fluid (ECF) volume are normal or near normal; however, total body water is increased.
Symptoms and Signs
Symptoms of SIADH are those of hyponatremia, which mainly involve central nervous system dysfunction
and generally occur when the effective plasma osmolality falls to < 240 mOsm/kg (< 240 mmol/kg).
Symptoms can be subtle and consist mainly of changes in mental status, including altered personality,
lethargy, and confusion. As the serum sodium falls to < 115 mEq/L (< 115 mmol/L), stupor, neuromuscular
hyperexcitability, hyperreflexia, seizures, coma, and death can result.
Diagnosis
Serum and urine osmolality and electrolyte measurements showing inappropriately high urine osmolality
compared to serum osmolality
Normal adrenal, thyroid, renal, cardiac and hepatic function
SIADH is suspected in patients who have hyponatremia and are euvolemic (ie, neither hyper- nor
hypovolemic on physical examination).
Laboratory tests should include serum and urine osmolality and electrolytes. Euvolemic patients should also
have thyroid and adrenal function tested. Hypo-osmolality in euvolemic patients should cause excretion of a
large volume of dilute urine (eg, osmolality < 100 mOsm/kg [<100 mmol/kg]) and specific gravity < 1.003).
Serum sodium concentration and serum osmolality that are low and urine osmolality that is inappropriately
high (120 to 150 mmol/L [120 to 150 mOsm/kg])) with respect to the low serum osmolality suggest volume
overload, volume contraction, or SIADH. Volume overload and volume contraction are differentiated
clinically.
When neither volume overload or volume contraction appears likely, SIADH is considered. Patients with
SIADH are usually euvolemic or slightly hypervolemic. BUN (blood urea nitrogen) and creatinine values are
normal, and serum uric acid is generally low. Urine sodium concentration is usually > 30 mEq/L (30 mmol/L),
and fractional excretion of sodium is > 1% (for calculation, see Evaluation of the Renal Patient).
Diagnosis of etiology should be pursued based on symptoms and signs. Because potentially causative drugs
are relatively commonly used, other etiologies must also be considered even when patients are taking such
a drug. In general, a chest x-ray should be done. Central nervous system (CNS) imaging can be reserved for
patients in whom a brain disorder is clinically suspected or no other cause for SIADH can be found.
Treatment
➢ Fluid restriction
➢ Sometimes a vasopressin receptor antagonist
➢ Sometimes hypertonic saline
When SIADH is present, severe water restriction (eg, 250 to 500 mL/24 hours) is generally required.
Additionally, a loop diuretic may be combined with IV 0.9% saline as in hypervolemic hyponatremia. Lasting
correction depends on successful treatment of the cause, particularly treating infection and stopping any
drug cause. When the underlying disorder is not correctable, as in metastatic cancer, and patients find
severe water restriction unacceptable, demeclocycline 300 to 600 mg orally every 12 hours may be helpful
by inducing a concentrating defect in the kidneys. However, demeclocycline is not widely used due to the
possibility of drug-induced acute kidney injury.
IV conivaptan, a selective vasopressin receptor antagonist, causes effective water diuresis without
significant loss of electrolytes in the urine and can be used in hospitalized patients for treatment of resistant
hyponatremia. Oral tolvaptan is another vasopressin receptor antagonist with similar action
to conivaptan. Tolvaptan use is limited to less than 30 days due to the potential for liver toxicity and it
should not be used in patients with liver or kidney disease
Hypertonic saline infusion should be reserved for patients with severe, symptomatic hyponatremia and
should be used cautiously because too-rapid correction risks complication,s such as the osmotic
demyelination syndrome. Typically, correction should aim to raise the serum sodium by no more than 4 to 6
mEq/L (4 to 6 mmol/L) which should be done over 4 to 6 hours.
Key Points
Patients with SIADH are euvolemic & have low serum osmolality but inappropriately high urine osmolality.
Despite the name, not all patients with SIADH have excessive vasopressin.
Causes include central nervous system disorders, lung disorders (particularly infections), certain cancers
(particularly lung cancer) and certain drugs.
Water restriction and treatment of cause may be adequate.
Some patients will also require a vasopressin receptor antagonist or hypertonic saline

154. Rosacea patient came to the pharmacy with


red swollen face suspected rosacea, you can tell
him to do:
a) Avoid Alcohol
Triggers worsen Rosacea: Sunlight, Heat, Hot beverages,
Spicy foods, vinegar, Alcohol, Use of astringents,
Application of topical corticosteroids to the face &
Emotional stress

155. What is the best local treatment?


Metronidazole

156. What can you add systemic?


Minocycline
157. Hospital accreditation means:
a) Assess hospital quality of care against standard of excellence
Hospital accreditation has been defined as “A self-assessment and external peer assessment process used by
health care organizations to accurately assess their level of performance in relation to established standards
and to implement ways to continuously improve”

158. Erectile dysfunction patient is taking Sildenafil 25mg but his partner complaining of
spontaneity. What do you think is the reason?
a) Taking it after alcohol consumption
b) Didn’t take it with a high fat meal
c) Sildenafil is short acting
Spontaneity in this context means "NOT obliged to have a scheduled act of sexual intercourse".
Sildenafil allows only a 4-hour window to have an enhanced sex with adequate erectile function.
The partner is complaining that they miss "spontaneity" and are compelled now to follow a "routine sex
schedule". A solution is tadalafil.

159. What is your suggestion?


a) Increase dose to 50 mg sildenafil prn
b) Change it to 5mg Vardenafil
c) Change it to 5mg Tadalafil daily
Do not drink large amounts of alcohol with sildenafil. Drinking too much alcohol can reduce your ability to
get an erection. Absorption of sildenafil is delayed by high-fat meals.
One of the main differences between short acting (Sildenafil)& long acting (Tadalafil 20 mg) is what is called
window of opportunity.it is 4 hours with Sildenafil and 33 hours with Tadalafil.So taking a lower dose on a
daily basis from Tadalafil (5 mg) can allow more spontaneous sexual activity.
Once daily Tadalafil has shown significant efficacy even after failure of on-demand treatment. In a
controlled cross-over study of on-demand versus daily Tadalafil treatment, 72% of the patients preferred
once daily administration, mainly because of superior and longer efficacy allowing a more spontaneous
sexual life.
Whence this bedroom-centric brand loyalty? Simple: Cialis lasts longer. Viagra and Levitra are effective for
approximately four hours, Cialis for 36. If a man takes Cialis on Friday evening, he can count on erection
assistance through part of Sunday. Both men and women say they like the fact that Cialis allows them to
take their eyes off that ticking clock and make love whenever they wish. So, for dating couples or new lovers
who prize sexual spontaneity, Cialis has a clear advantage.
Vardenafil 5-10 mg once daily PRN (window of sex = 8 hours), tadalafil 10-20 mg once daily PRN (window of
sex = 24 hours), or daily low-dose tadalafil (2.5 - 5 mg)

160. All are concerns of tertiary literature except:


a) Availability
b) Personal bias
c) Accuracy
d) Transcribing error

161. Which is not primary literature


a) Pharmacoeconomics
b) Case studies
c) Case review
Primary literature Secondary literature Tertiary literature
Description Original research articles Used to analyze, interpret, evaluate, Summarize and interpret the
(clinical studies, reports, etc.) or locate primary literature and offer findings of primary and
that provides detailed abstracting services (an abstract is a secondary literature to provide
description of the study brief synopsis of the study) fundamental knowledge.
design, intervention and Clinical guidelines, product
outcome, serve as a source of monographs, etc. are also used
information for other for development of tertiary
resources. references.
Advantages Current Fast access to primary literature. Easy to use
All reader to evaluate the Searchable → Researchers can obtain Convenient
study and read about design, information on certain topic. Usually Specialized
methodology, analysis, and index, highly quality, peer-reviewed Provide highly referenced, well
validity. literature. New literature is usually reviewed information
Unbiased evidence (if peer indexed within short period of time Its use is widely accepted in
reviewed before puplication) (weekly or monthly) i.e. current. with primary literature practice
Disadvantage Flows in design & Some delay in indexing (lag line) Long lag time (may take few
methodology can lead to Each database has a scope and index years to update a textbook –
questionable results and journals that suite this scope. outdated?)
conclusion Researchers may not retrieve all Inaccurate referencing may an
Reader must be able to assess information using one database. issue. Reader should be aware
the quality (ex, peer-reviewed It may be time consuming to search of potential bias (ex. personal
with sounding design) and through all available literature on bias of the author.
interpret primary literature certain topic. The quality of the 3rd reference
Adoption by practitioners may Research must have skills to search depends on the quality of 1st
take time database retrieve information (need research used)
Some topics may be key, words, MeSH, etc) Information peovided can be
researched using a single trial limited/censored by author
which can be misleading Space limitation
Inability to retrieve all literature
Examples Sources: peer reviewed Examples of databases: MEDLINE, Textbook, compendia, review
journals PubMed, embase, lowa drug articles, Pharmacist letters,
Examples: research articles, information service (IDIS). online resource such as
conference paper/posters, International Pharmacy Abstract MicroMedix, etc.
thesis (if includes an original (IPA). scopus, Cochrane library, and
research. CINHAL.
If it includes a review of Search using key words examples:
primary literature then it is review articles: literature reviews,
secondary), potency, case meta analysis, guidelines, abstracting
series and case reviews. sources (like MedWatch), newsletters
or bulletins, and journals specialized
in puplishing literature review.

162. Auxiliary label for oral nystatin suspension all except:


a) Room temperature
b) Keep in mouth as long as possible
c) Shake well
d) Discard after 14 days
e) Complete as directed
Auxiliary label for nystatin suspension: Shake oral suspension well prior to use. No refrigeration. Finish all.
Use with or without a meal. Swish and gargle in the mouth for as long as possible prior to swallowing.

163. Side effect of Pramipexole except?


Orthostatic hypotension, somnolence, confusion, hallucinations, nausea, vomiting, sudden sleep attacks;
caution patients about potential compulsive behaviours such as pathologic gambling or hypersexual
behaviour; caution patients about driving or operating dangerous machinery. Risk of parkinsonism-
hyperpyrexia syndrome with abrupt discontinuation; taper gradually. Drug holidays not recommended.

164. Baclofen AUXILLARY label


a) May cause drowsiness
b) Avoid Grape fruit
165. Patient with opioid overdose the patient come to emergency with miosis, then require to
look at which reference for Management
a) TC
b) CPS → in the yellow pages
c) Lexi
d) Micromedex
Any drug monograph to cover overdose = CPS

166. Patient has Onychomycosis (tinea pedis), you will refer all except:
a) Nail with dark (or change) colour
b) Thick nail
c) Preschool age
When should you see a health-care
provider?
➢ If your toes or fingers
become swollen, red,
painful or drain pus. This
may be a bacterial infection
that will need antibiotics.
➢ If you have a fungal
infection that is not clearing
up or you are getting other
infections on your skin.
➢ To make sure the medicine
you take by mouth is
working and not causing
any harm.
➢ If you develop any side
effects from your
medications such as rash,
nausea, vomiting, fatigue,
abdominal pain or dark-
coloured urine.

167. Schedule for area A, area B & area C in the pharmacy& give the cost of goods & average
inventory of each area separately
a) Ask about the turnover of the pharmacy
168. How to decrease (or adjust) this turn over & the choices was:
a) Increase the sales of section A
b) Decrease order from section C

169. In a hospital, a drug is about to be interchanged with another medication. All must be done
except? The Q is NOT about therapeutic interchange or auto-sub (no way), but about the
inclusion of a new formulary drug.
a) Approval of the ethical committee
b) Approval of the key doctors
c) If the new medication is at better price
d) Therapeutic index
e) Consult reference
At hospitals there is no importance of ethical issues in choosing the medication to be available in pharmacy

170. Patient do not want to quit smoking. Pharmacist told him you must quite due to your
illness (not serious). Pharmacist demonstrates
a) Autonomy
b) Veracity
c) Non maleficence
d) Paternalism

171. If iron is taken with levothyroxine: will increase TSH


Iron will decrease the absorption of levothyroxine (T4): Hypothyrodism & Hight TSH

172. CADTH Common Drug Review (CDR) objective


Reviews drugs and makes reimbursement recommendations to Canada's federal, provincial, and territorial
public drug plans, with the exception of Quebec, to guide their drug funding decisions.
Conducting reviews of the clinical, cost-effectiveness, and patient evidence for drugs and providing
formulary listing recommendation. Gives options for drug prices to compare between.

173. How to avoid serious effect from heparin & morphine in the hospital
a) Separate
b) Avoid buying potent concentration

174. Symptoms of hypothyroidism, what causes it?


a) Amiodarone /lithium

175. Patient is taking Bupropion for TID & miss morning dose what he can do
a) Skip dose, taking it next time
The usual recommended dose of sustained-release bupropion hydrochloride is 100 to 150 mg/day given
once daily. As with all antidepressants, the full antidepressant effect of BUPROPION SR may not be evident
until several weeks of treatment. In patients who are not responding to a dose of 150 mg/day the dose may
be increased up to a maximum of 300 mg/day. Dose increases should occur at intervals of at least 1 week. In
order to minimize the risk of seizures, single doses of BUPROPION SR must not exceed 150 mg. Doses greater
than 150 mg/day should be administered BID preferably with at least 8 hours between successive doses.
BUPROPION SR should be taken at the same time each day and no more than the recommended dose should
be taken each day. If the normal administration time has been missed, the dose should be skipped and
administration resumed at the normal administration time of the following day.
Patients should be advised to swallow BUPROPION SR tablets whole with fluids, and NOT to chew, divide,
crush or otherwise tamper with the tablets in any way that might affect the release rate of bupropion.

176. Neuropathic pain best local treatment


a) Menthol
b) Methyl subsalicylate
c) Lidocaine
CTC: Tramadol and topical lidocaine can be considered as second line. Combination therapy can be helpful.
Do not combine TCAs with SNRIs because of the potential risk of serotonin syndrome.
Topical treatments may be helpful as adjunctive therapy combined with oral analgesic, or as monotherapy
for elderly patients if oral agents are not tolerated. Topical lidocaine has demonstrated efficacy for relief of
neuropathic pain in several low-quality studies; however, a Cochrane review found no evidence from good-
quality randomized controlled trials to support its use for neuropathic pain.
The majority of studies involved a transdermal patch, although some studies investigated the topical cream;
the patch is not yet available in Canada. Other topical treatments, such as topical ASA, diclofenac, menthol,
clonidine, doxepin, isosorbide dinitrate spray and capsaicin cream require further study.
A high-dose capsaicin patch (8%) demonstrated superior pain relief compared with capsaicin cream (0.025–
0.075%), and similar rates of adverse events; the capsaicin patch is not yet available in Canada.

177. Absence seizure patient, he is taking Valproic & not controlled


a) Change the dose of Valproic
b) Change to Ethosuximide
c) Add Ethosuximide
A little tweak in the Q can change the answer to increase the dose (versions exist)
If absent seizure, start with Ethosuximide. But if combination generalized plus absence, I should start with
Valproic acid then if no effect, add one of following: Clobazan, Topiramate, Lamotrigine, Levticetam
Childhood Absence Epilepsy
For childhood absence epilepsy, ethosuximide and valproic acid are similarly effective in preventing seizures,
and both medications are superior to lamotrigine. Ethosuximide is associated with lower rates of attention
difficulties and fewer behavioural problems than valproic acid and is usually the drug of first choice in
childhood absence epilepsy. However, ethosuximide is not effective in preventing other seizure types such as
generalized tonic-clonic seizures, and should not be used as monotherapy in children with multiple seizure
types
178. Patient took Statin & ask what you
think if patient take it will cause
Myotoxity
a) Gemfibrozil
b) Ezetimibe
GEMFIBROZIL, ATORVASTATIN.
Either increases toxicity of the other by
pharmacodynamic synergism.
Increased risk of rhabdomyolysis.
High likelihood serious or life-threatening
interaction.
Contraindicated unless benefits outweigh risks
and no alternatives available.

179. Patient has asthma take salbutamol and nicotine patch. His asthma not controlled so what
the reason
a) Drug disease interaction
b) Drug drug interaction
c) Inappropriate medication

180. Patient try to suicide, depressed mood, feel guilty, no sleep. Which antidepressant to give?
a) Venlafaxine
b) Mirtazapine

181. Topical ttt for Shingles! Calamine. Docosanol (Abreva) for cold sores
Hints to be more comfortable:
➢ Use calamine lotion to stop the itching. A shower or cool bath (with or without oatmeal) may help.
➢ If your itching bothers you a lot, you can take an antihistamine. Ask your health-care provider which
one is best for you.
➢ Keep rash clean and dry. Trim fingernails to prevent scratching. Scratching can cause skin infections.
➢ Wear loose cotton clothing if possible. Avoid wool clothing.
➢ You can take acetaminophen or ibuprofen to treat pain and fever. Do not take acetylsalicylic acid
(Aspirin, ASA); it may cause a serious condition called Reye syndrome.
➢ If you develop eye problems, call your health-care provider immediately.

182. SE of carbamazepine
a) Visual problem
b) Gingivitis
c) Peripheral edema
Iminostilbene By blocking Na channels reduces abnormal impulses in brain.
Derivatives Generalized tonic-clonic, focal (partial) seizures or unclassified tonic-clonic seizures
Carbamazepine When added to existing anticonvulsant therapy, the drug should be added gradually while
the other anticonvulsants are maintained or gradually decreased, except for phenytoin,
which may be increased.
Initially, 100 to 200 mg once or twice a day depending on the severity of the case and
previous therapeutic history. The initial dosage is progressively increased, in divided doses,
until the best response is obtained. The usual optimal dosage is 800 to 1200 mg daily. In
rare instances some adult patients have received 1600 mg. As soon as disappearance of
seizures has been obtained and maintained, dosage should be reduced very gradually until a
minimum effective dose is reached.
TEGRETOL Suspension should be well shaken before use since improper re suspension
may lead to administering an incorrect dose. Since a given dose of TEGRETOL Suspension
produces higher peak carbamazepine levels than the same dose in tablet form, it is
advisable to start with low doses and to increase slowly to avoid adverse reactions. When
switching a patient from TEGRETOL Tablets to TEGRETOL Suspension, the same
number of mg per day should be given in smaller, more frequent doses (i.e., BID Tablets to
TID Suspension).
S.E: Q. Rash 5–10% (Steven Johnson Syndrome); ↑ liver enzymes; transient neutropenia
(monitor WBCs); aplastic anemia (rare); hyponatremia. blurred vision, nystagmus (rare),
ataxia, dizziness, somnolence, Very common: vomiting, nausea. Common: dry mouth and
throat. Uncommon: diarrhea, constipation, Common: edema, fluid retention, weight
increase, hyponatremia and blood osmolarity decreased due to antidiuretic hormone
(ADH)-like effect occurs, leading in rare cases to water intoxication accompanied by
lethargy, vomiting, headache, confusional state, neurological disorders. Very rare:
galactorrhea, gynecomastia.
Advantages: Linear pharmacokinetics. Gradual increase in dosage is advised.
Disadvantages: Substrate of CYP3A4 and potent inducer of several cytochrome P450.
Should be swallowed unchewed with a little liquid during or after a meal.
May decrease efficacy of hormonal contraceptives; adjunctive nonhormonal birth control is
recommended.
May worsen absence seizures; may produce or exacerbate myoclonus.
Q. HLA-B*1502 test used to identify those at risk for serious side-effects to carbamazepine
Monitoring
Complete blood counts, including platelets and possibly reticulocytes and serum iron,
should be carried out before treatment is instituted, and periodically thereafter.
Baseline and periodic evaluations of hepatic function must be performed, particularly in
elderly patients and patients with a history of liver disease.
Pre-treatment and periodic complete urinalysis and BUN determinations should be
performed.
Carbamazepine has been associated with pathological eye changes. Periodic eye
examinations, including slit-lamp funduscopy and tonometry are recommended.
183. Why we give Vit. D with Ca?
a) Vit D increases Active Ca absorption
b) To increase the intestinal absorption of passive ca
c) To increase the intestinal absorption of the active ca
1,25Dihydroxyvitamin D3 (1,25(OH)2D3) the hormonally active form of vitamin D, through its genomic
actions, is the major stimulator of active intestinal calcium absorption which involves calcium influx,
translocation of calcium through the interior of the enterocyte and basolateral extrusion of calcium by the
intestinal plasma membrane pump.

184. What is the negative superiority? Negative superiority is a term to describe the non significant
trials (P value more than or equal 0.05) where the null hypothesis is accepted)

185. Differentiation between allergic rhinitis and common (viral rhinitis) cold except?
a) Allergic rhinitis has no fever
b) Nasal discharge
CTMA: The common cold is characterized by a sore throat usually resolving within a few days, followed by
nasal congestion, rhinorrhea, sneezing and cough. Nasal discharge can sometimes be purulent and mistaken
for bacterial sinus infection. Fever is infrequent in adults but common in children. Symptoms peak around
day 2–4 and begin to resolve by day 7. For a small proportion of patients, symptoms such as cough can still
be present after 3 weeks. The common cold is usually a self-limiting illness confined to the upper respiratory
tract. It can sometimes predispose individuals to bacterial complications, such as otitis media (especially in
children via dysfunction of the eustachian tube), bacterial rhinosinusitis and pneumonia. It may also cause
exacerbations of asthma.
Common Cold- Symptoms and Signs
After an incubation period of 24 to 72 hours, cold symptoms begin with a scratchy or sore throat, followed
by sneezing, rhinorrhea, nasal obstruction, and malaise. Temperature is usually normal, particularly when
the pathogen is a rhinovirus or coronavirus. Nasal secretions are watery and profuse during the first days
but then become more mucoid and purulent. Mucopurulent secretions do not indicate a bacterial
superinfection. Cough is usually mild but often lasts into the 2nd week. Most symptoms due to
uncomplicated colds resolve within 10 days.
Allergic rhinitis: Symptoms and Signs
Patients have itching (in the nose, eyes, or mouth), sneezing, rhinorrhea, and nasal and sinus obstruction.
Sinus obstruction may cause frontal headaches; sinusitis is a frequent complication. Coughing and wheezing
may also occur, especially if asthma is also present.
The most prominent feature of perennial rhinitis is chronic nasal obstruction, which, in children, can lead to
chronic otitis media; symptoms vary in severity throughout the year. Itching is less prominent than in
seasonal rhinitis. Chronic sinusitis and nasal polyps may develop.
Signs include edematous, bluish-red nasal turbinates, and, in some cases of seasonal allergic rhinitis,
conjunctival injection and eyelid edema. Purulent sputum or significant lower respiratory tract symptoms
are unusual with rhinovirus infection. Purulent sinusitis and otitis media may result from the viral infection
itself or from secondary bacterial infection.
186. Psychosis,1st episode
mantaince dose? 1 year

187. Valproic acid cap caused


GIT side effects, what to do?
a) Change to soln...
b) Reduce dose by 25%...
c) Give Na divalproex (less
GIT)

188. Drug causes infertility


which reference to look at
a) Micromedex,
b) Briggs,
c) CPS"

189. Dr called you asking not to provide his patients with a written info about medication and
make your counselling verbal only as they non compliance not taking the medication when they
read the side effects what to do
a) Refuse the Dr request as it's their right to know about their medication
b) Phone patients, tell them that you will do counseling only verbally upon your Dr request

190. Insulin 70/30 dispensed instead of NPH 100 u for type II D.M. "taken at breakfast and
Supper, what he may experience
a) Noctureal hypoglycemia
b) Postprandial hypoglycemia

191. How to prevent dispensing error-


a) Double checking
b) Asking pt to identifying medications b4 giving it out

192. A pt. suffered from stroke and has aphasia, which is the best method to counsel this pt.
a) Speak slowly and ask simple yes and no questions.
b) Use sign and symbols
What is aphasia? It’s a language disorder that affects your ability to communicate. It’s most often caused by
strokes in the left side of the brain that control speech and language. People with aphasia may struggle with
communicating in daily activities at home, socially or at work. They may also feel isolated.
Aphasia doesn’t affect intelligence. Stroke survivors remain mentally alert, even though their speech may be
jumbled, fragmented or hard to understand.
193. Methadone regular question (past question)
Opioid Follows legal requirements for “straight narcotics”
Agonists Prescribers no longer require special exemption to prescribe
Q. Acute withdrawal symptoms in in-patients: 5–20 mg Q2–4H PO until stable (usually 20–40
methadone mg/day). If not to be continued as maintenance, taper by 5 mg/day over 1–2 wk
Methadose, Opioid Use Disorder: Initiate as per provincial guidelines, 5– 30 mg starting dose depending on
Metadol-D, risk stratification; may increase by 5–15 mg Q3 days until dose of 60–80 mg reached and then
may increase by 5– 10 mg Q7 days.
Maintenance: 40–80 mg/day PO; higher maintenance doses (60–100 mg) has better outcomes.
Missed 1-2 days: Give usual dose
Missed 3 days: Patient must be assessed by MD; do not provide dose until seen by MD
Missed 4 or more days: Major loss of tolerance. Do not provide dose until seen by MD, may be
restarted on low initial doses
Emesis (vomiting): Vomited doses are not replaced unless observed by pharmacy staff
➢ ½ the dose replaced if vomited in 15 minutes or less post ingestion
➢ Dose not replaced if vomited after 15 minutes post ingestion
Advantages: Potent opioid agonist; does not precipitate withdrawal; potentially superior
treatment retention.
Disadvantages: Greater risk of overdose during initiation; longer time to reach maintenance
dose (>35 days); less tolerable adverse effects; more drug interactions; greater risk of
abuse/misuse compared with buprenorphine/naloxone and thus more likely to require daily
witnessed ingestion.
Methadone is administered as a liquid mixed with a flavoured juice powder to minimize
tampering. It can also be administered using a two-cup method whereby the first cup contains
the methadone and the second cup is just juice. The second cup is taken to ensure that the
patient swallows the drug.
Adverse effects of Methadose are similar to those of other opioid analgesics, and represent an
extension of pharmacological effects of the drug class. The major hazards of opioids include
respiratory and central nervous system depression and to a lesser degree, circulatory depression,
respiratory arrest, shock and cardiac arrest. Sedation, dizziness, hypotension, diaphoresis,
weight gain, erectile dysfunction, nausea, vomiting, diarrhea, QTc prolongation.
Contraindicated with QT prolonging drugs as citalopram, domperidone, fluoxetine, amiodarone
Co-administration with a serotonergic agent, such as a SSRIs or a SNRIs, may increase the risk
of serotonin syndrome.
CNS depressants may increase risk of sedation and respiratory depression.
CYP3A4 inducers & inhibitors may decrease/increase methadone serum levels, respectively.
Main use is maintenance therapy; infrequently used for detoxification, which should be done
only in a supervised in-patient medical setting. High relapse rate without maintenance therapy.
Follow maintenance guidelines from the province or territory’s regulatory body.
The co-ingestion of alcohol with Methadose should be avoided as it may result in dangerous additive
effects, causing serious injury or death.
194. Pt on 3 ART missed a dose and call u what would u advise.
a) Double dose
b) Skip dose
ART may be Androgen replacement therapy

195. Calculation on vancomycin (repetition from 2013 may)

196. Question on pt. with COPD and some complex info- ans was FQ

197. High energy drink in the pharmacy u noticed its being abuse what would u do.
a) Remove to restricted area.
b) Insert alert information
c) Stop ordering the drink

198. Pt on a number of drug and qt interval was prolong which drug is that- sotalol
199. 4-month-old child wt 7.5 kg takes 40mls of amox, what to do.
a) Send to ER
b) Wait 48 hrs and observe for diarrhea

200. Family of 18yrs boy, 13yr girl, grand mother and mother coming to ask for their rx info.
a) 13 yrs can make decision for herself.
b) Mother can make decision for 13 yr girl

201. In which of the following situation can a pharmacist disclose confidential info
-pt is trying to harm self

202. Which vaccines do u not give to a 65 yrs. old man? MMR, unless HCP or HPV
https://www.cdc.gov/vaccines/schedules/downloads/adult/adult-combined-schedule.pdf

203. What is the role of provincial regulating authority

204. Which is not a role of health Canada- packaging and labelling


205. Pt with oozing leg ulcer what organism causing it
a) Atypical organism
b) Anerobes
c) Grame –ve
Q. Most Likely Pathogen involved in Diabetic Foot Infection
➢ Staph. aureus (MSSA or MRSA) causes most infections, beta-hemolytic streptococci (group A or B
most common)
➢ Strep. pyogenes (group A streptococcus) causes Erysipelas, P. aeruginosa causes Macerated foot.
➢ Gram-positive bacteria including enterococci, Gram-negative bacteria and anaerobic bacteria

206. A drug was voluntary recall by manufacture what would u do


a) Post notices in pharmacy
b) Note batch size of the drug
c) Note lot numbers

207. Pregnant woman wants drug info where would u contact- motherisk

208. Pt on buspiron and lorazepam, how would u advise


a) Avoid alcohol
b) Takes 3-5 weeks to take
effect
c) No cross activity
between them
209. Dr rx mefloquine to a pt going to a rural south Africa country. Pt has a number of
conditions. Which of the following is a contraindication? Anxiety
Contraindicated in patients with a history of seizures, depression, generalized anxiety disorder, psychosis,
schizophrenia or other psychiatric disorders, self-endangering behaviour, suicide attempts or suicidal
ideations.

210. What would u advise the pt? use mosquito net

211. Which of the follow vaccine would the person not need? Hep. B (not fecal oral
transmitted)

212. A table was given with figures and u are to calculate turn-over rate? How would u improve
sales or turn over from the figure given

213. Cal on NNT, Cal on NNH, Cal on OR

214. What is true of epipen? can be given through clothes


How to Store It:
Keep your EpiPen or EpiPen Jr at room temperature. Do not refrigerate. Do not drop.
Do not expose your EpiPen or EpiPen Jr to direct sunlight.
Do not keep your EpiPen or EpiPen Jr in a vehicle during extremely hot or cold weather.
Always keep your EpiPen or EpiPen Jr in the carrier tube to protect it from damage; however, the carrier
tube is not waterproof.
The blue safety release cap helps to prevent accidental injection. Keep the blue safety release cap on until
you need to use EpiPen or EpiPen Jr.
Occasionally inspect your EpiPen or EpiPen Jr solution through the viewing window. Replace
your EpiPen or EpiPen Jr if it is discolored or contains solid particles (precipitate) or if there are any signs of
leakage. The solution should be clear.
Discard if there are any signs of damage to the carrier or the EpiPen or EpiPen Jr.
Do not attempt to take the EpiPen or EpiPen Jr apart.
Replace your EpiPen or EpiPen Jr before the expiration date or after you use it.
Talk to your pharmacist about how to properly dispose of your expired EpiPen or EpiPen Jr.
Do not place this Consumer Information or any other objects in the carrier tube with your EpiPen
or EpiPen Jr, as this may prevent you from removing your EpiPen or EpiPen Jr quickly for use.
Keep out of the reach and sight of children.
Injection site:
You should ONLY inject EpiPen or EpiPen Jr into the outer side of your upper thigh—into the muscle.
Do not inject it into the: vein (intravenously (IV)), buttocks, hands, fingers, feet and toes
If you do, it can either cause dangerously high blood pressure or you or your child may not get the effect of
the emergency treatment that you or they need.
If you accidentally inject it into any of these areas, go right away to the nearest hospital (emergency room)
for further treatment.
If you inject a young child with EpiPen or EpiPen Jr hold their leg firmly in place and limit movement before
and during the injection to prevent injuries. Ask your healthcare provider to show you how to properly hold
the leg of a young child during injection.
If you have a thick layer of fat under your skin the epinephrine in EpiPen or EpiPen Jr may not reach your
muscle tissue. In some cases, this might make EpiPen and EpiPen Jr not work as well.

215. Pt return expired epipen what to do? accept the financial cost

216. Metoprolol – side effects


Orthostatic hypotension, worsening HF/fluid retention, bronchospasm (less with beta1-selective), dyspnea,
bradycardia, malaise, fatigue, asthenia, erectile dysfunction, may mask hypoglycemia.

217. Allupurinol side effects? Rash (SJS)

218. What is the first thing to do before clinical trial? Check for gap

219. Pt taking amiodarone, what would u not monitor? CrCl (renal failure not SE of
amiodarone)

220. Pt with intermittent diarrhea for the past 10 days what would u recommend
a) Refer (intermittent diarrhea is risk for IBD)
b) ORS

221. Mixing of insulin? withdraw regular first

222. Which is the cause of shortage of drug supply


a) Single supplier
b) Patient’s demend
c) Raw material for production/ Remember: quality assurance issue

223. A baby develop constipation after stopped breasting feeding what would u advise
a) Refer (if less than 2 years- red flag)
b) Glycerin supp

224. A case was given, in recording this in SOAP format what would be recorded in the A
section?
a) CYP3A4 is inhibited resulting in higher concentration of the drug
225. Case of dispensing error who to contact first?
a) Doctor
b) Patient
c) Nurse

226. Health care is covered by


a) Federal govt
b) Provincial
c) Insurance

227. Pt on erythropoietin what would u monitor


a) BP
b) Hb

228. Pt has crohn disease, no response to current medication what to do? give infliximab

An antimetabolite (azathioprine, 6-mercaptopurine, or methotrexate), an anti-tumor necrosis factor (TNF)


agent (infliximab, adalimumab, or certolizumab pegol), or a combination of both, can be used as 2nd-line
therapy after corticosteroids, and even as first-line therapy in preference to corticosteroids. These drugs,
guided by measurements of drug and antibody levels, achieve clinical success in most cases. When these
lines of treatment fail in patients for whom surgery is not feasible or appropriate, newer biologic
drugs including anti-integrins (eg, vedolizumab) or an anti-IL-12/23 antibody (eg, ustekinumab) can be used.
Furthermore, other biologic agents are emerging rapidly.
229. How would u maintain remission in this pt
a) Azathioprine
b) Cyclosporine
c) Mesalamine

230. Pt has rheumatoid arthritis in 3 or


more joints was on sulfasalazine but
there was no improvement why?
a) Advanced disease stage

231. Terbinafine counselling (athlete foot case- continue course 4 weeks)


a) Avoid alcohol
b) Take with fat diet

Terbinafine oral Therapeutic indications


1. Treatment of Terbinafine tablets sensitive fungal infections such as Tinea corporis, Tinea cruris and Tinea
pedis (caused by Dematophytes see Section 5.1) is considered appropriate due to the site, severity or extent
of the infection.
2. The treatment of onychomycosis (Terbinafine tablets-sensitive fungal infection of the nails) caused by
dermatophytes.
N.B. Orally administered Terbinafine tablets are not effective against Pityriasis versicolor.
The official local guidelines should be borne in mind, for example, national recommendations relating to the
correct use and prescription of antimicrobial drugs”.
Method of administration
Adults: 250mg once daily.
The duration of treatment varies according to the indication and the severity of the infection.
Skin infections
Likely durations of treatment are as follows:
Tinea pedis (interdigital, plantar/moccasin type): 2 to 6 weeks
Tinea corporis: 4 weeks
Tinea cruris: 2 to 4 weeks
Onychomycosis
The duration of treatment for most patients is between 6 weeks and 3 months. Treatment periods of less
than 3 months can be anticipated in patients with fingernail infection, toenail infection other than of the big
toe, or patients of younger age. In the treatment of toenail infections, 3 months is usually sufficient
although a few patients may require treatment of 6 months or longer. Poor nail outgrowth during the first
weeks of treatment may enable identification of those patients in whom longer therapy is required.
Complete resolution of the signs and symptoms of infection may not occur until several weeks after
mycological cure.
Additional information on special population
Liver impairment: Terbinafine tablets are contraindicated for patients with chronic or active hepatic disease
Renal impairment: The use of Terbinafine tablets has not been adequately studied in patients with renal
impairment and is therefore not recommended in this population.
Children
A review of safety experience with oral terbinafine in children, which includes 314 patients involved in the
UK Post Marketing Surveillance study, has shown that the adverse event profile in children is similar to that
seen in adults. No evidence of any new, unusual or more severe reactions to those seen in the adult
population have been noted. However, as data is still limited its use is not recommended.
Elderly
There is no evidence to suggest that elderly patients (aged 65 years or above) require different dosages or
experience side-effects different to those of younger patients. The possibility of impairment of liver or kidney
function should be considered in this age group (see Precautions).
Method of administration
The scored tablets are taken orally with water. They should preferably be taken at the same time each day
and can be taken on an empty stomach or after a meal.
232. Pt taking prednisolone for a long-time what side effects would you expects?
a) Mood change

234. Question on tacrolimus


Calcineurin inhibitors
Pimecrolimus They have anti-inflammatory activity & tacrolimus has also demonstrated antifungal properties.
Elidel $70 These agents have efficacy comparable to standard antifungal and topical corticosteroid
treatments in decreasing severity of erythema, scaling and pruritus.
Tacrolimus Use of topical calcineurin inhibitors on the eyelids has an established long-term safety profile.
Protopic Tacrolimus ointment is more effective and better tolerated than pimecrolimus cream.
Second-line Body: Rub a thin layer into affected area BID, Maintenance dose: Once daily, 2 days/wk.
Requires 2-4 wk to see effect. Maintain use at interval necessary to keep condition under control
Topical tacrolimus used with zinc pyrithione may be alternative to topical corticosteroids in
treating scalp seborrheic dermatitis.
Mild, transient skin burning at onset of therapy. Lacks longterm side effects of corticosteroids.

235. 120 kg woman needs contraceptive what would give? Nuva ring

236. Methotrexate was taking daily instead of weekly what side effects, would u expects?
Myelosuppression
237. Same pt taking folic acid to prevent?
a) Fetal neural tube defects or in megaloblastic anemia
b) HTN
c) MI
d) PVD
e) CV disease
Therapeutic indications
➢ In treatment of folate-deficient megaloblastic anaemia.
➢ For prophylaxis in chronic haemolytic states or in renal dialysis.
Symptoms and Signs
➢ Folate deficiency may cause glossitis, diarrhea, depression, and confusion. Anemia may develop
insidiously and, because of compensatory mechanisms, be more severe than symptoms suggest.
➢ Folate deficiency during pregnancy increases the risk of fetal neural tube defects and perhaps other
brain defects.

238. What’s true of horizontal luminar flow? filters air particle

239. What to evaluate in the process? Evaluate particle produce


240. You have subs. A conc. is 4% to be added to subs B to have a final volume of 50 ml. After
mixing both subs. The conc. of subs. A in the final solution was 0.1 %, Calculate the amount of subs.
B in Mg in each dose if you divided the total solution to 5 small doses. Conc. of subs B is 10% w/v
Answer:
C1V1=C2V2 4%*V1 = 0.1%*50 V1 = 1.25 ml which is the final volume of subs. A
Volume of subs. B = 50-1.25 = 48.75 ml
10mg ----- 100ml Xmg ----- 48.75ml X=4.875 mg 4.875mg/5 = 0.975 mg in each dose.

241. Female with vaginal candidiasis, in which of the following condition you may refer her to doctor
a) Renal problem
b) Heart problem
c) Diabetes
d) Lung problem

242. What is correct about Zopiclone:


a) Prescription can be verbally or written without refills.
b) Prescription should be written only.
c) Prescription should be verbally with refills.
d) Prescription can be written or verbally with refills

243. Several errors were reported from ward area regarding concentrated KCL solution, what would
be the best action to prevent such errors in the future:
a) Put concentrated KCL solution in a locked area.
b) Buy a premixed KCL solution bag.
c) Put a label stating (concentrated Solution) on KCL bags.
d) Remove from ward

244. To determine the safety of taking insulin in hospitals and avoid errors, what do you recommend?
a) Use mL unit in insulin syringe
b) Use the patient home insulin
c) Put insulin beside patient bed to decrease the time to take insulin
d) Use appropriate syringe unit that is clear enough to the patient and nurse

245. A mother came to you complaining of eye pain because her child put his finger into her eye, she
also has Irritation, Redness and inflammation. What would you recommend?
a) Cold compresses
b) Warm compresses
c) Refer to the DR.
d) Just wash with water and everything will be ok
246. A drug was given IV at dose of 500mg, oral bioavailability is 85%, what should be the oral dose
a) 425
b) 575
c) 615
d) 588

247. Benefit of auto-substitution in hospitals


a) Prevent new drugs from entering the formulary
b) Decrease the inventory on hand (inventory advantage)

248. Drug A give odd ratio 5 comparing drug to drug B it means or The odd ratio of drug A was (5)
in trial with drug B is that mean:
a) A more effective than B
b) A is more significant
c) B is significant than A
d) B is slightly significant than A
e) A is slightly significant than B
If the odds ratio is greater than 1, the risk of exposure is greater in cases than controls.
If the odds ratio is less than 1, the risk of exposure is smaller in cases than controls

249. Canadian pharmacist wants to send error (did not put ISMP)
a) CIHI
b) NSIR
c) CMIRPS

250. 56 years old female diagnosed with Osteoporosis. She was taking Vitamin D 400 IU and
Calcium supplement 500 mg per day. Her dairy consumption is equivalent to 700 mg per day.
What is your concern regarding Vitamin D and Calcium?
a) Too low dose and adequate dose
b) Too low dose and too high dose
c) Adequate dose and adequate dose
d) Two high dose and too low dose
Vitamin D 800-2000 IU /day
251. MD. Supplied a woman with Estrogel and told her to apply 2.5mg daily from day 1 to 25
what is the DTP
a) Too low dose
b) Wrong medication
c) No DTP
The right one should be 2.5g of gel about 1-2 actuation which contains 1.5-3 mg estrogen
Recommended Dose and Dosage Adjustment
Treatment is usually initiated with 2.5 g ESTROGEL, daily. ESTROGEL is usually administered on a cyclic schedule
from day 1 to day 25 of each calendar month or from day 1 to day 21 of a 28-day cycle.
The dose of ESTROGEL should be adjusted as necessary to control symptoms. Attempts to adjust the necessary
dosage should be made after two months of treatment. Breast discomfort and/or breakthrough bleeding are
generally signing that the dose is too high and needs to be lowered. However, if the selected dose fails to
eliminate the signs and symptoms of estrogen deficiency, a higher dose may be prescribed. For maintenance
therapy, the lowest effective dose should be used.
Missed Dose
If a dose of ESTROGEL has been missed, the missed dose should be taken as soon as possible. However, if it is
almost time for the next dose, the missed dose should be skipped and the regular dosing schedule should be
continued. The dose of ESTROGEL should not be doubled.
Administration
ESTROGEL Metered-Dose Pump
Two metered-actuations will deliver 2.5 g of gel (1.5 mg E2). All of the gel should be applied with the hands over a
large area of skin (>2000 cm2) in a thin, uniform layer.
To measure a 2.5 g dose of ESTROGEL (1.5 mg E2), press firmly on the pump once and apply the gel to one arm.
Repeat applying the gel to the opposite arm. It is recommended to apply ESTROGEL to both arms. Alternate sites
of application are the abdomen or the inner thighs. It is not necessary to rotate the site of
administration. ESTROGEL must not be applied to the breasts. ESTROGEL must not be applied to the face or to
irritated or damaged skin. Allow the gel to dry approximately 2 minutes before covering with clothing. ESTROGEL
does not stain or smell.
When a new metered-dose pump is opened, it may be necessary to prime the pump by pressing the pump once or
twice. The first metered-actuation may not be accurate and should therefore be discarded. The pump contains
enough gel for approximately a month's use (i.e. 64 metered-actuations). After that, the amount of gel delivered
may be lower and thus, it is recommended to change the pump.
ESTROGEL should be prescribed with an appropriate dosage of a progestin for women with intact uteri in order to
prevent endometrial hyperplasia/carcinoma. Progestin therapy is not required as part of hormone replacement
therapy in women who have had a previous hysterectomy.

252. After correction of the DTP. counsel her for all except? Apply on chest

253. A female patient weight is 138 bound and 57 years old; her serum creatinine level is 150. Her
CrCl will be:
a) 30-39
b) 40-49
c) 50-59
d) 60-69

254. Patient is getting IV Morphine (conc. 0.2mg/ml/hr) by a pump. Now the Dr. wants to increase
the conc. to (0.5/ml/hr). If you know that, the syringe used in the pump is 60 ml and the Morphine
stock conc. is 10 mg/ml. Calculate the amount of stock solution to be added to the current conc. to get
the desired conc.
Answer:
0.5-0.2 = 0.3 0.3*60 = 18 mg
10mg ------ 1 ml 18mg ----- x ml? X ml = 1.8 ml
255. 45 years old teacher with parkinsonism, she was embarrassed as she falls many times and now,
she is working part time instead of full time: What is your advice for this patient
a) Start treatment to avoid delaying of disease progression
b) Do not start treatment because of the side effects of the medication
c) Start the treatment as quality of life is affected
d) Change her work

256. What is most appropriate treatment


a) Pramipexole
b) Trihexyphenidyl
c) Selegiline

257. Which is fetal if given intrathecally: Vincristine


Vinca alkaloids (Vincristine, Vinblastine) are fatal if taken intrathecally
Vincristine causes neurotoxicity. Methotrexate also

258. What is true concerning Tallman lettering? HYDROmorphone morphine

259. Pharmacist concerned about not receiving the appropriate bonus after the annual review, in his
manager's opinion he was not performing well, while in his opinion he was doing five clinical checks
per week, which has increased to eight, which is a 60% improvement, what is an appropriate criticism
for that?
a) Pharmacist did not improve significantly over last year
b) Pharmacist was unfairly assessed
c) Objective was not measurable
d) There were no criteria set for measuring the objective

260. What is the rational for all pharmacies reporting medication errors into one authority?
a) Doing root cause Analysis for each individual error
Doing a trend for repeated errors = learning resource

261. Angina symptoms


a) Exercise tolerance
b) ECT
c) NYHA score
262. Diabetic patient travelled before and now and he comes again asking about best recommendation
against hepatitis A, what is the best recommendation
a) Wash your hands
b) Go to travel clinic

263. When to refer Herpes zoster patient? First signs of pain & burning
Refer when you see signs of a skin infection (redness, swelling, a feeling of heat in a specific area) Or the person
develops a severe cold or cough. Some people, especially adults, may get pneumonia

264. Sinusitis for 4 weeks and tried antihistaminic what to do?


a) Change
b) Refer
Refer to (ENT) specialist if a patient: Fails second-line therapy of Antibiotics, Experiences 4 or more episodes of
bacterial sinusitis per year, has chronic sinusitis that is not responding to medical therapy (symptomatic and
antibiotic), Has anatomic anomalies, Develops complications

265. What are the regulations for dispensing targeted substance, all are true except?
a) Transfer more than once

266. Patient on Warfarin and we started Statin what to do for the dose of warfarin? Monitor INR
As statin increase INR that may lead to decrease warfarin dose
MAY 2013
1. A pregnant woman at 36-week UTI what to take
a) Amoxicillin
b) Cloxacillin
c) Nitrofurantoin
d) Macrofurantoin
e) Azitromycin

2. UTI + pregnant + penicillin and ASA allergy, what


to give:
a) Nitrofurantoin
b) Cephalexin,
c) SMX/TMP,
d) Tetracycline
Cephalosporin cause cross sensitivity with penicillin
Nitrofurantoin taken with a lot of water because is very nephrotoxic, contraindicated with CrCl<60ml/min
and Pyelonephritis. Women >65 treat for 3 days in UTI
Contraindications
➢ Patients who are hypersensitive to nitrofurantoin or to any ingredient in the formulation or
component of the container.
➢ Patients with a history of anaphylactic reaction to nitrofurantoin.
➢ Patients with a history of cholestatic jaundice or hepatic dysfunction associated with prior
nitrofurantoin use.
➢ Patients with anuria, oliguria or serious renal impairment (ClCr <30 mL/minute or clinically
significant elevation of serum creatinine)
➢ Patients with glucose-6-phosphate dehydrogenase deficiency, erythrocyte enolase deficiency or
glutathione peroxidase deficiency, due to the risk of hemolytic anemia.
➢ Pregnant women at term (38–42 weeks’ gestation), during labour and delivery or when the onset of
labour is imminent, due to concerns of hemolytic anemia with immature erythrocyte enzyme systems
➢ Infants <1 month of age, due to the possibility of hemolytic anemia with immature erythrocyte
enzyme systems.
➢ In women when breastfeeding infants <1 month of age, premature or with glucose-6-phosphate
dehydrogenase (G6PD) deficiency, due to the risk of hemolytic anemia with immature erythrocyte
enzyme system.

3. Nitrofurantoin administration:
a) Take with food because of bitter after taste
b) Take on empty stomach
c) Take with food because of Diarrhea
Nitrofurantoin is administered orally and all dosage
forms should be taken with food or milk to minimize
gastric upset. Administration with food may also
enhance absorption.
Gastrointestinal side effects: Diarrhea, dyspepsia,
abdominal pain, constipation, emesis, sialadenitis
and pseudomembranous colitis (including that due
to C. difficile) have occurred.

4. What is the common cause of UTI? E. coli

5. What to advice patient has UTI about cranberry uses?


a) Effective in prevention
b) Used in males only
Evidence from clinical trials does not support
the use of cranberry products (juice, powder or
tablets) for the prevention of UTIs.
Consumption of large quantities of cranberry
juice may result in an interaction with
warfarin and raise INR. The use of cranberry
products is safe in pregnancy, but efficacy to
prevent infection in this patient population is
unknown.

6. 80 years old woman taking rosuvastatin 40 mg and developed Cystitis with symptoms.
Doctor gave her Nitrofurantoin 100 mg bid for 7 days. What could be a problem
a) Too high dose of Nitrofurantoin
b) Too low dose of nitrofurantoin
c) Change medication order
d) Stop rosuvastatin till the end of antibiotic treatment
Nitrofurantoin macrocrystals 50–100 mg Q6H PO
Nitrofurantoin monohydrate / macrocrystals 100 mg Q12H PO
Serious Warnings and Precautions
➢ Hemolytic anemia has occurred rarely during nitrofurantoin therapy. Discontinue nitrofurantoin if any signs
of hemolysis are present. Hemolysis will stop when the drug is withdrawn.
➢ Acute, subacute and potentially irreversible chronic pulmonary reactions have occurred rarely in patients
taking nitrofurantoin. Monitor patients, withdraw the drug immediately and take appropriate measures if
such a reaction occurs.
➢ Hepatic reactions such as hepatitis have rarely occurred and fatalities have been reported, especially with
prolonged use (>6 months). Chronic active hepatitis may occur insidiously. Monitor patients for changes in
liver function. Withdraw nitrofurantoin immediately if hepatitis occurs.
➢ Peripheral neuropathy is rare but may be severe or irreversible. Withdraw nitrofurantoin immediately if
numbness or tingling occurs.
➢ Severe hepatotoxicity, irreversible chronic lung toxicity and increased antibiotic resistance are associated
with long-term use. Suppressive therapy should be used only if the potential benefits exceeds the risks. It is
not recommended in the elderly.

7. The woman has cured from acute cystitis and now she has asymptomatic bacteriuria, should treat
her? No. Note that she is 80 years old so she cannot be pregnant, so no treatment

8. Patient take cortisone for long time, what do u tell him, except
a) Try to reduce salt intake
b) This drug may cause HTN
c) It has glucocorticoid like activity
Try to reduce salt intake, not tell pt that it may cause HTN

9. Patient mixed between Diamicron 30 mg 4 tabs am and valsartan 80 mg 1 tab so what happen
a) Increase pulse rate → Hypotension & Hyperglycemia
b) Decrease pulse rate
c) Decrease BG
d) Increase systolic
e) Increase diastolic

10. How to avoid error? Show patient.


Show the pt 3shan el3'alat 3ando howa, law el 3'alat mn elpharmacist yb2a separate or double check

11. 15-month child diarrhea and vomiting for past 36 hours


a) Wait 2-3 days
b) Refer to doctor (red flag: below 2 years)
c) Dispense over counter medication
12. Preservative should not be included with.
a) Large volume parenteral
b) Ear drops
c) Eye drop
d) Nasal spray
e) IM

13. What is NOT related to PIPEDA (Personal Information Protection & Electronic Documents Act)
rules
a) Don’t disclose personal information to anyone
b) To disclose personal information to third party
c) Use the personal information to be used in the pharmacy
d) To disclose personal information to Regulatory Authority

14. All are true about Common drug review CDR, except?
a) Used to help the manufacturers better market their products for better sales
b) Recommends to the provincial drug plans on which drugs should be included in the formulary
c) To develop policies for adaptation of drugs in formulary by provincial authorities
d) Regulated under CADTH Canadian Agency for Drugs and Technologies in Health
15. Patient is travelling to Madagascar & come from vacation, Pharmacist concern for all except:
a) Hepatitis A
b) Hepatitis B
c) Amebiasis
d) Giardiasis
“MMR, Difteria, Tetanus, Pertusis, Varicella, Polio, Yearly flu shots, Hep A, Malaria, Typhoid, Rapes, Yellow fever”

16. What other things he needs to


know to exclude infections other than
hepatitis?
a) Marijuana
b) Sexual activity

17. Hepatitis A prevention:


a) Immunoglobulin
b) Lamivudine,
c) Ribavirin for B, C

18. A woman with atopic dermatitis gave birth. 4 weeks ago, breast-feeding, and no allergic
or medical condition for the baby and she is receiving aid from the community and wants a
reliable method of contraception:
a) COC (no estrogen with lactation)
b) Intrauterine device
c) Ring with combined estrogen and program
d) Medroxy progesterone acetate injection (contraceptive of choice for lactating women, but from
week 6)
e) Diaphragm with spermicide
Although expulsion and perforation rates are higher for LNG-IUS when inserted immediately after delivery
compared to 6 weeks postpartum, immediate insertion after delivery, either vaginally or via cesarean section may
be considered.[85] An IUD can also be inserted immediately after a first- or second-trimester abortion. Immediate
insertion appears to be safe and effective and provides convenience. As well, it provides assurance of
contraception postpartum, ensures the patient is not currently pregnant, and decreases the risk of repeat
unintended pregnancies and abortions. Ensure good fundal placement; this can be difficult in the immediate
postpartum state with the larger uterine cavity.
Most women can use an IUD. Contraindications include the following:
• Current pelvic infection, usually pelvic inflammatory disease (PID), mucopurulent cervicitis with a suspected
STD, pelvic tuberculosis, septic abortion, or puerperal endometritis or sepsis within the past 3 months
• Anatomic abnormalities that distort the uterine cavity
• Unexplained vaginal bleeding
• Gestational trophoblastic disease with persistently elevated serum beta–human chorionic gonadotropin
(beta-hCG) levels (a relative contraindication because supporting data are lacking)
• Known cervical cancer or endometrial cancer
• Pregnancy
• For levonorgestrel-releasing IUDs, breast cancer or allergy to levonorgestrel
• For copper-bearing T380 IUDs, Wilson disease or allergy to copper
Conditions that do not contraindicate IUDs include the following:
• Religious beliefs that prohibit abortion because IUDs are not abortifacients (however, a copper IUD used for
emergency contraception may prevent implantation of the blastocyst)
• A history of PID, STDs, or ectopic pregnancy
• Contraindications to contraceptives that contain estrogen (eg, history of venous thromboembolism, smoking
> 15 cigarettes/day in women > 35, migraine with aura, migraine of any type in women > 35)
• Breastfeeding
• Adolescence

19. What can we do to prevent from giving expired medication to a patient?


a) Check expiry dates every 6 months (if every month, it would be better)
b) Check the expiry dates when you are giving the medication to the patient
The key is to set a policy whether this policy is to review inventory expiration dates on a prescheduled/regular
basis or to set a policy to write down the drug expiration and batch number of the stock bottle on the transaction
copy as recommended by the standards.
Any of these measures are appropriate but leaving things to chance and checking right before dispensing to the
patient is by far less appropriate.
Overall, as you can see make your best to understand the Q stem and the answer before making a decision,
especially that here you had the right idea that's supported by good practice and experience, yet you picked a less
appropriate choice.

20. Child OM recurrent had taken amoxicillin 2 months before so give him all except:
a) Amox /clav
b) Cefuroxime
c) Cloxacillin
d) Ceftriaxone
e) Azithromycin

21. Which case you cannot take Bismuth subsalicylate with patient have GERD:
a) Diabetes
b) Age
c) Renal failure
d) Hepatic failure
Also, second half of pregnancy, GIT ulcer due to aspirin, pediatric pt with infection or chickenpox due to Reye's
syndrome
22. Methadone 8.7-gram, Na benzoate 8 gram found in 870 mlqs, if you have this formula in the
pharmacy to prepare the methadone and you received, the following Rx from the MD: Prepare
Methadone 9 gram. Mitte 900 ml. what quantities you will mix to get the Rx filled?
Answer:
Methadone = 8.1/810 * 900 = 9 gm Na Benzoate = 1.15/810 * 900 = 1.27 gm

23. Federal funding utilized for the reimbursement of drug purchases of (k type)
a) First nations
b) Veterans
c) Financial contingent
Also, refugee, Inmates, RCMP

24. How to treat tumor induced hypercalcemia: Pamidronate


For acute management, parenteral Pamidronate, clodronate or zoledronic acid may be used.
Cinacalcet is given for chronic kidney disease hypercalcemia
Bisphosphonates inhibit osteoclasts. They are usually the drugs of choice for cancer-associated hypercalcemia.
Zoledronate can be given as a one-time dose of 4 to 8 mg IV and lowers serum calcium very effectively for an
average of > 40 days.
Pamidronate can be given for cancer-associated hypercalcemia as a one-time dose of 30 to 90 mg IV, repeated only
after 7 days. It lowers serum calcium for ≤ 2 weeks.
Ibandronate as a one-time dose of 4 to 6 mg IV can be given for cancer-associated hypercalcemia; it is effective for
about 14 days.
Etidronate 7.5 mg/kg IV once a day for 3 to 5 days is used to treat Paget disease and cancer-associated
hypercalcemia. Maintenance dosage is 20 mg/kg orally once a day, but the dose must be reduced when glomerular
filtration rate is low.
Repetitive use of IV bisphosphonates to treat hypercalcemia associated with metastatic bone disease or myeloma
has been associated with osteonecrosis of the jaw. Some reports suggest this finding may be more common with
zoledronate. Renal toxicity has been reported in patients receiving zoledronate. Oral bisphosphonates
(eg, alendronate or risedronate) can be given to maintain calcium in the normal range but are not generally used
for treating hypercalcemia acutely.
Denosumab, 120 mg SC every 4 weeks with additional doses on days 8 and 15 of the first month of treatment, is a
monoclonal antibody inhibitor of osteoclastic activity that can be used for cancer-associated hypercalcemia that
does not respond to bisphosphonates. Calcium and vit D given as needed to avert hypocalcemia.
25. How to monitor hypercalcemia
a) TSH
b) Albumin
Hypercalcemia Monitoring: Ca levels, serum Ca & albumin, Serum creatinine, Serum intact parathyroid hormone
(PTH)

26. Patient has depression and anxiety will travel to African country; his doctor
prescribed for him Mefloquine to protect against Malaria. What is the DTP
a) Medication for no indication
b) Inappropriate medication
c) Resistant
Anxiety, nightmares, depression and irritability are more common side effects of Mefloquine

27. Travelling to area of malaria so doctor prescribe Mefloquine


a) Take 1 day before travel
b) Take on day of travel
c) 1–2 weeks before travelling & continued until 4 weeks after leaving a malarious area.

28. What would you recommend?


a) Change Mefloquine to Chloroquine
b) Refer to doctor to choose the suitable med. for his case.

29. What to council him:


a) Avoid walking 10-2 pm
b) Sleep in bed net
c) Put sunscreen after DEET

30. Crohn treatment all stage


Status CDAI Score Description from ACG Guidelines
Remission < 150 Asymptomatic or without any symptomatic inflammatory sequelae
Mild to 150 to 220 Ambulatory and able to tolerate oral alimentation without manifestations of
moderate dehydration, systemic toxicity, abdominal tenderness, painful mass, intestinal
obstruction, or > 10% weight loss
Moderate to 220 to 450 Failed to respond to treatment for mild to moderate disease, or those with
severe more prominent symptoms of fever, significant weight loss, abdominal pain
or tenderness, intermittent nausea or vomiting, or significant anemia
Severe > 450 Persistent symptoms despite the introduction of conventional corticosteroids
or biologic drugs as outpatients, or individuals presenting with high fevers,
persistent vomiting, evidence of intestinal obstruction, significant peritoneal
signs such as involuntary guarding or rebound tenderness, cachexia, or
evidence of an abscess
31. Patient with Comedonal acne non-inflammatory, what to give?
a) Azelaic acid
b) Erythromycin
c) Topical retinoids (adapalene, tretinoin and tazarotene)
Used for non-inflammatory and inflammatory.

32. Patient has mild Comedonal acne without inflammation, what should he take
a) Azelaic acid
b) Glycolic acid
c) Sulfur
d) Erythromycin
N.B no retinoid in the choices

32. A person with acne and is experiencing severely inflamed face and taking multivitamins
and other medications was on benzoyl peroxide but wasn't working for him, so doctor
changed him to erythromycin and retinoid, what to worry about?
a) Drug drug interaction
b) Inappropriate medication
c) Too high dose
Treatment is for mild cases where the case here is sever

33. All are goals of therapy for treatment of Acne except


a) Eradication of microorganism
b) Slow the progression of signs and symptoms
c) Limit disease duration and recurrence.
d) Avoid psychological suffering
Goals of Therapy
➢ Clear existing lesions, Prevent new lesions
➢ Minimize scarring, Reduce dyspigmentation
➢ Minimize psychological impact

34. Calculation about Accuracy? Sensitivity = Weight * Error

35. All are monitored during Amiodarone treatment except:


a) Pulmonary fibrosis
b) Liver Enzymes
c) Ocular
d) Thyroid function
e) Blood and stool occult
36. Amiodarone late SE
a) Skin
b) Eye
Early SE: Headache, Lung, Eye (6 months), Late SE: Skin (5 years), Blue man
Oral amiodarone hydrochloride induces photosensitization in about 10% of patients. Sunscreen preparations or
protective clothing may afford some protection to individual patients experiencing photosensitization. Blue-grey
discoloration of exposed skin has been reported during long-term treatment. With discontinuation of therapy, the
pigmentation regresses slowly over a period of up to several years. The risk may be increased in patients of fair
complexion or those with excessive sun exposure, and may be related to cumulative dose and duration of therapy.

37. What not to give with Amiodarone? Metoprolol


Amiodarone should be used with caution in patients receiving ß-receptor blocking agents (e.g.,
propranolol, a CYP3A4 inhibitor) because of the possible potentiation of bradycardia, sinus arrest,
and AV block. If necessary, amiodarone can continue to be used after insertion of a pacemaker in
patients with severe bradycardia or sinus arrest.

38. Rheumatoid arthritis case, about DMARD


therapy, what is the first thing to monitor before
initiating therapy?
a) Lipid
b) Glucose tolerance test
c) Liver
Initial baseline monitoring with DMARDs? Infection
Chest X ray, liver, CBC, LFT,

Another version: Alarm symptom of biological


DMARDS? Fever or alopecia
39. Then he will start Methotrexate what is first thing to monitor
a) Chest x ray
b) Serum folic acid

40. Bulimia nervosa case. What the DOC:


a) Mirtazapine
b) Fluoxetine → also, Trazdone, Venlafaxine
c) Sertraline

41. It affects
a) Esophagitis in case of vomiting
b) Pancreatitis in case of excessive laxative use

42. What is the expected period to see effect from medication? 2-3 months

43. Patient with Anorexia should be referred to all except


a) Dietitian
b) Occupational therapist
c) Psychotherapist
d) Cognitive behavioral therapy
Anorxia Nervosa ttt: Domperidone, Metoclpramide

44. Patient coming in counselling area tell pharmacist he had noticed that the front shop
employee works a part-time at his restaurant and he is afraid that he will know about his
medication. What to tell him?
a) All medication profiles are confidential and front shop employees cannot have access to it.
b) All medication profiles are confidential and only pharmacist staff can access it
Choice no 1 is assuring his concerns which is the main here in this situation
While B said pharmacist only which not right, technician also has access

45. Patient have chronic liver disease, what score used.


a) Child BUGH
b) APACHE II: Acute Physiology, chronic health evaluation, patient in ICU within 24 hours
c) Modified ranking scale: Patient suffer from Stroke
Child BUGH score for diagnosis of liver disease especially cirrhosis, liver transplantation &during surgery
to decrease mortality

46. Patient has fibromyalgia taking Amitriptyline 10mg and gabapentin, also he takes Metoprolol
50mg, no improvement. What is the reason for not improving?
a) Metoprolol (B blockers cause fatigue)
47. What is the drug which taken in suboptimal dose? Amitriptyline

48. What to give?


a) Fluoxetine
b) Duloxetine
c) Nortriptyline
d) Tylenol 3

49. What to monitor


a) Absence of work
b) Sleep 8 hrs
• Tricyclic antidepressants at bedtime (e.g., amitriptyline starting at 5 mg and progressing slowly, every
2–3 weeks, to a maximum of 50 mg) can improve sleep and reduce pain and fatigue, if taken 1–2
hours before bedtime the effect will start at bedtime, and morning hangover will be lessened.
• Cyclobenzaprine
• The SNRI duloxetine provides a small but significant reduction in fibromyalgia pain but does not
improve fatigue, quality of life or sleep disturbances. Duloxetine may be considered as first-line drug
therapy in fibromyalgia patients with concomitant depression
• SSRIs are not as helpful as tricyclics but are usually better tolerated.
• Pregabalin

50. What is the goal of therapy?


a) Restrict pain
b) Dec her absences from work
51. Which drug can be added to control his HTN? Nifedipine XL

52. Patient have Fibromyalgia; want to take Soya supplements, where to look for
a) PSC
b) Natural product database
c) Licensed natural product database
The Licensed Natural Health Products Database contains information about natural health products that have
been issued a product licence by Health Canada.
• Products with a licence have been assessed by Health Canada and found to be safe, effective and of
high quality under their recommended conditions of use. You can identify licensed natural health
products by looking for the eight-digit Natural Product Number (NPN) or Homeopathic Medicine
Number (DIN-HM) on the label.
• This Licensed Natural Health Products Database is managed by Health Canada and includes
information on licensed natural health products, including:
• vitamin and mineral supplements
• herb and plant-based remedies
• traditional medicines like Traditional Chinese Medicines or Ayurvedic (Indian) Medicines
• omega 3 and essential fatty acids
• probiotics
• homeopathic medicines
• many everyday consumer products, like certain toothpastes, antiperspirants, shampoos, facial
products and mouthwashes
• What Information Can I Find Here?
• For every licensed product listed in this database, the following details are provided:
• product name
• product licence holder
• Natural Product Number (NPN) or Homeopathic Medicine Number (DIN-HM)
• product's medicinal ingredients
• product's non-medicinal ingredients
• product's dosage form
• product's recommended use or purpose (i.e. its health claim or indication)
• risk information associated with the product's use (i.e. cautions, warnings, contra-indications and
known adverse reactions)

53. Gouty arthritis and hypertensive, who has HF and taking Atorvastatin, what do you give him?
a) Indomethacin
b) Colchicine
c) Corticosteroid
Also, Losartan, fenofibrate decrease gout
54. What to tell him about most side effect of Colchicine
a) Tinnitis
b) Abdominal cramps

55. Patient should avoid with colchicine:


a) Taking colchicine with grapefruit
b) Take within 2hrs of dairy products
c) On empty stomach
Avoid alcohol, smoking & grapefruit with use of colchicine and
Gout.
Continue colchicine for 3-6 months to prevent recurrent flares

56. Patient with Pre cataract surgery in her left eye then will do it in right eye after 8 weeks, dr
rx for her 3 eye drops one was prednisolone and other 2 were moxifloxacin, sundilac (anti-
inflammatory) tid for 7 days then: What the best action to do:
a) Dispense all drops
b) Too high a dose
c) Too long duration

57. What was the reason for prescribing these eye drops?
a) Prevent infection and control inflammation
b) To decrease IOP
c) Decrease inflammation
Goals of therapy: control inflammation, prevent infection, maintain eye comfort, promote early visual
rehabilitation

59. After she did the surgery in her eye, she still has many lefts at home. What to do
a) She has to buy new eye drops
b) Hold for her till she needs them
c) Give her prednisolone only & use other two she has home
d) Give her other two & continue on prednisolone she has home

60. Unapproved use of drug “Off label” where to look all except
a) Drug facts & comparison
b) Medline
c) USP-DI
d) Merck manual
61. Baby after 13 months take:
a) Iron fortified
b) Cow pasteurized
c) Cow milk
lron-fortified infant formulas are an acceptable alternative until 9–12 months of age.
9–12 months, pasteurized whole cow milk may be introduced

62. A lot of hypertensive patient have treatment failure; pharmacist is requested to do


modification on treatment. Who should give him permission?
Physician → Alternatively, modify dose and tell Physician to approve

63. RRR Calculation with 10 event and 15 controlled and calculate NNT

64. Patient will start with high dose of prednisone for 6 months, what should u counsel him
a) Do not stop suddenly
b) Do not take missed two doses together
c) Take with food
Missed Dose If a dose is missed, then it should be taken as soon as possible. However, if it is almost time for the
next dose, then the missed dose should be skipped and regular dosing schedule resumed. Patients should not take
a double dose to make up for a missed one. Administration PREDNISONE tables should be taken orally, with water.

65. Prednisone monitoring:


a) Sexual dysfunction
b) Alopecia
c) Photosensitivity
d) Mood changes
e) Hypertension

66. COPD patient with acute exacerbation and hypertension in the ICU. What to give?
a) IV corticosteroid
b) Salmeterol

67. Patient in need to antibiotic: QA 2014


a) Amox/clav (hypertention= cardiac disease)
b) Doxycycline
Simple Amox, Doxycylcine, SMX/TMP. Complicated (cortisone, AB within 3-month, cortisone, ISH, FEV1 <50%)
Amoxclav, Floroquinlone, 2nd cephalosporin
68. Dementia monitoring all except
a) Vit B12
b) Folic acid
c) MMSE
d) TSH
e) Electroencephalogram
Also, CBC TSH Electrolyte, Kidney function, calcium, blood glucose

69. Patient with dementia prescribed Donepezil 10 mg, what the drug therapy problem?
a) Wrong med
b) Too high a dose
Donepezil – 1st line for Dementia, starting dose 5mg – up to 10mg.
Donepezil can cause N/V so should be titrated. If patient experiences s/e reduce dose.
Memantine: added to donepezil for moderate to severe [don’t confuse with amantadine- Parkinson’s]

70. When do you expect to see improvement in dementia case? 3-6 Months
71. Goals of therapy for dementia.
a) Delay joining health care facility.
b) Reverse cognitive impairment
c) Slow mental deterioration

72. Herbal drug where herbal guidelines or database herbal: OBSELETE


a) Lawrence,
b) PSC
c) Comp. of non-prescription
d) Herbs everyday

73. Young female pregnant patient taking ramipril. She comes in with an Rx of diclectin.
What is your concern?
a) Change Ramipril to methyldopa

74. Patient want to stop smoking, he took varenicline with starting dose 150 once for 3 days
then 150 bid what is DTP
a) Dose too high
Varenicline Act on nicotinic receptors. It is partial agonist that binds selectively to alpha4, beta2, nicotinic
Champix acetylcholine receptors with a greater affinity than nicotine.
0.5 mg daily PO for 3 days then BID for 4 days then 0.5–1 mg BID PO for 12 wk. If 1 mg BID
is not tolerated, can reduce to 0.5 mg BID.
Patient should quit smoking 1–2 wk after starting varenicline. If patient is still smoking 4 wk
after starting, reassess therapy. Can be continued for additional 12 wk if patient has benefited.
No tapering necessary when discontinuing. Efficacy is dose-related.
S.E: Q. Nausea (30%); may be mitigated by taking on a full stomach, increasing water intake
or reducing dose. May cause insomnia; take second daily dose at suppertime.
Neuropsychiatric side effects such as suicidal/homicidal ideation have been reported; monitor
closely for changes in mood/behaviour.
Close monitoring by health-care provider for those with pre-existing psychiatric disorders.
Varenicline, when used in conjunction with NRT, has shown to be more advantageous in
facilitating smoking cessation and equally as safe compared to using only varenicline. It may
increase risk of adverse effects. Further studies are needed to assess long-term efficacy and
safety of the combination.
The combination of varenicline and bupropion was studied in a group of people who were
unable to reduce their smoking by at least 50% after 1 week of NRT.
Compared with varenicline alone, subjects taking varenicline plus bupropion were more likely
to be smokefree at 8–11 weeks. Combination was statistically superior in men, but not women.
Does not induce CYP enzymes; excreted renally unchanged.
Contraindications: Pregnancy, breastfeeding, and children.
Drug Interactions: Insulin, NRT, warfarin, and theophylline.
Contact doctor if constipation, abdominal pain, appetite changes.
75. What is true about varenicline
a) It is contraindicated with amlodipine
b) It is contraindicated in hypertension
Special warnings and precautions for use
Effect of smoking cessation
Physiological changes resulting from smoking cessation, with or without treatment with CHAMPIX, may alter the
pharmacokinetics or pharmacodynamics of some medicinal products, for which dosage adjustment may be
necessary (examples include theophylline, warfarin and insulin). As smoking induces CYP1A2, smoking cessation
may result in an increase of plasma levels of CYP1A2 substrates.
Neuropsychiatric symptoms
Changes in behaviour or thinking, anxiety, psychosis, mood swings, aggressive behaviour, depression, suicidal
ideation and behaviour and suicide attempts have been reported in patients attempting to quit smoking with
CHAMPIX in the post-marketing experience.
A large randomised, double-blind, active and placebo-controlled study was conducted to compare the risk of
serious neuropsychiatric events in patients with and without a history of psychiatric disorder treated for smoking
cessation with varenicline, bupropion, nicotine replacement therapy patch (NRT) or placebo. The primary safety
endpoint was a composite of neuropsychiatric adverse events that have been reported in post-marketing
experience.
The use of varenicline in patients with or without a history of psychiatric disorder was not associated with an
increased risk of serious neuropsychiatric adverse events in the composite primary endpoint compared with
placebo (see section 5.1 Pharmacodynamic properties - Study in Subjects with and without a History of Psychiatric
Disorder).
Depressed mood, rarely including suicidal ideation and suicide attempt, may be a symptom of nicotine withdrawal.
Clinicians should be aware of the possible emergence of serious neuropsychiatric symptoms in patients attempting
to quit smoking with or without treatment. If serious neuropsychiatric symptoms occur whilst on varenicline
treatment, patients should discontinue varenicline immediately and contact a healthcare professional for re-
evaluation of treatment.
History of psychiatric disorders
Smoking cessation, with or without pharmacotherapy, has been associated with exacerbation of underlying
psychiatric illness (e.g. depression).
CHAMPIX smoking cessation studies have provided data in patients with a history of psychiatric disorders.
In a smoking cessation clinical trial, neuropsychiatric adverse events were reported more frequently in patients
with a history of psychiatric disorders compared to those without a history of psychiatric disorders, regardless of
treatment.
Care should be taken with patients with a history of psychiatric illness and patients should be advised accordingly.
Seizures
In clinical trials and post-marketing experience there have been reports of seizures in patients with or without a
history of seizures, treated with CHAMPIX. CHAMPIX should be used cautiously in patients with a history of
seizures or other conditions that potentially lower the seizure threshold.
76. Asthma case worsening on salbutamol and fluticasone what to give in hospital then what at home
a) Cortisone IV, Oxygen, then after discharge give oral prednisone at home.

77. Furosemide side effects,


a) Hyperkalemia
b) Hypoglycemia
c) Hypomagnesia → Decrease Mg, Na, Cl, K
d) Hypercalcemia
OH DANG: Ototoxicity, Hypokalemia, Dehydration, Allergy (Sulfa) except Ethacrinic acid, Nephritis intestinal, Gout
Arthritis. Sequence of ototoxicity (ethacrynic acid> furosemide> bumetanide)

78. Monitor of jaundice:


a) Albumin
b) Bilirubin
If no liver injury: ALP, Gamma Glutamyl Transferase
(GGT), Bilirubin, If there’s Liver injury: AST, ALT

79. Patient with Parkinsonism, Monitor all except


a) Akasethisa, or Restless leg syndrome

80. When should a doctor start treatment of


parkinson's disease:
a) When symptoms interfere with patient's
daily activity
In Parkinson’s treat when symptoms become
bothersome

81. Levodopa and Carbidopa monitor all except,


a) Blood glucose
b) Appetite
c) Urinary incontinence
Blood glucose (minor relevance). Appetite because LD-induced nausea can affect appetite.

82. Patient cannot keep sitting on chair taking High dose of risperidone this is called:
a) Dystonia
b) Akathisia
c) Dyskinesia
d) Late dystonia
Risperidone can cause tardive dyskinesia (so causes pseudo Parkinson symptoms)
Treated with, beta-blockers” Propranolol”, anticholinergics, clonidine, or benzodiazepines
83. Concerning Rivastigmine, what is
true?
Reassess in 3 months
Rivastigmine taken with food! (Breakfast and
dinner). Rivastigmine used for patients with
Lewy or Parkinson’s (Lewy was in the park near
the River and always have to bring food

Rivastigmine Inhibits non specific butyrylcholinesterase and reversible acetylcholinesterase or centrally


oral, selective arylcarbamate AchEi. Has short half life 2 hours, but able to inhibit AchEi to 10 hours.
transdermal Because of slow dissociation of carbamate enzyme, it is referred as pseudo-irreversible AchEi.
patch Initiated at 1.5 mg BID and increased to the minimum effective dose of 3 mg BID after 30 days.
The maximum dose is 6 mg BID PO. Not metabolized by cytochrome P450 system.
Oral: Adjust dose monthly; take with breakfast and dinner.
Patch:1 Exelon-5 patch daily; if well tolerated, increase to Exelon-10 patch after at least 4 wk.
If switching from oral rivastigmine, use Exelon-5 for patients taking <3 mg BID, and Exelon-10
for patients taking 3–6 mg BID
S.E: >10%: headache, dizziness, N/V, diarrhea, abdominal pain, anorexia. <10%: insomnia,
fatigue, syncope, dyspepsia, weight loss, UTI, rhinitis. Heart block, delirium & seizures are rare.
Patients should be monitored for symptoms of active or occult GI bleeding, especially those at
increased risk for developing ulcers, e.g., history of ulcer or receiving concurrent NSAIDS.
Patients may lose weight while taking cholinesterase inhibitors, including rivastigmine.
Therefore, the patient's weight should be monitored during therapy with EXELON PATCH.
Considering the possibility of an additive extra-pyramidal effect, the concomitant use of
metoclopramide and rivastigmine is not recommended.
Additive effects leading to bradycardia (which may result in syncope) have been reported with
the combined use of various beta-blockers (including atenolol) and rivastigmine.

84. What is of the goal of therapy (Demetia)? Increase time to long-term care
Goals of Therapy
➢ Alter the natural disease progression to meet patient’s and caregiver’s goals
➢ Treat cognitive, psychological and behavioural symptoms
➢ Alleviate caregiver burden. Minimize medication side effects.

85. If Parkinson patient suffer from


Nausea and vomiting what to give:
a) Domperidone
b) Metoclopramide
Give domperidone because metoclopramide
crosses the blood brain barrier and causes
CNS effects
86. Parkinson’s disease missing many doses and when. You called him he told you that he
cannot move it is very hard. so, what to do
a) Prepare them in blister pack
b) Try to make his doses once per day
‫ﻣﻔﯿﺶ وﻻ دوا ﺑﯿﺘﺎﺧﺪ ﻣﺮة واﺣﺪة ﻓﻰ اﻟﯿﻮم‬

87. Patient with first episode psychosis, for how long to maintain treatment period with Risperdal
a) One year
2nd episode 2-5 years

88. Patient with Warfarin & levetiracetam increased recently then pharmacist give
levofloxacin instead; patient took it for a week so what to see then what to do?
a) Space on shelf
b) Put label
c) Review stock bot with label
d) Decrease space bet ph and tec (cannot be answered without a scenario)
DDI Warfarin with Levofloxacin

89. Pharmacist Initial action:


a) Document
b) Refer to ER (in case of bleeding)
c) Disciplinary action with tech
Asses, Apologize, Refer, document. If bleeding refer to ER, If not close monitoring

90. Drug shortage related to:


a) Shortage of row material
b) New QA regulations
c) Wholesalers ordering too much

91. You are a pharmacist & you informed by manufacturer there will be a shortage for a specific
medication, what to do?
a) Try to do order right now, as much as you can
b) Contact drs in nearby clinic & let them save the quantity you have to some patients
c) Assess how many patients are using this medication
92. What the percentage of success for nicotine gum in smoking
cessation?
a) 5%
b) 20%
c) 40%
d) 55%

93. Renal patient with hyperphosphatemia and normal Calcium


level, What the best treatment?
a) Calcium carbonate
b) Insulin,
c) Sodium polystyrene
If hypercalcemia is not present, Start therapy with (calcium carbonate or calcium citrate)
If hypercalcemia develops, change to lanthanum, sevelamer carbonate or sevelamer hydrochloride

94. Drug recall in special area due to


a) Production
b) Cold Chain Transportation
c) Aseptic technique
d) Adverse effect

95. What to do if the cold chain of vaccines was broken?


a) Vaccines are affected by heat more than cold
b) Varicella should be stored between 2&8C
c) Report to public health unit
Call your public health unit for advice if you suspect that vaccine has
been exposed to temperatures below +2 °C or above +8 °C.
96. BDZ withdrawal symptoms after 2-3 days:
a) Lorazepam,
b) Diazepam,
c) Alprazolam,
d) Oxazepam
Short Acting: 1-2 days, Intermediate Acting: 3-5 days, Long Acting: 5-10 days

97. Patient admitted to a hospital because of Alcohol withdrawal


symptoms. What is the difference between alcohol withdrawal &
opioid withdrawal?
a) Opioids take long time
b) Alcohol withdrawal is more severe than opioid withdrawal effect
c) Opioid withdrawal more severe than Alcohol withdrawal effect

98. Alcohol withdrawal symptom:


a) Seizures
b) Miosis

99. Alcohol withdrawal can be treated with: Non pharmacological options


Opioid withdrawal within week. Alcohol withdrawal 2-3 days. High alcoholic withdrawal 5 days
Approximately two-thirds of patients with mild to moderate withdrawal symptoms can be managed
with supportive treatment and monitoring
Severe alcohol withdrawal benzodiazepines (>50 mg diazepam iv in the first hour or >200 mg in 3 hours)
and/or addition of a different GABA-active drug like phenobarbital (130–1430 mg, mean dose 390 mg

100. Upon discharge from the hospital, whom would you recommend for this patient?
a) Social worker
b) Nutritionist
c) Occupational therapist

101. Naltrexone is given for Alcohol withdrawal:


a) For long term treatment
b) For acute symptoms (clonidine)
Naltrexone, an opioid antagonist, decreases the relapse rate and number of drinking days in most patients who
take it consistently. Naltrexone 50 mg orally once/day is typically given, although there is evidence that higher
doses (eg, 100 mg once/day) may be more effective in some patients. Even with counseling, adherence rates with
oral naltrexone are modest. A long-acting depot form is also available: 380 mg IM once/month. Naltrexone is
contraindicated in patients with acute hepatitisor liver failure and in those who are opioid dependent.
Clonidine, the oldest alpha-2-agonist, given orally or transdermally has proved successful in reducing symptoms of
alcohol withdrawal, particularly hypertension and tachycardia, in patients with mild-to-moderate withdrawal.
However, there is evidence that clonidine is effective as monotherapy to prevent alcohol withdrawal seizures or
alcohol withdrawal delirium.
Acamprosate, a synthetic analogue of gamma-aminobutyric acid, is given as 2 g orally once/day. Acamprosate
may decrease the relapse rate and number of drinking days in patients who relapse.
Nalmefene, an opioid antagonist, and topiramate are under study for their ability to decrease alcohol craving.

102. When to start therapy for RA? Best early when diagnosed

103. Depression case with sexual dysfunction, hyperlipidemia and was taking metoprolol, Ator, ASA,
sildenafil, gliclazide. What to give him for depression treatment
a) Bupropion
b) Mirtazapine
c) Venlafaxine
d) TCAs
Mirtazapine cause weight gain and patient have high lipid

104. What may worsen his depression? Propranolol. (BB worsening depression)
Drug induced Depression: ACEI, acetazolamide, amphetamine/ cocaine withdrawal, anticonvulsants,
amantadine, barbiturates, BCPs, belimumab, benzos, bromocriptine, caffeine, chemotherapy some,
cimetidine, clonidine, dapsone, digoxin, disulfiram, efavirenz, ethambutol, ethanol, finasteride,
griseofulvin, haloperidol, hydralazine, interferon, isoniazid, isotretinoin, levodopa, mefloquine,
methyldopa, methylphenidate, methysergide, metoclopramide, metronidazole, nitrofurantoin, NSAIDs,
opiates, physostigmine, procainamide, progestins, propranolol, reserpine, rilpivirine, streptomycin,
steroids, sulfas, tetracycline & thiazides

105. How to prevent dispensing High KCL conc.?


a) Florescent label fatal if injected undiluted

106. Patient with cold sores 5 days ago, wants treatment to accelerate the healing?
a) Acyclovir oral
b) Valcyclovir
c) Docosanol cream
d) Nothing you can give can accelerate the healing
Treatment should initiate when symptoms appears. ABREVA shortens healing time and the duration of cold sore
symptoms including pain, burning, tingling and itching.
If patient <12 years or If > 6 times/year → Refer

107. New drug in the market where to find its storage conditions:
a) Manufacturer leaflet
b) Micromedex .. no storage
Pantoprazole counselling
c) Martindale .... no storage &CI 1- Should be taken before breakfast, with or without food.
2- Swallow the tablet(s) whole, with water. Do not crush or
chew the tablet(s).
108. Patient with GERD, What to council for 3- Keep taking until you have finished all your tablets,
Pantoprazole? as recommended by your doctor. Do not stop even
when you start to feel better.
a) Don't chew or crush 4- If you forget to take one dose, take a tablet as soon
as you remember, unless it is almost time for your
b) Need long term ttt next dose. If it is, do not take the missed tablet at all.
Improvement 2-4 weeks, if recurrence use it long term Never double-up on a dose to make up for the one
you have missed, just go back to your regular
Omeprazole have to keep in the original container schedule.

109. Ciprofloxacin side effects? Dysglycemia (Hyper and hypoglycemia)


COBTSH: Cardiovascular, Ocular, Bloodglucose, Tendinitis, Photosensitivity & CNS.
Tendinitis or tendon rupture requiring surgical repair, has occurred with the use of fluoroquinolone, primarily
involving the shoulder joint, hand or Achilles tendon. It appears to occur more often in men, the elderly or in
patients who have recently taken corticosteroids. If tendinitis occurs, fluoroquinolone therapy should be
discontinued, physical exercise should be avoided and a physician consulted.

110. Which advice can you tell to the patient who is on Lithium:
a) Limit caffeine intake
b) Drink a lot of fluids
Lithium cause polydipsia. Patient counselling: Keep your salt intake and coffee

111. Lithium SE on Thyroid? Hypothyroidism


Also cause weight gain, tremors, Diarrhea Alopecia, Psoriasis
Switch from IR to SR. Monitor thyroid, renal function and Lithium level
Toxicity: weight loss. Elevated Liver Enzyme is not a side effect

112. Which will react with Ciprofloxacin all except:


a) Phenytoin
b) Caffeine
c) Ferrous sulfate
d) Statin especially Pravastatin
113. Patient gets tired with he climbs a flight stair and walking few minutes in the sidewalk
next to him home, does not get shortness of breath when wearing his cloths and does not feel
tired when resting. Which NYHA stage?
a) NYHA I
b) NYHA II
c) NYHA III
d) NYHA IV

114. 4 years old child with pinworm and anal itching while other family members have no
symptoms. Father is wondering whether all family members should be treated because of the
cost of the medication: What to tell him?
a) All family members should be treated,
b) Child only needs to be treated,
c) Child can be treated with 2 doses and the rest of the family with a single dose
d) First all members treated, then only patient with symptoms will take repeated dose

115. What the DOC? Pyrantel pamoate (nonprescription)

116. How to determine the dose? Weight (Dose of Pyrantel Pamoate: 11 mg/kg)

117. What is alternative in case of Pyrantel Pamoate failure?


a) Mebendazole (prescription drug)
b) Metronidazole
c) Fluconazole
TREATMENT:
A single dose of any of the following, repeated in 2 weeks, is effective in eradicating pinworms (but not ova)
in > 90% of cases:
• Mebendazole 100 mg orally (regardless of age)
• Pyrantel pamoate 11 mg/kg (maximum dose of 1 g) orally (available over the counter)
• Albendazole 400 mg orally
Carbolated petrolatum (ie, containing carbolic acid) or other antipruritic creams or ointments applied to the
perianal region may relieve itching.
The following can help prevent the spread of pinworm:
Pinworm reinfestation is common because viable ova may be excreted for 1 week after therapy, and ova deposited
in the environment before therapy can survive 3 weeks. Multiple infestations within the household are common,
and treatment of the entire family may be necessary.
• Washing the hands with soap and warm water after using the toilet, after changing diapers, and before
handling food (the most successful way)
• Frequently washing clothing, bedding, and toys. Vacuuming the environment to try to eliminate eggs
• If people are infected, showering every morning to help remove eggs on the skin
118. Patient using levothyroxine and ferrous fumerate. Upon asking him about the use of the
medication he said that he was taking them both together all that time. What is the most appropriate
counsel a pharmacist should provide?
a) Separate 12 hours apart
b) Check TSH levels with your physician
c) Decrease dose of levothyroxine
d) Increase dose of levothyroxine
e) Stop levothyroxine

119. Patient treated with Levothyroxine


when should the doctor adjust the dose?
a) After 1 week
b) Two weeks
c) 3weeks
d) 4-6 weeks

120. Levothyroxine toxicity will cause.


a) Lethargy
b) Nervousness
c) Weight gain
d) Hypotension
e) Bradycardia

121. Creatinine clearance 23mmol/min/1.72


m2...chronic kidney disease, which stage?
a) Stage 1
b) Stage 2
c) Stage 3
d) Stage 4
e) Stage 5

122. A woman with very swollen hand, erythema, and fever do not sleep miis 2 days of
work doctor prescribe antibiotic so:
a) Will decrease swelling in 24 h (cellulitis monitoring)
b) Improve sleep
c) Decrease absence from work
123. Raynaud's disease and renal artery stenosis. How to treat hypertension?
a) ACEI
b) B blocker
c) CCB Nifedipine

124. What to give to pregnant with DVT?


a) Enoxaparin
b) Warfarin
c) Dabigatrin
d) ASA
LMWH is the anticoagulant of choice during pregnancy. UFH is used when LMWH is not available. LMWH
induces less bone loss than UFH and can be used daily. UFH and LMWHs do not cross the placental barrier.
Warfarin is usually avoided during pregnancy because of the potential for teratogenic effects.

125. Breastfeeding mother and wants to increase her lactation what to give:
a) Bromocriptine
b) Metoclopramide
c) Domperidone

126. Calculation Aminoglycoside IV in a hospital where they approximate the dose to


NEAREST 50 ... dose is 420 mg Stock 80 mg/2 ml how many ml will u take?

127. A person on phenytoin and came in to the hospital because of seizure, on cardiac
medications, diabetic and there are given values concerning his status, phenytoin level is below
the appropriate serum level, why did he get seizure?
a) Too low dose of phenytoin
b) Needs dual medication
c) Switch to CBZ

128. Doctor changed him to 460 mg phenytoin po, after 6 days they measured his serum levels,
and they were in the range, why is this not reliable?
a) Should measure 2 hours post dose
b) Should measure 6 hours post dose
c) He's taking an oral medication
Phenytoin at high doses becomes zero order kinetics because of saturation
Phenytoin clinically effective serum levels: 40-80 micromol/L
Peak serum concentrations are achieved between 3–12 hours following an oral dose
Phenytoin reaches steady state in 7-10 days (1-2 weeks)
Phenytoin may induce the metabolism of Plan B and therefore decrease its efficacy
Can give Depo, Patch, barrier method (Not oral). Phenytoin may cause folic acid deficiency
PHENYTOIN:
➢ P: P-450 interactions
➢ H: Hirsutism, Hyper Glycemia, Hypo thyroidism
➢ E: Enlarged gums
➢ N: Nystagmus (Involuntary movement of eyes)
➢ Y: Yellow-browning of skin, steven Johnson syndrome
➢ T: Teratogenicity
➢ O: Osteomalacia
➢ I: Interference with folic acid absorption (hence anemia)
➢ N: Neuropathies: vertigo, ataxia, and headache

129. Patient have gingival hyperplasia. What the most common cause of gingival hyperplasia:
a) Phenytoin
b) Valproic acid
c) Carbamazepine

130. So, what to do


a) Decrease dose
b) Change drug
c) Use proper oral hygiene
d) Use teeth brush right
e) Use mouthwash
If patient on Phenytoin and experiences gingival hyperplasia then refer to a dentist!
Do not brush up and down. Phenytoin Increase INR by unknown mechanism

131. BMI calculation

132. Anorexic patient treatment BMI <18.5, 5ft,7inches what is the minimum weight she should be?
a) 42
b) 48
c) 55
d) 60
133. Patient with anorexia with a height of 6 feet and 5 inches. According to standard guidelines
BMI < 18.5 is considered low weight. What is the minimum acceptable weight for this patient?
a) 30
b) 39
c) 40
d) 49
(none is right according to numbers here it should 70 kg)

134. All of the following cause cross sensitization or cross allergy with phenytoin except:
a) CBZ → most one
b) Lamotrigine
c) Valproic acid
d) Phenobarbital
Or Levetiracetam

135. Stevens–Johnson syndrome is due to administration of


a) Carbamazepine
b) Valproic acid
c) Phenobarbital
If dose titration...Lamotrigine
Skin rash Q. If a definite drug rash develops, the AED should be stopped
The AEDs most likely to cause skin rash immediately.
include phenytoin, carbamazepine and Continuation of AEDs after a skin rash develops can result in life-
lamotrigine, although rash has been threatening reactions.
reported with almost every AED. In patients with a history of AED related skin rash, consider an
Patients who have experienced skin rash AED with low risk of skin rash and not chemically related to the
with one AED are much more likely to offending agent.
develop a skin rash with another AED Introduce the AED slowly and monitor carefully for rash.
associated with high risk of rash. Q. Patients of Han Chinese and Japanese descent with certain HLA
Rash is most likely to occur within the status may be at higher risk of carbamazepine-associated rash.
first 6 wk of AED introduction, but can Consider checking HLA status before starting carbamazepine in
occur at any time during treatment. these patients.

136. Which of the following cause dose related Steven-Johnson Syndrome?


a) Phenytoin
b) Carbamazepine
c) Valproic acid
d) Lamotrigine
137. Least concern when using tertiary reference
a) Accuracy
b) Personal bias
c) Sponsor
d) Transcribing error

138. In dispensing a prescription by law, the pharmacist is responsible on all except


a) Accuracy
b) Efficacy
c) Appropriateness

139. What is reason to refer for candida?


a) Use OCP
b) Use antibiotic
First time, <12 year, Pregnant, Diabetic, Recurrent within two months, 4th time in same year, or if malodor so this
is bacterial then refer

140. A mother is getting Nystatin cream for her breast candidiasis and her baby is getting
Nystatin oral suspension for his oral candidiasis, which of the following is considered a good
advice for the mother:
a) Use a bottle to feed your baby during the time of treatment. (because the rest is wrong)
b) Use a dropper or cotton swap to apply the Nystatin drops into your baby`s mouth
c) Apply the Nystatin cream to the infected area just before feeding your baby.
d) Put nystatin drops in his mouth then breast feed him to ensure he swallow it
Shake oral suspension well prior to use. Swish and gargle in the mouth for as long as possible prior to swallowing.

141. Technician council on med while you were on phone, so you should
a) Advise her not to council
b) Council under supervision
c) Council on OTC med
142. What the least her to tell her?
a) Did you tell the patient about Side Effects?

143. How to decrease hospital pharmacy load of work


a) Decrease hours on weekend
b) Hire student and intern and pay them
c) Decrease overlap and use Automation
d) Decrease stuff in weekend

144. What is the reason doctor documenting in hospital pharmacy?


a) For patient interest
b) Do not depend on staff cognitive function
c) For reimbursement

145. CPS contains:


a) Off-label use of drugs
b) Canadian and American drugs
c) List of investigational drugs
d) Conversion from American units to Canadian units
Unit conversion, but it contains too some reference for/info about off-label use.

146. A mother with watery eyes and foreign body sensation and pain for 2 days because her
child put his finger in her eyes what to give:
a) Polymoxin B
b) Refer
Red flags...pain, foreign body

147. Patient with severe cellulitis, what to give for IV


a) Cloxacillin
b) Norfloxacin
c) Ceftazidime
Mild to mod: cephalexin po
Severe:1st line: IV cefazolin +/- po clindamycin. 2nd line iv clindamycin

148. What is the endpoint treatment of cellulitis?


a) Reduce inflammation
b) Negative culture of skin sample
c) Disappearance of lesions
Cellulitis monitor: No Erythema, No fever within 7days, Negative culture
149. Aseptic technique preparation contaminated due to:
a) Object
b) Personal
c) Technique

150. CPS contain


a) Contains approved indications
b) Unapproved indications
Unapproved indications found in eCPS

151. Androgen prescription. Where to look for regulations;


a) Health Canada
b) Targeted substances act
c) Narcotic & controlled substance act CDSA

152. Hospital pharmacy why to put policy & regulations


a) To be eligible for third party funds for
b) To ensure quality assurance
Hospital fulfills federal requirements, to be eligible for funds.

153. Girl want Plan B and the pharmacist did not want to dispense. What to do:
a) Give her this time only
b) Send her to the doctor
c) Send her to nearby pharmacy
Patient taking plan B the patient can take dimenhydrinate 30-60min before to prevent nausea

154. Plan B wanted and use it from 1 month, why do u give her
a) Justice
b) Fidelity
c) Paternalism
d) Autonomy
e) Beneficence

155. Therapeutic decision made after conducting trial on small group what is conflicting
a) Autonomy
b) Non maleficence
c) Veracity
d) Justice
156. Volunteer in an RCT, which ethical principal will conflict with concept of administering placebo
to him
a) Autonomy
b) Justice
c) Veracity

157. Aboriginal patient has believed that hearing something bad can happen to him, what is
conflicting with the pharmacist when he is trying to council her
a) Confidentiality
b) Autonomy
c) Veracity
d) Justice

158. Fish contains high amount of


a) Phosphorous
b) Potassium...

159. Which vitamins would you give to a patient undergoing hemodialysis:


a) Water soluble vitamins plus vit B complex
b) Vit ADEK
c) Minerals
d) Refer to physician to assess his need

160. Woman want to be pregnant soon so should take:


a) Multivitamins
b) Multivitamin with iron
c) 1mg folic acid
d) 5gm folic
e) ADEK
1 mg folic OTC (normal patient). 5 mg folic Shelf (CNS problem, obese, Neurotuble defect)
When give maternal multivitamin for woman wants to get pregnant, she should start 3 months before
161. When to start her vitamins for pregnancy
a) One month after pregnancy
b) Three months before pregnancy
c) First day
d) Continue at end of pregnancy

162. All of the following are true for the Calcitonin salmon spray, except?
a) It can not be used in Shell fish allergy
b) Calcium and vitamin D are recommended with calcitonin therapy.
c) It is used in alternate dose.
d) Prime when use
one spray (200 IU) once a day administered intranasally, alternating nostrils daily into one nostril only. You should
switch between each nostril every time you use Sandoz Calcitonin NS.
Your physician may prescribe calcium and vitamin D together with Sandoz Calcitonin NS to help retard the
progressive loss of bone mass. Upon, first use only, the pump must be primed. The product should be allowed to
reach room temperature before priming

163. Patient asthmatic has sulfa allergy don't give Dorzolamide/timolol, what is the reason?
a) Drug interaction between sulfa and dorzolomide. (caution)
b) Drug disease reaction: Timolol and Asthma (contraindication)
Also, Celecoxib, All ACEI except enalapril

164. Pt has cancer due to genetically causes, worried that her 13yrs old daughter will get it too.
Living with her mother and her son 19yrs old, which of the following support autonomy?
a) Share the decision with her mum
b) Share decision with her son
c) Share the decision with her daughter
d) She takes decision on her own
e) Her daughter takes decision on her own

165. Ca gluconate 3g if each 1 g gives 4.5 meq of cagluconate what is the mmole of ca
1gm --- 4.5m. eq 3gm --- X X = 3 * 4.5 / 1 = 13.5
m.mole = m. eq / valence = 13.5 / 2 = 6.75

166. Which one needs shaking


a) Nitroglycerin
b) Epipen
c) Tobradex
d) Miacalcin
e) Advair discus
167. All are true about use of Epipen except? Shake before use

168. Calculation of NNT, answer was 18 then?


a) 18 patients needed to show adverse outcome
b) 18 patients needed to show (positive) outcome

169. Old woman in care home what to do for her medications


a) Send her medication to home at easily open vial
b) Send her medication in blister pack
c) Let her come to take her medication
d) Let her son to take her medication

170. 45-year women obese height? weight? (BMI = 35) not smoking but parents do caffeine 4
cups eat too much diabetic so diagnosed for lipids what make u start medication
a) Age
b) Gender
c) Obesity
d) Diabetes → If Diabetes not in answers choose obesity
e) Family history of smoking

171. Hyperkalemia, asymptomatic, treatment with?


a) Na polysterene
b) Chloestyramine
c) Insulin with glucose DOC
Mild hyperkalemia
Patients with serum potassium < 6 mEq/L (< 6 mmol/L) and no ECG abnormalities may respond to diminished
potassium intake or stopping potassium-elevating drugs. The addition of a loop diuretic enhances renal potassium
excretion as long as volume depletion is not present.
Sodium polystyrene sulfonate in sorbitol can be given (15 to 30 g in 30 to 70 mL of 70% sorbitol orally every 4 to 6
hours). It acts as a cation exchange resin and removes potassium through the gastrointestinal mucosa. Sorbitol is
administered with the resin to ensure passage through the gastrointestinal tract. Patients unable to take drugs
orally because of nausea or other reasons may be given similar doses by enema. Enemas are not as effective at
lowering potassium in patients with ileus. Enemas should not be used if acute abdomen is suspected. About 1 mEq
(1 mmol) of potassium is removed per gram of resin given. Resin therapy is slow and often fails to lower serum
potassium significantly in hypercatabolic states. Because sodium is exchanged for potassium when sodium
polystyrene sulfonate is used, sodium overload (see Hypernatremia) may occur, particularly in oliguric patients
with preexisting volume overload.
In patients with recurrent hyperkalemia, avoidance of drugs that can induce hyperkalemia (see table Factors
Contributing to Hyperkalemia) is generally all that is needed. In patients who need ACE inhibitors and angiotensin
receptor blocking agents (eg, patients with chronic heart failure or diabetic nephropathy), the polymer
resin patiromer can be taken daily to help decrease gut absorption of potassium and prevent hyperkalemia.
Moderate to severe hyperkalemia
Serum potassium between 6 and 6.5 mEq/L (6 and 6.5 mmol/L) needs prompt attention, but the actual treatment
depends on the clinical situation.
If no ECG changes are present and renal function is intact, maneuvers as for mild hyperkalemia are usually
effective. Follow-up serum potassium measurements are needed to ensure that the hyperkalemia has been
successfully treated.
If serum potassium is > 6.5 mEq/L (6.5 mmol/L), more aggressive therapy is required. Administration of regular
insulin 5 to 10 units IV is followed immediately by or administered simultaneously with rapid infusion of 50 mL 50%
glucose. Infusion of 10% dextrose in water (D/W) should follow at 50 mL/hour to prevent hypoglycemia. The effect
on serum potassium peaks in 1 hour and lasts for several hours.
If ECG changes include the loss of P-wave or widening of the QRS complex, treatment with IV calcium as well as
insulin and glucose is indicated; 10 to 20 mL of 10% calcium gluconate (or 5 to 10 mL of 22% calcium gluceptate) is
given IV over 5 to 10 minutes. If the ECG shows a sine wave pattern or asystole, calcium gluconate may be given
more rapidly (5 to 10 mL IV over 2 minutes). Calcium antagonizes the effect of hyperkalemia on cardiac muscle.
Calcium should be given with caution to patients taking digoxin because of the risk of precipitating hypokalemia-
related arrhythmias. Calcium chloride can also be used but can be irritating to peripheral veins and cause tissue
necrosis if extravasated. Calcium chloride should be given only through a correctly positioned central venous
catheter.
The benefits of calcium occur within minutes but last only 20 to 30 minutes. Calcium infusion is a temporizing
measure while awaiting the effects of other treatments or initiation of hemodialysis and may need to be repeated.
A high-dose beta 2-agonist, such as albuterol 10 to 20 mg inhaled over 10 minutes (5 mg/mL concentration), can
lower serum potassium by 0.5 to 1.5 mEq/L (0.5 to 1.5 mmol/L) and may be a helpful adjunct. The peak effect
occurs in 90 minutes. However, beta 2-agonists are contraindicated in patients with unstable angina or acute
myocardial infarction.
Administration of IV sodium bicarbonate (NaHCO3) is controversial. It may lower serum potassium over several
hours. Reduction may result from alkalinization or from the hypertonicity due to the concentrated sodium in the
preparation. The amount of sodium contained in the infusion may be harmful for dialysis patients who also may
have volume overload. Another possible complication of IV sodium bicarbonate is that it acts to acutely lower the
ionized calcium concentration, which further exacerbates the cardiotoxicity of hyperkalemia. When sodium
bicarbonate is given, the typical dose is 3 ampules of 7.5% sodium bicarbonate in one liter 5% D/W infused over 2
to 4 hours. Bicarbonate therapy has little effect when used by itself in patients with severe renal insufficiency
unless acidemia is also present.
In addition to strategies for lowering potassium by shifting it into cells, maneuvers to remove potassium from the
body should also be done early in the treatment of severe or symptomatic hyperkalemia. Potassium can be
removed via the gastrointestinal tract by administration of sodium polystyrene sulfonate, but because the rate of
potassium removal is somewhat unpredictable, close monitoring is needed.
Patiromer is not recommended for use as an emergency treatment to acutely lower potassium because of its
delayed onset of action.
Hemodialysis should be instituted promptly after emergency measures in patients with renal failure or when
emergency treatment is ineffective. Dialysis should be considered early in patients with end-stage renal disease
and hyperkalemia because they are at increased risk of progression to more severe hyperkalemia and serious
cardiac arrhythmias. Peritoneal dialysis is relatively inefficient at removing potassium acutely.
172. Study followed during 5 years between amlodipine and Metoprolol, Patients were chosen
by chance. What study is that?
a) Cohort
b) RCT

173. Type of the head lice (generic and sp name)? Pediculosis capitis
Pediculosis Capitis in Head ‫ﻛﺎب ﻓ اﻟرأس‬
Pediculosis Corporis in body
Pediculosis Pubis in Pubic

174. Calculation for amlodipine and metoprolol, given numbers to calculate RRR? Not
specifying what control and what not answer was 10.4

175. Pharmacy community project (cannot remember name but not blueprint) aim at all except;
a) Changing the organization of the pharmacy
b) To be eligible for third party funding

176. Objective of pharmacy blueprint, except?


a) Layout of pharmacy
b) Development and improvement
c) Patient preference
d) Updated technology

177. What is true about capsaicin?


a) It may take one week to see an effect.
b) The burning sensation will go with continuous use.
c) Use a bandage with it.
d) Start action after 1 week
Capsicum The burning and painful sensations associated with capsaicin result from its chemical interaction
Derivatives with sensory neurons.
Q. Arnica gel may be comparable to topical ibuprofen for relief of OA pain.
Capsaicin Apply cream or ointment sparingly 3 or 4 times daily and massage into the affected area
Arnica gel Following application, the action of capsaicin has been noted to last 4–6 h.
Full therapeutic response may not be apparent for 1–2 wk in arthritic disorders or 2–4 wk in
neuralgias.
Regular application is essential for optimal pain relief.
Consider wearing gloves to apply capsaicin. Rub in well until no residue remains on the skin. If
not wearing gloves, wash hands with soap and warm water immediately after application.
However, if capsaicin is used on hands, refrain from washing hands for a minimum of 30 minutes.
Heating pads should not be used and hot baths or showers should be avoided immediately prior to
or following application since warm water or excessive sweating may intensify the localized
burning sensation. Do not wrap or cover area with a bandage.
Marketed as a nonprescription natural health product.
Avoid contact with eyes or open lesions.
If patients experience pain, swelling or burning from topical analgesics, advise them to stop using
the product and seek immediate medical attention.
S.E: Local pain, burning, stinging or erythema → This is usually a transient effect most prominent
in the 1st wk of treatment; it diminishes or disappears with continued use at the recommended
dose. If applied less frequently than recommended or used intermittently, the burning effect may
persist. Heat and extensive perspiration may intensify the burning effect.
Coughing → Likely due to inhalation of dried drug residue. Can be prevented by washing the
treated skin 30–40 min after application

178. Calculation about dosing and wants rate of infusion in ml/ kg, Wt 15 kg

179. Withdrawal symptoms of venlafaxine?


FINISH: Flu like, insomnia, nausea, imbalance, sensory disturbance & hyperarousal

180. How to treat? By giving low dose venlafaxine.


The syndrome can be reversed by restarting the antidepressant and tapering the dose more slowly.
Alternatively, if a slow taper is poorly tolerated, substitute with one dose of fluoxetine 10–20 mg po.

181. Patient on HCTZ 50mg and amlodipine 5mg, got hypokalemia. What is DTP?
a) Taking med, no need for it
b) DDI
c) Taking too low dose
d) Taking too high dose of HCTZ
182. A patient just started paroxetine 5 days ago; the pharmacist will call him to follow up on
which side effect?
a) Dry mouth
b) Insomnia
Commonly Observed Adverse Events: Dry mouth
Adverse Events Leading to Discontinuation of Treatment: Insomnia

183. Which interacts with SSRIs: Tramadol


Tramadol increases Serotonin Syndrome with SSRI

184. Which SSRI is most nausea and which least?


Most = fluvoxamine, Least = sertraline, Citalopram: fewest drug interactions
185. How to detect discontinuation of SSRI for a patient? Late refills

186. Calculation about % accuracy. Giving false + ve, false -ve, true +ve & true -ve

187. Long case about patient has leg amputation and take Oxycodone CR, Oxycodone PRN for
breakthrough pain and take Temazepam, Nabilon and Dr want to give him Methadone for pain
management 100mg to be dispensed in a weekly base so, what is wrong (DTP)
a) Too high methadone dose
b) Too much opioid use
c) Methadone should be dispensed in a daily basis
Methadone should not be used in opioid naive patients. The usual adult oral dose is 2.5 to 10 mg every 4
hours during the first 3 to 5 days, followed by a fixed dose every 8 to 12 hours depending on the patient's
requirements. In geriatric patients the dosage schedule could be given on a once daily basis.

188. Differentiation between methadone and buprenorphine/naloxone (obsolete)


a) No need for Doctor exempted with buprenorphine/naloxone
b) Give less respiratory distress
c) Is given parenteral

189. How to avoid confusion that may happen between distilled water and methadone stock solution?
a) Putting the water in transparent bottles & methadone stock in dark bottles
b) Pharmacists are only allowed to touch & prepare methadone
c) Keep the prepared methadone in the narcotic stock (obselete)

190. 6y child with lice allergic to ragweed, what to give? permethrin


Isopropyl Mystriste can be given to a patient with a ragweed or chrysanthemum Allergy
Permetherin is: Given to prego, Permetherin should be left in for 10 minutes and then rinsed.
C/I: chrysanthemum and Ragweed allergy

191. Child with long hair, treated 2 times


with Permethrin shampoo and not cured.
What is reason?
a) Hair not soaked well

192. What is the most expected side effect of


anti-lice shampoo
a) Burning and stinging
b) Hair thinning
c) Hair discoloration
193. Her mother which is pregnant got lice, what to give? Permethrin

194. Patient with lice and tried Nix (Permethrin) twice and failed to treat it, what to give?
a) Lindane
b) Pyrethrine
c) Butoxide,
d) Oral clindamycin
Permethrin 5% over night or lindane. Oral SMX/TMP + Permethrin 1%. Ivermectine oral repeated within 7days
Lindane is an effective pediculicide; however, there are concerns about neurotoxicity and bone marrow
suppression after percutaneous absorption. Lindane is currently not available in Canada.
Treatment failure of head lice may be due to resistance to topical agents with a neurotoxic mode of action,
including permethrin and pyrethrin/piperonyl butoxide.
In contrast, resistance to those products with a physical mode of action (isopropyl myristate 50%/cyclomethicone
50% and dimeticone) is unlikely to develop. Other explanations for treatment failure include misdiagnosis, lack of
adherence or reinfestation. Treatment with permethrin may fail if hair is not thoroughly soaked. Two bottles are
often needed for thick or long hair. In cases of resistance, switching to a different pharmacologic class may be
helpful, although resistance to permethrin may cross over to pyrethrins and other pyrethroids.
The following have been advocated:
permethrin 5% cream applied to scalp and left on for several hours or overnight;
oral sulfamethoxazole/ trimethoprim in combination with permethrin 1%;
topical crotamiton 10% applied to scalp and left on for 24 hours in adults;
oral ivermectin 200 mcg/kg repeated in 10 days or an alternative regimen of 400 mcg/kg repeated in 7 days.
Ivermectin is not approved for use in children weighing <15 kg.
Topical ivermectin 0.5% is effective for treatment of head lice, but is not available in Canada.
Topical ivermectin 1% is available in Canada for the treatment of rosacea but it has not been studied in the
treatment of lice or scabies.

195. Stroke patient attend to hospital with his wife in 3 hrs and after giving him Alteplase you
find that it was 4.30 hrs past and doctor decide to give him so:
a) What make the drug non effective (as time zone past)
b) He follows paternalism, as he did not ask the consent of his daughter
After Alteplase by 24 hrs & ensure that there is no
hemorrhage by CI give ASA 160mg once, then ASA
80-325 mg.
Must be given within 4.5 hours of symptoms,
INR>1.7, No previous stroke in last 3 months,
Patient must be>18 years old. Alteplase not used
with hemorrhagic stroke
196. After patient stabilized what to give
a) ASA 180mg single dose then 80-
325mg daily
b) Clopidogrel → Clopidogrel given if he was
previously on ASA

197. Who should agree to give Alteplase?


a) The patient. if conscious
b) His wife. if unconscious

198. Stroke patient reached the hospital after 3.5 hrs from symptoms & she is (semi paralysed),
doctor ask the pharmacist to prepare Alteplase if she is good candidate but 1hr later doctor said that
she is not good candidate why?
a) Because she has hemorrhagic stroke.

199. So, what the pharmacist can recommend? PCI + vit K


Hemorrhagic Intra-cerebral Hemorrhage:
Stroke Treat with prothrombin complex concentrate, vit K or fresh frozen plasma to reverse
coagulopathy
In acute phase:
➢ Administration of recombinant factor VIIa prevents hematoma growth, but is not
recommended for routine use because it ↑ risk of arterial thromboembolic phenomena.
➢ Recombinant activated factor seven (VIIa) is a vitamin K-dependent glycoprotein that is
structurally similar to human plasma-derived factor VIIa.
After the acute phase:
➢ Manage patients with intracerebral hemorrhage similarly to those with ischemic stroke,
except that anticoagulants should be avoided.

Subarachnoid Hemorrhage: (Calcium Channel Blockers)


➢ Nimodipine 60 mg PO Q4H for 3 weeks reduces risk of 2ry vasospasm and cerebral
infarction
➢ Crushing tablets for nasogastric administration is not recommended since bioavailability
reduces with time. If necessary, crush and administer immediately.
S.E: Hypotension (5%), nausea, bradycardia and rash.
In patients with hemorrhagic stroke, the use of LMWH or UH is controversial
Antithrombotic Drugs: Avoid in patients with hemorrhagic stroke.

200. What increases the credibility of Meta Analysis?


a) Case control studies are included,
b) Reviewed by an author before being published (NA)
Blind, random, reviewed and non-biased
201. When counting narcotics tech found 50 tabs missing & told the manager what is his initial
action:
a) Notify health Canada
b) Compare actual sales & purchase records vs computer
c) Call last patient took medication & check with him
d) Call wholesaler to make sure from quantity
Sales of straight narcotics must be recorded and kept in the sale record
In case of theft of narcotics must report to Office of Control Drug Substance within 10 days.

202. You are making the inventory checkup when you found one missed narcotic bottle, what
will be your appropriate action:
a) Call the last patient that you dispensed narcotics for him.
b) Report to health Canada.
c) Check since the last inventory checkup for this medication.

203. Pharmacist and physician working together to put brochure to give to newly health
professionals, what should be included?
a) Guidelines
b) Patient information
If all except: patient information

204. Pharmacist got a Rx of medication to dispense, pharmacist by mistake wrote down ( /dose) and
it should be (…/day) and Rx was dispensed consequently patient was transferred to ICU. Hospital will
make a committee; all will attend except?
a) Nurse who gave med
b) Pharmacist who dispensed
c) Physician who wrote rx
d) Member from pharmacy management

205. What is reason of documenting the above error in hospital?


a) To report to ISMP
b) To penalize pharmacist

206. Where would a pharmacist report a medication error to (He did not specify community or
hospital)?
a) Canadian Patient Safety Institute
b) Canadian Institute for Health Information
c) Canadian Medication Incident Reporting and Prevention System
207. Where to report error happened
a) Community of patient incident report
b) Community of medication incident
c) Office of controlled
d) Health Canada

208. Patient stressed because her mother had a hip fracture and she had to take care of her.
her profile shows pantoprazole, calcium carbonate and vitamin D. what is your concern?
a) Needs to start bisphosphonates,
b) Change calcium carbonate to calcium citrate
Ca Citrate is the only supplement that does not need acidic medium to be absorbed, so, can be taken with a PPI

209. 85-year-old Patient started Dabigatran, what you should monitor? Self-monitor bleeding

210. What to do before start Dabigatran


a) Kidney function Dabigatran is contraindicated in renal failure (CrCl <30ml/min)
b) Liver function
c) aPtt

211. 85 years old woman, living on her own, only have controlled HTN. Her daughters brought
her to ER because she is not feeling well since a couple of days; she is hearing that her husband is
running after her at her home! She does not want to stay in the hospital. Then she agreed to stay,
a better a team from the hospital should advise about next step for the woman, the team is:
a) Daughter and the social worker
b) Daughter, nurse and occupational therapist.
c) Daughter, nurse and social worker
The social worker can help with the following:
1) mental health resources and services,
2) family support & psychological support,
3) help within an inpatient psychiatric treatment center, AND
4) placement in specialized care homes.
PLEASE READ the following carefully to clarify the role of social services and social workers.
§ A social worker is likely one of the first professionals a patient has contact with when s/he seeks treatment in a
psychiatric setting.
§ Social workers will run an intake and evaluation for those patients such as insurance information, history of
medical & psychiatric treatment and details of the current problem.
§ Social workers are also responsible to develop a treatment strategy by consulting psychiatrists and psychologist.
§ Social workers do also importantly provide therapies within their scope of practice such as short-term
psychotherapy, CBT, group counseling, family therapy, supportive counseling, case management and advocacy.
Plus, the daughter can be a caregiver. Nurses do lot more than just administer meds.
212. The easiest dosage form for preparation an interchangeable drug is:
a) Suspension
b) Solution
c) Sustained release tablets
d) IV
Drugs with low therapeutic index are not usually interchangeable

213. Patient with potassium levels of 5.6 and, which stage of chronic kidney disease is he in? Mild
• Normal potassium level is (3.5-5)
• <5.5-5.8 → Non pharm until potassium is less than 5
• <6.5-7 → (No ECG changes) Insulin/Glucose
• >6.5-7 → calcium gluconate

214. Patient's profile indicates his K level is 5.6 taking drugs including Coversyl, he is
complaining from muscle pain. what to be done
a) Stop temporarily the ACE

215. Beta Carotene used in Age related Macular Degeneration contraindicated with:
a) Smoking
b) Pregnancy
c) Renal failure
d) Liver problems

216. Patient poor cannot pay so for long-term coverage for his medication, what to do?
a) Take his medication as samples from doctor
b) You give him medication (and not pay)
c) Look for another association that can pay for him

217. Addison disease patient had diabetes admitted hospital for ketoacidosis, how to treat?
a) IV hydrocortisone (in adrenal crisis) + low dose insulin (ketoacidosis)
b) Prednisone
c) Dexamethasone
Treatment Addison disease
➢ Hydrocortisone or prednisone
➢ Fludrocortisone
➢ Dose increase during intercurrent illness
Normally, cortisol is secreted maximally in the early morning and minimally at night. Thus, hydrocortisone
(identical to cortisol) is given in 2 or 3 divided doses with a typical total daily dose of 15 to 30 mg.
One regimen gives half the total in the morning, and the remaining half split between lunchtime and early evening
(eg, 10 mg, 5 mg, 5 mg). Others give two thirds in the morning and one third in the evening. Doses immediately
before bed should generally be avoided because they may cause insomnia. Alternatively, prednisone 5 mg orally in
the morning and possibly an additional 2.5 mg orally in the evening may be used. Additionally, fludrocortisone 0.1
to 0.2 mg orally once a day is recommended to replace aldosterone. The easiest way to adjust
the fludrocortisone dosage is to ensure that the renin level is within the normal range and that blood pressure and
serum potassium level are normal.
Normal hydration and absence of orthostatic hypotension are evidence of adequate replacement therapy. In some
patients, fludrocortisone causes hypertension, which is treated by reducing the dosage or starting a nondiuretic
antihypertensive. Some clinicians tend to give too little fludrocortisone in an effort to avoid use of
antihypertensives.
Intercurrent illnesses (eg, infections) are potentially serious and should be vigorously treated; the
patient’s hydrocortisone dose should be doubled during the illness. If nausea and vomiting preclude oral therapy,
parenteral therapy is necessary. Patients should be instructed when to take
supplemental prednisone or hydrocortisone and taught to self-administer parenteral hydrocortisone for urgent
situations. A preloaded syringe with 100 mg hydrocortisone should be available to the patient. A bracelet or wallet
card giving the diagnosis and corticosteroid dose may help in case of adrenal crisis that renders the patient unable
to communicate.
When salt loss is severe, as in very hot climates, the dose of fludrocortisone may need to be increased.
In coexisting diabetes mellitus and Addison disease, the hydrocortisone dose usually should not be > 30 mg/day;
otherwise, insulin requirements are increased.
Treatment of adrenal crisis
➢ Therapy should be instituted immediately upon suspicion of adrenal crisis. (CAUTION: In adrenal crisis, a
delay in instituting corticosteroid therapy, particularly if there is hypoglycemia and hypotension, may be
fatal.) If the patient is acutely ill, confirmation by an ACTH stimulation test should be postponed until the
patient has recovered.
➢ Hydrocortisone 100 mg is injected IV over 30 seconds and repeated every 6 to 8 hours for the first 24 hours.
Immediate intravascular volume expansion is done by giving 1 L of a 5% dextrose in 0.9% saline solution
over 1 to 2 hours. Additional 0.9% saline is given IV until hypotension, dehydration, and hyponatremia have
been corrected. Serum potassium may fall during rehydration, requiring replacement.
➢ Mineralocorticoids are not required when high-dose hydrocortisone is given. When illness is less
acute, hydrocortisone 50 or 100 mg IM every 6 hours can be given.
➢ Restoration of blood pressure and general improvement should occur within 1 hour after the initial dose
of hydrocortisone. Inotropic agents may be needed until the effects of hydrocortisone are achieved.
➢ A total dose of 150 mg hydrocortisone is usually given over the 2nd 24-hour period if the patient has
improved markedly, and 75 mg is given on the 3rd day. Maintenance oral doses of hydrocortisone (15 to 30
mg) and fludrocortisone (0.1 mg) are given daily thereafter, as described above. Recovery depends on
treatment of the underlying cause (eg, infection, trauma, metabolic stress) and
adequate hydrocortisone therapy.
➢ For patients with some residual adrenal function who develop adrenal crisis when under
stress, hydrocortisone treatment is the same, but fluid requirements may be much lower.
Pearls & Pitfalls
➢ When adrenal crisis is suspected, give hydrocortisone treatment immediately; any delay, including for
testing, may be fatal.
Treatment of complications
➢ Fever > 40.6° C occasionally accompanies the rehydration process. Except in the presence of falling blood
pressure, antipyretics (eg, aspirin 650 mg) may be given orally with caution. Complications of corticosteroid
therapy may include psychotic reactions. If psychotic reactions occur, then the hydrocortisone dose can be
reduced to the lowest level consistent with maintaining blood pressure and good cardiovascular function.
Antipsychotics may be temporarily required, but use should not be prolonged.
Key Points
➢ Addison disease is primary adrenal insufficiency.
➢ Weakness, fatigue, and hyperpigmentation (generalized tanning or focal black spots involving skin and
mucous membranes) are typical.
➢ Low serum sodium, high serum potassium, and high BUN (blood urea nitrogen) occur.
➢ Usually, plasma ACTH is high and serum cortisol levels are low.
➢ Replacement doses of hydrocortisone and fludrocortisone are given; doses should be increased during
intercurrent illness.
Correction of hyperglycemia and acidosis
➢ Hyperglycemia is corrected by giving regular insulin 0.1 unit/kg IV bolus initially, followed by continuous IV
infusion of 0.1 unit/kg/h in 0.9% saline solution.
➢ Insulin should be withheld until serum potassium is ≥ 3.3 mEq/L (≥ 3.3 mmol/L).
➢ Insulin adsorption onto IV tubing can lead to inconsistent effects, which can be minimized by preflushing
the IV tubing with insulin solution.
➢ If plasma glucose does not fall by 50 to 75 mg/dL (2.8 to 4.2 mmol/L) in the first hour, insulin doses should
be doubled. Children should be given a continuous IV insulin infusion of 0.1 unit/kg/h or higher with or
without a bolus.

218. Same patient after discharge from the hospital he should continue using:
a) Prednisone oral
b) Dexamethasone

219. What will you tell this patient regarding his medication? Except
a) This medication may increase your blood pressure.
b) Restrict Na intake, it could make fluid retention.
c) Dr. prescribed this medication for you because of its glucocorticoid activity.
d) Take Ca and vit D with corticosteroids, it could cause osteoporosis.
All except 3, Glucocorticoids

220. Caution when handling with: Hydroxyurea → Wear gloves when handle with Hydroxyurea

221. Which drug will be in breast-feeding?


a) High molecular wt
b) Low lipid
c) Low molecular, high lipid, high partition coefficient and low V
222. Tylenol 3 regulations:
a) Written only,
b) Verbal and written Rx, no transfer, no refill
c) Can be transferred

223. Tylenol 3 mitte cc you counsel all of the following are right except:
a) Call police for forgery
b) Call clinic to verify Rx
c) Ask patient why he saw his doctor
d) Call clinic to verify Rx with physician

224. New pharmacy service will care for all except


a) Offer from third part insurance
b) Physical appearance
c) Patient care
d) Local medical approval
Offer from third party insurance (doesn't exist, unless possibly preferred pharmacy networks or selective
contracting)

225. Pharmacist do medical reconciliation to:


a) Avoid allergy
b) Decrease ADR
c) Continuity of care
If K-type take 1,2. Medication Reconciliation helps to decrease adverse drug events by 15% & medication errors by
70%

226. What needs a prescription to be dispensed:


a) Vit B12 parenteral
b) Mupirocin 2% → schedule I
Vitamin B12 with Intrinsic Factor Concentrate need prescription

227. Which will be the least to cause a dispensing error:


a) QD for every day,
b) @ instead of at,
c) Leading zero in numbers less than one
c) Abbreviation of the medication name
228. Heavy smoker patient and know that smoking harm him, he tried to quit and failed. What stage
he is on?
a) Contemplation
b) Preparation
c) Relapse

229. He heard about new drug for smoking cessation so where to look in:
a) eCPS and Drug product database

230. Insulin pen and cartilage


a) Can be mixed
b) Not for visually impaired patients
c) Should be primed each time before use
Store in fridge, for patients with vision problem, Prime every timE

231. Patient had severe back pain, was on oxycodone and BDZ and many refills, coming to ask for
Tylenol 3, what to do. Ask patient first

232. Baby 18 month with Otitis media 3 episodes. In addition, take Amoxicillin 2 month ago. What
you will give him.
a) Amoxicillin/clav
b) Azithromycin
First line for otitis media in children >6 months is wait and watch, then Amoxicillin then Amoxicillin/clav

233. Natural skin mechanism for preventing skin and tissues from infections.
a) Moist and damp conditions
b) Sebaceous gland secretions
c) Decrease blood supply to the tissue
d) Availability of bacterial nutrients

234. Diabetic patient taking 10u NPH (short acting) at breakfast & bedtime and 4u regular insulin
before each meal he got confused before dinner took 4u NPH instead of regular so what to tell patient
a) Skip tonight doses
b) Take 6u NPH & regular
c) Take 4u NPH & regular insulin
d) Skip regular & take 6u NPH → if he already took his dinner
NPH is the only one which is cloudy
235. When to give ACEIs and ARB together with all the following except:
a) Diabetes without nephropathy
b) Refractory heart failure
c) NYHA III

236. 19 years old started on escitalopram, what do you monitor? Suicide.


The only one does not cause dry mouth
Insomnia is the most common side effect of Escitalopram
Escitalopram and citalopram: high QT prolongation because they are substrates for CYP 2C19. avoid some PPIs like
Rabeprazole and Omeprazole and Esomaprozole as they are strong inhibitors of 2C19 causes QTC prolongation (in
presence of PPIs don’t give more than 20mg of citalopram and escitalopram)

237. Patient has just finished PCI for MI and wants to quit smoking which of the following is the
most right
a) Use of NRT immediately after MI is unacceptable
b) Bupropion is acceptable to use post MI.
NRT are CI with recent MI or stroke except Bupropion
There is no clinical experience establishing the safety of bupropion in patients with a recent history of myocardial
infarction or unstable heart disease. Therefore, care should be exercised if it is used in these groups.

238. All of the following are symptoms of


nicotine withdrawal except:
a) Irritability,
b) Trouble sleeping,
c) Hungry appetite
d) Lack of concentration
e) Nausea and vomiting

239. Low HDL and high TG (good HDL) what the best medication suitable for him? Fenofibrate

240. Carbamazepine started at high dose, what do you recommend?


a) Decrease dose and titrate up weekly
b) Start with loading dose
c) Use SR formulations
Start low go slow to avoid dose dependant side effect
Carbamazepine is a substrate and an inducer of CYP 3A4. it metabolizes itself (inducer of its own metabolite)
Carbamazepine advantage over phenytoin due to easier dosing due to linear pk

241. Pregnant hypertensive patient, what is the DTP to change the meds?
a) Change the ACEIs to methyldopa.
242. Hypertensive Patient have one kidney, with renal artery stenosis, newly diagnosed for diabetes,
and does not want to start pharmacotherapy. What to do:
a) Agree with pt.
b) Support doctor Decision to Start
c) Give him Herbal product

243. What do you give for BP? Amlodipine. Amlodipine not renally cleared

244. Amlodipine side effect


a) Peripheral edema
b) Constipation

245. Now she has concern about diabetes and wanted weight loss, what do you give her?
a) Metformin (cause her creatinine was >30 so it's fine)

246. Patient with diabetes, HTN, Rynaud’s phenomenon, what to give


a) ACEI
b) Nifedipine XL
c) B. Blocker

247. Patient with psoriasis with infection and on Diovax (Calcipotril/Hydrocortison) and no
improvement. Doctor prescribed for him Prednisone tab. What is the DTP?
a) Double medication
b) Medication for no indication
c) Inappropriate medication for the case
• Psoriasis is a common inflammatory disorder affecting
the skin that has a genetic component and several
triggers (eg, trauma, infection, certain drugs).
• The most common skin findings are usually well-
circumscribed, erythematous papules and plaques
covered with silvery scales in plaque psoriasis, but
lesions differ between the other less common subtypes
of psoriasis.
• Psoriatic arthritis develops in 5 to 30% of patients and can cause joint destruction and disability.
• Diagnose based on the appearance and distribution of lesions.
• Use topical treatments (eg, emollients, salicylic acid, coal tar preparations, anthralin, corticosteroids, vitamin
D3 analogs, calcineurin inhibitors, tazarotene), particularly for mild disease.
• Use ultraviolet (UV) light therapy, usually for moderate or severe psoriasis.
• For extensive psoriasis, use systemic treatments, such as immunomodulatory (biologic)
agents, methotrexate, cyclosporine, retinoids, and/or other immunosuppressants.
248. What to give him?
a) Azithromycin
b) 6 mercaptopurine
c) Methotrexate Psoriasis Risk factors
1. Alcohol (excessive)
d) Isotretinoin 2. Smoking
3. Stress
4. Genetics
249. What you could advice this patient not to worsen Psoriasis? 5. Infection
6. Obesity
a) Avoid trauma 7. Pregnancy
b) Avoid infection 8. Trauma
9. Cold weather
c) Avoid cold weather

250. What the best medication used to treat rosacea?


a) Tretinoin topical
b) Topical corticosteroids
c) Clindamycin
d) Metronidazole topical
Metronidazole used for rosacea not for acne. Rosacea topical options: Metronidazole, Azelic acid

251. Patient with Rosacea, you can advise him with which of the following:
a) Avoid smoking
b) Avoid alcohol
c) Sun exposure is good for your case
d) Avoid exposure to cold weather
SHESHAA: Spicy food, Heat, Emotional stress, Sunlight, Hot Beverages, Alcohol, Application of Corticosteroids

252. Which of the following is a suitable systemic medication for Rosacea:


a) Minocycline
b) Amoxicillin
c) Cephalexin
Rosacea systemic options: Tetracycline Class

253. 60 years old male black + hypertension+ DM and taking hydrochlorothiazide. his BP is elevated
so how to control it:
a) ACEI
b) BBs
c) Add Spironolactone
d) Switch HCTZ to furosemide
Alternatively, increase HCTZ
254. PPI:
a) Should be spaced from antacid
b) Taken 30 min before breakfast
c) Useless to take PPI with H2Ra
If patient is on PPI and is not controlled: DOUBLE THE DOSE
You shouldn’t be on PPI for a very long time because causes osteoporosis
PPI initiated 2-4 weeks, if a response happens then continue for 4-8 weeks. = try to stop PPI after

255. What interacts with Co-trimoxazole


a) HCTZ
b) Metformin lactic acidosis
Diuretics (thiazides): in elderly patients concurrently receiving diuretics, mainly thiazides, there appears to be an
increased risk of thrombocytopenia with or without purpura.

256. Patient is being placed on 10mg Atorvastatin however she is very reluctant to starting the statin
therapy because her Dad has been using statins and had really bad muscle aches and myopathy, what
is the initial most appropriate approach the pharmacist will do?
a) Acknowledge her reluctance to taking the statin
b) Evaluate her chances of getting myopathy
c) Tell her it's a rare side effect and have no worries
d) Tell her it is a dose related side effect and she should not worry

257. All true about pneumococcal vaccine except?


a) Taken every 5 to 10 years
b) Influenza vaccine enhanced if taken with pneumococcal vaccine
c) Take >65yrs
Pneumococcal vaccine doesn’t protect against Endocarditis

258. Gardasil
a) Take on 6-month duration
b) Females 9-25years
c) All types of papilloma
d) Prevent against breast cancer
GARDASIL is a vaccine indicated in girls and women 9- through 45 years of age for the prevention of infection
caused by the Human Papillomavirus (HPV). Dose at 0, 2, and 6 months
It prevents against:
Cervical, vulvar, and vaginal cancer caused by HPV types 16 and 18
Genital warts (condyloma acuminata) caused by HPV types 6 and 11
259. Addison disease one they brought u a chart with equivalence to methyl prednisone so methyl
prednisone was 4 and prednisone was 5 in the chart and there was dexamethasone (the chart included
their glucocorticoid and mineralocorticoid levels) and he was taking methyl prednisone 15 mg BID
and he wanted to change it to prednisone so what dose would you give?
a) 20 mg
b) 40 mg
c) Change to a different corticosteroid

260. Baclofen SE: Dizziness


1st line for muscle spasticity is baclofen & tizanidine. TAZmanian devil spazzed out his BACk)
Muscle Baclofen is capable of inhibiting both monosynaptic and polysynaptic reflexes at the spinal
Relaxants level, possibly by hyperpolarization of afferent terminals, although actions at supraspinal sites
Baclofen may also occur and contribute to its clinical effect.
Start with 5 mg TID PO; increase gradually to maximum of 20 mg TID
S.E: Sedation, muscle weakness, nausea, dizziness. Hepatotoxicity (very rare).
Potential additive CNS depression with TCAs, opioids, benzodiazepines, antihypertensives.
Adjust dose gradually to minimize adverse effects or withdrawal symptoms.
In acute pain, short-term use as muscle relaxant. Not recommended in >65 y.
Abrupt discontinuation can result in confusion, seizures and hallucinations.

262. 1000 ml of 10% dextrose w/v dextrose was prepared. How many KCL will this make knowing
that 1 gram of dextrose gives 3.4 KCL?

263. A woman come taking a cough med contain codeine and u saw here yesterday in another
pharmacy taking same medication so
a) Call police
b) Refer her to doctor
c) Asses her cough
d) Do not sell here

264. T1/2 affected by: Volume of distribution

265. Child 10 years old being diagnosed with ADHD and his parents were trying to control it using
non pharmacological measures but was ineffective, which treatment should they initiate:
a) Dextroamphetamine = Methylphenidate
266. When should they see
improvement of his behaviour?
a) 1 week
b) 2 weeks
c) 4 weeks
d) 3 months

267. After 4 months the child showed no improvement and doctor changed his medication to a drug
which can be prescribed verbally and transferred easily, what is this?
a) Atomoxetine
b) Amphetamine

268. Atomoxetine, which side effect needs careful monitoring?


a) Suicidal ideation
b) Constipation
Patients with ADHD may have a slightly increased risk of suicidal thoughts and behaviours. Reports of
suicide-related events in patients taking stimulants and atomoxetine have prompted warnings that the
medications may contribute to this risk in some patients. The reported events occurred at varying
times during treatment, but the greatest concern was at the start or end of treatment, or when the
dosage was adjusted. Patients, family and friends should watch for these symptoms and report them
promptly. Health-care professionals should also monitor patients on medication for ADHD and consider
a change in treatment if concerns arise.
Both methylphenidate and amphetamines have been associated with increased risk of psychosis in
patients with ADHD. An observational study suggested the risk of new-onset psychosis in adolescents
and young adults with ADHD taking stimulants was 1 in 660, and that the rate was higher in those
taking amphetamines compared to methylphenidate. As a precaution, consider non-stimulant
options (e.g., atomoxetine and/or guanfacine) first in patients with a history of psychosis.
Health Canada and regulatory agencies in other countries have issued advisories about the possible
association of stimulants (and atomoxetine) with cardiovascular risks, such as sudden cardiac death in
those with underlying cardiac anomalies, and with adverse psychiatric symptoms, such as
hallucinations and agitation. The FDA and Health Canada have issued warnings that stimulants and
atomoxetine may be associated with rare reports of priapism in male children, adolescents and adults
taking these medications.
Inform patients (and parents/caregivers where appropriate) about potential adverse effects on sleep,
appetite, cardiovascular or psychiatric health, the possibility of prolonged erections (>2 hours), and the
need to seek medical attention if these events occur. The CADDRA 2018 guidelines contain useful tools
for monitoring adverse effects.
269. 39-year-old female on Escitalopram & started to feel some anxiety, shortage of sleep, inability to
listen, was recently diagnosed with ADHD, what is the most prominent symptom of ADHD?
a) Anxiety is a common symptom of adult
b) Inability to listen
c) Insomnia
Mood and anxiety disorders occur in about 25% of ADHD patients
Easily distracted, finding it difficult to listen to others in a conversation

270. What is the best course of action?


a) Keep Escitalopram & add atomoxetine
b) Change escitalopram to fluoxetine & add atomoxetine
c) Stop escitalopram & add atomoxetine

271. Which of the following is lead to pediatric lymphoma and cancer


a) Infliximab
b) Methotrexate
c) Leflunomide
d) Quinidine
e) Hydroxychloroquine
All other options given are CI in children except Infliximab

272. Effect of infliximab after 1 to 2 hrs.


a) Hypotension
b) Hypokalemia
c) Tinnitus at infusion site

273. Allergic rhinitis for 4 weeks and tried antihistaminic what to do?
a) Change
b) Refer
ACUTE SINUSITIS: Refer to ear, nose and throat (ENT) specialist if a patient: fails second-line therapy, experiences 4
or more episodes of bacterial rhinosinusitis per year, has anatomic anomalies & develops complications

274. Patient with signs of allergic rhinitis with watery eyes, nasal discharge and green respiratory
discharge for 3 weeks. She tried taking Loratadine but was ineffective. Why would you refer this
patient?
a) Because she had these symptoms for more than 2 weeks
b) Because green discharge indicates viral infection
c) Because loratadine was ineffective
275. Allergic rhinitis with no documented allergen, she has also atopic dermatitis what is the risk
factor here?
Atopic dermatitis may be associated with other atopic conditions such as asthma, allergic
rhinoconjunctivitis & food allergies.

276. Patient with allergic rhinitis and hypertension how to help her.
a) Desloratadine
b) Oxymetazoline
c) First generation Antihistaminic
d) Pseudoephedrine

277. Patient diagnosed with BPH and doctor started him on Finasteride. Which side effect should the
patient be aware of? Sexual dysfunction
Finasteride is the DOC in enlarged prostate (BPH)
Finasteride for Alopecia is 1mg (using its side effect Hirsutism as effect), dose for BPH is 5mg. Finasteride and
Dutasteride cannot be touched by women

278. When will the patient notice any improvement? After 6 months
Finasteride reduces the static component of bladder outlet obstruction over a period of several months to years
and may be accompanied by an improvement in urinary flow rates

279. After few months Finasteride was partially effective and doctor gave him Terazosin, what to
avoid
a) Pseudoephedrine
b) Saw palmetto
Alpha 1 adrenergic blocker (Doxazosin-Terazosin) cause first dose effect: SEVERE hypotension
(Doxazosin-Terazosin) can be used for Hypertension

280. Terazosin side effect: Sexual dysfunction


Also, cause first dose effect: SEVERE hypotension

281. Patient have migraine, doctor prescribe Sumatriptan to him. Target from using Sumatriptan.
a) Relief of nausea
b) Full resolve of pain in 2 hrs
c) Improvement is seen in 2 hrs

282. Sumatriptan S.E. Chest discomfort


Sumatriptan Nasal spray. Don’t open until ready. Don’t shake. Don’t prime (Only has one dose in it)
Instill one spray in one nostril while closing the other. Hold your head upright for 20 seconds after dose while
breathing in through your nose and out through your mouth
283. She was on Oral contraception and she has migraine is with aura, what is the DTB
a) Drug drug interaction bet Sumatriptan and OC
b) Risk of adverse effect of OC with her migraine
Oral contraceptives and propranolol may increase frovatriptan serum concentrations by 30–60%.

284. Migraine case last choice: Decrease of number of attacks per month
The ultimate goals of migraine preventive therapy are to:
• Reduce frequency, severity, and duration of attacks.
• Improve responsiveness to treatment of acute attacks
• Reduce level of disability.
• Maintain cost of care for migraine treatments.
• Reduce excessive overuse of acute medications

285. 15-month child diagnosed with meningitis. What antibiotic can reach CSF? Ceftriaxone

286. Meningitis child, All the following antibiotic pass BBB except
a) Ceftriaxone
b) Ampicillin
c) Gentamicin
d) Cloxacillin
e) Vancomycin

287. What type of gram +ve chain microorganism causing Meningitis?


a) Streptococcus pneumonia
b) L. Monocytogenes
c) Neisseria meningitis

288. Ibuprofen interacts with


a) Fluoxetine
b) Bupropion
In conclusion, our results confirm that particularly the use of SSRIs in combination with NSAIDs increases the risk of
gastrointestinal adverse effects, a 10 times higher risk than for SSRIs alone. Nonselective antidepressants do not
have this effect

289. Patient wants to know the effectiveness of Vitamin D in cancer, where reference will you search
for this information
a) Reviewed article about vitamin supplementation
b) Ask oncologist in cancer center
c) Therapeutic choices
d) Primary journal
290. Which reference to look for a new indication of vitamin D in treatment in cancer?
a) Primary article
b) Reviewed article

291. Which reference used to look for drug drug interactions?


a) Micromedex
b) DPD
c) Manufacture website

292. Patient takes medication for Parkinsonism and his doctor wants to give him a medication for
HTN, which of the following references would you check.
a) Micromedex
b) Remington
c) AHFS

293. Doctor wants to determine the solubility of certain substance. Which reference used to look for
a) Remington
b) goodman, gillman
c) TC

294. Dr asks about which SSRI give to patient for post traumatic disease, which reference to look for
a) CPS
b) Therapeutic Choice
c) Database

295. New drug A is in the market, where to find comparison between drug A and a standard drug B
a) Manufacturer website
b) Drug A monograph
c) Clinical guidelines
d) primary literature/clinical study/etc.

296. Which is the least likely to affect renal failure?


a) Atenolol
b) Pravastatin
c) Lisinopril
297. Patient with sore throat and culture is group A Beta Hemolytic and have Sulfa Allergy,
what to give?
a) Penicillin V
b) Azithromycin
c) SMX/TMP

298. Goal is to treat


a) Rheumatic fever
b) Rheumatoid arthritis
c) Febrile seizures
Rheumatic fever is a nonsuppurative, acute inflammatory complication of group A streptococcal pharyngeal
infection, causing combinations of arthritis, carditis, subcutaneous nodules, erythema marginatum, and chorea.
Diagnosis is based on applying the modified Jones criteria to information gleaned from history, examination, and
laboratory testing. Treatment includes aspirin or other nonsteroidal anti-inflammatory drugs, corticosteroids
during severe carditis, and antimicrobials to eradicate residual streptococcal infection and prevent reinfection.
A first episode of acute rheumatic fever (ARF) can occur at any age but occurs most often between 5 years and 15
years of age, which are the peak years of age for streptococcal pharyngitis. ARF is uncommon before 3 years and
after 21 years. However, preceding symptomatic pharyngitis is recognized in only about two thirds of patients with
ARF.

299. 80 years female with asymptomatic bacteriuria what to do?


a) Give antibiotic
b) Don’t give antibiotic
Asymptomatic Bacteriuria. Elderly, chronic cath., spinal cord injury: no indications for screening & no benefit in
treating; EXCEPT pre-genitourinary procedures, pre-op prosthetic, immunosuppression, pregnancy

300. Drug X 85% bioavailability in oral form. Complete serum concentration if IV dose is 500mg.
How much is oral dose to reach similar serum conc.? NO 588 in answers!!!!
a) 425
b) 500
c) 575

301. Given lipid% amino acid 7% dextrose 20% at a rate of 62.5ml/hr and dextrose 4% at rate
42ml/hr, weight 65kg
Calculate total protein/kg daily? 1.6 g/kg/day
Calculate total carbohydrate daily (1g=3.4kcal) 340.3 g 1157 k cal
302. Patient being treated for hepatitis C using Peginterferon. Why would you make sure that he will
not skip his dose?
a) Missing dose may cause drug resistance & treatment failure
b) To avoid side effect
c) Missing dose will cost the patient a lot
d) To avoid opposing interferon from being developed to obtain sustained virological response

303. What is the advantage of vertical laminar


hood over horizontal hood
a) Protection of the person working
Horizontal hood filters any air from entering the hood

304. Citalopram withdrawal symptoms


a) Flu like symptoms
b) Hallucinations

305. Patient with STEMI + 2 stents + hip fracture.


what should he take after being discharged from
hospital?
a) ASA lifetime and Clopidogrel for 1 month
b) ASA lifetime and Clopidogrel for 1 year
306. All are appropriate dosing of Methotrexate except:
a) 2.5-7.5 mg DAILY
Methotrexate causes diarrhea, anemia, and liver toxicity Methotrexate causes folic acid deficiency; give folic acid
5mg/week Methotrexate toxicity – give antidote FoliNIC acid (Leucovorin) Methotrexate Monitoring: Baseline CBC
and at every month Methotrexate does not cause cardio toxicity

307. Patient with acute back pain and hypertension and severe renal impairment and has allergy from
morphine. What should he take to control his pain?
‫ﻣن اﻷﻋﻠﻰ ﻗوة ﻟﻸﻗل‬
a) Celecoxib Fentanyl
Oxycodone
b) Naproxen Hydromorphone
Morphine
c) Codeine Codeine

d) Hydromorphone
e) Tramadol

308. Palliative care patient on


Hydromorphone not get benefit and get
pruritus. So, what to give for pain?
a) Morphine
b) Acetaminophen
c) Codeine
d) Fentanyl patch
If Oxycodone in choices will be the answer

309. Best to change the dressing to


a) IM morphine
b) Oral dose immediate release
morphine

310. Dentist prescribed lorazepam for a


patient. According to his scope of practice
which indication is related to his choice
of treatment
a) Insomnia
b) Panic attack
c) Prior surgery anxiety
d) No valid indication
Lorazepam NOT used for sleep due to rebound
anxiety Lorazepam- oLd- good for old people
Oxazepam- “O” for OLD people
311. Chlorhexidine which statement is true? May stain teeth if used for long term

312. Benzydamine which statement is false: Keep it long term in mouth before swallowing
Benzydamine oral used in cancer mouth ulcers

313. What is false about chemotherapy induced nausea and vomiting treatment?
a) Dimenhydrinate can be used
Antihistamines and Anticholinergics
The antihistamine dimenhydrinate and the antimuscarinic scopolamine are useful for treating vomiting due to
motion sickness but they are considered no more effective than placebo against CINV

314. Patient with cancer and Diverticulitis, how to monitor


a) AST
b) Endoscopy
c) INR
d) Liver
Diverticulitis is inflammation and/or infection of a diverticulum, which can result in phlegmon of the bowel
wall, peritonitis, perforation, fistula, or abscess. The primary symptom is abdominal pain. Diagnosis is by CT.
Treatment is with bowel rest, sometimes antibiotics, and occasionally surgery.
A colonic diverticulum is a saclike pouch of colonic mucosa and submucosa that protrudes through the muscular
layer of the colon; because it does not contain all layers of the bowel, it is considered a pseudodiverticulum (see
also Definition of Diverticular Disease). Many people have multiple colonic diverticula (diverticulosis). The
incidence of diverticulosis rises with increasing age; it is present in three quarters of people > 80 years.
Diagnosis
• Abdominal and pelvic CT
• Colonoscopy after resolution
Clinical suspicion is high in patients with known diverticulosis who present with characteristic abdominal
symptoms. However, because other disorders (eg, appendicitis, colon or ovarian cancer, inflammatory bowel
disease) may cause similar symptoms, testing is required.
Diverticulitis is evaluated with CT of the abdomen and pelvis with water-soluble contrast given orally and rectally;
IV contrast also is given when not contraindicated. However, findings in about 10% of patients cannot distinguish
diverticulitis from colon cancer. MRI is an alternative for pregnant and young patients.
Colonoscopy is often recommended 1 to 3 months after resolution of the episode to assess for cancer.
Treatment
• Varies with severity
• Liquid diet for mild disease; npo for more severe disease
• Sometimes antibiotics
• CT-guided percutaneous drainage of abscess
• Sometimes surgery
A patient who is not very ill is treated at home with rest and a liquid diet. Symptoms usually subside rapidly.
Patients with more severe symptoms (eg, pain, fever, marked leukocytosis) should be hospitalized, as should
patients taking prednisone (who are at higher risk of perforation and general peritonitis). Treatment is bed rest,
npo, and IV fluids.
Antibiotics
Antibiotics were traditionally recommended for all cases of acute diverticulitis whether or not they were
complicated. However, recent data suggest that antibiotics may not improve outcome in uncomplicated
diverticulitis, therefore, selected patients with acute uncomplicated diverticulitis can be managed conservatively.
(See also the American Gastroenterological Association's guidelines on management of acute diverticulitis.)
If antibiotics are used, they should cover gram-negative rods and anaerobic bacteria.
Oral antibiotic regimens that can be given to outpatients for whom treatment is elected include 7 to 10 days of
• Metronidazole (500 mg every 8 hours) plus a fluoroquinolone (eg, ciprofloxacin 500 mg every 12 hours)
• Metronidazole (500 mg every 8 hours) plus trimethoprim/sulfamethoxazole (800/160 mg every 12 hours)
• Amoxicillin (875 mg every 12 hours) plus clavulanate (125 mg every 12 hours)
• Moxifloxacin (400 mg once/day for patients unable to take penicillins or metronidazole)
IV antibiotic regimens for hospitalized patients are selected based on many factors, including the severity of illness,
risk of adverse outcome (eg, due to other illnesses, older age, immunosuppression), and likelihood of resistant
organisms. Many regimens exist.
Small pericolic abscesses up to 2 to 3 cm in diameter often resolve with broad-spectrum antibiotics and bowel rest
alone.
If response is satisfactory, the patient remains hospitalized until symptoms are relieved and a soft diet is resumed.
Percutaneous drainage or endoscopic ultrasound-guided drainage
CT-guided percutaneous or endoscopic ultrasound-guided drainage is becoming the standard of care for larger
abscesses (over 3 cm in diameter). However, abscesses that are multiloculated, inaccessible, or not improving with
drainage require surgical intervention.

315. Patient with Community acquired pneumonia and diabetic and the physician decided that she
does not need to stay in hospital during treatment. What is the DOC for her condition?
a) Levofloxacin
b) Erythromycin
c) Doxycycline
Levofloxacin is once daily dosing
and therefore has a dosing
benefit over ciprofloxacin
316. PCI immediate target: A door to dilatation time < 90 minutes
In developing the Heart Alert protocol, an interdisciplinary group of cardiologists, EM physicians, nurses, and
administrators convened to outline the actions and procedures necessary for achieving door-to-balloon times of
≤90 minutes.

317. Amikacin Most side effect: “Except hepatic”


a) Visual toxicity
b) Ototoxicity
Aminoglycosides can cause Ototoxicity and Nephrotoxicity. Signs include: Dizziness, Ringing in the ears, N/V.

318. Atrial flutter patient stabilized for 2 years and taking HCTZ for HTN has Reynaud's disease,
what to give to control heart rate
a) B. Blocker
b) Digoxin
c) Amiodarone
d) NDHP-CCB (Verapamil, diltiazem)
HCTZ & Digoxin: hypokalemia and hypomagnesemia may lead to arrhythmias.
BBs Implicated in Secondary Raynaud Phenomenon. Unlikely but controversial.

319. What the most important risk factor for stroke.


a) Age
b) Hyperlipidemia
c) Hypothyroidism
d) Gender
e) Hypertension
Vascular disease risk factors are stroke risk factors
(hypertension, smoking, diabetes mellitus, dyslipidemia,
excessive alcohol intake, body mass index, exercise and
family history of vascular disease or hemostatic disorders)

320. Patient have hypertension, MI, what is his score in CHADS2 score
a) 0
CHADS2 Score:
b) 1 0-1= ASA
2= Warfarin
c) 2
d) 3

321. A woman in menopause want to use HRT, why?


a) CVD
b) Decrease bone loss
c) UTI
322. Why is progesterone is added to her regimen
a) Decrease cervical cancer
b) Decrease breast cancer
c) Decrease endometrial cancer
Progesterone given with Estrogen to decrease the chance of endometrial cancer

323. Then she asks about what S/E of progesterone? Weight gain & edema

324. She read about a trial that Long term therapy of HRT is beneficial, what makes the pharmacist
says that trial is not reliable:
a) Sponsored by manufacturing company (conflict of interest)
b) Manufacture approval
c) It is a google article

325. Patient has PJP, HIV positive, he takes Efavirenz, Lamivudine, Tenofovir, he tells you that had
one sexual intercourse with HIV pt. and he starts to take Cotrimoxazole. So, he should monitor?
a) Hepatitis B and TSH
b) CD8, lipids
c) CD8, TSH
d) Lipid and Neutropenia

326. When his wife asks you where her husband got it
a) Tell her to speak with her husband
b) Tell her to do lab test
c) Tell her to ask his Dr
d) Tell her he got it from intercourse
1 if a type. 1 & 2 in k type. law heya gaya bt2ol enaha 3arfa w7'ayfa tt3edy yb2a 2ollaha e3mely lab test. law heya
mat3rafsh A only

327. What is your counselling to patient about this Rx?


a) Take Efavirenz with food
b) Take antiretroviral with or without food
c) Taken on an empty stomach, preferably at bedtime.
d) Do not take antiretroviral with cotrimoxazole
Efavirenz has NO effect on Thyroid

328. Sertraline counselling? Take with food to increase efficacy


Sertraline: Highest sexual dysfunction, Dry mouth, most one cause diarrhea, few drug interactions, Take with food
329. Calculation about camphor 5%, menthol 5%, basic cream 454 gm, the jar of cream is 454 gm,
the Pharmacist want use the full amount of cream jar, and how much should you add from Camphor.
Answer:
5 gm ----- 100 gm
X gm ------ 454 gm
X = 454*5/100 = 22.7 gm

330. Calculation about Vancomycin is given in a dose 500 mg q12h, after 2hrs of infusion (@11
o'clock) the conc. was 30 mg, after 7 hrs (@18 o'clock) the conc was 7 mg, what is the T ½?
a) 3.5 hour
b) 1.7 hour
t 1/2 = 0.693/ K where t1/2 = half life of drug K = rate constant
K = 2.303 x log Co where Co = initial concentration of drug
T C C = concentration of drug at time ‘t’

331. Tech. Got narcotic Rx 30 mg (straight narcotic) for 18 years old girl but he told you" we can't
dispense the Rx because this med is coming in 10mg, 15mg, what is your action?
a) Dispense it as 3 caps. Of 10 mg
b) Dispense it as 2 caps of 15 mg
c) Call the dr & correct it verbally
d) Call the dr & ask him to send over a written Rx

332. Patient started on clarithromycin which one of following will have significant interaction
a) Phenytoin
b) Pravastatin
c) Warfarin
333. A woman tells you she has long shifts and is getting varicose veins, big blue veins at the back of
her leg. She asks about compression stockings, what do you tell her?
a) Need a prescription because she needs stockings with high compression
b) They work from the hip downwards to prevent DVT
c) Take them off at night, and put them on in the morning before work

334. Which med is doing QT prolongation with Clarithromycin?


a) Metoprolol
b) Itraconazole
c) Cetirizine
Clarithromycin: shake well and take with food. Do not fridge. Interacts with sotalol cause QT prolongation
Both clarithromycin and itraconazole are substrates and inhibitors of CYP3A, leading to a bidirectional drug
interaction. Clarithromycin may increase the plasma levels of itraconazole, while itraconazole may increase the
plasma levels of clarithromycin. Patients taking itraconazole and clarithromycin concomitantly should be
monitored closely for signs or symptoms of increased or prolonged pharmacologic effect.
335. Pharmacist is going to serve a new service in his pharmacy by giving vaccines, what is the first
thing for the ph. To think about
a) Check the provincial regulations for giving shots to patient (planning stage)
b) Give the pharmacists staff first aid course
c) Give the pharmacists staff course regarding the technique of giving vaccines
d) Put a post in the nearest clinic about the new service you are going to do.

336. Patient on some straight narc. (oxycodone), controlled narc., benzo& regular meds (I think most
of them was bid but the oxycodone was SR regularly & short acting prn) so the patient is taking from
each respectively: narcotics, controlled, targeted subs.& regular med:
a) 4,0,1,1
b) 4,2,1,1
c) 3,1,0,1
d) 2,1,1,1

337. Pharmaco-economic study


a) Social
b) Government
c) Patient

338. What can break the autonomy of the doctor? Plan cut back for the first Rx day supply

339. What is DUKE (Definite Infective Endocarditis)


criteria
Two positive major tests with IE Modified Duke Criteria
Established in 1994 by the Duke Endocarditis Service and revised
in 2000, the Duke criteria are a collection of major and minor
criteria used to establish a diagnosis of endocarditis.
A definite diagnosis of infective endocarditis can be established
if the following conditions are fulfilled: Two major criteria, One
major and three minor criteria, or Five minor criteria.
340. Diabetic patient taking Lantus at night and rapid insulin 3 times before meals, BG reading low at
bedtime so
a) Stop evening exercise
b) Adjust dose (increase dose of fast acting, supper)
c) Watch diet
d) Adjust Obesity

341. Pharmacist has workload, which of the following he can delegate for help.
a) Technician take a detailed medical information (BPMH) from patient
b) Pharmacy student independently check verbal orders
c) Technician sign on receiving order with schedule G medication

342. Hospital pharmacy suffering from workload, which assignments can the pharmacy manager
delegate;
a) Pharmacy students can counsel patients independently
b) Pharmacy students can transcript verbal prescriptions left on answer machine independently
c) Technician ask patients if they need pharmacist intervention in self-medication area
d) Pharmacy technician can copy the narcotic purchase orders from invoices to computer
N.B Copy narcotic purchase orders from invoices to book log not computer

343. What is the reason for documentation in hospital pharmacy, all except?
a) For patient interest
b) Do not depend on staff cognitive function
c) For reimbursement

344. Hospital pharmacy suffering from workload what to do:


a) Decrease hours on weekend
b) Hire student and intern and pay them.
c) Decrease overlap between shifts and automate your work

345. Patient with hepatic encephalopathy declaring end of the world, he was hospitalized, took
Lactulose, which test to do?
Serum ammonia
346. Giving Lactulose for treatment, so when you say it is effective?
a) Weight decreased by 5kg
b) More consciousness
c) 2-3 bowel movements/day
Aim is to produce 2 to 3 loose bowel movements per day. Consider decreasing the dose if >2 to 3 bowel movements
per day. Discontinue if severe diarrhea.
Lactulose solution therapy reduces blood ammonia levels by 25 to 50% which is, in general, paralleled by an
improvement in the patient's mental state and by an improvement in EEG patterns.

347. After the patient was discharged from the hospital, he was worried that the prescription he took
for Lactulose has no refill, he lives in other province, so tell the patient
a) He must get new prescription
b) He can contact his doctor to add the refill on the prescription
c) He can buy Lactulose without prescription from any pharmacy
Lactulose does not need water and can be given to renal patients
Hepatic encephalopathy give lactulose then Lactulose/Metronidazole
Stop treatment when psychosis goes away

348. If we add a substance A with concentration 4% to 10% of substance B to make a mixture of


A&B, we will not exceed 50ml. This is an ophthalmic mixture and should only contain 0.1% of
substance A. It will be given to patient in plastic fills of 10 ml each. So how many grams of substance
B will be in 10ml.
Answer: 0.975g

349. All are signs of anaphylactic shock except:


Bradycardia
Symptoms of Anaphylactic Shock: Tachycardia,
Wheezing, Seizures, Hives/ urticaria, dangerously low
BP, Dizziness, sweating, fainting, Swelling of the face
and throat, Chest pain, Flushing, headache, itching
350. What to counsel the pt on CART “Combination Anti retro viral therapy therapy”
a) All cart therapy can be taken at the same time of cotrimoxazole
b) Can be taken with or without food (NO DDI between CART and Cotriamoxazole)

351. Asian university student, recently come to Canada & studying English fluency, what is the best
technique to counsel this patient;
a) Simple language & visual pictures
b) Pause technique to make sure she understands
c) Provide her with manufacturer leaflet for more written details

353. Patient asks for his medication, he has hepatitis C and lives in an area hard to reach away from
his pharmacy, he needs a refill and pharmacy do not have enough in stock and patient needs it and
cannot come and get it later and it is going to be the weekend, what should the pharmacist do?
a) Mail the medication and cover mailing expenses, and ask patient to call for his refills earlier
next time
b) Mail the medication and ask the patient to pay for expenses.
c) Ask the patient to go pick the medication from near pharmacy after checking if they have
it in stock

354. Technician decides by himself to order this patient medication and stock it so that this problem
does not happen again. They have a policy not to stock expensive medication like this one
($100/box), what is the action of the pharmacist?
a) Return the medication
b) Take disciplinary action toward the technician
c) Review policy of ordering with all staff
d) Charge the insurance of the patient early.
355. After completion of 6 months patient had no more refills, this pharmacy is in a province where
pharmacist regulation allows him to do an extra refill, what will pharmacist should do.
a) Do an extra refill for him and dispense
b) Contact doctor to ask for a refill
c) Ask patient to visit his doctor for follow up

356. Beta-blockers are contraindicated in all of the following except


a) HR < 50
b) Peripheral vascular disease
c) Diabetes
Beta-blockers are C/I in systolic BP of less than 85. Avoid beta-blockers with Raynauds (because of constriction)
Beta Blocker START LOW GO SLOW! Beta Blockers reduce mortality in Heart failure (Bispropolol + Carvediol)
Where are you gonna go if you have heart failure: BC! ‫ﺑﯾﺳﯾن‬

357. GERD patient with mild symptoms increase after pizza and taking antacids, he is hypertensive,
What to suggest for him?
a) Refer to get PPI
b) Famotidine OTC
c) Sodium alginate
d) Daily magnesium

358. He was a smoker with half a pack


every 2 days, what will benefit him
a) Decrease weight
b) Stop smoking

359. What would make you refer this


patient
a) Increased nocturnal symptoms
b) Difficulty swallowing
360. Chronic untreated Hyperuricemia may lead to
a) Osteoarthritis
b) Rheumatoid arthritis
c) Chronic tophi
The optimal therapy for chronic tophaceous gout is prevention by aggressive management of acute gout and
correction of Hyperuricemia. The aims of therapy are to control pain and inflammation, typically with NSAIDs, and
to decrease serum uric acid levels. After several years of therapy, resorption of urate deposits will eventually lead
to disappearance of tophi

361. DTP between allopurinol and azathioprine


a) Allopurinol Increase Azathioprine effect so decrease dose to 1/4
Several case reports describe increased toxicity of mercaptopurine or azathioprine (which is metabolized to
mercaptopurine) in patients receiving concomitant allopurinol therapy. Significant bone marrow suppression,
including leukopenia, pancytopenia, and thrombocytopenia have occurred. Allopurinol inhibits xanthine oxidase,
the enzyme responsible for first- pass metabolism of mercaptopurine to inactive products, and thus causes
accumulation of mercaptopurine. It has been suggested that intravenous mercaptopurine may not be affected by
allopurinol in the same fashion, however, precautions are still reasonably employed

362. Patient hypertensive and on Sertraline therapy, taking Naproxen, DTB?


a) Interaction between naproxen and SSRI (increase GIT bleeding)
b) Interacts with hypertension

363. IBS patient, what to council?


a) Diet has great impact on disease
b) IBS may cause colon cancer

364. What can be used for abdominal


pain?
a) Loperamide
b) Desipramine
c) Codeine
Tricyclic Increase GI transit time, so they may be most appropriate for use in patients with IBS-D.
Antidepressants: Potent inhibitors of the neuronal reuptake of norepinephrine and serotonin into
Imipramine 25– presynaptic nerve terminals. Also block serotonergic, α-adrenergic, histaminic, and
50 mg QHS PO muscarinic receptors.
Amitriptyline Use low doses with gradual increases for select patients with intractable pain. Numerous
Desipramine other agents are available.
25–100 mg QHS S.E: Drowsiness, dry mouth, headache. Possible CNS excitation with serotonergic agents.
PO Avoid combined use with other agents that prolong QTc.
365. Acute pancreatitis is due to high levels of?
a) TG
Very high triglyceride levels (>10 mmol/L) are a risk factor for
pancreatitis.
Caused by Methotrexate and sitagliptin

366. Appropriate council for Doxycycline.


a) Taken on empty stomach
b) Cause photosensitivity
c) Taken with full glass of water

367. Most appropriate pharmacist should say about Rifampin


Rifampin should be taken on an empty stomach (1 hour before a meal or 2 hours after a meal)
with a full glass of water.

368. Acetaminophen toxic dose


a) 325mg *12 tab
b) 500mg *8 tab
c) 500*12 tab
d) 325 * 6 tab

369. How to manage


a) Chelation
b) NAC
c) Hemodialysis

370. What to give to prevent stroke in AF patient who is not compliant with warfarin monitoring?
a) Clopidogrel
b) ASA
c) ASA/dipyridamole
d) Dabigatran
e) Prasugrel
Apixaban, dabigatran, rivaroxaban approved for prevention of systemic embolism in patients with atrial
fibrillation

371. Patient has SE of Warfarin, what to give him?


a) Rivaroxaban
b) ASA/dipyridamole
c) Dabigatran
To switch from a VKA to XARELTO, stop the VKA and determine the INR. If the INR is ≤2.5, start XARELTO at the
usual dose. If the INR is >2.5, delay the start of XARELTO until the INR is ≤2.5
Pre-Operative Phase
If an invasive procedure or surgical intervention is required, XARELTO 10 mg, 15 mg and 20 mg should be stopped
at least 24 hours before the intervention, if possible, due to increased risk of bleeding, and based on clinical
judgment of the physician. XARELTO 2.5 mg should be stopped at least 12 hours before the intervention.
Post-Procedural Period
XARELTO should be restarted following an invasive procedure or surgical intervention as soon as adequate
hemostasis has been established and the clinical situation allows, in order to avoid unnecessary increased risk of
thrombosis.

372. What should you tell the patient who is on Rivaroxaban?


a) Laboratory monitoring should be performed periodically
b) Self-monitoring for bleeding
373. Women got non gonococcal infection, what to
council
a) Abstain from intercourse or use condoms along
treatment period
Non-gonococcal urethritis caused by chlamydia.
Azithromycin and Doxycycline are first line for chlamydia

374. Allegation calculation 140 mmol/L, 110 mmol/L, 3% NaCl (0.9 % = 154 mmol).
How many of 3% added to 5 L of 110 to prepare 140? Ans: 402 ml

375. Patient took illicit injection and he is


experiencing sweating, nausea, and vomiting,
abdominal cramps, diagnosed with endocarditis?
What is the cause of MH's symptoms?
a) S. pyogenes
b) Streptococcus viridians
c) Staph aureus

376. What to give his endocarditis?


a) Cloxacillin +/- Gentamicin
b) Clindamycin
c) Ampicillin
(Cloxacillin or cefazolin) +/- Gentamicin
Pathogen Regimen Duration Comments
Antibiotic treatment regimens for staphylococcal endocarditis 40%
Staphylococci Methicillin - Cloxacillin or 6 wk May use 2 wk → if the infective endocarditis is
No Prosthetic sensitive Cefazolin uncomplicated and right-sided only.
Material Methicillin - Vancomycin or 6 wk Vancomycin trough concentrations of 10–20
resistant Daptomycin as mg/L suggested. Drug of choice for treatment of
an alternative in MRSA IE and in patients with a type-1
selected patients hypersensitivity reaction to beta-lactams
Staphylococci Methicillin - Cloxacillin or ≥ 6 wk Gentamicin peak and trough concentrations are
Prosthetic sensitive Cefazolin + First 2 wk suggested to be maintained at 3–4 mg/L and <1
Valve Rifampin + mg/L respectively
Gentamycin
Methicillin - Vancomycin + ≥ 6 wk Vancomycin trough concentrations of 10–20
resistant Rifampin + First 2 wk mg/L suggested
Gentamycin
377. He is gonna have a dental surgery a few weeks later, what to give him?
a) Amoxicillin
b) Doxycycline
Endocarditis prophylaxis before dental procedure:
Amoxicillin 2g Po.
Endocarditis is not prevented by pneumococcal vaccine
If patient is already on an antibiotic that would be used for
prophylaxis of endocarditis use a different antibiotic from
another class to avoid resistance

378. You discovered dispensing error that had happened 4 days before between lasix and losec. you
called patient and discussed everything about this dispensing error, what not right to disclose
a) Name of pharmacist involved in error.
b) Reasons that lead to that dispensing error.

379. What the patient experienced


a) Polydipsia
b) Polyuria
c) Hypoglycemia.

380. All Canadian get health services primarily done by


a) Health Canada
b) Provincial government
Done by: Provincial. Funded by Health Canada

381. Cancer pt taking very expensive medication which organization he can contact to support him?
a) Canadian cancer society

382. What to look to confirm that a certain drug is approved by HC: DIN

383. Which of the following is the objective part of SOAP?


a) Vital signs
b) Monitoring parameters

384. Which of the following represent the Assessment part of SOAP:


a) Vital signs
b) Lipid profile
c) Smoking history
d) Angina symptoms
e) Framingham risk score
385. A patient with chronic alcohol addiction, hypertension, Penicillin allergy, which one is best
antibiotic for CAP in this patient?
a) Azithromycin
b) Clarithromycin
c) Cotrimoxazole
d) Moxifloxacin
NOVEMBER 2012
1. A patient was diagnosed with metastatic breast cancer and was started on chemotherapy. In
addition, she was given dexamethasone and granisetron as a prophylaxis before the
chemotherapy. However, she still feels nauseated after her dose. Which of the following
sentences is true?
a) Granisetron is more effective than ondansetron. (Have same efficacy)
b) Prochlorperazine can be taken as prn for her breakthrough symptoms.
c) Diphenhydramine can be used as regular rather than prn for motion sickness not CINV
d) Add dimenhydrinate to the existing regimen
e) Continue on the dexamethasone only
Also, metoclopramide, haloperidol, olanzapine, lorazepam, alprazolam can be used for breakthrough CINV

2. Which class of medication is most useful in anticipatory nausea and vomiting?


a) Selective serotonin reuptake inhibitors
b) Benzodiazepines
c) Antihistamines
d) Cannabinoids

3. Patient was admitted to the hospital due to a suicidal attempt. They found out that it was due
to clomipramine and this medication was stopped during his hospital stay. Then upon his
discharge. The patient was given all his pre- admission prescriptions except the clomipramine
Afterwards the patient went to his community pharmacy and got his prescription medications
that he got from the hospital. Then when the patient went home, he continued taking his
clomipramine pills that he had at home. How could you have prevented this error?
a) Do a medication reconciliation upon discharge
b) Supply a written information with this prescription
c) Do a call back service to follow-up
d) Community pharmacist to check his medical profile when dispensing the new
prescription
e) Ask the patient

4. What is the most common side effect of Escitalopram?


a) Dry mouth (nausea, agitation, fatigue)
b) Somnolence
c) Insomnia
d) Weight gain
e) Fatigue
Escitalopram: insomnia more than somnolence. Citalopram: somnolence more than insomnia
5. A nurse called you at the hospital pharmacy and asked you about the compatibility of
nitroglycerine and heparin parenteral formulation. What is the most appropriate source to get
this information?
a) Micromedex
b) CPS
c) Remington
d) Cochrane library
e) Merck Index –contains chemical info on drugs (like pharmacopeia)
If parenteral only  Remington

6. A patient with depression and was controlled over fluoxetine. She was complaining that since
she started medication, she has trouble sleeping and has insomnia. The physician is asking
about your recommendation, as he wants to solve this problem without using sedative
medication at night. What is your best recommendation?
a) Bupropion - insomnia
b) Sertraline – another SSRI
c) Mirtazapine - sedation
d) Nortriptyline -sedation
e) Moclobemide -insomnia
General management: If adverse effects are severe, persist for longer than 2 weeks or are intolerable to the
patient, consider (1) lowering dose or (2) switching agent.

7. What is the minimum time until you see a significant improvement in her case?
a) 1 week
b) 2 weeks
c) 4 weeks
d) 3months
e) 6 months
1‐2 weeks to improve sleep & appetite & energy. 4‐6 weeks to improve mood

8. A patient has severe Raynaud’s and Angina pectoris. All of the following medications will
benefit his angina, EXCEPT:
a) Felodipine
b) Diltiazem
c) Nadolol – B-blocker causes peripheral vasoconstriction NOT used in Raynaud’s
d) Nitroglycerin
DOC of Raynaud’s disease is Nifidipine ‫ﻗﺑل اﻟﺑرد‬ ‫ﻧ‬
9. A physician prescribed potassium supplements 20mEq TID to patient but this patient does
not want to take supplements. He asks the physician if he can eat bananas instead. If each large
banana has 602 potassium. How many bananas should the patient eat each day? (Mwt K+=39)
a) 2 1meq 39 then 780/602=1.30
b) 3
c) 4 20meq X and AS TID =1.3*3= 4

d) 5 X= 780

10. A physician prescribed a fortified eye drops for a patient. Tobramycin 13.5 mg/ml Gtt i ou
twice daily. How much of the 40 mg/ml stock solution should be added to 5 ml of 0.3% to get
the desired conc.?
a) 1 ml
b) 1.5 ml
c) 2 ml
d) 2.5 ml
0.3 % means 0.3 gm ‐‐‐‐‐‐‐‐ 100 ml X gm ‐‐‐‐‐‐‐ 1 ml X=1*0.3/100=0.003 gm = 3mg So 0.3%=3mg/ml
By allegation method:

11. A patient admitted to the hospital. She has hypertension and is taking valsartan 160 mg
daily. The physician increased her dose to 160 mg BID. A day later, her potassium level was
found to be 5.9 (normal 3.4-4.5) but had no ECG changes. What is the most appropriate action?
a) Stop Valsartan temporarily
b) Change valsartan to Lisinopril
c) Decrease the dose of valsartan
d) Change to other sartan
e) Stop K+ supplements and/or drugs inducing hyperkalemia
Nonpharmacologic Choices
Stop K+ supplements and/or drugs inducing hyperkalemia.
If necessary, resume K+ supplements at a reduced dose once the hyperkalemia is resolved.
Reduce dietary K+ intake to ≤60 mmol/day.
In mild hyperkalemia (plasma K+ 5–6 mmol/L) these measures are usually sufficient.
Increasing K+ level, ongoing K+ absorption, release of intracellular K+ or renal failure require further
measures, e.g., hemodialysis.
12. The physician asks you about your recommendation for her hyperkalemia.
a) IV fluids
b) Salbutamol
c) Calcium gluconate
d) Na polystyrene sulfonate
e) Na HCO3
Many patients will be hypovolemic and will need fluid resuscitation. The initial fluid of choice is NS
Sodium bicarbonate (NaHCO3) is usually reserved for hyperkalemia associated with significant metabolic
acidosis. It has a synergistic effect with insulin in the presence of mild acidosis. In the absence of low serum
bicarbonate concentration or pH, sodium bicarbonate has a smaller effect.
To avoid an acute increase in plasma K+ induced by an osmolality change, hypertonic NaHCO3 solutions
should not be used. The correction of acidosis in hypocalcemic patients may induce tetany. Insulin
administration is faster, more reliable and more effective than sodium bicarbonate.
The beta2‐agonist salbutamol is effective in lowering plasma K+. Concurrent administration of insulin and
salbutamol have a synergistic effect. High doses of nebulized salbutamol have an effect similar to IV
salbutamol; however, up to 50% of patients with chronic renal failure are resistant to this therapy.
Mild hyperkalemia
Patients with serum potassium < 6 mEq/L (< 6 mmol/L) and no ECG abnormalities may respond to
diminished potassium intake or stopping potassium‐elevating drugs. The addition of a loop diuretic
enhances renal potassium excretion as long as volume depletion is not present.
Sodium polystyrene sulfonate in sorbitol can be given (15 to 30 g in 30 to 70 mL of 70% sorbitol orally every
4 to 6 hours). It acts as cation exchange resin and removes potassium through the gastrointestinal mucosa.
Sorbitol is administered with the resin to ensure passage through the gastrointestinal tract. Patients unable
to take drugs orally because of nausea or other reasons may be given similar doses by enema. Enemas are
not as effective at lowering potassium in patients with ileus. Enemas should not be used if acute abdomen is
suspected. About 1 mEq (1 mmol) of potassium is removed per gram of resin given. Resin therapy is slow
and often fails to lower serum potassium significantly in hypercatabolic states. Because sodium is
exchanged for potassium when sodium polystyrene sulfonate is used, sodium overload (see Hypernatremia)
may occur, particularly in oliguric patients with preexisting volume overload.
In patients with recurrent hyperkalemia, avoidance of drugs that can induce hyperkalemia (see table Factors
Contributing to Hyperkalemia) is generally all that is needed. In patients who need ACE inhibitors and
angiotensin receptor blocking agents (eg, patients with chronic heart failure or diabetic nephropathy), the
polymer resin patiromer can be taken daily to help decrease gut absorption of potassium and prevent
hyperkalemia.
CTC: Cation‐exchange resins (sodium polystyrene sulfonate, calcium polystyrene sulfonate) promote
the exchange of Na+ and Ca++ for K+, respectively, in the bowel; they also bind calcium and magnesium.
Despite their theoretical value and their widespread clinical use with apparent efficacy, the K+‐lowering
effect of single‐dose resin‐cathartic therapy is subject to debate.
The addition of resins does not seem to increase bowel K+ removal above the effect of the diarrhea induced
by the simultaneous administration of osmotic or secretory cathartics.
Because cation‐exchange resins are constipating, it is suggested that they are given with a laxative.
13. Later, the nurse admitted that she has mistakenly given the patient 320 mg BID instead of
160 mg BID, she told the medical staff. Which the first action you would ensure that it is
initially done?
a) Ensure the nurse apologizes for the patient’s family member
b) Contact the manager to ensure the nurse got a disciplinary action
c) Ensure the nurse knows how to treat hyperkalemia
d) To document that incident.
e) Transfer the nurse to another department

14. AB is an emigrant from south Asia. She is malnourished, she has pulmonary and extra
pulmonary tuberculosis, and she has fever. She is currently taking INH, rifampin, ethambutol,
pyrazinamide and pyridoxine for 9 months. Why is she more susceptible of getting neuropathy?
a) She is malnourished – risk factors for neuropathy
b) She is a female
c) She has extra pulmonary
Prescribe pyridoxine 25 mg/day (1 mg/kg/day for children) to prevent peripheral neuropathy in patients
given isoniazid who have poor nutrition, alcoholism or other substance abuse disorders, diabetes, renal
failure, HIV infection, seizure disorders or other disorders that might predispose to neuropathy. Pregnant
and breastfeeding women should also receive pyridoxine with isoniazid. Consider prescribing pyridoxine to
all patients given isoniazid

15. Which of the following statements is true regarding the therapy why is she taking 4
medications?
a) She is taking 4 medications because
she is coming from a highly
resistant area.
b) She could have taken INH and
rifampin only for 9 months
c) Because she got also extra pulmonary
d) Because of her fever
e) Malnourished
Empiric treatment with Ethambutol, Isoniazid,
Rifampin and Pyrazinamide given prior to
sensitivity results for areas with INH resistance
≥4%.
In areas with INH resistance < 4%, Isoniazid,
Rifampin and Pyrazinamide without Ethambutol
can be used pending sensitivity results.
16. A pregnant woman just had a C-section and the physician prescribed her meperidine Q4H
for her pain. One day later, she told the nurse that her legs are restless and that she had muscle
twitches. What is the reason for her symptoms?
a) The oxytocin delayed effect of the surgery
b) The delayed effect of the epidural after the surgery
c) Neurotoxicity of meperidine.
d) Long bed resting (3 days)
Lithium and local anesthetic also cause muscle twitching

17. MK is a regular client at your pharmacy. He is always having EpiPen at home. He presented
today with a new Rx for epinephrine as he ran out of his EpiPen. Two days later, he came to the
pharmacy returning the epipen as he noticed that it has only 3 more months to expire. What is
the most appropriate action for the pharmacist?
a) Return the medication, dispose it in a safe way and accept the credit loss.
b) Send it back to the manufacturer and tell him it was defective.
c) Donate it to poor people
d) Refuse to return it.
e) Let the staff check the expiry dates every 6 months.

18. What is true about injection counselling?


a) You can even inject it on top of your clothes
b) You should keep it refrigerated
c) Shake well before injection
d) Take the 1st dose if improvement occurs, don’t seek medical attention
e) If improvement occurs with the 1st dose, wait up to 6 months to use the second dose

19. Zn can be used in all of the following, EXCEPT:


a) Sore throat
b) Cold sores
c) Eczema
d) Wound healing
e) Migraine

20. What is the highest evidence-based type of study?


a) RCT
b) Systematic review of RCTs
c) Case series study
d) Case report
e) Observational study
21. Which of the following is included in the “Objective” of the SOAP assessment?
a) Smoking history
b) Medication history
c) Allergies
d) Lipid profile

22. A patient is complaining that he wakes up


early and anxious. Which medication is he using?
a) Triazolam  Only short acting so effect
isn’t sustained through the night
b) Diazepam
c) Flurazepam
d) Clobazam
e) Oxazepam

23. All are important for a patient with


Parkinson’s, EXCEPT:
a) Nutrition
b) Exercise
c) Occupational therapist
d) Counseling the Family/caregiver
e) TENS (Trans - electric…) – deep brain
stimulation is used in advanced pd3
If no tens in Answer choose Nutrition
Nonpharmacologic Choices
 Provide patient education via books, websites, and local and national Parkinson societies (all
available through Parkinson Canada).
 Stress the importance of staying active and having a regular exercise routine. To improve motor and
nonmotor symptoms, the exercise program should include aerobic, strength, balance and gait
training, e.g., dance, tai chi, yoga.
 Encourage awareness of the important roles of health professionals such as speech, physical and
occupational therapists, and home care as the disease becomes more advanced.
 Some patients may benefit from surgery; see Surgery.

24. What ONLY provides you with evidence based:


a) Cochrane library
b) Medline
c) Meta-analysis
d) Pharmacy letter
e) Inpharma
25. ISMP stands for:
a) International System for Medical Practices
b) Institute for Scientific Methods Practice
c) Institute for Safe Medication Practices

26. CDR the common Drug Review, is administratively regulated by:


a) The Canadian Pharmacist Association
b) The Canadian Agency for Drug and Technology in Health (CADTH)

27. A patient had cataract and had an operation,


which of the following oral medication will
benefit him post-operatively?
a) Dexamethasone-eye drops
b) Acetazolamide –given orally to reduce IOP
c) Ketorolac -eye drops
d) Doxycycline –not used
e) Anti-histamine eye-drops

28. A patient was given Novolin ge (30/70) BID


instead of Novolin ge NPH 10 Units BID.
What is the potential side effect?
a) Given rapid instead of intermediate
b) Postprandial hypoglycemia
c) Fasting hypoglycemia
d) Nocturnal hypoglycemia
Dawn phenomenon‐this is when u get high blood sugar in the morning. because patient is using a rapid
acting instead it doesn’t last through the night and so sugar levels r high in the morning.

29. To avoid the above mistake what should you do? (need more details)
a) Double check while dispensing
b) Computer alert
c) Make the patient check his insulin before leaving the pharmacy
d) Put different concentration of insulin in different places

30. You are counselling patient about benefits of weight loss. You should include all EXCEPT:
a) Prevent osteoarthritis
b) Prevent rheumatoid arthritis (auto immune disease)
c) Enhance psychological wellbeing
d) Prevent DM
31. A patient is on Metoprolol 100 mg BID and nitroglycerin sublingual prn. He gets 2-3 angina
attacks per month. What is the DTP?
a) He is taking the wrong medication
b) He needs a medication he’s not
taking step up treatment by
adding CCB
c) He’s taking too high dose of
Metoprolol
d) He’s taking too low dose of
Metoprolol dose is fine

32. Patient on heparin. What is the most


important to monitor?
a. Factor Xa
b. aPPT
c. PT
d. INR
Laboratory monitoring is widely recommended to measure the anticoagulant effect of unfractionated
heparin and to adjust the dose to maintain levels in the target therapeutic range. The most widely used
laboratory assay for monitoring unfractionated heparin therapy is the activated partial thromboplastin time
(aPTT).

33. Protamine sulfate is an antidote for:


a. Dalteparin
b. Enoxaparin
c. Heparin
d. Warfarin
e. Bivalirudin

34. Physician wants to switch a patient from oxycodone to fentanyl patch. What is the most
appropriate recommendation? Patch is every 3 days
a. 12.5 mcg/12 hrs
b. 12.5 mcg/36 hrs
c. 12.5 mcg/48 hrs
d. 12.g mcg/24hrs
e. 12.5 mcg/72 hrs
35. RS is an elderly patient who presented to your pharmacy complaining of constipation. She
usually has one bowel movement daily. Now she has bowel movement every 2-3 days. She also
had MI 7 years ago and hypertension. What is your best advice for her?
a. Cascara
b. Mineral oil
c. Polyethylene glycol
d. Sodium sulfate
e. Magnesium hydroxide
DOC renal & cardiac dysfunction ...PEG
The osmotic laxatives lactulose and polyethylene glycol are safe and effective for long‐term use.
Polyethylene glycol (PEG) is also safe and effective for use in geriatric patients. Whenever possible, use PEG
instead of lactulose for the treatment of chronic constipation since it results in greater improvements in
stool frequency and stool form, relieves abdominal pain, and reduces the need for additional laxatives. The
osmotic laxatives magnesium citrate, magnesium hydroxide, magnesium sulfate and sodium phosphates are
less frequently recommended.

36. Same patient, which of the following medication does NOT cause constipation?
a. Calcium
b. Ferrous sulfate
c. Magnesium hydroxide  CAUSES DIARHEA
d. Bismuth subsalicylate
e. Aluminum hydroxide

37. MB is admitted to the hospital for the second time for the last six months. He has crohn’s
disease and he was already on prednisone and azathioprine. The physician prescribed him
infliximab. Which statement is true?
a) Infliximab is used when others medication gives no effect.
Do not treat unless 4th time. Infliximab is a huge immunosuppressant, only given IV, very expensive, used
when other drugs fail to induce remission.
Biologic Response Modifiers: Anti‐tumor Necrosis Factor‐alpha Therapies
Anti‐tumor necrosis factor‐alpha (TNF‐alpha) antibodies are useful in the management of moderate to
severe IBD, including in those patients with evidence of fistulizing disease. Anti‐TNF‐alpha agents
include adalimumab, certolizumab pegol, golimumab and infliximab. While available for use in
other inflammatory conditions, certolizumab pegol is not approved in Canada for the treatment of CD.
Infliximab is administered IV; adalimumab and golimumab are administered SC and are effective in
patients with IBD who experienced treatment failure with corticosteroids, immunomodulators or
infliximab. Any of these drugs is a potential first‐line choice depending on cost, safety, route of
administration and patient’s preference.
Biologic Agents: Anti‐TNF drugs
Infliximab, certolizumab, adalimumab, and golimumab are antibodies to tumor necrosis factor
(TNF). Infliximab, certolizumab, and adalimumab are useful in Crohn disease, particularly in preventing or
retarding postoperative recurrence. Infliximab, adalimumab, and golimumab are beneficial in ulcerative
colitis for refractory or corticosteroid‐dependent disease.
Infliximab is approved for Crohn disease and ulcerative colitis and is given as a single IV infusion of 5 mg/kg
over 2 hours. It is followed by repeat infusions at weeks 2 and 6. Subsequently, it is given every 8 weeks. To
maintain remission in many if not most patients, the dose needs to be increased or the interval needs to be
shortened within a year or so. The accepted therapeutic serum level is > 5 mcg/mL

38. Currently, what’s the most important goal of


therapy for this patient
a) Prevent the disease flare ups try to induce
remission to decrease hospitalizations
b) Improve long term quality of life
c) Complete cure
d) Prevent side-effects of drug
e) Avoid surgery (Not goal of therapy of IBD)

39. What is true to tell the technician during preparation of infliximab?


a) It shouldn’t be stored in the fridge before reconstitution (it should be stored between 2
and 8 degrees)
b) It should be stored at room temperature after reconstitution for 30 days (it can be stored
for 24 hrs if the solution is stabilized if not should be used within 3 hrs cuz contains no
preservatives.)
c) If particles are found during reconstitution, a filter of 22 micron should be used. (if
particles are found, it should not be used)
d) You can give it with other medications to decrease the amount of parenteral (no physical
biochemical compatibility studies have been conducted to evaluate the co-administration
with other agents)
e) Shaking will denaturate the protein
Avoid prolonged or vigorous agitation. Do not shake. Foaming of the solution on reconstitution is not
unusual. Allow the reconstituted solution to stand for 5 minutes

40. Adult female 42 years has urinary incontinence. She said it has been very bothersome lately
and that she could not go out without worrying about if she does not find a toilet. She also
added that there is no leakage when she coughs or exercises but she has been stressed for the
past few days. Now, she is requesting a medication that will be helpful for her.
a) Desmopressin  Antidiuretic hormone analogue used in children!
b) Oxybutynin  Anticholinergics first line in urge incontinence
c) Amitriptyline
TCAs can also be used but not amitriptyline. Only Desipramine, imipramine, Nortriptyline
Oral Estrogen and SNRI Duloxetine are used in stress incontinence
41. What type of incontinence is she complaining of?
a) Urge incontinence
b) Stress incontinence
c) Mixed incontinence
d) Functional incontinence
e) Overflow incontinence
Antimuscarinics (darifenacin, fesoterodine, oxybutynin, solifenacin, tolterodine, trospium) are first‐line
treatment for urge incontinence
Estrogen does not have a place in therapy in the treatment of postmenopausal urge incontinence.
Mirabegron in patients who have failed first‐line therapies.

42. What is the causative microorganism of cellulites?


a) Streptococcus group A B-hemolytic
b) Other streptococcal
c) H-influenza
d) Enterococci

43. Cellulitis can be treated by all of the following except?


a) Cephalexin
b) Cloxacillin
c) Norfloxacin  Quinolones play little role in skin infections.
d) Clindamycin
e) Amoxicillin/clavulanate

44. What is the duration of treatment of cellulitis?


a) 5 days
b) 3 days
c) 10 days (7-10 days)
d) 21 days
e) 28 days
Antibiotics are the treatment of choice, and selection is based on the presence or absence of purulence and
other risk factors for serious and/or resistant infection. Treatments are usually not given for a fixed interval
but are continued until there is satisfactory clinical response—but typically for not less than 1 week.

45. A pharmacist is working on a Friday shift that has been very stressful. He is speaking to the
pharmacist manager expressing how hectic and busy the day was; and that there was a backlog
in the prescriptions. He is asking the pharmacist manager whether it is possible that he leaves
early today and goes back to his family for the weekend. The manager was very understanding,
and he agreed. However, so that things get harder, the physician in a nearby clinic called the
manager and said that he has 17 patients with meningitis that he wants to send to the pharmacy.
What is the most appropriate response?
a) Tell the pharmacist that you should know your legal and ethical responsibilities and
should not leave until all the work is done.
b) Get additional staff to help the pharmacist with the anticipated and expected
workload and ask the pharmacist to stay as late as possible
c) Tell the physician to send those patients to another pharmacy that is 40 km away
d) Tell the pharmacist these patients have meningitis and their treatment should not be
delayed
e) Let the pharmacist go home early as agreed and let the physician send the patients
tomorrow morning

46. Hospitals follow the federal standards and principles in order to:
a) Get federal money from the Canada Assistance Plan
b) Get their full share of the federal transfers for the hospital services

47. A patient is taking alendronate weekly every Wednesday morning. She called you on
Wednesday afternoon 5 pm, saying that she missed her pill this morning and asking what to do.
What is your most appropriate response?
a) Advise her to take the pill now and take the next pill next Wednesday the same time
b) Advise her to take the pill tomorrow morning and reschedule her medication every
Thursday
c) Advise her to take the pill tomorrow morning and continue as her schedule every
Wednesday
d) Advise her to take the pill at bed-time on empty stomach
Missed Dose: Patients should be instructed that if they miss a dose of FOSAVANCE, they should take one
tablet on the morning after they remember. They should not take two tablets on the same day but should
return to taking one tablet once a week, as originally scheduled on their chosen day.

48. Which of the following textbooks does NOT have references?


a) Merck manual
b) Remington
c) Therapeutic choices
d) CPS
49. A study was done on 500-2000 patients and showed no significant difference between new
Drug X and placebo. However, another recent study showed a statistically significant difference
between Drug X and placebo when it was investigated on 20,000 patients. What is the most
appropriate reason?
a) Type I error of the recent study
b) The small sample size of the old study
c) Type II error of the recent study
d) Different sample size

50. A hospital pharmacist is considering doing a protocol for the orientation and training of new
technicians. What is the LEAST activity to include in the protocol?
a) “Shadowing” of another technician for a whole day
b) Rotate the technicians in the local hospitals
c) Give them the human resources policy to read
d) Update medical information by attending big medical event

51. A pharmacist in a community hospital is trying to increase the teamwork between the
pharmacy staff and other health care professionals (HCPs) in the area. Which of the following
will NOT contribute to achieving this endeavor?
a) Make a mentor system to integrate new staff
b) Plan a disciplinary seminar lunch
c) Do reports about conflict resolution to HCPs.
d) Send the pharmacist more often to the patient care area

52. They want to expand the pharmacy practice so they can refill medications for chronic
conditions. The physicians in your area are skeptical about this initiative. What is the most
appropriate action to do?
a) Bring up this topic whenever a physician calls you.
b) Let the patients talk about this issue whenever they visit their doctors.
c) Send the physicians a detailed letter about this initiative.
d) Meet the physicians on their local meetings and talk about it
e) Schedule a meeting and invite the physicians in your local area

53. A patient took phenytoin 20 mg and his phenytoin blood level was 100mg/ml. doctor
increased the dose to 40 mg but then the blood level was 625 mg /ml. If phenytoin followed
first order kinetics, what would be the blood level after the second dose?
a) 100
b) 50
c) 150
d) 200
54. What is the reason for this?
a) Due to increased protein binding of phenytoin at higher doses
b) Due to saturation of creatinine clearance at higher doses
c) Due to saturation of metabolism at high doses
d) Due to saturation of renal secretion at high doses
At low doses, its first order kinetics but at higher doses its zero order. I.e. because metabolic enzymes
become saturated at high doses, only a fixed amount of drug is metabolized per unit time. Therefore, at high
doses phenytoin metabolism becomes INDEPENDENT of concentration.

55. A pharmacist owned a new pharmacy and hired a new pharmacy manager. The owner
doesn’t want to work in the dispensary but wants to keep the financial decisions for him. All of
the following tasks can be done by the manager, EXCEPT:
a) Hiring staff
b) Signing contract with the nursing home
c) Increase salaries
d) Purchasing medications
e) Inventory

56. A pharmacist refuses to counsel a Pt. who takes half the prescribed dose although he
advised him several times before, he is violating:
a) Beneficence (or loyalty)
b) Justice
c) Autonomy

57. A patient had gonorrhea. What do you recommend? obsolete


a) Cefixime best because its oral
b) Ceftriaxone
c) Azithromycin

58. The physician decided to give her ciprofloxacin and he expects a chlamydia infection as
well. What’s your recommendation? obsolete
a) Add Ceftriaxone
b) Add Azithromycin preferred treatment for chlamydia
c) Continue on ciprofloxacin

59. All of the following need dose adjustment in renal dysfunction, EXCEPT:
a) Norfloxacin
b) Ciprofloxacin
c) Moxifloxacin
d) Levofloxacin
Geriatrics: Ciprofloxacin, levofloxacin and norfloxacin doses should be adjusted according to renal
function. Moxifloxacin does not require dose adjustment based on renal function.

60. MG is an asthmatic patient taking SABA but is uncontrolled. The physician wanted to add
inhaled CS but the patient refused because he does not want to be guinea pig. Which of the
following can be taken instead?
a) Oral CS
b) Add Salmeterol
c) Add Ipratropium
d) Add Montelukast
e) Increase SABA
People who do not want to take ICS can be given LTRA as 2nd line ttt. Ideally, they should be given ICS

61. Child has asthma, and he comes and his asthma is getting worse, he`s on SABA, if you do
not recommend to go to the doctor to get an ICS, you violate what?
a) Beneficence
b) Autonomy
c) Veracity
d) Justice

62. KB came to the pharmacy with a prescription for 20 Diazepam 10 mg QD and taper over 4-
5 days as directed. He said he has been drinking alcohol for the past 10 years, went to an alcohol
withdrawal center yesterday, and quitted. Today he is experiencing insomnia, agitation and
tremulousness. What is the DTP?
a) Patient is receiving too much drug
b) Patient is receiving wrong drug for indication
c) Don’t give the drug because he is at risk of substance abuse
d) Patient not receiving enough drug or therapy (TC: 5-10 mg IVor PO q 20 min)
Best practice is to administer thiamine to all patients with alcohol withdrawal. Should be administered prior
to iv dextrose, to avoid precipitation of Wernicke's encephalopathy.
63. The patient came back later complaining that the
quantity that has been dispensed was less than that
prescribed. You found out that there was a dispensing
error that has occurred
a) Double checking the number of tablets and
sign
b) Use electronic counter
c) Let the patient sign on a receipt that he has
received the correct amount
d) Alert on the computer

64. Atrial Fibrillation case. Patient 63 years old. Patient with gout, hypertension, dyslipidemia.
What is the most likely medication to cause atrial fibrillation?
a) Salbutamol  Causes supraventricular tachycardia & AF
b) Allopurinol
65. What will put him at risk of atrial fibrillation
induced stroke?
a) Hypertension
b) Age must be over 75
c) Dyslipidemia
d) Hyperthyroidism

66. What is the drug of choice for atrial


fibrillation?
a) Metoprolol  For rate control: b blocker.
b) Digoxin inadequate for rate control
c) Amiodarone not first line because of toxicity
d) Warfarin used for anticoagulation with CHADS2 score of >1
Start with a beta‐blocker or a calcium
channel blocker (Diltiazem or verapamil).
Digoxin is usually inadequate alone for rate
control. Because of its potential toxicity,
Amiodarone should be used only as a last
resort.
Dronedarone is without iodine‐related
organ toxicity. Health Canada recommends
that Dronedarone not be prescribed for
patients with heart failure, left ventricular
dysfunction or permanent atrial fibrillation.

67. What should be given to the


above patient to prevent stroke?
a) Antiplatelet
b) Anticoagulant from what we
know his chads2 score is 1.
c) Antiplatelet & anticoagulant
Anticoagulant used to prevent of stroke of
cardiogenic origin
Use ASA, Clopidogrel, or ASA +
dipyredamol Non‐cardiogenic origin.
CHADS2 Score: 0‐1= ASA, 2= Warfarin
68. Patient has Atrial fibrillation (Supra Ventricular) and his VVR is about 160. He also has BP
88/55, The dr. he is taking Docusate sodium for his constipation & everything else is good with
his health practitioner wants to treat his ventricular arrhythmia, all can be used except?
a) Amiodarone
b) Digoxin (contraindicated in ventricular arrhythmia)
c) BB
If the BP is low (won't be 88/55 mm Hg), then you would avoid arrhythmia meds interfering with BP (BB or
NDHP‐CCBs). It's that simple, especially in the old time (but not any more).
Contraindications
• Patients known to be hypersensitive to digoxin, other digitalis glycosides or any of the excipients.
• Patients with arrhythmias caused by cardiac glycoside intoxication.
• Patients with hypertrophic obstructive cardiomyopathy, unless there is concomitant atrial fibrillation and
heart failure, but even then, caution should be exercised if digoxin is to be used.
• Patients with supraventricular arrhythmias associated with an accessory atrioventricular pathway, as in
the Wolff‐Parkinson‐White syndrome, unless the electrophysiological characteristics of the accessory
pathway and any possible deleterious effect of digoxin on these characteristics have been evaluated. If an
accessory pathway is known or suspected to be present and there is no history of previous supraventricular
arrhythmias, digoxin is similarly contraindicated.
• Patients with intermittent complete heart block or second‐degree atrioventricular block, especially if there
is a history of Stokes‐Adams attacks.
• Patients with ventricular tachycardia or ventricular fibrillation.

69. If you want to control his VVR, we could give him


a) Nifedepine
b) Metoprolol
c) Acebutalol
d) Verapamil
Let's eliminate what won't apply.
Nifedipine: won't work efficiently on ventricular rate.
Verapamil is NOT a good option since there's already constipation.
Now, we've 2 BBs and the difference is that one has an ISA (acebutolol) and the other doesn't.
Although acebutolol can be used, the preferred agents are metoprolol and bisoprolol.

70. Camphor and menthol are being mixed with a base “cream”. What kind of mixture does
camphor and menthol make?
a) Eutectic
b) Geometric
c) Colloidal
Something else that starts with a “C”. Something else that starts with an “A”. Menthol and camphor, both
solids at room temperature, form a eutectic that is a liquid at room temperature.
71. A patient had two partial complex seizures in that past 6 months, but has not tried anything
for his seizures yet. What do you give him?
a) Start Carbamazepine and lamotrigine
b) Start Phenytoin
c) Start Phenobarbital
d) Wait for a third attack before initiating pharmacological treatment
e) Start gabapentin
Gabapentin is approved as adjunctive therapy for partial seizures.

72. Antiepileptic with highest hepatotoxicity (VPA>CBZ>phenytoin)


a) Valproic
b) Gabapentin
c) Lamotrigine
d) Topiramate
e) Carbamazepine

73. What is true about Gabapentin?


a) Gabapentin decreases oral contraceptive efficacy
b) Gabapentin requires dosage adjustment in renal dysfunction
c) Gabapentin is a Na channel blocker
Gabapentin requires dosage adjustment in hepatic dysfunction not metabolized at all so it’s safe in liver
failure

74. Which one of the following medications causes weight loss?


a) Topiramate
b) Carbamazepine
c) Phenytoin
d) Lamotrigine
e) Gabapentin

75. An 85-year-old obese patient had osteoarthritis for the past 2 years. She has been taking
acetaminophen and Ibuprofen but gave no effect. Which risk factor would prompt you to give
her cytoprotection?
a) Age  older patients at higher risk of GI bleed and she is taking NSAID
b) Obesity
c) Gender
d) Insomnia
e) Had osteoarthritis for 2 years
76. All can be used in sunburns except
a) NSAIDS (can use ibuprofen, acetaminophen for pain)
b) Lidocaine (risk of allergic contact dermatitis)
c) Topical corticosteroids (can be used topically because they decrease erythema)
d) Cold compresses
e) Oatmeal (colloidal oatmeal baths may provide symptomatic relief)
f) Oral antihistamine (can use oral diphenhydramine for itch control)
Topical anesthetic sprays associated with increased sensitization and should be avoided
Treatment
 Cold compresses
 Nonsteroidal anti‐inflammatory drugs (NSAIDs)
Further exposure should be avoided until sunburn has completely subsided. Cold tap‐water compresses and
oral NSAIDs help relieve symptoms, as may topical treatments (eg, aloe vera, other water‐based lotions).
Petrolatum‐based products such as petroleum jelly should be avoided in severe sunburns. Topical
corticosteroids are no more effective than cool compresses. Blistered areas should be managed similarly to
other partial‐thickness burns (see Initial wound care), with sterile dressings and silver sulfadiazine.
Ointments or lotions containing local anesthetics (eg, benzocaine) or diphenhydramine typically should be
avoided because of the risk of allergic contact dermatitis.
Early treatment of extensive, severe sunburn with a systemic corticosteroid (eg, oral prednisone 20 to 30 mg
2 times a day for 4 days for adults or adolescents) may decrease the discomfort, but this use is controversial.

77. All the statements are true when counseling about sunburn, EXCEPT:
a) Wear long sleeves
b) Avoid playing outdoors between 10 am and 4 pm (true because the sun is more harmful
in this period)
c) Use a sunscreen with mainly for UVB protection
d) Wide brimmed hat
The Canadian dermatology association recommends that sunscreens should offer broad spectrum UVA
protection, be minimally or nonperfumed, have an SPF of at least 30)
UVB is the primary cause of sunburn from sunlight. It does not penetrate glass. It can also cause
immunosuppression and skin cancer. Prefer UVA and UVB together
78. A patient went out in the sun and got sunburn much
more quickly than normal. She is wondering if one of
her medications may have caused this. Which of these
medications caused her to become sensitive to the sun?
a) Isotretinoin
b) Oral contraceptive
c) Nitrofurantoin

79. Upon initiating Isotretinoin, a baseline measurement should be taken for which of the
following medications?
a) Liver (need baseline CBC, AST, ALT, fasting lipid profile, pregnancy)
b) Glucose
c) Electrolytes
d) TG
e) Serum creatinine

80. Which of the following should be monitored on the next doctor visit?
a) Liver
b) CBC
c) TG
d) Glucose

81. Which of the following should be used with isotretinoin? Lip balm
Oral isotretinoin is the best treatment for patients with moderate acne in whom antibiotics are unsuccessful
and for those with severe inflammatory acne. Dosage of isotretinoin is usually 1 mg/kg once/day for 16 to
20 wk, but the dosage may be increased to 2 mg/kg once/day. If adverse effects make this dosage
intolerable, it may be reduced to 0.5 mg/kg once/day. After therapy, acne may continue to improve.
Most patients do not require a 2nd course of treatment; when needed, it is resumed only after the drug has
been stopped for 4 mo, except in severe cases when it may be resumed earlier. Retreatment is required
more often if the initial dosage is low (0.5 mg/kg). With this dosage (which is very popular in Europe), fewer
adverse effects occur, but prolonged therapy is usually required. Cumulative dosing has gained support; a
total dosage of 120 to 150 mg/kg resulted in lower recurrence rates, and some experts suggest a higher
cumulative dose of 220 mg/kg.
Isotretinoin is nearly always effective, but use is limited by adverse effects, including dryness of conjunctivae
and mucosae of the genitals, chapped lips, arthralgias, depression, elevated lipid levels, and the risk of birth
defects if treatment occurs during pregnancy. Hydration with water followed by petrolatum application
usually alleviates mucosal and cutaneous dryness. Arthralgias (mostly of large joints or the lower back)
occur in about 15% of patients. Increased risk of depression and suicide is much publicized but probably rare.
It is not clear whether risk of new or worsened inflammatory bowel disease (Crohn disease and ulcerative
colitis) is increased.
CBC, liver function, triglyceride, and cholesterol levels should be determined before treatment. Each should
be reassessed at 4 wk and, unless abnormalities are noted, need not be repeated until the end of treatment.
Triglycerides rarely increase to a level at which the drug should be stopped. Liver function is seldom
affected. Because isotretinoin is teratogenic, women of childbearing age are told that they are required to
use 2 methods of contraception for 1 mo before treatment, during treatment, and for at least 1 mo after
stopping treatment. Pregnancy tests should be done before beginning therapy and monthly until 1 mo after
therapy stops.

82. An 83 years old female patient broke her leg and she have a history of falling several times.
She lives alone in the house and has several bruises. Apart from sending her to the physician,
whom else should you refer her to
a) Social worker
b) Occupational therapist
c) Podiatrist
d) Physiotherapist
Occupational therapy (OT) focuses on self‐care activities and improvement of fine motor coordination of
muscles and joints, particularly in the upper extremities. Unlike physical therapy, which focuses on muscle
strength and joint range of motion, OT focuses on activities of daily living (ADLs) because they are the
cornerstone of independent living.
Basic ADLs (BADLs) include eating, dressing, bathing, grooming, toileting, and transferring (ie, moving
between surfaces such as the bed, chair, and bathtub or shower).
Instrumental ADLs (IADLs) require more complex cognitive functioning than BADLs. IADLs include preparing
meals; communicating by telephone, writing, or computer; managing finances and daily drug regimens;
cleaning; doing laundry, food shopping, and other errands; traveling as a pedestrian or by public
transportation; and driving. Driving is particularly complex, requiring integration of visual, physical, and
cognitive tasks.
Interventions
OT may consist of one consultation or frequent sessions of varying intensity. Sessions may occur in various
settings:
 Acute care, rehabilitation, outpatient, adult day care, skilled nursing, or long‐term care facilities
 The home (as part of home health care)
 Senior housing developments
 Life‐care or assisted‐living communities
Occupational therapists develop an individualized program to enhance patients’ motor, cognitive,
communication, and interaction capabilities. The goal is not only to help patients do ADLs but also to do
appropriate preferred leisure activities and to foster and maintain social integration and participation.
Before developing a program, a therapist observes patients doing each activity of the daily routine to learn
what is needed to ensure safe, successful completion of the activities. Therapists can then recommend ways
to eliminate or reduce maladaptive patterns and to establish routines that promote function and health.
Specific performance‐oriented exercises are also recommended. Therapists emphasize that exercises must
be practiced and motivate patients to do so by focusing on exercise as a means of becoming more active at
home and in the community.
Patients are taught creative ways to facilitate social activities (eg, how to get to museums or church without
driving, how to use hearing aids or other assistive communication devices in different settings, how to travel
safely with or without a cane or walker). Therapists may suggest new activities (eg, volunteering in foster
grandparent programs, schools, or hospitals).
Patients are taught strategies to compensate for their limitations (eg, to sit when gardening). The therapist
may identify various assistive devices that can help patients do many activities of daily living (see
Table: Assistive Devices). Most occupational therapists can select wheelchairs appropriate for patients’
needs and provide training for upper‐extremity amputees. Occupational therapists may construct and fit
devices to prevent contractures and treat other functional disorders.
Physical therapy aims to improve joint and muscle function (eg, range of motion, strength) and thus improve
the patient’s ability to stand, balance, walk, and climb stairs. For example, physical therapy is usually used
to train lower‐extremity amputees. On the other hand, occupational therapy focuses on self‐care activities
and improvement of fine motor coordination of muscles and joints, particularly in the upper extremities.
THE WORK OF A SOCIAL WORKER
Duties
Social workers’ tasks can be divided into four main groups: counselling, investigation and expert testimony,
program development and family mediation.
Counselling
Social workers help families living through difficult situations (poverty, spousal abuse, drug or alcohol
abuse, delinquency, etc.). The goal of a social worker is to help clients develop the skills they need to be able
to solve problems on their own.
Social workers often have a preference for specific areas of social work, which is why many specialize in
working with a particular group of clients. For example:
 Abused women
 Victims of crime
 Neglected children
 Drug addicts
 Seniors
Investigation and expert testimony
Many social workers, like those who work with neglected children, may be required to investigate
allegations of abuse or to do psychosocial evaluations of families. They can be required to testify as an
expert witness in hearings; as a result, they must always be very careful when documenting their
observations. They must understand the issues at play in the cases they investigate and remain objective,
even in the most difficult of situations they may encounter.

83. A patient with “osteoarthritis” and likes to swim 3times/week. She works as a hairdresser,
which is 2 km away from home. She smokes half a pack of cigarettes per day. She fell once
when she was young (7 years old) and broke her arm. She takes 1 cup of alcohol on weekends.
What is her risk factor for developing “osteoporosis”? (she had both conditions)
a) Smoking  big risk factor
b) Alcohol  her alcohol intake is moderate so it’s not a concern
c) Fracture history  only a risk factor if it is a fragility fracture. This was an injury
Osteoarthritis rheumatoid arthritis is a risk factor for osteoporosis
84. What advise should you give her?
a) Walk to work
b) Reduce your alcohol intake
c) Swim more often she already swims 3
days a week.
d) Change her work

85. A patient has asthma and takes Salbutamol


and Fluticasone. She is coming early for her
salbutamol. Her profile includes:
Salbutamol 2 puffs before exercise … Filled 10, 20, 50, and 70 days ago.
Fluticasone 2 puffs BID … Filled 10, 20, 50, and 70 days ago. (Dose can increase to 4 puffs
BID if there’s a cold). What is a sign that her asthma is uncontrolled?
a) Using salbutamol daily 15min before exercise
b) Waking up at night twice with a cough
c) High dose of fluticasone
d) Peak flow meter FEV > 90%

86. What recommendation would you give to her doctor?


a) Add Salmeterol since she is already taking ICS this is appropriate.
b) Add ipratropium
c) Oral prednisone
d) Add terbutaline

87. A patient has asthma and is taking Formeterol/corticosteroid regular and prn. She now has
bronchitis and the doctor prescribed ipratropium nasal spray and told her to inhale QID. What
should prompt you to contact the doctor?
a) She needs antibiotics right away
b) This is an inappropriate formulation  should be inhaler

88. A patient is taking oxycodone 30mg BID. The pharmacist was calling her to do a follow-up
and, on the phone, she said last week she decided to take 60 mg BID and since then her pain
control has been better and she has not experienced any side effects. What is the most
appropriate action for the pharmacist?
a) Tell her that she will not be allowed to get any more opioids.
b) Offer to call her doctor to tell him about her dose adjustment and get back to her.
c) Advise her about the side effects that come from high dosing
d) She is exceeding the maximum dose of this medication
e) As long as the pain is controlled it is safe to increase the dose
89. A patient has shingles and she was admitted to the hospital due to her severe and acute pain.
Her pain is now controlled. You delivered the medication to her as prescribed Amitriptyline 10
mg QHS. When she went home and read the leaflet, she decided not to take the medication due
to its side effects. What contributed to this problem?
a) No dialogue between the pharmacist and the patient
b) The physician did not give the patient sufficient information about the medication.
c) The leaflet contains detailed information

90. What should the pharmacist tell her?


a) The pain relief provided by this medication will outweigh any side effects.
b) The common side effects usually tolerated by most patients and can be managed.
c) This is a very low dose to cause these side effects
d) If you experience any side effects go right away to the doctor
e) Serious side effects do not happen to everyone
Low doses of TCAs used in neuropathic pain very well tolerated unlikely to give side effects.

91. A patient’s father comes in asking about his daughter’s medications (Oral contraceptives)
what two ethical principles the pharmacist is stuck between.
a) Veracity and autonomy
b) Autonomy and non-maleficence
Being truthful to father without impeding the daughters right to self‐determination (in this case, privacy)

92. A patient was treated for depression and took antidepressant then she went into mania. Her
doctor diagnosed her with bipolar disorder. What should the doctor switch her to?
a) Switch to lamotrigine
b) Switch to carbamazepine
c) Stop antidepressant and start lithium
d) Continue antidepressant and lithium together
Manic Episodes
Moderate to severe mania is often treated in hospital; mild mania, which by psychiatric definition is distinct
from hypomania, may be treated on an outpatient basis. The 1st step in treating mania is to assess for risk
of aggressive behaviour or violence to others, suicide, degree of insight and the ability to adhere to
treatment. If the patient is taking an antidepressant, it should be immediately discontinued. Complicating
conditions, particularly substance use disorders, will need attention.
Specific medication strategies for mania depend on whether the patient is already on maintenance therapy
and is experiencing a breakthrough episode or whether the individual is unmedicated. If the patient is
already taking a first‐line agent such as lithium, divalproex or a second‐generation (“atypical”)
antipsychotic, dosage adjustment may be sufficient after checking blood levels where appropriate. In
moderate‐to‐severe manic episodes, addition of another medication is usually necessary.
93. Manager wants to increase profitability in a new pharmacy. What is the first thing to do?
a) Increase opening hours
b) Increase local Advertising
c) Decrease staff
d) Delegate some of the pharmacy work to technicians

94. A depressed patient has recurrent and severe depression. It was his third time within this
year and he has been on his antidepressant for 9 months. How long should this patient be on an
antidepressant?
a) 1 year
b) 6 months
c) 1 more month
d) Indefinitely
After 1 episode, treat for 1 year and after 2 or
more episodes, treat for at least 2 years. This guy
had 3 so it should be at least 2 years which isn’t
an option here.
1‐ 1st episodes, continue for ~ 6 months 2‐ 4‐9
months after response
3‐ Pts with a history of 2+ episodes, treatment
longer (years –lifelong)
95. A patient has cholestatic jaundice with no hepatocellular damage. What levels should you
check?
a) Albumin
b) ALT
c) CK
d) ALP  usually raised in cholestatic jaundice
e) LDH
Clinical/biochemical evidence of cholestasis: 1‐ Elevated alkaline phosphatase, Gamma glutamyl transferase
[GGT], Later bilirubin

96. What is aim of demographic analysis in clinical studies?


a) Comparison between the population and the results
b) To help in Statistical analysis
c) Improve sales
d) To get powerful data
e) To be sure that both groups get same treatment
Definition of Demographics: Studies of a population based on factors such as age, race, sex, economic
status, level of education, income level and employment, among others. Demographics are used by
governments, corporations and non‐government organizations to learn more about a population's
characteristics for many purposes, including policy development and economic market research.

97. Patient culture affect his perception about treatment, what should the pharmacy manager do
to help his business regarding this aspect?
a) Assess the staff’s knowledge of different languages
b) Look at the local population culture
c) Assess the clientele that enter the pharmacy
d) Assess the impact of culture on Health and med (look into culture of demographics)
e) Educate the staff

98. A 4 months baby was just weaned and was started on the formula, his normal bowel
movement was 3 per day, now he doesn’t have any bowel movement in the last 36 hours, the
baby shows no symptoms, what should you do?
a) Refer to doctor
b) Wait a little bit longer
c) Give prune juice
d) Give infant mineral oil
e) Give infant glycerin suppository
99. What is the concern of the following Rx, Ferrous sulfate iii hs, Sildenafil 25, Terazosin,
Omeprazole?
a) Terazosin and sildenafil severe hypotension
b) Sildenafil and hypertension
SILDENAFIL increases effects of TERAZOSIN by pharmacodynamic synergism. Risk of hypotension. Possible
serious or life‐ threatening interaction.
OMEPRAZOLE will decrease the level or effect of FERROUS SULFATE by increasing gastric pH.

100. A patient was diagnosed with H. Pylori


and the dr. started him on eradication therapy
for 14 days. Lansoprazole 30 mg QD,
Clarithromycin 500 mg BID, Amoxicillin 1000
mg BID. What is the DTP?
a) Clarithromycin is too low
b) 14 days therapy is too long
c) Lansoprazole is sub-optimal dose
All PPIs are taken on a BID bases as a part of H. pylori
eradication

101. A researcher hypothesized that black cohosh is beneficial in menopause. He divided the
people into two groups, a group who takes black cohosh and a group who do not take black
cohosh and he followed them over time. The study was conducted from 1985 to 2002. Which
study design best describes this study?
a) Cohort
b) Case control
c) Case report
d) Case series
e) Double blind RCT

102. A trial for a new drug was conducted vs placebo. The new drug was tried on 5237 patients
and a placebo on 4993 patients. From the new drug 28 got the side effects and 7 from the
placebo. What is the odd ratio?
a) 2
b) 4
c) 5
Drug Placebo Total 5265 5000
Affected A 28 B7
Not‐Affected C 5237 D 4493
Odd ratio = AD/BC = 28*4493 / 7*5237 = 3.8
103. Which drug causes dizziness?
a) Nitrofurantoin
b) Omeprazole
c) Amoxicillin/clavulanate
d) Sodium docusate
Nitrofurantoin monograph: Central Nervous System  Dizziness, vertigo, asthenia, drowsiness, reversible
intracranial hypertension and cerebellar dysfunction have occurred.

104. Several errors were reported from the patient care area for KCl injection, as a pharmacy
manager what should you do?
a) Pharmacist prepare the infusion
b) Lock it in a cupboard and keep the key with one nurse in the patient care area
c) Put the label on the bin where you store the injections
d) Remove it completely from the ward and keep it in the pharmacy

105. Timolol eye drops counseling.


a) Discard once opened after 1 month
b) Once you open it you need to refrigerate
c) Shake before use
Invert the closed container and shake once before each use. It is not necessary to shake the container more
than once. Store at room temperature (15‐25°C). Protect from light and freezing. The contents of should not
be used for more than one month after the date on which the container is first opened.

106. A patient that believes in the biomedical health model, what is true?
a) Treatment occurs by Restoring the overall balance
b) Treatment occurs by Removing the organ affected
c) Treatment occurs by using traditional remedies rather than pharmacotherapy
d) Treatment occurs through a person who has healing powers, and shock
Removing affected organ or administering a pharmacotherapeutic medication.
The biomedical model of health focuses on purely biological factors and excludes psychological,
environmental, and social influences. It is considered to be the leading modern way for health care
professionals to diagnose and treat a condition in most Western countries.

107. Ferrous gluconate 300 mg TID given, what is the elemental iron he will get daily?
a) 105
b) 300
c) 900
Ferrous gluconate contains 35mg of elemental iron in 300mg (11.6%). Ferrous sulfate contains 60mg of
elemental iron in 300mg (20%). Ferrous sulfate did contain 90mg in 300mg (30%).
Ferrous Fumarate contains 99mg of elemental iron in 300mg (33%).
108. How to monitor Iron store in the body?
a) Ferritin
b) Hemoglobin
c) Hematocrit
A ferritin blood test checks the amount of ferritin in the blood. Ferritin is a protein in the body that binds to
iron; most of the iron stored in the body is bound to ferritin. The amount of ferritin found in the blood is the
same amount that is in the body.

109. Patient taking clonazepam, and she is travelling to Florida and she is asking if she can have
a 4 months’ supply, what is the most appropriate response from the pharmacist?
a) Transfer the prescription to a local pharmacy in Florida
b) Give her the 4 months’ supply and bill the insurance monthly
c) Tell her that the amount is restricted by the provincial regulatory authorities
d) Ask her physician to increase the daily dose to cover her trip
e) Mail her medications to Florida
As per provincial insurance plans (NOT regulatory authorities).

110. Which of the following could cause tolerance?


a) Clobazam
b) Amitriptyline
c) Valproic acid
Tolerance is a big issue with benzodiazepines: Clobazam is a benzodiazepine that is used in combination
with other medications to treat seizures caused by Lennox‐Gastaut syndrome, a severe form of childhood
epilepsy that also causes developmental and behavior problems. Clobazam may also be used for purposes
not listed in this medication guide.
Tolerance and physical dependence. Tolerance develops rapidly to the sleep‐inducing effects
of benzodiazepines. The anticonvulsant and muscle‐relaxant effects last for a few weeks before tolerance
develops in most individuals.

111. Verbal narcotic legal requirement, except?


a) Cannot be transferred
b) Keep in sales record not a legal requirement
c) Can be part-filled
d) Sales record done only if you sell it to another pharmacy

112. Fentanyl patch counseling, all except?


a) Can be applied to arm on the deltoid
b) Do not apply on broken skin
c) Fold in away the 2 adhesive halves facing each other inward and flush in the toilet.
d) First aid tape may be used for the non-adhesive endings of the patch
e) It should be stored in the fridge
f) Rotate site of application routinely
Store in the original pouch, in order to protect from light. This medicinal product does not require any
special temperature storage conditions.

113. Which of the following causes photosensitivity?


a) Isotretinoin
b) Nitrofurantoin
c) Medroxyprogesterone

114. Side effects of Isotretinoin?


a) Conjunctivitis
b) Ankle edema
Monitor: GPT KLP, BGL, pregnancy, TG, kidney, lipid,
photosensitivity

115. All of the following are side effects for


Valproic acid, EXCEPT:
a) Ataxia
b) Sedation
c) Mood changes
d) GIT side effects
e) Peripheral edema
Valproic is Anticonvulsant and mood‐stabilizing
drug. (never causes mood changes)

116. Patient on 40 mg atorvastatin, 500 mg clarithromycin new prescription for 10 days, what
should he do?
a) Dispense the prescription
b) Stop the statin temporarily for 2 weeks
c) Change to levofloxacin
d) Reduce the dose of clarithromycin to 250
But if we decrease the dose of atorvastatin that would be better
Note that pravastatin is the only one could be safely used in such case as it has no DDI
CPS "Lipitor" Caution should be used when co‐prescribing atorvastatin and appropriate clinical assessment
is recommended to ensure that the lowest dose necessary of atorvastatin is employed
117. The best test to calculate the renal function dosing.
a) Serum creatinine
b) Glomerular filtration rate
c) Creatinine kinase
d) Albumin to creatinine ratio
e) Alanine

118. A child is on amoxicillin 2mL once daily and his baby sitter accidentally gave him 10mL.
The child does not appear in any discomfort or pain. What should they do?
a) Take him to emergency
b) Observe the child for any diarrhea for 48 hours
c) Give him activated charcoal
Activated charcoal is recommended only for very large recent ingestions. For amoxicillin ingestions of less
than 250 mg/kg, treatment is not usually required in patients with normal renal function. Monitor fluid and
electrolyte status and renal function in patients with severe vomiting and/or diarrhea. Hemodialysis may be
useful following severe overdose with renal impairment.

119. Patient is on Metoprolol 50mg


bid and he is here early. When you
ask him, he tells you that the doctor
increased the dose to 100mg bid.
What should you do?
a) Call doctor to verify dose
b) Refuse to dispense
c) Give him the new dose.
Call Dr. and confirm (professionalism
and safety rather than veracity‐related)

120. Patient suffering from alcohol dependence & decided to stop. Two days ago, she went to
the doctor, he wrote her BDZ. When she went to the pharmacy to get the medication, she found
that the cashier was her neighbor. The patient was embarrassed and when you talk to her you
found that she didn’t want her neighbor to know anything about her. What should the you do?
a) Tell him alcohol dependence is nothing to be embarrassed of
b) Ask him to go to private counseling area and ensure that information is confidential
c) Tell the cashier discuss with him since he is his neighbor
d) Tell him the cashier will not disclose any information
e) Tell him he should disclose his information for his sake
121. In order to weigh a compound that that has 5% accuracy on a balance that has sensitivity
error of 4.5 mg. What is the maximum weighable amount?
a) 9mg
b) 45mg
c) 90mg
d) 180mg

122. MG is an obese diabetic patient on metformin 1000 mg bid and Gliburide 7.5mg BID.
HbA1c is above 8.5%. The Dr. wants to re-evaluate her HbA1C after 3 months and is asking
you about the best recommendation regarding the therapy?
a) Increase the sulfonylurea
b) Increase the biguanide (little additional benefit above 1500mg/day max is 2500mg)
c) Add meglitinide
d) Add thiazolidinedione
e) Add Sitagliptin
Sitagliptin is indicated in combination with a sulfonylurea (i.e., triple combination therapy) as an adjunct to
diet and exercise to improve glycemic control in adult patients with type 2 diabetes mellitus inadequately
controlled on metformin and a sulfonylurea.
Gliclazide dose: 40‐320mg/day po give in 2 divided doses if daily dose is greater than or equal to 160mg.
Glyburide: 20 mg/day

123. Three months later, the dr. realized that her HbA1C is still getting higher and it was 9-
11%. The Dr. is considering initiating insulin therapy with her oral medications. What is the
most appropriate insulin therapy for MG?
a) Insulin NPH in the evening
b) Insulin glargine at bedtime
c) Insulin (30 regular/70 NPH) in the morning and at bedtime
d) Insulin NPH in the morning and regular insulin after each meal
e) Insulin R before each meal
Insulin added to existing oral therapy when targets are not met is daily bedtime injection of basal insulin
either intermediate NPH or long acting insulin glargine or detemir

124. MG came later and she was complaining of shortness of breath, ankle edema and edema all
over body, which of the following medication – if she was given it- would have caused the
following symptoms?
a) Acarbose (flatulence, diarrhea, abdominal pain, cramps, nausea)
b) Pioglitazone (weight gain, fluid retention and hemodilution) CI in cardio problems
c) Metformin (Diarrhea, nausea, metallic taste, anorexia)
d) Gliclazide (hypoglycemia, weight gain)
e) Meglitinide (hypoglycemia)
125. Which medication should be stopped before IV contrast media?
a) Metformin to avoid an acute renal failure which may cause lactic acidosis
b) Acarbose
c) Pioglitazone
d) Gliclazide
e) meglitinide

126. AB presented to you at the pharmacy and said that he had high cholesterol level. He also
mentioned that the doctor was considering starting him on statin but AB was not convinced that
the benefits outweigh the risks of this medication. After a meaningful informative exchange
between you and the patient about the importance of statin therapy, he is still insisting on his
opinion. What is the best pharmacist response?
a) Explain to AB the side effects of statin therapy and the risk of untreated cholesterol
b) Wait until AB comes back with the RX later and then talk to him.
c) Let AB understand that his physician and pharmacist have better judgment on his health
and he should follow their advice.
d) Respect his point of view and his decision
e) Inform the patient that no problem this time as anyway he will be on statin anyway soon

127. A Lady came to your pharmacy and asked you about an evidence-based proved non-
prescription medication for premenstrual syndrome. What is your most appropriate
recommendation?
a) Zinc
b) Calcium
c) Black cohosh
d) Primrose oil
e) NSAID first line for dysmenorrhea

128. All of the following can be used to treat postmenpausal symptoms EXCEPT
a) Estrogen
b) Venlafaxine
c) Gabapentin
d) Raloxifen
Raloxifen in osteoporosis after menopause
Gabapentin 900 mg daily reduced hot flash frequency and severity
129. Female with asthma, has premenstrual cramps and takes ibuprofen and Tylenol no.1
regularly. She complains of increased asthma attacks recently. What can be contributing to this?
a) Caffeine
b) Acetaminophen
c) Codeine
d) Ibuprofen

130. Patient comes in to refill their prescription for 90 tablets of Atenolol 50mg daily. You
check their profile and find that their last refill was 60 days ago. The patient tells you the doctor
told him to increase the dose to 50mg bid. What do you do?
a) Call the doctor to verify if the dose has changed
b) Give him 7-day supply and ask him to increase dose
c) Fill the Rx for the same dose it is processed
d) Tell the patient to go get a new prescription

131. An 85-year-old patient complained about taking him one hour to fall asleep. He wanted a
medication to help him initiate sleep without hangover in the morning. After discussing with his
physician, what is your best recommendation?
a) Oxazepam (is indicated for insomnia and anxiety in the CPS)
b) Flurazepam (long acting BZD so has hangover effect also not recommended in elderly)
c) Zopiclone
d) Lorazepam (causes significant rebound effects such as anxiety and tension)
e) Diazepam (longest half-life so has hangover effect)
f) Triazolam should be avoided in elderly

132. AF is a 42-year-old woman who presented at your pharmacy and was asking about a
contraceptive method for herself. She is travelling for a two-month mountain climbing trip in
Nepal and her flight is in two weeks. She has been smoking ½ a pack of cigarettes per day for
the past 10 years. AF does not want to have her period during the trip. What is the most
appropriate thing you should do?
a) Tell her 2 weeks is a very short period for her hormonal level to be adjusted so he can do
nothing for her.
b) Combined oral contraceptive from now daily till the end of the trip
c) Medroxy progesterone injection
d) Levonorgestrel intrauterine system
e) Norethindrone
133. A woman was standing in one of the isles in front of the OTC section and she seemed
confused about which medication to choose. You, the pharmacist, when you saw her, you
realized that the she is your daughter’s teacher. You approached her and introduced yourself
saying,” Hi, I am the pharmacist. Which medication are you looking for?” She seemed hesitant
to answer and she looked shy. What is the best action the pharmacist should do as a next step?
a) Offer her that you go to a private counseling area.
b) Talk to her in a quite OTC isle area

134. In the previous case, which principle of ethics the pharmacist is upholding:
a) Autonomy
b) Confidentiality
c) Veracity
d) Conflict of interest
e) Beneficence

135. A dispensing error in a hospital should be reported to:


a) Canadian Pharmacist Association
b) The Canadian Agency for Drug and Technology in Health
c) The National system for Incident Reporting
d) Public health Agency of Canada

136. The Canadian Adverse Drug Reaction Newsletter (CARN) is done by:
a) Medeffect (Health Canada)
b) Canadian Pharmacist Association
c) ISMP
d) The Canadian Agency for Drug and Technology in Health

137. MJ is a patient with HIV taking Tenofovir 300mg, lamivudine, BID 7am and 7pm
Lopinavir/ritonavir at 7 pm, and levothyroxine qam. When they did a lab test, they found that
his triglycerides were high what the reason is for this.
a) Tenofovir
b) Lopinavir/ritonavir (hypertriglyceridemia is a side effect) protease inhibitor cause this
c) Interaction of levothyroxine with Lopinavir/ritonavir (no interaction)
d) Interaction between Tenofovir and Lopinavir/retonavir. (would cause renal problems)
138. MJ called you at 3:00 pm and told you that this morning he was vomiting because he drank
a lot of alcohol last night, so he skipped his medications as he woke up late this morning. He is
asking you what to do next.
a) Take all the morning medications now and at 7pm with take Lopinavir/ritonavir
b) Take only lamivudine and Tenofovir now and take the Lopinavir/ritonavir with
other antiviral with supper.
c) Skip all of today’s medications since he drank a lot of alcohol
d) Skip the morning medications and take the evening medications
e) Dose same time each day! in AM 30‐60 mins before breakfast or HS 4hr after supper, he
made shift to the dose 7am-7pm to 3pm-10pm

139. A patient is taking Omeprazole. Which one of his medications will interact with it?
a) Omeprazole will decrease the absorption of iron
b) Omeprazole will decrease the absorption of Ca (calcium absorption increased with acid)
c) Omeprazole will increase the absorption of concomitant medications
d) Omeprazole will decrease the absorption of concomitant medications
OMEPRAZOLE will decrease the level or effect of IRON by increasing gastric pH. Applies only to oral form of
both agents. Significant interaction possible, monitor closely.

140. AM is a 47-year-old patient with uncontrolled hypertension, and he injured his back badly
from lifting heavy objects and now he has severe pain going down to his sciatica.
What is your best advice for him to control his pain?
a) Naproxen (he could be on a diuretic) avoid NSAIDs in cardio issues
b) Oxycodone
c) Fentanyl patch (Don’t start with a patch unless oral therapy has failed)
d) Acetaminophen, codeine, caffeine
Another version: What is the best medication to start with?
a) Naproxen
b) ASA
c) Oxycodone
d) ASA + Codeine + methocarbamol
Sciatica is pain along the sciatic nerve. It usually results from compression of lumbar nerve roots in the lower
back. Common causes include intervertebral disk herniation, osteophytes, and narrowing of the spinal canal
(spinal stenosis). Symptoms include pain radiating from the buttocks down the leg. Diagnosis sometimes
involves MRI or CT. Electromyography and nerve conduction studies can identify the affected level.
Treatment includes symptomatic measures and sometimes surgery, particularly if there is neurologic deficit.
Symptoms and Signs: Pain radiates along the course of the sciatic nerve, most often down the buttocks and
posterior aspect of the leg to below the knee. The pain is typically burning, lancinating, or stabbing. It may
occur with or without low back pain. The Valsalva maneuver or coughing may worsen pain due to disk
herniation. Patients may complain of numbness and sometimes weakness in the affected leg.
Nerve root compression can cause sensory, motor, or, the most objective finding, reflex deficits. L5‐S1 disk
herniation may affect the ankle jerk reflex; L3‐L4 herniation may affect the knee jerk.
Straight leg raising may cause pain that radiates down the leg when the leg is slowly raised above 60° and
sometimes less. This finding is sensitive for sciatica; pain radiating down the affected leg when the
contralateral leg is lifted (crossed straight leg raising) is more specific for sciatica. The straight leg raise test
can be done while patients are seated with the hip joint flexed at 90°; the lower leg is slowly raised until the
knee is fully extended. If sciatica is present, the pain in the spine (and often the radicular symptoms) occurs
as the leg is extended. The slump test can also be done, similarly to the straight leg raise test, but with the
patient "slumping" (with the thoracic and lumbar spines flexed) and the neck flexed. The slump test is more
sensitive, but less specific, for disk herniation than the straight leg raise test.
Treatment
 Activity as tolerated, analgesics, and sometimes drugs that relieve neuropathic pain
 Physical therapy
 Sometimes oral or epidural corticosteroids
 Surgery for severe cases
Acute pain relief can come from 24 to 48 hours of bed rest in a recumbent position with the head of the bed
elevated about 30° (semi‐Fowler position). Measures used to treat low back pain, including nonopioid
analgesics (eg, NSAIDs, acetaminophen), can be tried for up to 6 weeks. Drugs that decrease neuropathic
pain (see Chronic Pain), such as gabapentin or other anticonvulsants or low‐dose tricyclic antidepressants
(no tricyclic is superior to another), may relieve symptoms. Oral gabapentin 100 to 300 mg at bedtime is
used initially and should be titrated up slowly to avoid adverse effects that might inhibit patient recovery.
As with all sedating drugs, care should be taken in the elderly, patients at risk of falls, patients with
arrhythmias, and those with chronic kidney disease.
Muscle spasm may be relieved with therapeutic heat or cold, and physical therapy may be useful. Whether
corticosteroids should be used to treat acute radicular pain is controversial. Given epidurally, corticosteroids
may accelerate pain relief, but they probably should not be used unless pain is severe or persistent. Some
clinicians try oral corticosteroids, but firm evidence of efficacy is lacking.
Surgery is indicated only for cauda equina syndrome or for unequivocal disk herniation plus one of the
following:
 Muscular weakness that is worsening or not resolving
 Other progressive neurologic deficits
 Intolerable, intractable pain that interferes with job or personal functions in an emotionally stable
patient and that has not lessened after 6 weeks of conservative treatment
Classic diskectomy with limited laminotomy for intervertebral disk herniation is the standard procedure. If
herniation is localized, microdiskectomy may be done; with it, the skin incision and laminotomy can be
smaller. Chemonucleolysis, using intradiskal injection of chymopapain, is no longer used.
Predictors of poor surgical outcome include
 Prominent psychiatric factors
 Persistence of symptoms for > 6 months
 Heavy manual labor
 Prominence of back pain (nonradicular)
 Secondary gain (ie, litigation and compensability)
141. All of the following should be included in AM’s counselling, EXCEPT:
a) Moderate exercise
b) Acupuncture
c) Complete bed rest (there is no advantage to bed rest in acute back pain with sciatica)
d) TENS (Trans-electric nerve stimulation)
Ice, heat, laser, ultrasound, massage, acupuncture, tens
TENS used for LOD f (low back pain, OA, dysmenorrhea, fibromyalgia)

142. Oral metronidazole affects the P450 2C9. Which medication will it interact with?
a) Allopurinol
b) Glyburide
c) Haloperidol
d) Metoprolol
HALOPERIDOL and METRONIDAZOLE both increase QTc interval. Potential for dangerous interaction; Use
with caution and monitor closely.
FUROSEMIDE and METRONIDAZOLE both increase QTc interval. Potential for interaction; monitor.
METRONIDAZOLE will increase the level or effect of PHENYTOIN by affecting hepatic enzyme CYP2C9/10
metabolism. Significant interaction possible, monitor closely.
METRONIDAZOLE will increase the level or effect of FLUVASTATIN by affecting hepatic enzyme CYP2C9/10
metabolism. Minor or non‐significant interaction.

143. A patient took pseudoephedrine and loratadine twice in the last 24 hours and had very high
blood pressure (160/95). Above patient probably which drug he is taking and cause this
symptoms combine with?
a) Desipramine (TCA)
b) Tranylcypromine (MAOI non selective)
c) Citalopram (SSRI)
d) Paroxetine (SSRI)
e) Clobazam (benzodiazepine derivative)
Trimipramine and venlafaxine raise blood pressure the most and sustain it from all antidepressants. After
which MAOIs increase blood pressure.

144. What is the patient experiencing? Hypertensive crisis

145. A 10-week pregnant lady came to you in the pharmacy and said she has burning and
itching in the vaginal area and it’s the first time for her to experience these symptoms and she
said she has a white- grey discharge and malodorous. Why would you refer this patient? K type
a) Because this is the first time for her to get candidiasis
b) Her symptoms are different from those of candidiasis (its not suppose to smell bad,
this maybe bacterial vagnosis)
c) A pregnant woman cannot take intravaginal medication
Refer: first episode, fever, pelvic pain, odor, under 12, pregnant, diabetes, immunocompromised, 2
infections in less than 2 months
Option A if written: because this is the first time for her to get these symptoms will be answer also

146. A teenager was skateboarding and fell on his knees. He presented to you in the pharmacy
with his knees excoriated, red, and something else. There was no dirt or debris in the wound.
After appropriate irrigation of the wound, what is the best action the pharmacist should do?
a) Give topical antibiotics
b) Cover the wound with an appropriate dressing
c) Refer him to the physician to check if there is an infection in the wound
d) Irrigate with saline
e) Recommend tetanus injection

147. IF you will refer the previous patient, which of the following statements does NOT prompt
you to refer
a) If blood stopped upon applying pressure
b) The wound has particles in it
c) The wound is inflamed Wound is deep and penetrating
d) If you get bitten by human or animal
e) The patient’s tetanus status is unknown

148. A regular client at your pharmacy called you on Saturday evening (not midnight) and she
said that she ran out of her medications and the doctor is on vacation and she asks you for a 2-
day advance until she sees her doctor on Monday morning. Her profile is as follows (in a table)
90 Amlodipine - last refill was 3 months ago and has no refills left 90 Ramipril - last refill was
3 months ago and has no refills left
90 Paroxetine - last refill was 3 months ago and has no refills left 30 Zopiclone – last refill was
2 weeks ago and has no refills left
Using your professional judgment as a pharmacist and your ethical/legal principles, what is the
most appropriate action you should do?
a) Advance her a two-day supply of all her medications
b) Advance her a two-day supply of all her medications except Zopiclone
c) Advance her a two-day supply of all her medications except Zopiclone and paroxetine
d) Advise her to go to a walk-in clinic and get an authorized prescription
e) Advise her to go the nearest emergency department to get an authorized prescription
149. A physician called you to ask about the newest treatment for multiple sclerosis. What is the
most suitable method for the pharmacist to get this information?
a) PubMed
b) E-therapeutics
c) E-cps
d) Clinical practice guidelines
e) Primary Literature

150. An obese diabetic with albuminuria woman newly diagnosed with hypertension. Her blood
pressure was 160/95. Her physician wanted her to try changing her life style for 3 months
before starting therapy and he advised her to have salt substitution in her diet.
Which statement is true?
a) DASH diet can be of same benefit as pharmacotherapy
b) Salt substitution has no benefit for her case
c) Reducing trans fatty acids help in salt restriction
d) Most of the patients can not accomplish salt substitution
e) Because she is a diabetic, she needs meds ASAP to reduce risk factors.

151. Three months later, this woman lost 6


kg and her blood pressure was 154/90. She
presented to you at the pharmacy with a
prescription Ramipril, 5mg daily (may be
valsartan). What is the DTP?
a) The dose of Ramipril is too low for
her case
b) Monotherapy is not a good option for
her
c) Inappropriate therapy
d) Dispense as it is

152. A woman came to the pharmacy for the second time this week to buy Senokot. What is the
most appropriate action you should do?
a) Advise her that she shouldn’t overdose
b) Ask her about her apparent overdose of this medication
c) Refuse to dispense her this medication
d) Send her to other pharmacy
e) Talk about laxative abuse & identify better ways she can deal with constipation lifestyle.
153. A patient is taking digoxin. All can put him at risk for digoxin toxicity except?
a) Hepatic impairment (some hepatic metabolism)
b) Renal impairment (renally excreted)
c) Cognitive decline (symptom)
d) Atrial fibrillation (digoxin treats this)
e) Decreased potassium level (predisposes toxicity)
Like gabapentin both excreted renally, and has no relation with hepatic. Plasma digoxin conc. profiles in
patients with acute hepatitis generally fell within the range of profiles in a group of healthy subjects.

154. A vegetarian patient taking warfarin all are monitored except


a) INR (always)
b) Protamine in diet (reverses anticoagulant effects of heparin)
c) All the other choices were relevant to warfarin
d) Vitamin k (affects INR, lots of vit K in green leafy veggies)

155. A patient with ADHD the doctor prescribed 10 mg Methylphenidate BID at 7am and at
3pm., (the question never said IR or ER). The child is well controlled but the mother told the
pharmacist that her son has insomnia. What is your best recommendation?
a) Take 2 doses in the morning (20 mg) at once
b) Switch to methylphenidate ER 2 tablets at noon
c) Switch to methylphenidate ER and take once at 7am and once at 3pm
d) Take one dose at 7am and the other at 12pm
e) Take just the morning dose.
Generally, should be given before 4pm to avoid insomnia. The earlier the better

156. The doctor added Atomoxetine to this patient’s medications. One week later, mother came
and said that her child is complaining of headache for the past week. What is your best advice?
a) Advise her to see the physician if the headache is bothersome and persistent (report to
doctor if persistent)
b) This is transient side effect and should not be a concern
c) Atomoxetine does not cause headache (it does)
d) Advise her to go to the nearest emergency (not an emergency it’s a common side effect)

157. What is the difference between methylphenidate and Atomoxetine?


a) Atomoxetine is a nonstimulant so it will cause somnolence
158. The mother moved to another town and wants to transfer her son’s prescriptions. What is
your best response? The son’s prescriptions had some refills left.
a) Transfer Atomoxetine but Methylphenidate cannot be transferred and needs a new
written prescription (you can’t transfer controlled drugs except BZD only once)
b) Transfer Atomoxetine but Methylphenidate cannot be transferred and needs a new
verbal prescription
c) He needs written RX for both Atomoxetine and methylphenidate

159. A patient has osteomyelitis in his sternum bone due to a Staph aureus infection. He has
been taking Cloxacillin IV 2g Q4H. The doctor wants to do a step-down oral therapy. What
should you do?
a) Cloxacillin 500 po q12h
b) Clindamycin 450 q6h
c) Ciprofloxacin 750 q12h
d) Azithromycin 500 qd
e) Co-trimoxazole
To treat osteomyelitis due to staph infection you can use Cloxacillin, cefazolin or clindamycin IV and then
Cloxacillin, cephalexin, clindamycin or amoxicillin‐clavulanate orally for completion of course

160. A patient got an infection from an IV central line. The culture revealed coagulase negative
gr+ve cocci. What is the microorganism?
a) Streptococcus Pyogenes (gram positive)
b) Staphlycoccus epidermidis (gram positive, coagulase negative)
c) Streptococcus A B- hemolytic (gram positive)
d) Enterococci (gram positive)
Coagulase negative staphylococci (CoNS) species such as Staphylococcus epidermidis and Staphylococcus
hemolyticus are commonly found on the skin and the mucous membranes of many individuals. Staph aureus
is gram positive, coagulase positive

161. A patient is 85 years old has osteoarthritis and she is taking prescribed acetaminophen 650
qid and OTC ASA/codeine/caffeine and Tylenol 1. Now she has increased her OTC dose to 4-6
tablets. What is the DTP?
a) The patient is at risk of GI side effects
b) The pharmacist is concerned about the constipation and thinks it does not outweigh the
pain control.
c) Addiction
d) I would be more concerned about the Tylenol overdose
162. A mother for a one-year-old child presented to you at the pharmacy and said that her child
has fever. He said his temperature is 39 and that he does not have seizures at night.
You should recommend all of the following, EXCEPT:
a) Give him acetaminophen not more than 5 doses per day
b) Remove excess clothing
c) If you’re going to sponge, do this 30 min after the antipyretic (more effective in the first
30 mins)
d) Ensure a good fluid intake
e) Wake him up at night to give him a dose

163. What is the gold standard to measure body temperature for this child?
a) Axillary thermometer
b) Rectal thermometer (chapter 12 in TC, gold standard for children 5 and under)
c) Oral thermometer
d) Tympanic thermometer
e) Temporal thermometer

164. What is a true statement to tell for the mother?


a) Most of elevated body temperature is due to bacterial infection
b) Axillary temperatures are usually lower than rectal and oral ones.
c) Water immersion might be needed if fever reached 39.5
d) The risk of high fever might lead to tremors and twitching
A rectal temperature is 0.5°F (0.3°C) to 1°F (0.6°C) higher than an oral temperature.
An ear (tympanic) temperature is 0.5°F (0.3°C) to 1°F (0.6°C) higher than an oral temperature.
An armpit (axillary) temperature is usually 0.5°F (0.3°C) to 1°F (0.6°C) lower than an oral temperature.

165. A woman has migraine with aura and she used to take Sumatriptan sc to relieve her
headache. She has two kids and she love biking with her kids in the weekends.
All the following statements for counseling about migraine headache are true, EXCEPT:
a) Avoid triggers (makes it better)
b) Avoid working in front of a computer screen for a long time (prevents it)
c) Try taking a nap when you have headache (makes it better)
d) Go for biking when you have headache
e) Turn off the lights (makes it better)
How can you treat a migraine?
Prevention
 Avoid foods and other things that may trigger a migraine.
 Some people find that biofeedback, relaxation, acupuncture and cognitive behavioural therapy help
to prevent migraine headaches.
Mild migraine headaches
 Many patients get relief by lying down in a dark, quiet room and applying a cold cloth or ice pack to
the head. Falling asleep often provides relief as well.
 Pain relievers such as ASA, acetaminophen, ibuprofen or naproxen sodium might help.
 It is important not to use pain relievers more than 15 days per month, or products containing codeine
or caffeine more than 10 days per month. If you use them more often, you may begin to get more
frequent or daily headaches called “medication‐overuse headache.”
 Seek medical advice if you have headaches every day or almost every day. You may be asked to stop
taking pain relievers for a while. If your headaches are severe, you will usually be prescribed other
medication.
Moderate or severe migraine headaches
 Seek medical advice. You may need a prescription medication.
 It is important not to use migraine therapy such as triptans and ergot medication more than 10 days
per month.
 You may need medication to treat nausea. Dimenhydrinate (Gravol) may help to reduce nausea. You
may also need prescription medication such as domperidone or metoclopramide to treat n or v.
 Go to an emergency room if you have a very severe headache and/or severe vomiting that starts
suddenly. Consider going to an urgent care facility if your migraine isn’t relieved by medication.
 You may be prescribed medication that you take every day to prevent migraines that happen often,
that are severe or that last a long time.
 Certain vitamins or herbal remedies may help to prevent migraines. Talk to a health‐care practitioner
before taking any medication to prevent migraines.
Other headaches
 There are many other less common types of headaches. Seek medical advice to find out what is
causing your headache. Many people who think they have “sinus” headaches are actually suffering
from migraine or another type of headache. Sinus headaches occur only if a sinus infection is present.
Severe, new or unusual headaches
Some headaches may be the first sign of a more serious health problem. Go to the emergency room or an
urgent care facility if:
 You suddenly have a severe headache that is worse than any you have had before.
 You have symptoms such as fever, stiff neck, drowsiness, confusion, seizures or a general feeling of
weakness, as well as a headache.

166. The same woman came back with a prescription for Rizatriptan Wafer. Which of the
following is a true statement about Rizatriptan Wafer?
a) It is absorbed from the buccal cavity (it’s swallowed orally)
b) Co-administration with alcohol is contraindicated. (not stated in the CPS)
c) It is used for migraine with nausea (it indicated for migraines with or without aura)
d) It is absorbed faster than Rizatriptan tablets
e) It is contraindicated with people who have difficulty swallowing (it dissolves in the saliva
so easy to swallow)
167. You are a hospital pharmacist. You discovered that one of patients had an order for Losec
and the technician who prepared the prescription misinterpreted it as Lasix. However, the
pharmacist who was there in that shift is on vacation today. The patient has been taking the
wrong medication for three days so far including this day. Who is the first person you should
contact?
a) The physician who wrote the prescription.
b) The pharmacist in charge of that shift
c) The technician who prepared the prescription
d) The patient’s family
e) The nurse on the patient care unit
Assuming he is still in hospital talk to nurse first to stop giving Lasix

168. The most important source for Candida albicans infections that would lead to it being
present in ur blood test
a) Mouth
b) Skin
c) Bladder
d) Colon (commensal bacterial in the gut flora)
Diagnosis
 Histopathology and fungal cultures
 Blood cultures
 Serum beta‐glucan testing
 T2Candida panel
Because Candida species are commensal, their culture from sputum, the mouth, the vagina, urine, stool, or
skin does not necessarily signify an invasive, progressive infection. A characteristic clinical lesion must also
be present, histopathologic evidence of tissue invasion (eg, yeasts, pseudohyphae, or hyphae in tissue
specimens) must be documented, and other etiologies must be excluded. Positive cultures of specimens
taken from normally sterile sites, such as blood, cerebrospinal fluid, pericardium, pericardial fluid, or
biopsied tissue, provide definitive evidence that systemic therapy is needed.
Serum beta‐glucan is often positive in patients with invasive candidiasis; conversely, a negative result
indicates low likelihood of systemic infection.
The T2Candida panel is a magnetic resonance assay that directly detects Candida species in whole blood
samples in 3 to 5 hours. It is highly sensitive and has an excellent negative predictive value (1). Other
molecular diagnostic testing is also available, including matrix‐assisted laser desorption ionization–time of
flight (MALDI‐TOF) mass spectrometry and polymerase chain reaction (PCR)‐based assays.
Ophthalmologic examination to check for endophthalmitis is recommended for all patients with candidemia.
Standard laboratory techniques often misidentify C. auris as C. haemulonii, C. famata, C. sake, or another
species. Matrix‐assisted laser desorption ionization‐time of flight mass spectrometry (MALDI‐TOF MS) is a
more reliable method for correct identification. A nucleic acid‐based test also is now available.
169. According to NAPRA, Iron is Schedule:
a) I
b) II
c) III
d) Unscheduled
Iron with more than 30mg elemental iron per solid dose is sch II, less than 30mg is unscheduled

170. AM is a patient with Parkinson’s on Levodopa/carbidopa 100/25 mg QID, Risperidone and


other medications. All of the following should be monitored with levodopa therapy, EXCEPT:
a) Blood glucose
b) Appetite
c) Blood pressure (can cause hypotension)
d) Urinary incontinence
Psychotic symptoms can develop depression with suicidal tendencies, can cause hallucinations & confusion

171. Which of AM’s medications may worsen his Parkinsons?


a) Risperidone (has Parkinsonian extrapyramidal side effects)
b) Domperidone doesn’t cross BBB
c) Quetiapine, clozapine no EPS
d) Olanzapine the worst in in EPS, it increases weight

172. A patient came to your pharmacy complaining that his eyes are itchy and red for the past
month. He also said that he has been using eye drops all this period to treat his eye. You suspect
hyperemia. Which of the following medications may cause his symptoms?
a) Tetrahydrozoline decongestant constricts blood vessels in eye making them red
b) Nedocromil treats hyperemia
c) Polymixin B
d) Levocabastin treats hyperemia
They all do

173. A cancer patient admitted to the hospital with altered consciousness. She had
hypercalcemia. Which of the following medication is the most appropriate for her case?
a) Pamidronate (this is IV bisphosphinate first line for acute tx in malignancy)
b) Prednisolone (Corticosteroid is appropriate therapy for hypercalcemia when its due to
hormonal therapy for breast cancer for example.)
c) Na polystyrene sulphonate
d) Insulin
Diuretic (can give this if there was fluid overload, but discouraged because fluid depletion can exacerbate
hypercalcemia)
174. What else should you monitor to know the underlying reason of hypercalcemia?
a) Creatinine
b) Serum albumin
c) TSH
d) Transaminases
e) Biopsy
Another opinion: coz he asks for the underlying reason not a monitor for hypercalcemia, hyperthyroid
increases the bone turnover rate resulting in hypercalcemia. This is the underlying cause, but the monitor is
albumin

175. MB presented to your pharmacy. He said he had intermittent diarrhea for the past 10 days
and that he has been tired the last month. What is the most appropriate advice you should give
to MB?
a) Advise him to take Loperamide
b) Advise him to have some bed rest and drink fluids
c) Advise him to visit his physician
d) Advise him to visit the nearest emergency department
e) Bisthmus subsalicylate
When to refer Diarrhea: fever, bloody stool, dehydration, more than 48hrs (as it could be drug induced),
Adult with abdominal pain or vomiting or more than 6 unformed stool per day, child < 6 moths, or vomits 4‐
6 time /day
Repeated constipation, or diarrhea, from DM may be sig of gastroparesis

176. What is true about the goal of ISMP?


a) Provide a safe working place for health care professionals
b) Decrease adverse drug reactions
c) Protect the patient
d) Some option about ADR
ISMP promotes safe medication practices. ISMP Canada's mandate includes analyzing medication incidents,
making recommendations for the prevention of harmful medication incidents, and facilitating quality
improvement initiatives.

177. A doctor called you about a patient that had a fungal toenail infection, what is your
recommendation?
a) Oral Terbinafine
b) Topical ciclopirox
c) Clotrimazole
d) Tolnaftate
e) Itraconazole
178. A patient on omeprazole. What is the best Calcium salt he can take?
a) Ca carbonate
b) Ca citrate (needs less acid for it to be absorbed, also the better option for the elderly)
c) Ca gluconate
d) Ca Cl

179. A hypertensive patient had sulfa allergy (anaphylaxis). Which diuretic to give?
a) Furosemide (loop diuretic)
b) Indapamide (thiazide like diuretic)
c) Metolazone (thiazide like diuretic)
d) Spironolactone
e) Hydrochlorothiazide (thiazide diuretic)
Diuretics that do not contain a sulfonamide group (e.g., amiloride, eplerenone, ethacrynic acid,
spironolactone, and triamterene) are safe for patients with an allergy to sulfa.
A number of drugs contain sulfur but are not sulfonamides; examples are Amoxicillin, clopidogrel, captopril,
omeprazole, ranitidine, spironolactone, and sulindac.
Environmental sulfur dioxide has profound respiratory effects on both healthy and asthmatic individuals,
Sulfhydryl‐containing drugs, such as captopril and penicillamine, are associated with serious cutaneous
allergies.
Acetazolamide, bumetanide, celecoxib, chlorothiazide, diazoxide, dorzolamide, furosemide, glyburide,
hydrochlorothiazide, indapamide, metolazone, valdecoxib, sumatriptan, torsemide, and zonisamide.

180. A pregnant woman vomited 8 times today and she is not drinking well and does not want
to take any medications. Her husband calls you at the pharmacy asking what to do. What is the
most appropriate response?
a) See or call your physician to prescribe Diclectin
b) Have some rest
c) Eat small snacks
d) Go to the emergency (can be severely dehydrated which is harmful to the fetus, needs to
be hydrated)

181. A patient is taking many eye drops. What advise you should provide for the patient to
reduce systemic effects of the eye drops?
a) Press on the lacrimal duct… with your finger
b) Separate 2 hours between different drops
c) Closed her eyes after each use
It can be limited to some degree by compressing the medial punctum and nasolacrimal sac on drop
application (press your finger firmly over your lids, next to the nose).
182. You are a pharmacy manager. A pharmacist came asking you if she can have 2 weeks, sick
leave as her family is going through some troubles and she wants to be with them. She has
worked hard and had a cumulative of 21 sick days. What is the most appropriate response from
the pharmacist?
a) I am not sure if the HR will consider this a good reason for a sick leave
b) You will have to bring a letter from your physician
c) Tell her to go to her doctor to provide a list with your medications
d) You will support her at the HR

183. Which of the following is proarrhythmic?


a) Sotalol
b) Ramipril
c) Digoxin
Sotalol is proarrythmic at higher doses, and may prolong the QTc interval, aggravate arrhythmias, and lead
to torsades de pointes (4%).

184. All are announced as highly risk in dispensing except


a) Vincristine
b) Lorazepam
c) KCL
d) Morphine
e) Heparin

185. Immunization every 10 years


a) Hepatitis B (give at 0,1,6 months)
b) Pneumococcal (Once per life)
c) Tetanus and diphtheria
d) Polio (0, 1-2, 6-12 months)

186. Patient is taking SC morphine; doctor is asking when to increase the dose at interval of:
a) 6 hrs
b) 12 hrs
c) 24 hrs  (according to CPS 2013) Dose every 4‐5 hr, but increasing dose is every 24hr

187. Pt with pneumonia PSI is 114 admitted to the hospital because (long case)
a) He is severely ill
b) Score is 114
c) He cannot take oral therapy
Note: PSI is pneumonia‐specific severity of illness. If score is or less than 90, treat as outpatient. Is or greater
than 91 treats as inpatient
Treatment for the above patient ICU iv therapy what is the empiric treatment
Empiric treatment is B‐lactam iv plus macrolide iv or respiratory fluoroquinolone iv.

188. Patient had hemorrhagic Stroke and the ambulance gave him with 81 mg ASA & sent him
to the hospital. The doctor at the hospital ordered for him alteplase. What is wrong about that?
a) Patient did not receive enough medication
b) Inappropriate medication for that patient alteplase is a clot buster! This guy is
bleeding!

189. Doctor decided to treat him. He acted with paternalism why:


a) Because he did not consult his family
b) Because he did not consult wife
c) He followed the general guidelines

190. Amlodipine and list of medications including ramipril monitor all except
a) Angioedema
ACE makes angioedema but does not make ankle edema. CCB edema (pulmonary or peripheral)

191. Tobramycin toxicity (ototoxicity)


a) Fatigue
b) Dizziness
c) Bradycardia
d) Hepatotoxicity

192. A pharmacist refused to dispense medication according to his moral beliefs and he
arranged with another pharmacy for the patient to pick the medication from. Which ethical
principle he worked with: (no non-mal)
a) Beneficence
b) Autonomy

193. Ulcerative colitis what to give as an interim therapy


a) Azathioprine
b) Sulfasalazine
c) 6 Mercaptopurine
d) Prednisone
Prednisone = used as a temporary intervention to induce remission = "INTERIM therapy"
NOT used on regular basis to maintain remission
194. A community pharmacy was sold, and the new owner appointed a pharmacist manager to
take care of all dispensary issues, except financial issues. Manager should do all except:
a) Sign for new contract with nursing home
b) Do staff performance appraisal
c) Be a narcotic signer
d) Transfer amounts directly from the current accounts for purchasing inventory
e) Adjust the scheduling

195. Which of the following could only the owner do


a) Salary increases
b) Appraisals
c) Choose patient care clinics
d) Something related to dispensing medications

196. In order to avoid bias in making recommendations by the pharmacist to the patient:
a) Follow conclusion of an author of a peer reviewed article in a journal
b) Follow opinions of local physicians
c) Give evidence-based recommendations

197. Pt took H-pylroi treatment for 2 weeks and he is asking you for a home diagnostic kits for
H-pylori. What to advise him as a pharmacist:
a) Not sensitive if used just after the treatment
b) It will give a false negative (testing before 28 days may give false negative results)
Urea breath tests use an oral dose of 13C‐ or 14C‐labeled urea. In an infected patient, the organism
metabolizes the urea and liberates labeled CO2, which is exhaled and can be quantified in breath samples
taken 20 to 30 minutes after ingestion of the urea. Sensitivity and specificity are > 95%. Urea breath tests
are well suited for confirming eradication of the organism after therapy.
False‐negative results are possible with recent antibiotic use or concomitant proton pump inhibitor therapy;
therefore, follow‐up testing should be delayed ≥ 4 weeks after antibiotic therapy and 1 week after proton
pump inhibitor therapy. H2 blockers do not affect the test.

198. 67-year-old lady with recurrent cystitis doctor gave her co-trimoxazole for single dose
a) 3 days
b) 5 days
c) 7 days
d) 10 days
Treatment of acute uncomplicated UTI is for 3 days.
Treat for 7 days in women with symptoms lasting more than one week or women with recurrent infections
in less than a month.
Treat for 3 days if the women are 65 years or older.
199. A patient just started on paroxetine 5 days ago, the pharmacist will call him to follow up on
a) Dry mouth
b) Insomnia

200. Objective of pharmacy blueprint, except?


a) Physical appearance of pharmacy
b) Development and improvement
c) Patient preference
d) Updated technology
Blue print responsible for: legislation, regulations, liability, continuous education, communication technique.
Except Design of the pharmacy

201. Cancer Patient on narcotic. Doctor prescribed docusate and senna. His doctor came to ask
why the doctor prescribed both medications and if he should take them. What should the
response of the pharmacist?
a) Tell her to take docusate regularly and senna prn
b) Call Dr. to ask why he prescribed laxatives
c) Tell her that he should take them both regularly

202. Patient has hypercalcemia, which other test would you do other than calcium blood levels.
a) TSH (PTH)
b) INR
c) Peripheral smear
d) Ferrous
e) Amylase
Need to test for serum ionized calcium or serum calcium and albumin, PTH, 24‐hr urine creatinine and
calcium, serum phosphate, alkaline phosphatase, total protein, serum creatinine and urea

203. Teacher wants to take Pramipexole, which of the following is not a side effect the patient
well experience?
a) Constipation
b) Orthostatic hypotension
c) Altered taste
d) Sudden sedation
MIRAPEX may cause unwanted effects such as nausea, constipation, sleepiness, dizziness, dream
abnormalities, amnesia (memory loss), fatigue, muscle weakness, restlessness, weight decrease, including
decreased appetite, weight increased, hiccups, accidental injury, confusion, increase in cholesterol,
aggressive behaviour, pneumonia, abnormal behaviour (reflecting symptoms of impulse control disorders
and compulsions), overeating, headache, hyperkinesia (unusually overactive), dystonia (inability of keeping
your body and neck straight and upright (axial dystonia)), in particular flexion of the head and neck (also
called antecollis), forward bending of the lower back (also called camptocormia) or sidewards bending of
the back (also called pleurothotonus or Pisa Syndrome), fainting, visual impairment, including double vision,
vision blurred and visual acuity reduced, shortness of breath, vomiting, heart failure, and peripheral oedema
(swelling of hands, ankles or feet). MIRAPEX does not usually affect people's normal activities. However,
some people may feel dizzy or sleepy while taking MIRAPEX, especially during the first few weeks of
treatment

204. Nausea & Vomiting will be affected in a


Cancer patient by all, EXCEPT:
a) Gender
b) Type of medication
c) Duration of therapy
d) Site of tumor
Although medication is the most likely cause of nausea and vomiting in a patient receiving cancer
treatment, rule out other potential causes (e.g., fluid/electrolyte abnormalities, bowel obstruction, CNS or
hepatic metastases, infections, radiation therapy). Other drugs (e.g., opioids, digoxin, antibiotics) may cause
or exacerbate nausea and vomiting; therefore, a thorough medication history is essential. Some
chemotherapeutic agents are more likely to cause nausea and vomiting than others. It is important to
consider the emetogenic potential, the dose of the chemotherapy agents and the expected pattern of emesis
of the chemotherapy regimen when choosing antiemetics.
Patient‐specific factors such as <50 years of age, female gender, and a history of motion sickness, nausea in
pregnancy, or chemotherapy‐induced nausea and vomiting (CINV) may raise the risk of experiencing nausea
and vomiting; therefore, antiemetic regimens must be tailored to the individual patient.

205. Calculations
 NNT. Answer: 91
 NNH. Answer: 25
 Standard error. Answer: 90

206. Calculation on drug prices based on daily treatment given the capsule price and frequency
where the insurance company will cover only the lowest price and a 10% higher of the lowest
one. Answer: 2 lowest cost options
207. Pharmacy performance last year is 8 as a turnover. This year, staring stock was 75,000 and
end of year stock was 25,000. Cost of goods: 150,000. How is the performance compared to last
year?
a) Better than last year
b) same as last year
c) Worse than last year
d) Does not meet financial obligations
Answer: by calculation: COG/av. inventory = 150,000/75,000 ‐ 25,000 = 3. Therefore, performance is worse

208. Patient asking about Omega 3 if it is effective for prevention of stroke. Where can the
pharmacist check this information:
a) Reputable manufacturer for Omega 3
b) Heart and Stroke Foundation Website
c) Primary literature

209. Diabetic patient who is on 2 drugs, doctor asked the best next addition to his treatment plan
a) Clinical practice guidelines
b) Clinical Pharmacology
c) Pharmacology textbook

210. Side effect of a drug, where would you find it:


a) CPS
b) TC

211. Patient has ragweed allergy. For nit removal what to use
a) Isopropyl myristate/dimethicone
b) Pyrethroid pamoate
c) Permethrin
d) Lindane

212. What else to suggest


a) Keep non washable stuff in plastic bag for 30 days
b) Patient need single treatment only
c) Keep cloth 24 hr in hot water for washing
d) Family member should be treated
e) Towel soaked in vinegar------ ok
213. Don't shake, all except:
a) Sumatriptan
b) Miacalcic
c) Nitroglycerin
d) Betaxolol (glaucoma)
Reduction of elevated intraocular pressure in conditions
such as ocular hypertension and open‐angle glaucoma.
Suspension must be well shaken before use. Store the bottle in the outer container.
Adults (including the elderly): recommended therapy is one drop of Betaxolol 0.5% Eye Drops to be instilled
into the affected eye(s) twice a day.
Children: No clinical studies have been performed to establish safety and efficacy in children. Therefore, this
product is currently not recommended for use in children.
When using nasolacrimal occlusion or closing the eyelids for 2 minutes, the systemic absorption is reduced.
This may result in a decrease in systemic side effects and an increase in local activity.
Intraocular pressure should be reassessed approximately four weeks after starting treatment because
response to Betaxolol 0.5% Eye Drops may take a few weeks to stabilise.
If necessary, concomitant treatment with miotics, adrenaline and/or carbonic anhydrase inhibitors can be
instituted. In order to prevent the active substance(s) from being washed out when additional ophthalmic
medication is used, an interval of at least 10 minutes between each application is recommended. The use of
two topical beta‐adrenergic agents is not recommended.
Transfer from a single antiglaucoma agent: Continue the agent and add one drop of Betaxolol 0.5% Eye
Drops in each affected eye twice daily. On the following day, discontinue the previous agent completely, and
continue with Betaxolol 0.5% Eye Drops.
When several antiglaucoma agents are being used, the patient should be assessed on an individual basis.
Adjustment should involve one agent at a time at intervals of not less than one week.
Patients should be instructed to remove soft contact lenses before using betaxolol.

214. Patient came for late refill, what is the likely DTP: No DTP

215. SAP - What`s true about SAP:


a) Manufacturer makes the finally decision
b) SAP authority certifies efficacy
c) The medication is provided free of charge
d) The medication is provided for only 3 months

216. All the following are among the responsibilities of Health Canada EXCEPT:
a) Providing NOC for marketed drugs
b) Review package label of marketed drug
c) Coordinating adverse drug reaction reports
d) Coordinating of poison control
217. Marketed drug, what`s the first step to done after registration:
a) Post marketing surveillance (Phase IV)
b) Advertising
c) Pricing by PMRB
d) Notice of Compliance

217. Calculations: t ½ for Vancomycin. Initial dose: 45 and then 40 after 10 hours. T ½ is:
Answer: 6 days

218. Patient with ascites and takes Spironolactone, what should we add:
a) Furosemide
b) Metolazone
c) HCTZ

219. Diabetes, on metformin and glyburide. 2 cups of alcohol per day and doubles it on the
weekend, tell the patient to avoid
a) Glyburide interacting with alcohol
b) Metformin interaction with alcohol (in interaction in CPS mentioned that ethanol
enhances hypoglycemia)
c) Excess alcohol will cause hypo OR HYPER (can’t remember) glycaemia
d) hypoglycemia in the weekend

220. All of these are taken on an empty stomach except?


a) Cefuroxime
b) Cloxacillin
c) Norfloxacin
Cefuroxime axetil tablets should be taken after food for optimum absorption.

221. The following drug is a verbal narcotic


a) Oxycodone+acetaminophen+caffeine
b) Hydrocodone
c) Fiorinal (butalbital+ ASA+codeine+caffeine)

222. Guy calls you from the States telling you he wants to buy a lot of pseudoephedrine to give
his family for the cold season. Ethically, you are worried about all EXCEPT:
a) Border trouble Border (security is not within my job or scope.)
b) You aren’t there to counsel the family members
c) It should only be for personal use
223. Hydrochlorothiazide:
a) Increases LDL
b) Increases TG
c) Increases LDL and TG
d) Increases TG and decreases LDL
It's from a class of medications called diuretics, more
commonly known as water pills.
High doses — 50 milligrams or more — of some diuretics,
including hydrochlorothiazide, can temporarily increase
your low‐density lipoprotein (LDL) cholesterol — the
"bad" cholesterol — and triglycerides

224. Hypothyroidism symptoms, all EXCEPT:


a) Oily skin
b) Dry skin
c) Cold intolerance
d) Constipation

225. A hospital pharmacist got order for a parenteral, he realized he doesn’t have all the
materials
a) Obtain stock from another institute
b) Order from manufacture and wait till it arrives
c) Call doctor and ask him to change the order
d) Fill order with what u have and monitor the patient
Obtain from a sister institution (borrow)
226. COPD patient wants to take Influenza, what to tell the patient
a) Tell him its ineffective after December
b) Not used for COPD
c) It can be used in most of the minor illness

227. What is the reason for documenting in the pharmacy? Enhance patient care

228. Patient is prescribed 187.5 mg TID for 10 days, he has two option: 125mg/5 ml (100 ml
bottle is for 3.50$) The other bottle is 250mg/5ml (100 ml bottle for 5.40$).
What is the most cost effective to give him?
a) 3 bottles of 125mg/ml

229. What is the minimal amount to give to the patient for EACH dose?
a) 3.75ml of 250mg/ml

230. What is correct about Horizontal Laminar flow


a) It is the most effective technique
b) Must never interfere with the air flow behind the object
c) Must not hang any IV bags above the object you’re working with
d) Objects must be virus free before entering
e) Can place non sterile objects into the hood

231. Patient has COPD, what do u recommend


a) Influenza yearly
b) Pneumoccoccal yearly
Ensuring COPD patients have currently recommended vaccines will help to reduce their risk of infection and
exacerbation. Vaccinate against influenza annually, early in the fall. Influenza vaccine reduces
exacerbations and death in patients with COPD. Recommend pneumococcal vaccination to all patients with
COPD who have no contraindications. Efficacy studies of pneumococcal vaccine in patients with COPD yield
conflicting results; however, pneumococcal vaccination is recommended because patients with COPD are at
increased risk of hospitalization and mortality from pneumonia.
Consider repeating the pneumococcal polysaccharide vaccine (Pneu‐P‐23) as a one‐time booster in high‐risk
patients (COPD plus ≥1 other risk factor) in 5 years. Additional boosters are not recommended.
232. Pharmacist found a patient who is the teacher of his child searching the isles of OTC and
trying to avoid asking him, he offered her a private counselling area, this uphold:
a) Confidentiality
b) Conflict of interest
c) Autonomy

233. A medication renally eliminated what is true:


a) Treatment of dehydration will increase elimination
b) Diuretics will increase elimination
c) Vitamin C will increase elimination
d) Increase dose will increase elimination

234. Technician in a hospital has used all her vacation days a now asking for more days because
her doctor told her that she is under stress and needs rest. What to do (k-type)
a) Ask to see report from her doctor
b) Check with HR
c) Ask to see medications she is using

235. A patient with hepatic disease with no intracellular involvement and was diagnosed with
cholestasis. What to monitor:
a) Albumin
b) Bilirubin
c) ALP
d) AST & ALT
e) LDH
Clinical/biochemical evidence of cholestasis 1‐ Elevated alkaline phosphatase [ALP]
2‐ Gamma glutamyl transferase [GGT] 3‐ Bilirubin, If there's cell injury (intracellular) monitor Bilirubin

236. On doing an interchangeability project in the pharmacy of hospital. As a pharmacist you


can do all except:
a) Prepare clinical-evidence support for your claim
b) Provide evidence of cost-effectiveness of the new drug to be added
c) Approach ethics committee for their approval
d) Try to have support from physicians

237. Pt. was on high dose of a drug. When he entered the hospital, they reduced his dose
because of operation and later released. After discharge by 1 month, he discovered that he was
on wrong dose when he went to refill his prescription. What caused this problem:
a) Medication reconciliation upon admission
238. Pharmacist to enhance the application of expansion of scope of practice which approved by
regularity authority and resisted by physicians:
a) Lobby patients to pressure physicians to accept it
b) Send written letters to physicians explaining the new changes
c) Attend local meetings of physicians in your area to explain changes and address
their concerns.

239. Hospital pharmacy manager to enhance co-operation between pharmacy staff and
physicians. Do all except:
a) Assign a mentor for new staff joining the pharmacy
b) Take staff in patients rounds
c) Assign tasks to staff in different hospitals
d) Attend inter-disciplinary lunch meetings.
e) Let technicians attend grand medical rounds

240. Regular pt. in your pharmacy travelling to USA for 4 months, asking for remaining refills
in her benzodiazepine Rx. Her insurance will not cover her for this period. What to tell her:
a) I can`t refill 4 months in advance
b) I will refill them and bill your insurance every month
c) I will ask the doctor to increase the dose so I can fill a larger qty.
d) I will mail it to you once your refills are due.
e) I will transfer your Rx to a pharmacy you choose in USA
Insurance covers three months only during travelling. Pay for more than three months

241. Pt. took amoxicillin, what isn’t a sign of anaphylaxis that require emergency:
a) Urticarial
b) Macupapular rash
c) Shortness of breath
d) Pharyngeoedema
The rash is described as Maculopapular or morbilliform; therefore, in medical literature, it is called
"amoxicillin‐induced morbilliform It starts on the trunk and can spread from there. This rash is unlikely to be
a true allergic reaction, and is not a contraindication for future amoxicillin usage, nor should the current
regimen necessarily be stopped. However, this common amoxicillin rash and a dangerous allergic reaction
cannot easily be distinguished by inexperienced persons, so a healthcare professional is often required to
distinguish between the two
242. What can be seen in community pharmacy:
a) Plastic tools are better than glass
b) Measuring tools should be changed every year
c) Cream can be prepared over paper slab or glass plate
d) Cylindrical measures should be kept under sterilization when not in use.

243. Glargine insulin side effect:


a) Lipoatrophy
b) Nocturnal hypoglycemia
c) Weight loss
d) Injection site reaction, lipodystrophy, pruritus, and rash
Hypoglycemia is the most common adverse effect of insulin, including LANTUS
The more common side effects that can occur with insulin glargine include:
 Low blood sugar. Symptoms may include: hunger. nervousness. ...
 Unexplained weight gain.
 Swelling in your arms, legs, feet, or ankles (edema)
 Reactions at the injection site. Symptoms may include: a small indent in your skin (lipoatrophy)

244. What is contraindication of Plan-B contraceptives:


a) Migraine with aura
b) History of bleeding
c) Pregnancy

245. Pt. newly married and do not want to take OC daily and needs to be pregnant once she
stops her pills. She is obese (weight is not given), what to recommend:
a) OC pills
b) IUS (Levonorgestrel)  Not used in newly married wife
c) Combined contraceptives patch
d) Nuva-ring
e) Medroxy-progesterone injection

246. What is the group side effect of fluoroquinolones?


a) Hepatotoxicity
b) Glucose abnormality
c) Neurotoxicity
d) Nephrotoxicity
247. Which of the following have drug interaction with Sertraline?
a) Meperidine (opioid+SSRI: Serotonin syndrome)
b) Naproxen
c) Codeine

248. Pregnant patient having vaginal discharge, itching, non-smelly, she had it before and was
told that it is a yeast infection, you refer her to doctor because:
a) She is pregnant

249. Old patient who needs to take the influenza vaccine but cannot due to his illness. He lives
in a long- term care home and sees his grandchild and family. What is the best choice?
a) Give the grandchild and family the vaccine

250. A child took 5 times the dose of amoxicillin suspension that he is supposed to get. What
should be done:
a) Refer to Dr.
b) Refer to ER
c) Monitor the baby
Signs and Symptoms
Serious toxicity is unlikely following large doses of amoxicillin. Acute ingestion of large doses of amoxicillin
may cause nausea, vomiting, diarrhea and abdominal pain. Acute oliguric renal failure and hematuria may
occur following large doses.
Recommended Management
Activated charcoal is recommended only for very large recent ingestions. For amoxicillin ingestions of less
than 250 mg/kg, treatment is not usually required in patients with normal renal function. Monitor fluid and
electrolyte status and renal function in patients with severe vomiting and/or diarrhea. Hemodialysis may be
useful following severe overdose with renal impairment.

251. IBD patient taking many


medications and had a flare up.
What do you give him?
a) Sulfalazine
b) Prednisone (interim use)
Flare up = acute attack
Corticosteroids are useful for acute
flare‐ups of most forms of IBD when
5‐ASA compounds are inadequate.
However, corticosteroids are not
appropriate for maintenance.
IV hydrocortisone 300 mg/day or methylprednisolone 16 to 20 mg 3 times a day is used for severe
disease; oral prednisone or prednisolone 40 to 60 mg once a day may be used for moderate disease.
Treatment is continued until symptoms remit (usually 7 to 28 days) and then tapered by 5 to 10 mg weekly
to 20 mg once a day. Treatment is then further tapered by 2.5 to 5 mg weekly depending on clinical
response, while instituting maintenance therapy with 5‐ASA or immunomodulators. Adverse effects of short‐
term corticosteroids in high doses include hyperglycemia, hypertension, insomnia, hyperactivity, and acute
psychotic episodes.
Hydrocortisone enemas or foam may be used for proctitis and left‐sided colon disease; as an enema, 100 mg
in 60 mL of isotonic solution is given once a day or twice a day. The enema should be retained in the bowel
as long as possible; instillation at night, with the patient lying on the left side with hips elevated, may
prolong retention and extend distribution. Treatment, if effective, should be continued daily for about 2 to 4
weeks, then every other day for 1 to 2 weeks, and then gradually discontinued over 1 to 2 weeks.
Budesonide is a corticosteroid with a high (> 90%) first‐pass liver metabolism; thus, oral administration may
have a significant effect on gastrointestinal tract disease but minimal adrenal suppression.
Oral budesonide has fewer adverse effects than prednisolone but is not as rapidly effective and is typically
used for less severe disease. Budesonide may be effective in maintaining remission for 8 weeks but has not
yet proved effective for long‐term maintenance. The drug is approved for small‐bowel Crohn disease, and an
enteric‐coated, delayed‐release form is available for ulcerative colitis. Dosage is 9 mg once a day. It is also
available outside the US as an enema.
All patients started on corticosteroids (including budesonide) should be given oral vitamin D 400 to 800
units/day and calcium 1200 mg/day. Corticosteroids should be used with caution in patients with chronic
liver disease including cirrhosis because bioavailability and clinical effects may be enhanced.

252. All are transmitted through water EXCEPT


a) Shigella
b) Hepatitis B (blood+sex)
c) Campylobacter

253. A child in the hospital was given Cotrimoxazole IV 0.5 ml Q 6 hours. Each ml contains 80
mg trimethoprim and 16 mg Sulfamethoxazole. The physician wants to switch him to
suspension containing 200 mg TMP and 40 mg SMX per 5 ml. how much volume should we
use for this child.
a) 0.5 ml QID
b) 2 ml BID
c) 4 ml BID
d) 4 ml TID
e) 8 ml BID
254. A patient with hypercalcemia which test do we use to confirm:
a) Ferritin
b) Serum iron
c) Urinary test
d) APTT
e) INR

255. A 65 yrs old male was diagnosed with MI, what are the markers of ACS, all EXCEPT
a) Troponin 1
b) Troponin 2
c) CK
d) Serum Creatinine
e) Myoglobin
Serum creatinine is not a marker, it is for monitoring

256. In a study the drug is given to adults but not children, so which principle was followed:
a) Paternalism
b) Veracity
c) Non- Maleficence

257. A preparation of 5% stock solution & you want to make .0125% so you use;
a) 5ml in 2 liters
b) 5ml in 1 liter
c) 2.5 ml in 2.5 liters

258. Which medication will increase the INR?


a) Indinavir (inhibitor)
b) Phenytoin (inducer)
c) CBZ (inducer)
d) Rifampin (inducer)

259. All medications would increase INR, EXCEPT:


a) Cholestyramine

260. Which drug must be should be dispensed pursuant to written Rx only:


a) Ketamine (straight narcotic)
b) T2
c) T3
d) Clobazam
261. All are menopause symptoms EXCEPT?
a) Flushing
b) Night sweats
c) Vaginitis
d) Mood changes
e) Breast tenderness
Other associated complaints of menopause, including
vaginal symptoms such as dryness, itching, vaginitis and
dyspareunia,

262. What would glatiramer be used for? (storage in fridge)


a) HIV infection
b) Multiple sclerosis
c) Hepatitis-C
d) Psoriasis
e) Gullian-berri syndroms

263. Patient is bringing methadone prescription from unauthorized (means has no exemption)
doctor. Her family doctor on vacation for three weeks, what is appropriate action of pharmacist?
a) Refuse to fill prescription
b) Phone to her family doctor and request prescription after he arrive
c) Tell new doctor to register temporarily
d) Right away fill her prescription

264. Monitoring of ezetimibe? Liver enzymes


CI: hepatic. Monitor: LFT's & CK. Fibrates the only one we should do renal function

265. Drug is recalled you should do all except


a) Post the recalling display in the pharmacy
b) Contact patients to whom the drug was dispensed
c) Look for the patch size and number which is affected

266. BG is asking for the emergency contra plan-B®, what will you ask her?
a) How many partners do you have?
b) Did you take oral contraceptive before?
c) When was your last menses?
267. A drug is given at the dose of 1g IV Q12H to a patient. The desired therapeutic level of the
drug in the blood is 15 – 20 mg/L. When blood drawn just before the next dose, the
concentration was found 10.57 mg/L. What is the appropriate dose of the drug so that the
desired therapeutic level is maintained?
a) 1250 mg IV Q12H
b) 1500 mg IV Q12H
c) 1750 mg IV Q12H
d) 1125 mg IV Q12H

268. Prednisolone 5mg tablets is prescribed with instruction of 10mg starting dose and tapering
½ tablet weekly. How many tablets are required?
a) 35 tabs
b) 105 tabs

269. In a community, the pharmacist arranged a seminar on medication error. All of the
following personnel are invited except?
a) Physician
b) Patients
c) Technicians
d) Nurse
e) Pharmacist

270. Symptoms of mild to moderate Alzheimer’s disease, except?


a) Slurred speech lost
b) Difficulty managing money and paying bills
c) Fatigue
d) Depression
e) Poor judgment
Early dementia symptoms
Recent memory is impaired; learning and retaining new information become difficult. Language problems
(especially with word finding), mood swings, and personality changes develop. Patients may have
progressive difficulty with independent activities of daily living (eg, balancing their checkbook, finding their
way around, remembering where they put things). Abstract thinking, insight, or judgment may be impaired.
Patients may respond to loss of independence and memory with irritability, hostility, and agitation.
Functional ability may be further limited by the following:
 Agnosia: Impaired ability to identify objects despite intact sensory function
 Apraxia: Impaired ability to do previously learned motor activities despite intact motor function
 Aphasia: Impaired ability to comprehend or use language
Although early dementia may not compromise sociability, family members may report strange behavior
accompanied by emotional lability.
Intermediate dementia symptoms
Patients become unable to learn and recall new information. Memory of remote events is reduced but not
totally lost. Patients may require help with basic activities of daily living (eg, bathing, eating, dressing,
toileting).
Personality changes may progress. Patients may become irritable, anxious, self‐centered, inflexible, or angry
more easily, or they may become more passive, with a flat affect, depression, indecisiveness, lack of
spontaneity, or general withdrawal from social situations. Personality traits or habits may become more
exaggerated (eg, concern with money becomes obsession with it).
Behavior disorders may develop: Patients may wander or become suddenly and inappropriately agitated,
hostile, uncooperative, or physically aggressive.
By this stage, patients have lost all sense of time and place because they cannot effectively use normal
environmental and social cues. Patients often get lost; they may be unable to find their own bedroom or
bathroom. They remain ambulatory but are at risk of falls or accidents secondary to confusion.
Altered sensation or perception may culminate in psychosis with hallucinations and paranoid and
persecutory delusions. Sleep patterns are often disorganized.
Late (severe) dementia symptoms
Patients cannot walk, feed themselves, or do any other activities of daily living; they may become
incontinent. Recent and remote memory is completely lost. Patients may be unable to swallow. They are at
risk of undernutrition, pneumonia (especially due to aspiration), and pressure ulcers. Because they depend
completely on others for care, placement in a long‐term care facility often becomes necessary. Eventually,
patients become mute. Because such patients cannot relate any symptoms to a physician and because older
patients often have no febrile or leukocytic response to infection. End‐stage dementia results in coma and
death, usually due to infection.

271. Which drug is contraindicated in pregnancy who has hypertension?


a) Nifedipine
b) Losartan
c) Amlodipine
Methyl Dopa, Nifidine XL, Labetaolol, Clonidine, Hydralazine

272. Which of the following should be monitored after she finishing isotretinoin treatment:
a) Triglycerides
b) Thyroid
c) blood glucose

273. New case: Diabetic patient taking metformin 1 g bid, does not want to be a ginuea pig for
trial and does not like injections, and is cost conscious (low cost medication). Why would the
pharmacist meet the patient to detail the information regarding - upholding with which ethic?
a) Autonomy
b) Veracity
c) Justice
274. What medication would be added next to give better control?
a) Glyburide
b) Sitagliptin

275. What to give to treat mild hyperkalemia?


a) Sodium polystyrene
b) ca gluconate
c) IV fluids
d) NaHCO3

276. Patient with sulpha allergy, what should be taken:


a) Ethacrynic acid
b) Indapamide
c) HCTZ

277. Patient with ascites and takes Spironolactone, what should we add:
a) Furosemide
b) Metolazone
c) HCTZ

278. Child has asthma, and he comes and his asthma is getting worse, he`s on SABA, if you do
not recommend to go to the doctor to get an ICS, you violate what?
a) Beneficence
b) Autonomy
c) Veracity
d) Justice

279. TB patient 28 yrs old and malnourished. Recently emigrated from south east asia. She is
on INH + Rifampin + pyranzamide + ethambutamol. When the nurse asks about her quadruple
regimen, what is true to tell her:
a) She is on four drugs because the area she came from is very resistant.

280. You are a pharmacy manager and there is a pharmacist about to finish his shift and he told
you how busy was today that there is still pending paper work to be done. Adding to this a
physician called and said he will send you 17 meningitis patients. What to do:
a) Tell the doctor you will dispense them tomorrow
b) Send the patients to the other pharmacy 40 km away
c) Force the pharmacist to stay
d) Call another pharmacist and ask the pharmacist to stay as late as he can
281. Pharmacy to improve profitability:
a) Increase local advertisement
b) Decrease staff
c) Increase working hours

282. Pregnant women has nausea and vomiting and husband called the pharmacist to ask what
they should do. Yesterday she vomited 8 times and now is feeling very tired
a) Refer to the ER for assessment

283. 3 months infant his mother came to your pharmacy she told you that she has just switched
her baby from breast feeding to formula instead. He used to have a bowel movement 3 times a
day. Now she is worry because he did not defecate from 36 hr what you should do
do not warrant treatment with mineral oil
a) Warrant treatment with mineral oil
b) Warrant treatment with prune juice
c) Warrant treatment with glycerine supp.
d) Refer to doctor

284. A child took 5 times the dose of amoxicillin suspension that he is supposed to get. What
should be done:
a) Refer to Dr.
b) Refer to ER
c) Monitor the baby

285. Use with caution in BPH


a) Benztropine (anticholinergic)
b) Furosemide

286. All of following are side effect of benztropine except


a) Miosis
b) Urinary retention
c) Constipation

287. Myotoxicity with statin is due to


a) Renal dysfunction
b) High dose of statin
c) Due to cholestyramine
288. Pharmacist student measure blood pressure of obese patient. BMI 33.5 what is true
a) Take at least 2 reading do average
b) Use adult cuff
c) Measure on standing position
d) Measure on both hands

289. Which vaccine should be given every 10 year?


a) D-TAP
b) Pneumocacal
c) Meningococcal

290. A New regulation allow the pharmacist for prescription adaptation; they want to expand
the pharmacy practice so they can refill medications for chronic conditions. The physicians in
your area are skeptical about this initiative. What is the most appropriate action to do?
a) Bring up this topic whenever a physician calls you.
b) Let the patients talk about this issue whenever they visit their drs.
c) Send the physicians a detailed letter about this initiative.
d) Meet the physicians on their local meetings and talk about it
e) Schedule a meeting and invite the physicians in your local area

291. Your pharmacy costumers have different ethnic background what would you do in your
pharmacy to improve communication with customer with diverse background:
a) Enroll your staff in communication training
b) Analyse different ethnic back ground of your customer
c) Test how diversity effect on customer health
d) Assess impact of beliefs on treatment and health

292. Academic detailing by pharmacists provides a service to physicians by?


a) Educating on improved prescribing legibility.
b) Advising on optimal patient interviewing techniques.
c) Recommending strategies to avoid medication wastage.
d) Providing current information on best prescribing practices.
e) Promoting the use of physician samples given to patients.

293. P VALUE, TYPE 1 error and type 2

294. Question on if it is possible to inject epipen through clothes


295. What of triptan can be used in pregnancy? Sumatriptan
Management Non-Drug methods should be First Line, occasional use of Tylenol may be sufficient.
during For women with frequent disabling headaches or who experience severe nausea and vomiting
Pregnancy leading to dehydration, the benefits of further drug treatment may outweigh the risks.
Ibuprofen and naproxen can be used during the 1st or 2nd trimester but should be avoided in the
later stages of pregnancy because they may cause constriction of the fetal ductus arteriosus.
ASA, Indomethacin, Barbiturates must be avoided in pregnancy.
Opioids (Meperidine, Codeine) must be avoided as it can depress fetal respiration.
Severe nausea can be treated with Metoclopramide or Prochlorperazine.
Ergot derivatives (vasoconstriction) cannot be used as they restrict uterine blood flow.
Triptans must be avoided as well (vasoconstriction).
In severe migraine and NV, Propranolol is the only preferred agent; although it is still
associated with intrauterine growth retardation and lower weight.
Eclampsia (a tonic-clonic seizure/HPT/NV/headaches/blindness during pregnancy that is present
only when the placenta is still there), may be prevented by Mg IV supplements.
Breastfeeding As in pregnancy, use nonpharmacologic measures first line during breastfeeding.
When medication is required, acetaminophen is the preferred abortive agent.
Ibuprofen is considered the NSAID of choice in breastfeeding.
Sumatriptan has been studied more than other triptans in lactation and is considered compatible
with breastfeeding, although it is prudent to avoid vasoconstricting agents in the initial
postpartum period; other triptans should be used with caution.
Metoclopramide, domperidone, dimenhydrinate and prochlorperazine are safe in breastfeeding.
For prophylaxis, propranolol and magnesium are the preferred options.
Valproic acid/divalproex sodium is considered compatible with breastfeeding.
Other prophylactic agents should be avoided or used with caution and monitoring of the infant.
Avoid ergot derivatives, barbiturates and opioids (including codeine).

296. Drug of choice of trigeminal neuralgia? Carbamazepine

297. Patient has orthostatic hypotension upon standing?


a) Treat with midodrine + octreotide
Nondrug treatment
Patients requiring prolonged bed rest should sit up each day and exercise in bed when possible. Patients
should rise slowly from a recumbent or sitting position, consume adequate fluids, limit or avoid alcohol, and
exercise regularly when feasible. Regular modest‐intensity exercise promotes overall vascular tone and
reduces venous pooling. Elderly patients should avoid prolonged standing. Sleeping with the head of the bed
raised may relieve symptoms by promoting sodium retention and reducing nocturnal diuresis.
Postprandial hypotension can often be prevented by reducing the size and carbohydrate content of meals,
minimizing alcohol intake, and avoiding sudden standing after meals.
Waist‐high fitted elastic hose may increase venous return, cardiac output, and BP after standing. In severe
cases, inflatable aviator‐type antigravity suits, although often poorly tolerated, may be needed to produce
adequate leg and abdominal counterpressure.
Increasing sodium and water intake may expand intravascular volume and lessen symptoms. In the absence
of heart failure or hypertension, sodium intake can be increased to 6 to 10 g daily by liberally salting food or
taking sodium chloride tablets. This approach risks heart failure, particularly in elderly patients and in
patients with impaired myocardial function; development of dependent edema without heart failure does
not contraindicate continuing this approach.
Drug treatment
Fludrocortisone, a mineralocorticoid, causes sodium retention, which expands plasma volume, and often
lessens symptoms but is effective only when sodium intake is adequate. Dosage is 0.1 mg po at bedtime,
increased weekly to 1 mg or until peripheral edema occurs. This drug may also improve the peripheral
vasoconstrictor response to sympathetic stimulation. Supine hypertension, heart failure, and hypokalemia
may occur; potassium supplements may be needed.
Midodrine, a peripheral alpha‐agonist that is both an arterial and a venous constrictor, is often effective.
Dosage is 2.5 mg to 10 mg po tid. Adverse effects include paresthesias and itching (probably secondary to
piloerection). This drug is not recommended for patients with coronary artery or peripheral arterial disease.
NSAIDs (eg, indomethacin 25 to 50 mg po tid) may inhibit prostaglandin‐induced vasodilation, increasing
peripheral vascular resistance. However, NSAIDs may cause GI symptoms and unwanted vasopressor
reactions (reported with concurrent use of indomethacin and sympathomimetic drugs).
Droxidopa, a norepinephrine precursor, may be beneficial for autonomic dysfunction.
Propranolol or other beta‐blockers may enhance the beneficial effects of sodium and mineralocorticoid
therapy. Beta‐blockade with propranolol leads to unopposed alpha‐adrenergic peripheral vascular
vasoconstriction, preventing the vasodilation that occurs when some patients stand.
Pyridostigmine and octreotide have been effective in small clinical studies.

298. Pentoxifylline (TRENTAL) interactions


299. Wilson disease treatment?
a) Penicillamine
b) Deferoxamine
Drugs Comments
Chelating The treatment of choice for Wilson disease, and treatment is lifelong.
Agents 1–2 g/day PO in 4 divided doses on an empty stomach
Penicillamine Give pyridoxine 25 mg daily with penicillamine to counteract its antipyridoxine effect.
S.E: Proteinuria (nephrotoxic), hematologic effects, mouth ulcers, diarrhea, reduced sense
of taste, decreased appetite, nausea, vomiting, hypersensitivity.
Decreased effect with antacids, iron, zinc. Decreases serum levels of digoxin.
Trientine Indicated in the treatment of patients who are intolerant of penicillamine.
S.E: Anemia. Reduced effect with iron
Elemental Used in patient’s intolerant of penicillamine and trientine.
zinc 50 mg (elemental zinc) TID PO between meals
S.E: GI disturbances. Decreases levels of quinolones, tetracycline, penicillamine

300. Case patient coming to your pharmacy, she has Sinusitis, hypertension, she had nasal
congestion one week ago. Now she is coming with sneezing, rhinorrhea, has a trip tomorrow to
USA, she is looking for a quick relief for her symptoms, what will you recommend
a) Diphenhydramine
b) Diphenhydramine & pseudoephedrine
c) Loratadine alone
d) See her Dr

301. Patient has allergic rhinitis, green sputum, she took loratadine but she still no improvement
these symptoms continued for 3 weeks, you will refer her to Dr for which reason?
a) Green color sputum
b) Because loratadine had no effect on her
c) Because symptoms continued for more than 2 weeks

302. Physician fees are covered by?


a) Mostly public/insurance and some out of pocket expenses
b) Solely public
c) Solely insurance
d) Solely out of pocket

303. Which comes first in the marketing of the drug?


a) NOC
b) PMBRP
c) RCT
304. Pt lost her ACI box with remaining of 42 days and insurance refuses to pay she wants the
Medicine without paying herself, what to advise her?
a) Get a pres with her husband name
b) Call the dr to change to another med
c) Advance her 7 days until get a pres new and u apply for insurance com later

305. Pt will travel 2 mo doesn't want to have period


a) Take low dose of continues daily untill return
b) Take low dose of norethindrone daily continue till return (in HAJ/ Social, sports
activities)

306. Patient was admitted to the hospital due to a suicidal attempt. They found out that it was
due to clomipramine and this medication was stopped during his hospital stay. Then upon his
discharge. The patient was given all his pre- admission prescriptions except the clomipramine
Afterwards the patient went to his community pharmacy and got his prescription medications
that he got from the hospital. Then when the patient went home, he continued taking his
clomipramine pills that he had at home. How could you have prevented this error?
a) Do a medication reconciliation upon discharge
b) Supply a written information with this prescription
c) Do a call back service to follow-up
d) Community pharmacist check his medical profile when dispensing the new prescription*
e) Ask the patient

307. ISMP stands for:


a) International System for Medical Practices
b) Institute for Scientific Methods Practice
c) Institute for Safe Medication Practices

308. A pharmacist is working on a Friday shift that has been very overloaded. He’s speaking to
the pharmacy manager expressing how hectic and busy the day was; and that there was a
backlog in the prescriptions. He’s asking the pharmacist manager whether it’s possible that he
leaves early today and goes back to his family for the weekend. The manager was very
understanging and he agreed. But so that things get harder, the physician in a near by clinic
called the manager and said that he has 17 patients with meningitis that he wants to send to the
pharmacy. What is the most appropriate response from the manager?
a) Tell the pharmacist that you should know your legal and ethical responsibilities and
shouldn’t leave until all the work is done. (workload)
b) Get additional staff to help the pharmacist with the anticipated and expected work
load and ask the pharmacist to stay as late as possible *
c) Tell the physician to send those patients to another pharmacy that is 40 km away
d) Tell the pharmacist these patients have meningitis and their treatment should not be
delayed
e) Let the pharmacist go home early as agreed and let the physician send the patients
tomorrow morning
f) Let the pharmacist go back early as he took a permission, and call the doctor to
send the patients and he will wait for them *

309. Hospitals follow the federal standards and principles in order to:
a) Get federal money from the Canada Assistance Plan
b) Get their full share of the federal transfers for the hospital services*
c) Increase their share in the money needed
Receive Federal fiscal tax transfer funds

310. A pharmacist in a community hospital is trying to increase the team work between the
pharmacy staff and other health care professionals (HCPs) in the area. Which of the following
will NOT contribute to achieving this endeavor?
a) Make a mentor system to integrate new staff
b) Plan a disciplinary seminar lunch
c) Do reports about conflict resolution to HCPs.
d) Send the pharmacist more often to the patient care area

311. The reason for dispensing errors related to environmental conditions in a community
pharmacy is:
a) Low light in the pharmacy
b) The climate inside the pharmacy
c) The work load*
d) Noise
e) Untrained staff

312. The patient came back later complaining that the quantity that has been dispensed was less
than that prescribed. You found out that there was a dispensing error that has occurred, what
would you do to prevent this error?
a) Alert on the computer
b) Checking the number of the stock tablets at the end of each day
c) Double checking the number of tablets and highlighting the amount on the
prescription hard copy*
d) Use electronic counter
e) Let the patient sign on a receipt that he has received the correct amount
313. 16-year-old female took plan B; her father comes in asking about his daughter’s
medications what two conflicting ethical principles the pharmacist is stuck between. << NO
CONFIDENTIALITY >>
a) Veracity and autonomy*
b) Autonomy and nonmalficience
c) Veracity and beneficience
d) Veracity and nonmalfecience
e) Justice and beneficience

314. Patient culture affect his perception about treatment, what should the pharmacy manager
do to help his business regarding this aspect?
a) Look at the local population culture
b) Assess the staff’s knowledge of different languages
c) Assess the clientele that enter the pharmacy
d) Survey the health beliefs of the different cultures
e) Do researches about socio-economic of the surrounding area
Assess impact of culture on health

315. Several errors were reported from the patient care area for KCl injection, as a pharmacy
manager what should you do?
a) Pharmacist prepare the infusion
b) Lock it in a cupboard and keep the key with one nurse in the patient care area
c) Put the label on the bin where you store the injections
d) Remove it completely from the ward and keep it in the pharmacy
e) Put a computer alert

316. A hospital is trying out a new protocol for insulin. What should they do? Pilot phase
a) Install it hospital wide and compare results prior and after the new protocol
b) Install the new protocol in two special units and compare error rate prior and after
c) Collect feedback about the new protocol

317. Elderly female is confused about taking her many medications. You can do all of the
following except?
a) Prepare her medications in a blister pack
b) Go visit her at home to organize her medications
c) Ask her what she is taking
d) Review her medication history in the pharmacy
318. Patient suffering from alcohol dependence & decided to stop. Two days ago, she went to
the doctor, he wrote her BDZ. When she went to the pharmacy to get the medication, she found
that the cashier was her neighbor. The patient was embarrassed and when you talk to her you
found that she didn’t want her neighbor to know anything about her. What should the you do?
a) Tell him alcohol dependence is nothing to be embarrassed of
b) Ask him to go to a private counseling area and ensure that the information is
confidential.
c) Tell the cashier discuss with him since he is his neighbor
d) Tell him the cashier will not disclose any information
e) Tell him he should disclose his information for his sake.

319. case for pt diabetic type 2, her dr prescribed for her metformin and tell her he may add
another one if her BSL still not controlled but she doesn’t want to take any untested medication
telling him she is not a guine pig and she dislikes the idea of insulin, if pharmacist wants to give
her information about diabetes and sessions, what he will uphold?
a) Beneficence
b) Autonomy
c) Paternalism

320. What ISMP do?


a) Gather information from illegal practice in pharmacy
b) Report about side effects and send them to pharmacy
c) Care about the safe medication practice in pharmacy
d) Something about the blue printed book
The Institute for Safe Medication Practices Canada is an independent national not‐for‐profit organization
committed to the advancement of medication safety in all healthcare settings. ISMP Canada works
collaboratively with the healthcare community, regulatory agencies and policy makers, provincial, national
and international patient safety organizations, the pharmaceutical industry and the public to promote safe
medication practices. ISMP Canada's mandate includes analyzing medication incidents, making
recommendations for the prevention of harmful medication incidents, and facilitating quality improvement
initiatives.
MAY 2012
1. What to Counsel a patient wanting to use Chlorhexidine 0.12% mouth rinse?
a) Stains teeth and tongue with prolonged use
b) Treats plaque caused by candida
c) Seperate it from fluoride containing toothpaste
d) Wash and swallow

2. Patient is using Benzymadine oral for mouth pain...counsel on all except


a) Keep in mouth for a few minutes before swallowing
b) It is a non steroidal anti-inflammatory used for your mouth
c) Benzydamine works by preventing the formation of prostaglandins.
Benzydamine also acts as a local anaesthetic, causing numbness and so pain relief at the site of application.
It should then be spit out, not swallowed (although accidental swallowing is unlikely to cause harm).

3. Patient comes to you complaining of major pain all over her leg, and neck and going down to
her back...She tells you she got into a motorcycle accident 2 years ago and has been in this pain
since...Doctors have ruled out any specific cause for the pain...She admits to having tried
marijuana 20 years ago in university and said it made her feel good and relaxed...She tried
Tylenol 1 before but got really nauseated. She has been getting social help...what is your best
advice as the Doctor plans to start her on Opioid for her pain?
a) Opioids will not relieve this type of pain that she is experiencing
b) Opioids are highly addicted and she may become addicted to them
c) She has a history of addiction and abuse so avoid opioids all together
d) she may be intolerable to the opioids, so it is not a good option for her
The pain doesn't seem to be somatic in nature.

4. She wants to start marijuana, what do you tell her


a) It is illegal to use marijuana
b) It is legal but she must obtain authorization from her physician
c) It is legal but she must obtain authorization before she can grow it
d) It is legal if she meets the criteria set out by the Canadian Marijuana

5. Patient has COPD, what do u recommend, all except?


a) Influenza yearly
b) Pneumococcal yearly
6. Regarding a newborn who you talked to his mother about conjugated pneumococcal
vaccine...it will prevent all except
a) Endocarditis
b) AOM
c) Meningitis
d) Ear tubectomy surgery (something along those lines)

7. Patient has Sulfa allergy and glaucoma, and ASTHMA...on a bunch of drugs, what is the
main interaction that would concern the pharmacist?
a) Timolol/Asthma

8. There is a cross allergy with one of his medications and his allergy, which drug may lead to
this
a) Dorzolamide
b) Timolol

9. Doctor insists on trial of the dorzolamide, regardless of the allergy...what do you do


a) Refuse to dispense as it is allergic
b) Refuse to dispense because of patient safety
c) Dispense but document your conversation with the doctor and the patient
d) Dispense and document counselling of the patient only

10. Depot MPA, what to monitor for safety and efficacy


a) Monitor calcium and vitamin d
(warning long term use cause
osteoporosis)
b) Breast cancer (contraceptives are CI in
breast cancer)
c) Weight (cause weight gain)
Women who currently have or have had breast
cancer should not use hormone contraceptives,
including DEPO-PROVERA, because breast cancer
may be hormonally sensitive. Women with a
strong family history of breast cancer or who have
breast nodules should be monitored with
particular care.
Women receiving DEPO-PROVERA should be
counselled regarding the importance of breast self-
examination. Clinical breast examination should be performed at regular intervals.
11. What to monitor for Alendronate safety and efficacy?
a) Calcium levels
b) BMD
c) Esophageal ulcers

12. Drug A, B, C, are on the same page of Formulary, and D is found on different page
a) Chewable
b) Sustained release formulation
c) Has different bioavailability

13. Patient has Osteoporosis, history of breast cancer, menopausal, what to give her
a) Teriparatide
b) Raloxifene (antagonist on breast/ agonist to bones)
c) Estrogen
d) Etidronate
Raloxifene is a selective estrogen receptor modulator (SERM) that may be appropriate for treatment of
osteoporosis in women who cannot take bisphosphonates. It is given orally once daily and reduces vertebral
fractures by about 50% but has not been shown to reduce hip fractures. Raloxifene does not stimulate the
uterus and antagonizes estrogen effects in the breast. It has been shown to reduce the risk of invasive breast
cancer. Raloxifene has been associated with an increased risk of thromboembolism.

14. Testosterone prescription mitte: 90, Repeats: 2x


a) Needs a partfill
b) Dispense as it is legal
c) Needs an interval for the refill

15. A provincial drug plan not supporting a chemotherapeutic drug because of lack of clinical
trials on the drug, they follow which ethical principle
a) Non- maleficence

16. A physician follows the autonomy of the patient in which scenario


a) Informs the patient of therapy he will undergo

17. Thiazides increase what?


a) LDL
b) TG
c) LDL and TG
18. Statins monitor all except
a) Serum creatine
b) Creatine kinase
c) Arthralgia
d) Liver tests

19. Patient has an osteomy and it gets really red and irritant due to all of the following except
a) Too frequent changing of the pouch
b) Drinking milk
An osteoma (plural: "osteomata") is a new piece of bone usually growing on another piece of bone, typically
the skull. It is a benign tumor. When the bone tumor grows on other bone it is known as
"homoplastic osteoma"; when it grows on other tissue it is called "heteroplastic osteoma".

20. What do u not ask a patient to differentiate between a common cold and allergic rhinitis
a) If she is experiencing sore throat
b) If she has nasal discharge (both, nasal discharge)
c) If she has a fever
d) Itchy eyes
e) Duration of her symptoms

21. His dad comes in and asks for his reports for tax purposes?
a) Must get patient consent

22. Patient has been Experiencing normal menstrual pain. She gets them every month and the
pain usually start with the onset of her menstrual cycle and lasts for about 2-3 days after. What
is your best recommendation to relieve her pain?
a) Tylenol
b) Ibuprofen
c) Acetaminophen, pamabrom, pyrilamine
d) Acetaminophen, codeine, caffeine
Dysmenorrhea is uterine pain around the time of menses. Pain may occur with menses or precede menses by
1 to 3 days. Pain tends to peak 24 hours after onset of menses and subside after 2 to 3 days. It is usually
sharp but may be cramping, throbbing, or a dull, constant ache; it may radiate to the legs.
Headache, nausea, constipation or diarrhea, lower back pain, and urinary frequency are common; vomiting
occurs occasionally. Symptoms of premenstrual syndrome may occur during part or all of menses.
Sometimes endometrial clots or casts are expelled.
Red flags: The following findings are of particular concern:
 New or sudden-onset pain
 Unremitting pain
 Fever
 Vaginal discharge
 Evidence of peritonitis
 Interpretation of findings
 Red flag findings suggest a cause of pelvic pain other than dysmenorrhea.
Treatment
Underlying disorders are treated.
Symptomatic treatment of dysmenorrhea begins with adequate rest and sleep and regular exercise. A low-
fat diet and nutritional supplements such as omega-3 fatty acids, flaxseed, magnesium, vitamin E, zinc, and
vitamin B1 are suggested as potentially effective.
Women with primary dysmenorrhea are reassured about the absence of structural gynecologic disorders.
Drugs
 If pain persists, NSAIDs (which relieve pain and inhibit prostaglandins) are typically tried. NSAIDs are
usually started 24 to 48 hours before and continued until 1 or 2 days after menses begin.
 If the NSAID is ineffective, suppression of ovulation with a low-dose estrogen/progestin oral
contraceptive may be tried.
 Other hormone therapy, such as danazol, progestins (eg, levonorgestrel, etonogestrel,
depot medroxyprogesterone acetate), gonadotropin-releasing hormone agonists, or
a levonorgestrel-releasing IUD, may decrease dysmenorrheal symptoms.
 Periodic adjunctive use of analgesics may be needed.

23. New Side effect appeared with a mediation, upon checking in the monograph the side effect
was unlisted, where would be the best place to look for this side effect
a) Manufacturer Medical information staff
b) Meta analysis
c) CPS
d) Martindale

24. Patient got an acute gout attack and the doctor started him on allopurinol and colchicine 0.6
mg bid for 3 days then 0.6mg once a day for 7 days...what to do as a pharmacist
a) Allopurinol should not be started initially at the sign of an acute attack as it may
cause a flare up

25. Upon starting colchicine what do you tell the patient to AVOID
b) Going out in the sun (no photosensitivity)
c) Using alcohol while taking the medication (no alcohol interaction)
d) Taking it with food (without regards to meal)
e) Operating heavy machinery (no dizziness)
Drug-Food Interactions
Studies of colchicine and grapefruit have conflicting results. Avoid grapefruit, Seville orange, lime and
pomelo. If grapefruit is consumed, clinical monitoring is required, especially for vomiting, alopecia and
pancytopenia.
Colchicine is rapidly absorbed from the GI tract. Food does not affect the rate but slightly decreases the
extent of absorption. Peak concentrations occur in 0.5–2 hours. Bioavailability is approximately 45%.
Yes, you can drink alcohol with colchicine. Alcohol does not affect how this medicine works.
However, drinking alcohol increases the level of uric acid in your blood. This could make your gout worse.

26. Long case about a patient having gastric ulcer, arrhythmia, heart failure and doctor
recommends Dronedarone, why is this patient contraindicated for this drug?
a) Arrhythmia
b) Ulcer
c) Heart failure
d) Renal impairment
Dronedarone is without iodine-related organ toxicity. Health Canada recommends that Dronedarone not be
prescribed for patients with heart failure, left ventricular dysfunction or permanent atrial fibrillation.

27. Amiodarone interact with?


a) Rosuvastatin
b) Enalapril
c) Verapamil
If k type so the right answer would be verapamil and rousvastatin but the major DDI with verapamil
Monograph Calcium Channel Antagonists
Amiodarone should be used with caution in patients receiving calcium channel antagonists (e.g., verapamil,
a CYP3A4 substrate, and diltiazem, a CYP3A4 inhibitor) because of the possible potentiation of bradycardia,
sinus arrest, and AV block. If necessary, amiodarone can continue to be used after insertion of a pacemaker
in patients with severe bradycardia or sinus arrest.
HMG-CoA Reductase Inhibitors
HMG-CoA reductase inhibitors that are CYP3A4 substrates (including simvastatin and atorvastatin) in
combination with amiodarone have been associated with reports of myopathy/rhabdomyolysis.

28. Academic detailing, which one is incorrect


a) Pharmacist plays a big role
b) Funded by health Canada
c) Funded by the pharmaceutical manufacturer

29. Canada health act...essential services in the hospital are funded by


a) Federal, provincial and insurance premium
b) Fully federal funded
c) Fully provincial only

30. ISMP Canada: (The Institute for Safe Medication Practices) Canada
a) Includes a method to prevent near misses, before reaching the patient
31. KCL was given to patient at a really high dose, you do all except
a) Correct the dose and make sure he is taking the correct dose
b) See his need for sodium polystyrene resins
c) Assess his need for insulin with glucose

32. A pharmacy manager notes that the pharmacy technician sells Tylenol 1 to her friends.
What should he do?
I. Tell the pharmacist that selling OTC narcotics needs his intervention (BTC)
II. Assess monitoring methods to decrease medication diversion
III. Tell the pharmacist that he should record sold OTC narcotic
Each hard, white, capsule-shaped tablet, imprinted with stylized "M" and "McNEIL" on one face and "No. 1"
on the other, contains 300 mg of acetaminophen, 15 mg of caffeine, and 8 mg of codeine phosphate.
Nonmedicinal ingredients: cellulose, cornstarch, magnesium stearate, and sodium starch glycolate

33. PTSD, what is the drug of choice


a) SSRI
b) Gabapentin
c) Imipramine
Drug therapy (eg, with selective serotonin reuptake inhibitors [SSRIs])
A wide variety of psychotherapies have been successfully used to treat posttraumatic stress disorder (PTSD).
SSRI or another drug therapy is often used as well.
If untreated, chronic PTSD often diminishes in severity without disappearing, but some people remain
severely impaired.
The primary form of psychotherapy used, exposure therapy, involves exposure to situations that the person
avoids because they may trigger recollections of the trauma. Repeated exposure in fantasy to the traumatic
experience itself usually lessens distress after some initial increase in discomfort.
Eye movement desensitization and reprocessing (EMDR) is a form of exposure therapy. For this therapy,
patients are asked to follow the therapist's moving finger while they imagine being exposed to the trauma.
Stopping certain ritual behaviors, such as excessive washing to feel clean after a sexual assault, also helps.
SSRIs may reduce anxiety and/or depression. Prazosin appears helpful in reducing nightmares. Mood
stabilizers and atypical antipsychotics are sometimes prescribed, but support for their use is scant.
Because the anxiety is often intense, supportive psychotherapy plays an important role. Therapists must be
openly empathic and sympathetic, recognizing and acknowledging patients’ mental pain and the reality of
the traumatic events. Early in treatment, many patients need to learn ways to relax and control anxiety (eg,
mindfulness, breathing exercises, yoga) before they can tolerate the exposure that tends to be a focus of
PTSD treatment.
For survivor guilt, psychotherapy aimed at helping patients understand and modify their self-critical and
punitive attitudes may be helpful.
34. Woman comes in with symptoms of a yeast infection. She checked online and found out
that she has a yeast infection. First time, great medical history.
a) Refer to her physician for better initial diagnosis

35. Goal of therapy for acne all except?


a) Eradicate the bacteria causing the acne
b) Prevent new lesion & clear current lesions
c) Prevent scarring
d) Minimize psychological damage
Goals of Therapy
- Clear existing lesions & Prevent new lesions
- Minimize scarring & Reduce dyspigmentation
- Minimize psychological impact

36. Teenager was taking benzyl peroxide for acne but saw no effect for the past 4 months. MD
prescribed topical tretinoin/erythromycin. What is the DTP?
a) None
b) Wrong medication for
indication
c) Not enough medication

37. How long will it take for him to


see effects?
a) 1-2 weeks
b) 4-6 weeks
c) 2-3 months
d) 4-6 months

38. MD decides to give him 40mg oral Isotretinoin. What is the DTP?
a) Dose too high
b) Wrong medication
c) Dose too low
d) No problem
Initial Therapy
The initial dose of EPURIS should be individualized according to the patient's weight and severity of disease.
In general, patients initially should receive EPURIS 0.5 mg/kg body weight daily for a period of two to four
weeks, when their responsiveness to the drug will usually be apparent. It should be noted that transient
exacerbation of acne is occasionally seen during this initial period. For optimal absorption, the daily dose of
EPURIS should be taken with food. Taking EPURIS without food decreases the rate and extent of absorption
by 21% and 33% (Cmax and AUCt). EPURIS should be taken in the nearest number of whole capsules, either
as a single dose or in two divided doses during the day, whichever is more convenient.
Maintenance Therapy
Maintenance dose should be adjusted between 0.1 and 1 mg/kg body weight daily and, in exceptional
instances, up to 2 mg/kg body weight daily, depending upon individual patient response and tolerance to
the drug.
A complete course of therapy consists of 12-16 weeks of EPURIS administration.
Patients may show additional improvement for up to several months after a course of EPURIS has been
completed. With effective treatment, appearance of new lesions will not normally be evident for a period of
at least three to six months

39. Doctor is starting patient, who is 60kg, on Isotretonoin. MD wants to start at 0.5mg/kg/day
for 4 weeks. After the 4 weeks, he wants to increase it to 1mg/kg/day. The total cumulative dose
is 120-150mg/kg. If he starts with 30mg daily for 4 weeks, what is the next step:
a) 30mg bid x 3 months
b) 30mg bid x 4 months
c) 30mg bid x 5 months
d) 60mg bid x 3 months
e) 60mg bid x 4 months
Initial dose is 0.5mg/kg/day for 4 weeks taken with food as OD or BID
Maintenance dose should be adjusted (0.1- 1) mg/kg/day for 12-16 weeks & this is the complete course for
isotretonione therapy.

40. Depressed with erectile dysfunction, hypercholesterolemia & hypertension, what to give
him
a) Bupropion
b) Mirtazapine
c) Venlafaxine
d) Paroxetine

41. What worsens his Depression? Propranolol

42. High TG levels and normal HDL, what to give him? Fenofibrate

43. Diovan 1 tablet every morning, Diamicron 4 tablets qam. If the labels are mixed the patient
takes his medication, he will experience
a) Increased pulse rate
b) Decreased pulse rate
c) Hypoglycemia
44. Patient calls a week later and tells u about the mix-up after realizing he is taking the wrong
doses base on reading the manufacturers recommended dose. What is the initial thing the
pharmacists should do?
a) Asses his status

45. How would you AVOID this error in the future


a) Show the patient the medication upon counselling
b) Ensure that the patient is receiving his appropriate meds
c) Use technology to scan the mediation
d) Separate them on the shelf

46. Which one needs renal adjustment


a) Cotrimoxazole (specially in the elderly)- Gabapentin
b) Moxiflox
c) Doxyxycline
d) Azithromycin
e) Clindamycin

47. Discontinuation syndrome for patient on Antidepressants, how would you best monitor
a) Check patient profile for late refills
b) Automated technique to refill their meds
c) Contact their family and friends and see if they’re taking their meds properly

48. Patient taking a drug and causes acute hepatitis, what to monitor
a) Aspartate aminotransferase
b) Alkaline phosphatase
In the initial diagnosis of acute hepatitis, viral hepatitis should be differentiated from other disorders
causing jaundice (see figure Simplified diagnostic approach to possible acute viral hepatitis).
If acute viral hepatitis is suspected, the following tests are done to screen for hepatitis viruses A, B, and C:
 IgM antibody to HAV (IgM anti-HAV)
 Hepatitis B surface antigen (HBsAg)
 IgM antibody to hepatitis B core (IgM anti-HBc)
 Antibody to hepatitis C virus (anti-HCV) and hepatitis C RNA (HCV-RNA) PCR
If any of the hepatitis B tests are positive, further serologic testing may be necessary to differentiate acute
from past or chronic infection (see table Hepatitis B Serology). If serology suggests hepatitis B, testing for
hepatitis B e antigen (HBeAg) and antibody to hepatitis B e antigen (anti-HBe) is usually done to help
determine the prognosis and to guide antiviral therapy. If serologically confirmed HBV infection is severe,
antibody to hepatitis D virus (anti-HDV) is measured.
Hepatitis B has at least 3 distinct antigen-antibody systems that can be tested:
 HBsAg
 Hepatitis B core antigen (HBcAg)
 HBeAg
HBsAg characteristically appears during the incubation period, usually 1 to 6 weeks before clinical or
biochemical illness develops, and implies infectivity of the blood. It disappears during convalescence.
However, HBsAg is occasionally transient. The corresponding protective antibody (anti-HBs) appears weeks
or months later, after clinical recovery, and usually persists for life; thus, its detection indicates past HBV
infection and relative immunity. In 5 to 10% of patients, HBsAg persists and antibodies do not develop; these
patients become asymptomatic carriers of the virus or develop chronic hepatitis.
HBcAg reflects the viral core. It is detectable in infected liver cells but not in serum except by special
techniques. Antibody to HBcAg (anti-HBc) usually appears at the onset of clinical illness; thereafter, titers
gradually diminish, usually over years or life. Its presence with anti-HBs indicates recovery from previous
HBV infection. Anti-HBc is also present in chronic HBsAg carriers, who do not mount an anti-HBs response. In
acute infection, anti-HBc is mainly of the IgM class, whereas in chronic infection, IgG anti-HBc predominates.
IgM anti-HBc is a sensitive marker of acute HBV infection and occasionally is the only marker of recent
infection, reflecting a window between disappearance of HBsAg and appearance of anti-HBs.
HBeAg is a protein derived from the viral core (not to be confused with hepatitis E virus). Present only in
HBsAg-positive serum, HBeAg tends to suggest more active viral replication and greater infectivity. In
contrast, presence of the corresponding antibody (anti-HBe) suggests lower infectivity. Thus, e antigen
markers are more helpful in prognosis than in diagnosis. Chronic liver disease develops more often among
patients with HBeAg and less often among patients with anti-HBe.
HBV-DNA can be detected in the serum of patients with active HBV infection.
Other tests
Liver tests are needed if not previously done; they include serum alanine aminotransferase (ALT), aspartate
aminotransferase (AST), alkaline phosphatase, and bilirubin. Other tests should be done to evaluate liver
function; they include serum albumin, platelet count, and prothrombin time/international normalized ratio
(PT/INR).

49. Patient got hepatic encephalopathy and the doctor placed him on Lactulose, what do we
monitor
a) Serum ammonia
b) Serum creatine

50. He is travelling to another city so what do u tell him


a) Lactulose does not need a prescription

51. When do u contact the doctor to verify a prescription for a narcotic


a) Dr from different municipality
b) You’re not familiar with the signature
c) He was a new patient
d) Greater than a 90-day supply
52. What will affect the autonomy decision of the physician by a provincial drug plan?????
a) Not covering vacation
b) Needing to authorize initial prescriptions

53. Patient with Vaginitis and is receiving metronidazole 500mg daily for 7 days, what is the
best advise for his patient
a) Use a barrier method
b) Avoid the sun as it may cause photosensitivity (no photosensitivity)
c) Take on an empty stomach (after a meal reduce nausea)
d) Douche everyday to remove odour

54. European drug where to look for this product? Martindale

55. Best ref for drug info for the pt? CTMA
a) E-CPS
b) Martindale

56. Which one of these scenarios follows a Paternalistic view?


a) Diabetic coordinator increases the patient’s antidiabetic dose based on blood results
b) Nurse evaluates the patient and gives him her best recommendation
c) Social worker meets with the patient upon discharge to talk about

57. No wt. gain


a) Lamotrigine
b) Gabapentin
c) Pregabalin
d) Amitriptyline

58. Goal of therapy for osteoporosis


a) Quality of bones
b) Quantity of bones
c) Prevent fractures

59. How to determine her absolute risk for osteoporosis


a) Asses her vit d and ca intake,
b) History of low trauma fractures
c) Her risk of falls from seizures
A fragility fracture occurs after less trauma than might be expected to fracture a normal bone. Falls from a
standing height or less, including falls out of bed, are typically considered fragility fractures.
60. Benzocaine cream/ointment/gel 0.1%. Ratio is 1:2:2 and he total quantity mitte 150g
What is the percentage of the benzocaine?
Ans: 0.02%

61. You have compound A which is 20% w/w salicylic acid, you also have compound B which
is 10% w/v salicylic acid. You want to prepare a 2% salicylic acid solution in which you end up
using 1% of each of the above compounds (A and B). You will use
a) 5ml of compound A and 10ml of compound B
b) 4ml if compound A and 8ml of comound B
Answer:
Compound A: 20 mg -------- 100 mg 1 mg ------ X mg X= 1*100/20 = 5 mg of compund A
Compound B: 10 mg -------- 100 ml 1 mg ------ Y ml Y = 1*100/10 =10 ml of Compound B

62. What do you tell a distressed mother whose child is complaining of croup
a) Usually in most children croup is self limiting within 48h
b) If his symptoms don't improve, he will require antibiotics (para influenza virus)
c) Usually croup patients benefit from Misting

63. What is a Nested case control study?


a) A cohort study group in a case control study
b) A case control group in a cohort study
c) A randomized controlled study

64. 81 years old patient, weighing 52kg has osteoarthritis in both knees. What is the most
effective topical choice?
a) Diclofenac
b) Capsaicin
c) Methyl salicylate

65. She then wants to try something natural for her knee arthritis? Hyaluronic acid

66. What is the least likely thing to help her?


a) Hydrotherapy
b) Weight loss (52 kg)
c) Knee brace
d) Cane
67. Pt got mild symptoms of heart burn after food. He has hypertension, what do u recommend
a) Recommend a light snack before sleep
b) Recommend alginates
c) Call doctor to get a prescription PPI
d) Give him antacids before meals
e) Give him famotidine non prescription

68. When would you have to refer such a patient?


a) He has belching
b) Difficulty swallowing

69. Patient with GERD and drinks 1-2 glasses of wine on the weekend
a) Exercise
b) Weight loss (most significant non-pharmacological)
c) Stop alcohol

70. Patient using eye drops for a month now for his dry and irritated eyes, he develops
hyperemia, it is due to
a) Tetrahydrozoline
b) Zinc sulphate
c) Something with alcohol
d) Sodium cromoglycate

71. Which of the medication had cardiac


toxicity?
a) Doxorubicin

72. Raynaud's question about patient on a bunch of drugs including Metoprolol, he started
getting cold extremities etc, it is may be due to what?
a) There is a drug-disease interaction

73. Which one of these is a concern for the pharmacist


a) Atorvastatin 80mg po daily
b) Insulin glulisine 40U (before breakfast, lunch and dinner)
c) Insulin Glargine 40U (at bedtime)
Basal/background Insulin Dose = 40-50% of Total Daily Insulin Dose
40 U= total daily bolus insulin (13 U before each meal)
74. Patient had a lot of chest pain and pain due to fibromyalgia is uncontrolled and is on many
medications. Medications were; Bisoprolol 5mg daily, Amitriptyline 10mg prn, etc…
What may be the possible reason for his symptom?
a) Bisoprolol is too low (5mg daily)

75. Which one of his medications may at a suboptimal level?


a) Amitryptyline suboptimal 10mg daily (still pain)

76. What's the counselling on insulin pen


a) Draw regular insulin first then NPH
b) Draw NPH then regular after

77. Energy drink is being sold in your pharmacy; the manufactures do not recommend this
product for patients under 18. as a pharmacist, what is the best thing to tell the manager?
a) Put a caution signage near the drinks to educate the customers
b) Restrict to adults’ only above age
c) Do not stock this product entirely

78. A daughter came to ur pharmacy with a prescription Rivastigmine for her Father. He started
to have dementia attacks what is the goal of therapy
a) Improve memory
a) Postpone him from long term care facility or Postpone nursing home admission
b) Decrease mortality
b) Prevent Cachexia
c) Reverse dementia

79. Rivastigmine
a) Monitor after a minimum of 3mo
b) It is proven to be the best treatment for alzheimers

80. In the above case what would u advice the pt


a) Rivastigmine is very effective in her father’s case
b) Not to take the medication with food because it can decrease the absorption of the
medication
Ans: A, coz it should take with breakfast…need to confirm?

81. Sotalol avoid with?


a) Clarithromycin (QT prolongation)
b) Cetirizine
82. Osteoarthritis – she was 87-year-old, 52 kg, and 160 cm, all are good for her except
a) Weight loss
b) Knee brace
c) Hydrotherapy
d) Leg cane and walker
e) Physiotherapist

83. Patient has hypothyroidism experiences all except


a) Dry skin
b) Nervousness (hyperthyroidism)
c) Cold extremities
d) Weight gain

84. Schedule 2 according to NAPRA


a) Make sure pharmacist initiates a conversation with the patient to ensure he is self
medicating correctly
Schedule I drugs require a prescription for sale. Schedule II drugs require professional intervention from the
pharmacist (e.g. patient assessment and patient consultation) prior to sale. Schedule III drugs must be sold
in a licensed pharmacy but can be sold from the self-selection area of the pharmacy.

85. All can lead to errors except


a) ml
b) QD
c) U
d) IU

86. What is the correct Tall Man lettering?


a) predniSONE and prednisoLONE
b) PREDNISONE and PREDNISOLONE
c) PREDnisone and PREDnisolone

87. Patient is on Oral Contraception and gets runny nose and


congestion, what seems to be a common cold. You will refer her
due to all of the following?
a) Dyspnea
b) Headache
Discontinue Medication at the Earliest Manifestation of the Following
A. Thromboembolic and cardiovascular disorders, such as
thrombophlebitis, pulmonary embolism, cerebrovascular disorders, myocardial ischemia, mesenteric
ischemia, mesenteric thrombosis and retinal thrombosis.
B. Conditions that predispose to venous stasis and to vascular thrombosis (e.g. immobilization after
accidents or confinement to bed during long-term illness). Other nonhormonal methods of contraception
should be used until regular activities are resumed. For use of oral contraceptives when surgery is
contemplated, see Warnings and Precautions, Peri-Operative Considerations.
C. Visual defects—partial or complete
D. Papilledema or ophthalmic vascular lesions
E. Severe headache of unknown etiology or worsening of pre-existing migraine headache
F. Increase in epileptic seizures

88. Which one of these medications may cause Biliary obstructive lithiasis?
a) Gentamycin
b) Vancomycin
c) Penicillin
d) Cloxacillin
e) Ceftriaxone

89. Patient comes to you complaining that he


has been recently experiencing Falls and
dizziness. He doesn't not know why. He never
felt this before. His medication profile was
Risedronate started 4 years ago, Perindopril
(titrated up to target dose over the last 6
months), A bunch of other meds started 3 or 4
years ago. What would be your best action in
response to this patient?
a) Hold the Perindopril and monitor the
patient
b) Hold the risedronate and see

90. Patient with Hospital acquired pneumonia and had a bunch of catheters, respiratory rate
high, heart rate high, and was being
treated for a motor vehicle accident, what
to give him now?
a) Ceftriaxone
b) Piperacillin/tazobactam
c) Ampicillin
91. After initiating the antibiotic, when do you expect his blood pressure, respiratory rate and
fever to normalize?
a) 2 h
b) 24
c) 72 (to a fibrile this is the answer)
d) 1 week

92. Monitor improvement except?


a) Sputum count
b) Blood culture
c) Chest x-ray
93. A new patient started on combination of Methotrexate and 6-MP; you should look out for
all of the following except?
a) Leucopenia
b) Thrombocytopenia
c) Sore throat and fever
d) Shortness of breath and peripheral edema

94. You then give him intermittent vincristine, what will you
observe
a) Peripheral neuropathy
b) Diarrhea

95. Patient being discharged and mentions to the pharmacist that he wants to share any changes
made to his therapeutic regimen with his other health care groups outside the hospital. What
will you do as a pharmacist?
a) Tell him to verbally tell his other health care groups of the changes that were made
b) Call his physician and inform him of the changes that occurred to his profile
c) Send out a letter to all his other health care groups
d) Give him a written summary report of the changes that were made to his profile
(medication reconciliation)

96. Patient Come to the pharmacy complaining of a white curdy discharge, no odour, but very
itchy area and bothersome. She is sure that her symptoms are due to candidiasis however this is
the first time she has ever experienced such symptoms. What can you do to help her?
a) Give her a 7-day supply of clotrimazole cream
b) Give her a one-day supply 150mg Fluconazole tablets
c) Giver her 3-day course of miconazole ovules for her candida infection
d) Refer her to her physican before you initiate any therapy

97. What is correct about Horizontal Laminar flow


a) It is the most effective technique
b) Must never interfere with the air flow behind the object
c) Must not hang any IV bags above the object your working with
d) Objects must be virus free before entering
e) Can place non sterile objects into the hood

98. Which med to prepare in biohazard cabinate??


a) Hydroxyurea
b) Valacyclovir
c) Digoxin
99. What makes a meta analysis reliable: large studies/
relevant (low heterogenicity)/ narrow CI
a) Large collection of small studies
b) Diverse nonhomogenous

100. You will need to adjust the dose of Pregabalin in


patient with?
a) Hemodialysis (Pregabalin 75% renally excreted)
b) Hepatic disease
c) Blood disorders

101. Methotrexate and what do u initially monitor


a) Initial Blood levels in the first months
b) Yearly monitoring of retinopathy

102. If patients accidently take it daily instead of weekly what is the first toxic side effect to
look for
a) Myelosuppression
b) Stomatitis

103. Clarithromycin may cause increase QT prolongation if given with?


c) Itraconazole
d) Cetrizine

104. What will increase INR with warfarin? Binge alcohol (large amount at once)
Acute ingestion of a large amount of alcohol may inhibit the metabolism of warfarin and increase INR.
Conversely, chronic heavy alcohol intake may induce the metabolism of warfarin. Moderate alcohol intake
can be permitted.

105. A hospital pharmacist got order for a parentral, he realized he doesn’t have all the
materials, he can do all of the following except?
a) Obtain stock from another institute (sister institute: borrow)
b) Order from manufacture and wait till it arrives
c) Call dr and ask him to change the order
d) Fill order with what u have and monitor the patient

106. Drug shortage can be due to


a) Price competition leading to lower manufacture profit
b) Different Regulations for Quality assurance
107. A pt had an acute gout attack and was treated, after that the dr. wants to initiate allopurinol
a) Titrate till you reach the target dose of allopurinol
b) Titrate dose till you reach the target serum uric acid level

108. Miacalcin was initiated for a patient with pain due to fracture, how to counsel...all Except
a) Avoid if you have shellfish allergy
b) Take ca and v.D supplements
c) Alternate nostrils daily
d) May cause common nasal irritation

109. Patient has Asthma (long case) taking salbutamol & ibuoprofen for pain prn and her
asthma is triggered by dust, pollen, bla bla...she started getting shortness of breath and
wheezing, what may trigger her asthma
a) Ibuprofen
b) Caffeine
c) Acetaminophen
d) Codeine

110. What do you counsel for this patient, All except?


a) Close your windows
b) Put protective covers on beds and pillows
c) Stay hydrated

111. How do you manage her acute attack in the future?


a) Intranasal ipratropium
b) Oral prednisone
c) Theophylline
d) Salmeterol
e) IV methyl prednisone (rescue medication in ICU- at home: avoid triggers)

112. Patient is 45 years old, obese, has


diabetes (bunch of other stuff in the case).
What leads you to initiate a lipid level test?
a) Obesity
b) Age
c) Gender
113. COMPUS, what does it do
Identifies and promotes evidence-
based, clinical and cost-effectiveness
information on optimal drug
prescribing and use.

114. What causes Torsade de point?


Hypokalemia

115. Calculate sensitivity & odds ratio of drug A relative to drug B


Gave you the tru negative, tru positive, false negative., false positive
For people that have the characteristic (as determined by the gold standard), record the number of people
who tested positive and the number of people who tested negative.
Do the same for people that do not have the characteristic (as determined by the gold standard). You will end
up with four numbers. People with the characteristic AND tested positive are the true positives (TP). People
with the characteristic AND tested negative are the false negatives (FN). People without the characteristic
AND tested positive are the false positives (FP).
People without the characteristic AND tested negative are the true negatives (TN) For example, let us suppose
you did the RPR test on the 1000 patients.
Among the 100 patients with syphilis, 95 of them tested positive, and 5 tested negatives. Among the 900
patients without syphilis, 90 tested positive, and 810 tested negatives. In this case, TP=95, FN=5, FP=90, and
TN=810
To calculate the specificity, divide TN by (FP+TN).
In the case above, that would be 810/ (90+810) = 90%. The specificity tells us how likely the test is to come
back negative in someone who does not have the characteristic. Among all people without the characteristic,
what proportion will test negative? 90% specificity is pretty good
To calculate the sensitivity, divide TP by (TP+FN).
In the case above, that would be 95/ (95+5) = 95%. The sensitivity tells us how likely the test is come back
positive in someone who has the characteristic. Among all people that have the characteristic, what proportion
will test positive? 95% sensitivity is pretty good.
Odd ratio: (EE / EN) / (CE / CN) = [(number of patients with events in experimentalgroup) / (number of subjects
with “non-events” in experimental group)] divided by [(number of patients with events in control group) /
(number of subjects with “non-events” in control group)]

116. Topiramate causes? Weight loss


117. Champix causes (varnecline) most common side? Nausa

118. Which drug is knowing “as a class effect” that causes hepatic choleastatic dysfunction?
a) Penicillins
b) Aminoglycosides
c) Estrogen
d) Nitrates

119. Giving a person an IV dose would increase? Peak concentration

120. A drug is totally eliminated by the kidneys what would affect the clearance of the drug
a) Sustained release formulation
b) Hepatic dysfunction

121. Which drug acts like hydralazine


a) Hydoxychlorthiazide
b) ACEI
c) Minoxidil

122. Clarithromycin susp counselling?


a) Shake well
b) Take full course
c) Keep in fridge

123. C. albicans, which is the main cause that would lead to it being present in ur blood test
a) Mouth
b) Bladder
c) Skin
d) Liver
e) Colon

124. Child with AOM, hes 25mo old, 12 mo old and used amoxicillinnow presents with otitis
media...he was treated with amoxicillin...what do u not give him now...all except
a) Amoxicillin
b) Amoxi/clav
c) Cefuroxime
d) Cloxacillin
125. What do you counsil the mother
a) Tell her that the pain and fever may go away within 24-72h after initiating antibiotic
b) Call doctor and tell him to give her prophylactic treatment
c) Tell her to use cold compresses to relive the pain and will not aggravate the fever
d) Avoid Acetaminophen for his pain cause hes a child

126. Patient with rosacea, he is taking metronidazole gel...her face gets dry irritant
a) Continue and give her emollient
b) Switch to metronidazole cream
c) Change medication
Skin (metrogel monograph)
If a reaction suggesting local irritation occurs,
patients should be directed to use the medication
less frequently, discontinue use temporarily, or
discontinue use until further instructions.
There were no reports of contact dermatitis
attributed to METROGEL during clinical trials.
However, there have been reports of contact
dermatitis/allergic reaction reported as post marketing adverse reactions. Physicians should be aware of
the possibility of skin sensitivity reactions and of cross-sensitization with other imidazole preparations, such
as clotrimazole and tioconazole.

127. Advantage of insulin pen, what do u tell them?


a) Not used for visually impaired
b) Prime before every use
c) Once opened keep it in the fridge
d) Adjust the ratio for premixed insulin

128. Which one may cause Hyperglycemia?


a) Removing the TPN while continuing insulin (hypoglycemia)
b) Giving 50U instead of 5U (hypoglycemia)
c) If patient took 0.5ml instead of 0.5u (hypoglycemia)
d) Giving a glargine long acting and not giving pre-meal insulin

129. Sumatriptan what to counsel on, can not take with??


a) Caffeine
b) Tylenol 1
c) Caffergot
4.5 Interaction with other medicinal products and other forms of interaction
There is no evidence of interactions with propranolol, flunarizine, pizotifen or alcohol.
There are limited data on an interaction with preparations containing ergotamine or another triptan/5-
HT1 receptor agonist. The increased risk of coronary vasospasms is a theoretical possibility and concomitant
administration is contraindicated.
The period of time that should elapse between the use of sumatriptan and ergotamine-containing
preparations or another triptan/5-HT1 receptor agonist is not known. This will also depend on the doses and
types of products used. The effects may be additive. It is advised to wait at least 24 hours following the use
of ergotamine-containing preparations or another triptan/5-HT1 receptor agonist before administering
sumatriptan. Conversely, it is advised to wait at least 6 hours following the use of sumatriptan before
administering an ergotamine-containing product and at least 24 hours before administering another
triptan/5-HT1 receptor agonist.
Undesirable effects can occur more frequently in cases of concomitant use of triptans and herbal
preparations containing St. John's wort (Hypericum perforatum).
An interaction may occur between sumatriptan and MAOIs and concomitant administration is
contraindicated.
There may be a risk of serotonergic syndrome also if sumatriptan is used concomitantly with lithium.
There have been rare post-marketing reports describing patients with serotonin syndrome (including altered
mental status, autonomic instability and neuromuscular abnormalities) following the use of SSRIs and
sumatriptan. Serotonin syndrome has also been reported following concomitant treatment with triptans
and SNRIs
Triptans and dihydroergotamine can cause coronary artery constriction and are thus contraindicated in
patients with coronary artery disease or uncontrolled hypertension; these drugs must be used with caution
in older patients and in patients with vascular risk factors. Lasmiditan, which has a much greater affinity for
serotonin 1F receptors than for 1B receptors, has no cardiovascular contraindications. (Triptans cause
vasoconstriction by activating 5-HT1B receptors.) Ubrogepant is an alternative; as of this time, it has no
cardiovascular precautions or contraindications and has no serious cardiovascular and gastrointestinal
effects.

130. How to manage a vaccine in the fridge, to keep it at its proper temperature
a) Keep it in a bar size fridge, just for pharmaceuticals
b) Adjust the temp from 0-8C
c) Regular size fridge but only keep it in the door
d) Remove any frost from the fridge
https://www.canada.ca/en/public-health/services/publications/healthy-living/national-vaccine-storage-handling-
guidelines-immunization-providers-2015.html#a3

131. Which is not a symptom of athlete’s foot?


Pain in the heel and soul of foot
132. Patient on clotrimazole for 1 week and feels better, what to tell him, all except
a) Switch his current therapy to tolnaftate
b) After lesions heal, can use tolnaftate prophylactic
c) Continue treatment for at least 2 weeks after lesions heal
Give patients the following instructions:
 Wash feet with soap and water every day.
 Dry feet thoroughly, paying special attention to areas between the toes.
 Change socks daily (more frequently if feet sweat).
 Wear socks made of natural, absorbent materials or synthetic blends, e.g., acrylic, cotton,
polypropylene, wool. Individuals with hyperhidrosis should avoid socks that are made of nylon.
 Avoid tight-fitting footwear.
 Allow shoes to dry completely before being worn again. This may take 2–3 days and it may be
necessary to alternate pairs of shoes on different days.
 Do not go barefoot in public places (e.g., swimming pool decks or gym change rooms)—wear foot
protection (e.g., sandals, pool shoes).
 Do not share personal items such as towels.
 Antiperspirants or absorbent powders (e.g., talcum or aluminum chloride) can be applied to the feet
to decrease sweating.
 In addition to the above measures, individuals with a history of tinea pedis may regularly apply a
dusting of antifungal powder such as tolnaftate once or twice daily on their feet to prevent further
recurrences.
 To prevent coagulation of powder and moisture buildup, antifungal powder should not be placed in
shoes.
Pharmacologic Therapy
Treatment of the skin with a topical antifungal agent (e.g., ciclopirox 1%, clotrimazole 1%, ketoconazole 2%,
miconazole 2%, terbinafine 1%) twice daily is the mainstay of therapy. Treatment typically continues for up
to 4 weeks, including 1–2 weeks after the lesions have disappeared, to prevent recurrences
Topical terbinafine 1% may have a slightly higher cure rate compared with other topical antifungals.
Inflamed infections may benefit from adding a topical anti-inflammatory such as betamethasone or
hydrocortisone; some antifungal and topical corticosteroid combinations exist commercially (e.g.,
clioquinol/ hydrocortisone, clioquinol/ flumethasone, clotrimazole/ betamethasone). If signs and
symptoms persist beyond 6 weeks, consider referral to a foot care specialist. The main causes of
treatment failure are incorrect diagnosis and inadequate treatment.
Oral fluconazole, itraconazole or terbinafine may be indicated for tinea pedis infections that are
resistant to topical treatment. Toenail involvement requires oral treatment.
There is no evidence that tea tree oil is effective in the treatment of tinea pedis, and if used it should not
be applied to open lesions.
The selection of dosage form is based on individual preference. Generally, ointments remain in contact
with the affected area for a longer period of time than creams; however, there is the danger of creating
an occlusive barrier, which promotes skin maceration and retards wound healing. Powders may be either
nonmedicated or medicated and are also absorbent. Solutions, sprays or foams applied directly to the
skin should be allowed to air dry.
133. Morphine and Heparin errors in the hospital pharmacy
a) Avoid purchasing potent conc or purchase ready made from manufacturer

134. Child has asthma, and he comes and his asthma is getting worse, hes on SABA, if you do
not recommend to go to the doctor to get an ICS, you violate what?
a) Beneficence
b) Autonomy
c) Veracity
d) Justice
e) Non-maleficence

135. Accutane Question, huge case. What is the first thing to monitor at baseline? TG

136. To manage his side effects, what to give? Lip balm emollient

137. What to monitor during and throughout the therapy? Liver functions
Correct (also liver, CBC & pregnancy test)

138. Sertraline for a depressed patient is on it for 4 months, experiencing a good response with
appetite and sleep, but got sudden cry spells, dry mouth and a bunch of depression symptoms.
What to tell her?
a) SSRI are more effective than TCA
b) Can use multiple SSRI together
c) Symptoms of sleep and appetite usually resolve first (1-2 weeks)

139. Which is probably caused by Sertraline


a) Dry mouth
b) Crying spells

140. All should be Empty stomach except


a) Cloxacillin
b) Efaverinze
c) Zafirlukast
d) Cefuroxime Auxetil
141. COPD patient wants to take Influenza. What to tell the patient
a) Tell him its ineffective after December
b) Not used for COPD
c) It can be used in most of the minor conditions
d) Preservative (thiomersal) will make symptoms worse
e) Only for very old, very sick and very young
Young, elderly, and patients with comorbidities

142. Patient coming to inquire about plan b, what is the most appropriate action
a) Offer her a private counselling area
b) Send her to the doctor

143. Patient is taking lorazepam 0.5 TID, she was 87yo and she has a caregiver...she had 90-day
supply, she came for an early refill for the second time now...What is the first most appropriate
thing to do?
a) Ask the caregiver for the administration report
b) Call the doctor and tell him there's abuse
c) Tell the nurse that there is drug diversion

144. What is the main reason for documenting in the community pharmacy?
a) To keep legal records of what happened
b) To enhance patient care
c) For legal purposes for the patient

145. Parkinson's disease, tell patient all except


a) Exercise
b) Counsel family and caregiver
c) Nutrition counselling

146. Diabetic patient wants to prevent any chance of getting diabetic foot, who do u invite
a) Nurse
b) Chiropractor
c) Orthopedist
d) Orthopath

147. Patient is 38 weeks pregnant and develops cystitis, what do u recommend


a) Amoxicillin
b) Cotrimoxazole
c) Nitrofurantoin
148. The point of “step down therapy” from IV to oral for what purpose
a) Increases compliance
b) Decrease resistance
c) Decreases nosocomial infections

149. Pharmacist has received a Rx consists of Hydrocortisone, Zn Oxide, Nitroglycerine 0.2 %


mitt 30 gm, he has nitroglycerine ointment 60 gm (2%) to prepare this Rx, how much
nitroglycerine will remain in the tube.
Answer:
C1*V1 = C2*V2 0.2%*30= 2%*V2 V2 = 0.2*30/2 = 3 gm
The remaining = 60-3 = 57 gm

150. Fiorinal c1/2 regulations


a) Verbal and written and no repeats
b) Verbal and written with repeats
c) Repeats with written prescription only
Depends on ingredients but generally no refills in narcotics. Fiorinal C0.5 & C0.25 are verbal narcotics
(Verbal narcotic part fill & no refills & no transfer)
While fiorinal only is controlled drugs part II (verbal& written with refills but no transfer)

151. Patient has A-fibb and his VVR is about 160...he also has BP 88/55, The dr. He is taking
Docusate sodium for his constipation and everything else is good with his health Wants to treat
his ventricular arrhythmia, all can be used except?
a) Amiodarone
b) Digoxin (contraindicated in ventricular arrythmia)
c) BB
Ans.C I think C bcz BB not recommended if BP < 90 (not sure)

152. If you want to control his VVR, we could give him?


a) Nifedepine
b) Metoprolol
c) Acebutalol
d) Verapamil
but if his blood pressure was corrected the first choice is BB

153. Patient has end stage breast cancer, she is taking morphine, what would u tell her
a) Addiction is not a concern for cancer patients
b) Cancer patients do not get addicted to opioids
154. Her daughter returns telling you that she still feels a lot pain and has been talking more
doses than prescribes, what to tell her
a) Since her pain is uncontrolled it is ok to take more than prescribed
b) Call the doctor to reassess her dose
c) Call the doctor and report abuse
d) Recommend to switch to fentanyl patch

155. Patient comes with symptoms of pinpoint pupil, no bowel movements, very low
respiration...this may be due to?
a) Cocaine
b) Narcotic Analgesics
c) Alcohol

156. Patient ingested acetaminophen tablets about 6hours ago, in which cause would he
develops acute toxicity?
a) 18 x 500mg tabs
b) 6 x 325mg
c) 12 x 500mg
d) 8 x 500mg
Max. Tolerated dose 4 gm, above 4 to 6 (7.5 gm) →hepatotoxicity, while 10 gm is fatal
The anti-dot is N-acetylcysteine & symptoms as acute hepatic necrosis.

157. What would you give him for his toxicity?


a) Protamine
b) NAC
c) Sodium bicarbonate

158. Patient has Osteoarthritis for 3 months ago and has been really healthy other than that...
and her doctor told her to self-medicate herself with acetaminophen so she has been taking
500mg TID prn, she heard from a friend that Ibuprofen may help relieve her symptoms better,
what is the best advice to give her?
a) Switch her to ibuprofen
b) Increase her dose of acetaminophen to 1000mg TID
c) Switch her to Tylenol 1 (caffeine and codeine)
d) Refer her to her doctor
159. Patient is travelling to USA, and wants to buy 1000 of pseudoephedrine 60mg and bring it
to his family and friends for the upcoming flu and cold season...All are ethical concerns for the
pharmacist except
a) Potential for drug abuse
b) Pseudoeph has many drug-drug interactions
c) Can only travel with medications for personal use only
d) There is No pharmacist-patient intervention with the other family members

160. Patient taking Ticaracillin/ Clavulinic 3.5 gm Q6H IV for an infection. Doctor decides he
wants to do a tooth extraction while in the hospital for the patient. What should you tell the
doctor regarding prophylaxis?
a) No need for prophylaxis as patient is already taking high dose of a broad-spectrum
antibiotic.
b) No need for prophylaxis as this is a minor procedure.
c) Give 2 gm of amox. 1 hour before procedure.
d) A dose of Clindamycin (can’t remember how much) before the procedure.
Drug Adult Dose Pediatric Dose
Standard Regimen Amoxycillin 2 g PO 50 mg/kg
Unable to Take Oral Ampicillin 2 g IM or IV 50 mg/kg IM or IV
Medications Cefazolin OR Ceftriaxone 1 g IM or IV 50 mg/kg IM or IV
Allergic to Penicillins Cephalexin 2 g PO 50 mg/kg
Clindamycin 600 mg PO 20 mg/kg
Azithromycin or Clarithromycin 500 mg PO 15 mg/kg
Allergic to Penicillins and Cefazolin or Ceftriaxone 1 g IM or IV 50 mg/kg IM or IV
Unable to Take Oral Clindamycin 600 mg IM or IV 20 mg/kg IM or IV
Medications

161. Patient has been stabilized after ACS, what not to monitor
a) Troponin levels (first 72 hours)
b) CBC
c) Chest pain

162. Drug is withdrawn from the market due to a bunch of reasons, what not to do?
a) Post a note in the pharmacy

163. Patient has nasal congestion, itchy eyes, water eyes, sneezing, and he started feeling
shortness of breath...usually his symptoms are only nasal congestion every year around this
time...he wants something that wont cause drowsiness...what do u recommended
a) Pseudoephedrine
b) Topical decongestants
c) Fexofenadine
d) Refer to the doctor
Ans.D as he has SOB

164. Patient is initiated on Antihypertensive drugs, when should you monitor the effect?
a) 4 days
b) 7days
c) 4 weeks
d) 6 weeks

165. Uncontrolled HTN may lead to all except


a) PAD
b) Tricuspid valve dysfunction
c) Hypertrophy
d) Stroke
e) Retinopathy

166. Domperidone has been questioned to cause Infertility, where would you look to find out
more about this topic
a) Micromedex
b) Drugs in Pregnancy and Lactation
c) Hansten and Hornes Drug Interactions Assessment and Management

167. Patient is receiving insulin and his BGL are


well controlled. His morning dose levels are
5.6mmol and his nightly dose is 6.3mmol. What
may be the cause for his 3am dose to be 11.3
mmol?
a) The Dawn phenomenon

168. All are considered goal of treatment for diabetic foot except?
a) Eradicate the infection
b) Relieve local pain
c) Allow mobilization on foot
d) Avoid pressure on foot
e) Enhance the blood flow to the leg
Cure the acute or chronic infection. Heal wounds (diabetic foot ulcers, traumatic wounds)
Restore limb function for activities of daily living with appropriate footwear and orthoses
Prevent recurrence
169. Gault Equation is useful when?
a) After an oral ingestion of creatine
b) Acute kidney deterioration for several days
c) Kidney dysfunction
In another format the answer is used when Cr is steady

170. Question on Patient taking Ferrous Gluconate TID at a dose of 300mg, what is the amount
of elemental iron in the the total daily dose
a) 35
b) 95
c) 55
d) 105
e) 900
(fumarate 100/300 – sulphate 60/300 – gluconate 35/300)

171. What should be monitored for his iron levels


a) Total iron binding capacity
b) Ferritin test
c) Hemoglobin concentration
d) Serum iron levels
Complete blood count (CBC), serum iron, iron-binding capacity, serum ferritin, transferrin saturation,
reticulocyte count, red cell distribution width (RDW), and a peripheral blood smear
Rarely bone marrow examination
Iron deficiency anemia is suspected in patients with chronic blood loss or microcytic anemia, particularly if
pica is present. In such patients, a CBC, serum iron and iron-binding capacity, and serum ferritin and
reticulocyte count are obtained (see table Typical Serum Values for Iron, Iron-Binding Capacity, Ferritin, and
Transferrin Saturation).
Iron and iron-binding capacity (and transferrin saturation) are measured because their relationship is
important. Various tests exist; the range of normal values relates to the test used and varies from
laboratory to laboratory. In general, normal serum iron is 75 to 150 mcg/dL (13 to 27 micromol/L) for men
and 60 to 140 mcg/dL (11 to 25 micromol/L) for women; total iron-binding capacity is 250 to 450 mcg/dL (45
to 81 micromol/L) and transferrin saturation is 20 to 50%. Serum iron level is low in iron deficiency and in
many chronic diseases and is elevated in hemolytic disorders and in iron-overload syndromes. The iron-
binding capacity increases in iron deficiency, while the transferrin saturation decreases.
Serum ferritin levels closely correlate with total body iron stores. The range of normal in most laboratories is
30 to 300 ng/mL (67.4 to 674.1 pmol/L), and the mean is 88 ng/mL (197.7 pmol/L) in men and 49 ng/mL
(110.1 pmol/L) in women. Low levels (< 12 ng/mL (27 pmol/L)) are specific for iron deficiency. However,
ferritin is an acute-phase reactant, and levels increase in inflammatory and infectious disorders (eg,
hepatitis), and neoplastic disorders (especially acute leukemia, Hodgkin lymphoma, and gastrointestinal
tract tumors). In these settings, a serum ferritin up to 100 ng/mL remains compatible with iron deficiency.
The reticulocyte count is low in iron deficiency. The peripheral smear generally reveals hypochromic red cells
with significant anisopoikilocytosis, which is reflected in a high red cell distribution width (RDW).
The most sensitive and specific criterion for iron-deficient erythropoiesis is absent bone marrow stores of
iron, although a bone marrow examination is rarely needed.

172. What schedule does this drug belong to according to NAPRA


a) U
b) 1
c) 2
d) 3

172. Patient has been prescribed Levothyroxine and takes his iron at breakfast with his
Levothyroxine, he will experience what?
a) Increase in TSH levels
b) Iron will not be absorbed because hes taking it on an empty stomach
c) Iron should be taken with a glass of milk
Absorption of levothyroxine (thyroxine) possibly reduced by antacids, proton pump inhibitors, calcium salts,
cimetidine, oral iron, sucralfate, colestipol, polystyrene sulphonate resin and cholestyramine (administration
should be separated by 4-5 hours).

173. Patient is being placed on 10mg Atorvastatin however she is very reluctant to starting the
statin therapy because her Dad has been using statins and had really bad muscle aches and
myopathy, what is the initial most appropriate approach the pharmacist will do?
a) Acknowledge her reluctance to taking the statin
b) Evaluate her chances of getting myopathy
c) Tell her its a rare side effect and have no worries
d) Tell her it is a dose related side effect and she should not worry

174. A pharmacy employee has noticed that his co-worker has been abusing his sick days lately,
as a manager of the pharmacy what is the best approach to go about this situation?
a) Tell him according to human resources it is his right and we can’t do anything about it
b) Tell him sick-days is part of the the workers right and i can not interfere
c) Monitor and review the recent sick days report and if you notice a trend in workers
abusing or taking too many sick days, then talk to the accused about the situation
d) Leave a letter in the accused worker’s letter box informing him of the accusation
175. Common Drug review (K-type)
a) Used to help the manufacturers better market their products for better sales
b) Recommends to the provincial drug plans on which drugs should be included in the
formulary
c) Pharmacoeconomics

176. Patient was on Levothyroxine 0.15mg, he was admitted to the hospital and upon contacting
his physician they discovered that he should be on 0.5mg instead and his dose was increased a
few months ago...how ever the patient had lost his prescription and the pharmacist just gave him
a repeat of what he originally had on his profile. Using a root and cause analysis, where can you
identify where the error occurred
a) There is no 0.5mg formulation
b) There was no dialogue between the physician and the patient regarding the change
of his dose to a higher dose
c) Pharmacist did not double check the patients’ profile and never compared it to the
original hard copy originally received from the patient
d) Pharmacist did not double check to see what the patient was taking on his profile

177. 1g of dextrose gives 3.4kcal, but we have 100g of Dextrose, so what will be the total
Calories
a) 0.34
b) 3.4
c) 340
d) 3400
e) 340 000

178. Patient comes in, 60yo, inquiring about the varicella zoster vaccine. His friend recently got
shingles and got herpes zoster neuralgia and was in too much pain. This patient wants to know
if this vaccine will reduce his risk of developing this condition but he hears that it is very
expensive, what do you recommend for him as a pharmacist based on the values provided
Gave you a bunch of Placebo risks and treatment risks and absolute risk reductions and all these
numbers
Calculate NNT...very easy – 1/ARR Answer was 357

179. What can you tell this patient about the vaccine?
a) Relative risk reduction is 66% so it is highly recommended
b) Absoulte risk reduction is ______________ so it is highly recommended
c) Tell him what all the values mean and make him compare the benefits with the cost
C may be correct bcz it`s pt. right to know all the info then decides!!
180. Patient then develops a rash on his arm and started experiencing acute pain on the site. He
was prescribed Acyclovir orally and started his therapy, what may help with his symptoms
a) Topical acyclovir will help prevent him from developing neuralgia
b) Topical acyclovir will treat the rash and increase healing time
c) Tell him to avoid contact with any one who have not had chicken pox before
d) Give him a standard dose opioid for his acute pain
Pharmacologic Choices
Antivirals
Systemic antiviral nucleoside analogues (acyclovir, famciclovir, valacyclovir) initiated within 72 hours of rash
onset reduce the duration of viral shedding, acute pain and the appearance of new lesions.
Increasing age (patients <50 years of age rarely develop postherpetic neuralgia),[60] severe pain and
extensive disease correlate with an increased risk of postherpetic neuralgia.
Oral acyclovir does not significantly reduce the risk of postherpetic neuralgia, and the effect of the other
nucleoside analogues on this sequela is not known.
Topical antivirals are not effective in the treatment of herpes zoster.
To prevent ocular complications, treat patients with ocular zoster even if the rash has been present for up to
7 days. Promptly refer patients with ocular complications to an ophthalmologist. Famciclovir and
valacyclovir may improve patient adherence because they can be given less frequently, e.g., TID dosing.
Patients with severe immunosuppression (such as hematopoietic stem-cell transplant recipients or patients
with lymphoproliferative malignancies) are at highest risk of VZV dissemination and visceral organ
involvement. Disseminated infection is defined as a widespread rash (>20 skin lesions) affecting 3 or more
dermatomes or spreading beyond the primarily affected and adjacent dermatome. Visceral dissemination
most often involves the CNS (e.g., cerebellar ataxia, encephalitis, transverse myelitis) or the lungs (e.g., viral
pneumonitis).
The drug of choice for treating dissemination in immunocompromised patients is acyclovir 10–15 mg/kg IV
every 8 hours. When the infection is under control, switch therapy to an oral antiviral drug until healing is
complete or for a minimum of 10–14 days (whichever is longer) to reduce the risk of disease relapse.
In patients co-infected with HIV and HSV-2, viral suppressive therapy with oral acyclovir 400 mg twice daily
was found to reduce herpes zoster incidence by 62% compared with placebo.
Corticosteroids, Analgesics and Opioids
Prevention of postherpetic neuralgia using antiviral therapy is not absolute. Analgesics, opioids, gabapentin,
pregabalin and corticosteroids may be used in the treatment of acute pain related to herpes zoster.
Corticosteroid therapy improves quality of life (resolution of acute neuritis, uninterrupted sleep and return
to normal activity) but does not accelerate healing or reduce the incidence of postherpetic neuralgia
compared with acyclovir alone.
The risk of immunosuppression with corticosteroids may hinder their use in high-risk patients, e.g., elderly,
patients with diabetes, hypertension, GI ulcers. Therefore, limit use of corticosteroids to healthy patients
with moderate-to-severe pain.
181. He then developed herpes zoster neuralgia, what is the best option that this patient will
require to treat his symptoms PHN
a) Gabapentin
b) Pregabalin
c) Tramadol
d) Codeine
e) Amitriptyline
Chronic Peripheral Neuropathic Pain
Although the evidence for the recommended therapeutic approaches is mainly from studies of patients with
postherpetic neuralgia and diabetic neuropathy, patients with phantom limb pain or other chronic
peripheral neuropathic pain may derive benefit. Tricyclic antidepressants (TCAs), gabapentinoids such as
gabapentin and pregabalin, and SNRI antidepressants (duloxetine, venlafaxine) are considered first-line
agents. When initiating pharmacotherapy, initial follow-up should be arranged in about 2 weeks and
regularly thereafter for monitoring.
Several RCTs indicate that pain may be taken from moderate or severe to mild in about one-half to two-
thirds of patients by a TCA. In a systematic review, SNRIs were estimated to have an NNT of 6.4—meaning
that about 1 of every 6 patients treated with the drugs would report clinically meaningful pain relief.
In this same review, gabapentinoids had an NNT of ~7 while TCAs had an estimated NNT of 3.6, suggesting
better efficacy than SNRIs and gabapentinoids. However, TCAs are known to have a higher rate of troubling
side effects.
Tramadol and topical lidocaine can be considered as second line. Combination therapy can be helpful. Do
not combine TCAs with SNRIs because of the potential risk of serotonin syndrome.
Management of postherpetic neuralgia
Management of postherpetic neuralgia can be particularly difficult. Treatments include gabapentin,
pregabalin, cyclic antidepressants, topical capsaicin or lidocaine ointment, and botulinum toxin injection.
Opioid analgesics may be necessary. Intrathecal methylprednisolone may be of benefit.

182. Huge case on an alcoholic patient admitted to the hospital...he is having withdrawal
symptoms and so on. What is a symptom that this patient may experience?
a) Seizures. Correct plus hyperthermia

183. What can you say about alcoholic withdrawal syndrome?


a) Alcoholic withdrawal symptoms treated with benzodiazepine
b) Opiod withdrawal is much more dangerous
c) Opioid withdrawal can be treated by multidisciplinary therapy

184. He then develops Hypophosphatemia, how would you treat it


a) Effervescent sodium phosphate
b) IV phosphate
c) Another form of phosphate
d) Milk
Hypophosphatemia is a serum phosphate concentration < 2.5 mg/dL (0.81 mmol/L). Causes include alcohol
use disorder, burns, starvation, and diuretic use. Clinical features include muscle weakness, respiratory
failure, and heart failure; seizures and coma can occur. Diagnosis is by serum phosphate concentration.
Treatment consists of phosphate supplementation
Symptoms and Signs
Although hypophosphatemia usually is asymptomatic, anorexia, muscle weakness, and osteomalacia can
occur in severe chronic depletion. Serious neuromuscular disturbances may occur, including progressive
encephalopathy, seizures, coma, and death. The muscle weakness of profound hypophosphatemia may be
accompanied by rhabdomyolysis, especially in acute alcoholism.
Hematologic disturbances of profound hypophosphatemia include hemolytic anemia, decreased release of
oxygen from hemoglobin, and impaired leukocyte and platelet function.
Treatment
 Treat underlying disorder
 Oral phosphate replacement
 IV phosphate when serum phosphate is < 1 mg/dL (< 0.32 mmol/L) or symptoms are severe
Oral treatment
Treatment of the underlying disorder and oral phosphate replacement are usually adequate in
asymptomatic patients, even when the serum concentration is very low. Phosphate can be given in doses up
to about 1 g orally 3 times a day in tablets containing sodium phosphate or potassium phosphate. Oral
sodium phosphate or potassium phosphate may be poorly tolerated because of diarrhea. Ingestion of 1 L of
low-fat or skim milk provides 1 g of phosphate and may be more acceptable. Removal of the cause of
hypophosphatemia may include stopping phosphate-binding antacids or diuretics or correcting
hypomagnesemia.
Parenteral treatment
Parenteral phosphate is usually given IV. It should be administered in any of the following circumstances:
 When serum phosphate is < 1 mg/dL (< 0.32 mmol/L)
 Rhabdomyolysis, hemolysis, or central nervous system symptoms are present
 Oral replacement is not feasible due to underlying disorder
IV administration of potassium phosphate (as buffered mix of K2HPO4 and KH2PO4) is relatively safe when
renal function is well preserved. Parenteral potassium phosphate contains 93 mg (3 mmol) phosphorus and
170 mg (4.4 mEq) potassium per mL. The usual dose is 0.5 mmol phosphorus/kg (0.17 mL/kg) IV over 6
hours. Patients with alcohol use disorder may require ≥ 1 g/day during total parenteral nutrition;
supplemental phosphate is stopped when oral intake is resumed.
If patients have impaired renal function or serum potassium > 4 mEq/L (> 4 mmol/L), sodium phosphate
preparations generally should be used; these preparations also contain 3 mmol/mL of phosphorus and are
thus given at the same dose.
Serum calcium and phosphate concentrations should be monitored during therapy, particularly when
phosphate is given IV or to patients with impaired renal function. In most cases, no more than 7 mg/kg
(about 500 mg for a 70-kg adult) of phosphate should be given over 6 hours. Close monitoring is done, and
more rapid rates of phosphate administration should be avoided to
prevent hypocalcemia, hyperphosphatemia, and metastatic calcification due to excessive calcium phosphate
product.
Key Points

 Acute hypophosphatemia most often occurs in patients with of alcohol use disorder, burns, or
starvation.
 Acute severe hypophosphatemia can cause serious neuromuscular disturbances, rhabdomyolysis,
seizures, coma, and death.
 Chronic hypophosphatemia may be due to hormonal disorders (eg, hyperparathyroidism, Cushing
syndrome, hypothyroidism), chronic diuretic use, or use of aluminum-containing antacids by patients
with chronic kidney disease.
 Hypophosphatemia is usually asymptomatic, but severe depletion can cause anorexia, muscle
weakness, and osteomalacia.
 Treat the underlying disorder, but some patients require oral, or rarely, IV phosphate replacement.

185. Upon discharge from the hospital, who would you recommend for this patient
a) Social worker
b) Nutritionist
c) Occupational therapist
d) Physiotherapist

186. Patient on asthma medication (all MDI’s) and got chest infection, his doctor decides to
start him on antibiotics and discontinue his LABA MDI and his Fluticasone MDI and put him
on a combination diskus...what do you check for in the patient
a) Past history of chest infections
b) Don’t remember this one but i think it was all 3
c) His previous asthma medication technique
187. Then the doctor put him on a diskus, what do u tell him all except
a) Wipe the mouth piece with a wet cloth
b) Once you hear a click the dose is loaded
c) It contains a dry powder medication

188. What will be present in the “P” part of SOAP


a) Allergies
b) History of smoking
c) Blood test
d) Chest x-ray
e) Initiation of antifungal therapy (PLAN)

189. How do you monitor the effectiveness of Furosemide?


a) Fluid input
b) Fluid output
c) Weight

190. Patient being prescribed mefloquine and naproxen, given many options and choose the
correct statement
a) You tell the patient not to take naproxen as it would interact with mefloquine
b) Counsel the patient to take the mefloquine at least 1 week before travel

191. The repeated question of Neofloxacin? I think related to Ca interaction

192. A patient has been stabilized on Li, has been prescribed ACEI, what to do?
a) Stop Li as the ACEI will increase its levels (don’t stop Li if he’s stabilized on it)
b) Monitor for signs of toxicity of Li as ACEI will inc. Li levels

193. D.O.C (drug of choice) for DVT prevention after hip replacement surgery:
a) Argotroban
b) Rivaroxoban
c) Leprutide
d) ASA
e) Clopidogrel
Apixaban, dabigatran and rivaroxaban are direct-acting oral anticoagulants that can be used for VTE
prophylaxis following elective total hip or total knee replacement surgery. Apixaban and rivaroxaban have
demonstrated superiority over enoxaparin.
194. Female on rizatriptan for her migraine, what is the DOC. For her to treat trigeminal
neuralgia?
a) CBZ (carbamazepine)
b) Oxcarbazepine
c) Gabapentin
d) Baclofen

195. Strongest Meta-analysis?


a) Randomized Control Trials (RCT) only {I’ve chosen this one bec. Case reports are
weak}
b) Mixture of RCT and case report

196. A female is on Lamotrigine and Divalproex wants to get pregnant, what to to do?
a) Don’t get pregnant unless 2 years has passed
b) Take folic acid before, during and after pregnancy
Review folate dose and timing

197. To change an antidepressant medication for a patient from Venlafaxine to Fluoxetine:


a) Stop venlafaxine and start fluoxetine immediately
b) Taper venlafaxine over 1 wk
c) Taper venlafaxine over 5 wks
Shifting from desvenlafaxine duloxetine venlafaxine (SNRIs) to citalopram, escitalopram, paroxetine and
sertraline: taper SNRI, start above SSRI at low dose*
Shifting from SNRI to fluoxetine: taper and stop SNRI, start fluoxetine at 10 mg
https://www.nps.org.au/assets/Products/Guidelines-switching-antidepressants_A3.pdf

198. A patient travelling to an area with low sanitation, all are may be infections except:
a) Entamoeba
b) Giardiasis
c) Hepatitis A
d) Hepatitis B
e) Fecalis

199. A diabetic patient’s wife called telling the pharmacist that her husband’s blood sugar level
is less than 2.8. What to tell her?
a) Give him sugar
b) Take him to ER
Early signs of hypoglycemia include sweating, hunger, nausea, shakiness, heart palpitations (heavy, fast
heartbeats), anxiety, feeling irritable, mood or behaviour changes, numb lips or tongue, and headache.
Tips to manage hypoglycemia:
 Always carry a source of fast sugar (such as glucose tablets) and a snack (such as 6 crackers and cheese or
peanut butter).
 At the first sign(s) of low blood sugar, check your blood glucose value and if less than 4 mmol/L eat one of
the following items that contain 15 grams of fast sugar (carbohydrate) right away:
 4 Dex4 glucose tablets
 6 Life Savers candies (chewed)
 1 tablespoon (15 mL) or 3 packets of sugar dissolved in water, or 1 tablespoon of honey
 3/4 cup (6 oz; 180 mL) juice or regular soft drink
 Think ‘15-15-15’ to treat hypoglycemia; treat with 1 of the above sources of 15 grams of fast sugar, wait 15
minutes and then retest your blood glucose. If your blood glucose is still less than 4 mmol/L, take 1 of the
items containing 15 grams of fast sugar listed above again. Eat your next meal at the regular time. If your
next mealtime is more than 1 hour away, eat a snack with 15 grams of carbohydrate and 1 source of protein
to prevent the return of hypoglycemia.
 If the hypoglycemia is severe (you need help from another person), eat 20 grams of carbohydrate (for
example 5 glucose tablets or 4 packets of sugar). Wait 15 minutes and then retest your blood glucose. If
your blood glucose is still less than 4 mmol/L, take 15 grams of carbohydrate.
 If you are having hypoglycemia and are unconscious or not able to swallow, caregivers or support persons
should get medical attention for you as soon as possible. Your diabetes health-care team should also be told
about the event.
 If you are taking the medication acarbose (Prandase/Glucobay) and have hypoglycemia, you must take
glucose tablets, milk or honey. Acarbose prevents other sugars from being quickly absorbed.
 For those at risk of severe hypoglycemia, support persons should be taught how to administer glucagon by
injection.
 It is important not to overtreat hypoglycemia. This can lead to high blood glucose and weight gain. Fifteen
grams of glucose will raise blood sugar by 2 mmol/L within 20 minutes.
 Avoid driving immediately following hypoglycemia. Be aware of provincial driving regulations
regarding hypoglycemia.
 Wear a MedicAlert bracelet or other diabetes identification at all times, particularly if you don’t recognize
the early signs of hypoglycemia easily.

200. Allergic rhinitis not controlled, patient on mometasone. Why would you call the doctor?
a) To recommend oral decongestant
b) To recommend nasal ipratropium
I think to recommend systemic CS or immunotherapy

201. Asthma patient with acute exacerbation in hospital, what to give?


a) I.V. methyl prednisolone
b) Inhaled salbutamol
May be correct but still I have to give him SABA may be plus ipratropium
201. What should the dr. prescribe to help manage similar future situations?
a) Oral prednisone Not sure depends on other options

202. COPD patient on salbutamol and is not controlled what should he take?
a) LABA Correct if tiotropium isn`t an option

203. Which of the following require to alert the patient about handling due to toxicity?
a) Hyalourinasae
b) Digoxin
c) Cool tar
d) Hudroxyurea
e) Methylsalicylate
f) Finasteride

204. Factor affecting the t1/2?


a) Css
b) Vd

205. Diabetic patient with ulcer in his foot, it is oozing, with discoloration. What kind of
infection is it?
a) Cellulitis
b) Diabetic foot
c) Osteomyelitis

206. Refrigeration condition: (can’t remember if it was k or A type)


a) 0-8 degrees C
b) Prevent frost from formation
I think it should be 2-8 degree, cool (8-15), room temp (15-25), freezing (-10)-TEFA

207. Reference questions:


Storage condition for a new medication: Drug manufacture
Off label use: pubmed. Adverse drug reaction: Medeffect website

208. CHF questions: A case of a man with description of the effort he can only do, what class of
NYHA is that:
a) I
b) II
c) III
d) IV
209. Counseling for him:
a) Restrict Na intake

210. A case of a patient that has been prescribed Tramadol and is taking rizatriptan, you will
recommend what to the patient?
a) Call the doctor to change tramadol as it interacts with rizatriptan
Tramadol is a 5-HT / NI reuptake inhibitor (serotonin synd)

211. A patient being prescribed Testosterone oral capsule, counseling (k-type)


a) Fridge  ( don’t refrigerate)
b) Empty stomach  (food enhances its bioavailability)
c) Worsening of BPH symptoms  (a side effect)

212. In a patient with an infection (not sure was it CAP or UTI) the step-down approach from
I.V to oral ttt in patient with normal body temperature and able to swallow is:
a) Normal WBC
b) Hemodynamically stable
If CAP we switch when GIT stable (no vomiting or diarrhea), a febrile for 16 hrs normal WBC, improvement in
cough & SOB and able to swallow oral.
So, if k-type I & III are correct and if A type I will choose A.

213. Reference which provides you with a list for nephrotoxic medications?
a) DPD
b) Clinical drug data

214. Reference for patient counseling:


a) e-CPS
b) Martindale

215. Upon forgery discovery after 6 months, what’s you FIRST action:
a) Call the RCMP (police)
b) Notify health Canada (OSC)
c) Ask the supplier if there is 1 bottle you forgot to supply

216. After ttt failure with pyrantel pamoate, what to do?


a) Refer to doc. To prescribe mebindazole
217. 2 cases for menopausal women, remember 1 only the one for osteoporosis: female patient
With blood pressure 150/95, with family history for breast cancer, but her mammogram is
normal. What’s the D.O.C?
a) Estrogen {I’ve chosen that answer bec. She’s under 160/100 bp, no self history of cancer
and estrogen will help with her menopausal symptoms}
b) Raloxifene
May be correct if patient is premenopause or early menopause with symptoms otherwise Raloxifene will be
better.

218. Hyponatremia causes what?


a) Seizures
b) Renal failure

219. A list of drugs is given and the question is which of these medications can cause SIADH
a) Paroxetine Correct (SSRI, Thiazides, Carbamazepine)

220. What is the ttt of SIADH? Hypertonic solution

221. What causes danger if injected I.V?


a) Dextrose 10 %
b) Nacl salt 0.45 %
c) Saline 0.9 %
d) Mixture of dextrose and saline
e) Sterile water for injection

222. Atrial fibrillation patient DOC? Digoxin


Amiodarone, flecainide sotalol, propafenone for rhythm control. BB & CC blockers for rate control.

223. A female with cold symptoms and in her profile, she has ASTHMA, what to do?
a) Refer (whenever you find an OTC case with a TC medical history u refer or think about
it as any combination of 2 chapters you refer: D)

224. List of medication, what causes increase in the heart rate? Salbutamol

225. All can cause weight gain EXCEPT? ROCQ


a) Lamotrigine (weight gain)
b) Risperidone
c) Aripiprazole
d) Quetiapine
All antipsychotics cause weight gain
226. What decreases cyclosporine levels? Phenytoin(inducer)

227. Metronidazole counseling;


a) Use condom
b) Use sunscreen bec. It causes photosensitivity
Correct is it is only in amber glass and condom if indicated in trichomoniasis

228. Patient with alcoholic encephalopathy and the doctor put him on lactulose, what’s the
endpoint that would make the doc. Stop the lactulose?
a) Stop bleeding
b) Return to consciousness
In another format it was end of psychosis which may be correct

229. List of medications in dosette, and tell you what’s your concern?
a) Zopiclone at lunch
b) Vitamin at bedtime
c) Amitriptyline PRN

230. HCTZ will:


a) Inc. LDL only
b) Inc. LDL and TG
c) Inc. TG only
d) Dec. LDL
Answer should be B (checked from cps monograph)

231. Furosemide will cause:


a) Hypocalcaemia
b) Hypercalcemia

232. Newfloxacin question. It is conc dependent killing like other fluoroquinolones. t1/2 is 12
hrs but dosing studied is 24hr. How would u adjust the dose in renal failure
a) Decrease dose keep the interval
b) Keep the dose increase the interval (dose dependent)
c) Keep the dose decrease the interval

233. How would u study the efficacy of the newfloxacin


a) Randomized trial control, comparative, not against placebo
b) Systemic review
c) Case series
234. Pharmacy manager wants to do narcotic medication record reconciliation. What policy he
should develop 1st?
a) Assign one pharmacy staff responsibility to recount and reconcile narcotic record
regularly
b) Print the sales record from computer system reconcile it with the paper record of the
narcotic

235. There was a question abt accuracy of two diagnostic procedure calculation...gave two
diagnostic test and number for false positive and false negative, true positive and true negative...

Diagnostic A
Diagnostic B
a) X %
b) Y %
c) Z %
http://en.wikipedia.org/wiki/Positive_predictive_value

236. Crohn’s disease pt. having severe flare up like diarhea, fever, etc. Already used 5-ASA,
AZA, budesonide. Last 2 wk he had flare up and was treated with prednisone, but now again he
has flare up. Why would u include infliximab in the therapy?
a) pt condition is not responding to corticosteroid
Patients rarely require chronic treatment with low-dose prednisone or purine antimetabolites. Biologics
are often not needed, but are effective in inducing and maintaining a clinical response in refractory
ulcerative proctosigmoiditis. Colectomy may be necessary in some patients despite the limited extent
of the disease.
Anti-TNF drugs
Infliximab, certolizumab, adalimumab, and golimumab are antibodies to tumor necrosis factor
(TNF). Infliximab, certolizumab, and adalimumab are useful in Crohn disease, particularly in preventing or
retarding postoperative recurrence. Infliximab, adalimumab, and golimumab are beneficial in ulcerative
colitis for refractory or corticosteroid-dependent disease.

237. MD wants to give him methotrexate. What do you monitor?


a) Blood work. Also, LFT

238. When preparing infliximab what would u tell the technician how to prepare the infusion?
a) Mix with other infusion as it is going to reduce the volume infused
b) Don’t shake it, as it is going to destroy the protein in it
239. As NAPRA developed regulation for pharmacy technician, what is the 1 st goal of making
this regulation? Guidelines and scope of practice
a) It provides the entry level technical skill required for the technician
b) Technicians are going to apply their skills in patient care
c) Technicians will be responsible for their therapeutic skill
d) They are going to supervise the pharmacy intern and students

240. An OTC cough and cold pharmaceutical company is going to fund an educational session
on cough and cold session. which action will make it optimal (LEAST)conflict of interest???
a) If u will provide info in the session about only the medication manufactured by the
company which had evidenced based
b) You will talk abt only the company meds which is equal or superior quality from the
comparator
c) If pharmacy staff don’t get any personal benefit from the funding

241. Isotretinoin—what would u monitor baseline starting the therapy for a teenage patient??
a) LFT
b) CBC
Ans: 1 st Pregnancy test. 2 nd Triglyceride level, 3 rd pancreatic test-amylase test.
TG 1st, liver functions 2nd

242. What would you monitor in the patient if he continues to take the isotretinoin??
a) LDL
b) Fasting glucose

243. What would u recommend the patient to use regularly in the above case???
a) Lipbalm  coz of photosensitivity s/e.
b) Face wash

244. Pt profile—Mother and father both are chain smokers. Mother had fracture. Obese and has
GERD (somethingelse also don’t remember), taking omeprazole, calcium carbonate, vitamin D.
pt is usually stressed how to manage his health. What will be beneficial for him?
a) Start bisphosphonate for her as osteoporosis prophylasix
b) Change calcium carbonate to calcium citrate

245. In the above patient what is the risk factor for hyperlipidemia?
a) Obesity
b) Family hx of smoking
246. After 6 month the pt came to ur pharmacy with an atorvastain Rx and his/ her LDL was
3.5, but the patient is reluctant to take the medication coz his/her father was using the same
medication and had very bad muscle pain. What is the first thing to do???
a) Acknowledge her complain abt fear abt s/e
b) Asses her 10yr- cv risk
c) Tell her that this s/e only occurs @ high dose

247. 8 m pregnant lady had cystitis pending result of culture and sensitivity, what can she take?
a) Nitrofurantion (contraindicated from 38 to 40)
b) SMX/TMP
c) Cephalosporin (don’t remember the exact one)
d) Amoxicillin
e) Ciprofloxacin
Ans: C- cephalexine…as amoxicillin is used if known culture sensitivity…n here it is pending. Nitrofurantoin is
avoided near term. Smx/tmp avoided in first term, smx in last 6 wks. Fluoroquinolones is CI.

248. Where can u find the potential drug interaction of a newly marketed drug?
a) Manufacture’s leaflet
b) Health canada drug data base
c) Therapeutic choices

249. Rheumatoid arthritis pt has 3 inflammed joints, morning stiffness >1hr. He was using HCQ
200mg BID for 6 months. did not get relievd from symptoms. now methotretaxe has been
introduces 15mg q wk. why the drug therapy was not helping the patient?? The pt wad also
taking other meds for pain (don’t remember)
a) Too lilte drug con cog HCQ
b) Too less time for drug trial
c) Less drug conc in the body d/t Drug interactions
d) HCQ was not effective for disease severity
Ans: D…HCQ is good for less than 2 swollen joints… (mild RA, not monotherapy)

250. What would you council the patient for


methotrexate??
a) Wear a wig as it had dramatic alopecia S/E
b) Take it on empty stomach as it will increase
absorption
c) Takes 2weeks to show full effect
d) Takes 3 months to show effect
251. If the pt took methotrexate qd instead of qwk for 2 wks. what s/e will be noticed?
a) Bone marrow suppression
S/E: bone marrow/ liver toxicity, pneumonitis, immunosuppression, malignancy.

252. pt had heart attack 6 years ago. Also


has hypertension, hyperlipidemia, taking
metoprolol, candesartan (one ACEI/ARB
not sure), statin, hydralazine. What is the
therapy problem?
a) Taking medication not for valid
indication
Note: hydralazine…it is not necessary in this pt.
The combination of isosorbide dinitrate plus
hydralazine reduces mortality and morbidity in
black patients with NYHA class III–IV HF
and is recommended in addition to
standard therapy (ACE inhibitor, beta-
blocker with or without MRA) in this
setting. Use of this combination may also
be considered in black patients with
NYHA class II HF and in other HF patients
who do not tolerate ACE inhibitors, ARBs
or an ARNI. Nitrate monotherapy is
valuable in treating symptoms of angina,
paroxysmal nocturnal dyspnea and
orthopnea. Nitrates have not been shown
to reduce mortality in the absence of
hydralazine. Consider hydralazine /
isosorbide dinitrate therapy in the patient
who cannot tolerate an ACEI or ARB.

253. Hydralazine is shortage. which medication had the most similar mechanism of action??
a) Minoxidil
b) Doxazosin
c) Ramipril
Vasodilator drugs can be classified based on their site of action (arterial versus venous) or by mechanism of
action. Some drugs primarily dilate resistance vessels (arterial dilators; e.g., hydralazine), while others
primarily affect venous capacitance vessels (venous dilators; e.g., nitroglycerine). Most vasodilator drugs,
however, have mixed arterial and venous dilator properties (mixed dilators; e.g., alpha-adrenoceptor
antagonists, angiotensin converting enzyme inhibitors).
254. What wouldn’t you ask pt with UTI to asses
the condition or what is not symptom of cystitis:
a) Flank pain
b) Suprapubic discomfort
c) Dysuria
d) Costal muscle pain
e) Hematuria

255. HIV pt taking medn zidovudine (not sure) +


tenofovir+ efavirenz. What to monitor except.
a) LFT
b) Renal function
c) Phosphate
d) TSH

256. What is the thing that we should asses for aseptic prep
a) Personal technique
b) Airflow in clean room

257. Which medication is contraindicated with zomig (zolmitriptan) nasal spray (K-type)?
I. Caffeine
II. Tyoenol# 1 (acetaminophen/caffeine/codeine)
III. Caffergot (caffeine/ergotamine)

pt is having gout attack (which he has been geeting very frequesntly now). Doctor prescribed
him Colchicine 0.6mg two tablet stat then 1 tab for 7 days. Allopurinol 1 tab daily. He also has
hyperlipidemia, blood pressure and taking a statin and ACEI. What is the DTP in this case?
a) ACEI interacts with Allopurinol
b) Need to change statin
c) Allopurinol and dx interaction
Because allopurinol should not start in acute attack, it can be added as a maintenance therapy. There is
drug int between colchicine and statin, which increase statin toxicity- myotoxicity.

258. In the above pt. what would your advice the pt. to AVOID?
a) Taking alcohol with colchicine
b) Taking colchicine with food
c) Operating heavy machinery while taking colchicine
d) Avoid grapefruit juice with colchicine
Avoid alcohol in gout. Colchicine is CI in pt. with renal and hepatic impairment who are also taking CYP3A4
or P-gp (cyclosporine) inhibitors. Fatalities reported with clarithromycin which inhibits both of above.
Drug-Food Interactions
Studies of colchicine and grapefruit have conflicting results. Avoid grapefruit, Seville orange, lime and
pomelo. If grapefruit is consumed, clinical monitoring is required, especially for vomiting, alopecia and
pancytopenia.

259. Metronidazole inhibits CYP 2C9. which of the following medications has an interaction
with metronidazole??
a) Furosemide
b) Gliclazide

260. What should you use in prinzmetal angina?


a) Nifedipine
b) Isosorbide dinitrate
c) Furosemide
d) Metoprolol
Prinzmetal's angina is treated with medications, specifically nitroglycerin (NTG), long-acting nitrates, and
calcium channel blockers ” all of which widen or open the blood vessels and improve blood and oxygen flow
to the heart muscle. Calcium channel blockers are generally prescribed first. If you continue to experience
episodes of chest pain, then a different class of calcium channel blocker or a long-acting nitrate may be
given. For people who do not respond well to either of these drugs, alpha blockers can be prescribed. Alpha
blockers lower blood pressure and reduce blood vessel narrowing by blocking the action of stress hormones.
Nitroglycerin is used when you are experiencing a spasm to stop it, thereby alleviating pain; it is not used as
a preventive medication. Nitroglycerin is usually given as a spray that you use in your mouth or a tablet that
you place under your tongue.
Risk factor modification is also important because most people with Prinzmetal's angina also have heart
disease and are therefore at risk for a heart attack. It is especially important to stop smoking because
smoking increases the likelihood of coronary artery spasms.5 By lowering your risk factors, you will reduce
your chances of developing atherosclerosis. In addition, people diagnosed with Prinzmetal's angina should
avoid exposure to cold and high stress situations since these can trigger spasms.
Treatment
 Calcium channel blockers
 Sublingual nitroglycerin
Average survival at 5 yr is 89 to 97%, but mortality risk is greater for patients with both variant angina and
atherosclerotic coronary artery obstruction. Risk increases with increasing obstruction.
Usually, sublingual nitroglycerin promptly relieves variant angina. Calcium channel blockers may effectively
prevent symptoms. Theoretically, beta-blockers may exacerbate spasm by allowing unopposed alpha-
adrenergic vasoconstriction, but this effect has not been proven clinically.
Oral drugs most commonly used are calcium channel blockers:
 Sustained-release diltiazem 120 to 540 mg once/day
 Sustained-release verapamil 120 to 480 mg once/day (dose must be reduced in patients with renal or
hepatic dysfunction)
 Amlodipine 5 to 10 mg once/day (dose must be reduced in elderly patients and in patients with
hepatic dysfunction)
In refractory cases, amiodarone may be useful.
Although all these drugs relieve symptoms, they do not appear to alter prognosis.

261. what is the minimum wt required for a weigh scale is it has 5% accurary & 0.45 sesitivity?

262. A child had ADHD. He is now going to be treated with methyphenidate 1mg TID. The
mother is worried about the abuse potential of the medication. As a pharmacist what should
your advice her
a) It has less abuse potential than long acing stimulant
b) Call the doctor to change the medication to lower abuse potential stimulant.
c) Tell the mother that abuse is less of an importance if the child will be properly
treated
d) Advice this mother not to give this medication to the child

263. What is not goal of therapy of ADHD:


a) Improve athletic performance
b) Decrease impulsivity
c) Decrease inattention

264. Fluoroquinolone class effect CHECK


a) Alopecia
b) Glycemic abnormalities
c) Nephrotoxity
d) Thrombocytopenia
e) Rhabdomyolysis

265. As a pharmacist u noticed that psychiatric pt are not compliance to their medication and
having D/C syndrome. What is the best action to take to 1 st detect any compliance issue?
a) Ask the pt’s family members abt their compliance
b) Check the history of their refilled medication and detect any late refills and act
accordingly
c) Give them automated compliance packs
266. pt. is having pneumonia attack and the suspected organism is Pseudomonas aeruginosa.
What is the empiric therapy? Hospital acquired pneumonia
a) Penicillin
b) Moxifloxacin
c) Clarithromycin
d) Piperacillin/ tazobactam

267. Which of the following question u should not ask the pt if he has common cold and allergy
rhinitis?
a) Ask about any nasal discharge, both have nasal discharge

268. Which of the following is the most paternalistic work from a healthcare professional?
a) A doctor after trying a lot of medication referred the pt to a specialist
b) A social assistance arranged for a stroke pt for living in an assisted housing
c) A nurse practitioner provided all medication options and rational of use of these
medications to the pt and gave her recommendation
d) Pharmacist informed all the s/e of the medication before pt decide whether or not to take
a medication

269. pt. is taking atorvastatin 40 mg qd for 6month now the LDL level is controlled but has high
TG. What the next option for this pt??
a) Increase the dose
b) Change it to rosuvastatin
c) Add ezetimibe
d) Add cholestyramine
e) Add fenofibrate
Add fenofibrate or nicotinic acid, as both of them have highest TG lowering effect. Rx files.

270. Taking cloxacillin 2 g IV q12H for osteomyelitis. What’s step down therapy?
a) Cloxacillin 500 tid Cloxacillin or cephalexin or clindamycin or amoxicillin-clavulanic
b) Clindamycin 600mg
c) Ciprofloxacin 750mg
d) Cefatizidime

271. Why do we do step down from IV to PO:


a) Decrease Drug-drug interaction
b) Decrease nosocomial infection
c) Decrease drug resistance
d) Increase patient compliance
272. Diabetes, on metformin and glyburide. 2 cups of alcohol per day and doubles it on the
weekend
a) Glyburide interacting with alcohol
b) Metformin interaction with alcohol
c) Excess alcohol will cause hypoglycemia
d) Hypoglycemia in the weekend

273. Worker accused the other for too many sick days. What does manager do?
a) Look at schedule and verify if that’s true and then speak to accused

274. How does one do a compound


a) Levigation
b) Trituration
q not complete. (correct is geometric dilution / mentioned in another format)
Trituration is the continuous rubbing or grinding of the powder in a mortar with a pestle. This method is
used when working with hard, fractural powders.
Levigation reduces the particle size by triturating it in a mortar or spatulating it on an ointment slab or pad
with a small amount of a liquid in which the solid is not soluble. The solvent should be somewhat viscous
such as mineral oil or glycerin. This method is also used to reduce the particle size of insoluble materials
when compounding ointments and suspensions.

275. 27-year-old obese male came into pharmacy asking about the safest erectile dysfunction
med. He has no medication but is taking a supplement from the internet. What do you tell the
patient about them?
a) They unlabelled ingredient because not standardized
b) All NPN numbers are safe
c) You should try natural before meds because they’re therapeutically better.
According to health Canada before providing a natural health product with NPN they evaluate its efficacy
and safety (bcz still nothing is absolutely safe)

276. BPH. Started finasteride. They found out that it’s not controlling it. Taking terazocin. Still
not controlled. Given patient profile. What is the DTP
a) Viagra and terazosin interaction

277. Patient with really bad C. Diff. can’t remember the question (sorry)

278. Where do you get S. viridans?


a) Mouth
b) Lung
c) Skin
d) Bowel
e) Urine
Ans.: A May be correct (logic bcz of prophylactic AB given bef. Dental procedures)

279. Child needs TPN with 0.5μmol/L of Se. The pre-made TPN contains 0.25μmol/L of Se.
You have stock containing 4mcg/mL Se. How much of the stock will you need to add to a
482mL TPN bag? MW of Se is 78.9. Mmol= wt. mg / mol. Wt
a) 1.8mL
b) 0.24 ml
c) 0.2 ml

280. Pt passed out and upon examination, you find he has pinpoint pupils. What was the cause?
a) Cocaine
b) Narcotic opioid
c) Amphetamine
d) Heroin (diacetyl morphine)

281. All of these are complications of chronic hypertension, EXCEPT:


a) Retinopathy
b) TIA
c) Tricuspid valve stenosis
d) Left ventricular hypertrophy
e) Peripheral neuropathy

282. When treating hyper-uricemia after an acute gout attack, how will you titrate the dose?
a) Until the target doses are reached
b) Until uric acid is at 50% of baseline levels
c) Until patient experiences side effects, then titrate down
d) Until number of attacks is 2/year
Aim of therapy is to reach uric acid lower than 360 umol/L

283. Patient comes in to refill their prescription for 90 tablets of Atenolol 50mg daily. You
check their profile and find that their last refill was 60 days ago. The patient tells you the doctor
told him to increase the dose to 50mg bid. What do you do?
a) Call the doctor to verify the dose and change it
b) Give the increased dose
c) Fill the Rx for the same dose it is processed as
d) Tell the patient to go get a new prescription
284. Lady tells you she has long shifts and is getting varicose veins. She asks about
compression stockings, what do you tell her?
a) Need a prescription because they have to be fitted
b) They work from the hip downwards to prevent DVT
c) Take them off at night, and put them on in the morning before work

285. Patient comes in with a prescription for 20 Diazepam 10mg daily, decrease dose every 4-5
days. He tells you he had alcohol dependence and went to rehab. His last drink was yesterday
and he is experiencing insomnia (and other withdrawal symptoms). What is his DTP?
a) Patient is receiving too much drug
b) Patient is receiving wrong drug for indication
c) Don’t give the drug because he is at risk of substance abuse
d) Patient not receiving enough drug or therapy
It`s indicated even in mild symptoms with dose range from 5-10 mg upto every 10 min)

286. New drug is approved in Canada and the doctor is asking you where to look for drug
interactions.
a) Product monograph
b) Drug interactions textbook
c) Textbook
d) Health Canada drug product database
New approved drug or drug indication:
 Manufacturer web site or customer line
 E.CPS for the product monograph after 3-4 week of the approval
 Health Canada web site for drug database.
 Checked DPD and also contains product monographs but sometimes not available

287. What is the side effect with topiramate? Weight loss


Plus, cognitive problems, kidney stones, wt loss, acute angle closure glaucoma & finger or toes parasthesia.

288. You have a prescription for Amoxicillin 187.5mg tid x 10 days. The available suspensions
are 100mL bottles of Amoxicillin 125mg/5ml and Amoxicillin 250mg/5mL. What is the
cheapest option? (I forget the cost for each, but just know how to calculate the dose you need
etc)
289. Pioglitazone can be used regardless of which of these conditions?
a) Renal dysfunction
b) Moderate hepatic disease
c) Heart failure
d) Edema
Contraindicated in hepatic disease, HF, rest or hip fracture as it causes osteoporosis & increase the tendency
of fractures plus salt & water retention. (TEFA)

290. Pregnant Lady comes in with UTI symptoms. She got a prescription for Nitrofurantoin
100mg bid for cystitis. What is wrong?
a) Change the antibiotic
b) Fill as given
c) Dose too high (50-100 mg once daily, adult dose)
d) Dose too low
Not sure about case but may be pregnant near term or took it previously within a short period or renally
impaired (most logic reasons)

291. What does not cause sexual dysfunction?


a) Atenolol
b) Terazosin
c) Thiazide
d) Fosinopril

292. Child with asthma is not controlled using salbutamol and montelukast. If you don’t
recommend steroid therapy with the family, you are breaking what?
a) Veracity
b) Autonomy
c) Beneficence
d) Justice

293. Guy calls you from the States telling you he wants to buy a lot of pseudoephedrine to give
his family for the cold season. Ethically, you are worried about all EXCEPT:
Border trouble
a) You aren’t there to counsel the family members
b) It should only be for personal use
c) Borders
294. What are the sugar levels you want in pregnancy? 3.8-5.2

295. Patient has been stable on Phenytoin dose for 6 months. Currently experiencing gingival
hyperplasia. What do you do?
a) Decrease phenytoin dose, start new epileptic to titrate
b) Refer to dentist to come up with oral hygiene plan
c) Stop phenytoin and start a new drug

296. Phenytoin drug shortage in the hospital, but you have fosphenytoin available. You do all
EXCEPT:
a) Call manufacturer to find out reason for the shortage
b) Tell the pharmacist to tell the doctors to stop prescribing phenytoin and switch everyone
to fosphenytoin
c) Save for the ICU patients
d) Let everyone know

297. They want to expand pharmacy practice. What CONTRAVENES with legislation?
a) Doctor diagnosed someone with HTN, pharmacist initiates ramipril 5mg
b) Pharmacist is allowed to make therapeutic substitution for insurance purposes
c) Patient comes in and pharmacist diagnoses and treats him for athletes’ foot with tinactin

298. Hydrochlorothiazide:
a) Increases LDL
b) Increases TG
c) Increases LDL and TG
d) Increases TG and decreases LDL
e) Increases LDL and decreases TG

299. Patient wants Marijuana for his disease state. What do you tell him?
a) Grow it
b) Go to your physician to request it
c) Go to a doctor with prescribing rights for controlled substances
d) Patient must fulfill the requirements for Marijuana growing something
300. Patient comes in with runny nose, you ask all EXCEPT to differentiate between cold and
allergic rhinitis:
a) How long they’ve had their symptoms?
b) Do they have a fever?
c) Do they have a runny nose?

301. Patient had croup, what do you counsel the parents on:
a) Symptoms will resolve in 48hrs
b) Send them to their doctor

302. Mom comes in saying she tried permethrin twice for her child, what is the most likely
reason for it is not working
a) She didn’t leave it long enough. Should be left for 10 min & repeated after 7-10 days

303. What do you recommend for her child to use? Isopropyl Myristate
Depends on other options but it`s correct as an option

304. Mom discovered that she has lice, and she is 2 months pregnant, what do you recommend
for her?
a) Permethrin
b) Soak head in vinegar

305. Patient has been acting crazy. Diagnosed with Hepatic Encephalopathy. What test would
you do? What do you want to decrease?
a) Lipase
b) Ammonia

306. Patient is being treated with Lactulose. When do you stop treatment?
a) When they lose 5kg
b) LFTs are normal
c) Psychosis goes away
d) Normal consciousness
As per TC target dose to achieve 2-3 BM/day then continue lactulose so C is more logic

307. Which one of these is schedule I:


a) Mometasone Nasal spray
b) Nitroglycerin spray (class 2)
c) IV diphemhydramine (class 2)
d) IV B12
e) Epipen (class 2)
308. Woman is menopausal for the past 6 years. Diagnosed with osteoporosis. Family history of
breast cancer. What do you give her?
a) Raloxifene
b) Calcitonin
c) Terperitide
d) Estrogen

309. Hypothyroidism symptoms, all EXCEPT:


a) Nervousness/ except oily skin
b) Dry skin
c) Cold intolerance
d) Constipation

310. pt comes in with dry mouth due to an ADR of a drug. The doctor doesn’t want to stop the
medication so he looks in the literature for a treatment. He decides to start pilocarpine eye drops
used in the pts mouth to apply on tongue or lips. What is correct?
a) Refuse to fill prescription and send him to another pharmacy
b) Document after discussing both benefits and risks with both the patient and doctor
c) Tell them it is an off-label indication that has toxic side effects
d) Call her doctor to change the order

310. Pneumococcal vaccine. What does it NOT cover?


a) Otitis media
b) Ear canal tube
c) Endocarditis
d) Meningitis
Ear canal infection or swimmer`s ear is caused by P. aureginosa (not covered by pneumococcal vaccine)

311. Woman comes in with symptoms of a yeast infection. She checked online and found out
that she has a yeast infection. First time, great medical history.
a) Refer to physician

312. When should you increase the dose of SC morphine?


a) Q6h
b) Q12h
c) Q24h
d) Q2days
e) Q3days
313. Patient experienced MI and is being treated. You monitor all EXCEPT?
a) Blood pressure
b) Heart rate
c) Troponin levels
d) Lipids
May be correct as only used during attack to stratify risk of pt. but check also if myoglobin is an option.

314. A patient had an acute gout attack and was treated, after that dr wants to initiate allopurinol
a) Titrate till you reach the target dose of allopurinol
b) Titrate dose till you reach the target serum uric acid level

315. Patient has end stage breast cancer, she is taking morphine, what would u tell her
a) Addiction is not a concern for cancer patients
b) Cancer patients do not get addicted to opioids

316. Her daughter returns telling you that she still feels a lot of pain and has been taking more
doses than prescribes, what to tell her
a) Since her pain is uncontrolled it is ok to take more than prescribed
b) Call the doctor to reassess her dose
c) Call the doctor and report abuse
d) Recommend to switch to fentanyl patch

317. Child needs TPN with 0.5 micromole/L of Selenium, your stock is 0.25 micromole/L & 40
microgram /ml, how much of 40 µg/ml of selenium would you add if you want to make 482mL
(M. Wt of selenium is 79).
Answer:
40 microgram ------- 1 ml X microgram -------- 1000 ml X = 1000*40/1 = 40000 microgram / L.
No. of m. mole = 40000 / 79 = 506.32 m. mole /L.

X+Y = 482 ml Y = 0.25/ (505.82+0.25) * 482 = 0.238 L.

318. HIV Patient that was kicked out of her house and possibly sexually assaulted. She is now
talking to a councilor that is part of the healthcare team. What should you tell the pharmacy
interns working with you to do? (pretest)
a) Do you tell him to attend the counseling session with the councilor?
b) Do you tell him to take the patient’s preliminary drug history?
c) Do you tell the patient that the intern must be present, because this is a teaching institution?
319. Patient taking Salbutamol MDI 2 puffs QID and Fluticasone 250 mcg BID and asthma not
controlled he’s taking Advil, and Tylenol 1 (codeine, caffeine, and acetaminophen).
Which drug is causing it?
a) Advil (ibuprofen) (answer is advil)

320. What would you suggest as add on therapy?


a) Switch Salbutamol to Formeterol.
b) Add Tiotropium.
c) Oral prednisone

321. Patient taking a whole bunch of drugs, they’re in a blister pack. What requires your
intervention He’s taking Psyllium TID and tech prepared it as Psyllium one pill in each
compartment (there are four compartments in the blister pack so this is wrong). All other
options didn’t require your intervention.

322. Question about a patient suffering discontinuation syndrome, what drug caused?
a) Venlafaxine.

323. How do you manage it? (pretest)


a) Give higher dose of venlafaxine
b) Give a lower dose of venlafaxine.
c) Tell him to go home and do non-pharm.

324. Patient with atrial fibrillation which drug doesn’t help?


a) Amlodipine (reflex tachycardia).

325. Patient has high ventricular response rate (something like that) what do you give him, she
also tried cardio conversion and it didn’t work?
a) Digoxin
b) Metoprolol
c) Carvedilol
d) Amlodipine
e) Verapamil

326. What would use as interim therapy for Colitis?


a) Prednisone (in exacerbations)
b) Azathioprine
c) Sulfasalazine
d) 6-mercaptopurine
N.B. Interim was the word they used, it makes no sense and it has nothing to do with anything

327. Alcoholic for 3 years and quit yesterday, dr prescribes benzos for withdrawal, what drug
therapy problem exists?
a) Wrong drug for indication.
b) Dose was too low (diazepam 10 mg QID for alcohol withdrawal symptoms)
c) Potential for drug disease interaction.

328. Patient on Lorazepam for years and wants to stop what do you?
a) Give him clonazepam and taper over a period of a few months.
Ans. The answer should be diazepam

329. Athelete’s foot which is not a symptom?


a) Bad odour.
b) Scaling, burning skin
c) Peeling and bleeding (no bleeding)
d) Pain in sole of foot and heel

330. Patient has urinary incontinence and she read about Kiegel exercise to help her decrease
the incontinence that would be able to help her learn this?
a) Chiropodist
b) Nurse
c) Occupational therapist
Continence Nurse OR Occupational therapist

331. What would be the most important measure of success for this patient’s therapy?
(menopause cause)
a) Decrease in incontinence
b) Decrease in insomnia
c) Decrease in hot flashes

332. Patient on lisinopril, and a bunch of other medications, potassium level 5.5 (shouldn’t be
above 5) which med. do u need to stop?
a) Lisinopril

333. What is inappropriate recommendation?


a) They gave us a bunch of treatments for hyperkalemia, one of which was Na bisulfite
enema, that’s what I put (don’t know if it’s right).
Wrong choice by uncle
334. Which drug does not have cross sensitivity with phenytoin?
a) Carbamazepine
b) Phenobarbital
c) Oxcarbamazepine
d) Gabapentin
Patients receiving anticonvulsant agents are often sensitive to more than one agent, and cutaneous
reactions are particularly common. It has long been known that the “aromatic” anticonvulsants
(phenobarbital, phenytoin, and carbamazepine) have a high rate of cross-reactivity for severe adverse
cutaneous reactions.

335. Long case about a patient having gastric ulcer, arrhythmia, heart failure and doctor
recommends Dronedarone, why is this patient contraindicated for this drug?
a) Arrhythmia
b) Ulcer
c) Heart failure (right answer according to eCPS)

336. Amiodarone interacts with which of the following drugs?


a) Erythromycin
b) Omeprazole
Ans. A due to increasing QTC prolongation.

337. She forgot and she ended up taking the metformin now she is having complications, what
test should you use to evaluate this?
a) Blood test to check electrolytes (what we put because she has lactic acidosis)

338. Who should you report this to?


a) Hospital management
b) Med effect (this is what I chose because it’s for rare and serious side effects)

339. How do you prevent this error from happening in the future?
a) I put something about a written reminder for the patient, can’t remember

340. Patient with neuralgia, has pain, taking amitryptilline, Naproxen, and Tyelnol 1 (but she
doesn’t like using it because it makes her have nausea). Dr. Wants to give Oxycodone IR BID
what would prevent him?
a) Dose is inssufficent.
b) She probably won’t tolerate it (what I put because she can’t tolerate the codeine in
Tyelnol 1)
341. What breaks a doctor’s autonomy?
a) When an insurance company sets specific disease criteria for drug approval (what I put).
b) Something about covering part of the drug cost

340. Which drug can’t have an interchangeable in a formulary?


a) Drug with low therapeutic index.

341. Child with lymphogranuloma, he is prescribed 6-mercaptopurine and methotrexate, what


don’t you monitor?
a) Bunch of blood tests
b) Shortness of breath and edema

342. Patient coming in with RX for Champix, he says that he doesn’t want to quit smoking but
he’s only coming because the doctor told him he should quit, all of the following are
appropriate actions except:
a) Tell him about risks of smoking
b) Council and dispense medication
c) Ask him why he doesn’t want to quit
d) Ask him why Dr. Wants him to quit
e) Invite him to a work shop to quit smoking

343. What is an example of paternalism?


a) Diabetes educator changes insulin regimen after measuring BGL.
b) Dr. Discusses alternative treatment with patient.
c) Social worker discusses living arrangement of stroke patient with him and his family
(that’s her job)
A can potentially be an example of paternalistic behavior if the patient was NOT involved.

344. Lay language counselling for seniors or something like that, what do you use?
a) Merck manuel (not sure)
b) eCPS (has a section in patient centered language)
c) Martindale

345. Patient prescribed TCA for burning pain, she’s not fully controlled, she admits to smoking
marihuana 25 years ago and now doctor wants to switch her to Opioid only therapy, what is
true:
a) She’s at high risk for abuse due to her history of smoking marihuana
b) This is inappropriate therapy for her condition
346. Long case about a patient with 4 migraine attacks per month, with severe nausea & aura,
she’s also taking COCs. What indicates necessity for Prophylactic therapy?
a) Aura
b) Number of attacks (right answer because it’s more than 4)

347. What is a DTP you notice in this case?


a) Possible Drug Disease interaction with oral contraceptive

348. Patient with Ostomy bag has irritation in the area around the ostomy bag, all of the
following could be causes except:
a) Too much dairy in diet
b) Bag not properly fitted
c) Possible fungal infection
d) Bag being changed too often

349. Patient requires treatment for osteoarthritis, she was taking 2000 mg of Tylenol QD and
now the pain is not controlled, what do you recommend?
a) Increase dose of Acetaminophen (what I chose because max dose of Tylenol not
reached).
b) Change to Ibuprofen
c) Change to Naproxen
d) Add Ibuprofen

350. Ginko Biloba is used for: Enhancing memory

351. Efavirenz has no S.E. on? Thyroid

352. Patient just had baby 2 weeks ago, what do you use for contraception? (they give you
active ingredient names)
a) Combined oral contraceptive
b) Progestrone only oral pills
c) Estrogen only
d) Plan B

353. Pt. Took amoxiclav for otitis media, what is true


a) Ear pain will improve in 24-36 hours

354. What affects TSH? Amiodarone


355. Patient has been stable on Phenytoin dose for 6 months. Currently experiencing gingival
hyperplasia. What do you do?
a) Decrease phenytoin dose, start new epileptic to titrate
b) Refer to dentist to come up with oral hygiene plan
c) Stop phenytoin and start a new drug
NOVEMBER 2011
1. A diabetic patient who has hypertension, angina, reynauds. He was taking a medication for
his high blood pressure and it was controlled. A few months later his blood pressure rose to
150/…? Which of the following would be the best add on to control his high blood pressure?
a) Nifedipine XL
b) Spironolactone

2. Patient suffered from acute gouty attack, took NSAID and it was controlled, now the
pharmacist is against the use of allopurinol for this patient due to:
a) It's only used for maintenance if the patient suffers from 2 acute attacks
b) It's not used, cause the patient was on ACEI and will decrease uric acid level after a while
c) Allopurinol is contraindicated wiz diabetes
d) Diet will improve uric

3. When to start gout prophylaxis? More than 2 attacks per year or uric acid more than 720
( double the normal )
Urate-lowering therapy includes xanthine oxidase inhibitors and uricosurics. Indications for the use of urate-
lowering therapy includes established diagnosis of gouty arthritis along with either tophus or tophi by
clinical exam, ≥2 attacks per year, chronic kidney disease stage 2 or worse (GFR ≤89 mL/min/1.73 m2), or
past urolithiasis. The optimal time to initiate urate-lowering therapy remains controversial. One
view is that the 1st attack is a late event in the gouty diathesis; even if further attacks do not occur, it
cannot be assumed that renal damage will not.
The other view is that because recurrence may be delayed for many years and chronic tophaceous gout
develop only in a minority, therapy can be delayed until recurrence or detection of tophi.
A treat-to-target approach has been recommended by ACR and EULAR. The minimum serum urate
target is <360 μmol/L, the saturation point of monosodium urate in the extracellular fluid.
A lower serum urate target (<300 μmol/L) is recommended in patients with more severe gout (e.g., patients
with tophaceous gout) to facilitate the improvement of signs and symptoms through the dissolution of urate
crystals. Once the urate crystals have dissolved, a maintenance serum target should be <360 μmol/L to
avoid the development of new tophi.

4. Can’t remember this case well.pt who has diabetes type 2 for which he takes metformin 1g
bid. His hba1c average is 7. He is also hypertensive for which he takes metoprolol 50 mg bid.
His hypertension is well controlled. He also takes amitriptyline 10 mg. He said that his limbs
feel cold and that he is still experiencing fibromyalgia...which is the following is the cause of
this complain as a result of a too high dose?
a) Metoprolol
b) Metformin
c) Amytriptyline------dose from 5—50 mg daily
5. Patient with epilepsy partial complex, what to give it is his 2nd attack what to give:
a) Carbamazepine
b) Lamotrigine
c) Phenytoin
d) Phenobarbital

6. long case about black man who is overweight,


he left his work then he has hypertension. Which
of the following is not a risk factor for CVD?
a) Stress
b) Diabetes
c) Obesity
d) Alcohol if taken within normal range

7. A patient suffering from Parkinson and said that he started to experience falls. He was a full-
time teacher but now he is working part time because of his situation. Which is true?
a) Patient should not start therapy now as the side effect may worsen his symptoms of falls
b) Medication should be started only when the disease is starting to affect quality of life

8. Medications causing photosensitivity (k-type):


a) Isotretinoin
b) Nitrofurantoin
c) Oral Contraceptives

9. A patient coming to your pharmacy complaining about the insulin cartridge that he filled
from your pharmacy yesterday as they were expired. How can you prevent it in future?
a) Encourage your staff to check your stock every 6 months for expired products.
b) Encourage your staff to check if the medication is expired right before dispensing
c) Let pt. to check

10. The same holy question about the kcl solution. Many patients die because of an excess
administration of Kcl...how to minimize this error?
a) Buy premixed bags
b) Put a label of concentrated solution

11. Glatiramer Is Used in Which Condition?


a) Gout
b) Multiple Sclerosis
c) SLE
12. To make an elderly pt who has difficulty falling asleep to sleep quickly without hangover
a) Diazepam
b) Flurazepam
c) Oxazepam  oxa & lora are best in elderly so used in dementia with insomnia &
agitation
d) Zopiclone

13. One has depression and insomnia the best for him is
a) Fluoxetine
b) Clobazam
c) Mirtazapine or any TCA or fluvoxamine

14. All are side effects of levodopa/carbidopa except:


a) Confusion
b) Hallucination
c) Nystagmus
Nausea, vomiting, orthostatic hypotension, dyskinesias, hallucinations, confusion, mental changes including
paranoid ideation and psychotic episodes, depression with or without development of suicidal tendencies,
and dementia. Convulsions also have occurred.
Rare: Weight gain or loss, edema, Leukopenia, hemolytic and non-hemolytic anemia, thrombocytopenia,
agranulocytosis. Urinary retention, hematuria, and priapism.
Arrhythmias, non-specific ECG changes, phlebitis. Sialorrhea, bruxism, hiccups, GI bleeding, flatulence,
burning sensation of tongue, development of duodenal ulcer.
Long term use of sinemet can produce mydriasis & precipitation of glaucoma, melanoma.
Risk of parkinsonism hyperpyrexia syndrome with abrupt discontinuation; taper gradually.
Serious Warnings and Precautions: Sudden Onset of Sleep
Patients receiving treatment with SINEMET (levodopa and carbidopa) and other dopaminergic agents have
reported suddenly falling asleep while engaged in activities of daily living, including the driving of a car,
which has sometimes resulted in accidents. Although some of the patients reported somnolence while on
SINEMET, others perceived that they had no warning signs, such as excessive drowsiness, and believed that
they were alert immediately prior to the event.
General
When patients already receiving levodopa are switched to SINEMET, levodopa must be discontinued
for at least 12 hours or more before SINEMET is started. SINEMET should be substituted at a dosage that will
provide approximately 20% of the previous levodopa dosage.
Patients who are taking SINEMET should be instructed not to take additional levodopa unless it is
prescribed by the physician.
Periodic evaluations of hepatic, hematopoietic, cardiovascular and renal function are recommended
during extended therapy with SINEMET (levodopa and carbidopa).
15. Lady walks in with a Rx of donepezil for her 85-
year-old husband. She says that he is a social drinker,
smokes, and is taking pravastatin 40 mg for his
cholesterol. Which of the following are risk factors for
his dementia?
a) Age
b) Alcohol
c) Dyslipidaemia
d) Smoking

16. Sexual dysfunction is mostly associated with:


a) Nadolol
b) Digoxin
c) Terazocin
Another version
Nadolol can be used in all of the following except; Raynaud’s phenomena and angina

17. Drug given 1g iv q 72 hours, its half life is 72 hours, its trough after 72 hours expected to be
15 mg/ml what will be the trough at steady state conc. (i think) if it is given at same interval 1 g
q 72 hours

18. Patient taking medroxyprogesterone acetate. How late she will be so that she would need
another injection?
a) 1 month
b) 3 weeks
c) 6 weeks
d) 1 week
The recommended dose for contraception is 150 mg of DEPO-PROVERA every 3 months, administered by
deep intramuscular injection.
To increase assurance that the woman is not pregnant at the time of the first administration, it is
recommended that this injection be given only within the 5 five days of the onset of a normal menstrual
period or, only within the first 5 days post-partum if not breastfeeding. If the woman has chosen to
breastfeed, discuss the risks of pregnancy and possible risks of DEPO-PROVERA to determine the most
appropriate course of action for the individual woman.
If administered within the first 5 days after the onset of a normal menstrual period, DEPOPROVERA is
effective from the day of injection. When DEPO-PROVERA is given later in the menstrual cycle it may not be
effective for the first 3 to 4 weeks after the injection and another method of contraception (non-hormonal)
should be used during this time.
After miscarriage or first trimester therapeutic abortion, the injection is normally given within 5 days of the
procedure and no extra precautions are required. After a late (second trimester) abortion, some further
delay is recommended to reduce the risk of heavy and prolonged bleeding, therefore, the first injection
should not be given until 4 weeks after the procedure.
The woman must return every 10 to 13 weeks for a repeat intramuscular injection to maintain contraceptive
effectiveness. Intervals between intramuscular injections must not exceed 13 weeks (3 months).
When switching from other contraceptive methods, DEPO-PROVERA should be given in a manner that
ensures continuous contraceptive coverage based upon the mechanism of action of both methods, (e.g.,
patients switching from oral contraceptives should have their first injection of DEPO-PROVERA within 7 days
after taking their last active pill).

19. To measure the effectiveness of spironolactone therapy in ascites measure:


a) Weight
b) Abdominal Girth
c) Electrolytes

20. To get the most updated data for drug schedules:


a) UpToDate
b) NAPRA
c) E-Therapeutic

21. Question about "biomedical therapy", what concept describe it:


a) Let the body treat itself by self immunity
b) Take remedial therapies that represent different cultures and beliefs
c) Let the body heal itself through emotional stimuli
d) Treat the body by putting it into a shock or someone with healing power
e) Cure illness of patient by keeping his cultural belief and values
f) Removing injured or affected body part and preventing damage to other body parts.
The biomedical model of medicine has been around since the mid-nineteenth century as the predominant
model used by physicians in diagnosing diseases.it has four core elements.
According to the biomedical model, health constitutes the freedom from disease, pain, or defect, thus
making the normal human condition "healthy". The model's focus on the physical processes, such as
the pathology, the biochemistry and the physiology of a disease, does not take into account the role of
social factors or individual subjectivity. The model also overlooks the fact that the diagnosis (that will affect
treatment of the patient) is a result of negotiation between doctor and patient.
It is however very limiting. By not taking into account society in general, the prevention of disease is
omitted. Many diseases affecting first world countries nowadays, such as heart disease and type 2 diabetes
mellitus are very much dependent on a person's actions and beliefs .
Biomedical  In this model, disease is defined as biophysical malfunction and goal of treatment is to
model correct the malfunction in order to cure the disease.
 The biomedical model includes pathophysiology of disease, objective tests and
therapeutic interventions at centre of patient care.
 However, it offers one dimensional approach to patient care that excludes the patient
experience of illness and how this affects other facets of life as work disability, finances,
and social networks because they are believed to lie outside of medicines responsibility.

22. Sulpha allergy what not to take:


a) Ferrous Sulphate
b) Sulfadiazine Cream
c) EpiPen with bisulfited preservative
d) Captopril

23. A hospital pharmacist wants to buy a new computer software he would consider all of the
following in which order or about pharmacy manager want to change computer system, what is
the correct order for the 4 steps?
a) Financial cost or Its price
b) Stake holder requirement
c) Ability of system to perform function
d) Try it first
e) See if it is compatible with your computer
f) Compare its advantage to the program you already have
BCFEDA

24. A pt take aminoglycoside about 420mg /24hr so:


a) Take it more frequent since has good renal function
b) Aminoglycoside can coz irreversible ototoxicity upon long use and high dose
Drug-Induced Ototoxicity
Factors affecting ototoxicity include
 Dose & Duration of therapy
 Concurrent renal failure
 Infusion rate
 Lifetime dose
 Coadministration with other drugs having ototoxic potential
 Genetic susceptibility
Ototoxic drugs should not be used for otic topical application when the tympanic membrane is perforated
because the drugs might diffuse into the inner ear.
Aminoglycosides, including the following, can affect hearing:
 Streptomycin tends to cause more damage to the vestibular portion than to the auditory portion of
the inner ear. Although vertigo and difficulty maintaining balance tend to be temporary, severe loss of
vestibular sensitivity may persist, sometimes permanently. Loss of vestibular sensitivity causes
difficulty walking, especially in the dark, and oscillopsia (a sensation of bouncing of the environment
with each step). About 4 to 15% of patients who receive 1 g/day for > 1 week develop measurable
hearing loss, which usually occurs after a short latent period (7 to 10 days) and slowly worsens if
treatment is continued. Complete, permanent deafness may follow.
 Neomycin has the greatest cochleotoxic effect of all antibiotics. When large doses are given orally or
by colonic irrigation for intestinal sterilization, enough may be absorbed to affect hearing, particularly
if diffuse mucosal lesions of the colon are present. Neomycin should not be used for wound irrigation
or for intrapleural or intraperitoneal irrigation, because massive amounts of the drug may be retained
and absorbed, causing deafness.
 Kanamycin and amikacin are close to neomycin in cochleotoxic potential and are both capable of
causing profound, permanent hearing loss while sparing balance.
 Gentamicin and tobramycin have vestibular and cochlear toxicity, causing impairment in balance and
hearing.
 Vancomycin can cause hearing loss, especially in the presence of renal insufficiency.
Some mitochondrial DNA mutations predispose to aminoglycoside ototoxicity.
Azithromycin, a macrolide, has also been shown in rare cases to cause both reversible and irreversible
hearing loss.
Viomycin, a basic peptide with antituberculous properties, has both cochlear and vestibular toxicity.
Chemotherapeutic (antineoplastic) drugs, particularly those containing platinum (cisplatin and carboplatin),
can cause tinnitus and hearing loss. Hearing loss can be profound and permanent, occurring immediately
after the first dose, or can be delayed until several months after completion of treatment. Sensorineural
hearing loss occurs bilaterally, progresses decrementally, and is permanent.
Ethacrynic acid and furosemide given IV have caused profound, permanent hearing loss in patients with
renal failure who had been receiving aminoglycoside antibiotics.
Salicylates in high doses (> 12 325-mg tablets of aspirin per day) cause temporary hearing loss and tinnitus.
Quinine and its synthetic substitutes can also cause temporary hearing loss.
Prevention
Ototoxic antibiotics should be avoided during pregnancy, because they can damage the fetal labyrinth. The
elderly and people with preexisting hearing loss should not be treated with ototoxic drugs if other effective
drugs are available. The lowest effective dosage of ototoxic drugs should be used and levels should be
closely monitored, particularly for aminoglycosides (both peak and trough levels).
If possible before treatment with an ototoxic drug, hearing should be measured and then monitored during
treatment; symptoms are not reliable warning signs. The risk of ototoxicity increases with the use of
multiple drugs with ototoxic potential and the use of ototoxic drugs excreted through the kidneys in patients
with renal compromise; in such cases, closer monitoring of drug levels is advised. In patients known to
have mitochondrial DNA mutations that predispose to aminoglycoside toxicity, aminoglycosides should be
avoided.
Key Points
 Drugs may cause hearing loss, dysequilibrium, and/or tinnitus.
 Common drugs include aminoglycosides, platinum-containing chemotherapy drugs, and high-dose
salicylates.
 Symptoms may be transient or permanent.
 Using the lowest possible dose of aminoglycosides and measuring drug levels during treatment may
prevent hearing loss caused by ototoxic drug use.
 Drugs are stopped if possible, but there is no specific treatment.
25. Patient has CVD and took self-medication last week he has HTN, then complain about
pedal edema and SOB, what medication can coz this of the OTC he takes:
a) Ibuprofen  sodium & water retention and NSAIDs induced asthma
b) Glucosamine
c) Docusate Na
d) Milk of Magnesia

26. Patient on phenelzine and the dr. Will shift it to fluoxetine. How many days washout
a) 7 days
b) 14 days
c) 5 days
From moclobemide -- 5 days. Any SSRI ------------------- 5 t1/2
Irreversible MAO --------------- 14 days Fluoxetine ---------------- 5 weeks
taper and stop MAOI, then wait 14 days for washout before starting fluoxetine§

27. The most evidence-based reference?


a) Pharmacy Letters
b) Cochrane Library
c) Drug Facts

28. Question about enema and regular use. Why enema should not be used continuously?
a) May cause continuous diarrhoea and irreversible
b) Loss of spontaneous tone of bowel  no movement except with the enema
c) Colon cancer
d) Ulcerative colitis
e) Eradication of colon normal flora
f) Body is getting used to
Rectal suppositories (e.g., glycerin) and enemas (e.g., sodium phosphates) may be used for shortterm
relief of constipation in patients who feel the urge to defecate but have difficulty evacuating. The
physical insertion of suppositories may stimulate anorectal motility, particularly in patients with spinal
cord dysfunction. Enemas increase the water content of stool and cause rectal distension, which
stimulates colonic peristalsis and prompts evacuation.
Side effects of continuous use of enema: Dependence, Perforation, Infection, Electrolyte imbalance & Flora
trouble
29. Dr. Orders dexamethasone prescription. You have 5ml vial contains 4mg/ml. Complete to a
final volume of 10 ml (containing 8mg/10ml) with a diluent. How many mls will you withdraw
from the vial?
4mg ----- 1ml 8mg ------- xml x = 8*1 / 4 = 2ml from ampoule

30. A man called the pharmacy and ask about his pregnant wife who has 8 times vomiting and
she could not drink anything. What is the proper suggestion?
a) Advice to call the dr for diclactin rx
b) Advise him to take his wife to the family physician
c) Advise to go to ER
d) Advise him to give her gastrulate

31. To avoid OTC interaction of severe drugs. What to do:


a) Do a seminar for all the patients about OTC drug interaction
b) To add any new OTC on the patient profile on the computer system
c) To try to print labels on the OTC

32. Cancer patient with neutropenia. Which medication can be given?


a) Filgrastim
b) Epoetin alpha
c) Oprelvekin (IL-11)

33. Patient take calcitonin spray; pharmacist should counsel all except:
a) Take vit D and calcium
b) Take this medication in alternative nostril each day
c) Allergy to shellfish should not use it
d) Prime before use

34. Question about methadone dispensing in a community pharmacy:


a) Max day supply is 3 days
b) Patient can drink his dose up to 12 hours from taking the medication
c) Pharmacy has to document doses dispensed and date of dispensing

35. Who is not allowed to prescribe narcotic:


a) Nurse Practitioner
b) Pharmacists
c) Physician
d) Dentist
e) Veterinary Doctor
Doctors of medicine, Dentists, Doctors of veterinary, Midwives, Podiatrists or chiropody, Nurse Practitioners

36. A case with gonorrhoea, discharge and urgency. What is the drug of choice?
a) Cefixime + azithromycin
b) Ceftriaxone
c) Doxycycline
d) Cotrimaxole
e) Penicillin

37. what organism accompanies gonorrhoea ordinarily


a) Chlamydia trichomatis
b) Streptococcus pneumonia
c) Neisseria meningitis
d) Candida albicans
Another version: Which of the following bacteria is responsible for non-gonococcal infection?
The 2 main causes of urethritis and cervicitis are gonorrhea (caused by Neisseria gonorrhoeae) and chlamydia
(caused by Chlamydia trachomatis). Nongonococcal, nonchlamydial causes of urethritis and cervicitis include
Mycoplasma genitalium, Ureaplasma urealyticum, T. vaginalis, (HSV), (VZV) and adenovirus.

38. In the above patient the bacteria are penicillinase producer. What is the doc:
a) Doxycycline
b) Cotrimoxazole
No ceftriaxone, azithro and cipro in the answer

39. Then she developed chlamydia what to


give or If they are both seen in the above case
whats doc combination?
a) Doxy+Azitro
b) Cefixim+Azitro
c) Ceftriaxon+Penicillin
d) Doxy+Ceftriaxone

40. A patient with CVD and diabetes what vaccine to


give except 48-year patient; no shingles
a) Flu
b) Pneumococcal
c) Tetanus
d) Varicella
e) Diphteria
41. 13-month child with meningitis to ttt
a) Cefazolin
b) Ceftriaxone + Ampicillin
c) Gentamycin

42. Diplococci chain G+ve bacteria causing meningitis


a) N. Meningitis
b) S. Pneumonia
c) S. Pyogenes

43. pt in precontemplation stage to stop smoking:


a) Talk to him about the adverse effects of
smoking on his health & Goals of smoking
cessation
b) Talk to him about NRT medication for
smoking cessation  preparation
c) Invite him or tell him about benefits of
cessation
Precontemplation-------------side effects of smoking

44. Contamination in laminar flow?


a) Hands
b) Clothes
c) Pyrogen
d) Bacteria

45. Which cause pseudo biliary liathais:


a) Amoxicillin
b) Ceftriaxone
c) Tobramycin
d) Vancomycin

46. Case about patient with acne, and on medroxyprogesterone, and also taking nitrofurantoin
for infection, eventually coming with sunburn., except
a) Wear long sleeves
b) Avoid direct sun from 10 am – 4 pm
c) Sun screen against UVB
d) Avoid tan beds
47. What causes phototoxicity: (k-type)
a) Nitrofurantoin
b) Isotretinoin
c) Medroxyprogesterone inj

48. All can be prepared by horizontal laminar flow except:


a) Analgesic
b) Amoxicillin
c) Antineoplastic

49. Horizontal flow all except:


a) Antibiotic
b) Chemotherapy

50. Tobramycin, which one we should not check? (nephron/ototoxicity)


a) Liver test
b) Post dose concentration
c) Predose concentration
d) Cr clearance

51. Which of the following signs and symptoms indicate tobramycin toxicity
a) Myalgia
b) Jaundice
c) Dizziness
d) Bradycardia
e) Fatigue

52. Treatment of PJP in HIV patient:


a) Ceftriaxone
b) Cotrimoxazole
c) Gentamicin

53. Gm +ve coci chain:


a) Staph  Clusters
b) Strep.  Cocci
c) Neisseria
54. A question about "composite endpoint" in a
drug trail that the pharmacist is involved in, so
what is true about it:
a) This endpoint measure rare endpoints
b) Measure many patient events
c) Measure CV mortality and morbidity
d) Used to evaluate several questions
A composite outcome consists of two or more component
outcomes. Patients who have experienced any one of the
events specified by the components are considered to
have experienced the composite outcome
Why to use a composite endpoint
A composite endpoint in a rct consists of multiple single endpoints that are combined in order to confront an
investigational drug with a higher number of events expected during the trial. For instance, the primary
composite endpoint may include mortality along with nonfatal endpoints such as hospitalization and cardiac
arrest in chronic heart failure patients, or along with myocardial infarction and stroke in hypertensives.
The major advantages in using a composite endpoint are: statistical precision and efficiency will be increased;
trials become smaller, less costly; and the results of promising new treatments will be available earlier. If more
than one outcome is important for efficacy evaluation, a composite endpoint can efficiently deal with the
issue of multiplicity. a summary measure for drug efficacy can be defined (e.g., the assessment of skeletal-
related events [sres] in trials for prevention or treatment of bone metastases).
http://www.appliedclinicaltrialsonline.com/appliedclinicaltrials/article/articledetail.jsp?id=324331
https://www.slideshare.net/medicres/fda-2013-clinical-investigator-training-course-clinical-trial-endpoints

55. 6 years old child the Dr. ordered a loading dose of Phenytoin 20mg/Kg infused with a rate
of 0.5 mg/Kg/min. If you've it in 100 ml bag. what is the rate of pumb that you must be adjusted
in ml/hr.?
0.5 mg ------- 1 kg ------- 1 min 20 mg ------- 1 kg ------- X X = 1*20/0.5 = 40 min.
100 ml --- 40 min Y ml 60 min Y = 60*100/40 = 150 ml So rate = 150 ml/ hr

56. Report adverse drug reaction:


a) Medeffect (health Canada)
b) Institute of safe medication practice ISMP

57. When to disclose patient information?


a) Patient intent to harm himself
b) Third party requires information
c) Patient has cognitive impairment
d) Pt under the legal age
58. Which of the following is not a side effect of lithium?
a) Elevated liver enzyme
b) Wt gain
c) Diarrhea
d) Hypothyroidim

59. You are a pharmacy manager, a patient came to take OTC pseudoephedrine and
diphenhydramine, he has BPH, as a manager what to do:
a) Change the place of drug drug interaction and drug disease interaction behind the counter
b) Make a system to add the OTC in the pt profile
c) Said to cashier to ask the pt about what he take

60. Parkinson's patient taking levodopa/carbidopa. Which of the following indicates failure of
effectiveness of levodopa/carbidopa therapy or decrease in the effect? (incomplete question)
I) On/Off Phenomenon
II) Wearing Off Phenomenon
III) Akathisia

61. Which of the following under federal?


a) Clobazam
b) Trazodone
c) Zopiclone
d) Tramadol

62. What will the most serious side effect of zidovudine:


a) Liver
b) Bone Marrow suppression
c) CNS

63. For which of these patients we can break the confidentiality if its needed?
a) HIV patient
b) Pt with cognitive problem
c) Pt intending to harm himself
d) Children under the age of 13
e) A prisoner on parole
64. Diabetic patient taking NPH twice daily, and regular insulin twice daily, his blood sugar
level was as follows: 7 am 11 mmol/l, 12 pm 7.5 mmol/l, 5pm 6.7 mmol/l, 9 pm 7.3 mmol/l
What to do?
a) Increase NPH at supper
b) Decrease NPH at supper
c) Increase regular at breakfast

65. Patient asking u about new drug that was recently added to the market, to give him more
info about the medication, u should search:
a) Drug data base
b) Drug manufacture leaflets
c) CPS

66. Pt going to Dominican Republic, asking if he needs to take prophylaxis for malaria, so the
pharmacist will; (as i remember)
a) Refer him to a travel clinic
b) Tell him to check a reference (can't remember its name)
c) Tell him to buy a prophylaxis kit with deet, citronella oil...
d) Refer to a travel agency
e) Tell him to search the net for more info.
For number b: it was pharmacist who will look in an antimicrobial book considering the region to where he
is travelling

67. a woman patient have migraine with aura, her


mother has breast cancer and his father has HTN, she
cannot use coc because all of the following except:
a) Hepatic problem
b) Migraine with aura
c) Hypertension  should be uncontrolled to be
a contraindication
d) Smoking above 35 years old

68. a question about PIPEDIA, when the pharmacist is allowed to use personal information:
a) If he's using info to collect data about demographic area around him
b) Using it in marketing and to know sales of certain medication
c) Using it with third party administration in the benefit of the patient
Third party insurance (for which disclosure may be a possibility)
69. A lady coming and need her lorazepam refills for 4 months as she is leaving to Florida, what
will u do: or Someone comes to pharmacy with a prescription of 30 tabs of lorazepam with 2
refills for every month, he is going to Florida for 40 days so he wants 40 days' supply. What we
can done for him?
a) Mail her medication when they r due to Florida
b) Give her the 4-month supply and bill her insurance monthly
c) Tell her that the quantity of refills is restricted to the max supply offered by the
provincial authorities (insurance)
d) Make her doctor write her a prescription with more intervals to cover the quantity she
needs during the 4 months

70. You are arranging a program for diabetic management, so u will advise all except:
a) Diabetic patient in ur area
b) Physician
c) Nurses
d) Local school boards

71. A lady is 5 feet 6 inch she is 73 kg she enjoys reading. She sees orlistat commercial and is
wondering if she can take orlistat?
a) She is not a good candidate for orlistat

72. Now what would you suggest reducing her weight?


a) Reduce fat in diet
b) Brief walk during break time
c) Join health club

73. A female marathon runner on psyllium for a year to control her constipation and she is fine
with it, but recently is constipated for 2 days and comes to your pharmacy for constipation and
abdominal pain she wants something that will give her quick relief as she has an athletic
competition?
a) Bisacodyl
b) Lactulose
c) Docusate sodium
d) Magnesium salt
e) Sodium polysorbate
f) castor oil
PEG high dose ideally plus electrolytes (1/2 or 1L Colyte) to avoid dehydration before the running
competition. All the rest are going to take days to kick in
74. After some time she has fallen down and has a fracture the doctor has prescribed her
codeine for pain, but she is worried about the constipation what will you give her?
a) Sennoside (codeine: opioids)
b) Docusate sodium

75. Now the above patient is pregnant and has constipation what will you give her
a) Bulk Laxative
b) Mineral Oil
c) Caster Oil

76. You got the following Rx, what will be the final amount of Mometasone cream? 10% Drug
X 1% Drug Y 0.05% Tretinoin cream aa Mometasone cream M: 60 gm
Drug X = 10 % * 60 gm = 6 gm Drug Y = 1 % * 60 gm = 0.6 gm
Remaining = 60 - 6 - 0.6 = 53.4 gm
Tretinoin cream = Mometasone cream Mometasone cream = 53.4 gm / 2 = 26.7 gm

77. Dose of li is 600mg q 8 hr, and its clearance is 1.6 l/hr.what is time to reach css 94%?
T1/2=18hr 100........18hr.........50.........18hr............25........18hr............12.......18hr............6
18*4 = 72 = 3days

78. If we change dose to 300mg q 8 hr. What is time to reach css 94%?
a) immediately
b) 1 day
c) 3 days
d) 6 days

79. What reason make the pharmacist verify a narcotic prescription:


a) Patient looks suspicious
b) The signature is unknown to the pharmacist
c) The order exceeds 90-day supply

80. Which is not taken in opioid naive people:


a) Fentanyl injection
b) Fentanyl patch
c) Hydrocodone extended release
d) Oxycodone extended release
81. What is your recommendation for using the fentanyl patch except?
a) Put it in refrigerator when you are not using it
b) After removing it throw it into the toilet and flush
c) Change the site of application each time
d) Throw it in the garbage.

82. Fentanyl patch, for how long a daily patch is used for?
a) 1 day
b) 2 days
c) 3 days
d) 4 days

83. Valproic side effect all except or valproic acid monitoring all except:
a) Sedation
b) Ataxia
c) Mood changes (valproic acid: mood stabilizer)
d) Peripheral edema
e) Alopecia

84. A question about the purpose of hospital accreditation:


a) To ensure that they operate according to standards of excellence
b) To ensure the equipment are up to date and keep patient confidentiality

85. why should a patient take sertraline with food:


a) Food increases its absorption
b) Food will decrease irritation of the stomach wall

86. Question about a study done on a new pain killer that the physician thinks it's great, but
when u check the statistical data of the study done on this medication compared to iboprofen
600 mg tid, u find that the p-value > 0.05 and the 95 % CI is 0.7 - 1.5, so u conclude that
a) There drug is more effective than iboprfen
b) The NSAID is similar in effect to the new medication
c) There's a significant difference between the two medication
d) The medication is ineffective
e) There's no significance between them
If range of confidence interval crosses 1, it's not significant. Range is already given 0.7‐1.5.
If range is before 1 (0.7‐0.9), the drug is more effective, if after 1 the placebo is more
effective. If range was 0.7‐0.9 then ans would be B
87. A study about safety of certain drugs in pregnancy can be done by all except:
a) Case control
b) Case series
c) Randomized double blinded
d) Studies done on risk versus
benefits

88. Why to refer a depressed person


to her doctor:
a) She complains from these
symptoms for the last 3
weeks

89. Pregnant take diclectin what is the action of this med.:


a) Pyridoxine and dicyclomine are vitamins
b) It will give prolonged action till the morning
Two (2) Diclectin delayed release tablets at bedtime to control nausea and vomiting occurring in the
morning; additionally, one (1) delayed release tablet in the morning and one (1) delayed release tablet mid-
afternoon to control symptoms throughout the day. The dosage schedule may be individualized according to
timing, duration, severity and frequency of the symptoms experienced by the patient. Diclectin can be
prescribed in any trimester of pregnancy.
Diclectin is a delayed-release formulation that works optimally when given 4 to 6 hours prior to anticipated
onset of symptoms. The delay in action may be prolonged when tablets are taken with food. However,
based on the available data, the above recommended dosage schedule should be followed (see Action and
Clinical Pharmacology, Pharmacokinetics).
Diclectin tablets being of a delayed release formulation should not be prescribed on an as needed basis
(prn). It is important that Diclectin is taken daily for optimal effect.
A gradual tapering dose of Diclectin is recommended at the time of discontinuation to prevent a sudden
onset of symptoms.
Missed Dose
In the event that a dose is missed, it should be taken as soon as possible. However, if it is almost time for the
next dose, the missed dose should be skipped. The prescribed dosing schedule should be continued.
Administration Diclectin tablets are to be taken orally. Diclectin tablets are a delayed release formulation
therefore they should not be crushed or split.
Mechanism of Action
Diclectin (doxylamine succinate and pyridoxine hydrochloride) provides the action of two unrelated
compounds. Doxylamine succinate (an antihistamine) and pyridoxine hydrochloride (vitamin B6) provide
anti-nauseant and antiemetic activity. The delayed action of Diclectin permits the nighttime dose to be
effective in the morning hours, when the patient needs it most.
Doxylamine can cross the blood-brain barrier and has a high affinity for H1 receptors in the brain.
The mechanism of action of Diclectin is unknown.
90. Pt has migraine with aura, which one you don’t recommend:
a) Take Nap in Dark Quit Room
b) Relaxation therapy
c) Cold compress
d) Go for a bike ride

91. Pt take phenelzine and has congestion and cough what to take:
a) Diphinhydramine + Phenylephrine + Chlophenarmine
b) Topical Oxymetazoline
c) Phenylephrine

92. A patient 42 yrs. old afraid of get osteoporosis as her mother has recently fallen and has
knee fracture. She drinks 3 milk cups (3x300 mg Ca) & 1 cup coffee, what to advise her:
a) Take Elemental Ca 500mg TID (daily Ca intake for 42 years old 400-1000)
b) Do Exercise
c) Stop Coffee (she takes only one cup- not more than 4 cups per day)
Nonpharmacologic Choices: Recommended for
everyone:
Regular exercise (especially impact type)
Fall prevention: minimize hazards for falling in the
home (e.g., remove throw rugs, install grab
bars in bathrooms, ensure adequate lighting),
assess drugs implicated in falls such as
benzodiazepines and other psychotropics, improve
strength and balance
Smoking cessation
Dietary measures: encourage adequate protein,
calcium and vitamin D intake, avoid excessive
alcohol (>2 drinks/day) and caffeine (>4 cups of coffee per day or equivalent)
93. She comes again to your pharmacy asking for additional therapy, what do you recommend?
a) Add Vitamin D

94. Now she came with vertebral column fracture, patient also using PPI for her GERD, what is
your recommendation or what should be the prophylaxis:
a) Calcitonin
b) Alendronate  given as half of the ttt dose
c) Etidronate
d) Rezidronate

95. A patient has 82 years old and BMI of 20kg/ m2. The patient has osteoarthritis, what should
be given to him?
a) Capsaicin cream (topical diclofenac)
b) Diclofenac and misoprostol
c) Aspirin
d) Morphine
e) Hyaluronic acid

96. She asked for an intraarticular injection for her arthritis; what should be given to her?
a) Hyaluronic Acid
b) Fentanyl Injection
c) Desmopressin Aoetate
d) Septra Injection
e) Meropenem Injection

97. Which one is correct about capsaicin:


a) Use warm pack to increase absorption (no, intensify burning sensation)
b) Effect in a week (1-2 weeks)
c) If no effect, stop in week (effect 1-2 weeks in osteoarthritis)
d) Burning sensation will disappear after 2 weeks of using (transient effect)
A burning sensation may occur at the site of application; This is usually a transient effect most prominent in
the 1st wk of treatment; it diminishes or disappears with continued use at the recommended dose.
Taking a hot bath or shower, excessive sweating or heating pads immediately prior to or after applying as
well as hot weather may intensify this sensation. In some patients, the burning may be severe.
Avoid use on mucous membranes, wounds, broken or irritated skin, or on areas where the skin barrier is
compromised. Keep away from lips and genital area. If contact occurs with mucous membranes, lips or
genital area, gently rinse with soap and cool water, or wet a cloth with cold water and apply compress
against the area until the burning sensation disappears. Do not use occlusive dressings or other bandages
on treated areas. Wash hands with soap and warm water immediately after applying cream.
98. A patient who recently lost his wife and has depression. He is taking paroxetine,
mirtrazapine, ipratropium and salbatumol. Now
he can’t sleep, this is due to which medication?
a) Amitriptyline
b) Paroxetine
c) Mirtazapine
d) Ipratropium
e) Salbutamol
Another version: patient had depression and insomnia. Which medication is the most
appropriate?
a) Paroxetine
b) Moclobemide
c) Amitriptyline/ Mirtazapine
d) Chloral hydrate

99. A patient 7 years old, with ADHD, on atomoxetine and methyl phenidate, his mother comes
to ask you ask his prescription to be transferred to your pharmacy, what should u tell her:
a) Transfer the medication to your pharmacy
b) Transfer the methylphenidate only
c) Transfer atomoxetine, and ask for written rx for methylphenidate
d) Transfer atomoxetine, and doctor can call the methylphenidate in

100. He is taking methylphenidate at 7 am and at 3 pm, but he is experiencing insomnia. What


to do?
a) Switch to SR at 12 pm
b) Switch to SR at 7 am and 3 pm
c) Add immediate release at 7 and 5 pm
d) Give methylphenidate at 7:00 am and 12:00 noon

101. A case about alcohol withdrawal and asking all are signs of alcohol withdrawal except:
a) Pinpoint pupil (opioid)
b) Agitation.
c) Nausea
d) Confusion and delerium
e) Depression
f) Fatigue
g) Dilated pupils
h) Insomnia

102. Alcohol withdrawal symptoms treatment? Diazepam (10 mg 4 times daily)

103. A patient comes with a complain about the sleeping medication is not working, and he
wakes up in the middle of the night anxious, what is the medication he's on:
a) Zopiclone
b) Flurazepam
c) Diazepam
d) Triazolam (short acting benzodiazepine)

104. He wants to take a medication for sleep but with no hangover effect which medication can
he take?
a) Oxazepam
b) Triazolam
c) Lorazepam

105. All the following are side effects of amiodarone except


a) Hepatoxicity
b) Renal toxicity
c) Phototoxicity
d) Pigmentation
e) Pulmonary toxicity
Note amiodarone no renal, no blood, no sugar, no bones, no joints

106. pt has hypersensitivity reaction, all are true except:


a) Symptoms happen immidiately after exposure
b) Symptoms can appear even after one hour or more
c) The pattern of symptoms is monophasic  as symptoms appear in different phases

107. which is not a sign of anaphylactic shock


a) Bradycardia
b) Rash
c) Angioedema
d) Tachypnoea
e) Low Blood Pressure

108. Dr. Gives Him EpiPen, What's Not True?


a) He Can Use EpiPen Over His Pants
b) He Should Shake EpiPen Before Using
c) He Should Keep EpiPen Out of Direct Sunlight
d) Effect Last 10-20 Min

109. What to do after pt. stabilize from anaphylactic shock:


a) Give Corticosteroid
b) Watch Pt 48 Hr
c) It May Be Delayed In 24hr
110. What reference to use if a customer is coming to ask you about the latest Canadian
guideline of malaria, so which one u will check:
a) E-CPS
b) Medline
c) Health Canada website  includes guidelines for several epidemic diseases
d) American guidelines database

111. Which reference u will use if you want to know the latest guidelines published and
protocol management in certain disease:
a) E-CPS
b) AHFS
c) Medline (pubmed)
d) Clinical practice guidelines

112. A long case about asthmatic patient who worked in the office as computer analyst, recently
he has moved to the basement. He is using salbutamol prn and Symbicort budesonide 100 mg 1
puff bid, drink one glass of wine every night...etc. His asthma is worsening, and he skip a day
from work, what is the goal of therapy in this patient:
a) Do not lose more work day

113. Which the probable cause for worsening of his


condition: his residence

114. What is the best intervention in managing his asthma?


a) Add prednisone oral
b) Increase his Symbicort dose.
c) Review inhaler technique with him or her
SYMBICORT should be taken every day as 2 puffs in the morning and 2 puffs in the evening. If you
miss a dose of SYMBICORT, you should take your next dose at the same time you normally do

115. A patient recently diagnosed with hypertension, works 2 jobs, eat all his food as junk food,
takes salt substitute supplement, smoker, 48 years old, what to give him for hypertension:
a) Nadolol
b) Clonidine
c) HCTZ
d) Ramipril
e) Furosemide
First-line therapy for uncomplicated hypertension. Effective in patients with ISH isolated systolic
hypertension, elderly and black patients. (effective in patients with moderate to severe renal dysfunction)
116. Cancer pt has systemic fungal infection, he has mild renal dysfunction, what to give?
a) Systemic Fluconazole
b) Orale Ketoconazole
c) Orale Itraconazole
d) Amphotericin-b

117. A 65-year-old man moved to florida and he has refills in your pharmacy, his son now came
to ask for the father's refills to mail it to his father or you mail the medications to florida
a) Refuse to give the son without authorisation from the father
b) Mail the medications to florida
c) The father has no longer the right to get his medications as he moved to florida
d) Medications are not allowed to cross the american borders

118. Goal of alzheimer medications is to?


a) Decrease disease progression
b) Decrease getting lost in the subway
c) Improve motor function
The goal of treatment is to slow down the progression of dementia-related impairments and to control
behavioural symptoms, which may be treated with a combination of psychotherapy, environmental
modifications, and medication.

There was question on marijuana sales. Learn about how to obtain marijuana from pharmacy
and sales about the same.
119. Health Canada marijuana
a) Legal and can be provided by health Canada directly in different variety.

120. Marijuana production at home


a) Health Canada allows 1 pot for patient with terminal illness such as cancer.

121. Marijuana treated people that have the permission of to deal with it
a) Can have inspection from authority.

122. What is included in a diabetic clinic?


a) Chiropractor
b) Physiotherapist
c) Podiatric
123. Pt. J. Is a 35 y. O. Female who is placed on lithium therapy. The suggested dose is 600 mg
q8h of lithium carbonate. The total body clearance of lithium is 0.44ml/s or 1.62l/h.the
biological half life is 18 hours. The mole weight of lithium carbonate is 74.
How long will it take to reach 94% of steady state?
a) 18 hours
b) 1 day
c) 1.5 days
d) 3 days  (94% steady state after 4-5 t1/2 = 4.5 x 18 = 81 hr = 3 days)
e) 5 days  (99% after 6.6 t1/2,100 % steady state after 7 t1/2)

124. The lithium dose is now changed to 300 mg q8h.how long will it take lithium to reach 94%
of steady state
a) 18 hours
b) 1 day
c) 1.5 days
d) 3 days  (94% steady state after 4-5 t1/2)
e) 5 days  (100 % steady state after7 t1/2)

125. Dr. Prescribes 20 meq k for a patient. But the person is not willing to take k. Dr. Told him
to take banana and one banana contain 602 mg k. How many bananas he need to cover 20 meq
k?
1 meq = 39 mg k 20 meq = 780 mg k 780 mg / 602 mg = 1.3 banana (ans)

126. A patient is with aspirin overdose; his blood PH is 7.2. Which of the following is true for
him?
a) Respiratory alkalosis
b) Metabolic acidosis.
c) Metabolic alkalosis.
d) Respiratory acidosis

127. What's the major risk factor for Alzheimer?


a) Dyslipidaemia
b) Age
c) HTN
d) Family History
e) Vascular Disease
All are risk factors, but age is the major one
Another version. Which one is not the risk
factor for dementia?
a) Older age
b) Emotional stress
c) Female gender

128. A woman having watery discharge, sneezing, itching and congestion. She has been taking
antihistamine for the past few days but irregularly. She doesn't feel better. What's the
appropriate action?
a) Take the antihistamine regularly for the next 2 weeks
b) Add nasal decongestant PRN to the antihistamine
c) Add oral decongestant to the antihistamine

129. She also has dry cough. So, what to give


a) Dextromethorphan
b) Dextromethorphan/Pseudoephedrine

130. 18-month infant has congestion in his nose. What should you do?
a) Normal Saline Drops
b) Xylometazoline
c) Pseudoephedrine Drops

131. He is suffering barking cough as well, what is the appropriate action to take? (barking
cough= croup)
a) Give Him Dextromethorphan
b) Give Him Guaifenesin
c) Refer to doctor
d) Nonpharmacological measures
132. A 4-year-old child having rhinorrhoea, nasal discharge, dry cough and mild fever. Which
of the following drug will you give?
a) Pseudoephedrine
b) Dextromethorphan
c) Chlorpheniramine
d) Loratadine
e) Saline nasal drops

133. Which of the following is a true statement about Rizatriptan Wafer?


a) It is absorbed from the buccal cavity  from stomach, swallow
b) Co-administration with alcohol is contraindicated
c) It is used for migraine with nausea
d) It is absorbed faster than Rizatriptan tablets
e) It is contraindicated with people who have difficulty swallowing

134. A patient has hypothyroidism coming to you in the pharmacy. He's complaining about
having all of the following symptoms except.
a) Weight Gain
b) Cold Intolerance
c) Oily Skin
d) Constipation
e) Confusion

135. Question about the risks associated with the use of hormonal contraception. All except ..

136. Elderly patient 84 years old who is immunocompromised. He's living with his daughter
and his grandchildren. The dr. Is hesitant about giving the flu vaccine. What's the most
appropriate action?
a) Give him the flu vaccine
b) Immunize his close contacts and caregivers*
c) Don’t recommend immunizing him because he is immunocompromised
d) Give him zanamivir as prophylaxis

137. Patient has ascites. He's currently taking spironolactone. What can you add to his therapy?
a) Furosemide
b) HCTZ

138. Which of the drug causes biliary pseudolithiasis?


a) Gentamycin
b) Vancomycin
c) Ceftriaxone
d) Cefotaxime
e) Penicillin

139. What is used for prophylaxis of appendectomy?


a) Cefazolin
b) Cefotetan Or Cefoxitin
c) Cephalexin
Appendectomy should be preceded by IV antibiotics. Third-generation cephalosporins are preferred. For
nonperforated appendicitis, no further antibiotics are required. If the appendix is perforated, antibiotics
should be continued until the patient’s temperature and white blood cell count have normalized or
continued for a fixed course, according to the surgeon’s preference. If surgery is impossible, antibiotics —
although not curative—markedly improve the survival rate. Although several studies of nonoperative
management of appendicitis (ie, using antibiotics alone) have shown high rates of resolution during the
initial hospitalization, a significant number of patients have a recurrence and require appendec tomy
during the following year. Thus, appendectomy is still recommended.

140. Where to find look-alike drugs?


a) ISMP
b) CPS
c) TC

141. Which of the following cannot be applied for sunburn?


a) Corticosteroid
b) Lidocaine
c) Calamine
d) Ibuprofen

142. A drug is given at the dose of 1g iv q12h to a patient. The desired therapeutic level of the
drug in the blood is 15 – 20 mg/l. When blood drawn just before the next dose, the
concentration was found 10.57 mg/l. What is the appropriate dose of the drug so that the
desired therapeutic level is maintained?
a) 1125 mg iv q12h
b) 1250 mg iv q12h
c) 1500 mg iv q12h
d) 2500 mg iv q12h
143. You have a stock solution of 10% w/v. You are required to dilute it as 1:5 by adding
suitable diluents. The total final volume of solution is 5 ml. What is the appropriate volume of
diluents and stock solution you need to make the required solution? [n-07]
a) 4 ml diluents and 1 ml stock solution
b) 4.5 ml diluents and 0.5 ml stock solution
c) 4.9 ml diluents and o.1 ml stock solution
10 % Soln. diluted as 1:5 become 2 %
C1 * V1 = C2 * V2 2% * 5 = 10% * V2 V2 = 2% * 5 / 10% = 1ml
So, 1 ml stock solution & 4 ml (rest of 5 ml needed) diluent

144. In the above question, what would the amount of drug in the final 5ml solution?
a) 10 mg
b) 100 mg
c) 200 mg
Final solution: 2% means 2 gm ------- 100 ml X gm ------- 5 ml
X=5*2 /100 = 0.1 gm = 100 mg

145. Tobramycin iv 400mg/24h was given to a patient. Peak concentration is required is >20mg
and trough concentration required is < 0.5 mg. Post dose peak is 28 mg/l and after 10 hours is 7
mg/l. Find the t1/2 of the drug.
a) 5 hours
b) 9 hours
c) 10 hours
d) 15 hours
Solution: ln c – ln co = kt Ln 28 – ln 7 = k x 10 3.33 – 1.94 = 10 k K = 0.139
So t1/2 = 0.693/0.139 = 5 hours

146. In the above patient what is the correct dose?


a) 400 mg/24 hours
b) 400 mg/36 hours
c) 400 mg/48 hours
d) 400 mg/72 hours

147. A drug a contain 10% and drug b contain 20% and dr. Wants to prepare 2% of normal
saline solution. Both drug A and drug B have 1%. How much drug a and drug b are needed for
this preparation?

148. A clinical trial report shows that a control drug group 338 patients out of 9332 patients
died and in the test drug group 320 patients out of 9650 patients died. Find the relative risk
reduction?
Solution:
338/9332*100 = 3.62% 320/9650*100 = 3.32%
RRR = (3.62 – 3.32) / 3.62 * 100 = 8.29% Ans. 8.3%

149. A clinical trial report shows that a control drug gives side effects in 50 patients among 500
and a test drug gives 25 among 500 patients. Find the number needed to treat?
Solution:
50/500*100 = 10% 25/500*100 =5%
Relative risk = 10% - 5% = 5% Number needed to treat = 1/5% = 20 (ans)
150. 30g drug is needed to cover the entire body area. If a woman needs 20% of the body bid
for 7 days, how many grams of the drug should be dispensed?
Solution:
30 g x 20% = 6g needed daily single application 6 g x 2 times x 7 days = 84 g (ans)

151. Which of the following drugs require a written prescription?


a) Ketamine
b) Diazepam
c) Zopiclone
d) Lorazepam
e) Ketorolac

152. Which of the following products require sales record?


a) Tylenol # 3
b) Ketamine
c) Phenobarbitol
d) Diazepam
e) Nalbuphine

153. Canadian health act includes all except?


a) Accessibility
b) Portability
c) Universality
d) Affordability
e) Comprehensiveness

154. The following narcotic drugs have expired in your pharmacy. You want to return it to the
distributor for destruction. What is the appropriate action?
a) Wait until written authorization from the office of the narcotic substances.
b) Give back these drugs to the distributor and ask for payment.
c) Destroy in front of intern

155. A drug is recalled. Pharmacist will do all of the following except?


a) Post your display in the pharmacy
b) Look for batch size & number which is affected
c) Check the computer to whom the drug is dispensed and call patient
d) Check the lot number of the available stock

156. Person is overdose with methanol. How can we treat?


a) Ethanol
b) Activated charcoal
c) Ipecac

157. A patient was diagnosed with peptic ulcer disease due to h. Pylori infection. His doctor
prescribed him the following medicines.
 Omeprazole 20mg bid x 7 days
 Clarithromycin 250 mg bid x 7 days
 Amoxycillin 1g bid x 7 days.
What is the pharmacist’s appropriate action?
a) Dispense as written
b) The appropriate dose of clarithromycin is
500mg bid x 7 days.
c) Ask the patient to take all medicines in empty
stomach

158. Which reference you will check for off-label indication of a drug?
a) CPS
b) Martindale
c) Therapeutic choices
d) Medline

159. Which of the following drugs to be given to acute acetaminophen overdose without delay?
a) N-acetylcysteine
b) Ipecac emesis
c) Charcoal
160. A vitamin d capsule contains 1.25mg. Each 100mcg = 400 iu. If you want to prepare 60
gm containing 10,000 iu, how many capsules will you need? 2 capsule (ans)
Solution:
1.25 mg x 1000 = 1250 mcg Now 100 mcg = 400 iu 1250 mcg = 5000 iu
No. Of capsule = 10,000 iu / 5000 iu = 2 cap

161. An eight months pregnant woman was suffering from uncomplicated UTI. Best medication
for this patient:
a) Fosfomycin
b) Smx/tmp
c) Ciprofloxacin
d) Amoxicillin
e) Cephalexin
162. Which of the following is not true regarding aldara (imiquimod)?
a) Apply 3 times per week at bedtime
b) Keep the product under refrigeration
c) Wash hands before and after application
d) Keep the application area open
e) Use fresh pack every time
Storage and Stability: Store between 15–25°C. Avoid freezing.

163. Which of the following prescription prompt pharmacist intervention while dispensing?
a) Azithromycin tid
b) Warfarin 5mg once daily
c) Levothyroxine 50mcg once daily

164. An elderly patient with history of open-heart surgery recently now turned into
osteomyelitis. Culture shows s. aureous sensitive to cloxacillin, vancomycin and clindamycin
and resistant to penicillin and cephalosporins because of lactamase production. The patient is
allergic to penicillin. Which of the
following drugs should be given?
a) Vancomycin
b) Cloxacillin
c) Cephalexin
d) Ceftriaxone

165. in the above case what will be


the duration of therapy?
a) 1-2 w
b) 2-4 w
c) 4-6 w
d) 6-8 w

166. In the above case, what should be monitored?


a) C-Reactive Protein
b) Troponin
c) Creatinine clearance

167. A patient with acute coronary syndrome came to emergency room. Which is given first?
a) ASA
b) Clopidrogel
c) Nitroglycerine SL Tab
d) Warfarin
e) Dalteparin

168. Patient is on isotretinoin therapy; all need to be monitored except?


a) Liver function test
b) Pregnancy test
c) Kidney function
d) CBC with differential

169. What are the counselling of isotretinoin?


a) Take on empty stomach
b) Treatment usually results in few months
c) Use a birth control method at least one month before the therapy starts

170. Which of the following causes sexual dysfuntion?


a) Sotalol
b) Thiazide diuretics
Erectile 5-alpha reductase inhibitors acetazolamide alcohol (acute) alcohol (chronic) alpha-blockers
dysfunction anti-androgens barbiturates carbamazepine cimetidine clofibrate clonidine digoxin GnRH
analogues ketoconazole lithium MAOIs metoclopramide opioids phenothiazines phenytoin
spironolactone SSRIs thiazide diuretics tricyclic antidepressants
Delayed 5-alpha reductase inhibitors alpha-blockers
ejaculation antipsychotics SSRIs
Hypoactive 5-alpha reductase inhibitors alcohol (chronic) anti-androgens barbiturates BBs carbamazepine
sexual desire GnRH analogues ketoconazole MAOIs opioids phenytoin spironolactone SSRIs TCAs

171. CARN newsletter part of which?


a) Medeffect
b) ISMP
c) CPS

172. Turnover rate calculation question then answer the question turn over when calculated was
what does that mean?
a) Inventory is moving slowly
b) Finanicially not stable
c) No, this is ideal  4 – 6 times / year

173. Intent to treat analysis, which patients are included?


a) Randomized
b) Withdrawn patients
https://www.ncbi.nlm.nih.gov/pmc/articles/PMC5654877/
Intention-to-treat analysis: A statistical analysis for randomized trials that includes all of the patients who
were randomized to a treatment arm regardless of whether or not they finished the study. An intention-to-
treat analysis is considered to mimic clinical practice more closely than an analysis that includes just the
patients who completed the study.

174. Premenstrual syndrome PMS, which medication has an evidence for usage in pms
a) Soya
b) Black Cohosh
c) Evening Primrose Oil
d) Calcium
Black cohosh has been used in the management of menopausal symptoms including vaginal dryness,
although evidence of efficacy is generally of low quality. Black cohosh can cause nausea and headache and
is contraindicated in pregnancy and breastfeeding.
Soy isoflavones may improve hot flash scores and vaginal dryness but not night sweats.

175. Patient comes to ICU with haemorrhagic stroke and blood pressure 180/110. What is the
following was wrong in the prescription and what is the right action taken by the physician?
a) Pravastatin taken at bedtime
b) Alteplase can’t be given.

176. Which of the following drugs cause leukopenia or neutropenia?


a) Carbamazepine
b) Lamotrigine
Leukopenia occurs transiently in about 10% and persistently in about 2% of carbamazepine-treated patients
and typically develops within the first 3 months of treatment.

177. A lady from south east Asia 45 kg weight and malnourished, diagnosed with tuberculosis
and gets RFP+INH+PZA+ETB+B6VIT, she develops neuropathy; what's the cause?
a) Being malnourished
b) Being under weight
c) Being a woman
d) Being asian

178. Someone is using sulfasalazine; which vitamin may be required:


a) Vitamin B12
b) Folic Acid
c) Vitamin
Aminosalicylates are considered low risk for use
in pregnancy or while breastfeeding.
Due to potential antifolate effects, women taking
sulfasalazine may be safely switched to 5-ASA or
are generally advised to supplement with 2 mg of
folic acid daily starting before conception and
continuing throughout their pregnancy.

179. The following are true about Sulfasalazine EXCEPT:


a) It acts systemically in the treatment of Ulcerative Colitis
b) Discolors urine and may color skin orange yellow.
c) Reduced absorption of folic acid and digoxin.
d) Take after meals to reduce GI distress and to facilitate passage into intestine.
e) Maintain adequate fluid intake to prevent crystalluria and stone formation.
5-aminosalicylic acid (5-ASA) includes both mesalamine (2 linked 5-ASA moieties) and sulfasalazine (a
sulfapyridine linked to a 5-ASA moiety with an azo bond). The term 5-ASA generally refers to mesalamine
with sulfasalazine being differentiated and referred to directly by name. 5-ASA may be administered orally
or rectally. Oral preparations are formulated to release 5-ASA at specific sites in the gastrointestinal tract
since efficacy is dependent on luminal concentration. Salofalk and Pentasa release 5-ASA in the small bowel,
allowing it to be available in the small bowel and colon.
Sulfasalazine, olsalazine, Asacol and Mezavant release 5-ASA primarily in the colon.
All oral 5-ASA compounds, irrespective of their formulation, are equally effective and safe for induction of
remission and prevention of relapse of mild to moderate UC. 5-ASA is ineffective for induction or
maintenance of pharmacologically or surgically induced remission of CD while sulfasalazine may have a
modest benefit for induction of remission in mild colonic CD.
Sulfasalazine has the least favourable adverse effect profile, which includes nausea, headache, rash,
hemolytic anemia, hepatotoxicity and reports of reversible oligospermia. However, many of these
effects are minor and dose-related, and the majority of these events (>90%) are related to the
sulfapyridine moiety, which is not present in the 5-ASA preparations. In addition, the structure of
sulfapyridine is similar to that of sulfamethoxazole and may trigger a cross reaction in patients with a
history of allergy to sulfonamide antibiotics. Adverse reactions to 5-ASA therapies include hypersensitivity
reactions and, infrequently, renal toxicity in the form of interstitial nephritis.

180. Which of the following is not risk factor OR is not a trigger for psoriasis:
a) Sun
b) Wind
c) Aspirin Use
d) Pregnancy
e) Cold weather
f) Trauma
g) Infection
h) Vit D deficiency
(Non-steroidal anti-inflammatory drugs (NSAIDs) have been reported to induce or exacerbate psoriasis. ...
There was no clear association between aspirin and risk of psoriasis or PsA. In conclusion, long-term
acetaminophen and NSAIDs use may be associated with an increased risk of PsA)

181. Doctor called asking for patient counselling question, which reference to check?
a) CPS
b) Micromedex
c) Google Search

182. Dr. Asked about a new drug dosing. Where to check it:
a) CPS
b) Guideline
c) TC

183. Patient has Parkinson's disease he has 2-3 conditions he wants to know if any of his
medication will interact with Parkinson's drugs, which reference will he see?
a) Google search
b) Micromedex
c) Drug product database
d) Medline
e) Dipiro pharmacotherapy
184. Blister-pack patient which of the following is incorrect?
a) Alendronate given in the evening or bedtime.
b) Calcium given after meals

185. Total parenteral nutrition question about proteins and carbohydrate (check ansel)

186. A patient comes to your pharmacy and the patient sits down and keeps getting back up
what symptom is this patient experiencing?
a) Akathisia
187. This is his first episode of psychosis how long will this patient be treated for?
a) At least one year

Codeine scenario
188. Patient comes to you and tells you that he is taking 60 mg bid instead of 30 mg bid which
is his prescribed dose of codeine for pain control, what will you do?
a) Discuss with the doctor regarding his increase in dose

189. Now the doctor has prescribed him fentanyl what is the correct dosing for fentanyl
a) 25 mcg/ hr for 72 hrs

190. Palliative care Patient taking more than 2 narcotics what is your least concern
a) Abuse potential with taking too many opioids

191. In a hypersensitivity reaction all is true except?


a) Corticosteroids are used once the patient is stable
b) Hypersensitive reactions occur in 3-5 mins (acute)
c) Hypersensitive reactions occur in 1-2 hours (delayed)
d) It occurs in monophasic system
e) Need to monitor the patient for the next 48 hours

192. What is the goal of ISMP?


a) Collecting all the errors in pharmacy and trying to find a general pattern of error.
b) To prevent patient harm
c) Gathers information on incidents
Mission is to advance patient safety worldwide by empowering the healthcare community, including
consumers, to prevent medication errors.

193. All the following abbreviations can cause error except?


a) Abbreviated drug name
b) Using @ instead of at
c) Using qd for once daily
d) Using leading zeros for less than one (least to cause error)

194. Medical reconciliation goal is to


Medication reconciliation is the process of comparing a patient's medication orders to all
of the medications that the patient has been taking.
This reconciliation is done to avoid medication errors such as omissions, duplications,
dosing errors, or drug interactions.

195. For SOAP analysis in a diabetic what is subjective


a) Gender
b) Body mass index
c) Medical history
d) Blood pressure

196. For which of the following condition showing picture and gesture will be beneficial except
a) Hearing problems
b) Cognitive disability
c) Low literacy
d) Language barrier
e) Emotional distress

197. A pharmacy manager notices that a technician dispenses and exempted codeine product
while the other pharmacist is busy; what is the pharmacy manager supposed to do?
a) Regulate sale of exempted codeine products by pharmacist.
b) Pharmacist has to counsel patient for dispensing exempted codeine products
c) Pharmacy manager has to intervene in drug diversion.

198. Pharmacy manager suspect that one of the staff members diverge narcotics within the
pharmacy. What will the pharmacy manager do?
a) Call police for the investigational strategies.
b) Arrange the staff members to watch him
c) Dismiss the technician
d) Restrain the technician to discipline

199. Pharmacist working in a community pharmacy and see lady taking codeine syrup. He saw
the same lady take the syrup in another pharmacy in which he was working yesterday. What
will be the first action of the pharmacist?
a) Ask how the old cough syrup is working in her condition.

200. Patient has diarrhea from last 2 weeks and has chronic fatigue for the last month what will
you do?
a) Refer her to the doctor
b) Refer her to emergency
201. Pharmacist has ordered a vaccine the vaccine has two parts api + diluent. The diluent needs
to be stored at 15-25 c or 2-8 c whereas the api needs to be stored in the freezer what will you
do to maintain cold chain and storage?
a) Store diluents in the fridge and api in freezer until dispensing.

202. Doctor calls you to help a senior patient who is having difficulty remembering to take
multiple drugs all of the following strategies are helpful except
a) Using easy open vials for dispensing.
b) Send her medication in blister back
c) Let her son to take her medication

203. A patient comes to your busy pharmacy with a rx for losec many other patients were
waiting to pick up their medication. Patient comes back and picks up lasix instead what is the
reason for this error (K-type)
a) Pharmacy was busy
b) Drugs look alike
c) Because patient drop rx and came after 20 min

204. A patient is found to have osteoporosis with a history of diabetes he is taking the following
medication prednisone, alendronate, calcium and vitamin d which of the following is a concern
to the pharmacist
a) Drug drug interaction
b) Drug disease interaction (prednisone: hyperglycemia)
Note if they ask for a DTP in this question the patient requires an additional medication for which he is not
receiveing appropriate therapy. Because the patient is on chronic prednisone, he also requires a PPI

205. A patient is taking gemfibrozil, atenolol he has been newly prescribed clarithromycin he is
also taking tadalafil, and lisinopril. Which of the following drug drug interaction occurs?
a) Tadalafil and clarithromycin
b) Lisinopril and clarithromycin
c) Gemfibrozil and clarithromycin
CYP3A4 inhibitors (e.g., cimetidine, clarithromycin, efavirenz, erythromycin, grapefruit juice, itraconazole,
ketoconazole, ritonavir) can significantly decrease metabolism of PDE5 inhibitor.
CYP3A4 inducers (e.g., carbamazepine, phenobarbital, phenytoin, rifampin) may decrease efficacy of the
PDE5 inhibitor.

206. Which is seen in a community pharmacy?


a) Cylindrical measure should be kept under sterile conditions when not in use
b) Glass measures should be replenished after one year of use
c) The use of plastic is preferred over glass especial for coal tar
d) Cream can be prepared on a glass slab or a paper pad

207. Federal government plans, which one doesn’t cover?


a) Children < 25 years
b) Veterans
c) Natives or aboriginals
d) RCMP

208. For a drug to be used in pregnancy which of the following would you consider; except?
a) Clinical trial, animal study for teratogenicity
b) Clinical report
c) Randomised controlled Trial for study

209. Heparinoids except?


Heparinoids are glycosaminoglycans which are derivatives of heparin.
They reduce swelling and helps with healing, also, used to treat: bruises, internal bruising (haematoma),
varicose veins and other inflamed veins (phlebitis), piles (haemorrhoids) and itchy bottom
It's safe to use heparinoid while you're pregnant or breastfeeding.

210. Publication bias definiton


Tendency of pharmaceutical companies and researchers to handle the reporting of experimental results
Publication Publication bias is a type of bias that occurs in published academic research.
bias It occurs when the outcome of an experiment or research study influences the decision
whether to publish or otherwise distribute it.

211. A pharmacist wants to send a medication back for credit to the manufacturer which of the
following doesn’t need prior authorization to send back
a) Tranylcypromine
b) Diazepam
c) Ketamine
d) Amphetamine
e) Testosterone

212. Which of the following is a targeted substance according to the drug controlled and
substance act?
a) Clonazepam
b) Ketamine
c) Amphetamine
d) Atomoxetine
e) Ketoralac

213. A pharmacist wants to organize a seminar for hypertension he will consult all of the
following except
a) Hospital
b) Community doctor
c) Heart and health society (heart and stroke foundation)
d) Community nurse
e) All of the school board

214. A newly diagnosed patient with hypertension and BPH which of the following can be
given?
a) Fosinopril
b) Terazosin
c) Clonidine

215. A lady has found a new job. Now her company is providing her medical insurance with
monthly premium of 100 dollars she has decided that in one month she will get insurance she
comes into the pharmacy for advice she occasionally takes antibiotics. Your advice would be?
a) When you have severe disease take insurance
b) Premium of other employees would reduce
c) It will help in increasing health canada expenses
d) Patient has to find an exclusion clause if she can later get on insurance plan

216. All of the following are time dependant killing antimicrobials except?
a) Penicillin
b) Clindamycin
c) Erythromycin
d) Levofloxacin
e) Ceftriaxone
Time dependant antibiotics are B lactams, cephalosporins, macrolides, clindamycin, lizenolide,
carbapenenems. Concentration dependent antibiotics are Quinilones, aminoglycosides & azithromycin

217. Gonorrhea. What is the treatment for non gonococcal infection?


a) Ceftrioxone + azithromycin
b) Ceftazidime + penicillin
c) Doxycycline + ciprofloxacin
d) Spectinomycin

218. A pregnant woman has gonococcal infection which of the following is the drug of choice
a) Spectinomycin
b) Cefixime (ceftriaxone)
c) Doxycycline
d) Levofloxacin

219. She was also found to have chlamydia infection which of the following is drug of choice
a) Azithromycin
b) Doxycycline
c) Ceftriaxone
d) Penicillin

221. Mild renal impairment and fungal infection


a) Nystatin
b) Amphotericin b
c) Fluconazole
d) Ketoconazole
e) Itraconazole

222. In olanzapine you would monitor all of the following except


a) White blood cells
b) Blood glucose level (it increases risk of diabetes)
c) Lipid profile (it causes weight gain and increases risk of dyslipidemia)
d) Liver function (it causes hepatic transaminase elevation)
N.B. other adverse effects: dizziness, sedation, orthostatic hypotension, anticholinergic effects and
extrapyramidal symptoms

223. Pharmacy owner/manager wants to check staff member evaluation within time he would
check
a) Work efficiency
b) Check his availability regarding his position

224. When a pharmacist dispensing a drug, he should check all the following in medication
except
a) Efficacy
b) Appropriateness
c) Accuracy
d) Safety

225. A patient with endocarditis, what is the cause?


a) S. viridians
b) S. aureus
c) P. aeruginosa
d) Strep. pneumonia

226. It is resistant to penicillin (no amoxicillin or ampicillin in the question), what to give?
a) Vancomycin
b) Gentamicin

227. What should be monitored in the above case, after choosing the antibiotic?
a) Renal function

228. A patient with diabetes and increased triglycerides and is taking gemfibrozil all of the
following are counselling parameters except
a) Use peanut oil / coconut oil

229. A patient who is now living in the USA sent his son to collect his refill for the next 6
months. The patient had filled his prescription 10 weeks ago, and his insurance company only
covers 3-month refills. So, what will you do?
a) Tell him he is only allowed to fill for 3 months
b) Tell him he can take the whole 6 months but he has to pay for the uncovered 3 months
c) Tell him that since his dad now lives in the usa he isn’t entitled for refills from canada
d) Tell him that his dad should get a physician in the states
230. Pt with DM plus BP, what is the DOC?
a) ACEI
b) BBs
c) Diuretics

231. Someone who has HF and peripheral edema, he is using spirinolactone 100 mg, but no
response (edema). What do you recommend:
a) Add furosemide
b) Reduce fluid intake

232. For improvement monitoring in above patient, what do you recommend: Weighing patient
233. After using furosemide 80 mg, there is no response, what do you recommend:
a) Increase furosemide dose (double dose)

234. QT prolongation?
a) Sotalol
b) Propranolol

235. Which antibiotic has interaction


with sotalol
a) Quinolones

236. What is the causative microorganism of


non-purulent cellulites?
a) Streptococcus group A B-hemolytic
b) S. aureus
c) H-influenza
d) Enterococci

237. It can be treated by all of the following


except?
a) Cephalexin
b) Cloxacillin
c) Norfloxacin
d) Clindamycin
e) Amoxicillin/clavulanate

238. duration of treatment of cellulitis?


a) 5 days
b) 3 days
c) 10 days
d) 21 days
e) 28 days

239. What is the endpoint treatment of cellulitis?


a) Reduce inflammation
b) Negative culture of skin sample
c) Disappearance of lesions
Cellulitis monitor: No Erythema, No fever within 7days & Negative culture

240. pt. came home after STEMI, he


had atrial fibrillation & Cardiogenic
shock; what you should give?
a) Warfarin
b) Clopidogrel
c) ASA

241. A patient with itchy watery eyes


and diabetes, when will you not refer
her to the doctor?
a) Blurred vision
b) Itchy watery eyes
c) Her diabetes

242. The above patient is diagnosed with glaucoma and she is given latanoprost you would
counsel her to
a) Store it in the fridge
b) Causes eye pigmentation

243. Patient having iron anemia, all are true except:


a) Large mean corpuscular volume MCV
b) Hemoglobin deficiency
c) Increased capacity for binding with iron
d) Low hematocrit

244. A case who was taking atorvastatin for a long time now after increasing its dose from 10
mg to 20 mg she gets myalgia, but you should find this among lots of information given in the
case, then it asks whats the cause of aches.
it was repeated qs but slightly different. It had something to do with diet?

245. Diabetic pt 48 yrs old pt give vaccine, except?


a) Flu vaccine
b) Zostavax(shingles)
c) Tetanus
d) Chicken pox (varicella vaccine)
If you're 50 or older, get Shingrix. Get Shingrix even if you already had shingles, because you can get the
disease more than once. Your risk of shingles and complications increases as you age. You need 2 doses of
Shingrix. Get the second dose 2 to 6 months after you get the first dose.

246. All the following drugs can be given in acute gout except:
a) Naproxen
b) Colchicine
c) Sulfinpyrazone
d) Prednisolone

247. A woman had unprotected sex 2 days ago, her last period was 14 days ago, you should
advise her of all of the following except:
a) Go to the doctor
b) Plan b is an option for her
c) Take a pregnancy test
d) Go to the women’s health clinic
e) Tell her to contact the assaulted women’s hotline for counselling

248. A pregnant woman, advise her of all of the following except:


a) Take vitamin A supplements (teratogenic)

249. A woman asks you what she should do with her leftover prenatal vitamins:
a) She can continue taking them after delivery

250. Pharmacoeconomic studies are done to benefit:


a) Society
b) Insurance companies
c) Patients

251. Insurance qs about wife’s plan which covers son not pt. He wants son’s asthma drug for
himself….
a) No, you can’t dispense

252. A patient is experiencing shortness of breath, his legs are in pain and swelling, what is the
diagnosis:
a) Deep venous thrombosis (DVT)
DVT may occur in ambulatory patients or as a complication of surgery or major medical illness. Among high-
risk hospitalized patients, most deep vein thrombi occur in the small calf veins, are asymptomatic, and may
not be detected.
When present, symptoms and signs of DVT (eg, vague aching pain, tenderness along the distribution of the
veins, edema, erythema) are nonspecific, vary in frequency and severity, and are similar in arms and legs.
Dilated collateral superficial veins may become visible or palpable. Calf discomfort elicited by ankle
dorsiflexion with the knee extended (Homans sign) occasionally occurs with distal leg DVT but is neither
sensitive nor specific. Tenderness, swelling of the whole leg, > 3 cm difference in circumference between
calves, pitting edema, and collateral superficial veins may be most specific; DVT is likely with a combination
of ≥ 3 in the absence of another likely diagnosis (see table Probability of Deep Venous Thrombosis).
Low-grade fever may be present; DVT may be the cause of fever without an obvious source, especially in
postoperative patients. Symptoms of pulmonary embolism, if it occurs, may include shortness of breath and
pleuritic chest pain.
Common causes of asymmetric leg swelling that mimic DVT are
 Soft-tissue trauma
 Cellulitis
 Obstruction of a pelvic vein
 Obstruction of a lymphatic vessel in the pelvis
 Popliteal bursitis (Baker cyst) that obstructs venous return
Less common causes include
 Abdominal or pelvic tumors that obstruct venous or lymphatic return
 Symmetric bilateral leg swelling is the typical result of use of drugs that cause dependent edema (eg,
dihydropyridine calcium channel blockers, estrogen, high-dose opioids), venous hypertension (usually
due to right heart failure), and hypoalbuminemia; however, such swelling may be asymmetric if
venous insufficiency coexists and is worse in one leg.

253. Depo provera, after how long if u miss dose, you can get pregnant or they mean max gap
which u cannot give between 2 inj is?
a) 1 week
b) 2 weeks
Interval between injections must not exceed 13 wk.

254. A patient using ipratropium, salbutamol, steroid inhaler, and still experiences a copd
exacerbation, so what is the appropriate action:
a) Antibiotic + systemic corticosteroids
b) Systemic corticosteroid only
c) Long acting beta 2 agonist + systemic corticosteroids

255. A patient with lung cancer is taking morphine to control his pain; his family is worried
about the use of morphine, what you should say:
a) Morphine addiction isn’t a major concern in cancer patients

256. A patient with pneumocystis carnii pneumonia, what is the drug of choice:
a) Co-trimoxazole

257. A patient traveling within 45 minutes, what should you give him to avoid motion sickness:
a) Scopolamine (onset of action after 4 hours)
b) Dimenhydrinate (onset of action after 30 minutes)
Treatment
 Prophylactic drugs (eg, scopolamine, antihistamines, antidopaminergic drugs)
 Nondrug prophylaxis and treatment measures
 Antiemetic drugs (eg, serotonin antagonists)
 Sometimes IV fluid and electrolyte replacement
People prone to motion sickness should take prophylactic drugs and use other preventive measures before
symptoms start; interventions are less effective after symptoms develop. If vomiting occurs, an antiemetic,
given rectally or parenterally, can be effective. If vomiting is prolonged, IV fluids and electrolytes may be
required for replacement and maintenance.
Pregnant women should treat motion sickness as they would treat nausea and vomiting during early
pregnancy.
Scopolamine
Scopolamine, an anticholinergic prescription drug, is effective for prevention, but efficacy in treatment is
uncertain. Scopolamine is available as a 1.5-mg transdermal patch or in oral form. The patch is a good
choice for longer trips because it is effective for up to 72 hours. It is applied behind the ear 4 hours before its
effect is required. If treatment is needed after 72 hours, the patch is removed and a fresh one is placed
behind the other ear. The oral form of scopolamine is effective within 30 minutes and is given as 0.4 mg to
0.8 mg 1 hour before travel and then every 8 hours as needed.
Anticholinergic adverse effects, which include drowsiness, blurred vision, dry mouth, and bradycardia, occur
less commonly with patches. Inadvertent contamination of the eye with patch residue may cause a fixed and
widely dilated pupil. Additional adverse effects of scopolamine in the elderly can include confusion,
hallucinations, and urinary retention. Scopolamine is contraindicated in people who are at risk of angle-
closure glaucoma.
Pearls & Pitfalls
 If an elderly person becomes confused and develops a fixed, dilated pupil while traveling,
consider scopolamine toxicity (as well as intracranial hematoma with brain herniation).
Scopolamine can be used by children > 12 years in the same dosages as for adults. Use in children ≤ 12 years
may be safe but is not recommended due to the higher risk of adverse effects.
Antihistamines
The mechanism of action for antihistamines is probably anticholinergic. All effective ones are sedating;
nonsedating antihistamines do not appear to be effective. These drugs can be effective for prevention and
possibly treatment. Anticholinergic adverse effects may be troublesome, particularly in the elderly.
Beginning 1 hour before departure, susceptible people may be given
nonprescription dimenhydrinate, diphenhydramine, meclizine, or cyclizine in the following doses:
 Dimenhydrinate: Adults and children > 12 years, 50 to 100 mg orally every 4 to 6 hours (not to exceed
400 mg/day); children 6 to 12 years, 25 to 50 mg orally every 6 to 8 hours (not to exceed 150 mg/day);
children 2 to 5 years, 12.5 to 25 mg orally every 6 to 8 hours (not to exceed 75 mg/day)
 Diphenhydramine: Adults, 25 to 50 mg orally every 4 to 8 hours; children ≥ 12 years, 25 to 50 mg orally
every 4 to 6 hours; children 6 to 11 years 12.5 to 25 mg orally every 4 to 6 hours; children 2 to 5 years,
6.25 mg orally every 4 to 6 hours
 Meclizine: Adults and children ≥ 12 years, 25 to 50 mg orally every 24 hours
 Cyclizine: Adults, 50 mg orally every 4 to 6 hours; children 6 to 12 years, 25 mg 3 or 4 times a day
Cyclizine and dimenhydrinate can minimize vagally mediated gastrointestinal symptoms.
Antidopaminergic drugs
Promethazine 25 to 50 mg orally 1 hour before departure and then twice a day appears to be effective for
prevention and treatment. The dosage in children 2 to 12 years is 0.5 mg/kg orally 1 hour before departure
and then twice a day; it should not be used in children < 2 years because of the risk of respiratory
depression. Adding caffeine may increase efficacy. Metoclopramide may also be effective, but evidence
suggests it is less so than promethazine. Adverse effects include extrapyramidal symptoms and sedation.
Benzodiazepines
Benzodiazepines (eg, diazepam) may also have some benefit in the treatment of motion sickness but do
have sedative effects.
Serotonin antagonists
Serotonin (5-HT3) antagonists, such as ondansetron and granisetron, are highly effective antiemetics.
Examples of potential dosages for ondansetron are as follows:
 Adults: 4 mg to 8 mg orally every 8 to 12 hours
 Children 6 months to 10 years: 8 to 15 kg, 2 mg orally; > 15 kg, 4 mg orally
Nondrug measures
Susceptible people should minimize exposure by positioning themselves where motion is the least (eg, in the
middle of a ship close to water level, over the wings in an airplane). Also, they should try to minimize the
discrepancy between visual and vestibular stimuli. If traveling in a motor vehicle, then driving or riding in
the front passenger seat, where vehicle motion is most evident (or where motion is most visible), is best.
When traveling on a ship, viewing the horizon or land masses is usually better than viewing a cabin wall.
Whatever the form of transportation, reading and rear-facing seats should be avoided. A supine or
semirecumbent position with the head supported is best. Sleeping can also help by reducing vestibular
sensory input. In space adaptation syndrome, movement, which aggravates the symptoms, should be
avoided.
Adequate ventilation helps prevent symptoms. Consuming alcoholic beverages and overeating before or
during travel increase the likelihood of motion sickness. Small amounts of fluids and bland food consumed
frequently are preferred to large meals during extended travel; some people find that dry crackers and
carbonated beverages, especially ginger ale, are best. If travel time is short, food and fluids should be
avoided.

Adaptation is one of the most effective prophylactic therapies for motion sickness and is accomplished by
repeated exposure to the same stimulus. However, adaptation is specific to the stimulus (eg, sailors who
adapt to motion on large boats may still develop motion sickness when on smaller boats).
Alternative therapies
Some alternative therapies are unproven but may be helpful. These alternative therapies include wristbands
that apply acupressure and wristbands that apply electrical stimulation. Both can be safely used by people
of all ages. Ginger 0.5 to 1 g, which can be repeated but should be limited to 4 g/day, has been used but has
not been shown to be more effective than placebo.

258. Contraceptive, High body weight >90kg? patch is not good for her

259. What is wrong when we instruct patient about using Sumatriptan spray?
a) Tilt your head back when using the spray and stay at that position for 20 seconds
b) Prime the spray several times before using it  no shake no prime
c) Put the spray in one nostril and keep other one closed
The nasal spray should be administered into one nostril only. The device is a ready-to use single-dose unit
and must not be primed before administration. Patients should be advised to read the patient instruction
leaflet regarding the use of the nasal spray device before administration

260. Enteric coated aspirin is used for all except:


a) Prolongation of action
b) To protect the drug from damage by the stomach acidity
c) To protect the stomach lining from damage by the drug

261. Zidovudine, what to monitor?


a) Blood tests
b) CNS
c) Pancreatitis

262. To delegate some duties to technicians, to decrease the workload on the pharmacist, all can
be a barrier except:
a) Pharmacist’s attitude
b) Law
c) Technician’s ability
d) Pharmacist availability

263. pt is having heartburn he took calcium then antacid last night, he took large meal last night,
now having chest pain, what to do?
a) Refer to Doctor
b) Refer to emergency

264. Acetaminophen overdose, what to monitor first?


a) AST
b) ALT
c) Bilirubin

265. Tall man lettering found in? ISMP

266. Ibuprofen cannot be given to? Dehydrated person

267. Which of the following is not RA dose of MTX or is mtx dose for RA
a) 15 mg daily
b) 2.5 daily
c) 7.5 mg weekly
d) 7.5 mg three times weekly
e) 2.5 mg three times weekly
RA dose 20 mg, Psoriasis dose 30 mg

268. All are SE of MTX except


Nausea, malaise, flulike aches, headache, oral ulcers, transient loose stools; bone marrow and liver toxicity
(rare), pneumonitis (rare), immunosuppression, malignancy (rare). Alcohol restriction may minimize
hepatotoxicity.

269. A child with fever and sore throat, chills and pharyngitis. We get sample from throat and
make culture which shows group A beta haemolytic strep, what's doc?
a) Penicillin V
b) Clindamycin
c) Azithromycin
270. In the above case what's the rational for giving antibiotic
a) Preventing rheumatic fever
b) Decreasing the length of sickness
c) Decreasing pain
d) Preventing endocarditis

271. A patient taking clarithromycin and atorvastin what would you ask?
a) About cholesterol levels
b) Ask dr to change meds

272. A solution made up of substance a and substance b with a total volume of 50 ml. The
concentration of the stock solution of substance a is 4%, and you want to make a final solution
of 0.1%. What volume of substance b will you use? 48.75 ml
C X V = C’ X V’ 0.1 x 50 = 4 x V’ therefore, V’ = 1.25 ml
Volume of substance b = 50 – 1.25 = 48.75 ml (most important step)

273. Packaging insert enclosed with a vial containing 1 gm ampicillin Na powder specially that
3.5 ml of sterile H2O is added to the powder and the resulting concentration is 250mg/ml.
Using this information what volume of sterile h2o should be added to make solution containing
100mg/ml? Ans 9.5 ml (question repeated in 2010 november)
250mg  1ml 1000mg  X = 4mg (as they have minus 0.5mg) and it was 3.5mg
Step1---- 100mg …………1ml 1000mg……….x
X = 10ml (since the subtract in the above so we will subtract here) 9.5ml

274. The daily dose of a drug is 1.5gm/ml administered in 3 divided doses. A vial containing 2
gm of the drug is reconstituted by 19 ml of 0.9% nacl (sp.gr 1 gm/ml) to produce 20 ml
suspension. How many ml of suspension is administered in each dose? Ans: 5ml
Solution:
Step #1.1.5gm/ml in 3 divided dose means 0.5g in each dose
Step# 2. 2000mg (2gm) ……20ml x……………………1ml
So, X =100mg/ml 100mg  1ml 500mg  X X = 500/100 = 5ml

275. What is the concentration w/v% of drug in the reconstituted solution in above question?
Answer: 2 gm are in 20 ml X gm are in 100 ml, therefore X = 10% w/v

276. What is concentration w/w %of the solution prepared in question?


Answer: % w/w = 2 / (19 + 2) = 0.095 x 100 = 9.5% w/w

277. A patient is prescribed fentanyl patches he is also taking naproxen what to do?
a) Stop the naproxen 1 day after because the fentanyl patches have started working
b) Stop the naproxen immediately
c) Keep taking the naproxen for as needed pain
d) Dont fill the naproxen prescription

278. A patient is experiencing nausea and vomiting with levodopa, what to give?
a) Bromocriptine
b) Take with food
Levodopa may be taken with food early in therapy to ease nausea; it may be taken on an empty stomach in
more advanced disease to help manage motor fluctuations.
A change in diet to foods that are high in protein (such as meat, fish, dairy products, seeds and nuts) may
delay the absorption of levodopa and may not work as well as it should.
279. 55-year lady, work as a teacher & newly diagnosed with Parkinson, she is experiencing
poor hand writing and she fall down two times falls 2 times and change his full-time job with
part time job, what is the treatment for that case?
a) Pramipexole
b) Selegiline
c) Levodopa
d) Nothing yet
Bromocriptine, pramipexole and ropinirole are effective as monotherapy in the early stages of the disease,
and as adjunctive therapy with levodopa for patients with more advanced motor complications

280. The patient has started to experience dyskinesia what to do


a) Stop COMT and levodopa
b) Decrease levodopa dose and add dopamine agonist

281. A patient had a cold sore which had pus in it, what not to use? Capsaicin

282. Which topical medication is not used for zoster pain


a) Capsaicin

283. Patient on steroids with SLE and taking calcium and alendronate, what is DTP?
a) Patient is not getting medication for conditions required i.e. PPI prophylaxis

284. A patient getting right sided HF due to?


a) Naproxen
b) Milk of magnesia
c) ACEI

285. A patient with acute exacerbation of COPD & taking fluticasone + laba, what to give?
a) Give antibiotic
b) Increase dosing

286. Patient with DM, bilateral renal stenosis, Reynaud’s, his CrCL is 52, hypertensive
What to give for hypertension
a) Amlodipine
b) Beta Blocker
c) ACEI
Another version:
A patient has bilateral kidney stenosis he is taking BB and amitriptyline, what medication is he
not using
ACEI

287. Patient has pneumonia and is taking amoxicillin for 3 days and no improvement in
symptoms and worsening of shortness of breath and fever, what do you recommend?
a) Check the parameters and reassess the medication
b) The patient has not been taking the medication for a long enough time
c) The patient is taking the wrong medication
d) Check blood in stool question
e) Consult your dr to change medication
f) Continue your amoxi

288. Patient has pneumonia and using few antibiotics including gentamycine 420 mg daily for
10 days ago, what is DTP:
a) High dose of gentamicin
289. His dr. Calls you to ask about the medication, what do you recommend:
a) Macrolide

290. Patient is taking iron supplement what not to do


a) Take it with milk to increase absorption
b) Tell her to see md because of dark blood
c) Ask if seeing fresh blood
d) Suggest ferrous sulphate instead of gluconate as it has more iron

291. Diabetic patient with cardiac heart failure, which medication not to give
a) Pioglitazone
Thiazolidindiones cause fluid retention, edema worsening HF.
Antihyperglycemic agent with demonstrated cardiovascular outcome benefit (empagliflozin, canagliflozin,
liraglutide) should be added to reduce the risk of major cardiovascular events. The use of semaglutide has
also resulted in a reduction of major adverse CV events.

292. Diabetes patient sick day management, what not to do


a) Stop insulin
b) Avoid caloric intake

293. A patient with pain and cannot taking take oral opioids due to constipation. His md. Has
prescribed naproxen and fentanyl all are true except.
a) At this point in time the fentanyl patch will not treat his acute pain (note it will take
around 24 hours from time of administration to work check CPS)

294. Patient has experienced second seizure in 2 years what to do


a) Start seizure medication
b) Give prophylaxis seizure medication
c) Do not start medication

295. A black patient with hypertension and diabetes his diabetes is well controlled without
medication, which is not a risk factor for this patient?
a) Obesity

296. You are a pharmacist and believe against plan b a patient comes in to your pharmacy
looking for plan b there is no other pharmacist working today, what will you do?
a) Send her to another pharmacy
b) Forget your beliefs and values and give her plan b

297. A patient is given levothyroxine, what would you recommend?


a) Takes 1 week to start working
b) Space 8 hours from ibuprofen
c) Take empty stomach

298. Clincial guidelines


a) Contain evidence only
b) Are updated annually/yearly
c) Are a compliation of information reviewed from experts
299. Which of the following acyclovir, famciclovir, valcyclovir is most potent?
Most potent famciclovir. Least potent valcycovir
Acyclovir and valacyclovir 200 mg bid for 4-month cold sore suppression therapy. Reccurence 1 more month
Genital herpes 400 mg bid 3-6 months giving all 3 acyclovir, famcyclovir, valacyclovir
Acyclovir dose 200-400 mg 5 times a day with or without food.

300. Which of the following acyclovir, famciclovir, valacyclovir is most well tolerated?
Famciclivir and valacyclovir has better bioavailability with oral dosing.
Note for recurrent cold sore you use these meds. If it’s the first time you apply. Ice. Benzocaine or abreva
topical acyclovir ointment not recommended for cold sore

301. We are going to make a 0.8% hydrocortisone oint with a 2% bulk and petrolatum. The
total should be 50 g. so, we should how much of bulk? 20 g

302. An old woman with chronic pain uses oxycodone ER 10 mg bid + morphine 10 mg prn
and is controled. After years in follow up pharmasist finds out that she increases her dose to 20
mg bid, and now she feels much better, what should pharmacist do?
a) Tell her that she is doing something wrong
b) Tell her that she can't get any pain killer anymore from the pharmacy.
c) Call the dr and describe the situation to increase the dose
d) Everything is ok and he shouldn't do anything
e) Explain to patient about side effects of extra dose of narcotics

303. For above case dr changes the medication to fentanyl patch, what is the best option to
begin the therapy
a) 24 mcg/day
b) 24mcg/2days
c) 24 mcg/3days

304. The above case gets insomnia what is the medication which doesn't make hangover and
helps to sleep more rapidly?
a) Oxazepam
b) Diazepam
c) Flurazepam
d) Zopiclone

305. Which One Gives Bitter Taste?


a) Oxazepam
b) Diazepam
c) Zopiclone

306. A probation officer calls you to get the profile of a prisoner, when you work in that prison,
to rule out the use of drugs after a urine test, what you do?
a) Ask for a consent from the prisoner
b) Give him the information

307. Which of these medications are most probably being abused?


a) Primerose oil
b) Dimenhydrinate
c) Mineral oil

308. Which one is right about evra patch?


a) It is not different from oral COC in terms of deep vein thrombosis
b) A woman who is over 35 y and smokes can use it
c) It can be used in a p with migrain with aura

309. Which one is not a contraindication for HRT?


a) Breast cancer related to hormones
b) Liver problem
c) Migraine with aura
d) Bleeding with unknown cause from uti
e) Deep vein thrombosis
310. Someone is getting 1000 ml infusion for TPN, dextrose 20% with 20 ml /h and dextrose
5% + saline 0.9% with 10 ml/h, what is the total energy he gets? (1 g dextrose gives 3.2 kcal)
Two q about protein and lipid in that case but i can’t remember the exact numbers, but the
information was in ml/kg/h and you needed to convert the information to another units many
times, so tricky

311. What is losartan not used in


a) Peripheral neuropathy
b) Heart failure
c) Left ventricular dysfunction
d) Hypertension
Losartan is used in Essential hypertension and left ventricular hypertrophy. Type II diabetic patient with
proteinuria and hypertension. Cozaar is also indicated to delay the progression of renal disease as measured
by the occurrence of doubling of serum creatinine and end stage renal disease and to reduce proteinuria.

312. A question about the concentration of phenytoin and its relation with blood albumin
concentration or in conditions like
a) Alcoholic liver disease
b) Decreased blood albumin
c) Phenobarbital co-administration

313. A very busy customer who says her dr told her that she has high cholesterol now whats the
first question you tell her?
a) What are her other risk factors (age, lifestyle, diet, family history)?
b) Try to educate her about cholesterol treatment
c) Go to emergency

314. She says (above case) she drinks one glass of wine every afternoon and many other things
about her lifestyle, she uses bicycle, eats in restaurants usually, etc... Whats not your
recommendation for her?
a) Reduce alcohol

315. When a drug with first order pharmacokinetic is given 1 gram gives 15 miligram trough
after 72 h, if we continue dosing when it gets to the steady state what will be the trough
concentration?
a) 30mg

316. A low risk case with diabetes type 2 and high triglyceride whats the doc
a) Lovastatin
b) Niacin
c) Gemfibrozil
d) Cholestyramin
e) Ezetimibe

317. Which one is not the symptom of zoster?


a) Oozing
b) Burning sensation
c) Vesicular
d) Symmetrical

318. Cancer patient on MTX dr. adds vincristine, what SE to experience? Neurotoxicity

319. Case of a56 yrs old male teacher who had complainig about poor hand writing and due to
this he changed his job from full time to part tim, now he came to the pharmacy and asked for
councelling about parkinson treatment:
a) Good for use sooner to prevent progress
b) Until there is not any inteference with his daily activity he can postpone to take the
medication.

320. Which medication is used for above patiet?


a) Amantadine
b) Anticholinergic
c) Selegiline

321. Patient is on levodopa we want to add bromocriptine


a) Titrate bromocriptine gradually
b) Stop levodopa and add bromocriptine
c) Decrease levodopa gradually and increase bromocriptine gradually
When patients already receiving levodopa are switched to SINEMET®, levodopa must be discontinued for at
least 12 hours or more before SINEMET® is started. SINEMET® should be substituted at a dosage that will
provide approximately 20% of the previous levodopa dosage

322. Patient is a computer programmer and has partial seizure and it is his second seizure. He
wants to start medication. Which one is the most suitable option?
a) Lamotrigine
b) Valproic acid
c) Gabapentin
d) Topiramate

323. Patient who has open angel glaucoma. And he is on dorzolamide. Which one should be
monitored?
a) IOP
b) Vision loss
324. A patient had OP and GERD. Which was is the most suitable option to take at night?
a) Etidronate
b) Alendronate

325. Patient has rosasea who apply gel metronidazole for her skin. She experienced extreme dry
skin. What should we do?
a) Gel metronidazole is continued and add emolient
b) Change to cream with erythromycin base

326. Which one induce more impotence:


a) Beta Blocker
b) ACEI
c) Furosemide

327. Patient is using imiquimod for anal wart, what you not recommend:
a) Apply at night and wash in the morning
b) Keep it in refrigerator
c) Wash your hand before and after applying
d) Apply it every other night

328. What do you give for prophylaxis before appendix surgery?


a) Ciprofloxacin
b) Ceftriaxone
c) Cefexime
d) Gentamycin
e) Cefazolin

329. Pateint is using loratadine 10 mg once a day for runy nose and congestion, but not
controlled:
a) Continue
b) Change antihistamine
c) Add decongestant

330. Which one is not correct about resperidone:


a) Has effect on posive symp.
b) Has effect on negative symp
c) First generation has lesser extrapyramidal symp
d) It is serotonin agonist
331. Tobromycin monitoring axcept:
a) Creatinine
b) Liver
c) Ototoxicity
d) Post dose level
e) Prior dose level

332. dose dependant side effect of tubromycine;


a) Ototoxicity
b) Bradycardia
c) Dizziness

333. CBZ sign of toxicity:


a) Gingivitis
b) Bruising and petechia
c) Drowsiness

334. Differentiate between atomoxetine and amphetamine:


a) Atomoxetine is not a stimulant and cause sedation

335. An obese patient (BMI?), who her mother has history of breast cancer and her father has
history of MI. What is your main concern to not give her contraceptive patch?
a) Father history of mi
b) Her mother breast cancer
c) Because of her wt.

336. Patient is travelling to domenican. He asked you about recommendation to prevent


malaria. Where you have to check for this patient?
a) Call and ask dr.
b) Check a book (don’t remember)
c) Intoduce him website
d) Send him to a travel medicine clinic

337. A patient brings back the medication to your pharmacy, what is correct:
a) Take back the medication and give him credit
b) Cannot take the medication back and give him money back

338. What does it mean with "hospital accreditation"


a) Assess hospital quality of care against standard of excellence
Hospital accreditation has been defined as “A self‐assessment and external peer assessment process used by
health care organizations to accurately assess their level of performance in relation to established standards
and to implement ways to continuously improve”
MAY 2011
1. What are the risk factors for diabetic foot ulcer?
a) Peripheral vascular disease, neuropathy, smoking
b) PAD, age
c) Neuropathy, smoking
d) PAD, neuropathy
Risk factors for development of foot ulcers in diabetics include: Peripheral vascular disease, Neuropathy,
Poor glycemic control, Cigarette smoking, Diabetic nephropathy & Previous foot ulcerations / amputations

2. Diabetic foot, infection caused by?


a) Pseudomonas aeruginosa
b) Mycoplasma
c) Anaerobic
d) Gram negative

3. What medication should be given to the infection?


a) Amoxicillin/Clavulanate
b) Cefotaxime + cloxacillin
c) Ceftazidime + clindamycin
d) Piperacillin/tazobactam

4. The community pharmacist decided to give a presentation on diabetic foot, which health care
worker will he invite for the presentation?
a) Podiatrist
b) Chiropodist
c) PT

5. Which anti-arrythmic drug will increase heart rate?


a) Sotalol
b) Digoxin (positive inotropic, negative chronotropic)
c) Amiodarone
d) Lisinopril
Cardiovascular Abnormalities: Exacerbation of arrhythmia has had a reported incidence of about
2% to 5% in most series (new ventricular fibrillation, incessant ventricular tachycardia, increased
resistance to cardioversion, and paroxysmal polymorphic ventricular tachycardia (torsades de pointes).
In addition, symptomatic bradycardia or sinus arrest with suppression of escape foci has occurred in 2% to
4% of patients. Congestive heart failure has occurred in approximately 3% of patients. Second degree AV
block and left bundle branch block (LBBB) have occurred in less than 1% of patients, vasculitis and
angioedema have also been reported. Hypotension independent of – as well as associated with ‐
discontinuation of cardiopulmonary bypass following open heart surgery has also been reported (see
“WARNINGS AND PRECAUTIONS, Cardiovascular”).

6. What kind of monitoring is required before initiating isotretinoin therapy?


a) Liver tests
b) Heart
c) Kidney
d) Psychiatric
Check for depression. Check triglyceride, pregnancy, liver function…need to check

7. Which agent causes iris pigmentation?


a) Latanoprost
b) Dorzolamide
c) Brimonidine

8. A patient having glaucoma and is allergic to Sulfa drugs, so which drug do you recommend:
(k-type)
a) Latanoprost (first line)
b) Betoxalol
c) Dorzolamide

9. Dr insists on trial of the dorzolamide, regardless of the sulfonamide allergy. What do you do?
a) Refuse to dispense as it is allergic
b) Refuse to dispense because of patient safety
c) Dispense but document your conversation with the doctor and the patient
d) Dispense and document counselling of the patient only
Not absolutely contraindicated, instead comes under "WARNING"

10. Patient with Rx of (Andriol) testosterone 80 mg bid Mitte: 30 refills :3. Pharmacist decided
not to dispense medication, what is the cause?
a) Dose is high
b) Interval not given for refills
c) Can be written only not verbal
d) No repeats for verbal
e) Has to be written as part fills
11. What should the pharmacist counsel the patient on andriol?
a) Take on empty stomach
b) Keep it in the fridge
c) Effect will appear after 6 months
d) Testosterone is not a good choice for adenopause
e) Worsen benign prostatic hyperplasia
Usually, a daily dosage of 120‐160 mg divided in two doses, taken once in the morning and once in the
evening for 2‐3 weeks is adequate. Subsequent dosage (40‐120mg daily) should be based on the subsequent
testosterone levels and/or clinical effect obtained during therapy.
Missed Dose: Should you forget a dose, take your dose at the next scheduled time. Do not take a double
dose of this medicine. To ensure adequate absorption, Andriol® (testosterone undecanoate capsules) must
be taken with a meal and swallowed without chewing.
Store between 15‐30°C. Protect from light and moisture. Do not refrigerate. Keep blister in the outer carton.

12. Patient profile of an obese woman who needs OC but prefers something that is not taken
daily and wishes to be pregnant in 1 year. What is the best choice for such patient?
a) Transdermal patch
b) Intrauterine ring (Nuva ring)
c) Medoprogesterone inj
d) Continuous oral contraceptives

13. Patient case on asthma. Moderate asthma patient taking salbutamol prn, flovent bid,
experienced 4 exacerbations in 1 year, what could be the DTP?
a) Med adherence (salbutamol not prn)
b) Needs more cortisone
c) Needs prophylaxis

14. Asthmatic patient now placed on Montelukast. The benefits include:


a) Improvement in lung function
b) Exercise tolerance
c) Reduction in Salbutamol use
d) Discontinuation of Fluticasone

15. CHF Patient on irbesartan/HCTZ for hypertension. He finds it hard to climb stairs and he
has edema. Now experiencing shortness of breath while walking & subside once got home
when rest, what is the DTP?
a) Needs more drug
b) Receiving medication not indicated
c) Not receiving medication for indication (edema)
d) Experiencing adverse effect of medication
16. Above patient, What Non pharms. Option can help him?
a) Decaffeinated coffee
b) Salt substitute
c) Compress stocking
d) CoQ10
e) Aerobic exercise
Compression stocking (given in non edematous individual as it prevents edema PSC 923)
COQ10 given in NYHA Class III and IV and chronic heart failure who are receiveing digoxin, ACEI, ARB, CCB
Nonpharmacologic Choices
- Sodium: decrease sodium intake to <88 mmol/day (2 g Na+ or 5 g NaCl).
- Fluid: ↓ fluid intake based on individual patient assessment of volume and disease state.
- Posture: supine position improves cardiac output; elevate legs.
- Stockings: consider supportive compression stockings.
- Paracentesis for ascites: performed under the direction of a specialist.
- Ultrafiltration therapy: if renal insufficiency is significant, refer to a specialist for this therapy.
17. You are a community pharmacist who is doing a presentation for the community in diabetes
type II, what information is not useful?
a) Type of monitors & how to use them
b) How to mix insulins (mainly type I diabetes)
c) Complications of DM

18. Patient had acute renal failure, which drug can be used safely?
a) ACEI
b) B Blocker
c) Gliclazide (used in stages 1,2,3)
Gliclazide Monograph: CONTRAINDICATIONS
 Known hypersensitivity or allergy to gliclazide, other sulfonylureas, sulfonamides, or to any of the
 excipients of this product (for a complete listing, see HOW SUPPLIED, Composition section).
 Unstable and/or insulin dependent diabetes mellitus, particularly juvenile diabetes, diabetic
ketoacidosis, diabetic pre‐coma and coma.
 During stress conditions such as serious infection, trauma or surgery.
 In the presence of severe hepatic impairment.
 In the presence of severe renal impairment.
 Treatment with miconazole via systemic route or oromucosal gel.
 Pregnancy and lactation

19. Which of these factors can lead to hyperglycemia?


a) Excess doses of insulin
b) Missing meals
c) Basal insulin with no prandial insulins/ or taking 0.1 ml insulin instead of 1 ml

20. Medical error. A nurse gave heparin to a patient. She gave him 10 times more concentration
than the prescribed concentration. A pharmacist found the error but the patient already got the
heparin for several days. What is NOT the appropriate action of the pharmacist?
a) Observe patient & decide if he needs vitamin K. (Antidote Protamine sulphate)
b) Observe bleeding.
c) Let the doctor know it
d) Change it to the correct concentration
e) Stop heparin until the aPTT level is decrased.

21. You will do all of the following to prevent this error from happening again except?
a) Purchase prefilled vial
b) Put different strength (standardize concentrations)
22. Rivaroxaban- monitoring includes (No monitoring)
a) INR
b) Bleeding
c) Liver

23. Patient using insulin pump – insulin lispro used in pump, patient complaining of morning
hyperglycemia- contributing factor is
a) Dawn effect
b) Too much food at dinner
c) Insulin lispro is short acting
Insulin pumps are small, computerized devices that mimic the way the human pancreas works by delivering
small doses of short acting insulin continuously (basal rate). The device also is used to deliver variable
amounts of insulin when a meal is eaten (bolus)
Insulin pump high blood sugar
When pumping, high blood glucose (BG) must be taken seriously as insulin pumps deliver only rapid‐
acting insulin. Without any long‐acting insulin in your body, BG can rise quickly if the insulin flow is
accidentally interrupted and a condition called diabetic ketoacidosis (DKA) can develop

24. Patient on phenytoin 300mg HS. Counsel on all except;


a) Take With food
b) Take 1 capsule 100 tid (300 mg at once at bedtime as per Rx)
c) Will tell her the interaction between Phenytoin and alcohol
d) Will talk to her about the SE
e) Will talk to her that she should make monitoring
Phenytoin acid (tablet, suspension) and phenytoin sodium (capsules) may be administered orally.
Phenytoin sodium parenteral solution may also be administered IV. There is approximately an 8%
increase in drug content with the free acid form over that of the sodium salt; phenytoin sodium 100
mg is approximately equivalent to phenytoin acid 92 mg. This should be taken into consideration
when a patient is switched from one form to the other; serum level monitoring may be necessary.
If dosage adjustment is required, phenytoin’s nonlinear pharmacokinetics must be taken into
consideration. Maintenance doses should generally only be increased in small increments, often by
no more than 10% of the total daily dose. Some experts suggest that, in adults, if the steady‐state
serum level is less than 28 μmol/L, the dose should be increased by 100 mg/day. If the concentration
is between 28 and 48 μmol/L, the dose can be increased by 50 mg/day. If the concentration is greater
than 48 μmol/L, the dose should be increased by 30 mg/day or less. The time required to reach
steady‐state after each dosage adjustment (usual range 7–10 days) may be prolonged (up to several
weeks), and must be taken into account in order to avoid premature and unnecessary modifications
which might result in toxicity or loss of seizure control.
Drug‐Food Interactions
The presence of food may affect phenytoin absorption, but may also reduce its gastrointestinal side
effects. Taking phenytoin with food on a consistent basis may minimize any food‐associated
fluctuations in bioavailability. Phenytoin bioavailability can be significantly reduced in the presence of
enteral nutrition products. Tube feeds should be held for 2 hours before and after phenytoin doses

25. A pregnant woman. Three months of pregnant. She was diagnosed with diabetes recently.
She has no history of diabetes. What is the target of fasting glucose level for her?
a) < 6
b) < 5.3
c) < 7.8
Aim to achieve an HbA1c level of ≤6.5% (≤6.1% if safely achievable).
FPG/Preprandial Glucose (mmol/L)  < 5.3
1‐hour Postprandial Glucose (mmol/L)  < 7.8
2‐hour Postprandial Glucose (mmol/L)  < 6.7
If insulin cannot be given give GLYBURIDE PREFFERED, metformin can be given as well

26. While counseling this woman, the pharmacist should tell her the following hypoglycemic
symptoms EXCEPT?
a) Tremor
b) Excessive hunger
c) Sweaty skin
d) Anxiety
e) Fruity smell in breathing
Hypoglycemia is most commonly the result of either a missed meal or an unusual amount of exercise.
Frequent hypoglycemic events may lead to reduced autonomic symptoms (hypoglycemia unawareness).
Teach patients to account for diet and physical activity when planning insulin treatment regimens.
Mild to moderate hypoglycemia is manifested by autonomic symptoms: sweating, tremors, tachycardia,
hunger, nausea and a general sensation of weakness. It can easily be treated with an oral source of sugar.
Fifteen grams of glucose (e.g., ¾ cup of juice or regular soft drink, 6 Life Savers) will usually raise the blood
glucose approximately 2 mmol/L within 20 minutes.
Severe hypoglycemia requires assistance in its recognition and/or treatment.
Neuroglycopenic symptoms such as confusion, altered behaviour, difficulty speaking and disorientation can
progress to seizures and coma, and prevent the patient from appropriately treating the hypoglycemic
episode. If the patient is conscious, an oral glucose preparation consisting of 20 grams of carbohydrate
should be used, preferably as glucose tablets or equivalent. Glucose gel is not a preferred treatment
option. It must be swallowed for significant effect, and the blood glucose response is very slow.
In unconscious patients with no IV access, 1 mg of glucagon IM or SC temporarily increases blood glucose,
allowing for the intake of oral carbohydrate. Glucagon is not effective in malnourished patients or in
alcohol‐induced hypoglycemia. If IV access is available, the treatment of choice is 20–50 mL of 50% dextrose
IV over 1– 3 minutes.
Assess specifically for hypoglycemia unawareness and use strategies to recover awareness, such as relaxing
glycemic targets for up to 3 months and increasing patient self‐monitoring of BG.
27. What is the goal for monitoring Fasting Blood glucose level in a pregnant patient?
a) To prevent complication such as Ectopic pregnancy and neonatal complication
Risks of diabetes during pregnancy
Diabetes during pregnancy increases fetal and maternal morbidity and mortality. Neonates are at risk of
respiratory distress, hypoglycemia, hypocalcemia, hyperbilirubinemia, polycythemia, and hyperviscosity.
Poor control of preexisting (pregestational) or gestational diabetes during organogenesis (up to about 10
weeks gestation) increases risk of the following:
 Major congenital malformations
 Spontaneous abortion
Poor control of diabetes later in pregnancy increases risk of the following:
 Fetal macrosomia (usually defined as fetal weight > 4000 grams or > 4500 grams at birth)
 Preeclampsia
 Shoulder dystocia
 Cesarean delivery
 Stillbirth
However, gestational diabetes can result in fetal macrosomia even if blood glucose is kept nearly normal.

28. Patient 5 foot and 6 inch and 75 kg she likes reading and she doesn’t like the clubs. She
going on vacation and wants to lose weight, she said she wish to try orlistat. What is your least
concern?
a) Money
b) Lifestyle as She needs to increase her activity
c) She is a good candidate for orlistat
d) Decrease diet
Indications for the use of apply to obese patients with a BMI* ≥ 30 kg/m2 or a BMI ≥ 27 kg/m2 in the
presence of other risk factors (e.g., hypertension, type 2 diabetes, dyslipidemia, excess visceral fat).
BMI = wt/height m2 = 75/ 1,682= 22.32

29. What you will recommend for her?


I. Decrease fat content in her diet
II. Do some briskly walking and activities in her break time
III. Tell her to go to the health clubs

30. Least weight gain medication is?


a) Lamotrigine
b) Valproic acid
c) Topiramate
31. Alzheimer patient 65 years old, Hypertension pt,
agitated & has sleep problems & taking dimenhydrinate.
What may aggravate his Alzheimer?
a) Age
b) Smoking
c) Diphenhydramine

32. Patient taking Levodopa and Carbidopa 200/25 for 5


years, in which of the following cases would you consult
doctor except:
a) On off effect
b) End of dose
c) No movements (=no tremors=control)

33. Increased the dosage of Levodopa/carbidopa. All are possible side effects except?
a) Ataxia
b) Orthostatic hypotension
c) Peripheral edema
d) Lip grimacing
e) Hallucinations
Q. Levodopa Preparations
Levodopa Levodopa, a dopamine precursor that is converted to dopamine in the brain by with enzyme
dopa decarboxylase, appears to correct akinesia, rigidity and tremors of Parkinson’s disease by
Levodopa / the formation of dopamine at the nigro striatial dopaminergic site.
carbidopa, Although used alone when first discovered, it is now combined with a DOPA decarboxylase
SINEMET® inhibitor (carbidopa or benserazide) to inhibit peripheral transformation to dopamine, thus
immediate enhancing distribution to the brain, reducing the amount of levodopa required for optimal
release therapeutic benefit and minimizing acute side effects such as nausea and vomiting.
SINEMET® When patients already receiving levodopa are switched to SINEMET®, levodopa must be
CR discontinued for at least 12 hours or more before SINEMET® is started. SINEMET® should
controlled be substituted at a dosage that will provide approximately 20% of previous levodopa dosage.
release. When patients receiving levodopa monotherapy or SINEMET® switched to SINEMET® CR,
this medication must be discontinued at least 8 hours before therapy with SINEMET® CR is
started. Dosage with SINEMET® CR 200/50 should be substituted at an amount that
Levodopa / eventually provides approximately 10 to 30 % more levodopa per day. The interval between
carbidopa, doses should be prolonged by 30 to 50 percent. This is because when Compared to Sinemet
intestinal gel immediate-release formulation, bioavailability of Sinemet CR is 25–30% lower and duration
of action 25–30% longer.
Because entacapone enhances the bioavailability and therefore central effects of levodopa, it
may be necessary to adjust dosage of levodopa during the initial days to weeks of entacapone
therapy in order to reduce levodopa-related dopaminergic adverse reactions, e.g., dyskinesias,
nausea, vomiting and hallucinations. In some cases, it may be necessary to reduce the daily
dosages of levodopa by about 10-30%. This can be achieved through either reducing the
dose of the levodopa preparation itself, or by extending the interval between doses, according
to the clinical condition of the patient.
Levodopa may be taken with food early in therapy to ease nausea; it may be taken on an
empty stomach in more advanced disease to help manage motor fluctuations
A change in diet to foods that are high in protein (such as meat, fish, dairy products, seeds and
nuts) may delay the absorption of levodopa and may not work as well as it should.
S.E: Nausea, vomiting, orthostatic hypotension, dyskinesias, hallucinations, confusion, mental
changes including paranoid ideation and psychotic episodes, depression with or without
development of suicidal tendencies, and dementia. Convulsions also have occurred.
Rare: Weight gain or loss, edema, Leukopenia, hemolytic and non-hemolytic anemia,
thrombocytopenia, agranulocytosis. Urinary retention, hematuria, and priapism.
Arrhythmias, non-specific ECG changes, phlebitis. Sialorrhea, bruxism, hiccups, GI bleeding,
flatulence, burning sensation of tongue, development of duodenal ulcer.
Long term use of sinemet can produce mydriasis & precipitation of glaucoma, melanoma.
Risk of parkinsonism hyperpyrexia syndrome with abrupt discontinuation; taper gradually.
Drug holidays not recommended.
Levodopa / carbidopa, intestinal gel: Complications related to percutaneous intrajejunal tube
placement e.g., infection, intestinal obstruction
When levodopa or a dopamine agonist are initiated, domperidone (10–20 mg 30 minutes prior
to each dose) can be helpful in minimizing gastric upset or orthostatic hypotension.
Antihypertensives, diuretics, tricyclic antidepressants may increase hypotensive action
Wearing off: Short duration response or "end dose" effect. The motor complications include
"off periods" of immobility or greater severity of the other parkinsonism symptoms & various
abnormal movements. This is due to decrease synthesis and storage of dopamine generated
from endogenous or exogenous levodopa.
On-off fluctuations: Most severe form of wearing off effect (abruptly freezes).
Sudden changes in movement control, which can last for a variable period of time.
‘On’ time is when levodopa is working well and your symptoms are controlled.
‘Off’ time is when levodopa is no longer working well and symptoms such as tremor, rigidity
and slow movement re-emerge.
Dyskinesia generally occurs when levodopa and other dopaminergic medications are at their
most effective. It is thought that an increased sensitivity to dopamine in the brain as a result of
treatment, combined with the natural progression of Parkinson’s, gives rise to dyskinesia.
Less commonly, dyskinesia can also occur when levodopa is just starting to take effect or
when it is wearing off – this is known as ‘diphasic dyskinesia’.
Avoid VB6 supplements as it reduces the effectiveness
Monitoring:
 Frequent monitoring of BUN, creatinine levels, and hepatic function because liver is
where the drug is decarboxylated.
 Also, it is important to test for intraocular pressure in patients with glaucoma.
 Testing for peripheral neuropathy before and while on levodopa is also very important.
Patients should also be regularly monitored for dyskinesia.
 Patients need to be observed for psychotic behavior and hallucinations when on
dopaminergic medications. Confusion and excessive dreaming can be accompanied
with hallucinations. Thus, patients with a history of past psychiatric disorders should
not be treated with levodopa.
 As an extra precaution, patients should be monitored for melanoma. It is not known yet
whether the risk of melanoma is increased due to levodopa use or Parkinson disease.

34. Parkinson’s patient now complaining of agitation and insomnia –what could be a good
choice
a) Haloperidol
b) Mirtazepine,
c) Bupropion
d) Olanzapine
The aim of antiparkinson medication dose reduction is to achieve a balance between improving drug‐related
psychotic symptoms and not significantly worsening the motor symptoms of PD. If additional measures are
needed for chronic PDP treatment, the use of second‐generation antipsychotics, such as clozapine,
pimavanserin, or quetiapine, must be considered. The first‐generation antipsychotics (eg, fluphenazine,
haloperidol) are not recommended.

35. Patient wants to try a natural herb; you agree to their choice. What ethical principal are you
following?
a) Autonomy
b) Beneficence
c) Paternalism

36. Patient on levothyroxine 0.1mg for 30 yrs. Also taking zopiclone, atorvastatin, now
complaining of fatigue and feeling slow. What could be the DTP?
a) DI between atorvastatin and zopiclone
b) Less levothyroxine
c) Compliance

37. Case on RA. Patient was on hydroxychloroquine, now starting methotrexate. Counselling
should include
a) Empty stomach
b) Takes 2 weeks to show benefit
c) Check for toxicity in 1 month
d) Causes hairloss so buy a wig
38. Patient is female came to pharmacy has vaginal symptoms burning, pruritis, cheesy curd
discharge. Patient has this first time but patient knows vaginal candidiasis and asks for OTC
medication. What is appropriate response to be given by pharmacist?
a) No medication to be given
b) Advise patient to go to Dr for advice (first time)
c) Eat Yogurt.
Most fungal vaginitis is caused by C. albicans (candidiasis), which colonizes 15 to 20% of nonpregnant and
20 to 40% of pregnant women.
Risk factors for candidal vaginitis include the following:
 Diabetes
 Use of a broad‐spectrum antibiotic or corticosteroids
 Pregnancy
 Constrictive nonporous undergarments
 Immunocompromise
 Use of an intrauterine device
Candidal vaginitis is uncommon among postmenopausal women except among those taking
systemic hormone therapy.
39. Patient female came to pharmacy complaining of vaginal symptoms. Which of the
following to be considered for Dr referral?
a) Itch
b) Fishy smell
c) Curd like discharge
Note: right ans, bec fishy smell is sym of bacterial infection….so it need referral.

40. She brought a prescription for metronidazole, what to counsel?


a) No sun as it causes phototoxicity
b) Use Barrier method in intercourse
c) Should be taken in empty stomach
it can cause headache, insomnia, discoloration urine, avoid
alcohol (disulfiram like reaction), reversible reduction of serum
lipid, gynecomastia, urticara

41. What is true about SSRIs? Better than TCAs

42. Patient has cellulitis- the wound is wheeping, Oozing


and honey colored, what is the causative organism?
a) S. Aureus

43. What is the DOC? Cloxacillin PO X 5 days

44. Benzydamine mouthwash counseling - all expect?


a) Dilute with warm water into a 1:1 solution
b) Don't eat hot foods directly after it
c) Short acting
d) For pharyngitis & mucositis caused by radiation therapy
e) Swish in the mouth for as long as possible then swallow

45. Which reference does not support its recommendations or DI info is available in all except?
a) TC
b) Merck
c) Remington
d) CPS
46. to look for S.E. which reference all xpt?
a) Remington
b) Micromidix
c) CPS
d) TC

47. Which is the highest level of reference?


a) RCT
b) Meta analysis
c) Case control

48. Which drug may cause urinary retention?


a) Oxybutinin (DOC: Used in Urinary Incontinence)

49. Patient taking terbinafine oral – counsel


a) Needs 2 weeks to work
b) Causes GI upset,
Baseline Hepatic
function tests.
c) Use tolfonate powder topically
Before prescribing TERBINAFINE
Tablets, a baseline liver function
test should be performed to assess
any pre‐existing liver disease since
hepatotoxicity may occur in
patients with and without pre‐
existing liver disease. Periodic
monitoring (after 4‐6 weeks of
treatment) of liver function tests is
recommended.
TERBINAFINE should be
immediately discontinued in case of
elevation of liver function tests.

50. Soy milk given in?


a) Cows milk lactose intolerance
b) Cows milk protein allergy
c) Sucrose intolerance
As it comes from plants, soy milk is naturally free of cholesterol and low in saturated fat. It also contains
no lactose. Soybeans and soy milk are a good source of protein, calcium (when fortified), and potassium.
Soy Protein Free of cow's milk protein and lactose. They are iron-fortified and are designed to meet the
Isolate– nutritional needs of term-born infants.
Based They are recommended for term infants with galactosemia or congenital lactase deficiency.
Formulas They can be used as a supplement to breastfeeding for infants of mothers who follow vegetarian
diet or for infants whose mothers wish to feed them nonanimal protein-based formula.
They are not recommended for infants with cow's milk protein–induced enteropathy or
enterocolitis, as 30–60% will also be sensitive to soy. However, infants who have an
immunoglobulin E–associated reaction to cow's milk protein may tolerate soy formulas.

51. Patient with probable shingles- what is the red flag to refer to doc except?
a) Severe pain
b) Rash for 7 days
c) Eye involvement
d) Age older 70 yrs
e) HIV patient
Shingles presents as a painful
rash, usually in 1 or 2 adjacent
dermatomes, the rash
develops into vesicles that
rupture and crust over and
usually heal in 2– 4 weeks.

52. Which is to be given with food


a) Amoxicillin (Empty stomach)
b) Cefuroxime
c) Metronidazole (1 Hr Before or 2 Hr after food)
d) Cephalexin

53. Which of the following does not need to be adjusted in renal impairement?
a) Norfloxacin
b) Ciprofloxacin
c) Ofloxacin
d) Moxifloxacin
e) Levofloxacin
Moxi also effective against Anaerobes, and it is not effective in UTI so don’t recommend.
54. Patient has cough, shortness of breath, pleuritic chest pain, hemoptysis, sputum production.
Doctor diagnosed him with CAP. The major Causative agent is?
a) S. pneumonia
b) Chlamydophila pneumoniae
c) E. coli
d) H. influenza

55. In the above patient, all need to be monitored in a few days except?
a) Fever
b) Respiration rate
c) Blood pressure
d) Chest x-ray

56. Patient with fibromyalgia – has already tried amitryptiline, gabapentin – still no relief what
could be the next choice of therapy?
a) Imipramine
b) Doluxetine
c) Nortryptiline
Duluxetine  FANDU – Fibromyalgia, Anxiety, Neupathic pain, Depression, Urinary Incontinence}

57. Which of the following medications is most likely to cause loose stool or diarrhea?
a) Misoprostol
b) ARBs
c) Glyburide
d) Ramipril

58. Patient is female and has depression is taking some other medications. Dr prescribed
Sertraline. She took it for 4 weeks, but still complaining and there is no improvement with
depression, she is experiencing crying spells, mood changes, what is the appropriate action?
a) Sleep and appetite will improve earlier in depression
b) Change within same group.
c) Wait to 6 months

59. Which of the following is a SE of sertraline?


a) Crying spells
b) Dry mouth
c) Mood changes
d) Fatigue
e) Less concentration
60. Which of the following medications has drug interaction with sertraline except?
a) Meperidine
b) Codeine (Serotonin syndrome with opioids)
c) Naproxen (increase GI bleeding with NSAIDs)
d) Ibuprofen
e) Celecoxib
With meperidine: Can cause weakness, hyperreflexia, and incoordination; so, appropriate observation of the
patient for acute and longterm adverse events is advised.
Sertraline may reduce the effectiveness of codeine. Codeine is a CYP2D6 substrate while SSRIs, including
sertraline and most tricyclic antidepressants, inhibit the biochemical activity of the drug metabolizing
isozyme, cytochrome P450 2D6

61. Question on risedronate. Osteoporosis patient was given alendronate, risedronate, what is
true about risedronate?
a) Should be used for 18 months only
b) Take on empty stomach
The optimal duration of bisphosphonate treatment for osteoporosis has not been established. The need for
continued treatment should be re‐evaluated periodically based on the benefits and potential risks of APO‐
RISEDRONATE on an individual patient basis.
Missed Dose
 Patients should be instructed that if they miss a 150 mg dose of RISEDRONATE (1 tablet of 150 mg),
and the next month’s scheduled dose is more than 7 days away, they should take the missed tablet
in the morning after the day it is remembered.
 Patients should then return to taking their APO‐RISEDRONATE 150 mg as originally scheduled. If a
dose of RISEDRONATE 150 mg is missed, and the next month’s scheduled dose is within 7 days,
patients should be instructed to wait until their next month’s scheduled dose and then continue
taking RISEDRONATE 150 mg. Patients should not take more than 150 mg of APO‐RISEDRONATE
within 7 days.

62. They are not working for him. What


would you give? Teriparatide

63. What is the side effect of Teriparatide?


(teriparatide=PTH=hypercalcemia)
a) Hypercalcemia
b) Orthostatic hypotension
64. Which of these agents can be used with oral contraceptives?
a) Gabapentin
b) Carbamazepine
c) Phenytoin
d) Topiramate

65. Patient on H2RAs best calcium salt to give to such patient


a) Calcium citrate
b) Acetate
c) Gluconate
d) Carbonate

66. Which can be destroyed without waiting for notice from controlled office?
a) Tylenol 1
b) Benzodiazepines
c) Codeine
d) Clozapine (antipsychotic)

67. Canada health Act is useful for


a) Transferring funds to hospitals
b) Tax exemptions to hospitals

68. Common drug Review is administrated by:


a) CPhA
b) Physician’s association
c) NAPRA
d) Natural drug and therapeutics committee
e) CADTH Canadian agency for drugs and tech in health

69. Question on correct Tall man lettering


a) predniSONE and PrednisoLONE
b) PredniSOLONE – prednisone
c) PREDNISOLONE-PREDNISONE

70. Question on how to reduce risk of medication error by nurses in wards


a) Automation
b) Remove look alike sound alike drugs
71. Reports of ADRs are sent to
a) Provincial committee
b) NAPRA
c) Medeffect/ Canada vigilance
d) CARN= Canadian Adverse Reaction Newsletter

72. A doctor calls to ask you about the latest changes in guidelines that happened yesterday.
Where can you look for this information
a) Cochrane review
b) Website of speciality
c) Merck index

73. Patient reporting ADR – where can information on similar cases in other patients be found?
a) Google
b) CPS
c) Medline

74. Which decreases cyclosporine levels


a) Erythromycin
b) Phenytoin
c) Azole
d) Propafenone
SJW increase the level of cyclosporine and other CYP3A4 inhibitor also increase toxicity of cyclosporine

75. Question on sunburn- all are appropriate except


a) Oral ibuprofen
b) Lidocaine
c) Wet compress
d) Hydrogel
Lidocaine is not useful even worse the situation. Topical antihistamine also not useful and Oral useful one is
first generation antihistamine which helps in sleeping.

76. Question on how to avoid a sunburn. all except


a) Avoid sun at peak time
b) Use umbrellas
c) Wear Long sleeves
d) Use sunscreen with only UVB
77. Question on compounding apparatus in pharmacy:
a) Should be changed yearly
b) Use plastic spatula for coal tar
c) Use glass or disposable paper for ointments
d) Place in sterile area

78. Patient came from Africa and she got infected with tuberculosis and gave 4 combinations.
Rifampin, Isoniazid, Ethambutol, Pyrizinamide. Which one is true regarding medication?
a) Rifampin inhibits metabolism of other drugs (it induces metabolism)
b) Pyrazinamide and ethambutol have liver problems
c) Ethambutol has optic side effects have regular eye checkup
d) Blue green coloration
A is wrong. C is right. For option B, Pyrazinamide has hepatotoxicity, not ethambutol has.)

79. Why is the patient at risk of peripheral neuropathy?


a) Four antibiotic regimens
b) Low weight patient
c) Malnutrition

80. Lady, smoker, has COPD uses salbutamol prn and now has emphysema can’t walk for a
block. Dr prescribed combination of Ipratropium and Salbutamol. What are the goals of
treatment?
 Prevent disease progression & Improve exercise tolerance
 Decrease or eliminate breathlessness and other respiratory symptoms
 Reduce the frequency and severity of exacerbations & health‐related quality of life
 Reduce impairment, disability & mortality

81. What is the DOC? No comorbidity: Amoxicillin/Azithro- Comorbidity:


levofloxacin/Amoxclav
a) Levofloxacin
b) Clindamycin
c) Azithromycin
d) Budesonide

82. Patient doesn’t like to use metered dose inhaler. What is the first concern for pharmacist?
a) Call dr to change it
b) Contact respirologist
c) Ask patient why she doesn’t want to use it
83. Patient with emphysema and AE COPD – is taking more frequent doses of SABA What else
should the therapy include:
a) Increase ipratropium
b) Increase LABA
c) Add steroid
d) Give oral steroid

84. Asthma patient consulting. What can be under the category of “Plan” of SOAP note?
a) Check if you experience nocturnal symptoms, Subjective
b) Check your peak flow meter level (in moderate to severe persistent asthma), objective
c) Check smoking history, Subjective
d) Monitoring follow-up

85. Prednisolone tab 5mg dose 10 mg/day then reduce by ½ a tab per week. How many tabs are
needed to fill this prescription? 14+10.5+7+3.5= 35
a) 75
b) 35
c) 50
d) 125

86. How many mls of 4% solution A is required to fill RX to make 50 mls of final solution so
that the final concentration of the solution A is 0.1%
a) 1.75
b) 1.25
c) 0.55

87. How much elemental ferrous will the patient receive daily from the following rx;
Ferrous gluconate 300 mg TID.
a) 35
b) 105
c) 200
d) 300
Ferrous gluconate 35 mg 300mg
Ferrous Sulphate 60‐65 in 300‐325 mg
Ferrous fumarate 100 mg in 300 mg tab
Ferrous succinate 60mg in 100 mg tab
88. What is the schedule of ferrous gluconate 300mg?
a) Schedule I
b) Schedule II
c) Schedule III
d) Unscheduled
e) Schedule F
Iron supplements are regulated as schedule 2 drugs (behind the counter).
Iron containing <30 mg dose is schedule III (over the counter).

89. A final solution containing 140 mmol is required – you have a solution containing 110
mmol –how much 3% NaCl id needed to make 140 mmol (0.9% has 154 mmol)

90. Vancomycin trough levels measured are 15 mg. if a second dose is given at t1/2 72 hrs what
should be the next trough level?
a) 40 mg/litre
b) 60 mg/litre
c) 15mg/litre
d) 30mg/litre
Imp: Vancomycin C peak = 18‐26 mg./litre and C trough = 5 ‐10 mg/litre

91. Patient taking Phenytoin initial dose 100mg and plasma level was 20mcg/ml. dose increased
to 200mg, then plasma levels measured was 160 mg/L. Patient experienced nausea and ataxia.
What is the expected final concentration after increasing to 200 mg assuming that phenytoin
obeys 1st order?
a) 20mcg/ml
b) 40mcg/ml

92. Phenytoin follows first order kinetics. If the first dose is 100mg what should be the second
dose to maintain CSS
a) 100 mg
b) 200mg
c) 250 mg

93. Drug A 100 death, 1000, Drug B 50, 1000. What is NNH of Drug A? 20

94. Accreditation of hospital means?


a) Looking for excellence of practice and performance in hospital
https://accreditation.ca/faq/?fbclid=IwAR0ITzGlSqpjj82gTc9lvEsDuQmYpeMrNc22UzpnvPHTvP-
ZcEZXZkQilsw
Accreditation is an ongoing process of assessing health and social services organizations against standards
of excellence to identify what is being done well and what needs to be improved. Accreditation is important
because it helps create better health care and social services for you, your family, and your community. It
allows organizations to understand how to make better use of their resources, increase efficiency, enhance
quality and safety, and reduce risk. Accreditation provides health and social services organizations an
independent, third‐party assessment of their organization using standards built upon best practices used
and validated by similar organizations around the world.

95. Most cause of errors in hospital pharmacy


a) Less trained stuff
b) Work overload

96. Hospital medication error where to report:


a) ISMP
b) Med error database
c) Hospital association
For medication incident  report to CIHI National System for Incident Reporting (NSIR).

97. ADR’s in hospital are reported to which organisation?


a) Health Canada MedEffect Canada Vigilence
98. A patient is taking clonazepam. He has refills remaining on his prescription. he is now
going away to the US for a holiday. How may he get his refills?
a) He can be advanced the remaining quantity
b) You can mail the refill quantity at the time of refill
c) Tell him that he cannot get his refills as he will be out of country so the provincial plan
does n’t apply
d) Give 4 mos supply and bill insurance monthly
e) Set-up transfer to Florida

99. A patient with Crohn’s disease. Very severe symptoms of diarrhea and abdominal pain. The
followings are the initial goal of therapy EXCEPT.
a) Cure the disease
b) Prevent surgery (non pharmacological of chron's)
c) Avoid use of steroids
d) Symptoms remission
e) Prevent recurrence
f) Improve abdominal pain and diarrhea
g) Prevent possible drug adverse reactions

100. He admitted to emergency with flare up of CD with more than 10 loose bowel movements,
fistulas inflamed. The symptoms are not controlled by IV corticosteroids. What can be tried?
a) Infliximab
b) Methotrexate
c) Sulfasalazine
d) Prednisolone

101. What can be used to prevent recurrence?


a) Corticosteroids
b) Azathioprine
c) Methotrexate
d) Mesalamine
e) Cyclosporine

102. Pt admitted to emergency department. Patient is in confused state has history of liver
cirrhosis and Dr gave lactulose in hospital. What will be end point to stop lactulose treatment?
a) Until he loses 3 kg of body weight
b) Till attain normal AST values
c) Until patient comes to normal state or normal mentation
d) Until patient is not hallucinating.
103. What is elevated in the above patient?
a) Ammonia
b) AST
c) ALT
d) Alc. dehydrogenase

104. Till when you’ll give lactulose for this patient?


a) Decrease weight 5 kg
b) Until liver enzymes normalized
c) Regain consciousness

105. Patient is released from hospital and patient got Rx of lactulose to continue the treatment.
He is moving to another province. What can the pharmacist advice about his medication?
a) Only written permitted
b) Can take only one time with no refills
c) Rx from different province not allowed
d) He can only refill in the residing province
e) He can get lactulose in another province as it is OTC medicine
f) He needs to get a new prescription in the new province

106. What is the proper procedure to mix insulins?


a) Draw up regular then analogue
b) Draw up analogue then regular
https://my.clevelandclinic.org/health/drugs/11728-insulin-injection-two-bottle-injection-instructions

107. You have a prescription to prepare a solution that must contain drug A 10mg/ml. The
solution you already have contains 5mg/ml. How much of another solution containing drug A
4mg/ml must be used to get the final strength?

108. Doctor is starting patient, who is 60kg, on Isotretinoin. Patient took 0.5mg/kg/day for 4
weeks. After the 4 weeks, doctor wants to increase it to 1mg/kg/day. The total cumulative dose
is 120-150mg/kg. If he starts with 30mg daily for 4 weeks, what is the next step:
Minimum cumulative dose = 120mg * 60Kg =7200mg
mg taken dose = 30mg/daily *28 days = 840 mg
So, the dose should be taken = 7200 - 840 = 6360 mg
If patient takes 30 mg BID = 30mg *2 = 60 mg Daily
No. of days = 6360 / 60 = 106 days = 3.5 months
As this is the minimum cumulative dose. So, the dose should be used for more than 3.5 month
the answer is 30 mg BID for 4 months
109. Which forms of drugs cannot be given through an NG tube?
a) Enteric coated
b) Contents of a capsule
c) Sugar coated
d) Uncoated tablets

110. What type of health care system is implemented in Canada?


a) Mostly federal with some private
b) Mostly private with some federal

111. What needs to occur first during a marketing process


a) Post marketing surveilence
b) NOC
c) PMDRB
d) CDR

112. Cases on hypertension - patient on diamicron 4 tabs daily and diovan 1 tab daily. The
prescription sig was mixed up – what should be monitored for this patient
a) Slow pulse
b) Fast pulse
c) Increased systolic
d) Increased diastolic

113. Side effects of diltiazem


a) Constipation, Headache, Hypotension, Dizziness
114. Patient in sotalol for cardiac arrythmias, what could be the DTP? QT Prolongation

115. Patient has sexual dysfunction what could be the most probably cause? HCTZ

116. Uncomplicated gonorrhea in pregnant patient– treatment choice


a) Erythromycin
b) Cefixime

117. The woman also has Chlamydia. What is the treatment?


a) Erythromycin
b) Cefixime

118. Which medication can cause photosensitivity?


a) Sulfacetamide
b) Isotretinoin
c) Nitrofurantoin
d) Oral contraceptive
Common side effects of Bleph‐10 sulfacetamide eye drops include: eye stinging/burning/redness/itching
/irritation, temporary blurred vision, eyelid itching, eyelid swelling, or sensitivity to light.

119. 2 questions on timolol  Glaucoma Therapy


Drugs Mechanism Side effects Comments
Beta-Blockers, ophthalmic (BLT)
Betaxolol Decrease IOP by Local: Stinging, dry eyes, Effective ocular hypotensive
Levobunolol inhibiting the formation conjunctivitis (rare). agents that lack significant
Timolol of aqueous humor. Systemic: Bronchospasm, ocular side effects.
Timolol, gel- Timolol and betaxolol exacerbation of CHF, dyspnea, Avoid in asthmatics or
forming provided visual field bradycardia, hypotension, patients with obstructive lung
protection. syncope, depression, impotence, disease; caution in patients
st
1 line agents 1 drop QAM or Q12H altered response to with a history of syncope or
Gel: Shake container hypoglycemia, reduction of bradycardia.
once before each use. high-density lipoproteins. C.I when pulmonary and
1 drop once daily cardiac diseases are present

120. Patient has Addisons taking Prednisone PO. She now been hospitalized due to DKA. What
should be given for his addisons?
a) IV dexamethasone
b) IV hydrocortisone + low dose insulin
c) PO prednisone
d) Inhaled Beclomethasone
121. What you should advise her;
a) ICs cause hypokalemia and recommend her to get K supplement
b) ICS cause hypertension

122. Glargine SE
a) Wt. gain
b) Lipodystrophy
c) Cancer
d) Hyperglycemia

123. Patient on 30/70 taking it BID noon and evening –values throughout the day
2200- 5.6 0300-7.5 0800-11.1
What adjustments should be made to the patient’s treatment plan?
a) Increase evening dose
b) Increase noon dose
c) Reduce evening dose
d) Add another dose at noon

124. Which principal mostly contradicts pharmaceutical care


a) Autonomy
b) Privacy
c) Justice

125. HIV patient’s wife comes to the pharmacy asking how he contracted the disease. She is
worried that she may now have HIV too; you tell her that (K-type)
a) Her husband had sex with someone
b) To get tested for HIV
c) To discuss this with her husband

126. A medication is under investigation. To find out if the metabolism of this medication is
inhibited by CYP3A4, what medication should be administered?
a) Ritonavir (3A4 inhibitor)
b) Phenytoin (inducer)
c) Rifampin (inducer)
d) Warfarin
e) Clarithromycin
Clarithromycin inhibits CYP3A4 activity by an irreversible mechanism‐based inhibition which occurs when it
is metabolized by CYP3A4 to form reactive a nitrosoalkane via N‐demethylation.
127. Patient taking sildenafil- the pharmacist should counsel:
a) Don’t take alcohol with it
b) Duration of effect is 4 hours
c) Starts working in 6 hours

128. Patient had tinea pedis – having recurrent infections – counselling should include
a) Use tolfonate as prophylaxis
b) Treat for at least two weeks
c) Switch to tofonate powder
d) Keep using clotrimazole (after the lesions disappear to avoid recurrence- whole course)

129. A new medication is released. This is a kind of fluoroquinolone antibiotics. According to


the general information of this class, what is true?
a) It can change glucose level.
b) Nephrotoxic

130. Case on pain therapy- patient given naproxen and fentanyl patch at discharge from
hospital. Patient applied the patch at noon and now its evening and patient is still experiencing
pain. What to counsel
a) Use naproxen for breakthrough pain
b) Apply another patch
c) Go back to emergency
131. Teriparatide treatment –counselling-
a) Treatment lasts for 2 yrs
b) Take with bisphosphonates
c) Reduce calcium supplements

132. Teriparatide monitoring should include


a) Blood calcium levels
b) Hypocalcaemia
c) Uric acid levels
d) SCR

133. Atopic dermatitis case - All of the following are good advices to manage AD except?
a) Use a steroid
b) Use bath oil after bathing
c) Use calamine lotion
d) Use lanolin (woolfat) products

134. The atopic dermatitis patient is going out on vacation & will be out in the sun mostly.
What should you consel (K-type)?
a) Avoid steroids
b) Avoid the sun
c) Use sunscreens

135. Which is not a secondary symptom for Parkinson’s Disease?


a) Restless leg syndrome
b) Constipation
c) Drooling

136. What should be monitored if patient is taking valprioc acid


a) CBC
b) Liver/rash/GI upset/DI with lamotrigine.
c) Heart
d) Weight

137. Patient with osteoarthiritis is taking Acetaminophen. Wants to try something topical. What
could be a good choice
a) Capsaicin
138. Bupropion patient, working but when experiencing stress and anxiety, smokes to relief.
What to do?
a) Nicotine gum
b) Nicotine patch
c) Nicotine inhaler prn
d) Increase bupropion
e) Relaxing technique

139. Patient is female and 26-year-old has sleeping problem. Patient taking zopiclone already
and some other medical condition also. Now, has a new RX for Bupropion. All of the following
are important to ask before giving bupropion except:
a) Head injury
b) Seizures
c) Bulimia Nervosa
d) Taken MOA 14 days before
e) Pregnant

140. Which of the following antidepressants is associated with the highest rate of hyponatremia
(SIADH)?
a) Mirtazapine
b) Moclobemide
c) Bupropion
d) Paroxetine
SSRI, TCA and MAOi, Chlorpropamide, carbamazepine, phenothiazine, Cyclophosphamide and antidiuretic
hormone insufficiency

141. Treatment of SIADH? Desmopressin


 Demeclocycline; Fludrocortisone  To maintain Na Level
 Furosemide  to decrease fluid
 Decrease water intake
 Varopression antagonist Tolvaptan, conivaptan
 Li

142. Patient with influenza and Asthma – what could be good treatment choice
a) Amantadine (Only for Influ A)
b) Zanamavir (Rhinitis side effects exacerbate Asthma)
c) Osaltamavir (For both Influ A and B)
143. Major side effect of Varenicline? (Alpha4-Beta2 agonist)
a) Insomnia
b) Nausea
c) Suicidal ideation
d) CV problems

144. What is true about Varenicline?


a) Can’t be used in pregnancy
b) Nicotine gum has better effects

145. What does COMPUS do?


Canadian Optimal Medication Prescribing and Utilisation Services (COMPUS) identifies, evaluates, promotes
and facilitates best practice in drug prescribing and use among health care providers and consumers.
Objective: To optimize drug related health outcomes and cost‐effective use of drugs.

146. Know the difference between merck Manual and merck index
Merck Manual Merck Index
The Merck Manuals (outside U.S. and Canada The MSD Provides data, descriptions of
Manuals) are medical references published by the chemicals, drugs, and biological.
American pharmaceutical company Merck & Co. (known The book contains subject matter
as MSD outside the United States and Canada), that cover includes human and veterinary
a wide range of medical topics, including disorders, tests, drugs, biological and natural
diagnoses, and drugs. products, agricultural chemicals,
These manuals were originally developed in book form, industrial & laboratory chemicals,
but have been updated and converted to digital formats, and environmentally significant
including websites and mobile apps. Digital versions also compounds
include audio, 3D models, and animations.

147. Medical error. You told your patient to take methotrexate daily (the instruction was “once
a week” on the prescription). What is the most important side effect?
a) Bone marrow suppression

148. The pharmacist found the error later. However, the patient had already took the medication
for a while. What is the most appropriate action of the pharmacy owner?
b) Report to college of pharmacy
c) Report to Health Canada
d) Report to ISMP
e) Do not apologize because it can give him grounds for a legal sue.
f) Offer to give financial settlement for any possible medical problems.
149. Dementia patient. The cognitive function was suddenly worsened and the test result
became to moderate level from mild level. He said he took a medication recently. What can be
the possible medication?
a) Oxybutynin

150. Not Risk factor for dementia?


a) Age
b) Hyperlipidemia
c) Stress

151. If the above patient experiences insomnia and agitation, what would be the most
appropriate treatment option?
a) Mirtazapine
b) Lorazepam (given when others fail)
c) Haloperidol
d) Olanzapine
e) Trazodone
Trazodone for Behavioural and Sleep Disturbances
Trazodone, a serotonergic antidepressant, is often used successfully to manage agitated behaviour, with
some RCT evidence to support its use. Trazodone is also used to treat disrupted sleep/wake cycles and
“sundowning” (worsening of behaviour as darkness falls).
In a systematic review of various medications used to treat sleep disturbances associated with Alzheimer
disease, only low‐dose trazodone had some evidence of a beneficial effect based on an RCT of 30 patients. In
this small trazodone study, 50 mg QHS administered for 2 weeks to community‐dwelling patients with AD
increased daily sleep time by 43 minutes with no cognitive or functional impairment and no other serious
adverse effects. The review also concluded that no benefit was seen with melatonin and ramelteon (not
available in Canada); studies for other widely prescribed drugs (e.g., lorazepam, oxazepam, temazepam,
triazolam, tryptophan, zolpidem) were excluded.
Benzodiazepines
Data on the efficacy of benzodiazepines for RBD are conflicting. Although their use can result in
oversedation, falls and worsening cognition, benzodiazepines are sometimes indicated for severe agitation,
especially when other agents fail.
Low doses of a short‐acting agent without active metabolites (e.g., lorazepam 0.5–1 mg, oxazepam 5–10
mg, temazepam 15 mg) may be tried. In an acute situation, to manage severely agitated patients,
lorazepam 0.5–1 mg can be mixed in the same syringe with haloperidol (0.5, 1 or 1.5 mg) and given IM every
8 hours for a maximum of 3 days. A double‐blinded, randomized, controlled comparison of IM lorazepam
and IM olanzapine found them to be equally effective.
152. What you’ll say for spouse of dementia husband
a) Stop progression or no more worsening
b) Improve memory function
c) Patient will not get lost again in the metro
d) Improve behavioral symptoms of the patient in future symptoms

153. What antibiotic should be used as a prophylaxis before appendectomy surgery?


a) Cefoxitin or cefotetan (Long acting)
According to several studies, antibiotic prophylaxis should be administered before every appendectomy.
When the patient becomes afebrile and the white blood cell (WBC) count normalizes, antibiotic treatment
may be stopped. Cefotetan and cefoxitin seem to be the best choices of antibiotics.

154. Clarithromycin can increase the blood level of digoxin. What enzyme is involved?
a) CYP3A4
b) CYP2C9
c) P-glycoprotein
Digoxin is thought to be a substrate for the efflux transporter, P‐gp. Clarithromycin is known to inhibit P‐gp.
When clarithromycin and digoxin are administered together, inhibition of P‐gp by clarithromycin may lead
to increased exposure to digoxin.
Elevated digoxin serum concentrations have been reported in patients receiving clarithromycin tablets and
digoxin concomitantly.
In post‐marketing surveillance some patients have shown clinical signs consistent with digoxin toxicity,
including potentially fatal arrhythmias. Serum digoxin levels should be carefully monitored while patients
are receiving digoxin and clarithromycin simultaneously.

155. You gave a patient Lasix instead of Losec (omeprazole) which the patient should take.
When you found out this error, the patient had already taken Lasix (furosemide) for seven days.
There was no toxicity sign. What is the most appropriate lab test?
a) Serum creatinine
b) Blood lipid
c) Blood glucose
d) Uric acid
Monitor Serum Electrolytes and renal function. Also cause hearing loss in high dose or rapid IV infusion SE
hyper urecemia

156. What is the appropriate action to prevent the above error?


a) Give the medication in the original container
b) Put warning label on all ward medication
c) Put the sound drug in closed box
d) Give courses to the patient to teach them how is it danger
157. What is the side effect of furosemide? Hypomagnesemia/ hypocalcaemia/hypokalemia/
hyperglycemia/ hyperuricemia
a) Hypercalcemia
b) Hyperkalemia
c) Hypomagnesemia
d) Hyperphosphatemia

158. There were dispensing errors between hydromorphone and morphine. Nurses were
confused with morphine with hydromorphone and gave hydromorphone instead of morphine.
How would you been able to prevent this error?
a) Write a comment in red to separate opioids from others
b) Make tall man lettering like HYDROmorphone and morphine
c) hydromorPHONE or hydromorPHINE

159. A medication is renally eliminated. What is true?


a) When diuretics are used, the elimination is increased.
b) When dehydration is treated, elimination is increased.
c) When Vit C is used, elimination is increased.
d) When the dosage of the medication is increased, the elimination is increased.

160. A patient is an IV drug abuser. He got bacterial endocarditis and right-sided failure. What
are the symptoms?
a) Hypothermia, weight loss, fatigue
b) Fever, splenomegaly, weight gain
c) Fever, heart murmur, weight
loss
d) Fever, left-sided failure, …
e) Hyperthermia, ………
Sx: chills weakness, dyspnea, night sweats,
malaise, Fever, heart murmur common MAY
or MAY NOT have embolic phenomenon,
spleenolmegally, skin menifestations

161. Which of the followings interacts with amiodarone?


a) Metoprolol
b) Pravastatin
c) Ramipril
ONLY 2 Drugs that decrease Amiodarone concentration are Phenytoin and Cholestrymine (PC)
Beta Blockers
Amiodarone should be used with caution in patients receiving ß‐receptor blocking agents (e.g.,
propranolol, a CYP3A4 inhibitor) because of the possible potentiation of bradycardia, sinus arrest,
and AV block. If necessary, amiodarone can continue to be used after insertion of a pacemaker in
patients with severe bradycardia or sinus arrest.
HMG‐CoA Reductase Inhibitors
HMG‐CoA reductase inhibitors that are CYP3A4 substrates (including simvastatin and atorvastatin) in
combination with amiodarone have been associated with reports of myopathy/rhabdomyolysis.

162. While a patient is on amiodarone, what should NOT be monitored?


a) ALT, AST
b) Chest X-ray
c) TSH
d) Occult blood test (NO BONE, NO RENAL, NO BLOOD, NO SUGAR)
e) Electrolytes K, Mg
f) HR, ECG

163. What is NOT the adverse effect which can show up in the early phase of treatment?
a) Eye deposit (happens at 6 months of therapy)
b) Skin color change

164. A patient has Prinzmetal's angina. He does not have symptoms on activity. What is the
best medication choice?
a) Amlodipine
b) Metoprolol
c) Ramipril
d) Nitroglycerin SL (no symptoms)
e) ISDN
Treatment
 Calcium channel blockers & Sublingual nitroglycerin
Average survival at 5 yr is 89 to 97%, but mortality risk is greater for patients with both variant angina and
atherosclerotic coronary artery obstruction. Risk increases with increasing obstruction.
Usually, sublingual nitroglycerin promptly relieves variant angina. Calcium channel blockers may effectively
prevent symptoms. Theoretically, beta‐blockers may exacerbate spasm by allowing unopposed alpha‐
adrenergic vasoconstriction, but this effect has not been proven clinically.
Oral drugs most commonly used are calcium channel blockers:
 Sustained‐release diltiazem 120 to 540 mg once/day
 SR verapamil 120 to 480 mg once/day (dose must be reduced with renal or hepatic dysfunction)
 Amlodipine 5 to 10 mg once/day (dose must be reduced in elderly and in pts with hepatic dysfunction)
In refractory cases, amiodarone may be useful.
165. A 58 yr male hypertension patient. BP 150/? First time prescription for HTN. He is now
taking salt substitution. What is the most appropriate medication for him?
a) Metoprolol
b) HCTZ
c) Prazosin
d) Clonidine
e) Ramipril

166. You are working for a small hospital. Indefinite supply shortage of fosphenytoin is
expected. What is the most appropriate action?
a) Get as much as supplies from the available sources.
b) Assess any possible therapeutic alternatives to fosphenytoin and get sufficient
supply.
c) Restrict dispensing to patients.

167. The factors that can worsen psoriasis EXCEPT


a) Cool weather (triggers psoriasis)
b) NSAIDs
c) Systemic infection
d) Local trauma
e) Pregnancy

168. Psoriasis patient (knee psoriasis) using fluocinolone, wants to go to Mexico for vacation
for a week. What should you counsel. Or Patient is leaving for Mexico has eczema in elbows
using hydrocortisone cream 1% what to advise patient and he wants to spend some time in sun
a) Avoid sun exposure while on vacation
b) Use sunscreen lotion with A & B protection
c) Discontinue using Hydrocortisone or fluocinolone cream while on vacation
d) Don’t go in sun it can exacerbate eczema.

169. A woman was brought to emergency room. She said she ingested 50 “iron” pills. However,
the pill bottle did not have any label. The doctor asked you for help. What is the most
appropriate action?
a) Find what medication it is.
b) Contact local poison center.
170. Child needs TPN with 0.5 micromole/L of Selenium, your stock is 0.25 micromole/L & 40
microgram /ml, how much of 40 µg/ml of selenium would you add if you want to make 482mL
(M. Wt of selenium is 79).
Answer:
40 microgram ------- 1 ml X microgram -------- 1000 ml
X = 1000*40/1 = 40000 microgram / L. No. of m. mole = 40000 / 79 = 506.32 m. mole /L.

X+Y = 482 ml Y = 0.25/ (505.82+0.25) * 482 = 0.238 L.

171. The followings are the symptoms of althlete’s foot


EXCEPT.
a) Bad odour
b) Pain on the foot (heels and sole)
c) Itchy
Generally speaking, prescription medications are needed to treat
athlete's foot if:
 The infection hasn't cleared after four weeks of self‐
treatment.
 The infection goes away but comes back (recurs).
 The infection is spreading to other parts of the body (such as the nails, groin, or hands).
If you have athlete's foot and diabetes, you should see your doctor right away; don't bother with home
treatment. Athlete's foot can cause dry, cracked skin, which can leave people with diabetes vulnerable to
serious complications such as cellulitis, bacterial infections, or skin ulcers.2
Call your doctor immediately or seek urgent care if:
 Your foot is swollen and developing red streaks.
 There is a profuse discharge of pus or other fluids.
 You develop a high fever and other signs of infection.
If your athlete's food fails to respond to over‐the‐counter medications, your physician will usually prescribe
you prescription‐strength versions of the same topical medications. These include:
 Lamisil (terbinafine)
 Spectazole (econazole)
 Mentax (butenafine)
 Lotrimin (clotrimazole)
 Micatin (miconazole)
 Naftin (naftifine)
 Ertaczo (sertaconazole)
 Exelderm(sulconazole)
 Tinactin (tolnaftate)1
172. A Patient is 30 years old has high fever, stiff neck and feeling like getting seizure admitted
to emergency and was diagnosed with bacterial meningitis. Which of the following causes
meningitis (Ktype)?
a) S. pneumoniae
b) H. influenza
c) N. gonorrhea

173. What is the empiric treatment for the above symptoms?


a) Ampicillin
b) Cloxacillin
c) Cefotaxime + Vancomycin
d) Clindamycin

174. The patient above has severe symptoms (fever, etc) what can be given at this moment?
a) Dexamethasone IV

175. A patient took antibiotic recently and now has C. difficle diarrhea. The symptoms are
severe. With consideration of the severity, what is the most appropriate treatment option?
a) Metronidazole iv
b) Metronidazole po
c) Vancomycin iv
d) Vancomycin po
Combination of metro and vanco oral preferred for very severe condition or IV

176. The daughter of the patient came to the pharmacy and asked why her mother got infected.
What is the appropriate explanation?
a) Due to the antibiotic, a certain bacterium can overgrow and infect the patient.
b) The bacteria are normal flora, but it can infect the patient.
c) The patient contacted with someone who had the infection
d) The patient ate something wrong.

177. A patient was taking lorazepam for a long period for anxiety. The patient is now about to
discontinue it. What is the proper procedure?
a) Discontinue lorazepam gradually for 3 weeks
b) Discontinue lorazepam gradually for 3 months
c) Change it to clorazepam and discontinue gradually for 3 weeks
d) Change it to clorazepam and discontinue gradually for 3 months
178. When you want to see the efficacy of a new medical compound, what is the most
appropriate investigation?
a) Cohort
b) Randomized Controlled Trial with Placebo

179. Study on a new antihypertension and followed up for 5 years wt type of study is it?
a) Cohort
b) Observational
c) Case control
d) Randomized

180. A patient with osteoarthritis, dyslipidemia and diabetes. Current medications are celecoxib,
metformin, statin, low dose aspirin, and etc. He filled 180 caps of celecoxib last months but
came for the next refill (early refill). The instruction for celecoxib was 100mg bid. What is the
drug therapy problem?
a) He requires an additional drug therapy but is not getting it.
b) He is not taking the medication appropriately.
c) He has adverse drug reaction.
181. He admits that he took more celecoxib tablets more than instructed. What is the
appropriate action as a pharmacist?
a) Tell him to contact Dr. and let him assess his condition.
b) Give him OTC analgesics
c) Tell his doctor to increase the daily dosage of celecoxib.

182. A patient with hypertension comes for refill. He says that he has high cholesterol level and
asks you if there are any good dietary supplements that can help him. He also says his Dr. told
him to take a medication called statin, but he refused it because he believed that the risks of
statin are greater than the benefit. What should pharmacist tell him?
a) Respect the patient’s opinion and belief.
b) Show him evidence-based data about the benefit of statin.

183. A patient is newly diagnosed with hypertension and got a prescription of ramipril once a
day. However, he sometimes skipped taking it. At the last follow up with his doctor, doctor said
his BP is persistently high. What is the most appropriate action?
a) Tell him the risks of persistently uncontrolled blood pressure
b) Discuss with the patient the reason the patient was not taking the medication.
c) Call Dr. and ask to change the medication for a simpler one.

184. A pregnant woman. This is the second time of pregnancy. She experiences morning
sickness. She asks you about non-pharmacologic options. What is the best reference?
a) Patient self care
b) Motherisk
c) Drugs in pregnancy and lactation

185. Acute attack of gout admitted to emeregency department. Patient is taking some other
meds also. Dr gave Colchicine 0.6 mg. What to counsel for Colchicine?
a) Do not crush
b) Don’t take with food
c) Avoid grape fruit juice.
S/E: nausea, vomiting, diarrhea, abdominal cramp, neuropathy, myopathy, bone marrow suppression.
Can be taken with/without food. It is only antiinflimatory.

186. In a Compounding sterile preparation pharmacy, parenteral injections were made. What is
the filter size to be used?
a) 0.22 u
b) 0.33 u
187. Which is an appropriate step in cold chain maintenance of vaccines in pharmacy
a) Store vaccines at 0-8C
b) Store them in a separate bar fridge in the pharmacy
c) Stack them up on top of one another
d) Place them with enough room and maintenance temperature etc.
Bar fridge is something which doesn’t have proper temperature control
Store bet 2‐8c, store in middle compartment, donot use in door, stat or near the floor.
Use Domestic frost‐free cross fridge refrigerator.

188. A pharmacist owner has purchased a new pharmacy. He doesn’t work there and he
appointed a pharmacy manager. He gave permission to pharmacist regarding all the activities of
pharmacy but keeping financial issues to himself. All the following activities can be done by
manager except:
a) Taking decision of front store items
b) Taking decision of stocks and ordering
c) Salaray raises for technicians.

189. Patient is on insulin NPH 30/70 in morning before breakfast and evening before supper.
The following are the blood levels of sugar 12 noon  5.4, 3pm  5.4, 8 am 11.6. What
insulin dose has to be adjusted?
a) Increase insulin before supper
b) Increase morning dose of insulin
c) Decrease morning dose of insulin
d) Add short acting insulin in mid morning

190. Common drug review comes under


a) NAPRA
b) CADTH
c) Provinical pharmacy registries

191. Patient is female taking ASA, metformin, glyburide and vitamin D and due for MRI Scan.
Which medication has to be stopped 2 days before scan?
a) ASA
b) Metformin
c) Glyburide
d) VitaminD
192. Patient forgot to skip dose and MRI was done. Blood work was done and Na level was
177meq/litre and K level was OK. Serum Creatinine was normal. Calculate CrCL.Weight
given.
(Use the crockfort formula for calculation based on Ideal body weight)
IBW= 50 kg+ 2.3 (over 5’’) for male, for female = 45.5+ 2.3 (over 5”)
CrCL = 1.2 (140‐ patient’s age, yrs) (TBW, Kg) / (serum creatinine, umol/L)
(for female multiply above eq with 0.85)
If serum creatinin is not given, and patient is above 65 then use value 1.

193. Where do you report this incident of patient taking the medication before scan?
a) ISMP
b) CISMP (Canadian institute of safe Medication practices)
Note: in ISMP. I don’t know what is cismp? So, for error report it to ismp.
Helth Canada‐ check adverse effect and report it to health Canada, Canada vigilance, med effect.

194. Patient came to pharmacy with depressed mood and suicidal symptom last 2 weeks. What
action should the pharmacist take
a) Call the Doctor

195. Dr gave Venlafaxine. Patient has suicidal thoughts. Main Side effect which is of more
concern and should contact Dr for the same?
a) Venlafaxine has to be changed to other since has suicidal ideation
b) Venlafaxine dose is not correct

196. After 4 months patient comes back and wants to know if he can stop now what pharmacist
will say?
a) You can stop medication right
away
b) Can continue medication
long time to prevent relapse
c) Feel better in mood in 1 week.
d) Continue medication for 4 to 8
weeks for better outcome
e) You will have to take these
medications lifelong.
197. Now, the patient has come back with prescription of fluoxetine. Patient asked the
pharmacist when to start fluoxetine. Which of the following true regarding starting the
medication? (question on washing periods refer RX)
a) Stop venlafaxine and start fluoxetine
b) Stop venlafaxine for 1 week and start fluoxetine after 1 week
c) Decrease dose of venlafaxine over period of 1 week then start fluoxetine after then

198. Nurse gave 40 instead of 4 units. What pharmacist will do in hospital pharmacy setting?
a) Change the stock order
b) Use Prefilled syringes
c) Decrease ward stock

199. Indication of medication, Dr wants to know which is not a good source?


a) Therapeutic choices
b) CPS
c) Merck Index
The Merck Index is an encyclopedia of chemicals, drugs and biologicals with over 10,000 monograph on
single substances or groups of related compounds published online by the Royal Society of Chemistry
Monographs in The Merck Index typically contain:
 a CAS registry number
 synonyms of the substance, such as trivial names and International Union of Pure and Applied Chemistry
nomenclature
 a chemical formula
 molecular weight
 percent composition
 a structural formula
 a description of the substance's appearance
 melting point and boiling point
 solubility in solvents commonly used in the laboratory
 citations to other literature regarding the compound's chemical synthesis
 a therapeutic category, if applicable
 caution and hazard information

200. Patient has GERD which symptom you will refer to Doctor?
a) Dysphagia
b) Heartburn
c) Horseness
201. what you’ll recommend for the above patient?
a) PPI
b) Ranitidine
c) Aluminum hydroxide
Trivial to Mild Gastroesophageal Reflux Disease
Most people with mild symptoms do not seek medical attention and will obtain symptomatic relief with
antacids, alginates or histamine H2‐receptor antagonists (H2RAs). Additional therapy becomes necessary if
these agents fail to sufficiently control symptoms or GERD severity increases.
Moderate to Severe Gastroesophageal Reflux Disease
If antacids or H2RAs alone are not effective, or if there is endoscopic evidence of erosive esophagitis, the
most effective and common approach is an 8‐week course of any of the PPIs. The goal is to eliminate
symptoms, heal erosive esophagitis and prevent complications.

202. New medication follows first order kinetics. Mine ffective concentration = 16 ug/litre.
MinToxiconcn=20 ug/litre. Patient takes 300 mg of medication. Cpeak = ……Ctrough=……
If Cpeak of 16 and Ctrough of 15 required. What dose to be given to maintain these Cpeak and
Ctrough levels?
Note: don’t know how to calculate it??????
203. Which of the following medication shows serious adverse effects when administered
intrathecally?
a) Baclofen
b) Methotrexate
c) Vincristine
Vincristine has a high neurotoxicity level. If given intrathecally by accident, it can cause ascending
radiculomyeloencephalopathy, which is almost always fatal.

204. 850 patients in experimental group out of 2000 patients enrolled in a study got headache.
800 patinets in control group.
Calculate NNH. (Number needed to harm)
Calculate relative risk for the same.

205. In a study, patients were selected randomly and given medication and observed for period
of 5 years. What kind of study this is?
a) Case controlled
b) Randomized prospective

206. Ibuprofen and Naproxen were tested and results were found within 95% of confidence
interval = 0.3 - 0.8. Pharmacist analyzes this and what should he say to Dr?
a) There is no therapeutic difference between the 2 drugs
b) Therapeutic efficacy same of the 2 drugs

207. Patient is HIV positive and had unprotected sex with someone. Patient prescribed
tenofovir, lopinavir, ritonavir and zidovudine. Which of the following is true regarding Rx?
a) Tenofovir best in renal impairment
b) Take medications on empty stomach
c) Tenofovir and Ritonavir increases lipid level
Note: Ritonavir and all PI except unboosted Atz are associated with hyperlipidemia.
Tenofovir S/E is renal toxicity. No need to take med on empty stomach.

208. Patient’s wife came to pharmacy and asked pharmacist how husband got AIDS? (Ktype)
a) Tell her that husband had unprotected sex
b) Advise her to go to doctor to test HIV
c) Discuss matter with husband

209. Digoxin interaction. In which of the following cases, pharmacist will consult doctor?
a) Drug interaction
b) Drug side effect
210. Pharmacist put caffeine containing products in front of pharmacy. In the same area, there is
school, all teenagers buying more caffeine products and sale of these products is high. What
action pharmacist should take?
a) Stock more of these drugs
b) Put age restriction
c) Remove from shelf
d) Come to pharmacy for medications
e) Plan an educational seminar in area of school

211. Ischemic stroke patient came to hospital and did CT scan and left-brain hemorrhage
present. Dr decided to give Atleplase.As a pharmacist what will you tell the Dr?
a) Alteplase not to be given to this patient
b) Tell Dr that there are better meds than atleplase

212. If Dr decided to give Alteplase, in which of the following case Dr is showing paternalism?
a) Consulting with wife for treatment
b) Follow hospital protocol
c) Don’t discuss with anyone and give treatment.

213. 33-year-old female delivered baby and decided to breastfeed and now is on 5th week after
delivery wants to try contraception method
a) Medroxypregestrone injection
b) Low estrogen contraceptive
c) Combined contraceptive
d) Diaphragm
e) Cervical cap
However, the long‐term effects on the child are not fully understood. It is recommended that DEPO‐
PROVERA not be administered until 6 weeks postpartum in women who are breastfeeding to avoid risk of
exposure of the neonate to steroid hormones.
Norethindrone Monograph: Lactation
In most women, progestin‐only contraceptives, such as MICRONOR, do not affect the quantity and quality of
breast milk or length of lactation. However, isolated post‐marketing cases of decreased milk production has
been reported. Studies with various orally administered progestin‐only contraceptives have shown that
small amounts of progestins pass into the breast milk of nursing mothers resulting in detectable steroid
levels in infant plasma.
No adverse effects have been found on the health, growth or development of the infant.
214. She started taking contraception and she has itchy nipples and sore. Baby has oral thrush.
Lady got Rx of clotrimazole and baby has nystatin what pharmacist will counsel?
a) Advise to use cream on nipples and wash before breast feeding
b) Give nystatin drops to baby before breastfeeding

215. Now, baby is 13 months old is doing well and started eating solid foods and has no food
allergies. Which of the following will recommend as milk supplement?
a) Soya milk protein
b) Pasteurized cow milk
c) Iron fortified infant formula
d) Hydrogenated infant formula

216. Child has vomiting and has motion sickness. And wants to go on ride. What can be given?
a) Dimenhydrinate
b) Scopalamine
c) Meclizine

217. Patient is looking for Loratidine and Pseudoephedrine combination and asked pharmacist
where can he find. Upon discussion, you know he is hypertensive. What to counsel?
a) Make a big signboard for these medications
b) Make a list of OTCs in his profile

218. Counseling of Tenofovir except:


a) Take it in the morning
b) Taken orally without regard to food
Dosing Considerations
In adults and pediatric patients weighing ≥25 kg, DESCOVY is taken orally once daily with or without food
DESCOVY is a fixed dose combination (FDC) of FTC and TAF.
DESCOVY should not be used alone and should be administered in combination with other antiretrovirals
such as non‐nucleoside reverse transcriptase inhibitors, protease inhibitors, or integrase inhibitors.
In the presence of a pharmacokinetic enhancer (i.e., ritonavir or cobicistat (COBI)), the dose of DESCOVY
should be 200 mg/10 mg (FTC/TAF).
DESCOVY should not be coadministered with products containing any of the same components, FTC or TAF
(ATRIPLA, BIKTARVY, COMPLERA, EMTRIVA, GENVOYA, ODEFSEY, STRIBILD, Symtuza, TRUVADA, and
VEMLIDY); or with products containing lamivudine (3TC, Combivir, Kivexa, Triumeq, and Trizivir) or tenofovir
disoproxil fumarate (TDF) (ATRIPLA, COMPLERA, STRIBILD, TRUVADA, and VIREAD); and DESCOVY should
not be administered with adefovir dipivoxil (HEPSERA).
DESCOVY is not indicated for use as a pre‐exposure prophylaxis (PrEP) to reduce the risk of sexually
acquired HIV‐1 in adults at high‐risk.
219. Patient is 72 years old and is forgetful these days. Dr. prescribed Donepezil. What is the
goal of treatment?
a) Increasing score of MME
b) Stop disease progression
c) Treating behavioral disturbance

220. Which of the following medication causes Parkinson symptoms?


a) Metoclopramide
b) Domperidone
c) Cetrizine

221. Which has to be shaken except?


a) Nitroglycerin
b) Calcitonin
c) Betoxolol
d) Imitrex

222. Late side effect of Amiodarone


Pulmonary fibrosis, hepatic dysfunction, aggravation of arrhythmias.
Various GI, dermatological, neurological, ophthalmologic and thyroid abnormalities.

223. Patient has right sided endocarditis infection. Which of the following are symptoms?
a) Fever and shortness of breath
b) Cardiac murmur, fever, spleenomegaly
c) Cardiac murmur, lung infection

224. Which of the following effective against endocarditis?


a) Cloxacillin
b) Amoxicillin
c) Fluroquinolones

225. 45-year-old lady came to pharmacy has GERD takes omeprazole for GERD.
Mother had hip fracture and asking if she is at risk. She drinks 3 cups of milk everyday. She is
obese and occasionally drinks coffee. What would you recommend as pharmacist?
a) Tell her to take 500 mg Ca bid (3cups= 900 mg Ca)
b) Stop taking omeprazole
c) Stop drinking coffee (no more than 4 cups per day allowed)
d) Do exercise
226. Also advise her to take
a) Vitamin D 400 units bid
b) Tell her to do bone density test
Note: Bone density test not done in women under 50 years old.

227. In future to decrease risk of vertebral fracture she can take:


a) Alendronate
b) Oral contraceptive
c) Raloxifene

228. You can find references in all except:


a) Therapeutic choices
b) Remington
c) CPS
d) Martindale

229. Information regarding Rx medication (K type)


a) TC
b) Merck manual
c) Patient self care

230. Off label uses found in? Medline

231. Dr called asking regarding recent Cardiovascular guidelines. What is electronic source to
gather this information?
a) e-therapeutics

232. Grape fruit juice interaction fastest way to check:


a) CPS
b) TC
c) Merck index
d) Handbook of clinical trials

233. Adverse effects of medications found in all except;


a) Handbook of clinical trials
b) CPS
c) Martindale
d) Merck Index
234. Where can you find medication removed from Canadian market to find generic equivalent
of the same?
a) National drug database
b) Health Canada drug database
c) Other options were databases not books

235. A lady wants to go on vacation in 2 weeks and wants to do hiking there. She doesn’t want
to get periods there and doesn’t want to take tablets everyday. She wants to get pregnant after
sometime. What does the pharmacist say?
a) Tell lady 2 weeks is too short
b) Nuvaring
c) Depo provera injection
d) Evra patch

236. Patient came to pharmacy returning an unopened insulin vial and expired which was given
in error by the pharmacist to prevent this error from happening again?
a) Inventory cycling every month
b) Complaining about expired medication
c) Check before dispensing

237. Celiac disease for patient. Technician is not aware of this disease. What the pharmacist has
to do to increase awareness?

238. Patient has recurrent infection of Athelte’s foot using Clotrimazole cream 1% which of the
foll is true except?
a) Stop using Clotrimazole and use Tolfnate
b) Advise him to use Tolfnate powder for prophylaxis
c) Apply cream bid

239. Patient is taking Haloperidol and changed to other medication. Which of the foll is true?
(check sig and doses for haloperidol)

240. The purpose of medication reconciliation (refer rx files page 122)


a) Giving trade name
b) Use least expensive medication
c) To prevent polypharmacy

241. Which of the following solely depend on physician?


a) Special access program
242. Which of the following is true regarding special access program?
a) To get medication not available in Canada
b) It is manufacturer’s decision whether to give medication or not

243. Which of the following is available in chewable form?


a) Montelukast
b) Zafirlukast.

244. QT prolongation which of the following causes QT prolongation? (K type)


a) Fexofenadine
b) Cimetidine
c) Ketoconazole

245. Blood pressure checking, what to counsel on BP machine?


a) Sit on a chair and put her legs on the floor
b) If the device is reliable one measurement will be efficient

246. In Breastfeeding woman, which of the following affects penetration of drug in breast milk
to child except?
a) Molecule is highly plasma bound
b) High molecular weight
c) Low molecular weight
d) Low volume of distribution
High mol wt, high protein binding, no active metabolite, low volume of distribution

247. Patient took 40 pills of Iron admitted to emergency department. Patient’s family got bottle
of iron pills and don’t know what it is as the bottle without a label. The ER doctor called the
pharmacy, what you do to help doctor?
a) Check books reference with product identification.
b) Check for iron salt identification online
c) Give the number of poison control centre
d) Call physician to check his take iron or not
e) Give symptoms to diagnose toxicity

248. The patient was given with antidote deferroxamine, what is the MOA?
a) Chelating agent
249. 6 hours after ingestion of Iron pills what can be given for treatment?
a) Activated charcoal
b) Gastric lavage
a) Hemodialysis or Blood dialysis
Phlebotomy ‐ first choice (drawing 500 mL of whole blood weekly or biweekly as tolerated) will ultimately
normalize body iron stores (target serum ferritin <50 µg/L); when repeated weekly, may takeup to 2 years
Chelating Agents, deferoxamine: used for those who are unable to receive phlebotomy due to other causes
of iron overload (e.g., transfusions for thalassemia).

250. Patient taking Hydromorphone 30mg for pain bid prn. Pain not controlled patient taking tid
patient is out of refills. Dr is out of town. What will you do?
a) Tell him to go to walk in clinic
b) Give her 1-day emergency supply
c) As a regular custom give him 1 pill.

251. Dr prescribed Fentanyl.What to counsel?


a) 25mcg/hr put in morning and remove at night
b) 25mcg/hr use 2 patches
c) Change every 72 hours

252. BPH – Efficacy comparing action of finasteride and tamsulosin, which one affects size of
prostrate?
a) Finasteride
b) Tamsulosin

253. Terazosin advantage over Finasteride except


a) Terazosin has lesser hypotension effect than Finasteride
b) Maximal response seen in 4 - 6 weeks while finasteride effect seen within 6 months.
c) NOT influenced by prostate size.
d) No sexual dysfunction

254. Patient has acne and skin irritation. Which formulation better gel or cream?
a) Gel not better because of alcohol content.

255. Silver sulfadiazine? for burns


Broad antibacterial spectrum, limited applicability, multiple dressing changes, poorer wound healing than
other dressings, questionable efficacy.
Only fair penetration, sulfonamide sensitivity (rash), leukopenia.
256. Test and placebo analysis and 2x2 analysis. Which of the following parameters used in
analysis and what is the study?
a) Chisquare method (for randomized studies)
Note: Nested study means group within group study

257. In retrospective study, information ins detail for patient: what is more bias for these types
of studies?
a) Information bias (recall bias)

258. Patient is complaining of burning using capsacin cream, after 4 days of using the cream.
What to tell?
a) Stop capsicacin
b) Allergic to capsicacin
c) Patient not using properly
d) Burning sensation will go away with continuous use

259. Which of the following increases INR of warfarin?


a) Rifampin
b) Cimetidine
c) St john’s wort

260. Counseling about warfarin. lady had DVT in hospital had heparin. Discharged from
hospital gave warfarin, what to counsel? (refer to Rx file page 11)
a) Check INR everyday
b) Check INR every week
c) Vitamin K

261. What to Monitor in patient taking metoprolol and simvastatin?


a) CPK
b) LFTs.

262. The above patient developed Myalgia, what is the cause?


a) Simvastatin.
b) Metoprolol

263. To check interaction between simvastatin and grapefruit juice, what you will use?
a) CPS
b) DPD
c) Micromedex
264. Isotretinoin SE?
a) Constipation  it causes sever diarrhea
b) Conjunctivitis

265. Your major concern for isotretinoin? psychiatric disorder

266. Isotretinoin monitoring; CBC, liver, pregnancy, lipid, glucose

267. To get most recent for hypertension published and evident this year, what you will use?
a) E-therapeutics
b) Cochrane org
c) Micromedix
d) Journal

268. A family moved to your neighbourhood and child taking methyl phenidate
a) You can get a verbal Rx from their Dr
b) Ask them to get a new prescription
c) Give them same medication with no Rx.

269. After 2 days they came with a Rx with atomoxetine and methylphenidate and the dr said
the methylphenidate regular should be taken 7:00 and 15:00 and the mother came and
complained the the boy had insomnia last night what u would recommend for her?
a) Rescheduling the 7 and 12:00
b) Switch to controlled and taken HS
c) Take the 10mg twi tablets at 7 am regular

270. She was asking about the atomoxetine, what you’ll say
a) Its not a stimulant drug for the ADHD and may cause somnolence

271. The women told u later her son got headache after taking atomoxetine wt. u will say?
a) If the headache persists, see the DR
b) No problem and its regular SE (transient SE)
c) Its signs of toxicity and you must go to emergency
d) Tolerate it after while

272. Which of the following gout medication contribute in RF?


273. Patient reconciliation means?
a) Don’t intervene with patient believes
b) Check consistency of the regimen in different stages.

274. Patient don’t want use narcotic pill after dental surgery she laid off from her work and she
got the social services she was taking amitriptyline for other indication and she get improved wt
u will do?
a) Tell her that the dr know the best for her
b) Contact the dr and tell him to switch to amitryptyline

275. After amitryptylin she got drymouth and Dr prescripe for her pilocarpine?
a) Refuse to fill it as its not officially used
b) Tell her its an offlable medication for her
c) Health team concerned with this case contacted u to ask u possibility of taking opioid

276. What u will say? Its not recommended as she has a history of upusing marijuana

277. Marijuana requirement


a) Its illegal to sell or purchase it
b) Its legal but patient have to get it
c) It has to prescriped by authorized specialist
d) Should obeys marijuana agency requirement

278. Nitrofurantoin is given with food for?


a) Avoid bitter taste
b) Increase bioavailability
c) Decrease diarrhea

279. All excreted renally except?


a) Moxiflocin
b) Ciprofloxacin
c) Ofloxacin

280. All given on empty stomach except?


a) Cefuroxime auxitile
b) Cloxacillin
281. Patient with MTX mentoring except?
a) Eye monitoring
b) CBC

282. In patient taking MTX instead taking it weakly he got it daily wt SE will appear after 14
days?
a) Myelosuppression
b) Pancriatitis

283. Pregnant with chronic cystitis?


a) Fosfamycin
b) Amoxicillin
c) Nitofurantoin

284. Role of federal government is all except?


a) Prevent Infectious disease spreading
b) Advise Canadian travelers prevent disease
c) Give required vaccine

285. You got robbed in pharmacy, what is your 1st action?


a) Report it to the Office of controlled substance
b) Call manager
c) Call police station

286. Patient taking 100 mg Cp was 20mcg/ml and when the dose increased to 200mcg/ml the
Cp was 60mcg/ml wts the cause of this phenomena?
a) Metabolism saturation
b) Renal excrtion saturation

287. Diabetic Patient with pneumonia wants ttt in home ttt?


a) Ceftazdime IM and clindamycin
b) Amox / Clav
c) Levofloxacin (or macrolide + B lactam)

288. 70 yrs old Patient discharged from hospital with redness and pain on his right arm was
taking cloxacillin iv q4h wt u will give him?
a) Cloxa Qid po
289. Which of the following is schdual I?
a) Mupirocin cream
b) Orphenadrine
c) Gravol
d) Miclizine
Meclizine or its salts: schedule I ‐ For human use ‐ when sold in concentrations greater than 25 mg per
dosage unit.
Meclizine or its salts: schedule III ‐ when sold in concentrations of 25 mg or less per dosage unit

290. Whichh of following cause clarithromycin QT prolongation?


a) Fexofenadine
b) Cetirizine
c) Ketoconazole

291. Obese Patient 25 years with withdrawal symptoms (-ve symptoms) and hearing weird stuff
which med you’ll recommended?
a) Olanzapine
b) Haloperidol
c) Clozapine
d) Ziprasidone

292. What you will monitor to the medication you recommend?


a) Qt-prolongation
b) Hyperglycemia
c) Hypertension

293. Asthmatic taking zafirlukast. The role of zafirlukast all xpt?


a) Stop ICs
b) Decrease salbutamol use
c) Exercise induce asthma

294. S.E. of ICS inhaler depends on all xpt?


a) Device used
b) Technique of using
c) Dose of inhaled ICs
295. Patient taking Valsartan 160 QD and dr increased to bid, the patient developed
asymptomatic hyperkalemia, what you’ll give him?
a) Ca gluconate
b) Salbutamol
c) Na polystyrene SO4
Stop offending agent + IV fluids

296. 70 yrs. patient has diabetic type I & hypertension which of the following is not
appropriate?
a) Nadolol
b) HCTZ
c) Nifedipine XL

297. Patient got a new job and the premium insurance for medicine will cost him 100$/month
he has no chronic symptoms just he got same Rx for antibiotics, what you gonna tell him?
a) You’ll recommend him to subscribe
b) Ask if it was for a limited time
c) Tell wait until having chronic disease
d) U’ll tell him its not a good deal

298. SAP special access program is done for?


a) Selling unapproved medication

299. Federal government will pay for all xpt?


a) Nursing home care
b) Third specialist

300. Provinces should obey Canada health principle to?


a) Get funding

301. Funding of Acute hospital services comes from?


a) Fedral taxation, provincial taxation
b) Fedral taxation only

302. Chron’s disease goal of ttt (K-type)


a) Avoid surgery
b) Prevent se of med
c) Improve condition
303. Mother came with her baby one-year old; he was vaccinated but she worried about swollen
and pain soreness and redness, what you’ll give her?
a) Ibuprofen
b) Acetaminophen (not given unless fever)
c) Cold compress
d) Lidocaine spray (before vaccination)
e) Benzocain cream

304. Female 26 yrs obese came to ask for contraception as she needs something temporarily as
she may want get pregnant after your and don’t worry about the pills, what to give her?
a) Nuvaring
b) Levonorgestrel
c) Medroxy progestrone

305. Woman call and told you she used Nitrofurantoin and got stomatitis. Neither her Dr. nor
her pharmacist recognized it. what will u do?
a) Google it
b) Inform the patient taking the same medication
c) Make a report to health canada

306. You’ll make a day for chronic back pain patients; you’ll invite all except?
a) Anesthetic Therapist
b) Occupational Therapist
c) Dietician
d) Physical Therapist

307. Patient taking simvastatin, fluoxetine, meloxicam, what you will council him about?
a) Interaction between fluoxetine and meloxicam
b) Simvastatin and meloxicam
c) Simvastatin causing myopathy

308. all of the following are side effects of Donepezil, except?


a) Diarrhea
b) Urinary retention
c) Headache
d) Syncope
e) Worsening asthma
S.E: Cholinesterase inhibitors: theoretically, these agents may lower seizure threshold, increase the risk of GI
ulceration or bleeding, or exacerbate COPD or asthma.
Donepezil: >10%: headache, nausea, diarrhea. <10%: vomiting, anorexia, fatigue, sleep disturbance,
syncope, muscle cramps, urinary frequency. Bradycardia (uncommon), heart block (uncommon),
rhabdomyolysis (uncommon), neuroleptic malignant syndrome (uncommon).

309. For religious occasion some people fast the whole day; so, what you’ll recommend for DM
patient: http://guidelines.diabetes.ca/healthcareprovidertools/ramadan-and-diabetes
a) Don’t fast
b) Give him glargine in night
c) Increase insulin in the morning
310. New immigrant female, you felt that she isn’t comfortable talking with you despite her
perfect language; so, what you will do?
a) Talk to her in front of her husband
b) Offer her to talk to your female colleague
c) Keep eye contact with her

311. Patient sexually assaulted asking you for plan B u’ll


a) Give her plan B
b) Give her the phone number of organizations
c) Tell her that u’ve to report it to the police

312. Which of the following is harshing paternalism (violate paternalism):


a) Diabetic educator will adjust insulin dose after measuring glucose level
b) Dr will refer the patient to 3rd specialist
c) Social worker will talk to stroke pt and told them she will change some bad habits
d) New medication pharmacist will follow-up with him

313. Nurse gave a concentrated KI to patient and other nurse catch the problem and asked you
to check, you found that medical entry was correct. What is the best thing to do?
a) Close the cupboard having KI with key
b) Take away of ward the KI bottles
c) Put labels warning label

314. Patient with scratchy throat, rinorrhea and congestion history of using desloratidin PRN
wt’s ur recommendation?
a) Continue with desloratadine regularly
b) Topical pseudoephedrine
c) Oral pseudoephedrine

315. Female CHF patient complain from constipation from 2 weeks in 1st week she had 1
bowel movement every other day then two timed per week then once per week
a) Senna
b) Docusate sodium
c) Mgso4
d) Mineral oil
316. Obese patient taking insulin and still his glucose level uncontrolled, what to give?
a) Metformin
b) Pioglitazone
c) Glyburide
d) Sitagliptine

317. Pt required 5L of 140 mmol/L dialysate, available product already contains 110 mmol/L.
How many mL of 3% NaCl needed (0.9% NaCl contains 154 mmol/L)? Ans- 292ml

318. Patient with liver dysfunction will affect all except?


a) Tissue binding
b) Protient binding
c) Degree of cirhossis
d) Blood flow to liver
e) Internsic metabolic activity

319. In hospital 10 pharmacies grouped together for all of the following causes except?
a) Purchasing power
b) Increase non-formulary medications

320. You runout of codiene 40 mg and want to fill a prescription now wt u can do?
a) Get loan by writtin request and should be returned it back within 48 hours
b) Get loan by writtin or verbal request and should be returned it back within 48 hours
c) Buy it by written request from other pharmacy and register it in sales registry

321. How to mix creams in a pharmacy?


a) Use disposable paper
b) Measuring strile tubes
c) Plastic morter preferred over the glass morters

322. A clinical trial about 2 vitamins X and T may decrease the risk of MI the result is hazard
ratio HR 0.63. (CI 95%:0.3-0.8), so what does that mean:
a) T decrease risk of MI more than X
b) X decreases the risk of MI more than T
c) There is a trend that T may have a role in MI
d) There is no big difference between x and T in decreasing MI
When Hr is below 1 it means it decreasing the risk and if it above 1 it means increasing the risk
And 95 percent it is significant difference so D is wrong, if HR was above 1 so it could cause Mi but it
is below 1 so C is wrong and as HR between T and X is 0 ,63 it means researcher saying he is 95
percent confident that drug X decrease Mi 63 percent more as compare to Drug T and his confidence
level is this can be between 30 to 80 perecent.
If range of confidence interval crosses 1, it's not significant. Range is already given 0.7‐1.5.
If range is before 1 (0.7‐0.9), the drug is more effective, if after 1 the placebo is more
effective. If range was 0.7‐0.9 then ans would be B

323. Drug tested given to adult not children this obeys:


a) Non-malficeince
b) Veracity
c) Confidentiality

324. Case talking about an error done by pharmacist, what is the appropriate action?
a) Report to Institute of safty and Side effect
b) Report to collage of pharmacy
c) Appologize for the patient
d) Tell the Manager

325. What is least likely to cause an error?


a) mL (milliliter)
b) IU (international unit)
c) QD (once daily)
d) Trailing zero after decimal point (most)
e) g (microgram)

326. You are a pharmacist working in a hospital for quality assurance and found the prescriping
errors increased in the weekends, wt u’ll do?
a) Make a committee to investigate
b) Teach Dr about the dangers of worng prescriping

327. Vaccine with its diluents orderd by a pharmacy upon dr’s order.the manufacturars
instructions insists to keep the vaccine under -15C while its diluents under 6C, what’s your
appropriate action?
a) Keep vaccine in freezer and diluents in the fridge
b) Keep both in the freezer
c) Deliver the medication to the Dr’s office for direct use

328. All causing constipation all xpt;


a) Mg salt
b) CaCO3
329. Asymptomatic cystitis with which of the following groups you should initiate ttt?
a) Elderly
b) Pregnant
c) Smokers
d) All patient who asymptoms should initiate ttt

330. Nitrofurantoin SE all except;


a) Pneumonitis
b) Hepatic
c) Renal dysfunction
d) Headache

331. 50 yrs old women taking low COCs for her menopausal symptoms after 2 years she has
vaginal symptoms she has urge incontinence, what you will change in her regimen?
a) Ultra low dose COC
b) Estrogen cream
c) Local progestin

332. Patient taking allopurinol and having renal failure taking naproxen in acute attack which is
wrong in this regimen?
a) Naproxen isn’t a good choice in this case

333. After the acute attack is resolved the Dr decided to give him antihyperuricemic, you will
titrate the dose until?
a) Reach the target dose
b) Normal uric acid level
c) Decrease 50% of total uric acid

334. Risk factor of UTI all except;


a) Coronary artery syndrome
b) Age
c) Female gender
Age: The rate of UTIs in women gradually increases with age. Incomplete bladder emptying, which allows
the residual urine to be rapidly infected by bacteria present.
335. What to do in case of medication shortages
336. Error done by Dr and the pharmacist detect it and dispense it as it was prescribed. All of
the following should be in the investigation committee except
a) The pharmacy manager
b) One of the hospital’s legal team
c) The pharmacist
d) The dr
e) Patient.

337. What to counsel for ophthalmic preparations?


a) Blink hard
b) Put eye drop first then eye ointment
c) Look down and put drops
d) Move eyeballs up and pull eyelid down

338. What can you say to patient to change from precontemplative stage to contemplative stage
of smoking?
 Encourage patients to discuss their smoking openly,
 reinforce relevant health consequences of smoking, but avoid judgmental comments,
 when possible, use a personalized approach to initiating a dialogue, e.g., “I am concerned about the
effect smoking is having on your asthma
 Empower patients with belief in their ability to quit.

339. Influenza information provided to 1ry health care? Triaging of the infected patient

340. Root Cause analysis committee benefit?


 A systematic approach aimed at discovering the causes of close calls and adverse events for the
purpose of identifying preventative measures.
 RCA teams look beyond human error to identify system issues that contributed to or resulted in the
close call or adverse event. I.e: Used for retrospective study.
 The goal is to answer what happened, why did it happen, and what can be done to prevent it from
happening again?
 The process includes document reviews and interviews with the parties involved in the event.
 Flow diagramming, cause and effect diagramming, and identifying root causes and contributing
factors help to organize the events and determine why an error occurred.
 Based on the root causes and contributing factors, actions can be developed to prevent the error
from recurring.
 Measuring the outcome of an intervention is also planned in order to determine the success of the
RCA.
 Tools to assist the team include triggering questions, the five rules of causation, and action hierarchy
340. What to do to decrease the error of putting wrong label on a medication
a) Show the tablets to the patient while counseling
b) Use electronic scanner

341. Where to find withdrawn product?


Health Canada Drug monographs and availability. Contains all licensed products in Canada
Drug Product Database is managed by Health Canada (the federal regulator of therapeutic products)
Database It does NOT provide medical advice on the use of products but contains drug specific
information on all drugs APPROVED for use in Canada & discontinued drugs.
The Only complete list of marketed drugs available, updated nightly.

342. Framingham cardiac risk score contains all except;


a) Family history
b) Smoker history
c) HDL
d) LDL
e) BP
FRS involves age, smoking, diabetes, cholesterol level, systolic BP & HDL‐C. Double the FRS when there is a
family history of premature CV disease (modified FRS)

343. Onychomycosis pt used itraconazole cream for 3 months but no improvement, what is the
DTP?
a) Not correct medication
b) Not enough period of time
c) Not correct dosage form  should be systemic not topical
Oral terbinafine is first‐line treatment when OM is caused by dermatophytes; itraconazole is first‐line
treatment when OM is caused by yeasts and no dermatophyte moulds. Note that supportive evidence is based
on toenail OM; fingernail OM occurs less frequently and often involves yeasts.

344. What wrong with Rx?


a) Azithro for two day
b) Fluticasone 1 bottel
c) Fluconazole STAT
d) Simvastatin OD (no strength was mentioned)

345. Improvement of pneumonia detected by all xpt?


a) Fever
b) Spirometry
c) Level of consciousness
d) X-ray
346. TIA was taking ASA prior the stroke? Switch to clopidogrel

347. Female patient with recurrent UTI should be treated by? take cotrimoxazole for 3 days

348. The Dr prescribe ciprofloxacin for her what of the following is incorrect?
a) Wrong medication for valid indication

349. Patient wih renal failure taking clarithromycin (time dependent)?


a) Give the same dose and increase interval
b) Reduce dose and keep interval

350. Athlete’s foot patient can experience all these symptoms except?
a) Pain
b) Fissures & minor bleeding
c) Itching
Signs and symptoms
 Itching, peeling, scaling, vesiculation, patchy hyperkeratinization and inflammation occurs between
the toes are the main clinical signs. Malodor may be present.
 The acute form is characterized by fissuring, scaling and peeling and the skin between toes appears
white, macerated and soggy.
 Chronic form is characterized by hyperkeratotic, scaly eruptions on weight bearing surfaces of feet,
heels, soles and borders of the feet.

351. He is using miconazole cream, you’ll advice him all xpt;


a) Change to oral tolenaftate
b) Treat for 4 weeks

352. Situation that may cause hypoglycaemia. except


a) 5mls conc 100 U/ml but administered 0.5 mls
b) No food by mouth, insulin continued
c) TPN, continue Insulin
d) Give NPH and regular every meal
e) bata something sa NPH na may food

353. Which one is false regarding Tranexamic acid?


a) Concomitant use of hormonal contraception exacerbates thrombotic risk.
b) Can be used in pregnancy & lactation
c) Need dose adjustment in pt with kidney problems
d) Can cause n & v, diarrhea, visual disturbances, ↓ bp.
e) Used to prevent or reduce bleeding in heavy periods (menorrhagia) in pt < 18 yrs of age
Indications Hereditary angioneurotic oedema.
Increased local fibrinolysis when the diagnosis is indicative of hyperfibrinolysis, as with
conization of the cervix, dental extraction in patients with coagulopathies (in conjunction
with antihaemophilic factor) epistaxis, hyphaema, and menorrhagia (hypermenorrhea).
Contra‐ Patients with a history or risk of thrombosis should not be given CYKLOKAPRON
indications (tranexamic acid), unless at the same time it is possible to give treatment with
anticoagulants. The preparation should not be given to patients with acquired disturbances
of colour vision. If disturbances of colour vision arise during the course of treatment the
administration of the preparation should be discontinued.
Patients with active thromboembolic disease, such as deep vein thrombosis, pulmonary
embolism and cerebral thrombosis.
Patients with subarachnoid haemorrhage: the limited clinical experience shows that a
reduced risk for re‐bleeding is offset by an increase in the rate of cerebral ischaemia.
Haematuria
Hypersensitivity to tranexamic acid or any of the ingredients.
Warnings Visual disturbances including visual impairment, vision blurred, impaired color vision have
been reported with tranexamic acid. For patients who are to be treated for several weeks
with tranexamic acid, an ophthalmic check‐up is advisable (sharpness of vision, colour vision,
fundus, field of vision, etc.) if possible, before treatment is initiated and regularly during
treatments.
Combination hormonal contraceptives are known to increase the risk of venous
thromboembolism, as well as arterial thromboses such as stroke and myocardial infarction.
Because CYKLOKAPRON is an antifibrinolytic, concomitant use of hormonal contraception
and CYKLOKAPRON may further exacerbate this increased thrombotic risk.
The safety of CYKLOKAPRON (tranexamic acid) during pregnancy has not yet been
established. No harmful effects have been reported.
Tranexamic acid is secreted in the mother's milk at a concentration only a hundredth of the
corresponding serum levels (Eriksson et al, 1971). The investigators are of the opinion
that tranexamic acid can be given during lactation without risk to the child.
Clinical experience in menorrhagic children under 18 years of age is not available.
Tranexamic acid may cause dizziness and therefore may influence the ability to drive or use
machines.
In patients with serum creatine concentrations of 120 to 250 µmol/L, 15 mg orally or 10 mg
intravenously tranexamic acid per kg body weight twice daily. At serum creatine levels of
250 to 500 µmol/L the dosage should be 15 mg orally or 10 mg intravenously per kg body
weight at 24‐hourly intervals, and at serum creatine levels of 500 µmol/L or more, the same
dose should be given at intervals of 48 hours between doses.
Adverse Gastrointestinal symptoms (nausea, vomiting, diarrhea) occur but disappear when the dose
Effects is reduced.
Isolated cases of dizziness or reduced blood pressure have been reported.
Allergic dermatitis has been reported less commonly.
354. What is the intervention with Schedule II?
You should be available when patient requires assistant, you are required to interevene
when there is a purchase, needs prescription

355. Metronidazole, developed rash? tell patient it is transient

356. When changing medications, safety, efficacy and


tolerability should be:
a) Discussed only at the time of change
b) Not linked to the reporting of ADRs
c) Excluded from documentation
d) All considered as part of pharmacovigilance

357. NPN- Stands for?


a) Natural Product Number for vitamins,
calcium supplements, probiotics, mineral oil.

358. Bio-marker if not candidate for statin – His CRP – Sample PEBC

359. Liver problem what does not contribute. Hyperkalemia

360. Warfarin use & CHADS score

361. Amoxicillin maculopapular rash  allergic reaction

362. Study by “chance” – randomized

363. Medical team, what are the benefits?


a) To help patient with professional
b) Collaborate team input
c) To provide comprehensive outcome of the patients help.

364. Why does 8 yo patient given carbamazepine instead of phenytoin?


a- aesthetic purposes?

365. Medical error, if you did not disclose to patient what did you uphold? Veracity.

366. Clinical trial, you did not give to children what ethical uphold? Non-maleficence
367. Patient insist to take other med. If you followed her choice what did you practice?
Autonomy

368. What is the special authority that physician has? Drug formulary

369. What is the advantage of penfill cartridge? Already mixed

370. What can you do to ensure the patient get the most out of his medication? Digoxin,
Calcium, Levothyroxine
a) Call the patient how to take medication
b) Make followup
c) Review how to take med
d) Give blister pack

Patient has DM, BP, Reynauds. He is taking Metformin 1g BID, Metoprolol 50 mg BID
(titrated for 6 mos), Atorvastatin 10 mg, Amitryptilline 10 mg qhs, BP = 130/80, FBG 6-11,
371. Which medication is too high a dose?

372. Which medication is suboptimal? Atorvastatin

373. SINEMET IR to change to SINEMET Controlled Release, what do you do?


a) Wait for 8 hours
b) Decrease intervals between doses
c) Decrease dose
When patients already receiving levodopa are switched to SINEMET®, levodopa must be discontinued for at
least 12 hours or more before SINEMET® is started. SINEMET® should be substituted at a dosage that will
provide approximately 20% of previous levodopa dosage.
When patients are receiving levodopa monotherapy or SINEMET® (levodopa and carbidopa), this medication
must be discontinued at least 8 hours before therapy with SINEMET® CR is started. Dosage with SINEMET®
CR 200/50 should be substituted at an amount that eventually provides approximately 10 to 30 percent
more levodopa per day. The interval between doses should be prolonged by 30 to 50 percent. This is because
when Compared to Sinemet immediate‐release formulation, bioavailability of Sinemet CR is 25–30% lower
and duration of action 25–30% longer.
Because entacapone enhances the bioavailability and therefore the central effects of levodopa, it may be
necessary to adjust the dosage of levodopa during the initial days to weeks of entacapone therapy in order
to reduce levodopa‐related dopaminergic adverse reactions, e.g., dyskinesias, nausea, vomiting and
hallucinations. In some cases, it may be necessary to reduce the daily dosages of levodopa by about 10‐30%.
This can be achieved through either reducing the dose of the levodopa preparation itself, or by extending
the interval between doses, according to the clinical condition of the patient.
374. Which should not need witness for destruction?
a) Buprenorphine
b) Mepridine
c) Pentazocine
d) Olanzapine

375. Patient rushed to the hospital with signs and symptoms of respiratory depression,
decreased bowel movement, miosis, What toxicity?
a) Opioid
b) Alcohol

376. What can you remove in the shelf which can cause abuse? Dimenhydrinate

377. All of the following are needed to be record in the narcotics sales report, except?
a) Telephone
b) Name of doctor
c) Details of patient
d) Pharmacist
e) Manufacturer

378. Drug A 4%, Drug B 10%, Drug A and B is mixed to have 0.1% concentration of Drug A.
How many mL is placed to get 250 mL of mixed solution

379. Dilution problem: Drug avail 5% to make 0.0125%. Get 2.5ml of 5% then dilute to
1000mL of water

380. Initial act robbery what to do? Give the narcotic


Do not compromise safety of pharmacy staff or patients.

381. Patient on gliclazide has new medication Metformin, what to monitor?


a) Creatinine clearance
b) Liver function

382. Gabapentin side effects: hyperglycemia


Post‐marketing adverse events that have been reported, which may have no causal relationship to
gabapentin., are as follows: agitation, anaphylactic reaction, angioedema, blood creatine phosphokinase
increased, blood glucose abnormal, drug rash with eosinophilia and systemic symptoms, fall,
gynaecomastia, hepatic function abnormal, hepatitis, hepatitis cholestatic, hepatitis fulminant,
hyperglycemia, hypoglycemia, hypersensitivity, hyponatremia, jaundice, loss of consciousness, pancreatitis,
pulmonary oedema, renal failure acute, rhabdomyolysis, sexual dysfunction (including changes in libido,
ejaculation disorders and anorgasmia), StevensJohnson syndrome.

383. Patient out of Percocet, will be seeing doctor 2 days from now, needs refills, what to do?
a) Go to walk in clinic

384. Isotretinoin, 0.5 mg/kg for the first week then increased 1 mg/kg. Cummulative 120-150
mg/kg. Patient weighs 60 kg
a) 30 mg bid for 3 mos
b) 40 mg bid for 4 mos
c) 60 mg bid for 4 mos
d) 60 mg bid for 5 mos
e) 40 mg bid for 5 mos

385. Elderly has asthma, had Influenza what to give?


a) Influenza vaccine
b) Amantadine
c) Oseltamivir
d) Zanamivir

386. Patient with HIT, what to give?


a) Dalteparin
b) Enoxaparin
c) Tinzaparin
d) Fondaparinux
e) Nandroparin

387. Patient has hypertension, diabetes,


diabetes neuropathy. Which disease
may have cause constipation?
a) Hypertension
b) Diabetes

388. Child what to use to relieve


congestion? Nasal saline

389. Woman with hep B – what to


follow up in care plan?
a) Immunize her son
NOVEMBER 2010
1. A patient is asking for OTC drug for heart burn & GERD problem, what will you to tell him
for immediate symptomatic relief?
a) Famotidine has longer duration but antacids have faster onset of action
b) Famotidine has many drug interactions
c) Cimetidine is the worst in DDI
d) She can take OTC famotidine

2. Two weeks ago after he took famotidine, he came today complaining from pain radiating to
his neck and shoulders. What is your advice for him?
a) Continue taking famotidine
b) Arrange to see your doctor as soon as it is convenience for you
c) Go to the emergency

3. Corticosteroids side effects all except


a) Hirsutism
b) Weight gain
c) Systemic lupus erythematosus
SLE is autoimmune disease treated by corticosteroids

4. Which statement is correct about mood stabilizers?


a) Depressed mood responds more slowly to mood stabilizers
b) Therapeutic levels of Li, carbamazepine and Valproic are well established
c) After the episode is resolved you should maintain the therapeutic level constant
d) Need monitoring periodically to the drug level
e) Symptoms resolved within 1-2 months
f) Maintaining therapeutic level is not important for all drugs just the ones with low
therapeutic index

5. MA is customer in your pharmacy you noted that it is the second time this week to buy
Senokot-S ® (Senna-Docusate) what you should do:
a) Advise her not to take more than prescribed
b) Ask her about her apparent to overdose
c) Call her doctor and tell him that she takes overdose
6. A pharmacist noticed that the pharmacy technician sells Tylenol 1 to her friends. What
should he do?
a) Tell the pharmacy technician that selling OTC narcotics needs his intervention
b) Assess monitoring methods to decrease medication diversion
c) Tell the pharmacist that he should record sold OTC narcotic

7. All of the following are symptoms of anaphylaxis except?


a) Urticaria (hives)
b) Pharyngeal swelling
c) Blisters on the trunk
d) Tachycardia
Also, except fever and bradycardia

8. In dispensing a prescription by law, the pharmacist is responsible on all except?


a) Accuracy
b) Efficacy
c) Appropriateness
d) Potency of drug
e) Safety

9. Antiepileptic drug may cause folate deficiency in children?


a) Phenytoin
b) Carbamazepine
c) Valproic acid

10. A father comes to pharmacy asking if his 16 years old daughter taking any contraceptive pill
or not. Pharmacist decides not to disclose the information. In this case, which code of ethics
conflict the pharmacist experiencing?
a) Autonomy & veracity
b) Justice & veracity
c) Autonomy & justice
Note: there was no confidentiality in the choices

11. Doctor called pharmacist and requested him not to tell patient about side effect of certain
drug because if patient knows, he will refuse taking the medication. If the pharmacist accepts
that. Both pharmacist & physician work with
a) Veracity
b) Paternalism  seeking Beneficence
c) Autonomy  Violating autonomy
12. Counselling about Sumatriptan nasal spray all except
a) Do not open till you are ready to use
b) Prime many times before use  No prime, no shake
c) Instill one spray in one nostril while closing the other one
d) Hold your head backward for 20 seconds

13. Metabolic acidosis may be caused by all except


a) Diabetic ketoacidosis
b) Thiazides overdose
c) ASA toxicity
d) Polyethylene glycol poisoning
e) Metformin overdose
Thiazides and loop diuretic its metabolic alkalosis and other diuretics metabolic acidosis

14. A patient is with ASA overdose; his blood PH is 7.2, which of the following is true for him?
a) Respiratory alkalosis
b) Metabolic acidosis
c) Metabolic alkalosis
d) Respiratory acidosis
His PH is 7.2; this means that he has not developed acidosis yet. Therefore, answer is respiratory Alkalosis

15. Which drug can be used with tranylcypromine?


a) Dextromethorphan
b) Fluoxetine
c) Phenytoin
d) Pseudoephedrine
e) Meperidine

16. PT takes BDZ for insomnia, but he wakes up early with anxious daytime & experiencing
frequent awakenings during his sleep. What Benzodiazepine is the patient most likely on?
a) Flurazepam
b) Triazolam
c) Oxazepam
d) Diazepam
e) Chlordiazepam
f) Cholrazepate
Triazolam has short half life responsible for his symptoms
17. Pharmacist received this prescription Testosterone undecanoate 40 mg cap, M 90 cap,
Dispense 30. Why the pharmacist could not dispense this prescription?
a) It lacks time interval
b) Dose is high
c) Can be written only not verbal
d) No repeats for verbal
e) Has to be written as part fills

18. What is correct about testosterone undecanoate


a) Increases symptoms of benign prostatic hyperplasia
b) Capsules should be put in the fridge
c) Taken with food
d) It is taken 6-month to show effect

19. KB is treating from cancer with methotrexate and azathioprine, KB may suffer from all
except
a) Neutropenia
b) Cardiotoxicity
c) Hepatotoxicity
Anticancer drug with cardiotoxicity: Doxorubicin

20. Adding vincristine to KB may be result in:


a) Increase the risk of cardio toxicity
b) Increase nausea& vomiting
c) Increase the risk of neurotoxicity
d) Increase the risk of neutropenia

21. Patient takes alendronate, you can assess the efficacy and safety by monitoring?
a) Gastritis
b) Bone mineral density
c) Serum Ca++ level

22. For protection of 2 years old child from sun burn, you can do all except?
a) Use sun block
b) Allow playing outdoor when it is cloudy
c) Use wide-brimmed hats
d) UVA is present all day and can penetrate through clouds, windows and clothing.
23. Allopurinol counselling
a) Take on empty stomach
b) Restrict fluid intake
c) If you note any rash contact your physician
d) Increases INR of warfarin
Allopurinol is better tolerated when taken with meals
Patients receiving allopurinol should receive a high fluid intake (e.g., 2.5–3 L daily)
Rare reports of enhanced anticoagulant effect with patients taking vitamin K antagonists

24. Drug X is not metabolized & excreted unchanged from the kidney. It is given IV with dose
100 mg. Now we want to switch the IV to oral dose, what is the oral dose if Vd: 0.25 L/kg, AUC
I.V.: 150 mg.h/L, AUC Oral: 100 mg.h/L. Given it is 100% metabolized & cleared.
Answer
F = AUC oral * dose oral / AUC IV * dose IV Oral dose= 150*100/100=150

25. Pharmaceuticals are considered the second major expenditure in health Canada, what is the
first one?
a) Hospitals
b) Physician salaries
Hospitals > Pharmaceuticals > Physician fees

26. When using ticlopidine monitor all except


a) CBC
b) Neutrophills
c) Creatinine
d) LFT
e) Triglycerides
f) Glucose level
All except glucose and kidney. Medoxiflox, ticlopidin  no renal adjustment
Gabapentin and digoxin safe with hepatic patient
Ticlopidin activated hepaticly  Ticlopidin is not eliminated by renal excretion, and routine dosage
adjustments are not required in patients with renal impairment

27. A, B, C are interchangeable, drug D has the same active ingredient of drug C but it was
enlisted in a separate formulary, why?
a) It may be sublingual
b) It may be SR tab
c) It is not bioequivalent for drugs A, B & C
III only. But if he said that A, B, C are normal tab => so answer is all
28. American patient came to your pharmacy his medications (Ramipril 10mg, metformin 500
mg) are over. He needs 2 days’ supply. If you gave him the medications you worked with
a) Justice
b) Nonmaleficence
c) Veracity

29. A dr Rxed for 8-year-old girl codeine 20mg her father came to the pharmacy to dispense it,
the available codeine strength 5, 10, 15mg, what should you do?
a) Loan the remaining quantity from another pharmacy & refund it within 48hr within 5
days
b) Tell him to Buy from another pharmacy by verbal or written order
c) Tell him to buy from another pharmacy by written order and keep it in the sale
record
d) Tell him the strength is not available and you can’t dispense it.
e) Dispense a mixture of 2*10 mg to adjust the dose
f) Call the MD to write a new Rx matching with what’s available

30. A patient brought Rx to rph for codeine in a pharmacy. The same rph while working on
another Pharmacy found the same patient asking for codeine + caffeine (Schedule—ll??), what
rph will tell him?
a) Dependency might develop if he takes too much codeine (telling behind the counter)

31. All can be used in treatment of hot flashes except


a) Estrogen
b) Progesterone
c) Raloxifene  it causes hot flashes
d) Venlafaxine
e) Gabapentin

32. Erythromycin causes all of the following, EXCEPT:


a) Nephrotoxicity
b) Diarrhea
c) QT prolongation
d) Nausea & Abdominal pain
e) Photosensitivity
Stevens‐Johnson syndrome and toxic epidermal necrolysis have been reported. Allergic Reactions Urticaria,
mild skin eruptions and anaphylaxis have been reported.
33. Doctor called you asking about the uses of certain drug according to evidence depending
study where can you look for.
a) Cochrane Library
b) CPS
c) Meta analysis
d) RxFiles

34. Penicillin powder 5000 000IU when we add 23 ml of solvent its conc. Becomes 200 000
IU/ml. How much solvent should be added to make conc. 125 000 IU/0.5 ml
a) 15 ml
b) 18 ml
c) 20 ml
d) 25 ml
Soln: Calculate powder volume
200 000 ‐‐‐‐‐‐ 1 ml 5000 000 ‐‐‐‐‐‐‐‐ X ml X= 25 so powder volume = 25 – 23 = 2ml
Solvent volume 250 000 ‐‐‐‐‐ 1ml 5000 000 ‐‐‐‐‐ X ml X = 20 so solvent volume = 20 – 2 = 18 ml

35. How much coal tar powder you should be added to 120gm 5% coal tar ointment to make
concentration of 20% in white petrolatum or You have 120 gm of 15 % coal tar ointment. So
how much coal tar so you need to add this ointment to make 20% in white petrolatum?
a) 22.5 ml
b) 30 ml
c) 37.5 ml
Soln:
(Powder.) 100 15 part
20
(Oint.) 5 80 part
80 part = 120 gm 15 part =X so X = 15 * 120 / 80 = 22.5

36. You need to prepare 5 L of 140 mmol/L nacl. The available conc. is 110 mmol/L, how
much NaCl 3% should be added to obtain the required conc (0.9% NaCl = 135.8 mmol/L)
Soln:
Calculate conc. Of nacl by mmol/L 0.9 → 135.8 mmol 3→X so X= 452.6 mmol/L
Calculate the needed amount of nacl by allegation
110 312.66 part (of 110 mmol/L)
140
452.6 30 part (of 452.6 mmol/L)
No of parts = 342.66 342.66 →5000 30 → X so X= 437.75 ml
37. Best way to avoid drug name confusion is tall letter typing. A correct example of this
technique is:
a) PredniSONE / predniSOLONE
b) PredNISone / predNISOLone
c) PredniSONE / PREDnisolone

38. Which vaccine should be taken every 10?


a) TD
b) Influenza
c) Pneumococcal conjugate
d) Typhoid

39. Drug Utilization Review K-Type


a) Collecting previous data
b) Has definitive concept
c) Needs small patient intervention in the future
Drug Utilization Reviews (DUR), also referred to as Drug Utilization Evaluations (DUE) or Medication
Utilization Evaluations (MUE). DURs are including three categories:
• Prospective ‐ evaluation of a patient's therapy before medication is dispensed
• Concurrent ‐ ongoing monitoring of drug therapy during the course of treatment
• Retrospective ‐ review of therapy after the patient has received the medication
While drug utilization effectiveness is retrospective only

40. The health authority sent brochures about influenza to pharmacists to increase readiness for
the next flu season, what should be included in the brochures?
a) How to meet the shortage in flu medications
b) How to deal with symptomatic patient
c) The percentage of drug resistance cases last year

41. You are arranging a campaign about hypertension. You can look for information from all
except
a) Walking clinic
b) School camp

42. If you will arrange a meeting and preparing a brochure for that meeting it should contain all
except?
a) Success stories
b) Cost for participation
c) Place of the meeting
d) Time of the meeting
43. You are a pharmacy manger. One of the employees accuses another worker saying that he
misusing his sick vacations, what you should do
a) Arrange meeting between accused & accuser to know the truth
b) Tell the accuser that sick vacation is a negotiating right & there is little things to do
c) According to human rights, you cannot invade personal life & there is nothing to do
d) Call doctor of the accused employee to confirm that he was sick

44. Patient using Rizatriptan wafer, what is true sentence?


a) Absorbed from the buccal cavity
b) Alcohol should be avoided during treatment
c) Has longer duration of action than tablet
d) Used in migraine with nausea & propranolol increases bioavailability of Rizatriptan

45. Clarithromycin suspension?


a) Keep in the fridge
b) Shake well before use
c) Take with food
All macrolide: keep out of fridge except Erythromycin

46. At the hospital you discovered a dispensing error in heparin infusion in which the heparin
concentration was 1mg/ml instead of 0.1 mg/ml (10 times error). You can do all the following
except
a) Assess the need for vit K
b) Tell the doctor
c) Replace the wrong conc by the right one
d) Stop heparin till aptt become OK
Vit K has no role with Heparin, it should be protamine sulfate

47. To prevent this error, you can do


a) Order ready prepared conc from the manufacture

48. Drug X is used to decrease mortality rate, these are the result
Group NO Drug X Placebo
50 30% 25%
100 26% 30%
300 32% 36%
500 28% 33%
Overall 25% 31%
This result mean:
a) Overall relative risk reduction is 6% this is absolute risk reduction
b) Drug X has different effect in different groups
c) Difference in mortality is very small to indicate the effectiveness of the drug
When population is 50, the percentage to decrease mortality is 30, when it is 100 it is 26, we will expect
when it is 300, it will decrease again, but it increases not decrease, so random results

49. Patient came to your pharmacy he has MI (his profile includes many cardiac drugs) He
asked for a chair because he cannot stand till finishing the prescription, which scale you can use
to assess his case?
a) New York Heart Association
b) Framingham risk score
NYHA III  Symptoms with less than ordinary activities

50. A clinical trial about 2 vitamins X & T that may decrease the risk of myocardial infraction
the result for T is HR 0.63 (95% CI: 0.3-0.8).so that is mean
a) T decrease risk of MI more than X
b) X decreases the risk of MI more than T
c) There is a trend that T may has a role in MI
d) There is no big different between X&T in decreasing MI

51. The provincial authority does all except:


a) Assess the complains related to pharmacy practice
b) Investigate the need for practice sites in certain area
c) Show steps for whom need to enter the practice field
d) License the practice sites
Note: Practice site mean pharmacy

52. 3 months infant, his mother came to your pharmacy she told you that she has just switched
her baby from breastfeeding to formula instead, now she is worry because he did not defecate
from 36hrs, what you should do?
a) Wait and see
b) Warrant treatment with mineral oil
c) Warrant treatment with prune juice
d) Warrant treatment with glycerin supp.
e) Refer to doctor
Diarrhea 48hrs  refer. Constipation  5‐days in adult
53. In a hospital pharmacy there is a certain concentration of morphine solution. The nurse is
requesting preparation of another concentration to be available; you will think about all except?
a) Cost
b) Wastage
c) Dispensing error due the presence of many concentrations of morphine
d) Effectiveness of the new concentration

54. In a hospital pharmacy the most dispensing errors were in heparin & morphine what can be
done to decrease such errors
a) Do not order high concentrations
b) Put these drugs in a locked area in the pharmacy
c) Separate them
d) Computer alarm
Computer alarm warn staff about the differences between Heparin and morphine is one of
recommendations

55. In which reference you can look for dose


adjustment in case of renal failure?
a) AHFS  also CPS and TC

56. In which reference will you look in new investigational drug


a) Compendium of Pharmaceuticals and Specialties CPS
b) American Hospital Formulary selected Drug Information AHFS
c) Martindale
d) Cochrane
Investigation drugs  Martindale, Pubmed, medline, fact and comparison, unlisted drug FDA website

57. Patient is taking HRT sometime she suffers from bothersome vasomotor symptoms, which
regimen she is using
a) Estrogen
b) Progesterone
c) Estrogen + progesterone daily
d) Estrogen daily +14 days progesterone monthly
e) Estrogen daily +14 days progesterone every 3 months

58. Drug will be recalled from the market due to its adverse effect on stomach, the physician
called you and asked you to order quantity from this drug because he see that stomach bleeding
is a minor effect. What you should do
a) Stock a big quantity as the doctor requested
b) Do not sell or stock this drug since the time you received the recall order
c) Call the suppliers and order all quantity they have

59. And what you shouldn’t do


a) Post the recalling display in the pharmacy
b) Contact patients to whom the drug was dispensed
c) Look for the patch size and number which is affected

60. A father came to your pharmacy and he was worried about his child who ingested nystatin
cream what you should do
a) Send to the emergency
b) Monitor for 24 hrs for drowsiness
c) Tell him not to worry, nystatin is not absorbable orally
Nystatin is poorly absorbed from the GI tract, intact skin and mucous membranes. Detectable blood
concentrations are not obtained after usual dose administration.

61. A responsible from caring home for disabled children requested OTC staff to keep them for
emergency you will do all except
a) Label alcohol “for external use only”
b) Give her generic instead of Cortate ® (hydrocortisone 0.5%) cream
c) Ask her to bring a prescription for Ipeca syp when she need it
d) Counsel about Ipecac syp & how to use it
e) Ipeca syrup is OTC product

62. In hospital patient has received 1 gm vancomycin, after 2 hr the blood conc was 35 mmol/L.
T½ of Vancomycin is 3 days. After 72 hr the conc was 17 mmol/L & he has received a second
dose of 1 gm vancomycin. The blood conc was 15mmol/L, what is the steady state trough conc.
Of Vancomycin
a) 30 mmol/L
b) 15 mmol/L
c) 45 mmol/L

63. When should he take the third dose?


a) 1 day after the second dose
b) 2 days after the second dose
c) 3 days after the second dose
d) 5 days after the second dose
64. To add drug to hospital formulary as interchangeable. All are right except
a) It should have the same acquisition price
b) The patency of drug Y is still valid
c) Drug Interchangeability, must meet the following requirements:
d) Same amount of same active ingredient
e) Comparable pharmacokinetics
f) Same clinically significant formulation characteristics
g) To be administered in the same way as the drug prescribed

65. Non-pharmacologic advice for recent liver transplant patient with UTI:
a) Do not drink fluids
b) Avoid Echinacea
Echinacea stimulates the immune system and used to prevent and treat the common cold and other
respiratory infections. Echinacea should be used with caution in atopic individuals. It is contraindicated in
immunosuppressed patients, including those with HIV and autoimmune diseases or those taking
immunosuppressant medication.
Saw palmetto, African plum tree (Pygeum africanum), stinging nettle and pumpkin seeds used in treating
BPH and their role remains unclear
Oil of evening primrose, dong quai, ginseng: Studies have failed to show a beneficial effect for reducing hot
flashes
American ginseng reduces the duration of the common cold by about 6 days if used daily for up to 4 months.
Decreased INR with concomitant warfarin use. American ginseng has demonstrated significant postprandial
blood glucose lowering when 3 g is used prior to a 25 g glucose challenge
Based on evidence of efficacy in patients with depression, St. John's wort is considered a potential first‐line
monotherapy option for MDD of mild to moderate severity.

66. To exclude forgery of a prescription, you can consider all of the following except
a) Prescriber is well known
b) Unrecognized hand writing
c) Prescriber has prescribed for someone before
d) Prescriber works in the local area

67. Doctor called you and asked about the treatment of certain disease, which resource you will
use?
a) Therapeutic choices CTC
b) Compendium of Pharmaceuticals and Specialties

68. Patient with diabetes & taking atorvastatin for high LDL, now the level of LDL is ok but he
has high TG, what to do?
a) Add Cholestyramine
b) Add Fenofibrate
c) Add Ezetimibe
d) Change to rosuvastatin
e) Increase dose of atorvastatin

69. A woman came to your pharmacy. She has greyish vaginal discharge without itching what
should you do
a) Give her miconazole vaginal cream
b) Refer her to physician
c) Bacterial Vaginosis

70. Which antibiotic should be taken with food?


a) Cefuroxime axetil
b) Penicillin
c) Residronate
d) Itraconazole

71. To see the overall and decrease liabilities you can


a) Assess the economic characteristic in the surrounded area
Overall = Total liabilities / Net worth

72. You can find drugs interactions in all of


these resources except
a) Drug facts & comparison
b) Remington
c) Hansten & Horn’s
d) Clin-Alert

73. Pharmacy manager has vacancies of pharmacist &technician what should he do before hire
new staff
a) Prevent any overlap
b) Assess work load in each shift
c) Decrease work hour in week end

74. Patient has asthma he is taking salbutamol inhaler, today he comes with a prescription for
ipratropium. What is the main concern of the pharmacist before dispensing this prescription?
a) Risk of arrhythmia from combination of a beta blocker & ipratropium
b) Assess his need for corticosteroids
75. Patient with Asthma is taking a β-Blocker and corticosteroids. Asthma is still not controlled.
What is the next option?
a) Add Ipratropium
b) Increase dose of Corticosteroids

76. The technician prepared the following prescription as a blister pack


R: Calcium 650 mg iii daily. Digoxin QD PO. ASA QD PO. Senokot TID
What the pharmacist will pay attention to:
a) Calcium in three components morning, noon & supper
b) 1 Digoxin in the morning
c) 1 ASA in the dinner
d) 1 Senokot tab in 3 components

77. Patient has angina and his profile includes Nifidipine, from the collaboration relationship
between pharmacist & physician. If the pharmacist called the physician and asked him to
change Nifidipine, which code of ethics the pharmacist would follow
a) Non maleficence
b) Justice

78. Patient has schizophrenia he is on Haloperidol today he comes with prescription of


Risperidone instead of haloperidol. He asks for a chair because he cannot stand for a long time,
you noticed that while he is setting, he stands many times and bends his back. Before changing
haloperidol, doctor should assess?
a) Extra pyramidal effect
b) Anticholinergic effect

79. This is the first episode for the patient and his symptoms have resolved with Risperidone,
what is the recommended duration for therapy
a) 6 months
b) 1 year

80. Which side effect that patient suffers from


a) Akathisia (restlessness, unable to sit)
b) Acute dystonia (Rethmic jurks, twitches repetitive movement or abnormal posture)
c) Tardive dyskinesia (hyperkinetic movement of face, protrusion of tong)

81. A female is asking for the emergency contra plan-B®. What will you ask her?
a) How many partners do you have?
b) Did you take oral contraceptive before?
c) When was your last menses?

82. What is the contraindication for plan-B?


a) Ectopic pregnancy
b) Planning for pregnancy
Women with known or suspected pregnancy. A pregnancy test should be performed before taking Plan B.
A history of ectopic pregnancy need not be considered a contraindication to use of this emergency
contraceptive method. However, physicians should be alert to the possibility of an ectopic pregnancy in
women who become pregnant or complain of lower abdominal pain after taking Plan B.
Patients with undiagnosed abnormal vaginal bleeding, a pregnancy test should be performed before taking
Plan B.

83. LM is 32 years old female. She has allergic rhinitis. She is asking for OTC medication for
her symptoms. She is also taking OC & topical erythromycin for her acne. Which question
pharmacist should not ask to differentiate between allergic rhinitis and common cold?
a) Is there any nasal discharge?
b) Do you have fever?
c) For how long do you have symptoms?

84. When to refer, if she has


a) Dyspnea
b) Headache
c) Symptoms for more 5 days
A & B If K‐type, she is taking OCP

85. A 50 years old male was diagnosed with depression. He has sexual dysfunction,
dyslipidemia and hypertension. His profile includes atorvastatin, propranolol, ASA,
hydrochlorothiazide and sildenafil. What is DOC for his case?
a) Mirtazapine
b) Bupropion
c) Fluoxetine
d) Citalopram

86. Which drug worsens his depression


a) Sildenafil
b) Propranolol
c) Atorvastatin
87. Now this patient suffers from an acute gout attack, what is the drug of choice in his case
a) Indomethacin  not in hypertension or kidney failure
b) Acetaminophen  not used in Gout
c) Colchicine  close monitoring due to atorvastatin
d) Corticosteroid  not in hypertension or dyslipidemia
Acute Gout: ICC  Indomethacin, Colchicine, Cortisone

88. Patient came to the hospital he was diagnosed with arrhythmia. His blood pressure is 88/60.
All the following drugs can be used in treatment of arrhythmia except?
a) Propranolol
b) Sotalol
c) Ramipril
d) Nadolol
e) Digoxin

89. What is DOC for this patient


a) Propranolol
b) Diltiazem
c) Digoxin
Most patients will need drugs to achieve rate control, with the exception of some patients with intrinsic AV
nodal disease (usually elderly individuals). Pharmacologic control of heart rate should initially be attempted
with a beta‐blocker or a calcium channel blocker (diltiazem or verapamil).
Digoxin is usually inadequate alone for rate control. Because of its potential toxicity, amiodarone should be
used only as a last resort.

90. Cancer Patient came with a prescription for chemotherapy induced nausea & vomiting
included: Metoclopramide starting 3 days before chemo, Corticosteroids prn &
Diphenhydramine. Two days later, he complained from funny sensation in his mouth Feeling of
restlessness, facial spasm, involuntary movement. Which drug is the cause for this symptom?
a) Metoclopramide
b) Corticosteroids
c) Diphenhydramine

91. What will be your action?


a) Tell him this is a side effect of corticosteroids and continue medications
b) Tell him this is due to metoclopramide and continue your medications
c) Refer him to physician
92. DV is patient in your pharmacy. His son came with a written prescription for DV. He asked
you to dispense 6 months’ supply of DV medications to send them for his father who is now in
Florida. DV has an insurance plan which allow him to take 3 months supply. What is correct
about concerning his health coverage?
a) DV’s prescriptions can be transferred to an American pharmacy
b) DV is covered where ever he is
c) Permission from the father to his son to pick up his medication
a) Father is no longer entitled to his ODB covered medications since he no longer lives in
Canada.

93. What is least important concern of the pharmacist about this case
a) His son needs to bring a written consent from DV to act instead of him with health
care authority
b) Written prescription is considered a consent

94. From your knowledge about private insurance plan in Canada, what is correct about this
case?
a) DV can take 6 months’ supply if he pays cash

95. SD has ascites. He was using spironolactone but no improvement. Which drug can be added
to spironolactone to enhance diuresis?
a) Hydrochlorothiazide
b) Furosemide
c) Metolazone
d) Amiloride

96. An assessment of the treatment effectiveness can be done by measuring


a) Electrolyte
b) Creatinine
c) Weight  losing from 1-1.5 kg/day

97. MT is a diabetic patient. Now he has osteomyelitis, the organism is MRSA, so he was
treated by Vancomycin. Advises for diabetic patients to avoid diabetic foot are all of the
following except
a) Monitor foot daily
b) Do not go bare foot
c) Take ASA 81 mg daily
98. If MT is allergic to the used antibiotic, what is the suitable substituent?
a) Linezolid
b) Cloxacillin

99. What is the recommended duration of treatment?


a) 6 weeks  4‐6 weeks
b) 2 months
c) 6 months

100. EB performed checkup lab tests for lipid profile, His doctor told him that his cholesterol
level is high so he prescribed for him atorvastatin 10 mg daily. EB decided not to take any
medication until the next lab time after 6 months and depend on changing life style & natural
products. EB in lunchtime at work buy takeaway foods. He also drinks one glass of wine every
night, what is best action for the pharmacist toward EB decision?
a) Talk with him about the cardiovascular risks of high cholesterol
b) Show respect to his decision
c) Speak with him about cholesterol chelating therapy

101. Which changes in life style will not benefit in decrease the risks of cardiovascular disease?
a) Take food from home
b) Prepare healthy food
c) Alcohol
d) Exercise

102. GM has severe symptoms of rheumatoid arthritis, she took Hydroxychloroquine 200mg
BID for 6 months, but now it is ineffective, so she went to her doctor who changed it to
methotrexate 15 mg injection weekly. GM takes caco3 daily, Why Hydroxychloroquine was
ineffectiveness
a) CACO3
b) Low dose
c) Medication is affected by Diuretics
d) Medication is not indicated as an initial treatment for severe cases
e) Short course

103. What is main concern about using methotrexate


a) Alopecia
b) Myelosuppression
c) Taken on empty stomach to increase its absorption
104. With methotrexate, all of the following should be monitored except?
a) Liver enzymes
b) CBC
c) Thyroid

105. If a dispensing error has happened in methotrexate dose, as a result GM has taken
methotrexate daily instead of weekly for 1 week what should the pharmacist do?
a) Withhold apologizing, because it is a ground for legal action
b) Report to health Canada
c) Report to the provincial authority
d) Offer to pay the cost of GM treatment
e) Better report to ISMP

106. MU is a pregnant woman she has gonorrhea. What is the treatment of choice?
a) Cefixime
b) Amoxicillin 7 days

107. What is the suggested antibiotic for treatment concurrent Chlamydia?


a) Azithromycin
b) Erythromycin
c) Metronidazole

108. Female patient diagnosed with irritable bowel syndrome. She suffers from attacks of
diarrhea and constipation but the most bothersome is diarrhea. She took psyllium but is not
working. What will you recommend treatment for her?
a) Give Loperamide
b) Give bran
c) Give methylcellulose
Loperamide: Used in traveler diarrhea except when patient have fever or blood

109. She came to your pharmacy complaining from the worst attack of constipation. She had
not bowel movement 2days ago. She has no fever but has severe pain. What is your action
a) Refer her to doctor due to her high risk of colon cancer
Referral alarm symptoms: recent consistent change in bowel habit, fever, anemia, persistent daily diarrhea
or constipation, history of colon cancer, abdominal mass, bleeding
110. OD is a student in the third year of the university. He has asthma, his profile is
Salbutamol inhaler 1-2 buff 150min before exercise……………. last refills
10 days ago
20 days ago
50 days ago
Beclomethasone inhaler 1puff BID ………………………………… last refills
10 days ago 50 days ago
Which indicate that his asthma is uncontrolled?
a) He needs daily salbutamol before exercise
b) He wakes up last week with asthma symptoms
c) High dose of cortisone (BID)

111. Which medication does he need?


a) Oral prednisone
b) Salmeterol
c) Theophylline
d) Patient has cellulitis

112. What is the suspected microorganism?


a) Haemophilus influenzae
b) Group A Streptococcus
c) Mycobacterium avium

113. What is the recommended duration of therapy?


a) 10 days
b) 21 days

114. Which antibiotic is not suitable for treatment of this patient


a) Cefazolin
b) Levofloxacin
c) Cephalexin
d) Clindamycin

115. MF has heart failure. (His profile includes ACE inhibitor), his doctor told him that there is
a drug is beneficial for him but if it is titrated quickly, it will cause serious adverse effects.
What is that drug?
a) Furosemide  generally no need for titration, started IV in HF
b) Ramipril
c) Carvedilol  needs titration for 2-4 weeks
116. Which drug should be added to decrease risk of mortality?
a) Digoxin
b) Beta blocker
c) Ca channel blocker
d) Increase dose of ACE Inhibitor
Prescribe only BBs that have been shown to reduce mortality: bisoprolol, carvedilol and metoprolol
succinate (not available in Canada). Metoprolol tartrate is available in Canada, but has not been shown to
reduce mortality in patients with HF.
Candesartan has reduced cardiovascular mortality and hospitalizations for HF. Valsartan has reduced
morbidity but not mortality.
An aldosterone antagonist (eplerenone or spironolactone) used to reduce mortality and morbidity

117. Diabetic patient treated with Metformin 500 mg TID & Glyburide 2.5 mg BID. His blood
glucose readings are: At Morning  6.5 mmol/L, After Supper  10.3 mmol/L, Hba1c 10.
Hba1c indicates the medications effect in
a) 1 month
b) 2 months
c) 3 months
d) 5 months

118. What is the best action for this patient?


a) Increase evening metformin
b) Increase evening glyburide
c) Decrease morning metformin
d) Decrease morning glyburide
e) Add NPH insulin
Metformin up to 2000mg, Glyburide up to 20mg, Gliclazide up to 300mg, then add Sitagliptin.

119. A patient takes PPI; He needs Ca supplement which Ca salt will you recommend for him
a) Caco3
b) Ca citrate
c) Ca acetate
d) Ca gluconate

120. Which is not a trigger factor for psoriasis


a) Pregnancy
b) Trauma
c) Hot weather
d) Systemic Infection
e) Stress
121. Which study type is not suitable for studying nausea & vomiting in pregnancy?
a) Double blinded randomized clinical trial

122. A 3-month pregnant female who has recently developed vulvovaginal candidiasis
infection. What’s the drug of choice?
a) Clotrimazole for 3 days
b) Clarithromycin
c) Ciprofloxacin 7 days
d) Nystatin vaginal cream daily for 14 days is a safe recommendation

123. Health Canada provides drug insurance for all except


a) Children
b) Indian affairs
c) Royal Mountain Police
d) Inuit
e) Military forces
Note: health Canada also provides insurance for prisoners

124. The broadest pharmacoeconomic perspective is


a) Patient
b) Pharmacist
c) Physician
d) Insurer
e) Society
If no society then choose insurer

125. Do not shake before use, all except:


a) Nitroglycerin N.S
b) Sotalol N.S
c) Livostin E.d
Livostin should be shaken before each application. Patients should clear the nasal passages prior to
administering

126. Patient received 1gm IV dose of drug X, blood samples were taken 2 hours later and were,
At 10 00 ---- 60 mmol/L. At 22 00 ---- 45 mmol/L. Calculate T ½ of drug X
Answer:
Log C = log Co ‐ KT /2.303 Log 45 = log 60 – K * 12 / 2.303 k= 0.02397
T1⁄2 = 0.693 / 0.02397 = 28.9 hr
127. All are side effects of Ondansetron except?
a) Constipation
b) Diarrhea
c) Headache 11%
d) Constipation 4%

128. Occlusive bases include all of the following, EXCEPT:


a) White petrolatum
b) Eucerin
c) Oil in water emulsion

129. Used in local treatment of neuropathic pain


a) Lidocaine
b) Capsicum
Don't cover lidocaine because it increases systemic absorption. It causes hypotension and bradycardia

130. Which drug cannot be use in morphine allergic patient?


a) Meperidine Increasing in potency decrease in allergy
Fentanyl
b) Codeine Hydromorphone
Morphine
c) Ketorolac Oxycodone
Codeine

131. The most drug diversion in intensive care unit ICU is?
a) Fentanyl
b) Morphine
c) Codeine
d) Acetaminophen
e) Nitrous Oxide

132. All of the following is are the Common SE of Atomoxetine EXCEPT


a) Weight gain
b) Abdominal pain
c) Decreased appetite
d) Dyspepsia
e) Sleep disturbance
S.E: Headaches, rhinorrhea, upper abdominal pain, nausea, sedation, vomiting, decreased appetite,
dizziness, fatigue, emotional lability and small increases in heart rate and blood pressure.
Significant: suicidal ideation, sudden cardiac death, liver toxicity, exacerbation of tics. May increase QTc
interval. Methylphenidate withdrawal cause suicidal ideation
133. A pharmacist wants to delegate some responsibilities to the technician to reduce his
workload. All of the following can be a barrier except
a) Manager attitude
b) Pharmacist availability
c) Technician availability
d) Federal law
Pharmacist availability should not affect tasks delegation to the technician because it is the technician that
will be doing the tasks not the pharmacist

134. Who is responsible for dispensing errors in hospital pharmacy?


a) Physician
b) Pharmacist
c) Nurse
d) Technician

135. Treatment of hypoglycemia where blood glucose level <4 mmol/L


a) 4-5 dextrose tab
One Dextrose tab contains 3gm. And as we need 15gm => 5 tab

136. Raloxifene can be used in postmenopausal women to


a) Treat osteoporosis
b) Treat hot flashes
c) Decrease risk of DVT

137. Newfloxacin cannot be interchanged with other medications if:


a) AUC ↑
b) AUC ↓
c) Therapeutic index narrow
d) Therapeutic index wide

138. The easiest dosage form for preparation an interchangeable drug is:
a) Suspension
b) Solution
c) Sustained release tablets

139. M 6 yo child. He has hyperacidity, does not want to take ranitidine pill (ranitidine is
sparingly soluble in Alcohol), what would be the option to give him ranitidine?
a) Elixir
b) Syrup form
c) Suspension
d) Paper powder
e) Solution form

140. What is right about mixing insulin


a) Regular insulin should be withdrawn first then NPH
b) NPH insulin should be withdrawn first then Regular

141. Patient told you that he forgot his insulin after opening in the kitchen counter and asked
how many days it will be effective.
a) 28 days
b) 14 days
c) Till expired

142. You have 1% Atropine sulphate solution. You want to prepare atropine e.d. How much
nacl USP should be added to the formula to be isotonic (1% atropine is isotonic)
Atropine sulfate 1% 7.5 ml Sterile water DS 15 ml
Soln:
Since Atropine Sulfate 1 % is isotonic, we have to confirm that sterile water is isotonic too.
Water used = 15 ‐ 7.5 = 7.5 ml
Isotonic means 0.9 gm ‐‐‐‐‐‐‐‐ 100 ml X gm ‐‐‐‐‐ ‐‐ 7.5 ml
X = 7.5*0.9 / 100 = 0.0675 gm = 67.5 mg

143. Odd ratio of drug A is 5 times greater than drug B, what does this mean?
a) Drug A is more effective than B
b) Drug B is more effective than A
c) Drug A is slightly effective than B
d) Drug B is slightly effective than A
e) There is no difference between A and B

144. Which group of drugs has first dose phenomena in which it causes severe hypotension
a) Beta blockers
b) Αlpha adrenergic blockers

145. What is the main cause of dispensing errors in hospital pharmacy


a) Unsuitable environment
b) Inadequate training for the staff
c) Workload
146. In opioid dependence treatment, what is right about using Naltrexone
a) It can be repeated till the symptoms stopped
b) It reduces the respiratory toxicity
c) Patients should remain opioid-free for a minimum of 7-10 days before starting
Naltrexone, a long‐acting opioid antagonist, is ineffective in decreasing long‐term opioid use, even with
psychosocial support. In addition, naltrexone‐based withdrawal results in no benefit compared to other
detoxification treatments
Naloxone: no respiratory depression, short acting can be repeated
Naltrexone: respiratory depression, long acting not repeated
Naltrexone used mainly in withdrawal in maintenance of abstinence

147. Patient was diagnosed with hypothyroidism. He is taking levothyroxine. 3 weeks later, his
TSH level is low 0.2mu.ml. What is the best action?
a) Decreasing dose of Levothyroxine
b) Increasing dose of Levothyroxine
c) Withholding Levothyroxine for 2 weeks, then re-test.
d) Montior TSH, T4 and then increase the dose
e) Monitor T4, stop the drug and monitor TSH
Dose adjustment should be done in 6‐8 weeks

148. Female patient was diagnosed with hypothyroidism, currently on Levothyroxine. Now, she
is pregnant, what is the suitable action?
a) Increase dose of Levothyroxine
b) Decrease dose of Levothyroxine
c) Change to methimazole
d) Change to propylthiouracil

149. Most prominent side effect of Zidovudine is:


a) Kidney toxicity
b) Neuropathy
c) Liver damage
d) Peripheral neuropathy
e) Nephrotoxicity
Zidovudine S.E: Nausea, headache, malaise, fatigue, rash, myositis, myocarditis, anemia, leukopenia,
(AZT) hepatic steatosis, elevated liver enzymes, lactic acid and CK. Longterm use associated with
pyrimidine peripheral lipoatrophy.
analog Additive hemotoxicity with other agents, e.g., anemia with dapsone, foscarnet, ganciclovir,
pentamidine, ribavirin. Pharmacologic antagonism with stavudine.
Avoid combined use of AZT and ribavirin or stavudine as are activated by the same
intracellular pathways. Available as an oral syrup.
150. Polydipsia & diarrhea are signs of Lithium
a) Toxicity
b) Overdose
c) Adverse effects
Vomiting & Diarrhea  Overdose
Lithium nephrotoxicity will occur within a month of onset of use of the drug, manifested polyuria and
polydipsia. Dyspepsia and diarrhea are side effects, if came together,
But diarrhea alone toxicity, here polydipsia means dehydration which will decrease sodium level leads to
lithium toxicity

151. 25 years old female came to your pharmacy asking for plan-B. She told you that she was
sexually assaulted. What should you do?
a) Give her plan-B
b) Give her phone numbers of sexual assault authorities
c) It obliges for pharmacist to tell the police about any sexual assault

152. A solution consists of substance A& substance B with total volume 50ml. We have stock
of substance A with conc of 4%. We want to make a final solution of 0.1% substance A. What
is volume of substance B should be used?
Soln:
C1*V1=C2*V2 4*V1 = 0.1*50 so V1= 1.25 ml
Volume of substance B = 50‐1.25 ml = 48.75 ml

153. Vitamin D capsule contains 1.25mg. Each 100mcg=400IU of Vit D. If you want to prepare
60gm containing 10000IU.How many capsules do you need?
Soln:
1.25mg*1000=1250mcg 100mcg = 400IU 1250mcg = X IU so X = 5000IU
No of cap = 10000 / 5000 = 2 caps

154. A woman from USA came to your pharmacy. She want to buy 1000 of pseudoephedrine
for her family to be ready for the next flu season. What is the main concern of the pharmacist to
refuse her request?
a) Pseudoephedrine is a precursor for methylphenidate
b) USA board allows passage of personal use only of pseudoephedrine
c) Pseudoephedrine has many drug interactions
d) It is hard to the pharmacist to counsel her relatives

155. Which of the following is not a S.E of phenytoin?


a) Grasping of lips
b) Gingival hyperplasia
c) Rash 5–10%
d) Hirsutism and acne.
e) Constipation

156. Which is not a goal of therapy of obesity?


a) Decrease risk of cardio vascular diseases
b) Decrease risk of sleep apnea
c) Enhance psychological effect of wellbeing
The overall aim is to reduce excess body fat for health and not for cosmetic reasons; reducing weight by 5–
10% can result in important health benefits. Minimum goal: stabilize and prevent further weight gain,
prevent weight regain, Prevent and treat obesity‐related comorbidities and complications

157. KU is a student in high school. It is important to speak with him about hepatitis B which
spreads by:
a) Sexual contact
b) Water
c) Food

158. You are a pharmacy manger, the clinical counsellor told you that the pharmacist has
increased his duties by 30% to 50%, you can do all except:
a) Delete unnecessary activities which do not add benefit to the pharmacy
b) Offer to increase his salary
c) Do another job description includes his new activities

159. Manager of pharmacy putting the work’s schedule, he is concerned with all except
a) Workers are treated equally
b) Schedule template from month to month

160. Mother came to pharmacy with a baby suffering from croup (stridor/barking cough), what
would be appropriate step to do?
a) Watchful wait for 48 hours
b) Refer to dr.
c) Refer to emergency
d) Give her dexamethasone

161. You received an Rx of androgen, where can you check the regulation of this prescription?
a) Compendium of Pharmaceuticals and Specialties
b) Controlled drug and substance act
c) Food and Drug Administration website
162. Occurs naturally & decreases incidence of pneumonia? Thick sputum

163. KM is patient in your pharmacy. Today he came with this prescription Lorazepam M 100
tab 3 refills. What should you do?
a) Do not fill this prescription as it should not has refill
b) Do not fill it as it needs time interval for refills
c) Fill it as written
Controlled drugs are the only one need intervals

164. KM will move to another province to be near to his daughter, so he asks you to transfer his
prescription to another pharmacy close to her house, what is correct about this case:
a) This prescription cannot be transferred
b) The prescription can be transferred without refills
c) The prescription can be transferred with its refills but only once

165. 1 week ago, you dispensed Nitrofurantoin to a patient. Today she came to the pharmacy
and told you that she got stomatitis. Her doctor and dentist did not find any cause for her
stomatitis. What should you do?
a) Report to Medeffect (health Canada)
b) Report to Institution of Safe Medication Practice
c) FDA
d) Canadian pharmacist journal

166. A mother came to the pharmacy. Her child has Cystic Fibrosis she told the pharmacist that
he should has a role in helping her child. She is worry because he is taking many medications.
The pharmacist can ask her about all except:
a) Immunization record
b) Family history of dyslipidemia
c) Medication history
d) Any known allergy

167. MG is your regular patient for 3 years. His medication profile includes Ramipril,
ticlopidine & oxycodone he has no refills for any of them. He told you that he forgot to go to
his physician because he thought that he has enough quantity of medications. His doctor is
away until next Monday& he has discovered the he does not have enough oxycodone. What is
your best action?
a) Send him to a walk-in clinic to bring a prescription
b) Advance him some tablets till he brings authority from his physician on Monday
c) Try to call an emergency physician
168. The most dispensed drugs in Canada
a) CV drugs
b) GI drugs
c) Anti-depressants

169. What is the treatment of pneumocystis


jirovecii in HIV patients? Cotrimoxazole

170. To assess the accuracy of a web site. You may consider all of the following except
a) Number of references used
b) Sponsor of the web site
Evaluating Internet information: To select information from internet few key criteria to consider.
 Authorship; Who and their credential
 Referencing; Credible references like primary
 Disclosure; Is there any potential conflict of interest on the part of author? (Conflict of interest,
publication bias, research funding sources, research ethics (pregnancy, children, placebo not included &
chemo drugs).
 Currency. What is the date of last revision? Start with tertiary.
Search Techniques: Boolean ("advanced") versus non‐Boolean search
 Boolean = supports the use of "AND". OR", "NOT", etc.
 Non‐Boolean = ONLY one keyword could be used; e.g., e‐CPS

171. A pharmacist has received prescription for hemorrhoid formula consists of hydrocortisone,
zinc oxide & nitroglycerin 0. 2% Mitt 30 gm He will use Nitrol® ointment 60 gm (nitroglycerin
2%) to prepare this formula. How much Nitrol® ointment will remain in the tube?
a) 27gm
b) 57gm
c) 48gm
Soln:
Required nitroglycerin is 0.2% in 30 gm 0.2 ‐‐‐ 100 X ‐‐‐ 30 X= 0.06 gm
Amount of nitroglycerin in Nitrol oint. 2 ‐‐‐ 100 X ‐‐‐ 60 X = 1.2 gm
Amount needed from Nitrol oint
1.2 ‐‐‐ 60 0.06 ‐‐‐ X X= 3gm so amount remaining = 60 – 3 = 57gm

172. What is the method used for preparing this formula?


a) Trituration (powder + powder)
b) Levigation (powder + liquid or cream)
c) Geometrical dilution
d) Pulverization by intervention
173. Patient is treating from her third depression episode. She was treated by citalopram. 3
months ago, her doctor switched her to Fluoxetine responds well to fluoxetine & symptoms is
fully controlled. She should continue treatment with fluoxetine:
a) 9 months more
b) Indefinitely

174. The cost of necessary physician services is:


a) Fully insured & funded from general revenue
b) Funded from provincial tax

175. ASA S.E? Stomach bleeding. Black tarry stool. Rash

176. Patient taking ASA. But it caused him stomach S.E. What can be taken instead of ASA?
a) Celecoxib
b) Naproxen
c) Mefenamic acid
d) Ketorolac

177. A community pharmacy has been sold to a new owner. He hired a manager to be
responsible on the dispensing activity but he kept the financial responsibilities for him self
The manager can do all of the following except:
a) Be a consecutive manager for narcotics
b) Overview the performance of other workers
c) Purchase orders directly to account & contracts with nursing homes
d) Dispense medications
e) Increases the salary of staff
f) Purchase narcotic

178. Which thing will be sole for the new owner?


a) Discuss for hiring new staff
b) Increase staff salary
c) Overview front merchandise

179. What is the main area can the owner work in to increase his profitability
a) Increase opening hours
b) Increase advertising in the local area
c) Decrease pharmacy staff
180. Newfloxacin, a new antibiotic, shows concentration dependent killing. Newfloxacin
efficiency data are gathered from?
a) Double blind placebo studies
b) Comparative prospective studies
c) Singe blind studies

181. In patients with renal failure, what adjustment of the above newfloxacin is appropriate?
a) Keep dose but increase interval
b) Decrease dose and decrease interval
c) Maintain both dose and interval
d) Decrease both dose and interval

182. Newfloxacin is a new antibiotic related to fluoroquinolones. It is still under researches to


be used in a dose 400mg BID. If this dose is ineffective what is the best action
a) Keep dose but increase interval
b) Decrease dose and decrease interval
c) Maintain both dose and interval
d) Decrease both dose and interval
Concentration dependent antibiotics Time dependent antibiotics
Those antibiotics which eradicate pathogenic Those classes of antibiotics whose killing response is
bacteria by achieving high concentration at dependent on time are termed as time dependent
the site of binding are termed as concentration antibiotics.
dependent antibiotics. In renal disease decrease dose of the drug but kept at
In renal disease increase intervals of drug the same intervals.
& Keep the same dose Higher concentration of such drugs does not result in
These antibiotics show optimum response in greater killing of organism. “The inhibitory effect can be
killing bacteria(bactericidal) when their effective because their concentration exceeds the MIC
concentration is either equal or greater than for the microorganism. Hence, these antibiotics are
10 times above the MIC (minimum inhibitory referred to as time-dependent antibiotics.
concentration) at the site of infection for For time-dependent drugs, the pharmacodynamic
certain target micro-organism. parameter can be simplified to the time that serum
The pharmacodynamic parameter of such concentrations remain above the MIC during the dosing
antibiotics can be simplified as a peak/MIC interval (t > MIC)”. They show optimum killing response
ratio. when the time that the drug remains above the MIC is
Examples: either equal or greater than 50% of the dosing interval.
Aminoglycosides, flouroquinolones & Examples:
Azithromycin are the classes of antibiotics Different antibiotics whose response is time dependent
which show concentration dependent killing are penicillins, cephalosporins, carbapenems,
of organism. monobactams), clindamycin, macrolides (erythromycin,
clarithromycin), oxazolidinones (linezolid).
183. From your knowledge about fluoroquinolone, which substance that may be biologically
interact with newfloxacin
a) Erythromycin
b) Calcium  all metal cations
c) Warfarin
d) Digoxin

184. KU is a diabetic & HTN Pt, Came to pharmacy for his monthly medication supply. He is
taking Diamicron® (gliclazide) 80 mg. Today he came with a new prescription as his doctor
changed him from regular Diamicron® to MR formula
R/ (D/C Diamicron 80), Diamicron SR 30mg 4tab QD M: 60 (2 boxes)
Diovan® (Valsartan) 1-tab QD M: 60 (2 boxes)
If a dispensing error has happened as Diamicron’s label was fitted on Diovan & Diovan’s label
was fitted on Diamicron. KU has taken the medications as written on the labels for a week till
he read the manufacture leaflet and noted the difference in the dose. If KU did self-monitoring
for himself. He would find:
a) Increase in pulse rate
b) Decrease in pulse rate
c) Hyperglycemia
d) Increase in systolic blood pressure
e) Increase in diastolic blood pressure

185. If KU came to the pharmacy & told the pharmacist about the error. What is the first action
of the pharmacist?
a) Assess the patient current state
b) Call his doctor

186. What is the most appropriate immediate step after discovering the error?
a) Try to get hold of the Pharmacist responsible.
b) Talk to the nurse since she administered the medications to the patient this morning.
c) Report the pharmacist to the OCP.
d) Speak to the next physician you see about the error.

187. How can the pharmacist do to prevent such error?


a) Show the boxes to patient when counselling
b) Separate pronounced similar drugs on the shelves
188. All are associated with Dysmenorrhea, EXCEPT:
a) A decrease in the level of Prostaglandins
b) An increase in the level of Prostaglandins
c) An increase in Bradykinins
Dysmenorrhea occurs as prostaglandins are released from lysing endometrial cells in the luteal phase of
an ovulatory cycle. Women with dysmenorrhea have higher concentrations of PGF2‐alpha and PGE2 in
their menstrual fluid than women who do not complain of pain on menstruation.

189. DY came to the pharmacy asking for Iron supplement. He was in a walking clinic 6
months ago and the doctor prescripe for him iron supplement, but he did not buy any iron
supplement since that time. You noticed that he looks pale, you may not dispense iron for DY
& refer him if he has all except:
a) Dark stool as that may indicate stomach bleeding
b) Fresh blood in stool as it may indicate bleeding from colon

190. If you dispense Iron for DY you will advise him to do all except:
a) Take with milk to decrease stomach upset

191. All are lab tests for anemia except:


a) Hematocrit
b) Albumin bound ferric it is rarely used to diagnose anemia as a lab test
c) Hemoglobin
d) Serum ferritin
e) Mean cell volume (MCV)

192. SW is 9 years old. He takes methylphenidate about 2 years for ADHD. Today he came to
your pharmacy and told you that he wants to stop his medication because it makes him drowsy
at school and he found difficulty to fall sleep at night, furthermore he fells embarrassed when he
takes the medication at school. What is your advice to SW?
a) Suggest changing to ER formula
b) Tell him don not be embarrassed

193. If SW accept your advice what should you do


a) Send fax to his doctor with the new medication which you suggest
b) Phone the doctor and tell him about your suggestion
c) Ask SW to see his doctor to discuss with him your suggestion
194. GH has asthma. She lives in a basement apartment. She works as an officer in a company.
Last week she was absent 2 days from work, as she could not sleep due to asthma symptoms.
Her profile includes
 Ventolin® HFA (salbutamol)200 mcg i-ii puff PRN
 Symbicort® Turbohaler (budesonide 200mcg & Formeterol 6 mcg) i puff BID
What is the goal of the treatment in the current situation?
a) Decrease needs for oral prednisone
b) Decrease absence day from work

195. What is your recommendation for SW treatment?


a) Add tiotropium
b) Increase dose of Symbicort
c) Ask her to assess plan for asthma control
d) Go to allergy specialist
e) Alternatively, add LTRA
1 inhalation of SYMBICORT 200 TURBUHALER as needed in response to symptoms to control asthma. If
symptoms persist after a few minutes, an additional inhalation should be taken. Not more than 6
inhalations should be taken on any single occasion. The maximum recommended total daily dose is 8
inhalations.

196. What is the factor that may trigger her asthma?


a) Occupation
b) Pollen
c) Exercise
d) Food
e) Current residence

197. MB was diagnosed with osteoporosis; she also has osteoarthritis in her right knee. Her foot
has broken because something heavy fall on her foot but now she is ok. She works as
hairdresser her work locates 2 km far from home. She takes daily 1 glass of wine daily & smoke
½ packet of cigarette. What is the risk factor for her osteoporosis?
a) Smoking
b) Alcohol
c) Fracture
d) Osteoarthritis
Another version: All are the risk factors for osteoporosis, except:
a) Cervical cancer
b) History of breast cancer
c) Smoking
d) Family history
e) Female gender

198. What is the most appropriate advice for a patient with Osteoporosis who works within a
walking distance from his house?
a) Walk to work
b) Control risk of fall at home
c) Decrease alcohol intake
d) Increase Alcohol
e) Drive to work

199. She has heard that Strontium is beneficial for osteoporosis. Which reference you will use
to find this information
a) Medline
b) Compendium of Pharmaceuticals and Specialties
c) Compendium of therapeutic choices

200. DP had shingles & she suffers from neuropathic pain. Her doctor has prescribed for her
Amitriptyline 10 mg QD. The medication was delivered for her with a written note. When she
read the adverse effects of Amitriptyline, she decided that she will not take it. Which factor that
has a role in of her decision?
a) Lack of communication with the pharmacist

201. What should the pharmacist say when he knows her decision?
b) S.E do not occur with this low dose
c) Common S.E are tolerated & resolved in most patients

202. FR was admitted to the hospital. He suffers from UTI symptoms, which includes fever,
dysuria, frequency and flank pain. He was treated empirically with IV cefotaxime. The culture
showed that the micro-organism is G -ve E-coli. FR has not any allergy. What is the
recommended treatment for FR?
a) Cotrimoxazole
b) Moxifloxacin not renally excreted
c) Norfloxacin
d) Nitrofurantoin
203. What is correct about pyelonephritis in this case
a) Treat for 6 months
b) Treat from 10-14 days
c) Oral is given so that patient can go home but it has relatively less efficacy
Oral therapy is less expensive and decrease stay time in hospital, but has high failure rate
I cannot remember it but it was a right answer 100%

204. A mother came to your pharmacy. His son has pediculosis nit and head lice. He is allergic
to ragweed. What will you recommend for him?
a) Tea tree oil
b) Permethrin cream rinse
c) White vinegar
d) Pyrethrine/Piperonyl butoxide
e) Isopropyl myristate

205. What should you tell her in counselling?


a) All family member should be treated prophylactically
b) In case of treatment failure other medication class may be beneficial
c) Unwashed clothes should be kept in a plastic bag for 30 days, Bed linen should be put in
a soap water for 24 hours

206. OD is in the last stage of breast cancer. Her doctor prescribed for her morphine for pain
control. Her family is worry about the addiction potential of morphine. What should be your
action?
a) Call her doctor to change morphine
b) Tell them that addiction is not a concern in cancer patients

207. Today his son came to the pharmacy asking for early refill for the drug, what is your
action?
a) Ask him about her dose schedule
b) Call his doctor & discuss with him her need for
c) Refuse his request because there is no early refill for narcotics

208. YT is 75 years old male. He has Alzheimer disease many times he was lost in the sub way
station his doctor prescribed for him donepezil 10 mg daily and told his wife that this drug will
benefit his case. What is important to tell the patient about donepezil?
a) It will show effect after 3- 6 months at least
209. His wife asked you “What is the benefits of this drug to her husband”, What will you tell
her?
a) It will increase his score in Mini-Mental State Examination
b) It will decrease his lost times
c) Delaying progression of disease

210. LO is 3 months infant. He was diagnosed with meningitis. The culture showed that
microorganism is G+ ve streptococci. What is the M.O?
a) Staphylococcus aurous
b) Streptococcus Pyogenes
c) Streptococcus pneumonia
d) Neisseria meningitis

211. What is the recommended antibiotic?


a) Gentamicin
b) Amikacin
c) Ceftriaxone

212. 1 hour ago, GF came to the hospital with difficulty in speaking. His heart rate was high &
BP 180/95. He was diagnosed with ischemic stroke. What is the most appropriate initial action?
a) Give Dalteparin
b) Administer Alteplase
c) Decrease his BP
d) Heparin
e) ASA

213. Today he discharged from the hospital with minor disability, which drug he should take to
prevent Stroke?
a) Warfarin life long
b) Clopidogrel for one year and ASA for life
c) ASA
d) Ticlopidine

214. RQ came to your pharmacy with a prescription of hydrocortisone. Her doctor told her that
she has Addison disease. What should you tell her when you dispense this medication for her?
a) You should take Ca & vit D
b) An increase in blood pressure may occur
c) Your doctor prescribed hydrocortisone due to its glucocorticoid effect
d) Limit salt intake as it increases risk of edema caused by this drug
215. If she cannot tolerate hydrocortisone, which one can be used instead
a) Prednisone
b) Dexamethasone
c) Betamethasone
Mineralocorticoids: Fludrocortisone, Hydrocortisone, prednisone responsible for sodium & H2o retention and
used in Addison disease
Glucocorticoids: anti‐inflammatory and all of them have glucocorticoid effect except Fludrocortisone.
Hydrocortisone & Prednisone have mixed effects

216. KU is smoker woman. She came to your pharmacy asking about medication for smoking
cessation. She has tried many times to stop smoking but she failed, furthermore she worries
about gaining weight. All of the following can be used for smoking cessation except:
a) Nicotine patches
b) Bupropion
c) Varenicline
d) Amitriptyline

217. Nortriptyline is approved for smoking cessation for a course of 12 weeks. All of the
following are causes of her failure except?
a) Loss of motivation she is coming by herself
b) High cost
c) Weight gain
d) High craving

218. While LN was travelling in long flight his leg became red and swollen. After arrival, he
admitted to hospital, His creatinine clearance is low. What is recommended treatment for LN?
a) Enoxaparin
b) Heparin
c) ASA

219. Enoxaparin excreted renally so not used if crcl<30ml/min. The treatment can be assessed
by measuring:
a) PT
b) Aptt
c) INR
220. Patient with Dyslipidemia, MD prescribed Statins, patient does not want to use statins.
After a long meaningful discussion, the patient still refuses. What is appropriate?
a) Insist that he takes Statin prescribed.
b) Respect his autonomy.
c) Talk to him about the benefits of Statins over others.
d) Make his wife convince him.

221. Which of the following organisms is a cause of meningitis in a 15-month-old infant? Lab
results identified a Cocci in chains.
a) Streptococcus Pneumonia
b) Nisseria meningitis
c) Haemophilus influenza type B

222. Patient with gangrene in diabetic foot ulcer, which is the causative microorganism?
a) Staph. Aureus
b) Escherichia coli
c) Legionella Sp.

223. What is an appropriate treatment for the above case?


a) IV Ceftriaxone
b) Penicillin
c) Metronidazole
Osteomyelitis☹ Palpation of bone at base of ulcer Treatment of osteomyelitis may require 4–6
S. aureus (MSSA or MRSA) is the most common weeks of parenteral or several months of
pathogen, but other less virulent organisms may be oral antimicrobial therapy.
pathogenic, e.g., coagulase-negative staphylococci. Oral Options
Treat with IV therapy or long-term oral antimicrobial Amoxicillin/clavulanate, Cephalexin,
therapy using agents that are well absorbed from the GI Clindamycin, Cloxacillin, Doxycycline,
tract and have good distribution to bone and tissue Ciprofloxacin or levofloxacin plus
Surgical débridement indicated to remove necrotic debris, clindamycin or metronidazole, Linezolid,
abscess or sequestrum Moxifloxacin, SMX/TMP, SMX/TMP plus
Therapy should base on culture results whenever possible. clindamycin or metronidazole.
If MRSA present / suspected, add vancomycin or Parenteral Options
linezolid. Carbapenem, Clindamycin PO/IV or
If clinical improvement isn’t observed (e.g. resolution of metronidazole PO/IV plus third-generation
erythema, edema, heat, draining sinus, coverage of bone cephalosporin, Daptomycin, Linezolid,
with soft tissue), consult with a specialist. Piperacillin/tazobactam.
224. Patient is looking to buy Plan B. You learn that she was sexually assaulted but she did not
mention it to her MD. What is appropriate?
I. Call police
II. Give her the pill
III. Give her phone number for help
I only III only I & II II & III I, II and III

225. Ways by which Hepatitis B is transmitted include:


a) Sexually
b) Orally
c) Airborne

226. Female patient uses OTC laxative (Glycerine) regularly. She is looking for an additional
laxative to have in hand for “Just in case”. What is most appropriate?
a) Refer to MD
b) Give her laxative
c) Educate her about laxative abuse
d) Educate her about eating disorders
e) Call 911

227. Size of pore in water filter is:


a) 0.33
b) 0.22
c) 0.12
d) 0.44
e) 0.60

228. All of the following can cause sexual dysfunction, EXCEPT:


a) Clonidine  impotence
b) Hydrochlorothiazide
c) ACE

229. Patient is taking 6-Mercaptopurine + Methotrexate, monitor all, EXCEPT:


a) Meylosuppression
b) Infection
c) Constipation
230. A patient is using Methotrexate. How often it has to be monitored?
a) Every week
b) Everyday
c) Every month along with X-ray
d) Every 6 month
e) Every year
Monitor: Baseline CBC, LFTs, albumin, creatinine, hepatitis B and C serology, chest x‐ray; monthly × 3
months, then Q1–3 months. Consider HIV screening in high‐risk patients.

231. Vitamin D is given with calcium for what purpose?


a) Increased absorption
b) Increased resorption
c) Decreased excretion
d) Increased excretion

232. What is NOT a side effect of Clarithromycin?


a) Taste disturbances
b) Phototoxicity
c) Diarrhea
d) Constipation
e) QTc interval prolongation

233. Side effects of medications CAN NOT be found in:


a) Remington
b) Merk
c) USP DI – VOL. 1
From which book you will not get diagnostic results? Remington pharmaceutical

234. Immunization to be given every 10 years are: (more than one correct answer was given)
a) Tetanus
b) Influenza vaccine
c) Hepatitis B vaccine

235. Migrane with aura. Which medication does not cause migrane?
Drug‐induced causes (other than MOH): Alcohol • Cocaine • Methylphenidate • Caffeine withdrawal •
Corticosteroids • Nitrates (nitroglycerin) • SSRI • Oral contraceptives
Drugs associated with intracranial HTN (leading to HA): Tetracycline antibiotics • Trimethoprim‐
sulfamethozaxole • Corticosteroids • Isotretinoin • Tamoxifen
236. All are useful advice for migrane, EXCEPT:
a) Lie in a dark room
b) Avoid loud noise
c) Take a bike ride
d) Get good night sleep

237. Retrospective drug utilization review is:


I. Done after dispensing
II. Done before dispensing
III. Done during research trials
I only III only I and II II and III I, II and III

238. Which Combined oral contraceptive does not cause breakthrough vasomotor symptoms
a) Estrogen
b) Estrogen + Progesterone
c) Estrogen + Progesterone for 3 weeks then stop for 1 week.

239. Goal of treatment in Alzheimer’s disease include:


a) To delay going to an elderly long-term care facility
b) To halt the progression of the disease
c) To reverse the disease

240. Which organism is responsible for cellulitis:


a) Group A streptococcus
b) H. Influenza
c) M. Catarrhalis
d) Mycoplasma

241. All drugs are used to treat the above case, except?
a) Cephalaxin
b) Cloxacillin
c) Norfloxacin
d) Amoxi/clav
e) Clindamycin

242. What is the duration of treatment for cellulitis?


a) 3 days
b) 10 days
c) 21 days
243. DOC for MRSA
a) Linezolide IV
b) Cloxacillin IV
c) Clindamycin IV

244. All are true about Common drug review CDR (part of CADTH), except?
a) Used to help the manufacturers better market their products for better sales
b) Recommends to the provincial drug plans on which drugs should be included in the
formulary
c) To develop policies for adaptation of drugs in formulary by provincial authorities
d) Regulated under CADTH Canadian Agency for Drugs and Technologies in Health

245. LK recent HbA1c value was 10 (high). Patient is on Metformin BID and Glyburide TID.
Sugar reading is high in afternoon and evening, but is low in the morning. What is the most
reasonable intervention?
a) Increase Glyburide evening dose
b) Increase Metformin evening dose
c) Increase both Glyburide and Metformin in the morning
d) Decrease glyburide in morning

246. Jill, a female Lawyer, walks into the pharmacy with a prescription for Atorvastatin. Her
Physician has explained to her that she has “bad cholesterol”. She does not want to use the
medication. Jill has one glass of red wine with supper every day. What is appropriate with
regards to her situation?
a) Ask her what does she know about “bad cholesterol”
b) Advice her to diet and exercise
c) Give her a Fibrate instead

247. What is not a relevant advice for Jill?


a) Stop alcohol
b) Try to exercise
c) Eat healthy

248. Patient previously diagnosed Schizophrenia. All previous psychotic episodes were
successfully managed with Rivastigmine. He has recently experienced his 4th psychotic episode.
What is the most appropriate duration of maintenance treatment for his most recent episode?
a) Take Rivastigmine indefinitely.
b) Take Rivastigmine for at least 5 years.
c) Take Rivastigmine for at least 1 year.
d) Take Rivastigmine for 3 months only.
249. Rivastigmine counselling?
I. Take with food
II. Is used for dementia associated with Parkinson’s disease
III. Is a Cholinesterase inducer
I only I and II III only II and III I, II and III

250. Following are all of the side effects of rivastigmine except?


>10%: headache, dizziness, nausea/vomiting, diarrhea, abdominal pain, anorexia.
<10%: fatigue, insomnia, syncope, dyspepsia, weight loss, UTI, rhinitis. Heart block
(rare), delirium (rare), seizures (rare).

251. Pt with Dementia, given Donpezil 10 mg, got nausea, your action:
a) Give Rivastigmine
b) Give dimenhydrinate
c) Give 5 mg Donpezil
The initial daily dose (5 mg) of donepezil is usually taken at night, but can be taken in the morning if sleep
disturbances occur.

252. Risk factors for dementia include:


a) Age
b) Sex
c) High BMI
d) Smoking

253. Pt. with depression is on antidepressant, Mania being precipitated, what is the proper
action?
a) Give Lithium with the antidepressants
b) Giv Lithium and stop the antidepressants.

254. All of the following are true:


a) Depressive episode requires more time to improve than manic symptom
b) Duration of treatment in the manically depressed is longer for manic phase than it is
for the depressed phase.

255. With regards to Schedule II controlled substances:


a) Pharmacist is required to be available in case a patient need help
b) Pharmacist is required to intervene
c) No need for pharmacist intervention
d) Technician is qualified for counseling
e) A prescription is required
f) Can be sold anywhere

256. A Benzodiazepine prescription with 3 repeats, 60 tablets. (Can’t remember the question
well)
a) Pharmacist can give to patient
b) Pharmacist can give only 20 (first prescription with no refills) can be transferred

257. What’s not indicated in atrial fibrillation as a first measure?


a) Cardioversion
b) Beta Blockers
c) ACEI
d) CCB

258. CARN is Canadian Adverse Reaction Newsletter published by


a) PUB-med
b) ISMP
c) MedEffect
d) CPhA

259. Pt. with CAP, his fever


39, PSI score 130, Bp is
normal, admitted to the
hospital, why dr would give
him IV therapy?
a) He is severely ill
b) PSI Score is high
c) He cannot take oral
therapy

260. Organism suspected to


be in ICU?
a) P. aeruginosa

261. Treatment for the above patient ICU IV therapy what is the empiric treatment
a) B-lactam IV plus macrolide IV
b) Respiratory fluoroquinolone IV.
262. CAP efficiency of therapy can be monitor by
a) Spirometry
b) Symptom resolving
c) Chest X ray
Discharge the patient when the following criteria are met in addition to those above: absence of
complications from the pneumonia (e.g., empyema); absence of complications from comorbid illnesses
(e.g., MI); absence of complications from treatment (e.g., severe adverse drug reactions); physiological
stability as indicated by an oxygen saturation of ≥92% while breathing room air for those who do not have
COPD (for patients with COPD, a return to baseline status is desirable), pulse rate of <100 beats/minute
and respiratory rate ≤24 breaths/minute

263. What provides the most protection against Community Acquired Pneumonia (CAP)?
a) Low ph of saliva
b) Aspiration???
c) Low ph of stomach
d) Closure of epiglottis

264. A patient with morphine sensitivity, what agent cannot be used?


a) Acetaminophen-codeine preparation
b) Meperidine

265. A patient with shingles. What is the most appropriate non-prescription choice?
a) Lidocaine
b) Calamine Lotion

266. All of the following are likely to decrease recurrence of MI, EXCEPT?
a) Nitroglycerine
b) CCBs
c) Metoprolol
d) ACE inhibitors

267. All of the following do not require shaking, Except


a) NTG spray
b) Miacalcin
c) Flovent Diskus
d) Levocabastine
268. Insulin is stored outside of the fridge. It continues to be effective for a maximum period of:
a) 28 days
b) 20 days
c) 7 days

269. Patient with kidney dysfunction. His crcl is 10. What is most likely elevated?
a) Ca++
b) K+
c) PO43-
d) Mg++
In renal failure, acute or chronic, one most commonly sees patients who have a tendency to develop
hypervolemia, hyperkalemia, hyperphosphatemia, hypocalcemia, and bicarbonate deficiency (metabolic
acidosis). Sodium is generally retained, but may appear normal, or hyponatremic, because of dilution from
fluid retention. Following the relief of a urinary tract obstruction, hypovolemia, hyponatremia (true loss of
sodium), hypokalemia, hypocalcemia, hypomagnesemia, and bicarbonate loss are most apt to occur.
Electrolyte imbalances after urinary diversion vary depending on the site of urine diversion.

270. A patient with Addison’s disease was admitted to the ICU for exacerbation. A similar
emergency for the patient was controlled successfully 6 months ago with Dexamethasone. Best
therapeutic choice in ICU to control his condition is:
a) IV Dexamethasone
b) IV Hydrocortisone
c) Oral Prednisone
d) IV Prednisone

271. What is NOT a risk factor for Rheumatoid Arthritis?


a) Age
b) Gender
c) Smoking
d) Genetic factors
e) Coffee consumption
Onset can occur at any age, including childhood, but most frequently starts between the ages of 40 and 50.
Rheumatoid arthritis affects women 3 times more frequently than men.
Non‐modifiable Prognostic Factors: Moderate to high disease activity according to composite measures
(e.g., DAS28) or individual measures (e.g., swollen joint count, high acute phase reactants) Presence of RF
and/or Anti‐CCP, especially at high levels Presence of erosions on radiograph at baseline Increased MBDA
Failure of 2 or more csDMARDs High degree of physical disability at onset
Modifiable Prognostic Factors: Smoking, Sedentary lifestyle, Delay in treatment initiation,
272. Most commonly prescribed class of medication in Canada is:
a) Hormones
b) Cardiovascular drugs
c) Antibiotics
d) Antidepressants

273. Parkinson’s patient recently had his Levodopa/Carbidopa dose increased by his physician.
All of the following should be monitored EXCEPT:
a) Peak dose tremors???
b) Wearing off
c) Sudden muscle freezing
d) Hallucinations
e) Response fluctuations during the day

274. A patient is on Omeprazole. What Calcium preparation is best suitable for this patient?
a) Calcium Carbonate
b) Calcium Citrate
c) Calcium Succinate
d) Calcium Fumarate

275. Which Vitamin is depleted in Steatorrhea?


a) Vitamin K  Decrease absorption of fat ‐soluble vitamins ADEK.
b) Biotin
c) Vitamin C
d) Vitamin B

276. All are proper when counseling about Cacitonin, EXCEPT:


a) Avoid in Shell fish allergy
b) Should be primed when used for the first time
c) Alternate nostrils

277. Donepezil side effects are all EXCEPT:


a) Urinary incontinence???
b) Headache
c) Fatigue
d) Diarrhea
e) Weight loss
278. Side effect of Chemotherapy noticed after 1 - 2 weeks is:
a) Bone marrow toxicity
b) Myelosupression
c) Alopecia

279. A mistake was made when preparing a parenteral Warfarin solution. As a result, 10 X the
required dose was administered. Best course of action to prevent this error from occurring in the
future is:
a) Buy pre-mixed parenteral solution from outside.
b) Use a different strength of stock solution.
c) Stop using Warfarin for the time being.

280. Patient is being treated for Major Depression for the third time. Most appropriate duration
of treatment is:
a) 6 months
b) 1 year
c) Continue medication indefinitely

281. Atorvastatin monitoring includes all, EXCEPT:


a) CK
b) Creatinine Phosphatase
c) Liver function
d) Serum Creatinine

282. Syncope is most likely to occur with:


a) Αlpha Blockers
b) BB
c) ACE inhibitors
d) Diuretics

283. Which of the following is most likely to interact with Cyclosporine?


a) Phenytoin
b) Ranitidine
c) Ramipril
d) Pravastatin
The induction of the cytochrome P‐450 enzyme system by barbiturates, phenytoin and rifampicin markedly
accelerated the elimination of cyclosporine, potentially causing inadequate immunosuppression and acute
rejection.
284. A family of a patient is concerned about their mother taking a high dose of Morphine for
Cancer pain. Most appropriate Pharmacist response is:
a) Morphine is not addictive.
b) Patient can be switched to a less addictive pain killer.
c) Morphine addiction is not a primary concern in cancer pain treatment.
d) Talk to the nurse about this.

285. Son wants to pick up a Morphine refill early for his mother who has Cancer. What is most
appropriate?
a) Do not give him any Morphine.
b) Talk to his mother about this.
c) Call M.D. to discuss the situation.
d) Call M.D. to recommend Fentanyl.

286. Patient has run out of his Percocet. His M.D. is out of town and will be back in two days.
What is most appropriate?
a) Make a walk-in clinic call in a prescription for Percocet.
b) Send him to a walk-in clinic.
c) Lend him two pills until his M.D. is back.
d) Refuse to give him anything.

287. Where do Physician’s salaries come from?


a) Federal & Provincial governments.
b) Primarily from insurance.
c) Hospitals.

288. 10 meq of Ca++ is needed. Available at the pharmacy is 10% of cacl2. How much cacl2
would you need?
Available to you is 120 mg of a 5% Ointment. How much Ointment should you add to that so
you get 20% Ointment?

289. All affects Contraceptives, EXCEPT:


a) Gabapentine
b) Carbamazepine
c) Phneytoin
290. Patient has Hypertension and Angina. Which of the following is most likely to prolong this
patient’s life?
a) Digoxing
b) Metolazone
c) Hydroclorothiazide
d) Carvidolol

291. What to monitor in Triptan treated patients for effectiveness and toxicity (K-type)
a) Esophagitis

292. An I.V. dose of Cotrimoxazole administered every 6 hours gives 80 mg Sulfamethoxazole


and 16 mg Trimethprim. Physician wants to switch the patient to an oral regimen of 200 mg
Sulphamethoxazole and 20 mg Trimpthoprim. How much oral Cotrimoxazole should the
patient receive?
a) 8 ml
b) 4 ml
c) 2 ml
Answer:
0.5 ml Q6hr = 0.5 * 4 = 2 ml /Day
SMT: 80 mg ‐‐‐‐‐‐‐‐ 1 ml X mg ‐‐‐‐‐‐‐‐ 2 ml X= 2*80/1 = 160 mg
Liquid 200 mg ‐‐‐‐‐‐‐‐5 ml 160 mg ‐‐‐‐‐‐‐‐ Y ml Y = 160*5/200 = 4 ml/Day

293. What is NOT a side effect of Ticlopidine?


a) Neutropenia
b) Leukopenia
c) Thrombotic thrombocytopenic purpura (TTP)
d) Prolonged bleeding time

294. A patient with DVT, COPD, Endocarditis in right ventricle. Patient is feeling pain in leg
and edema. What is most likely the cause of his signs and symptoms?
a) COPD exacerbation
b) Left ventricle failure and COPD
c) Left ventricle failure
d) DVT

295. Which of the following medications causes QT prolongation?


a) Sotalol
b) Dimenhydrinate
c) Doxylamine
296. What does Capsaicin cause on application?
a) Burning sensation
b) Discoloration
c) Itching
d) Hair removal

297. A technician selling Codeine to a friend while Pharmacist is busy. What is true:
a) Pharmacist intervention is required for exempted products
b) Manager should get involved due to apparent drug use diversion issues

298. Uncomplicated gonorrhoea?


Cefixime 400 mg once
Ceftriaxone 125 mg IM once indicated for gonorrhea of the pharynx
Ciprofloxacillin 500 mg oral once indicated for gonorrhea of the pharynx
Ofloxacillin 500mg once oral avoid during pregnancy
Levofloxacillin 250 mg oral once” avoid during pregnancy

299. Tachycardia exacerbated by which of the following drug:


a) Metoprolol
b) Nadolol
c) Amoldipine
d) Ramipril

300. which drug cause tachycardia in pt. receiving antiarrthymia drug therapy?
a) Lidocaine
b) Quinidine  toxic level induces ventricular tachycardia.
c) Propafenone
d) Dronedarone

301. Ventricular tachycardia DOC:


a) Verapamil
b) Metoprolol
c) Nadolol

302. Seminar of hypertension what should not be included in the leaflet?


a) Name of the successful candidate
b) Cast to participant
303. Which of the following is consistent with endocarditis?
a) Heart murmur, fever, splenomegaly
b) Fever, heart murmur, left ventricular failure
c) Hypotension, headache, heart murmur
d) Fever. murmur, right sided failure.

304. Ascites patient taking spironolactone, BP 130/85, HR 85 beats /min, what to do?
a) Add Furosemide
b) Give Nacl
c) Give blood

305. A patient using triamterene plus HCTZ, his renal clearance is less than 30ml/min, which
diuretic should
a) Give furosemide
b) Give furosemide + metalozone

306. Clopidogrel side effects, all except:


a) Rash
b) bleeding
c) Diarrhea
d) Purpura

307. Treatment of Methanol toxicity? https://www.ncbi.nlm.nih.gov/books/NBK482121/


a) Ethanol
b) Activated charcoal
c) Ipecac
Treatment options for methanol toxicity include supportive care, fomepizole (Antizole, 4‐Methylpyrazole or
4MP), ethanol, dialysis and theoretically, folate. Fomepizole is the antidote for toxic alcohols, and its
mechanism of action is the inhibition of alcohol dehydrogenase. Ethanol may also be utilized therapeutically
to inhibit alcohol dehydrogenase when fomepizole is unavailable. There are advantages and disadvantages
to either treatment. Fomepizole is more easily dosed, does not cause any inebriation, strongly inhibits
alcohol dehydrogenase, but is fairly expensive. Ethanol is less expensive but is harder to dose accurately,
requires close monitoring of the serum ethanol concentration, and causes inebriation that may necessitate
intensive care monitoring

308. Goal of treatment for rheumatoid arthritis


 Fully control signs and symptoms of the disease, i.e., pain, stiffness and fatigue
 Halt radiographic progression and joint damage
 Maintain physical function and work capacity; and maximize quality of life
 Obtain rapid clinical improvement with a goal of 50% improvement within 3 months and ideally
clinical remission.
 Remission means the absence of disease activity as assessed by a clinician (swollen and tender
joints), patient (global assessment of disease activity) and laboratory results (CRP and/or ESR).

309. A nurse called you at the hospital pharmacy and asked you about the compatibility of
nitroglycerine and heparin parenteral formulation. What is the most appropriate source to get
this information?
a) Micromedex
b) CPS
c) Remington
d) Cochrane library
e) Merck Index
Merck Index contains chemical info on drugs (like pharmacopeia)
If parenteral only........Remington

310. Patient takes 5mg prednisone tab. She is taking 10mg/day for the 1st week then dose
decreased by half tab every week. Calculate total tab required for 4 weeks.
1st week: 10 x 7 = 70 mg 2nd week: 7.5mg x 7 = 52 mg
3rd week: 5mg x 7 days= 35 mg 4th week: 2.5 mg x 7 day = 17.5 mg
Total =70+52+35+17.5 = 174.5 mg / 5mg per Tablet = 35 tablets

311. 80. You are critically appraising a study for a new cancer medication which examined the
drug’s ability to reduce mortality. The study enrolled 1000 patients in total, of which 500 were
randomized to the placebo (control) group & 500 were randomized to study drug (experimental)
group. In the study group, 80 patients died. In the placebo group, 100 patients died.
Control group Experimental group
Died 100 (C) 80 (A)
Lived 400 (D) 420 (B)
Calculate control event rate (CER), experimental event rate (EER), absolute risk reduction,
relative risk, relative risk reduction?
Control Event Rate CER = C/ (C+D) = 100 / (500) = 0.2 = 20 %
Experimental Event Rate EER = A / (A+B) = 80/ (80 + 420 = 80/500 = 0.16 = 16.00%
ARR = CER – EER = 20 % ‐ 16 % = 4 %
RR = EER/CER = 16% / 20% = 0.8 = 80%
RRR = CER – EER / CER = 20% ‐ 16% / 20% = 0.2 = 20%
NNT = 1 / ARR x 100 = 1 / 4 x 100 = 25
OR = (A/B) / (C/D) = (80/420) / (100/400) = 0.1904 / 0.25 = 0.7616
312. Onset of action is the same for 2 drugs, what to monitor?
a) Rate of absorption
b) AUC
c) Cmax

313. You have a Glaucoma eyedrops stock solute of 10% W/V. you are required to dilute it as
1:5 by adding suitable diluent, the total final volume of solution is 5 ml. what is the appropriate
volume of diluents and stock solution you need to make the required solute & what would be
the amount of drug in the final solution?
Answer:
10 % Soln. diluted as 1:5 become 2 %
C1*V1 = C2*V2 2%*5=10%*V2 V2=2%*5/10%=1ml
So, 1 ml stock solution & 4 ml (rest of 5 ml needed) diluent
Final solution: 2% means 2 gm ‐‐‐‐‐‐‐‐ 100 ml X gm ‐‐‐‐‐‐‐ 5 ml
X=5*2 /100 = 0.1 gm = 100 mg

314. Patient is using imiquimod cream (aldara) for anal wart, what you not recommend:
a) Apply at night and wash in the morning
b) Keep it in refrigerator
c) Wash your hand before and after applying
d) Apply it every other night

315. Cancer patient taking Methadone, his doctor is out of town, the available doctor is not
registered to prescribe Methadone, what to do as a pharm.?
a) Do not give him as you are not allowed to
b) Fill prescription expecting doctor will sign it when he returns
c) Call doctor and get verbal prescription
d) Ask available doctor to register himself temporarily to prescribe Methadone

316. A technician took a narcotic, the pharmacy manager suspicion, what will be his Action:
a) Tells the staff to watch him
b) Fire him
c) Put him away from the narcotic dispensing area

317. What should be done if narcotics are stolen from the pharmacy?
a) Report to the office of controlled substances within 10 days
b) Report to RCMP within 10 days
c) Report to college within 3 days
d) Report to health canada
318. Question about medication reconciliation program in hospitals
Medication reconciliation is the process of comparing a patient's medication orders to all of the medications
that the patient has been taking. This reconciliation is done to avoid medication errors such as omissions,
duplications, dosing errors, or drug interactions.

319.Health Canada special access program, what is true?


The SAP authorizes a manufacturer to sell a drug that cannot otherwise be sold or distributed in Canada.
Drugs considered for release by the SAP include pharmaceutical, biologic, and radio‐pharmaceutical
products not approved for sale in Canada.
The manufacturer is the one responsible for pricing the product and decides to sell it or not

320. Tobramycin IV 400 mg/24 hr was given to a patient, peak conc. is > 20mg & rough conc.
required is < 0.5 mg. post dose peak was 28 mg/L.& after 10 hr. was 7 mg/L. find the T1/2 &
the correct dose.
Answer:
Log C = Log Cₒ ‐ (k*t /2.303) Log 7 = Log 28 ‐ (10K /2.303) k=0.14
T1/2= 0.693 / K = 0.693 / 0.14 = 4.99 hr (5 hr.)
400 mg / 24 hr ‐‐‐‐‐‐‐‐ 28 mg/ L X mg / 24 hr ‐‐‐‐‐‐‐ 20 mg / L X = 20*400 / 28 = 285.7 mg
285.7 mg ‐‐‐‐‐‐‐ 24 hr 400 mg ‐‐‐‐‐‐‐‐ Y Y = 400*24/285.7 = 33.6 hr.

321. 4-year-old child rhinorrhea, nasal congestion, what to give?


a) Only acetaminophen/ibuprofen
b) Nasal saline drops
c) Oxymetazoline mist

322. MDD person with suicidal tendency, which drug to be avoided?


a) Venlafaxine
b) Sertraline
c) Clomipramine
Rigorous clinical monitoring for suicidal ideation or other indicators of potential for suicidal behaviour is
advised in patients of all ages. The risk of suicide attempt must be considered, especially in depressed
patients; the smallest quantity of drug, consistent with good patient management, should be provided to
reduce the risk of overdose with this drug.

323. Pt. Taking Ciprofloxacin for 7 days for E. coli isolated from his urine, although he was
asymptomatic. Then he took Ceftriaxone/Metronidazole for his pain [planned for 24 days]. He
got C. difficille diarrhea [Pseudomembranous colitis]. Why this problem happened?
a) A wrong drug for valid indication
b) Drug has no indication
c) Drug - drug interaction
d) Drug - food interaction
324. What not to monitor: [ i.e., Monitor all except]
a) C. difficille cytotoxin weekly assay
b) No fever on 3-4 days of antibiotic ttt
c) No abdominal pain on 3 - 4 days
d) Improvement in 5-7 days from diarrhea

325. You have compound A which is 20% w/w salicylic acid, you also have compound B which
is 10% w/v salicylic acid. You want to prepare a 2% salicylic acid solution in which you end up
using 1% of each of the above compounds (A and B). You will use 5ml of compound a and
10ml of compound B -4ml if compound A and 8ml of compound B
a) 8 mg of A &16 ml of B
b) 7 mg of A & l4 ml of B
c) 6 mg of A & 12 ml of B
d) 5 mg of A & 10 ml of B
e) 3 mg of A & 5 ml of B
Answer:
Compound A: 20 mg ‐‐‐‐‐‐‐‐ 100 mg 1 mg ‐‐‐‐‐‐ X mg X= 1*100/20 = 5 mg of compund A Compound
B: 10 mg ‐‐‐‐‐‐‐‐ 100 ml 1 mg ‐‐‐‐‐‐ Y ml Y = 1*100/10 =10 ml of Compound B

326. What is covered by the government?


a) Hospital lab tests
b) Emergency ambulance equipments

327. Patient with face psoriasis, which is true?


a) Take calcium as it helps
b) Avoid emollient on the face
Hydrocortisone 1% cream may be sufficient to control psoriasis of the face as long as it is applied regularly.
At first, applications should be in the morning and at bedtime. As improvement occurs, decrease the
frequency of application to bedtime only and eventually to every 2–3 nights.
In more severe or resistant facial psoriasis, stronger topical corticosteroids may be used on nonresponsive
areas or for acute flares. Avoid long‐term use (several months) of medium‐ or higher‐strength corticosteroids
on the face as these products have been associated with skin thinning, striae, telangiectasias, purpura & acne
Topical calcineurin inhibitors such as tacrolimus and pimecrolimus are approved for the treatment of atopic
dermatitis but not psoriasis. However, these agents have been used in patients with facial psoriasis since
they are not associated with cutaneous atrophy. Available evidence to date does not support concerns
about long‐term risk of malignancy with these agents. Use the minimum amount of these medications for
the shortest time necessary.
328. Pt. On Simvastatin 20 mg, the dr prescribe Fenofibrate 100 mg for control, he got
Muscle pain, the pharmacist told him to stop ttt 4 then to?
a) Give Simvastatin and Niacin
b) Dose titration of Simvastatin
c) Give Ezetimibe only

329. Prochloroperazine is prescribed for a cancer pateint for delayed nausea, but she is afraid
from the funny effects, what to tell her:
a) Nausea is a concern for you
b) Dr sees it will work for you
c) This side effect doesn’t probably happen
d) There are other ttt options
Prochlorperazine side effects: Sedation, anticholinergic effects (dry mouth, blurred vision, constipation,
nasal congestion, urinary retention), extrapyramidal effects, hypotension, hypersensitivity, pancytopenia
(rare).

330. For new drug, if the manufacturer wants to protect his invention, he must ask for?
a) Patent
b) Brand name
c) Trade mark
d) Copy right

331. You saw [for the second time] a mother threating her child worthlessly, what is your
action?
a) Encounter, document for the future episodes
b) Bask her, is everything ok
c) Call her husband
d) Ask the child
e) This is not a concern for you, it is a discipline matter

332. Where you can find the validity of use of antidepressant in pregnancy?
a) Product monograph
b) Mother risk program
c) Briggs

333. Probation crime, the probe officer called you to know the pt. Medications, because
they need to do urine tests for drugs, what is y action:
a) Give him, they are a part of his care
b) Donn’t release without consent
334. Contraindicated in sulfonamide allergy:
a) Captopril
b) Silver sulfadiazine

335. A case of GERD, for what reason you refer to dr.:


a) A dyspepsia interferes with sleep
b) Melena

336. Used for diagnosis of cirrohosis:


a) Alanine transferase
b) Lactate dehydrogenase

337. The toxic dose of acetaminophen?


a) 2 gm
b) 4 gm
c) 7 gm
Max. Tolerated dose 4 gm, above 4 to 6 (7.5 gm) →hepatotoxicity, while 10 gm is fatal. The anti‐dot is N‐
acetylcysteine & symptoms as acute hepatic necrosis.

338. Diagnostic liver test for acetaminophen toxicity?


Obtain liver function tests (LFTs). Aspartate aminotransferase (AST) and alanine aminotransferase (ALT)
concentrations begin to rise within 24 hours after an acute ingestion and peak at about 72 hours. In
severe overdose, transaminase elevation can be detected as early as 12‐16 hours post‐ingestion.

339. A divorced man comes to you, he lost his job & his insurance coverage, he is asthma pt 20
years ago, he wants to dispense his son’s puffer for himself instead of his son. What shall You do?
a) Refuse to dispense except for the son only'
b) Call the mother to take her permission
c) Dispense only this time
d) Give him a sample you were offered from physician '
e) Call the wife

340. A case of pt has headache, migraine, asthma, coming for refill of salbutamol. His profile
shows: Amitriptyline 1, Tylenol 3, Salbutmol [2 wks ago], Nicotine gum. What is your
conclusion?
a) Drug for no indicaton
b) Indication and no drug??? May need Cs
c) Drug-drug interaction
Amytriptyline with codeine  increase your risk of becoming very drowsy and having breathing problems
341. Teenager comes to your pharmacy with severe, inflamed acne, open comedone and closed
comedones which drug can be used?
a) Glycolic acid 2%  Available without prescription.
b) Azaleic acid 15%  In Canada it is not officially indicated for use in acne.
c) Isotretinoin  for severe acne
d) Clindamycin 1%  Avoid using as monotherapy to limit bacterial resistance.

342. What kind of monitoring is required before initiating isotretinoin therapy?


a) Liver tests
b) Heart
c) Kidney
d) Psychiatric
Check for depression. Check triglyceride, pregnancy, liver function…need to check

343. What would u recommend the patient to use regularly in the above case???
a) Lipbalm  coz of photosensitivity s/e.
b) Face wash

344. 48 years old teacher with parkinsonism, she was embarrassed as she falls many times and
now, she is working part time instead of full time: What is your advice for this patient
a) Start treatment to avoid delaying of disease progression
b) Do not start treatment because of the side effects of the medication
c) Start the treatment when quality of life is affected or When symptoms interfere with
life activities
d) Change her work

345. What is the drug suitable to start?


a) L-dopa
b) Amantadine
c) Entacapone
d) Trihexphenidyl
e) Pramipexole

346. When To check HbA1C for type 2 DM pregnant woman?


a) 3 months
b) 1 month
c) 6 months
347. Which is not taken in opioid naive people:
a) Fentanyl injection
b) Fentanyl patch
c) Hydrocodone extended release
d) Oxycodone extended release
a) Morphine

348. Auxialary label for Sucralfate:


a) Causes drowsiness
b) Do not crush the tablet
c) On empty stomach, half hour before food
d) Avoid in pregnancy and pt under 18

349. Pt. On Catbamazepine, prescribed Etythromycin, the best action:


a) Give clarithromycin
b) Give azithromycin
c) Replace carbamazepine with phenytoin
d) No problem

350. Where to find unapproved indication of drug?


a) Martindale
b) AHFS
c) RxFiles

351. What is not associated with Iron deficiency?


a) Hemoglobin
b) Hematocrit
c) Macrocytic anemia
d) Serum ferritin

352. Where to search for most recent evidence-based medicine:


a) Drug Product Database.
b) Systemic review of randomized trials
c) Cochrane library
d) Pharmacy letter
353. A study has been conducted on 2 groups of smokers from 1985 till 1997. Somo other
Scenario written then the following question What type of study is that?
a) Cohort Study
b) Case control
c) Delphi method

354. To protect the baby from sunburn, all are true except?
a) Do outdoor activity in cloudy weather
b) Use sunscrean at least spf 30
c) Avoid going out in the sun from 10 am to 4 pm

355. (SD) in a mental health care center to keep a first aid box (for kids) all of the following you
will recommend except?
a) Prescription of lpecac  to induce vomiting incase of toxicity
b) Label on the bottle of the alcohol to rub for disinfection
Why we put in first aid box alc bottle for disinfection, we put only alc pads, it’s safer also.

356. A lady came to pharmacy, she does not believe in allopathic medicine and has faith on
herbal product, which of the following is incorrect
a) Echinacea (cold/cough)
b) Cranberry for UTI
c) St. Johnswort for antipsychotic
d) Prune for constipation
e) Ginsing for BPH  It is saw palmetto

357. Patient has seizures, Asthma, & Influenza. what to give him to treat Influenza?
a) Influenza vaccine
b) Oseltamivir
c) Zanamivir
d) Amantadine
e) Atazanavir ꞏ
Influenza vaccine (for prophylaxis, not for treatment)
Zanamivir (Cl in Asthma because it is available only as an inhaler)
Amantadine (Cl in seizures). Atazanavir (not for influenza)

358. Question on erectile dysfunction (scenario)


a) Sildenafil
b) Verdinafil
c) Ginseng
359. All are side effects of Plan B except?
a) Nausea
b) Depression
c) Mood disorder
d) Breast tenderness
e) Galactorrhoea

360. P & T committee members


Pharmacy and Therapeutics Committee (P&T)
 A sub‐committee of Medical Advisory Committee MAC
 Formal line of communication between the medical staff and the pharmacy department of
hospital. Acts as link between pharmacy department and other disciplines and health care
providers.
 Mission is to promote safe, rational & cost‐effective distribution, utilization and administration of
drugs.
 Usually composed of at least 3 physicians, a pharmacist, a representative of nursing staffs, quality
improvement managers and administrators.
PTC’s Mission:
Provide safe, rational, and cost‐effective prescribing, distribution and administration of drugs and
therapeutic agents within the hospital setting.

361. If you prepare this by adding neutral base (petrolatum), which method will you apply for
compounding?
a) Levigation
b) Geometric dilution
c) Trituration

362. Which is/are oleaginous soft base? K type,


a) Eucerine
b) Soft petrolatum
Hydrocarbon or Oleaginous Bases: Hydrocarbon bases neither contain nor absorb water. These bases are
used for their emollient effect and are occlusive. They are non‐water washable. These bases are chemically
inert and may be useful in preparing ointments containing water‐insoluble or water‐incompatible drugs.
There are three major types of ointment bases: oleaginous (greasy) bases, which are water repellent; water‐
absorbing bases, which are greasy but allow absorption of water, and water‐miscible bases (vanishing
creams or water‐soluble bases).
Petrolatum (which may be stiffened with wax) is the most widely used greasy ointment base and is suitable
for the incorporation of oleaginous materials.
363. kind of milk for 1-year child

Type Uses
Iron-fortified infant an acceptable alternative until 9–12 months of age If the mother chooses not to
formulas breastfeed exclusively
Cow's Milk–Based The most commonly used substitution or supplement to breast milk.
Formulas
Lactose-Free Cow's Used if Parents suspect that symptoms of gassiness or fussiness in infancy related to
Milk–Based lactose or after about of infant diarrhea where there may be a temporary, 2ry
Formulas disaccharidase deficiency.
Lactose-free formulas may contain residual amounts of lactose and galactose, which
makes them an inappropriate choice for infants with galactosemia.
Soy Protein Isolate– Free of cow's milk protein and lactose. They are iron-fortified and are designed to
Based Formulas meet the nutritional needs of term-born infants.
They are recommended for term infants with galactosemia or congenital lactase
deficiency.
They can be used as a supplement to breastfeeding for infants of mothers who follow
vegetarian diet or for infants whose mothers wish to feed them nonanimal protein-
based formula.
They are not recommended for infants with cow's milk protein–induced enteropathy
or enterocolitis, as 30–60% will also be sensitive to soy. However, infants who have
an immunoglobulin E–associated reaction to cow's milk protein may tolerate soy
formulas.
Partially hydrolyzed The perceived benefits of partially hydrolyzed whey formulas, such as fewer spitting-
whey formulas up episodes and softer stools, are likely related to the beta-lactoglobulin (which
remains soluble in the stomach) moving faster to the upper jejunum.
These benefits may be more pronounced in children with underlying GERD.
Extensively They are recommended for infants with intolerance to intact cow's milk protein
hydrolyzed protein– and soy protein.
containing formulas Extensively hydrolyzed formulations benefit infants with malabsorptive diseases
such as short bowel syndrome, liver disease, cystic fibrosis and intractable diarrhea.
Infants with cholestasis and lymphangiectasia also benefit from extensively
hydrolyzed formulations, especially those containing a higher percentage of medium-
chain triglycerides.
Amino Acid–Based They are designed for infants with severe milk protein hypersensitivity and are also
Formulas used for infants with malabsorption-associated diseases who have persistent
symptoms when receiving a partially hydrolyzed formula.
Pre-thickened Commercially available pre-thickened formulas for treatment of regurgitation and
Formulas vomiting.
Formulas for These formulas are designed to meet the accelerated growth needs of infants born
Premature Infants prematurely.
Follow-up Formulas Designed for infants between 6 and 24 months of age who are consuming some
complementary foods.
364. vaccine take every year
a) Pneumococcal vaccine
b) Influeza vaccine
c) Tetanus vaccine
d) Dukoral vaccine

365. The main goal for randomized trials is?


a) Decrease the bias in allocation of treatment group

366. In a hospital setting, which of the following is not considered as a reason for drug errors?
a) Checking patient compliance
b) The rx stage
c) The dispensing stage
d) Measuring the dose by nurse

367. During sterilization process and aseptic technique, which of the following is considered as
a source of contamination?
a) Person working close to the hood

368. It’s important to document error in the pharmacy due to all of the following except?
a) For patient interest
b) Do not depend on staff cognitive function
c) Helping in reimbursement from insurance companies

369. A parole officer calls you to ask about the medication that one of the prisoners. Taking that
would help in criminal investigation. Host should you do in this case?
a) Don’t disclose any information except when you get a consent from the Prisoner.
b) The parole officer is in the center of heath care and you can give him the Information he
needs.
c) Because the patient is a prisoner; he lost his confidentiality rights and you can disclose
the information to parole officer

370. Pt had H. pylori, he came to return Metronidazole, Omeprazole, clarithromycin after 3


days of use due to S/E, what is true?
a) It is legal to return
b) lt is illegal to return or give credit
c) It is illegal to return it but you can give credit
371. A pharmacy adopting a new system of care for the patients. All can be done except:
a) Offer an incentive for the patients who use this service

372. A pharmacist wants to adopt new service for geriatric home. What is the most appropriate
step he should begin with?
a) Make a survey about the geriatric homes around the area

373. What is the best reason for a manager to stock a new released prescription drug in a
community Pharmacy?
a) Stock it according to local demand for this medication

374. All should follow the Narcotic Destruction Regulation EXCEPT


a) Ketamine
b) Pentazocine
c) Nabilone
d) Olanzapine

375. Two drugs A and B. Given the total body clearance values, renal clearance, many other
values. Which drug has a higher first path metoholism?
A) Oral form
Drug A The same question is in 2010s example from questions in the
PEBC'website

376. Canada health act includes all except:


a) Sustainability
b) Comprehensiveness
c) Portability
d) Accessibility

377. What is considered proper action if a pharmacist is required to prepare a pharmaceutical


preparation and didn't find enough quantity of a certain ingredient?
a) Send to a nearby institution to get the required quantity
b) Tell the patient that it is not available and advice to come tomorrow
c) Prepare the preparation with the available & continue in the second day when the new
order comes
378. In influenza out break, it is required to make a brochure for 1ry health care professional
what information is most appropriate to include?
a) Clinical guidelines for treatment of symptomatic patients

379. You are manager in pharmacy and you decided to change the computer system. Which of
the following steps is the most important?
a) The stakeholder’s intention to change it
b) Identify the defects in your computer system
c) The financial means availability
d) Try new computer software

380. What to monitor daily in CHF except?


a) Heart rate
b) BP
c) Weight
d) Potassium
e) Dyspena

381. How do you recognize morphine addiction from the Rx?


a) Morphine long acting is Rx ed for the patient
b) Morphine long acting is Rxed for the patient together with morphine immediate release
c) Morphine is Rxed for the same patient by the same MD every time
d) LA morphine is Rxed with multiple part fills predated in the Rx

382. To find information about stroke?


a) PSC
b) Primary reference
c) Heart and stroke foundation

383. Question on altiplase occlusion criteria

384. Which preparation doesn't contain preservatives:


a) Total Parenteral Nutrition
b) Eye drops
c) Ear drops
d) Nasal spray
385. Which of the following for hypercalcemia?
a) Pamidronate
b) Calcitonin
c) Ca chloride

386. Ginko biloba and INR effect?


Ginko Biloba is used for: Enhancing memory
Specific natural medicines reported to affect warfarin sodium therapy include the following:
 Bromelains, danshen, dong quai (Angelica sinensis), garlic, and Ginkgo biloba, ginseng, and
cranberry products are associated most often with an INCREASE in the effects of warfarin sodium.
However, the effects of ginseng can be variable (increased or decreased effect of warfarin sodium)
and the combination should be avoided or more careful monitoring is warranted.
 Coenzyme Q10 (ubidecarenome) and St. John’s wort are associated most often with a DECREASE in
the effects of warfarin sodium.

387. pt colored yellow green diarrhea, what to do?


a) Give advil
b) Refer to doctor

388. Vitamin D3 dosage form?


Osteoporosis Canada recommends routine vitamin D for all Canadian adults year‐round:
 Adults 19–50 y (including pregnant or breastfeeding women): 400–1000 units daily.
 Adults >50 y and those at high risk: 800–2000 units daily
Obese, limited sun exposure, non‐European populations: 2000 units/day
Canadian Cancer Society:
 Adults during fall and winter months: 1000 units/day
 Adults ≥ 50 y, dark skin or little sun exposure: 1000 units/day all year round
Canadian Pediatric Society:
 Pregnancy and breastfeeding: consider 2000 units/day, especially during winter months.
Maintenance range: 400–2000 units/day.
For severe deficiency (levels <25 nmol/L), consider bolus dosing followed by maintenance: 2000–4000 units
of D3 daily for 8–20 wk or 600 000 units of D2 over 8 wk, e.g., 50 000 units weekly for 8–12 wk
Appropriate vitamin D levels may improve dietary absorption of calcium.

389. Which of the following interacts with Linezolid?


a) Moclobemide (Serotonin Syndrome)
b) Amitryptiline
c) Amlodipine
d) Nitrazepam
MAY 2010
1. Mr JP is one of your regular patients came to your pharmacy with 3 unused medicines. He
purchased these medicines 2 months ago. He wants to return it to the pharmacist for destruction.
Mr LP, another patient of you, overheard the conversation between you and JP. He saw that the
same medicines he was using last few months. Now he lost his job and he does not have the
insurance coverage for buying his medicines. LP requested you to give him these medicines.
What is the appropriate action you should take?
a) Give him these medicines, as he is a regular customer of you and he cannot buy
medicines for him.
b) Tell him that you do not know the storage condition of these medicines for last few
weeks and these may harm him.
c) Tell him that it is illegal to dispense these medicines to him.

2. A drug is given at the dose of 1g IV Q12H to a patient. The desired therapeutic level of the
drug in the blood is 15 – 20 mg/L. When blood drawn just before the next dose, the
concentration was found 10.57 mg/L. What is the appropriate dose of the drug so that the
desired therapeutic level is maintained?
a) 1125 mg IV Q12H
b) 1250 mg IV Q12H
c) 1500 mg IV Q12H
d) 1750 mg IV Q12H
e) 2500 mg IV Q12H

3. You have a stock solute of 10% W/V you are required to dilute it as 1:5 by adding suitable
diluent, the total final volume of solution is 5 ml. what is the appropriate volume of diluents and
stock solution you need to make the required solute & what would be the amount of drug in the
final solution?
a) 4 ml diluents and 1 ml stock solution
b) 4.5 ml diluents and 0.5 ml stock solution
c) 4.9 ml diluents and o.1 ml stock solution
Answer:
10 % Soln. diluted as 1:5 become 2 % C1*V1 = C2*V2 2%*5=10%*V2
V2=2%*5/10%=1ml
So, 1 ml stock solution & 4 ml (rest of 5 ml needed) diluent
4. In the above question, what would the amount of drug in the final 5ml solution?
a) 10 mg
b) 100 mg
c) 200 mg
2% means 2 gm ‐‐‐‐‐‐‐‐ 100 ml X gm ‐‐‐‐‐‐‐ 5 ml X=5*2 /100 = 0.1 gm = 100 mg

5. Tobramycin iv 400mg/24h was given to a patient. Peak concentration is required is >20mg


and trough concentration required is <0.5 mg. Post dose peak is 28 mg/L and after 10 hours is 7
mg/L. Find the T1/2 of thedrug.
a) 5 hours
b) 9 hours
c) 10 hours
d) 15 hours
Solution: ln C – ln Co = Kt ln 28 ln 7 = K x 10 3.33 – 1.94 = 10 K
K = 0.139 So t1/2 = 0.693/0.139= 5 hours

6. In the above patient what is the correct dose?


a) 400 mg/24 hours
b) 400 mg/36 hours
c) 400 mg/48 hours
d) 400 mg/72 hours
Tips for solution: Drug concn after 5 h = 200mg Drug concn after 10h = 100mg
Drug concn after 50 h = 0.39mg which is < 0.5mg i.e., trough concn
JN comment: use conc not dose i.e 28 14 7 3.5 1.75 0.9 0.45

7. Tobramycin, all should be monitored except?


a) Creatinine clearance
b) Liver function tests
c) Serum levels
d) Eighth-nerve function

8. A physician prescribed potassium supplements 20 m. Eq TID to patient but this patient


does not want to take supplements. He asks the physician if he can eat bananas instead. If each
large banana has 602 potassium. How many bananas should the patient eat each day?
(M. wt K = 39).
Answer:
20 m. Eq TID = 60 m. Eq per day m. Eq = M. Wt * Valency = 39 * 1 = 39 mg Each day = 60mEq = 60*39=
2340 mg
Each banana contains 602 mg No. of banana = 2340/6002 = 3.88
9. Which drug is used to treat Anticipatory Nausea & Vomiting in chemotherapy:
a) Dimenhydrinate
b) Ondansetron
c) Metoclopramide
d) Lorazepam
e) Steroids

10. What is not true about ondansetron


a) Cause diarrhea
b) Cause constipation
c) Tachycardia
d) QTc prolongation
S.E: constipation and headache the most common. Bradycardia, diarrhea, dizziness, QTc interval
prolongation, sedation, transient increase in laboratory values of LFTs.

11. Which on is used in scabies treatment?


a) Lindane 1%
b) Permethrin 5%
c) Crotamiton 5 %  should be 10%
d) Sulpher 7 %  should be 10%

12. Pharmacist makes a meeting to discuss with the pt the advantages of statins vs risk. By
doing this meeting, the pharmacist upholds which ethical principle?
a) Paternalism
b) Veracity
c) Autonomy
d) Beneficience
e) Non-maleficience

13. Which is photosensetive?


a) Tetracyclin
b) Isotritinoin
c) Phenazopyridin

14. Patient takes 5mg prednisone tab. She is taking 10mg/day for the 1st week then dose
decreased by half tab every week. Calculate total tab required
Answer:
1st week: 10 x 7 = 70 mg 2nd week: 7.5mg x 7 = 52 mg
3rd week: 5mg x 7 days= 35 mg 4th week: 2.5 mg x 7 day = 17.5 mg
Total =70+52+35+17.5 = 174.5 mg / 5mg per Tablet = 35 tablets

15. Addison disease patient had diabetes admitted hospital for ketoacidosis, all these symptoms
can be experienced by addison’s pt except?
http://www.addisonsociety.ca/what-is-addisons-disease.html
Addison's disease develops when the adrenal glands, which are above the kidneys, are not able to make
enough of the hormones cortisol and, sometimes, aldosterone.
Symptoms of Addison's Disease: salt craving, weakness and fatigue, hyperpigmentation of skin and mucous
membranes, weight loss, nausea, vomiting, constipation or diarrhea, low blood pressure, abdominal pain,
lethargy, confusion, fainting, particularly on standing up & muscle complaints

16. What to give for the treatment of Addison's disease in hospital


a) IV hydrocortisone + low dose insulin
b) Prednisone
c) Dexamethasone
Addison’s Disease (decreased cortisol) treated by: Hydrocortisone IV or Cortisone IV
Both are partial mineralocorticoid partial glucocorticoids

17. After discharge from the hospital he should continue using:


a) Prednisone oral
b) Dexamethasone

18. What will you tell this patient regarding his medication?
a) This medication may increase your blood pressure.
b) Restrict Na intake, it could make fluid retention.
c) Dr. Prescribed this medication for you because of its glucocorticoid activity.
d) Take Ca and vit D with corticosteroids, it could cause osteoporosis.
All except 3, Glucocorticoids

19. EK is a 25-year-old female who presents to the pharmacy requesting (levonorgestrel) Plan
B® for emergency contraception following an episode of unprotected sex 12 hours ago. After
speaking with EK, the pharmacist decides she is a good candidate to receive Plan B®. The
pharmacist should include all of the following information in counseling EK, EXCEPT:
a) The effectiveness of Plan B® in EK’s situation is likely to be better than 90%.
b) Plan B® works by dislodging an implanted fertilized egg from the endometrium.
c) EK should take two tablets of Plan B® as a single dose.
d) Side effects may include nausea, vomiting, breast tenderness, cramps and spotting.
e) The total dosage for one complete regimen is 1.50 mg
20. What first thing u ask when some one need plan B
a) How many partners do you have?
b) Did you take oral contraceptive before?
c) When was your last menses?

21. What is contraindication for plan B


a) Abnormal vaginal bleeding
b) Pregnancy or suspected pregnancy
c) Hypertension
d) Angina

22. All are used for acute gout attack except


a) Colchicine
b) Naproxen
c) Sulphinpyrazon

23. pt has crohns disease taking cortisone and methotrexate but no effect, what to give
a) Mesalamine
b) Infliximab
c) Sucralfate

24. The followings are the initial goal of therapy EXCEPT.


a) Cure the disease
b) Prevent surgery
c) Avoid use of steroids
d) Symptoms remission & Prevent recurrence
e) Improve abdominal pain and diarrhea

25. With Infliximab, what should be monitored during the first 2 hours during infusion?
a) FBC (Full Blood Count) FBC & LFTs done with Methotrexate and Leflunomide
b) LFTs (Liver Function Tests)
c) WCC (White Cell Count)
d) Chest pain, fever, hypotension
e) Visual symptoms and assessing for blurred vision

26. Pt have fever and stiff neck diagnosed with meningitis what DOC
a) Ceftriaxon and vancomycin
b) Gentamycin
27. Lady travelling to india she wants to make antibiotic prophylaxis for traveller diarrhea. Last
year she tried cotrimoxazol and no effect, what to give?
a) Ciprofloxacin
b) Norfloxacin
c) Azithromycin
Drug of choice for southeast Asia and Indian subcontinent.

28. P.J. is a 35-year-old female who is placed on Lithium therapy. The suggested dose is 600mg
q8h of lithium carbonate. The total body elearance of lithium is 0.44mL/s or 1.621 L/h. The
biogical half life is 18 hours. The molecular weight of lithium carbonate is 74. The number of
m m o l t h e p a t i e n t s w i l l r e c e i v e p e r d o s e o f l i t h i u m c a r b o n a t e ( Li ) w i l l b e :
a) 8
b) 16
c) 24
d) 32
Dose = 600mg q8h = 1800mg / day 1 mole = 74 gm 1 mmole = 0.074 gm
0.074gm present in 1 mmol So, 0.6g (600mg) will be = 0.6 / 0.074 = 8.11 mmol

29. The number of mEq per dose the patient will receive is:
a) 8
b) 16
c) 24
d) 32
1 mole = 74gm Eq Wt = 74/2 = 37 1 mEq = 37/1000 = 0.037gm = 37mg
37mg present in 1 mEq So, 600mg will be = 600 / 37 = 16.22 mEq

30. How long will it take to reach 94 % of steady state?


a) 18 hours
b) 1 day
c) 3days
d) 5 days
Css is 99% = t1/2 * 6.65 = 18 x 6.65 =119.7 hrs = 4.98/ day
Css is 95% = t1/2 * 4.32 = 18 x 4.32 = 77.76hrs = 3.24/day
Css is 90% = t1/2 * 3.32 = 18 x 3.32 = 59.76hrs = 2.49/day

31. The lithium does is now changed to 300mg q8h. How long will it take lithium to reach 94%
of steady state?
a) Immediately
b) 1 day
c) 1.5 days
d) 3 days
e) 5 days

32. New drug available in the market, where can u fine the info?
a) e-CPS
b) Drug fact and comparision
c) Medline
d) RxTx

33. A pregnant patient comes to your pharmacy with nausea. She is using diclectin (doxylamine
+ pyridoxine), but her nausea is not controlled. She asks you whether she can use
dimenhydrinate along with diclectin. Your response will be
a) It is contraindicated with Diclectin.
b) Not enough data is available for this combination of drugs.
c) Yes, you can use it and it is safe to use both together
d) It can be used intermittently, not regularly
Since dimenhydrinateis recommended by Canada's Motherisk program for augmenting doxylamine
/pyridoxine (Diclectin) therapy in pregnancy and it is considered generally safe in pregnancy, it can be
considered for intermittent therapy of motion sickness in pregnant patients. Promethazine may also be
used.

34. Which of the following product is not chemically related to opioids?


a) Codeine
b) Tramadol
c) Heroin

35. A woman has diabetes and hypertension. Now, she became pregnant. Which of the
following drug should be discontinue 1st?
a) Ramipril
b) Ranitidine
c) Atenolol
d) Omeprazole
e) Insulin
ACEi are teratogenic and contraindicated in pregnancy

36. Which of the following drugs require a written prescription?


a) Ketamine
b) Diazepam
c) Zopiclone
d) Lorazepam
e) Ketorolac

37. Which of the following products require sales record?


a) Tylenol # 3
b) Ketamine
c) Phenobarbitol
d) Diazepam
e) Nalbuphine

38. Canadian Health Act includes all except?


a) Accessibility
b) Portability
c) Universality
d) Affordability
e) Comprehensiveness

39. A patient presented a prescription of prochlorperazine for nausea & vomiting. After reading
the leaflet, the patient decided not to take the medication. What is the right course of action of
the pharmacist for this patient?
a) Tell patient that physician has written this medication, which is best for you
b) Offer the alternative mediation, which has less side effect
c) Call the physician to change the drug

40. Doctor prescribed ferrous sulfate 20 mg/kg/day for a patient weighing 20 lbs. You have
ferrous sulfate 300 mg/5ml solution in stock. How much ml stock solution you need?
Answer:
20 lbs = 9.09 kg 9.09 kg x 20mg/kg/day = 181.82mg / day = 3.03 ml = 3 ml

41. Which opioid has derived from morphine?


a) Fentanyl
b) Codeine
c) Oxycodone
d) Meperidine

42. What is APACHE score?


APACHE II ("Acute Physiology and Chronic Health Evaluation II") is a severity of
disease classification system (Knaus et al., 1985), one of several ICU scoring systems.
After admission of a patient to an intensive care unit, an integer score from 0 to 71 is
computed based on several measurements; higher scores imply a more severe disease and
a higher risk of death. Gcs for consciousness level.

43. A 37 years old asthma patient is using Salbutamol inhaler prn. During last month, the use of
salbutamol has increased than before. During last 2 weeks, the patient has used 4/5-time daily
prn. His doctor prescribed him inhaled corticosteroid. However, the patient does not want to use
inhaled corticosteroid. He asked you any other alternative. What is the most appropriate
medicine for him?
a) Salmeterol
b) Montelukast
c) Tiotropium
d) Oral corticosteroid  only in exacerbation of asthma
e) Ipratropium

44. Patient is on Zidovudine. Which of the following you will monitor?


a) Lipase
b) Creatinine
c) Potassium
d) CBC
‫ﺯﻳﺪﺍﻥ ﺑﻴﻠﻌﺐ ﻣﺎﺗﺶ ﺑﻴﺘﺬﺍﻉ ﻋﻠﻰ ﻗﻨﺎﺓ ﺳﻰ ﺑﻰ ﺳﻰ‬

45. A capsule contains 20 mcg/400 units. Doctor prescribed 1.25 mg and patients need 100,000
units in total. How many capsules he needs?
a) 2
b) 4
c) 8
d) 10

46. A clinical trial report shows that a control drug group 338 patients out of 9332 patients died
and in test drug group 320 patients out of 9650 patients died. Find the relative risk reduction?
Solution:
338/9332*100 = 3.62% 320/9650*100 = 3.32%
RRR = (3.62 – 3.32) / 3.62 * 100 = 8.29% Ans. 8.3%

47. In a community, the pharmacist arranged a seminar on medication error. All of the
following personnel are invited except?
a) Physician
b) Patients
c) Technicians
d) Nurse
e) Pharmacist

48. Symptoms of mild to moderate Alzheimer’s disease, except?


a) Slurred speech lost
b) Difficulty managing money and paying bills
c) Fatigue
d) Depression
e) Poor judgment

49. A drug is recalled. Pharmacist will do all of the following except?


a) Post your display in the pharmacy
b) Look for batch size & number which is affected
c) Check the computer to whom the drug is dispensed and call patient
d) Check the lot number of the available stock

50. In case of dispensing error, which aspects a pharmacist should consider?


a) Inform the patient, ask for apology and inform the physician
1‐ Inform patient (to asses) 2‐ physician 3‐ Apology 4‐ document

51. In a hospital pharmacy, there is a lot of wastage and error of KCl solution. The pharmacist
can manage loss of wastage by all of the following except?
a) Decrease number of orders
b) Decrease wastage
c) Proper handling and storage

52. Why carbamazepine is preferred over phenytoin for the treatment of complex partial
seizure?
a) Saturated pharmacokinetics of carbamazepine
b) Easy dosing of carbamazepine

53. Which of the following is not a dose related side effect of carbamazepine?
a) Dizziness
b) Rash
c) GIT - Nausea
d) Visual disturbance
54. A clinical trial report shows that a control drug gives side effects in 50 patients among 500
and a test drug gives 25 among 500 patients. Find the number needed to treat?
Solution:
50/500*100 = 10% 25/500*100 =5%
Absolute Risk = 10% - 5% = 5%
Number needed to treat = 1/5% = 20 (Ans)

55. All of the following medicine cause insomnia,


except.
a) Selegilin
b) Gabapentin  sedation
c) Omeprazole
d) Fluoxetine
e) Paroxetine

56. The following narcotic drugs have expired in your pharmacy. You want to return it to the
distributor for destruction. What is the appropriate action?
a) Wait until written authorization from the Office of the Narcotic Substances
b) Give back these drugs to the distributor and ask for payment
Wait for all xpt benzo, send for knowledge but not have to wait authorization.
No return for narcotics.

57. 30g drug is needed to cover the entire body area. If a woman needs 20% of the body BID
for 7 days, how many grams of the drug should be dispensed?
Solution:
30 g X 20% = 0.6g needed daily single application
0.6 g x 2 times x 7 days = 8.4 g (ans)

58. What scale is used to evaluate liver disease?


a) MELD score (Model of End Stage Liver Disease)
The Model for End‐Stage Liver Disease, or MELD, is a scoring system for assessing the severity of chronic
liver disease. It was initially developed to predict death within three months of surgery in patients who had
undergone a transjugular intrahepatic portosystemic shunt (TIPS) procedure, and was subsequently found
to be useful in determining prognosis and prioritizing for receipt of a liver transplant. This score is now used
by the United Network for Organ Sharing (UNOS) and Eurotransplant for prioritizing allocation of liver
transplants instead of the older Child‐Pugh score.
59. You have two different brands of same drug. How would find which drug has better efficacy?
a) Same Clearance of the drug from the body
b) Same Time to peak concentration of the drug in the body (Same AUC)
c) Same Rate of absorption
It’s only A, coz it's the SAME drug so the same kinetics for both just the rate of absorption may be different
due to the different brands
Same clearance from the body is an indication for toxicity not efficacy

60. A pharmacist returned from maternity leave. She is not coming on time, do more mistakes
in dispensing medicine, leaving early in the evening (before time). What manager will do?
a) Fire her right away
b) Hire one more pharmacist to cover up
c) Warn her regarding disciplinary action

61. A 69-year-old obese female with osteoarthritis of both knees has been taking ibuprofen
1800 mg daily for the last few months. She got some relief on her knee pain but developed
‘stomach pain’ for the last 2 weeks. Antacid provided some relief, but she developed diarrhea.
Which of the following would be the most appropriate next option?
a) Have the patient take OTC famotidine with the ibuprofen
b) Suggest the patient take max recommended doses of acetaminophen instead of ibuprofen
c) Decrease the ibuprofen dose to 1200mg daily
d) See her physician to get a prescription for a PPI
Anyone over 50 yr  refer. Here 2 red flags, stomach pain & >50 yr

62. A 65 years old male patient has developed signs and symptoms of Alzheimer’s disease. All
of the following are the risk factor of Alzheimer’s disease EXCEPT?
a) Female gender
b) Family history
c) Age
d) Emotional stress

63. Alzheimer’s goal all except?


a) Decrease the progression of disease
b) Decrease the burden of caregiver
c) Prevention of disease

64. DOC for mild to moderate Alzheimer’s disease?


a) Donepezil
b) Memantine
c) Rivastigmine
d) Galantamine

65. His doctor has prescribed Donepezil 10mg daily. After one month, the patient developed
severe nausea, vomiting and diarrhea. What is your appropriate suggestion?
a) Decrease the dose of donepezil to 5mg
b) Give dimenhydrinate prior to donepezil
c) Switch to another choline esterase inhibitor
d) Stop the medication temporarily and start a later
e) Change to another class

66. A 4-year-old child having rhinorrhea, nasal discharge, dry cough and mild fever. Which of
the following drug will you give?
a) Pseudoephedrine
b) Dextromethorphan
c) Chlorpheniramine
d) Loratadine
Avoid 1st generation due to sedation, decreased learning abilities in children

67. Which of the following causes black tongue?


a) Carbamazepine
b) Bismuth Subsalicylate  B=B Black for Bismuth
c) Nitrates

68. A patient was prescribed 25mg methotrexate weekly for control of his RA. However, the
dispensing pharmacist mistakenly labelled it as 25mg daily. If the patient has already taken 7
days medicine, what will the symptoms will appear EXCEPT?
a) Leukopenia
b) Thrombocytopenia
c) Constipation
d) Anemia
e) Bone Marrow Depression

69. Which of the following drug is appropriate to reduce the toxicity of methotrexate?
a) Leucovorin
b) Methanol
c) Meperidine
d) Morphine
e) Ethanol
70. Person is overdose with methanol. How can we treat?
a) Ethanol
b) Activated charcoal
c) Ipecac

71. Which of the following is the common side effect of Clopidogrel Except?
a) Skin rash
b) Constipation
c) Cough
d) Dyspnea
e) Neutropenia

72. Which of the following medicines cause least nausea and vomiting?
a) Sertraline
b) Venlafaxine
c) Paroxetine
d) Fluoxetine
e) Fluvoxamine (most nausea & vomiting)

73. Patient with angina, hypertension & hyperlipidemia. Which medicine is appropriate for him?
a) Acebutalol
b) Lisinopril
c) HCTZ
d) Verapamil
e) Diltiazem
BBs are 1st line agents in hypertension for patients less than 60 and in comorbid condition mainly angina
I believe Acebutalol. Angina ttt (BNC)
Verapamil and Diltiazem => nothing to do with HTN ==> not an option for me
Acebutalol used with HTN and angina, add to that it has no effect on lipid => perfect option
ACE... TC did not say much about it; it is not even mentioned in the drug table.

74. A 65-year-old patient complains of weakness and fatigue. He explains that these symptoms
have worsened over several days. He is taking felodipine 5mg daily for her hypertension,
tamsulosin 0.4mg daily for her BPH, amiodarone 400mg daily for ventricular arrhythmia. He
denies the use of herbal products or OTC medications. He does not smoke but take occasional
social drinking. His pulmonologist is performed lab works which shows BP 150/95mg Hg, HR
90 beats/min, temp 38.9 deg Celsius, and respiratory rate 20 breaths/min. Other lab results are
within normal limits, but ESR is outside of normal limits. What is the concern of the
pulmonologist?
a) Felodipine side effects
b) Tamsulosin side effects
c) Amiodarone side effects
d) Hypertension
e) Increased heart rate

75. A patient was diagnosed with peptic ulcer disease due to H. Pylori infection. His doctor
prescribed him the following medicines.
Omeprazole 20mg BID x 7 days
Clarithromycin 250 mg BID x 7 days
Amoxicillin 1g BID x 7 days.
What is the pharmacist’s appropriate action?
a) Dispense as written
b) The appropriate dose of clarithromycin is 500mg BID x 7 days
c) Ask the patient to take all medicines in empty stomach

76. A 29 years old patient known to you came with a new prescription of oxycodone 20mg
daily for 20 doses. What is the reason pharmacist will not verify the prescription?
a) The prescription is from a different area
b) The prescriber is unknown to you.
c) The signature of the prescriber is illegible
d) The prescription is from a different region.

77. A 45-year-old male patient diagnosed with having DVT recently. What is the appropriate
medicine for him?
a) Warfarin
b) Dalteparin
c) ASA
d) Heparin
e) Clopidogrel

78. For which of the following disease conditions a pharmacist can break patients’ right of
confidentiality?
a) Heart diseases
b) Liver diseases
c) HIV infection
d) Psoriasis
e) Rheumatoid arthritis
79. Which reference you will check for off-label indication of a drug?
a) CPS
b) Martindale
c) Therapeutic Choices
d) Medline

80. To find which drug is approved in Canada for post-traumatic stress, which reference you
will check?
a) Martindale
b) CPS
c) Therapeutic Choice

81. Signs and symptoms of dehydration except?


a) Dry mouth
b) Loss of appetite
c) Dark color skin
d) Polyuria
e) Fatigue and weakness

82. A physician calls pharmacist to ask about a laxative for colonoscopy. Before recommending
any laxatives, what pharmacist should consider?
a) Patient is on a diuretic
b) Patient has diabetic
c) Patient has heart problem

83. A patient is unconscious and doctor feeding him via nasogastric tube to the jejunum. Which
of the following medicines cannot be crushed or broken?
a) Omeprazole tab
b) Diltiazem Controlled Delivery
c) ASA enteric coated tab
d) Nifedipine

84. You require a solution of phosphorus 5 mmol/10ml. You have 100ml stock solution, which
contain 25mmol phosphorus and 20mmol potassium. What amount of stock solution you need
to have the solution of phosphorus 5 mmol/10ml? 20 ml stock solution

85. A patient came from USA. He got some lab reports in Canada. Which reference should you
use to convert the units of lab results to US units?
a) CPS
b) Micromedex
c) Therapeutic Choices

86. The same above patient gave a prescription from USA. What is the course of action for you?
Ask the patient to have co-signed it from the Canadian doctor
a) Fill the prescription right away
b) Send him to another pharmacy
c) Send to a walk-in clinic

87. All are alcohol overdose symptoms except?


a) Pinpoint pupil
b) Depression
c) Fatigue
d) Dilated pupils
e) Insomnia

88. Patient did suicidal attempt, she found depressed, so which antidepressant should not be
prescribed to the patient?
a) Citalopram
b) Clomipramine
c) Bupropion
d) Venlafaxin
e) Mirtazepin

89. She is anxious, and has insomnia and do not want any sedatives at night so what to give?
a) Mirtazepin
b) Bupropion
c) Venlafaxin

90. She is now suffering from manic symptoms so what to give??


a) Discontinue therapy and add Lithium
b) Continue therapy add lithium

91. A woman came to your pharmacy for purchasing plan B. Due to personal belief, the
pharmacist does not want to sell the Plan B tab to the lay. What is the appropriate action?
a) Send her to another pharmacy
b) Put all personal belief aside and give her plan B tab
c) Tell her that he does not sell plan B tablet
92. Which of the following drugs you will use for opioids withdrawal symptoms?
a) Alprazolam
b) Triazolam
c) Diazepam
d) Midazolam
e) Temazepam

93. A patient has developed Acetaminophen overdose. What is the minimum quantity of
developing acute hepatotoxicity?
a) 12ml of syrup with dosage of 100mg/ml
b) 10 tablets of 500 mg
c) 20 tablets of 325mg (6500)
d) 25 tablets of 325mg (8125)
e) 15 tablets of 650mg (9750 mg)

94. Which of the following drugs to be given to acute Acetaminophen overdose immediately?
a) N-Acetylcysteine
b) Ipecac emesis

95. A pharmacist was working in the night shift. A man with a face mask approached to the
pharmacist with a knife and asked the pharmacist to give him all of narcotics. What is the
appropriate action? [K-type]
a) Give him what he wants
b) Call police
c) Delay him by asking some question

96. What is the next course of action for the pharmacist?


a) Report to the office of the Control Drug Substance
b) Report to College of pharmacy
c) Report to RCMP

97. JD is a 40 yr old cancer patient and he got the prescription of prochlorperazine. He heard
that the drug causes jerky movement and hand shaking. He asks the pharmacist about the drug.
So, what will you do for JD?
a) The DR prescribes you this drug with risk to benefit ratio.
b) The pharmacist has a concern about side effect
c) The pharmacist has concern to dispense medication
d) The side effect is rarely occurred in this dose
98. A patient calls your pharmacy tonight that a dispensing error has occurred today morning by
a dispensing pharmacist. What is the appropriate first action?
a) Ask the patient to bring back the medicine
b) Ask the patient whether the medication is ingested
c) Ask the patient to return the payment.

99. A vitamin D capsule contains 1.25mg. Each 100mcg = 400 IU. If you want to prepare 60
gm containing 10,000 IU, how many capsules will you need?
Solution:
1.25 mg x 1000 = 1250 mcg Now 100 mcg = 400 IU 1250 mcg = 5000 IU
No. of capsule = 10,000 IU / 5000 IU = 2 Cap

100. A patient is having anticipatory nausea and vomiting with chemotherapy. Which class of
drugs is suitable for him?
a) Serotonin antagonist
b) Benzodiazepines
c) Phenothiazines
d) Corticosteroids

101. In a hospital pharmacy, it is observed that the prescription error has increased recently.
Which of the following factors mostly contribute to the increased dispensing error?
a) Inexperience or lack of training of the pharmacists and technicians
b) Inefficient light
c) Inappropriate working environment
d) Increased workload of the staff

102. Aspirin toxicity with the level given of PH 7.2, Co2, and HCO3 all are less than normal.
what is the Expected problem?
a) Respiratory acidosis followed by
decompensated metabolic alkalosis.
b) Metabolic acidosis compensated by
respiratory alkalosis
c) Respiratory alkalosis compensated by
Metabolic acidosis
d) Metabolic acidosis compensated by
respiratory acidosis
To manage any acid base disturbance question, follow t
his sequence
1‐ check for PH Normal is 7.35 ‐ 7.45
If less than this range then acidosis. If more, alkalosis
2‐ check for Pco2 (35‐45 mmHg) we must memorize
these values.
If more than 45 then lung is the cause of acidosis and
this case will be respiratory acidosis.
This happens in cases of respiratory depression, COPD exacerbation or any case of reduced
respiratory activity
3‐ check for HCO3 level (22 ‐ 26 mmol)
If more than 26 then metabolic alkalosis but it less than 22 it’s metabolic acidosis
A patient has the following symptoms of toxicity, mydriasis, tachycardia, and hypotension.

103. A pharmacist wants to delegate some responsibilities to a pharmacy technician to reduce


his workload. All of the following can be a barrier except:
a) Pharmacist’s attitude
b) Pharmacist’s availability
c) Technician’s availability
d) College law

104. Which of the following products increase the INR?


a) Fluconazole
b) Carbamazepine
c) Phenytoin
d) Ginseng
e) Garlic (increases bleeding due to its antiplatelet effect but not the INR)

105. Warfarin acts on all of the following blood clotting factors, EXCEPT-
a) VII
b) IX
c) X
d) XI

106. A diabetic patient comes to emergency room. Blood tests suggest that he has diabetic
ketoacidosis. What intravenous treatment is appropriate for him?
a) Regular insulin IV
b) NPH
c) NPR + Regular insulin
d) Insulin detemir
e) Insulin glargine
107. A patient came to your pharmacy with concern regarding side effect of a drug. Which is
the best reference?
a) CPS
b) Heath Canada website
c) Manufacturer’s leaflet
d) PubMed
e) Therapeutic Choices

108. A 55-year-old patient with Parkinson’s disease complaining of bothersome handshaking.


He was working fulltime but now has become part-time. He is also considering early
retirement. He is worried about starting any medicines. What should you tell him?
a) Don’t use any medicines
b) Start medicine if symptoms are bothersome or impair daily activities
c) Medicines will cure your disease
d) Start medicine immediately to avoid disease progression

109. An elderly patient has been using 30mg IV morphine Q12H for his breakthrough cancer
pain. He and his relatives are concerned about developing addiction. They came to your
pharmacy to ask your opinion. What is appropriate to tell them?
a) It may develop addiction if you use long-term
b) Do not worry; he should use morphine for reducing his cancer pain.

110. You overheard that a cashier is explaining a side effect of an OTC medicine to a patient.
What is the best thing to do?
a) See whether the cashier advising right
b) Offer the patient to go to the counselling room talk to technician later
c) Intervene and talk to the patient and scold the technician in front ofthe patient

111. A woman came to your pharmacy and she said that her doctor advised Kegel exercise for
her. You advise her to go to
a) Chiropodist
b) Chiropractor
c) Occupational therapist

112. Target BP for patient with DM type 2


a) < 130/80
b) < 140/90
c) < 135/85
113. An eight months pregnant woman was suffering from uncomplicated UTI. Best medication
for this patient:
a) Fosfomycin
b) SMX/TMP
c) Ciprofloxacin
d) Amoxicillin

114. Which of the following is NOT the goal of UTI?


a) Eradication of symptoms within 12 hours

115. Which of the following is NOT true regarding IMIQUIMOD?


a) Apply 3 times per week at bed time
b) Keep the product refrigerated
c) Wash hands before and after application
d) Keep the application area open
e) Use fresh pack every time

116. An HIV patient with PJP infection. What is the DOC?


a) Cotrimoxazole
b) Doxycycline
c) Azithromycin

117. Which of the following prescription prompt pharmacist intervention while dispensing?
a) Azithromycin TID
b) Warfarin 5mg once daily
c) Levothyroxine 50mcg once daily

118. A patient with uncomplicated UTI has creatinine clearance of 30ml/min.Which of the
following drug will not be suitable for this patient?
a) Nitrofurantoin
b) Cotrimoxazole
c) Ciprofloxacin
d) Ofloxacin
Avoid if crcl<40‐60

119. Which of the following microorganism cause travelers’ diarrhea?


a) E. coli (and shigela, salmonella)
120. A patient with DM-2 was taking oral antidiabetic drug, but his sugar level is not
controlled. His present HbA1C is 8.2. This will indicate the blood sugar level of past
a) 2 months
b) 1 year
c) 6 months
d) 3 months
e) 4 months

121. A 23 years old young woman attended a party. After returning to home, she has rash, itching,
fever, difficulty to breath. You discovered that she is suffering from anaphylaxis from antibiotic
use. Which of the following symptom is considered the least thing to send her to the emergency?
a) Fever, blisters in trunk, bradycardia
b) Skin rash, Swollen mouth, SOB
c) Tachycardia
d) Fever

122. The same lady came back to you with epipen and diphenhydramine. Which of the
following you should not advise the lady to do?
a) She should always keep the medication with her
b) Give epipen and then rub the skin gently for better absorption
c) Give dimenhydrinate first and if no relief, then give epipen
d) Leave epipen enjection for a while
e) Epipen can be used over clothes
f) Keep the needle in at body for a few seconds after injection

123. A patient with COPD, acute exacerbation, confirmed with MycoplasmaPneumoniae. He is


also taking Atorvastatin for his high cholesterol level. Which of the following will be good
choice for this patient to combat with this exacerbation?
a) Amoxicillin
b) Erythromycin
c) Clarithromycin
d) Azithromycin

124. Which of the following is NOT the goal of dispensing/filling a prescription?


a) Appropriateness
b) Effectiveness (and safety)
c) Potency
d) Accuracy
125. There is an outbreak of Influenza in Long-term care facility. Which of the following will
be justified to start immediately? Neuraminidase inhibitors, Osetlamivir (Tamiflu)

126. In on influenza outbreak, it is required to make a brochure for 1ry health care
professionals. What information is most approp iate to include?
a) Clinical guidelines for treatment of symptomatic patients.
b) Pt information

127. A patient diagnosed with breast cancer, and her lab test shows Hypercalcemia (lab test was
3.4 and normal levels were given) what is the DOC for Hypercalcemia?
a) Pamidronate
b) Insulin
c) Corticosteroids
d) Na polystyrene
e) CaCl2

128. An elderly patient with osteomyelitis came to you. Culture was done. The result shows the
sensitivity to cloxacillin, Vancomycin, Cephalexin. The patient is allergic to penicillin. Which
of the following drugs should be given?
a) Vancomycin
b) Cloxacillin
c) Cephalexin
d) Ceftriaxone

129. Minimum duration of therapy for this patient? 4-6 weeks

130. Patient with acute coronary syndrome came to the emergency room. Which is given first?
a) ASA
b) Clopidogrel
c) Nitroglycerine SL tab
d) Warfarin
e) Dalteparin

131. Which drug is contraindicated in pregnancy who has hypertension?


a) Nifedipine
b) Losartan  ARBs are C/I in pregnancy
c) Amlodipine
d) Methyldopa
132. DOC for patient who has enlarge prostate gland?
a) Sildenafil
b) Finasteride
c) Tamsulosin

133. Which of the following drugs can be used to control the heart rate?
a) Metoprolol
b) Sotalol
c) Amiodarone

132. Patient with Ischemic attack of angina to hospital. After treatment, he was released. He is
allergic to ASA. What should be given for as maintenance?
a) Clopidogrel
b) Ticlopidine

133. What is true about Isotretinoin? Isotretinoin is used to treat severe acne (nodular and or
inflammatory acne) that cannot be cleared up by other acne treatments, including antibiotics.

134. Isotretinoin causes dryness of the body, mouth. What should be used?
Moisturizing creams, lip balms

135. What is the counselling of Isotretinoin?


a) Take on empty stomach
b) Treatment usually results in few months
c) Use a birth control method at least one month before the therapy starts
2‐3 months for optimal response. Usually, 3‐4 months for complete suppression

136. The patient is on Isotretinoin therapy, what is not needed to monitor?


a) Liver function test
b) Pregnancy test
c) Kidney function
d) CBC with differential
The tests may include:
Pregnancy test (beta‐HCG) for women and girls of child‐bearing potential (you will probably be asked to
have this performed even if you tell your dermatologist you are not sexually active, still a teenager, or your
partner has been sterilised). Please do not be offended.
Blood fats (cholesterol and triglyceride levels). These are most reliable if measured on a fasting sample, i.e.,
no food for some hours (perhaps first thing in the morning, eating your breakfast after the test has been
completed).
Liver function tests. Occasionally, isotretinoin may disturb liver function; this requires monitoring but if the
reaction is mild the drug can usually be continued. Rarely, it causes a symptomatic hepatitis: the drug must
then be discontinued. Drink minimal alcohol while taking isotretinoin, as alcohol also affects the liver.
Blood count: this is to check for anaemia and to monitor white cell count and platelets (clotting cells).

137. 13 months baby’s mother wants to wean, what is most appropriate


a) Soya milk
b) Pasteurized cow’s milk
c) Cow’s milk-based formula
d) Iron fortified formula
e) Hydrolyzed milk

138. Which of the following is not a side effect of Lithium?


a) Elevated liver enzyme
b) Weight gain
c) Diarrhea
d) Hypothyroidism

139. A patient comes to emergency with ACS. Which drug should be given first?
a) ASA
b) Alteplase
c) IV nitroglycerin
Another version: Which of the following can be use in the primary prevention of the ACS?
a) Alteplase
b) Metoprolol
c) ASA
d) Isosorbide dinitrate

140. Which of the following you will measure in ACS, EXCEPT


a) CK
b) ECG
c) Troponin I
d) Troponin M
e) aPTT
141. A patient comes to physician with CVD. He is suffering from stress due to loss of job. His
mother died due to stroke at age of 62 year and father died at 59 years due to heart attack. He is
obese, smokes and drinks 4 drinks of alcohol daily. What is not a risk for him?
a) Family history
b) Smoking
c) Alcohol
d) Obesity
e) Stress

142. A patient is taking losartan, atorvastatin. He discovered to have BPH. Which of the
following will be the most appropriate treatment?
a) Prazosin
b) Tamsulosin
c) Finasteride
d) Terazosin

143. Now prostate is enlarged, what to use?


a) Prazosin
b) Tamsulosin
c) Finasteride

144. If this same patient needs an herbal remedy. What ourld you recommend?
a) Saw Palmetto
b) Ginko biloba
c) St. john’s wort
d) Echinacea

145. 7-year-old child will go to New York with family, what will you recommend for nausea
and vomiting?
a) Dimenhydrinate
b) Scopolamine patch
c) Diphenhydramine

146. Has approved topical use in neuropathetic pain


a) Venlafaxine
b) Capsaicin
c) Diclofenac
d) Lidocaine
147. A divorced man comes to you. He is an asthma patient for 20 years. He lost his job and his
insurance coverage; His child has coverage through his mother’s insurance, now want to take
his son’s puffers for himself instead, what will pharmacist do
a) Give some sample drug which was left by doctors
b) Refuse to dispense except for the son only
c) Call the mother to take her permission
d) Dispense only this time

148. Pharmacist is in hospital ethics committee, what is appropriate for a trial of new
antiarrhythmic drug?
a) It is ethical to do placebo control trial
b) It is ethical to do trial against gold standard

149. A woman gave-birth a baby 4 weeks ago, now she wants to use contraception. What is
most appropriate for her?
a) COC
b) Condom for her partner
c) Progesterone only pill
d) IUD
e) Progesterone injection

150. A patient with MI is released from hospital. Which of the following drugs is not
recommended after his released?
a) Isosorbide dinitrate
b) Furosemide
c) Lisinopril
d) ASA

151. A doctor wants to Rx. Tacrolimus for a lady for atopic dermatitis (an unapproved
indication), so what does unapproved indication mean:
a) Not in the product monograph
b) Physicians cannot legally rx it
c) Pharmacist cannot legally dispense it
d) The manufacturer cannot legally promote the unlabelled indications to drs
e) The physician can rx on his own responsibility.
152. What is wrong about Tacrolimus?
a) Causes transient skin irritation
b) Patient can use it for long-term intermittent
c) Patient will be more photosensitive
d) Effect will appear in one week
e) Apply occlusive dressing (all vit D derivatives …no occ)
f) Immunosuppressive can be used in psoriasis
g) Avoid grape fruit

153. What is incorrect about academic detailing?


a) To improve prescribers’ prescription
b) Pharmacist plays a big role
c) Funded by health Canada
d) Funded by the pharmaceutical manufacturer  it is funded federally

154. All are appropriate measures in sleep hygiene, except


a) Exercise 2 hours before going to bed, avoid wind exercise before sleep
b) Don’t drink lots of fluids before going to bed
c) Don’t look at any watch while you are in the bed
d) Perform a wind-up dctivity just before sleep

155. Patient using PPI, what should not be used concomitantly


a) Antacids
b) H2 antagonist

156. Which will increase blood pressure


a) Metoprolol
b) Propafenone  since it causes tachycardia
c) ACEI

157. Kj 38-year-old person having low back pain, which drug you used for initial therapy?
a) Naproxen
b) Morphine
c) Tylenol#1 Acetaminophen with caffeine and codeine
d) Meperidin
e) Fentanyl
For acute uncomplicated low back pain, NSAIDs are effective for pain relief, particularly during the first few
weeks, but there is no evidence that one NSAID or COX‐2 inhibitor (celecoxib) is more effective than another.
158. After 6 month of therapy she came to Dr for check up in clinical data, morphine was found
in urine, so which of the following drug gives morphine in urine?
a) Codeine
b) Morphine
c) Fentanyl
d) Meperidine
e) Acetaminophen

159. A lady seek emergency for her husband who has COPD and suffers SOB, fever, upper
respiratory infection. At hospital his PSI= 115. Dr meet staff to decide what to do. For which
reason, this patient needs to be hospitalized?
a) Heart failure with NYHA class III-IV
b) Don't remember
c) Don't remember
d) PSI

160. Most common causative organism in CAP require hospital admission


a) Streptococcus Pneumonia
b) Pseudomonas aeruginosa
c) H. Influenza
d) L. Crypto monogenesis

161. Patient in ICU, what is the DOC (BM or BFQ)


a) Ceftazidime
b) Levofloxacin
c) Ciprofloxacin

162. What will create no doubt


a) µg = microgram
b) mL = millilitre

163. A patient admitted to hospital. Body temperature. 39 deg., heart beat 25/min, BP 125/80.
Why she has given IV therapy?
a) Acute ill, may be oral drug not better absorbed
b) A form of septic shock
164. A woman is suffering from UTI. What is not correct?
a) Dysurea
b) Cloudy urine
c) Urinary frequency
d) Lower abdominal pain-----------indicative for pyelonephritis with the fever
e) Urine urgency

165. A patient has a fungal toenail infection; he is taking digoxin. Which of the allowing do you
recommend?
a) Tolnaftate
b) Terbinafine
c) Griseofulvin
d) Ketoconazole
e) Fluconazol
f) Nystatin

166. What is true about horizontal laminar air-flow cabinet


a) Should not hang IV bags above objects
b) Should not place anything behind objects or Avoid the barrier to the materials and
the filter.
c) Viruses are present in laminar airflow

167. During sterilization process and asceptic technique. Which of the following is considered
as a source of contamination? "
a) Equipment
b) Persons working close to the hood

168. All these drugs can cause hirsutism, except?


a) Minoxidil
b) Finasteride
c) Sildenafil
d) Prednisolone
e) Phenytoin

169. Pharmacy manager wants to buy new computer system for hospital; he will see all the
following matter?
1. Financial status
2. Demands of the stakeholder or the stakeholder’s intention to change it
3. Quality of system or Identify the defects in your computer system
4. Software program or Try the new computer software
What is the most to least consideration?
a) 4-3-2-1
b) 1-3-2-4
c) 2-4-3-1
d) 3-2-4-1

170. Which drug has no drug drug interaction with OCP?


a) Phenytion
b) Phenobarbital
c) Primidone
d) Gabapentin
e) Topiramate

171. What will not cause weight gain


a) Sitagliptin
b) Pioglitazone

172. Diabetics patients taking Gliclazide Bid, Metformin 500mg Bid. His blood glucose reading
in the morning, before lunch and before night meal were high. What to do now?
a) Increase gliclazide in the morning
b) Increase metformin at night
c) Diet control
d) Use metformin three times
e) Use NPH in the evening

173. What is true for insulin Pen?


a) Cartridge cannot be substituted with different manufacturer or cartridges from
different manufactures are unexchangable
b) Pen should place in Refrigerator after open or when pen is preloaded, should be
refrigerate
c) It allows mixing of different insulin types

174. A patient wants 1000 tabs for his friends in the USA (pseudoephedrine), when the
pharmacist asks, the patient says that he needs the medication for his friends back home. The
pharmacist refuses to give the drug for all reasons except or All are ethical concerns for the
pharmacist except?
a) Pseudo is a base for making methylphenidate & has Potential for drug abuse
b) Absence of a pharmacist-patient interaction
c) Pseudoephedrine has a high potential for interaction
d) Can only travel with medications for personal use only
e) USA authority permits up to 3 gm
f) It is illegal in USA

175. Which narcotic drug causes CNS effects?


a) Meperidine  causes neurotoxicity
b) Morphine
c) Oxycodone

176. Which drug is exempted narcotics?


a) 3.3 mg Codeine/5 ml and Phenylephrine + NH4Co3
b) 5ml codeine syrup (19.8mg)
c) 5ml of codeine+NH4Co3

177. Which drug is used for falling sleep and no hang over effects in the morning?
a) Triazolam
b) Zopiclone
c) Diazepam

178. Which drug causes pro-arrhythmia


a) Propafenone
b) Sotalol  causes QT
c) Amiodarone  causes QT

179. A patient has asthma and migraine. He is using Amitriptyline, Salbutamol and nicotine
patch (3 times revious refill and full course was given), he is coming now to have his
salbutamol refill and complaining of chest tightness and ineffectiveness of salbutamol since
along time. what is reason?
a) Drug - disease interaction
b) Drug - drug interaction
c) He is not using appropriate drug  he should use ICS
d) This patient is taking too much medication
e) This patient uses medication that he is no indication

180. A patient using omeprazole for GERD, not controlled, what to do?
a) Double the dose of omeprazole
181. Which of the following drugs a pregnant woman should not touch?
a) Digoxin
b) Framicetin plaster
c) Dutasteride

182. Below is the pharmacokinetics on two beta-blockers. The kidneys and liver metabolize
both.
Drug X Drug Y
Total clearance (ml/min) 1100 460
Renal clearance (ml/min) 110 90
VdL 3.3 2.0
Protein binding (%) 85% 90%
Which of the following undergo extensive 1st pass hepatic elimination?
a) Drug X oral
b) Drug Y oral
c) Drug Y rectal
1100 ‐110 = 900 hepatic elimination more than renal

183. If drug Y is produced by one manufacturer only and is going to be discontinued soon, what
the hospital pharmacists will do?
a) Procure same drug from manufacturer in large quantities
b) Stock pile the drug since it is going to be discontinued
c) Procure the drug that has similar indications and let the physician know about it

184. Doctor prescribed a drug that is not available with you because the manufacturer has short
supply right now. What would be the appropriate action you can do except?
a) Inform the doctor about it and request to prescribe alternative drug
b) You do not have anything to do
c) Ask the manufacturer when this drug will be available

185. A lady came Pharmacy with her 8yrs child for prescription refilled, Pharmacist noticed that
she is shouting her child; it is third time Pharmacist noticed the same incidence, what is
Pharmacist course of action?
a) Asked her everything is all right
b) Talk to child separately and asked him how often she did shout
c) Talk to Child’s father
d) Report to police
1st time nothing. 2nd time document. 3rd time talk to her
186. If pharmacist reports it, which organization he should report?
a) Child aid society
b) Police
c) Public health
d) Nurse

187. A patient is suffering from osteoarthritis, which drug not used for long time?
a) Ketorolac  only for short term use and in renal colic
b) Diclofenac/ misoprostol
c) Acetaminophen
d) Indomethacin

188. Phenytoin has initial concentration is 20mg/hr with Css 10mg/hr, now dose is increased to
40mg/hr, what will be Css level now?
a) 10
b) 20
c) 30
d) 40

189. A 37 years old man suffering from asthma, using salbutamol MDI since he was 8 years
old. Which counseling’s are required, except?
a) Use Spacer
b) Assessment asthma
c) Salbutamol use frequency

190. A new drug produce side effect 10%, studies among 100 patients, and 10 % with placebo
effect with same 100 patients. What types of problems can be assumes from the above studies?
a) Type 1 error
b) Type 2 error
c) Sample size to small
So, no difference exists so type 2‫ﺍﻋﺘﻘﺪ ﻫﻨﺎ ﻳﻘﺼﺪ ﺍﻥ ﻣﻔﻴﺶ ﺍﺧﺘﻼﻑ‬
191. A patient come to Pharmacy with sertraline, while the patients pick the prescription, the
cashier informed him that sertraline causes sexual dysfunction, the pharmacy manager noticed
it, what is course of action by pharmacy manager.
a) Pharmacy manager asked cashier you are not allowed to tell presence of patient
b) Pharmacy manager solve the problem first with patients and talk to cashier
separately later
c) Allow cashier solve it first and then talk to cashier

192. In a hospital pharmacy which factor leads to dispensing error?


a) In adequate training of technician
b) Communication problem
c) Electronic dispensing order

193. What is common cause of prescription error in a community pharmacy?


a) Pharmacist workload
b) Inadequate light

194. A patient with INR 4.5. What to do?


a) Stop warfarin for one day & reduce the dose (less than 5 and no bleeding)
b) Take vitamin K orally
c) Take vitamin K containing foods

195. Atorvastatin 10mg daily for 4 months and then DR add fenofibrate and patient has muscle
pain so what would you suggest to patient?
a) Stop all medication and add ezetimibe
b) Start atorvastatin 10 mg and then increase the dose
c) Start atorvastatin and add niacin

196. Patient is suffering from bipolar disorder. What is the DOC?


a) Lithium
b) Carbamazepine
c) Lamotrigine

197. Contraindication of Zolmitriptan?


Ischemic heart disease, sustained hypertension, pregnancy, basilar or hemiplegic
migraine.

198. What is the right posture of measuring BP in pharmacy?


a) Sitting on the chair and feet on the floor
199. Patient has rosacea and feeling itching, red skin and the factors should be avoided except -
a) Sunlight
b) Alcohol
c) Smoking
d) Stress
e) Spicy food

200. Amox/clav suspension patient counselling?


I. Shake well before use.
II. Refrigerate the suspension and discard any unused portion after 10 days.
III. Avoid sun exposure.

201. All causes photosensitivity except


a) Minocycline
b) Ciprofloxacin
c) Isotretinoin
Tetracycline, doxycycline, nalidixic acid, voriconazole, amiodarone, hydrochlorothiazide, naproxen,
piroxicam, chlorpromazine and thioridazine are among the most commonly implicated medications.

202. Lady is on OC and taking phenytoin, what is true?


a) Phenytoin increases the clearance of the OC, and decreases the effect.

203. Dementia patient, having symptoms of Acute Agitation best drug to initiate to the therapy
is (it was a senario)
a) Lorazepam

204. A drug a contain 10% and drug B contain 20% and dr. wants to prepare 2% of normal
saline solution. Both drug A and drug B have 1%. How much drug A and Drug B are needed
for this preparation?
a) 1 gr of 10% & 2 gr of 20%
b) 4 gr of 10% & 6 gr of 20%
c) 6 gr of 10% & 6 gr of 20%

205. Patient is coming in the hospital emergency department with the symptoms of tachycardia,
hypotension, irregular heart rhythms on addition to it he has some dryness of the mucosa. These
symptoms seem like the overdose of which of the following medication??
a) Amitriptyline
Note: It’s the symptoms of TCA overdose and this was the only TCA in the option.
Another version: A patient came to the emergency having dilated pupil, increased heart rate,
dry mouth. Which agent could be the cause?
a) Morphine
b) Amitriptyline

206. Non-pharma for the back-pain patient (Senario)


a) Complete bed rest
b) Tens therapy
c) Acupuncture
d) Do routine work as tolerated.

207. In the hospital pharmacy the Dr. Intern is asking pharmacist recommendation about the
systemic corticosteroid use in the COPD for his one of the patients, best reference for the quick
search is?
a) Secondary Internet source
b) Tertiary reference book
c) Case study
d) Medical jernoual

208. Goal of therapy for the RA is all of the following except.


 Fully control signs and symptoms of the disease, i.e., pain, stiffness and fatigue
 Halt radiographic progression and joint damage
 Maintain physical function and work capacity; and maximize quality of life
 Obtain rapid clinical improvement with a goal of 50% improvement within 3 months and ideally
clinical remission.
 Remission means the absence of disease activity as assessed by a clinician (swollen and tender
joints), patient (global assessment of disease activity) and laboratory results (CRP and/or ESR).

209. Digoxin overdose Question. Pattient have hypokalemia, nausea, vomiting, weakness &
blurred vision. these symptoms are due all to except
a) Digoxin
b) Hydrochlorothiazide

210. Digoxin level is found 2.5ng/ml (Normal is 0.6 - 1.0 nmol/l). What will u do?
a) Decrease the dose
b) Skip the next dose
c) Stop digoxine for next 2 days
d) Stop digoxine and administer antidot.
211. What is true about lithium?
a) Take an empty stomach
b) Minimize salt intake
c) Avoid dehydration and maintain Na balance
Note: Lithium with or after food, as per CPS and maintain Na level and avoid dehydration to avoid Lithium
toxicity.

212. Now patient had been diagnosing with hypertension and his DR prescribe him Fosinopril,
so what is the mechanism of interaction between fosinopril and lithium?
a) All ACEIs elevate Li+ levels (potential toxicity)

213. The pt experienced side effects. All of the following are Lithium side effects except:
a) Weight gain
b) Hypothyroidism
c) Increasing liver enzymes

214. What should be done to prevent drug interaction?


a) DR should prescribe hydrochlorothiazide instead of fosinopril
b) Change lithium to lamotrigine
Patients stabilized on lithium therapy who receive a thiazide diuretic may require a reduction of lithium
dosage to avoid accumulation and toxicity, since there is often a 20 to 40% reduction of renal lithium
clearance. Furosemide appears to be less likely to affect lithium clearance.
Reduce lithium dose and check serum level if long‐term treatment with these agents is required.
Stop lithium temporarily during acute intermittent illnesses causing fluid and electrolyte losses.
For lithium‐associated cognitive impairment, check lithium level and thyroid function. Lowering the dose or
using a slow‐release formulation may improve cognitive function.

215. Now patient symptoms are getting worsen, so what will be the reason?
a) Non adherence to the therapy

216. Drug given 200 mg by IV bolus infusion. If we are giving 200 mg each tablet, which is
35% bioavailable to the IV drug. So, how many tablets needed to switch the patient form IV
dose to oral to get the equivalent amount of the drug in the body?
100% * 200 mg = 35% * X X = 571.428
Each tablet contains 200 mg. so, for 571.428mg, it’s 3 tab.

217. To cover the whole-body part needed drug amount is 30 mg, but for the 18-year-old boy to
over 10% of the body part, apply BID for 7 days, what quantity is to be dispanced?
30 mg  100 % 3mg  10 % 3mg * 2 (BID) * 7 days = 42 mg.
218. Concerning special access programs drugs SAP, which is true?
a) Can be requested by the Practitioner, which is not available in the market for the
gerenal population use.
b) The manufacturer is responsible for product pricing and decides to sell it or not.

219. Where to report medical error that happens in the hospital?


a) Canadian hospital pharmacy residency board (CSHP)
b) Canadian Public health agency
c) Canadian hospital society
d) Canadian public health association

218. All of the following are the functions of fedral government except?
a) Provide funds to the provincial govt.
b) Provide the fund for the hospital.
c) Hospital research cost
d) Manufacture regulation???
(Not sure about options 100% but I think manufacture related was the ANS. Sorry.)

219. Most personal expensive thing after the drug cost for the patient?
a) Dr.’s Salary
b) Hospital expences
c) vaccination cost
d) Lab test fees.
Note: In the question asked about the 2nd most expences. So, 1st is drug expences and 2nd is Lab test fees.
All other options were paid by the governamnet.

220. Which of the following statement is most appropriate to ask the patient who already made
decision to quit the smoking?
a) How many cigarettes he smokes per day?
b) When he smokes his 1st cigarette?
c) Are you ready to quit smoking in next 3 months?
d) Change brand
e) Quit smoking slowly

221. He is on depression medication, what is the best smoking cessation treatment for him?
a) Non- Pharmacological choices
b) Nicotine Patch
c) Bupropion
d) Venlaflexine
Note: As patient was already on the Antidepressant medication and non pharm won’t work alone.
222. Which is the most appropriate advice for this patient to quite smoking?
a) Try to quit in less stress or in the weekend.

223. patient was on salbutamol and Ipratropium but symptoms are not controlled what is the
next appropriate stape for his physician?
a) Add triotropium BID
b) Add oral prednisone therapy
c) Add fluticasone BID

224. In the hospital pharmacy from the lab results you find that patient is on Levothyroxine, his
TSH level is decrease (value was given) what is the best recommendation?
a) Skip the dose for 2 days
b) Decrease the dose of levothyroxine
c) Lower the dose of levothyroxin and repeal TSH after 6 weeks.

225.mAll are cardiac markers, except?


a) Troponin
b) CK-MB
c) Serum CR

226. JD is 45-year-old male patient suddenly he got the chest pain radiating to his left arm, his
DR diagnose the prinzmetal angina. What will the DR prescribe to this patient?
a) Propranolol
b) Amlodipine
c) Atenolol
d) Isosorbide dinitrate
e) Ramipril
The drug of choice is CCBs (nifedipine, amlodipine). Acute chest pain use nitroglycerine SL spray or tab

227. What is true about Parkinson’s disease?


a) Start the treatment as soon as possible
b) Start the treatment if symptoms are bothersome.
c) Delay the drug therapy because of the side effects associated with the use of drug.

228. A patient taking Insulin NPH and Regular in the morning and at dinner time, and from the
values given you find that both A.M. And P.M. Glucose levels are low, so what should you do:
a) Decrease both A.M. And P.M. Doses of NPH
b) Decrease only A.M. Dose of NPH
c) Decrease A.M. Dose of Regular and P.M. Dose of NPH
229. Patient run out of some diabetic, hypertension medication and Zopiclone. It’s weekend and
he have dr.’s appointment on Monday. He phones to the pharmacy and asking for 2 days supply
of medication. Which is the best course of action you will take as a pharmacist by using the
code of ethics.
a) Give medication except Zopiclone
b) Refer to emergency department
c) Give him all 5 medication (there were 5 drugs listed in total)

230. A patient is on Atorvastatin and Losartan and is on a diet. Two weeks ago, her
Atorvastatin dose was increased. She has been eating Citrus Fruit lately. She comes to you
saying that she is experiencing aches and GIT problems, what is the cause
I. Increased dose of Atorvastatin
II. Roughage food
III. Grape fruit juice intraction
It’s because the increase the dose, and he was eating citrus fruit so, intraction with grape fruit juice. Don’t
know about the Roughage food. My ans was E. Plz check.

231. So what you would advice the above patient:


a) Go to the emergency straight away
b) Stop the drug for now and go see the doctor
c) This is a normal side effect of the drug and she should continue taking it

232. All of the following are the Side effects of Zidovudine except?
Zidovudine (AZT) S.E: Nausea, headache, malaise, fatigue, rash, myositis, myocarditis, anemia,
pyrimidine analog leukopenia, hepatic steatosis, elevated liver enzymes, lactic acid and CK. Longterm
use associated with peripheral lipoatrophy.
Additive hemotoxicity with other agents, e.g., anemia with dapsone, foscarnet,
ganciclovir, pentamidine, ribavirin. Pharmacologic antagonism with stavudine.
Avoid combined use of AZT and ribavirin or stavudine as are activated by the same
intracellular pathways. Available as an oral syrup.

233. Patient was on irbesartan, all are the correct recommendation for this patient except
a) Tell patient to eat banana and potasium rich diet.
Note: ARBs increase the K+ level, so this is wrong advice.

234. Which reference should you check for pharmacokinetics?


a) Merck Manual
b) Micromedex
c) CPS
235. TN, patient with the stright narcotic prescription at 10 PM and he had same prescription 4
days ago. He said he let the know about the 1st dr to the recent one. Best course of the action for
the pharmacist is?
a) Don’t feel the priscription
b) Refuse to feel as he is coming 10 pm, maybe he is abusing the drug.
c) Call the second dr. And discuss the matter with him.
d) Fill the priscription and counsel him.

236. Helicobacter pylori testing is important for all EXCEPT?


a) Duodenal Ulcer
b) Stomach Ulcer
c) Gastroesophageal Reflux Disease (GERD)
d) Peptic ulcer

237. Patient had an allergic reaction to clotrimoxazole. Which of the following drug should be
avoided in this patient?
a) Trimethoprim
b) Captopril
c) Silver sulfadiazine cream

238. Insulin can be stored at the room temp. for how many days?
a) 21
b) 15
c) 30
d) 45

239. Pharmacy is promoting blister pack; the pharmacy manager can do all these professional
steps except?
a) Post in in the pharmacy window.
b) Post in near the counter
c) Go and talk to the potential prescriber of the pharmacy
d) Give discount

240. Publication bias means: The bias of the editors to certain publication results.

241. A 27 years old woman is suffering from depression and insomnia. What is the appropriate
drug for her?
a) Mirtazapine
b) Bupropion
c) Paroxetine

242. Isotritinoin, all should be monitered, except?


a) Hyperglycemia
b) Serum electrolyte
c) Lipid

243. MD is 30-year-old male patient talking asking cashier about ST John wart. Pharmacist
intervenes between them. MD explains to pharmacist he is depressed from last two weeks, and
he heard about this medication and he wants pharmacist suggestion. Pharmacist told him to visit
his DR for further assessment but patient refuses. He only wants to buy St John wart. You agree
with patient and you give him ST John wart. So, which ethical principal you followed.
a) Autonomy
b) Nonmaleficience
c) Paternalism
d) Beneficience
e) Fidelity

244. Again after a week MD comes with the prescription of paroxetin and he is asking
technician about sexual dysfunction. Technician replies that many patients taking the same
medication so don’t worry. So, what will pharmacist do?
a) Pharmacist intervene and tells tech. badly in front of patient that its not his job
b) Tell the patient about his obliges and then talks to tech. separately
c) Make a next meeting and explain everyone the role of tech.
d) Make a note in tech. appraisal form

245. Pharmacist is living near the house of a female patient and he tells her sister that she tries
to suicide. So, which ethical principal is being violated by pharmacist??
a) Nonmaleficience
b) Veracity
c) Confidentiality
d) Autonomy

246. In Canada, which of the following is the publicly funded (tax paying)?
a) Hospital
b) Physician salary
c) Diagnostic test
d) Pharmacist salary
247. What is the normal fasting blood sugar level for diabetic pregnant woman?
a) 5.5 to 6.5
b) 6 to 7
c) 6.5 to 7.5
Aim to achieve an HbA1c level of ≤6.5% (≤6.1% if safely achievable).
FPG/Preprandial Glucose (mmol/L)  < 5.3
1‐hour Postprandial Glucose (mmol/L)  < 7.8
2‐hour Postprandial Glucose (mmol/L)  < 6.7

248. JD is 34-year pregnant lady she has burning and pain while urination. Her DR diagnose
UTI All are the symptoms of UTI except?
a) Burning sensation
b) Internal dysuria
c) Hematuria
d) White vaginal discharge

249. What will be treatment for her?


a) Fosfomycin
b) Co-trimoxazole
c) Ciprofloxacin
d) Cefuroxime

250. When would the symptoms relieved?


a) Symptoms relieves in 12 hrs
b) Symptoms relieves in 24 hrs
c) Symptoms completely eradicate in 7days

251. You are going to do a seminar for Cystic fibrosis patients, you will invite all except?
a) Physiotherapist
b) Occupational therapist
c) Respiratory therapist
d) Physician

252. Stroke, whom you invite? Neurologist

253. In hospital director invite all except?


a) Patient
b) Nurse
c) Physician
d) Technician
254. Drug x and drug y easily substituted in all cases, except?
a) Drug X and Y has same indication
b) Less or similar side effect
c) Both has same frequency of absorption

255. Which benzodiazepine shows withdrawal symptoms after 2-3 days of abrupt withdrawal?
a) Alprazolam
b) Oxazepam?
c) Flurazepam
d) Temazepam
e) Diazepam

256. Ezetimibe monitoring


a) Glucose
b) Creatinine kinase  Monitor liver function and CK.

257. Weight loss


a) Acarbose
b) Meglitinide
c) Metformin

258. A hospital pharmacist is considering doing a protocol for the orientation and training of
new technicians. What is the LEAST activity to include in the protocol?
a) “Shadowing” of another technician for a whole day
b) Rotate the technicians in the local hospitals
c) Give them the human resources policy to read???
d) Update medical information by attending big medical event

259. JK is the 40 yr old male patient suffering from acute pain. His DR prescribes him
morphine for acute pain. He has no history of cancer. What is true?
a) Addiction is a major concern
b) Side effects are major concern

260. Why is it important to perform medication reconciliation, all are important EXCEPT?
a) To create a complete list of the medications the patient takes on regular basis
b) To reduce adverse drug events due to drug discrepancies
c) To review total cost of received therapy
d) To ensure the patient is receiving appropriate therapy
261. Class 4 CAP patient risk is 110 which of the following organism may present
a) S. aureus
b) P. aeruginosa
c) H. influenza

262. What is the drug of choice?


a) Levofloxacin
b) Ceftazidime
c) Penicillin

263. Patient with arrhythmia was given anti-arrhythmic. Which of the following drug causes
increase in heart rate?
a) Propafenone
b) Procainamide
c) Amiodarone
d) Sotalol

264. JD is a 6-week pregnant woman and she vomits 8 times in last 24 hrs. She doesn’t want to
take any medication and she is not drinking enough fluid. Her husband come to your pharmacy
and asks for some help. So, what will you suggest him??
a) Give him OTC ginger tablet
b) Give him OTC dimenhydrinate
c) Send her to emergency
d) Refer her to DR for diclectin

265. Now she got baby, and after 4 week of pregnancy she need contraception and she does not
have any allergy, so which contraceptive method is best for her?
a) Medroxyprogesterone
b) Low dose of ethinyl estradiol
c) Norgestrel
d) Diaphragm with spermicide
e) Partner should use condom

266. She is on breastfeeding, fungal infection and son got infection in mouth. DR prescribes
clotrimazole cream for mother and nystatin drops for child, so what you advise her??
a) Wash clotrimazole cream and then breatfeed
b) Nystatin drop given to child and then breastfeed
c) Use a feeding bottle with breast milk in it feed the baby until cured.
d) Stop breastfeeding until bacterial infection gone
e) Use cotton swabs to spread nystatin

267. After 14 weeks of breast feeding, she wants to stop breastfeeding. So which baby formula
she should give to baby?
a) Milk formula enriched with Iron
b) Soya milk enriched with Iron
c) Hydrolyzed milk enriched with Iron
d) Pastuerized cow milk complet (3.2S%)

268. A woman is pregnant by 4 month and she is depressed and DR wants to prescribe
venlafaxin. DR asks pharmacist. So which reference book pharmacists will refere??
a) Primary journal
b) Health Canada website
c) Motherrisk programme
d) Product monograph

269. 20. A 48-year-old woman is coming to complain of ho flushes and insomnia. She has no
history of breast cancer. She is having her menopause. Which of the following is the most
appropriate to treat her vasomotor symptoms?
a) Progesterone
b) Estrogen
c) Venlafaxine

270. Patient taking ASA and Warfarin for arterial fibrillation, all consider risk factor except?
a) Diabetes
b) Hyperthyroidism
c) CHF
d) HT
e) Age > 75
271. Patient has ascites and his lab values k is 6.8 meq/lit all other values are normal, which
drug is choosen for initial therapy?
a) Furosemide
b) Spironolactone
c) Furosemide plus spironolactone
d) Hydrochorthiazide

272. Now he got confusion so which drug should be added?


a) Lactulose

273. Renal clearance 60ml/min and has acute gout which drug should be avoided?
a) Allopurinol

274. Side effect of allopurinol


a) Skin rash
b) Increase dyslipidemia

275. Patient is on salbutamol and if you do not tell symbicort will help you so which ethical
principal you will break?
a) Beneficience
b) Justice
c) Fidelity

276. 80 yr widow and she is diabetic, BP 190/110 what will be the DOC?
a) Perindopril

277. Patient taking 200mg iv bioavailability of oral liquid solution is 38% so how many 200mg
tablets to be dispense to achieve equivalent

278. Prednisone tablet counseling: k type


I. Should not be exposed to the sun after taking pills
II. Take with food

279. Auxialary label for Sucralfate:


a) Causes drowsiness
b) Do not crush the tablet
c) On empty stomach, half hour before food
d) Avoid in pregnancy and pt under 18
280. Heparin induced thrombocytopenia. What is true
a) Liperudin is a therapeutic alternative for heparin in case of HIT.
b) Happens only in veins.
c) Happens after exposure to heparin by 2 to 3 days.
Heparin‐induced thrombocytopenia (HIT) is a potentially devastating immune mediated adverse drug reaction caused
by the emergence of antibodies that activate platelets in the presence of heparin. Despite thrombocytopenia,
bleeding is rare; rather, HIT is strongly associated with thromboembolic complications involving both the arterial and
venous systems. HIT may develop in two distinct forms: type I and type II. HIT type I (also known as heparin‐
associated thrombocytopenia) is a non‐immunologic response to heparin treatment, mediated by a direct interaction
between heparin and circulating platelets causing platelet clumping or sequestration. HIT type I affects up to 10% of
patients, usually occurs within the first 48–72 h after initiation of heparin treatment, and is characterised by a mild
and transient thrombocytopenia (rarely <100 000/mm3), often returning to normal within 4 days once the heparin is
withdrawn
Another version: Which treatment can be used to treat Heparin Induced Thrombocytopenia?
a) Dalteparin
b) Warfarin
c) Lepirudin
d) Alteplase

281. A patient has Celiac disease; he must be referred to? Dietician

282. The food for this patient must be free from: Gluten

283. Patient is on carbamazepine & the doctor prescribed Amoxicillin for him as he has
mycoplasma pneumonia. What u should do in this case?
a) Call the dr to change amoxicillin to clarithromycin.
b) Call the dr to change amoxicillin to azithromycin
c) Give the medicine as written

284. A study has been conducted on 2 groups of smokers from 1985 till 1997. Some other
scenario written then the following question: What type of study is that???
a) Cohort study
b) Case report
c) Randomized controlled trial

285. Which one of the following caused prompts you to suspect the authenticity of the Rx?
a) The patient came late at 10 pm
b) The patient looks suspicious
c) The dr name and signature are not clear
286. The main goal for randomized double-blind study trials is:
a) Decrease the bias in allocation of treatment groups
b) Give a more valuable result

287. You are planning to conduct a lecture about diabetic foot infection. Which of the
following specialties could be invited?
a) Orthopodist
b) Chiropractor
c) Occupational therapist

288. A patient has suicidal ideation. Which of the following should not be given?
a) Bupropion

289. Calcium carbonate is given to patients with kidney failure. The rx shows: Chew one tab 3
times daily prior to meals. What is the purpose?
a) Prevent any severe acid reflux
b) Prevents phosphorus loss as the patient with kidney failure

290. A pharmacy student is having training in a pharmacy. After counseling a patient, the
patient came back and complained to the manager that she didn’t understand anything. What
action should the manager take?
a) Review counseling process with the student before patient counseling the next time
b) Give counseling materials for the student to read
c) Send him to a disciplinary action

291. A patient has migraine on Zolmitriptan. Which of the following is contraindicated? K type
a) Caffeine
b) Paracetamol + codeine + caffeine
c) Cafergot (Caffeine + ergotamine)

292. What is a risk factor for constipation?


a) Age
b) Diabetes
c) Female gender
d) Alcohol

293. A COPD patient developed bronchitis. The dr Rxed lpratropium nasal spray. Which of the
following is correct? Improper formulation to treat the diagnosis of medical condition
294. A calculation about how many capsules of vitamin E needed. Repeated one and very easy.
Answer was 2 capsules.

295. Which of the following is considered as an examp1e of tall man lettering in hospitals?
a) predniSONE and predniSOLONE
b) PREDnisone and PREDnisolone
c) prednisONE and prednisolONE

296. In a hospital setting, which of the following is not considered as a reason for drug errors?
a) Checking patient compliance
b) The rx stage
c) The dispensing stage
d) Measuring the dose by a nurse
e) Preparation
f) Counselling
g) Follow-up

297. What is not serotonergic symptoms


a) Sweat
b) Tremors
c) Anxiety
Serotonin syndrome:
Rare, and may occur when multiple serotonergic agents are used MAOIs have the highest risk and
require a minimum 2-wk washout period before another serotonergic agent is initiated
The serotonin syndrome may occur if the SSRI, MAOi, SSRI, TCA, are combined with Tramadol,
Meperidine, Amphetamine, Dopamine, Cocaine, Methyldopa, Dextromethorphan, Caffeine,
Lithium, SSRI, MAO Inhibitors, TCAs, St. John wort.
Amitriptyline, mirtazapine and trazodone are unlikely to cause serotonin syndrome
Signs and symptoms of Serotonin syndrome
 Neurobehavioral: confusion, agitation, seizures, coma, delirium, hyperreflexia.
 Autonomic: Hyperthermia (fever), diaphoresis (sweating), tachycardia, hypertension, diarrhea
 Neuromuscular: Myoclonus, rigidity, tremor (shivering), ataxia, dilated pupil and nystagmus
Management:
 Stop the drug(s) and refer patient to the hospital
 Most cases are mild and resolve spontaneously within 24 to 72 hours.
 Pharmacist has to contact the physician and tell the patient to withhold the serotoninergic agent.
 Benzodiazepines, propranolol, and cyproheptadine, a serotonin antagonist, have been used
successfully.
298. For this patient, pharmacist can give all the following counselling for a good for
prevention, except:
a) Do not wear cotton socks
b) Don't walk bare foot in public area
c) Have always short nails

299. A patient with acne comes after using Benzoyl Peroxide gel complaining of dryness in
the face and mild improvement only in acne. What do you advise him?
a) Continue using the gel as this is normal in the beginning and give him a moisturizer
for the dryness.
b) Stop it, use cream instead

300. A 56 years old lady gets menopause 6 years ago; she has family history of breast cancer
(mother and Sister) and family history of hip fracture. Her mammography recently done is
correct, and her bone mass test score is - 3, she has asthma and use long time inhaled
fluticasone. She recently had a hip fracture and she is afraid she will also have osteoporosis later
in her life. She is having a healthy life and drinks 3 cups of milk day. What do you advise her?
a) Take calcium and vitamin D daily
b) Make bone mass density yearly

301. She went to the MD. Which of the following can he prescribed to prevent hip fractures?
a) Calcitonin.
b) Raloxifene
c) Calcitonin
d) Teraparatide
e) Alendronate 10 mg daily

302. A calculation about least amount of amoxicillin suspension in ml required for treating a
small boy given the dose and weight and number of days.
and the correct answer was 200 ml.

303. If you have 1 liter of glucose 10%. Knowing that each gram gives 3,4 kcal. How many
kcal it will give you?

304. Methadone 8.1 gm Na Benzoate 1.15 gm Liquid qs 810 ml


If you have this formula in the pharmacy to prepare the methadone and you received
the following Rx from the MD: Prepare Methadone 9 gram. Mitte 900 ml.
What is the amount of Methadone & Na benzoate to prepare 900 ml?
Answer:
Methadone = 8.1/810 * 900 = 9 gm
Na Benzoate = 1.15/810 * 900 = 1.27 gm

305. A patient discovered he has diabetes mellitus and now is on Metformin. Which of the
following is considered the least important to do?
a) Measure his BP
b) HbA1c
c) Serum creatinine
d) Albumin/creatinine ratio
e) FBSL
‫ﻫﻮ ﻣﻤﻜﻦ ﺗﻘﻴﺲ ﺍﻟﻀﻐﻂ ﺯﻳﺎﺩﺓ ﻟﻜﻦ ﺍﻟﺒﺎﻗﻴﻴﻦ ﺍﺳﺎﺳﻰ ﺧﺼﻮﺻﺎ ﺍﻧﻪ ﺑﻴﺎﺧﺪ ﻣﻴﺘﻔﻮﺭﻣﻴﻦ‬

306. Dr. prescribe OFF label drug, what you can tell the patient:
a) Off label come after post surveillance of the drug
b) Health canada doesn't give NOC for OFF label indication
The term “off‐label” refers to any use of a drug beyond what Health Canada has reviewed and authorized to
be marketed in Canada and as indicated (reason the drug is used) on the product label.

307. Schedule 3 drugs means:


a) The pharmacist should be available to help the patient with making appropriate self
medication selection.
Schedule • Prescription not required, Over the counter OTC
III • Drugs accessible and visible to public but must be within 10 m of dispensary where
pharmacist can answer patient questions
• Ex: Plan B. Emergency contraception, Bacitracin and its salts for eye/ear,
fluconazole 150 mg, lactulose, hydrocortisone 0.5%, 1%. Naproxen 200 mg base or
220 mg salt, maximum package size more than 6 g. Ibuprofen 400 mg or < 600 mg
modified release package size more than 18 g.

308. A parole officer calls you to ask about the medication that one of the prisoners, he is saying
that this would help in criminal investigation. What should you do in this case?
a) Don’t disclose any information except when you get a consent from the prisoner.
b) The parole officer is in the center of health care and you can give him the information he
needs.
c) Because the patient is a prisoner, he lost hi confidentiality rights and you can disclose the
information to the parole officer.
309. A pharmacist wants to adopt a new service for geriatric home. What is the most
appropriate he should begin with?
a) Make a survey about the geriatric homes round the area
b) Offer an incentive for the patients who use this service

310. The risk factors of clostridium difficile diarrhea


include all except.
a) Female gender
b) PPIs
c) Age > 70 years
d) Recent flouroquinolone use

311. What is the advantage of propylthiouracil over


methamizole?
a) Prevents conversion of T4 to T3
b) Can be used during pregnancy
c) Can be used in children
Antithyroid Concentrated in the thyroid, Risk of skin rash, MMI preferred first line agent
Agents where they inhibit both the allergic reaction, as it has long half life,
oxidative processes required agranulocytosis, especially in children,
Methimazole for iodination of tyrosyl hepatotoxicity (rare). breastfeeding, except during the
(MMI) groups and the condensation Warn patient to stop if 1st trimester of pregnancy when
(coupling) of iodotyrosines rash, fever, sore throat PTU is preferred.
Propylthiouracil to form T3 & T4. or jaundice develop. Avoid PTU in children.
(PTU) PTU also blocks peripheral Develop a gradual MMI 1st choice when
conversion of T4 to T3. neutropenia, which is Pretreatment measurement of
These drugs have no effect most likely to occur in neutrophil count recommended.
on thyroglobulin already 1st 90 days of treatment. Thyroid status influences
stored in the gland. So, their They must be stopped response to warfarin.
clinical effects may be above five days prior to Monitor INR when anti-thyroid
delayed until thyroglobulin a thyroid scan, RAIU or medication dose altered.
stores are depleted) treatment with 131I.

312. A patient has diabetes and renal failure deyeloped Hypertension. What is the
best treatment option?
a) Lisinopril (ACE)
Add ACEIs or ARB in those with hypertension or albuminuria to delay progression of KD.
313. A patient is going to have surgery and has latex allergy. Where to find about if the IV set
will cause him allergy or no?
a) Martindale
b) CPS

314. A dr Rxed for an 8-year-old girl codeine 20 mg. Her father came to the pharmacy to
dispense it. The available codeine strengths are 5, 10, 30 mg. What should you do?
a) Tell him the strength is not available and you cannot dispense it.
b) Dispense a mixture of 2*10 mg to adjust the dose
c) Call the MD to write a new Rx matching with what’s available
Pahramcist can do Adapting: change in dose, formulation, regimen, duration and route. Renew or adapt
prescriptions with exception of narcotics and controlled and targeted substance.

315. What is considered as a proper action if a harmacist is required to prepare a


pharmaceutical preparation and didn't find enough quantity of a certain ingredient?
a) Send to a nearby institution to get the required quantity.
b) Tell the patient that it is not available an advice to come tomorrow
c) Prepare the preparation with the availab quantity and continue in the second day when the
new order comes.

316. A major depressive disorder patient has a Rx for Bupropion at 9 AM, 2 PM, 7 PM 150 mg
*3 times. He forgot the morning dose and remembered it at 12 PM. What proper action to
advise him?
a) Skip this dose and take the next dose at scheduled time.
b) Reschedule the dose and tell him to take the doses at 12 PM, 5 PM, 10 PM.
c) Tell him to take 2 tabs now and continue 5 scheduled
BUPROPION SR should be taken at the same time each day and no more than the recommended dose should
be taken each day. If the normal administration time has been missed, the dose should be skipped and
administration resumed at the normal administration time of the following day.

317. Which of the following is least considered in Rheumatoid arthritis?


a) Cartilage inflammation

318. How do you recognize morphine addiction of m the Rx?


a) Morphine long acting is Rxed for the patient
b) Morphine long acting is Rxed for the patient together with morphine immediate release
c) Morphine is Rxed for the same patient by the same MD everytime
d) If morphine is Rxed with multiple part fill predated in the Rx.
319. For a new drug, if the manufacturer wants to protect his invention, he must ask for:
a) Patent
b) Trade mark
c) Copyright

320. In which reference we can find informations about Omega 3?


a) CPS
b) Martindale
c) Compendium of patient self care

321. A 75 years old man using bisphosphonate, broken chest ribs, what to do?
a) Add calcitonin
b) Change to Teriparatide
c) Add calcium and vitamin D

322. In insomnia, all are good to help for treatment of insomnia, except:
a) Decrease napping in daytime
b) Decrease caffeine, breverage and nicotine near bed tim
c) Increase exercise near bed time

323. Which opioid can give rapid addiction.


a) LA opioids
b) LA & SA opioids
c) LA opioids & LA benzodiazepines
d) LA opioids & methadone
e) LA opiolds & fentanyl

324. Alcohol withdrawal treatment, which one is used?


a) Alprazolam
b) Lorazepam
c) Diazepam
d) Midazolam
Benzodiazepines reduce the autonomic hyperactivity of GABA receptors that occurs during withdrawal. Also,
reduce agitation/tremor, hallucinations, seizures. Goal is suppression of symptoms with no more than mild
sedation. Administered based on CIWA‐Ar score. If CIWA‐Ar score increases and/or does not improve after 4
doses, refer to emergency department.
 Diazepam: CIWA‐Ar score ≥10: 20 mg Q1‐2H PO until symptoms resolve and CIWA‐Ar score <8 on 2
consecutive readings, at least 1 h apart.
 Lorazepam: preferred in elderly and patients with hepatic dysfunction. CIWA‐Ar score ≥10: 1–2
mg Q2–4H PO or SL until symptoms resolve and CIWA‐Ar score <8 on 2 consecutive readings, at
least 1 h apart

325. Father brings his son prescription to the pharmacist


Rx: Morphine 5mg/d Mite 60 tablets Repeat 3
Pharmacist has in stock syrup 5mg/ml, tablets10mg, 20mg. What pharmacist can do?
a) Give 30 tablets l0mg and tells father to give half a tablet/day
b) Give 10 tablets 20 mg and gives quater/day
c) Give 60 ml syrup 5mg/ml and tells father to give l ml/day

326. A man returns to pharmacy some expired medications: Lorazepam, Diazepam, Oxycodone,
Morphine, Codeine/Acetaminophen. By which way pharmacist can destroy these medications?
a) Destroy with other expired medications
b) Ask for official letter to destroy narcotics and destroy lorazepam and diazepam with
others
c) Ask for official letter to destroy benzodiazepines and destroy opioids with other
medications
d) Wait for answer of official letter from ragular and send narcotics and benzodiazepines for
destruction

327. Alcohol overdose can give:


a) Metabolic acidosis
b) Respiratory acidosis
c) Metabotic alkalosis
d) Respiratoty alkalosis

328. Obese lady has heart faileur and diabetes.she is on metformin 1000mg bid, she refuse to
use insulin and don’t want to be guinea pig her diabetes is not control, hbA1c > 8%. Dr and
pharmacist decide to talk her about her situation and show her medicationand give her
information about which ethical they uphold?
a) Beneficence
b) Autonomy
c) Paternalisim
d) Confidentiality
329. A patient with type 2 diabetes and HF using metformin but still diabetes not controlled
(HbA1c 8.2), what is appropriate recommendation?
a) Add pioglitazone
b) Add Gliclazide
c) Add acarbose
d) Add Sitagliptin

330. Pt 34 years old asthma since childhood he is suffering headache and he is on amitriptylline
Tylenol 3, Salbutamol inhaler, NRT patches 21 mg 4weeks,14 mg 4 weeks and 7 mg 2 weeks.
What is about his treatment
a) Drug drug interaction
b) Drug-disease interaction
c) Overuse of medication
General dosing instructions involve 6 wk of use of highest strength (21 mg for Nicoderm or Habitrol, 15 mg
for Nicorette) followed by 2 wk at the intermediate strength then 2 wk at the lowest strength. Peak: 2–6 h

331. What is wrong in this prescription


a) Amitriptylline
b) Tylenol 3
c) Salbutamol inhaler
d) NRT

332. A CHILD gets a prescription of medication at dose of 20mg/kg/day, nurse give him
infusion at rate 0.5 mg/kg/min. Available sachet containing l00ml. At which rate she can give
him the complete infusion
a) 8ml/h
b) 25ml/h 50ml/h
c) 75ml/h
d) 150 ml/h

333. BMI for a patient weighting 87 kgs and his heigh is 1.73 m
a) 26.7kg/m2
b) 28.4kg/m2
c) 29.4kg/m2
d) 30.4kg/m2
334. A Pt male with osteoporosis taken alendronate 70 mg/wk with hypogonadism, severe
cough and 3 ribs broken taken also ca 1200 mg and vitamin d 1000 iu what can dr. what can be
add?
a) Change Alendronate to Risedronate
b) Increase the dose of calcium and vitamin d supplements
c) Give testestron
d) Increase the dose of alendronate

335. All are autoimmune diseases except


a) Systemic lupus erythromatus
b) DM
c) MS
d) Osteoarthritis

336. What is not used in unrinary incontinence?


a) Thiazide

337. Patient have cancer, she stoped chemotherapy from 14 days, now she got fever, chill what
to do?
a) Tell her go to emergency
b) Give acetarminophen

338. Who cann’t write narcotic


a) Dr / veterinian / podiatrist
b) Dr / veterinian / dentist
c) Dr / veterinian / chiropractor

339. Depot (oily ampoule) can be given by?


a) IM
b) IV
c) SC
d) Intraarticular

340. Caution in use of medroxy progesterone if you have


a) Amenorrhea
b) Osteoprosis
341. All are se of amiodarone
except
a) Headache
b) Hypotension
c) Diarrhea
d) Skin discoloration
e) Corneal deposition

342. Pt has history of MI and now


has recent acute stroke attack he
is taking aspirin, what he should
take next
a) Warfarin
b) ASA + dipyridamole
c) Clopidogrel

343. What is the first treatment


for acute stroke?
a) ASA
b) Ramipril
c) Metoprolol

343. Pt get stroke 6 hrs ago why getting alteplase is not acceptable to him?
a) Because he got stroke 6 hrs ago

344. Who is responsible for pharmacy practice


a) Health Canada
b) Provinces
c) Colleges of pharmacy
NAPRA wasn’t in choices.

345. Father of 16 years old girl, she is taking contraceptive & now, he is asking if her daughter
taking contraceptives or not. What conflict the pharmacist is on?
a) Non maleficence and veracity
b) Autonomy and veracity
No confidentiality and veracity in choices
346. To decrease the load on the pharmacist what could deligate to the tech
a) Ordering medication
b) Labeling prescription
c) Labeling container

347. Which is not risk factor that increase NSAID renal toxicity
a) Hepatic disease and ascites
b) CHF

348. Which cause renal faileur


a) IV gentamycin

349. Storage of iron in body is determind by


a) Ferritin
b) Transferrin
c) TIBC
d) MCV

350. Most elemental iron is found in


a) Ferrous fumarate
b) Ferrous gluconate
c) Ferrous sulphate

351. All are correct about digoxin except?


a) May cause AV block
b) May cause ventricular arrhythmia
c) May cause atrial fiberillation

352. Pt taking tylanol 3 he has constipation and using docusate but no effect what to do?
a) Using senna

353. Which cause d-d interaction with clarithromycin


a) Digoxin

354. Woman get depression dr. prescribe sertralin all are true except
a) It takes 6 months to see response
b) If no response, switch to same class
355. Woman asked pharmacist, that she is afraid to go outside, she has urinary incontinence, she
went to Dr., and it is not accompanied with Cough or sneeze or excercise she is little pit stress
which type of Incontinence is this?
a) Urge
b) Stress
c) Over flow
d) Mixed

356. What is the treatment of cough with chest congestion?


a) Pseudoephedrine
b) Oxymetazoline
c) Guafenesin
d) Dextromethorphan

357. Symptoms of overdose of opioid are all of the


following except?
a) Hyperventillation
b) Pinpoint pupil
c) Blue lips

358. Patient have RA polyarticular, he is taking


voltaren, and another NSAID, he was stabilized for
certain time, but symptoms are getting worse,
asking for external
a) You should protect area around
b) Don't apply to wide area
c) Hot compress when added are very
beneficial

359. What is the ttt?


a) Sulfasalazine
b) Hyalurinidase
c) Etanercept

360. What is not ttt of trigeminal neuralgia?


a) Lamotrigine
b) Gabapentin
c) Carbamazepine
361. What is the side effect of lamotrigine?
Lamotrigine Blocks sodium channels, as well as high voltage-dependent calcium channels
Generalized tonic-clonic, absence, myoclonic, tonic/atonic, focal (partial) seizures or
unclassified tonic-clonic seizures
S.E: Rash 5–10%, insomnia, Hepatotoxicity and hematological toxicity.
Advantages: BID dosing; broad spectrum; no enzyme induction (few interactions); some
patients report feeling more “alert”; increasing evidence for monotherapy; increasing use for
primary generalized seizures.
Disadvantages: Very slow dose titration. Metabolism markedly inhibited by valproic
acid/divalproex sodium and increased by enzyme inducing AEDs (carbamazepine, phenytoin,
phenobarbital, primidone).
Addition of hormonal contraceptives may reduce lamotrigine serum levels by up to 50%.
Expensive at high doses. Only available in oral form.

362. What is the first choice of treating open angle glaucoma?


a) Timolol
b) Pilocarpine

363. Pt. Have bacterial blepharitis with swollen red eye what the ttt? Polymexin/amphotiricin

364. Dr. Ordered benzodiazepine 20 tablets, 5 times refill, patient want to transfer to another
pharmacy, what is correct?
a) You can transfer only 20 tablet 5 times
b) You can transfer 20 tablets only once

365. Tylenol # 3 is taken for how long if not mentioned by Dr.


a) TID when needed for 7 days
b) BID for 10 days
c) TID for 10 days

366. All are sympoms of hypothyroidism except


a) Weight gain
b) Oily skin acne
c) Constipation
d) Cold intolerance

367. All are used as prophylaxis of migraine except?


a) Nifidipine
b) Proporanolol
c) Topiramate
368. All are used to monitor COPD except?
a) Spirometer
b) CBC

369. Monitoring of alendronate?


a) Cr. Cl
b) BMD
c) Hypocalcemia

370. Loosing weight can help all except?


a) Prevent DM
b) Improve RA
c) Prevent degenerative bone degeneration

371. Progestrone is used in COC to?


a) Decrease breast cancer
b) Decrease endometrial cancer

372. All are true about tamoxifene except


a) Used to treat breast cancer.
b) Its progesterone and antiestrogen

373. All are symptoms of estrogen deficiency except


a) Nervousness
b) Breast tenderness
c) Vaginitis
d) Amenorrhea

374. Dr prescribed testesteron decanoate, all are true about testosterone decanoate except?
a) Can be prescribed verbally
b) Can be refilled
c) Can be transferred
d) Require record keeping for 2 years

375. Treatment of UTI take 3-5 days in some cases, where regular treatment 10 -14 days is for?
a) Acute cystitis
b) Hospital acquired acute UTI
c) Pyelonephritis
d) Prostatitis
e) Complicated UTI

376. PT took an overdose of dimenhydrinate and died what is the cause of death
a) Bronchospasm
b) Delirium
c) Bradycardia
Accidental antihistamine overdose occurs frequently in infants and children. Symptoms of dimenhydrinate toxicity in
children may resemble atropine overdosage, and include dilated pupils, flushed face, excitation, hallucinations,
confusion, ataxia, intermittent clonic convulsions, coma, cardiorespiratory collapse, and death. Symptoms may be
delayed up to 2 hours after ingestion; death may occur within 18 hours.

377. TPN for child


Glucose aa
k 40 mmole/l
Na
Dr prescribed to be given in ratte 6 ml/hr pharmacist wish to change dose of glucose and aa and
to maintain equivalent amount of K to be given at a rate of 14 ml/hr what the conc of k in new
formula
a) 7mmol
b) 30 mmol
c) 36mmol

378. ADHD pt, dr. prescribed methylphenidate, mother is concerned that her child might get
addicted, what is true?
a) Methylphenidate doesn't cause addiction
b) If not treated now, when he will grow, he will get addicted

379. 83 years old man have alzehimer, and he is agitated, drug of choice
a) Bupropion
b) Mirtazapine
c) Haloperidol
d) Respiridone

380. Pharmacist treat all patient with equality, what ethic is this?
a) Veracity
b) Justice
c) Beneficiance
d) Non-maleficiance
381. There is 10 mg/ml injection of a drug in hospital formularly, nurse want to add 2 mg/ml
injection in hospital formulary, what is the least concern for the pharmacist to consider adding
2mg/ml
a) Does the effectiveness of the drug will decrease if diluted?
b) Wastage of drug

382. When can the insurance company pay for the new prescription?
a) After 2 weeks
b) After 1 month
c) Immediately
d) After 3 days

383. pt using nitroglycerin patch for certain time now he got chest pain, what he does?
Keep the patch for 24 hrs

384. What is true about Spiriva


a) Contain pd for inh
b) Taken once daily as it has long t half

385. Which one need refrigerator?


a) Ketorolak
b) Viroptic
c) Iodoxuridin

386. RRR = 40 ARR = 8 What NNT?


a) 3
b) 5
c) 7
d) 13
e) 19

387. Patient is taking ramipril for HTN, now his blood pressure is controlled, he is asking if he
can take it every other day, because he forgetting to take. What is appropriate
a) Give him drug dispencer for the whole month
b) Tell him about the importance of taking the drug daily and consequence if not
388. Which vaccine is taken every 10 years?
a) Tetanus
b) Hepatitis B
c) Cholera
d) Mumps
e) Typhoid

389. pt taking haloperidol and get tremors and rigidity, all can be done except?
a) Add tranylcypromine
b) Add benztropine
c) Decrease haloperidol dose
November 2009
1. Ky 50-year-old man with alzheimer, agitation, bizarre behavior, two days back he had fought
with his wife vigorously and he has psychomotor symptoms, what is the best therapy for him?
a) Diazepam
b) Risperidone  Improves –ve symptoms and + ve symptoms
c) Haloperidol
d) Trazodone
e) Amitriptyline
With exception of clozapine, SGAs are now considered a first‐line treatment choice. Clozapine is reserved for
treatment‐resistant schizophrenia due to the risk of agranulocytosis and need for regular blood monitoring.

2. After treating with first generation anti psychiatric drug then patient develop EPS side effect,
all are the DOC of this symptoms, except?
a) Trihexyphenidyl
b) Procyclidine
c) Benztropine
d) Dantroline
Extrapyramidal side effects (EPS; dystonia, Prevention is key—use SGAs first-line.
parkinsonism, akathisia, tardive dyskinesia, tardive If EPS occurs, first reduce dose; consider switch to
dystonia, tardive akathisia) SGA if on FGA.
Prophylactic use of anticholinergics (benztropine,
procyclidine, trihexyphenidyl) is not recommended
even with FGAs, and should usually be used only on a
short-term basis to treat parkinsonism associated with
FGAs. Anticholinergics are generally not
recommended with SGAs
For akathisia: if dose reduction is not effective, beta-
Assessment: rating scales such as Simpson-Angus Scale, blockers (e.g., propranolol 10–120 mg/day) are the
Barnes Akathisia Scale or ESRS are useful to assess EPS treatment of choice with monitoring for hypotension.
and the Abnormal Involuntary Movement Scale or the Benzodiazepines also provide symptom relief.
ESRS is used to assess TD Anticholinergics are ineffective
Monitoring: baseline assessment in antipsychotic-naïve For acute dystonia (acute torticollis, oculogyric
first-episode patients, crisis): benztropine or diphenhydramine IM, followed
In multiple-episode patients when initiating a new by reduction in dose or switch to SGA
antipsychotic, and in firstepisode and multiple-episode For tardive dyskinesia: there is no evidence-based
patients whenever dosage of antipsychotic is changed; treatment— prevention is key.
assess weekly for 2–4 wk or until EPS resolves. Use SGAs first-line. Antiparkinsonian medications
In stable patients, assess for TD every 6 months or more are not effective and may worsen symptoms.
often in patients at higher risk (on FGAs, erratic If TD occurs, suggest consultation with a psychiatrist.
medication adherence or intermittent treatment, female, Consider switching to an SGA.
age > 55, substance abuse, diabetes) For persistent, severe TD, consider clozapine trial
3. A patient is taking moclobemide, atenolol and tylenol 1 for arthritic pain. he got cough and
severe congestion, needs something what is your recommendation?
a) Dextromethorphan
b) Pseudoephedrine
c) Xylometazoline
d) Gargle.

4. Now, she is on pseudoephedrine, after three days she feels muscle rigidity & hypertension.
She visited her doctor and saw her BP is 200/110. This is due to which of the following drugs:
a) Pseudoephedrine
b) Moclobemide
c) Atenolol
d) Dextromethorphan
Contraindications: Severe hypertension or severe CHD. Angle‐closure glaucoma or urinary retention.
Concurrent or recent (i.e., within 2 weeks) therapy with an MAO inhibitor.

5. What are these symptoms for?


a) Serotonin syndrome.
b) Hypertensive crisis.
c) Hypertension
d) Hyperglycemia
e) Hypothyroidism
Hypertensive crisis (severe ↑ in blood pressure > 180/110 that can lead to a stroke, can be treated by acute
administration of short‐acting oral agents, such as captopril, clonidine, or labetalol).

6. 6-years old boy is taking methylphenidate for ADHD what is true about it?
a) Can prescribe both oral and written with repeats
b) Has to keep sales report
c) Only written prescriptions are accepted not oral
d) It is controlled drug part-2
e) It is narcotic preparations

7. His parents are concerned about the medication addiction, what will you do in this situation?
a) Manage the interval of medication, which will help to release constant amount of effect
b) Tell parents to wait and watch without medication, if there is any improvement in the
symptoms. (no medication)
c) Tell parents it is better to start medication rather being worried about addiction of the
medication
d) Tell there is no addiction with stimulants
8. All are adverse effects of methylphenidate except?
a) Weight loss
b) Insomnia
c) Dizziness
d) Headache
e) Constipation
Common, usually transient: anorexia, insomnia, weight loss, irritability, dizziness, weepiness, headache,
abdominal pain. (monitor weight & appetite every 6 months)
Transient ‐ stop and re‐evaluate: “zombie‐like” effects, psychotic reactions (such as hallucinations),
agitation, tachycardia, hypertension, growth failure (Monitor growth suppression, record weight and height
at baseline and then every 3–6 months), rebound hyperactivity, leukopenia, blood dyscrasias.
Monitor patient for suicidal thoughts; consider a change in treatment if concerns arise.
Overdose symptoms ‐ stop and retitrate: “glassy eyes,” insomnia, hyperactivity. Significant: sudden cardiac
death reported; neurologic symptoms; exacerbation of tics; avoid in patients with a history of cardiovascular
conduction disturbances, hypertension, acute psychotic episodes and hyperthyroidism.

9. All of the following should be improved in ADHD treatment, except


a) Athlete performance
b) Academic performance.
c) Social involvement
d) Self-esteem.
e) Decrease restlessness
f) Increase impulsivity.

10. What is not symptom of alzheimer’s?


a) Language difficulties
b) Cognitive impairment
c) Ongoing loss of memory
d) Insomnia
e) Psychomotor symptoms
Cognitive Symptoms Non-Cognitive Symptoms Functional Sx.
(Present throughout the illness) (Behavioural symptoms, less Inability to care for
1. Amnesia: Memory Loss & Disorientation predictable) self:
2. Aphasia: Impairment of language ability 1. Depression  Dressing
3. Anomia: inability to remember names 2. Psychotic: E.g. Hallucinations  Bathing
4. Agnosia: Inability to recognize things/people etc. and delusions  Toileting
5. Apraxia: Unable to perform a task/movement 3. Behavioral disturbances: E.g.  Eating
when instructed when they understand and want to Aggression, uncooperativeness,
do said task wandering
6. Impaired sexual function
11. What lab test will monitor for alzheimer patient? Except.
a) Folic acid
b) Vitamin B12
c) Montreal Cognitive Assessment (MoCA)
d) Mini-Mental State Examination (MMSE)
e) Echoencephalogram

12. Which one is not a goal of alzheimer’s disease?


 Alter the natural disease progression to meet patient’s and caregiver’s goals
 Treat cognitive, behavioural and psychological symptoms
 Alleviate caregiver burden. Minimize medication side effects.

13. Patient started donepezil 10 mg, after one month, the patient developed severe nausea,
vomiting and diarrhea. What is your appropriate suggestion?
a) Decrease dose of donepezil
b) Stop donepezil
c) Switch to another drug.
The initial daily dose (5 mg) of donepezil is usually taken at night, but can be taken in the morning if sleep
disturbances occur. After 4 weeks, try increasing to the target dose of 10 mg/day. Adjust dose after 4 wk.
Donepezil was effective in 3‐ to 6‐month trials in patients with mild to moderate Alzheimer disease (MMSE s
core of 10 – 26).

14. Rivastigmine which of the following is true


a) Reassess in at least 3 months
b) Take on empty stomach
c) No drug interaction with CYP3A4
d) It is proven to be the best treatment for alzheimers dementia
Taken with food in divided doses in the morning and evening. The missed dose should be taken immediately
or at the next scheduled dose. Doses should not be doubled
Monitor effect in two weeks in the beginning, adjust every 4 weeks and duration of therapy from 3‐6 month
15. What is side effect of rivastigmine expect?
a) Dyspepsia
b) Nausea
c) Vomiting
d) Wt. Gain
e) Insomnia
Rivastigmine S.E:
>10%: headache, dizziness, nausea/vomiting, diarrhea, abdominal pain, anorexia.
<10%: fatigue, insomnia, syncope, dyspepsia, weight loss, UTI, rhinitis. Heart block (rare), delirium (rare),
seizures (rare).
Cholinesterase inhibitors as well as Alzheimer's disease can be associated with significant weight loss.
Patients may lose weight while taking cholinesterase inhibitors, including rivastigmine. Therefore, the
patient's weight should be monitored during therapy with EXELON PATCH.

16. What is the major side effect of sertraline?


a) Dry mouth
b) Somnolence
c) Drowsiness
d) Fatigue
Nausea, tremors, diarrhea, dry mouth, sexual dysfunction, increased risk of GI bleeding. Somewhat
stimulating, take with food to ↑absorp on, weak CYP2D6 inhibitor

17. Senior patient with MDD major depressive disorder, what is first line agent?
a) Amitriptyline
b) Desipramine
c) Citalopram
d) Bupropion

18. Patient now is suffering from sexual dysfunction. Doctor called you to check the best
medication for depression with the least sexual dysfunction:
a) Fluoxetine
b) Citalopram
c) Mirtazapine
d) Venlafaxine
e) Trazodone
f) Paroxetine
g) Amitriptyline
h) Bupropion
i) Moclobemide
19. He is now having insomnia with weight loss. Which of the following antidepressants you
will recommend?
a) Fluvoxamine
b) Meclobomide
c) Sertraline
d) Trazodone
e) Mirtazepine

20. Refer to dr for depression why?


a) 3 wks. of daily symptoms

21. Patient took much dimenhydrinate tablets, all are overdose symptoms of this drug except?
a) Bronchospasm
b) Delerium
c) Bradycardia
Accidental antihistamine overdose occurs frequently in infants and children. Symptoms of dimenhydrinate
toxicity in children may resemble atropine overdosage, and include dilated pupils, flushed face, excitation,
hallucinations, confusion, ataxia, intermittent clonic convulsions, coma, cardiorespiratory collapse, and
death. Symptoms may be delayed up to 2 hours after ingestion; death may occur within 18 hours. In adults,
500 mg or more of dimenhydrinate may cause extreme difficulty in speech and swallowing, and produces a
psychosis indistinguishable from that of atropine poisoning. CNS excitation may be preceded by sedation,
leading to a cycle of CNS excitation, seizures, and postictal depression.
Treatment: Treatment of dimenhydrinate toxicity is symptomatic and supportive. Emetics are usually
ineffective but in the absence of seizures, early gastric lavage (with an endotracheal tube with cuff inflate in
place to prevent aspiration of gastric contents) may be beneficial. Patients should be kept quiet, to minimize
CNS stimulation; seizures may be treated with diazepam in adults and phenobarbital in children (additional
methods may include IV sodium bicarbonate, or IV physostigmine salicylate in children). Mechanical
respiratory assistance may be required.

22. Dyspepsia when to refer? Melena


See your health‐care provider If you have?
 Frequent heartburn or stomach upsets or the symptoms last for more than 2 weeks. Your symptoms
should not require more than 2 weeks of continuous medication every 6 months. See your health‐care
provider if the symptoms do not go away after treatment.
 Severe abdominal pain or Pain that stays in the same spot on the side of your abdomen
 Unexplained weight loss of more than 3 kg in the past 6 months
 New feelings of shortness of breath or chronic tiredness
 Difficulty or painful swallowing, Persistent vomiting, Coughing up blood
 Blood in the stool or black stools that look like tar
23. All are side effects of moclobemide except?
a) Nausea
b) Insomnia
c) Dizziness
d) Restless leg syndrome
e) Edema
Moclobemide Short acting, reversible selective inhibitors of monoamine oxidase type A, the enzyme,
RIMA which inactivates the neurotransmitters, epinephrine, norepinephrine and dopamine.
Does not require the same dietary restrictions as irreversible MAOIs.
Moclobemide is a well-tolerated alternative to SSRI or SNRI agents, particularly in
patients with a significant anxiety component to their depressive episode.
S.E: Nausea, insomnia, dizziness, headache, abdominal fullness, GI upset, dry mouth.
Use with irreversible MAOIs may lead to potentially fatal reaction initially presenting with
tremor, agitation, hypomania, hyperthermia and/or hypertension.
Avoid sympathomimetics, meperidine; reduce dose with cimetidine.
Caution with opioids, antihypertensives, antipsychotics, SSRIs, selegiline, excessive
tyramine, alcohol.

24. Patient is taking bupropion for smoking cessation. But he is smoking occasionally when feel
craving. After two weeks he feels is not working, what is your advice?
a) Add nicotine patch
b) Add nicotine gum
c) Change to another class
d) Nonpharmacological therapy
e) Add Nicotine inhaler
Bupropion May be safely combined with NRT (monitor for treatment emergent hypertension). However,
based on the currently available evidence, the combination of bupropion and NRT cannot be recommended
for routine use.
Desire to Patient reports level of desire Intense craving may require alternate treatment.
smoke decreasing to minimal (or none) by Encourage behavioural changes to decrease desire;
end of therapy (3–6 months); empathize with patient’s difficulty and strongly
cravings may never completely end encourage perseverance.
for some.

25. All the following drugs are used for migraine headache except?
a) Topiramate
b) Valproic acid
c) Gabapentin
d) Riboflavin
e) Nifedipine
26. What is the shortest acting or fastest acting migraine preparation?
a) Raiment rizatriptan
b) Nasal zolmitriptan wafer
c) Ergot nasal preparation
d) Oral sumatriptan
e) Sumatriptan S.C

27. Patient is taking naratriptan & he wants to take additional dose, how can take it?
a) After 2 hrs
b) After 3 hrs.
c) After 4 hrs.
d) After 8 hrs.

28. Woman came with a prescription for Rizatriptan Wafer. Which of the following is a true
statement about Rizatriptan Wafer?
a) It is absorbed from the buccal cavity (its swallowed orally)
b) Co-administration with alcohol is contraindicated. (no interactions stated)
c) It is used for migraine with nausea (It causes nausea, it’s indicated for migraines with
or without aura)
d) It is contraindicated with people who have difficulty swallowing (it dissolves in the saliva
so very easy to swallow and absorb from stomach)
e) It is absorbed faster than Rizatriptan tablets
The bioavailability and Cmax of Rizatriptan were similar following administration of tablets and wafers, but
the rate of absorption is somewhat slower with wafers.

29. Oily injection can be given by which route?


a) IM
b) SC
c) IV
d) Intrathecal.
e) Intrajoint

30. Which is longest duration of action, among opioids?


a) Oxycodone
b) Fentanyl patch
c) Hydromorphone
d) Meperidine
e) Morphine
31. There is a significant risk associated with drug diversion in hospitals specially with
a) Mepiridine
b) Codeine
c) Fentanyl
d) Tramadol

32. Drug is used for post herpetic neuralgia, Except


a) Capsaicin topical
b) Lidocaine topical
c) Venlafaxine oral
d) Carbamazepin
e) Pregabalin

33. Carbamazepine used for trigeminal neuralgia, patient gets partial response, what is further
step?
a) Add gabapentin
b) Change to gabapentin
c) Add baclofen
d) Change to amitriptyline
If some relief is achieved with carbamazepine but side effects are unacceptable, a good strategy is to switch
to oxcarbazepine or reduce dose of carbamazepine to tolerability and add baclofen.
Gabapentin, pregabalin, clonazepam and valproic acid may be tried if other strategies fail.

34. Opioids are used for restless leg syndrome, what drug suitable for daily use?
a) Morphine
b) Propoxyphene
c) High dose of codeine
d) Methadone
Methadone is a mu opioid receptor agonist and NMDA antagonist given once daily for the treatment of
moderate to severe opioid use disorder.
Oral form once daily, long half‐life (36‐48 HRS). (Slow Withdrawal Symptoms).

35. Pt just diagnosed with open angle glaucoma, what to give:


a) Acetazolamide
b) Timolol  1st line agents
c) Pilocarpine
d) Apraclonidine
e) Brinzolamide
f) Latanoprost
36. Patient is on levodopa/carbidopa for parkinson disease, now he is experiencing wearing off
and dyskinesia both, what action should do?
Wearing off: Short duration response or "end dose" effect. The motor complications include "off periods" of
immobility or greater severity of the other parkinsonism symptoms and various abnormal movements. This
is due to decrease synthesis and storage of dopamine generated from endogenous or exogenous levodopa.

37. Drug of choice of absence seizure?


a) Ethosuximide
b) Phynetoin
c) Valproic acid
d) Leviteracetam

38. Patient has allergic conjunctivitis, what is doc? K type


a) Flurbiprofen drops
b) Antihistamine
c) Polysorbate drops

39. Bacterial conjunctivitis what can u give:


a) Oxymetazoline for the redness (E.D)
b) polymyxinB/gramicidin ED
c) Bacitracin (E.D)
For mild cases in adults: polymyxinB/gramicidin ED for 7–10 days and 2 days after symptoms resolved.
Moderate to severe, broad spectrum antibacterial trimethoprim/polymyxinB, ophthalmic erythromycin
ointments or bacitracin.
Tobramycin is sufficiently effective in the topical treatment of conjunctivitis.
Ciprofloxacin, gatifloxacin, moxifloxacin, ofloxacin for serious infections such as bacterial keratitis.
40. Hypothyroidism patient & start taking levothyroxine. You will counsel him about which of
the following?
a) Take empty stomach and remain in upright position
b) You will get effect after two weeks.
c) Give 4 - 6 hours gap between ibuprofen & levothyroxine.
d) If you experience flue like symptoms, see your doctor.
Peak therapeutic effect at a given dose of levothyroxine sodium may not be attained for 4‐6 weeks
Absorption may be reduced by antacids, Ca salts, cholestyramine (separate administration by at least 6 h),
colestipol, iron salts.
Toxic symptoms are nervousness, palpitation, intolerance to heat.

41. TSH going down 0.1 (normal 0.4 to 0.6). What action should be taken?
a) Decrease the dose of levothyroxine.
b) Increase the dose of levo
c) Stop levothyroxine for 2 weeks, and resume again.
If there is no severe symptoms i think reduce the dose is good option. LT4 t1/2 is 5 weeks, so C is wrong

42. All are the symptoms of hypothyroidism except:


a) Oily skin
b) Wt. Gain
c) Memory loss
d) Constipation.
e) Cold sensitivity.
Symptoms: fatigue, impaired memory, constipation, cold intolerance, changes in skin or hair, weight gain,
appearance (coarse features, dry skin and hair), hypertension, bradycardia, delayed relaxation phase of
reflexes, extreme cases which may present with myxedema coma, hearing loss and Decreased libido.

43. In the above patient, which lab one goes down?


a) Ft4
b) t3
c) TSH.

44. Patient is type 1 diabetic and strong family history how often check HbA1C test?
a) Every 6 months
b) Every 3 months
c) Every 1 month
 HbA1c, a measure of glycated hemoglobin which serves as an indicator of blood glucose control in the
prior 3–4 month. In all patients who have been started on antihyperglycemic agents, aim to reach the
desired HbA1c target within 3–6 months through dosage titration or addition of other agents
 For patients on insulin: check every 3‐4 months
 For patients on oral antihyperglycemic or nutritional therapy: check every 6 months.
 A higher HbA1c of up to 8.5% may be more appropriate if the risk of hypoglycemia outweighs the
benefits of tight control, e.g., in frail elderly patients, those with limited life expectancy or patients with
a history of recurrent severe hypoglycemia
 HbA1c measurements every 3 months for patients who have not achieved target values; testing every 6
months may be acceptable in stable patients who consistently meet glycemic targets.

45. All can be done for sick day management except


a) Reduce calorie intake or avoid food contain carbohydrate
b) Test your blood glucose and urine ketones every 2 hours, 24 hours a day.
c) If there are symptoms of dehydration, call 911.
d) Hold SADMANS medications

46. What is true about glucagon?


a) Can be used for beta blocker toxicity
b) Indicated for emergency treatment of severe hypoglycemia
c) Active when taken orally or parenterally
d) Induce acetylcholine
e) Improve appetite
GLUCAGON (glucagon for injection, rDNA origin) is a polypeptide hormone identical to human glucagon
which is manufactured by recombinant DNA technology and has the same molecular structure as animal
sourced glucagon. Glucagon causes an increase in blood glucose concentration. Glucagon acts only on liver
glycogen, converting it to glucose. Parenteral administration of glucagon relaxes smooth muscle of the
stomach, duodenum, small bowel, and colon.
Glucagon is inactive orally because it is destroyed in the GIT before it can be absorbed. Can’t give in adrenal
/pancreatic tumor
GLUCAGON (glucagon for injection, rDNA origin) should be given only if patients are unconscious or
unresponsive and unable to ingest oral glucose. After intramuscular injection, the patient will normally
respond within 10 minutes. If the patient does not respond within 10 minutes, intravenous glucose must be
administered as soon as an IV access can be established. Because glucagon is of little or no help in states of
starvation, adrenal insufficiency, or chronic hypoglycemia, intravenous glucose should be used for treatment
of hypoglycemia in those conditions

47. Patient is type 2 diabetic, he is taking glyburide 7.5mg, metformin 1000mg twice daily, still
his blood sugar level is not control, what you will do best?
a) Increase glyburide dose
b) Increase metformin dose
c) Add acarbose
d) Add insulin
e) Add thiazolidinediones
The initial dose is 5 mg daily (2.5 mg in patients over 60 years of age) administered with breakfast or a first
meal and should be continued for 5 to 7 days.
Depending on the response, the dosage should then be either increased or decreased by steps of 2.5 mg. The
maximum daily dose of GLYBURIDE is 20 mg (higher doses normally have no additional effect on control of
metabolic state). Occasionally, control is maintained with 2.5 mg daily. The majority of cases can be
controlled by 5 to 10 mg (1 to 2 tablets) daily given as a single dose during or immediately after breakfast.
Patients who eat only a light breakfast should defer the first dose of the day until lunchtime. If more than 10
mg (2 tablets) daily is required, the excess should be taken with the evening meal. It is very important not to
skip meals after the tablets have been taken.

48. After three months above patient face side effects as edema, wt. Gain and leg swollen, what
drug would cause these side effects?
a) Glyburide
b) Metformin
c) Insulin
d) Pioglitazone
e) Acarbose

49. A patient is taking NPH/regular insulin as follows:


time of dose NPH dose regular insulin
7am 12u 6u
5am 6u 6u
blood sugar measurement gives the following results:
7am 11 – 15 mmol/l
5am 3 – 5 mmol/l
what to do?
a) Decrease NPH dose in morning and increase regular insulin dose in the afternoon.
b) Increase NPH dose in morning and decrease regular insulin dose in the afternoon.
c) Decrease NPH dose in morning and decrease regular insulin dose in the afternoon.
d) Increase NPH dose in morning and increase regular insulin dose in the afternoon.

50. All are side effects of chlorpromazine, except?


a) Glucose intolerance.
b) Galactorrhea.
c) Photosensitivity.
d) Orthostatic hypotension.
e) Dry mouth
Dopamine All act as a dopamine antagonist or by enhancing gastric emptying.
Antagonists Q. Chlorpromazine historically has been the drug of choice for persistent hiccups.
Chlorpromazine Parenteral: 25–50 mg IV over 30–60 min. Oral: 25–50 mg TID – QID PO × 2–3 days
S.E: Anticholinergic effects, extrapyramidal effects, hypotension, sedation.
Additive sedative effects with CNS depressants, including alcohol.

51. All of the following agent can used in pregnancy? Except


a) Folic acid
b) Vit. B12
c) Calcium
d) Beta carotene
e) Iron

52. Sexually active woman presents to pharmacy asking for advice on how she treat her white
creamy vaginal discharge. Which of the following is not cause of her symptoms?
a) Diabetic
b) Using OCP
c) Sexually active
d) Vaginal douching

53. Lady want to go for vacation and working overtime for money. She has sedentary lifestyle
and no medical condition. Height 5' 6" and weight is 59 kgs. One of her friends told her to take
orlistat tab for wt. Reduction. What is following statement is true?
a) Give orlistat
b) She is not eligible to take orlistat
Indications for the use of apply to obese patients with a BMI* ≥ 30 kg/m2 or a BMI ≥ 27 kg/m2 in the
presence of other risk factors (e.g., hypertension, type 2 diabetes, dyslipidemia, excess visceral fat).

54. As a pharmacist you’ll discuss with her all of the following except
a) Her diet
b) Cost/affordability
c) Encourage walking and dietary
modification

55. Patient has coronary artery disease with


risk factor, we have to control LDL
cholesterol, what is the limit?
a) Less than 5
b) More than 2
c) Less than 3
d) Less than 2
56. Which drug should take with food? K type
a) Lovastatin
b) Fluvastatin
c) Pravastatin

57. Drug increase warfarin level except?


a) H2 blockers

58. Patient BP reading-greater than 140/90 with risk factor, how many times to check doctor?
a) Three office visits
b) Go to emergency
c) Two office visits
d) Four office visits
e) Five office visits
140 ‐ 160 + no risk factors ‐‐‐‐ 5 visits 140 or more + risk factors ‐‐‐‐ 2 visits
160 ‐179 + no risk ‐‐‐‐ 3 visits 180 or more ‐‐‐‐ 1 visit

59. What drug can be used in heart failure patient except?


a) Metoprolal
b) ACE inhibitor
c) ARBS
d) Verapamil
e) Spironolactone

60. DOC for BP in pregnancy patient?


a) Hydralazine
b) Captopril
c) Irbesartan
Methyldopa, Labetalol, Oxprenolol, acebutolol, Pindolol, Propranolol, Metoprolol, Nifedipine
XL, Clonidine, Hydralazine

61. 42 years old patient with chronic stable angina, he is on nitroglycerine patch. He is also
taking ACEI and ASA, what will you give to this pt.?
a) Beta blocker
b) Warfarin
c) Statin
d) Increase dose of ACEI
62. The patient comes after a few months because his angina is not controlled all of the
following are appropriate option except?
a) Tells him to wear the patch 24 hours a day
b) Add metoprolol
c) Add nifedipine
d) Increase nitroglycerin dose

63. Patient is on betablocker for heart failure and patient has diabetics. What is best option
among betablocker?
Acebutolol

64. After parking his car patient walks 200 yards feels dyspnea and fatigue, and diagnosed as
left heart failure. What is NYHA functional classification?
a) Class 1
b) Class 2
c) Class 3
d) Class 4

65. All can increase digoxin toxicity


except?
a) Renal failure with no dose
adjustment
b) Hepatic failure with no dose
adjustment
c) A decrease in cognitive behavior
which affects patient’s self-care
d) No monitoring
66. Digoxin adverse effects all except?
a) Atrial fibrillation
b) Ventricular arrhythmia
c) Visual disturbance
d) Heart block

67. Pt on HCTZ, ACEI and digoxin, what is required to monitor in this pt.
a) Potassium
b) Increase urination
c) Increased water in body

70. Monitoring on digoxin, except?


a) LFT
b) CBC
c) Renal function test
Patients receiving TOLOXIN (Digoxin Tablets, C.S.D. and Digoxin Oral Solution, C.S.D.) should have their
serum electrolytes and renal function (BUN and/or serum creatinine) assessed periodically; the frequency of
assessments will depend on the clinical setting. The use of therapeutic doses of digoxin may cause
prolongation of the PR interval and depression of the ST segment on the electrocardiogram. Digoxin may
produce false positive ST‐T changes on the electrocardiogram during exercise testing. These
electrophysiologic effects reflect an expected effect of the drug and are not indicative of toxicity.

71. Pt with acute. NSTEMI what will be the initial step to be done to this pt.
a) ASA
b) Betablocker
c) ACEI
d) Alteplase
e) LMWH

72. Pt with acute ischemic stroke from 6 hours now in hospital. Pt was 60 years old, having BP
and cholesterol. What is the reason we cannot give him alteplase at this time?
a) Age
b) Symptoms from 6 hours
c) BP
d) Cholesterol
e) Medication used before
STEMI: Give early in STEMI within 6 hours for maximum reduction of mortality/morbidity.
STROKE: No significant benefit was demonstrated when treatment was delayed by more than 4.5 hours.
73. same pt, now his intracranial hemorrhage is excluded by CT scan. What will be added to
this patient’s therapy
a) ASA
b) Warfarin
c) B blocker
d) Ticlopidine
e) Heparin
CTC: If intracranial hemorrhage is excluded by CT scan but alteplase is not indicated, give ASA
160 mg immediately. Follow with ASA 80–325 mg daily.

74. Pt with CHF which of the following medication is used to decrease mortality in this pt.
a) ACEI
b) CCB
c) ASA
d) Spironolactone

75. Patient has atrial fibrillation, what is DOC maintenance for post MI?
a) Aspirin
b) Warfarin
c) Aspirin + dipyridamole
d) Aspirin + clopidogrel
e) Clopidogrel

76. Side effect of amiodarone except?


a) Gastric reflux & Occult blood in stools
b) Visual disturbance
c) Headache
d) Pulmonary fibrosis
e) Peripheral skin pigmentation

77. Which one should not be taken with amiodarone?


a) Metoprolol
b) Ramipril
c) Pravastatin
With amiodarone: Reduce doses of BBs, digoxin, procainamide, quinidine and warfarin by 50%.
78. For monitoring amiodarone, which test is not necessary?
a) LFT
b) Chest x-ray
c) Hypokalemia
d) Renal function
Monitor LFT transaminases and Serum TSH Q 6m, eye exam, Chest X‐Ray at baseline & annually
No data on dosage adjustment is available for the oral formulation. Based on a single‐dose clinical trial with
the intravenous formulation, no dosage adjustment is required for patients with renal dysfunction, end‐
stage renal disease or dialysis.

79. Which is not an overdose symptom of amiodarone?


a) Hypotension
b) AV block
c) Hepatotoxicity
d) Bradycardia
e) Nephrotoxicity
There have been cases, some fatal, of amiodarone hydrochloride overdose. Overdose may lead to severe
bradycardia and to conduction disturbances with the appearance of an idioventricular rhythm, particularly in
elderly patients or patients on digitalis therapy.
One report of the acute ingestion of a single 8 g dose of oral amiodarone hydrochloride by a healthy 20‐year‐
old female has been reported. At first assessment, the patient was conscious and profuse perspiration and a
slight tachycardia were the only abnormal findings on clinical observation. Slight bradycardia was observed
during the second and third day; thereafter, QT interval and heart rate returned to normal. No clinical adverse
events were documented over the subsequent 3‐month monitoring period.
Intravenous Amiodarone
There have been cases, some fatal, of amiodarone overdose. Effects of an inadvertent overdose of
I.V. amiodarone include hypotension, cardiogenic shock, bradycardia, AV block, and hepatotoxicity.
Hypotension and cardiogenic shock should be treated by slowing the infusion rate or with standard therapy:
vasopressor drugs, positive inotropic agents and volume expansion. Bradycardia and AV block may require
temporary pacing. Hepatic enzyme concentrations should be monitored closely. Neither amiodarone nor DEA
is dialyzable.
Overdosage Management
If an overdose should occur, gastric lavage or induced emesis should be employed to reduce absorption, in
addition to general supportive measures. The patient's cardiac rhythm and blood pressure should be
monitored, and if clinically significant bradycardia ensues, a ß‐adrenergic agonist or a temporary pacemaker
should be used. Hypotension with inadequate tissue perfusion should be treated with positive inotropic
and/or vasopressor agents. Neither amiodarone nor its metabolite is dialyzable.

80. Question about 12 lead ECG?


81. Risk factor of venus thromboembolism except?

82. Patient has COPD in the last three months, he has CHF and peripheral vascular deficiency,
now he is experiencing pain, left leg swollen and edema. What would cause these symptoms?
a) Intermittent claudication
b) Pulmonary edema
c) Raynaud’s phenomenon
d) DVT and emboli
e) Left ventricular dysfunction

83. What is the first line agent for raynaud’s phenomenon?


a) Nonpharm
b) Nifedipine xl
c) Diltiazem
Dihydropyridine First-line agents in patients with primary and secondary Raynaud phenomenon and they
CCBs: reduce the frequency rather than the severity of attacks
Amlodipine Nifedipine, extendedrelease Adalat XL: 30 mg PO 30–60 min before cold exposure
Felodipine Felodipine, extendedrelease Plendil: 5– 10 mg PO 60 min before cold exposure
Nifedipine Amlodipine Norvasc 5 mg PO 60 min before cold exposure
Use 30−60 minutes before cold exposure, or regularly during the winter months.
If not well tolerated, nondihydropyridine CCBs may be considered, but less effective.
Higher doses may be used in patients with severe primary or secondary Raynaud
phenomenon, or if ulcers are present, e.g., systemic sclerosis.
84. What are precipitating factors of asthma except?
Precipitating factors:
Environmental allergens, occupational exposures. Irritants such as smoke, exercise, cold air.
Drugs such as ASA or BBs, preservatives as sulfites. Viral respiratory infections, rhinitis, sinusitis, GERD.

85. Non pharmacological treatment of asthma all except


a) Open all window during summer seasons
b) Encouraging smoking cessation to the patient and family members if smokers
c) Keeping the pet outdoors and restricting them to very few rooms

86. Acute exacerbation of asthma came to ER of hospital. What will be the initial treatment
given to this patient in order to relieve his symptoms?
a) Salmeterol inhaler
b) Terbutaline inhaler
c) Ventolin tab
d) Corticosteroids inhaler
e) Ipratropium

Another version:
87. Adult pt. With acute asthma in
hospital
a) Methyl prednisone
b) Bronchodilators inhaler
c) Prednisone po.
d) Hydrocortisone iv
e) Ipratropium inhaler

88. While discharging the above pt.


What will you tell him to take in order to
prevent asthma exacerbation during
exercise, he is on terbutaline prn and
beclomethasone inhaler?
a) Take terbutaline before exercise
b) Take low dose prednisolone
c) Add salmeterol
d) Add prednisone
e) Increase dose of beclomethasone
f) Give combination of SABA+ ICS, with salbutamol prn
89. A patient has asthma was taking salbutamol, fluticasone and salmetrol MDI. She presents
with a prescription for clarithromycin 500mg bid to be used at the onset of action, also she has a
prescrtiption for advair diskus. All of the following are necessary for the pharmacist to assess
the patient’s case and fill the prescription except?
a) Drug allergies
b) Previous chest infections
c) Technique of previouly inhaled medications
d) Need for addition of new inhaler

90. Kid with asthma, taking salbutamol and fluticasone, admitted to hospital with an acute
exacerbation, after discharge, give him at home
a) Prednisone
Child with asthma not controlled what is true statement except?
a) Child having one inhaler in home and another in school

91. A case about a patient using Ipratropium. The patient asks you about Tiotropium as an
alternative, what should you tell the patient?
a) Long acting so used once daily
b) It is more effective in asthma than COPD
c) Reduces the number of COPD exacerbations
d) Improves survival

92. About advair diskus device what is wrong


statement?
a) Wash mouth piece with wet clothes
b) Rinse mouth and gargle with H2O after each
dose to prevent occurrence of candidiasis
c) Once you hear a click the dose is loaded
d) It contains a dry powder medication
e) Contains counter

93. pt with COPD now diagnosed with CAP, what


would be the DOC in this pt for CAP?
a) Azithromycin
b) Ciprofloxacin
c) Cephalexin
d) Amoxicillin
Organism Recommended Antibiotics
Streptococcus Penicillin nonresistant (Minimum inhibitory concentration MIC <2 mg/L):
pneumoniae  Initial therapy: penicillin G, amoxicillin
 Alternatives: macrolide, PO cephalosporins (cefprozil, cefuroxime), IV cephalosporins
(cefuroxime, ceftriaxone, cefotaxime), clindamycin, doxycycline, respiratory
fluoroquinolones
Penicillin resistant (MIC ≥2 mg/L):
 Initial therapy: cefotaxime, ceftriaxone, respiratory fluoroquinolone (PO or IV)
 Alternatives: vancomycin, linezolid, H.D amoxicillin (3 g/day for penicillin MIC ≤4 mg/L)
Haemophilus 2 or 3rd generation cephalosporin, amoxicillin/clavulanate, fluoroquinolones, doxycycline,
nd

influenza azithromycin, clarithromycin; amoxicillin monotherapy if nonbeta-lactamase producing


Staphylococcus  Methicillin-susceptible MSSA: cloxacillin, cefazolin, clindamycin
aureus  Methicillin-resistant MRSA: vancomycin, linezolid, tigecycline
Legionella sp. Legionnaires’ disease - severe form of pneumonia
 Fluoroquinolones or azithromycin; Alternative: doxycycline
Spread by human-made building water systems.
M. pneumoniae  Macrolides or tetracyclines
C. pneumoniae  Alternative: fluoroquinolones
Coxiella Cause Q fever - mild disease with flu-like symptoms.
burnetii  Doxycycline or fluoroquinolones;
 Alternative: macrolides (although some strains may be resistant)
Pseudomonas  Antipseudomonal beta-lactam Imipenem/cilastatin plus ciprofloxacin or aminoglycoside;
aeruginosa  Alternative: aminoglycoside plus ciprofloxacin

94. Which of the following tests is not proper for pt. With COPD and just release from hospital
due to pneumonia.
a) Sputum analysis
b) Peak flow meter test
c) Chest x ray
d) Blood analysis
I think C, it takes time to return to normal so no need to check when discharge from hospital.

95. Management of ascites patient is using spironolactone, but it is not effective, what you will
add for relieve syndrome?
a) Furosemide
b) Metolazone
c) Amiloride
Drugs Comments
Q. The diuretic of choice for high plasma aldosterone levels in patients with ascites that cause
Spironolactone sodium and fluid retention.
Initial: 100–200 mg/day PO Increase Q5–7 days to 400 mg/day PO
S.E: Hyperkalemia, hyperchloremic metabolic acidosis, gynecomastia and mastalgia in men.
Furosemide Helps to control serum potassium levels. Combination of sprionolactone and furosemide may
enhance diuresis versus spironolactone alone.
Initial: 40 mg/day PO Increase by 20–40 mg/day to achieve diuresis (up to 160 mg/day)
S.E: Nausea, anorexia, fatigue, weakness, decreased serum Na+, Cl-, K+, Mg++,
hyperuricemia, hyperglycemia, volume depletion, metabolic alkalosis, rash.
Q. Can be added if ascites is refractory to spironolactone and furosemide.
Metolazone Combination of furosemide and metolazone can produce profound diuresis, causing volume
depletion and electrolyte abnormalities, e.g., hypochloremic metabolic alkalosis, hypokalemia.
Amiloride Can be substituted for spironolactone if intolerable side effects develop.
S.E: Hyperkalemia, gynecomastia, muscle cramps, hyperchloremic metabolic acidosis.
Paracentesis The procedure is used to remove fluid from the peritoneal cavity, particularly if this cannot be
achieved with medication. The procedure is not painful and does not require sedation.

96. Side effect of pig interferon?


Immunomodulators Used in patients who persistently have lower HBV DNA levels & elevated serum
Peginterferon alfa-2a aminotransferases.
Not active orally, but S.E: Flulike syndrome, bacterial infections, fatigue, fever, muscle aches, asthenia, weight
administered SC, IV, loss, diarrhea, nausea, headache, irritability, depression, anxiety, difficulty concentrating
Intralesionally. and sleeping, memory loss, hair loss, retinopathy (particularly in those with preexisting
hypertension), soreness and redness at injection site, neutropenia, thrombocytopenia.
ONCE WEEKLY Neutropenia (<0.75 × 109/L) and thrombocytopenia (<50 × 109/L) are managed by dose
reductions.
Contraindications: pregnancy, decompensated liver disease, severe cardiac disease, solid
organ transplant (except liver), ongoing/untreated alcohol abuse or injection drug use,
severe/untreated major depression or psychosis, renal failure& acute hepatitis B.

97. What virus transmission through fecal-oral route?


a) Hep B
b) Hep A
c) Hep C
HAV HBV HCV HDV HEV
Transmission Fecal, Percutaneous, Percutaneous As HBV Fecal,
Oral route Sexual contact, Perinatal Oral route
Perinatal, blood & Blood
other body fluids. transfusion.

98. Patients suffer from moderate crohns disease, he is on prednisone, after 4 weeks no
remission occurs. What you will add along this drug?
a) Add methotrexate
b) Add infliximab
c) Continue for 4 months
d) Change to IV steroids
99. Now, his condition improved. What to be give to maintain remission?
a) Cyclosporine
b) Infliximab.
c) Methotrexate

100. What is drug of choice for


irritable bowel syndrome with
constipation?
a) Loperamide
b) Psyllium
c) Amitriptyline
d) Senna
e) Eluxadoline

101. Patient has BPH and hypertention, what is the monotherapy? (confusing scenario)
a) Finasteride
b) Captopril
c) Terazosin
d) Metoprolol
102. BPH pt should avoid? TCA colmipramine.
Risk Factors
 Aging: Not common in men < 40 years of age
 Family history: Having a close male relative (father or brother) with prostate problems
 Medications: Antihistamines (Decreased parasympathetic tone), Decongestants (Increased sphincter
tone via alphaadrenergic receptor stimulation), Diuretics (Increased urine production), Opiates
(Impaired bladder contractility), Tricyclic antidepressants (Anticholinergic effects).
 African Americans/Canadian males, Caribbean descent.
 Comorbidities: Diabetes, heart disease & obesity can increase risk

103. 59-year-old MALE, who suffers from Parkinsonism, complains of urinary incontinence for
the last 3 years now. He voids 12 to 15 times a day and has to get up 3 to 4 times at night as
well. He complains of not being able to get to the washroom. He had a stroke 2 years back and
has mild hypertension currently. He takes 6 to 8 cups of coffee a day, and doesn’t consume
fluids at all. What type of incontinence is suffering from?
a) Stress
b) Overflow incontinence
c) Functional incontinence
d) Mixed incontinence
e) Urge incontinence
Involuntary leakage of moderate to large amounts of urine due to inability to delay voiding when urge. Causes
include bladder wall hyperactivity or instability and CNS disorders (e.g., parkinsonism, stroke, spinal cord
injury, multiple sclerosis).

104. Which of the following causes urinary incontinence?


a) Furosemide
b) Hydroxyzine
c) Levodopa/Carbidopa
d) Respiredone

105. What is drug of choice for the above patient?


a) Oral estrogen
b) Oxybutynin
c) Estrogen cream
d) Imipramine
e) Nonpharm. Bladder training (timed voiding)

106. Absolutes contraindication to combined oral contraceptive pills?


Q. Absolute Contraindications to Combined Oral Hormonal Contraception:
 Breast cancer or hormone‐dependent cancer
 Cerebrovascular disease, history of cerebrovascular accident
 Complicated valvular heart disease
 History of VT, PE, known thrombogenic mutations e.g., factor V Leiden; prothrombin mutation; protein S,
protein C, antithrombin deficiencies; or other known coagulation‐factor deficiency, MI or vascular disease.
 Diabetes with microvascular complications
 < 6 weeks postpartum if breastfeeding
 Migraines with aura at any age. Hypertension (SBP ≥160 mm Hg or DBP ≥100 mm Hg)
 Severe cirrhosis or liver tumor. Smoker >35 years of age (≥15 cigarettes/day)

107. Treatment for acne with contraception?


a) Additional use of barrier method

108. What is the use of medroxyprogesterone oral pill?


a) Dysmenorrhea
b) Decreases endometrial cancer
c) Decreases breast cancer
PROVERA (medroxyprogesterone acetate) is indicated for:
 Hormonal replacement therapy, to oppose the effects of estrogen on the endrometrium and
significantly reduce the risk of hyperplasia and carcinoma;
 Functional menstrual disorders due to hormonal imbalance in non‐pregnant women, in the absence of
organic pathology;
 Adjunctive and/or palliative treatment of recurrent and/or metastatic endometrial carcinoma;
hormonally ‐ dependent, recurrent metastatic breast cancer in postmenopausal women.
 For indications not including breast cancer, PROVERA should be prescribed only to women with intact
uteri.

109. DOC for


endometriosis?
a) GnRH
b) Estrogen
c) Danazol
d) Clomiphene

110. What are menopause


symptoms except?
a) Flushing
b) Night sweats
c) Vaginitis
d) Mood changes
e) Breast tenderness
Vasomotor symptoms, commonly known as hot flashes and night sweats, are the most common presenting
complaint of menopausal women.
Other associated complaints of menopause, including vaginal symptoms such as dryness, itchiness, vaginitis
and dyspareunia, generally persist or worsen with aging due to low estrogen levels. Some women notice
decreasing libido and an alteration in sexual function with the onset of menopause.
Sleep disturbances (difficulty sleeping, fragmented sleep), mood changes (depression, anxiety), headaches and
joint pain have also been reported in the menopausal transition and may continue after menopause.

111. J.G. is a 25-year-old female (body weight: 80 kg) who presents to the pharmacy requesting
for Plan-B® (Levonorgestrel) for emergency contraception following an episode of unprotected
sex 72 hours ago. After speaking with J.G., the pharmacist determines she is a good candidate
to receive Plan-B®. All of the following counseling information for J.G. can be provided by the
pharmacist, EXCEPT:
a) If you vomit within 2 hours of taking Plan B®, an additional dose might be warranted.
b) You might experience a breakthrough bleeding a few days after taking Plan-B®.
c) Plan-B® dose should preferably be doubled in females weighing ≥ 80 kg who need
an immediate emergency hormonal contraception.
d) Plan-B® can be safely used more than once per cycle.

112. Patient is using tadalafil for what condition refer patient to doctor?
a) Headache
b) Flushing
c) Extended priapism
d) Dyspepsia

113. Which one is not used in acute gout attack?


a) Sulfinpyrazone
b) Naproxen
c) Colchicine
d) Prednisolone.
NSAIDs, colchicine or oral corticosteroids are appropriate first‐line options in therapy of acute gout.

114. Sulfinpyrazone is used as


a) Uricosuric
b) Anti-inflammatory & Analgesics
c) Tubular reabsorption of uric acid
d) Urinary antiseptic.
CTC: While officially approved as a uricosuric, sulfinpyrazone is rarely used for treating gout as it requires
monitoring for blood dyscrasias and has other noteworthy adverse effects including increased risk of bleeding
and skin rashes.
115. Patient on allopurinol; you should monitor?
a) Rash
Monitoring and Laboratory Tests
CBC, liver and renal function tests (especially at start of therapy), serum uric acid concentrations every 2–5
weeks during dose titration phase of treatment. Monitor for signs and symptoms of hypersensitivity reaction
(e.g., rash, pruritis, elevated hepatic transaminases) and hydration status.
Occupational Hazards: Drowsiness may occur. Patients should be cautioned not to engage in activities where
alertness is mandatory until their response to the drug is known.

116. Which is not a risk factor for hyperuricemia?


a) Higher amounts of meat and sea food
b) Niacin
c) Underweight
d) Smoking/alcohol
Drugs and Conditions Associated with Hyperuricemia and Gout
 Drugs: Alcohol, Cyclosporine, Cytotoxic chemotherapy, Diuretics (thiazide and loop), Ethambutol,
Interferon + ribavirin, Levodopa, niacin, Pyrazinamide, Salicylates (low‐dose), Tacrolimus, Teriparatide.
 Conditions: Excessive alcohol intake, Atherosclerosis, Chronic kidney, glomerular, interstitial renal
disease, Diabetes, Hyperlipidemia, Hypertension, Ischemic heart disease, Lead intoxication, Metabolic
syndrome, Myeloproliferative disorders and some cancers, Obesity, Urolithiasis history, rarely genetic
or acquired causes of uric acid overproduction.
 Dietary factors: excessive protein diet from red meats, organ meats & shellfish,  Purine intake
(shellfish, vegetables such as asparagus, cauliflower, spinach, beans, peas & mushrooms)

117. Patient has RA with poly


joints, he applied capsaicin
cream; but not effective. what
is the first line therapy for
him?
a) Corticosteriods
b) Sulphasalazine
c) Combination therapy
d) Methotrexate
MTX: Recommended to be the
drug of first choice or part of the
initial treatment strategy.
118. Risk factors of osteoporosis?
Older Adults (≥50 y) Younger Adults (<50 y)
Age ≥65 y Fragility fracture, Prolonged use of
Clinical risk factors for fracture (men age 50–64 y, corticosteroids, use of other high-risk
menopausal women): vertebral compression, fracture fragility, medications, e.g., aromatase inhibitors,
fracture after age 40, prolonged use of corticosteroids, use of androgen deprivation therapy, Hypogonadism
other high-risk medications, e.g., aromatase inhibitors, or premature menopause (<45 y),
androgen deprivation therapy, parent with hip fracture, Malabsorption syndrome, Primary
osteopenia identified on x-ray, current smoking, high alcohol hyperparathyroidism, Other disorders strongly
intake, low body weight (<60 kg) or major weight loss (>10% associated with rapid bone loss or fracture.
of weight since age 25), rheumatoid arthritis. Race (asian & caucasian).

119. Management of sports injuries?


Acute treatment is best summarized by the RICE protocol:
 Rest injured part for short time period; mobilize when feasible as early mobilization aids healing.
 Ice: Wrap an ice bag or cold pack in a damp, thin cloth and apply to the injured area for 15– 20 minutes
at a time, at least QID for the 1st 48 hours (or longer if swelling continues).
 Compress with an elastic bandage if there is swelling such as in an ankle sprain.
 Elevation: Try to elevate the injured part above the heart.

120. Who should avoid estrogen oral capsule? K-type


a) Soy allergy
b) Peanut allergy
c) Lactose allergy
PREMARIN (conjugated estrogens sustained release tablets) contains lactose.
In patients with rare hereditary galactose
intolerance, lactase deficiency or gIucose‐
galactose malabsorption, the severity of
the condition should be taken into careful
consideration before prescribing
PREMARIN. Patient should be closely
monitored.
Progesterone like prometrium soy allergy

121. All of the following are


treatments of rosacea, Except?
a) Avoids peak sunlight
b) Avoid spicy foods
c) Topical metronidazole
d) Topical corticosteroids
e) Avoid alcohol
122. About application of physical sunscreens with acne rosacea medications?
There are 2 main kinds of sunscreens: chemical and physical.
Chemical sunscreens must be absorbed by the skin to be effective, so they should be applied after cleansing
but before acne medication.
Physical sunscreens (containing zinc or titanium) remain on surface of the skin to reflect the sun & may stop
acne medication from being absorbed or being effective if they applied first; so, physical sunscreens should be
applied after acne medication.
If your acne medication contains benzoyl peroxide, do not use it at the same time as a sunscreen. Apply the
sunscreen during the day and the benzoyl peroxide at night.

123. Treatment of atopic dermatitis?

124. Patient allergic with ragweed, what is DOC for headlice?


a) Pyrethrin
b) Permethrin 1%
c) Isopropyl myristate 50%

125. About non pharm for headlice?


a) Put washable cloths into plastic bags and tight closed for 15 days

126. A woman of 25 years old came to pharmacy with her two years old son for the treatment of
her scabies. Which one is not suitable?
a) Lindane 1% cream
b) Permethrin 5%
c) Crotamiton
Lindane is an effective pediculicide; however, there are concerns about neurotoxicity and bone marrow
suppression after percutaneous absorption. Lindane is currently not available in Canada.
Cannot be used by children <10 yrs old, elderly, pregnancy and lactation, seizures.

127. Drug of choice of


folliculitis?
a) Topical mupirocin

128. Following lab test required


for iron def anemia, Except?
a) TIBC
b) Serum ferritin
c) Hematocrit
d) Hemoglobulin level
e) Occult blood

129. Dr. prescribed Ferrous


gluconate 300 mg TID, what is
the elemental iron he will get daily?
a) 105
b) 300
c) 900
Ferrous gluconate contains 35mg of elemental iron in 300mg (11.6%). Ferrous sulfate contains 60mg of
elemental iron in 300mg (20%). Ferrous sulfate did contain 90mg in 300mg (30%).
Ferrous Fumarate contains 99mg of elemental iron in 300mg (33%).

130. Drug of choice for hypercalcemia?


131. 20 years male using illicit drug inj, his lab result shows HIV and HEP negative, he has
endocarditis problem. What microorganism cause his problem?
a) Strep. viridines
b) Staph. aureus
c) Strep. epidermis
d) Strep. Pneumoniae
The Gram‐positive bacteria staphylococci 40% (Staph. aureus, MSSA, MRSA, Coagulase‐negative staphylococci
(e.g. S. epidermidis), streptococci 20% (Viridans streptococci ‐ VGS, S. bovis) and enterococci 10% (E. faecalis,
E. faecium) account for the majority of pathogens causing IE.

132. Symptoms of endocarditis?


a) Heart murmur, heart failure, embolism.
b) Heart murmur, splenomegaly, aneurism.
c) Heart murmur, heart failure, aneurism.
Infective endocarditis may involve the heart valves and surrounding tissues on either the left or right side of
the heart, although bilateral involvement has been described. Metastatic infection to extracardiac sites may
occur. Complications include heart failure, periannular abscess, mycotic aneurysm and glomerulonephritis.
Emboli to extracardiac sites such as the brain, lung or kidney may also occur

133. What is the DOC of this patient?


a) Cloxacillin
b) Cephalexin
c) Azithromycin
d) Clindamycin
Pathogen Regimen Duration Comments
Antibiotic treatment regimens for staphylococcal endocarditis 40%
Staphylococci Methicillin - Cloxacillin or 6 wk May use 2 wk  if the infective endocarditis is
No Prosthetic sensitive Cefazolin uncomplicated and right-sided only.
Material Methicillin - Vancomycin or 6 wk Vancomycin trough concentrations of 10–20
resistant Daptomycin as mg/L suggested. Drug of choice for treatment of
an alternative in MRSA IE and in patients with a type-1
selected patients hypersensitivity reaction to beta-lactams
Staphylococci Methicillin - Cloxacillin or ≥ 6 wk Gentamicin peak and trough concentrations are
Prosthetic sensitive Cefazolin + First 2 wk suggested to be maintained at 3–4 mg/L and <1
Valve Rifampin + mg/L respectively
Gentamycin
Methicillin - Vancomycin + ≥ 6 wk Vancomycin trough concentrations of 10–20
resistant Rifampin + First 2 wk mg/L suggested
Gentamycin
134. What lab test do for identification RA? Except (2 questions)
a) ESR
b) CBC
c) CRP
d) Morning stiffness
e) ANA
Anti‐CCP = antibody to cyclic citrullinated protein, CBC, CRP, ESR, RF = rheumatoid factor, liver and renal blood
panel, Radiographs of hands and feet.

135. Drug interaction of levofloxacin with calcium supplement?


Divalent or trivalent metal cations, e.g., calcium, iron salts, zinc (including within multivitamins), antacids
containing aluminum or magnesium, buffered didanosine, sucralfate chelated fluoroquinolones and reduced
their oral absorption.
Space oral doses:
 ciprofloxacin: 2 h before or 6 h after
 levofloxacin, norfloxacin: 2 h before or 2 h after
 moxifloxacin: 4 h before or 8 h after

136. A 70-year-old female comes at your pharmacy interested to learn about vaccines. Medical
history indicates chronic obstructive pulmonary disease (COPD) x 4 years and herpes viral
infections. Current medications include tiotropium/olodaterol 2.5/2.5 mcg per actuation, 2
actuations inhaled once daily and salbutamol 100 mcg/puff 1–2 puffs TID–QID PRN. Which
vaccines would you recommend patient to get annually?
a) Tetanus, zoster and influenza vaccine
b) Influenza vaccine, Pneumococcal and zoster vaccine
c) Pneumococcal vaccine and influenza
d) Influenza vaccine only

137. 65 years old pneumonia patient admitted in hospital ward, what is the doc?
a) Levofloxacin oral
b) Ciprofloxacin oral
c) Aminoglycoside IV
d) Erythromycin

138. Patient diagnosis as tuberculosis and taking treatment, how you could check whether
patient is improving or not? Scenario
a) Chest x ray
b) CBC
139. Treatment regimen-rifampin-resistant tuberculosis how long take treatment?
a) 12 months
b) 9 months
c) 2 years
Resistance Regimen Initial Continuation
Isoniazid INH + Option 1: RMP + EMB + PZA + Option 1: RMP + EMB + FQN daily
resistant RMP + FQN daily × 2 months or 3 times weekly × 4–7 months
disease PZA + Option 2: INH + RMP + EMB + PZA Option 2: continue daily or switch to 3
EMB ± daily × 2 months times weekly × 4–7 months
FQN Option 3: INH + RMP + EMB + PZA Option 3: RMP + EMB daily or 3
daily × 2 months times weekly × 10 months
Rifampin INH + Option 1: INH + PZA daily + Option 1: INH + PZA +
resistant PZA + aminoglycoside daily or 3 times aminoglycoside daily or 3 times
disease EMB ± weekly × 2 months weekly х 7 months
FQN or Option 2: INH + PZA + EMB + FQN Option 2: INH + EMB + FQN daily or
amino- daily × 2 months 3 times weekly × 10–16 months
glycoside Option 3: INH + PZA + EMB daily × Option 3: INH + EMB daily or 3 times
2 months weekly × 16 months

140. A 24-year-old man affected with meningitis, which organism causes meningitis? K type
a) S. Pneumoniae
b) H. Influenza
c) N. Gonorrhea

141. What is the DOC of same patient? Ceftriaxone + Vancomycin


Age Group Bacteria Empiric Antibacterial Regimen
Infants <1 month Streptococcus agalactiae (Group B Cefotaxime + Ampicillin
streptococcus), E. coli, Listeria monocytogenes Add gentamicin if early neonatal
(rare), Neisseria meningitidis (rare), S. meningitis suspected due to synergy
pneumoniae (rare), Other Enterobacteriaceae. for Group B streptococcal infections
Children ≥1 month S. pneumoniae, N. meningitidis, S. agalactiae Ceftriaxone or cefotaxime +
(Group B streptococcus), Haemophilus Vancomycin
influenzae type b, E. coli (rare), L.
monocytogenes (rare), Other Enterobacteriaceae
Adults < 60 y E. coli S. pneumoniae N. meningitidis Ceftriaxone or cefotaxime ±
L. monocytogenes (rare) Vancomycin
Adults with cellular S. pneumoniae N. meningitidis H. influenzae Ceftriaxone + Vancomycin +
immune deficiency type b L. monocytogenes Ampicillin
Humoral immune S. pneumoniae N. meningitidis Ceftriaxone or cefotaxime +
deficiency states H. influenzae type b Vancomycin
Any age: CSF leaks Staph. epidermidis S. aureus S. pneumoniae Ceftriaxone or cefotaxime +
or skull fractures, N. meningitidis H. influenzae b, S. pyogenes Vancomycin
head trauma Enterobacteriaceae Pseudomonas sp.
142. Which drug used as add on therapy in meningitis? Dexamethasone
Q. Patients frequently experience poor outcomes due to a brisk inflammatory response that
Adjunctive causes neuronal damage. Limiting the inflammation by administration of corticosteroids
Corticosteroids could potentially decrease neurologic sequelae such as hearing loss and disability.
Dexamethasone Strongly consider the addition of rifampin to the antibacterial regimen if resistant
pneumococci are isolated in a patient who has received dexamethasone.
Q. If CS used, they should be initiated either before or with the first dose of antimicrobials.
Children:
 Reduce severe hearing loss in children with H. influenzae type b meningitis.
 Protect against hearing loss in children with S. pneumoniae meningitis.
 Recommended dose for children is 0.6 mg/kg/day in 4 divided doses for 2–4 days.
 This should be initiated either before or with the 1st dose of antimicrobials.
Adults:
 No impact on mortality but reduce hearing loss and short-term neurologic sequelae.
 Dexamethasone 10 mg every 6 hours for 4 days has been used, commencing before or
with the 1st dose of antibacterials.

143. What is DOC of mild diabetic foot infection?


a) Ciprofloxacin
b) Clindamycin
c) Moxifloxacin
d) Piperacillin/tazobactam.

144. Duration of therapy for osteomyelitis?


a) Three weeks
b) 4-6 weeks
c) 6-8 weeks
d) Three months
e) Two weeks

145. 28 years old female infected from gram negative bacilli (gastrointestinal pelvic infection)
what is DOC of her? Piperacillin + Aminoglycoside

146. Which organism cause urinary tract infection? E-coli

147. What is the first line agent of acute uncomplicated UTI? Scenario type
a) Ciprofloxacin
b) Cephalexin
c) Nalidixic acid
d) SMX/TMP
148. all these diseases are needed to treat for 14 days, except?
a) Acute complicated UTI
b) Severe complicated UTI
c) Acute prostatitis  4 – 6 wk
d) Pyelonephritis

149. DOC of pseudomembranous colitis?


Severity Criteria Treatment
Mild Diarrhea plus any additional signs or Vancomycin 125 mg QID PO ˣ 10 days.
moderate symptoms not meeting severe or complicated Fidaxomicin 200mg PO BID x 10 days
disease criteria Metronidazole 500 mg TID PO ˣ 10 days
WBC ≤15 x 10^9/L and a serum creatinine
(SCr) level of <133 mcmol/L
Severe disease Serum albumin < 3 g/dL + 1 of the following: Vancomycin 125 mg QID PO ˣ 10 days or;
WBC ≥15 ˣ 10^9/L & Abdominal tenderness Fidaxomicin 200mg PO BID x 10 days
Severe and Any of the following attributable to CDI: Vancomycin 500 mg QID PO +
complicated Admission to ICU for CDI Metronidazole 500 mg Q8H IV +
disease Hypotension ± required use of vasopressors Vancomycin 500 mg in 500 mL saline QID
“Fulminant” Fever ≥38.5 °C. Mental state changes PR as enema
Ileus or significant abdominal distension Surgical consultation suggested
WBC ≥35 ˣ 10^9/L or < 2 ˣ 10^9/L
Serum lactate levels >2.2 mmol/L
End organ failure, e.g., renal failure
Recurrent Recurrent CDI within 8 wk of completion of Repeat metronidazole or vancomycin pulse
CDI therapy regimen. Consider FMT Fecal microbiota
transplantation after 3 recurrences.

150. Metronidazole ER used for which of the following condition?


a) Bacterial vaginosis
b) Pseudomembranous colitis
c) Endocarditis
d) Giardiasis
Bacterial Vaginosis (BV). FLAGYL ER 750 mg tablets are indicated in the treatment of BV in non‐pregnant women.
To reduce the development of drug‐resistant bacteria and maintain the effectiveness of FLAGYL ER and
other antibacterial drugs, FLAGYL ER should be used only to treat or prevent infections that are proven or
strongly suspected to be caused by susceptible bacteria. When culture and susceptibility information are
available, they should be considered in selecting or modifying antibacterial therapy. In the absence of such
data, local epidemiology and susceptibility patterns may contribute to the empiric selection of therapy.
DOSAGE AND ADMINISTRATION
Bacterial Vaginosis: 750 mg once daily by mouth for seven consecutive days. FLAGYL ER 750 mg tablets should
be taken under fasting conditions, at least one hour before or two hours after meals. The optimum extended‐
release characteristics of FLAGYL ER 750 mg are obtained when the drug is taken under fasting conditions
151. SE of citalopram? dry mouth.
CNS effects: anxiety, agitation, insomnia, headache, extrapyramidal effects.
GI effects: nausea, vomiting, diarrhea, constipation, increased risk of upper GI bleeding.
Others: dry mouth, increased sweating, sexual dysfunction, SIADH with hyponatremia.
Respiratory System Disorders: Infrequent: bronchitis, coughing, dyspnea, pneumonia. Rare: asthma,
bronchospasm, increased sputum, laryngitis, pneumonitis, respiratory disorder.

152. Empiric treatment of oral labial herpes? Acyclovir


Antiviral agents, include prescription agent acyclovir, famciclovir and valacyclovir
The nonprescription topical agent docosanol 10% (abreva) prevents the herpes simplex virus from spreading
to healthy cells. Adults and children >12 y: Apply to affected area at the 1st sign of pain, itching, burning,
redness or tingling every 3 h for 5–10 days.

153. Side effect of zidovudine or Monitoring parameters of zidovudine? K type


a) Leukopenia
b) Myocarditis
c) Anemia
Zidovudine Nucleoside Reverse Transcriptase Inhibitors (NRTIs):
(AZT) S.E: Nausea, headache, malaise, fatigue, rash, myositis, myocarditis, anemia, leukopenia,
pyrimidine hepatic steatosis, elevated liver enzymes, lactic acid and CK. Longterm use associated with
analog peripheral lipoatrophy. Available as an oral syrup.
Additive hemotoxicity with other agents, e.g., anemia with dapsone, foscarnet, ganciclovir,
pentamidine, ribavirin. Pharmacologic antagonism with stavudine.
Avoid combined use with ribavirin or stavudine as they activated by same intracellular pathway.
Hematological parameters should therefore be carefully monitored.
Periodic monitoring of both liver function tests

154. What would glatiramer use for?


a) HIV infection
b) Multiple sclerosis
c) Hepatitis-C
d) Guillain-barre syndrome
20 mg daily SC. Injection site pain, post‐injection reaction, flushing, chest pain, anxiety, hypertonia, joint
pain, weakness, nausea, palpitations.

155. HIV infected patient with PJP resistant to SMX/TMP. what is DOC? Dapsone
CD4 <200 cells/mcL or thrush (Pneumocystis jirovecii pneumonia) Primary and secondary prophylaxis
PCP  Preferred prophylactic therapy for PCP is SMX/TMP.
Oral candidiasis  Alternatives: dapsone PO, atovaquone PO or monthly inhaled pentamidine.
 Stop prophylactic if CD4 > 200 cells/mcL × ≥ 3 months
 Treatment of Oral candidiasis with azole antifungal agents.
156. 70 yrs. Old lady taking medication for cancer, since yesterday her body temp has been
raised above normal. What is appropriate action?
a) Give acetaminophen
b) Non pharms
c) Send to emergency stat
d) Tell her it is normal, it will resolve without medication
e) Ask her to go to doctor in next 1-2 weeks

157. A young family visiting to indian area which is resistant to azithromycin. What is
prophylactic treatment for traveler’s diarrhea?
a) Sulfamethoxazole
b) Ciprofloxacin
c) Loperamide
d) Bismuth subsalicylate

158. After taking medication patient face black tongue and passing black stool. What would
cause this syndrome?
a) Sulfamethoxazole
b) Ciprofloxacin
c) Loperamide
d) Bismuth subsalicylate
Intestinal It may have antibacterial activity, antisecretory and anti-inflammatory properties
Adsorbants Adults: Prophylaxis: 524 mg (2 tablets) or 30 mL suspension QID PO
Bismuth Treatment: 524–1048 mg (2–4 tablets) or 30–60 mL suspension Q30–60 min PO PRN
subsalicylate (maximum 4.2 g/24 h). Children: Not recommended
S.E: Blackening of stools and tongue, mild tinnitus.
Avoid in patients taking anticoagulants, therapeutic doses of salicylates or in whom
salicylates are contraindicated, e.g., children and pregnant women.
Decreases absorption of tetracyclines.
May decrease number of unformed stools passed but may not speed up illness recovery.
Chemoprophylaxis with BSS is also not recommended for young children due to concerns
of excessive salicylate absorption and risk of bismuth encephalopathy at higher doses

159. Cancer chemotherapy patient is taking dexamethasone for nausea and vomiting, still
vomiting is not control, what should you add for control vomiting?
a) Ondansetron
b) Lorazepam
c) Nabilone
d) Metoclopramide
160. Ondansetron s/e all except
Serotonin 5- Selectively block 5-HT3 receptors in periphery (visceral vagal afferent fibers) & in brain (CTZ).
HT3 Receptor 1st gen. has equivalent efficacy and toxicity when used for the prevention of acute
Antagonists chemotherapy induced N&V.
Granisetron 1st Palonosetron is the preferred 5-HT3RA & oral form is effective in preventing both acute and
gen. delayed N&V associated with moderately emetogenic.
Ondansetron 1st When administered IV, it is effective both acute and delayed nausea and vomiting associated
gen. with moderately emetogenic and acute nausea and vomiting associated with highly emetogenic.
Palonosetron 2nd If it is unavailable, use any other 5-HT3RA.
gen. Single-agent efficient, but when used in combination with a corticosteroid and NK-1 receptor
antagonist, efficacy improved for high emetogenic regimens.
S.E: constipation and headache the most common. Bradycardia, diarrhea, dizziness, QTc
interval prolongation, sedation, transient increase in laboratory values of LFTs.
High-dose ondansetron increase risk of serious cardiac arrhythmias. So, maximum
recommended single IV dose has been lowered to 16 mg.

161. What is doc of anticipatory nausea and vomiting?


a) Ondansetron
b) Lorazepam
c) Nabilone
d) Metoclopramide

162. Patient is taking psyllium for constipation past 3 days, still facing difficulties, what will
you give him immediate relief?
a) Continue with psyllium for 2 weeks and increase fluid intake
b) Discontinue psyllium and take bisacodyl
c) Add senna PRN
d) Non pharms
e) Add stool softener

163. Ampicillin 5,000,000 units when added to 23 ml of water yield 200,000 units / ml. How
much water needed to make it 250,000 units /0.5 ml.
a) 10ml
b) 4ml
c) 6ml
d) 8ml
Answer:
200,000 Us -------- 1 ml 5,000,000 Us -------- X ml X = 1*5,000,000/200,000 = 25 ml
Amount of Ampicillin = 25 - 23 = 2 ml.
250,000 Us ----- -- 0.5 ml 5,000,000 Us ------- X ml X = 5,000,000*0.5/250,000 = 10 ml
Amount of water = 10 - 2 = 8 ml
164. What drug should keep in refrigerator prior to dispensing?
a) Lactic acid bacilli cap
b) Virotic (Trifluridine) eye drops
c) Clotrimazole sy
d) Epipen
e) Sodium cromoglycate eye drops
f) Pilocarpine 1 %
g) Timolol 1 %

165. A vial contain 1 million units heparin


specifies that 3.75ml of solvent are added to
dry power the result concentration is 200000
units/ml. On basic of this information, how
many ml per day of sterile water injection
should be used to prepare for a patient
2000units/kg/ml? Patient wt. is 165 pounds
(same type-2 questions)
a) 3ml
b) 5ml
c) 4ml
d) 10ml
e) 8ml

166. We will prepare 500ml of a 5% glucose solution containing 15 meq potassium (k+) of kcl
solution + 20 meq sodium (Na+) of NaCl solution. What is concentration of cl ion in 1 litre?
a) 15 meq
b) 35 meq
c) 70 meq
Answer
Total m. Eq of Cl = 15 + 20 = 35 m. Eq
35 m. Eq -------- 500 ml X m. Eq -------- 1000 ml X= 1000*35/500 = 70 m. Eq / L.

167. Lansoprazole for a child. Drug powder is insoluble in water and partially soluble in
alcohol, drug should have to give a child, what is appropriate formulation?
a) Suspension
b) Elixir
c) Solution
d) Drops
e) Powder paper
168. What is most contamination of laminar flow hood?
a) Human
b) Sterile product
c) Disinfectant
d) Gloves

169. A vertical laminar flow hood would be the best choice when preparing a parenteral
formulation of?
a) Doxorubicin
b) Diclofenac

170. One question from cleaning of laminar flow hood, except?


a) Work close to the edge of it

171. A regular patient of yours has a methadone rx from unauthorized doctor. Her family doctor
on vacation for three weeks, this news pharmacist well known, what is appropriate action of
pharmacist?
a) Refuse to give him the medication as it is not illegal
b) Call the prescriber to register himself temporarily
c) Calculate the equivalent dose of acetaminophen with codeine 15mg and caffeine and get
a verbal rx and give it to him.
d) Right way fills her prescription
e) Phone to her family doctor and request prescription after he arrive
172. If not doing diagnoses for a terminally ill patient, what ethical principle is followed?
a) Autonomy
b) Paternalism
c) Confidentiality
d) Veracity

173. Holding ttt on pediatric and give it to adult, what ethical principle is violated?
a) Non maleficeince
b) Paternalism
c) Justice

174. Meaning of peer-reviewed article?


Peer review: Assessment of a clinical trial by experts for scientific merit, participant safety, and
ethical considerations.

175. Patient using sulphapyridine for ulcerative colitis, what deficiency is likely to occur:
a) Vit D
b) Folic acid
c) Vit b12
d) Vit k
e) Calcium

176. NaCl dissolve in 2.5 litres of water, what is amount of Na+ mmol/ml?
a) 100 mmol/ml
b) 6 mmol/ml
c) 200 mmol/ml
d) 148 mmol/ml
e) 98 mmol/ml

177. We need to prepare 20 meq of NaCl. How much solution of NaCl 14.5% to be needed?
a) 6.1 ml
b) 8.1ml

178. Patient wants emergency contraceptives. What question will you ask first?
a) When was the last menstruation?
b) Do you have multiple sex partner?
c) Do you have taken any contraceptives before?
d) Why you want it?
179. Plan b can be given in all the following except?
a) Abnormal vaginal bleeding
b) Pregnancy or suspected pregnancy
c) Hypertension
d) Angina
e) Migraine

180. How many grams of Kcl require to prepare 2


litres of solution containing 5 meq/ml.mol (wt of
Kcl 74.5)?
a) 745g
b) 74.5g
c) 5g
d) 7450g
e) 7.45g

181. Participants are randomized into 2 groups each group receives treatment with washout
period. What type of study is this?
a) Crossover
b) Cohort study
c) Randomized
d) Cross sectional

182. Methyl prednisolone acetate (depo-medrol) injection was given to a patient one week
before and his pain was relieved. Now his condition become worse after 7 days. He wants to
give the same injection again. What is the pharmacist advice?
a) It cannot be given within 2 weeks.
b) It cannot be given more than 3 time in the same joint per year.
Methylprednisolone Large joints: 20– 80 mg, Medium joints: 10-40 mg, Small joints: 4–10 mg
acetate Maximum dose: 3 injections/joint/year. No clinically significant systemic effects.
Depo-Medrol Inexpensive, safe and effective therapy for individual joints (especially hips/knees).
Minimize joint activity for 3 days following injection. Benefits last 4– 6 wk.

183. Which is the best source to find lactate containing product?


a) AHFS
b) Merck index
c) CPS
d) USP-DI vol1
184. Which source should be used to adjust dose of carbamazepine for renal failure patients?
a) Merck manual
b) Merck index
c) AHFS information center
d) Martindale extra pharmacopoeia

185. Rx: codeine 150mg to make solution 60ml


Sig 1 tsp tid, how much codeine received by the patient daily?
a) 40mg
b) 37.5mg
c) 30mg
d) 42.5mg

186. Pharmacy manager wants to improve quality of his staff’s function; what action can take?
a) Pharmacy technician involves in compounding
b) Technician helps to order the medication
c) Give joint program to both pharmacist and technician

187. A 16 years old girl is taking contraception pills from a pharmacy. Her father asked the
pharmacist what medication his daughter is taking. The pharmacist is new and does not know
how to handle the situation and hesitates to give the information to her father. Which two
ethical principle contradict?
a) Veracity and autonomy
b) Justice and autonomy
c) Fidelity & non-malifeceince
d) Confidentiality & veracity

188. A noncompliance patient does not listen to pharmacist, and he is taking half dose of his
blood pressure medication because it causes drowsiness. Pharmacist avoid giving him
counseling because he does not listen which one is violated
a) Beneficience
b) Justice
c) Autonomy
d) Nonmaleficience
189. Patient is in USA; his son brings prescription and ask his medication. What pharmacist
will do?
a) Fill the prescription
b) Do not fill it
c) Fill the prescription if border officer allows
Patient or patient’s agent is responsible for sending medications to destinations outside of Canada.
As a general rule, the Food and Drug Administration (FDA) does not allow prescription medications
to be mailed to the U.S. However, if an individual is there temporarily and needs to have their
prescription medication sent, the following should be followed: Ask their physician to write a letter
explaining that they are under their care, and that they have prescribed the drugs for their use.

190. P.j. is a 35-year-old female who is placed on lithium therapy. The suggested dose is 600mg
q8h of lithium carbonate. The total body clearance of lithium is 0.44ml/s or 1.621 l/h. The
biological half life is 18 hours. The molecular weight of lithium carbonate is 74.
190. How long will it take to reach 94 % o f steady state?
a) 18 hours
b) 1 day
c) 3 days
d) 5 days
(94% steady state after 4-5 t1/2 = 4.5 x 18 = 81 hr = 3 days)
(99% after 6.6 t1/2,100 % steady state after 7 t1/2)

191. The lithium does is now changed to 300mg q8h. How long will it take lithium to reach
94% of steady state?
a) Immediately
b) 1 day
c) 1.5 days
d) 3 days
e) 5 days

192. What is the DOC for schizophrenia?


a) Olanzapine
b) Clozapine
c) Buspirone
d) Sertraline
193. Which of the following deteriorate parkinson’s disease?
a) Risperidone
b) Galactorrhea
c) Buspirone.
d) Sertraline.
Drug‐induced parkinsonism such as 1st and 2nd generation antipsychotics and central dopamine‐blocking
antiemetics, e.g., metoclopramide and prochlorperazine

194. Which one causes sexual dysfunction?


a) Beta-blockers
b) ACEIs
c) Amiodarone
d) CCBs

195. How many ml of 0.9%(w/v) solution of NaCl can be prepared from 50tabs of 1.8g/tab?
a) 20 lts
b) 10 lts
c) 15 lts
d) 12 lts
e) 11 lts

196. Patient taking triazolam. Which one can be used for tapering?
a) Chlordiazepoxide
b) Oxazepam
c) Temazepam
d) Diazepam
e) Lorazepam.
There are no studies comparing different tapering approaches. The recommended approach is to taper
slowly, in collaboration with the patient.
Strategies usually include gradual dose reductions by approximately 25% every 2 weeks then by 12.5% every
2 weeks near the end followed by planned periodic drug‐free nights.
If the dosage form of the current BZRA does not allow for the taper rate required, consider either requesting
an extemporaneously compounded preparation or switching to lorazepam or oxazepam.
Switching to a long‐acting benzodiazepine, such as diazepam, has not demonstrated improved cessation
rates or a reduction in the incidence of withdrawal effects compared to tapering short‐acting BZRAs.
197. All of the following causes withdrawal symptoms within 2 to 3 days, except?
a) Diazepam
b) Lorazepam
c) Alprazolam
d) Temazepam
e) Oxazepam

198. Blood level of phenytoin is found very low. How can you increase blood level?
a) Increase one third and monitor effect following day
b) Increase one third and monitor effect after one week
c) Double the dose and monitor effect following day
d) Double the dose and monitor effect one week
How Much Should I Increase the Phenytoin Dose if my Patient is not Responding to Treatment?
Guidelines for dosing adjustments based on phenytoin plasma concentrations have been proposed for adults
with epilepsy without clinically significant renal or hepatic disease:
 for plasma phenytoin concentrations less than 7 µg/ml, a dosage increase of 100 mg/day is
recommended; or if Serum level below 28 umol/L increase 100mg/day
 for plasma concentrations between 7 and 12 µg/ ml, the dose may be increased by 50 mg/day; or
Serum level 28‐48 increase by 50mg/ day
 if the plasma concentration is greater than 12 µg/ ml, the dose may increase by 30 mg/day or Serum
level greater than 48 then 30mg/day
 Dosage increases when the plasma level is above 16 µg/ml should only be done with caution as even
a small increase may result in toxicity.
Dose changes should be more conservative in patients with reduced protein binding (e.g. in hypoalbuminemia
or in renal impairment) as changes in drug concentration will be exaggerated in these cases.
Time to steady‐state is highly variable, 1–8 wk.
Increase 10% To avoid over saturation of liver enzyme which will result in unpredictable increase in serum
level and monitor in 7‐10 days
But if IV monitor after 1hr. If used oral Loading dose monitor after 24 hrs

199. For using depoprovera progesterone, which of the following patient taken with caution?
a) Osteoporosis  increase Ca and vit D intake
b) Amenorrhea
c) Renal failure
d) Liver dysfunction
200. Which vaccine can be given every 10 years?
a) Diphtheria
b) Tetanus
c) Flu vaccine
d) Varicella

201. RR 40% and AR 8%. What is NNT? 3

202. Rx: Hydrocortisone power 2.4 g


Menthol 0.6 g
Champhor 0.6 g
Betamethasone cream, ointment 2:1 total wt. of product is 120 g w/w
How much qty of betamethasone cream and ointment for fill this prescription?
a) 60g cream + 60g ointment
b) 80g cream + 40g ointment
c) 77.6g cream + 38.8g ointment
d) 58.2g cream + 58.2g ointment
e) 40g cream + 80g ointment

203. Cancer patient having chronic pain using codeine. Pain is not relieved. What is next option?
a) Meperidine

204. A physician gives a prescription as follows: diazepam 30 tabs repeat 5 times.


a) Prescription is valid
b) The prescription is not valid
Invalid: even though refills are permitted for BZD when the prescription is written, the interval is missing
205. Which drug is not considered in a patient with hypertension and heart failure?
a) Ramipril
b) Metoprolol
c) Amlodipine,
d) Verapamil (note: avoid NDHP-CCB in HF)

206. 2g power mixed with 19 ml of solution, (specific gravity of solution is 1) to make 20 ml.
Patient need one teaspoonful per day, how many days would cover this solution? With scenario
total 3 question continue
a) 5 days
b) 3 days
c) 4 days
d) 2 days

207. Which of the following drug you will recommend empirically preoperative in c-section
surgery?
a) Cefazolin
b) Ceftazidime
c) Cefuroxime
d) Cephalexin
e) Ceftriaxone
Cefoxitine & Cefotatan are Better, If Allergic to Pencillin Use Vanco Or Clinda and Genta Or Metro and Genta

208. Which drug can cause sever toxicity and need to counsel pt. About it
a) Digoxin
b) Hydroxyurea
HYDREA (hydroxyurea) should be administered under the supervision of a physician experienced in the use
of cancer chemotherapeutic agents. Treatment with HYDREA should not be initiated if bone marrow
function is depressed. HYDREA may produce bone marrow suppression; leukopenia is generally its first and
most common manifestation.
Overdosage Symptoms: Acute mucocutaneous toxicity has been reported in patients receiving hydroxyurea
at a dosage several times the therapeutic dose. Soreness, violet erythema, edema on palms and foot soles
followed by scaling of hands and feet, severe generalized hyperpigmentation of skin, and stomatitis have
also been observed.

209. For sterilization purposes, steam is preferred than hot-air because:


a) Lower thermal heat
b) Higher thermal heat
c) Oxidation of bacteria
210. Auxillary label of clarithromycin? K type
Sometimes has a bitter aftertaste, the suspension should be taken with food and/or juice.
After reconstitution: Do not store above 25°C. Causes diarrhea or vomiting (15%).
Shake well before use and Discard unused portion after 14 days.

211. What is function of pharmacy and therapeutic committee? Except


Pharmacy and Therapeutics Committee (P&T)
 A sub-committee of Medical Advisory Committee MAC
 Formal line of communication between the medical staff and the pharmacy department of hospital. Acts
as link between pharmacy department and other disciplines and health care providers.
 Mission is to promote safe, rational & cost-effective distribution, utilization and administration of drugs.
 Usually composed of at least 3 physicians, a pharmacist, a representative of nursing staffs, quality
improvement managers and administrators.
Role of committee
 Evaluates and standardizes medication use within the hospital
 Sets standards for drug use (Automatic substitution orders, and Automatic stop orders)
 Evaluates the ongoing use of drugs for effectiveness
 Maintains the hospital formulary (Hospital formulary guiding principles, principles of adding drugs and
deleting drug in formulary) with optimal & cost-effective therapy
 Recommend drugs to be added to the formulary, DOESN’T decide – MAC does this!
 Review use of approved medication for unapproved indications and make recommendations
 Complete Drug utilization review of expensive drugs for cost-effectiveness

212. Canadian health act is responsible for all except?


a) Hospitalization
b) Pharmaceuticals
c) Physician fee
The five criteria of the Canada Health Act are: a) Public Administration b) Comprehensiveness c) Universality
d) Portability e) Accessibility)
Comprehensiveness The P/T plan must insure all medically necessary services.
What medically necessary services are covered according to the Act?
1. Physician services if deemed medically necessary by the province (some services like
doctor’s notes, may not be deemed necessary)
2. Hospital care services If deemed necessary by province (some cosmetic surgeries may
not be covered)
3. Anything the province deems medically necessary e.g, diagnostics
4. Prescription drugs are not covered under the Act except through (2) above, i.e.
medically necessary “hospital care services”
213. What is NOT related to PIPEDA (Personal Information Protection & Electronic
Documents Act) rules?
a) Don’t disclose personal information to anyone
b) To disclose personal information to third party
c) Use the personal information to be used in the pharmacy
d) Patient information can not be used for publicity
e) It has data base of people who identify the thefts.

214. Doctor contact pharmacist asks particular indication is not available in literature. Like off
label indication, what is appropriate action of pharmacist?
a) Dr can his judgement and take responsibility

215. Who can prescribe narcotic medicine?


* Doctors of medicine Dentists Doctors of veterinary medicine
* Midwives Podiatrists or chiropody Nurse Practitioners

216. Which of the following can be sold without sales report (exempted preparation)?
a) Hydrocodone + asap
b) Hydromorphone preparation
c) Oxycodone + codeine preparation
d) ASAP + caffeine + codeine
e) Oxycodone

217. Which is not present in narcotic purchase record or All of the following should be included
in the narcotic & controlled drug register except
a) Name of the supplier
b) Name of the drug
c) Name of the doctor or Pharmacist name
d) Name of the patient
Narcotic Purchases
A pharmacist, upon receipt of a narcotic from a licensed dealer, shall forthwith enter in a book, register or
other record maintained for such purposes, the following:
1. The name and quantity of the narcotic received;
2. The date the narcotic was received; and
3. The name and address of the person from whom the narcotic was received.
Where the records are maintained manually in a register or in a computer program, those records are to be
retained for at least two years. Where records are maintained by filing invoices in chronological order for at
least two years, pharmacists may consider filing copies of the invoices. Original invoices are financial records
and may have to be retained for up to 7 years for the purposes of Revenue Canada. Pharmacists should verify
the Revenue Canada requirements with their accountant.
218. You are a hospital pharmacist. You discovered that one of patients had an order for Losec
and technician who prepared prescription misinterpreted it as Lasix. However, the pharmacist
who was there in that shift is on vacation today. The patient has been taking the wrong
medication for three days so far including this day. Who is the first person you should contact?
a) The physician who wrote the prescription.
b) The pharmacist in charge of that shift
c) The technician who prepared the prescription
d) The patient’s family
e) The nurse on the patient care unit

219. In hospital pharmacy, a pharmacy technician can do all except or the role of a certified
technician with additional training could include all except?
a) Administer drug to patient
b) Prepare medication & product labelling
c) Checking the completeness of the prescription
d) Maintaining and updating patient profile
e) Preparation of chemotherapeutic preparation
f) Adjust dose with renal patient

220. Pharmacy profession in canada is regulated by or in canada, the legislations that regulate
the practice of pharmacy?
a) NAPRA
b) Health canada
c) Regulatory body in each territory/province.

221. Which ethical principle will be seeking if the clinical trial for a drug not used in children?
a) Beneficence
b) Non maleficence
c) Justice
d) Veracity

222. Which of the references can find therapeutic option? K type


a) Therapeutic choices
b) Literatures
c) Patient selfcare

223. Type 2 statistical error in a study comparing 2 drug treatment regimens occur
a) The data shows no difference between 2 treatment regimens and a difference
actually does exist
224. Type-2, p value is greater than 0.05 what it is mean? It is not significance

225. A patient on ciprofloxacin, isotretinoin, OCP & phenazopyridine. He got photosensitivity


while baby sitting. Which of the meds causes these symptoms?
a) Ciprofloxacin
b) Isotretinoin
c) Phenazopyridine

226. What can the pharmacist advice the above patient, except?
a) Use occlusive zinc oxide
b) Wear long sleeves
c) Avoid the sun from 10am +3 pm
d) Cool compress
e) Oatmeal bath
f) Topical corticosteroid
g) Use lotion that can protect only UVB.

227. Furosemide is used in CHF. How can you monitor its efficacy?
a) Edema
b) BUN
c) Urine output
d) Weight measurement
e) Hematocrit
228. stroke patient having atrial fibrillation. What will you give for 2ry prevention of stroke?
a) ASA
b) Warfarin
c) ASA + clopidogrel

229. Which parameter does not to check for etanercept?


TNF-alpha A recombinant monoclonal antibody that binds to TNF-α, thereby interfering with
Inhibitors TNFI endogenous TNF-α activity by blocking its interaction with cell surface receptors.
Adalimumab Adalimumab: 40 mg Q2 wk SC.
Certolizumab: 400 mg at wk 0, 2 and 4, then 200 mg Q2 wk SC. May give 400 mg Q4 wk
Certolizumab SC as maintenance dose
Etanercept: 25 mg twice weekly or 50 mg once weekly SC
Etanercept Golimumab: 50 mg once monthly on same date each month SC.
(acts against both S.E: Injection-site reactions; infections (including TB); newonset psoriasis; increased risk
TNF and ) of lymphoma (children and adolescents), leukemia autoimmune phenomena.
Monitoring: Baseline CBC, LFTs, creatinine, hepatitis B and C serology, PPD and chest
Golimumab x-ray to assess for latent TB. May consider screening for ANA.
Susceptibility to or presence of serious and/or recurrent infection; SLE, demyelinating
disease and heart failure are relative contraindications.

230. All are the responsibilities of health canada, except?


a) Price monitoring of the marketed product
b) Approval of new products
231. Cephalexin 6% and placebo 10%, n=100, what is the result
a) The result is non- significant because small number of sample size

232. Patient was being withdrawn from corticosteroid; this is because he experienced all of the
following except or Patient on Corticosteroid, physician will start to withdraw the
corticosteroids, the concerns related to corticosteroids withdrawal is?
a) Cushing syndrome
b) Adrenal insufficiency
c) Fluid retention
d) Oral thrush
e) GI disturbance (Nausea, Fatigue & Dizziness)

233. The package inserts enclosed with a vial containing 1g of ampicillin sodium powder
specifies that when 3.5 ml of sterile water is added to the powder the resulting concentration is
250 mg per ml. Using this information what column of sterile water should be added to make a
solution containing 100 mg per ml? (problem)
a) 3.5 ml
b) 5.0 ml
c) 7.0 ml
d) 9.5 ml
e) 10.0 ml
Answer:
250mg present in 1ml So 1000mg will be in = 1000 / 250 = 4ml
We are adding 3.5ml SWFI. So, the volume occupied by Ampicillin is: 4 – 3.5ml = 0.5ml
We have to make 100mg/ml
100mg present in 1ml So 1000mg will be in = 1000 / 100 = 10ml
We know that 0.5ml occupied by Ampicillin. So, the amount of SWFI = 10 – 0.5 = 9.5ml

234. A patient got an infection from an IV central line. The culture revealed coagulase negative
gr + ve cocci. What is the microorganism?
a) Streptococcus Pyogenes (gram positive)
b) Staphlycoccus epidermidis (gram positive, coagulase negative)
c) Streptococcus A B- hemolytic (gram positive)
d) Enterococci (gram positive)
Coagulase negative staphylococci (CoNS) species such as Staphylococcus epidermidis and Staphylococcus
hemolyticus are commonly found on the skin and the mucous membranes of many individuals. Staph aureus
is gram positive, coagulase positive.
235. A patient who is now living in the USA, sent his son to collect his refill for the next 6
months. The patient had filled his prescription 10 weeks ago, and his insurance company only
covers 3-month refills. So, what will you do:
a) Tell him he is only allowed to fill for 3 months
b) Tell him he can take the whole 6 months but he has to pay for uncovered 3 months.
c) Tell him that since his dad now lives in the USA, he isn’t entitled for refills from canada
d) Tell him that his dad should get a physician in the states

236. A patient being treated with norfloxacin because she has dysuria, burning, fever, pain (i.e.
UTI infection), so what is the most common pathogen causing this:
a) Pseudomonas aeruginosa
b) Enterobacteriaceae
c) E. Coli
d) Klebsiella pneumonia

237. Furosemide side effect


include
a) Decrease Mg
b) Increase Na
c) Increase Ca
d) Decrease glucose

238. Patient with ASA toxicity and on patient profile is pco2 is 30 (normal is 32-34), Po2 is
normal, bicarbonate is 4 (6-7 normal) and PH is 7.3 (7.45-7.55 normal), so the patient is
having?
a) Metabolic acidosis with respiratory compensation
b) Metabolic alkalosis with respiratory acidosis
To manage any acid base disturbance question, follow this sequence
1‐ check for PH Normal is 7.35‐7.45. If less than this range then acidosis. If more .... alkalosis
2‐ check for Pco2 (35‐45 mmHg) we must memorize these values. If more than 45 then lung is the cause of
acidosis and this case will be respiratory acidosis. This happens in cases of respiratory depression, COPD exa
cerbation or any case of reduced respiratory activity
3‐ check for HCO3 level (22‐26 mmol). If more than 26 then metabolic alkalosis but it less than 22 it’s
metabolic acidosis

239. When a manufacturer of a product sends a part of the order and then promises to complete
the rest later, this is called
a) Back order
b) Promissory order
240. Which of the following is not a side effect of NTG?
a) Bradycardia
b) Hypotension
S.E: Headache, syncope (rare), nausea, tachycardia, palpitation, hypotension, dizziness, flushing, weakness.
Tolerance develops unless a nitrate‐free period of 10‐12 hrs is used each day.

241. Which monitoring is not required for dalteparin (LMWH)


a) APTT
b) INR
c) Bleeding
d) Rash
e) Vital signs
f) Hemoglobin
g) Ferritin
h) Leucocytes

242. All the following are scope of provincial college of pharmacy, except
a) Safety of pharmacist work place
b) Pricing of pharmaceuticals
c) Registration of member of pharmacy

243. A busy pharmacist. So, in his last checking what is the lest important check
a) Doctors identification number
b) Dosing interval
c) Name & address of patient

244. Which one is not a straight narcotic


a) Fiorinal 1/2 ((butalbital + ASA+ codeine 30 mg)
b) Tylenol 4 (Codeine Phosphate. 60 mg + Acetaminophen. 300 mg)
c) Acetaminophen 325 + codeine 5mg/ml

245. All are found in CPS, except


a) Off-label use of drugs
b) Brand & generic name
c) Clinical information
d) Monographs
246. Which reference book will you see for lactose allergy
a) CPS
b) Therapeutic choices
c) Martindale

247. How many grams of cyclosporine will a 182 lb patient receive over 7 successive days at
dosage rate of 12 mg/kg/day?
a) 0.99
b) 6.9
c) 15.32

248. In bioequivalent studies, it is important to determine all of the following except


a) Vd
b) AUC
c) C max
d) T max

249. Which one is correct about methyl salicylate?


a) Don’t apply heat.
b) You should protect area around
c) Don’t apply to wide area
d) Hot compress when added are very beneficial

250. In a randomized double-blind study, if you have not taken placebo control trial group,
which of the ethical principles are broken?
a) Justice
b) Non-maleficence
c) Autonomy
It maybe verasity

251. What is wrong about antineoplastic drug


a) Solution should be prepared in a vertical laminar floe hood
b) Should be prepared within 8h of intended use
c) Multiple use of a syringe with needles is not recommended
d) Should be discarded after 8h of initial use.
e) Throw remaining in chemical waste
Should be hazardous waste
252. What is the treatment of chest congestion?
a) Guaifenesin
b) Epinephrine
c) Xylometazoline
d) Oxymetazoline
e) Dextromethorphan ꞏ

253. The daily dose of pyrantel pamoate is 11mg/kg for 3 consecutive days how many mls of
Pamoate suspension containing 50mg/ml should be given to a patient weighing 100 lbs?
a) 10ml
b) 22ml
c) 30ml
d) 50ml
e) 66ml
Answer:
Dose = 11mg/kg = 11 x 45.45 = 499.95mg We have a suspension of 50mg/ml
50mg present in 1ml So, 499.95mg will be in = 499.95 / 50 = 9.99ml for 1 day.
For 3 days = 9.99 x 3 = 29.97ml

254. How much quantity of sodium fluoride needed to make 120 ml solution with how many
grams of a substance of which 1 ml gives 2 ppm concentration if dissolved in 150 ml.
a) 3.0 gm
b) 3.6 mg
c) 36.0 mg
d) 360 mg

255. Symptoms of anaphylactic


reaction for which pt need to go ER
except?
a) Urticaria
b) Dyspnea
c) Swelling of the tongue
d) Rash in trunk

256. Which of the fallowing are false regarding EPIPEN injection?


a) Take off clothes before injection
b) There is some residual after injection
c) Pull back the plunger until you see blood
d) If symptoms are still there after 10 min, inject anothr epipen.
257. Patient on opioid medication & suffering from constipation for a few months. Dr.
Prescribe docusate sodium as PRN. What pharmacist cn suggest:
a) Take docusate daily instead of PRN.
b) Take docusate daily instead of PRN and senna 1-2 daily as PRN

258. Patient having high blood pressure and diagnosed with depression, took citalopram and
experienced sexual dysfunction what is best drug for him?
a) Fluoxetine
b) Venlaflexin
c) Bupropion
d) Buspirone

259. Bupropion side effect and monitoring.


Rigorous clinical monitoring for suicidal ideation or other indicators of potential for suicidal behaviour is
advised in patients of all ages given an antidepressant drug. This includes monitoring for agitation‐type
emotional and behavioural changes.
All patients with hepatic or renal impairment should be closely monitored for possible adverse effects (e.g.
insomnia, dry mouth, seizures) that could indicate high drug or metabolite levels.
Monitoring of blood pressure is recommended in patients who receive the combination of bupropion and
nicotine replacement.
In the event of overdose, hospitalization is advised. Ensure an adequate airway, oxygenation, and ventilation.
Monitor cardiac rhythm (ECG) and vital signs. EEG monitoring is also recommended for the first 48 hours post‐
ingestion.

260. CI of Bupropion are all of the following except.


a) MI
b) Seizure
c) Bulimia nervosa.
d) Hepatic impairment
261. Cefazolin dose is 1.5 gm/day given every eight hrs. The stock available is 2 gm vials to be
reconstituted by 19ml of normal saline (0.9% NaCl, 1 g/ml) in order to get 20ml solution. How
many ml soln we need for every dose?
a) 5.0 ml
b) 10.0 ml
c) 6.67 ml
d) 16.67 ml
e) 20.0ml
Answer:
1.5gm/day if you need a dose every 8 hours, that means 3 times a day
so, 1.5gm/3 = 500mg per dose 2000mg ‐‐‐‐‐ 20mls 500mg ‐‐‐‐ X x = 5ml

262. What is the cone. (%w/v) of reconstituted solution? 10.0% w/v

263. What is the cone. (%w/w) of reconstituted solution. 9.0 w/w

264. How much energy of kcal will get from 1000 ml 5% w/v dextrose (1g dextrose
monohydrate = 3.4 kal).
a) 120 kcal
b) 170 kcal
c) 190 kcal
d) 200 kcal

265. What amount of 10% oint. Stock is needed to prepare 30g of 0.2% ointment
a) 6.0mg
b) 60mg
c) 600mg
d) 300mg

266. The patient need 1.5mg/kg/day bid dose. How much you need for a boy weighed 48.4lb for
one-week supply.
a) 10.5 mg
b) 105 mg
c) 231mg
d) 462 mg

267. Gtt ii ou tid means? instill 2 drops into both eyes three times a day.

268. Gtt ii as tid means? instill 2 drops into left ear three times a day.
269. Pharmacist saw a pharmacy technician is placing HCTZ, digoxin, ASA & psyllium
capsules 3 times daily in dosette. What is the pharmacist concern?
a) Pharmacist should be worried about psyllium on a TID basis given with other
medications.

270. Which of the following is not-responsible for the renal dysfunction?


a) Digoxin
b) ACE inhibitor
c) Gentamycin
d) Furosemide

272. Pt have irregular eating habits and sedentary lifestyle. He is on metformin and glyburide
still the blood sugar is not under control, his HbAlc is 9.5%. What will you do? Pt was with
normal weight.
a) Add thiazolidinediones
b) Add metiglinides
c) Add acarbose
d) Add orlistat
Metiglinides: Taken just prior to meals to reduce postprandial glucose elevations and should be omitted if
meal is missed. ACARBOSE May reduce metformin bioavailability.

273. If we gave insulin, how


would u dose it?
a) Give in morning, supper,
and bedtime
b) Morning and with each
meal
c) Morning and bedtime only
d) Morning only
e) Supper only

274. Doctor prescribe above pt. With some medication but now he is having adverse effect like
weight gain, fluid retention and increase HDL and decrease the which of the following have the
potency of giving this side effect?
a) Pioglitazone
b) Metformin
c) Gluyburide
d) Insulin
e) Nateglinide
275. Symptoms of sever hypoglycemia
 Mild to moderate hypoglycemia has autonomic symptoms: sweating, tremors, tachycardia, heart
palpitations (heavy, fast heartbeats), hunger, nausea, numb lips or tongue, headache and a general
sensation of weakness.
 Severe hypoglycemia requires assistance in its recognition and/or treatment. Neuroglycopenic
symptoms such as confusion, anxiety, feeling irritable, altered behaviour, difficulty speaking and
disorientation can progress to seizures and coma that prevent the patient from appropriately treating
the hypoglycemic episode.

276. You will tell all the following to pt. Starting on antidepressant, except?
a) Take medication daily, even if u feel better
b) May take 2 to 6 weeks to give effect.
c) Don't stop without checking your physician
d) You may take medication for few days and if side effect bothers you, stop taking the
medication.

277. Pt on paroxetine from last one & half month. Now physician wants to change the regimen
as pt. Was under weight and malnourished, need something which gives good effect with
weight and insomnia was bothering him. What will you recommend for this patient?
a) Bupropion
b) Venlafaxine
c) Mirtazapine
d) Fluoxetine
e) Olanzapine

278. Which of the following benzodiazepines have least discontinue symptoms


a) Lorazepam
b) Alprazolam
c) Oxazepam
d) Triazolam
e) Chlordiazepoxide

279. Which of the following has no or least addiction potential?


a) Morphine
b) Oxycodone
c) Meperidine
d) Codeine
280. Which is not a serotonin syndrome symptom?
a) Agitation
b) Anxiety
c) Bp fluctuation
d) Shivering
e) Confusion
Serotonin syndrome:
Rare, and may occur when multiple serotonergic agents are used MAOIs have the highest risk and require a
minimum 2-wk washout period before another serotonergic agent is initiated
The serotonin syndrome may occur if the SSRI, MAOi, SSRI, TCA, are combined with Tramadol,
Meperidine, Amphetamine, Dopamine, Cocaine, Methyldopa, Dextromethorphan, Caffeine, Lithium, SSRI,
MAO Inhibitors, TCAs, St. John wort.
Amitriptyline, mirtazapine and trazodone are unlikely to cause serotonin syndrome
Signs and symptoms of Serotonin syndrome
 Neurobehavioral: confusion, agitation, seizures, coma, delirium, hyperreflexia.
 Autonomic: Hyperthermia (fever), diaphoresis (sweating), tachycardia, hypertension, diarrhea
 Neuromuscular: Myoclonus, rigidity, tremor (shivering), ataxia, dilated pupil and nystagmus
Management:
 Stop the drug(s) and refer patient to the hospital
 Most cases are mild and resolve spontaneously within 24 to 72 hours.
 Pharmacist has to contact the physician and tell the patient to withhold the serotoninergic agent.
 Benzodiazepines, propranolol, and cyproheptadine, a serotonin antagonist, have been used
successfully.

281. What is true for a patient on NRT therapy but still having craving to smoke.
a) Give inhaler or gum for breakthrough craving.

282. Psychosis pt. Now getting pseudo parkinson symptoms which of the following is not
responsible for this?
a) Haloperidol
b) Risperidone
c) Levodopa
d) Clozapine

283. Which of the following is not responsible for increase in dopamine?


a) Metoclopramide
b) Haloperidol
c) Domperidone
d) Bromocriptine
284. Which of the following has decrease in appetite as a side effect?
Zinc lozenges.
Endocrine
Endocrine side effects have included the reduction of high‐density lipoprotein (HDL) in males.
Due to an accompanying decrease in low density lipoproteins (LDL), the LDL/HDL ratio remains relatively
unchanged and little risk is assumed in terms of coronary heart disease. Triglyceride levels have not been
shown to be affected.
Gastrointestinal
Gastrointestinal irritation appears to be dose‐related.
Gastrointestinal side effects have included an unpleasant taste (80%), nausea (20%), mouth irritation (24%),
dry mouth (12%), gastrointestinal upset (10%), distortion of taste, abdominal pain, vomiting, and diarrhea.
Nervous system
Nervous system side effects have rarely included dizziness and headache.

285. Pt with acute pain, taking tylenol # 3, still no improvement. DOC should be.
a) Oxycodone.

286. What is true about acyclovir except


a) Topical acyclovir is used in cold sore.
b) Oral is for herpes zoster, which should be used within 24- 72 hour.
c) Used in viral herpes.

287. Ques on diabetic neuropathic pain. All are used to relieve neuropathic pain, except
a) Carbamazepine  ttt of trigeminal neuralgia
b) Amytriptyline
c) Gabapentin
d) Duloxetine

288. Child pt. With fever & history of seizure, what to give for fever to this child.
a) Acetaminophen.
b) Sponge with alcohol
c) Refer to Dr.

289. Which drug does not interact with oral contraceptives?


a) Phenytoin
b) Phenobarbitone
c) Primidone
d) Gabapentin
e) Carbamazepine
290. All of the following are recommended in pregnancy except.
a) Folic acid
b) Vitamin A
c) Iron
d) Vitamin D
e) Vitamin B6

291. All are true about vitamin A, except


a) It is antioxidant
b) Large dose gives toxicity
c) Recommended in smokers
d) Recommended in non-smokers
e) Side effect is liver toxicity

292. Ashma control failure for child indicated by:


a) Getting frequent cold per year
b) One salbutamol inhaler in school and one at home
c) More wake up at night due to breathing difficulties

293. Ques on declectin in pregnancy, what is true (5 big sentences)


a) Hydroxyzine and pyridoxine are vitamins that help reduce nausea.
b) Diclectin is long acting and will help reduce morning nausea when taken at bedtime.
c) Diclectin works as a hypnotic and will reduce daytime nausea.
d) Doxylamine is an H2 antagonist that will reduce acid reflux which is causing the nausea.
e) Doxylamine and pyridoxine are meant to replace nutrients lost by vomiting

294. For which of the following direction, you can fill the prescription without quantity for
tylenol#3.
a) Tit bid for 10 days
b) T 2-3-tab bid for 10 days
c) Tilt q 4 to 6 hr. For pain for 10 days
d) Tit q4h pm for pain for 10 days
e) Tlt q4h for pain
295. What will you give for secondary
prevention of stroke, pt. Having no other
medical condition or complications.
a) Aspirin
b) Warfarin
c) ASA + dipyridamole
d) ASA + warfarin
e) LMWH

296. Pt with high cholesterol and diabetes


and using metformin, drinking red wine
in dinner. What will you say to this pt. Except?
a) Decrease calories intake less than 300mg per day
b) Restrict fat intake to 20% calories
c) Limit sugar intake and increase fiber.
d) Exercise.
e) Count wine calories with your food calories too
f) Restrict saturated and trans-fat in diet.

297. All of the following are true about niacin, except


a) Has flushing as side effect, more with immediate release fonn
b) Give ASA or NSAIDS to decrease flushing
c) XL has most flushing effect

298. Which of the following will increase carbamazepine?


a) Clarithromycin
b) Phenytoin
Induces several cytochrome P450 isoenzymes, potentially increases the clearance of many drugs, such as OCs,
warfarin, risperidone or TCAs. Clearance may be reduced by CYP3A4 inhibitors as clarithromycin,
erythromycin, grapefruit juice, itraconazole or ketoconazole.

299. What will you monitor for patient on atorvastatin and having muscle pain
a) Increase AST & ALT
b) Increase CK
c) CBC
d) Thyroid function
e) Urine color change
Liver function tests should be performed before the initiation of treatment, and repeated as clinically
indicated. There have been rare postmarketing reports of fatal and non‐fatal hepatic failure in patients taking
statins, including atorvastatin. If serious liver injury with clinical symptoms and/or hyperbilirubinemia or
jaundice occurs during treatment with ATORVASTATIN, promptly interrupt therapy. If an alternate etiology is
not found, do not restart ATORVASTATIN.
Myopathy, defined as muscle pain or muscle weakness in conjunction with increases in creatine kinase (CK)
values to greater than ten times the upper limit of normal, should be considered in any patient with diffuse
myalgia, muscle tenderness or weakness, and/or marked elevation of CK. Patients should be advised to report
promptly any unexplained muscle pain, tenderness or weakness, particularly if accompanied by malaise or
fever. Patients who develop any signs or symptoms suggestive of myopathy should have their CK levels
measured.

300. Pt with CHF & BP, you will tell all of the following to pt except?
a) Decrease caffeine, alcohol & salt intake
b) Increase exercise
c) Rest

301. Sleep apnea is not risk factor for: Complications may include: Stroke, Heart failure, Hypertension,
Atrial fibrillation, Diabetes, Urinary Incontinence, Insomnia.

302. Black pt. With CHF & NYHA III, on which medication you will start with?
a) Beta blocker + spironolactone

303. Pt with diabetes mellitus on certain medication now he got rx for ramipril, pt. Curious
about new medication & said, I do not have BP, why my doctor gave me this medication?
a) To maintain blood pressure
b) To protect renal function
c) For prevention of retinopathy
d) To maintain heart functioning
e) As a blood thinner

304. Mother came to pharmacy, her dughter has nits and was told to leave school from last 10
days. Mother worried as daughter is missing her school. What will you recommend to this pt.,
pt. if you know that she is allergic to ragweed?
a) Give permethrin
b) Give isopropyl myristate
c) Give Pyrethrins/Piperonyl Butoxide
d) Soak in vinegar
e) Give lindane lotion
Nit After the treatment, the dead nits will still be attached to your hair.
removal To make it easier to remove them you can wet the hair with vinegar and water for 30-60
minutes or apply a product that contains formic acid (8%).
You can remove nits in 3 ways: Use your fingers to gently pull out any hairs that have nits.
Use tweezers to pull nits off the hair. Use a fine-tooth (nit) comb. Start at the scalp and comb
to the end of hair. Clean comb with soap and hot water when you are finished.
No-nit” policy requiring children be free of nits before returning to school has not been
effective in mitigating outbreaks. Consequently, it is recommended that parents of an
affected child be notified, and that the child not be sent home early but receive treatment
with an effective pediculicide that evening, and return to school the next morning

305. What other measures will you give to her mother


a) Treat other house members to prevent getting lice
b) Soak all washable in hot water for 2 to 3 days
c) Non washable items in plastic airtight bags for 10 to 15 days
d) Rewash hairs within 3 to 4 days to remove nits

306. You will say all of the following while


counselling to using spiriva handihaler, except.
a) Shake well before use
b) Capsule must not be swallowed.
c) Should be used with caution in patients with
narrow-angle glaucoma or urinary retention.
d) Breathe in slowly and deeply from the
HandiHaler mouthpiece, until your lungs are
full.

307. Pt with CHF, BP, age 50 years. Which of the


following medication is cl in this pt.?
a) Carvedilol
b) Ramipril
c) ASA
d) Amlodipine
e) Spironolactone

308. Side effect of ciprofloxacin, except?


a) Taste change
b) Phototoxicity
c) Hypertension  hypotension
d) Anemia
e) Dry mouth
CIPROFLOXACIN may be taken before or after meals. Absorption is faster on an empty stomach. Patients should
be advised to drink fluids liberally and avoid taking dairy products or antacids containing magnesium or aluminum.
Ciprofloxacin should be administered at least 2 hours before or 6 hours after antacids and mineral
supplements containing magnesium or aluminum, as well as sucralfate, VIDEX (didanosine) chewable/buffered
tablets or pediatric powder, metal cations such as iron, and multivitamin preparations with zinc
Although ciprofloxacin may be taken with meals that include milk, simultaneous administration with dairy
products alone, or with calcium‐fortified products should be avoided, since decreased absorption is possible. It
is recommended that ciprofloxacin be administered at least 2 hours before or 6 hours after substantial calcium
intake (>800 mg)

309. Pt with DVT and leg surgery, what is initial DOC?


a) ASA
b) LMWH + warfarin
c) LMWH only
d) Heparin
e) Clopidogrel

310. Reason for pseudoephedrine not allowed to be given in large quantity, and taken through
the boarders.
a) Due to addiction potential
b) Prodrug for crystal meth
c) Hazardous if taken in large quantity

311. Which one needs to be shaken well before use?


a) Levocabastin spray
b) Salmeterol diskus
c) Symbicort Turbuhaler
LIVOSTIN nasal spray is available as a microsuspension, the bottle should be shaken before each application.
Salmeterol diskus: Inhale rapidly and deeply. Continue to take a full, deep breath.
Symbicort Turbuhaler: Breathe in deeply and forcefully.

312. Which of the following is having least urinary retention as side effect
a) Risperidone  least one
b) Cetirizine
c) Chlorpheniramine

313. Vaccine. Pt is travelling and want to avoid food contamination what to give.
a) Dukoral
b) Salmonella vaccine
c) Twnirix
314. You will give all of the following non-drug measures to above pt., except
a) Wash hands with soap and water before eating.
b) Drink only boiled, bottled or carbonated beverages.
c) Eat fruit (including tomatoes) only if it has been washed in safe water and peeled.
d) Eat fresh raw meats or fish.

315. Which of the following is best used to relive the symptoms of a GERD.
a) Antacids like Al, Mg
b) Ranitidine
c) Alginates
d) PPIs
e) Milk

316. Goal treatment of dementia


 Alter the natural disease progression to meet patient’s and caregiver’s goals
 Treat cognitive, psychological and behavioural symptoms
 Alleviate caregiver burden. Minimize medication side effects.

317. Meperidine with SSRI? Contraindicated


Based on the mechanism of action of SSRIs and the potential for serotonin syndrome, caution is advised
when SSRIs hydrobromide is coadministered with other drugs or agents that may affect the serotonergic
neurotransmitter systems, such as tryptophan, triptans, serotonin reuptake inhibitors, lithium, St. John's Wort,
fentanyl and its analogues, dextromethorphan, tramadol, tapentadol, meperidine, methadone and
pentazocine.

318. Drug utilization effectiveness is


a) Retrospective
b) Prospective
c) Reactive
319. A physician asked the pharmacist to tell the patient that the drug prescribed is a
multivitamin and not a hypnotic so that he may not commit suicide. Which ethics do the
physician looks for the patient?
a) Beneficence
b) Autonomy
c) Non-Maleficence

320. Which is counseling to patient with ophthalmic ointment?


a) Close his eyes hardly to distribute the ointment.
b) Look down while applying the ointment
c) Apply the eye ointment along the upper lid
d) Squeeze your eyes strongly after applying the ointment
e) Remove the excess ointment from the eye lid
f) If you are also using an eye drop, use the eye drop and then the ointment

321. Lady with emphysma, she is smoker for 41 years. She had asthma and takes salbutamol
and Budesonide, her dr. Rx ipratropium & azithromycin; she got worsen; why?
a) Low dose of salbutamol.
b) DDI between salbutamol and azithromycin.
c) Low dose of ipratropium.
d) Deterioration due to smoking continuity.

322. What should be the best treatment option for this lady?
a) Bupropion + nicotine replacement therapy.
b) Increase dose of salbutamol.
c) Increase dose of ipratropium

323. Rosiglitazone side effects?


Highest risk of Weight gain, fluid retention, edema worsening HF, mild BP lowering, macular degeneration;
↑ risk of fractures & bladder cancer.

324. Insulin pen, which one is true?


a) Different cartridges of different manufactures can be used in the same device
b) Prime every time before use
c) Not suitable for patients with impaired sites
d) Good for mixing different types of insulin
325. S.E of isotretinoin. Isotretinoin monitoring test
Systemic Drug Therapy for Acne. Q. Retinoids Isotretinoin Accutane, Clarus, Epuris
It remains the most powerful anti-acne agent, with the majority of patients achieving clearing and sustained
remission, even in the most severe cases.
It is recommended as first-choice therapy for severe papulopustular or moderate nodular acne and for
nodular or conglabate acne for many reasons: clinical effectiveness, prevention of scarring and quick
improvement of a patient’s quality of life, including minimizing depression.
Avoid taking vitamin A as Oral isotretinoin is a natural metabolite of vitamin A.
0.5 mg/kg/day PO for the first month, increasing to 1 mg/kg/day as tolerated (with a goal cumulative dose of
120 – 150 mg/kg). Alternative lowdose regimen (particularly for treatment of resistant or quick-relapsing
moderate acne): 0.25–0.4 mg/kg/day.
A complete course of therapy consists of 12-16 weeks of Isotretinoin administration.
S.E: Teratogenicity. Common: mucocutaneous dryness, myalgia, arthralgia, photosensitivity, Headache. Rare:
hypertriglyceridemia, mood disorder, possibly suicide ideation, pseudotumor cerebri, erythema multiforme,
Stevens-Johnson syndrome, toxic epidermal necrolysis.
Blood potassium increased, blood alkaline phosphatase increased, blood bilirubin increased, blood urea
increased, elevated platelet counts, eosinophil count increased, false positive tuberculosis test, gamma-
glutamyltransferase abnormal, blood cholesterol increased, glucose urine present, haematocrit decreased,
protein urine, thrombocytopenia, WBC count decreased. Elevations in levels of serum creatine kinase CPK
(monitor)
Patients should be advised to use a skin-moisturizing ointment or cream and a lip balm from the start of
treatment as isotretinoin is likely to cause dryness of the skin and lips. When necessary a sun-protection product
with high protection factor of least SPF 15 should be used.
It is recommended that blood donation for transfusion purposes be deferred during therapy with EPURIS and
for one month after discontinuation of treatment.
Patients who experience tinnitus or hearing impairment should discontinue EPURIS treatment and be referred
for specialized care for further evaluation.
Isotretinoin has been associated with inflammatory bowel disease (including regional ileitis, colitis and
hemorrhage) in patients without a prior history of intestinal disorders. Patients experiencing abdominal pain,
rectal bleeding or severe diarrhea should discontinue EPURIS immediately.
Tetracyclines: rare cases of benign intracranial hypertension (pseudotumor cerebri): allow 7 days washout after
stopping tetracyclines before starting isotretinoin.
The patient should have two negative pregnancy tests (β-hCG in urine or serum) before starting therapy with the
first pregnancy test conducted at initial assessment when the patient is qualified for therapy by the physician.
The patient then should have a second pregnancy test with a sensitivity of at least 25 mIU/mL with a negative
result, performed in a licensed laboratory, within 11 days prior to initiating therapy. The patient has had two or
three days of the next normal menstrual period before therapy is initiated.
Pregnancy test must be repeated monthly for pregnancy detection during treatment and at one month after
discontinuation of treatment. The dates and results of the pregnancy tests should be documented.
Females must not become pregnant while taking Isotretinoin or for at least one month after its discontinuation.
Treatment should be discontinued if the patient develops any of the following reactions: rash, especially if
associated with fever and/or malaise, conjunctivitis (red or inflamed eyes); blisters on legs, arms or face and/or
sores in mouth, throat, nose or eyes; peeling skin or other serious skin reactions.
The following tests are required before starting Isotretinoin, at first month, then as clinically indicated:
 Serum blood lipid determinations (under fasting conditions) should be performed before Isotretinoin is
given and then at intervals (one month after the start of therapy) until the lipid response to Isotretinoin is
established (which usually occurs within four weeks), and also at the end of treatment.
 Complete blood count and differential: for early detection of leukopenia, neutropenia,
thrombocytopenia and anemia.
 Liver function tests: Increases in about 15% of ALT, AST, ALP baseline levels have been reported.
Liver function tests should be monitored before treatment and at regular intervals during treatment (one
month after the start of treatment and at least three-month intervals thereafter) unless more frequent
monitoring clinically indicated.
 Blood glucose levels: all patients and in particular patients with known or suspected diabetes should
have periodic blood sugar determinations.
 Serious Adverse Event Warnings include psychiatric disorders (depression, psychosis and, rarely,
suicidal ideation, suicide attempts, suicide, and aggressive and/or violent behaviors). Monitor
psychiatric illness before dispensing isotretenoin.

326. Woman on isotretinoin and want to be pregnant after ttt how long she has to wait
a) 1 mo
b) 2 mo
c) 3 mo
It is recommended that a woman wait one month after stopping isotretinoin before trying to
become pregnant. Usually, isotretinoin is no longer found in a woman's blood 4‐5 days after
the last dose and most of its by‐products should be gone within 10 days after the last dose.

327. A new drug with no supporting literature is refused to enter a formulary; this is in
compliance to which principle?
a) Non-maleficence

328. In canada most cases of liver cirrhosis are due to?


a) Ethanol
b) Hepatitis A
c) Hepatitis C

329. What is true about tamoxifen?


a) Low emetogenic potential of chemotherapy agent (<30%)
Tamoxifen is an estrogen antagonist with some estrogenic activity, and it is classified as a selective estrogen
receptor modulator (SERM). It is used for first‐line therapy in the treatment of estrogen receptor–positive
breast cancer.
Binds to estrogen receptors in the breast tissue, result in depletion (down‐regulation) of estrogen receptors,
and the growth‐ promoting effects of the natural hormone and other growth factors are suppressed
Side effects: Inhibition of estrogen cause Hot flashes. Low emetogenic or least nauseating anticancer drugs.
Vaginal bleeding, menstrual irregularities and risk of endometrial cancer.
Tamoxifen is a prodrug, and most of the therapeutic effect in treating breast cancer stems from its
metabolite, endoxifen. Since cytochrome P450 (CYP) 2D6 is the most important enzyme in the production of
endoxifen, drugs that inhibit CYP2D6 would be expected to reduce tamoxifen efficacy. In addition to drug–
drug interactions (DDI) involving CYP2D6, there is growing evidence that enzyme inducers can substantially
alter the disposition of endoxifen, reducing tamoxifen efficacy

330. A randomized double control trial found that the rate of surgical site infection (SSI) is 6%
With cefazoline and placebo 10%, sample size is 100. So, what you can say that error occur b/c.
a) Placebo is more effective than cefazolin
b) Cefazolin is more effective than placebo
c) The results are statistically significant
d) The results are statistically insignificant
e) The effects of cefazolin and placebo are identical

331. A type ii error could have occurred because of which of the following
a) The investigators were not blinded
b) The subjects were not blinded
c) The Sample size was too small
d) There was no control arm
e) Type ii error did not occur

332. The most life-threatening drug induced-arrythmia in quinidine is:


a) Torsade de pointes
b) Atrial fibrillation
c) Sinus bradycardia
d) Sinus tachycardia
e) Ventricular premature beat

333. A patient who has COPD was admitted to the hospital due to acute exacerbation. All of the
following are true regarding his treatment except
a) Fluid replacement helps to remove mucus.
b) Ipratropium improves symptom of breathlessness.
c) Salbutamol improve symptom of breathlessness.
d) Inhaled corticosteroids given for maintenance.
e) Antibiotics use to treat exacerbation.
334. A 15-year-old obese male just been diagnosed with diabetes type 1. His mother also has
type 1 diabetes. He has been prescribed NPH insulin and metformin. All of the following are
the concern for this patient in the next month except?
a) To prevent diabetic nephropathy
b) To prevent macrovascular complications
c) To prevent peripheral neuropathy

335. Patient experiencing nausea and vomiting by donepezil (aricept)


a) The dose of donepezil
b) Change to other cholinesterase inhibitor
c) Discontinue donepezil
d) Decrease dose from 10mg to 5 mg

336. Physician is calling to take your advice about best drug for a fungal toenail infection?
a) Clotrimazole
b) Nystatin
c) Terbinafine  For 12 – 24 weeks

337. You recommend all of the following measures for a toenail fungal infection except
a) Clean feet and dry completely with a towel after bath (this will discourage fungal growth)
b) Do not walk bare foot on public places
c) Avoid cotton socks.
d) Use antifungal sprays and powders to absorb sweat and prevent rubbing.
e) Keep nails short and clean.

338. Which is the correct statement regarding post-antibiotic effect.


a) Exhibited by most of antibiotics
b) Even after the fall of C max drug shows effect.
c) Even after stopping the dose the effect lasts for few days.
d) All antibiotics have that effect
e) Continuous immunity after termination of that antibiotic

339. Patients prescribed to narcotics some days before today after 3 days got another rx, what is
pharmacist appropriate action
a) Refuse to fill
b) Talk with first dr
c) Talk with second doctor who rx today
d) Give him medicine as it is
340. HIV goals of therapy
 The primary goal of therapy, to preserve the health of the infected individual, is readily
achievable with current combination antiretroviral therapy cART, and in most cases, HIV
can be considered a chronic manageable condition.
 Prolong survival, improve quality of life. Slow disease progression,
 Delay/prevent emergence of drug‐resistant strains of HIV & decrease viral replication,
 Prevent transmission. Prevent/reverse immunologic impairment

341. Counselling of HIV meds (with or without food.... etc.)

342. Empiric treatment of gonorrhea infection? Scenario type


a) Penicillin-G
b) Doxycycline
c) Erythromycin
d) Cefixime
e) Cefuroxime

343. If the organism that caused gonorrhea is penicillinase producing, what should you give
him:
a) Ceftriaxone + azithromycin
b) Cotrimoxazale
c) Doxycycline + gentamycin

344. After his lab. results are in, you discover that the above patient is also infected with
chlamydia, so, what else should you give him:
a) Penicillin-G
b) Doxycycline
c) Erythromycin
d) Cefixime
e) Cefuroxime

345. What you should advice him about how long he should avoid sex?
a) 3 days
b) 1 week
c) 2 weeks
d) No need to avoid sex
346. DM patient, fasting is low and post prandial is too high?
a) Increase the morning insulin dose and decrease the evening one.

347. What causes nephrotoxicity? Gentamicin

348. Md calls to ask for a dose of prednisone for a kid, where to find? AHFS

349. Lamotrigine study well, a few questions on it.


Lamotrigine Blocks sodium channels, as well as high voltage-dependent calcium channels
Generalized tonic-clonic, absence, myoclonic, tonic/atonic, focal (partial) seizures or
unclassified tonic-clonic seizures
S.E: Q. Rash 5–10%, insomnia, Hepatotoxicity and hematological toxicity.
The most commonly observed adverse experiences associated with the use of adjunctive
therapy with lamotrigine (incidence of at least 10%) were dizziness, headache, diplopia,
somnolence, ataxia, nausea, and asthenia.
1 %: anorexia, weight gain, amnesia, concentration disturbance, confusion, emotional
lability, nervousness, nystagmus, paresthesia, thinking abnormality and vertigo
Advantages: BID dosing; broad spectrum; no enzyme induction (few interactions); some
patients report feeling more “alert”; increasing evidence for monotherapy; increasing use
for primary generalized seizures.
Disadvantages: Very slow dose titration.
Medications that induce lamotrigine glucuronidation include carbamazepine, phenobarbital,
phenytoin, primidone, rifampin, lopinavir/ritonavir and atazanavir/ritonavir
Medications that neither inhibit nor induce lamotrigine glucuronidation include olanzapine,
oxcarbazepine, felbamate, gabapentin, levetiracetam, pregabalin, topiramate & zonisamide
Valproic acid is an inhibitor of lamotrigine glucuronidation
Q. Addition of hormonal contraceptives reduce lamotrigine serum levels by up to 50%.
Expensive at high doses. Only available in oral form.

350. A pharmacist wants to make a campaign for pts, he wants to get the largest number of pts,
how to invite, all except:
a) Consult his pt profiles
b) Get names from heart and stroke foundation
c) Ask in the local health club

351. He wants to advertise about the campaign by asking a brochure, it will contain all except:
a) Working hours of the pharmacy
b) Cost of contributing in the campaign
352. A hospital formulary contains drug A, B, C at the concentration of 200mg, it wants to add
drug D 200 mg but not interchangeable with them. This could be because;
a) It's a sublingual tab
b) It’s an SR formulation
c) Not bioequivalent with them

353. A pharmacist wants to search for ‘pamidronate’ on medline, he types the word ’drug’, all
will appear to him except:
a) Koda kimble applied therapeutics

354. Rx/ Resorcinol - 5%


Zinc oxide Mg state aa 30%
Ale qs 180ml.
What is the amount of magnesium stearate used?
Ans; this was tricky cozu have to take care that u is using equal amounts of zinc oxide ans
mag, so u’ll divide the 30 % by 2 to get the amount of mag, then i’ll get the percentage as usual.

355. What affects the medication in elderly?


a) Intestinal decreases
b) Heart circulation decreases

356. Cancer pt had last chemo 2 weeks ago and presents to u today with severe nausea,
vomiting, fever? Refer to ER

357. Which is seen in a community pharmacy?


a) Cylindrical measure should be kept under sterile conditions when not in use
b) Glass measures should be replenished after one ear of use.
c) The use of plastic is preferred over glass especial or coal tar.
d) Cream can be prepared on a glass slab or a paper pad.

358. Patient has a history of nausea and vomiting and is depressed. Which is the least
antidepressant to cause nausea and vomiting?
a) Mirtazapine
b) Venlafaxine
c) Sertraline
d) Paroxetine
e) Amitriptyline
Adverse Effect Management
GI upset (nausea, constipation, diarrhea) Usually diminishes after 1–2 wk of therapy
Nausea: fluvoxamine (37%), venlafaxine (37%), Nausea: once-daily dosing; take with food; have
paroxetine (26%), sertraline (26%), vilazodone (24%), small, frequent meals; avoid strong smells
vortioxetine (23%), citalopram (21%), desvenlafaxine Constipation: increase fibre intake, fluids, laxatives,
(22%), fluoxetine (21%), duloxetine (20%) exercise
Constipation: fluvoxamine (18%), venlafaxine (15%), Diarrhea: have small, frequent meals; take
paroxetine (14%), duloxetine (11%) probiotics; limit caffeine, alcohol, spicy food; fluid
Diarrhea: vilazodone (29%), sertraline (18%), replacement
paroxetine (11%)

359. Which of the following is not an autoimmune disease?


a) Rheumatoid arthritis
b) Grave’s disease
c) SLE
d) Multiple sclerosis
e) Cushing’s syndrome

360. How do you withdraw the above patient from flurazepam


a) Withdraw gradually
b) Shift to diazepam and withdraw gradually
c) Give zaleplon

361. What is usually done in a pharmacy?


a) Keep balance in open air
b) Use a glass or paper pad for preparation of compounds
c) Plastic is better than glass in preparing coal tar.
d) Glass wear should be kept in sterile conditions
e) Glass cylinders should be changed every year

362. All are predisposing factors for urinary tract infection, except:
a) Short urethra
b) Low urinary Ph  this is a natural defense mechanism
Urine is an ideal medium for bacterial growth; factors that make it unfavorable for bacterial growth include
a pH less than 5, presence of organic acids and high levels of urea. Frequent urination is also known to
decrease the risk of UTI.

363. A married couple going to the caribbean, which vaccine should they take: (k-type)
a) Hepatitis A
b) Hepatitis B
c) Influenza
364. A type 2 diabetic patient is taking glyburide 2.5 mg OD and metformin 500 mg BID, his
Blood sugar levels are still high after 1 month of treatment, what should you do:
a) Switch him to insulin
b) Increase dose of glyburide

365. Which of the following, has the least potential for abuse?
a) Benzodiazepine
b) Buspirone

366. Two studies were done on two types of drug, the first one involved 500 patients and the
results were not statistically (p<0.05) and both drugs showed a decrease in mortality between
10%- 14%. The second study involved 2000 patients and the results were statistically
significant 12%-10.8%. The difference in results is due to:
a) Different sample in each study

367. You are a manager & informed that the pharmacist working today may not renewed his
license, what you should do?
a) Keep him away from counseling patients immediately (away from direct contact with pts)
b) Call the licensure agent and check yourself
c) Ask him to make sure if he renewed it or not yet. If not so send him home

368. Case, one will be travelling, come asking you about prophylaxis for malaria, you will
advise him?
a) Citronella is taken once daily
b) Prophylaxis is only for immunocompromised patients
c) Can be exposed to woods
d) Sleep in mosquito net

369. NSAIDS contraindicated with all except


a) Hypertension
b) Gout
c) Ulcer

370. Which oral contraceptive that if u miss pregnancy will happen?


a) Progesterone only pills
b) Combined oral contraceptives
c) IUD
d) Depo provera injection
Consistent timing of intake required; if dose missed by >3 h or missing 1 pill or more, a backup method is
required x 48 h.
371. Turn over calculation total sale/ total inventory

372. How to increase inventory? Get rid of inventory before the next midterm

373. Which One Gives Bitter Taste?


a) Oxazepam
b) Diazepam
c) Zopiclone
S.E: Impaired cognitive function and, rarely, anterograde amnesia and transient global amnesia.
Bitter/metallic taste. May cause dosedependent, next‐day impairment of activities requiring alertness,
including driving a car, despite the patient feeling fully awake.

374. Grandmother gave overdose tylenol to child, what to do? assess liver function by dr

375. American pt. Who runs out of supply, what pharmacist do? give 2 days supply.

376. Which one is risk factor of stroke?


Risk factors (HPT, DM, dyslipidemia, excessive alcohol intake, smoking, high BMI, low exercise, family
history of vascular disease or hemostatic disorders)

377. To reduce dispensing error all except? Tech ask pt. about indication of the drug

378. Doc of cellulitis? Cloxacillin

379. First thing to monitor in insomnia is? Sleep

380. Parents asking about amphetamine for their son, what to say?
a) Along with the benefits there are some SE

381. Which eye drop without anticholinergic? Xalatan

382. Fosamax direction of use.


The recommended dosage is one 70 mg tablet once weekly or one 10 mg tablet once daily.
FOSAMAX must be taken at least one‐half hour before the first food, beverage, or medication of the day
with plain water only. Other beverages (including mineral water), food, and some medications are known to
reduce the absorption of FOSAMAX. Waiting less than 30 minutes will lessen the effect of FOSAMAX by
decreasing its absorption into the body.
FOSAMAX should only be taken upon arising for the day. To facilitate delivery to the stomach and thus
reduce the potential for esophageal irritation, a FOSAMAX tablet should be swallowed with a full glass of
water (200‐250 mL). To facilitate gastric emptying, FOSAMAX oral solution should be followed by at least 60
mL (a quarter of a cup) of water. Patients should not lie down for at least 30 minutes and until after their
first food of the day. FOSAMAX should not be taken at bedtime or before arising for the day. Failure to
follow these instructions may increase the risk of esophageal adverse experiences.
All patients must receive supplemental calcium and Vitamin D, if dietary intake is inadequate.

383. All of the following are true for the Calcitonin salmon spray, except?
a) It can not be used in Shell fish allergy
b) Calcium and vitamin D are recommended with calcitonin therapy.
c) It is used in alternate dose.
d) Prime when use
one spray (200 IU) once a day administered intranasally, alternating nostrils daily into one
nostril only. You should switch between each nostril every time you use Calcitonin NS.
Your physician may prescribe calcium and vitamin D together with Sandoz Calcitonin NS to
help retard the progressive loss of bone mass. Upon, first use only, the pump must be primed. The product sh
ould be allowed to reach room temperature before priming
MAY 2009
1. NS,16 yrs old girl suffering from acne, taking Ovral (Norgestrel and Ethinyl Estradiol) &
Ibuprofen. She has not been complaining of acne for the last year. Now her acne is pustular &
inflamed. What are the risk factors that aggrevate her acne?
a) Age
b) Gender
c) Ovral
d) Ibuprofen
Hormonal changes such as androgenic and antiestrogenic progestogens, found in oral contraceptives is an
aggrevating factor of acne.

2. What do you advise her:


a) Go to the doctor
b) Facial scrub
c) Bezoyl Peroxide + Clindamycin
d) Exfoliative therapy
e) Benzoyl peroxide
Benzoyl peroxide Alone, or in combination, it is part of the standard of care for mild to moderate papular‐
pustular acne. 2‐3 months may be needed to see significant improvement after both topical & systemic agents

3. If dr prescribed isotretinoin for her acne. For how long must she not get pregnant after
finishing the course of ttt:
a) 1 month
b) 2 months
c) 1 week
d) 2 weeks
Females must not become pregnant while taking EPURIS or for at least one month after its discontinuation.

4. 16-year-old boy, with severe acne, Dr Rx Isotretinoin for him as he tried antibiotics for few
months but no cure. His mother is refusing saying she read it has many side effects especially
liver toxicity. The boy is mainly concerned about the look of his face and the social impact of
the acne & scars on his friends. From a non malifecence point of view, what is your action:
a) Call the physician to cancel it & to prescripe other medication
b) Tell her he has got the drug which the dr feel is good for him
c) Tell her that this dose will cause no problems
d) Discus risk & benefit with them & Tell her that every drug has its benefits and risks
e) Refuse to dispense because of the mother opinion
f) Tell him to continue on antibiotics for longer duration
5. What should be monitored intitially with isotretinoin
a) Gastric Acid Amylase
b) Renal SC
c) Liver SGPT
Blood potassium increased, blood alkaline phosphatase increased, blood bilirubin increased, blood urea
increased, elevated platelet counts, eosinophil count increased, false positive tuberculosis test, gamma‐
glutamyltransferase abnormal, blood cholesterol increased, glucose urine present, haematocrit decreased,
protein urine, thrombocytopenia, WBC count decreased.
Elevations in levels of serum creatine kinase CPK (monitor)
Patients should be advised to use a skin‐moisturizing ointment or cream and a lip balm from the start of
treatment as isotretinoin is likely to cause dryness of the skin and lips. When necessary a sun‐protection
product with high protection factor of least SPF 15 should be used.
Liver function tests should be monitored before treatment and at regular intervals during treatment (one
month after the start of treatment and at least three‐month intervals thereafter) unless more frequent
monitoring clinically indicated.
Serious Adverse Event Warnings include psychiatric disorders. Blood glucose levels & Serum blood lipid

6. What is the mode of natural prevention method of acne?


CTC: A systematic review of complementary and alternative medicines (CAMs) for acne concluded that there
is some low‐quality evidence from single trials that tea tree oil and bee venom may reduce total skin lesions
in acne. The same review cautioned that there is a lack of evidence to support the use of other CAMs such as
aloe vera, copaiba essential oil, dried fruit of Berberis vulgaris, seaweed oligosacchrides, acupuncture or
wetcupping in the treatment of acne.
Natural health products can cause adverse effects; future studies need to assess safety. Methodological and
reporting quality limitations weaken any evidence. The use of nonstandardized botanical preparations
should be discouraged in favour of traditional, quality‐controlled preparations that have evidence of efficacy
Niacinamide (nicotinamide) is the active form of niacin (vit. B3), reported to have antimicrobial, sebostatic
and anti‐inflammatory properties when applied topically but evidence of clinical effectiveness is scant &
inconclusive

7. To find Compatibility between Heparin and Nitroglycerin refer to OR A nurse is asking you
about adding heparin to an anxiolytic in one bag. (i.e. DDI) what reference you should refer to:
a) CPS
b) Micromedex
c) AHFS
d) Martindale
e) USPDI
f) Drug Interaction Facts
Alternative to CPS: Drug monographs, Drug identification, interactions, IV compatibility, Calculations, Patient
education, off label uses & Toxicology. Micromedex, pre‐2019 CPS and LexiComp have drug identification tools.
8. If pharmacist refuse to do counselling for terminally ill patient, what ethical principle violated?
a) Autonomy
b) Beneficience
c) Confidentiality
d) Veracity
e) Justice
Members should provide fair and equitable access to pharmacy services and deliver consistent quality of
care to all patients regardless of socio‐economic status, culture, disease state or any other related factor
that might unfairly bias patient care

9. Woman got depression, dr. Prescribed sertraline, all are true except:
a) It takes 6 months to see response
b) Take with food to increase absorption
c) If no response, then switch within same class
A minimum therapeutic dose should be achieved in the first 2 weeks of treatment, and increased if
necessary, over the next 4– 6 weeks.

10. Side effect of sertraline: Dry mouth


S.E: Nausea (high), tremors, diarrhea, dry mouth, sexual dysfunction, risk of GI bleeding

11. Now, she is very confused, what is the DDI:


a) Meperidine
b) Codeine
c) Omeprazole
d) Naproxen
Rare postmarketing reports describe patients with weakness, hyperreflexia, and incoordination following the
combined use of a selective serotonin reuptake inhibitor (SSRI) and 5‐HT1 agonists (triptans).
If concomitant treatment with sertraline and a triptan (e.g., almotriptan, sumatriptan, rizatriptan,
naratriptan, zolmitriptan), tricyclic antidepressants, or other drugs with serotonergic activity including but
not limited to amphetamines, fentanyl (and its analogues, dextromethorphan, tramadol, tapentadol,
meperidine, methadone and pentazocine), fenfluramine and tryptophan is clinically warranted, appropriate
observation of the patient for acute and long‐term adverse events is advised.
Rigorous clinical monitoring for suicidal ideation or other indicators of potential for suicidal behavior is
advised in patients of all ages. This includes monitoring for agitation‐type emotional and behavioral changes.
Close monitoring of glycemia in patients treated with ZOLOFT & oral hypoglycemics or insulin is recommended
since their dosage of insulin and/or concomitant oral hypoglycemia drug may need to be adjusted
Altered anticoagulant effects, including increased bleeding, have been reported when SSRIs and SNRIs are co‐
administered with warfarin. Patients receiving warfarin therapy should be carefully monitored.
ZOLOFT should be introduced with care in patients with a seizure disorder and should be avoided in patients
with unstable epilepsy; patients with controlled epilepsy should be carefully monitored. ZOLOFT should be
discontinued in any patient who develops seizures.
12. Drug Addict patient developed Cellulitis with inflamed shoulder. In Hospital, they took
biopsy from sternum bone & found S. aureus. Which is sensitive to Cloxacillin, Vancomycin &
linzolide. & resistant to penicillin. What is true about it?
a) This is MRSA
b) This is MSSA

13. What antibiotic would be given I.V for him?


a) Vancomycin
b) Cloxacillin
c) Cotrimox.

14. How long is the course of ttt:


a) 1 week
b) 2-3 weeks
c) 4-6 weeks

15. Woman with very busy life & has Asthma & migraine patient is coming for refill for
Salbutamol. N.B. the question was a big table indicating the medical history of the patient,
which was as follows: Amitriptyline, Salbutamol, Nicotine patch 21 stopped, Nicotine patch 14
stopped, Nicotine patch 7 stopped. What is the Drug related problem for this patient (DTP)?
a) Drug drug interaction
b) Drug disease interaction
c) Receiving wrong medication
d) Medication Required
e) Medication error
As he is not receiving ICS (inhaled corticosteroid) for the asthma, or any rescue medication for the migraine

16. What Non-Pharmacological to advice her for migraine?


a) Take a nap
b) Swimming

17. All are used as prophylaxis of migraine except


a) Nifidipine
b) Propranolol
c) Topiramate
BBs without ISA (e.g., propranolol, nadolol, metoprolol). TCAs (e.g., amitriptyline, nortriptyline).
Ca channel blockers (e.g., flunarizine, verapamil). Serotonin (5‐HT2) receptor antagonists (e.g., pizotifen),
valproic acid/divalproex sodium, topiramate, candesartan, lisinopril, gabapentin.
NSAIDs (e.g., naproxen sodium 550 mg twice daily for 1 week per month for menstrual migraine prophylaxis
Natural Health Products e.g: Butterbur 75 mg. CO‐Q10 (300 mg/day) & Melatonin 3 mg.
18. A patient with hypercalcemia which test do we use to confirm:
a) Ferritin
b) Serum iron
c) Urinary test
d) Aptt
e) INR
Serum ionized calcium or serum calcium and albumin, serum intact parathyroid hormone (PTH) before
giving bisphosphonates, serum parathyroid hormone‐related peptide (PTHrP) if PTH low, 24‐hour urine
collection to measure calcium‐to‐creatinine ratio, serum phosphate, alkaline phosphatase, total protein,
serum creatinine and urea.

19. What is your recommendation for


the ttt of hypercalcemia:
a) HCTZ
b) Na poly styrene
c) Etidronate
d) Insulin

20. Fosamax (Alendronate)


counseling: (k-type)
a) Tell pt not to eat for 30 min.
b) On empty stomach
c) Take with juice or food
Food & beverages other than plain water may markedly reduce absorption & effectiveness of alendronate.
ALENDRONATE must be taken at least one‐half hour before the first food, beverage, or medication of the
day with plain water only. Do NOT take ALENDRONATE with: mineral water, coffee or tea & juice
Do not take ALENDRONATE if you are pregnant or breast‐feeding. Do NOT chew or suck on tablets.
After taking your ALENDRONATE tablet do not lie down—stay fully upright (sitting, standing or walking) for
at least 30 minutes and do not lie down until after your first food of the day.
Take one ALENDRONATE tablet 70 mg once a week. If you miss a dose, just take one dose
of ALENDRONATE on the morning after you remember. Do not take two doses on the same day. Return to
taking one dose once a week, as originally scheduled on your chosen day.

21. Monitoring of alendronate


a) CrCl
b) BMD
c) Hypocalcemia
d) All
Serum calcium and symptoms of hypocalcemia should be monitored during therapy with ALENDRONATE.
22. A 65 yrs old male was diagnosed with MI,
all are markers of ACS, except:
a) Troponin 1 & 2
b) CK-MB
c) Serum Creatinine
d) Myoglobin

23. What do you give to initiate therapy:


a) ASA
b) Clopidogrel
c) Ticlopedine
d) Dalteparin
ASA Should be initiated promptly and
continued long term. ASA is routinely
combined with a second antiplatelet
from a different class. Clopidogrel
(plus ASA) significantly reduced the
incidence of major adverse cardiac
events in patients with ACS compared
with ASA alone, and reduced the
incidence of recurrent ischemia, an
effect which evident within few hours.

24. The above patient given


warfarin, so which medication
will increase the INR:
a) Phenytoin
b) CBZ
c) Phenobarbital
d) Fluconazole
e) Cholestyramine

25. Why give Sinemet 30 mins before the breakfast?


a) To avoid fatty acid interaction
b) To prevent drooling while eating
c) To prevent difficulty swallowing dyrin eating
d) Amino acids in food may interact with its absorption
e) Pyridoxine in food may interact in its activity
f) Tyramine rich food may interact in its activity
g) It can be taken in the night
26. Which of the following is not a side effect of Levodopa/Carbidopa?
Levodopa Levodopa, a dopamine precursor that is converted to dopamine in the brain by with enzyme
dopa decarboxylase, appears to correct akinesia, rigidity and tremors of Parkinson’s disease by
Levodopa / the formation of dopamine at the nigro striatial dopaminergic site.
carbidopa, Although used alone when first discovered, it is now combined with a DOPA decarboxylase
SINEMET® inhibitor (carbidopa or benserazide) to inhibit peripheral transformation to dopamine, thus
immediate enhancing distribution to the brain, reducing the amount of levodopa required for optimal
release therapeutic benefit and minimizing acute side effects such as nausea and vomiting.
SINEMET® When patients already receiving levodopa are switched to SINEMET®, levodopa must be
CR discontinued for at least 12 hours or more before SINEMET® is started. SINEMET® should
controlled be substituted at a dosage that will provide approximately 20% of previous levodopa dosage.
release. When patients receiving levodopa monotherapy or SINEMET® switched to SINEMET® CR,
this medication must be discontinued at least 8 hours before therapy with SINEMET® CR is
started. Dosage with SINEMET® CR 200/50 should be substituted at an amount that
Levodopa / eventually provides approximately 10 to 30 % more levodopa per day. The interval between
carbidopa, doses should be prolonged by 30 to 50 percent. This is because when Compared to Sinemet
intestinal gel immediate-release formulation, bioavailability of Sinemet CR is 25–30% lower and duration
of action 25–30% longer.
Because entacapone enhances the bioavailability and therefore central effects of levodopa, it
may be necessary to adjust dosage of levodopa during the initial days to weeks of entacapone
therapy in order to reduce levodopa-related dopaminergic adverse reactions, e.g., dyskinesias,
nausea, vomiting and hallucinations. In some cases, it may be necessary to reduce the daily
dosages of levodopa by about 10-30%. This can be achieved through either reducing the
dose of the levodopa preparation itself, or by extending the interval between doses, according
to the clinical condition of the patient.
Levodopa may be taken with food early in therapy to ease nausea; it may be taken on an
empty stomach in more advanced disease to help manage motor fluctuations
A change in diet to foods that are high in protein (such as meat, fish, dairy products, seeds and
nuts) may delay the absorption of levodopa and may not work as well as it should.
S.E: Nausea, vomiting, orthostatic hypotension, dyskinesias, hallucinations, confusion, mental
changes including paranoid ideation and psychotic episodes, depression with or without
development of suicidal tendencies, and dementia. Convulsions also have occurred.
Rare: Weight gain or loss, edema, Leukopenia, hemolytic and non-hemolytic anemia,
thrombocytopenia, agranulocytosis. Urinary retention, hematuria, and priapism.
Arrhythmias, non-specific ECG changes, phlebitis. Sialorrhea, bruxism, hiccups, GI bleeding,
flatulence, burning sensation of tongue, development of duodenal ulcer.
Long term use of sinemet can produce mydriasis & precipitation of glaucoma, melanoma.
Risk of parkinsonism hyperpyrexia syndrome with abrupt discontinuation; taper gradually.
Drug holidays not recommended.
Levodopa / carbidopa, intestinal gel: Complications related to percutaneous intrajejunal tube
placement e.g., infection, intestinal obstruction
When levodopa or a dopamine agonist are initiated, domperidone (10–20 mg 30 minutes prior
to each dose) can be helpful in minimizing gastric upset or orthostatic hypotension.
Antihypertensives, diuretics, tricyclic antidepressants may increase hypotensive action
Wearing off: Short duration response or "end dose" effect. The motor complications include
"off periods" of immobility or greater severity of the other parkinsonism symptoms & various
abnormal movements. This is due to decrease synthesis and storage of dopamine generated
from endogenous or exogenous levodopa.
On-off fluctuations: Most severe form of wearing off effect (abruptly freezes).
Sudden changes in movement control, which can last for a variable period of time.
‘On’ time is when levodopa is working well and your symptoms are controlled.
‘Off’ time is when levodopa is no longer working well and symptoms such as tremor, rigidity
and slow movement re-emerge.
Dyskinesia generally occurs when levodopa and other dopaminergic medications are at their
most effective. It is thought that an increased sensitivity to dopamine in the brain as a result of
treatment, combined with the natural progression of Parkinson’s, gives rise to dyskinesia.
Less commonly, dyskinesia can also occur when levodopa is just starting to take effect or
when it is wearing off – this is known as ‘diphasic dyskinesia’.
Avoid VB6 supplements as it reduces the effectiveness

Monitoring:
 Frequent monitoring of BUN, creatinine levels, and hepatic function because liver is
where the drug is decarboxylated.
 Also, it is important to test for intraocular pressure in patients with glaucoma.
 Testing for peripheral neuropathy before and while on levodopa is also very important.
Patients should also be regularly monitored for dyskinesia.
 Patients need to be observed for psychotic behavior and hallucinations when on
dopaminergic medications. Confusion and excessive dreaming can be accompanied
with hallucinations. Thus, patients with a history of past psychiatric disorders should
not be treated with levodopa.
 As an extra precaution, patients should be monitored for melanoma. It is not known yet
whether the risk of melanoma is increased due to levodopa use or Parkinson disease.
27. The most common side effect with Flouroquinolones
a) Neurotoxicity.
b) Hepatotoxicity
c) Nephrotoxicity
d) Hyper or hypoglycemia
e) Hyponatremia
The common side effects of the fluoroquinolones are gastrointestinal disturbances, headaches, skin rash and
allergic reactions. Less common but more severe side effects include QT prolongation, seizures, hallucinations,
tendon rupture, angioedema and photosensitivity.

28. Meperidine: (5 confusing statements)


a) If given in renal failure, will cause toxicity (seizures)
b) Can be prescribed with Selegiline
Meperidine is metabolized chiefly in the liver. The plasma elimination half‐life is normally 3 to 4 hours, but
this may be extended considerably in the presence of significant hepatic disease. In patients with cirrhosis,
bioavailability may be increased as much as 80%.
Meperidine undergoes N‐demethylation to normeperidine, which may then be hydrolyzed to normeperidinic
acid and subsequently conjugated. Normeperidine has a considerably longer plasma elimination half‐life (15
to 20 hours) than its parent molecule. Normeperidine has an excitatory effect on the CNS, which is linked to
tremors, muscle twitches, and seizures observed in patients with overdosage. In the presence of renal
insufficiency, normeperidine elimination is reduced.
The combination of meperidine and selegiline can provoke serious systemic reactions (serotonin syndrome).
Use of this combination is contraindicated. Avoid use with atomoxetine, bupropion, dextroamphetamine,
dextromethorphan, meperidine, pseudoephedrine, TCAs, MAO‐A inhibitors, SSRIs. Serotonin syndrome
Due to its adverse effect profile, meperidine (pethidine) should not be used for pain that will last more than
3 hours (Meperidine is not considered a first‐line option in the treatment of acute pain and should not be
given for pain that is expected to last more than 3 hours where morphine is a better choice. Limit its use to
short‐term (i.e., 24–48 hours) due to the accumulation of normeperidine, a neurotoxic metabolite that can
cause seizures in some patients and CNS effects such as tremors, hyperreflexia, hallucinations. Avoid
using meperidine in patients with renal failure or liver disease and in those who have received MAOIs in the
past 14 days. In some hospitals, meperidine has been removed from formulary because of these concerns.

29. If a person is allergic to codeine, what can be given:


a) Fentanyl (Duragesic).
b) Morphine
c) Hydrocodone
d) hydromorphone
There is very little evidence available to guide the choice of opioid in a patient with a history of anaphylactic
allergic reaction to a particular opioid analgesic. Alternative analgesics should be considered. Allergic and
anaphylactic reactions to the opioid analgesics are rare and ill‐defined. Opioids cause endogenous histamine
release to varying degrees, and histamine causes a number of allergies like symptoms.
If an alternative opioid is to be considered, the use of an opioid with different chemical and structural
properties may be considered. However, patients who exhibit true allergy to an opioid analgesic should be
monitored extremely carefully if another opioid is to be used.
Alternative analgesics such as paracetamol or NSAIDs should be considered.
Based upon theoretical considerations, a patient who continues to need an opioid after demonstrating a
true allergy to morphine or a semi‐synthetic opioid a trial of a synthetic opioid may be considered.
Patients should be monitored carefully if an agent from another class is substituted. Co‐administration of an
anti‐histamine or glucocorticoid may be considered.

30. What are the conditions in which you cannot give venlafaxine:
a) HTN
b) Hypothyroid
c) Angina
Venlafaxine Immediate Release: asthenia, sweating, nausea, constipation, anorexia, vomiting, somnolence,
dry mouth, dizziness, nervousness, anxiety, tremor, blurred vision, and abnormal ejaculation/orgasm and
impotence in men.
Venlafaxine XR: abnormal dreams, anorexia, dizziness, dry mouth, nausea, nervousness, somnolence,
sweating, and tremor as well as abnormal ejaculation/orgasm in men.
Dose‐related increases in blood pressure have been reported in some patients treated with venlafaxine.
Also, rare cases of hypertensive crisis and malignant hypertension have been reported in normotensive and
treated‐hypertensive patients in post‐marketing experience. It is recommended that patients receiving
venlafaxine has their blood pressure evaluated before starting venlafaxine and monitored regularly during
treatment.
Rigorous clinical monitoring for suicidal ideation or other indicators of potential for suicidal behaviour is
advised in patients of all ages. This includes monitoring for agitation‐type emotional and behavioral
changes.
Measurement of serum cholesterol levels (including a complete lipid profile/fractionation and an
assessment of the patient's individual risk factors) should be considered especially during long‐term
treatment.
31. In the ER which of the following will have drug diversion:
a) Nitrous oxide
b) Fentanyl

32. Finasteride, all true xpt:


a) Used for BPH, Alopecia, but dose in Alopecia is less than in BPH
b) Better synergism with alpha blockers for BPH
c) For alopecia, topical is better than oral
d) It should not be handled by pregnant females
e) Growing of hair is dose-related
f) In PBH, early clinical response seen within 6 months.

33. What is NOT true about Sulfasalazine:


a) Has 5-ASA as the active component
b) Causes photosensitivity, watch out for sunlight
c) Taken with food & 6-8 water glasses
d) Act systemically
Systemic administration is a route of administration of medication, nutrition or other substance into
the circulatory system so that the entire body is affected.
Administration can take place via enteral administration (absorption of the drug through the gastro
intestinal tract) or parenteral administration (generally injection, infusion, or implantation). Contrast
with topical administration where the effect is generally local

34. Sulfasalazine must take it with a supplement of or SSZ causes deficiencies of


a) Folic acid
b) Vit B12
c) Iron
d) Ca
Oral sulfasalazine inhibits the absorption and metabolism of folic acid and may cause folic acid deficiency.

35. What is the role of cholinesterase inhibitors in Alzheimer all xpt:


a) Improve cognition
b) Improve function
c) Delay progression
d) Halt progression
Cholinesterase Inhibitors: Donepezil, Rivastigmine, Galantamine. All are of equal efficacy, benefits are small
to moderate, mainly stabilize disease.
36. Drugs that causes SIADH syndrome of inappropriate antidiuretic hormone secretion?
a) Furosemide
b) Lithium
c) Sertraline
d) CCB
Although a growing number of drugs have been reported to produce SIADH, most published reports concern
vasopressin and its analogues, thiazide and thiazide‐like diuretics, chlorpropamide, carbamazepine,
antipsychotics, antidepressants (SSRI, TCA and MAOi), phenothiazine, Cyclophosphamide and NSAIDS
What are the symptoms of SIADH?
 The clinical manifestations of SIADH are predominantly attributable to hyponatraemia and serum
hypo‐osmolality. The severity of the signs and symptoms depends on the degree of hyponatraemia
and the rapidity with which the syndrome develops.
 Nausea or vomiting. Cramps or tremors. Depressed mood, memory impairment. Irritability.
Personality changes, such as combativeness, confusion, and hallucinations. Seizures. Stupor or coma.

37. How to correct it?


a) Use desmopressin
b) Spironolactone
c) HCTZ
d) Normal saline
Desmopressin can be given to increase the urine
concentration and reduce free‐water loss, thereby
limiting autocorrection of serum sodium level.
Drug‐induced SIADH usually resolves following
cessation of the offending agent(s).
Additional measures are required in patients with
symptomatic hyponatraemia, including fluid
restriction and intravenous sodium chloride and/or
furosemide therapy. Careful monitoring is essential, with particular attention paid to the rate and extent of
correction of the hyponatraemia.

38. Some tablets were collected from the floor; how do you identify them:
a) Micromedix
b) AHFS
c) Remington
d) CPS
CPS has CANADIAN: Drug Products information (product images & identification tool pre 2019), indications,
doses & dose calculation, units of measure, adverse reaction, interaction, contraindications.
Micromedex, pre‐2019 CPS and LexiComp have drug identification tools.
39. All need witness to destroy xpt:
a) Duloxetine (cymbalta)
b) Lorazepam
c) Codeine
d) Morphine 10mg

40. Dr. asking you about a SE of MXT (does MXT cause pruritus or not), what reference do
you check:
a) CPS
b) Remington
c) Trissel
d) PDR

41. In a study the drug is given to adults but not children, so which principle was followed:
a) Paternalism
b) Veracity
c) Non- Malificience
Members refrain from participating in behaviours/attitudes which could potentially result in harm and
utilize their professional judgment to make every reasonable and conscientious effort to prevent harm to
patients and society.

42. Patient have severe crohn’s


disease, taking cortisone & MTX,
but no effect, what to give
a) Infliximab
b) 5-ASA
c) Budesonide

43. Patient is now cured from crohn’s


disease & we want to give something
for maintenance.
a) Mesalamine
b) Infliximab
Biologic therapy; Infliximab and
adalimumab are effective in inducing and
maintaining remission in patients with moderate to severe CD and in inducing closure of perianal fistulae.
Combining infliximab with azathioprine is more effective than infliximab monotherapy.
44. A doctor wants to Rx. Tacrolimus for a lady for atopic dermatitis
(an unapproved indication). What does unapproved indication mean:
a) Not in the product monograph
b) Physicians cannot legally Rx it
c) Pharmacist cannot legally dispense it
d) The manufacturer cannot legally promote the unlabelled
indications to Drs
e) The physician can Rx on his own responsibility.

45. What is wrong about Tacrolimus:


a) Causes transient skin irritation
b) Patient can use it continuously
c) Patient will be more photosensitive
d) For use in children <2 y & immunocompromised patients
e) Can be used in face area.

46. The acquisition cost of tacrolimus is 2.15 $/gm. & the mark-up is 15 % & the dispensing fee
is 11 $. What would be the price for 30 gms of tacrolimus?
Answer:
So, cost is $2.15/ g and we need the cost for 30  $2.15/g X 30g = $ 64.5
Then markup is %15. So, 64.5 X 0.15 = $ 9.6.  9.6 + 64.5 = 74.10$
Now we add the dispensing fee of $11 so $74.10 + $11 = $85.10

47. What are the non-pharmacological measures that you should advice the patient with:
a) Do not use petrolatum on the face
b) Exposure to the sun will exacerbate the condition
c) Use benzoyl-peroxide to remove scales
d) Use scrap to remove scales

48. A guy brings back expired drug to the pharmacy. The pharmacist can do all except:
a) Reimburse him
b) Take & discard
c) Offer to give back to the manufacturer & give him back his money
d) Take it back & give it to the homeless

49. A 61 yrs old lady with breast cancer since 2 yrs., OA, HTN, COPD, & using ibuprofen,
acetaminophen for her pain. In order to assess her condition, you will ask her all except:
a) History of medication & Allergies
b) These symptoms are since when?
c) Do you have GERD or smoking Tobacco?
d) At what age she got her menopause

50. Why would you recommend her to take cytoprotection with ibuprofen?
a) History of arthritis
b) Age
c) Female gender
d) Risk of ulcer due to NSAIDs use
e) Pain not relieved
Prevention of Consider using a gastroprotective agent for all patients on chronic ASA or NSAID therapy who
PUD during have risk factors for PUD: > 65 years of age
ASA or NSAID 1) Use of high-dose or multiple NSAIDs
Therapy 2) Concomitant use of corticosteroids, antiplatelet agents (e.g., clopidogrel), anticoagulants
(e.g., warfarin, new oral anticoagulants) or SSRIs.
3) Severe comorbidity, e.g., HF, COPD, chronic renal or hepatic disease, malignancy
4) History of gastric or duodenal ulcer or upper GI bleeding
Accepted gastroprotective strategies include once-daily PPIs, misoprostol 800 µg daily (in 4
divided doses) and substitution of a traditional NSAID with a COX-2 inhibitor.
Eradication of H. pylori prior to initiation of ASA or NSAID therapy may reduce the risk of
symptomatic ulcers.

51. A patient taking robaxacet (Acetaminophen/Methocarbamol Caplets) for a long time


because it is not a Rx drug (OTC). What advice you should give him:
a) Avoid alcohol
b) Causes hepatotoxicity
c) You might feel dizzy & lightheadedness
d) N, V, D, are common problems
52. Which of the following is scheduel I?
a) Mupirocin
b) Dimenhydrinate
c) Orphenadrine
Orphenadrine hydrochloride is a schedule I. Orphenadrine citrate is a schedule III

53. A patient with an Rx & he is in a hurry, the tech. counseled him. So, what do you do?
a) Tell the tech. Not to counsel again
b) Tell the tech. To council only on otc
c) Call the patient to fix any mistake

54. A patient called the pharmacist & told him that she was in the pharmacy in the morning &
there was an error, you admit it. So, what is the most important thing you will do:
a) Assess the risk to the patient
b) Check how it happened
c) Check who made the mistake
d) Do nothing

55. Principles of Canada health act:


a) Affordability
b) Pricing
c) Accessibility
Public Administered and operated on non-profit basis by a public authority accountable to P/T
administration government
Universality The provincial and territorial plans must cover all residents on uniform conditions.
Portability The provincial and territorial plans must cover all residents when they travel within
Canada. Limited coverage is also required for travel outside the country.
When a resident move to another province, they can continue to use their original health
care insurance card for 3 months. This gives them enough time to register for the new
plan and receive their new health insurance card.
Accessibility Provide all insured persons reasonable access to medically necessary hospital and
physician services without financial or other barriers.
Comprehensiveness The P/T plan must insure all medically necessary services.
What medically necessary services are covered according to the Act?
1. Physician services if deemed medically necessary by the province (some services like
doctor’s notes, may not be deemed necessary)
2. Hospital care services If deemed necessary by province (some cosmetic surgeries may
not be covered)
3. Anything the province deems medically necessary e.g, diagnostics
4. Prescription drugs are not covered under the Act except through (2) above, i.e.
medically necessary “hospital care services”
56. Provinces & Territories fulfill all Canada Health Act Principles in order to be eligible to:
a) Have money (federal taxes) to cover what they spent on the needed medical survices
b) Have money (from the federal taxes) to cover what they spent on the needed medical
survices including the services, the patient payed for

57. You are in a busy pharmacy, what can you delegate to the tech.?
a) Take new Rx from old patients
b) Council on medications
c) Council on OTC

58. In hospital, adrug is about to be interchanged with another one. All must be done except:
a) Approval of the P/T committee
b) Approval of the key doctors
c) If the new medicine is at better price
d) The therapeutic index
e) Consult referee

59. Parameters for bioequivalence. All except:


a) Tmax
b) Cmax
c) AUC
d) Vd
Bioequivalent: Pharmaceutical equivalent + rate and extent are equal
Rate = “Cmax” and “Tmax” Extent = Area under the curve (AUC)

60. Patient took Phenytoin 20 mg, his phenytoin blood levels became 100 μmol/L, after a week
his Dr gave him Phenytoin 40 mg, his phenytoin blood levels jumped to 625 μmol/L, why did
this happen?
a) Because of saturation of Phenytoin hepatic elimination
b) Because of saturation of Phenytoin renal elimination
c) Increased protein binding of phenytoin
Phenytoin is eliminated by hepatic metabolism only (CYP 2C9). As the dosing rate increases, hepatic
enzymes become saturated, so clearance decreases & drug accumulates.
Since clearance is dose dependent, half‐life also changes with dose & concentration.

61. According to the above question, imagine what would his phenytoin plasma levels had been
if it followed 1st order kinetics?
a) 10 μmol/L
b) 20 μmol/L
c) 50 μmol/L
d) 100 μmol/L
e) 200 μmol/L

62. A phenytoin dose was to a child (100 mg) the serum level was (5 nm). It was not effective
so the dose was increased to (200mg). Assuming first order kinetics the serum level should be:
a) 10 nm  100 mg -------- 5 nm 200 mg -------- X nm X=200*5/100 = 10 nm
b) 20 nm
c) 5 nm

63. A patient coming from the USA, when you check his BGL it is 6.2 mmol/L. so how do you
tell him this in American units. Which book do you refer to?
a) CPS
b) PSC
c) Merck index
Compunding Remington (The Science and Practice of Pharmacy)
American Stability/solubility of ingredients for compounding.
Basic information about compounding techniques and ingredients.
Merck Index Provides data, descriptions of chemicals, drugs, biological & solubility.
American The book contains subject matter includes human and veterinary drugs,
biological and natural products, agricultural chemicals, industrial and
laboratory chemicals, and environmentally significant compounds.

64. Again, this patient comes to your pharmacy & his flight is delayed for 1 week & needs
methylphenidate for his 1 week, what you do:
a) Give MF
b) Ask for an authorized Rx
c) Do not give MF

65. When an Rx for BDZ is transferred from one pharmacy to another. Do you need a copy
from the original Rx? https://www.ocpinfo.com/practice-education/practice-tools/fact-sheets/transfers/
A prescription may be transferred either under the signature of a member who is practising at the pharmacy
transferring the prescription or verbally by a member who is practising at the pharmacy making the
transfer.
keep records of:
 a copy of the prescription written by the practitioner or the record made in accordance with the
practitioner’s verbal prescription;
 the name and business address of both the transferring pharmacist and the pharmacist receiving the
prescription transfer;
 if applicable, the specified interval between refills.
66. A patient newly diagnosed with DM + HTN, BP 142/95. He is drinks 8-10 coffees/day. He
is 100 kg. what do you advice him:
a) Take <10 mmol of salt
b) Take <15 mmol of salt
c) Exercise 1-2 times / week
d) Eat a low-fat diet
Weight loss of 4 kg or more if overweight (target BMI: 18.5–24.9 kg/m2; waist circumference <102 cm in
men and <88 cm in women)  Reduce by 4.5 kg reduce B.P −7.2/−5.9 mmHg
Healthy diet—high in fresh fruits, vegetables, soluble fibre and low‐fat dairy products, low in saturated fats
and sodium, e.g., Q. DASH eating plan reduce B.P by −11.4/−5.5 mm/hg
Consider sodium intake target of <2000 mg (88 mmol) per day. Reduce by 1800 mg (78 mmol)/day reduce
B.P −5.8/−2.5 mm/hg

67. The above patient was advised by the Dr. to buy a BP monitor, so advise the him to keep his
BP below:
a) 130/80
b) 135/85
c) 150/85

68. To find solvent for 30% urea, refer to?


a) Micromedex
b) Remington
c) USP-DIV
d) AHFS

69. A patient with bronchoconstriction COPD & glaucoma. What is the doc for his glaucoma?
a) PG analogue Latanoprost
b) BBs timolol
c) CAIs dorzolamide

70. The above patient is given latanoprost, you would counsel her to
a) Store it in the fridge
b) Causes eye pigmentation or stains iris
XALATAN may gradually increase the pigmentation of the iris. This effect has predominantly been seen in
patients with mixed coloured irides, i.e., blue‐brown, grey‐brown, green‐brown or yellow‐brown. The eye
colour change is due to increased melanin content in the stromal melanocytes rather than to an increase in
the number of melanocytes. This change may not be noticeable for several months to years. Typically, the
brown pigmentation around the pupil spreads concentrically towards the periphery of the iris and the entire
iris or parts of the iris become more brownish. Store unopened bottle under refrigeration (2 to 8°C). Protect
from light. During shipment, the bottle may be maintained at temperatures up to 40°C for a period not
exceeding 8 days. Once opened, bottle may be stored at room temperature up to 25°C, for up to six weeks.
71. What is the most common SE of lithium?
a) Tremors
b) Headache
c) Diarrhea
d) Weight gain
e) Hyperhidrosis
The more common persistent adverse effects include fine tremor of the hands, fatigue, thirst and polyuria.

72. A 22 yrs old lady with asthma & she has Rx of


salbutamol & she is not controlled (she has SOB,
asthma). What is the gold standard will show that
asthma is not in control?
a) 1 attack/week
b) 2 attacks/week
c) 4 attacks/week

73. When you notice that she is not regularly using Cs, what could be the reason?
a) Afraid of growth suppression
b) Cataract
c) Fungal infection of the mouth
74. What will you advise her to do?
a) Go to the Dr.
b) Use Cs continuously & assess in 2 weeks
c) Addition of LA B-agonist

75. Patient with diarrhea, when referral to the Dr. is most important:
a) Stomachache
b) Melena
c) Back pain
Red Flags: See a health‐care provider if you have any of these symptoms:
 Blood or abnormal mucus in stool or stool looks black
 Extensive abdominal cramping or pain & Severe pain in your belly
 Fever >38.5˚C (over 38.5°C or 101°F)
 Frail elderly & Young age (< 2 y).
 Immunocompromised (e.g., HIV infection, immunosuppressants)
 Persistent vomiting for > 4 h
 Pregnancy or Presence of chronic medical conditions (e.g., DM, HF, kidney dysfunction)
 Recent use of antibiotics particularly those associated with Clostridium difficile colitis (e.g.,
clindamycin, ampicillin, cephalosporins)
 Persistent, chronic & Severe diarrhea (> 6 unformed stools per day for >48 h) & Worsening diarrhea
 Symptoms of dehydration (thirsty, weak or lightheaded, dry mouth or tongue, not urinating as usual)
 Weight loss due to diarrhea

76. Patient got stroke 6 hour ago, what is the first sign for stroke?
a) Slurry speech
b) Confusion
c) Headache
First headache, confusion, blurred vision, fainting, incoherent speech, dysarthia (difficulty of speech)

77. What is the first treatment of acute stroke?


a) ASA
b) Ramipril
c) Metoprolol
If intracranial hemorrhage is excluded by CT scan but alteplase is not indicated, give ASA 160 mg
immediately. Follow with ASA 80–325 mg daily.
When alteplase is used, wait until intracranial hemorrhage is excluded by CT scan 24 hours after treatment,
then give ASA 160 mg once followed by ASA 80–325 mg daily.
Administer ASA as a suppository or via nasogastric tube to dysphagic patients.

78. Why using alteplase is not acceptable to him? Because he got stroke 6 hour ago
79. A female with stress incontinence, how is it caused?
a) Urethral hypermotility & intrinsic sphincter deficiency.
The most common form of UI in women; it is the consequence of weakened pelvic floor muscles.
Involuntary urine leakage is the result of increased intra‐abdominal pressure (e.g., from laughing, coughing
or exercise) causing pressure inside the bladder to overcome the ability of compromised urethral sphincter
to close urethra.
Intrinsic sphincter deficiency is a less common form of stress incontinence and may occur after pelvic
surgery, irradiation, antiincontinence procedure, vaginal birth or in conjunction with neurologic problem

80. The ttt of the above patient would be:


a) Desmopressin
b) Pelvic floor muscle training
c) Oxybutinin
d) Conjugated estrogen

81. UTI, & fever. In the last year she had UTI 3 times & no drug plan, so what is the most
common bacteria:
a) E. Coli
b) P. Aerogenosa
c) E. Bacteriacea

82. The DOC would be:


a) Moxifloxacin
b) Nitrofurantoin
c) TMP
d) Amoxicillin

83. A patient with genital herpes & taking acyclovir, what is the expected outcome?
a) Prevent recurrence
b) Decrease pain
c) Decrease duration
Treatment is effective in reducing the severity and duration of symptoms if initiated up to 7 days after onset.

84. arrhythmia case & patient suffer from urinary incontinence, finally the question is which
antiarrhythmic drug with anticholinergic side effects or all can be used for this patient, except?
a) Quinidine
b) Procainamide
c) Disopyramide
d) Sotalol
Urinary retention may occur in patients of either sex, but males with benign prostatic hypertrophy are at
particular risk. If acute urinary retention develops, RYTHMODAN therapy should be temporarily discontinued,
except in occasional instances, in which continued control of the arrhythmia with RYTHMODAN is considered
mandatory. In such cases, overriding measures should be taken (e.g., catheter drainage or operative relief).
If RYTHMODAN is discontinued, and later reintroduced, a lower dose should be used
Disopyramide phosphate should be avoided in patients with glaucoma. In patients with a history or family
history of glaucoma, intraocular pressure should be measured before initiating treatment.

85. An 18 yrs old girl was taken to the hospital after a suicidal attempt. She was then diagnosed
with depression, anxiety, instability (& many others –she was a psycho). She plans to repeat the
suicidal attempt once she gets out of the hospital. So, you advise her psychiatrist not to Rx:
a) Bupropion
b) Moclobemide
c) Desipramine
d) Paroxetin
e) Citalopram
Rigorous clinical monitoring for suicidal ideation or other indicators of potential for suicidal behaviour is
advised in patients of all ages given an antidepressant drug. This includes monitoring for agitation‐type
emotional and behavioural changes.

86. A lady with CAP, the dr. advised that she does not need hospitalization, so what is the
antibiotic of choice:
a) Doxycycline
b) Ampicillin
c) Floroquinolone
Organism Recommended Antibiotics
Streptococcus Penicillin nonresistant (Minimum inhibitory concentration MIC <2 mg/L):
pneumoniae  Initial therapy: penicillin G, amoxicillin
 Alternatives: macrolide, PO cephalosporins (cefprozil, cefuroxime), IV cephalosporins
(cefuroxime, ceftriaxone, cefotaxime), clindamycin, doxycycline, respiratory
fluoroquinolones
Penicillin resistant (MIC ≥2 mg/L):
 Initial therapy: cefotaxime, ceftriaxone, respiratory fluoroquinolone (PO or IV)
 Alternatives: vancomycin, linezolid, H.D amoxicillin (3 g/day for penicillin MIC ≤4
mg/L)
Haemophilus 2 or 3rd generation cephalosporin, amoxicillin/clavulanate, fluoroquinolones, doxycycline,
nd

influenza azithromycin, clarithromycin; amoxicillin monotherapy if nonbeta-lactamase producing


Staphylococcus  Methicillin-susceptible MSSA: cloxacillin, cefazolin, clindamycin
aureus  Methicillin-resistant MRSA: vancomycin, linezolid, tigecycline
Legionella sp. Legionnaires’ disease - severe form of pneumonia
 Fluoroquinolones or azithromycin; Alternative: doxycycline
Spread by human-made building water systems.
M. pneumoniae  Macrolides or tetracyclines
C. pneumoniae  Alternative: fluoroquinolones
Coxiella Cause Q fever - mild disease with flu-like symptoms.
burnetii  Doxycycline or fluoroquinolones;
 Alternative: macrolides (although some strains may be resistant)
Pseudomonas  Antipseudomonal beta-lactam Imipenem/cilastatin plus ciprofloxacin or
aeruginosa aminoglycoside
 Alternative: aminoglycoside plus ciprofloxacin

87. A lady, her child was sent back from school coz of pediculosis. The whole school was
infested. What do you advice?
a) Cut the hair short
b) Soak hair for 1 hour in vinegar to remove the nits
c) TTT pets as well
d) If lice away from host they die (Didn’t mention 48 hours)

88. In resistence to lice, all can be done except


a) Oral clindamycin
b) Repeat
c) Switch to another agent
Treatment Failure of head lice treatment may be due to resistance, misdiagnosis, lack of adherence, or
failure of re-infestation. Treatment with permethrin may fail if hair is not thoroughly soaked. Two
head lice bottles are often needed for thick or long hair.
 If the lice/scabies are truly resistant, switching to another class may be helpful.
 One regimen has been Permethrin 5% cream overnight on scalp.
 Suggested Oral Sulfamethoxazole-Trimethoprim in combination with Permethrin
 Topical crotamiton 10% applied to scalp and left on for 24 hours in adults.
 Oral ivermectin 200 mcg/kg repeated in 10 days or an alternative regimen of 400
mcg/kg repeated in 7 days.
Ivermectin is not approved for use in children weighing <15 kg.
Topical ivermectin 0.5% effective for treatment of head lice, but is not available in Canada.
Topical ivermectin 1% is available in Canada for the treatment of rosacea but it has not been
studied in the treatment of lice or scabies.

89. Q about coal tar


 In Psoriasis, it is used with UVB lamp “UVA with Psoralen or UVB with tar”
 It is better to use plastic better than glass in preparing of coal tar
 Contraindicated in patients with acutely inflamed or broken skin, pustular psoriasis and in the presence
of any skin infections. Coal tar should not be used near the eyes, around the genital, groin or rectal area.
 It is recommended that women avoid use of coal tar during pregnancy or breastfeeding.
 Because of carcinogenic potential, some recommend that chemical gloves be worn during extemporaneous
preparation of coal tar formulations.
 Coal tar should be used cautiously with drugs that have known photosensitivity or phototoxic potential.
 To avoid falls, patients should be warned that coal tar preparations can make the bathtub slippery.

90. Mometasone is a CS that:


a) Has the worst effect on hypothalamic-pituitary-adrenal HPA axis
b) Causes growth retardation
c) Has to be shaken before use
Prior to administration, NASONEX nasal pump should be primed by actuating the pump 10 times (until a
uniform spray is observed). If the spray pump has not been used for 14 days or longer, it should be reprimed
with 2 actuations, until a uniform spray is observed, before next use. Shake container well before each use.
Patients should be instructed on the correct method of use, which is to blow the nose, then insert the nozzle
carefully into the nostril, compress the opposite nostril and actuate the spray while inspiring through the
nose, with the mouth closed.

91. A lady going to a business meeting in Chicago & want a drug for her runny nose, post nasal
dripping, congested nose. She has Sinusitis, hypertension, she is otherwise healthy. She is
looking for a quick relief for her symptoms, what will you recommend?
a) Nasal Phenylephrine
b) Oral Loratidine
c) Oral Pseudoephedrine
d) Oral Diphenhydramine & nasal pseudoephedrine???
e) See her Dr

92. A patient wants 1000 tabs for his friends in the USA (pseudoephedrine), when pharmacist
asks, the patient says that he needs the medication for his friends back home. The pharmacist
refuses to give the drug for all reasons except:
a) Pseudoephidrine is a base for making methylphenidate
b) Absence of a pharmacist-patient interaction
c) Pseudoephedrine has a high potential for interaction
d) Drugs are allowed into USA only for personal use
Ephedrine or pseudo‐ephedrine — strong nasal‐decongestants — are legally used to manufacture medicinal
products against flu or allergies, but they are also key products used in illicit production of methamphetamine.
Only 1.5 kg of ephedrine/pseudoephedrine is needed to obtain 1 kg of methamphetamine. Assuming that one
tablet contains 60mg of pseudoephedrine, 25 000 tablets are needed to obtain 1 kg of methamphetamine.

93. Patient weight 55 Ibs was prescribed Amoxicillin which comes in 2 concentrations
125mg/5ml (3.2 $ /100 ml bottle) & 250mg/5ml (5.4 $ /100 ml bottle). The Dr. prescribed 7.5
mg/Kg to be given TID for 10 days. How many bottles will be more economic to dispense this
prescribtion?
a) 2 bottles of 125
b) 2 bottles of 250
c) 3 bottles of 125
d) 1 bottle of 125 + 1 bottle of 250
Answer:
Weight = 55Ibs/2.2 = 25 Kg Dose = 7.5mg/Kg*25*3times/day*10days=5625 mg
125 mg ‐‐‐‐‐‐‐ 5 ml X mg ‐‐‐‐‐‐‐‐ 100 ml X = 125*100/5 = 2500 mg
250 mg ‐‐‐‐‐‐‐ 5 ml X mg ‐‐‐‐‐‐‐‐ 100 ml X = 250*100/5 = 5000 mg
So, to get 5625 mg It's better to get a bottle of 125mg/5ml & a bottle of 250mg/5ml
Cost = 3.2+5.4 = 8.6 $

94. Preparation of 5% stock solution, you want to make 0.0125% so you use.
a) 5 ml in 2 liters
b) 5 ml in 1 liter
c) 2.5 ml in 2.5 liter
Answer:
C1*V1 = C2 * V2 0.0125% * V1 = 5 % * V2 V1/V2 = 5%/0.0125%=5/0.0125=400
So, 5 ml in 2 L= 5 / 2000 = 1/400 5 ml in 1 L= 5 /1000 = 1/200 2.5 ml in 2.5 L= 2.5 /2500 = 1/1000

95. You have a progesterone suppository prescription that you have to compound in the
pharmacy, you want to select the base to be used so you refer to:
a) Remington
b) CPS
c) Martindale
d) AHFS

96. Another calculation where you are supposed to calculate the base to prepare progesterone
suppository. The answer was: 3.75

97. All can be given written & verbal order except:


a) Fiorinal C1/2
b) Hydrocodone
c) Pentobarbital
d) ASA+ codeine

98. Methadone 8.1 gm Na Benzoate 1.15 gm Liquid qs 810 ml


What is the amount of Methadone & Na benzoate to prepare 900 ml?
Answer:
Methadone = 8.1/810 * 900 = 9 gm
Na Benzoate = 1.15/810 * 900 = 1.27 gm
99. Patient in hospital is taking IV Cotrimoxazole (each ml contain 80 mg SMT / 16mg TMP)
0.5 ml Q6hr. Dr. want to shift him to liquid (200 mg SMX / 40 mg TMP) in each 5 ml, how
many of liquid is needed to provide the equivalent dose as was taken IV.
Answer:
0.5 ml Q6hr = 0.5 * 4 = 2 ml /Day
SMT: 80 mg -------- 1 ml X mg -------- 2 ml X= 2*80/1 = 160 mg
Liquid 200 mg --------5 ml 160 mg -------- Y ml Y = 160*5/200 = 4 ml/Day

100. A representative comes with a new drug Y-NSAID & said it is better than X-NSAID in ttt
of menstrual pain due to it’s enteric coating, longer BA. So, pharmacist concludes that (k-type):
a) More appropriate for chronic cases than menstrual pain.
b) The SR effect is due to the EC
Enteric coated tablets just protect stomach, and it has delayed effect but I don’t think it prolongs duration

101. Avery long case about Gonorrhea in man, then they ask about the
M.O causing non-gonococcal infection
a) Plasmodium
b) Amoeba
c) Chlamydia
The 2 main causes of urethritis and cervicitis are gonorrhea (caused by Neisseria
gonorrhoeae) and chlamydia (caused by Chlamydia trachomatis). Nongonococcal,
nonchlamydial causes of urethritis and cervicitis include Mycoplasma genitalium,
Ureaplasma urealyticum, T. vaginalis, (HSV), (VZV) and adenovirus.

102. 1st line ttt of Gonorrhea treatment


a) Cefixime
b) Quinolone
c) Ampicillin
Preferred treatment: Cefixime, Ceftriaxone (Preferred antibiotic in pregnant and breastfeeding women)

103. Gonorrhea always come as mixed infection (i.e. gonococcal & non-gonococcal) so what is
the best treatment for the original case
a) Cefriaxone + PenicillinV
b) Ceftriaxone + Azithromycin
c) Cefotaxime + Penicillin V
d) Cefotaxime + Azithromycin

104. After culture it was proven Chlamydia, what should be given? Azithromycin
Preferred treatment. Doxycycline 100mg for 7 days & azithromycin 1gm single dose
105. Female with vaginal itching & fishy odour:
a) Refer to her Dr because she has fishy odour
b) Give her OTC treatment for candidiasis

106. She came back with an Rx for metronidazole vaginal supp., what to counsel her?
a) Douche with warm water to decrease the fishy odour
b) Use condoms during intercourse
c) Avoid taking alcohol

107. A patient on many drugs (HCTZ+OHTERS) he also has renal failure + CHF + HTN & his
weight increased 3 kg in the last week, what would be your advice:
a) Switch from HCTZ to Furosemide

108. Patient on salbutamol prn, & medium potency corticosteroid & just added montelukast, all
of the following are the benefits of montelukast except:
a) Decrease exercise induced asthma
b) Decrease nocturnal asthma
c) Decrease the need for corticosteroids
d) Decrease the number of times salbutamol will be needed
e) Abolish the use of corticosteroids

109. Which medication does not cause weight gain or all causes weight gain, except?
a) Gabapentin
b) Lamotrigine
c) CBZ
d) Phenytoin
e) Olanzapine
f) Amitryptilline
g) Steroids
110. What is true about Cardiac Glycosides: (k-type)
a) Low therapeutic index
b) Used for arrhythmia & AF
c) Causes –ve inotropic & +ve chronotropic

111. A black patient with HTN & DM, his father has history of angina, his mom has history of
cancer, he drinks a lot of alcohol, fired from his job, under a lot of pressure. What is not a risk
factor for HTN:
a) Stress
b) DM
c) Alcohol
d) Mom’s history
e) Dad’s history
Cardiovascular risk factors
 Non‐modifiable: age > 54 years, male, family history of premature CVD (age<55 male and < 65 female)
 Modifiable: sedentary lifestyle, poor diet, abdominal obesity, smoking, dysglycemia, stress, non‐
adherence, cigarette and alcohol use, high sodium intake, diabetes and dyslipidemia.
Causes of hypertension aggravation:
 Alcohol (excessive use), licorice, salt, Excess thyroid hormone: exogenous or endogenous.
 EPO, midodrine, MAOIs, NSAIDs, OC, sex hormones, corticosteroids, stimulants (cocaine, nicotine,
amphetamines, Appetite suppressants: ephedra, caffeine,
 Sympathomimetic decongestants, venlaxafine, calcineurin inhibitors (cyclosporine, tacrolimus).
 Medical causes of HTN: Kidney disease, Thyroid disease, Pheochromocytoma, Hyperaldosteronism,
Sleep apnea & Obesity.

112. Then he was given ACE-I, for what reason ACE inhibitor is prescribed? (I cannot
remember what was the Q about, but it was not about the inappropriateness of ACE-I in black
population) (k-type)
a) To control BP
b) Decrease CVD
c) Increase circulation in foot

113. Tylenol # 3 is taken for how long if not mentioned by Dr.


a) TID when needed for 7 days
b) BID for 10 days
c) TID for 10 days
It is a narcotic preparation, need specific quantity and duration to be written in the prescription
114. You do not tell a patient that he is about to die because of:
a) Non-maleficence
b) Beneficence
c) Autonomy

115. 62 years female in palliative care, grade 4 cancer & he is on 30 mg codeine bid, 40 mg
morphine qid & fentanyl. He has excrociating pain, he wants to increase threshold of dose, what
u have to do?
a) Increase threshold, risk of addiction minimal
b) No, increase the threshold
c) Give acetaminophen and ibuprofen
d) Increase threshold with secondary Dr opinion
Metastatic cancer ... we should not be afraid of addiction as pain is always intolerable and wherethere is
pain, there is no overdose or addiction

116. A lady with problems so having poor sleep, confusion, fatigue, we give her antidepressant
to help to:
a) Improve mood
b) Improve sleep
c) Get back to work

117. A pharmacist wants to improve dispensing care & ↓ dispensing errors. He must do all xpt:
a) Let the tech. enter the data manually.
b) Using automation for filling prescriptions
c) Double checking while dispensing

118. You are a pharmacy manager and there is a pharmacist with above than normal sick leaves.
He asked for a vacation but it was refused. Then, on Saturday morning, he called sick. What is
the first step a manager should do?
a) Apply disciplinary action
b) Call him to find out, how many days he’ll be off
c) Fire him
d) Call his Dr to verify he is sick  illegal

119. A drug excreted renally, how can you enhance further excretion:
a) Alkalinizing the urine
b) Acidifying the urine
c) Reversing dehydration (increasing water intake)
It depends on drug nature, acidify with basic drugs & alkalinize with acidic drugs.
120. A diabetic patient taking glyburide 2.5 mg bid, metformin 500 Mg tid, he has hypoglycemia
right after exercise at 11 am. What do you advice him:
a) Increase the dose of MF
b) Increase the dose of glyburide
c) Decrease MF
d) Decrease glyburide
e) Give him a snack before exercise

121. Which drug must be written:


a) Ketamine
b) Tylenol 2 (acetaminophen 300 mg + codeine 15 mg + caffeine 15 mg)
c) Tylenol 3 (acetaminophen 300 mg + codeine 30 mg + caffeine 15 mg)
d) Clobazam
e) Fiorinal C1/2 (butalbital + ASA+ codeine 30 mg).
f) Phenobarbital

122. A lady is on several medications Digoxin, Enalapril, ASA & zopiclone. She wants to stop
them due to experiencing metallic taste. What medication is causing this problem:
a) Zopiclone
b) Omeprazole
c) Digoxin
d) Amitrypituine
e) Enalapril
Zopiclone: Treatment should rarely exceed 7–10 days.
S.E: Impaired cognitive function and, rarely, anterograde amnesia and transient global amnesia.
Bitter/metallic taste. May cause dosedependent, next‐day impairment of activities requiring alertness,
including driving a car, despite the patient feeling fully awake.

123. Which one of these has to be shaken?


a) Miacalcin (calcitonin-salmon)  prime
b) Salbutamol inhaler  shake, prime 1st time and if not used for 5 days
c) Imitrex (sumatriptan nasal spray)  no prime, no shake
d) NG spray  do not shake, need priming in 1st time or if not used for 14 days.
e) Stadol (Butorphanol Nasal Spray, 10 mg/mL)  prime, no shaking
f) Livostin spray (levocabastine Nasal Spray)  should be shaken before each
application. Before using the pump delivery system for the first time, the pump
reservoir should be filled up by priming until a fine spray is delivered.
124. A Dr wants to make pamphlets for respiratory patients and asks the pharmacist for help, all
should be included in the pamphlet except:
a) Show the refills to the dr. on file
b) Teach the pt. how to use expiratory flow meters
c) Ask the pharmacist for advice about medication
d) Mention the SE of the drug
Another version
Physician asks pharmacist to prepare a pamphlet to distribute to patients about importance of
refills, he should do all except
a) Mention to bring refills for physician on regular schedules to refill it
b) Mention the team strategy of physician, pharmacist and patient
c) Tell patients to follow up with pharmacy in case of any deviation from the dose

125. A newly diagnosed pt of hypothyroidism comes with a Rx of levothyroxine


a) Start with low dose and increase gradually

126. pt. with salbutamol also T4, NRT and sulfa allergy, which drug is inappropriate
a) T4
b) Salbutamol
c) Ibuprofen

127. Which has to be taken empty stomach


a) Fluvastatin & Lovastatin w
b) Cloxacillin Take 1 h before or 2 h after meals.
c) Clindamycin w/wo & full glass of water
d) Levothyroxin
e) Cefuroxime axetil w

128. which of the above medicines does not cause pseudomemberanous colitis
a) Fluvastatin
b) Cloxacillin
c) Clindamycin
d) Cefuroxime axetil

129. A guy with knife comes to your pharmacy and asks for hydrocodone. what would you do?
a) Try to detain him in pharmacy
b) Ask him why he needs it and try to make a conversation
c) Advise him to stop taking drugs
d) Give him what he wants & try to make mental note
130. As an effective manager of that pharmacy what would u do?
a) Report to college of physicians
b) Report to provincial college
c) Call the victim support to help the assaulted pharmacist
d) Report to narcotics and drug control board

131. An 8 yr old boy is started on methylphenidate for alternate weeks (One-week MP and one-
week Placebo). Mother is concerned that her child might get addicted, what is true
a) Methylphenidate doesn’t cause addiction
b) If not treated now, when he will grow, he will get addicted
c) Use SR formula
d) Use a long acting formula ex: concerta, biphentin

132. How can we monitor if the drug gives effect or not?


a) Weight Loss
b) Nausea
c) Insomnia
d) Hyperactivity
e) Impulsive behaviour
CONCERTA tablet uses Osmotic Controlled‐Release Oral Delivery System (OROS) to release the medication in
a controlled way throughout the day. Part of the CONCERTA tablet dissolves right after you/your child
swallow it in the morning, giving you/your child an initial dose of medication. The rest of the medication is
slowly released during the day to keep improving the symptoms of ADHD. CONCERTA helps increase
attention and decrease impulsiveness and hyperactivity in patients with ADHD. Impulsive behaviour needs a
long time to be modified so it is not an indicator for the drug effectiveness
Blood pressure should be monitored at appropriate intervals in patients receiving CONCERTA, especially in
patients with hypertension.
Suppression of growth (i.e. weight gain and/or height) has been reported with the long‐term use of
stimulants in children. Therefore, patients requiring long‐term therapy should be carefully monitored.
Patients who are not growing or gaining weight as expected should have their treatment interrupted.
Regular monitoring for the emergence or worsening of tics or Tourette’s syndrome during treatment
with methylphenidate is recommended at every dose adjustment and every visit, and treatment
discontinued if clinically appropriate
It is recommended to monitor patients beginning treatment with CONCERTA for the appearance of, or
worsening of, aggressive behaviour, marked anxiety or agitation, in which case consider
discontinuing methylphenidate.
It is recommended for patients treated with ADHD drugs that caregivers and physicians monitor for signs of
suicide‐related behaviour, including at dose initiation/optimization and drug discontinuation.

133. Evidence-based literature? Cochrane library


134. A manufacturer of sterile products must have all except:
a) Certification of people
b) Certification of equipment
c) Administration of I.V.
d) Preparation procedure

135. A pharmacist present in a clinic, saw a lady yell at her 8 yr old child and called him
worthless. What should pharmacist do?
a) Approach the lady and ask what’s going on
b) Call her husband to ask
c) Take the child aside and investigate more
d) Be vigilant for future incident

136. Legally, what is the pharmacist supposed to do in the above case?


a) Call the police
b) Call child support
c) Shout at the lady

137. Rx from emergency for 200 tabs narcotics, surprised the pharmacist calls the doc who says
he wrote only 20 tabs, it means the Rx is forged, what should you not do?
a) Call police
b) Refuse to fill
c) Detain pt. in pharmacy

138. Factors affecting secretion of drug in breast milk, except?


a) Pka
b) O/W solubility
c) Protein binding
d) Partition coefficient
e) Particle size

139. BN is a pharmacist and he is out of Androgel Cream calls in other pharmacyl (a block
away) to enquire about availability of the product. If BN sends the patient to other pharmacy
and ask him to get it from there as it is available there, what ethical principle he followed?
a) Autonomy
b) Beneficience
c) Confidentiality
d) Veracity
e) Justice
140. With vancomycin what do u monitor
a) Renal functions
b) Hepatic functions
c) Hypertension
Vancomycin is excreted rapidly by the kidneys; serum levels may increase dramatically following IV
administration in patients with decreased renal function. Dose must be adjusted based on renal function.
There is an increased risk of ototoxicity, neurotoxicity and nephrotoxicity in patients with impaired renal
function, advanced age or dehydration. Prolonged or aggressive therapy as well as concomitant use of
ototoxic and nephrotoxic agents such as aminoglycosides may increase the risk of ototoxicity or
nephrotoxicity. Monitor blood pressure during infusion for hypotension. (red man syndrome)
Neutropenia has been reported in patients receiving vancomycin but is rapidly reversible after discontinuing
the drug. Monitor leukocyte counts in patients receiving prolonged vancomycin therapy or concomitant
therapy with drugs that may cause neutropenia.

141. Patient with the following: Na decreased, K decreased, Cl decreased (normal values given
along). what will the patient progress to?
a) Hypotension
b) Renal failure
c) Arrhythmia
d) Atrial flutter

142. When to measure HbA1c for Type II DM?


a) 2-3 months
b) 6 months
c) 6 weeks

143. Obese woman, has HPT. She is recently diagnosed with psychosis, what to give her?
a) Clozapine
b) Olanzapine
c) Haloperidol
d) Ziprasidone
e) Zuclopenthixol

144. A lady with hyperkalemia was taking acetaminophen for the last month. How the
pharmacist know that it is drug induced Hk?
a) Morning Hk
b) Evening Hk
c) Hk after exercise
d) Hk with stress
145. The above patient should be treated as
a) Stop acetaminophen and use I.V. Ketorolac
b) Increase the dose of Tylenol
c) Gradually withdraw Tylenol

146. Grandmother gave 30 ml of Tylenol instead of 10 ml. Parents are very worried about this
condition. What is the appropriate action for parents?
a) Wait 48 hrs with monitoring vomiting and diarrhea
b) Refer to emergency
c) Give antidote
d) Continue with regular dose
e) Call poison center control
f) Wake him multi times at night to check the breath
In adults, hepatotoxicity may occur after ingestion of a single dose of more than 7.5 g (adults) or 150 mg/kg
(children) of acetaminophen; a dose of 10 g or more is potentially fatal. However, reports have indicated
hepatic necrosis with a single dose of 6 g and death occurring with a single dose of 13 g. Nonfatal overdoses of
12.5–31.5 g have also been reported.
Signs and Symptoms
Early symptoms (nausea, vomiting, weakness, diaphoresis) usually occur after acute ingestion of
an acetaminophen overdose large enough to cause hepatic toxicity. However, since some patients may exhibit
few or none of these early signs, in cases of suspected acetaminophen overdose, antidotal therapy should
begin as soon as possible. A latent period of 24–36 hours exists between ingestion and the onset of symptoms
of hepatic injury. Laboratory evidence usually appears within 24–48 hours if severe hepatotoxicity is to occur.
Therefore, liver function tests (AST or ALT) should be monitored for up to 48 hours after an acute ingestion.
Following the latent period, vomiting, pain in the upper right quadrant and manifestations of hepatic failure
including the onset of coma, may ensue. Maximum hepatic necrosis appears 2–5 days following overdose.
Signs include gross elevation of ALT, AST, increased bilirubin, hypoglycemia and increased prothrombin time.
Renal dysfunction following acetaminophen overdose is rare overall but quite common in cases where liver
injury or failure occurs.
In addition to hepatic and renal damage, there are rare reports of pancreatitis, clotting defects, and
myocardial damage with ST segment abnormalities, T wave flattening and pericarditis.
Recommended Management
 In cases of acute overdose, consultation with a Poison Control Centre is recommended.
 In cases of hepatotoxicity associated with subacute acetaminophen overdose, consultation with a
toxicologist is recommended.
Treatment of acute acetaminophen overdose includes supportive measures, gut decontamination with
activated charcoal, and prompt administration of acetylcysteine as an antidote.
Laboratory determinations include plasma acetaminophen levels, AST, ALT, prothrombin time, bilirubin,
creatinine, urea, blood glucose and electrolyte concentrations. A single dose of activated charcoal is
recommended, ideally within 1 hour of ingestion of overdose, and may be of benefit if given up to 4 hours
postingestion. Hemodialysis may be helpful in certain cases, e.g., massive overdoses or acute renal failure.
147. What to monitor in the above patient?
a) Respiratory acidosis
b) Respiratory Alkalosis
c) Metabolic Acidosis
d) Metabolic Alkalosis
Data suggest that acute acetaminophen overdose can lead to mitochondrial poisoning that results in
lactic acidosis, a cause of anion gap metabolic acidosis. Lactic acidosis may be a marker of severity in
paracetamol poisoning, both in those presenting early as a reflection of mitochondrial inhibition by NAPQI,
and in those presenting later as a marker of hepatic damage. It is thus important to measure it and act on
the results appropriately in severe cases of paracetamol overdose.

148. Child with febrile seizures choose best option:


a) Do not wakeup for medicine
Febrile seizures are diagnosed in children 6 months to 5 years of age who have fever > 38° C that is not
caused by a central nervous system infection and who have had no previous afebrile seizures.
Simple febrile seizures last < 15 minutes, have no focal features, and do not recur within a 24‐hour period.
Complex febrile seizures last > 15 minutes continuously or with pauses, or have focal features, or recur
within 24 hours.
Diagnosis is clinical after exclusion of other causes. Treatment of seizures lasting < 5 minutes is supportive.
Seizures lasting ≥ 5 minutes are treated with IV lorazepam, rectal diazepam, or intranasal midazolam and, if
persistent, IV fosphenytoin, phenobarbital, valproate, or levetiracetam.
Maintenance drug therapy is usually not indicated.
Available antipyretics also have analgesic effects, which can decrease discomfort if present.
Most guidelines for children recommend consideration of antipyretics only for febrile children who appear
distressed or unwell. Evidence indicates that antipyretic therapy does not prevent febrile seizures, a
phenomenon that may occur in children about 6 months to 6 years of age

149. COPD patient is already on salbutamol & tiotropium bromide & still not controlled. So,
you recommend him to take:
a) Oral Prednisone
b) Salmeterol
c) Theophylline
d) Fluticasone
e) Fluticasone + Salmeterol

150. You must advise him all of the following except:


a) H. influenza vaccine every year
b) Pnumococcal vaccine every 5 years
c) Avoid smoking and triggers
d) Tdap vaccine every 10 years
151. To avoid sound alike or look alike
medicine error, you consider all except:
a) Drugs with look alike package
b) Medicine with same strength
c) Drugs close to each other on shelf
d) Newer drugs in market

152. Smoker, obese, heartburn, what to


recommend for heartburn or What non
pharmacological measures for GERD?
a) Lower bed 6 inches
b) Eat larger meals
c) Raise bed 6 inches
d) Eat smaller meals
e) Walking after food
f) Decrease fatty food

153. A 35 yr old lady with migraine for the past 5 yrs doesn’t like the tablet, so you gave
rizatriptan wafers, what is true about them?
a) You put under your tongue, they melt & absorbed from the buccal cavity
b) Co-administration with alcohol is contraindicated
c) It is used for migraine with nausea
d) It is absorbed faster than Rizatriptan tablets  same absorption
e) It is contraindicated with people who have difficulty swallowing
Dissolve orally in saliva and then swallow and absorb from stomach

154. The lady needs something for fast relief, what would be the best option?
a) Oral Naproxen
b) DHE nasal spray
c) Rizatriptan Tablet
d) Sumatriptan SC.
155. If she doesn’t get relief, after how long you repeat the dose
a) 2hrs
b) 4hrs
c) Do not repeat if no relief from 1st dose.

156. Rivastigmine which of the following is true


a) Reassess in at least 3 months
b) Take on empty stomach  taken with food in divided doses in the morning and evening.
c) No drug interaction  potent interactions
d) It is proven to be the best treatment for alzheimers dementia

157. 4 yr old kid with asthma uses salbutamol 8-10/w upto 3-4 mths and uses fluticasone too.
How will you asses that there is improvement and signs are well controlled
a) Attack < 3wks
b) Wakeup 1 / m
c) Decrease no of going to school

158. What is the best advice for the above pt.?


a) Nebulizer
b) Oral
c) MDI with Spacer
d) Turbohaler
e) Diskus

159. If the signs of above pt. worsen, what will you do?
a) Increase dose of cortisone inhaler
b) Increase salbutamol
c) Give montelukast

160. Which of the quinalones can be given in renal failure?


a) Ciprofloxacin
b) Ofloxacin
c) Norfloxacin
d) Levofloxacin
Levofloxacin and moxifloxacin, also termed “respiratory quinolones,” are suggested as first‐line initial therapy
in the outpatient management of CAP in patients with medical comorbidities (e.g., chronic lung, heart, liver or
renal disease; diabetes; alcoholism; malignancies; asplenia; or immunosuppression) or risk factors for drug‐
resistant S. pneumoniae such as the use of antimicrobials within the past 3 months. They are also first‐line
agents for CAP in patients admitted to hospital (non‐ICU) or if Legionella spp. is the suspected pathogen.
161. A lady with graves’ disease dr prescribed methimazole also she is diabetic, hypertensive
and angina. How can she assess that medicine is working?
a) Wt. loss
b) Increased energy
c) Constipation

162. TPN calculation.


Stock solution available: NaCl additive 60mmol/L & KCL additive 30mmol/L
We have to make NaCl 4mEq/L, KCL 3mEq/L & Mix in 25% dextrose solution.
How much KCL and NaCl mix is needed for 1 L solution?

163. 10% cream w/v and 4 % w/v Hc available and mix in 50ml such that final volume will not
go above 50 ml how much (ml) 10% cream is in final solution?

164. Case. 48 years man, hypertensive, has benign prostatic hyperplasia (BPH), now has
symptoms of parkinsonism appears as falls, and difficulty in handwriting. When to initiate
treatment?
a) When symptoms interfere with for his parkinsonism
b) When symptoms interfere with the Cognitive functions
c) When symptoms interfere with the Activity of Daily Living (ADL)
d) When symptoms interfere with the Negative functions

165. What is the best option for this patient?


a) Pramipexole
b) Selegiline
c) Amantadine
d) Trihexphenedyle
e) Entacapone
Dopamine agonist e.g bromocriptine, pergolide pramipexole, ropinirole
N.B In this question, you must think about the patient age 48 years old is young so cannot start Dopamine
agonist, also must think about his BPH, so avoid drugs with anticholinergic side effect, also regarding
Entacapone it is not indicated as monotherapy but as add on therapy with the sinemet to decrease its side
effects,also must think about which drugs may affect his blood pressure. It was a tricky question; I was so
confused what to select

166. What is not true about ondansetron


a) Cause constipation
b) Less effective than Dexamethasone in Chemotherapy induced Nausea
c) Very effective in Chemotherapy induced Nausea
167. Regarding dexamethasone in chemotherapy, which is wrong
a) Most widely used corticosteroid
b) Can be used in delayed nausea
c) Can be used in acute nausea
d) All are right

168. Incase of anticipatory nausea of Chemotherapy induced nausea and vomiting?


a) BDZ lorazepam
b) Metoclopramide
c) Nabilone

169. What is true regarding Chemotherapy induced nausea and vomiting


a) Prevention of acute is the best way to prevent chronic

170. Case for patient with osteoarthritis, osteoporosis, work as hair styler, her life style: drink 2
glass of wine / week, smoke ½ pack of cigarette per day, live 2 Km apart from her work:
what are risk factors for this case?
a) Alcohol
b) Caucasian
c) Stress
d) Smoking
N.B the answer is smoking for this case, because her alcohol intake is low compared with the standard for
her (9 cups / week for women), also stress is not a risk factor for neither osteoporosis nor osteoarthritis

171. What is your advice to this case?


a) Walk to work
b) Swimming
c) Switch work to decrease the standing time
Exercise and Physiotherapy: are the key initial management strategies in OA. Exercise programs can also
reduce pain, improve physical function and reduce the need for total hip replacement by 44% in patients
with hip OA. Aquatic exercise can provide short‐term benefits for hip and knee OA pain. The benefits of
swimming can be similar to cycling exercises

172. She took Acetaminophin high dose, not yet controlled, what is ur advice for OA
a) See your physician for further assessment

173. She came to ask about Strontium for her case, where to look for info? CPS
Strontium ranelate is not available or approved for osteoporosis in Canada. However, some patients
obtain strontium ranelate from Europe, or purchase other nonprescription strontium salts, which are
available in Canada.
Strontium is a naturally occurring mineral that replaces calcium in bone matrix and causes a modest
reduction in bone resorption in conjunction with a similarly modest increase in bone formation

174. Case for plantar wart, a worker has the


full symptoms, also his colleagues at work
are suffering same symptoms, his medical
condition: hypertension, diabetes. What
would you recommend for his plantar wart
symptoms?
a) Salicylic acid 40%
b) Salicylic acid 17%
c) Salicylic acid 27%
d) Salicylic acid 12%
e) Refer him to the doctor

175. Case about a diabetic patient, working as a forest planter, he spends 3 days cutting the trees
in the forest, ran out his insulin medication, his sugar blood level rises cause Ketoacidosis. Now
he is hospitalized, what type of insulin to be injected intravenously at the hospital as an initial
treatment for his case
a) NPH
b) Regular
c) NPH/Regular
d) Glargine
e) Detemir
Management of Diabetic Ketoacidosis (DKA)
 Fluids: Patients are always significantly volume depleted. Give IV NS 500 mL for 4 h, then 250 mL for 4
h (faster if in shock), then individualize.
 Potassium: Potassium chloride is the preparation of choice. Do not give unless urine is being produced;
need to wait for IV fluids to improve volume status. Rehydration & insulin will drive K+ levels down.
 Insulin: Do not give insulin if K+ is <3.3 mmol/L, as insulin will drive K+ into cells and drop levels further.
Once K+ >3.3 mmol/L, insulin can be infused at 0.1 units/kg/h using 2nd IV line. When anion gap closed
and patient eating without nausea, overlap IV insulin with SC insulin by 2 h before stopping IV. DKA can
rapidly reoccur if waiting for SC insulin to peak. Never stop or hold insulin in DKA.
 Bicarbonate: Not first‐line therapy and infrequently used. Consider giving 1 ampoule of sodium
bicarbonate in 200 mL D5W over 1 h if acidosis is severe and ICU is being considered (e.g., pH <7,
lowered LOC).
 Supportive care: Keep the patient warm and rested.
176. Question about the Open Formulary
There are two types of formularies, open and closed. An open formulary places no limits on prescriber, so any
medication, brand, generic, strength, dosage can be prescribed. However open type of formulary is rarely
used. The closed formulary is selected list of drugs from each class of therapy, so prescriber will select from
listed formulary.

177. Warfarin all factors are affected by Vit K except


a) Factor II
b) Factor VII
c) Factor IX
d) Factor X
e) Factor XI
N.B Vitamin K affect factors 2,7,9,10,10x. So, the answer for this q was the last one factor 11 is not affected

178. Case for MI after hospitalization, give all EXCEPT


a) ASA
b) Nitrate
c) Furosemide
d) ACEI
e) B blocker

179. What is the defensive mechanism of the body to decrease the tissue & skin infection?
a) Increase the Sebacious & fatty secretion
b) Moist & damp skin
N.B. this q was not clear, I selected the moist & damp skin because it was the simplest option
Others were so complicated; I cannot memorize them.

180. Case, one come asking you about prophylaxis for malaria, you will advice him
a) Citronella is taken Once daily
b) Prophylaxis is only for Immunocopromised patients
c) Can be Exposed to Woods
d) Sleep in Mosquito net

181. He said that one of his friends took Chloroquine, Can I take it
a) Chloroquine causes CNS side effects
b) Chloroquine treatment will continue for longtime after return back so cost will be too high
c) The treatment choice depends according to area you will travel to & the resistance

182. A tech. receives an Rx of 30 mg codeine & he consults you that codeine is only available
as 15, 25, 60 mgs. So, what do you tell him to do: (I cannot remember the answers)?
183. A patient is coming with prescription of Lorazepam, prescriped 7 month ago. She wants to
transfer this prescription to your pharmacy, as she said the prescription still having 2 refills,
when you call her pharmacy you realized that you cannot take this prescription because?
a) It was written 7 month ago, so she has to go back to her dr. to get a new prescription
b) Prescription has been transferred before so cannot be transferred now

184. Monitoring for Ticlopidine includes all EXCEPT or SE of ticlopedine all except
a) Diarrhea
b) Hematauria or Melena
c) Rash
d) Bleeding
e) Jaundice
f) Hypernatremia
g) Neutropenia
Ticlopidine
 250 mg twice daily is comparable to ASA for prevention of all vascular events in patients at risk and
may be slightly superior for stroke prevention.
 Advise patients to take ticlopidine tablets with meals to minimize gastrointestinal adverse events.
 S.E: Diarrhea, skin rash and neutropenia are the common adverse events. ↑ theophylline levels.
 Dyspepsia, Nausea, Gastrointestinal pain, Vomiting, Flatulence, Anorexia, Purpura, Pruritus & Dizziness
 Should be discontinued 14 days prior to elective surgery or dental extraction. For urgent surgery,
effects of ticlopidine may be reversed by transfusion of platelets, but not by administration of fresh
frozen plasma.
 All patients should be monitored for signs and symptoms of bleeding during treatment with ticlopidine.
Monitor complete blood count with differential prior to initiating ticlopidine therapy every 2 weeks
during the first three months of therapy and periodically thereafter.
 Monitor alkaline phosphatase and serum transaminase levels at baseline and during the first four
months of therapy if liver dysfunction is suspected

185. All of the following have pharmacodynamic interaction Except?


a) Propranolol / Verapamil
b) Warfarin / Clarithromycin
c) Digoxin / Quinidine
d) Phenytoin / Rifampin
e) ACEI / NSAIDs
Dynamic is the effect of drug on body & Kinetic is effect of body on drug
So, if enzymes inhibited or induced by any reason, the effect of body on drug will be altered
Verapamil and propranolol are a great example of dynamic as both lower the blood pressure
186. You are a manager, saw the cashier council the patient on the side effect of the medication,
you will
a) Talk to the cashier after the patient will leave the pharmacy
b) Talk to the cashier infront of the patient
c) Invite the patient to the council area and talk to him then talk to the cashier later

187. All are transmitted through water Except


a) Shigella
b) Hepatitis B
c) Campylobacter

188. The most inconvenient oral contraceptive or lack of compliance with OC causing
pregnancy, mainly due to
a) Estrogen
b) Monophasic pills
c) Bi-phasic pills
d) Triphasic pills
e) Progestin only pills

189. The Goal of treatment of Acute gout (k type)


a) Decrease pain
b) Decrease inflammation & swelling
c) Decrease Hyperuricemia

190. Case for patient with Hypertension, ulcer, hypercholesterolemia, gout, the Best treatment
for acute gout is
a) Colchicine
b) Naproxen
c) Prednisone
d) Allopurinol
e) Sulphinpyrazine
N.B. Naproxen cannot be taken in hypertensive patient. Prednisone not used in hypertensive,
hypercholesterol & ulcer patient. Allopurinol & sulphinpyrazine not for Acute cases
Another version
Patient with acute gout & HTN taking ACEI, dr. prescribed NSAID, why he should not use
allopurinol?
a) Worsen symptoms of gout
b) It is given in recurrent acute attack
c) Better to control by diet
191. The most appropriate treatment to treat Hyperuricemia
a) Colchicine
b) Naproxen
c) Prednisone
d) Allopurinol
e) Sulphinpyrazine

192. Goals of therapy with Allopurinl is


a) Pain relief
b) Reduce inflammation
c) Decrease hyperuricemia

193. Side effect of allopurinol


a) Skin rash
b) Increase dyslipidemia
S.E: Skin rash, GI upset, xanthine stones (rare), severe skin adverse reactions includes Stevens‐Johnson
syndrome and toxic epideram necrolysis (TEN), drug reaction with eosinophilia and systemic symptoms with
fever, generalized exanthematous pustulosis, hepatotoxicity, worsening renal failure, can be fatal.
If rash occurs stop allopurinol immediately and do not restart.

194. Patient DM+HTN+Gout on, Metformin, Allopurinol, BB and diuretic, what is this patient
problem
a) Not getting a renoprotective agent
b) Not getting a drug for gout
c) BP not controlled

195. patient has encephalopathy, he was hospitalized, took Lactulose, which test to do?
a) Serum ammonia

196. Lactulose, most common side effect?


Hyperosmotic Beneficial in the treatment and prevention of hepatic encephalopathy due to its ability to reduce
Agents ammonia levels, and reduce liver-related morbidities and all-cause mortality.
Lactulose 45 mL every hour PO until bowel movement and clinical improvement, then maintain with 15–
Q. Schedule III 45 mL 1–4 times daily PO (titrate to produce 2–3 loose bowel movements per day)
S.E: Bloating, flatulence, cramps, diarrhea, hypokalemia (rare)

197. After the patient was discharged from the hospital, he was worried that the prescription he
took for Lactulose has no refill, he lives in other province, so tell the patient
a) If he will need Lactulose, he must get new prescription
b) He can contact his dr to add the refill on the prescription
c) He can buy Lactulose without prescription from any pharmacy
198. A Dr. whom you know came to your pharmacy with his wife, get his prescription pad &
wrote antibiotic prescription for his wife, he forgot to write any refills, what you will do?
a) You may dispense because the prescription is not for narcotics
b) You will not dispense because prescription has no refills
c) You will not dispense because the customer is not your pharmacy customer
d) You will not dispense because it is a legal issue
e) You may dispense because the issue is ethical not legal
N.B. the sentences were so close & tricky in this question

199. Hypercholesterolemia, hypertension &


gout, taking Atorvastatin, his lab values are
as follows: LDL level Ok, HDL level low,
TG level high. So, what to do?
a) Add Ezetimibe  add if LDL is high
b) Add Fibrate
c) Increase dose of Atorvastatin
d) Switch to Rosuvastatin
e) Add Niacin
As far as no ator doses mentioned, will go for C or D to avoid DD interaction. Maximum dose of atorvastatin
is 80 mg/day.
Statin is enough to lower both TG & LDL so we don’t add Fibrates except TG exceeds 10 mmole or if the
patient has only TG is high

200. What drug to monitor when given with fenofibrate


a) Statin
b) Ramipril
c) Nifedipine
d) ASA
Caution when combining with statins; potential additive hepatotoxicity and myotoxicity.

201. Nurse coming to ask about insulin, what is true about pen cartridges:
a) Used vial can be refrigerated for 60 days
b) Taking many times from the vial destroy the insulin components
c) It allows mixing Glargine or Detemir with NPH
d) Cartridge from different manufacturers are unexchangable
e) When pen is preloaded, it should be refrigerated
202. You are a pharmacist in your first day in the pharmacy, when you came at the new day
shift in the morning, a bag left on the counter, when you open it you found 2 vials of vaccine
having their label refrigerate at 2-8 °C, your action will be
a) Wait to see the manager and tell him in the night shift
b) Return the vials to the fridge, to sell it
c) Contact the manufacture to ask for refund
d) Investigate who is responsible & document in an incident report

203. A patient entering your pharmacy, telling you that his dr. prescribes for him nicotine patch,
but he doesn’t want to quite smoking, you will discus with him the most important things for
him now, all except
a) Why his dr. gave him the patch today
b) What his risk factors
c) Why he donot want to quite smoking
d) Invite him for the upcoming smoke cessation event in your pharmacy

204. Symptoms of extraphoesgeal GERD is


a) Nausea
b) Diarrrhea
c) Cough
d) Headache

205. He took antacids, which statement is true about antacids?


a) Effective if taken before and with meals
b) Act instantly
c) Don’t work if pain started already

206. Patient needs Famotidine, what to tell her?


a) Famotidine works on Cyt P 450 with many interactions
b) Antacids work faster
c) Famotidine gives more long-lasting effect than antacids

207. A trial for Flu treatment shows the following results, the number of patients who got flu
was as follows: The Drug 25 case in 500 patients, the Placebo 50 case in 500 patients. Calculate
the number need to treat NNT?
The answer is:
as 25 case in 500, so if one case in how many patients (x), i.e.
25 ------------ 500 1 ------------ X X = 20
208. Drug: 24 out of 100 pt get cured; while 32 get harm. Placebo: 15 out of 100 get cured,
while 12 get harm. Calculate NNH?
The answer is:
NNH = 1 / ARI = 1 / (32/100 - 12/100) = 5 patients

209. You received the following prescription


Rx 10% Hydroquinone powder
1% Ascorbic acid powder
0.05% Tretinoin ointment aa mometasone ointment
Mitte 60 gram
What is the amount of mometasone in this preparation?
a) 30
b) 26.7
c) 26.85
d) 27.3
e) 27.6
N.B. the answer is calculating the amount of the powder first
Powder = 10% + 1% = 6.6 gram Then 60 ‐ 6.6 = 53.4
aa means equal amount of both ingredients,
so, 53.4 / 2 = 26.7 which is the amount of mometasone or tretinoin ointment

210. If we add a substance A to 10% of substance B to make a mixture of A+B, we will not
exceed 50 ml. We use stock solution of substance A. If we know that this mixture is to treat
glucoma and to be effective it should be at least containing 0.1% of substance A. So, what will
be the volume of substace B in the mixture.
Answer:
C1 x V1 = C2 x V2 0.1% X 50 ml = 4% X ? ? = 1.25 ml of substance A
B = 50 -1.25 = 48.75 ml

211. Solution contain 5 million units diluted by 8 ml distilled water to be 500,000 units/ml.
How many mls of distilled water to be its concentration 125,000 units/0.5 ml.
Answer: 500,000 ------- 1 ml 5000,000 ------? = 10 ml (volume of solution)
Volume of powder = 10 - 8 = 2 ml
For 125,000 ----- 0.5 ml 5000,000 --------? = 20 ml
Volume of distilled water needed = 20 ml – 2ml (powder volume) = 18 ml

212. Peer reviewed article means


a) Its material and methodology were reviewed by respectful peers before publication
b) Its been reviewed after publication from many respectful peers
213. The most effective smoking cessation drug is?
a) Nicotine patch
b) Nicotine gum
c) Bupropion
d) Vareniceline

214. Cholestyramine side effects?


S.E: Common: constipation (>10%), bloating, abdominal fullness, flatulence,
increased triglycerides, increased transaminases (reversible).
Rare: hyperchloremic acidosis, cholecystitis, cholelithiasis, pancreatitis,
malabsorption syndrome, GI bleeding, peptic ulceration.

215. DOC travellers diarrhea prevention?


a) Typhoid
b) Twinrix
c) Dukoral

216. Hemorrhoids red flags, we should make referral to Dr for hemorrhoids if all except?
a) Bleeding
b) < 12 years old
c) Blood in stool
d) Flatulence
See your health‐care provider if any of the following things happen:
 The hemorrhoid does not go back in place after a bowel movement
 You have bleeding from your rectum (back passage)
 The problem lasts longer than 7 days
 Stool or mucus leaks from your rectum between bowel movements

217. Hospital managmenet wants to increase safety, which of the following is appropriate?
a) Interactions with nurse and px more
b) Tech transcribes rx
c) Md types rx into computer

218. All are side effects of amiodarone except:


a) Headache
b) Hypotension
c) Skin discoloration
d) Corneal diposition
S.E: "4Ps" Pulmonary damage (irreversible) Peripheral T4‐T3, Hypo or Hyperthyroidism
Pigmentation: deposits in skin cause bluish tinge skin color. Phototoxicities.
Corneal deposits cause blurred vision, hepatic toxicity, optic neuritis, erectile dysfunction, and photophobia.
Amiodarone is an inhibitor of CYP3A4 and p‐glycoprotein. Avoid grapefruit juice.
Monitor LFT transaminases and Serum TSH Q 6m, eye exam, Chest X‐Ray at baseline & annually
Reduce doses of BBs, digoxin, procainamide, quinidine and warfarin by 50%.

219. All are sympoms of estrogen deficiency except


a) Osteoporosis
b) CV risk
c) Cancer risk
d) Urogenital aging
e) Hot flushes
a) Nervousness
b) Breast tenderness
c) Vaginitis
d) Amenorrhea

220. which of the following drugs would cause QT prolongation if given with Clarythromycin
a) Fexofenadine
b) Cetrizine
c) Loratadine
d) Chlorpheniramine
QTc prolongation reported with concomitant use of
loratadine and amiodarone.

221. What is the drug of choice for DVT?


a) Dalteparin
b) Dipyridamole
c) Clopidogrel

222. Pregnant, Morning sickness using


Diclectin, Dr ask if he could add
Dimenhydrinate, your response is
a) NO
b) Yes, Safely
c) Yes, but only intermittently
d) Yes, although not enough trials
CTC: Since dimenhydrinate is recommended for augmenting pyridoxine treatment in pregnancy, it can be
considered for intermittent therapy of motion sickness in pregnant patients. Promethazine may also be used.
223. Lithium use in pregnancy?
If possible, lithium should be avoided in pregnancy, especially during the first trimester, as it freely crosses
the placenta. Administration of lithium during the first trimester may result in an increased incidence of
cardiac and other anomalies, especially Ebstein anomaly (a congenital heart defect characterized by anterior
displacement of the tricuspid valve and enlarged right chambers, often with atrial septal defect & arrhythmias)
The increased risk of cardiac malformations (1%) appears to be dose‐dependent, with a three‐fold increased
risk at doses >900 mg/day.
The risks of lithium treatment during pregnancy must be weighed against the risk of relapse of mood disorder;
the decision is personalized on a case‐by‐case basis. Some experts recommend tapering and discontinuing
lithium when planning a pregnancy or upon discovery of pregnancy, and reinstating therapy after 3 months’
gestation when organogenesis is complete. Others recommend planning a pregnancy during disease remission
and using the lowest effective dose. If lithium is used in the first trimester, fetal echocardiography is
recommended to monitor for cardiac anomalies. Supplementation with 5 mg folic acid is also recommended.
Some experts recommend avoiding a single daily dose of lithium and splitting it into divided doses to avoid
fetal exposure to peak serum levels. Serum lithium concentrations should be more frequently monitored
(monthly and weekly/biweekly during the last month) and the dosage adjusted, if necessary, since lithium
clearance may be increased during pregnancy.
Immediately postpartum, renal clearance of lithium may decrease to pre‐pregnancy levels. To decrease the
risk of postpartum lithium toxicity in the mother or newborn, dosage of the drug should be reduced 1 week
prior to delivery or when labor begins. Some experts recommend holding lithium when labour begins.
Nursing Women
Lithium is transferred into breast milk in highly variable concentrations ranging from 30–100% of maternal
serum levels. Due to immature renal function and the risk of toxicity in the newborn, lithium exposure
through breastfeeding should be avoided, when possible, until the infant is several months old. If lithium is
used during breastfeeding, the mother should be educated about signs and symptoms of lithium toxicity and
the increased risk posed by dehydration in the infant. Monitor lithium concentrations and consider renal and
thyroid function in infants exposed to lithium through breastfeeding.

224. Lithium counseling


a) Adequate Salt
b) Adequate protein
c) Adequate Alcohol
Avoid ACEI, ARB, NSAIDS, Spironolactone. Drink 8 to 10 glasses of water or other liquids every day.
Keep your salt intake about the same. Keep your caffeine intake about the same
Take lithium with food or milk. This will reduce possible digestive side effects like nausea, vomiting, diarrhea,
and abdominal pain. Lithium should be administered after meals for optimal absorption and improved
tolerability. Avoid alcoholic beverages.
228. Common Side effect of Nifedipine
a) Rash
b) Tremors
c) QT interval prolongation
d) Hyperhidrosis
The most common adverse effects and those that most frequently result in discontinuation are CV & nervous
system effects related to the vasodilatory action of CCBs, e.g., pedal edema, flushing, palpitations, headache.
These are reported more often with dihydropyridines, but can occur when nondihydropyridines are used.
Serious side effects include angina, heart failure (due to negative inotropic and chronotropic effects
associated more often with nondihydropyridines), pulmonary edema, tachycardia, bradycardia, excessive
hypotension, skin rashes, arthritis and transient blindness (for nifedipine).
Extrapyramidal effects, galactorrhea and mental depression are specifically associated with flunarizine.

229. Signs of Tardive Dyskinesia?


 Tardive dyskinesias (TDs) are involuntary movements of the tongue, lips, face, trunk, and extremities
that occur in patients treated with long‐term dopaminergic antagonist medications.
 There is no evidence‐based treatment— prevention is key. Use SGAs first‐line.
 Antiparkinsonian medications are not effective and may worsen symptoms.
 If TD occurs, suggest consultation with a psychiatrist. Consider switching to an SGA.
 For persistent, severe TD, consider clozapine trial

230. Which one is used for scabies


a) Lindane 1 %
b) Permethrin 5%
c) Crotamiton
d) All can be used

231. Permethrin cream case, Patient with many allergies, which of these is ok to use with
Permethrin?
a) Permethroid cream or lotion Allergy
b) Rageweed allergy
c) Chrysthemamus allergy
d) Ginkgo Allergy
e) Feverfew allergy

232. Obese, Smoker, HTN, DM, Dental problems already on Nicotine patches but has cravings
at night wants to quit, what is the most important factor in assessing his case
a) His willingness to quit

233. What should be done: Nicorette Inhaler PRN for cravings


234. Cephalexin label
a) May cause drowsiness
b) Don’t take with antacids or milk
c) Complete course of treatment
Cephalexin Shake well – take on an empty stomach - expect mild diarrhea
suspension Reconstituted suspension should be stored in a cool place (6°C-15°C) or in a refrigerator
(2°C-8°C). After reconstitution: to be used within 10 days

235. Asymptomatic bacteriurea should be treated in:


a) All patients
b) Only pregnant women
c) Only diabetic patients
d) Only in hypertensive patients
Should not be treated except in pregnancy or shortly before transurethral resection of prostate.
Treatment in nonpregnant women does not reduce development of symptomatic UTI or complications but
contributes to harmful outcomes include adverse effects, reinfection with more resistant bacteria and possibly
an increased short‐term incidence of symptomatic UTI.

236. A pregnant woman has cystitis, what to give her?


a) Amoxicillin
b) Fosfomvcin
c) TMP/SMX
d) Ciprofloxacin
e) Nitrofurantoin
Management of Treat asymptomatic bacteriuria & symptomatic cystitis with 3 - 7 days course of amoxicillin
UTI during (if the organism is known to be susceptible), amoxicillin/ clavulanate, cephalexin, fosfomycin
Pregnancy tromethamine or nitrofurantoin with appropriate follow-up.
Nitrofurantoin is usually avoided near term because of the risk of inducing hemolytic anemia in
the fetus or newborn, especially in those with G6PD deficiency; however, this toxicity is rare.
Fosfomycin tromethamine may also be used in pregnancy when the organism is susceptible.
Ceftriaxone is the preferred empiric therapy for treating pyelonephritis in pregnancy. Step
down to oral therapy once the patient is stabilized and urine culture results are available.
Avoid trimethoprim and SMX/TMP in the 1st trimester of pregnancy.
Avoid SMX in the last 6 weeks of pregnancy & Avoid fluoroquinolones in pregnancy.

237. Pt comes to take codeine syr, she came in another pharmacy yesterday u worked there,
what to do?
a) Give her number of Abuse society
b) Refuse to give
c) Refer to physician for a stronger cough medication
238. Patient take a lot of codeine to USA; you refuse to give due to all Except:
a) No Pharmacist-Patient interaction
b) Drug will be taken by US customs
c) Abuse

239. US citizen in Canada for 2 day, he is out of medication, you decide to give hime 2-day
supply for what reason:
a) Non Malifecence
b) Beneficence
c) Justice
d) Autonomy

240. Patient on Morphine from another province, forgot her pills, her Dr faxed you an Rx for
her to give her morphine, what to do?
a) Call physician to verify then dispense
b) Refuse to give
c) Send her to a walk-in clinic

241. A Mother needs receipts for family medications to give to insurance, what to do?
a) Obtain permission from family members

242. A Female comes to your pharmacy, she has been sexually assaulted & wants plan B
1. Call the police & report it
2. Give her number of the female assault (don't remember name of the organization)
3. Give her plan B

243. To measure renal function before giving a drug, what to monitor?


a) Glomerular filtration rate
b) Serum Ceatinine
c) Creatine kinase

244. You are a male pharmacist, a female patient comes to you, she is new to Canada, while
you were taking her information, you realized she is uncomfortable with the eye contact, what
would you do?
a) Call for her a female colleague to talk with her
b) Offer that you continue the interview on the phone
c) Ignore it & maintain the eye contact
245. Difference between allergic rhinitis and rhinosinusitis?
a) Post nasal discharge
b) Itchy eyes and nose  in allergic rhinitis
c) Fever  in rhinosinusitis
d) Sore throat
e) Duration
Differential Diagnosis of Upper Respiratory Tract Conditions

246. A mother coming to your pharmacy, she looks unclean, holding her 4-month baby in her
arms hose diaper is dirty, looks malnourished, she is asking for a formula to feed her baby, you
will suggest
a) Soya formula
b) Iron fortified formula
c) Omega 3 fatty acid syrup
247. In addition to referring her to treat her baby, you will recommend for her a social society,
you are following which ethics
a) Beneficience
b) Dignity
c) Veracity
d) Paternalism

248. Preparing cream and forgot about the diluent, where to look for information?
a) Remington (or merck index)

249. Some Preparation in the pharmacy in the flow hood, what is the pore size for the filter
a) 0.22
b) 0.33

250. When you were doing it u found out u are out of the material needed, what to do?
a) Get some stock from a close by institution
b) Get some stock from special access program

251. Counseling on dose and way of administration of L-thyroxine


T4 dosage, 1.6 µg/kg/day (adults).
12.5 ‐ 25 µg/day for patient with CAD or elderly as high doses associated with increased risk of fracture in
elderly patients. It takes 6 weeks to attain a new steady state.
Q. Take morning on empty stomach.
Absorption may be reduced by antacids, Ca salts, cholestyramine (separate administration by at least 6 h),
colestipol, iron salts. T3 is not reported to interact with iron salts.
Increased TSH with estrogens and PPIs. Response to warfarin may vary.

252. To minimize SE of high dose L-thyroxine, what should be given?


a) Propranolol (for tachycardia)
Beta‐blockers ameliorate the symptoms of adrenergic excess and are usually used adjunctively in the
management of Graves disease or toxic nodules. Propranolol and nadolol can decrease the conversion of L‐T4
to T3. Use beta‐blockers with caution in patients with asthma, obstructive respiratory disorders or Raynaud
phenomenon.

253. Pregnant with common cold, what is wrong:


a) Loratadine can be used despite lack of evidence
b) Chlorpheniramine is safe
c) Common cold is self limiting anyway
d) Nasal decongestant is very dangerous
254. Asthmatic, living in basement, computer tech on Symbicort BID and salbutamol
(excessively), Many missed work days, therapy should be aimed to
a) Improve lung function
b) Minimize work absence

255. What is your advice to him


a) See a respirologist
b) Give him an action plan

256. What contributes to his poor condition?


a) His job
b) His basement apartment
c) Excessive salbutamol

257. IBS patient with diarrhoea


predominant, what is not beneficial?
a) Bismuth SS
b) Diet modification
c) Lopermaide

258. for IBS patient to rule out other illnesses, what to do EXCEPT?
a) Upper gastroscopy
b) Colonscopy
c) Barrium swallow
d) Ultrasound

259. Which of the following AB should be discarded if


out of fridge for 24 hours?
a) Amoxicillin (7 days RT, 14 days in fridge)
b) Erythromycin estolate (room termp)
c) Amoxiclav susp (refrigerate & use in 7-10 days)
d) Clarithromycin

260. Best measure for temp for one-year old child is?
a) Axillary
b) Rectal
c) Oral
d) Ear
261. Patient is divorced, tried Marijuana once before & felt relaxed, he had a car accident a year
ago & has back injury & pain. He doesn't go to work & is depressed, use caution while giving
opioids because of:
a) High risk of addiction
b) Opioids won't give effect for his pain
c) High risk of opioid side effects

262. 1st line for Dysmenorrhea


a) NSAID
b) COC
c) Primerose oil

263. Centrally acting drug for male


sexual dysfunction
a) Tadalfil
b) Papaverine
c) Phentolamine
d) Alprostadil
e) Sildenafil
Phentolamine produces its therapeutic actions by competitively blocking alpha‐adrenergic receptors (primarily
excitatory responses of smooth muscle and exocrine glands), leading to a muscle relaxation and a widening of
the blood vessels. This widening of the blood vessels results in a lowering of blood pressure.
Phentolamine given by injection causes blood vessels to expand, thereby increasing blood flow. When injected
into the penis (intracavernosal), it increases blood flow to the penis, which results in an erection. This
medicine should not be used as a sexual aid by men who do not have erectile dysfunction.

264. Osteoarthritis, what can be used, all except


a) Diclofenac
b) Naproxen
c) Celecoxib
d) Ketorolac (use for acute pain)
265. A Patient just had dental surgery, his dentist prescribed opioid for him, but the patient
doesn't want to take it (didn't mention why) & prefers a pain killer OTC. What will you percept
is the reason behind the patient's decision?
a) Doesn't trust his dentist
b) Still under the effect of the anesthesia so he doesn't know how painful it will be
c) I don't remember the rest of the options

266. what will you do in the above case?


a) Tell him he must take it
b) Tell him its his choice
c) Discuss with him concerning his reasons

267. 25months old child diagnosed with recurrent AOM. He got first 1st occurrence 11 months
ago and 2nd occurrence 2 months ago. Patient’s mother is coming with high dose amoxicillin
prescription. Which drug is most appropriate for this patient?
a) Amoxicillin-clavulanate
b) Cloxacilin
c) Azithromycin.
d) Cefprozil

268. 1st line in chronic prostatitis


a) Nitrofurantoin PO
b) Ciprofloxacin PO
c) Ciprofloxacin IV
d) Fosomycin PO
e) Gentamycin IV
Acute bacterial E. coli, Enterobacteriaceae, P. aeruginosa, Aminoglycoside Fluoroquinolone IV
prostatitis Staphylococcus aureus, others IV ± cloxacillin IV or PO × 4 wk
Acute onset chills, fever, perineal and low back ± ampicillin (if or
pain, irritative and obstructive voiding. Enterococcus is a SMX/TMP PO × 4
The prostate is tender, swollen, indurated and concern) wk
warm. IV Cloxacillin is
Prostatic massage is not recommended because it useful only if S.
may cause bacteremia. aureus is known to
be present.
Chronic E. coli (80%), Klebsiella spp, P. aeruginosa, Fluoroquinolone SMX/TMP
bacterial Proteus spp, other PO × 4–6 wk PO × 4–6 wk
prostatitis Common cause of recurrent UTIs in older men;
increases with age. Intermittent urinary infection
presenting as cystitis; history of recurrent UTIs.
269. Child with ADHD on Methylphenidate immediate release (10 mg BID), he takes his meds
at 7:00 & 15:00, & he is having insomnia. what to do?
a) Change the time to 7:00am & 12:00pm
b) Take 20 mg QD at at 7:00
c) Take 20 mg QD at noon
d) Take SR 10 mg BID at 7:00 & 15:00
e) Take SR 20 mg QD at 12:00

270. Objective measure for pneumonia? >30 breath /min

271. Which of the following is active against Pseudomonas? Ceftazidime


(Ticarcillin, Piperacillin, Imipenem: Antipseudomonal B‐Lactam) (Cipro, Levo, Amg)

272. Pharmacy manager in annual assessment of employees should do all except


a) Compare employees’ performances

273. I ordered a vaccine for a patient. The vaccine should be stored at -5.6"C or cooler. Its
diluents came with it, it should be kept at 2” C. what to do?
a) Keep the vaccine & Diluent in the delivery packages in the freezer
b) Keep the vaccine & Diluent in the delivery package in the refrigerator
c) Keep the vaccine & Diluent in the delivery package & send to the Dr's office right away
d) Keep the vaccine in the freezer & the diluents in the refrigerator
e) Keep the vaccine & diluents in the delivery package & call the patier-it to come & take
them right away

274. HTN, Dr Rx Non-Pharmacological diet, what should u say


a) DASH diet is sometimes as effective as some antihypertensive medications

275. Methadone legislation? Federally approved dr to rx it


In the past, physicians required exemptions to prescribe methadone, but As of May 2018, physicians do NOT
require exemptions to prescribe methadone.

276. Nitrofurantoin:
a) Take with food because of bitter after taste
b) Take on empty stomach
c) Take with food because of diarrhea
Vomiting, diarrhea, anorexia, dyspepsia, abdominal pain and constipation have occurred. These effects are
likely dose‐related and can be minimized by administering the drug with food.
277. Doxorubicin most commone Side effect is
a) Cardiomyopathy (monitor CBC and hepatic function)
Heart failure Anthracyclines (Daunorubicin, Doxorubicin)
(in months to Prevention: Limit lifelong cumulative doses. Assess cardiac function periodically.
years) Regimens with longer infusion times (>6 h) may have lower risk.
Prophylaxis with statins, beta-blockers and ARBs are under prospective evaluation, having
shown some trends for benefit.
Consider discontinuing if serial ejection fraction results drop 25% from baseline.
Dexrazoxane (cardioprotective) used in certain patients (e.g., those with metastatic disease)
whose lifetime cumulative anthracycline dose exceeds safe thresholds or who have underlying
cardiac risk factors. Given IV at a dose of 10 times that of doxorubicin or epirubicin within 30
min of their administration
S.E: Myelosuppression, injection site reaction, phlebitis, nausea
Management: Clinically indistinguishable from HF due to other causes. Treat symptomatically.

278. An error in prescribing, all are invited for a meeting except?


a) Doctors
b) Pharmacists
c) Patients
d) Nurses

279. Dr ask you about a new indication of a drug, where to look for it
a) Micromedex
b) Medline
c) Medical letter

280. Patient with edema and constipation, he is taking tylenol 3 and on docusate but no effect &
what to do for his constiption?
a) Mineral oil
b) Add senna
c) Lactulose

281. Capsaicin, what is true


a) Used on large areas
b) Apply bandage
c) Burning will fade away with repeated use

282. Smoker, quitted already, Obese, you call his physician to?
a) Prescribe him statin
b) Precribe him Vitamin C
283. A hospitalized lady, has a catheter, Infected, so what to do?
a) Cranberry Juice
b) Remove the catheter if feasible

284. All are important in Chemotherapy induced N & V treatment except


a) Site of tumor
b) Type of Chemo Agent
c) Route of Administration
d) Duration of treatment

285. Pt with Osteoarthritis, on Acetaminophin, suffering from pain in lots of joints, you
recommend to him:
a) See your physician for disease assesment
b) Take advil
c) See your physician to increase acetaminophin

286. A patient tells you about a medical site he went through, what shouldn’t concern you
a) Sponsors of the site
b) Updatness of the site
c) Content of the site

287. What is not used in throat infections


a) Erythromycin
b) Ciprofloxacin
c) Amoxicillin
d) Penicillin
Penicillin V Amoxicillin Cefadroxil Cephalexin Azithromycin Clarithromycin Clindamycin

288. Depot (oily ampoule) can be given


a) IM
b) IV
c) SC
d) Intraarticular

289. Patient have hypokalemia, N&V, weakness, blurried vision, he is taking digoxin
hydrochlorothiazide, these symptoms are due to all except?
a) Vomiting
b) Digoxin  hyperkalemia
c) Hydrochlorothiazide
290. All are correct about digoxin except:
a) May cause av block
b) May cause ventricular arrhythmia
c) May cause atrial fibrillation

291. Who is responsible for pharmacy practice? Note: there was no NAPRA
a) Health canada
b) Each province and their college of pharmacist

292. Father of 16-year-old girl, she is taking contraception, father came and asked if his
daughter is using contraception, what conflict the pharmacist is on
a) Non malificiancy and veracity
b) Autonomy and veracity
Note: there was no confidentiality and veracity

293. To decrease load on pharmacist, what can be delegated to a technician


a) Ordering medications
b) Labeling prescription
c) Labeling container

294. Which is not a risk factor that increases NSAID renal toxicity
a) Hepatic disease and ascites
b) CHF

295. All are true about clopidogrel except


a) It is used as a primary treatment for ACS
b) NSAIDs & PPIs may reduce its efficacy
c) Bleeding, rash, purpura are main side effects
d) Discontinue 5 days prior to bypass surgery to decrease bleeding.
296. 83 y old man have alzehimer, and he is agitated, drug of choice?
a) Bupropion
b) Mirtazapine
c) Haloperidol
d) Respiridone

297. Least drug causing sexual dysfunction?


a) Spironolactone
b) Digoxin
c) Atenolol
d) Ramipril

298. A female having menopause and want to use hormonal therapy. All of the following can
occur if she doesn’t use HRT, except?
a) Hot flashes
b) Risk of CVS events
c) Osteoporosis
d) Vaginal atrophy
e) Susceptible to urinary Infections

299. Benzydamine mouthwash counseling - all expect?


a) Dilute with warm water into a 1:1 solution
b) Don't eat hot foods directly after it
c) For pharyngitis & mucositis caused by radiation therapy
d) Swish in the mouth for as long as possible then swallow

300. Which benzodiazepine shows withdrawal symptoms after 2-3 days of abrupt withdrawal:
a) Alprazolam
b) Oxazepam
c) Flurazepam
d) Temazepam
e) Diazepam

301. What is the treatment of cough with chest congestion?


a) Pseudoephedrine
b) oxymetazoline
c) Guanfasin
d) Dextromethorphan
e) chlorpheniramine
302. Woman asked pharmacist, that she is afraid to go outside, she has urinary incontinence,
she went to Dr., and it is not accompanied with cough or sneeze or exercise, she is little bit
stressed, whuch type of incontinence is this
a) Urge
b) Stress
c) Overflow
d) Mixed

303. What is the treatment:


a) Oral estrogen
b) Amitriptyline
c) Oxybutynin
d) Desmopressin

304. Symptom of overdose of


opioid?
a) Hyperventilation
b) Hypoventilation
note: there is no bradycardia, no
hypotension
Respiratory depression (reduced respiratory rate and/or tidal volume, Cheyne‐Stokes respiration, cyanosis),
extreme somnolence progressing to stupor or coma, skeletal muscle flaccidity, cold or clammy skin, and
sometimes hypotension and bradycardia. Severe overdosage may result in apnea, circulatory collapse, cardiac
arrest and death. Miosis can be one characteristic of morphine derivative overdose.
Mydriasis can take place in terminal narcosis, severe hypoxia, or as a toxic effect of meperidine & oxycodone.
Hyponatremia, hypoglycemia and seizures, as well as typical opioid effects of CNS and respiratory depression,
occur in tramadol overdoses.

305. DM Patient is taking metformin 1000 mg bid and taking glyburide, but no effect and
glucose level is high after food, what to do
a) Add acarbose
b) Add meglitinide
c) Add glitazones
d) Increase metformin
e) Increase glyburide
They both reduce postprandial Glucose
306. Now dr. Wants to add insulin, which one to choose
a) Lispro
b) Glargine
c) NPH/Regular 70/30

307. Patient is taking NPH/regular in morning and evening


7 am  glucose level 3-5 6 pm  glucose level 9-11 What to do?
a) Increase morning NPH and decrease evening NPH

308. Patient have OA polyarticular, he is taking voltaren, and another NSAID, he was stabilized
for certain time, but symptoms are getting worse, asking for external medicine, what is wrong
about methylsalicylate
a) You should protect area around
b) Don’t apply to wide area
c) Hot compress when added are very beneficial
For external use only. Do not apply to wounds or damaged skin and avoid contact with eyes and mucous
membranes. Discontinue use if rash or irritation occurs. If condition persists for more than 7 days, consult
physician. Do not bandage after application and avoid external sources of heat such as heating pads. Do not
use if you are allergic to salicylates or are taking anticoagulant medications. Keep out of reach of children.
Store in a cool place.

309. Now what is treatment


a) Sulfasalazine
b) Hyalurinidase
c) Etanercept
Monotherapy with sulfasalazine considered if patients contraindicated for MTX.
It may also be considered for some patients with early, very mild disease and absence of poor prognostic
factors.

310. What is not a serotonorgic symptom


a) Sweat
b) Tremor
c) Anxiety
Serotonin syndrome:
Rare, and may occur when multiple serotonergic agents are used MAOIs have the highest risk and require a
minimum 2‐wk washout period before another serotonergic agent is initiated
The serotonin syndrome may occur if the SSRI, MAOi, SSRI, TCA, are combined with Tramadol, Meperidine,
Amphetamine, Dopamine, Cocaine, Methyldopa, Dextromethorphan, Caffeine, Lithium, SSRI, MAO Inhibitors,
TCAs, St. John wort.
Amitriptyline, mirtazapine and trazodone are unlikely to cause serotonin syndrome
Signs and symptoms of Serotonin syndrome
 Neurobehavioral: confusion, agitation, seizures, coma, delirium, hyperreflexia.
 Autonomic: Hyperthermia (fever), diaphoresis (sweating), tachycardia, hypertension, diarrhea
 Neuromuscular: Myoclonus, rigidity, tremor (shivering), ataxia, dilated pupil and nystagmus
Management:
 Stop the drug(s) and refer patient to the hospital
 Most cases are mild and resolve spontaneously within 24 to 72 hours.
 Pharmacist has to contact the physician and tell the patient to withhold the serotoninergic agent.
 Benzodiazepines, propranolol, and cyproheptadine, a serotonin antagonist, have been used
successfully.

311. What is not a treatment of trigeminal neuralgia


a) Lamotrigine
b) Gabapentin
c) Carbamazepin
Antiepileptic Drug Carbamazepine, but not effective for other types of neuropathic pain unless there is a
shock‐like component. Give doses before meals.
If some relief is achieved with carbamazepine but side effects are unacceptable, a good strategy is to switch to
oxcarbazepine or reduce dose of carbamazepine to tolerability and add baclofen.
Gabapentin, pregabalin, clonazepam and valproic acid may be tried if other strategies fail.

312. Patient have fungal infection in his toenail, and he is taking digoxin, what is the best
treatment
a) Terbinafine
b) Itraconazole
c) Fluconazole
d) Nystatin
Duration: Fingernails: 6–12 wk. Toenails: 12– 24 wk.

313. What is the SE of lamotrigine?


Rash 5–10%, insomnia, Hepatotoxicity and hematological toxicity.
The most commonly observed adverse experiences associated with the use of adjunctive therapy with
lamotrigine (incidence of at least 10%) were dizziness, headache, diplopia, somnolence, ataxia, nausea, and
asthenia.
Dizziness, diplopia, ataxia, and blurred vision were dose‐related and occurred more commonly in patients
receiving carbamazepine in combination with lamotrigine than in patients receiving other AEDs that
induce lamotrigine glucuronidation in combination with lamotrigine. Reduction of the daily dose and/or
alteration of the timing of doses of concomitant antiepileptic drugs and/or lamotrigine may reduce or
eliminate these symptoms. Clinical data suggest a higher incidence of rash in patients who are receiving
concomitant valproic acid, or non‐inducing AEDs
314. Patient have bacterial blepharitis, with swollen red eye, what is treatment?
a) Gramicidin / polymyxin B

315. Dr. ordered benzodiazepine 20 tablets, 5 times refill, patient want to transfer to another
pharmacy, what is correct
a) You can transfer only 20 tablets
b) You can transfer 20 tablet * 5 times

316. All are sympoms of hypothyroidism, except?


a) Weight gain
b) Oily skin, acne
Symptoms such as fatigue, impaired memory, constipation, cold intolerance, changes in skin or hair, weight
gain. appearance (coarse features, dry skin and hair), hypertension, bradycardia, delayed relaxation phase of
reflexes, extreme cases which may present with myxedema coma, hearing loss and Decreased libido.

317. What causes Diarrhea?


a) Calcium
b) Magnesium
c) Aluminium
d) Bismuth subsalicylate

318. Treatment of acute asthma (Note: there was no salbutamol)


a) Ipratropium
b) Inhaled corticosteroid
c) Salmetrol
d) Sodium cromoglycate

319. Now patient entered hospital, what to give?


a) IV methyl prednisolone

320. Patient now was discharged, what is used to prevent second exacerbation of acute attack
a) PO prednisone

321. All are used to monitor COPD except


a) Spirometry
b) CBC
c) Chest X-ray
d) Peak flowmeter
322. Lossing weight can help all except
a) Prevent DM
b) Improve RA
c) Prevent degenerative bone degradation

323. Progesterone is used in COC TO


a) Decrease breast cancer
b) Decrease endometrial cancer

324. All are true about tamoxifen except


a) Used to treat breast cancer
b) It is progestin and anti estrogen
Tamoxifen is an estrogen antagonist with some estrogenic activity, and it is classified as a selective estrogen
receptor modulator (SERM). It is used for first‐line therapy in the treatment of estrogen receptor–positive
breast cancer.
Binds to estrogen receptors in the breast tissue, result in depletion (down‐regulation) of estrogen receptors,
and the growth‐ promoting effects of the natural hormone and other growth factors are suppressed
Side effects: Inhibition of estrogen cause Hot flashes. Low emetogenic or least nauseating anticancer drugs.
Vaginal bleeding, menstrual irregularities and risk of endometrial cancer.
Tamoxifen is a prodrug, and most of the therapeutic effect in treating breast cancer stems from its
metabolite, endoxifen. Since cytochrome P450 (CYP) 2D6 is the most important enzyme in the production of
endoxifen, drugs that inhibit CYP2D6 would be expected to reduce tamoxifen efficacy. In addition to drug–
drug interactions (DDI) involving CYP2D6, there is growing evidence that enzyme inducers can substantially
alter the disposition of endoxifen, reducing tamoxifen efficacy.

325. Female, obese, saved money by working overtime to go to Cuba, she comes in to your
pharmacy & wants to try Orlistat to lose weight quickly. Her physical activity is minimal. what
would you advise her?
1) Pay a registration to join a health club
2) Adjust her diet
3) Walk around the work corridors during her break
a) 1 & 3
b) 1 & 2
c) 2 & 3
d) 1, 2 & 3
e) none

326. Dr. Prescribed testosterone decanoate, all are true about testosterone decanoate except?
a) Can be prescribed verbally
b) Can be refilled
c) Can be transfered
d) Require record keeping for 2 years
Another version: Regarding Testosterone refills:
a) The intervals between the refills must be specified
b) Can be part-fills
c) Can not be refilled under any circumstances
d) Can be refilled with no restrictions
P.s. this was a really weird question, because choice no. a is correct & choice no. b also correct

327. Treatment of UTI for 3-5 days in some cases where the regular treatment is for 10-14 days
is used for
a) Acute cystitis
b) Hospital acquired acute UTI
c) Pyelonephritis
d) Prostitis
e) Complicated UTI

328. Patient took an overdose of dimenhydrinate and died, what is the cause of death?
a) Bronchospasm
b) Delerium
c) Bradycardia
Accidental antihistamine overdose occurs frequently in infants and children.
Symptoms of dimenhydrinate toxicity in children may resemble atropine overdosage and include dilated
pupils, flushed face, excitation, hallucinations, confusion, ataxia, intermittent clonic convulsions, coma,
cardiorespiratory collapse, and death.
Symptoms may be delayed up to 2 hours after ingestion; death may occur within 18 hours.
In adults, 500 mg or more of dimenhydrinate may cause extreme difficulty in speech and swallowing, and
produces a psychosis indistinguishable from that of atropine poisoning. CNS excitation may be preceded by
sedation, leading to a cycle of CNS excitation, seizures, and postictal depression.
Treatment:
Treatment of dimenhydrinate toxicity is symptomatic and supportive. Emetics are usually ineffective, but in
the absence of seizures, early gastric lavage (with an endotracheal tube with cuff inflate in place to prevent
aspiration of gastric contents) may be beneficial. Patients should be kept quiet, to minimize CNS stimulation;
seizures may be treated with diazepam in adults and phenobarbital in children (additional methods may
include IV sodium bicarbonate, or IV physostigmine salicylate in children). Mechanical respiratory assistance
may be required.
Positive and negative mode of ion mobility spectrometry (IMS) and ion mobility spectrometry/mass
spectrometry (IMS/MS) have shown efficacy for the preliminary screening of emergency patients suspected
of dimenhydrinate and another drug overdose.

329. TPN for child aa glucose, k 40 mmole/L, Na, Ca


Dr. prescribed to be given in rate 6 ml/hr, pharmacist wish to change dose of glucose and aa and
to maintain equivalent amount of k to be given at a rate of 14 ml/ hr, what is concentration of k
in new formula
a) 7 mmole/L
b) 30 mmole/L
c) 36 mmole/L

330. Mr. fox had a prescription for 3 months, filled from 2 and half month, his son came with a
new prescription and asked the pharmacist to fill the new prescription as his father has
transferred to florida now. His insurance plan can cover 3 month/prescription
What is the least concern for the pharmacist?
a) He can not fill prescription as father is living now in USA
b) Tell the son that his father should see a doctor in USA
c) Tell the son he needs signed paper from father that he approves to get the medicines

331. When can the insurance company pay for the new prescription
a) After 2 weeks
b) Immediately
c) After 6 months

332. Patient is taking ramipril for HTN, now his blood pressure is controlled, he is asking if he
can take it every other day, because he is forgetting to take, what is appropriate
a) Give him drug dispenser for whole month
b) Tell him about importance of taking drug daily and consequences if not
333. Which is taken every 10 years
a) Tetanus
b) Hepatitis B
c) Cholera
d) Typhoid
e) MMR

334. Patient taking haloperidol, got tremor and rigidity, all can be done aexcept
a) Add tranylcypromine
b) Add benztropine
c) Decrease dose of haloperidol

335. Which drug have no DDI with OCP


a) Phenytoin
b) Phenobarbital
c) Primidone
d) Gabapentin
e) Topiramate

336. There were many questions about Addison disease and treatment
 Addison disease is primary adrenal insufficiency.
 Weakness, fatigue, and hyperpigmentation (generalized tanning or focal black spots involving skin
and mucous membranes) are typical.
 Diagnosis is clinical and by finding elevated plasma adrenocorticotropic hormone (ACTH) with low
plasma cortisol.
 Low serum sodium, high serum potassium, and high BUN (blood urea nitrogen) occur.
 Usually, plasma ACTH is high and serum cortisol levels are low.
 Replacement doses of hydrocortisone and fludrocortisone are given; doses should be increased during
intercurrent illness.

337. Patient have fever and stiff neck, diagnosed with meningitis, what is DOC?
a) Ceftriaxone and vancomycin
b) Gentamycin

338. Lady travelling to india, she wants to take antibiotic prophylaxis for traveler diarrhea, last
year she tried cotrimoxazole but no effect, what to give?
a) Ciprofloxacin
b) Azithrmycin
c) Bismuth subsalicylate
339. Pt. j. IS a 35 Y. O. female who is placed on lithium therapy. The suggested dose is 600 mg
q8h of lithium carbonate. The total body clearance of lithium is 0.44ml/s or 1.62l/h. The
biological half life is 18 hours. The mole weight of lithium carbonate is 74.
How long will it take to reach 94% of steady state?
a) 18 hours
b) 1 day
c) 1.5 days
d) 3 days----- (94% steady state after 4-5 t1/2)
e) 5 days----- (99% after 6.6 t1/2,100 % steady state after7 t1/2)

340. The lithium dose is now changed to 300 mg q8h.How long will it take lithium to reach
94% of steady state
a) 18 hours
b) 1 day
c) 1.5 days
d) 3 days----- (94% steady state after 4-5 t1/2)
e) 5 days----- (100 % steady state after7 t1/2)

341. Patient has runny nose; nasal congestion, & itchy throat. He has been taking a 2nd
generation antihistamine irregularly for 2 weeks. His symptoms have slightly improved. He
now came to your pharmacy asking for a medication to relieve his symptoms. What would you
recommend?
a) Replace the 2nd generation antihistamine with 1st generation antihistamine
b) Use the 2nd generation antihistamine regularly
c) Add a nasal decongestant to the 2nd generation antihistamine
d) Add an oral decongestant to the 2nd generation antihistamine

342. Which of the following anti-TB medications cause ocular side effects?
a) Ethambutol
b) Isoniazide
c) Rifampin
d) Pyrazinamide

343. Patient was prescribed Methotrexate 15 mg weekly, but he took the Methotrexate 15 mg
daily for 2 weeks. what side effect to expect?
a) Myelosuppression
b) Depression
c) Heart arrhythmias
Oral overdose of methotrexate is often due to incorrect dosage and administration (e.g., daily rather than
weekly administration by patients). Symptoms include leukopenia, thrombocytopenia, anemia, pancytopenia,
bone marrow suppression, mucositis, stomatitis, oral ulceration, nausea, vomiting, GI ulceration and GI
bleeding.
Symptoms of intrathecal overdose include headache, nausea and vomiting, seizure or convulsion and acute
toxic encephalopathy.

344. In the past case, what should the pharmacist do?


a) Suggest to pay for the patient's treatment
b) Report to ISMP
c) Report to the collage
d) Ignore it

345. Which of the following text books contain NO


references?
a) Merck mannual
b) TC
c) Remington
d) Mertindale

346. A recent immigrant to Canada has TB, she is a female, she is 28 yrs old & she is
malnourished. She is taking for her TB; isoniazid, Rifampin, Ethambutol, Pyrazinamide &
Pyridoxine. She has high risk to develop Peripheral neuropathy, because;
a) She has TB
b) She is malnourished
c) She is a female
d) She is 28 yrs old
She is malnourished, therefore the amount of vitamine B6 in her diet is not enough to protect her from the
peripheral neuropathy caused by isoniazide

347. Patient with Dyslipidemia, Alzeihmer & has 85 years old. he is obese & has Hypertension.
what is the cause of his alzeihmer?
a) Dyslipidemia
b) Age
c) Weight
d) HTN
Risk Factors
 Non modifiable: Age > 65, Family history, Genetics, Females > males (evidence is not strong)
 Modifiable: Vascular risk factors, Atrial fibrillation, hypertension, CHD, diabetes, obesity, smoking
 Others: History of head trauma, Lower socioeconomic class, Education, Depression & Alcohol
348. Elderly in nursing home & has pneumonia, what is the possible causative organism?
a) Strept. pneumonia
b) M. pneumonia
c) Legionella
d) Chlamydia pneumophilia
e) Mouth anaerobes
Merck: "Common nursing home‐acquired pneumonia pathogens include gram‐negative bacilli, Staphylococcus
aureus, Streptococcus pneumoniae, Haemophilus influenzae, anaerobes, and influenza viruses"

349. What is the most common cause of pneumonia in preschool children?


a) Listeria monocytogens
b) E. coli
c) S. aureus
d) Strept. Pneumonia
Age Group Bacteria Empiric Antibacterial Regimen
Infants <1 month Streptococcus agalactiae (Group B Cefotaxime + Ampicillin
streptococcus), E. coli, Listeria monocytogenes Add gentamicin if early neonatal
(rare), Neisseria meningitidis (rare), S. meningitis suspected due to synergy
pneumoniae (rare), Other Enterobacteriaceae. for Group B streptococcal infections
Children ≥1 month S. pneumoniae, N. meningitidis, S. agalactiae Ceftriaxone or cefotaxime +
(Group B streptococcus), Haemophilus Vancomycin
influenzae type b, E. coli (rare), L.
monocytogenes (rare), Other Enterobacteriaceae
Adults < 60 y E. coli S. pneumoniae N. meningitidis Ceftriaxone or cefotaxime ±
L. monocytogenes (rare) Vancomycin
Adults with cellular S. pneumoniae N. meningitidis H. influenzae Ceftriaxone + Vancomycin +
immune deficiency type b L. monocytogenes Ampicillin
Humoral immune S. pneumoniae N. meningitidis Ceftriaxone or cefotaxime +
deficiency states H. influenzae type b Vancomycin
Any age: CSF leaks Staph. epidermidis S. aureus S. pneumoniae Ceftriaxone or cefotaxime +
or skull fractures, N. meningitidis H. influenzae b, S. pyogenes Vancomycin
head trauma Enterobacteriaceae Pseudomonas sp.

350. Pneumonia causes: (I can't remember the rest of the options)


a) Hypoglycemia
b) Hyperglycemia

351. Patient on Heparin, & he took an overdose of Heparin, to manage this overdose, do all of
the following except:
a) Give vitamin K
b) Give protamine sulphate
352. Patient with Congestive heart failure. He is on Furosemide. He has shortness of breath.
what to advise him?
a) Compression stockings
b) Salt substitution

353. Which of the following cause hyperkalemia?


a) Furosemide
b) Omeprazote
c) Aliskiren
(Aliskiren is new antihypertension medication. it is a direct rennin inhibitor)
Hyperkalemia Hypokalemia
Drug-induced: ACEIs, ARBs, BBs, aliskiren, Drug-induced: aminoglycosides, amphotericin B,
aminocaproic acid, cyclosporine, digoxin overdose, antipseudomonal penicillins, beta2-agonists, caffeine,
drospirenone, heparin, K+ supplements, K+ sparing foscarnet, insulin, laxatives, licorice, long-term
diuretics, ketoconazole, NSAIDs, penicillin G corticosteroid therapy, loop and thiazide diuretics,
potassium, trimethoprim or TMP/SMX, theophylline, tocolytic agents.
pentamidine, succinylcholine, tacrolimus, alfalfa and . Diarrhea, vomiting
nettle. . Inadequate dietary intake
. Renal failure, hyporeninemic hypoaldosteronism. . Familial history (Bartter or Gitelman syndrome)
. Diabetes, adrenal insufficiency. . Mineralocorticoid excess, e.g., primary aldosteronism
. Familial history of hyperkalemia . Renovascular disease, Metabolic alkalosis
. Acidosis, Massive transfusions . Osmotic diuresis (diabetes), Hypomagnesemia
. Crush injury, trauma, hemolysis, tumor lysis. . Increased sweat loss, Dialysis/plasmapheresis

354. Patient on statin, what should be monitored:


a) Liver function tests
b) Creatine kinase
c) Lipase
Measure lipid levels and liver enzymes when starting statin therapy. Unless there is a concern regarding
elevated triglyceride levels, lipid levels may be drawn in the nonfasting state. Lipid levels may be repeated 4
weeks after starting treatment or following a dose change to asses treatment response. Repeat liver enzyme
testing after 3 months or if liver toxicity suspected. If symptoms of myopathy develop, measure creatine
phosphokinase levels.
355. A child on atomoxetine & Methylphenidate, his mother wants to transfer his file to another
pharmacy:
a) Atomoxetine can not be transferred
b) Methylphenidate can not be transferred, & needs a new verbal Rx from the Dr
c) Methylphenidate can not be transferred & atomoxetine can be transfered

356. A patient is taking omeprazole & needs Ca supplement, which Ca salt should he take?
a) Calcium Carbonate
b) Calcium Citrate
c) Calcium chloride

357. Patient with hypertension & diabetes, what not to give him?
a) Valsartan
b) Nadolol
c) Thiazide
d) Lisinopril

358. A patient, takes no medications, except for occasional antibiotics. He got a new job & was
offered insurance plan, he will have to pay 100$ per month for insurance. what to advice him?
a) Do not do the insurance plan because you don't have a chronic illness
b) Do not do the insurance plan because this will increase the financial expectations of the
insurance companies
c) Tell him to ask the insurance company if there is exclusion policy if he postponed
d) Tell him to postpone joining the insurance company until he has a chronic illness
It is better for patient to have an insurance plan to use incase of disease or illness but here C is reasonable

359. Duties of Public Health Canada; all except:


a) Provides vaccination schedules for both adults & infants
b) Chronic disease control & treatment
c) Prevents and controls infectious diseases
d) Promotion of travel infection prevention
The role of the Public Health Agency of Canada is to:
 Promote health; Prevent and control chronic diseases and injuries;
 Prevent and control infectious diseases;
 Prepare for and respond to public health emergencies;
 Serve as a central point for sharing Canada's expertise with the rest of the world;
 Apply international research and development to Canada's public health programs; and
 Strengthen intergovernmental collaboration on public health and facilitate national approaches to
public health policy and planning.
360. Child is going to be vaccinated, mother is afraid of the pain, what to give him?
a) Acetaminophen
b) Aspirin
c) Penicillin
d) Naproxen

361. All of the following occur in liver failure, except;


a) Decreased metabolism
b) Decreased elimination
c) Decreased plasma protein binding
d) Decrease tissue protein binding
e) Cirrhosis
Liver failure can lead to:
1) coagutopathy (The liver can no longer synthesize enough of the proteins that help blood clot)
2) Jaundice (The liver can no longer adequately process bilirubin‐a waste product formed when old red blood
cells are broken down)
3) Portal hypertension, ascites & hepatic encephalopathy)

362. Which of the following cause Neurocallapse if given intrathecally?


a) Methotrexate
b) Baclofen
c) Corticosteroid
d) Vincristine
ERROR BEST PRACTICE
VinCRIStine and Should never be administered intrathecally because they can be neurologically fatal.
other vinca alkaloids Dispense vinCRIStine and other vinca alkaloids in a minibag of a compatible solution
inadvertently and not in a syringe.
administered by the An effective prevention strategy that reduces the risk of inadvertently administering
intrathecal route vinca alkaloids via the intrathecal route is to dilute the drug in a minibag that contains a
volume that is too large for intrathecal administration (e.g., 25 mL for pediatric patients
and 50 mL for adult patients).

363. Side effects of Inhaled Corticosteroids depends on all except


a) Dose
b) Frequency
c) Other inhaled Drugs
d) Device used
e) Technique by which the patient inhaled
364. Marijuana regulations (there were 3 question about it, it needs to be studied & understood
in details to be able to answer the question)
(I studied it from here: http://www.hc-sc.gc.ca/dhp-mps/marihuana/index-eng.php)

365. If I didn't tell a patient in the hospital that he had wrong medication, then I am breaking the
principle of:
a) Veracity

366. The money spent on hospitals come from:


a) Federal taxes
b) Provintial taxes
c) Loans from the government

367. A Patient with emphysema & it getting worse & he is smoking, why is his emphysema
getting worse. (there were a lot of other details about the patient's condition, but I don't
remember them. I was given his chart, & he was compliant on his meds)
a) Because he is smoking
b) Because he is not compliant on his meds

368. To apply the "Subjective" part of SOAP:


a) Check the patient's profile
b) Check Side effects
c) Check Drug-Drug Interactions
SOAP Meaning Example
SUBJECTIVE • What the patient reports Signs & symptoms, when they
• Info from patient’s perspective started, etc.
OBJECTIVE • What the provider reports/measures Diagnostics, Laboratory results,
(Findings) x-ray, vital signs BP 150/90, SrCr
(80), FBG 6.5 mmol.
ASSESSMENT Interpretation of subjective & objective information Working diagnosis
If writer is physician, assessment will be a disease state Drug interaction due to CYP450
or condition diagnosis & explain reason for diagnosis.
The pharmacist SOAP notes, assessment will identify
DTP and explains why DTP needs to be corrected.
PLAN • Action plan Discontinue Drug X and initiate
• Monitoring parameters Drug Y
369. Patient has HTN, he took overdose of valsartan for 3 days till now. Today his blood
potassium is 5.9 mmol/L & no arrhythmia & no acidosis, what to do for him?
a) Stop valsartan
b) Decrease dose of valsartan
c) Decrease frequency of valsartan
d) Change valsartan to lisinopril
e) Change valsartan to losartan

370. What is the appropriate treatment for his case?


a) Soduim polystyrene sulphate
b) Calcium gluconate
c) Hemodialysis
d) NaHC03
Cation-exchange Promote exchange of Na+ and Ca++ for K+, respectively, in bowel; and also bind Ca & Mg.
resins S.E: Constipating; risk of bowel ulceration or necrosis. Avoid administration of sorbitol with
Ca polystyrene calcium polystyrene sulfonate as it causes colonic necrosis & intestinal obstruction.
sulfonate, Calcium polystyrene sulfonate  Use if sodium overload is a concern.
Na polystyrene The duration of drug contact with the mucosa may be a risk factor. A cleansing enema
sulfonate (sodium-free) is recommended to reduce this risk.

371. Which of the following Rx is wrong?


1. Fluconazole 15 mg Stat
2. Digoxin .... mg (I don't remember how many mgs was written in the exam)
3. Aspirin 81 mg
a) l
b) 1 & 3
c) 1, 2 & 3
d) 2 & 3
e) 1 &2

372. Symptoms of hypoglycemia


 Mild to moderate hypoglycemia has autonomic symptoms: sweating, tremors, tachycardia, heart
palpitations (heavy, fast heartbeats), hunger, nausea, numb lips or tongue, headache and a general
sensation of weakness.
 Severe hypoglycemia requires assistance in its recognition and/or treatment. Neuroglycopenic
symptoms such as confusion, anxiety, feeling irritable, altered behaviour, difficulty speaking and
disorientation can progress to seizures and coma that prevent the patient from appropriately treating
the hypoglycemic episode.
373. Duties of a pharmacist concerning scheduale 2:
a) Be there so if the patient wants to ask something about it
b) Intervene if he sees that the drug is inappropriate for the patient
c) I don't remember the rest of the options
Schedule • Requires prescription for sale, The Highest risk.
I • Most drugs in schedule F (FDA) and some drugs that were listed under Schedule E.
• Some drugs are listed in Schedule I of the NAPRA schedules, but are not listed under Schedule
F, The Food and Drugs Act Regulations.
• Must be signed and released by a pharmacist
Schedule • Does not require prescription
II • Requires professional intervention by a pharmacist before sale
• Not visible or accessible i.e behind the counter BTC or under the counter
• Ex: Tylenol # 1 (codeine 8 mg + acetaminophen 300 mg + caffeine 15mg), Nitroglycerin SL,
Regular insulin, Epipen, most vaccines, Iron supplement (>30 mg), Vit D3 drops & Vit B12 inj
Schedule • Prescription not required, Over the counter OTC
III • Drugs accessible and visible to public but must be within 10 m of dispensary where pharmacist
can answer patient questions
• Ex: Plan B. Emergency contraception, Bacitracin and its salts for eye/ear, fluconazole 150 mg,
lactulose, hydrocortisone 0.5%, 1%. Naproxen 200 mg base or 220 mg salt, maximum package
size more than 6 g. Ibuprofen 400 mg or < 600 mg modified release package size more than 18 g.
Schedule • Sold without professional supervision & sold from any retail outlet
U • Labeling is adequate for self selection without supervision and to ensure safety
• Ex: Bacitracin and its salts for topical use, Naproxen 200 mg base or 220 mg salt, maximum
package size 6 g. Ibuprofen 400 mg or < 600 mg modified release package size less than 18 g.

374. In Diabetes type 2, patient will fast for a month from morning till sunset, what to give him
during the fasting period?
a) Do not give him anything
b) Glyburide
c) Insulin
d) Metformin, low dose

375. Patient taking TCA, has Dry Mouth, his Dr prescribed for him pilocarpine eye drops to be
applied inside each cheek, what can you do?
a) Call the college of physicians & report this Dr
b) Tell the patient not to take it
c) Tell patient this is not a labeled indication for Pilocarpine & let the patient decide
d) Argue with the Dr about the fact that treatment of Dry mouth is not a labeled indication
for pilocarpine eye drops
376. ADHD Child on Atomoxetine, he has evening headaches, what should you tell his mother?
a) Go to the Dr if it becomes persistent & bothersome
b) It's a normal side effect & it will go by itself
Common, usually transient: anorexia, insomnia, weight loss, irritability, dizziness, weepiness, headache,
abdominal pain. (monitor weight & appetite every 6 months)
Transient ‐ stop and re‐evaluate: “zombie‐like” effects, psychotic reactions (such as hallucinations), agitation,
tachycardia, hypertension, growth failure (Monitor growth suppression, record weight and height at baseline
and then every 3–6 months), rebound hyperactivity, leukopenia, blood dyscrasias.

377. Difference between Atomoxetine & Methylphenidate


a) Atomoxetine is not a stimulant, & thus cause sedation not insomnia

378. Donepezil will. cause all except:


a) Nausea
b) Diarrhea
c) Urinary Retention
Donepezil Selective and have greater affinity for AchEi in brain than periphery.
Piperidine- Reduces the hydrolysis of acetylcholine, increasing the amount available in the synaptic cleft.
based Donepezil was effective in 3- to 6-month trials in patients with mild to moderate Alzheimer
Centrally disease (MMSE score of 10 – 26).
active It was also effective in moderate to severe Alzheimer disease (MMSE 0 – 17), and is the only
reversible, non cholinesterase inhibitor approved for all disease severities
competitive. The initial daily dose (5 mg) of donepezil is usually taken at night, but can be taken in the
morning if sleep disturbances occur.
After 4 weeks, try increasing to the target dose of 10 mg/day. Adjust dose after 4 wk
S.E: Cholinesterase inhibitors: theoretically, these agents may lower seizure threshold, increase
the risk of GI ulceration or bleeding, or exacerbate COPD or asthma.
Donepezil: >10%: headache, nausea, diarrhea. <10%: vomiting, anorexia, fatigue, sleep
disturbance, syncope, muscle cramps, urinary frequency. Bradycardia (uncommon), heart block
(uncommon), rhabdomyolysis (uncommon), neuroleptic malignant syndrome (uncommon).
Cholinesterase inhibitors: theoretical concern regarding antagonistic effect of combined therapy
with cholinesterase inhibitors and drugs with anticholinergic activity.
Additive bradycardic effects when used with BBs or CCBs; few reports of actual interactions.
Toxicity may be increased by inhibitors of CYP2D6 or CYP3A4 such as paroxetine,
erythromycin, prednisone, grapefruit juice.
Effectiveness may be reduced by inducers of CYP2D6 or CYP3A4 such as carbamazepine,
phenytoin, rifampin.

379. What most reasonable expectation for a Dementia patient taking Donepezil after 3-6 months?
a) No worsening of symptoms
b) Improvement of mini-mental side effects.
c) Number of times he gets lost wilt decrease
380. A 65 yr old male has Diabetes, hypertension, retinopathy, neuropathy. He has diabetic
foot, Peripheral artery disease & he smokes. Now, has a leg ulcer, his diabetic foot is due to:
a) Age, PAD & DM
b) PAD & DM
c) Neuropathy & DM
d) PAD, DM & Neuropathy

381. The Dr suspects pseudomonas. Which of the following microorganisms can be the cause?
a) Enterococci
b) Anaerobes
c) G-ve bacteria
d) All can cause
Most Likely Pathogen involved in Diabetic Foot Infection
 S. aureus (MSSA or MRSA) causes most infections, beta‐hemolytic streptococci (group A or B most
common)
 Strep. pyogenes (group A streptococcus) causes Erysipelas, P. aeruginosa causes Macerated foot.
 Gram‐positive bacteria including enterococci, Gram‐negative bacteria and anaerobic bacteria

382. What to give him?


Infection Severity Antimicrobial Agent
Localized infections: Frequently treated with outpatient oral
Neither limb- nor life-threatening antimicrobial therapy (7 days)
Usually associated with cellulitis surrounding an ulcer  Amoxicillin/clavulanate
Purulent debris may be present at the base of the ulcer  Cephalexin, Clindamycin
Usual organisms: aerobic Gram-positive cocci  Cloxacillin, Doxycycline
(Staphylococcus aureus and beta-hemolytic streptococci)  Linezolid - SMX/TMP
More extensive infections: The choice of oral vs. parenteral therapy should
Includes more extensive cellulitis, plantar abscess and deep be guided by the extent of the infection and the
space infections. patient’s overall clinical status
Initial antimicrobial therapy against staphylococci, Oral Options
streptococci, anaerobes and common Enterobacteriaceae.  Amoxicillin/clavulanate, Linezolid,
Empiric treatment targeting Pseudomonas aeruginosa is Moxifloxacin
generally unnecessary unless risk factors present, e.g.,  Ciprofloxacin, levofloxacin, SMX/TMP +
history of foot soaking, severe or chronic infection clindamycin or metronidazole.
Patients who are not toxic may be treated with débridement Parenteral Options
and oral antimicrobial therapy  1st, 2nd or 3rd generation cephalosporin +
Patients who are ill or toxic despite moderate local signs are metronidazole.
treated as having a severe infection:  3rd generation cephalosporin +
Limb- or life-threatening, Frequently polymicrobial Clindamycin
Immediate hospitalization, early surgical débridement and  Daptomycin, Linezolid, Carbapenem,
parenteral antimicrobial therapy Piperacillin/tazobactam.
If MRSA present or suspected, add vancomycin or linezolid
Q. Osteomyelitis Palpation of bone at base of ulcer Treatment may require 4–6 weeks of parenteral
S. aureus (MSSA or MRSA) is the most common pathogen, or several months of oral antimicrobial therapy.
but other less virulent organisms may be pathogenic, e.g., Oral Options
coagulase-negative staphylococci.  Amoxi/clav, Cephalexin, Clindamycin,
Treat with IV therapy or long-term oral antimicrobial Cloxacillin, Doxycycline, Linezolid,
therapy using agents that are well absorbed from the GI tract Moxifloxacin, SMX/TMP.
and have good distribution to bone and tissue  Ciprofloxacin, levofloxacin, SMX/TMP
Surgical débridement indicated to remove necrotic debris, plus clindamycin or metronidazole.
abscess or sequestrum Parenteral Options
Therapy should base on culture results whenever possible.  3rd generation cephalosporin plus PO/IV
If MRSA present / suspected, add vancomycin or linezolid. Clindamycin or PO/IV metronidazole
If clinical improvement isn’t observed (e.g. resolution of  Daptomycin, Linezolid, Carbapenem,
erythema, edema, heat, draining sinus, coverage of bone Piperacillin/tazobactam.
with soft tissue), consult with a specialist.
NOVEMBER 2008
1. Finasteride all are true except
a) Low dose for alopecia and high dose for BPH
b) Topical is more effective
c) Inhibit conversion of testosterone
d) Cause impotency
e) More effective locally than orally in
alopecia

2. All of the following autoimmune disease


EXCEPT:
a) Multiple sclerosis
b) Rheumatoid arthritis
c) Cushing’s sydrome
d) Grave’s disease
e) SLE
f) Osteoarthritis

3. RJ come to your pharmacy suffering from acute gout attack. the patient is smoker and taked
sinemet CR for parkinsonism. He is also alcoholic. All of the following increase uric acid level
except:
a) Smoking
b) Sinemet CR
c) Alcohol
d) Hyperlipidemia
e) Hypertension
f) Obesity
Drugs and Conditions Associated with Hyperuricemia and Gout
 Drugs: Alcohol, Cyclosporine, Cytotoxic chemotherapy, Diuretics (thiazide and loop), Ethambutol,
Interferon + ribavirin, Levodopa, Nicotinic acid (niacin), Pyrazinamide, Salicylates (low-dose),
Tacrolimus, Teriparatide.
 Conditions: Excessive alcohol intake, Atherosclerosis, Chronic kidney, glomerular, interstitial renal
disease, Diabetes, Hyperlipidemia, Hypertension, Ischemic heart disease, Lead intoxication, Metabolic
syndrome, Myeloproliferative disorders and some cancers, Obesity, Urolithiasis history, rarely genetic
or acquired causes of uric acid overproduction.
 Dietary factors: excessive protein diet from red meats, organ meats & shellfish,  Purine intake
(shellfish, vegetables such as asparagus, cauliflower, spinach, beans, peas & mushrooms)
4. Dr will prescribe to him all of the following except or which durg is not used in acute gout?
a) Colchicine
b) Naproxen
c) Indomethacin
d) Sulfinpyrazone
e) Allopurinol
f) Cortisone
Pharmacologic Choices
 The earlier therapy is started, the more quickly the attack will be resolved.
 Do not stop or alter the dose of urate-lowering drugs during an acute attack, because symptoms may be
exacerbated or prolonged.
 Q. NSAIDs, colchicine or oral corticosteroids are appropriate first-line options in therapy of acute gout.
Treatment should be initiated within the first 24 hours of acute gout attack onset.
 For acute attacks, recombinant interleukin-1 beta (IL-1) receptor inhibitors such as anakinra and
canakinumab may be considered for use in patients who have failed or cannot take colchicine, NSAIDS
or corticosteroids.

5. What is the target blood pressure in chronic kidney failure?


a) <140/90
b) <130/80
c) <120/80
d) None of the above

6. In ischemic stroke patient, which of the following is not inclusive criteria for Alteplase
therapy?
a) <3hrs
b) No neurological disabling
c) Symptoms persist for >60 mins
d) >6hrs
e) All of the above

7. In above case. which is a DOC to


prevent recurrent stroke?
a) Clopidogrel
b) ASA + Dipyridamole
c) ASA
d) Acetaminophen
e) Ticlopidine
8. Acute attack of angina, he can use all of the following except:
a) ASA
b) Clopidogrel
c) Nitroglycerine patch
d) ACEI
e) Nitroglycerin lingual

9. Pt in hospital did stent to open coronary artery, what medication do you give when he leaves
the hospital?
a) ASA
b) Clopidogrel
c) Ticlopidine
d) Dipyridamole+ ASA
e) All of the above
If k type chooses both, if a type, choose clopidogrel

10. Patient with MI and Asthma, which b blocker is preferable?


a) Acebutalol
b) Propranolol
c) Nadolol
d) Metoprolol
e) All of the above

11. RJ is 56 years taking Amiodarone, you should counsel him about side effets. All of the
following are side effects of Amiodarone except:
a) Conjunctivitis  corneal microdeposits
b) Skin discolouration
c) Hypotension
d) Bradycardia
e) Pulmonary fibrosis
f) Fatigue
g) Nausea & vomiting
Another version: Q about LATE side effect of amiodarone
a) Skin discoloration
b) Eye pigmentation
Eye pigmentation is not a side effect…. Skin discolouration is a side effect of amiodarone
Rxtx
Because of the extensive distribution of amiodarone in body tissues, and the prolonged time required for its
elimination from the body following discontinuation of long‐term therapy, the relationship between adverse
reactions and dosage and duration of therapy, has not been fully established. For some adverse reactions—
for example, corneal microdeposits—a relationship to dosage and duration of therapy has been established,
so that corneal deposits are reversible with dose‐reduction or with discontinuation of therapy. However, for
other adverse reactions—for example, fibrosing alveolitis or peripheral neuropathy—the dose relationship
and the reversibility of the adverse reaction have not been established. Certain gastrointestinal reactions
(e.g., nausea, vomiting, constipation, and bad taste) and central nervous system reactions (e.g., fatigue,
headaches, vertigo, nightmares, and sleeplessness) occur frequently at the initiation of therapy when high
doses are used. These may disappear on reduction of the dose. The time and dose relationship of adverse
events are under continued study.
The most serious and potentially life‐threatening adverse effects associated with the use of amiodarone
hydrochloride are pulmonary fibrosis, the aggravation of arrhythmias, and cirrhotic hepatitis. Published
data reflecting the North American experience with chronic oral amiodarone hydrochloride therapy suggest
that amiodarone‐associated adverse drug reactions are very common, having occurred in approximately
75% of patients taking 400 mg or more per day; these adverse events have led to the discontinuation of
amiodarone treatment in 7% to 18% of patients. The adverse reactions most frequently requiring
discontinuation of amiodarone hydrochloride have included pulmonary infiltrates or fibrosis, paroxysmal
ventricular tachycardia, congestive heart failure, and elevation of liver enzymes. Other symptoms causing
discontinuations less often have included visual disturbances, solar dermatitis, blue skin discoloration,
hyperthyroidism, and hypothyroidism.

12. Amiodarone monitor, all except


a) Liver function AST, ALT
b) TSH
c) Stool analysis
d) Chest x-ray (pulmonary fibrosis)
e) Bruising

13. Amiodarone will interact with


a) Metoprolol
b) Diltiazem
D/D: amiodarone increases effect of B‐blockers + Ca‐channel blockers + warfarin. Plus, increase plasma level
of quinidine‐ procainamide‐ flecainide‐ digitalis‐ diltiazem

14. Old man, hospitalized, HIV. What not to advise?


a) Not to take antacids with HIV medication

15. His wife asks if this HIV due to STD, all except
a) Say yes, he got it through sexual transmission
b) Tell her to ask your husband
c) Tell her to go and do HIV test
16. MJ is an HIV patient. Now he come to the pharmacy comlaining of increase amylase and
pin-point pain, this may due to:
a) Didanosine
b) Lamivudine
c) Zidovudine
In acute pancreatitis serum lipase and serum amylase are elevated however serum lipase are slightly more
sensitive in both major causes of pancreatitis gallstone and alcoholic associated acute pancreatitis.
Drug induced: EtoH, CHC (Estrogen/ EE  ↑ TG), An ‐HIV medictions (didanosine, PIs), isotretinoin ↑ TG,
incretin‐based drugs DPP ‐ 4 Inhibitors (Alogliptin, Linagliptin, Saxagliptin & Sitagliptin), (GLP‐1) Agonists
(Dulaglutide, Semaglutide, Liraglutide & Lixisenatide

17. Which of the following anticancer drugs can cause Renal toxicity?
a) Cyclophosphomide
b) Digoxin
c) Lithium
d) Cisplatin
e) Amlodipine
Nephrotoxicity Prevention varies. Correct electrolyte and metabolic
(days to weeks) Vigorous hydration and alkalinization of the urine abnormalities.
Carmustine, with sodium bicarbonate is required (45–50 mEq/L Maintain intravascular volume and urine
Ifosfamide, of IV fluid) to prevent nephrotoxicity before, during output.
Methotrexate, and after administration of high doses of Dose reductions and aggressive
Mitomycin, methotrexate. hydration with subsequent chemotherapy
Platinums, Saline-based hydration and forced diuresis with treatment.
Streptozocin mannitol reduces cisplatin nephrotoxicity. Dialysis in severe cases.

18. Pharmacist is going to prepare anticancer drug in hospital. Which laminar flow hood is
used:
a) Vertical Laminar flow Hood
b) Horizental Laminar flow Hood

19. Which of the following is most Common cause of Contamination during lamina, flow
preparation?
a) Labcoat
b) Hands of Personal
c) Object
d) Technique
20. Above prepared or compounded preparation, after bow much time can be used?
Beyond-use dates for sterile preparations
According to Single-dose Single-dose vials include pharmacy bulk vials if the manufacturer has
type of vial labelled them as single-dose vials.
container If the vial is punctured in a primary engineering control PEC that maintains
ISO Class 5 air quality, the BUD is 6 hours. Six hours after initial needle
puncture, the vial can no longer be used.
Once the vial is removed from the ISO Class 5 PEC, it must be discarded.
If the vial or another single-dose container is opened or punctured in an
environment with air quality worse than ISO Class 5, the BUD is 1 hour.
Open ampoule No storage of an open ampoule is permitted; as such, no BUD applies.
Multiple-dose A multiple-dose container will be labelled as such by the manufacturer.
container (e.g., Multiple-dose containers usually contain a preservative.
vial) The BUD is 28 days, unless otherwise specified by the manufacturer.
If there is visible contamination before 28 days (or the manufacturer’s expiry
date), the container must be discarded.
According to
risk of
microbial
contamination

Preparations
prepared for
immediate use
or prepared
in segregated
compounding
areas

21. Pharmacist prepared chocolate Metronidazole suspension. what is on the label


I. This preparation is good for one year or Expiry after 1 year
II. Store in amber colour bottle & avoid sun light
III. Shake well before use

22. Rx Polysporine OU
a) Apply once daily
b) Apply to both eyes
c) Apply in right eyes
d) Apply in left eye
e) None of the above
23. Child with croup in emergency dept. Doctor call you to know which medication is good
choice. You can refer which of the following book?
a) TC
b) CPS
c) Martindale

24. Which is wrong about croup?


a) Need antibiotics
b) Hospital, barking at night worsens
c) Relieved within 48 hours.
d) Treat by using prednisolone

25. Patient is using Latanoprost for glaucoma. Which of the following is a major side effect of
Latanoprost?
a) Enlarge eyelash and pigmentation

26. Patient comes to pharmacy. Technician prepared tablet with open hand in front of the
patient. What to do?
a) Send him to home and dispense the medication
b) Say to go away and fill the tablet by himself
c) Review the aseptic technique and dispense the tablets

27. Patient with depression comes to phannacy. He is taking sertraline. Cashier is standing on
computer. Pharmacist is standing in the pharmacy.
a) Can I take St. John wort ask to cashier?
b) Cashier give him counseling and pharmacist listening from pharmacy
c) Cashier says its good medication, take it
d) Pharmacist immediately intervene and address the concern of patient and discuss
the matter later with cashier

28. Patient comes to pharmacy and ask to know about flax-seed. Pharmacist can refer which of
the following book to get more information?
a) CPS
b) TC
c) PSC
d) Good manufacture information pamphlet
29. DOC for Cellulitis: Cephalosporin (Cephalexin). Cephazolin IV (in severe)

30. Length of treatment for Cellulitis: 7 to 10 days

31. Patient is going to take Vancomycin. In which condition oral vancomycin is given:
a) Intensive MRSA
b) Meningitis
c) Treatment of C. difficile in P colitis

32. Patient with hematogenious osteomylitis. Which bacteria is responsible? S. Aurus

33. Acarbose:
a) Cause flatulence diarrhea
b) Post prandial absorption delay
c) Inhibit alpha glucosides
d) All are correct
Alpha- Acarbose, Inhibit intestinal Can increase risk Potentiates other antihyperglycemic
glucosidase Glucobay alpha-glucosidases of hypoglycemia agents.
Inhibitors 50–100 mg resulting in delayed when combined May reduce metformin bioavailability.
TID PO with digestion of starch with insulin or No weight gain; not absorbed.
Lowers each meal; and disaccharides secretagogues. Contraindicated in IBS & IBD.
HbA1c by start low and and reduce post Flatulence, Acarbose prevents other sugars from
≤1%. go slow prandial glucose Q. diarrhea, being quickly absorbed, so use glucose
levels. Do not abdominal pain, to treat hypoglycemia of acrabose.
Miglitol significantly inhibit cramps, nausea Acarbose: 30 min before meals/with
(not in Canada) intestinal lactase. first bite of meals

34. All of the following are benefits of insulin pump EXCEPT


a) Mix different types of insulin's
https://www.canada.ca/en/health-canada/services/healthy-living/your-health/medical-
information/insulin-pumps.html

35. FG is 65 years old he has hypothyroidism. Doctor prescribe Levothyroxine for him after 10
weeks he made analysis for TSH, but he found that Level of the TSH is 0.1 mmol, Normal is
<0.3 - 0.6. So, what you will ask the patient about?
a) Hold for two weeks then research again
b) Decrease dose of levothyroxine
c) Increase dose of levothyroxine
d) Do not change the dose
36. GH is 60 years old female. She suffers from hypothyroidism. Doctor prescribe levothyroxin
0.25 mcg but pharmacist dispense 0.25 mg instead. So, all of the following are expected side
effects except?
a) Tachycardia
b) Cold intolerance
c) Diarrhea
d) Weight loss
e) Irritability

37. Pt with hyperthyroidism, high BP, high heart rate. Which antihypert is best option
a) ACEI
b) B-Blocker
c) CCB
Used for BP and it dec HR and used for hyperthyroidism

38. Methimazole in hyperthyroidism patient:


a) Inhibit thyroid hormone synthesis
b) If stop treatment, disease will recur
c) Patient will feel improvement already in 2 weeks
d) You have to discontinue indefinitely if you get a rash
Methimazole is actively transported into the thyroid gland. It is part of the thionamide class of drugs that
inhibits the formation of thyroid hormones by blocking the incorporation of iodine into tyrosine residues of
thyroglobulin. It also inhibits the coupling reaction of these residues to form iodothyronines. Methimazole is
believed to interfere with the oxidation of iodide ion and iodotyrosyl groups via inhibition of the peroxidase
enzyme. Methimazole does not inactivate stored or circulating T3 and T4 hormones, or the exogenous
thyroxine administered as part of therapy. Unlike propylthiouracil, methimazole does not inhibit the
peripheral conversion of T4 to T3. Skin rash may occur in 3–5% of patients using methimazole. Rash may
resolve if treated, but switching to an alternate antithyroid drug may be required. Methimazole should be
discontinued in the presence of exfoliative dermatitis.
39. RF is 65 years old male he has CHF for two years. came to your pharmacy today. just he
walked from parking lot to the pharmacy and climbed three stairs, he suffered from dyspnea and
shortness of breath. According to NYHA classification of heart failure which grade he is in it:
a) I
b) II
c) III
d) IV
e) V

40. His case is worsening, so you have to do all except:


a) Change hydrochlorothiazide to furosemide
b) Increase hydrochlorothiazide
c) Decrease digoxin
d) Discontinue verapamil

41. Old pat with heart problem suffers from hypothyroidism. Dr. wants to start levothyroxin?
a) Decrease dose in heart patient
b) Increase dose in pregnancy
c) Used in treatment of hypothyroidism
SYNTHROID should be used with caution in patients with cardiovascular disorders, including angina,
coronary artery disease, and hypertension, and in the elderly, who have a greater likelihood of occult
cardiac disease. In these patients, levothyroxine sodium therapy should be initiated at lower doses than
those recommended in younger individuals or in patients without cardiac diseases.

42. You are a clinical pharmacist. You are planning to organize a meeting for osteoporosis. You
will invite all the following categories except;
a) Postmenopausal women
b) Patient with anorexia nervosa
c) Elderly
d) Women made oophrectomy

43. Risk factors for osteoporosis, except?


a) Obesity
b) Alcohol
c) Smoking
d) Long period on corticosteroids
44. Patient taking nitroglycerin, you should advise all except:
a) Use NG. three times 5 minutes apart before ask for emergency
b) If headache go to physician
c) Prime several times
d) NTG spray can be given before exercise
e) Prime before 1st dose and reprime if not used for 14 days

45. Patient taking COC, which of the following missed pill there are less chances of pregnancy.
a) Do not use condom
b) Miss 2 - 3 tab in last week
c) Miss a pill in the 2nd week
d) Miss l to 2 pills in the first week
The greatest risk of pregnancy is when you miss pill at the beginning or at the very end of the pack.
Another version
Pt misses OC pills during the month, what is true to tell her except
a) Ovulation starts at the first week of menstruation
b) Ovulation starts at the second week of menstruation
c) If you miss a pill in the 2nd week, pregnancy possibility increases

46. All of the following are symptoms of menopause EXCEPT


a) Hot flashes
b) Tachycardia
c) Sweating
d) Insomnia
e) Bradycardia
hot flashes, night sweats, sleep problems, nausea, dizziness, anxiety, palpitations (pounding or racing
heartbeat), fatigue (secondary), memory problems (secondary), mood changes or irritability (secondary)

47. KS is pregnant woman (1st trimester) she is suffering from UTI infection. She can take:
a) Amoxicillin
b) Erythromycin
c) TMP/SMX
d) Ciprofloxacin.

48. With symptoms of UTI, goal of therapy except?


a) Relief of symptoms within 24 hours.

49. Which of the following is responsible for UTI? E. coli.


50. Symptoms of allergic conjunctivitis
ocular itching, tearing, redness and mild eyelid swelling

51. ACEI + dry cough what is alternative: ARB

52. Left ventricular heart failure results in which type of edema: Pulmonary edema

53. Which is not used in left heart failure?


Non dihydropyridine CCBs – verapamil & deltiazem

54. Which cause impotence


BBs, statins (rare), thiazide diuretics, K- sparing diuretics, cimetidine, clonidine,
methyldopa, digoxine, MAOIs, finasteride, TCAs, SSRIs

55. A pregnant lady has heartburn, what do you recommend?


 Lifestyle modification is the cornerstone for treating mild symptoms, if this fails, begin nonsystemic
drug therapy with antacids except NaHCO3 (fluid retention & metabolic alkalosis)
 Alginates, Sucralfate, Ranitidine are also safe for use during pregnancy and breastfeeding. In more
refractory cases, PPIs may become necessary, and appear to be safe in pregnancy and breastfeeding.

56. Lady working as IT and spend a lot of time sitting on her computer. Her hight 5’6” and
weight 73 Kg, waist circumference < 100, she is going to Hawaii and want to lose some weight.
She came asking for orlistat, what is true?
a) She is good candidate to orlistat.
b) No need to use orlistat

57. What to advise her:


a) Join a health club
b) Brisk walking during her lunch hour
c) Low fat diet & reduce calories

58. The duration of insulin can be stored in room temperature (after opening vial)?
a) 4weeks
b) 6weeks
c) 8 weeks
The standard recommendation from all the insulin manufacturers is that a vial of insulin you are using can
be kept at room temperature for up to 28 days.
59. Don’t Shake all of the following except or which of the following require auxillary label
“shake well befroe use”
a) Calcitonin
b) Imitrix
c) Nitroglycerin spray
d) Betoxolol eye drop

60. Chronic Ulcerative colitis what is the DOC maintenance therapy?


a) Azathioprine
b) Sulfasalazine
c) 6 Mercaptopurine
d) 5-ASA
e) Prednisone

61. Elderly patient 84 years old who is immunocompromised. He's living with his daughter and
his grandchildren. Dr. is hesitant about giving flu vaccine. What's the most appropriate action?
a) Give him the flu vaccine
b) Immunize his close contacts and caregivers
c) Don’t recommend immunizing him because he is immunocompromised
d) Give him zanamivir as prophylaxis

62. Treatment in chemotherapy (choice of agent) depend on all except


a) Dose of anticancer drug
b) Type of drug
c) Route of administration
d) Site of tumor

63. Regarding chemotherapy


induced nausea and vomiting all are
true except:
a) Dexamethasone is the best
corticosteroid in N and V
treatment
b) Ondansetron best in delayed
N and V
c) Dimenhydrinate is used in N
and V incuded cancer
chemotherapy
d) Metoclopramide best in breakthrough N and V
64. Niacin action
a) Decrease LDL & increase HDL
b) Decrease total cholesterol
c) Decrease TG
Niacin – Strongly inhibits lipolysis in adipose tissue, thereby reducing production of free fatty acids.
Vitamin B3  Start with 50 mg TID PO; double dose Q5 days to 1.5 – 2 g/day.
(Nicotinic Acid)  Usual dose if tolerated: 1.5– 4 g/day divided TID PO after meals
Derivatives S.E: Common: hot flushes and pruritus, dry skin, acanthosis nigricans (reversible),
reactivation of peptic ulcer, GI disturbances, increased blood glucose, glucose intolerance.
Greatest HDL Rare: torsades de pointes, severe hepatotoxicity (more frequent with slow-release
raising effect 15 formulation), increased uric acid, transaminases.
to 35%, Use caution if using with statins because of potential hepatotoxicity and myopathy.
Flushing: Reassure patients that symptoms abate with time. A “no-flush” formulation
TG reduction 20 combining inositol and niacin purports to reduce this adverse effect.
to 50% (higher Administration of ASA 325 mg prior to taking niacin decrease flushing side effect
dosage) Niacin, extended release: Less flushing because taken at bedtime.
Monitor blood glucose, uric acid, transaminases at 3, 6 and 12 months, then yearly.
CI: severe peptic ulcer disease, uncontrolled hyperglycemia, severe gout, hepatic disease.
Glucose intolerance is not an absolute contraindication to the use of niacin.
Avoid hot drinks, hot showers, spicy foods, alcohol for 1–2 h after a dose.
Tolerance develops within several weeks. Avoid missing a dose.

65. Nested case control study?


 A nested case-control study is a type of study design where new case controls are applied into
cohorts that were defined before the study begins.
 Compared with case-control study, nested case-control study can reduce 'recall bias' and temporal
ambiguity, and compared with cohort study, it can reduce cost and save time.
 The drawback of nested case-control study is non-diseased persons from whom the controls are
selected may not be fully representative of the original cohort, due to death or failure to follow-up
cases.

66. Business man he is so busy. Doctor prescribeD ciprofloxacin 250 mg t.i.d. and he is already
on CaCo3 two times daily. He is complaining of so much drugs and afraid he may miss some
doses, what you will do:
a) Stop taking CaCO3 till he finishes his course of ciprofloxacin.
b) Contact his doctor to shift to Levofloxacin
c) Take both drugs as prescribed.
d) Take ciprofloxacin two times and CaCo3 once
Although, ciprofloxacin may be taken with meals that include milk, simultaneous administration with dairy
products, alone, or with calcium‐fortified products should be avoided, since decreased absorption is possible.
It is recommended that ciprofloxacin be administered at least 2 hours before or 6 hours after substantial
calcium intake (>800 mg)
67. In order to council patient on Methotrexate:
I. Avoid alcohol
II. Photosensitivity
III. Tell doctor when taken with other drugs
Q. A folic acid antagonist that inhibits cytokine production and purine nucleotide biosynthesis,
Methotrexate leading to immunosuppressive and anti-inflammatory effects.
Weekly IM injections can induce and maintain remission in patients with CD, but it has not
15–25 mg been demonstrated to be effective for inducing or maintaining remission in patients with UC.
IM/SC weekly S.E: Nausea, flulike aches, headache, oral ulcers, bone marrow and liver toxicity, pneumonitis,
immunosuppression, lymphoma.
Alcohol restriction may minimize hepatotoxicity. NSAIDs may increase serum concentrations.
Take leucovorin (folinic acid) 1 mg OD on a different day than the methotrexate, separated by
≥24 hours to reduce severity of adverse effects.
Contraindicated in pregnancy and breastfeeding owing to its teratogenic and cytotoxic effects.
Recommend effective contraception. Discontinue 3 – 6 months prior to attempted conception.
Q. Baseline assessment should include complete blood count (CBC) with differential and
platelet counts, hepatic enzymes, renal function tests and chest xray. Monitor hematology at
least monthly, and hepatic enzymes and renal function every 1–2 months.

68. Pt using Azathioprine, 6- Mercaptopurine & Methotrexate together, have the following side
effects except OR What NOT to monitor??
a) Leucopenia
b) Fever, sore throate
c) Shortness of breath

69. Canadian health act covers all except?


a) Prescription drugs
b) Doctor’s salaries
c) Pharmaceuticals
d) Lab. tests

70. Mother come to you because her child has Lice. and she tried with lindane 2 times but still
suffering from itching. so, what is the most appropriate to advice her: (K type)
a) There is effective oral OTC
b) Treatment is repeated over 3 weeks till effect
c) There is resistance to species
71. Mother wants permethrin cream for her child treating scabies, which of the following is
INCORRECT counseling?
a) Treat bedmates
b) Child shouldn’t go to school till free of nits
c) Wash all bedlinens and dry in hot air
d) Only treat sexual partner
e) Apply on the whole body

72. Monitoring for zidovudine?


a) Lactic acid
b) Myocarditis
c) Anemia, neutropenia
d) All
Zidovudine S.E: Nausea, headache, malaise, fatigue, rash, myositis, myocarditis, anemia, leukopenia,
(AZT) hepatic steatosis, elevated liver enzymes, lactic acid and CK.
pyrimidine Longterm use associated with peripheral lipoatrophy.
analog Available as an oral syrup.
Additive hemotoxicity with other agents, e.g., anemia with dapsone, foscarnet, ganciclovir,
pentamidine, ribavirin.
Pharmacologic antagonism with stavudine.
Avoid combined use with ribavirin or stavudine as they activated by same intracellular pathway.

73. Sick day mangement, all except? decrease sugar intake

74. Function of drug utilization committee?


 Remove drugs that are not used a lot from formulary
 Change expensive drug to cheaper ones

75. Which anticoagulant used for post MI Pt to prevent stroke?


a) Warfarin
b) Clopidogrel
c) Ticagrelol
d) ASA

76. All drugs used for chronic spasticity except?


a) Baclofen
b) Tizanidine
c) Sertraline
d) Gabapentin
e) Dantrolene
77. P.J. is a 35-year-old female who is placed on Lithium therapy. The suggested dose is 600mg
q8h of lithium carbonate. The total body elearance of lithium is 0.44mL/s or 1.621 L/h. The
biogical half life is 18 hours. The molecular weight of lithium carbonate is 74. How long will it
take to reach 94% of steady state
a) 18 hours
b) 1 day
c) 3 days
d) 5 days
Css is 99% = t1/2 * 6.65 = 18 x 6.65 =119.7 hrs = 4.98/ day
Css is 95% = t1/2 * 4.32 = 18 x 4.32 = 77.76hrs = 3.24/day
Css is 90% = t1/2 * 3.32 = 18 x 3.32 = 59.76hrs = 2.49/day

78. Patient of asthma taking salbutamol. Symptoms are present in night. Which drug pharmacist
can add?
a) Corticosteroid
b) LABA
c) LTRA
d) Theophyline

79. Athlete foot patient comes to pharmacy and said from last 7 days I am using clotrimazole
but there is no improvement. Councel him?
a) Wait for 2-3 weeks
b) Wait for 4 months
c) Consult doctor

80. treatment of hyperkalemia? Insulin + glucose (very important topic maybe 2 q’s)

81. Which of the following needs sales record?


a) Testosterone
b) Tylenol -3
c) Tylenol -2
d) Dexamphetamine
e) Phenobarbital

82. Anaphylaxis symptoms include all of the following except:


a) Bradycardia
b) Rash
c) Swollen tongue
d) Fluctuation in BP.
83. Patient comes to you with a history of anaphylaxis. He asked for Epipen injection. What is
true about EpiPen injection counselling?
a) You can even inject it on top of your clothes
b) You should keep it refrigerated
c) Shake well before injection
d) Take the 1st dose if improvement occurs, don’t seek medical attention
e) If improvement occurs with the 1st dose, wait up to 6 months to use the second dose
Do not freeze and do not refrigerate it.

84. Cyclophosphamide SE? Hemorrhagic cystitis


Use MESNA Prevent of high‐dose cyclophosphamide‐induced hemorrhagic cystitis.

85. AJ hypertensive patient, now he has stable angina. Which of the following shouldn’t be taken?
a) Statin
b) ACEI
c) ARBs
d) Clopidogrel
e) Nitroglycerine patch

86. pt taking 5mg prednisone tab. She is taking 10mg/day for the 1st week then dose decreased
by half tab every week. Calculate total tab required
Answer:
1st week: 10 x 7 = 70 mg 2nd week: 7.5mg x 7 = 52.mg
3rd week= 5mg x 7 days= 35mg 4th week= 2.5 mg x 7 day = 17.5
Total = 175 / 5 = 35 tablets
87. You are working in aseptic area which of the following should be prevented in aseptic
technique or aseptic technique of sterile soln, must be free of
a) Microorganism
b) Particulate matter
c) Pyrogen

88. Preservatives are not added to? Large volume parenterals

89. A patient is admitted to the hospital and was put on IV infusion. After few days, she
developed infection caused by coagulase (-) gram (+) microorganism from her blood culture.
Which of the following is the pathogen?
a) Pneumococcus
b) Strep pneumonia
c) Staph epidermidis
a) d)Strep group B
Coagulase (‐) gram (+) – S. epidermidis; Coagulase (+) and gram + ve is S. aureus

90. Mother of child 28-month-old called your pharmacy for complaining of ear pain and mild
fever. Recurrent AOM 1 month, 23rd month and 25th month. Previously treated with high dose
amoxicillin. What you will not recommend to this child?
a) Cefprozil
b) Cloxacillin
c) Clindamycin
d) Azithromycin
e) Amoxycilline clavulanate

91. Pathogen: Viral or Bacterial


The primary bacterial pathogens involved in acute otitis media are Streptococcus pneumoniae, Haemophilus
influenzae and Moraxella catarrhalis.

92. Same case, what is true councelling point of the pharmacist to the patient’s mother upon
discharge?
a) Kids if not treated, may lead to mastoditis
b) Put cold compress behind the ear to relieve pain
c) Don’t give acetaminophen for fever because fever is a marker for AOM to resolve
d) Use psuedoephidrine for congestion

93. pt with pneumonia. Which organism that led pt to go to hospital? P. aurigenosa


94. All the following side effects are due to decrease of estrogen, except:
S/E of estrogen deficiency: Dyspareunia, vaginitis, vaginal bleeding, oligomenorrhea, hot
flash, mood change, alteration of sexual function, night sweats and decrease libido. breast
tenderness

95. Add progesterone to estrogen to prevent all of the following except:


a) Breast cancer
b) Endometrial cancer
c) Mood changes

96. RF is 45 years old female. She is suffering from cancer. She came to you with a prescription
of morphine sulphate, senna & docusate. She told it is too much for her to take all these laxatives.
what you will counsel her:
a) Call dr. To see why laxative regimen
b) Take senna when you feel the need
c) Change to lactulose
d) Take docusate regularly and senna prn.
e) Take both regularly as this is recommended for her case.

97. Auxillary label “keep in fridge”? or Which of the following refrigerate before selling?
a) Viroptic “trifluridine” eye drops
b) Diclofenac eye drop

98. Practice of pharmacy regulated by?


a) Provinicial regulatory authorities.
b) Canada health
c) NAPRA

99. Tobramycin used to treat Osteomyelitis. You have to monitor all of the following except:
a) Serum tranaminase
b) Ototoxicity
c) Nephrotoxicity

100. Unapproved indication found in:


a) Medline
b) CPS
c) Martindale
d) Therapeutic choices
e) Patient self care
101. Three key words: “Ramipril”. “Cough”, “Review”, on Medline; what reference will he see
or which citation will come up on the internet?
a) TC
b) Pharmacotherapy journal
c) CPS
d) Clinical pharmacology
e) Drug facts and comparision

102. Interaction between ACEIs and indomethacin?


a) Inhibition of bradykinin by ACEIs
b) Enzyme induction by indomethacin
c) Displacement of captopril by indomethacin from binding site
d) Inhibition of prostaglandin synthesis by indomethacin

103. Patient has UTI & Can’t empty bladder (renal failure), which one he should not take :
a) Meperidine
b) Ketorolac
c) Celecoxib
d) Naproxen

104. Allopurinol all except;


a) Start with acute attack
b) Colchicines given in acute attack
c) Hyperuricimea treated even if there is no acute attack.

105. Allopurinol SE, when to refer to Dr.


a) Severe rash
b) Palpitation
c) Anxiety
d) Depression

106. ACEIs monitor which is true?


a) Potassium & Magnesium
b) Potassium & serum creatinine
c) Sodium & calcium
d) Serum creatinine
e) Serum electrolyte
f) Hematocrit
107. Brand drug, now there is a new generic for it. what we will consider for its generic or
which of the following should considered for Bioequivalent drug:
I. T max
II. C max
III. AUC
a) I only
b) III only
c) I & II
d) II & III
e) I, II, & III

108. 2 drugs, orally given (same route of admin, but one was absorbed in blood before the other
due to:
a) Lower fat solubility
b) Lower dissolution rate
Poor oral bioavailability for many drugs is generally attributed to poor solubility in the gastrointestinal
fluids, poor gut membrane permeability and/or extensive hepatic first‐pass elimination.

109. patient complains of red, hot and swelling skin. On discussion, she mentioned that she is
on acuttane & OCP. All of the following can be done except?
a) Aloe gel
b) Ibubrofen
c) Prednisone
d) Bacitracin oint

110. BDZ stop, after 24 hours withdrawl symptoms appear 3-5 days later; what drug
a) Lorazepam
b) Diazepam
c) Flurazepam
d) Chlordiazopoxide
e) Clonazepam

111. Patient comes to you with acne taking some medication, which of these not photosensitive:
a) Isotretinoin
b) Erythromycin
c) Clindamycin
d) Retin A
e) Tretinoin
112. Betamethasone 0.1% cream, petrolatum, glaxal base 1:2:2 M 150gm
Calculte betamethsone quantity?
150 / 5 parts = 30gm/part as betamethasone is 1 part, so we need quantity in 30gm
0.1gm ‐‐‐‐‐ 100gm X gm ‐‐‐‐‐ 30 gm X = 30*0.1/100 = 0.03gm = 300mg

113. Pt on MAOI. what meds to avoid??


SSRIs, SNRIs, TCAs, triptans, meperidine, dextromethorphan and tryptophan due to high risk of serotonin
syndrome. Also, Drugs sympathomimetic, ephedrine, and pseudoephedrine
Food to avoid: Aged cheese (cheddar, blue and swiss), fava bean, smoked meats, wine, liquor, yeast, raisins,
chocolate, dry salami, pepperoni, sausage, bananas, figs, tea and coffee.

114. RJ is 56 years old patient is suffering from parkinsonism. He is taking Sinemet CR. In
order to monitor parkinsonism symptoms, you should monitor all of the following except:
a) Cutting vegetables
b) Bradykinesia
c) Unilateral symptoms
d) Akathesia
Other versions
S.E of L-dopa except? Nystagmus
When starting treatment with sinemet watch for? urine input and output

115. Which symptoms indicate poor response to Sinemet:


I. On-off phenomenon
II. Wearing of phenomenon
III. Akathesia
a) I only
b) III only
c) I & II
d) II&III
e) I, II & III

116. In order to provide more control for RJ symptoms the doctor decide to Bromocriptine to
Sinemet CR. So, what should be done:
I. Discontinue Levodopa for few days before giving Bromocriptine.
II. Start with test dose of Bromocriptine at night.
III. Decrease dose of Levodopa and increase dose of Bromocriptine gradually.
a) I only
b) III only
c) I & II
d) II & III
e) I, II & III

117. SG is 25 years old patient has psoriasis; he is on fluticasone. He thinks it may relieve with
sun. so, he decided to go to Caribbean. You will advise him with:
I. Use UVA & UVB sun blocker.
II. Stop corticosteroid.
III. Sun may worsen psoriasis.
a) I only
b) III only
c) I & II
d) II & III
e) I, II & III
As with all psoriasis remedies, consistency and persistence are keys to a successful outcome when using
sunlight to treat psoriasis. Short, multiple exposures to sunlight are recommended. To get the most from the
sun, give all affected areas equal and adequate exposure, but avoid overexposure and sunburn. Be aware
that it can take several weeks or longer to see improvement. It is a good idea to see your dermatologist
regularly to watch for sun damage.

118. Atopic dermatitis risk factors except:


a) Food allergy
b) Female gender
c) Skin allergy
d) Asthma

119. Flouxetine not taken with:


a) Mepiridine
b) Ibuprofen
c) Celecoxibe

120. Patient suffering from schizophrenia. he is on Haloperidol. He switched to Respiridone. He


should monitor all except:
a) Anticholinergic
b) Blood pressure
c) Serum level of Respiridone.
121. Alzehimer goal of therapy except:
a) Delay progression
b) Delay death due to Alzheimer
c) Treatment will delay caregivers need
d) Treatment improves functioning

122. ADHD, what are the goals of therapy?


a) Athlete improvement achievement
b) Social concentration
c) Functional improvement
d) Restlessness improvement

123. Folic acid, all are true except?


a) Use only during pregnancy
b) Continue to take after pregnancy
c) If neural tubule effect history in family use 5 mg

124. Urinary retention aggravated by: Amitriptylline


Urinary retention has been described with the use of drugs with anticholinergic activity (e.g., antipsychotic
drugs, antidepressant agents and anticholinergic respiratory agents), opioids and anaesthetics, α‐
adrenoceptor agonists, benzodiazepines, NSAIDs, detrusor relaxants and calcium channel antagonists.
Elderly patients are at higher risk for developing drug‐induced urinary retention, because of existing co‐
morbidities such as benign prostatic hyperplasia and the use of other concomitant medication that could
reinforce the impairing effect on micturition.
Drug‐induced urinary retention is generally treated by urinary catheterization, especially if acute, in
combination with discontinuation or a reduction in dose of the causal drug.

125. SH is a very busy lawyer. She has high cholesterol level. She also smoker. And due she is
so busy; she always eats fast food. Sometimes she rides a bike at evening around her house. She
drinks one glass of wine at night. All of the following are beneficial for her except:
a) Cook food at home and take it with her
b) Go to work by bike
c) Quiet smoking
d) Eat regular diet
e) Decrease alcohol intake
126. Patient with stroke. he admitted for hospital for three days. Today he will be discharged.
Doctor prescribes for him Simvastatin. He should monitorall except;
a) LDL level
b) Cholesterol level
c) Liver enzymes
d) Creatin kinase
e) Compliance

127. Decrease estrogen in post menopause will lead to all except:


a) Breast tenderness
b) Insomnia
c) Vaginal dryness
d) Non-bacterial infection
e) Osteoporosis

128. Corticosteroid has the following side effects except:


I. Slow gait
II. Bruising
III. Puffy face
a) I only
b) III only
c) I & II
d) II & III
e) I, II & III

129. What is the most appropriate regarding Tacrolimus counseling?


I. Cause burning and stinging upon application
II. Used for long term intermittent use
III. Apply occlusive bandage after application
a) I only
b) III only
c) I & II
d) II & III
e) I, II & III
Another version: Calcinurin inhibitors.
a) Not used for long time b/c of Cancer
b) Not used in children < 2 years
c) 2nd line thearpy
d) Apply in thin layer and avoid UV exposure
130. Capsaicin side effects include:
a) Burning
b) Cooling
c) Used intermittently
d) Side effect disappear after two weeks

131. Patient is feeding with nutrient through tube. All of the following could be crushed except;
a) Omeprazole
b) Digoxin
c) E.C. Aspirin
d) Diltiazem CD

132. Furosemide side effects include all of the following except:


a) Hypermagnesemia
b) Hyponatremia
c) Impotence
d) Hyperglycemia
e) Hypokalemia

133. Medical rep. with new drug all except:


a) Company research

134. Error in hospital invite all except:


a) Patients
b) Physicians
c) Pharmacists
d) Nurses
e) Medical director

135. Man come to suffering from depression. Doctor prescribe Paroxetin for him. The
pharmacist can ask all the following questions, except?
a) When was the last time you visited the doctor?
b) Is he under more stress these days?
c) Current medication
d) Has there been a loss of a family member “?
136. This patient ask you about St John’s wart, you will give it him applying which ethics:
a) Paternalism
b) Beneficence
c) Non-maleficence
d) Autonomy

137. You will be a lecturer in lecture of erectile dysfunction for some patients. You will discuss
all of the following except:
a) Side effects
b) Intimacy
c) Drug-drug interaction
d) Causes

138. Verapamil differ than Nifedipine in: or Difference between DHP and NDHP CCB??
a) Decrease Ca influx into cells
b) AV block
c) Increase Ca outside cells
d) DHP works more on peripheral vasodilation
e) NDHP works more on the heart and has less peripheral vasodilation, more negative
chronotropic effect

139. Glaucoma parameters:


I. Decrease visual acuity
II. Decrease optic nerve damage
III. Decrease intraocular pressure
a) I ONLY
b) III ONLY
c) I & II
d) II & III
e) I, II & III

140. Ipratropium bromide nasal spray you will advise which of the following;
I. Prime twice before use
II. Use it away from eye
III. Use for long period cause rebound congestion
a) I ONLY
b) III ONLY
c) I & II
d) II & III
141. Patient suffering from headache. He is diabetic three years ago. He is hypertensive. You
should refer him to physician for all except:
a) Headache
b) Blurred vision
c) Itchy, runny tears
d) History of asthma
e) History of diabetes

142. Mother has her son 8-month-old with vomiting and diarrhea 36 hours what to advise:
a) Refer to doctor
b) Wait two days then refer to doctor
c) Give ORS for 5 days
d) Give Kaolepectin

143. Cough due to ACEI occur due to accumulation of:


a) Leukotriene
b) Bradykinine
ACE inhibition increases the cough reflex. The mechanisms underlying ACE inhibitor‐induced cough are
probably linked to suppression of kininase II activity, which may be followed by an accumulation of kinins,
substance P and prostaglandins.

144. All are true to convert from ACEI to ARBs except:


a) Hyperkalemia
b) Angiodema
c) Cough
d) Bilateral renal artery stenosis

145. Which drug used in hypertension with hyperlipidemia:


a) Acebutalol
b) Atenolol
c) Carvedilol
d) Propranolol
e) Hydochlorothiazide

146. European drug found in all except;


a) Martindale
b) European pharmacopeia
c) Index nominum
d) Cps
147. Which drug return to wholesaler without permission: Tranylcypromine

148. Send signed order to wholesaler within 5 days: Ativan (Lorazepam)

149. Patient with seizure which is wrong:


a) Valproic acid cause hepatic dysfunction in early treatment.
b) Carbamazepine causes gingival hyperplasia.

150. Patient had two seizures of grand mal epilepsy. His doctor prescribes phenytoin for him.
but he suffered from gingival hyperplasia as a side effect of phenytoin. You will advice him
with all of the following except:
a) Go to dentist.
b) Check your teeth & use tooth brush regularly
c) Use dental floss regularly
d) Reduce dose
e) Change to another
f) Vit B 6 is given

151. Vincristine has the following side effect:


a) Neuropathy
Vincristine has a high neurotoxicity level. If given intrathecally by accident, it can cause ascending radiculom
yelo encephalopathy, which is almost always fatal.

152. Omega-3 containing products, where to find information:


a) Goodman & Gillman
b) CPS
c) Primary literature
d) Martindale

153. Patient with hyperthyroidism if you need fast relief of symptoms what to give:
a) Propranolol
b) Levothyroxine
c) Amlodipine

154. Addison disease what to give:


a) I.V. Hydrocortisone
b) I.V. Dexamethasone
c) P.o. Cortisone
d) P.o. Dexamethasone
155. Lady with 85 years old admitted to hospital with emphysema, she is suufering from asthma
5 years ago, she is hypertensive and diabetic. She smokes 1 pack per day. Which of the
following made her status worsen?
a) Asthma
b) Hypertension
c) Diabetes
d) Smoking
e) Female gender

156. When she admitted to hospital, her doctor prescribes Metered Dose Inhaler for her, but she
refuses to take MDI due to bad compliance with her, you will advice her with which of the
following:
a) Take MDI because it is of much profit for the hospital
b) Change MDI with other alternative less cost
c) Convince her with MDI because it is the most effective in her case

157. Patient with moderate hypertension which of the following is not the first choice:
a) Hydrochlorothiazide
b) ACEI
c) Metoprolol
d) ARBs
e) Amlodipine

158. Diabetic patient since many years his profile is as following: Metformine 1000 mg B.I.D.
Glyburide 7.5 mg once. He is not controlled what to advise him:
a) Increase biguanide dose
b) Increase sulfonylurea dose
c) Add Acarbose
d) Add glitasone
e) Add insulin

159. After a while he suffer from Dyspnea and shortness of breath and edema, which of the
following drugs cause these side effects:
a) Metformine
b) Glyburide
c) Acarbose
d) Rosiglitasone
160. If you want to add Insulin to his profile which one of the following is most suitable:
a) NPH-Regular 70-30
b) Regular only
c) NPH only
d) NPH-Lispro 70-30
e) Lispro-Regular

161. Diabetic patient on Insuline and has the following measurements:


11 --- 8 A.M, 5 --- 5 P.M. 6 --- 3 A.M.
What to give him:
I. Increase NPH in the morning
II. Increase regular in the morning
III. Increase NPH in the evening
a) I only
b) III only
c) I & II
d) II & III
e) I, II & III

162. Patient will undergo a procedure of Hip replacement he will take which of the following:
I. Enoxaparin
II. Deltaparin
III. Heparin
a) I only
b) III only
c) I & II
d) II & III
e) I, II & III

163. II gtt O.U. T.I.D


a) Two drops each ear three times daily
b) Two drops right eye three times daily
c) Two drops left eye three times daily
d) Two drops each eye three times daily

164. When dispense Andriol” testosterone” you look in:


a) Narcotic Act
b) Controlled substance Act
165. Drug of choice in anticipatory nausea and vomiting:
a) Metochopramide
b) Dimenhydrinate
c) Benzodiazepines
d) Diphenhydramine

166. Methylphenidate:
a) Prescribed by vet., dentist & medicine
b) Prescribed by nurse, dentist & medicine
c) Prescribed by pharmacist, dentist & medicine
d) Prescribed by vet, pharmacist & medicine
e) Prescribed by pharmacist, nurse & medicine

167. Lithium interact with with all of the following except:


a) Diclofenac
b) Phenytoin
c) Amitriptylline
d) ACEI
e) ARBs

168. Patient comes to you in the pharmacy with sunburn. you will advice with all except:
a) Cortisone
b) Celecoxibe
c) Ibuprofen
d) Bacitracin
e) Indomethacin

169. Patient come to you with a history of endocarditis. And now he will make a dental
procedure. what will you give him as a prophylaxis?
a) Amoxicillin
b) Cloxacillin
c) Clindamycin

170. You are a pharmacist in hospital and you want to add a bacteriostatic to all of the
following except:
a) TPN
b) Eye drops
c) Oral syp
171. RJ is patient comes to your pharmacy with a prescription of multiple drugs. he has
congestive hear failure two years ago and his doctor prescribe benzodiazepine for him to relieve
his anxiety. At the same time his prescribe psyllium for his constipation. His profile as
following:
1. Digoxin once daily
2. Psyllium T.I.D
3. Benzodiazepine once daily
4. Multivitamine once daily
5. Amlodipine once daily
You will dispense these medications in a blister pack for breakfast, lunch, dinner & bedtime, the
four blister packs are full. To where you will be interested to look after:
a) Digoxin once A.M.
b) Benzodiazepine once at bedtime
c) Multivitamine once in the lunch
d) Psyllium three times daily
e) Amlodipine at dinner

172. SJ is 73 years old female. She admitted to hospital with acute exacerbation of asthma.
After a while she got an infection due to co aggulase –ve pathogen. which organism do you
suspect to be?
a) Strept.pyogenes
b) Gp B-streptococcus
c) Staph aureus
d) Haemophilus influenza
e) Moraxella catarrhalis

173. Patient suffering from petit mal epilepsy he is now on Carbamazepine but he suffers from
some side effects of it. in order to decrease these side effects, you have to decrease its dose. If
you decrease the dose all side effects will be decreased except:
a) Rash

174. All of the following drugs has antidote except:


a) Salicylate
b) Digoxin
c) Warfarin
d) Acetaminophen
e) Codeine
175. Doctor prescribe Codeine syp. for patient, but he still has pain. What you will advice him:
a) Discuss with the patient drawbacks of codeine
b) Discuss with the patient to go to Dr. for more potent.

176. SG is an asthmatic patient he is on Salbutamol, but he is not controlled because which of


the following:
a) Awake two night / week
b) Require 2 puffs before exercise.
c) Respirometry reading is 85% of normal

177. What to give him:


a) Oral corticosteroid
b) Ipratropium
c) Oral theophylline
d) Salmeterol

178. MG is 40 years old male he suffering from renal failure one year ago. Now he got CAP.
Which of the following he shouldn’t take:
I. Ceftazidim
II. Levofloxacin
III. Ciprofloxacin
a) I only
b) III only
c) I & II
d) II & III
e) I, II & III

179. Which is the susceptible microorganism: Haemophilus influenza

180. CPS doesn’t contain which of the following:


a) Approved indication
b) Therapeutic classification
c) Vaccination schedules
d) Non approved indication

181. How much KCl needed to prepare 1L if 5 ml contain 4 meq of KCl “K= 39.5 Cl = 35”
mEq= (wtxvalency)/Mwt
4 = {wt x1) /74.5  wt = 298mg / 5ml
298 mg ‐‐‐‐‐‐ 5 mI X mg ‐‐‐‐‐‐ 1000ml X = 59600 mg = 59.6gm
182. Patient has a recurrent Tenia pedis. He is treated with Clotrimazole. You will advise him
with all of the following except:
a) Tolnaftate for recurrent treatment
b) Tolnaftate for prophylaxis
c) Tolnaftate used for at least 2 weeks after recovery.
d) Stop miconazole and give tolnaftate
e) spray tolnaftate between toes and foot
f) Keep miconazole bid
g) Change socks everyday
h) alternate shoes on # days

183. FJ IS 45 years old Obese male has hypertension & he is newly diagnosed with Diabetes.
He tries to lose weight by exercise but after 2 months he loose only 10 Ib but his diabetes still
not controlled. what to advise him;
a) Refer to dietitian
b) Continue exercise for another 2-3 months
c) Give Metformin
d) Give Rosiglitazone

184. The good control of his blood pressure will give protection from:
I. Peripheral vascular disease
II. Heart attack
III. Peripheral neuropathy
a) I only
b) III only
c) I & II
d) II & III
e) I, II & III

185. FJ is a diabetic patient he is on oral hypoglycemic. Monitor all of the following every
week or every day except:
a) Hb A1c
b) Blood sugar

186. RJ is 75 years old he has GERD; all of the following are risk factors except:
a) Lying down after eating.
b) Age
187. Closed angle glaucoma. Avoid all except;
a) Amitriptyline
b) Sertraline
c) Haloperidole
d) Metoprolol
e) Doxylamine

188. When you search for a new drug in market, you will look in:
a) Product monograph
b) CPS
c) TC
d) Martindale

189. Evidence–based is present in: Cochrane review

190. FG is a renal failure patient with CAP his score is 115, why he should take I.V.:
a) Acute ill patient & he can’t take oral
b) Oral give delayed effect
c) PSI score must take I.V. till the end of infection
d) Must febrile for 24 hours

191. Building including many doctors & many clinics. the entrance of the building has changed.
what’s the most negative action?
a) Relocate off-site building
b) Retire 3 doctors in one office
c) Two doctors leave building.

192. RF has an infection due to MRSA:


a) Penicillin
b) Ampicillin
c) Amoxicillin
d) Vancomycin
e) Methicillin

193. Regarding Varicella-zoster which is right:


a) Contagious if crusted
b) It is herpes zoster
c) Its is contagious before appearance fo lesions
d) It is passive immunity
194. H2-Antagonist all except:
a) H2-Antagonist faster than antacid
b) OTC is effective as prescription
c) PPI more effective than H2-anagonist.

195. Erythromycin cardiotoxicity increased with:


I. Ketoconazole
II. Ritonavir
III. Rifampicin
a) I only
b) III only
c) I & II
d) II & III
e) I, II & III

196. Patient suffer diarrhea & abdominal cramp for two days. His profile is as the following:
Alendronate once weekly. Hydrochlorothiazide twice daily. Amoxicilline twice daily for seven
days. and finished two days ago. What to council this patient:
a) Take loperamide
b) These symptoms are due to antibiotic and resolved with time
c) Refer the patient to physician as these symptoms is due to C. difficile
d) Advise the patient to increase fluid intake

197. Home nursing increase morphine: Check with nurse

198. Non-profit organization for caring of mentally retarded children.


a) Keep drug away of children
b) Storage of drugs
c) No Ipeca without prescription

199. Report office of controlled drugs to destroy it: Acetaminophen + codeine

200. What to check in nurse department pharmacy:


a) Storage condition
b) Overlock
c) Drug indication
d) Contamination
201. Tinea pedis symptoms except:
a) Fissure & scaly between toes
b) Malodour
c) Pain in the Sole & heel of foot

202. Valeriana used in:


a) Depression
b) Anxiety
c) Insomnia

203. A ph provides products from a pharmacy to children’s home, he will do all except:
a) Ask for a prescription for ipeca only OTC up to 30ml.

204. Codeine 60 mg, mitte: 60, rx: 3 x 10 days. What is true?


a) Max amount dispensed 10 days
b) Repeat this rx
c) Refill
d) Dr. Can phone pharmacist to dispense 10 days
e) Record in sales in way obvious to audit

205. Pt 77 years old, they are afraid to give him for what reason?
a) His age more than 60 years old
b) He had MI 2 years ago
c) His BP 190/50

206. Mother brings Rx for her son 3 years old having OM, this Rx was for 3 day ago: what is
true to tell her
a) Waiting is good to watch in case of her son’s symptoms resolve alone
b) In all age, waiting is not good as it will lead to mastoiditis
c) Is not good to wait if less than 2 years old

207. Mother has a kid taking ADHD treatment, she is worried about addiction, what is true???
a) Methylphenidate is stimulant, does not cause dependence
b) Give treatment quickly 3 month and stop
c) Give one month, stop, repeat after 2 months to avoid addiction
208. Pt has dyslipediemia, hypertension, CVD, all will improve his vascular symptoms except
(he is smoking 1 pack a day, drinking 1 cup of wine a day)
a) Eat fibre food
b) Stop smoking
c) Stop alc intake
d) Doing weight bearing exercise 3 times weekly
e) Keeping blood pressure < 120/80

209. Dr asks for unapproved use of drug, which reference


a) Micromidex
b) Pharmacological clinical data
c) Medline

210. Which vaccine is given every 10 years?


a) Varicella
b) DPT
c) Pneumonia
d) Diphtheria

211. Pt was recently diagnosed with bad cholesterol, what is the pharmacy first concern
a) Pt knowledge about cholesterol (maybe)
b) Start it immediately
c) Reduce diet

212. Old man coming out of hospitals, can be given all vaccines, except
a) Varicella
b) Pneumonia
c) Influenza
d) Dipheteria
e) DPT

213. Patient taking glyburide, acarbose, today he woke up having dry eye, with weak left eye
vision, he asks if these symptoms related to acarbose, glyburide. what is true?
a) Apply tear 2 drops, it will resolve the dry eye
b) Go immediately to physician
c) Refer him to optometrist

214. Pt has dysphagia, all can cause his symptoms except? H. pylori
215. Pt. With b-hemolytic pneumonia, fever, sore throat, all are goals of treatment except
a) Suppurative
b) Prevent transmission to relatives
c) Prevent rh fever
d) Prevent febrile fever

216. Female pt needs Plan B, what is the most appropriate to do if pharmacist with moral issues
and doesn’t want to conflict beneficence.
a) Tell her pharmacy doesn’t provide this medication
b) Refuse to dispense
c) Pharmacists put his moral aside for pt beneficence
d) Ph keeps his morals but also for beneficence gives her a list of
e) Pharmacies that sell it

217. What causes Parkinsonism


a) Metoprolol
b) Metoclopramide

218. Nasal sprays cause systemic absorption, pt takes a lot of them, what is true:
a) Take each separately 10 minutes apart
b) Take some in the morning while others at night
c) Put your finger on tear duct
d) Open eyelid to ensure liquid in gap

219. Pt has grave’s disease, takes Metamucil, all information is true except:
a) Synthetic preparation is good in disectomy
b) Eldery, good for them perp in T3

220. Which are autoimmunie disease all except


a) Cushing syndrome
b) SLE
c) Rh arthirits
d) Myoxedema
e) Osteoarthiritis

221. Female came to the pharmacy with an itchy vagina, when to refer to dr?
a) Fishy odour
b) Severe itchy
c) Clumpy curdy discharge
222. Pt with CHF, hospitalized, now ok, coming out. What to do except
a) Monitor BP < 120/80
b) Salt substitute
c) Healthy food
d) Exercise 3 times weekly

223. What is the active ingredient of cannabis?


 Delta‐9‐tetrahydrocannabinol (THC) and Cannabidol (CBD) are the main components of cannabis
plant.
 THC is a partial CB1 and CB2 agonist → pleasure/feeling high/euphoria due to dopamine release.
 CBD modulates the CB1 and CB2 receptors → s mula on of serotonin and adrenergic receptors →
analgesic, anti‐inflammatory, ↓ nausea, ↓ anxiety and ↓ seizures. It counteracts the psychoactive
effects of THC.

224. Laminar vertical flow, what is false? Use of solutions 24 hours of preperations

225. For sterile solution what is true


a) There is a maximum number of acupunctures to keep it sterile

226. All transmitted when drink water except:


a) Hep B
b) Garidia Lambelia
c) Hep A

227. Mother came to pharmacy with her son covered skin with redness neck, arms, and wants
your advice
a) Cortisone 5 % cream
b) Cold compress
c) Oatmeal baths

228. Nurse gives wrong KCL strength, to avoid this, you are a pharmacy manager, what to think
it is appropriate to do except?
a) Double check preperations
b) Buy commercial KCL from market
c) Make 2 personnels for checking on KCl soln prep
229. How to prepare 50 mg prednisolone; insoluble in water, sol in alc 1:900 dose 250 mg/day,
what is true
a) Soln in isopropyl alc
b) No of tab needed is 50 tabs
c) Use amber glass container

230. R.x: hydromorphone


Mitte:cc
Uc diet
What can you do?
a) Hold it as its clearly a fraud
b) Call the walk-in clinic to confirm the dose
c) Call the walk-in clinic to confirm direction
d) Call dr. To confirm prescription
e) Call the police to confirm fraud

231. Pt with suicidal attempts, has amitritptyline Rx what to do:


a) Refuse to refill
b) Tell him to go to another pharmacy
c) Give him advice not to over dose
d) Give in small proportions

232. Sterilaztion what is false:


a) Used distilled water as products is already sterilized
b) Using gloves
c) Waste products are rid of in………manner

233. Long use of corticosteroids eye drops? Increase IOP

234. Which is maintained in Sale Report:


a) Dexamphetamine
b) Phenteraime
c) Nandrolone
d) Altrazolam

235. Diabetic HT, 1st choice


a) ACEI
b) HCTZ
c) Spironolactone
236. How frequently to monitor?
a) 2 days
b) 7 days
c) 4 weeks
d) 2 months

237. Retrospective case study: based on


a) Already gives medication to pt
b) Presassuming results
c) Self patient ideas

238. Case study best accurate results


a) Randomized case study
b) Cohort case study

239. Child has OM, advise mother;


a) Do not use acetaminophen
b) Use cold compression on ear
c) Use AB symptoms will resolve 48-72 hours after abuse
d) Ask dr. To use ab as prophylaxis to inhibit recurrence frequently

240. PT has COPD; lost consciousness, hospitalized, severe emphysema, smoke 1 pack of
cigareetes a day, shortness of breath, chest xray………bilateral lobe …. classified IV >115
what makes this pt require IV ab treatment immediately
a) Loss of consciousness, won’t be able to swallow
b) Chest X-ray
c) Classified pt as IV

241. What empiric treatment in this case


a) Clindamycin
b) Ceftazidine
c) Ciprofloxacin

242. Which is m.o in this case


a) P. Aeurginosa (s pneumonia, h influenza, m. catarralis, klebsiella)
b) Staph……
c) Strep…….
243. Rx. Lorazepam
Signature is illegible with head letter on Rx indication a physician name in the neighbourhood,
what pharmacist reaction.
a) Call dr. To clarify Rx
b) Call dr. To clarify indication
c) Ask pt to clarify indication
d) Call clinic on Rx to identify physician
e) Call health Canada to notify them about fraud

244. New hiring post for technicians, ph put principals in list include all except.
a) Technician has right to refuse delegations
b) Licensed technicians supervise their own work

245. Pharmacist always sick on weekends (went over his sick days limit), he called next
weekend to call in sick
a) Apply penalty
b) Do nothing as he within his sick day limit
c) Call him and see if his illness is during all weekends or not
d) Call dr. To check his sickness
e) Check with his collegues his sick days

246. A hospital pharmacist is considering doing a protocol for the orientation and training of
new technicians. What is the LEAST activity to include in the protocol?
a) “Shadowing” of another technician for a whole day
b) Rotate the technicians in the local hospitals
c) Give them the human resources policy to read
d) Update medical information by attending big medical event

247. Old man living with his granddaughter, his daughter worried about her 2 y.o, 7y.o 10 y.o.
Insisted to give him influenza shot but he refused. What can she do?
a) Put him in homecare
b) Immunize caregiver and family
c) Give him zanamavir as prophylaxis

248. Leukotrine added to asthma treatment?


a) Decrease SABA inhaler
b) Decrease LABA inhaler
c) Discontinue corticosteroid (oral)
d) Decrease dose of ICS
249. Pheytoin, carbamazepine, dr. will shift due to:
a) Easier dose
b) Carbamazepine reaches saturated kinetics faster
Note: phenytoin works by saturation kinetics and CBZ by linear one.

250. Pt has dry nonproductive cough, what to check in his profile


a) Antihypertensives
b) Inhalers

251. Which is not primary literature?


a) Pharmacoeconomics
b) Case studies
c) Case review

252. A man came (no work) wants to take inhaler instead his son, divorced, no insurance, can’t
pay for the medication, what to do
a) Call the mother totake permission to dispense
b) Refuse to dispense, dispense for son only
c) Give him non pharm adicse to improve asthma

253. You noticed pt took yesterday form another pharmacy OTC codeine syrup; what to do?
a) Refuse to give him
b) Tell her no more in pharmacy
c) Tell her to go to dr. To increase dose of cough syrup
d) Explain to her about opioid dependence

254. Rizatriptan CI
a) HTN
b) Diabetes
c) Peripheral vascular diseases?

255. NSAID DDI with captopril


a) Inhibits PG synthesis which increases captopril level
b) Displaces captopril from protein binding sites
c) Interefer with renal excretion which increases captopril
d) Antagonize the effect of captobril (monitor for HF, blood pressure, renal function)
256. Absolute bioavailability is comparing
a) 2 oral solns
b) 2 IV soln
c) One oral and one IV soln

257. Recent new information on drug is found in


a) Journal pharmacotherapy
b) Fact and comparison
c) TC
d) Applied therapeutics

258. Ampicllin 5,000,000 units, when added to 23 ml of H2O yields 200,000 unit/ ml, how
much H2O needed to prepare 250,000 units/ 0.5 ml?
200,000 ……1 ml 5,000,000…… x X= 25 ml
Given that the final conc had water……. 25 - 23 = 2 ml of Water is needed
Ans: 8 ml

259. Dose of child 20 mg/ kg, rate of infusion 0.5mg/kg/min, knowing that wt of child = 33 lb,
calculate time of infusion.
20 x 33/2.2 = 300 mg 300 mg /o.5 = 600min

260. Drug renaly excreted with no active metabolite. Find oral dose of drug if AUC(IV) of IV
dose 100 mg is 150
answer:
F = AUC oral/ AUC IV
No metabolite means F = 1 1 = AUC oral / 100 AUC oral = 100 mg

261. Dr. prescribed drug A to be prepared form stock soln of Drug B which is 10 %. How much
is to be added to ………gm 5% of drug to have a final conc of 20 %.

262. How many mmoles of NaCl of 4 MEq of Na and CL in 2 L soln knowing that (M. WT of
Na = 23, Cl = 35.5)
ans: mmole = meq/valency
MEq = (wt (gm) / mwt) x 1000 x 2
you have 2 L of 3 % ……….49 gm NaCl. How much of NaCL do u need to make it isotonic

263. Betamethasone cream 30 gm to cover whole body, what is the amound used to cover 10%
BID for 7 days 30x 10 /100 = 3 gm 3 x2 x 7 = 42
264. Hydromorphone 0.2 mg/ml increased to 0.5 mg/ ml we have ampoules of hydromorphone
60 ml, if we have in market containing 10 mg/ml. How many mls of ampoules used to make
0.5mg/ml
a) 1.8 ml
b) 18 ml
c) 12 ml
d) 32 ml

265. Pt taking SR morphine 15 mg BID, Physcian switched to liquid morphine 5 mg/ml, every
4 hrs. Calculate how many ml /dose
a) 1 ml
b) 2 ml
c) 5ml

266. SE of Viagra? Dyspepsia, headache

267. Pt with strept sore throat and allergic to penicillin what to give? Erythromycin

268. Intranasal steroids. which of following is LONG TERM side effect? Septal perforation

269. Rifampin provided in the market as 5mg/5ml. adult dose is 300 mg BID, Child dose 10
mg/Kg. You have 4 adults & 20 children weight 20 Kg & 10 Children weight 30 Kg. How
many ml of Rifampin will be needed to dispense these doses?
Answer:
Adult: 4 adults * 300 mg * 2 per day = 2400 mg/day
20 Children 10 mg ------- Kg X mg ------- 20 kg X = 20*10/1 = 200 mg * 20 = 4000 mg
10 Children 10 mg ------- Kg Y mg ------- 30 kg Y = 30*10/1 = 300 mg * 10 = 3000 mg
Total = 2400+4000+3000 = 9400mg
5 mg ------- 5 ml 9400 mg ------- X ml X = 9400*5 / 5 = 9400 ml

270. Pt with hypertension and diabetes which vaccine not needed?


a) Pneumonia
b) Flu
c) DT booster
d) Diphtheria booster
e) Varicella vaccine
271. Oral Vancomycin is useful in;
a) MRSA
b) C. difficile
c) MSSA

272. Doctor wants to prescribe Penlazocin and ask the pharmacist about its legislation. The
pharmacist says;
a) Written Rx with no refills
b) Written Rx with refills
c) Can be be a written or verbal Rx with refills

273. Increased blood flow cause all of the following diseases (except;)
a) Diabetes
b) Cerebral vascular disease
c) Kidney disease
d) Coronary artery disease

274. Hypertensive patient taking HCT develops edema and renal disease, all can be done
except;
a) Increase HCT
b) Switch to Furosemide

275. Patient is taking Amiodarone-Digoxin-Pravastatin, Amiodarone interacts with;


a) Digoxin
b) Pravastatin

276. Pharmacist in the hospital discovers that a patient was given the wrong potassium
concentration and now his situation is life threatening, the pharmacist will do all of the
following except?
a) Call the doctor and ask him to correct the dose then give the correct dose to the patient
b) Tell the doctor that the patient needs Na polystyrene sulphonate
c) Tell the doctor that the patient needs insulin + glucose

277. How to avoid this mistake in future?


a) Assign two peopre to be responsibre for dispensing potassium
b) Buy USP Potassium form (international)
278. Prednisolone tablets
a) Amber glass
b) Protect from light
c) With food

279. Auxilliary labels for Metronidazole include


a) Empty stomach
b) Amber glass bottles
c) Avoid antacids
d) Prevent sun exposure

280. Long case about a patient who is in the hospital with shortness of breath, flushing and
cramps. He should be giving an IV dose because;
a) He has an acute onset of a disease and need an lV dose
b) Oral dose did not give a good action
c) He has a septic shock so he needs an immdiate lV dose

281. All of following drugs Cause sexual dysfunction except?


a) Celecoxib
b) Finasteride
c) Spironolactone
d) Despiramine
e) Metoprolol

282. Most toxic for kidney;


a) Cisapride
b) Vincristine

283. The pharmacy that opens near a hospital center (physician offices), a walk-in clinic,
physiotherapy offices ...etc. Which office affects the pharmacy the most if it were to close;
a) Physician office
b) Walk-in clinic

284. Good relation between pharmacist and patient. which ethic least to apply in this situation?
a) Veracity
b) Autonmy
c) Paternalism
d) Non-maleficincy
e) Justice
285. A mental home caregiver asked the pharmacist about some topics regarding 1st aid, all
except?
a) Use alcohol with external label
b) Take about safe storage and out of children reach
c) Give him USP cortisone

286. Pharmacist in the hospital, check on the nurse about all of the following except?
a) Drug use
b) Storage of drug
c) Cleaning area of drug

287. There has been a decrease in the number of prescriptions in dispensing area, the most
approperiate action of the manager is;
a) Decrease number of pharmacists
b) Decrease number of technicians
c) Change expert cashier with a new student
d) Decrease expense inventory to all staff

288. Technician with cold symptomps, while counting tablets, he sneezed on them. As a
pharmacist, what should you do? Or; Pharmacy Technician has a cold and was counting tablets
without wearing gloves; what is the manager action?
a) Dispense the dosage form then apply disciplinary action against the tech
b) Discard the dosage form, and send the tech home for rest
c) Discard the dosage form, keep the tech away from the dispensing area & review the
aseptic techniques with him/her
d) Discard the dosage form and apply disciplinary action

289. Lorazepam Rx: Mitte: 600 tablets. Action of the pharmacist is all of the following except
a) Call police and tell them about this doctor
b) Call the doctor to verify Rx
c) Try to get a telephone number and call the doctor
d) Refuse to dispense the Rx

290. Pharmacist from his personal responsibility is to refuse to sell oral contraceptives. What’s
the pharmacist action when a patient brings in a Rx for contraceptives
a) Refuse to give it to her
b) Tell her to go for assessment first
c) Send her to another pharmacy
291. 435 gm sucrose is dissolved in 935 ml of water to make 505 ml volume. what is the
strength (w/w)?
a) 8.6
b) 70
c) 58
d) 86

292. Magic mouth wash formula:


Tetracycline 2 gm
Dihydro 240 ml
Mix to use 15ml PC & HS. lf it's available as Tetracycline 125 mg/5ml, how many mt of
Tetracycline is needed for this preparation?
a) 10
b) 30
c) 80
2000MG ‐‐‐‐‐‐‐ X 125MG ‐‐‐‐‐‐‐‐‐‐ 5ML X = 80ML

293. Which of following is easier to be interchangeable?


a) Oral liquid
b) SC
c) IV

294. Dorzolamide MOA?


a) Decrease IOP
b) Increase flow

295. When should pharmacist object?


a) SSRI for depression
b) Galantamine for dementia
c) Trazadone for depression

296. Which principle where pharmacist respect opinion of pt? Autonomy

297. What is principle where pharmacist gives his 100% effort to pt?
a) Beneficence
b) Veracity
c) Autonomy
298. Definition of Meta analysis?
They gave us long definitions. make sure you goggle it and understand it

299. Pt comes to you and tell you he doesn’t like the MAOI he is taking what to ask him?
a) Why doesn’t he like it?
b) Review with him the technique of use of drug
c) Change it to something else

300. Who regulate pharmacy profession in Canada? NAPRA

301. Q. about bupropion in smoking cessation, which is false?


a) Least effect in smoking cessation
b) 4 weeks are enough for treatment

302. Lamotrigine SE
Lamotrigine Blocks sodium channels, as well as high voltage-dependent calcium channels
Generalized tonic-clonic, absence, myoclonic, tonic/atonic, focal (partial) seizures or
unclassified tonic-clonic seizures
Initial: 25 mg Q2 days PO to 50 mg/day, depending on concurrent AED therapy; after 2
wk give same dose BID × 2 wk, then increase by 100 mg/day at 1- to 2-wk intervals
Usual maintenance: 200–400 mg/day in 2 divided doses
S.E: Q. Rash 5–10%, insomnia, Hepatotoxicity and hematological toxicity.
The most commonly observed adverse experiences associated with the use of adjunctive
therapy with lamotrigine (incidence of at least 10%) were dizziness, headache, diplopia,
somnolence, ataxia, nausea, and asthenia.
1 %: anorexia, weight gain, amnesia, concentration disturbance, confusion, emotional
lability, nervousness, nystagmus, paresthesia, thinking abnormality and vertigo
Advantages: BID dosing; broad spectrum; no enzyme induction (few interactions); some
patients report feeling more “alert”; increasing evidence for monotherapy; increasing use
for primary generalized seizures.
Disadvantages: Very slow dose titration.
Medications that induce lamotrigine glucuronidation include carbamazepine, phenobarbital,
phenytoin, primidone, rifampin, lopinavir/ritonavir and atazanavir/ritonavir
Medications that neither inhibit nor induce lamotrigine glucuronidation include olanzapine,
oxcarbazepine, felbamate, gabapentin, levetiracetam, pregabalin, topiramate & zonisamide
Valproic acid is an inhibitor of lamotrigine glucuronidation
Q. Addition of hormonal contraceptives reduce lamotrigine serum levels by up to 50%.
Expensive at high doses. Only available in oral form.
303. Valproic acid SE
Valproic acid Drug of 1st choice for patients with mixed primary generalized seizures (generalized tonic-
Initial: clonic, myoclonus, absence). Enhance GABA action at inhibitory synapses
250 mg BID PO; S.E: Nausea, weight gain, tremor, hair loss, blood dyscrasias, hepatotoxicity (rare), edema
increase by 250 (rare) menstrual irregularities, pancreatitis, teratogenicity.
mg/day Q3–4
Advantages: Often may use BID dosing; broad spectrum; no hepatic enzyme induction;
days as necessary
very low incidence of rash; cognitive effects generally less than with other older AEDs.
Maintenance: Q. No interactions with oral contraceptives.
750–1000 Disadvantages: Inhibits hepatic enzymes; therefore, many potential drug interactions. Least
mg/day in 2–4 desirable option for women of childbearing potential.
divided doses Neural tube defects (avoid in pregnancy). Don’t prescribe to female in age of pregnancy

You might also like